You are on page 1of 1040

Revised Edition

Master Resource Book for

JEE Main
Mathematics
5000+
Specially Prepared Questions for JEE Main with
Complete Theory
2 Levels Exercises
Exams Questions

PRAFFUL K AGARWAL

ARIHANT PRAKASHAN (Series), MEERUT


Master Resource Book for

JEE Main

ARIHANT PRAKASHAN (Series), MEERUT


All Rights Reserved

© Author
No part of this publication may be re-produced, stored in a retrieval system or by any
means, electronic, mechanical, photocopying, recording, scanning, web or otherwise
without the written permission of the publisher. Arihant has obtained all the information
in this book from the sources believed to be reliable and true. However, Arihant or its
editors or authors or illustrators don’t take any responsibility for the absolute accuracy of
any information published and the damage or loss suffered thereupon.
All disputes subject to Meerut (UP) jurisdiction only.

Administrative & Production Offices


Regd. Office
‘Ramchhaya’ 4577/15, Agarwal Road, Darya Ganj, New Delhi -110002
Tele: 011- 47630600, 43518550

Head Office
Kalindi, TP Nagar, Meerut (UP) - 250002
Tel: 0121-7156203, 7156204

Sales & Support Offices


Agra, Ahmedabad, Bengaluru, Bareilly, Chennai, Delhi, Guwahati,
Hyderabad, Jaipur, Jhansi, Kolkata, Lucknow, Nagpur & Pune.

ISBN 978-93-25792-26-5

PO No : TXT-XX-XXXXXXX-X-XX
Published by Arihant Publications (India) Ltd.

For further information about the books published by Arihant, log on to


www.arihantbooks.com or e-mail at info@arihantbooks.com
Follow us on
Master Resource Book for

JEE Main

PREFACE
In sync with the recent changes in the test pattern and format of JEE Main (Joint Engineering Entrance), it
is my pleasure to introduce Master Resource Book in Mathematics for JEE Main, for the Students
aspiring a seat in a reputed Engineering College. JEE Main is a gateway examination for candidates
expecting to seek admission in Bachelor in Engineering (BE), Bachelor of Technology (B.Tech) and Bachelor
of Architecture (B.Arch) at Indian Institutes of Information Technology (IIITs), National Institutes of
Technology (NITs), Delhi Technological University and other Centrally Funded Technical Institutes (CFTIs).

JEE Main is also an examination which is like screening examination for JEE Advanced
(The gateway examination to India’s most reputed Technical Institutes, Indian Institutes of Technology
IITs). Only the top 2.2 lacs students passed in JEE Main will be able to attempt JEE Advanced. Gradually,
the number of students aspiring for the seat in the Engineering College has increased rapidly in the last
5 Years or so.

This year nearly 10 lacs students appeared for JEE Main and only a few were able to reserve a seat in the
college of their choice, so there is a cut throat competition among the aspirants. Thus, it calls for a
systematic mastery of all the subjects of the test with paramount importance to problem-solving. Most of
the books now in the market have become repetitive with scant respect to the needs of true and effective
learning. This book has been designed to fulfill the perceived needs of the students as such.

— This book comprehensively covers all the topics of JEE Main Mathematics syllabus. The chapters have
been sequenced according to the syllabus of class 11th & 12th. Each chapter has essential theoretical
discussion of the related concepts with sufficient number of solved examples, practice problems and
other solved problems. In each chapter previous years' questions of AIEEE and JEE Main have been
included to help students know the difficulty levels and nature of questions asked in competitive exams
at this level.

— All types of questions have been included in this book: Single Correct Answer Types & Numerical
Value Questions.

— This is the only book which has its subject matter divided as per class 11th & 12th syllabus.

It is hoped this new effort will immensely benefit the students in their goal to secure a seat in the
prestigious engineering college, and would be convenient to teachers in planning their teaching
programmes. Suggestions for further improvement are welcome from the students and teachers.

Prafful K Agarwal
Master Resource Book for

JEE Main

CONTENTS
PART I
Chapters from Class 11th Syllabus
1. Sets 3-16
2. Fundamentals of Relations and Functions 17-25
3. Sequence and Series 26-65
4. Complex Numbers 66-106
5. Quadratic Equations 107-131
6. Permutations and Combinations 132-155
7. Mathematical Induction 156-164
8. Binomial Theorem and its Applications 165-191
9. Trigonometric Function and Equations 192-230
10. Properties of Triangles, Heights and Distances 231-257
11. Cartesian Coordinate System 258-284
12. Straight Lines 285-316
13. Circles 317-350
14. Parabola 351-376
15. Ellipse 377-408
16. Hyperbola 409-434
17. Introduction to Three Dimensional Geometry 435-444
18. Limits and Derivatives 445-471
19. Mathematical Reasoning 472-489
20. Statistics 490-511
21. Fundamentals of Probability 512-527
Master Resource Book for

JEE Main

PART II
Chapters from Class 12th Syllabus
22. Matrices 531-553
23. Determinants 554-589
24. Relation and Functions 590-607
25. Inverse Trigonometric Functions 608-630
26. Continuity and Differentiability 631-666
27. Differentiation 667-698
28. Applications of Derivatives 699-772
29. Indefinite Integration 773-817
30. Definite Integration 818-852
31. Area Bounded by Curves 853-885
32. Differential Equations 886-929
33. Vector Algebra 930-966
34. Three Dimensional Geometry 967-1002
35. Advanced Probability 1003-1032
Master Resource Book for

JEE Main

SYLLABUS
UNIT 1 Sets, Relations and Functions UNIT 6 Binomial Theorem and
Sets and their representation; Union, intersection and its Simple Applications
complement of sets and their algebraic properties; Binomial theorem for a positive integral index,
Power set; Relation, Types of relations, equivalence general term and middle term, properties of
relations, functions;. one-one, into and onto functions, Binomial coefficients and simple applications.
composition of functions.
UNIT 7 Sequences and Series
UNIT 2 Complex Numbers and
Arithmetic and Geometric progressions, insertion
Quadratic Equations of arithmetic, geometric means between two given
Complex numbers as ordered pairs of reals, numbers. Relation between A.M. and G.M. Sum
Representation of complex numbers in the form a+ib upto n terms of special series: ∑ n, ∑ n2, ∑ n3.
and their representation in a plane, Argand diagram, Arithmetico - Geometric progression.
algebra of complex numbers, modulus and argument
(or amplitude) of a complex number, square root of a UNIT 8 Limit, Continuity
complex number, triangle inequality, Quadratic
equations in real and complex number system and their
and Differentiability
solutions. Relation between roots and Real valued functions, algebra of functions,
co-efficients, nature of roots, formation of quadratic polynomials, rational, trigonometric, logarithmic
equations with given roots. and exponential functions, inverse functions.
Graphs of simple functions. Limits, continuity and
UNIT 3 Matrices and Determinants differentiability.
Matrices, algebra of matrices, types of matrices, Differentiation of the sum, difference, product and
determinants and matrices of order two and three. quotient of two functions. Differentiation of
Properties of determinants, evaluation of deter-minants, trigonometric, inverse trigonometric, logarithmic
area of triangles using determinants. Adjoint and exponential, composite and implicit functions
evaluation of inverse of a square matrix using derivatives of order upto two. Rolle's and
determinants and elementary transformations, Test of Lagrange's Mean Value Theorems. Applications of
consistency and solution of simultaneous linear derivatives: Rate of change of quantities,
equations in two or three variables using determinants monotonic - increasing and decreasing functions,
and matrices. Maxima and minima of functions of one variable,
tangents and normals.
UNIT 4 Permutations and Combinations
Fundamental principle of counting, permutation as an UNIT 9 Integral Calculus
arrangement and combination as selection, Meaning of Integral as an anti - derivative. Fundamental
P (n,r) and C (n,r), simple applications. integrals involving algebraic, trigonometric,
exponential and
UNIT 5 Mathematical Induction logarithmic functions. Integration by substitution,
Principle of Mathematical Induction and its simple by parts and by partial fractions. Integration using
applications. trigonometric identities.
Master Resource Book for

JEE Main

Evaluation of simple integrals of the type and a circle with the centre at the origin and
condition for a line to be tangent to a circle,
dx , dx , dx , dx , equation of the tangent. Sections of cones,
x2 ± a2 Öx 2 ± a 2 a2 – x2 Öa 2 – x 2
equations of conic sections (parabola, ellipse and
dx dx , (px + q) dx , hyperbola) in standard forms, condition for y = mx
,
ax 2 + bx + c Öax 2 + bx + c ax 2 + bx + c + c to be a tangent and point (s) of tangency.
(px + q) dx ,
Öax 2 + bx + c Öa 2 ± x 2 dx and Öx 2 – a 2 dx UNIT 12 Three Dimensional
Geometry
Integral as limit of a sum. Fundamental Theorem of Coordinates of a point in space, distance between two
Calculus. Properties of definite integrals. Evaluation of points, section formula, direction ratios and direction
definite integrals, determining areas of the regions cosines, angle between two intersecting lines. Skew
bounded by simple curves in standard form. lines, the shortest distance between them and its
equation. Equations of a line and a plane in different
UNIT 10 Differential Equations forms, intersection of a line and a plane, coplanar
Ordinary differential equations, their order and degree. lines.
Formation of differential equations. Solution of
differential equations by the method of separation of
UNIT 13 Vector Algebra
variables, solution of homogeneous and linear Vectors and scalars, addition of vectors, components
differential equations of the type dy +p (x) y = q(x) of a vector in two dimensions and three dimensional
dx space, scalar and vector products, scalar and vector
triple product.
UNIT 11 Coordinate Geometry
Cartesian system of rectangular coordinates in a plane, UNIT 14 Statistics and Probability
distance formula, section formula, locus and its Measures of Dispersion: Calculation of mean,
equation, translation of axes, slope of a line, parallel and median, mode of grouped and ungrouped data.
perpendicular lines, intercepts of a line on the Calculation of standard deviation, variance and mean
coordinate axes. deviation for grouped and ungrouped data.
Ÿ Straight lines Probability: Probability of an event, addition and
multiplication theorems of probability, Baye's
Various forms of equations of a line, intersection of
theorem, probability distribution of a random variate,
lines, angles between two lines, conditions for
Bernoulli trials and Binomial distribution.
concurrence of three lines, distance of a point from a
line, equations of internal and external bisectors of
UNIT 15 Trigonometry
angles between two lines, coordinates of centroid,
orthocentre and circumcentre of a triangle, equation Trigonometrical identities and equations.
of family of lines passing through the point of Trigonometrical functions. Inverse trigonometrical
intersection of two lines. functions and their properties. Heights and Distances.

Ÿ Circles, conic sections UNIT 16 Mathematical Reasoning


Standard form of equation of a circle, general form of Statements, logical operations and implies, implied
the equation of a circle, its radius and centre, by, if and only if. Understanding of tautology,
equation of a circle when the end points of a contradiction, converse and contra positive.
diameter are given, points of intersection of a line
01
Sets
The theory of sets was developed by German Mathematician Georg Cantor IN THIS CHAPTER ....
(1845-1918). The concept of sets is widely used in the foundation of relations,
functions, logic, probability theory, etc. Sets
Representation of Sets
Sets Power Set
In Mathematics, a set is a collection of well-defined distinct object or Venn Diagram
elements. The elements that make up a set can be any kind of things : people Operations on Sets
letters of alphabet, numbers, geometrical shape, variables or even other sets.
Laws of Algebra of Sets
Generally, sets are denoted by capital letters A, B, C , . . . and its elements are
denoted by small letters a , b, c, . . . . . Cardinal Number of a Finite and
Infinite Set
Let A is a non-empty set. If x is an element of A, then we write ‘x Î A’ and
read as ‘x is an element of A’ or ‘x belongs to A’. If x is not an element of A,
then we write ‘x Ï A’ and read as x is not an element of A’ or ‘x does not belong
to A’.
e.g. A = Set of all vowels in English alphabets.
In this set a , e , i , o and u are members.

Representation of Sets
We can use the following two methods to represent a set.
(i) Listing Method In this method, elements are listed and put within a
braces { } and separated by commas.
This method is also known as Tabular method or Roster method.
e.g. A = Set of all prime numbers less than 11 = { 2, 3, 5, 7}
(ii) Set Builder Method In this method, instead of listing all elements of
a set, we list the property or properties satisfied by the elements of set
and write it as
A = { x : P ( x )} or { x| P ( x )}
It is read as ‘A is the set of all elements x such that x has the property
P ( x ).’ The symbol ‘:’ or ‘|’ stands for such that.
This method is also known as Rule method or Property method.
e.g. The set A = { 1, 2 , 3, 4, 5, 6, 7, 8} is written in set builder form
A = { x : x Î N and x £ 8}
4 JEE Main Mathematics

Different Types of Sets (vii) Subset and Superset


Let A and B be two non-empty sets. If each element of
(i) Empty (Void/Null) Set set A is an element of set B, then set A is known as
A set which has no element, is called an empty set. subset of set B. If set A is a subset of set B, then set B
It is denoted by f or { }. is called the superset of A.
e.g. A = Set of all odd numbers divisible by 2 Also, if A is a subset of B, then it is denoted as A Í B
and read as ‘A is a subset of B’.
and B = {x : x Î N and 5 < x < 6 }
Thus, if x Î A Þ x Î B, then A Í B
Such sets which have atleast one element, are called
non-void set. If x Î A Þ x Ï B, then A Í/ B
and read as ‘A is not a subset of B.’
Note If f represents a null set, then f is never written with in braces
e.g. If A = {1, 2, 3} and B = {1, 2, 3, 4, 5}
i.e. {f} is not the null set.
Here, each element of A is an element of B. Thus,
(ii) Singleton Set A Í B i.e. A is a subset of B and B is a superset of A.
A set which have only one element, is called a singleton set. Note
e.g. A = Set of even prime number • Null set is a subset of each set.
and B = { x : x 2 £ 0, x Î R } • Each set is a subset of itself.
• If A has n elements, then number of subsets of set A is 2 n .
(iii) Finite Sets
A set having finite number of elements is called a finite set.
(viii) Proper Subset
e.g. A = { a , b, c, d }. If each element of A is in set B but set B has atleast
one element which is not in A, then set A is known as
Here, A is a finite set as it has four elements proper subset of set B. If A is a proper subset of B, then
(finite number of elements) it is written as ‘A Ì B ’ and read as A is a proper subset
of B.
(iv) Infinite Sets
A set which does not contain finite number of elements is e.g. If N = {1, 2, 3, 4, …}
called infinite set. and I = {…, –3, –2, –1, 0, 1, 2, 3, …}
e.g. A set of all prime numbers = { 2, 3, 5, 7, 11, 13, K } then NÌI
Here, A has not finite number of elements, hence it is an If A has n elements, then number of proper subsets is
infinite set. 2n - 1.

(v) Equivalent Sets (ix) Comparability of Sets


Two finite sets A and B are said to be equivalent, if they Two sets A and B are said to be comparable, if either
have the same number of elements. A Ì B or B Ì A or A = B, otherwise, A and B are said to
e.g. If A = { 1, 2 , 3} and B = { 3, 7, 9 } be incomparable.
Number of elements in A = 3 e.g. Suppose A = { 1, 2, 3}, B = { 1, 2, 4, 6} and C = { 1, 2, 4}
and number of elements in B = 3 Since, A Ë B or B Ë A or A ¹ B
\ A and B are equivalent sets. \ A and B are incomparable.
But C Ì B
(vi) Equal Sets
\ B and C are comparable sets.
If A and B are two non-empty sets and each element ofset A
is an element of set B and each element of set B is an element (x) Universal Set
of set A, then sets A and B are called equal sets.
A set that contains all sets in given context is called
Symbolically, if x Î A Þ x ÎB the universal set. The universal set is generally
and x ÎB Þ x Î A denoted
e.g. A = {1, 2, 3} and B = {x : x Î N , x £ 3 } by U .
Here, each element of A is an element of B, also each This set can be chosen arbitrarily for any discussion of
element of B is an element of A, then both sets are called given sets but after choosing it is fixed.
equal sets. e.g. Suppose A = {1, 2, 3}, B = {3, 4, 5} and C = {7, 8, 9}
Equal sets are equivalent sets while its converse need not to \ U = {1, 2, 3, 4, 5, 6, 7, 8, 9} is universal set for all
be true. three sets.
Sets 5

Example 1. Which of the following is a singleton set? -1± 5 ± 5 -1


Þ t= =
(a) { x : x < 1, x Î I} 2 2
(b) { x : x = 5, x Î I} 5 -1
Q Î(0 , 1]
(c) { x : x 2 = 1, x Î I} 2
(d) { x : x 2 + x + 1 = 0 , x Î R} 5 -1
\ t= is one of the solution.
2
Sol. (a) (a) { x :| x| < 1, x Î I } = { x : - 1 < x < 1, x Î I } = {0 }
Case II If|t - 1| + |t - 2| = 1, for 1 < t £ 2,
(b) { x :| x| = 5, x Î I } = { x : x = ± 5 } = { ± 5 }
then t 2 - t + 2 = 1, t 2 - t + 1 = 0 have no real solutions
(c) { x : x2 = 1, x Î I } = { x : x = ± 1} = { ± 1}
Case III If|t - 1| + |t - 2| = 2t - 3, for t > 2
ìï -1 ± 12 - 4 ´ 1 üï
(d) { x : x2 + x + 1 = 0 , x Î R } = í x : x = , x Î Rý then t 2 - t + 2 = 2t - 3
ïî 2(1) ïþ Þ t 2 - 3t + 5 = 0 have no real solution.
ì -1 ± 3i ü Since, S be the set of all real roots of the given equation,
= íx : x = , x Î Rý = { f}
î 2 þ then S is a singleton set.
[Q x is real but here x is a complex number]
Hence, option (a) is correct. Power Set
Let A be a non-empty set, then collection of all possible
Example 2. Set A has m elements and Set B has n subsets of set A is known as power set. It is denoted by
elements. If the total number of subsets of A is 112 more than P ( A).
the total number of subsets of B, then the value of m × n is ..... e.g. Suppose A = {1, 2, 3}
(JEE Main 2020)
\ P ( A) = {f, {1}, {2}, {3}, {1, 2}, {2, 3}, {3, 1}, {1, 2, 3}}.
Sol. (28.00) It is given that n( A) = m and n(B ) = n
(a) A Î P ( A)
and 2 m = 2 n + 112. [Q number of subsets of set A and B are (b) { A} Ï P ( A)
2 m and 2 n respectively]
m n 4
Þ 2 - 2 = 2 (7) Properties of Power Set
n m-n 4 3
Þ 2 (2 - 1) = 2 (2 - 1) (i) Each element of a power set is a set.
On comparing n = 4 and m - n = 3 (ii) If A Í B, then P ( A) Í P ( B)
\ m=7 (iii) Power set of any set is always non-empty.
So, m × n = 28 (iv) If set A has n elements, then P ( A)has 2n elements.

Example 3. Let S be the set of all real roots of the (v) P ( A) Ç P ( B) = P ( A Ç B)


x x x x
equation, 3 (3 - 1) + 2 = |3 - 1| + |3 - 2|. Then, S (vi) P ( A) È ( B) Í P ( A È B)
(JEE Main 2020) (vii) P ( A È B) ¹ P ( A) È P ( B)
(a) is a singleton 8. P(P(......N times (A))) = 2^N(no.of elements in P[A]).
(b) is an empty set Example 4. If set A = {1, 3, 5}, then number of elements in
(c) contains at least four elements P{P( A)} is
(d) contains exactly two elements (a) 8 (b) 256 (c) 248 (d) 250
Sol. (a) Given equation Sol. (b) Given, A = {1, 3, 5}
in any 3 x (3 x - 1) + 2 = |3 x - 1| + |3 x - 2| \ n {P( A)} = 23 = 8
question if
mod appears Let 3 x = t > 0 , " x Î R, so equation is \ n [P {P( A)}] = 2 8 = 256
just break it t 2 - t + 2 = |t - 1| + |t - 2| …(i)
using its
critical points ì1 - t + 2 - t , 0 < t £ 1 Example 5. If A = {f, {f}}, then the power set of A is
ï
and solve it by Q |t - 1| + |t - 2| = ít - 1 + 2 - t , 1 < t £ 2 (a) A (b) {f , {f}, A}
cases ït - 1 + t - 2 , t >2 (c) {f, {f}, {{f}}, A} (d) None of these
î
ì3 - 2t , 0 < t £ 1 Sol. (c) We have,
ï
=í 1 , 1< t £ 2 A = {f , {f}}
ï2t - 3 , t >2 Subset of set A are f , {f}, {{f}}, {f , {f}}
î
Now, Case I If|t - 1| + |t - 2| = 3 - 2t , for 0 < t £ 1, then \ Power set of A i.e. P( A) = {f , {f}, {{f}}, {f , {f}}}
t 2 - t + 2 = 3 - 2t Þ t 2 + t - 1 = 0 Þ P( A) = {f , {f}, {{f}}, A}
6 JEE Main Mathematics

Venn Diagram (ii) Intersection of Two Sets


If A and B are two sets, then intersection of A and B is a
A set is represented through a diagram is called Venn set of all those elements which are in both A and B.
diagram. The intersection of A and B is denoted by A Ç B and read
In Venn diagram, the universal set is represented by a as ‘A intersection B ’.
rectangular region and a set is represented by circle or a Symbolically,
closed geometrical figure inside the universal set. Also,
an element of a set A is represented by a point within the A Ç B = {x : x Î A and x Î B }
circle of set A. If x Î A Ç B Þ x Î A and x Î B
e.g. If U = {1, 2, 3, 4, …, 10} and A = {1, 2, 3} and if x Ï A Ç B Þ x Ï A or x Ï B
Then, its Venn diagram is as shown in the figure. The Venn diagram of A Ç B is as shown in the figure and
the shaded region represents A Ç B.
10 U
U U U
4 8
1 A B
2 A B A B
3 A
5 9

6 7
AÇB A Ç B when neither AÇB=f
when A Í B or A Ç B = A A Í B nor B Í A (no shaded region)
Operations on Sets e.g. If A = {1, 2, 3, 4} and B = {4, 3, 5, 6}
We introduce some operations on sets to construct new \ A Ç B = {3, 4}
sets from the given ones.
General Form
(i) Union of Two Sets
The intersection of a finite number of sets
Let A and B be two sets, then union of A and B is a set of
all those elements which are in A or in B or in both A1 , A2 , A3 , . . . , A n is represented by
n
A and B. It is denoted by A È B and read as ‘A union B ’. A1 Ç A 2 Ç A3 Ç . . . Ç A n or Ç Ai
i =1
Symbolically, A È B = {x : x Î A or x Î B }
n
Clearly, xÎAÈB Symbolically, Ç Ai = { x : x Î Ai for all i}
i =1
Þ x Î A or x Î B
If xÏAÈB (iii) Disjoint of Two Sets
Þ x Ï A and x Ï B Two sets A and B are known as disjoint sets, if A Ç B = f
The venn diagram of A È B is as shown in the figure and i.e. if A and B have no common element. The Venn
the shaded portion represents A È B. diagram of disjoint sets as shown in the figure

U U U U

A A B A B
B A B

AÈB A È B when neither A È B when A and B AÇB


(when A Í B) A Í B nor B Í A are disjoint sets
e.g. If A = {1, 2, 3}
e.g. If A = {1, 2, 3, 4} and B = {4, 5, 6},
and B = {4, 8, 5, 6} then AÇB={ } = f
\ A È B = {1, 2, 3, 4, 5, 6, 8}. \ A and B are disjoint sets.

General Form (iv) Difference of Two Sets


The union of a finite number of sets A1 , A2 , . . . , An is If A and B are two non-empty sets, then difference of
represented by A and B is a set of all those elements which are in A but
n not in B. It is denoted as A - B. If difference of two sets is
A1 È A 2 È A3 È . . .È A n or È A i
i =1 B - A, then it is a set of those elements which are in B
n but not in A.
Symbolically, È Ai = { x : x Î Ai for atleast one i} Hence, A - B = {x : x Î A and x Ï B }
i =1
Sets 7

and B - A = {x : x Î B and x Ï A} A¢ U
If x Î A - B Þ x Î A but x Ï B shaded region is A'
A
and if x Î B - A Þ x Î B but x Ï A
The Venn diagram of A - B and B - A are as shown in the
figure and shaded region represents A - B and B - A.
If U is a universal set and A Ì U ,
U U then A¢ = U - A = {x : x ÎU but x Ï A}
B A
i.e. x Î A Þ x Ï A¢
A B
The Venn diagram of complement of a set A is as shown
in the figure and shaded portion represents A¢.
A–B A–B e.g. If U = {1, 2, 3, 4, 5, …}
when A Í B, i.e., (A – B = f) when B Í A
and A = {2, 4, 6, 8, …}
U U \ A¢ = U - A = { 1, 3, 5, 7, …}
A B A B Note
• f = U¢ • f¢ = U • ( A¢ )¢ = A
• A È A¢ = U • A Ç A¢ = f
A – B when neither A – B when A and B are
A Í B nor B Í A disjoint sets. Clearly, Example 6. If X = {4n - 3n - 1: n Î N} and
A–B=A
Y = { 9 (n - 1) : n Î N}, where N is the set of natural numbers,
e.g. If A = {1, 2, 3, 4} and B = {4, 5, 6, 7, 8} then X È Y is equal to
\ A - B = {1, 2, 3} and B - A = {5, 6, 7, 8} (a) N (b) Y - X
(c) X (d) Y
Important Points Sol. (d) We have, X = { 4 n - 3n - 1 : n Î N }
● A- B¹ B- A Þ X = {0 , 9, 54, 243, L } [put n = 1, 2, 3, L ]
● A - B Í A and B - A Í B
and Y = {9(n - 1) : n Î N }
● A - f = A and A - A = f
Þ Y = {0 , 9, 18, 27, L} [put n = 1, 2, 3, L ]
● The sets A - B and B - A are disjoint sets.
It is clear that X Ì Y .
(v) Symmetric Difference of Two Sets \ X ÈY = Y
If A and B are two sets, then set ( A - B) È ( B - A) is Example 7. Consider the two sets A = {m Î R : both the
known as symmetric difference of sets A and B and is
denoted by ADB. The Venn diagram of ADB is as shown roots of x 2-(m + 1) x + m + 4 = 0 are real} and B = [ - 3, 5).
in the figure and shaded region represents ADB. Which of the following is not true? (JEE Main 2020)
U (a) A - B = ( - ¥ , - 3) È (5, ¥)
(b) A Ç B = {- 3}
A B (c) B - A = ( - 3, 5)
(d) A È B = R
Sol. (a) Given sets A = {m ÎR : both the roots of
AD B
x2 - (m + 1) x + m + 4 = 0 are real} and B = [ - 3, 5)
e.g. A = {1, 2, 3} and B = {3, 4, 5, 6},
Q Roots of x2- (m + 1) x + m + 4 = 0 are real, m ÎR
then ADB = ( A - B) È ( B - A)
\ D ³ 0 Þ (m + 1) 2 - 4(m + 4) ³ 0
= {1, 2} È {4, 5, 6} = {1, 2, 4, 5, 6}
Þ m2 - 2m - 15 ³ 0
Note 2
Þ m - 5m + 3m - 15 ³ 0
• Symmetric difference can also be written as
ADB = ( A È B) - ( A Ç B)
Þ m (m - 5) + 3 (m - 5) ³ 0
Þ (m + 3) (m - 5) ³ 0
• ADB = BDA (commutative)
Þ m Î ( - ¥ , - 3] È [5, ¥) [Q A = ( - ¥ , - 3] È [5, ¥)]
(vi) Complement of a Set \ A - B = ( - ¥ , - 3) È [5, ¥)
The complement of a set A is the set of all those elements A Ç B = {- 3}, B - A = ( - 3, 5) and A È B = R
which are in universal set but not in A. It is denoted by Hence, option (a) is correct.
A¢ or Ac.
8 JEE Main Mathematics

Laws of Algebra of Sets Sol. (a) Given sets A = {x Î R :| x| < 2},


and B = {x Î R :| x - 2| ³ 3}
For three sets A, B and C
then, A = {x Î R : - 2 < x < 2}
(i) Idempotent law
and B = {x Î R :( x - 2) Î ( -¥ , - 3] È [3, ¥)}
(a) A È A = A
{x Î R : x Î ( -¥ , - 1] È [5, ¥)}
(b) A Ç A = A
\ B - A = R - ( -2, 5)
(ii) Identity law
(a) A È f = A Cardinal Number of a Finite
(b) A Ç U = A and Infinite Set
(iii) Commutative law The number of distinct elements in a finite set A is called
cardinal number and it is denoted by n( A). And if it is not
(a) A È B = B È A
finite set, then it is called infinite set.
(b) A Ç B = B Ç A e.g., If A = { -3, - 1, 8, 10, 13, 17}, then n( A) = 6
(iv) Associative law
(a) ( A È B) È C = A È ( B È C ) Properties
(b) A Ç ( B Ç C ) = ( A Ç B) Ç C If A, B and C are finite sets and U be the finite universal
set, then
(v) Distributive law
(i) n( A È B) = n( A) + ( B) - n( A Ç B)
(a) A È ( B Ç C ) = ( A È B) Ç ( A È C ) (ii) n( A È B) = n( A) + n( B), if A and B are disjoint.
(b) A Ç ( B È C ) = ( A Ç B) È ( A Ç C ) (iii) n( A - B) = n( A) - n( A Ç B)
(vi) De-Morgan’s law (iv) n( ADB) = n( A) + n( B) - 2n ( A Ç B)
(a) ( A È B)¢ = A¢ Ç B¢ (v) n( A È B È C ) = n( A) + n( B) + n(C )
(b) ( A Ç B)¢ = A¢ È B¢ - n( A Ç B) - n( B Ç C ) - n( A Ç C ) + n( A Ç B Ç C )
(vi) n (number of elements in exactly two of the sets
(c) A - ( B Ç C ) = ( A - B) È ( A - C )
A, B, C )
(d) A - ( B È C ) = ( A - B) Ç ( A - C ) = n( A Ç B) + n( B Ç C ) + n(C Ç A) -3n( A Ç B Ç C )
(vii) (a) A - B = A Ç B¢ (vii) n (number of elements in exactly one of the sets
(b) B - A = B Ç A¢ A, B, C ) = n( A) + n( B) + n(C ) - 2n( A Ç B)
(c) A - B = A Û A Ç B = f - 2n( B Ç C ) - 2n( A Ç C ) + 3n( A Ç B Ç C )
(viii) n( A¢ È B¢ ) = n( A Ç B)¢ = n(U ) - n( A Ç B)
(d) ( A - B) È B = A È B
(ix) n( A¢ Ç B¢ ) = n( A È B)¢ = n(U ) - n( A È B)
(e) ( A - B) Ç B = f
(x) n( B - A) = n( B) - n( A Ç B)
(f) A Ç B Í A and A Ç B Í B
Example 9. In a town of 10000 families it was found that
(g) A È ( A Ç B) = A
40% families buy newspaper A, 20% families buy newspaper
(h) A Ç ( A È B) = A B and 10% families buy newspaper C, 5% buy A and B, 3%
(viii) (a) ( A - B) È ( B - A) = ( A È B) - ( A Ç B) buy B and C and 4% buy A and C. If 2% families buy all of
three newspapers, then the number of families which buy A
(b) A Ç ( B - C ) = ( A Ç B) - ( A Ç C )
only, is
(c) A Ç ( BDC ) = ( A Ç B) D ( A Ç C ) (a) 4400 (b) 3300
(d) ( A Ç B) È ( A - B) = A (c) 2000 (d) 500
(e) A È ( B - A) = ( A È B) Sol. (b) n( A) = 40% of 10000 = 4000 , n(B ) = 2000 ,
n(C) = 1000 , n( A Ç B ) = 500, n(B Ç C) = 300 , n(C Ç A) = 400,
Example 8. If A = {x Î R :| x| < 2} and n( A Ç B Ç C) = 200
B = {x Î R :| x - 2| ³ 3}; then (JEE Main 2020) \ n( A Ç B Ç C ) = n{A Ç (B È C) ¢} = n( A) - n{A Ç (B È C)}
(a) B - A = R - ( -2, 5) = n( A) - n( A Ç B) - n( A Ç C) + n( A Ç B Ç C)
(b) A - B = [ -1, 2) = 4000 - 500 - 400 + 200
(c) A È B = R - (2, 5) = 3300
(d) A Ç B = ( -2, - 1)
Sets 9

Example 10. A survey shows that 63% of the people in a (a) 13.5 (b) 13
city read newspaper A whereas 76% read newspaper B. If x % (c) 12.8 (d) 13.9
of the people read both the newspapers, then a possible value Sol. (d) Let the population of city is 100.
of x can be
Then, n( A) = 25, n(B ) = 20 and n( A Ç B ) = 8
(a) 55 (b) 29
A B U
(c) 65 (d) 37
Sol. (a) Given, n ( A) = 63, n (B) = 76 and n ( A Ç B) = x
17 8 12
We know that, n(U)=100

n ( A È B ) = n ( A) + n (B ) - n ( A Ç B )
Þ 100 ³ 63 + 76 - x
Þ x ³ 139 - 100 ³ 39 Venn diagram

and n ( A Ç B ) £ n( A )
So, n( A Ç B) = 17 and n( A Ç B) = 12
Þ x £ 63
According to the question,
\ 39 £ x £ 63
Percentage of the population who look into advertisement is
Example 11. Two newspapers A and B are published in a é 30 ù é 40 ù
=ê ´ n( A Ç B) ú + ê ´ n( A Ç B) ú
city. It is known that 25% of the city population reads A and ë100 û ë100 û
20% reads B while 8% reads both A and B. Further, 30% of é 50 ù
those who read A but not B look into advertisements and 40% +ê ´ n( A Ç B) ú
ë 100 û
of those who read B but not A also look into advertisements,
æ 30 ö æ 40 ö æ 50 ö
while 50% of those who read both A and B look into =ç ´ 17÷ + ç ´ 12÷ + ç ´ 8÷
è100 ø è100 ø è100 ø
advertisements. Then, the percentage of the population who
look into advertisements is (JEE Main 2019) = 5 .1 + 4 .8 + 4 = 13 .9
Practice Exercise
ROUND I Topically Divided Problems
Sets, Their Representation and Venn Diagram and Operations on Sets
Types of Sets 8. If A and B are two given sets, then A Ç ( A Ç B) c is
1. If X n = ìí z = x + iy :|z|2 £ üý for all integers n ³ 1 .
1 equal to
î nþ (a) A (b) B (c) f (d) A Ç Bc
¥
Then, I X n is 9. The set A = { x : x Î R, x 2 = 16 and 2 x = 6 }is equal to
n=1
(a) f (b) {14, 3, 4}
(a) a singleton set (c) {3} (d) {4}
(b) not a finite set
(c) an empty set 10. If A and B are two sets, then ( A È B) ¢ È ( A ¢ Ç B) is
(d) a finite set with more than one element equal to
(a) A ¢ (b) A
2. The set A = { x :|2 x + 3|< 7 } is equal to (c) B¢ (d) None of these
(a) D = { x : 0 < x + 5 < 7}
11. Three sets A, B and C are such that A = B Ç C and
(b) B = { x : - 3 < x < 7}
B = C Ç A, then
(c) E = { x : - 7 < x < 7}
(d) C = { x : - 13 < 2x < 4} (a) A Ì B (b) A É B
(c) A = B (d) A Ì B¢
3. Let A = {1, { 2, 3}}. Then, the number of subsets
of A, is
12. The set ( A È B È C) Ç ( A Ç B ¢ Ç C ¢ ) ¢ Ç C ¢
(a) 2 (b) 4 (c) 8 (d) 7 is equal to
(a) B Ç C ¢ (b) A Ç C
4. Let n( A) = m and n( B) = n, if the number of subsets (c) B¢ Ç C ¢ (d) None of these
of A is 56 more than of subsets of B, then m + n is
equal to 13. Let S1, S2 and S3 be three sets defined as
(a) 9 (b) 13 (c) 8 (d) 10 S1 = { z Î C :|z - 1|£ 2 }
5. Let A be a set represented by the squares of S2 = { z Î C : Re((1 - i) z) ³ 1}
natural numbers and x, y are any two elements S3 = { z Î C : Im ( z) £ 1}
of A, then Then, the set S1 Ç S2 Ç S3 (JEE Main 2021)
(a) x - y Î A (b) xy Î A (a) is a singleton
x (b) has exactly two elements
(c) x + y Î A (d) Î A
y (c) has infinitely many elements
(d) has exactly three elements
6. If A and B are two non-empty subsets of a set X
such that A is not a subset of B. Then, 14. Let A, B, C be three sets such that A È B È C = U ,
(a) B is a subset of A where U is universal set. Then,
(b) A and B¢ are non-disjoint sets {( A - B) È ( B - C) È (C - A)}¢ is equal to
(c) A and B are disjoint sets (a) A È B È C (b) A È (B Ç C )
(d) A is a subset of B¢ (c) A Ç B Ç C (d) A Ç (B È C )
7. If P = {q : sin q - cos q = 2 cos q } and 15. If aN = { an : n Î N } and bN Ç cN = dN , where
Q = {q : sin q + cos q = 2 sin q } are two sets. Then, a, b, c Î N and b, c are coprime, then
(a) P Ì Q and Q - P ¹ f (b) Q Ë P (a) b = cd (b) c = bd
(c) P Ë Q (d) P = Q (c) d = bc (d) None of these
Sets 11

16. Universal set, U = { x : x 5 - 6 x 4 + 11x 3 - 6 x 2 = 0 }, 20. Out of 64 students, the number of students taking
A = { x : x 2 - 5 x + 6 = 0 } and B = { x : x 2 - 3x + 2 = 0 }. Mathematics is 45 and number of students taking
Then, ( A Ç B) ¢ is equal to both Mathematics and Biology is 10. Then, the
(a) {1, 3} (b) {1, 2, 3} number of students taking only Biology is
(c) {0, 1, 3} (d) {0, 1, 2, 3} (a) 18 (b) 19
(c) 20 (d) None of these
17. For any two sets A and B, if A Ç X = B Ç X = f and
A È X = B È X for some set X, then 21. In a group of 50 people, two tests were conducted,
(a) A - B = A Ç B one for diabetes and one for blood pressure. 30
(b) A = B people were diagnosed with diabetes and 40 people
(c) B - A = A Ç B were diagonsed with high blood pressure. What is
(d) None of the above the minimum number of people who were having
diabetes and high blood pressure?
Cardinal Number of a Finite (a) 0 (b) 10
and Infinite Sets (c) 20 (d) 30
18. There is a group of 265 persons who like either 22. Let X be the universal set for sets A and B. If
singing or dancing or painting. In this group, 200 n( A) = 200, n( B) = 300 and n( A Ç B) = 100, then
like singing, 110 like dancing and 55 like painting. n( A ¢ Ç B ¢ ) is equal to 300 provided n ( X ) is equal to
If 60 persons like both singing and dancing, 30 like (a) 600 (b) 700
both singing and painting and 10 like all three (c) 800 (d) 900
activities, then the number of persons who like only 23. In a college of 300 students, every student reads
dancing and painting is 5 newspaper and every newspaper is read by
(a) 10 (b) 20 60 students. The number of newspaper is
(c) 30 (d) 40 (a) atleast 30 (b) atmost 20
50 n
(c) exactly 25 (d) None of these
19. Let U X i = U Yi = T, where each X i contains 10
i =1 i =1 24. Out of 800 boys in a school, 224 played Cricket,
elements and each Yi contains 5 elements. If each 240 played Hockey and 336 played Basketball. Of
element of the set T is an element of exactly 20 of the total, 64 played both Basketball and Hockey;
sets X i ¢ s and exactly 6 of sets Yi ¢ s, then n is equal 80 played Cricket and Basketball and 40 played
to (JEE Main 2020) Cricket and Hockey; 24 played all the three games.
(a) 50 (b) 15 The number of boys who did not play any game is
(c) 45 (d) 30 (a) 128 (b) 216
(c) 240 (d) 160

Only One Correct Option 4. If A = {( x, y) : x 2 + y 2 = 25 } and


1. If A = {( x, y) : y = e- x } and B = {( x, y) : y = - x }. Then, B = {( x, y) : x 2 + 9 y 2 = 144 }, then A Ç B contains
(a) A Ç B = f (b) A Ì B (a) one point
(c) B Ì A (d) A Ç B = {(0, 1), (0, 0)} (b) two points
2. If A1, A2 , A3, K A100 are sets such that n( Ai ) = i + 2 (c) three points
100 (d) four points
A1 Ì A2 Ì A3 K Ì A100 and I Ai = An then n( A) is
i=3
5. For any three sets A1, A2 , A3. Let
equal to B1 = A1, B2 = A2 - A1 and B3 = A3 - ( A1 È A2 ), then
(a) 3 (b) 4 (c) 5 (d) 16 which of the following statement is always true.
(a) A1 È A2 È A3 É B1 È B2 È B3
3. The number of elements in the set
(b) A1 È A2 È A3 = B1 È B2 È B3
{ x Î R : (|x|- 3)|x + 4|= 6 } is equal to [JEE Main 2021] (c) A1 È A2 È A3 Ì B1 È B2 È B3
(a) 3 (b) 2 (c) 4 (d) 1
(d) None of the above
12 JEE Main Mathematics

-1
6. If A = ìí x : cos x > ,0 £ x £ püý and 10. In a class of 140 students numbered 1 to 140, all
î 2 þ even numbered students opted Mathematics
1 p
B = ìí x : sin x > , £ x £ püý, then
course, those whose number is divisible by 3 opted
î 2 3 þ Physics course and those whose number is divisible
ì p 2p ü by 5 opted Chemistry course. Then, the number of
(a) A Ç B = í x : £ x £ ý
î 3 3þ students who did not opt for any of the three
ì -p 2p ü courses is (JEE Main 2019)
(b) A Ç B = í x : £x£ ý
î 3 3þ (a) 42 (b) 102 (c) 38 (d) 1
ì -5 p 5p ü
(c) A È B = í x : £x£ ý Numerical Value Type Questions
î 6 6þ

11. Let X = { n Î N : 1 £ n £ 50 }. If A = { n Î X : n is
ì
(d) A È B = í x : 0 £ x £ ý multiple of 2} and B = { n Î X : n is a multiple of 7},
î 6þ
then the number of elements is the smallest subset
7. If A = {( x, y) : y = e2 x , x Î R } of X containing both A and B is ………… .
and B = {( x, y) : y = e-2 x , x Î R }, then A Ç B contains (JEE Main 2020)

(a) No points 12. Suppose A1, A2 , . . . , A30 are thirty sets each having
(b) One points 3 elements and B1, B 2 , . . . , Bn are n sets each
(c) Two points having
(d) Three points 30 n
3 elements. Let È Ai = È B j = S and each
8. If A = ìí ( x, y) : y = , 0 ¹ x Î Rüý
4 i =1 j =1
î x þ elements of S belongs to exactly 10 of Ai ’ s and
and B = {( x, y) : y = x, x > 0, x Î R } exactly 9 of B j ’ s. The value of n is equal to
(a) A ÇB = f
13. Let S = {1, 2, 3, K, 50 }. The number of non-empty
(b) A Ç B is singleton set
(c) A Ç B has infinite elements subsets A of S such that product of element in A is
(d) A Ç B has two elements even, is 2 m ( 2 n - 1), then the value of ( m + n) is
9. Let A, B and C be sets such that f ¹ A Ç B Í C. equal to ............ .
Then, which of the following statements is not 14. The maximum number of sets obtainable from A
true? (JEE Main 2019) and B applying union and difference operation is
(a) B Ç C ¹ f ............ .
(b) If ( A - B) Í C, then A Í C
(c) (C È A ) Ç (C È B) = C
15. In a factory 70% of the workers like oranges and
(d) If ( A - C ) Í B, then A Í B 64% like apples. If x% like both oranges and apples,
then the minimum value of x is ............ .

Answers
Round I
1. (a) 2. (a) 3. (b) 4. (a) 5. (b) 6. (b) 7. (d) 8. (d) 9. (a) 10. (a)
11. (c) 12. (a) 13. (c) 14. (c) 15. (c) 16. (c) 17. (b) 18. (a) 19. (d) 20. (b)
21. (c) 22. (b) 23. (c) 24. (d)

Round II
1. (a) 2. (c) 3. (b) 4. (d) 5. (a) 6. (a) 7. (b) 8. (b) 9. (d) 10. (c)
11. (29) 12. (45) 13. (50) 14. (8) 15. (34)
Solutions
Round I \ tan q =
1
´
2+1
= ( 2 + 1)
ì 1ü ì 1ü 2 -1 2+1
1. Given, X n = í z = x + iy :| z |2 £ ý = í x2 + y2 £ ý
î nþ î nþ \ P =Q
ì 1 ü 8. A Ç ( A Ç B)c = A Ç ( A c È Bc ) (Qby De-Morgan’s law)
\ X1 = { x2 + y2 £ 1}, X 2 = í x2 + y2 £ ý
î 2þ = ( A Ç A c ) È ( A Ç Bc )
ì 1ü = f È ( A Ç Bc ) = A Ç Bc
X3 = í x2 + y2 £ ý Þ X ¥ = { x2 + y2 £ 0}
î 3þ 9. Since, x2 = 16 Þ x = ± 4
¥
\ I X n = [X1 Ç X 2 Ç X3 Ç L Ç X ¥ = { x2 + y2}] = {0} and 2x = 6 Þ x = 3
n =1 Hence, no value of x is satisfied.
¥
\ A=f
Hence, I X n is a singleton set.
n =1 10. We have, ( A È B)¢ È ( A ¢ Ç B)
2. Given, set A = { x :|2x + 3| < 7} = ( A ¢ Ç B¢) È ( A ¢ Ç B)
Now, |2x + 3| < 7 Þ - 7 < 2x + 3 < 7 [by De-Morgan’s law]
= A ¢ Ç [(B¢ È B)] [distributive law]
Þ - 7 - 3 < 2x < 7 - 3 Þ - 10 < 2x < 4
= A¢ Ç U = A¢
Þ - 5 < x < 2 Þ 0 < (x + 5) < 7
11. Given, A = B Ç C and B = C Ç A
3. We have, A = {1, {2, 3}}
Then, A = (C Ç A ) Ç C
Number of element in A = 2
A = ( A Ç C ) Ç C = A Ç (C Ç C ) [associative]
Number of subsets of A = 22 = 4
= A ÇC
4. Since, total possible subsets of sets A and B are 2m and A =C Ç AÞ A = B
2n, respectively.
12. ( A È B È C ) Ç ( A Ç B¢ Ç C ¢ )¢ Ç C ¢
According to given condition,
= ( A È B È C ) Ç ( A¢ È B È C ) Ç C ¢
2m - 2n = 56
m- n = (f È B È C ) Ç C ¢
Þ 2 (2 - 1) = 23 ´ (23 - 1)
n
= (B È C ) Ç C ¢
On comparing both sides, we get
= (B Ç C ¢ ) È f = B Ç C ¢
2n = 23 and 2m - n = 23
Þ n = 3 and m - n = 3 13. For|z - 1|£ 2 , z lies on and inside the circle of radius 2
units and centre (1, 0).
Þ m = 6 and n = 3
Y
Now, m + n = 6 + 3 = 9
(0, 1)
5. Let x, y Î A. Then, Im(z)=1
2 2
x = m , y = n for some m, n Î N S1 Ç S2 Ç S3
X¢ X
Þ xy = (mn )2 Î A (1, 0)
6. Since, A Ë B therefore it is not necessary that B is
subset of A. x+y=1

U
A B
For S 2, let z = x + iy
Now, (1 - i )(z ) = (1 - i )(x + iy)
Re((1 - i )z ) = x + y Þ x + y ³ 1
Þ S1 Ç S 2 Ç S3 has infinity many elements
14. We have, {( A - B) È (B - C ) È (C - A )}
And also, it is not necessary that A and B are disjoint.
Obviously, A and B¢ are not disjoint. = ( A - B)¢ Ç (B - C )¢ Ç (C - A )¢
[by De-Morgan’s law]
7. Since, cos q ( 2 + 1) = sin q
= B ÇC Ç A
Þ tan q = 2 + 1
[Q ( A - B)¢ = B, (B - C )¢ = C , (C - A )¢ = A ]
and sin q ( 2 - 1) = cos q
= A Ç B ÇC
14 JEE Main Mathematics

15. Given, aN = { an : n Î N } 22. Given, n ( A ) = 200, n (B) = 300, n ( A Ç B) = 100


\ bN = { bn : n Î N } and cN = { cn : n Î N } We know that, n ( A È B) = n ( A ) + n (B) - n ( A Ç B)
Also, given bN Ç cN = dN \ n ( A È B) = 200 + 300 - 100 = 400
\ bc Î bN Ç cN or bc Î dN Also, n ( A ¢Ç B¢ ) = n{( A È B)¢ }
\ bc = d [Q b and c are coprime] = n (X ) - n ( A È B)
[Q coprime numbers are those numbers, whose Þ 300 = n (X ) - 400
common factor is 1. e.g., (i) 3 and 5 (ii) 5 and 9 etc.]
Þ n (X ) = 700
16. U = { x : x5 - 6x4 + 11x3 - 6x2 = 0} = {0, 1, 2, 3}
A = { x : x2 - 5x + 6 = 0} = {2, 3} 23. Let number of newspaper be x. If every student reads
one newspaper, the number of students would be 60x.
and B = { x : x2 - 3x + 2 = 0} = {2, 1}
\ ( A Ç B)¢ = U - ( A Ç B) Since, every student reads 5 newspapers.
x ´ 60
= {0, 1, 2,3} - {2} = {0, 1, 3} \ Number of students = = 300
5
17. Given, A Ç X = B Ç X = f
Þ x = 25
So, A and X , B and X are disjoint sets.
Also, A È X = B È X Þ A = B 24. Here, n (C ) = 224, n (H ) = 240, n (B ) = 336,
n (H Ç B ) = 64, n (B Ç C ) = 80, n (H Ç C ) = 40,
18. Let D denotes dancing, P denotes painting and S
n (C Ç H Ç B ) = 24
denotes singing.
\ n (D È P È S ) = 265, \ n (C c Ç H c Ç Bc ) = n [(C È H È B )c ]
n (S ) = 200, n (D ) = 110, n (P ) = 55, = n (U ) - n (C È H È B)
n (S Ç D ) = 60, n (S Ç P ) = 30 = 800 - [224 + 240 + 336 - 64
and n (D Ç P Ç S ) = 10 - 80 - 40 + 24]
Q n (D È P È S ) = n (D ) + n (P ) + n (S ) - n (D Ç P ) = 800 - 640 = 160
- n (P Ç S ) - n (S Ç D ) + n (D Ç P Ç S )
\ 265 = 110 + 55 + 200 - n (D Ç P ) - 30 - 60 + 10 Round II
Þ 265 = 285 - n (D Ç P ) Þ n (D Ç P ) = 20 1. Given that, A = {(x, y) : y = e-x }
\ Persons who like only dancing and painting and B = {(x, y) : y = - x}
= n (D Ç P ) - n (D Ç P Ç S ) = 20 - 10 = 10 Now, drawing both curves
19. According to the given information y=e–x Y
number of distinct elements in y=–x
50
50 ´ 10
U Xi = 20 = 25
i=1
X¢ X
n
n ´5
and number of distinct elements in U Yi =
i=1 6
50 n
Q U Xi = U Yi = T [given]

i=1 i=1
n ´5 Since, both graphs do not intersect each other at any
Þ 25 = Þ n = 30
6 point.
20. Let M and B denote the number of students taking \ AÇB=f [disjoint]
Mathematics and Biology. 2. It is given that A1 Ì A2 Ì A3 K Ì A100
Then, n (M ) = 45, n (M Ç B) = 10, n (M È B) = 64 100
\ n (B) = n (M È B) - n (M ) + n (M Ç B) Q I Ai = A Þ A3 = A
i =3
= 64 - 45 + 10 = 29
Þ n (only B) = n (B) - n (M Ç B) = 29 - 10 = 19 Þ n ( A ) = n ( A3 ) = 3 + 2 = 5
21. Given, n (D ) = 30, n (B) = 40 3. Case I x £ - 4
As, n (D Ç B) = n (D ) + n (B) - n (D È B) (- x - 3) (- x - 4) = 6
= 40 + 30 - n (D È B) Þ (x + 3) (x + 4) = 6
= 70 - n (D È B) Þ x2 + 7 x + 6 = 0
As total number of people is 50, therefore maximum Þ x = - 1 or - 6
value of n (D È B) is 50. but x£ -4
Hence, minimum value of n (D Ç B) = 70 - 50 = 20 x= -6
Sets 15

Case II x Î (- 4, 0) 7. Given, A = {(x, y) : y = e2x , x Î R}


(- x - 3) (x + 4) = 6 Þ A is the set of all points on the graph of y = e2x
Þ - x2 - 7x - 12 - 6 = 0 and B = {(x, y) : y = e-2x , x Î R}
Þ x2 + 7x + 18 = 0 Þ B is the set of all points on the graph of y = e-2x
Now, we plot the graph of given sets
D < 0 Þ No solution
Case III x ³ 0 y = e–2x
Y y = e2x
(x - 3) (x + 4) = 6
Þ x2 + x - 12 - 6 = 0
Þ x2 + x - 18 = 0 X¢
O
X
- 1 ± 1 + 72
x=
2 Y¢
73 - 1
\ x= only Since, the graph of y = e and y = e-2x intersect at one
2x
2
point.
4. Clearly, A is set of all points on the circle x2 + y2 = 25 4
8. Given, A = {(x, y) : y = , 0 ¹ x Î R } Þ A is the set of all
and B is set of all points on ellipse x2 + 9 y2 = 144. x
These two intersect at four points P , Q , R and S. points on the graph of xy = 4
and B = {(x, y) : y = x, x > 0, x Î R}
Hence, A Ç B contains four points
Þ B is the set of all points on the graph of y = x, x > 0
Now, we plot the graph of given sets.
(0, 5)
x2+y2=25 Y
(0, 4) y=x
Q P
xy = 4
X¢ X
(–12, 0) (– 5, 0) (5, 0) (12, 0)

R S
(0, –4)
x +9y2=144
2 Y¢
(0, –5)
Since, the graph of xy = 4 and y = x, x > 0 intersect at one
point.
5. We have, B1 = A1 Þ B1 Ì A1 \ A Ç B ¹ f and A Ç B is a singleton set.
B2 = A2 - A1 Þ B2 Ì A2 9. According to the question, we have the following Venn
diagram.
B3 = A3 - ( A1 È A2) Þ B3 Ì A3
Here, A Ç B Í C and A Ç B ¹ f
Q B1 È B2 È B3 Ì A1 È A2 È A3
C
Þ A1 È A2 È A3 É B1 È B2 È B3
A B
6. We have,
1
cos x > - and 0 £ x £ p
2
2p
Þ 0£x£ and 0 £ x £ p A ÇB
3
2p ì 2p ü
Þ 0£x£ Þ A = íx : 0 £ x £ ý Now, from the Venn diagram, it is clear that
3 î 3þ
1 p B Ç C ¹ f, is true
Also, sin x > and £ x £ p Also, (C È A ) Ç (C È B) = C È ( A Ç B) = C is true.
2 3
p 5p ì p 5p ü If ( A - B) Í C, for this statement the Venn diagram is
Þ £x£ Þ B = íx : £ x £ ý
3 6 î 3 6þ
A B
ì p 2p ü
Q A Ç B = íx : £ x £ ý
î 3 3þ
ì 5p ü
and A È B = íx : 0 £ x £ ý C
î 6þ
16 JEE Main Mathematics

From the Venn diagram, it is clear that 11. Given X = { n Î N : 1 £ n £ 50}, then
if A - B Í C, then A Í C. A = { n Î X : n is multiple of 2}
Now, if ( A - C ) Í B, for this statement the Venn = {2, 4, 6, 8, ...... , 50}
diagram.
and B = { n Î X : n = is multiple of 7
A {7, 14, 21, 28, 35, 42, 49} Q 14, 28, 42 Î A
B \ Smallest subset of X containing elements of both A
and B have elements = n ( A ) + n (B) - n ( A Ç B)
C = 25 + 7 - 3 = 29

From the Venn diagram, it is clear that 12. If elements are not repeated, then number of elements
A Ç B ¹ f, A Ç B Í C and A – C = f Í B but A Í B. in A1 È A2 È A3 È . . . È A30 is 30 ´ 5. But each element is
used 10 times, so
10.
30 ´ 5
S= = 15 …(i)
A B 10
Similarly, if elements in B1 , B2, K , Bn are not repeated,
then total number of elements is 3n but each element is
C
repeated 9 times, so
3n
S=
Let A be the set of even numbered students then 9
é 140 ù 3n
n( A) = ê = 70 Þ 15 = [from Eq. (i)]
ë 2 úû 9
[[ × ] denotes greatest integer function] Þ n = 45
Let B be the set of those students whose number is
13. Given, Set S = {1, 2, 3, K 50}.
é 140 ù
divisible by 3, then n (B) = ê = 46 Total number of non-empty subset of ‘S’ = 250 - 1
ë 3 úû
Now, number of non-empty subset of ‘S’ in which only
[[ × ] denotes greatest integer function] odd number. {1, 3, 5, K 49} occurs = 225 - 1
Let C be the set of those students whose number is
So, the required number of non-empty subsets of ‘S ’
divisible by 5, then
such that product of elements is even.
é 140 ù
n (C ) = ê = 28 (250 - 1) - (225 - 1) = 250 - 1 - 225 + 1
ë 5 úû
= 250 - 225 = 225 (225 - 1)
[[ × ] denotes greatest integer function]
Here, m = n = 25
é 140 ù
Now, n ( A Ç B) = ê = 23 Q m + n = 25 + 25 = 50
ë 6 úû
[numbers divisible by both 2 and 3] 14. A È B, A - B, B - A
é 140 ù ( A È B) - ( A - B) = B
n (B Ç C ) = ê =9
ë 15 úû ( A È B) - (B - A ) = A
[numbers divisible by both 3 and 5] A D B = ( A - B) È (B - A )
é 140 ù ( A È B) - ( A D B) = A Ç B
n (C Ç A ) = ê = 14
ë 10 úû and ( A - B) - A = f
[numbers divisible by both 2 and 5] Thus, the required number is 8.
é 140 ù 15. Let the total number of worker be 100.
n (A Ç B ÇC ) = ê =4
ë 30 úû A, the set of workers who like oranges and B, the set of
[numbers divisible by 2, 3 and 5] workers who likes apples.
and n ( A È B È C ) = Sn ( A ) - Sn ( A Ç B) + n ( A Ç B Ç C ) So, n ( A ) = 70, n (B) = 64, n ( A Ç B) = x
= (70 + 46 + 28 ) - (23 + 9 + 14) + 4 Also, n ( A È B) £ 100
= 102 Þ n ( A ) + n (B) - n ( A Ç B) £ 100
\Number of students who did not opt any of the Þ 70 + 64 - x £ 100
three courses Þ x ³ 134 - 100
= Total students n ( A È B È C ) Þ x ³ 34
= 140 - 102 = 38 Q The minimum value of x is 34.
02
Fundamentals of
Relations and Functions
In Mathematics we come across many relations such as number m is less than IN THIS CHAPTER ....
number n, line l is parallel to line m. In all these, a relation involves pairs of
Ordered Pair
objects in certain order. A special type of relation called function. Concept of
function plays very important role in Mathematics, since it captures the idea Cartesian Product
of a mathematically precise correspondence between one quantity with the Relations
other.
Different Types of Relations
Inverse Relation
Ordered Pair Composition of Relations
Two elements a and b listed in a specific order form an ordered pair, denoted
Functions or Mappings
by ( a , b). In an ordered pair ( a , b); a is regarded as the first element and b is
the second element. Classification of Functions
It is evident from the definition that Algebra of Real Functions
(i) ( a , b) ¹ ( b, a ) Inverse Function
(ii) ( a , b) = ( c, d ) iff a = c, b = d Composition of Functions

Equality of Ordered Pair


Two ordered pairs ( a1 , b1 ) and ( a2 , b2 ) are equal iff
a1 = a2 and b1 = b2
i.e., ( a1 , b1 ) = ( a2 , b2 )
Þ a1 = a2 and b1 = b2
Thus, it is evident from the definition that (1, 2) ¹ (2, 1) and (1, 1) ¹ (2, 2).

Cartesian Product
Let A and B be two non-empty sets. The cartesian product of A and B is
denoted by A ´ B and is defined as the set of all ordered pairs ( a , b), where
a Î A and b Î B.
Symbolically, A ´ B = {( a , b) : a Î A and b Î B}
18 JEE Main Mathematics

Important Points Example 1. Let Z be the set of integers. If


2
● If A ¹ B, then A ´ B ¹ B ´ A A = {x Î Z : 2( x+ 2 )( x - 5x + 6 )
= 1} and B = {x Î Z : -3 < 2 x - 1 < 9},
● If A has p elements and B has q elements, then A ´ B then the number of subsets of the set A ´ B, is (JEE Main 2019)
has pq elements. 12 18 15
(a) 2 (b) 2 (c) 2 (d) 210
● If A = f or B = f, then A ´ B = f . 2
Sol. (c) Given, set A = { x Î Z : 2( x+ 2)( x -5 x + 6 )
= 1}
● Cartesian product of n sets A1 , A2 , A3 , . . . , An is the set 2
( x + 2)( x -5 x + 6 )
of all n-tuples ( a1 , a2 , a3 , . . , an ), ai Î Ai , i = 1, 2, 3, . . . , n Consider, 2 = 1 = 2º
n
Þ ( x + 2) ( x - 3) ( x - 2) = 0
and it is denoted by A1 ´ A2 ´ . . . ´ An or Õ Ai .
i =1 Þ x = -2, 2, 3
Þ A = {-2, 2, 3}
Properties of Cartesian Product Also, we have set B = { x Î Z : - 3 < 2x - 1 < 9}
If A, B and C are three sets, then Consider, -3 < 2x - 1 < 9, x Î Z
(i) (a) A ´ ( B È C ) = ( A ´ B) È ( A ´ C ) Þ -2 < 2x < 10 , x Î Z Þ -1 < x < 5, x Î Z
(b) A ´ ( B Ç C ) = ( A ´ B) Ç ( A ´ C ) Þ B = {0 , 1, 2, 3, 4}
(ii) A ´ ( B - C ) = ( A ´ B) - ( A ´ C ) So, A ´ B has 15 elements.
(iii) A ´ B = B ´ A Û A = B \ Number of subsets of A ´ B = 215 .
(iv) If A Í B Þ A ´ A Í ( A ´ B) Ç ( B ´ A) [Q if n( A) = m, the number of possible subsets = 2 m ]
(v) If A Í B Þ A ´ C Í B ´ C
(vi) If A Í B and C Í D Þ A ´ C Í B ´ D Inverse Relation
(vii) ( A ´ B) Ç (C ´ D ) = ( A Ç C ) ´ ( B Ç D ) If R is a relation on set A , then the relation R -1 on A
(viii) A ´ ( B¢È C ¢ )¢ = ( A ´ B) Ç ( A ´ C ) defined by R -1 = {( b, a ) : ( a , b) Î R } is called an inverse
(ix) A ´ ( B¢ Ç C ¢ )¢ = ( A ´ B) È ( A ´ C ) relation to A.
(x) If A and B have n common elements, then A ´ B and Clearly, domain ( R -1 ) = range ( R ); range ( R -1 ) = domain ( R )
B ´ A will have n 2 common elements.
e.g. Let A = {1, 2, 3} and let R = {(1, 2), (2, 2), (3, 1), (3, 2)}
Then, R being a subset of A ´ A, it is a relation on A.
Relations Clearly, 1R 2 ; 2R 2 ; 3R1 and 3R 2.
Let A and B be two non-empty sets. Then, a relation R
Domain ( R ) = {1, 2, 3} and range ( R ) ={2, 1}
from A to B is a subset of A ´ B.
Also, R -1 = {(2, 1), (2, 2), (1, 3), (2, 3)}
Thus, R is a relation from A to B Þ R Í A ´ B. If R is a
relation from a non-empty set A to a non-empty set B Domain ( R -1 ) ={2, 1} and range ( R -1 ) ={1, 2, 3}
and if ( a , b) Î R, then we write aRb which is read as ‘a
is related to b by the relation R.’ Example 2. If R = {( x, y) : x, y Î Z , x 2 + 3y 2 £ 8} is a relation
If ( a , b) Ï R, then we write aRb
/ and it is read as ‘a is not on the set of integers Z, then the domain of R-1 is
related to b by the relation R ’. (JEE Main 2020)

Let A and B be two non-empty finite sets having p and q (a) {-1, 0 ,1} (b) {- 2, - 1,1, 2} (c) {-2, - 1, 0 ,1, 2} (d) {0, 1}
2 2
elements respectively, then total number of relations Sol. (a) Given relation, R = {( x, y) : x, y Î Z , x + 3y £ 8}
form A to B = 2 pq. For, y 2 = 0 , x2 = 0 , 1, 4 For, y 2 = 1, x2 = 0 , 1, 4
For, y 2 = 4, x2 Î f
Domain and Range of Relations
\ Range of R is possible values of y = {- 1, 0 ,1}
Let R be a relation from A to B. The domain of R is the
set of all those elements a Î A such that ( a , b) Î R for \ Domain of R -1 = Range of R = {- 1, 0 , 1}
some b Î B.
\ Domain of R = { a Î A : ( a , b) Î R , "b Î B} Composition of Relations
and range of R is the set of all those elements b Î B such Let R Í A ´ B, S Í B ´ C be two relations. Then,
that ( a , b) Î R for some a Î A. composition of the relations R and S denoted by
\ Range of R = { b Î B : ( a , b) Î R , "a Î A}. SoR Í A ´ C and is defined by ( a , c) Î SoR , iff $b Î B s.t.
Here, B is called the codomain of R. ( a , b) Î R , ( b, c) Î S
e.g. Let A = {1, 2, 3} and B = {3, 5, 6} e.g. Let A = {1, 2, 3}, B = { a , b, c, d }, C = { a , b , g }
Let aRb Þ a < b R Í ( A ´ B) = {(1, a ), (1, c), ( 2 , d )}
Then, R = {(1, 5), (2, 5), (3, 5), (1, 6), (2, 6), (3, 6)} S Í ( B ´ C ) = {( a , a ), ( c, g ), ( d , b )}
\ Domain of R = {1, 2, 3}, range of R = {5,6} Then, SoR Í ( A ´ C ) = {(1, a ), (1, g ), ( 2 , b )}
and codomain of R = {3, 5, 6}
Fundamentals of Relations and Functions 19

Note Example 4. The domain of the definition of the


• One should be careful in computing the relation RoS. Actually, SoR starts 1
with R and RoS starts with S. function f ( x) = + log10( x3 - x) is
4 - x2
• In general, SoR ¹RoS (JEE Main 2019)
• (SoR )-1 = R -1oS -1, known as reversal rule. (a) ( -1, 0) È (1, 2) È (3, ¥) (b) ( -2, - 1) È ( -1, 0) È (2, ¥)
(c) ( -1, 0) È (1, 2) È (2, ¥) (d) (1, 2) È (2, ¥)
Example 3. If the relation R : A ® B, where A = {1, 2, 3, 4} and
1
B = {1, 3, 5} is defined by R = {( x, y) : x < y, x Î A, y Î B}, then RoR-1 is Sol. (c) Given function f ( x) = + log10( x3 - x)
4 - x2
(a) {(1, 3), (1, 5), (2, 3), (2, 5), (3, 5), (4, 5)}
For domain of f ( x)
(b) {(3, 1), (5, 1), (5, 2), (5, 3), (5, 4)}
4 - x2 ¹ 0 Þ x ¹ ± 2 …(i)
(c) {(3, 3), (3, 5), (5, 3), (5, 5)}
3
(d) None of the above and x - x > 0 Þ x( x - 1)( x + 1) > 0
From Wavy curve method,
Sol. (c) We have, R = {( x, y) : x < y , x Î A, y Î B}
Q R = {(1, 3), (1, 5), (2, 3), (2, 5), (3, 5), ( 4, 5)} + +
and R -1 = {(3, 1), (5, 1), (3, 2), (5, 2), (5, 3), (5, 4)} –¥ – +¥
Thus, RoR -1 = {(3, 3), (3, 5), (5, 3), (5, 5)}. –1 0 – +1

Functions or Mappigs From Eqs. (i) and (ii), we get the domain of f ( x) as
Let A and B be two non-empty sets. Then, a function f from set ( -1, 0) È (1, 2) È (2, ¥).
A to B is a rule which associates elements of set A to elements
of B such that all elements of set A are associated to elements Classification of Functions
of set B in unique way.
If f associates x Î A to y Î B, then we say that y is the image of
Constant Function
Y
the element x and denote it by f ( x ) and write as y = f ( x ). The Constant function is a
element x is called the pre-image or inverse image in B. function in which it doesn’t (0, 2) y=2
f matter what is the input
A function is denoted by f : A ® B or A ¾® B. value (domain) because X¢ X
the output value (range) O
Note will be same.
• There may exist some elements in set B which are not the images of any Y¢
OR
element in set A .
• To each and every independent element in A there corresponds one and Let k be a constant, then function f (x) = k, " x Î R
only one image in B. is known as constant function.
• Every function is a relation but every relation may or may not be a function. Domain of f ( x ) = R and Range of f ( x ) = { k}
• The number of functions from a finite set A into finite set B = [n(B)][n( A )]
Polynomial Function
Domain, Codomain and Range of a Function The function y = f ( x ) = a0 x n + a1x n - 1 +......+ an ,
Let f : A ® B, then A is known as domain of f while B is known
where a0 , a1 , a2 , ..... , an are real coefficients and n
as codomain of f.
is a non-negative integer, is known as a polynomial
Also, set f ( A) = { f ( x ) : x Î A} is known as range of f.
function. If a0 =/ 0, then degree of polynomial
Clearly, f ( A) Í B
function is n.
e.g., Let A = {1, –1, 2, –2}, B = {1, 4, 9}
Domain of f ( x ) = R, range varies from function to
x2 2
f : A ¾® B i.e. f ( x ) = x function.

1 1 Rational Function
–1
2
4 If P ( x ) and Q( x ) are polynomial functions, Q( x ) ¹ 0,
–2 9 P( x)
then function f ( x ) = is known as rational
A B Q( x )
From the figure, it is clear that domain of function function.
= { 1, - 1, 2 , - 2} Domain of f ( x ) = R - { x : Q( x ) = 0 }
and range of function = { 1, 4} . and range varies from function to function.
Also, codomain of function = { 1, 4, 9} .
20 JEE Main Mathematics

Irrational Function Modulus Function


The function containing one or more terms having Function y = f ( x ) =|x|is known as modulus function.
non-integral rational powers of x are called irrational ì x, x ³ 0
function. y = f ( x ) =|x|= í
î- x, x < 0
5x3/ 2 - 7x1/ 2
e.g. y = f(x) = Domain of f ( x ) = x Î R
x1/ 2 - 1
and Range of f ( x) = [0, ¥ )
Domain = varies from function to function. Y

Identity Function y=–x


y=x

Function f ( x ) = x , " x Î R is known as identity function. if x ³ 0


if x < 0
It is straight line passing through Y
y=x X¢ X
origin and having slope unity. O
Domain of f ( x ) = R and Range of X¢
O
X Y¢
f(x) = R
Y¢ Properties of Modulus Function
Square Root Function (i) For any real number x, x 2 =|x|.
The function that associates every Y
y = Öx (ii) If a and b are positive real numbers, then
positive real number x to + x is
(a) x 2 £ a 2 Û|x| £ a Û - a £ x £ a
called the square root function, i.e.,
(b) x 2 ³ a 2 Û|x|³ a Û x £ - a or x ³ a
f(x) = + x. X¢
O
X (c) a 2 £ x 2 £ b2 Û a £|x| £ b Û x Î [- b, - a ] È [a , b ]
Range of f ( x ) = [0, ¥ ) . Y¢ (iii)|x ± y| £ |x|+| y|
(iv)|x + y| ³||x|-| y||
Exponential Function
A function of the form f ( x ) = a x , a is a positive real Greatest Integer Function
number, is an exponential function. The value of the For any real number x, the greatest integer function [x ] is
function depends upon the value of a for 0 < a < 1 , equal to greatest integer less than or equal to x.
function is decreasing and for a > 1 , function is
increasing. In general, if n is an integer and x is any number
satisfying n £ x < n + 1 , then [x ] = n. It is also known as
Domain of f ( x ) = R integral part function.
and Range of f ( x ) = [0, ¥ ) e.g. If 2 £ x < 3, then [x ] = 2
y = ax Y Y y = ax Domain = R
(0, 1)
(0, 1) Range = I
X¢ X X¢ X
O O Y
3
Y¢ Y¢
0<a<1 a>1 2
(a) (b) 1
X¢ X
–3 –2 –1 0 1 2 3 4
Logarithmic Function –1
Function f ( x ) = loga x ,( x , a > 0) and a =/ 1, is known as –2
logarithmic function. –3
Domain of f ( x ) = ( 0, ¥ ) Y¢
and Range of f ( x ) = R
Y Y Properties of Greatest Integer Function
y = logax, a > 1 y = logax, 0 < a < 1 If n is an integer and x is any real number between n and
n + 1, then
X¢ X X¢ X
O O (1, 0)
(1, 0) (i) [- n ] = - [n ]
Y¢ Y¢ (ii) [x + n ] = [x ] + n
(a) (b)
(iii) [- x ] = - [x ] - 1, x is not an integer.
Fundamentals of Relations and Functions 21

(iv) [x + y ] ³ [x ] + [ y ] (iv) Multiplication of two real functions


(v) [x ] > n Þ x ³ n + 1 fg( x ) = f ( x ) g( x )
(vi) [x ] < n Þ x < n (v) Quotient of two real functions
(vii) [x + y ] = [x ] + [ y + x - [x ]], for all x , y Î R æfö f(x)
ç ÷ (x) = , g( x ) ¹ 0
é 1ù é 2ù é n - 1ù è gø g( x )
(viii) [x ] + ê x + ú + ê x + ú + . . . + ê x + = [nx ],
ë nû ë nû ë n úû
1 æ px ö
n ÎN Example 6. If f ( x) = - tan ç ÷ , - 1 < x < 1 and
2 è2 ø
Signum Function g ( x) = 3 + 4x - 4x 2 , then domain ( f + g ) is given by
The function defined by é1 ù é1 ö é 1 ö é 1 ù
ì -1 , x<0 (a) ê , 1ú (b) ê , - 1÷ (c) ê - , 1÷ (d) ê - , - 1ú
ë2 û ë2 ø ë 2 ø ë 2 û
|x| ï
f(x) = = í 0, x=0
x ï 1,
1 æ p xö
x>0 Sol. (c) Given, f ( x) = - tan ç ÷ , where - 1 < x < 1
î 2 è 2 ø
is called the signum function. Given, domain of f ( x) is d1 = ( -1, 1)
Y
For domain of g ( x), 3 + 4x - 4x2 ³ 0 Þ (2 x - 3)(2 x + 1) £ 0
y=1
1 é 1 3ù
x>0 \ Domain of g ( x) is d 2 = ê - , ú
ë 2 2û
X¢ X é 1 ö
O Hence, domain of ( f + g ) = d1 Ç d 2 = ê - , 1÷
ë 2 ø
y = –1
–1
x<0 Properties of Composition of Function
Y¢ (i) The composition of functions is not commutative.
Domain = R i.e. fog ¹ gof
Range = { -1 , 0 , 1} (ii) The composition of functions is associative.
i.e. fo( goh ) = ( fog)oh
Example 5. The domain of the function
(iii) The composition of any function with the identity
1 function is the function itself.
f ( x) = is
2
[ x] - [ x] - 6 i.e. If f : A ® B , then foI A = I B of = f .
(a) ( -¥ , - 2) È [ 4, ¥) (b) ( -¥ , - 2] È [ 4, ¥) Example 7. If f ( x) = ( ax 2 + b)3, then the function g such
(c) ( -¥ , - 2) È ( 4, ¥) (d) None of these
that f {g ( x)} = g {f ( x)} is given by
Sol. (a) f ( x ) is defined, if [ x]2 - [ x] - 6 > 0 æ b - x1/3 ö
1/ 2
1
(a) g ( x ) = ç ÷ (b) g ( x ) =
Þ ([ x] - 3)([ x] + 2) > 0 è a ø ( ax + b)3
2

Þ [ x] < - 2 or [ x] > 3 1/ 2
æ x1/3 - b ö
But [ x] < - 2 Þ [ x] = - 3, - 4, - 5, . . . (c) g ( x ) = ( ax2 + b)1/3 (d) g ( x ) = ç ÷
\ x < -2 è a ø
1/ 2
Also, [ x] > 3 Þ [ x] = 4, 5, 6, . . . æ x1/3 - b ö
\ x³4 Sol. (d) Let g ( x ) = ç ÷
è a ø
Domain of f = ( -¥ , - 2) È [ 4, ¥) 1/ 2
æ x1/3 - b ö
\ f {g ( x )} = f ç ÷
Algebra of Real Functions è a ø
3
Let f : X ® R and g : X ® R be any two real functions, é æ x1/3 - b ö ù
= êa ç ÷+ bú = ( x1/3)3 = x
where X Ì R. êë è a ø úû
(i) Addition of two real functions and g {f ( x )} = g {( ax2 + b)3}
( f + g) ( x ) = f ( x ) + g( x ) 1/ 2
æ ax2 + b - b ö
(ii) Subtraction of two real functions =ç ÷ = ( x2)1/ 2 = x Þ g {f ( x )} = f {g ( x)}
è a ø
( f - g) ( x ) = f ( x ) - g( x ) 1/ 2
(iii) Multiplication by a scalar æ x1/3 - b ö
Thus, g (x) = ç ÷
(a f ) ( x ) = a f ( x ), a , x Î R è a ø
Practice Exercise
ROUND I Topically Divided Problems
Cartesian Product 8. The relation R defined on set A = { x : x < 3, x Î I }
1. If two sets A and B are having 99 elements in by R = {( x, y) : y = x } is
common, then the number of elements common to (a) {(- 2 , 2), (- 1, 1), (0, 0), (1, 1), (2 , 2)}
each of the sets A ´ B and B ´ A are (b) {(- 2 , - 2), (- 2 , 2), (- 1, 1), (0, 0), (1, - 2), (1, 2),
(a) 299 (b) 99 2 (c) 100 (d) 18 (2 , - 1), (2 , - 2)}
(c) {(0, 0), (1, 1), (2 , 2)}
2. Consider the following statements
I. If A Ç B = f, then either A = f or B = f. (d) None of the above
II. For a ¹ b, { a , b} = { b, a } and ( a , b) ¹ ( b, a). 9. Let R be the relation from A = { 2 , 3, 4, 5 } to
III. If A Í B, then A ´ A Í ( A ´ B) Ç ( B ´ A). B = {3, 6, 7, 10 } defined by ‘x divides y’, then R -1 is
IV. If A Í B and C Í D, then A ´ C Í B ´ D. equal to
Which of these is/are correct? (a) {(6, 2), (3, 3)}
(a) Only (II) (b) Only (I) (b) {(6, 2), (10, 2), (3, 3), (6, 3), (10, 5)}
(c) Only (IV) (d) (II), (III) and (IV) (c) {(6, 2), (10, 2), (3, 3), (6, 3)}
(d) None of the above
3. If A = { x : x 2 - 5 x + 6 = 0 }, B = { 2, 4 }, C = { 4, 5 }, then
10. R is a relation from {11, 12 , 13} to {8, 10, 12 } defined
A ´ ( B Ç C) is
by y = x - 3. The relation R -1 is
(a) {(2, 4), (3, 4)}
(a) {(11, 8), (13, 10)}
(b) {(4, 2), (4, 3)}
(b) {(8, 11), (10, 13)}
(c) {(2, 4), (3, 4), (4, 4)}
(c) {(8, 11), (9, 12), (10, 13)}
(d) {(2, 2), (3, 3), (4, 4), (5, 5)}
(d) None of the above
Relations 11. If R is a relation from a set A to the set B and S is a
4. Let A = {1, 2 , 3}. The total number of distinct relation from B to C, then the relation SoR
relations that can be defined over A, is (a) is from C to A (b) is from A to C
(a) 29 (b) 6 (c) does not exist (d) None of these
(c) 8 (d) None of these
12. Let a relation R be defined by
5. Let n( A) = m and n( B) = n. Then, the total number R = {( 4, 5), (1, 4), ( 4, 6), (7, 6), (3, 7)}. The relation
of non-empty relations that can be defined from A
R -1oR is given by
to B is
(a) {(1, 1), (4, 4), (7, 4), (4, 7), (7, 7)}
(a) mn (b) nm - 1
(c) mn - 1 (d) 2mn - 1 (b) {(1, 1), (4, 4), (4, 7), (7, 4), (7, 7), (3, 3)}
(c) {(1, 5), (1, 6), (3, 6)}
6. If R = {( x, y) : x, y Î I , x 2 + y 2 £ 4 } is a relation in I, (d) None of the above
then domain of R is
(a) {0, 1, 2} (b) { - 2 , - 1, 0} Functions
(c) { - 2 , - 1, 0, 1, 2} (d) None of these 13. If f ( x) = ax + b, where a and b are integers,
7. Let R be a relation on N defined by x + 2 y = 8. The f ( -1) = - 5 and f (3) = 3, then a and b are equal to
domain of R is (a) a = - 3 , b = - 1
(b) a = 2 , b = - 3
(a) {2, 4, 8} (b) {2, 4, 6, 8}
(c) a = 0 , b = 2
(c) {2, 4, 6} (d) {1, 2, 3, 4} (d) a = 2 , b = 3
Fundamentals of Relations and Functions 23

14. The domain of definition of the function 22. If the functions are defined as f ( x) = x and
æ5x - x ö 2
g ( x) = 1 - x , then what is the common domain of
f ( x) = log10 ç ÷ is
è 4 ø the following functions
f + g, f - g, f / g, g / f , g - f
(a) [1, 4] (b) [1, 0] (c) [0, 5] (d) [5, 0]
f ( x)
2 1 where ( f ± g)( x) = f ( x) ± g ( x), ( f / g)( x) =
15. The range of the function f ( x) = x + is g ( x)
x2 + 1
(JEE Main 2021)
(a) [1, ¥ ) (b) [2, ¥ ) (a) 0 £ x £ 1 (b) 0 £ x < 1
é3 ö (c) 0 < x < 1 (d) 0 < x £ 1
(c) ê , ¥ ÷ (d) None of these
ë 2 ø 1- x
23. Let f : [0, 1] ® [0, 1] defined by f ( x) = ,0 £ x £1
16. If f : R ® R is defined by f ( x) = [ 2 x ] - 2[ x ], " x Î R, 1+ x

where [ x ] is the greatest integer not exceeding x, and let g : [0, 1] ® [0, 1] be defined by
then the range of f is g ( x) = 4 x (1 - x), 0 £ x £ 1, then fog and gof is
(a) { x Î R : 0 £ x £ 1} (b) {0, 1} (2x - 1) 2 8x (1 - x) (2x - 1 ) 8 (1 - x )x
(a) , (b) ,
(c) { x Î R : x > 0} (d) { x Î R : x £ 0} 1 + 4x - 4x 2 (1 + x) 2 1 + 4x - 4x 2 (1 + x) 2
17. Let f : (1, 3) ® R be a function defined by (2x + 1 ) 2 8 (2x + 1 ) 2 8 (1 - x )
(c) , (d) ,
x[ x ] 1 + 4x + 4x 2 (1 + x) 2 (1 + x ) 2 (1 + x ) 2
f ( x) = , where [ x ] denotes the greatest integer
1 + x2 24. If f : R ® R, g : R ® R and h : R ® R are such that
£ x. Then, the range of f is (JEE Main 2020) f ( x) = x 2 , g ( x) = tan x and h( x) = log x, then the
æ2 3ù æ3 4ö æ2 4ù
(a) ç , ú È ç , ÷ (b) ç , ú p
è5 5û è4 5ø è5 5û value of ( ho ( gof ))( x), if x = will be
4
æ3 4ö æ2 1ö æ3 4ù
(c) ç , ÷ (d) ç , ÷ È ç , ú (a) 0 (b) 1 (c) –1 (d) p
è5 5ø è5 2ø è5 5û
2
x 25. If [ x ] - 5[ x ] + 6 = 0, where [ × ] denote the greatest
18. Let f : R ® R be defined by f ( x) = ,
1 + x2 integer function, then
x Î R . Then, the range of f is (JEE Main 2019) (a) x Î [3, 4] (b) x Î (2, 3]
é 1 1ù (c) x Î [2 , 3] (d) x Î [2, 4)
(a) ê - , ú (b) (-1, 1) - {0}
ë 2 2û 26. If f ( x) = cos (log x), then
é 1 1ù é æxö ù
(c) R - ê - , ú (d) R - [-1, 1] 1
ë 2 2û f ( x) f ( y) - ê f ç ÷ + f ( xy) ú has the value
2 ë è yø û
19. The domain of the real valued function 1
2 2 (a) -1 (b) (c) –2 (d) 0
f ( x) = 5 - 4 x - x + x log ( x + 4) is 2
(a) -5 £ x£1 Numerical Types Questions
(b) - 5 £ x and x ³ 1
27. Let A and B two sets containing 2 elements and 4
(c) -4 < x£1
elements respectively. The number of subsets of
(d) f
A ´ B having 3 or more elements is ............ .
20. The domain of the function
28. If two set A and B having 99 elements in common,
f ( x) = exp ( 5 x - 3 - 2 x 2 ) is
then the number of elements common to each of the
(a) [3 / 2, ¥ ] (b) [1, 3 / 2] sets A ´ B and B ´ A is a b, then a + b is equal to
(c) (- ¥ , 1] (d) (1, 3 / 2) ............ .
21. The domain of the function f defined by 29. The relations on the set A = { x = |x|< 3, x Î Z } is
1 defined by R = {( x, y) = y = |x|, x ¹ - 1}.
f ( x) = 4 - x + is equal to
x2 - 1 Then the number of elements in the power set of R
(a) (-¥ , - 1) È (1, 4] is ............ .
(b) (-¥ , - 1] È (1, 4] 30. If A and B are two sets such that n ( A Ç B ) = 9,
(c) (-¥ , - 1) È [1, 4] n ( A Ç B) = 10 and n ( A È B) = 24, then the value of
(d) (-¥ , - 1) È [1, 4) n ( A ´ B), is equal to ............ .
24 JEE Main Mathematics

Answers
Round I
1. (b) 2. (d) 3. (a) 4. (a) 5. (d) 6. (c) 7. (c) 8. (a) 9. (b) 10. (b)
11. (b) 12. (b) 13. (b) 14. (a) 15. (a) 16. (b) 17. (d) 18. (a) 19. (c) 20. (b)
21. (a) 22. (c) 23. (a) 24. (a) 25. (d) 26. (d) 27. (219) 28. (101) 29. (16) 30. (210)

Solutions
Round I 10. Let A = { 11, 12, 13 }, B = { 8, 10, 12 }
1. n [( A ´ B ) Ç (B ´ A )] = n [( A Ç B ) ´ (B Ç A )] \ R = {(11, 8), (13, 10)}
= n ( A Ç B ) ´ n (B Ç A ) R-1 = {(8, 11), (10, 13)}
= 99 ´ 99 = 992 11. Given, R Í A ´ B and S Í B ´ C , we have
2. Statement I If A Ç B = f, then it is not necessary that SoR Í A ´ C
A = f or B = f. \ SoR is a relation from A to C .
Statement II It is true { a , b} = { b, a } and (a , b) ¹ (b, a )
12. Given, R = {(4, 5), (1, 4), (4, 6), (7, 6), (3, 7)}
Statement III By properties of cartesian product
\ R-1 = {(5, 4), (4, 1), (6, 4), (6, 7), (7, 3)}
If A Í B, then A ´ A Í ( A ´ B ) Ç (B ´ A )
Þ R-1oR = {(4, 4), (1, 1), (4, 7), (7, 4), (7, 7), (3, 3)}
Statement IV Also A Í B and C Í D, then
A ´ C Í B ´ D by properties of cartesian product. 13. Given, f (x ) = ax + b
3. Given, A = { x : x2 - 5 x + 6 = 0 } Also, f (- 1 ) = - 5 and f (3) = 3
\ -5 = - a + b and 3 = 3a + b
= { x : (x - 2)(x - 3) = 0} = { 2, 3} On solving, we get
and B = { 2, 4 } and C = { 4, 5 } a = 2, b = - 3
Now, B Ç C = { 2, 4} Ç { 4, 5 } = { 4 } 14. For f (x) to be defined,
\ A ´ (B Ç C ) = { 2, 3 } ´ {4 } = {(2, 4), (3, 4)} 5 x - x2
³ 1 Þ x2 - 5x + 4 £ 0 Þ (x - 4) (x - 1) £ 0
4. Q n ( A ´ A ) = 32 = 9. So, the total number of subsets of 4
A ´ A is 29 and a subset of A ´ A is a relation over the \ x Î [1, 4]
set A. 1 æ x2 ö
15. Given, f (x) = x2 + = (x 2
+ 1 ) - ç 2 ÷
5. Given, n ( A ) = m and n (B ) = n x2 + 1 è x + 1ø
\Total number of relations from A to B = 2mn é x2 ù
= (1 + x2) ê1 - 2 ú ³ 1, " x Î R
\Total number of non-empty relations from A to B ë (x + 1)2 û
= 2mn - 1 Hence, the range of f (x) is [1, ¥ ).
6. Given, R = {(x, y) : x, y Î I , x2 + y2 £ 4} 16. Given, f (x) = [2x] - 2[x], " x Î R
= {(0, 0), (0, –1), (0, 1), (0, –2),…, (–2, 0)} If x is an integer, then f (x) = 0 and if x is not an
\ Domain of R = { x : (x, y) Î R} = { -2 , - 1, 0, 1, 2} integer, then f (x) is either 1 or 0.
\ Range of f (x) = {0, 1}
7. Given, R = {(x, y) : x + 2 y = 8, x, y Î N }
17. The given function f : (1, 3) ® R, defined by
8-x
x + 2y = 8 Þ y = x(1)
2 , x Î (1, 2)
x[x] 1 + x2
\ R = {(2, 3), (4, 2), (6, 1)} f (x) = =
1 + x2 x(2)
\ Domain of R = { x : (x, y) Î R} = {2 , 4, 6} , x Î [2, 3)
1 + x2
8. Given, set is A = { x : x < 3, x Î I } x
, x Î (1, 2)
A = { x : - 3 < x < 3, x I } = { - 2, - 1, 0, 1} 1 + x2
Q f (x) =
Also, R = {(x, y) : y = x } 2x
, x Î [2, 3)
\ R = {(- 2, 2), (- 1, 1 ), (1, 1 ), (0, 0), (2, 2)} 1 + x2
9. Given, A = {2, 3, 4, 5} and B = {3, 6, 7, 10} is a decreasing function, so
\ R = {(2, 6), (2, 10), (3, 3), (3, 6), (5, 10)} 2(3) 3
lim f (3 - h ) ¾® =
Þ R-1 = {(6, 2), (10, 2), (3, 3), (6, 3), (10, 5)} h® 0 1+9 5
Fundamentals of Relations and Functions 25

4 24. { ho( gof ) }x = ho{ g (x2)} = ho(tan x2) = log(tan x2)


f (2) =
5
p ì æ p ö üï
2
2 1 ï
Similarly, f (x ® 2- ) tends to and f (x®1+ ) tends to . At x = , { ho( gof ) }(x) = log í tan ç ÷ ý
5 2 4 ïî è 4ø ï
þ
æ2 1ö æ3 4ù pö
So range of the given function ‘f’ is ç , ÷ È ç , ú æ
è5 2ø è5 5û = log ç tan ÷
è 4ø
Hence, option (d) is correct. = log 1 = 0
x
18. Let y = Þ yx2 - x + y = 0 2
25. Given [x] - 5[x] + 6 = 0
1 + x2
Þ [x]2 - 3[x] - 2[x] + 6 = 0
Q x Î R, so D ³0 Þ 1 - 4 y2 ³ 0
Þ [x]([x] - 3) - 2([x] - 3) = 0
é 1 1ù
Þ (1 - 2 y) (1 + 2 y) ³ 0 Þ y Î ê - , ú Þ ([x] - 3) ([x] - 2) = 0
ë 2 2û
– + – Þ [x] = 3 or [x] = 2
–1/2 1/2 Þ x Î [3, 4) or x Î [2, 3)
é 1 1ù \ x Î [2, 4)
So, range is ê - , ú .
ë 2 2û 26. We have, f (x ) = cos (log x )
2
19. For f (x) to be define,d 5 - 4x - x ³ 0 and x + 4 > 0 é æ xö1 ù
\ f (x ) f ( y ) -
ê f çè y ÷ø + f (xy )ú
Þ - 5 £ x £ 1 and x > - 4 Þ - 4 < x £ 1 ë 2 û
5 x - 3 - 2x 2 1
20. Given, f (x) = e = cos (log x ) cos (log y) - [2 cos (log x ) cos (log y)]
2
For domain of f (x), = cos (log x ) cos (log y) - cos (log x ) cos (log y) = 0
3
2x2 - 5x + 3 £ 0 Þ (2x - 3) (x - 1) £ 0 Þ 1 £ x £ 27. A ´ B contains (2 ´ 4) = 8 elements
2
é 3ù The number of subsets of A ´ B having 3 or more
\ Domain of f (x) = ê1, ú elements = (Number of subsets A ´ B )
ë 2û
1 - (Number of subsets with at most 2 elements)
21. Given, f (x ) = 4 - x + = 28 - (1 + 8 + 28) = 256 - 37 = 219
x2 - 1
For domain of f (x),
28. We have, n [( A ´ B) Ç (B ´ A )]
4 - x ³ 0 and x2 - 1 > 0 = n [( A Ç B) ´ (B Ç A )] = n ( A Ç B) × n (B Ç A )
Þ x £ 4 and x2 > 1 = n ( A Ç B) × n ( A Ç B) = 99 ´ 99 = 992
Þ x £ 4 and x Î (- ¥ , - 1 ) È (1, ¥ ) = a = 99 and b = 2
\ x Î (- ¥ , - 1 ) È (1, 4 ] Q a + b = 99 + 2 = 101
22. f (x) + g (x) = x + 1 - x, domain [0, 1] 29. We have,
f (x) - g (x) = x - 1 - x, domain [0, 1] A = { x :|x| < 3, x Î Z } - { - 2, - 1, 0, 1, 2}
g (x) - f (x) = 1 - x - x, domain [0, 1] and R = {(x, y) = y = |x|, x ¹ - 1}
f (x)
=
x
, domain [0, 1) R = {(-2, 2) (0, 0), (1, 1), (2, 2)}
g (x) 1-x Clearly, R has four elements, so the number of
g (x) 1-x elements in set of R is 24 = 16
= , domain (0, 1]
f (x) x
30. We have,
So, common domain is (0, 1).
n ( A Ç B ) = 9, n ( A Ç B) = 10 and n ( A È B) = 10
1 - 4 x (1 - x )
23. fog = f [4x (1 - x) ] = Þ n ( A ) - n ( A Ç B) = 9, n (B) - n ( A Ç B) = 10
1 + 4x (1 - x )
1 - 4 x + 4 x2 (2x - 1 )2 and n ( A ) + n (B) - n ( A Ç B) = 24 …(i)
= =
1 + 4x - 4x 2
1 + 4x - 4x2 Þ n ( A ) + n (B) - 2n ( A Ç B) = 19 …(ii)
æ1 - xö æ1 - xö æ 1 - xö From Eqs. (i) and (ii), we get
and gof = g ç ÷ =4 ç ÷ ç1 - ÷
è 1 + xø è 1 + xø è 1 + xø n ( A Ç B) = 5, n ( A ) = 14, n (B) = 15
4(1 - x ) æ 1 + x - 1 + x ö 8x (1 - x) Hence, n ( A ´ B) = 14 ´ 15 = 210
= ç ÷=
(1 + x ) è 1+ x ø (1 + x)2
03
Sequence and
Series
In Mathematics, the word ‘sequence’ is used in the same way as it is in IN THIS CHAPTER ....
ordinary English. When we say that a collection of objects listed in a
Sequence
sequence, we usually mean that the collection is ordered in such a way that it
has an identified first member, second member, third member and so on. Series
Progression
Sequence
Arithmetic Progression (AP)
A set of numbers arranged in a definite order according to some definite rule
is called a sequence. Insertion of Arithmetic Mean
(AM) between Two Numbers
Series Geometric Progression (GP)
If a1 , a2 , . . . , an is a sequence, then the expression a1 + a2 + . . . + an + . . . is Insertion of Geometric Mean
called the series. The series is said to be finite or infinite according as the (GM) between Two Numbers
given series is finite or infinite.
Relation among Arithmetic Mean,
e.g. (i) 2 + 4 + 6 + 8 + K + 20 is a finite series. Geometric Mean
(ii) 1 + 3 + 5 + . . . is an infinite series. Arithmetico-Geometric
Progression
Progression
It is not necessary that the terms of a sequence always follow a certain To Find nth Term by Difference
pattern or they are described by some explicity formula for the nth term. Method
Those sequences whose terms follow certain patterns are called progression. Sum of n Terms of Special Series
OR Sn, Sn2 and Sn3
If the terms of a sequence are written under specific conditions, then the
sequence is called progression.
Sequence and Series 27

Sol. (c) Let the common difference of AP,


Arithmetic Progression (AP) a1, a2, a3 , ..., an is ‘d ’, the common difference of the AP,
A sequence is said to be an arithmetic progression, if the b1, b2, b3 , ..., bm is ‘d + 2 ’.
difference of a term and its previous term is always same. Q a40 = - 159 and a100 = - 399
i.e. an + 1 - an = Constant ( = d ), " n Î N Þ a1 + 39d = - 159 ...(i)
The constant difference, generally denoted by d, is called and a1 + 99d = - 399 ...(ii)
the common difference. From Eqs. (i) and (ii), we get
OR 60d = - 240 Þ d = - 4 and a1 = - 3
An arithmetic progression (AP) is a sequence whose Now, as b100 = a70
terms increase or decrease by a fixed number. This fixed Þ b1 + 99(d + 2) = a1 + 69d
number is called the common difference of the AP.
Þ b1 = - 198 - 30d - 3
In other words, if a1 , a2 , a3 , . . . , an are in AP, then Þ b1 = - 201 + 120 Þ b1 = - 81
a2 - a1 = a3 - a2 = . . . = an - an - 1 = d
If a is the first term and d is the common difference, then Example 3. If the sum of the series
AP can be written as a , a + d , a + 2 d , K , { a + ( n - 1) d } 3 1 4
20 + 19 + 19 + 18 + ... upto nth term is 488 and the nth
e.g. (i) 1, 3, 5, 7, … (ii) 2, 4, 6, … 5 5 5
term is negative, then (JEE Main 2020)
The nth term of an AP (a) nth term is - 4 (b) nth term is - 4
2
Let a be the first term d be the common difference and l 5
be the last term of an AP, then nth term is given by (c) n = 60 (d) n = 41
Tn = l = a + ( n - 1)d, where d = Tn - Tn - 1 Sol. (a) Given series is
The nth term from last is Tn = l - ( n - 1)d 3 1 4
20 + 19
+ 19 + 18 + … upto nth term = 488
5 5 5
The sum of n terms of an AP né æ 2öù é 1 nù
Suppose there are n terms of a sequence, whose first Þ ê(2 ´ 20) + (n - 1) ç - ÷ ú = 488 Þ nê20 + - ú = 488
2ë è 5øû ë 5 5û
term is a, common difference is d and last term is l, then
sum of n terms is given by Þ n[101 - n] = 488 ´ 5 = 2440
n n Þ n 2 - 101n + 2440 = 0
S n = [2a + ( n - 1)d ] = [a + l ]
2 2 Þ n 2 - 40n - 61n + 2440 = 0
Þ (n - 40)(n - 61) = 0 Þ n = 40 or 61
Example 1. If a1, a2, a3, ..., an are in AP and a1 + a4 + a7 + æ 2ö 1 22
... + a16 = 114 , then a1 + a6 + a11 + a16 is equal to Q T40 = 20 + 39 ç - ÷ = [100 - 78] = >0
è 5ø 5 5
(JEE Main 2019)
æ 2ö 1
(a) 64 (b) 76 (c) 98 (d) 38 and T61 = 20 + 60 ç - ÷ = [100 - 120 ] = - 4 < 0
è 5ø 5
Sol. (b) Given AP is a1, a2, a3 , ¼ , an Q nth term is negative, so value of ‘n’ is 61 and value of
Let the above AP has common difference ‘d’, then nth term is - 4.
a1 + a4 + a7 + ¼ + a16 Hence, option (a) is correct.
= a1 + ( a1 + 3d) + ( a1 + 6d) + ¼ + ( a1 + 15d) 30

= 6a1 + (3 + 6 + 9 + 12 + 15)d Example 4. Let a1, a2, ..... a30 be an AP, S = å ai and
i =1
= 6a1 + 45d = 114 (given) 15
Þ 2a1 + 15d = 38 …(i) T= å a( 2i - 1). If a5 = 27 and S - 2T = 75, then a10 is equal to
Now, a1 + a6 + a11 + a16 i =1

= a1 + ( a1 + 5d) + ( a1 + 10d) + ( a1 + 15d) (a) 42 (b) 57 (c) 52 (d) 47


= 4a1 + 30d = 2(2a1 + 15d) Sol. (c) We have, S = a1 + a2 + ¼ + a30 = 15 [2a1 + 29 d ] …(i)
= 2 ´ 38 = 76 [from Eq. (i)] [where d is the common difference]
é n ù
Example 2. The common difference of the AP êëQ S n = 2 [2a + (n - 1) d ]úû
b1, b 2, K, b m is 2 more than the common difference of AP
a1, a2 , ..., an. If a40 = - 159, a100 = - 399 and b100 = a70, then and T = a1 + a3 + ¼ + a29
b1 is equal to (JEE Main 2020) 15
= [2a1 + 14 ´ 2 d)] [Q common difference is 2d]
(a) 81 (b) -127 2
(c) -81 (d) 127 Þ 2T = 15 [2a1 + 28 d ] …(ii)
28 JEE Main Mathematics

From Eqs. (i) and (ii), we get Important Results Related to AP


S - 2T = 15 d = 75 [Q S - 2T = 75]
Þ d =5 (i) If a fixed constant C is added to (or subtracted from)
each term of a given AP, then the resulting sequence
Now, a10 = a5 + 5 d = 27 + 25 = 52
is also an AP with the same common difference as
Example 5. Let a1, a2, a3, ... be an AP, such that that of the given AP.
a1 + a2 + ... + ap (ii) If each term of an AP is multiplied by (or divided by)
p3 a6
= ; p ¹ q, then is equal to a non-zero fixed constant C, then the resulting
a1 + a2 + a3 + ... + aq q3 a21 (JEE Mains 2013) sequence is also an AP with common difference C
41 121 11 121 æ 1 ö
(a) (b) (c) (d) times ç or times÷ the previous.
11 1681 41 1861 è C ø
a1 + a 2 + K + a p p3 (iii) If a1 , a2 , a3 ,K and b1 , b2 , b3 ,K are two AP’s, then
Sol. (b) Given that, =
a1 + a 2 + K + a q q3 a1 ± b1 ,a2 ± b2 , a3 ± b3 ,Kis also an AP with common
difference d1 ± d2.
p
[ 2a1 + ( p - 1) d ] (iv) A sequence obtained by multiplying or division of
2 p3
Þ = 3 corresponding terms of two AP is not an AP.
q
[ 2a 2 + (q - 1) d ] q
2 (v) If in an AP, S p = q and S q = p, then S p + q = - ( p + q ).
where, d is a common difference of an AP. (vi) Three numbers in AP can be taken as a - d , a , a + d ,
2a 1 + ( p - 1) d p 2 five numbers in AP can be taken as
Þ =
2a 2 + (q - 1) d q 2 a - 2 d , a - d , a , a + d , a + 2 d, where the common
difference is d.
d
a1 + ( p - 1)
2 (vii) Four numbers in AP can be taken as
Þ 2 =p
d q2 a - 3d , a - d , a + d , a + 3 d , where common difference
a 2 + (q - 1) is 2 d.
2
On putting p = 11and q = 41, we get (viii) If an , an + 1 and an + 2 are three consecutive terms
d of an AP , then 2an + 1 = an + an + 2
a1 + (11 - 1) 2
2 = (11) (ix) In an AP, the sum of terms equidistant from the
d ( 41) 2 beginning and end is constant and equal to the
a 2 + ( 41 - 1)
2 sum of first and last term,
a1 + 5 d 121 a6 121 i.e. a1 + an = a2 + an - 1 = a3 + an - 2 = K
Þ = Þ =
a 2 + 20 d 1681 a 21 1681 (x) Any term of an AP (except the first) is equal to half
the sum of terms which are equidistant from it,
Example 6. In a potato race 20 potatoes are placed in a 1
i.e. an = ( an - k + an + k ), k < n
line at intervals of 4 m with the first potato 24 m from the 2
starting point. A contestant is required to being the potatoes 1
back to the starting place on at a time. How far would he run and for k = 1, an = ( an - 1 + an + 1 )
2
in bringing back all the potatoes?
(xi) Tn = S n - S n - 1 ( n ³ 2).
(a) 2485 m (b) 2480 m
(c) 2482 m (d) None of these
(xii) A sequence is an AP, if its nth term is of the form
An + B, i.e. a linear expression in n. In such a case
Sol. (b) According to the given condition, the sequence becomes, the common difference is A i.e. the coefficient of n.
24, 28, . . . , 20 th term, (xiii) If sum of n terms of any sequence is quadratic
Here, a = 24,d = 28 - 24 = 4,n = 20 expression in n (i.e. S n = an 2 + bn + c), then
n sequence is AP.
\ S n = [2a + (n - 1)d ]
2 (xiv) The common difference of an AP can be zero,
20 positive or negative.
= [2 ´ 24 + (20 - 1) 4]
2
Example 7. Five numbers are in A.P., whose sum is 25
= 10 ( 48 + 76) 1
and product is 2520. If one of these five numbers is - , then
= 10 ´ 124 2
= 1240 the greatest number amongst them is (JEE Main 2020)
The contestant is required to bring the potatoes back to the 21
(a) 7 (b) 16 (c) 27 (d)
starting place = 2S n = 2 ´ 1240 = 2480 m 2
Sequence and Series 29

Sol. (b) Let five numbers, which are in A.P. is Arithmetic Mean
a - 2d, a - d, a, a + d , a + 2d .
If we insert one arithmetic mean between two numbers
According to given information,
a+b
5a = 25 Þ a = 5 a and b, then Arithmetic mean =
2
and a( a2 - d 2)( a2 - 4d 2) = 2520 ● The sum of n arithmetic means between a and b is n

Þ 5(25 - d 2)(25 - 4d 2) = 2520 times of arithmetic mean of a and b


Þ (d 2 - 25)( 4d 2 - 25) = 504 æ a + bö
i.e. A1 + A2 + K + An = n ç ÷ = nA
è 2 ø
Þ 4d 4 - 125d 2 + 625 = 504
● The arithmetic mean of n positive numbers
Þ 4d 4 - 125d 2 + 121 = 0 a1 , a2 , a3 , K , an is
Þ 4d 4 - 4d 2 - 121d 2 + 121 = 0 a + a2 + a3 + K + an
AM = 1
Þ 4d 2(d 2 - 1) - 121(d 2 - 1) = 0 n
121
Þ d 2 = 1, Example 8. Suppose x and y are two real numbers such
4 that the rth mean between x and 2y is equal to rth maan
11 between 2x and y, when n arthmetic means are iserted
Þ d = ± 1, ±
2 n +1 y
between them in both the cases, - is equal to
If d = 1, then terms are 3, 4, 5, 6, 7, and r x
if d = -1, then terms are 7, 6, 5, 4, 3, and (a) 2 (b) 1 (c) 4 (d) 3
11 1 21 Sol. (b) Let A1, A2, A3 , L , An are n arithmetic means between two
if d = , then terms are -6, - , 5, , 16.
2 2 2 numbers a and b, then a1, A1, A2, A3 , Ar , L , An , b are in AP.
11 Let d be the common difference, then
When a = 5 and d = ± , then one of these five numbers of
2 b = a + (n + 1) d [Q l = a + (n - 1) d ]
1
A.P. is - and the greatest number amongst them is 16. b-a
and d=
2 n +1
( b - a)
\ Ar = a + rd = a + r ×
Insertion of Arithmetic Mean n +1
a(n - r + 1) + rb
(AM) between Two Numbers Þ Ar = …(i)
n +1
Let A1 , A2 , . . . , An ; n arithmetic means are inserted Now, put a = x and b = 2y in Eq. (i), we get
between two numbers a and b such that a , A1 , A2 , . . . , An , x (n - r + 1) + 2ry
Ar = …(ii)
b form an AP. n +1
Here, total number of terms are ( n + 2) and common Again, put a = 2x and b = y in Eq. (i), we get
difference be d (say). 2x (n - r + 1) + ry
Ar = …(iii)
\ b = ( n + 2) th term = a + ( n + 2 - 1) d n +1
b- a From Eqs. (ii) and (iii),
Þ d= x (n - r + 1) + 2ry 2x (n - r + 1) + ry
n +1 =
n +1 n +1
æb - aö n a + b
\ A1 = a + d = a + ç ÷= Þ - x (n - r + 1) = - ry
è n + 1ø n +1 y n - r +1
Þ =
æ b - a ö ( n - 1) a + 2 b x r
and A2 = a + 2d = a + 2 ç ÷= y n +1
è n + 1ø n +1 Þ = -1
x r
æb - aö n +1 y
In general, Ar = a + r ç ÷ \ - =1
è n + 1ø r x
( n - r + 1) a + r b
= , " r = 1, 2 , ... , n Geometric Progression (GP)
n +1
(1 - 1 + 1)a + 1( b) A geometric progression (GP) is a sequence of numbers,
(i) If r = 1 and n = 1, then A1 = whose first term is non-zero and each of the term is
1+1 obtained by multiplying its just preceding term by a
a+b constant quantity. This constant quantity is called the
Þ A1 = = Mean common ratio of the GP.
2
30 JEE Main Mathematics

In other words, if a1 , a2 , a3 , ... , an are in GP, then 5


= {(10 - 1) + (100 - 1) + (1000 - 1) + . . . to n terms}
a2 a3 an 9
= =K= =r (say)
a1 a2 an - 1 5
= {10 + 100 + 1000 + . . . + n terms - (1 + 1 + . . . + n terms)}
9
r is known as common ratio of GP.
5 ì10(10 n - 1) ü
If a is the first term and r is the common ratio, then GP = í - ný
9 î 10 - 1 þ
can be written as a , ar , ar 2 , K , ar n - 1. ( a ¹ 0)
5
e.g. 2, 4 , 8, 16, . . . = {10 n + 1 - 10 - 9n}
81
The nth term of a GP (General Term)
Example 10. If a , b, c are pth, qth and rth terms of a GP,
Let a be the first term, r be the common ratio and l be the
last term of a GP, then nth term is given by then (q - r) log a + (r - p) log b + ( p - q) log c is equal to
Tn (a) p + q + r (b) 1
Tn = l = ar n - 1, where r = (c) -pqr (d) 0
Tn - 1
l Sol. (d) Let A and R be the first term and common ratio of the
The nth term from last is, Tn ¢ =
rn - 1 given GP. Then, a = AR p - 1
Þ log a = log A + ( p - 1) log R …(i)
The sum of n terms of a GP Similarly, log b = log A + (q - 1) log R …(ii)
Suppose there are n terms of a sequence, whose first and log c = log A + (r - 1) log R …(iii)
term is a, common ratio is r and last term is l, then sum Now, (q - r) log a + (r - p) log b + ( p - q) log c
of n terms is given by
= (q - r) {log A + ( p - 1) log R }
ì a(r n - 1) + (r - p) {log A + (q - 1) log R }
ï , when r > 1
ï r - 1n
+ ( p - q) {log A + (r - 1) log R }
ï a(1 - r ) = log A [q - r + r - p + p - q ]
Sn = í , when r < 1
+ log R [ p(q - r) + q(r - p) + r( p - q)
ï 1-r
ï na when r = 1 - (q - r) - (r - p) - ( p - q)]
ï = log A × 0 + log R × 0 = 0
î
If, terms are infinite, then sum of GP is, Example 11. Let an be the nth term of a GP of positive
a 100 100 200
S¥ =
1-r
, when|r| < 1 terms. If å a 2n + 1 = 200 and å a2n = 100, then å an is equal
n=1 n=1 n=1

Note to (JEE Main 2020)


• If a, b, c , d … are in GP, they are also in continued proportion (a) 300 (b) 175 (c) 225 (d) 150
a b c 1
i.e. = = =K= (say) Sol. (d) It is given that an is the nth term of a GP of positive
b c d r
terms, such that
• If first term of a GP of n terms is a and last term is l. Then, the 100
S a2n + 1 = 200
product of all the terms of the GP is (al )n / 2 . n =1

• If a, b, c are in AP, as well as in GP, then a = b = c . Þ a3 + a5 + a7 + K + a201 = 200


Þ ar + ar 4 + ar 6 + K + ar 200 = 200
2
Example 9. The sum of the series
[where a is the first term of GP and r is the common ratio]
5 + 55 + 555 + . . . to n terms is
Þ ar 2(1 + r 2 + r 4 + K + r198) = 200
1 5
(a) (10 n + 1) (b) (10 n + 1 - 9n - 10) æ1 - r 200 ö
80 81 Þ ar 2ç ÷ = 200 …(i)
5 5 è 1 - r2 ø
(c) (10 n - 1 - 8n - 1) (d) (10 n + 1 + 9n + 10) 100
81 81 and S a2n = 100 (given)
n =1
Sol. (b) 5 + 55 + 555 + . . . to n terms
Þ a2 + a4 + a6 + K + a200 = 100
= 5 {1 + 11 + 111 + . . . to n terms}
5 Þ ar + ar3 + ar5 + K + ar199 = 100
= {9 + 99 + 999 + . . . to n terms}
9 æ1 - r 200 ö
[multiply numerator and denominator by 9] Þ ar ç ÷ = 100 …(ii)
è 1 - r2 ø
Sequence and Series 31

From Eqs. (i) and (ii), we get = [ x2 + x3 + x4 + ... ] - [y 2 + y3 + y 4 + .... ]


150 x2 y2
r = 2 and a = = - as| x| < 1and|y| < 1
2 200 - 1 1- x 1-y
200 a(r 200 - 1) x2(1 - y) - y 2(1 - x) ( x2 - y 2) - ( x2y - y 2x)
\ S an = Þ ( x - y) S = =
n =1 r -1 (1 - x)(1 - y) (1 - x)(1 - y)
150 (2 200 - 1) ( x - y) [( x + y) - xy ]
= ´ Þ ( x - y) S =
200
2 -1 2 -1 (1 - x) (1 - y)
= 150 x + y - xy
Þ S=
Hence, option (d) is correct. (1 - x) (1 - y)

Example 12. Let S be the sum of the first 9 terms of the Important Results Related to GP
2 3
series {x + ka} + {x + (k + 2) a} + {x + (k + 4) a} (i) If each term of a GP is multiplied (or divided) by a
4 non-zero constant C ( ¹ 0), then the resulting
+ {x + (k + 6) a} + ... where a ¹ 0 and x ¹1.
sequence is also a GP with same common ratio.
10
x - x + 45 a ( x - 1) (ii) If a , ar , ar 2, … and a ¢ , a ¢ r ¢ , a ¢ r ¢ 2, … are two GP’s,
If S = , then k is equal to
x -1 (JEE Main 2020) then the sequences aa¢, aa ¢ (rr ¢ ), aa ¢ (rr ¢ )2, … and
2
(a) -5 (b) 1 (c) - 3 (d) 3 a a ær ö a ær ö
, ç ÷, ç ÷ , … obtained by the products
Sol. (c) It is given that a¢ a ¢ è r ¢ ø a ¢ è r ¢ ø
S = {x + ka} + {x2 + (k + 2) a} and quotients of the corresponding terms of the
given GP’s are also GP’s with common ratios rr ¢ and
+ {x3 + (k + 4) a} + {x4 + (k + 6) a} + .....+ upto 9 terms r
respectively.
Þ S = {x + x2 + x3 + x4 + K+ x9} r¢
+ a{k + (k + 2) + (k + 4) } + .....+ (k + 16)} (iii) The reciprocals of the terms of a GP also form a GP.
x ( x9 - 1) æ9ö (iv) If a1 , a2 , a3 , … is a GP of positive terms (i.e., ai > 0,
= + aç ÷ {2k + (9 - 1)2}
x -1 è2ø " i ), then log a1 , log a2 , log a3 , K is an AP and the
converse is also true in this case.
x10 - x
= + 9a (k + 8) (v) The odd number of terms in a GP should be taken
x -1
x10 - x + 9a (k + 8) ( x - 1)
as …
= a a a
x -1 ar3 , ar 2, ar, a , , 2 , 3 , K where common ratio is
r r r
x10 - x + 45a ( x - 1)
= [given] r while the even number of terms in a GP should be
x -1
taken as …
So, on comparing, we get a a a
k + 8 = 5 Þk = -3 ar5 , ar3 , ar, , 3 , 5 , … where common ratio is r 2.
r r r
Hence, option (c) is correct.
(vi) If a1 , a2 , a3 , ... , an are in GP, then a1k , a2k , a3k , ... , ank
Example 13. If| x | < 1,|y | < 1and x ¹ y, then the sum to will also be in GP whose common ratio is r k.
infinity of the following series
(vii) If a1 , a2, a3 , ... , an are in AP, then a a1 , a a2 ,
( x + y) + ( x 2 + xy + y 2) + ( x3 + x 2y + xy 2 + y3)+ … is
a a3 , ... , a an will be in GP whose common ratio is a d .
(JEE Main 2020)
x + y + xy x + y - xy (viii) If a1 , a2 , a3 ,K and b1 , b 2 , b3 ,K are in GP, then the
(a) (b)
(1 + x) (1 + y) (1 - x) (1 - y) sequence(s) a1 ± b1 , a 2 ± b 2 , a 3 ± b3 ,…., will not be a
x + y + xy x + y - xy GP.
(c) (d)
(1 - x) (1 - y) (1 + x) (1 + y) (ix) In a GP, the product of two terms equidistant from
the beginning and end is a constant and is equal to
Sol. (b) Given series of infinite terms, if| x| < 1,|y| < 1and x ¹ y is
the product of first term and last term, i.e. if
S = ( x + y) + ( x2+ xy + y 2) + ( x3 + x2y + xy 2 + y3) +…
a 1 , a 2 , a 3 , K , an - 2, an - 1 , an are in GP, then
On multiplying ( x - y) both sides, we get a1an = a2an - 1 = a3 an - 2 = K
Þ ( x - y) S = ( x2 - y 2) + ( x3 - y3) + ( x4 - y 4) + ....
32 JEE Main Mathematics

1/ ( n + 1)
Example 14. Three numbers form an increasing GP. If the æ bö
Þ r=ç ÷
middle number is doubled, then the new numbers are in AP. èaø
The common ratio of the GP is (JEE Main 2014) 1/ ( n + 1)
æ bö
(a) 2 - 3 (b) 2 + 3 (c) 3 - 2 (d) 3 + 2 \ G1 = ar = a ç ÷
èaø
a
Sol. (b) Let the three numbers in GP be , a , ar. 2/ ( n + 1)
r æ bö
G2 = ar 2 = a ç ÷
Also, the numbers form an increasing GP. èaø
a k/ ( n + 1)
So, r > 1. Now, , 2a , ar in AP. æ bö
r In general, Gk = a ç ÷ , " k = 1, 2 , 3, ... , n
a
èaø
\ 4a = + ar 1/ (1 + 1) 1/ 2
r æ bö æ bö
If k = 1 and n = 1, then G1 = a ç ÷ =aç ÷ = ab
Þ r 2 - 4r + 1 = 0 Þ r = 2 + 3 [Q r ¹ 2 - 3 ] èaø èaø

Example 15. If ax3 + bx 2 + cx + d is divisible by ax 2 + c , Geometric Mean


then a , b, c , d are in
If we insert one geometric mean between two numbers a
(a) AP (b) GP and b, then
(c) HP (d) None of these
Geometric mean = ab.
Sol. (b) Since, ax3 + bx2 + cx + d is divisible by ax2 + c.
Therefore, when ax3 + bx2 + cx + d is divided by ax2 + c, the Important Points
remainder is zero. ● The product of n geometric means between a and b is
bc equal to the nth power of geometric mean of a and b
\ d- =0
a i. e. , G1 × G2×K × Gn = ( ab)n / 2 = G n
b d
Þ bc = ad Þ = 1
a c ● The product of n geometric means between a and is 1.
a
Þ a , b, c, d are in GP.
● The geometric mean of n positive numbers
Example 16. The sum of the first three terms of a GP is a1 , a2 , a3 , K , an is GM = ( a1 a2 a3 K an )1/ n .
S and their product is 27. Then all such S lie in (JEE Main 2020) ● If A and G be the AM and GM between two positive
(a) ( - ¥ , 9] (b) [ - 3, ¥) numbers, then the numbers are A ± A2 - G 2 .
(c) ( - ¥ , - 9] È [3, ¥) (d) ( - ¥ , - 3] È [9, ¥)
● If A and G are respectively AM and GM between two
a
Sol. (d) Let, the three terms of GP are , a, ar. positive numbers a and b, then the quadratic equation
r
a
having a , b as its roots is
As, it is given that´ a ´ ar = 27 Þ a = 3 x 2 - 2 Ax + G 2 = 0
r
a æ1 + r + r 2 ö
and S = + a + ar Þ S = 3 ç ÷ Example 17. If m is the AM of two distinct real numbers
r è r ø
l and n (l , n > 1) and G1, G2 and G3 are three geometric means
Þ 3r 2 + (3 - S) r + 3 = 0 [Q r ¹ 0 ] between l and n, then G14 + 2G24 + G34 equals (JEE Main 2015)
Q r ÎR
\ D ³ 0 Þ (3 - S) 2 - 4 (3) (3) ³ 0 (a) 4 l 2mn (b) 4 lm2n (c) 4 lmn 2 (d) 4 l 2m2n 2

Þ (3 - S - 6) (3 - S + 6) ³ 0 Sol. (b) Given, m is the AM of l and n


Þ ( S + 3) ( S - 9) ³ 0 \ l + n = 2m …(i)
Þ S Î ( -¥ , - 3] È [9, ¥) and G1, G 2, G3 are geometric means between l and n
\ l, G1, G 2, G3 , n are in GP.
Let r be the common ratio of this GP.
Insertion of Geometric Mean (GM) \ G1 = lr Þ G 2 = lr 2
between Two Numbers 1

Let G1 , G2 , . . . , Gn ; n geometric means are inserted 3 4 ænö 4


Þ G3 = lr Þ n = lr Þ r=ç ÷
between two numbers a and b such that èlø
a , A1 , A2 , . . . , An , b form a GP. Now, G14 + 2G 24 + G34 = (lr) 4 + 2(lr 2) 4 + (lr3) 4
Here, total number of terms are ( n + 2) and let common = l 4 ´ r 4(1 + 2r 4 + r 8) = l 4 ´ r 4(r 4 + 1) 2
ratio is r. 2
n æn + lö
n + 2-1 = l4 ´ ç
2 2
÷ = ln ´ 4m = 4 lm n
\ b = ( n + 2)th term = ar l è l ø
Sequence and Series 33

Relation among Arithmetic Mean Example 19. The sum of the series
and Geometric Mean 1 + 2 × 2 + 3 × 2 2 + 4 × 23 + . . . + 100 × 299 is
Let a and b be two real positive and unequal numbers (a) 100 × 2100 + 1 (b) 99 × 2100 + 1
and A, G , H are arithmetic, geometric and harmonic (c) 99 × 2 99 - 1 (d) 100 × 2100 - 1
mean respectively between them.
a+b Sol. (b) Let S = 1 + 2 × 2 + 3 × 2 2 + 4 × 23 + . . . + 100 × 2 99 …(i)
Then, A= , G = ab
2 It is an arithmetico-geometric series.
a+b On multiplying Eq. (i) by 2 and then subtracting, we get
Now, A-G = - ab
2 S = 1 + 2 × 2 + 3 × 2 2 + 4 × 23 + . . . + 100 × 2 99
2
a + b - 2 ab æ a - b ö 2S = 1× 2 + 2 × 2 2 + 3 × 23 + . . . + 99 × 2 99 + 100 × 2100
= =ç ÷ >0
2 è 2 ø - S = 1 + 2 + 2 2 + 23 + . . . + 2 99 - 100 × 2100
\ A-G > 0 [Q a and b are positive] 1 (2100 - 1 )
Þ A>G …(i) Þ -S = - 100 × 2100
2 -1
Example 18. The minimum value of 2 sin x + 2 cos x is Þ - S = 2100 - 1 - 100 × 2100
(JEE Main 2020) Þ - S = - 1 - 99 × 2100
1 1
-1 + 1-
(a) 2 -1 + 2
(b) 21 - 2
(c) 2 2 (d) 2 2 Þ S = 99 × 2100 + 1

Sol. (d) As 2 sinx and 2 cosx are positive quantities for "x Î R.
To Find nth Term by Difference Method
So, according to AM-GM inequality, we have
If T1 , T2 , ... , Tn are terms of any series and their
2 sin x + 2 cos x
³ 2 sin x × 2 cos x Þ (2 sin x + 2 cos x) ³ 2 ´ 2(sin x+ cos x)/ 2 difference ( T2 - T1 ), ( T3 - T2 ),( T4 - T3 ) , …, ( Tn - Tn - 1 )
2
are either in AP or in GP, then Tn and S n of series may
æp ö
Now, sin x + cos x = 2 sinç + x÷ Î [– 2 , 2 ] be found by the method of differences.
è4 ø
\The minimum value of 2 + 2 cos x is minimum value of
sin x Let S n = T1 + T2 + T3 + . . . + Tn
1
1+ (sin x + cos x )
1
1+ (– 2) 1-
1 Again, S n = T1 + T2 + . . . + Tn - 1 + Tn
2 2 and it is equal to 2 2 =2 2.
S n - S n = T1 + ( T2 - T1 ) + ( T3 - T2 ) + . . . + ( Tn - Tn -1 ) - Tn
Arithmetico-Geometric Þ Tn = T1 + ( T2 - T1 ) + ( T3 - T2 ) + . . . + ( Tn - Tn - 1 )
Progression (AGP) Þ Tn = T1 + t1 + t2 + t3 + . . . + tn - 1
A sequence whose each term is obtained by multiplying where t1 , t2 , . . . are terms of the new series.
corresponding terms of AP and GP is called an
arithmetico-geometric progression. Example 20. The sum of the infinite series
If a , ( a + d ), ( a + 2d ), K , [a + ( n - 1) d ] is an AP and 4 9 16
1+ + + + K ¥ is
1, r , r 2 , K , r n - 1 is a GP, then on multiplying 3 32 33
(a) 4/2 (b) 9/2 (c) 4/9
corresponding terms of AP and GP, a progression
(d) 1
a , ( a + d ) r , ( a + 2d ) r 2 , K , [a + ( n - 1) d ] r n - 1 is formed
which is called an arithmetico-geometric progression. Sol. (b) This is clearly not an arithmetico-geometric series,
since 1, 4, 9, 16, … are not in AP. However, their successive
Method of Solving AGP Series differences ( 4 - 1), (9 - 4), (16 - 9) , … are in AP.
4 9 16
Let AGP series Let S¥ = 1 + + 2 + 3 + K ¥
3 3 3
a + ( a + d )r + ( a + 2d )r 2 + . . . + { a + ( n - 1)d }r n - 1
1 1 4 9
S¥ = + 2 + 3 + K ¥
Types of problem can be solved by using the direct 3 3 3 3
method given below. On subtraction, we get
(i) The sum of n terms of arithmetico-geometric is 2 3 5 7
S¥ = 1 + + 2 + 3 + K ¥
a dr(1 - r n - 1 ) [a + ( n - 1) d ] r n 3 3 3 3
Sn = + - ,r ¹1
1-r (1 - r )2 1-r 1 2 1 3 5
Þ × S¥ = + 2 + 3 + K ¥
(ii) The sum of infinite terms of arithmetico-geometric 3 3 3 3 3
progression, é 1ù
a dr êëmultiplying both sides by 3 úû
S¥ = + , |r| < 1
1 - r (1 - r )2
34 JEE Main Mathematics

On subtracting the two series, we get Tn = (nth term of 3, 5, 7, . . .) ´ (nth term of1, 2, 3, . . .) 2
æ 4ö 2 2 2 = [3 + (n - 1)2][1 + (n - 1)1]2
ç ÷ × S¥ = 1 + + 2 + 3 + K ¥
è9ø 3 3 3 = (3 + 2n - 2)(n) 2 = (2n + 1)n 2 = 2n3 + n 2
2æ 1 1 ö 2 1
= 1 + ç1 + + 2 + K ¥ ÷ = 1 + ´ =2 Now, S = STn = 2Sn3 + Sn 2
3è 3 3 ø 3 æ 1ö
ç1 - ÷ é ì n(n + 1) ü ù
2
è 3ø
2 êQ Sn3 = í ý ú
æ 9ö 9 é n(n + 1) ù n(n + 1)(2n + 1)
ê î 2 þ ú
\ S ¥ = ç2 ´ ÷ = =2 ê +
è 4ø 2 ë 2 úû 6 ê 2 n(n + 1)(2n + 1) ú
ê Sn = ú
ë 6 û
Sum of n terms of Special Series n(n + 1) é n(n + 1) 2n + 1ù
= êë2 ´ +
2 2 3 úû
Sn, Sn 2 and Sn 3 n(n + 1) é 3n(n + 1) + 2n + 1ù
= êë úû
Generally, find the sum of n terms of any series, we use 2 3
S before the nth term of the series. n(n + 1)
= ´ (3n 2 + 3n + 2n + 1)
Sum of some special series is given below. 6
(i) Sum of n natural numbers n(n + 1)(3n 2 + 5n + 1)
=
= 1 + 2 + 3 + 4 +K+ n 6
n( n + 1)
= Sn = Example 22. The sum of series
2
3 ´ 13 5 ´ (13 + 23) 7 ´ (13 + 23 + 33)
(ii) Sum of squares of n natural numbers 2
+ 2 2
+ + ....... + upto 10th
2 2 2 2 2 1 1 +2 12 + 2 2 + 32
= 1 + 2 + 3 + 4 +K+ n
term, is (JEE Main 2019)
n( n + 1) ( 2n + 1)
= Sn 2 = (a) 680 (b) 600 (c) 660 (d) 620
6
(iii) Sum of cubes of n natural numbers Sol. (c) Given series is
3 3
= 1 + 2 + 3 +K+ n3 3 3 ´ 13 5 ´ (13 + 23) 7 ´ (13 + 23 + 33)
+ + + ...
2
12 12 + 2 2 12 + 2 2 + 33
n 2( n + 1)2 é n ( n + 1) ù
= Sn3 = =ê ú = ( Sn )2 So, nth term
4 ë 2 û (3 + (n - 1)2)(13 + 23 + 33 ... + n3)
Tn =
Note 12 + 2 2 + 3 2 + K + n 2
1 1 1 1 n 2
• + + +K+ = æ n (n + 1) ö
1× 2 2 × 3 3 × 4 n(n + 1) n + 1 (2n + 1) ´ ç ÷
è 2 ø
1 1 1 1 1 =
• + +K+ = - n(n + 1)(2n + 1)
1× 2 × 3 2 × 3 × 4 n(n + 1) (n + 2 ) 4 2(n + 1) (n + 2 ) 6
n(n + 1) (n + 2 ) é n
é n(n + 1) ù
2 n
2 n(n + 1)(2n + 1) ù
• 1× 2 + 2 × 3 + K + n(n + 1) = 3
3 êQ r S= 1 r = ê 2 ú andr S= 1 r = ú
ë ë û 6 û
• 1× 2 × 3 × 4 + 2 × 3 × 4 × 5 + K + n(n + 1) (n + 2 ) (n + 3)
3n(n + 1) 3 2
1 So, Tn = = (n + n)
= n(n + 1) (n + 2 ) (n + 3) (n + 4) 2 2
5
Now, sum of the given series upto n terms
Example 21. The sum of the series 3
S n = STn = [ Sn 2 + Sn]
2
3 ´ 12 + 5 ´ 2 2 + 7 ´ 32 + . . . is
3 é n(n + 1)(2n + 1) n(n + 1) ù
= ê +
n(n + 1)(n 2 - 5n - 1) n(n + 1)(3n 2 + 5n + 1) 2ë 6 2 úû
(a) (b)
6 6 3 é10 ´ 11 ´ 21 10 ´ 11ù
\ S10 = ê +
(c)
n(n - 1)(3n 2 - 5n - 1)
(d) None of these 2ë 6 2 úû
6 3 3
= [(5 ´ 11 ´ 7) + (5 ´ 11)] = ´ 55 ( 7 + 1)
Sol. (b) Let given series is S = 3 ´ 12 + 5 ´ 2 2 + 7 ´ 3 2 + . . . 2 2
3
First, we will split the given series into two parts which are = ´ 55 ´ 8 = 3 ´ 55 ´ 4
2
3, 5, 7, … and12, 2 2, 3 2, . . . and find the nth term of each part
= 12 ´ 55 = 660
separately to find the nth term of the given series.
Practice Exercise
ROUND I Topically Divided Problems

General Term of AP 8. A carpenter was hired to build 192 window frames.


1. { an } and { bn } be two sequences given by The first day he made five frames and each day,
1 1 1 1 thereafter he made two more frames than he made
an = ( x) 2
n n
+ ( y) 2 and bn = ( x) 2
n n
- ( y) 2 for all the day before. How many days did it take him to
finish the job?
n Î N , then a1 a2 a3. . . an is equal to
(a) 11 (b) 10 (c) 12 (d) 14
x+ y x- y xy
(a) x - y (b) (c) (d)
bn bn bn 9. In a cricket tournament 16 school terms
participated. A sum of ` 8000 is to be awarded
2. If the roots of the equation
x 3 - 12 x 2 + 39 x - 28 = 0 are in AP, then their among themselves as prize money. If the last
common difference will be placed team is awarded ` 275 in prize money and
(a) ± 1 (b) ± 2 (c) ± 3 (d) ± 4 the award increases by the same amount for
successive finishing places, amount will the first
3. The interior angles of a polygon are in AP. If the
place team received is
smallest angle be 120° and the common difference
(a) ` 720 (b) ` 725 (c) ` 735 (d) ` 780
be 5, then the number of side is
(a) 8 (b) 10 (c) 9 (d) 6 10. If q1, q 2 , q 3, . . . , q n are in AP, whose common
1 1 1 difference is d, then sin d (sec q 1 sec q 2 + sec q 2 sec q 3
4. If , , be consecutive terms of an AP,
+ . . . + sec q n -1 sec q n ) is equal to
b- c c- a a- b
then ( b - c) 2 , ( c - a) 2 , ( a - b) 2 will be in (a) tan qn - tan q2 (b) tan qn + tan q1
(a) GP (b) AP (c) tan qn - tan q1 (d) None of these
(c) HP (d) None of these 11. If a1, a2 , a3 , . . . , a4001 are terms of an AP such that
5. A man arranges to pay off a debt of ` 3600 by 1 1 1
+ + ... + = 10 and
40 annual instalments which are in AP. When a1a2 a2 a3 a4000 a4001
30 of the instalments are paid, he dies leaving one-third a2 + a4000 = 50, then|a1 - a4001| is equal to
of the debt unpaid. The value of the 8th instalment is (a) 20 (b) 30
(a) ` 35 (b) ` 50 (c) 40 (d) None of these
(c) ` 65 (d) None of these 12. If a1, a2 , . . . , an are in AP with common difference d,
6. Given that n AM’s are inserted between two sets of then the sum of the series
numbers a , 2 b and 2 a , b where a , b Î R. Suppose sin d ( cosec a1 cosec a2 + cosec a2 cosec a3 + . . . +
further that mth mean between these sets of cosec an -1 cosec an ) is
numbers is same, then the ratio a : b is equal to (a) sec a1 - sec a n (b) cot a1 - cot a n
(a) (n - m + 1): m (b) (n - m + 1): n (c) tan a1 - tan a n (d) cosec a1 - cosec a n
(c) n : (n - m + 1) (d) m : (n - m + 1)
13. Let S1, S2 , . . . be squares such that for each n ³ 1,
7. A man saved ` 66000 in 20 yr. In each succeeding the length of a side of Sn equals the length of a
year after the first year he saved ` 200 more than diagonal of Sn + 1. If the length of a side of S1 is 10
what he saved in the previous year. How much did cm, then for which of the following values of n, the
he save in the first year? area of Sn less than 1 sq cm ?
(a) ` 1450 (b) ` 1400 (a) 7 (b) 6
(c) ` 1470 (d) ` 1480 (c) 9 (d) None of these
36 JEE Main Mathematics

14. If a, b, c are in AP, then 10 a x + 10 , 10 b x + 10 , 10 c x + 10 23. If the sum and product of the first three terms in
( x ¹ 0) are in an AP are 33 and 1155, respectively, then a value
(a) AP (b) GP only when x > 0 of its 11th term is (JEE Main 2019)
(c) GP for all x (d) GP only when x < 0 (a) 25 (b) –36 (c) –25 (d) –35
15. If 19th term of a non-zero AP is zero, then its
(49th term) : (29th term) is (JEE Main 2019) 24. If log 2 , log( 2 n - 1) and log ( 2 n + 3) are in AP, then n
(a) 1 : 3 (b) 4 : 1 (c) 2 : 1 (d) 3 : 1 is equal to
2 2 2
16. If a , b , c are in AP, then which of the following is 5 3
(a) (b) log 2 5 (c) log3 5 (d)
2 2
also in AP ?
(a) sin A ,sin B,sin C (b) tan A , tan B, tan C 25. Given sum of the first n terms of an AP is 2 n + 3 n2 .
(c) cot A , cot B, cot C (d) None of these Another AP is formed with the same first term and
double of the common difference, the sum of n
17. Between 1 and 31 are inserted m arithmetic means
terms of the new AP is
so that the ratio of the 7th and ( m - 1)th means is
(a) n + 4n 2 (b) 6 n 2 - n
5:9, then the value of m is
(c) n 2 + 4n (d) 3 n + 2n 2
(a) 13 (b) 14 (c) 11 (d) 12
26. If the ratio of the sum of n terms of two AP’s be
Sum of n Terms in AP (7 n + 1) : ( 4 n + 27), then the ratio of their 11th
18. If a1, a2 , a3, . . . , a2 n are in AP, then the value of terms will be
a12 - a22 + a32 - a42 + K + a22 n - 1 - a22 n is equal to (a) 2 : 3 (b) 3 : 4 (c) 4 : 3 (d) 5 : 6
n n 27. If the 2nd, 5th and 9th terms of a non-constant AP
(a) (a12 - a 22n ) (b) (a12 - a 22n )
2n - 1 2n + 1 are in GP, then the common ratio of this GP is
n (JEE Main 2016)
(c) (a12 + a 22n ) (d) None of these 8 4 7
2n - 1 (a) (b) (c) 1 (d)
5 3 4
19. If the sum of first 11 terms of an AP., a1, a2 , a3, . . .
28. If the first term of an AP is a and the sum of the
is 0 ( a1 ¹ 0), then the sum of the AP.,
first p terms is zero, then the sum of its next
a1, a3, a5 , . . . , a23 is ka1, where k is equal toMain 2020)
(JEE
q terms is
121 121 72 72 a ( p + q)q - a ( p + q)q
(a) - (b) (c) (d) - (a) (b)
10 10 5 5 p-1 p-1
20. If the first term of an AP is 3 and the sum of its a ( p - q)q
(c) (d) None of these
first 25 terms is equal to the sum of its next 15 p-1
terms, then the common difference of this AP is
(JEE Main 2020)
29. The sum of all two digit positive numbers which
1 1 1 1 when divided by 7 yield 2 or 5 as remainder is
(a) (b) (c) (d) (JEE Main 2019)
6 5 4 7
(a) 1256 (b) 1465 (c) 1356 (d) 1365
21. Let a1, a2 , ¼ , an be a given AP. Whose common
30. If a1, a2 , a3, . . . , a24 are in arithmetic progression
difference is an integer and Sn = a1 + a2 + ¼ + an .
and a1 + a5 + a10 + a15 + a20 + a24 = 225, then
If a1 = 1, an = 300 and 15 £ n £ 50, then the ordered
a1 + a2 + a3 + . . . + a23 + a24 is equal to
pair ( Sn - 4 , an - 4 ) is equal to (JEE Main 2020)
(a) 909 (b) 75 (c) 750 (d) 900
(a) (2490, 249) (b) (2480, 249)
(c) (2480, 248) (d) (2490, 248)
31. If a1, a2 , a3, . . . , an are in AP, where ai > 0 for all i,
1 then value of
22. If the 10th term of an AP is and its 20th term is 1 1 1
20 + + ... + is equal to
1 a1 + a2 a2 + a3 an - 1 + an
, then the sum of its first 200 terms is
10 n -1 n+1
(JEE Main 2020)
(a) (b)
(a) 50
1
(b) 100 a1 + a n a1 + a n
4 n -1 n+1
1 (c) (d)
(c) 50 (d) 100 a1 - a n a1 - a n
2
Sequence and Series 37

32. Let the sequence, a1, a2 , a3, . . , a2 n , form an AP, then 40. Let a, b, c, d and p be any non-zero distinct real
a12 - a22 + a32 - ... + a22 n - 1 - a22 n is equal to numbers such that ( a 2 + b2 + c2 ) p 2 - 2
n 2n ( ab + bc + cd) p + (b2 + c2 + d 2 ) = 0. Then,
(a) (a12 - a 22n) (b) (a 22n - a 12) (JEE Main 2020)
2n - 1 n -1
n (a) a , c, p are in AP (b) a , c, p are in GP
(c) (a 12 + a 22n) (d) None of these
n+1 (c) a , b, c, d are in GP (d) a , b, c, d are in AP

33. Let S1 be the sum of first 2 n terms of an arithmetic 41. If x, 2 y and 3z are in AP, where the distinct
progression. Let S2 be the sum of first 4 n terms of numbers x, y, z are in GP, then the common ratio of
the same arithmetic progression. If ( S2 - S1) is the GP is
1000, then the sum of the first 6 n terms of the 1 1
(a) 3 (b) (c) 2 (d)
arithmetic progression is equal to (JEE Main 2021) 3 2
(a) 1000 (b) 7000 42. The length of three unequal edges of a rectangular
(c) 5000 (d) 3000 solid block are in GP. The volume of the block is
34. Consider the following statements 216 cm 3 and the total surface area is 252 cm 2 .
I. If an denotes the nth term of an AP, then The length of the longest edge is
an + k + an - k (a) 12 cm (b) 6 cm (c) 18 cm (d) 3 cm
an = a a
2 43. Let a1, a2 , . . . . , a10 be a GP. If 3 = 25, then 9
a1 a5
II. In an AP, if the sum of m terms is equal to the
sum of n terms, then the sum of ( m + n) terms equals
(JEE Main 2019)
is always zero.
(a) 53 (b) 2(52) (c) 4(52) (d) 54
Which of the statement is given above is/are
correct?
44. If A is the arithmetic mean and G1, G2 be two
(a) Only I (b) Only II
geometric means between any two numbers, then
(c) Both I and II (d) None of these G12 G22
+ is equal to
G2 G1
35. The sum of integers from 1 to 100 that are divisible
(a) 2 A (b) A
by 2 or 5 is
(a) 3000 (b) 3050 (c) 3 A (d) None of these
(c) 4050 (d) None of these 45. If the pth and qth terms of a GP are q and p,
36. The sum of the integers from 1 to 100 which are not respectively, then ( p - q)th term is
1 1
divisible by 3 or 5 is
æ qp ö p - q æ qq ö p - q
(a) 2489 (b) 4735 (c) 2317 (d) 2632 (a) ç q ÷ (b) ç p ÷
èp ø èp ø
General Term of GP 1
æ pq ö p - q
37. If a , b, c are in GP and log a - log 2 b, log 2 b - log 3c (c) ç p ÷ (d) None of these
èq ø
and log 3c - log a are in AP, then a , b, c are the
length of the sides of a triangle which is 46. If a , b, c, d and p are different real numbers such
(a) acute angled (b) obtuse angled
that ( a 2 + b2 + c2 ) p 2 - 2 ( ab + bc + cd) p
(c) right angled (d) equilateral
+ ( b2 + c2 + d 2 ) £ 0, then a , b, c, d are in
38. If a and b are roots of the equation x 2 - 3x + a = 0
and g and d are roots of the equation (a) AP (b) GP
x 2 - 12 x + b = 0 and a, b, g, d form an increasing (c) HP (d) ab = cd
GP, then the values of a and b are respectively 47. If the roots of the cubic equation ax 2 + dx 2 + cx + d
(a) 2, 16 (b) 4, 8 are in GP, then
(c) 2, 32 (d) None of these (a) c3 a = b3 d (b) ca3 = bd3
3 3
39. If a be the AM between b and c and GM’s are G1 (c) a b = c d (d) ab3 = cd3
and G2 , then G13 + G23 is equal to 48. If x, y, z are in GP and a x = b y = cz , then
(a) abc (b) 2abc
(a) log a c = logb a (b) logb a = log c b
(c) 3abc (d) 4abc
(c) log c b = log a c (d) None of these
38 JEE Main Mathematics

49. In an increasing, geometric series, the sum of the 57. Let a 1, b1 are the roots of x 2 - 6 x + p = 0 and a 2 , b 2
second and the sixth term is
25
and the product of are the roots of x 2 - 54 x + q = 0. If a 1, b1, a 2 , b 2
2 form an increasing GP. Then, the value of ( q - p) is
the third and fifth term is 25. Then, the sum of 4th, (a) 500 (b) 520 (c) 540 (d) 560
6th and 8th terms is equal to (JEE Main 2021)
58. In a GP the sum of three numbers is 14, if 1 is
(a) 35 (b) 30 (c) 26 (d) 32 added to first two numbers and subtracted from
50. If a , b, c be in GP, then log a n , log bn , log cn will be third number the series becomes AP, then the
greatest number is
(a) AP (b) GP
(a) 8 (b) 4 (c) 24 (d) 16
(c) HP (d) None of these
59. Suppose a , b, c are in AP and a 2 , b2 , c2 are in GP. If
51. Let an be nth term of the GP of positive numbers. 3
100 100 a < b < c and a + b + c = , then the value of a is
2
Let å a2 n = a and å a2 n - 1 = b, such that a ¹ b ,
1 1 1 1 1 1
n =1 n =1 (a) (b) (c) - (d) -
2 2 2 3 2 3 2 2
then the common ratio is
a b a b 60. In a GP, first term is 1. If 4T2 + 5T3 is minimum,
(a) (b) (c) (d)
b a b a then its common ratio is
2 2 3 3
52. The product of three consecutive terms of a GP is (a) (b) - (c) (d) -
5 5 5 5
512. If 4 is added to each of the first and the second
of these terms, the three terms now form an AP. 61. Three numbers form a GP. If the 3rd term is
Then, the sum of the original three terms of the decreased by 64, then the three numbers thus
given GP is (JEE Main 2019) obtained will constitute an AP. If the second term
(a) 36 (b) 28 (c) 32 (d) 24 of this AP is decreased by 8, a GP will be formed
again, then the numbers will be
53. If three distinct numbers a, b and c are in GP and (a) 4, 20, 36 (b) 4, 12, 36
the equations ax 2 + 2 bx + c = 0 and (c) 4, 20, 100 (d) None of these
dx 2 + 2 ex + f = 0 have a common root, then which
one of the following statements is correct?
(JEE Main 2019)
62. If AM of two numbers is twice of their GM, then the
d e f ratio of greatest number to smallest number is
(a) d , e and f are in GP (b) , and are in AP (a) 7 - 4 3 (b) 7 + 4 3
a b c
d e f (c) 21 (d) 5
(c) d , e and f are in AP (d) , and are in GP
a b c 63. Let a, b and c be the 7th, 11th and 13th terms
54. Let a, b and c be in GP with common ratio r, where respectively of a non-constant AP. If these are
1 a
a ¹ 0 and 0 < r £ . If 3a, 7b and 15c are the first also the three consecutive terms of a GP, then
2 c
three terms of an AP, then the 4th term of this is equal to (JEE Main 2019)

AP is (JEE Main 2019) 7


(a) 2 (b)
2 7 13
(a) 5a (b) a (c) a (d) a
3 3 1
(c) 4 (d)
2
55. The value of common ratio for which three
successive terms of a GP are the sides of a triangle is 64. If x, y and z are in AP and tan - 1 x, tan - 1 y and
(a) r > 0 (b) r Î (0, 1) tan - 1 z are also in AP, then (JEE Main 2013)
æ 5 - 1 5 + 1ö
(c) r Î ç , ÷ (d) None of these (a) x = y = z (b) 2x = 3 y = 6z
è 2 2 ø (c) 6x = 3 y = 2z (d) 6x = 4 y = 3z
56. In the four numbers first three are in GP and last
Sum of n Terms in GP
three are in AP whose common difference is 6.
If the first and last numbers are same, then first 65. If a, b and c be three distinct real numbers in GP
number will be and a + b + c = xb, then x cannot be (JEE Main 2019)
(a) 2 (b) 4 (c) 6 (d) 8 (a) 4 (b) 2 (c) -2 (d) -3
Sequence and Series 39

66. The sum of n terms of the series 75. The sum of


1.4 + 3.04 + 5.004 + 7.0004 +K is (x + 2)n - 1 + (x + 2)n - 2 (x + 1) + (x + 2)n - 3 (x + 1)2 +
4æ 1 ö 4æ 1 ö K + ( x + 1) n - 1 is equal to
(a) n 2 + ç1 + ÷ (b) n 2 + ç1 - n ÷
9è 10n ø 9è 10 ø (a) (x + 2) n-2
- (x + 1) n
(b) (x + 2)n - 1 - (x + 1)n - 1
4æ 1 ö (c) (x + 2)n - (x + 1)n (d) None of these
(c) n + ç1 - n ÷ (d) None of these
9è 10 ø 76. If S denotes the sum to infinite and Sn the sum of n
67. If the sum of the second, third and fourth terms of 1 1 1
terms of the series 1 + + + +L such that
a positive term GP is 3 and the sum of its sixth, 2 4 8
1
seventh and eighth terms is 243, then the sum of S - Sn < , then the least value of n is
the first 50 terms of this GP is (JEE Main 2020)
1000
1 49 1 50 (a) 8 (b) 9 (c) 10 (d) 11
(a) (3 - 1) (b) (3 - 1) ¥
26
2 50
26
1 50
77. If sum of the series å r n = S, for|r|< 1, then sum
(c) (3 - 1) (d) (3 - 1) n= 0
13 13 ¥
¥
68. If f ( x + y) = f ( x) f ( y) and å f ( x) = 2, x, y Î N , where
of the series å r 2n
is
n= 0
x =1
S2 2S S2
N is the set of all natural numbers, then the value (a) S 2 (b) (c) (d)
2S + 1 S2 - 1 2S -1
f ( 4)
of is (JEE Main 2020) 78. The sum of the infinite series
f ( 2)
2 7 12 17 22
2 1 1 4 1+ + + + + + K is equal to
(a) (b) (c) (d) 3 32 33 34 35
3 9 3 9
9 15 13 11
69. (666K6) 2 + (888K8) is equal to (a) (b) (c) (d)
4 4 4 4
n digits n digits
4 4 79. If 1 + cos a + cos2 a + . . . ¥ = 2 - 2 , then a , (0 < a < p)
(a) (10n - 1) (b) (10 2n - 1)
9 9 is
4 p p p 3p
(c) (10n - 1) 2 (d) None of these (a) (b) (c) (d)
9 8 6 4 4
70. If 1 + l + l2 + K + ln 80. An infinite GP has the first term ‘ x ’ and sum 5,
2 4 8 16
= (1 + l) (1 + l ) (1 + l ) (1 + l ) (1 + l ), then x belongs to
then the value of n is (where n Î N ) (a) x < -10 (b) -10 < x < 0
(a) 32 (b) 16 (c) 31 (d) 15 (c) 0 < x < 10 (d) x > 10
¥ ¥ ¥
71. If f ( x + y) = f ( x) × f ( y) for x, y Î N and f (1) = 2 , 81. If a = S x n , b = nS= 0 y n , c = nS= 0 ( xy) n , where
n=0
then the value of a for which
n |x|,|y|< 1 , then
S f ( a + k) = 16 ( 2 n - 1) is
k =1
(a) abc = a + b + c (b) ab + bc = ac + b
(c) ac + bc = ab + c (d) ab + ac = bc + a
(a) 2 (b) 3 (c) 4 (d) 5
72. The number 111...1 (91 times) is a/an Relation Among AM and GM
(a) even number (b) prime number 82. The minimum value of 4 x + 41 - x , x Î R is
(c) not prime (d) None of these (a) 2 (b) 4 (c) 1 (d) 0
n n
73. The value of the sum å å r s
Srs 2 3 , where 83. If one GM, g and two AM’s, p and q are inserted
r =1 s =1
between two numbers a and b, then
Srs = 0, if r ¹ s and Srs = 1, if r = s, is ( 2 p - q)( p - 2 q) is equal to
(5n - 1) 6 n 5n6n 5 n (a) g 2 (b) - g 2 (c) 2 g (d) 3 g 2
(a) (b) (6 - 1) (c) (d) (5 - 1)
4 5 n+1 4
84. If a, b and c are in AP and p, p ¢ are the AM and GM
74. Let n ( > 1) be a positive integer, then the largest respectively between a and b, while q, q¢ are the AM
integer m such that ( nm + 1) divides and GM respectively between b and c, then
(1 + n + n2 + . . . + n127 ), is (a) p 2 + q 2 = p 2 + q ¢2 (b) pq = p¢ q¢
(a) 32 (b) 63 (c) 64 (d) 127 2 2
(c) p - q = p ¢ - q ¢ 2 2
(d) None of these
40 JEE Main Mathematics

85. The product of n positive numbers is unity. n (n + 1)(n + 2) n (n + 1)


(a) (b)
6 2
Their sum is
1 n 2 + 3n + 2
(a) a positive integer (b) equal to n + (c) (d) None of these
n 2
(c) divisible by n (d) never less than n 96. The sum of the series
86. If 2 p + 3q + 4 r = 15, then the maximum value of 1 2 3
2 4
+ 2 4
+ + … to n terms is
1+1 +1 1+2 +2 1 + 32 + 34
p 3q5 r7 is
54 × 35 55 × 77 n (n 2+ 1) n (n + 1)
(a) 2180 (b) (c) (d) 2285 (a) (b)
215 217 × 9 n2+ n+1 2 (n 2+ n + 1)
n (n 2- 1)
87. If a + 2 b + 3c = 12 , ( a , b, c Î R + ), then ab2 c3 is (c) (d) None of these
3 6 2 (n 2+ n + 1)
(a) ³ 2 (b) ³ 2
(c) £ 26 (d) None of these 97. Some identical balls are arranged in rows to form
an equilateral triangle. The first row consists of one
Arithmetico-Geometric Progression ball, the second row consists of two balls and so on.
4 7 10 If 99 more identical balls are added to the total
88. nth term of the series 1 + + + + … will be
5 52 53 number of balls used in forming the equilateral
3n + 1 3n - 1 3n - 2 3n + 2 triangle, then all these balls can be arranged in a
(a) (b) (c) (d)
5n -1 5n 5n -1 5n -1 square whose each side contains exactly 2 balls less
89. The sum of the series 1 + 2 ´ 3 + 3 ´ 5 + 4 ´ 7 +. . . than number of balls each side of the triangle
contains. Then, the number of balls used to form
upto 11th term is [JEE Main 2019, 9 April Shift-II]
the equilateral triangle is (JEE Main 2019)
(a) 915 (b) 946 (c) 916 (d) 945
(a) 262 (b) 190
90. The sum of n terms of the infinite series (c) 225 (d) 157
1 × 32 + 2 × 5 2 + 3 × 7 2 + . . . ¥ is 98. The sum of the series 1 × 3 × 5 + 2 × 5 × 8 + 3 × 7 × 11 + . . .
n n
(a) (n + 1) (6n 2 + 14n + 7) (b) (n + 1) (2n + 1) (3n + 1) upto n terms is
6 6
n (n + 1) (9n 2+ 23n + 13) n (n - 1) (9n 2 + 23n + 12)
(c) 4n 3 + 4n 2+ n (d) None of these (a) (b)
6 6
3 5 7
91. 1 + + + . . . ¥ is equal to (n + 1) (9n 2 + 23n + 13) n (9n 2 + 23n + 13)
2 2 2 23 (c) (d)
6 6
(a) 3 (b) 6 (c) 9 (d) 12
99. The nth term of the series
92. If Tn denotes the nth term of the series 13 13 + 2 3 13 + 2 3 + 33
+ + + … will be
2 + 3 + 6 + 11 + 18 + . . . , then T50 is 1 1+3 1+3+5
(a) 49 2 - 1 (b) 49 2 n 2+ 2n + 1
(a) n 2+ 2n + 1 (b)
(c) 50 2 + 1 (d) 49 2 + 2 8
93. Sum of n terms of series 12 + 16 + 24 + 40 + . . . will be n 2+ 2n + 1 n 2- 2n + 1
(c) (d)
4 4
(a) 2 (2n - 1) + 8n (b) 2 (2n - 1) + 6n
(c) 3 (2 n- 1) + 8n (d) 4 (2n - 1) + 8n 100.For any odd integer n ³ 1,
n3 - ( n - 1) 3 + . . . + ( -1) n - 113 is equal to
Sum of n Terms of Special Series 1 1
(a) (n - 1)2 (2n - 1) (b) (n - 1)2 (2n - 1)
2 3 2 4
94. The sum of the series 1 + 3x + 6 x + 10 x + . . . ¥ will
1 1
be (c) (n + 1)2 (2n - 1) (d) (n + 1)2 (2n - 1)
1 1 1 1 2 4
(a) (b) (c) (d)
(1 - x) 2 1-x (1 + x) 2 (1 - x) 3 101.The nth term of the series
1 + 2 + 5 + 12 + 25 + K is
95. Let Sn denote the sum of the cubes of the first n
1
natural numbers and sn denote the sum of the (a) (n - 1 ) (n - 2) (b) n (n - 1 ) (n - 2) + n
3
n
Sr (c) n (d) None of these
first n natural numbers. Then, å sr
is equal to
r =1
Sequence and Series 41

Only One Correct Option 9. The sixth term of an AP is equal to 2. The value of
1. Number of identical terms in the sequence 2, 5, 8, the common difference of the AP which makes the
11, ... upto 100 terms and 3, 5, 7, 9, 11, ... upto 100 product T1 T4 T5 least, is given by
terms, are (a) 8/5 (b) 5/4
(a) 17 (b) 33 (c) 50 (d) 147 (c) 2/3 (d) None of these
-1 -1 -1
2. If positive numbers a , b , c are in AP, then the 10. After striking the floor, a certain ball rebounds
product of roots of the equation (4/5)th of height from which it has fallen. Then, the
total distance that it travels before coming to rest,
x 2 - kx + 2 b101 - a101 - c101 = 0, ( k Î R) is
if it is gently dropped from a height of 120 m is
(a) > 0 (b) < 0
(a) 1260 m (b) 600 m
(c) = 0 (d) None of these
(c) 1080 m (d) None of these
3. If a1, a2 , a3, . . . , a20 are AM’s between 13 and 67,
11. Concentric circles of radii 1, 2, 3, … , 100 cm are
then the maximum value of a1 a2 a3 . . . a20 is equal
drawn. The interior of the smallest circle is
to
coloured red and the angular regions are coloured
(a) (20) 20 (b) (40) 20 (c) (60) 20 (d) (80) 20
alternately green and red, so that no two adjacent
4. If the set of natural numbers is partitioned into regions are of the same colour. Then, the total area
subsets S1 = {1}, S2 = { 2 , 3), S3 = { 4, 5, 6 } and so on. of the green regions in sq cm is equal to
Then, the sum of the terms in S50 is (a) 1000p (b) 5050p
(a) 62525 (b) 25625 (c) 4950p (d) 5151p
(c) 62500 (d) None of these ¥ ¥
p
5. Jairam purchased a house in ` 15000 and paid
12. For 0 < q <
2
, if x = å cos2 n q , y = å sin 2 n q ,
n=0 n=0
` 5000 at once. Rest money he promised to pay in ¥

annual instalment of ` 1000 with 10% per annum


z= å cos2 n q sin 2 n q, then
n=0
interest. How much money is to be paid by Jairam?
(a) xyz = xz + y (b) xyz = xy + z
(a) ` 21555 (b) ` 20475
(c) xyz = x + y + z (d) xyz = yz + x
(c) ` 20500 (d) ` 20700
6. 150 workers were engaged to finish a piece of work 13. The sum of n terms of an AP is a n( n - 1). The sum
in a certain number of days. 4 workers dropped the of the squares of these terms is
second day, 4 more workers dropped the third day a2
(a) a 2n 2(n - 1) 2 (b) n (n - 1)(2n - 1)
and so on. It takes eight more days to finish the 6
work now. The number of days in which the work 2a 2 2a 2
(c) n (n - 1)(2n - 1) (d) n (n + 1)(2n + 1)
was completed is 3 3
(a) 15 (b) 20 (c) 25 (d) 30 8 16 128
14. Let S = + + . . . + 18 , then
5 65 2 +1
7. If the sum of the roots of the equation
1088 545
ax 2 + bx + c = 0 be equal to the sum of the (a) S = (b) S =
545 1088
reciprocals of their squares, then bc2 , ca 2 , ab2 will
1056 545
be in (c) S = (d) S =
545 1056
(a) AP (b) GP
(c) HP (d) None of these 15. The sum of the infinite terms of the series
5 9 13
8. If p, q, r are in AP and are positive, the roots of the 2 2
+ 2 2
+ 2 + … is
3 +7 7 + 11 11 + 15 2
quadratic equation px 2 + qx + r = 0 are all real for
1 1 1 1
r p (a) (b) (c) (d)
(a) -7 ³4 3 (b) -7 <4 3 18 36 54 72
p r
(c) All p and r (d) No p and r 16. Let x, y be positive real numbers and m, n positive
integers. The maximum value of the expression
42 JEE Main Mathematics

xm yn 24. If 32 sin 2 a-1, 14 and 34 - 2 sin 2 a are the first three


is (JEE Main 2019)
(1 + x 2 m ) (1 + y 2 n ) terms of an AP for some a, then the sixth term of
1 1 m+ n this AP is (JEE Main 2020)
(a) (b) 1 (c) (d)
2 4 6mn (a) 81 (b) 65
1 2 2 3 3 4 (c) 78 (d) 66
× × ×
2 2
17. The sum of 3 + 3 2 2 + 2 2 + … upto n 25. If the sum of the first 20 terms of the series
1 1 + 2 3 13 + 2 3 + 33
log x + log x + log x + … is 460, then x
terms is equal to (7 1 / 2 ) (7 1 / 3 ) (7 1 / 4 )
n -1 n is equal to (JEE Main 2020)
(a) (b)
n n+1 (a) 72 (b) 71/ 2 (c) e2 (d) 746/ 21
n+1 n+1
(c) (d) 26. The sum of all natural numbers ‘n’ such that
n+2 n
100 < n < 200 and HCF (91, n)>1 is
18. If the sum of first n terms of an AP is cn2 , then the (JEE Main 2019)
sum of squares of these n terms is (a) 3203 (b) 3303
n (4n 2 - 1) c 2 n (4n 2 + 1) c 2 (c) 3221 (d) 3121
(a) (b)
6 3 20
1
n (4n 2 - 1) c 2 n (4n 2 + 1) c 2
27. The sum å k 2 k is equal to (JEE Main 2019)
(c) (d) k=1
3 6
11 11
(a) 2 - (b) 1 -
19. The consecutive digits of a three digit number are 219 220
in GP. If the middle digit be increased by 2, then 3 21
(c) 2 - 17 (d) 2 - 20
they form an AP. If 792 is subtracted from this, 2 2
then we get the number constituting of same three 28. The sum of first 20 terms of the sequence
digits but in reverse order. Then, number is 0 .7 , 0 .77 , 0 .777 , ¼ , is (JEE Main 2013)
divisible by 7
(a) (179 - 10- 20 )
(a) 13 (b) 49 (c) 19 (d) 11 81
7
20. For an increasing AP a1, a2 , . . . , an , (b) (99 - 10- 20 )
9
if a1 + a3 + a5 = - 12 and a1a3a5 = 80, then which of 7
the following is/are true? (c) (179 + 10- 20 )
81
(a) a1 = - 10 (b) a 2 = - 1 7
(d) (99 + 10- 20 )
(c) a 3 = - 13 (d) a 5 = - 3 9
21. The sum of the first three terms of a GP is S and 29. Let the sum of the first n terms of a non-constant
their product is 27. Then all such S lie in n( n - 7)
AP a1, a2 , a3. . . . .be 50 n + A, where A is a
(JEE Main 2020) 2
(a) (- ¥ , 9] constant. If d is the common difference of this AP,
(b) [- 3, ¥ ) then the ordered pair ( d, a50 ) is equal to
(c) (- ¥ , - 9] È [3, ¥ ) (JEE Main 2019)
(d) (- ¥ , - 3] È [ 9, ¥ ) (a) (A, 50 + 46A) (b) (50, 50 + 45A)
22. If 210 + 2 9 × 31 + 2 8 × 32 + . . . . + 2 × 39 + 310 = S - 211, (c) (50, 50 + 46A) (d) (A, 50 + 45A)
10
then S is equal to (JEE Main 2020) 30. Let å f ( a + k) = 16 ( 210 - 1), where the function f
311 k=1
(a) + 210 (b) 311
2 satisfies f ( x + y) = f ( x) f ( y) for all natural numbers
(c) 2 × 311 (d) 311 - 212 x, y and f (1) = 2. Then, the natural number ‘a’ is
23. If 1 + (1 - 2 2 × 1) + (1 - 4 2 × 3) + (1 - 6 2 × 5) + (JEE Main 2019)
2
L + (1 - 20 × 19) = a - 220 b, then an ordered pair (a) 2 (b) 4
(c) 3 (d) 16
(a , b) is equal to (JEE Main 2020)
(a) (11, 97) (b) (10, 103) 31. Let Sn = 1 + q + q2 + K + qn and
2 n
(c) (10, 97) (d) (11, 103) æ q + 1ö æ q + 1ö æ q + 1ö
Tn = 1 + ç ÷+ç ÷ +K+ ç ÷ , where q is
è 2 ø è 2 ø è 2 ø
Sequence and Series 43

a real number and q ¹ 1. 15 (12 + 2 2 + . . . + 5 2 )


+ + . . . up to 15 terms is
If 101C1 + 101C2 × S1 + K + 101C101 × S100 = aT100 , then a 11 (JEE Main 2019)
is equal to (JEE Main 2019) (a) 7510 (b) 7820 (c) 7830 (d) 7520
100
(a) 2 (b) 202
(c) 200 (d) 299 Numerical Value Types Questions
3 3 3 3 3
1 +2 1 + 2 +3 æ1 1 1 ö
32. The sum of series 1 + + + ... log 2.5 ç + +
è 3 32 33
+ ... to ¥ ÷
ø
1+ 2 1+ 2 +3 36. The value of (0.16) is equal to
3 3 3
1 + 2 + 3 + K + 15 1 3 ...... . (JEE Main 2020)
+ - (1 + 2 + 3 + K + 15)
1 + 2 + 3 + K + 15 2
37. If m arithmetic means (AMs) and three geometric
is equal to (JEE Main 2019) means (GMs) are inserted between 3 and 243 such
(a) 620 (b) 660 (c) 1240 (d) 1860 that 4th AM is equal to 2nd GM, then m is equal to
(JEE Main 2020)
33. Let a1, a2 , a3, . . . be a GP such that a1 < 0,
9 20
38. The sum å (1 + 2 + 3 + K + k) is
a1 + a2 = 4 and a3 + a4 = 16. If å ai = 4l, then l is k= 1 (JEE Main 2020)
i =1

equal to (JEE Main 2020)


7
n( n + 1) ( 2 n + 1)
511
39. The sum, å 4
is equal to ....... .
(a) - 171 (b) 171 (c) (d) - 513 n=1 (JEE Main 2020)
3
34. If the sum of the first 40 terms of the series, 40. The number of terms common to the two APs 3, 7,
3 + 4 + 8 + 9 + 13 + 14 + 18 + 19 + . . . is (102)m, then 11, …, 407 and 2, 9, 16, ...., 709 is ..... .
(JEE Main 2020)
m is equal to (JEE Main 2020)
(a) 10 (b) 25 (c) 5 (d) 20 41. If the arithmetic mean and geometric mean of the
pth and qth terms of the sequence - 16, 8, - 4, 2,
35. The sum of the following series
............ satisfy the equation 4 x 2 - 9 x + 5 = 0, then
9 (12 + 2 2 + 32 ) 12 (12 + 2 2 + 32 + 4 2 )
1+6 + + p + q is equal to (JEE Main 2021)
7 9

Answers
Round I
1. (c) 2. (c) 3. (c) 4. (b) 5. (c) 6. (d) 7. (b) 8. (c) 9. (b) 10. (c)
11. (b) 12. (b) 13. (c) 14. (c) 15. (d) 16. (c) 17. (b) 18. (a) 19. (d) 20. (a)
21. (d) 22. (d) 23. (c) 24. (b) 25. (b) 26. (c) 27. (b) 28. (b) 29. (c) 30. (d)
31. (a) 32. (a) 33. (d) 34. (c) 35. (b) 36. (d) 37. (b) 38. (c) 39. (b) 40. (c)
41. (b) 42. (a) 43. (d) 44. (a) 45. (a) 46. (b) 47. (a) 48. (b) 49. (a) 50. (a)
51. (a) 52. (b) 53. (b) 54. (c) 55. (c) 56. (d) 57. (c) 58. (a) 59. (d) 60. (b)
61. (c) 62. (b) 63. (c) 64. (a) 65. (b) 66. (b) 67. (b) 68. (d) 69. (b) 70. (c)
71. (b) 72. (c) 73. (b) 74. (c) 75. (c) 76. (d) 77. (d) 78. (c) 79. (d) 80. (c)
81. (c) 82. (b) 83. (b) 84. (c) 85. (d) 86. (c) 87. (c) 88. (c) 89. (b) 90. (a)
91. (d) 92. (d) 93. (d) 94. (d) 95. (a) 96. (b) 97. (b) 98. (a) 99. (c) 100. (d)
101. (b)

Round II
1. (b) 2. (b) 3. (b) 4. (a) 5. (c) 6. (c) 7. (a) 8. (a) 9. (c) 10. (c)
11. (b) 12. (b) 13. (c) 14. (a) 15. (d) 16. (c) 17. (b) 18. (c) 19. (b) 20. (a)
21. (d) 22. (b) 23. (d) 24. (d) 25. (a) 26. (d) 27. (a) 28. (c) 29. (a) 30. (c)
31. (a) 32. (a) 33. (a) 34. (d) 35. (b) 36. 4.0 37. 39 38. 1540 39. 504 40. 14
41. 10
Solutions
Round I 40
5. Given, 3600 = [ 2a + (40 - 1) d ]
æ a a K an ö æ a1a 2 K a n - 1 ö 2
1. a1a 2 K a n = bn ç 1 2 ÷ = a nbn ç ÷
è bn ø è bn ø Þ 3600 = 20 (2a + 39 d )
é 1 1 ù Þ 180 = 2a + 39 d …(i)
n-1 n-1 æ a a K an - 1 ö
= ê (x) 2 - ( y) 2 úç 1 2 ÷ After 30 instalments one-third of the debt is unpaid.
ê úè bn ø 3600
ë û Hence, = 1200 is unpaid and 2400 is paid.
æ a1a 2 K a n - 2 ö x- y 3
= bn - 1 × a n - 1 ç ÷ =K= 30
è bn ø bn Now, 2400 = {2a + (30 - 1) d }
2
2. Since, the given equation is cubic, therefore we take \ 160 = 2a + 29 d …(ii)
three roots. On solving Eqs. (i) and (ii), we get
Let the roots be a - d , a , a + d. a = 51, d = 2
Sum of three numbers in AP = 3a = 12 Now, the value of 8th instalment
Þ a = 4 is a root. = a + (8 - 1) d
\The given equation x 3 - 12x2 + 39x - 28 = 0 can be = 51 + 7.2 = ` 65
rewritten as m (2 b - a )
(x - 4) (x2 - 8x + 7 ) = 0 6. mth mean between a and 2 b is a + .
n+1
\ x = 1, 4, 7 or 7, 4, 1 m (b - 2 a )
and mth mean between 2 a and b is 2 a + ×
\ d = ±3 n+1
3. Let the number of sides of the polygon be n. Then, the According to the given condition,
sum of interior angles of the polygon m (2 b - a ) m (b - 2a )
p a+ =2a +
= (2n - 4) = (n - 2) p n+1 n+1
2 Þ m (2 b - a ) = a (n + 1 ) + m (b - 2 a )
Since, the angles are in AP and a = 120° , d = 5
n Þ a (n - m + 1 ) = bm
Therefore, S n = [2a + (n - 1 )d ] a m
2 Þ =
n b n -m + 1
Þ [2 ´ 120 + (n - 1)5 ] = (n - 2) 180
2 7. Here, n = 20,S n = 66000, d = 200
Þ n 2 - 25n + 144 = 0
Let first term is a.
Þ (n - 9) (n - 16) = 0 n
Þ n = 9, 16 Q Sn =[2a + (n - 1 )d ]
2
Take n = 16 20
\ 66000 = [2 ´ a + (20 - 1 )200]
T16 = a + 15d = 120°+15(5° ) = 195°, which is impossible, 2
an interior angle cannot be greater than 180°. Þ 6600 = 2a + 19 ´ 200
Hence, n =9 Þ 2a = 6600 - 3800
2 2 2
4. Now, we assume (b - c) , (c - a ) , (a - b) are in AP, then Þ 2a = 2800
we have Þ a = 1400
2 2 2 2
(c - a ) - (b - c) = (a - b) - (c - a ) Hence, he save in the first year is ` 1400.
Þ (b - a ) (2 c - a - b) = (c - b) (2 a - b - c) …(i) 8. Clearly, the series is 5, 7, 9, 11, …
1 1 1 It is an AP, where a = 5 and d = 2, S n = 192
Also, if , , are in AP, then
b - c c- a a - b n
1 1 1 1 Þ S n = [2a + (n - 1 )d ]
- = - 2
c- a b - c a - b c- a n
b + a -2 c c+ b -2 a Þ 192 = [2 ´ 5 + (n - 1 )2]
Þ = 2
(c - a ) (b - c) (a - b) (c - a ) n
Þ 192 = ´ 2(5 + n - 1 )
Þ (a - b) (b + a - 2 c) = (b - c) (c + b - 2 a ) 2
Þ (b - a ) (2 c - a - b) = (c - b) (2 a - b - c) Þ 192 = n (n + 4)
which is equal to Eq.(i), so our hypothesis is true. Þ 192 = n 2 + 4n
Sequence and Series 45

Þ n 2 + 4n - 192 = 0 1æ1 1 ö 4000


= ç - ÷= = 10 (given)
Þ 2
n + 16n - 12n - 192 = 0 d è a1 a 4001 ø a1a 4001
Þ n (n + 16) - 12(n + 16) = 0 Þ a1a 4001 = 400 …(i)
Þ (n + 16)(n - 12) = 0 a1 + a 4001 = a 2 + a 4000 = 50 …(ii)
Þ n = - 16, 12 \ (a1 - a 4001 )2 = (a1 + a 4001 )2 - 4a1a 4001
But n ¹ - 16 = (50)2 - 1600
Q Number of days are always positive. Þ |a1 - a 4001|= 30
\Required number of days = 12
12. Since, a1 , a 2, a3 ,... , a n are in AP.
9. Here, n = 16,S n = 8000 Þ d = a 2 - a1 = a3 - a 2 = . . . = a n - a n - 1
Since, we consider last place team as a first term, \ sin d (cosec a1cosec a 2 + ... + cosec a n - 1 cosec a n )
a = 275 sin (a 2 - a1 ) sin (a n - a n - 1 )
= + ... +
Let common difference be d. sin a1 sin a 2 sin a n - 1 sin a n
n (sin a 2 cos a1 - cos a 2 sin a1 )
Q S n = [2a + (n - 1 )d ] =
2 sin a1 sin a 2
16 (sin a n cos a n - 1 + cos a n sin a n - 1 )
\ 8000 = [2 ´ 275 + (16 - 1 )d ] +K+
2 sin a n - 1 sin a n
Þ 1000 = (550 + 15d )
= (cot a1 - cot a 2) + (cot a 2 - cot a3 )
Þ 15d = 450 + K + (cot a n - 1 - cot a n )
Þ d = 30 = cot a1 - cot a n
Now, T16 = 275 + (16 - 1 )30
13. Given, xn = xn + 1 2
= 275 + 450
= 725 \ x1 = x2 2 , x2 = x3 2 , ... , xn = xn + 1 2
x1
\The first place team will get the prize ` 725. On multiplying x1 = xn + 1 ( 2 )n Þ xn + 1 =
( 2 )n
10. Since, q1 , q2, q3 , . . . , qn are in AP. x1
Hence, xn =
Þ q2 - q1 = q3 - q2 = . . . = qn - qn - 1 = d …(i) ( 2 )n - 1
Now, taking only first term x12
sin d sin (q2 - q1 ) Area of S n = xn2 = n -1
<1
sin d sec q1 sec q2 = = 2
cos q1 cos q2 cos q1 cos q2
Þ 2n - 1 > x12 (Q x1 = 10)
[from Eq. (i)]
\ 2n - 1 > 100
sin q2 cos q1 - cos q2 sin q1
= But 27 > 100,28 > 100 etc.
cos q1 cos q2
sin q2 cos q1 cos q2 sin q1 \ n - 1 = 7, 8, 9, K Þ n = 8, 9, 10, K
= -
cos q1 cos q2 cos q1 cos q2 14. Since a , b, c are in AP.
= tan q2 - tan q1 \ 2b = a + c
Similarly, we can solve other terms which will be Þ 2 b x = (a + c) x for all x
tan q3 - tan q2, tan q4 - tan q3 , . . .
Þ 2 b x + 20 = (a + c) x + 20 for all x
\ sin d (sec q1 sec q2 + sec q2 sec q3
Þ 2 (b x + 10) = (ax + 10) + (cx + 10)
+ . . . + sec qn - 1 sec qn )
\ 102(b x + 10) = 10( a x + 10) + ( c x + 10)
= tan q2 - tan q1 + tan q3 - tan q2
Þ (10b x + 10 )2 = 10a x + 10 × 10c x + 10
+ . . . + tan qn - tan qn - 1
Þ 10a x + 10 , 10b x + 10 , 10c x + 10 are in GP for all x
= - tan q1 + tan qn = tan qn - tan q1
1 1 1 15. Let tn be the nth term of given AP. Then, we have t19 = 0
11. Now, + + ... +
a1a 2 a 2a3 a 4000a 4001 Þ a + (19 - 1)d = 0 [Q tn = a + (n - 1)d ]
1 æ a - a1 a3 - a 2 a - a 4000 ö Þ a + 18d = 0 …(i)
= ç 2 + + ... + 4001 ÷ t49 a + 48d - 18d + 48d
d è a1a 2 a 2a3 a 4000a 4001 ø Now, = = [using Eq. (i)]
t29 a + 28d - 18d + 28d
1æ1 1 1 1 1 1 ö
= ç - + - + ... + - ÷ 30d
d è a1 a 2 a 2 a3 a 4000 a 4001 ø = = 3 :1
10d
46 JEE Main Mathematics

16. Q a 2, b2, c2 are in AP. Þ a1 + 5d = 0 …(i)


Þ 2 2
sin B - sin A = sin C - sin B2 2 Now, it is given that
Þ sin C (sin B cos A - cos B sin A ) a1 + a3 + a5 + .... + a 23 = ka1
12
= sin A (sin C cos B - cos C sin B ) Þ [2a1 + (12 - 1) (2d )] = ka1
2
On dividing by sin A sin B sin C , we get é æ a öù
Þ 6 ê2a1 + 22ç - 1 ÷ ú = 5a1 [from Eq. (i)]
2 cot B = cot A + cot C ë è 5 øû
Þ cot A , cot B, cot C are in AP. é 10a1 - 22a1 ù 72
Þ 6ê úû = 5a1 Þ k = - 5
17. Let A1 , A2,... , Am be m arithmetic means between 1 ë 5
and 31. Hence, option (d) is correct.
Þ Then A1 , A2,... , Am 31 is an AP with common 20. Let the common difference of the AP having first term as
diffference. 3 is ‘d’, so according to the question
31 - 1 30 é b-aù S 25 = S 40 - S 25 Þ 2S 25 = S 40
d= = êëQ d = n + 1 úû
m+1 m+1 25
Þ 2´ [(2 ´ 3) + (25 - 1)d ]
\ A7 = 1 + 7d 2
7 ´ 30 m + 211 40
=1 + = = [(2 ´ 3) + (40 - 1)d ]
m+1 m+1 2
Þ 5 [6 + 24d ] = 4 [6 + 39d ]
and Am-1 = 1 + (m - 1)d
1
30 Þ 30 + 120d = 24 + 156d Þ 36d = 6 Þ d =
=1 + (m - 1) 6
m+1
Hence, option (a) is correct.
31m - 29
= 21. For the given AP.
m+1
n n
A7 5 Sn = [a1 + a n ] = [1 + 300] …(i)
Now, = (given) 2 2
Am-1 9
As, a1 = 1 and a n = 300
m ´ 211 5
= and 300 = 1 + (n – 1)d, where d is the common
31m - 29 9 difference of AP.
Þ 9m + 1899 = 155m - 145 299 23 ´ 13
Þ d= =
Þ 146m = 2044 n -1 n -1
Þ m = 14 Q d is an integer so n - 1 = 23, or 13
18. Q a1 , a 2, a3 , ... , a 2n are in AP. Þ n = 24 or 14
As it is given that, 15 £ n £ 50
\ a 2 - a1 = a3 - a 2 = K = d
So, n = 24 Þ d = 13
\ a12 - a 22 + a32 - a 42 + ... + a 22n - 1 - a 22n a n - 4 = { a + (n - 1) d }
= (a1 - a 2) (a1 + a 2) + (a3 - a 4 ) (a3 + a 4 ) a 20 = {1 + (20 - 1) ´ 13}
+ K + (a 2n - 1 - a 2n ) (a 2n - 1 + a n ) = 1 + 19 ´ 13
= (- d ) (a1 + a 2) + (- d ) (a3 + a 4 ) a 20 = 248
+ K + (- d ) (a 2n - 1 + a 2n ) 20
\ S n- 4 = S 20 = [1 + 248] = 10 ´ 249 = 2490
= (- d ) (a1 + a 2 + K + a 2n ) 2
é 2n ù 22. Let first term and common difference of an AP is ‘a’ and
= (- d ) ê (a1 + a 2n )ú
ë 2 û ‘d ’ respectively, then
1
æ a - a1 ö é a 2n - a1 ù a10 = a + 9d = (given) …(i)
= - ç 2n ÷ [n (a1 + a 2n )] êQ d = ú 20
è 2n - 1 ø ë 2n - 1 û
n 1
= (a12 - a 22n ) and a 20 = a + 19d = (given) …(ii)
2n - 1 10
From Eqs. (i) and (ii), we get
19. Let the ‘d’ is the common difference of the AP
1 1 1 1
a1 , a 2, a3 ,..... (a1 ¹ 0), so sum of first eleven terms is 10d = - = Þd =
10 20 20 200
S11 = 0 [given] 1 9 1
11 and a= - =
Þ [2a1 + (11 - 1) d ] = 0 20 200 200
2
Sequence and Series 47

\ Sum of first 200 terms 27. Let a be the first term and d be the common difference.
200 é æ 1 ö 1 ù Then, we have a + d, a + 4d, a + 8 d in GP,
S 200 = ê ç2 ´ ÷ + (200 - 1)
2 ë è 200 ø 200 úû i.e. (a + 4d ) 2 = (a + d ) (a + 8 d )
1 201 1 Þ a + 16 d 2 + 8ad = a 2 + 8ad + ad + 8 d 2
2
= [2 + 200 - 1] = = 100
2 2 2 Þ 8 d 2 = a d Þ 8d = a [Q d ¹ 0]
23. Let first three terms of an AP as a - d, a, a + d. Now, common ratio,
a + 4d 8 d + 4d 12 d 4
So, 3a = 33 Þ a = 11 r= = = =
a+d 8d + d 9d 3
[given sum of three terms = 33
and product of terms = 1155] 28. Let the common difference be d.
Þ (11 - d )11 (11 + d ) = 1155 [given] Sum of first p terms = 0
Given, Sp = 0
Þ 112 - d 2 = 105
p
Þ 2
d = 121 - 105 = 16 Þ [2a + ( p - 1 )d ] = 0
2
Þ d = ±4 Þ 2a + ( p - 1 )d = 0
So the first three terms of the AP are either 7, 11, 15 2a
or 15, 11, 7. Þ d=- …(i)
p-1
So, the 11th term is either 7 + (10 ´ 4) = 47
\Required sum of next q terms Sum of first
or 15 + (10 ´ (-4)) = - 25
( p + q) terms - Sum of first p terms
24. Since, log 2 , log(2n - 1 ) and log (2n + 3) are in AP. = Sp + q - Sp
\ 2 log (2n - 1 ) = log 2 + log (2n + 3) p+ q
= [2a + ( p + q - 1 )d ] - 0
Þ (2n - 1 )2 = 2 (2n + 3) 2
Þ (2n - 5) (2n + 1 ) = 0 p+ q é æ -2 a ö ù
= ê2a + ( p + q - 1 ) ç ÷ ú [from Eq. (i)]
n
As 2 cannot be negative hence, 2 ë è p - 1øû
2n - 5 = 0 p+ q é1 p + q - 1ù
Þ 2n = 5 Þ n = log 2 5 = ´ 2a ê - ú
2 ë1 p-1 û
25. Here, T1 = S1 = 2 (1 ) + 3 (1 )2 = 5 é p - 1 - p - q + 1ù
= ( p + q) a ê ú
T2 = S 2 - S1 = 16 - 5 = 11[Q S 2 = 2(2) + 3 (2)2 = 16] ë p-1 û
T3 = S3 - S 2 = 33 - 16 = 17 - a ( p + q)q
=
[Q S3 = 2(3) + 3 (3)2 = 33 ] p-1
Hence, sequence is 5 , 11 , 17.
29. Clearly, the two-digit number which leaves remainder
\ a = 5 and d = 6
2 when divided by 7 is of the form N = 7k + 2
For new AP,
[by Division Algorithm]
A = 5 , D = 2 ´ 6 = 12 For, k = 2, N = 16
n k = 3, N = 23
\ S ¢n = [2 ´ 5 + (n - 1 )12]
2
M M
= 6 n2 - n k = 13, N = 93
26. Let S n and S¢ be the sums of n terms of two AP’s and T11 \12 such numbers are possible and these numbers
and T ¢11 be the respective 11th term, then forms an AP.
n 12 æ n ö
[2 a + (n - 1 ) d ] Now, S = [16 + 93] = 654 çQ S n = (a + l)÷
Sn 2 7n + 1 2 è 2 ø
= = (given)
S n ¢ n [2a ¢ + (n - 1 ) d ¢ ] 4 n + 27 Similarly, the two digit number which leaves
2 remainder 5 when divided by 7 is of the form
(n - 1 ) N = 7k + 5
a+ d
2 7n + 1
Þ = For k = 1, N = 12
(n - 1 )
a¢ + d ¢ 4 n + 27 k = 2, N = 19
2
M
Now, put n = 21, we get
k = 13, N = 96
a + 10 d T 148 4
= 11 = = \13 such numbers are possible and these numbers
a ¢ + 10 d ¢ T ¢11 111 3
also forms an AP.
48 JEE Main Mathematics

13 æ n ö Þ 2na + 2n [6n - 1] = 1000


Now, S¢ = [12 + 96] = 702 çQ S n = (a + l)÷
2 è 2 ø 1000
Þ 2a + (6n - 1) d =
Total sum = S + S ¢ = 654 + 702 = 1356 n
6n 1000
30. Given that, Now, S 6n = [2a + (6n - 1) d ] = 3n × = 3000
2 n
a1 + a5 + a10 + a15 + a 20 + a 24 = 225
34. Statement I Since, nth term of an AP is a n.
Þ (a1 + a 24 ) + (a5 + a 20 ) + (a10 + a15 ) = 225
an + k + an - k é a + bù
Þ 3(a1 + a 24 ) = 225 \ an = êëQ AM = 2 úû
2
Þ a1 + a 24 = 75 …(i)
Statement II Let a be the first term and d be the
[Q in an AP the sum of the terms equidistant from
the beginning and the end is same and is equal common difference of an AP.
to the sum of the first and last term] According to the given condition,

\
n
a1 + a 2 + ... + a 24 = (a + l ) =
24
(a1 + a 24 ) Sm = S n
2 2 m n
\ [2a + (m - 1 )d ] = [2a + (n - 1 )d ]
= 12 ´ 75 = 900 [from Eq. (i)] 2 2
31. Since, a1 , a 2, a3 ,... , a n are in AP. Þ (m - n )2a + (m2 - n 2 - m + n )d = 0
Then, a 2 - a1 = a3 - a 2 = ... = a n - a n - 1 = d Þ (m - n )[2a + (m + n - 1 )d ] = 0 …(i)
where d is the common difference of the given AP. m+ n
\ S(m + n) = [2a + (m + n - 1 )d ] = 0
Also, a n = a1 + (n - 1 ) d 2
m+ n
Then, by rationalizing each term = (0) [Q from Eq. (i)]
1 1 1 2
+ + ... + =0
a 2 + a1 a3 + a 2 a n + a n -1
35. The sum of integers from 1 to 100 that are divisible by
a 2 - a1 a3 - a 2 a n - a n -1
= + + ... + 2 or 5
a 2 - a1 a3 - a 2 a n - a n -1 = Sum of series divisible by 2+ Sum of series divisible
=
1
( a 2 - a1 + a3 - a 2 + ... + a n - a n - 1 ) by 5 - Sum of series divisible by 2 and 5
d = (2 + 4 + 6+ ...+100) + (5 + 10 + 15 + ... + 100)
1 a n + a1 - (10 + 20 + 30 + ... + 100)
= ( a n - a1 ) ´
d a n + a1 50 20
= {2 ´ 2 + (50 - 1 ) 2} + {2 ´ 5 + (20 - 1 ) 5}
2 2
1 æ a n - a1 ö 1 é (n - 1 ) d ù n -1 10
= ç ÷= ê ú= - { 10 ´ 2 + (10 - 1 ) 10}
d è a n + a1 ø d ë a n + a1 û a n + a1 2
= 25 (102) + 10 (105) - 5 (110)
32. Since, a1 , a 2, a3 ,... , a n form an AP.
= 2550 + 1050 - 550 = 3050
\ a 2 - a1 = a 4 - a3 = ... = a 2n - a 2n -1 = d
Let S = a12 - a 22 + a32 - a 42 + ... + a 22n - 1 - a 22n 36. Let S = 1 + 2 + 3 + ... + 100
100
= (a1 - a 2) (a1 + a 2) + (a3 - a 4 ) (a3 + a 4 ) = (1 + 100) = 50 (101 ) = 5050
2
+ K + (a 2n - 1 - a 2n ) (a 2n - 1 + a 2n )
Let S1 = 3 + 6 + 9 + 12 + ... + 99
é 2n ù
= - d (a1 + a 2 + ... + a 2n ) = - d ê (a1 + a 2n )ú …(i) = 3 (1 + 2 + 3 + 4 + ... + 33)
ë 2 û
33
Also, we know a 2n = a1 + (2n - 1 ) d = 3× (1 + 33) = 99 ´ 17 = 1683
2
a - a1 a - a 2n
Þ d = 2n Þ -d= 1 Let S 2 = 5 + 10 + 15 + ... + 100
2n - 1 2 n -1
= 5 (1 + 2 + 3 + ... + 20)
On putting the value of d in Eq. (i), we get
20
n (a1 - a 2n ) (a1 + a 2n ) n = 5× (1 + 20) = 50 ´ 21 = 1050
S= = (a12 - a 22n ) 2
2n - 1 2n - 1
Let S3 = 15 + 30 + 45 + ... + 90
2n 4n
33. S 2n = [2a + (2n - 1) d ], S 4n = [2a + (4n - 1) d ] = 15 (1 + 2 + 3 + ... + 6)
2 2
6
4n 2n = 15 × (1 + 6) = 45 ´ 7 = 315
Þ S 2 - S1 = [2a + (4n - 1) d ] - [2a + (2n - 1) d ] 2
2 2 \ Required sum = S - S1 - S 2 + S3
= 4an + (4n - 1) 2nd - 2na - (2n - 1) dn = 5050 - 1683 - 1050 + 315
= 2na + nd [8n - 2 - 2n + 1] = 2632
Sequence and Series 49

37. Since, a , b, c are in GP. Þ (ap - b)2 + (bp - c)2 + (cp - d )2 = 0


Þ b2 = ac Þ ap - b = 0 = bp - c = cp - d
b c d
and log a - log 2 b, log 2 b - log 3c and log 3c - log a are Þ p= = =
a b c
in AP.
Þ a , b, c, d are in GP.
Þ 2 (log 2 b - log 3c) = log a - log 2 b + log 3c - log a
\ b2 = ac and 2 b = 3c 41. Since, x, 2 y and 3z are in AP.

Þ b=
2a
and c =
4a \ 4 y = x + 3z …(i)
3 9 And x, y, z are in GP.
5a 10a \ y = rx and z = xr 2
Since, a+b= > c, b + c = >a,
3 9
On putting the values of y and z in Eq. (i), we get
13a
c+ a = >b 4xr = x + 3xr 2
9
Þ 3r 2 - 4r + 1 = 0
It implies that a , b, c form a triangle with a as the
greatest side. Þ (3r - 1 )(r - 1 ) = 0
1
Now, let us find the greatest ÐA of D ABC by using the Þ r = ,1
cosine formula. 3
4a 2 16a 2 1
+ - a2 \ r= [Q r ¹ 1]
2 2 2 3
b + c -a 29
cos A = = 9 81 =- <0
2 bc 4a 4a 48 42. Let the edges of rectangular block are
×
3 9 a , ar , ar 2 …(i)
\ The ÐA is obtuse. Now, volume = 216 cm3
38. Q a , b, g , d are in GP. a (ar )(ar 2) = 216 [Q volume of cuboid = l ´ b ´ h]
Let a = A, b = Ar, g = Ar 2, d = Ar3 Þ (ar )3 = 216
Q a and b are the roots of the equation x2 - 3x + a = 0, Þ ar = 6 cm …(ii)
then And total surface area = 252 cm2
a + b = 3 Þ A (1 + r ) = 3 … (i) 2[a (ar ) + ar (ar 2) + a (ar 2)] = 252
g and d are the roots of the equation x2 - 12x + b = 0, \From Eq. (ii),
then
2(6a + 36r + 36) = 252
g + d = 12
Þ 12(a + 6r + 6) = 252
Þ A r 2(1 + r ) = 12 … (ii)
Þ a + 6r = 15 …(iii)
On solving Eqs. (i) and (ii), we get æ6ö
Þ a + 6 ´ ç ÷ = 15 [Q from Eq. (ii)]
A = 1, r = 2 èaø
Þ a = 1, b = 2 , g = 4, d = 8 Þ a 2 - 15a + 36 = 0
\ a = ab = 1 ´ 2 = 2 Þ (a - 12)(a - 3) = 0
and b = gd = 4 ´ 8 = 32 Þ a = 3, 12
39. Q b, a , c are in AP. \From Eq. (iii),
\ 2a = b + c …(i) when a = 3, r = 2
and b, G1 , G2, c are in GP. 1
when a = 12, r =
\ G1 = br, G2 = br 2 and c = br3 , 2
where r be the common ratio of GP. On putting above values in Eq. (i), we get
Now, G13 + G32 = (br )3 + (br 2)3 Edges are 3, 6, 12 or 12, 6, 3.
2 Hence, the length of the longest edge is 12 cm.
æ cö æ cö
= b3 r3 + b3 r 6 = b3 ç ÷ + b3 ç ÷
è bø è bø 43. Let r be the common ratio of given GP, then we have the
2 2 following sequence
= b c + bc = bc(b + c) = 2abc
a1 , a 2 = a1r , a3 = a1r 2, ... , a10 = a1r 9
\ G13 + G32 = 2abc
Now, a3 = 25 a1
40. Given equation, Þ a1r 2 = 25 a1 Þ r 2 = 25
(a 2 + b2 + c2) p2 - 2 (ab + bc + cd ) p + (b2 + c2 + d 2) = 0 a 9 a1r 8
Þ (a 2p2 - 2apb + b2) + (b2p2 - 2bpc + c2) Consider, = = r 4 = (25)2 = 54
a5 a1r 4
+ (c2p2 - 2cpd + d 2) = 0
50 JEE Main Mathematics

44. Let A be the arithmetic mean between two numbers a


47. Let , a and a r be the roots of the equation
a and b. Again, let G1 and G2 be the two geometric mean r
between two numbers a and b. ax3 + bx2 + cx + d = 0.
a+b
\ A= and G1 = ar ,G2 = ar 2, a b
2 \ Sum of the roots, + a + ar = -
1 r a
1/3
æ bö 2+ 1 æ bö æ1 ö b
where r=ç ÷ =ç ÷ …(i) Þ a ç + 1 + r÷ = - …(i)
èaø èaø èr ø a
G12 G22 2 2
a r a r 2 4
\ + = 2
+ Sum of product of the roots taken two at a time,
G2 G1 ar ar a a c
× a + a × ar + ar × =
æ bö r r a
= a + ar3 = a + a ç ÷ [Q from Eq. (i)]
èaø
2æ 1 ö c
é a + bù Þ a ç + r + 1÷ = …(ii)
= a + b = 2A èr ø a
êëQ A = 2 úû
a d
Product of the roots, × a× a r = -
45. Let first term be A and common ratio be R. r a
Given, pth term, T p = q and qth term, Tq = p. 3 d
Þ a =- …(iii)
AR p - 1 = q and ARq - 1 = p …(i) a
AR p - 1 q On dividing Eqs. (ii) by (i), we get
\ =
ARq - 1 p æ1
a 2ç + r + 1÷
ö c
1 èr ø a
p-q q æ qö p- q =
Þ R = ÞR= ç ÷ æ1 ö b
p è pø a ç + 1 + r÷ -
èr ø a
On putting the value of R in Eq. (i), we get c
p -1 p -1 Þ a=- …(iv)
æ qö p-q æ pö p-q
b
A×ç ÷ = qÞ A = q×ç ÷ From Eqs. (iii) and (iv),
è pø è qø
3
Now, ( p + q)th term, T p + q = AR p + q - 1 æ cö d c3 d c3 d
ç- ÷ = - Þ - =- Þ =
p -1 p + q -1 è bø a b3 a b3 a
æ pö p - q æ q ö p-q
Þ c3 × a = d × b3
= q×ç ÷ ´ç ÷
è qø è pø
48. Given, x, y, z are in GP and a x = by = cz
p -1 p + q -1 p - q - p + 1 + p + q -1
1- +
p-q p-q p-q Since, x, y, z are in GP.
q q
= p + q -1 p -1
= p + q -1 - p + 1
- Þ y2 = xz …(i)
p-q p-q p-q
p p We have,
p 1 1
a x = by = cz = k (say); k > 0
qp-q æ q p ö p - q æ q p ö ( p - q)
= q
= ç q÷ = ç q÷ On taking log both sides, we get
èp ø èp ø
p p-q x log a = y log b = z log c = log k
log k log k log k
46. We have, Þ x= ,y= ,z=
log a log b log c
(a 2 + b2 + c2) p2 - 2(ab + bc + cd ) p + (b2 + c2 + d 2) £ 0 …(i)
On substituting the values of x, y and z in Eq. (i),
LHS = (a 2p2 - 2abp + b2) + (b2p2 - 2bcp + c2)
we get
+ (c2p2 - 2cdp + d 2) 2
æ log k ö log k log k
= (ap - b)2 + (bp - c)2 + (cp - d )2 ³ 0 …(ii) ç ÷ = ×
è log b ø log a log c
Since, the sum of squares of real number is
non-negative. (log k)2 (log k)2
Þ =
From Eqs. (i) and (ii), we get (log b) 2
log a × log c

(ap - b)2 + (bp - c)2 + (cp - d )2 = 0 Þ (log b)2 = log a × log c


Þ ap - b = 0 = bp - c = cp - d log a log b
Þ =
b c d log b log c
Þ = = =p
a b c é log e x ù
\ logb a = log c b êQ log y = log y xú
\ a , b, c and d are in GP. ë e û
Sequence and Series 51

49. Let a be the first term and r be the common ratio of GP. 8 æ2 ö
Þ 24 = + 8r + 4 Þ 20 = 4 ç + 2r ÷
25 r èr ø
Þ ar + ar5 =
2 2
Þ 5 = + 2r Þ 2r 2 - 5r + 2 = 0
Þ ar 2 ´ ar 4 = 25 r
Þ a 2r 6 = 25 Þ 2r 2 - 4r - r + 2 = 0
Þ ar3 = 5 Þ 2r (r - 2) - 1(r - 2) = 0
5 Þ (r - 2) (2r - 1) = 0
Þ a= 3
r 1
Þ r = 2,
5r 5r5 25 2
Þ + 3 =
r3 r 2 \ Three consecutive terms are
1 2 5 8
Þ +r = , 8, 8 (2) Þ 4, 8, 16
r2 2 2
Put r2 = t Sum of these terms = 4 + 8 + 16 = 28
t2 + 1 5
Þ = 53. Given, three distinct numbers a , b and c are in GP.
t 2
Þ 2t 2 - 5t + 2 = 0 \ b2 = ac …(i)
and the given quadratic equations
Þ 2t 2 - 4t - t + 2 = 0
ax2 + 2bx + c = 0 …(ii)
Þ (2t - 1) (t - 2) = 0
1 1 dx2 + 2ex + f = 0 …(iii)
Þ t = , 2 Þ r2 = , 2 For quadratic Eq. (ii),
2 2
Þ r= 2 the discriminant D = (2b)2 - 4ac
= ar3 + ar5 + ar7 = 4(b2 - ac) = 0 [from Eq. (i)]
= ar3 (1 + r 2 + r 4 ) Þ Quadratic Eq. (ii) have equal roots, and it is equal to
= 5 [1 + 2 + 4] = 35 b
x = - , and it is given that quadratic
a
50. If a , b, c are in GP, then b2 = ac Eqs. (ii) and (iii) have a common root, so
Taking log on both sides, we get 2
æ bö æ bö
2 log e b = log e a + log e c d ç - ÷ + 2e ç - ÷ + f = 0
è aø è aø
Þ 2n log e b = n log e a + n log e c
Þ db2 - 2eba + a 2f = 0
Þ 2 log e bn = log e a n + log e cn
Þ d (ac) - 2eab + a 2f = 0 [Q b2 = ac]
Þ log e a n , log e bn , log e cn be in AP.
Þ dc - 2eb + af = 0 [Q a ¹ 0]
51. Let r be the common ratio. Þ 2eb = dc + af
100
å a 2n Þ
e dc af
2 = 2+ 2
n =1 a 2 + a 4 + a 6 + K + a 200 b b b
\ =
100
a1 + a3 + a5 + K + a199 [dividing each term by b2]
å a 2n - 1
æ eö d f
n =1 Þ 2ç ÷ = + [Q b2 = ac]
3 5 199 è bø a c
a1 (r + r + r + K + r )
= =r d e f
a1 (1 + r 2 + r 4 + K + r198 ) So, , , are in AP.
a b c
a
Þ =r
b 54. It is given that, the terms a , b, c are in GP with
a 1
52. Let the three consecutive terms of a GP are , a and ar. common ratio r, where a ¹ 0 and 0 < r £ .
r 2
Now, according to the question, we have So, let, b = ar and c = ar 2
a Now, the terms 3a, 7b and 15c are the first three terms
× a × ar = 512 Þ a3 = 512
r of an AP, then
Þ a =8 ... (i) 2(7b) = 3a + 15 c
Also, after adding 4 to first two terms, we get Þ 14ar = 3a + 15ar 2 [as b = ar, c = ar 2]
2
8
+ 4, 8 + 4, 8r are in AP Þ 14r = 3 + 15r [as a ¹ 0]
r Þ 15r 2 - 14r + 3 = 0
8 Þ 15r 2 - 5r - 9r + 3 = 0
Þ 2 (12) = + 4 + 8r
r
52 JEE Main Mathematics

Þ 5r (3r - 1) - 3(3r - 1) = 0 1
Þ r=-
1 3 2
Þ (3r - 1) (5r - 3) = 0 Þ r = or
3 5 From Eq. (ii), we get
æ 1ù 1 éæ 1ö ù
as, r Î ç0, ú , so r = a ê ç - ÷ - 1ú = 6 Þ a = - 4
è 2û 3 ë è 2 ø û
Now, the common difference of AP = 7b - 3a On putting the value of a and r in Eq. (i), the required
æ7 ö 2a numbers are 8, - 4, 2 , 8.
= 7ar - 3a = a ç -3÷ = -
è3 ø 3 57. Given, a1 , b1 are the roots of the equation
æ -2 a ö x2 - 6 x + p = 0
So, 4th term of AP = 3a + 3 ç ÷=a
è 3 ø
Þ a1 + b1 = 6 …(i)
55. Let r be the common ratio. Þ a1 × b1 = p …(ii)
If r ³ 1, then ar 2 is the greatest term. and a 2, b 2 are the roots of the equation
\ ar 2 < a + ar x2 - 54x + q = 0
Þ r2 - r - 1 < 0 Þ a 2 + b 2 = 54 …(iii)
1± 1+4 Þ a2 × b2 = q …(iv)
Þ r=
2 Since, a1 , b1 , a 2, b 2 are in GP.
1± 5 \ a1 = a, b1 = ar, a 2 = ar 2, b 2 = ar3
Þ r= Now, a1 + b1 = 6
2
æ1 - 5 1 + 5 ö \ a + ar = 6 …(v)
Þ r Îç , ÷ and a 2 + b 2 = 54
è 2 2 ø
ar 2 + ar3 = 54 …(vi)
1
As r ³ 1, 1 £ r < ( 5 + 1) From Eqs. (v) and (vi), we get
2
r2 = 9 Þ r = ± 3
In case, 0 < r <1
Q a (1 + r ) = 6 = a (1 ± 3) = 6
a < ar + ar 2
a = - 3, 3 / 2
Þ r2 + r - 1 > 0
q - p = a 2r5 - a 2r = a 2r (r 4 - 1)
-1 ± 5 1
Þ r= Þ r < - ( 5 + 1) æ3ö
2
2 2 = ç ÷ (3) (81 - 1)
è2ø
1
or r > ( 5 - 1) 9
2 = ´ 3 ´ 80 = 540
4
5 -1
As 0 < r <1 Þ < r <1 a
2 58. Let three numbers in GP are , a , ar.
r
æ 5 -1 5 + 1ö
\ r Îç , ÷ From the given condition,
è 2 2 ø a
+ a + ar = 14
a r
56. Let the four numbers be , a , ar ,2ar - a …(i) æ1 ö
r Þ a ç + 1 + r ÷ = 14 …(i)
èr ø
where first three numbers are in GP and last three
a
in AP. and + 1, a + 1 and ar - 1 will be in AP, then
Given that, the common difference of AP is 6, so r
a a
ar - a = 6 …(ii) 2 (a + 1 ) = + 1 + ar - 1 = (1 + r 2) …(ii)
r r
a From Eqs. (i) and (ii), we get
and also given = 2ar - a
r a = 4 and r = 2
a So, required numbers are 2, 4, 8.
Þ = 2 (ar - a ) + a
r Hence, greatest number is 8.
a
Þ = 2 (6) + a [from Eq. (ii)] 59. Since, a , b, c are in AP.
r
\ b = a + d , c = a + 2d,
æaö
Þ ç ÷ - a = 12 where d is a common difference, d > 0
èrø
Again, since a 2, b2, c2 are in GP.
Þ a (1 - r ) = 12r \ a 2, (a + d )2 and (a + 2d )2 are in GP.
Þ - 6 = 12r [from Eq. (ii)] Þ (a + d )4 = a 2 (a + 2d )2
Sequence and Series 53

or (a + d )2 = ± a (a + 2d ) 62. Let the numbers be x and y.


Þ a 2 + d 2 + 2ad = ± (a 2 + 2ad ) According to the question,
On taking (+ ) sign, d = 0 (not possible as a < b < c) x+ y
= 2 xy
On taking (- ) sign, 2
2a 2 + 4ad + d 2 = 0 Þ x + y = 4 xy …(i)
2
æ1 ö æ1 ö Þ 2
(x + y) = 16xy
Þ 2a 2 + 4a ç - a ÷ + ç - a ÷ = 0
è2 ø è2 ø And we know that,
é 3 1ù (x - y)2 = (x + y)2 - 4xy
êëQ a + b + c = 2 Þ a + d = 2 úû
\ (x - y)2 = 16xy - 4xy = 12xy
Þ 4a 2 - 4a - 1 = 0 Þ x - y = 2 3xy …(ii)
1 1
\ a= ± On solving Eqs. (i) and (ii), we get
2 2
1 x = (2 + 3 ) xy
Here, d = -a >0
2 and y = (2 - 3 ) xy
1 x (2 + 3 ) xy
So, a< \Required ratio = =
2 y (2 - 3 ) xy
1 1
Hence, a= - (2 + 3 )2 = 7 + 4 3
2 2
63. Let A be the Ist term of AP and d be the common
60. Given, a = 1 and 4T2 + 5T3 is minimum. difference.
Let r be the common ratio of the GP, then \ 7th term = a = A + 6d [Q nth term = A + (n - 1)d]
\ 4T2 + 5T3 = 4 (ar ) + 5(ar 2) [Q a = 1] 11th term = b = A + 10d
Þ 4r + 5r 2 = f (r ) [say] …(i) 13th term = c = A + 12d
Þ r (4 + 5r ) = f (r ) Q a, b, c are also in GP
f (r ) = 0 \ b2 = ac
r (4 + 5r ) = 0 Þ r = 0, - 4 / 5 Þ ( A + 10 d )2 = ( A + 6 d ) ( A + 12 d )
Þ A 2 + 20 Ad + 100 d 2 = A 2 + 18 Ad + 72d 2
Þ 2 Ad + 28 d 2 = 0
f(r)
–2/5 Þ 2d ( A + 14 d ) = 0
.

–4/5 O Þ d = 0 or A + 14 d = 0
( –2, –4
) But d ¹ 0 [Q the series is non constant AP]
5 5
Þ A = - 14 d
We know that, if a > 0, quadratic expression \ a = A + 6 d = - 14 d + 6 d = - 8d
b and c = A + 12 d = - 14 d + 12 d = - 2d
ax2 + bx + c has least value at x = - .
2a a - 8d
Þ = =4
From the graph it is clear that, minima occurs of point c - 2d
æ -2 -4 ö 64. Since, x, y and z are in AP.
ç , ÷.
è 5 5 ø \ 2y = x + z
-2 Also, tan - 1 x, tan - 1 y and tan - 1 z are in AP.
\ r=
5
\ 2 tan - 1 y = tan - 1 x + tan - 1 z
2 2
61. Let a , ar , ar are in GP and a , ar , ar - 64 are in AP, æ 2y ö æ x+ zö
we get Þ tan -1 ç ÷ = tan - 1 ç ÷
è 1 - y2 ø è 1 - xz ø
a (r 2 - 2r + 1 ) = 64 …(i)
x+ z x+ z
Again, a , ar - 8, ar 2 - 64 are in GP. Þ =
1 - y2 1 - xz
\ (ar - 8)2 = a (ar 2 - 64)
Þ a (16r - 64) = 64 …(ii) Þ y2 = xz
On solving Eqs. (i) and (ii), we get Since x, y and z are in AP as well as in GP.
r = 5 ,a = 4 \ x= y=z
Thus, required numbers are 4, 20, 100.
54 JEE Main Mathematics

65. Let b = ar and c = ar 2, where r is the common ratio. 68. Given functional relation f (x + y) = f (x) × f ( y)
Then, a + b + c = xb Þ f (x) = a x
Þ a + ar + ar 2 = xar ¥

Þ 1 + r + r 2 = xr … (i) [Q a ¹ 0] Since, å f (x) = 2, so,|a | < 1


x =1
1 + r + r2
Þ x= Þ a + a 2 + a3 + ¼ = 2
r
1 a 2
=1 + r + Þ = 2 Þ 3a = 2 Þ a=
r 1-a 3
2
1
We know that, r + ³ 2 (for r > 0) f (4) a 4 æ2ö 4
So, = = a2 = ç ÷ =
r f (2) a 2 è3ø 9
1
and r + £ - 2 (for r < 0) [using AM ³ GM] 69. (666 K 6) = 6 + 6 ´ 10 + 6 ´ 102 + K + 6 ´ 10n - 1
r n digits
1 1
\ 1 + r + ³ 3 or 1 + r + £ -1 = 6 (1 + 10 + 102 + K + 10n - 1 )
r r
Þ x ³ 3 or x £ -1 6 2
= (10n - 1 ) = (10n - 1 )
Þ x Î (- ¥ ,-1] È [3, ¥ ) 9 3
Hence, x cannot be 2. 8
Similarly, (888 K 8) = (10n - 1 )
Alternate Method n digits 9
From Eq. (i), we have 4 8
Hence, required sum = (10n - 1 )2 + (10n - 1 )
1 + r + r 2 = xr 9 9
Þ r 2 + (1 - x)r + 1 = 0 4
= (102n - 2 × 10n + 1 + 2 × 10n - 2)
For real solution of r , D ³ 0. 9
4
Þ (1 - x)2 - 4 ³ 0 = (102n - 1 )
9
Þ x2 - 2 x - 3 ³ 0
1 (1 - ln + 1 ) 1 - ln + 1
Þ (x - 3)(x + 1) ³ 0 70. LHS = =
1-l 1-l
Þ x Î (-¥ , - 1] È [3, ¥ )
and RHS = (1 + l ) (1 + l2) (1 + l4 ) (1 + l8 ) (1 + l16 )
\ x cannot be 2. (1 - l )(1 + l ) (1 + l2) (1 + l4 ) (1 + l8 ) (1 + l16 )
=
66. 1.4 + 3.04 + 5.004 + K (1 - l )
= (1 + 3 + 5 + ... ) + (0.4 + 0.04 + 0.004 + ... ) (1 - l2) (1 + l2) (1 + l4 ) (1 + l8 ) (1 + l16 )
=
n é4 4 ù 1-l
= [2 + (n - 1 ) × 2] + ê + + K upto n terms ú
2 ë 10 102 û (1 - l32)
=
é æ1ö ù
n
1-l
ê1 - ç ÷ ú
4 ê è 10 ø ú 1 - ln + 1 1 - l32
= n2 + \ =
10 ê 1 - 1 ú 1-l 1-l
ê 10 ú
ë û Þ 1 - ln + 1 = 1 - l32
4 æ 1 ö \ n + 1 = 32
= n 2 + ç1 - n ÷
9è 10 ø Þ n = 31

67. Let the first term and common ratio of the GP be ‘a’ and 71. Given f (x + y) = f (x) × f ( y), x, y Î N and f (1) = 2
‘r’ respectively, so according to the given information f (2) = f (1 + 1) = f (1) × f (1) = 4 = 22
2 3
ar + ar + ar = 3 …(i) f (3) = f (2 + 1) = f (2) × f (1) = 8 = 23
and ar5 + ar 6 + ar7 = 243 …(ii) f (4) = f (3 + 1) = f (3) × f (1) = 16 = 24
From Eqs. (i) and (ii), we get M M M M
\ r 4 = 81 Þ r = 3 [Q terms are positive]
f (n ) = f (n - 1 + 1) = f (n - 1) f (1) = 2n
\ 3a + 9a + 27a = 3 n
1 Now, å f (a + k) = f (a + 1) + f (a + 2)
Þ a + 3a + 9a = 1 Þ a = . k =1
13 + f (a + 3) + L + (a + n )
\ Sum of first 50 terms of the GP be = f (a ) f (1) + f (a ) f (2) + f (a ) f (3) + L + f (a ) f (n )
1 (350 – 1) 1 50
S50 = = (3 – 1) = f (a ) [ f (1) + f (2) + f (3) + L + f (n )]
13 3 – 1 26
Sequence and Series 55

= f (a ) [2 + 4 + 8 + L + 2n ] = f (a ) [2 + 22 + 23 + L + 2n ] 1 1 1
76. Given series = 1 + + + +L
2 4 8
é 2 (2 - 1) ù
n
n
= f (a ) ê ú = 2 f (a ) (2 - 1) …(i) 1
ë 1 û Here, a = 1, r =
2
n
1 é a ù
But å f (a + k) = 16 (2n - 1) …(ii) \ S= =2 êS ¥ = ú …(i)
k =1
1-
1 ë 1 - rû
2
From Eqs. (i) and (ii), we have
é æ1ö ù
n
2 f (a ) (2n - 1) = 16(2n - 1) 1 ê1 - ç ÷ ú
êë è2ø ú é a (1 - r n ) ù
Þ 2 f (a ) (2n - 1) = 2 ´ 8 (2n - 1) Sn = û
and êQ S n = , r < 1ú
Þ 2 f (a )(2n - 1) = 2 f (3)(2n - 1) [Q f (3) = 8]
1
1- ë 1 - r û
2
Þ f (a ) = f (3)
æ 1ö
= 2 ç1 - n ÷ …(ii)
Þ a =3 è 2 ø
72. Let S = 1 + 10 + 102 + ... + 1090 It is given that,
1
S - Sn <
91 1000
1 × (10 - 1 )
= æ 1ö 1
10 - 1 Þ 2 - 2 ç1 - n ÷ < [from Eqs. (i) and (ii)]
è 2 ø 1000
(1013 )7 - 1 1013 - 1 2 1
= ´ Þ 2 -2 + n <
1013 - 1 10 - 1 2 1000
= [(1013 )6 + (1013 )5 + (1013 )4 + ... + 1] 1 1
<
´ (1012 + 1011 + ... + 1 ) 2n-1 1000
é 1 1 ù
\ It is the product of two integers and hence not Þ 2n - 1 > 1000 êëQ a < b Þ a > búû
prime.
n n Þ n - 1 ³ 10
73. å å Srs2r3s = 2 × 3 + 22 × 32 + 23 × 33 + ... + 2n × 3n Þ n ³ 11
r =1 s =1
So, the least value of n is 11.
6 (6n - 1 ) ¥
[as S rs = 0, if r ¹ s and S rs = 1, if r = s) = 77. Given, å rn = S
6 -1
n= 0
6 ù
= (6n - 1 )ú 2 1
5 û Þ 1 + r + r + ... ¥ = S Þ =S
1-r
74. Since, nm + 1 divides 1 + n + n 2 + K + n127 S -1
Þ 1 = S - rS Þ r =
1+ n+ n +K+ n 2 127 S
Therefore, is an integer. ¥
nm + 1 Now, å r 2n = 1 + r 2 + r 4 + ... ¥
n= 0
1 - n128 1
Þ ´ m is an integer. 1 1 S2
1-n n +1 = = =
1 - r2 æS -1ö
2
S - (S - 1 )2
2
(1 - n 64 )(1 + n 64 ) 1-ç ÷
Þ is an integer when largest è S ø
(1 - n )(nm + 1 )
S2 S2
m = 64 = =
(S - S + 1 ) (S + S - 1 ) (2 S - 1 )
75. Given, sum 2 7 12 17 22
ì 2 n - 1ü 78. s=1 + + + + 4 + 5 +K
n - 1ï æ x + 1ö æ x + 1ö æ x + 1ö ï 3 32 33 3 3
= (x + 2) í 1 + ç ÷ + ç ÷ + K + ç ÷ ý
è x + 2 ø è x + 2 ø è x + 2 ø
îï þï s 1 2
= + +
7
+ K... ¥
3 3 32 33
ì æ x + 1ö ü
n
ï1 - ç ÷ ï 2s 1 5 5
= 1 + + 2 + 3 + K... ¥
ï è x + 2ø ï 3 3 3 3
= (x + 2)n - 1 í ý
ï 1 - æç x + 1 ö÷ ï ì ü
ï è x + 2ø ï 2s 4 5 ï 1 / 3 ï 5 4 13
î þ = + í ý= + =
3 3 3 ï1 - 1ï 6 3 6
(x + 2)n - 1 {(x + 2)n - (x + 1 )n } × (x + 2) î 3þ
=
(x + 2)n 13
s=
= (x + 2)n - (x + 1 )n 4
56 JEE Main Mathematics

79. We have, 1 + cos a + cos 2 a + K ¥ = 2 - 2 1


=
æ a - 1ö æ b - 1ö
1 1-ç ÷ç ÷
Þ =2 - 2 è a øè b ø
1 - cos a
From Eqs. (vi) and (vii)
1 1
Þ 1 - cos a = =1 + c=
1
2- 2 2 ab - (a - 1) (b - 1)
1 3p ab
Þ cos a = - Þ a=
2 4 ab
Þ c=
a × b - { ab - a - b + 1}
80. As we know, sum of infinite terms of GP,
ab
ìï a Þ c=
, |r|< 1 ab - ab + a + b - 1
S¥ = í 1 - r
ïî ¥ , |r|³ 1 ab
Þ c=
x a + b -1
\ S¥ = =5 (thus|r|< 1 )
1-r Þ ca + cb - c = ab Þ ac + bc = ab + c
x
Þ 1-r = 82. We know, AM ³ GM
5 4
5-x 4x +
Þ r= exists only when|r|< 1 \ 4x ³ 4x ´ 4
5 2 4x
5-x
Þ -1 < <1 Þ
4 4
4x + x ³ 2 4 Þ 4x + x ³ 4
5
4 4
\ -10 < - x < 0 Þ 0 < x < 10
¥ 83. Since, g = ab. Also, a , p , q and b are in AP.
81. a=å x n
…(i) b-a
n=0 So, common difference d is .
3
¥
b - a 2a + b
b=å yn …(ii) \ p=a + d=a + =
n=0 3 3
¥ b - a a + 2b
q=b-d=b- =
c = å (xy)n …(iii) 3 3
n=0 Now, (2 p - q)( p - 2q)
From Eq. (i), (4a + 2b - a - 2b) (2a + b - 2a - 4 b)
¥ = ×
a=å xn = 1 + x + x2 + x3 + L + ¥ 3 3
n=0 = - ab = - g 2
1
= [Q|x | < 1] …(iv) 84. Since, a , b and c are in AP.
1-x
¥ \ 2b = a + c …(i)
b=å yn = 1 + y + y2 + y3 + L + ¥ Also, p is AM between a and b
n=0 \ 2p = a + b …(ii)
1 Also, p¢ is GM between a and b
= [Q | y | < 1] …(v)
1- y \ p¢ = ab …(iii)
1 Similarly, 2q = b + c and q¢ = bc …(iv)
From Eq. (iv), a =
1-x æ a + b ö
2
æ b + c ö
2
Now, p2 - q2 = ç ÷ -ç ÷
Þ a (1 - x) = 1 Þ a - ax = 1 è 2 ø è 2 ø
a -1 1
Þ x= …(vi) = [a 2 + b2 + 2ab - (b2 + c2 + 2bc)]
a 4
1 1
From Eq. (v), b = = [(a 2 - c2) + 2b(a - c)]
1- y 4
b -1 (a - c)
Þ y= …(vii) = (a + c + 2b)
b 4
¥ æ a - cö
=ç ÷ (2b + 2b) [Q from Eq. (i)]
Now, c=å (xy)n è 4 ø
n=0
æ a - cö
c = 1 + xy + (xy)2 + (xy)3 + L + ¥ =ç ÷ 4b = (a - c)b …(v)
è 4 ø
1
= [Q|x| < 1,| y| < 1 Þ|xy| < 1] and p¢2 - q¢2 = ( ab )2 - ( bc )2
1 - xy
Sequence and Series 57

= ab - bc = b(a - c) …(vi) 90. Given series is 1 × 32 + 2 × 52 + 3 × 72 + ... ¥


\From Eqs. (v) and (vi), we get This is an arithmetico-geometric series whose nth term
p 2 - q 2 = p ¢2 - q ¢2 is equal to
Tn = n (2n + 1 )2 = 4n3 + 4n 2 + n
85. Given that, x1x2x3 ... xn = 1 …(i) n n

We know that, AM ³ GM
\ Sn = å Tn = å (4n3 + 4n 2 + n )
1 1
æ x1 + x2 + x3 + ... + xn ö 1/ n n n
\ç ÷ ³ (x1x2x3 ... xn ) n
è n ø =4 å n3 + 4 å n2 + S n
1
1 1
= (1 )1/ n = 1 [from Eq. (i)] 2
én ù 4 n
Þ x1 + x2 + x3 + .... + xn ³ n = 4 ê (n + 1 )ú + n (n + 1 ) (2n + 1 ) + (n + 1 )
ë2 û 6 2
\ x1 + x2 + x3 + . . . + xn é 4 1ù
can never be less than n. = n (n + 1 ) ê n 2 + n + (2n + 1 ) + ú
ë 6 2û
2 p 2 p 2 p 3q 3q 4r 4r n 2
+ + + + ... + + + ... + = (n + 1 ) (6n + 14n + 7 )
3 3 3 1 54 5 7 7 6
4244 3 14 4244 3
86. Since,
5 times 7 times 91. This progression is an arithmetico-geometric series.
15 a dr
\ S¥ = +
3 5
æ 2 p ö æ 3q ö æ 4r ö
7
1 - r (1 - r )2
³ 15 ç ÷ ç ÷ ç ÷ [Q AM ³ GM]
è 3 ø è5ø è7ø 1 2
= +
233547 55 77 55 77 1 - 1 /2 (1 - 1 /2)2
Þ p3 q5 r7 £ 1 Þ p3 5 7
q r £ £ 2 2
33 55 77 23 32 47 217 9 = + = 4 + 8 = 12
1 /2 1 /4
87. Given that, a + 2b + 3c = 12
and a , b, c are positive real numbers. 92. Let S = 2 + 3 + 6 + 11 + 18 + . . . + Tn
Now, AM ³ GM or S = 2 + 3 + 6 + 11 + . . . + Tn
a + b + b + c+ c+ c 6 2 3 On subtracting, we get
Þ ³ ab c 0 = 2 + 1 + 3 + 5 + 7 + . . . - Tn
6
a + 2b + 3c 6 2 3 Þ Tn = 2 + [1 + 3 + 5 + . . . (n - 1 ) term]
Þ ³ ab c Þ ab2c3 £ 26
6 n -1
=2 + [2 + (n - 1 - 1 )2]
88. The given series is clearly an AG, the corresponding AP 2
is 1 + 4 + 7 + 10 + ... having nth term = 3n - 2 . Tn = 2 + (n - 1 )2
1 1 \ T50 = 2 + (50 - 1 )2 = 492 + 2
and corresponding GP is 1 + + 2 + ... having nth term
5 5 93. Let S n = 12 + 16 + 24 + ... + Tn
1
= n -1 S n = 12 + 16 + ... + Tn
5
3n - 2 On subtraction
Hence, required nth term of the series is n - 1 .
5 0 = 12 + 4 + 8 + 16 + ... - Tn
4(2n - 1 - 1 )
89. Given series is Þ Tn = 12 + = 2n + 1 + 8
2 -1
1 + (2 ´ 3) + (3 ´ 5) + (4 ´ 7) + …upto 11 terms.
Now, the rth term of the series is a r = r (2r - 1) S n = STn = 22 + 23 + 24 + ... + 8n
\Sum of first 11-terms is 22(2n - 1 )
= + 8n = 4 (2n - 1 ) + 8n
11 11 11 11 2 -1
S11 = å r (2r - 1) = å (2r 2 - r ) = 2 å r 2 - å r 94. Let S = 1 + 3x + 6x2 + 10x 3 + ... ¥
r =1 r =1 r =1 r =1
11 ´ (11 + 1)(2 ´ 11 + 1) 11 ´ (11 + 1) xS = x + 3x2 + 6x 3 + ... ¥
=2 -
6 2 On subtracting, we get
é n
n (n + 1 )(2 n + 1 ) n
n (n + 1) ù S (1 - x ) = 1 + 2x + 3x2 + 4x 3 + ... ¥
êQ å r 2 = and å r = ú
êë r = 1 6 r =1 2 úû x (1 - x ) S = x + 2x2 + 3x3 + ... ¥
æ 11 ´ 12 ´ 23 ö æ 11 ´ 12 ö Again on subtracting, we get
=ç ÷-ç ÷
è 3 ø è 2 ø S [(1 - x ) - x (1 - x )] = (1 + x + x2 + x 3 + ... ¥ )
= (11 ´ 4 ´ 23) - (11 ´ 6) 1 1
Þ S [(1 - x ) (1 - x )] = Þ S=
= 11(92 - 6) = 11 ´ 86 = 946 1-x (1 - x )3
58 JEE Main Mathematics

r 2(r + 1 )2 98. Let S n = 1 × 3 × 5 + 2 × 5 × 8 + 3 × 7 × 11 + . . . + nth term


n n
Sr
95. å sr
= å 4
r (r + 1 )
\ Tn = n (2n + 1 )(3n + 2)
r =1 r =1
2 \ S n = STn = Sn (2n + 1 )(3n + 2)
n
r (r + 1 ) 1 n = Sn (6n 2 + 7n + 2)
= å 2
=
2
å (r 2 + r )
= S(6n3 + 7n 2 + 2n )
r =1 r =1
1 é n (n + 1 )(2n + 1 ) n (n + 1 ) ù = 6Sn3 + 7Sn 2 + 2Sn
= +
2 ëê 6 2 ûú é n (n + 1 ) ù
2
7n (n + 1 )(2n + 1 ) n (n + 1 )
1 =6 ê úû + +2
= (n )(n + 1 ) (2n + 1 + 3) ë 2 6 2
12 n (n + 1 ) é 6n (n + 1 ) 7(2n + 1 ) ù
n (n + 1 )(n + 2) = êë + + 2ú
= 2 2 3 û
6
n (n + 1 ) é 18(n 2 + n ) + 28n + 14 + 12 ù
96. Given series is = ê ú
1 2 3 2 ë 6 û
+ + + ... + n terms
2
1+1 +1 4 2
1+2 +2 4
1 + 32 + 34 n (n + 1 ) æ 18n 2 + 46n + 26 ö
= ç ÷
Let Tn be the nth term of the series 2 è 6 ø
1 2 3
+ + +… n (n + 1 ) 2(9n 2 + 23n + 13)
2
1+1 +1 4 2
1+2 +2 4
1 + 32 + 34 = ´
2 6
n n
Then, Tn = = n (n + 1 )(9n 2 + 23n + 13)
2
1+ n + n 4
(1 + n 2)2 - n 2 =
6
n
= 2 13 13 + 23 13 + 23 + 33
(n + n + 1 ) (n 2 - n + 1 ) 99. Let Sn = + + + ...
1 1+3 1+3+5
1é 1 1 ù
= -
2 êë n 2 - n + 1 n 2 + n + 1 úû \ Tn =
13 + 23 + 33 + . . . + n3
1 + 3 + . . . + (2n - 1 )
1é 1 1 ù
= ê - ú
2 ë 1 + (n - 1 )n 1 + n (n + 1 ) û Sn3
= [Q1 + 3 + . . . + (2n - 1 ) = n 2]
1 é1 1 ù n2
\ T1 = - [n (n + 1 ) / 2]2 (n + 1 )2 n 2 + 2n + 1
2 êë 1 1 + 1 × 2 úû = = =
1é 1 1 ù n2 4 4
T2 = ê -
2 ë 1 + 1 × 2 1 + 2 × 3 úû 100. Let S n = n3 - (n - 1 )3 + . . . + (-1 )n - 113
1é 1 1 ù Here, n is an odd integer.
T3 = ê -
2 ë 1 + 2 × 3 1 + 3 × 4 úû
S n = 13 - 23 + 33 - 43 + . . .+ n3
… … …
= [13 + 23 + 33 + . . . + (n - 1 )3 + n3 ]
… … …
1é 1 1 ù - 2 [23 + 43 + 63 + . . . + (n - 1 )3 ]
Tn = ê - ú é æ n - 1ö ù
3
2 ë 1 + (n - 1 )n 1 + n (n + 1 ) û
= Sn3 - 2 ´ 23 ê13 + 23 + 33 + . . . ç ÷ ú
êë è 2 ø ú
On adding all these equations, we get û
n
1é 1 ù n (n + 1 ) é æ n - 1 ö3 ù
å Tr = 2 ê1 - 1 + n (n + 1 ) ú = 2 (n 2 + n + 1 ) = Sn3 - 16 ê S ç ÷ ú
ë û
r =1
êë è 2 ø úû
97. Let there are n balls used to form the sides of equilateral 2 2
é n (n + 1 ) ù éæ n - 1ö æ n - 1 öù
triangle. =ê úû - 16 ê çè 2 ÷ø çè 2 + 1÷ø ú
According to the question, we have ë 2 ë û
n (n + 1) é æ n - 1 ö 2 æ n + 1 ö 2ù
+ 99 = (n - 2)2
2 16 êç ÷ ç ÷ ú
êë è 2 ø è 2 ø úû
2
n 2(n + 1 )
Þ n 2 + n + 198 = 2 [n 2 - 4n + 4] = -
4 4
Þ n 2 - 9n - 190 = 0 Þ n 2 - 19n + 10n - 190 = 0
Þ (n - 19)(n + 10) = 0 Þ n = 19, - 10 (n + 1 )2 2
= [n - (n 2 + 1 - 2n )]
Þ n = 19 [Q number of balls n > 0] 4
Now, number of balls used to form an equilateral (n + 1 )2
n (n + 1) 19 ´ 20 = (2n - 1 )
triangle is = = 190. 4
2 2
Sequence and Series 59

101. Let nth term of the series is Tn and sum is S, then 3. Q13 , a1 , a 2, ... , a 20 , 67 are in AP.
S = 1 + 2 + 5 + 12 + 25 + 46 + 77 + K + Tn æ 13 + 67 ö
\a1 + a 2 + a3 + K + a 20 = 20 ç ÷ = 800
S = 1 + 2 + 5 + 12 + 25 + 46 + K + Tn - 1 + Tn è 2 ø
On subtraction, we get Also, AM > GM
0 = 1 + 1 + 3 + 7 + 13 + 21 + 31 + K + (Tn - Tn - 1 ) - Tn a1 + a 2 + K + a 20
Þ ³ (a1a 2a3 K a 20 )1/ 20
\Tn = 1 + { 1 + 3 + 7 + 13 + 21 + K + upto (n - 1 )} 20
Þ 40 ³ (a1 × a 2 × a3 K a 20 )1/ 20
Let (n - 1 )th term and S of the series 1 + 3 + 7 + 13 + K
Hence, maximum value of a1 × a 2 × a3 K a 20 is (40)20.
are tn - 1 and S¢ respectively, then
S ¢ = 1 + 3 + 7 + 13 + 21 + K + tn - 1 4. From symmetry, we observe that S50 has 50 terms. First
S ¢ = 1 + 3 + 7 + 13 + K + tn - 2 + tn - 1 term of S1 , S 2, S3 , S 4 ,... , S50 are 1, 2, 4, 7, … , 50.
On subtraction, we get Let Tn be the first term of nth set. Then,
0 = 1 + 2 + 4 + 6 + 8 + K + (tn - 1 - tn - 2) - tn - 1 S = T1 + T2 + T3 + ... + Tn
\ tn - 1 = 1 + 2 { 1 + 2 + 3 + 4 + K upto (n - 2)} Þ S = 1 + 2 + 4 + 7 + 11 + ... + Tn - 1 + Tn
1 or S = 1 + 2 + 4 + 7 + ... + Tn - 1 + Tn
= 1 + 2 × (n - 2) (n - 1 ) = n 2 - 3n + 3
2 Therefore, on subtracting
Þ tn = (n + 1 )2 - 3 (n + 1 ) + 3 0 = 1 + [1 + 2 + 3 + 4 + ... + (Tn - Tn - 1 )] - Tn
= n2 - n + 1 n (n - 1 )
or 0 =1+ - Tn
\ Tn = 1 + { 1 + 3 + 7 + ... upto (n - 1 )} 2
n -1
n (n - 1 )
=1 + å (n 2 - n + 1 ) Þ Tn = 1 +
2
1
n -1 n -1 n -1 Þ T50 = First term in S50 = 1226
=1 + å n2 - å n + å 1 Therefore, sum of the terms in S50
1 1 1
50
1 1 = [2 ´ 1226 + (50 - 1 ) ´ 1]
=1 + n (n - 1 ) (2n - 1 ) - n (n - 1 ) + (n - 1 ) 2
6 2
= 25 (2452 + 49) = 25 (2501 ) = 62525
1
= n (n - 1 ) (n - 2) + n
3 5. It will take 10 yr for Jairam to pay off ` 10000 in
1 10 yearly installments.
Hence, Tn = n (n - 1 ) (n - 2) + n
3 Q He pays 10% annual interest on remaining amount.
\ Money given in the first year
Round II 10000 ´ 10
= 1000 + = 1000 + 1000
1. Q Common terms are 5, 11, 17, … 100
\ Tn = 5 + (n - 1 ) 6 = 6n - 1 = ` 2000
100th term of the first sequence Money given in second year
= 2 + (100 - 1 ) 3 = 299 = 1000 + interest of (10000 - 1000)
and 100th term of the second sequence 9000 ´ 10
= 1000 + = 1000 + 900
= 3 + (100 - 1 ) 2 = 201 100
2
Now, 201 > 6n - 1 Þ n £ 33 = ` 1900
3
Similarly, money paid in third year = ` 1800 etc.
Þ n = 33 (Q n Î N )
-1 -1 -1
So, money given by Jairam in 10 yr will be ` 2000,
2. Q a , b , c are in AP.
` 1900, ` 1800, ` 1700, …
\ a , b, c are in HP.
Which is in arithmetic progression, whose first term
Now, for numbers a101 , b101 , c101
a = 2000 and d = -100
AM > GM
Total money given in 10 yr
101
a + c101 10
Þ > ( ac )101 > b101 [Q ac > b] = [2(2000) + (10 - 1 ) (-100)]
2 2
Þ 2b101 - a101 - c101 < 0 …(i) = ` 15500
Now, product of roots of given equation Therefore, total money given by Jairam
2b 101
-a 101 101
-c = 5000 + 15500
= <0 [from Eq. (i)]
1 = ` 20500
60 JEE Main Mathematics

2
6. Let the number of days be n. é p + rù
Þ êë 2 úû ³ 4 pr [from Eq. (i)]
Hence, a worker can do
æ 1 ö Þ p2 + r 2 - 14 pr ³ 0
ç ÷ th part of the work in a day.
è 150n ø 2
ærö ærö
Þ ç ÷ - 14 ç ÷ + 1 ³ 0 (Q p > 0)
According to the given condition, è pø è pø
1
[150 + 146 + 142 + ... + upto (n + 8)] ´ =1 ær ö
2
terms 150n Þ ç - 7÷ - 48 ³ 0
èp ø
n+8
Þ [300 + (n + 8 - 1 ) (- 4)] = 1 2
2 ær ö 2
Þ ç - 7÷ - (4 3 ) ³ 0
Þ (n + 8) (272 - 4n ) = 300n èp ø
Þ 4n 2 + 60n - 2176 = 0 r
Þ -7 ³4 3
Þ 2
n + 15n - 544 = 0 p
Þ n = 17, - 32 9. Q T6 = 2 Þ a + 5d = 2
We do not take negative value Now, let P = T1T4T5
\ n = 17 = a (a + 3d )(a + 4d )
Therefore, number of total days = (2 - 5d )(2 - 2d )(2 - d )
= 17 + 8 = 25 = 2(4 - 16d + 17d 2 - 5d3 )
2
7. Given equation is ax + bx + c = 0 and let the roots On differentiating w.r.t. d, we get
are a , b. dP
Now, = 2 (-16 + 34d - 15d 2)
b c dd
So, a + b = - and ab = .
a a dP
Put = 0 for maxima or minima
Now, a 2 + b 2 = (a + b)2 - 2 a b dd

b2 2c -16 + 34d - 15d 2 = 0


= 2- 2 8
a a Þ d = and
3 5
1 1 a2 + b2
Now, + 2= 2 2 d 2P
a 2
b a b Also, = 2 (34 - 30d )
dd 2
b2 2 c æ d 2P ö
-
2 b2 - 2 ac ç ÷ >0
= a 2a = è dd 2 ø d = 2/3
c c2
a2 Thus, P is least.
According to the given condition, Thus, the value of d = 2 /3 .
1 1 10. Initially the ball falls from a height of 120 m. After
a+b = 2 + 2
a b striking the floor, it rebounds and goes to a height of
2
b b - 2ac 4 4
Þ - = ´ (120) m. Now, it falls from a height of ´ (120) m
a c2 5 5
4 é4 ù
Þ - bc2 = ab2 - 2a 2c and after rebounding goes to a height of ê (120)ú m.
5 ë5 û
Hence, 2 a 2c = ab2 + bc2
This process is continued till the ball comes to rest.
Þ ab2, ca 2, bc2 or bc2, ca 2, ab2 be in AP.
Hence, the total distance travelled is
8. Since, p, q and r are positive and are in AP. é4 2 ù
æ4ö
p+ r 120 + 2 ê (120) + ç ÷ (120 + . . . ¥ )ú
\ q= …(i) êë 5 è5ø úû
2
Q The roots of the equation px2 + qx + r = 0 are real. é4 ù
ê 5 (120) ú
Þ q2 ³ 4 pr = 120 + 2 ê = 1080 m
4 ú
ê 1- ú
ë 5 û
Sequence and Series 61

16 n æ 8r ö
11. 100 14. S= å ç 4 ÷
è 4r + 1 ø
r =1
4
3 16 n æ 1 1 ö
2 G
1 G
=2 å ç 2 - ÷
è 2 r - 2 r + 1 2 r2 + 2 r + 1 ø
r =1
R
æ 1 1 1 1 1 1 ö
= 2 ç1 - + - + -K+ - ÷
è 5 5 13 13 481 545 ø
æ 1 ö 1088
= 2 ç1 - ÷=
è 545 ø 545
5 + (n - 1 )4
15. Q Tn =
p [(r22 - r12) + (r42 - r32) + ... + 2
(r100 - 2
r99 )] [3 + (n - 1 )4]2 [7 + (n - 1 )4]2

= p (r1 + r2 + r3 + r4 + . . . + r100 ) 1ì 1 1 ü
= í - ý
[Q r2 - r1 = r4 - r3 = . . . = r100 - r99 = 1 ] 8 î (4n - 1 )2 (4n + 3)2þ

= p (1 + 2 + 3 + . . . + r100 ) \ S n = T1 + T2 + K + Tn
= 5050p sq cm 1ì1 1 1 1 1 1 1 ü
= í 2 - 2 + 2 - 2 + 2 -K + 2
- 2ý
¥ 8 î3 7 7 11 11 (4n - 1 ) (4n + 3) þ
12. Since, x= å cos 2n q
1ì1 1 ü 1 æ1 ö 1
n=0
= í 2- 2ý
Þ S¥ = ç - 0÷ =
Þ 0
x = cos q + cos q + cos q + . . . ¥ 2 4 8 î3 (4n + 3) þ 8 è9 ø 72
which is an infinite GP with first term 1 and common xm yn 1
16. Consider, =
ratio cos 2 q such that 0 £ cos 2 q £ 1 (1 + x )(1 + y2n ) (xm + x-m )( yn + y- n )
2m

1
\ x= [sum of an infinite GP] By using AM ³ GM (because x, y Î R+ ), we get
1 - cos 2 q
(xm + x-m ) ³ 2 and ( yn + y- n ) ³ 2
1 1
Þ x= …(i) [Q if x > 0, then x +
³ 2]
sin 2 q x
1 1
1 Þ (xm + x-m )( yn + y- n ) ³ 4 Þ m £
Similarly, y = …(ii) (x + x-m )( yn + y- n ) 4
cos 2 q
1
1 \ Maximum value = .
and z= …(iii) 4
1 - cos q sin 2 q
2

On substituting respective values of x, y, z in the given 17. The general term is


n n+1
options, we get the option (b) is correct. ×
2 2 1 1 1
13. Since, S n = an (n - 1 ) Tn = 3 = = -
1 + 2 + 33 + K + n3
3
n (n + 1 ) n n + 1
Now, nth term of the series is 1 n
Tn = S n - S n - 1 \ Sn = 1 - =
n+1 n+1
Þ Tn = an (n - 1 ) - a (n - 1 )(n - 2)
18. S n = cn 2
Þ Tn = a (n - 1 )[n - (n - 2)] = 2a (n - 1 )
Again, let the sum of squares of n terms of the series is S n - 1 = c (n - 1 )2 = cn 2 + c - 2cn
S1 such that Tn = 2cn - c
n Tn2 = (2cn - c)2 = 4c2n 2 + c2 - 4c2n
S = T12 + T22 + T32 + ... + Tn2 = å Tr2
Sum = S Tn2
r =1
n 4c2 + n (n + 1 ) (2n + 1 )
= + nc2 - 2c2n (n + 1 )
Þ S= å { 2a (r - 1 )}2 6
r =1
2c2 n (n + 1 ) (2n + 1 ) + 3nc2 - 6c2 n (n + 1 )
n =
3
Þ S= å 4a 2(r - 1 )2
nc2 (4n 2 + 6n + 2 + 3 - 6n - 6) nc2 (4n 2 - 1 )
r =1 = =
3 3
é1 ù
Þ S = 4a 2 ê (n - 1 )n{2(n - 1 ) + 1 )}ú
ë6 û 19. Let the three-digit number be xyz. According to the
given condition, we have
2a 2
\ S= n (n - 1 )(2n - 1 ) y2 = xz …(i)
3 2( y + 2) = x + z …(ii)
62 JEE Main Mathematics

2
Þ 100x + 10 y + z - 792 = 100z + 10 y + x æ 10 ´ 11 ö æ 10 ´ 11 ´ 21 ö
Þ 11 - 8 ç ÷ +4ç ÷ = a - 220 b
Þ x- z =8 …(iii) è 2 ø è 6 ø
On squaring Eqs. (ii) and (iii) and subtracting, we have 10 ´ 11 æ 10 ´ 11 21 ö
4xz = 4( y + 2)2 - 64 Þ 11 - ´4 ç ´2 - ÷ = a - 220 b
2 è 2 3ø
Þ y2 = ( y + 2)2 - 16 [using Eq. (i)]
Þ 11 - (20 ´ 11)(110 - 7) = a - 220 b
Þ y=3
Þ x + z = 10 [using Eq. (ii)] Þ 11 - 220(103) = a - 220 b
Þ x = 9, z = 1 Þ a = 11 and b = 103
Hence, the number is 931 = 72 ´ 19. Þ (a , b ) = (11, 103)
20. a1 + a3 + a5 = - 12 Hence, option (d) is correct.
a + a + 2d + a + 4d = - 12(d > 0) 24. First three terms of an AP are given as
a + 2d = - 4 …(i) 32sin 2a - 1, 14, 34 - 2sin 2a
a1a3 a5 = 80
32sin 2a 34
a (a + 2d )(a + 4d ) = 80 \ 28 = + 2sin 2a
3 3
or a (-4)(-4 - 2d + 4d ) = 80 [from Eq. (i)]
or (-4 - 2d )(-4 + 2d ) = - 20 Þ d = ± 3 Let, 32sin 2a = x
Since, AP is increasing, so d = + 3; a = - 10. x 81
So, 28 = +
a1 = - 10; a 2 = - 7 ü 3 x
ï Þ x2 - 84x + 243 = 0
Hence, a3 = a + 2d = - 10 + 6 = - 4ý
a5 = a + 4d = - 10 + 8 = - 2ïþ
2
Þ x - 81x - 3x + 243 = 0
a Þ (x - 81)(x - 3) = 0 Þ x = 3, 81
21. Let the three terms of GP are , a , ar.
r Þ 32sin 2a = 31 or 34 Þ 2 sin 2 a = 1 or 4
a 1
As, it is given that ´ a ´ ar = 27 Þ a = 3 Þ sin 2 a = [Q sin 2 a Î [-1, 1]]
r 2
a æ 1 + r + r2 ö So, first three terms of given AP are 1, 14, 27
and S = + a + ar Þ S = 3 ç ÷
r è r ø \ Sixth term, a 6 = 1 + (6 - 1)13 = 1 + 65 = 66
2
Þ 3r + (3 - S ) r + 3 = 0 [Q r ¹ 0] 25. Given series is
Q r ÎR S = log(71/ 2 ) x + log(71/ 3 ) x + log(71/ 4 ) x + ..... + upto 20 terms
2
\ D ³ 0 Þ (3 - S ) - 4 (3) (3) ³ 0 Þ S = 2 log7 x + 3 log7 x + 4 log7 x + ....+ upto 20 terms
Þ (3 - S - 6) (3 - S + 6) ³ 0 Þ S = (2 + 3 + 4 + .... + 21) log7 x = 460 [given]
Þ (S + 3) (S - 9) ³ 0 Þ S Î (-¥ , - 3] È [9, ¥ ) æ 21 ´ 22 ö
Þ ç – 1÷ log7 x = 460
22. Let, l = 210 + 29 × 31 + 28 × 32 + …… + 2 × 39 + 310 …(i) è 2 ø

3 3 11 Þ (231 – 1) log7 x = 460


So, l = 29 × 31 + 28 × 32 + ¼¼ + 310 + …(ii)
2 2 Þ log7 x = 2 Þ x = 72
Subtracting Eq. (ii) from Eq. (i), we get 26. The natural numbers between 100 and 200 are 101, 102,
1 311 103, …, 199.
- l = 210 -
2 2 Since, 91 = 13 ´ 7, so the natural numbers between 100
and 200 whose HCF with 91 is more than 1 are the
Þ l = 311 - 211
numbers which are either divisible by 7 or 13.
Q l = 210 + 29 × 31 + 28 × 32 + ¼ +
So, the required sum of numbers between 100 and 200
2 × 39 + 310 = S - 211 [given]
= (sum of numbers divisible by 7)
\ 311 - 211 = S - 211
+ (sum of numbers divisible by 13)
Þ S = 311
- (sum of numbers divisible by 91)
23. Given series is 14 8
1 + (1 - 22 × 1) + (1 - 42 × 3) + (1 - 62 × 5) + ... = å (98 + 7r ) + å (91 + 13r ) - (182)
r =1 r =1
+ (1 - 202 × 19) = a - 220 b
10 æ 14 ´ 15 ö æ8 ´9ö
= (98 ´ 14) + 7ç ÷ + (91 ´ 8) +13 ç ÷ - (182)
Þ 1+ å [1 - (2n )2(2n - 1)] = a - 220 b è 2 ø è 2 ø
n =1
= 1372 + 735 + 728 + 468 - 182
10 10
Þ 1 + 10 - 8 å n3 + 4 å n 2 = a - 220 b = 3303 - 182 = 3121
n =1 n =1
Sequence and Series 63

20
æ1ö 1 2 3 4 20 1 n-7
27. Let S = å kçè 2k ÷ø ÞS = 2 + 22 + 23 + + ¼ + 20 …(i) Þ [2a1 + (n - 1)d ] = 50 + A
2 4
2 2 2
k =1
1 æ 7 ö n
æ1ö Þ [2a1 + nd - d ] = ç50 - A ÷ + A
On multiplying by ç ÷ both sides, we get 2 è 2 ø 2
è2ø
æ d ö nd æ 7 ö n
S 1 2 3 19 20 Þ ç a1 - ÷ + = ç50 - A ÷ + A
= + + + ¼ + 20 + 21 …(ii) è 2ø 2 è 2 ø 2
2 22 23 24 2 2
On comparing corresponding term, we get
On subtracting Eq. (ii) from Eq. (i), we get
d 7
S 1 1 1 1 20 d = A and a1 - = 50 - A
S - = + 2 + 3 + ¼ + 20 - 21 2 2
2 2 2 2 2 2
A 7
1æ 1 ö Þ a1 - = 50 - A [Q d = A]
ç1 - 20 ÷ 2 2
S 2è 2 ø 20
Þ = - 21 Þ a1 = 50 - 3 A
2 1 2
1-
2 é a (1 - r n ) ù So a50 = a1 + 49d
êQ sum of GP = , r < 1ú = (50 - 3 A ) + 49 A [Q d = A]
ë 1-r û
1 20 1 10 11 = 50 + 46 A
= 1 - 20 - 21 = 1 - 20 - 20 = 1 - 20
2 2 2 2 2 Therefore, (d , a50 ) = ( A , 50 + 46 A )
11 30. Given, f (x + y) = f (x) × f ( y)
Þ S = 2 - 19
2
Let f (x) = lx [where l > 0]
28. 0.7 + 0.77 + 0.777 + … + upto 20 terms Q f (1) = 2 (given)
7 77 777
= + + + ¼ + upto 20 terms \ l =2
10 102 103 10 10 æ 10 ö
é1 11 111 ù So, S f (a + k) = S la+ k = la ç S lk ÷
=7ê + + + ¼ + upto 20 terms ú k =1 k =1 è k=1 ø
ë 10 102 103 û
= 2a [21 + 22 + 23 + ......+210 ]
7é9 99 999 ù
= ê + + + ¼ + upto 20 terms ú é 2(210 - 1) ù
9 ë 10 100 1000 û = 2a ê ú
7 éæ 1ö æ 1 ö æ 1 ö ë 2 -1 û
= ê çè1 - 10 ÷ø + çè1 - 102 ÷ø + çè1 - 103 ÷ø [by using formula of sum of n-terms of a GP having
9 ë first term ‘a’ and common ratio ‘r’, is
+¼+ upto 20 terms]
7 a (r n - 1) ù
= (1 + 1 +¼+ +¼+ upto 20 terms) Sn = , where r > 1ú
9 r -1 û
æ1 1 1 öù
-ç + + +¼+ upto 20 terms ÷ ú
è 10 102 103 øû Þ 2a+ 1 (210 - 1) = 16 (210 - 1) (given)
Þ 2a+ 1 = 16 = 24 Þ a + 1 = 4 Þ a = 3
é 1 ìï æ 1 ö üï ù
20
ê í1 - çè ÷ø ý ú 31. We have, S n = 1 + q + q2 + ¼ + qn
7ê 10 ïî 10 ïþ ú
= ê20 - ú æ q + 1ö æ q + 1ö æ q + 1ö
2 n
9 1 and Tn = 1 + ç ÷+ç ÷ + ¼+ ç ÷
ê 1- ú è 2 ø è 2 ø è 2 ø
ê 10 ú
ë û Also, we have
é a (1 - r n ) ù 101
êQ Sn = , where 1 > rú C1 + 101C 2S1 + 101C3S 2 + ¼ + 101C101S100 = aT100
êë 1- r úû
101 101 101
Þ C1 + C 2(1 + q) + C3 (1 + q + q2)
7é 1 ìï æ 1 ö üï ù
20
= ê20 - í1 - ç ÷ ý ú + ¼+ 101
C101 (1 + q + q2 + ¼ + q100)
9ê 9 ïî è 10 ø ï ú
ë þû
= a × T100
7 é 179 1 æ 1 ö 20 ù (1 - q2) 101 æ 1 - q3 ö
= ê + ç ÷ ú Þ 101
C1 + 101
C2 + C3 ç ÷
9 êë 9 9 è 10 ø úû 1-q è1-qø
7 æ 1 - q4 ö æ 1 - q101 ö
= (179 + 10- 20 ) + 101C 4 ç ÷ + ¼ + 101C101 ç ÷
81 è1-qø è 1-q ø
29. Given AP, is a1 , a 2, a3 ,… having sum of first n-terms æ 1 - rn ö
= a × T100 [Q for a GP, S n = a ç ÷, r ¹ 1 ]
=
n
[2a1 + (n - 1)d ] è1-r ø
2 1
[where, d is the common difference of AP] Þ [{101C1 + 101
C2 + ¼ + 101
C101 }
n (n - 7) 1-q
= 50n + A (given)
2 - {101C1q + 101
C 2q2 + ¼ + 101 C101 q101 }] = a × T100
64 JEE Main Mathematics

1 Þ ar 2 + ar3 = 16 Þ ar 2(1 + r ) = 16 …(ii)


Þ [(2101 - 1) - ((1 + q)101 - 1)] = aT100
(1 - q) From Eqs. (i) and (ii), we get
[Q nC 0 + nC1 + ¼ + nC n = 2n] r 2 = 4 Þ r = ±2 …(iii)
101 101
2 - (q + 1) From Eqs. (i) and (iii), we get
Þ =a
1-q 4
a = > 0 (rejected), if r = 2 and a = - 4 < 0, if r = - 2
é q + 1 æ q + 1ö
2
æ q + 1ö ù
100 3
ê1 + +ç ÷ + ¼+ ç ÷ ú 9
è 2 ø è 2 ø ú
êë 2 û Now, å ai = a1 + a 2 + a3 + ...... + a 9
i =1
é æ q + 1ö ù
101
-4((-2)9 - 1) 4
ê 1-ç ÷ ú = = - (512 + 1)
2101
- (q + 1) 101
è 2 ø ú -2 - 1 3
Þ = a ê1 ×
1-q ê q+1 ú 4
1- = - (513) = 4l (given)
ê 2 ú 3
ë û
q+1 Þ l = - 171
[Q q ¹ 1 Þ q + 1 ¹ 2 Þ ¹1]
2 34. Given series of 40 terms is
a [2101 - (q + 1)101 ] 3 + 4 + 8 + 9 + 13 + 14 + 18 + 19 + ¼+ upto 40 terms
= Þ a = 2100
(1 - q) × 2100 = (3 + 8 + 13 + 18 + ¼ + upto 20 terms)
13 + 23 13 + 23 + 33 + (4 + 9 + 14 + 19 + ¼ + upto 20 terms)
32. Given series, S = 1 + + + ... + 20 20
1+2 1+2+3 = [(2 ´ 3) + (20 - 1)5] + [(2 ´ 4) + (20 - 1)5]
2 2
13 + 23 + 33 + K + 153 1
- (1 + 2 + 3 + K + 15) Q Sum of first n-terms of AP.
1 + 2 + 3 + K + 15 2 n
S n = [2a + (n - 1)d ]
= S1 - S 2 (let) 2
13 + 23 13 + 23 + 33 = 10[6 + (19 ´ 5)] + 10[8 + (19 ´ 5)]
where, S1 = 1 + + +K+
1+2 1+2+3 = 60 + 950 + 80 + 950 = 2040 = 102 m (given)
13 + 23 + 33 + K + 153 Þ m = 20
1 + 2 + 3 + K + 15
2 35. General term of the given series is
æ n (n + 1) ö
15 ç ÷ 3r (12 + 22 + K + r 2) 3r [r (r + 1) (2r + 1)]
15 3
1 + 23 + K + n3 è 2 ø Tr = =
= å = å 2r + 1 6(2r + 1)
n =1 1 + 2 + K + n n =1
n (n + 1 )
1 3
2 = (r + r 2)
é 2
n (n + 1) ù
n 2 n
æ n (n + 1) ö 15 15
êQ å r3 = ç ÷ and å r = ú 1
êë r = 1 è 2 ø r =1 2 úû
Now, required sum = å Tr = 2 å (r3 + r 2)
r =1 r =1
15 15
n (n + 1) 1 1 ïì é n (n + 1) ù n (n + 1) (2n + 1)ïü
2
= å = å (n 2
+ n) = íê + ý
2 2 2 îï ë ú
û
n =1 n =1 2 6 þï n = 15
1 é 15 ´ 16 ´ 31 15 ´ 16 ù
= + 1 ìï n (n + 1) é n 2 + n 2n + 1 ùüï
2 êë 6 2 úû = í ê + úý
2 ïî 2 ë 2 3 ûïþ
é n n (n + 1)(2n + 1) ù n = 15
êQ å r 2= ú 1 ì n (n + 1) (3n 2 + 7n + 2)ü
êë r = 1 6 úû = í ý
2î 2 6 þ n = 15
1
= [(5 ´ 8 ´ 31) + (15 ´ 8)]
2 1 15 ´ 16 (3 ´ 225 + 105 + 2)
= ´ ´ = 7820
= (5 ´ 4 ´ 31) + (15 ´ 4) = 620 + 60 = 680 2 2 6
æ1 1 1 ö
1 1 15 ´ 16 log 2. 5 ç + + + ... to ¥÷
and S 2 = (1 + 2 + 3 + K + 15) = ´ = 60 36. Since, (0.16) è3 3 2 3 3 ø
2 2 2
æ 1/3 ö æ1 ö
Therefore, S = S1 - S 2 = 680 - 60 = 620 log 2.5 çç ÷÷ log 2.5 ç ÷
è 1 - 1/3 ø è 2ø
= (0.16) = (0.16)
33. Let first term and common ratio of given GP a1 , a 2, a3 ,
log 2.5 ( 2) -1 log 22.5
… are a1 = a < 0 and ‘r’ respectively. 16 æ 100 ö
= =ç ÷
Now, a1 + a 2 = 4 (given) 100 è 16 ø
Þ a + ar = 4 Þ a (1 + r ) = 4 …(i) æ 10 ö
2 log 2.5 2
log 2.5 22
a3 + a 4 = 16 (given) =ç ÷ = (2.5) = 22 = 4
è4ø
Sequence and Series 65

37. The 4th AM out of m AMs inserted between 3 and 243 is 40. The terms, which are common in above A.P.s are also in
243 - 3 an AP have common difference = LCM { d1 , d2}
A4 = 3 + 4 …(i)
m+1 (where d1 and d2 are common differences of given two
and the 2nd GM out of three GMs inserted between 3 APs)
æ 1 ö2 and LCM {4, 7} = 28 = d (let)
ç æ 243 ö 3 +1÷ 1/ 2 Now, let mth term of first AP is first common term of
and 243 is G2 = 3 ç ç ÷ ÷ = 3(81) = 27
è 3 ø second AP, which is nth term of second AP.
è ø
So, 3 + (m - 1)4 = 2 + (n - 1)7
æ 240 ö
Q A4 = G 2 Þ 3 + 4 ç ÷ = 27 Þ 4m - 1 = 7n - 5 Þ 7n - 4m = 4
èm + 1ø So, m = 6 and n = 4
æ 240 ö
Þ 4 ç ÷ = 24 Þ m + 1 = 40 Þ m = 39 So, first term which is common, is 3 + (6 - 1)4 = 23
èm + 1ø Now, let there are p terms common to the two given
20 APs, so 23 + ( p - 1)28 £ 407
38. The sum å (1 + 2 + 3 + ¼ + k) Þ 28 p £ 412 Þ p £ 14 . 7
k =1
20
k(k + 1) 1 20 2 So, p = 14, therefore 14 terms are common to the two
= å 2
= å (k + k)
2 k =1
given APs.
k =1
41. Given, 4x2 - 9x + 5 = 0
1 é 20(20 + 1)(40 + 1) 20(20 + 1) ù
= ê + úû Þ (x - 1) (4x - 5) = 0
2ë 6 2
5
1
= [(10 ´ 7 ´ 41) + (10 ´ 21)] Þ AM = , GM = 1 [Q AM > GM ]
2 4
Again, for the series
= 5[287 + 21] = 5 ´ 308 = 1540
7 - 16, 8, - 4, 2 ......
n (n + 1)(2n + 1)
39. The sum, å 4 æ -1 ö
p -1
n =1 pth term t p = - 16 ç ÷
è 2 ø
1 7 1 7
= å
4 n =1
n (2n 2 + 3n + 1) = å (2n3 + 3n 2 + n )
4 n =1 æ -1 ö
q -1
qth term tq = - 16 ç ÷
è 2 ø

2
æ 7 ´ (7 + 1) ö 7(7 + 1)(14 + 1) 7(7 + 1) ù
t p + tq
= ê2 ´ ç ÷ +3 + úû 5
4 êë è 2 ø 6 2 Now, AM = = and GM = t ptq = 1
2 2
1 p+ q-2
= [2 ´ (7 ´ 4)2 + (28 ´ 15) + (7 ´ 4)] æ 1ö
4 Þ 162 ç - ÷ =1
è 2ø
= (8 ´ 49) + (7 ´ 15) + 7 = 7 [56 + 15 + 1]
= 7(56 + 16) Þ (- 2)8 = (- 2)( p + q - 2)
= 7 ´ 72 = 504 Þ p + q = 10
04
Complex Numbers

The equation x 2 + 9 = 0 or x 2 = - 9 has no solution in the set of real numbers. IN THIS CHAPTER ....
There can not be a real number x whose square is - 9, so we need to extend
Complex Numbers
the real number system to a larger system so that we remedy this solution.
Here, we will defined the system of complex number to solve the equation Equity of Complex Numbers
ax 2 + bx + c = 0, where b2 - 4ac < 0. Representation of a Complex
Number in a Plane

Complex Numbers Algebra of Complex Numbers

The numbers of the form x + iy are known as complex numbers. Conjugate of a Complex Numbers

Here, x and y are real numbers and i = -1 is iota. Triangle Inequality


Square Root of a Complex
The complex number is usually denoted by z and its set is denoted by C.
Numbers
e.g. 7 + 2 i , 0 + i , 1 + 0i , C = { x + iy : x , y Î R , i = -1 }
Concept of Rotation
etc., are complex numbers. De-Moivre’s Theorem
Integral Powers of Iota (i) nth Roots of Unity
i = -1 is called the imaginary unit. Application of Complex Numbers
2 3 4 in Coordinate Geometry
Also, i = - 1 , i = - i, i = 1
Equation f a Circle
In general i 4n
= 1 , i 4n + 1 = - i, i 4n + 2 = - 1, i 4n + 3 = - i, for any integer n.
e.g. i1998 = i 4 ´ 499 + 2 = - 1
Note
• The sum of any four consecutive powers of i is zero. i .e. ,
in + in + 1 + in + 2
+ in + 3
=0
• - a = i a, when a is any real number, then, - a -b = i a i b = - ab
But - a - b = (- a)(- b ) = ab is wrong.
Complex Numbers 67

Example 1. The sequence S = i + 2 i 2 + 3 i3 + 4 i 4 + L upto Q Re( z) = 0


100 terms simplifies to, where i = -1 2 - 6 sin 2 q
\ = 0 Þ 2 = 6 sin 2 q
1 + 4 sin 2 q
(a) 50 (1 - i) (b) 25 i
1 1
(c) 25 (1 + i) (d) 100 (1 - i) Þ sin 2 q = Þ sin q = ±
3 3
Sol. (a) S = i + 2i 2 + 3i3 + L + 100i100 æ 1 ö -1 æ 1 ö
Þ q = sin - 1 ç ± ÷ = ± sin ç ÷
S × i = i 2 + 2i3 + L + 99i100 + 100i101 è 3 ø è 3ø
S (1 - i) = i + i 2 + i3 + L + i100 - 100 i101
S (1 - i) = (i + i 2 + i3 + i 4 ) + (i 5 + i6 + i7 + i 8)
Equality of Complex Numbers
Let z1 = x1 + iy1 and z 2 = x2 + iy2 are two complex
+ L + (i97 + i98 + i99 + i100) - 100i numbers, then these two numbers are equal, if
Þ S (1 - i) = 0 + 0 + L + 0 - 100i x1 = x2 and y1 = y2
[Q i n + i n + 1 + i n + 2 + i n + 3 = 0 ] i.e. Re( z1 ) = Re( z 2 )
-100i -100i (1 + i) and Im( z1 ) = Im( z 2 )
Þ S= =
1- i (1 - i) (1 + i) e.g. If z1 = 2 - iy and z 2 = x + 3 i are equal, then
Þ S = - 50 (i - 1) 2 - iy = x + 3 i
\ S = 50 (1 - i) Þ x = 2 and y = - 3
Two complex numbers cannot be compared i.e., no
Representation of Complex Numbers greater complex number can be find in two given complex
in the form x + iy numbers.
Let z = x + iy is a complex number, then x is called the
Example 3. The real values of x and y for which the
real part of z and is denoted by Re ( z ) and y is called the
imaginary part of z and is denoted by Im ( z ). following equality hold, are respectively
e.g. If z = 7 + 4i , then Re( z ) = 7 and Im ( z ) = 4 . ( x 4 + 2 xi ) - (3x 2 + iy) = (3 - 5 i ) + (1 + 2 iy)
A complex number z is said to be purely real, if Im( z ) = 0 (a) 2, 3 or -2 , 1 /3 (b) 1, 3 or -1 , 1 /3
and is said to be purely imaginary, if Re ( z ) = 0. (c) 2 , 1 /3 or -2 , 3 (d) None of these
The complex number 0 = 0 + i 0 is both purely real and Sol. (a) The given equality can be rewritten as
purely imaginary. ( x4 - 3x2) + i(2x - y) = 4 + i(2y - 5)
Every real number ‘a’ can be written as a + i 0. Therefore, Þ x4 - 3x2 = 4 and 2x - y = 2y - 5
every real number is considered as a complex number,
whose imaginary part is zero. Þ x4 - 3x2 - 4 = 0 and 2x - 3y + 5 = 0
Þ ( x2 - 4) ( x2 + 1) = 0
2 + 3i sin q
Example 2. A value of q for which is purely Þ x=±2 [Q x2 ¹ - 1]
1 - 2i sin q
1
imaginary, is (JEE Main 2016) \ At x = 2 , y = 3 and at x = - 2 , y =
3
p p æ 3ö -1 æ 1 ö
(a) (b) (c) sin -1ç ÷ (d) sin ç ÷ 3
3 6 è 4 ø è 3ø æ 1 ö x + iy
Example 4. Let ç -2 - i ÷ = (i = -1,
) where x and
2 + 3i sin q è 3 ø 27
Sol. (d) Let z = is purely imaginary.
1 - 2i sin q y are real numbers, then y - x equals (JEE Main 2019)
Then, we have Re( z) = 0 (a) 91 (b) 85 (c) – 85 (d) – 91
2 + 3i sin q 3 3
Now, consider z = x + iy æ 1 ö é –1 ù
1 - 2i sin q Sol. (a) We have, = ç - 2 - i ÷ = ê (6 + i) ú
27 è 3 ø ë3 û
(2 + 3i sin q) (1 + 2i sin q )
= x + iy 1
(1 - 2i sin q ) (1 + 2i sin q) Þ =- (216 + 108i + 18i 2 + i3)
27 27
2 + 4i sin q + 3i sin q + 6i 2 sin 2 q 1
= =- (198 + 107i)
12 - (2i sin q) 2 27
2 + 7i sin q - 6 sin 2 q [Q( a + b)3 = a3 + b3 + 3a2b + 3ab 2 and i 2 = - 1]
=
1 + 4 sin 2 q On equating real and imaginary part, we get
2
2 - 6 sin q 7 sin q x = - 198 and y = - 107
= +i Þ y - x = - 107 + 198 = 91
1 + 4 sin 2 q 1 + 4 sin 2 q
68 JEE Main Mathematics

Representation of a Complex Number Example 5. Let i = -1, define a sequence of a complex


in a Plane number by z1 = 0, z n + 1 = z n2 + i for n ³ 1. In the complex
Complex numbers can be represented as follows plane, how far from the origin is z111?
(a) 1 (b) 2 (c) 3 (d) 110
Geometrical Representation of a
Complex Number Sol. (b) z1 = 0 , z n + 1 = z n2 + i,n ³ 1
The complex number z = x + iy may be represented z2 = z12 + i at n = 1
graphically by the point P, whose rectangular coordinates z2 = 0 + i Þ z2 = i
are ( x , y ) . Thus, each point in the plane is associated z3 = z 22 + i Þ z3 = i 2 + i = - 1 + i
with a complex number. z 4 = z32 + i Þ z 4 = ( -1 + i) 2 + i
In the figure, P defines z = x + iy. It is customary to = 1 +i 2 - 2i + i = - i
choose x-axis as real axis and y-axis as imaginary axis. z 5 = z 42 + i
Such a plane is called argand plane or argand Þ z 5 = ( -i) 2 + i = - 1 + i
diagram or complex plane or gaussian plane. M M M
Y Hence, z111 = - 1 + i
Imaginary axis
P(x, y) By above, it is clear that for odd suffix of z = - 1 + i
y \ Distance from origin =|z111|
q

O x
X
M ReaI axis = ( -1) 2 + (1) 2 = 2

Eulerian Form of a Complex Number



We have, eiq = cos q + i sin q and e- iq = cos q - i sin q
These two are called Euler’s notations.
Distance of P from origin is OP = x 2 + y 2 . It is called the
Let z be any complex number such that|z| = r and
modulus of z and angle of OP with positive direction of arg( z ) = q. Then, z = x + iy = r (cos q + i sin q ) can be
x-axis is called argument of z. represented in exponential or Eulerian form as
\ tan q =
y æ yö
or q = tan-1 ç ÷ z = reiq = r (cos q + i sin q )
x èxø
5 5
æ 3 iö æ 3 iö
Trigonometrical or Polar Representation of Example 6. Let z = çç + ÷÷ + çç - ÷÷ . If R( z ) and
è 2 2ø è 2 2ø
a Complex Number I( z ) respectively denote the real and imaginary parts of z, then
Let z = x + iy is a complex number which is denoted by a (JEE Main 2019)
point P ( x , y ) in a complex plane, then (a) R( z) > 0 and I( z) > 0 (b) I( z) = 0
OP =|z| and Ð POX = q = arg ( z ) (c) R( z) < 0 and I( z) > 0 (d) R( z) = - 3
5 5
OM x æ 3 iö æ 3 iö
In DPOM , cos q = = Sol. (b) Given, z = ç + ÷ +ç - ÷
OP |z| Y è 2 2ø è 2 2ø
P(x, y)
Þ x =|z|cosq 3 i æ p pö
r Q Euler’s form of+ = ç cos + i sin ÷ = e i ( p/6 )
and sin q =
PM
=
y
q 2 2 è 6 6ø
OP |z| X¢ X
O M 3 i æ -p ö æ pö - ip /6
and - = cosç ÷ + i sinç - ÷ = e
Þ y =|z|sinq 2 2 è 6 ø è 6ø
\ z = x + iy i
5p
-i
5p

Þ z =|z|cos q + i|z|sin q
Y¢ So, z = ( eip/6 ) 5 + ( e-ip/6 ) 5 = e 6 + e 6
Þ z =|z|(cos q + i sin q ) æ 5p 5p ö æ 5p 5p ö
= ç cos + i sin ÷ + ç cos - i sin ÷
è 6 6 ø è 6 6 ø
Þ z = r (cos q + i sin q )
æ yö [Q eiq = cos q + i sin q]
where, r =|z|and q = tan-1 ç ÷ 5p
èxø = 2 cos
6
This form of z is known as polar form. p
\ I( z) = 0 and R( z) = -2 cos = - 3 < 0
In general, polar form is 6
z = r [cos( 2 np + q ) + i sin ( 2 np + q )] é 5p æ pö pù
êQ cos 6 = cosçè p - 6 ÷ø = - cos 6 ú
where, r =|z|, q = arg ( z ) and n Î N . ë û
Complex Numbers 69

Þ (1 + i ) (3 - i ) x - 2 i (3 - i ) + (2 - 3 i ) (3 + i ) y + i (3 + i ) = 10 i
Algebra of Complex Numbers
Þ ( 4 + 2 i) x - 6 i - 2 + (9 - 7i ) y + 3 i - 1 = 10 i
Addition of Complex Numbers Þ ( 4x - 2 + 9y - 1) + i (2x - 6 - 7y + 3) = 10 i
Let z1 = x1 + iy1 and z 2 = x2 + iy2 are two complex Þ ( 4x + 9y - 3) + i(2x - 7y - 3) = 10 i
numbers, then On equating real and imaginary parts on both sides, we get
z1 + z 2 = x1 + iy1 + x2 + iy2 4x + 9y = 3 …(i)
= ( x1 + x2 ) + i ( y1 + y2 ) and 2x - 7y = 13 …(ii)
Þ Re( z1 + z 2 ) = Re( z1 ) + Re( z 2 ) On solving Eqs. (i) and (ii), we get
and Im( z1 + z 2 ) = Im( z1 ) + Im( z 2 ) x = 3, y = -1

Properties of Addition of Complex Numbers Example 8. Let z Î C with Im ( z ) = 10 and it satisfies


(a) z1 + z 2 = z 2 + z1 (commutative law) 2z - n
= 2i - 1 for some natural number n, then
(b) z1 + ( z 2 + z3 )= ( z1 + z 2 ) + z3 (associative law) 2z + n (JEE Main 2019)
(c) z + 0 = 0 + z (where, 0 = 0 + i 0) (a) n = 20 and Re( z) = - 10 (b) n = 40 and Re( z) = 10
(additive identity law) (c) n = 40 and Re( z) = - 10 (d) n = 20 and Re( z) = 10

Subtraction of Complex Numbers Sol. (c) Let z = x + 10i, as Im( z) = 10 given.


Let z1 = x1 + iy1 and z 2 = x2 + iy2 are two complex Since, z satisfies,
2z - n
numbers, then = 2i - 1, n Î N,
2z + n
z1 - z 2 = ( x1 + iy1 ) - ( x2 + iy2 )
Put z = x + 10i, we get
= ( x1 - x2 ) + i ( y1 - y2 )
\ (2x + 20i - n) = (2i - 1) (2x + 20i + n)
Þ Re( z1 - z 2 ) = Re( z1 ) - Re( z 2 )
Þ (2x - n) + 20i = ( -2x - n - 40) + ( 4x + 2n - 20)i
and Im ( z1 - z 2 ) = Im ( z1 ) - Im ( z 2 ) On comparing real and imaginary parts, we get
Multiplication of Complex Numbers 2x - n = - 2x - n - 40 and 20 = 4x + 2n - 20
Þ 4x = - 40 and 4x + 2n = 40
Let z1 = x1 + iy1 and z 2 = x2 + iy2 are two complex
Þ x = - 10 and - 40 + 2n = 40
numbers, then
Þ n = 40
z1 × z 2 = ( x1 + iy1 ) ( x2 + iy2 )
So, n = 40 and x = Re ( z) = - 10
= ( x1x2 - y1 y2 ) + i( x1 y2 + x2 y1 )
Þ z1 × z 2 = [ Re ( z1 ) Re( z 2 ) - Im ( z1 ) Im ( z 2 )] Division of Complex Numbers
+ i [Re( z1 )Im ( z 2 ) + Re( z 2 )Im ( z1 )] Let z1 = x1 + iy1 and z 2 = x2 + iy2 ( ¹ 0) are two complex
numbers, then
Properties of Multiplication of
z1 x1 + iy1
Complex Numbers =
z 2 x2 + iy2
(a) z1 × z 2 = z 2 × z1 (commutative law)
1
(b) ( z1 × z 2 ) z3 = z1( z 2 × z3 ) (associative law) = 2 [( x1x2 + y1 y2 ) + i ( x2 y1 - x1 y2 )]
x2 + y22
(c) If z1 × z 2 = 1 = z 2 × z1 ,then z1 and z 2 are multiplicative z1
inverse of each other. e.g. The value of , where z1 = 2 + 3 i and z 2 = 1 + 2 i , is
z2
(d) (i) z1( z 2 + z3 ) = z1 × z 2 + z1 × z3 (left distribution law)
1 1 - 2i
(ii) ( z 2 + z3 )z1 = z 2 × z1 + z3 × z1 (right distribution law) \ z 2-1 = =
1 + 2 i (1 + 2 i ) (1 - 2 i )
Example 7. The real values of x and y, if 1 - 2i 1 2
= = - i
(1 + i ) x - 2 i (2 - 3 i )y + i 1+ 4 5 5
+ = i , are respectively
(3 + i ) (3 - i ) z1
Thus, = z1 × z 2-1
(a) 3, - 1 (b) 3, 1 z2
(c) -3 , 1 (d) -3 , - 1 æ1 2 ö
(1 + i ) x - 2 i (2 - 3 i )y + i
= (2 + 3 i ) ç - i÷
Sol. (a) + =i è5 5 ø
(3 + i ) (3 - i )
æ 2 6ö æ 4 3ö 8 1
Þ {(1 + i ) x - 2 i } (3 - i ) + {(2 - 3 i ) y + i } (3 + i ) = ç + ÷ + i ç- + ÷ = - i
è 5 5ø è 5 5ø 5 5
= i (3 + i ) (3 - i )
70 JEE Main Mathematics

Alternate Solution Properties of Conjugate of


Here, x1 = 2 , y1 = 3, x2 = 1 and y2 = 2 Complex Numbers
z1 1
\ = {( x1x2 + y1 y2 ) + i( x2 y1 - x1 y2 )} If z , z1 and z 2 are complex numbers, then
z 2 x2 + y2
(i) ( z ) = z
1
= {( 2 ´ 1 + 3 ´ 2) + i ( 3 ´ 1 - 2 ´ 2)} (ii) z + z = 2 Re( z )
12 + 22
(iii) z - z = [2 Im ( z )]
1
= {( 6 + 2) + i ( 3 - 4)} (iv) z = z Þ z is purely real.
5
(v) z = - z Þ z is purely imaginary.
8 1
= - i (vi) zz = {Re( z )} 2 + {Im ( z )} 2 =|z|2
5 5
(vii) z1 + z 2 = z1 + z 2
1+ a
Example 9. If a = cos q + i sin q, then is equal to (viii) z1 - z 2 = z1 - z 2
1- a
q q (ix) z1z 2 = z1 × z 2
(a) i cot (b) i tan
2 2 æz ö z
q q (x) ç 1 ÷ = 1 , z 2 ¹ 0
(c) i cos (d) i cosec è z2 ø z2
2 2
(xi) z n = ( z )n
Sol. (a) Given, a = cos q + i sin q
1 + a 1 + cos q + i sin q Example 10. If w = a + ib, where b ¹ 0 and z ¹ 1, satisfies
Now, =
1 - a 1 - cos q - i sin q æ w - wz ö
(1 + cos q) + i sin q (1 - cos q) + i sin q the condition that ç ÷ is purely real, then the set of
= ´ è 1- z ø
(1 - cos q) - i sin q (1 - cos q) + i sin q
values of z is
sin 2 q + 2i sin q - sin 2 q (a) | z| = 1 , z ¹ 2 (b) | z| = 1and z ¹ 1
=
1 + cos2 q - 2 cos q + sin 2 q (c) z = z (d) None of these
q q w - wz
i 4 sin × cos Sol. (b) Let z1 = be purely real.
= 2 2 1- z
q w - wz w - wz
4 sin 2 Þ z1 = z1 Þ =
2 1- z 1- z
q Þ w - wz - wz + wz × z = w - zw - wz + wz × z
= i cot
2 Þ (w - w ) + (w - w)| z|2 = 0
(w - w )(1 - | z|2 ) = 0
Conjugate of a Complex Number Þ | z |2 = 1 [as w - w ¹ 0, since b ¹ 0]
Let z = x + iy is a complex number, then conjugate of z is \ | z | = 1as z ¹ 1
denoted by z and is equal to x - iy.
Y Example 11. If x + iy = (1 + i )(1 + 2i ) . . . (1 + ni ), then the
P(z) value of x 2 + y 2 is equal to
(a) 2.5 … (1 + n 2) (b) 2.4 … (1 + n 2)
q
X¢ X (c) 2.5 … (1 - n 2) (d) None of these
O -q
Sol. (a) Given, x + iy = (1 + i )(1 + 2i ) . . . (1 + ni ) …(i)
Q(z) \Conjugate of above complex number is
x - iy = (1 - i )(1 - 2i ) . . . (1 - ni ) …(ii)

On multiplying Eqs. (i) and (ii), we get
Thus, z = x - iy
x2 - (iy) 2 = (12 - i 2)[1 - (2i ) 2 ] . . . [12 - (ni ) 2 ]
Geometrically, the conjugate of z is the reflection (image)
Þ x2 + y 2 = (1 + 1)(1 + 4) . . . (1 + n 2)
of point z in the real axis.
e.g., If z = 3 + 4i , then z = 3 - 4i = 2 × 5 . . . (1 + n 2)
Complex Numbers 71

Modulus of a Complex Number Example 12. Find the locus of a complex number
Let z = x + iy is a complex number, then modulus of a z - 5i
z = x + iy, which satisfy the equation = 1.
complex number z is denoted by|z| . z + 5i
Y
x P(x, y) (a) X-axis (b) Y-axis
(c) origin (d) None of these
(0, y)
y2

y
Sol. (a) Put z = x + iy in the given equation.
+
Öx 2

x + iy - 5i
\ =1
q x + iy + 5i
X¢ X
O (x, 0) x + i(y - 5)
Y¢ Þ =1
x + i(y + 5)
\ |z| = x 2 + y 2 = { Re ( z )} 2 + {Im ( z )} 2 x + i(y - 5) z1 | z1|
Þ =1 Q =
e.g. If z = 4 + 3 i is a complex number, then x + i(y + 5) z2 | z2 |

|z| = 42 + 32 = 16 + 9 = 25 = 5 \ x2 + (y - 5) 2 = x2 + (y + 5) 2

In geometrical representation, the modulus of a complex Þ x2 + (y - 5) 2 = x2 + (y + 5) 2


number is equal to the distance between origin and point Þ x + y - 10y + 25 = x2 + y 2 + 25 + 10y
2 2

P. i.e. OP = x 2 + y 2 Þ 20y = 0 Þ y = 0
Hence, it lies on X-axis.
Properties of Modulus of Complex Numbers z -a
(i)|z| ³ 0 Þ|z| = 0, iff z = 0 and|z| > 0, iff z ¹ 0 Example 13. If (a ÎR) is a purely imaginary number
z +a
(ii) -|z| £ Re( z ) £|z| and -|z| £ Im( z ) £|z| and| z| = 2, then a value of a is (JEE Main 2019)
(iii)|z| =|z| =| - z| =|- z| 1
(iv) zz =|z| 2 (a) 2 (b) (c) 1 (d) 2
2
(v)|z1z 2| =|z1||z 2| z-a
In general,|z1z 2z3K z n | =|z1||z 2||z3|...|z n | Sol. (d) Since, the complex number ( a ÎR) is purely
z+a
z |z | z-a z-a
(vi) 1 = 1 , ( z 2 ¹ 0) imaginary number, therefore + =0 [Q a ÎR]
z 2 |z 2| z+a z+a
(vii)|z1 ± z 2| £|z1| +|z 2| Þ zz - az + az - a 2 + zz - az + az - a 2 = 0
In general,|z1 ± z 2 ± z3 ± K ± z n | Þ 2 z
2
- 2 a 2 = 0 [Q zz = z ]
2

£|z1| +|z 2| +|z3| + K +|z n | 2


Þ a2 = z =4 [| z | = 2, given]
(viii)|z1 ± z 2| ³||z1| -|z 2||
Þ a=±2
(ix)|z n | =|z|n
(x)||z1| -|z 2|| £|z1 + z 2| £|z1| +|z 2| Argument/Amplitude of
Thus,|z1|+|z 2|is the greatest possible value of|z1 + z 2|
and||z1| -|z 2||is the least possible value of|z1 + z 2| .
Complex Numbers
(xi)|z1 ± z 2|2 = ( z1 ± z 2 ) ( z1 ± z 2 ) Let z = x + iy be a non-zero Y
2 2 complex number and can be
=|z1| +|z 2| ± ( z1z 2 + z1z 2 ) represented in the form
N P(x, y) = (r, q)
=|z1|2 +|z 2|2 ± Re( z1z 2 ) z = r(cos q + i sin q ) where, r is
y2

the modulus and q is the


+
Öx 2

=|z1|2 +|z 2|2 ± 2|z1||z 2|cos(q 1 - q 2 ) y(= r sin q)


argument of z.
r=

(xii) z1z 2 + z1z 2 = 2|z1||z 2| cos (q 1 - q 2 ) q


From the figure, let z be X¢ X
where, q 1 = arg ( z1 ) and q 2 = arg( z 2 ) O x(= r cos q) M
represented by a line OP
z inclined at an angle q with the
(xiii)|z1 + z 2|2 =|z1|2 +|z 2|2 Û 1 is purely imaginary. Y¢
z2 positive direction of x-axis or
(xiv)|z1 + z 2|2 +|z1 - z 2|2 = 2 {|z1|2 +|z 2|2 } the angle measured in the counter clockwise direction
and the distance of the point from O in the direction q is
(xv)|az1 - bz 2|2 +|bz1 + az 2|2 r, i.e., z . Then, in right angled DOPM, right angled at M,
= ( a 2 + b 2 ) (|z1|2 +|z 2|2 ), where a , b Î R we have
y æ yö æ yö
Note If z is unimodular, then| z| = 1. Now, if f( z) is a unimodular, tan q = Þ q = tan-1 ç ÷ or arg ( z ) = tan-1 ç ÷
then it always be expressed as f( z) = cos q + i sin q, q Î R. x èxø èxø
72 JEE Main Mathematics

Principal Value of Argument = r [ - cos q + i sin q ]


The value of q of the argument which satisfies the = - r (cos q - i sin q )
inequality - p < q £ p is called the principal value of the = -w
argument. 3 + i sin q
Example 15. If , q Î [0, 2 p], is a real number,
Principal values of the argument are q , p - q , - p + q , - q 4 - i cos q
according as the complex number lies on the Ist, IInd, then an argument of sin q + i cos q is (JEE Main 2020)
IIIrd or IVth quadrant.
æ 4ö æ3ö
Y
(arg z = q)
(a) p - tan -1ç ÷ (b) - tan -1ç ÷
(arg z = p – q) è3ø è 4ø
x<0
æ 4ö æ3ö
y>0 x>0 (c) tan -1ç ÷ (d) p - tan -1ç ÷
y>0 è3ø è 4ø
q q
X¢ X Sol. (a) Given complex expression
q q x>0
x<0 3 + i sin q 3 + i sin q 4 + i cos q
y<0 y<0 , q Î[0 , 2p ] = ´
4 - i cos q 4 - i cos q 4 + i cos q
[on rationalization]
(arg z = –p + q) (arg z = -q) (12 - sin q cos q) + i( 4 sin q + 3 cos q)
Y¢ =
16 + cos2 q
Note Argument of 0 is not defined. 3 + i sin q
Q is a real number, so 4 sin q + 3 cos q = 0 …(i)
Properties of Argument of Complex Numbers 4 - i cos q
Now, argument of sin q + i cos q lies in either quadrant
If z1 , z 2 and z3 are three complex numbers, then
second or fourth because according to Eq. (i) sin q and cos q
(i) arg ( z1z 2 ) = arg ( z1 ) + arg ( z 2 ) + 2kp ( k = 0 or 1 or - 1) are opposite in sign, so either,
In general, arg ( z1z 2z3K z n ) = arg ( z1 ) + arg ( z 2 ) cos q ½
arg(sin q + i cos q) = p - tan -1½ [for second quadrant]
+ arg ( z3 ) + K + arg ( z n ) + 2kp ½ sin q ½
æ 4ö æ cos q 4ö
( k = 0 or 1 or - 1) = p - tan -1ç ÷ ç from Eq. (i) =- ÷
è3ø è sin q 3ø
æ z1 ö
(ii) arg ç ÷ = arg ( z1 ) - arg ( z 2 ) + 2kp ( k = 0 or 1 or - 1) cos q ½ æ 4ö
è z2 ø or, arg(sin q + i cos q) = - tan -1½ = - tan -1ç ÷
½ sin q ½ è3ø
æzö
(iii) arg ç ÷ = 2 arg ( z ) + 2kp ( k = 0 or 1 or - 1)
èzø Triangle Inequality
n
(iv) arg ( z ) = n arg ( z ) + 2kp ( k = 0 or 1 or - 1) In any triangle, sum of any two sides is greater than the
æz ö æz ö third side and difference of any two sides is less than the
(v) If arg ç 2 ÷ = q , then arg ç 1 ÷ = 2kp - q, where k Î I third side.
è z1 ø è z2 ø
(i) z1 ± z 2 £ z1 + z 2
(vi) arg ( z ) = - arg ( z )
(ii) z1 ± z 2 ³ z1 - z 2
(vii) If arg ( z ) = 0 Þ z is real.
(iii) In general,
(viii) arg ( z1z 2 ) = arg ( z1 ) - arg ( z 2 ) z1 + z 2 + . . . + z n £ z1 + z 2 + z3 + . . . + z n
p
(ix)|z1 + z 2| =|z1 - z 2| Þ arg ( z1 ) - arg ( z 2 ) =
2 Example 16. If|z1 - 1| < 1,|z 2 - 2| < 2,|z3 - 3| < 3,
(x)|z1 + z 2| =|z1| +|z 2| Þ arg ( z1 ) = arg ( z 2 ) then|z1 + z 2 + z3|
(a) is less than 6
(xi) If|z1| £ 1,|z 2| £ 1, then
(b) is more than 3
(a) |z1 - z 2|2 £ (|z1| -|z 2|)2 + [arg ( z1 ) - arg ( z 2 )]2 (c) is less than 12
(b) |z1 + z 2|2 ³ (|z1| +|z 2|)2 - [arg ( z1 ) - arg ( z 2 )]2 (d) lies between 6 and 12
Sol. (c) Q |z1 - 1| < 1;|z 2 - 2| < 2 and|z3 - 3 | < 3
Example 14. Let z and w be two non-zero complex numbers
\ |z1 + z 2 + z3| = |( z1 - 1) + ( z 2 - 2) + ( z3 - 3) + 6|
such that| z | = |w| and arg( z ) + arg(w) = p. Then, z is equal to
Þ|z1 + z 2 + z3| £ |( z1 - 1)| + |( z 2 - 2)| + |( z3 - 3) + 6|
(a) w (b) - w (c) - w (d) w
Þ|z1 + z 2 + z3| £ |z1 - 1| + |z 2 - 2| + |z3 - 3| + 6
Sol. (c) Let| z | = |w| = r and arg(w) = q Þ|z1 + z 2 + z3| £ 1 + 2 + 3 + 6
Also, arg( z) + arg(w) = p Þ arg( z) = p - q \ |z1 + z 2 + z3| £ 12
Now, z = r [cos( p - q ) + i sin( p - q )] Hence,|z1 + z 2 + z3| is less than 12.
Complex Numbers 73

On solving the equations x2 + y 2 = 14 and x2 - y 2 = 4, we get


Square Root of a Complex Number
x2 = 9 and y 2 = 5
Let a + ib is a complex number such that a + ib = x + iy ,
\ x = ± 3 and y = ± 5
where x and y are real numbers.
Since, xy > 0 , it follows that x and y are of the same sign.
Now, a + ib = x + iy Þ ( x + iy )2 = a + ib
\ x = 3, y = 5 or x = - 3, y = - 5
Þ ( x 2 - y 2 ) + 2 ixy = a + ib
So, ( 4 + 3 -20 )1/ 2 = ( 4 + 6 i 5)1/ 2
2 2
Þ x -y =a …(i) = ± (3 + 5 i ) …(i)
and 2xy = b …(ii) \ ( 4 - 3 - 20 ) 1/ 2
= ± (3 - 5 i ) …(ii)
Now, ( x + y 2 )2 = ( x 2 - y 2 )2 + 4 x 2 y 2
2
1/ 2 1/ 2
Hence, ( 4 + 3 -20 ) + ( 4 - 3 -20 ) =±6
Þ ( x 2 + y 2 )2 = a 2 + b2 [on adding Eqs. (i) and (ii)]
Þ ( x 2 + y 2 ) = a 2 + b2 [Q x 2 + y 2 > 0] …(iii) Alternate Solution
Here, x = 4, y = 6 5
On solving Eqs. (i) and (iii), we get
\ ( 4 + 3 -20 )1 / 2 + ( 4 - 3 -20 )1/ 2
æ1ö
x 2 = ç ÷ ( a 2 + b2 + a )
è 2ø = 2 { 4 2 + (6 5) 2 + 4}
æ1ö = 2 ( 16 + 180 + 4)
and y 2 = ç ÷ ( a 2 + b2 - a )
è 2ø
= 2 (14 + 4) = 36 = ± 6
æ1ö
Þ x = ± ç ÷ [ a 2 + b2 + a ]
è 2ø
Concept of Rotation
æ1ö Let z1, z 2 and z3 be the vertices of a DABC described in
and y = ± ç ÷ [ a 2 + b2 - a ]
è 2ø anti-clockwise sense. Draw OP and OQ parallel and equal
If b is positive, then the sign of x and y from Eq. (ii) will to AB and AC, respectively. Then, point P is z 2 - z1 and Q
be same i.e., is z3 - z1. If OP is rotated through Ða in anti-clockwise
é æ1ö ù sense it coincides with OQ.
æ1ö
a + ib = ± ê ç ÷ ( a 2 + b2 + a ) + i ç ÷ ( a 2 + b2 - a ) ú z - z1 ia
z3 - z1 OQ
êë è 2 ø è 2ø CA ia
úû Then, = (cos a + i sin a ) = e = 3 e
z 2 - z1 OP BA z 2 - z1
If b is negative, then the sign of x and y will be opposite,
i.e., Y
C(z3)
é æ1ö æ1ö ù
a + ib = ± ê ç ÷ ( a 2 + b2 + a ) - i 2 2
ç ÷ ( a + b - a)ú
êë è 2 ø è 2ø úû
B(z2)
Q(z3 – z1)
Note If x, y Î R, then A(z1)
2 2
• x + iy + x - iy = 2 ( x + y + x)
a P(z2 – z1)
• x + iy - x - iy = i 2 ( x2 + y2 - x) X
O

Example 17. The value of é OQ CA ù


êQ DOPQ and DABC are congruent, \ =
( 4 + 3 - 20 )1/ 2 + ( 4 - 3 - 20 )1/ 2 is ë OP BA úû
(a) ± 6 (b) 0 (c) ± 5 (d) ± 3 æ z - z1 ö
or amp ç 3 ÷ =a
Sol. (a) We may write, è z 2 - z1 ø
( 4 + 3 -20 ) = ( 4 + 6 i 5)
Example 18. If z1, z 2 and z3 are vertices of an isosceles
Let ( 4 + 3 -20 )1/ 2 = ( x + iy)
triangle and right angled at z 2, then
Then, ( 4 + 6 i 5)1/ 2 = ( x + iy)
(a) z12 + z 22 + z32 = 2 z 2 ( z1 + z3)
Þ 4 + 6 i 5 = ( x2 - y 2) + (2xy) i
(b) z12 + 2 z 22 + z32 = 2 z 2 ( z1 + z3)
Þ x2 - y 2 = 4 and 2xy = 6 5
\ x2 + y 2 = ( x2 - y 2) 2 + 4x2y 2 (c) 2 z12 + z 22 + z32 = 2 z 2 ( z1 + z3)

= (16 + 180) = 196 = 14 (d) z12 + z 22 + 2 z32 = 2 z 2 ( z1 + z3)


74 JEE Main Mathematics

3
Sol. (b) If z1, z 2 and z3 are vertices of an isosceles triangle and é æ 2 2p 2 2 2p ö æ 2p 2p ö ù
right angled at z 2 i.e. |z3 - z 2| = |z1 - z 2| ê çè sin 9 - i cos 9 ÷ø + çè sin 9 + i cos 9 ÷ø ú
=ê ú
z3 - z 2 |z3 - z 2 | i p/ 2 ê 2p 2p ú
i.e. = e 1 + sin - i cos
z1 - z 2 |z1 - z 2 | êë 9 9 úû
z3
Þ z3 - z 2 = i ( z1 - z 2) C éæ 2p 2p ö ì 2p 2p üù
3

Þ z32 + z 22 - 2 z 2z3 = - ( z12 + z 22 - 2 z1z 2) ê çè sin 9 + i cos 9 ÷ø × íî sin 9 - i cos 9 + 1ýþ ú


=ê ú
Þ z12 + 2z 22 + z32 = 2 z1z 2 + 2 z 2z3 ê 2p 2p ú
Bz 1 + sin - i cos
Þ z12 + 2 z 22 + z32 = 2z 2 ( z1 + z3) 2 z1A êë 9 9 úû
3 3
æ 2p 2p ö æ 2 2p 2p ö
= ç sin + i cos ÷ = ç - i sin + i cos ÷
è 9 ø è 9 ø
De-Moivre’s Theorem 9
3
9
3
é 2p 2p ù é æ 2p ö æ 2p ö ù
A simple formula for calculating powers of complex = i3 ê cos - i sin = - i ê cos ç - ÷ + i sinç - ÷
numbers in the form of cos q and sin q is known as ë 9 9 úû ë è 9 ø è 9 ø úû
de-Moivre’s theorem. If n is a rational number, then é 2p 2p ù
= - i ê cos - i sin
(cos q + i sin q )n = cos n q + i sin n q ë 3 3 úû
[according the De-Movier’s theorem]
Applications of De-Moivre’s Theorem é æ 1ö 3 ù 3 i 1
(i) If z = (cos q 1 + i sin q 1 ) (cos q 2 + i sin q 2 ) = - iê ç - ÷ - i ú=- + = - ( 3 - i)
ë è 2 ø 2 û 2 2 2
...(cos q n + i sin q n )
8
Then, z = cos(q 1 + q 2 + K + q n ) æ p pö
ç1 + sin + i cos ÷
+ i sin(q 1 + q 2 + K + q n ) Example 20. The expression ç 8 8 ÷ is equal to
(ii) If z = r (cos q + i sin q ) and n is a positive integer, ç1 + sin p - i cos p ÷
then è 8 8ø
é æ 2kp + q ö æ 2kp + q ö ù (a) 1 (b) - 1 (c) i (d) - i
( z )1/ n = r1/ n ê cos ç ÷ + i sin ç ÷ú 8
ë è n ø è n øû æ p pö
ç1 + sin + i cos ÷
where, k = 0, 1, 2 , 3, K ,( n - 1) Sol. (b) ç 8 8
p p÷
(iii) (cos q - i sin q )n = cos nq - i sin nq ç 1 + sin - i cos ÷
è 8 8ø
1 8
(iv) = (cos q + i sin q )-1 = cos q - i sin q éæ p pö
2
æ 2 p 2 p ö
ù
cos q + i sin q ê çè cos + sin ÷ + i ç cos - sin ÷ú
16 16 ø è 16 16 ø ú
(v) (sin q ± i cos q )n ¹ sin nq ± i cos nq =ê 2
êæ p pö æ p p öú
n ê çè cos + sin ÷ - i ç cos2 - sin 2 ÷ ú
é æp ö æp öù ë 16 16 ø è 16 16 øû
(vi) (sin q + i cos q )n = ê cos ç - q ÷ + i sin ç - q ÷ ú
ë è 2 ø è 2 øû éæ p pö æ p p öù
8

é ê çè cos 16 + sin 16 ÷ø + i çè cos 16 - sin 16 ÷ø ú


æ np öù é æ np öù =ê ú
= ê cos ç - nq ÷ ú + i ê sin ç - nq ÷ ú
ë è 2 øû ë è 2 øû ê cos p + sin p - i æç cos p - sin p ö÷ ú
n êë 16 16 è 16 16 ø úû
(vii) (cos q + i sin f ) ¹ cos nq ± i sin nf 8
é p p ù
Note ê cos 16 (1 + i) + sin 16 (1 - i) ú
=ê ú
• 1 = cos 0 + i sin 0 • -1 = cos p + i sin p p p
p p p p ê cos (1 - i) + sin (1 + i) ú
• i = cos + i sin • - i = cos - i sin ë 16 16 û
2 2 2 2 8
é p æ1 - i ö p ù
3 8 ê cos +ç ÷ sin ú
æ 2p 2p ö æ1 + i ö ê 16 è1 + i ø 16 ú
ç1 + sin + i cos ÷ =ç ÷
Example 19. The value of ç 9 9 ÷ is è1 - i ø ê p æ1 + i ö p ú
ç1 + sin 2 p - i cos 2 p ÷ ê cos 16 + çè 1 - i ÷ø sin 16 ú
ë û
è 9 9 ø 8
é p p ù
(JEE Main 2020)
ê cos 16 - i sin 16 ú
1 1 1 1 = i8 ê
(a) - ( 3 - i) (b) - (1 - i 3) (c) ( 3 - i) (d) (1 - i 3) p p ú
2 2 2 2 ê cos + i sin ú
3
ë 16 16 û
æ 2p 2p ö é 8p 8p ù
ç1 + sin + i cos ÷ cos - i sin
9 9 4 2 ê 16 16 ú = - 1
Sol. (a) Given expression, ç = (i ) ê
2p 2p ÷ 8p 8p ú
ç 1 + sin - i cos ÷ ê cos + i sin ú
è 9 9 ø ë 16 16 û
Complex Numbers 75

= 24 + 9 a – 24 [Q1 + a + a 2 = 0 ]
Cube Roots of Unity Þ 9 a = a + ba [given]
Let x = 3 1 Þ x 3 - 1 = 0 Þ ( x - 1) ( x 2 + x + 1) = 0 \ a = 0 and b = 9
-1 + i 3 -1 - i 3 \ a+ b =9
Therefore, x = 1, ,
2 2
If second root be represented by w , then third root will Example 22. If z 2 + z + 1 = 0, where z is a complex
be w2. number, then find the value of
\ Cube roots of unity are 1, w , w2; 1 is a real root of unity æ 1ö
2
æ 2 1ö æ 3 1ö
2
æ 6 1ö
2 2
çz + ÷ + çz + 2 ÷ + çz + 3 ÷ + ... çz + 6 ÷
and other two i.e., w and w2 are conjugate complex of each è zø è z ø è z ø è z ø
other. (a) 5 (b) 12 (c) –12 (d) 5

Properties of Cube Roots of Unity Sol. (b) \ z 2 + z + 1 = 0


3 3r
(i) w = 1 or w =1 -1 ± 12 - 4(1) - 1 ± 3i
z= =
(ii) w3 r + 1 = w, w3 r + 2 = w2 2(1) 2

(iii) 1 + wr + w2r = 0, if r is not a multiple of 3 - 1 + 3i


Þ z = w , w2, where w =
2
= 3, if r is multiple of 3
- 1 - 3i
(iv) Each complex cube root of unity is square of other and w2 =
2
and also reciprocal of each other. 2 2 2 2
æ 1ö æ 2 1ö æ 3 1ö æ 4 1ö
(v) If a is any positive number, then a1/ 3 has roots \ çz + ÷ + çz + 2 ÷ + çz + 3 ÷ + çz + 4 ÷
è zø è z ø è z ø è z ø
a1/ 3 (1), a1/ 3 ( w), a1/ 3 ( w2 ) and if a is any negative 2 2
æ 1ö æ 1ö
number, then a1/ 3 has roots - a
1/ 3
,- a
1/ 3
w, + ç z 5 + 5 ÷ + ç z6 + 6 ÷
è z ø è z ø
1/ 3
- a w2. Put z = w , we get
2 2 2 2
Y æ 1ö æ 2 1ö æ 3 1ö æ 4 1 ö
(vi) The cube roots of unity when çw + ÷ + çw + 2 ÷ + çw + 3 ÷ + çw + 4 ÷
represented on complex plane w 1 è wø è w ø è w ø è w ø
2 2
lie on vertices of an equilateral æ 1ö æ 1ö
X¢ + ç w5 + 5 ÷ + ç w6 + 6 ÷
triangle inscribed in a unit –1 O 1
X è w ø è w ø
circle having centre at origin.
= ( w + w2) 2 + ( w2 + w) 2 + (1 + 1) 2 + ( w + w2) 2
One vertex being on positive w2 –1
real axis. + ( w2 + w) 2 + (1 + 1) 2

= ( - 1) 2 + ( - 1) 2 + 4 + ( - 1) 2 + ( - 1) 2 + 4 (Q1 + w + w2 = 0 )
Important Relations
= 1 + 1 + 4 + 1 + 1 + 4 = 12
(i) x 2 + xy + y 2 = ( x - yw)( x - yw2 )
Similarly, for putting z = w2, we get the result 12
(ii) x 2 - xy + y 2 = ( x + yw)( x + yw2 )
(iii) x 3 + y3 = ( x + y )( x + yw)( x + yw2 ) Application of Complex Numbers in
3 3
(iv) x - y = ( x - y )( x - yw)( x - yw ) 2
Coordinate Geometry
Example 21. If a and b are real numbers such that Distance between Two Points
-1 + i 3 Distance between two points P ( z1 ) and Q( z 2 ) is
(2 + a ) 4 = a + ba , where a = , then a + b is equal to
2 Q(z2)
(JEE Main 2020) P(z1)
(a) 24 (b) 33 (c) 9 (d) 57
PQ =|z 2 - z1|
Sol. (c) It is given that, (2 + a) 4 = a + ba, where a, b Î R
Section Formula
–1 + i 3
and a = × Let R( z ) divides a join of P ( z1 ) and Q( z 2 ) in the ratio m : n.
2
(i) If R( z )divides the line segment PQ internally, then
–1 + i 3
\a= is non-real complex root of unity and we n
2 m
Q(z2)
know that a3 = 1 and 1 + a + a 2 = 0 , so P(z1) R(z)
(2 + a) 4 = 16 + 32 a + 24 a 2 + 8 a3 + a 4 mz 2 + nz1
z=
= 16 + 8 a + 24( a + a 2) + 8 + a [Q a3 = 1] m+n
76 JEE Main Mathematics

(ii) If R( z )divides the line segment PQ externally, then (iii) If a 1 and a 2 are slopes of two lines in a complex
n plane, then
m
R(z) (a) lines will be parallel, if a 1 = a 2.
Q(z2)
P(z1) (b) lines will be perpendicular, if a 1 + a 2 = 0.
mz 2 - nz1 (iv) Slope of a line PQ joining two points P ( z1 ) and
z=
m-n z - z2
Q ( z2 ) = 1 .
Note z1 - z 2
• Distance of a point P ( z) from the origin =| z| . (v) Length of perpendicular from a point P ( z1 ) to the
z + z2 line
• If R ( z) is a mid-point of PQ, then z = 1 .
2 |az1 + az1 + b| |az1 + az1 + b|
az + az + b = 0 = =
C(z3) |a| +|a | 2|a |
A(z1) B(z2)
(vi) The coordinates of centroid of a DABC whose vertex
• Three points will be collinear, if for A ( z1 ), B( z2 ), C ( z3 ), then z + z 2 + z3
are A( z1 ), B( z 2 ) and C( z3 ), is G( z ) = 1
AB + BC = AC 3
i .e. | z1 - z2| + | z2 - z3| = | z1 - z3|
(vii) The triangle whose vertices are z1 , z 2 and z3 is
equilateral iff
Equation of Perpendicular Bisector 1 1 1
P(z1) + + =0
z1 - z 2 z 2 - z3 z3 - z1

R(z)
or z12 + z 22 + z32 = z1z 2 + z 2z3 + z3 z1
é ( z - z3 )( z1 - z 4 ) ù
(viii) If arg ê 2 ú = ± p , 0 (or purely real),
ë ( z1 - z3 )( z 2 - z 4 ) û
then the points z1 , z 2 , z3 and z 4 are concyclic.
Q(z2) æ z - z1 ö
(ix) arg ç ÷ = 0 Þ Locus of z is a straight line
If P ( z1 ) and Q( z 2 ) are two fixed points and R( z ) is an è z - z2 ø
equidistant point from P and Q. Then, passing through z1 and z 2.
|z - z1| =|z - z 2| Þ|z - z1|2 =|z - z 2|2
Þ ( z - z1 )( z - z1 ) = ( z - z 2 )( z - z 2 ) Equation of a Circle
Þ z( z1 - z 2 ) + z ( z1 - z 2 ) =|z1|2 -|z 2|2 A circle is the locus of all those points which
Hence, R( z )lies on perpendicular bisector of P ( z1 ) and Q( z 2 ). are equidistant from the given fixed point. This fixed
point is called the centre and the fixed distance is called
Equation of Straight Line radius.
(i) Parametric form Equation of line joining points (i) Equation of a circle whose radius is r and centre is
P ( z1 ) and Q( z 2 ) is z = tz1 + (1 - t )z 2 , where t Î R . C( z 0 ), is|z - z 0| = r. If the centre of circle lies on the
(ii) Non-parametric form Equation of line joining origin, then equation of circle is|z| = r .
points P ( z1 ) and Q( z 2 ) is P(z)
z z 1 r
z1 z1 1 = 0
C(z0)
z2 z2 1

Þ z( z1 - z2 ) - z ( z1 - z2 ) + z1 z2 - z2 z1 = 0
(iii) General equation General equation of straight (ii) The general equation of a circle is
line is az + az + b = 0, where a is a complex number zz + az + az + b = 0, where a Î C and b Î R . Centre
and b is a real number. of circle is at - a and radius is |a|2 - b.
Some Important Results (iii) If P ( z1 ) and Q( z 2 ) are the vertices of diameter of a
(i) Three points z1 , z 2 and z3 will be collinear, if circle, then equation of circle is
z1 z1 1 ( z - z1 )( z - z2 ) + ( z - z2 ) ( z - z1 ) = 0
z2 z2 1 = 0 R(z)
z3 z3 1
a P(z1) Q(z2)
(ii) Slope of line az + az + b = 0 is - .
a
Complex Numbers 77

Nature of Circle Þ ( x - 2) 2 + y 2 = 4[( x - 3) 2 + y 2 ]


The general equation of circle z z + az + a z + b = 0 Þ x2 + 4 - 4x + y 2 = 4( x2 + 9 - 6x + y 2)
represents a Þ 3x2 + 3y 2 - 20 x + 32 = 0
2
(i) real circle, if a > b 20 32
2
Þ x2 + y 2 - x+ =0
(ii) point circle, if a = b 3 3
2 Standard equation of circle is x2 + y 2 + 2gx + 2fy + c = 0 .
(iii) an imaginary circle, if a < b
-10 32
\ g = , f = 0, c =
3 3
Position of a Point with respect to Circle
æ10 ö
Let the general equation of circle is z z + az + a z + b = 0 \ Certre of circle = ( - g , - f ) = ç ,0÷
è3 ø
whose centre z 0 and radius a.
2
There are following position with respect to circle are æ10 ö 32 100 32
\ Radius = g 2 + f 2 - c = ç ÷ + (0) 2 - = -
given below. è3ø 3 9 3
(i) If the point ( z 0 ) lies inside the circle, then z - z 0 < a. 100 - 96 4 2
= = =
(ii) If the point ( z 0 ) lies on the circle, then z - z 0 = a. 9 9 3
(iii) If the point ( z 0 ) lies outside the circle, then
Example 24. Let z1 and z 2 be two complex numbers
z - z 0 > a.
satisfying| z1| = 9 and| z 2 - 3 - 4i | = 4. Then, the minimum
z -2 value of| z1 - z 2 | is
Example 23. If = 2 represents a circle, then centre (JEE Main 2019)
z -3 (a) 1 (b) 2 (c) 2 (d) 0
and radius are Sol. (d) Clearly| z1| = 9, represents a circle having centre C1(0 , 0)
æ10 ö 2 æ - 10 ö 2 and radius r1 = 9.
(a) ç ,0÷, (b) ç ,0÷,
è3 ø 3 è 3 ø 3
and| z 2 - 3 - 4i| = 4 represents a circle having centre C 2(3, 4)
æ10 ö 2 and radius r2 = 4.
(c) ç , 1÷ , (d) None of these
è3 ø 3 The minimum value of| z1 - z 2| is equals to minimum
Sol. (a) Let z = x + iy distance between circles| z1| = 9
x + iy - 2 and| z 2 - 3 - 4i| = 4.
\ Given equation becomes, =2
x + iy - 3 Q C1C 2 = (3 - 0) 2 + ( 4 - 0) 2 = 9 + 16 = 25 = 5
Þ ( x - 2) + iy = 2 ( x - 3) + iy and |r1 - r2| = |9 - 4| = 5 Þ C1C 2 =|r1 - r2|
\ Circles touches each other internally.
Þ ( x - 2) 2 + y 2 = 2 ( x - 3) 2 + y 2
Hence,| z1 - z 2|min = 0
Practice Exercise
ROUND I Topically Divided Problems
Algebraic Operations and 9. The square root of 7 + 24 i are
Square Root of Complex Number (a) ± (3 + 4 i ) (b) ± (3 - 4 i )
1. Number of integral values of n for which the (c) ± (4 + 3 i ) (d) ± (4 - 3 i )
quantity ( n + i) 4 , where i 2 = - 1 is an integer, is q + ir p + iq 1 + iz
10. Given z = , then = , if
(a) 1 (b) 2 (c) 3 (d) 4 1+ p 1+ r 1 - iz
2 n
(1 + i) 2 n (a) p 2 + q 2 + r 2 = 1 (b) p 2 + q 2 + r 2 = 2
2. The complex number 2n
+ , n Î I, (c) p 2 + q 2 - r 2 = 1 (d) None of these
(1 + i) 2n
is equal to 11. If 3 + i = ( a + ib) ( c + id), then
(a) 0 (b) 2 æ bö ædö
tan -1 ç ÷ + tan -1 ç ÷ has the value
(c) {1 + (-1)n } × i n (d) None of these èaø ècø
3. The imaginary part of p p
(a) + 2 np , n Ï I (b) np + ,n ÎI
1/ 2 1/ 2 3 6
(3 + 2 -54 ) - (3 - 2 - 54 ) can be p p
(JEE Main 2020) (c) np - , n Î I (d) 2np - , n Î I
3 3
(a) - 6 (b) - 2 6 (c) 6 (d) 6
1 12. The value of x 4 + 9 x 3 + 35 x 2 - x + 4 for
4. Real part of is
1 - cos q + i sin q x = - 5 + 2 -4 is
1 1 1 (a) 0 (b) –160
(a) - (b) (c) tan q / 2 (d) 2 (c) 160 (d) –164
2 2 2
5. If Re ( z2 ) = 0,|z|= 2, then 13. Given that the equation z2 + ( p + iq) z + r + is = 0,
(a) z = + 2 ± i 2 (b) z = + 3 ± i 3 where p, q, r and s are real and non-zero roots, then
(c) z = + 3 ± i 2 (d) None of these (a) pqr = r 2 + p 2s (b) prs = q 2 + r 2p
2 2
6. There is only one way to choose real numbers M (c) qrs = p + s q (d) pqs = s 2 + q 2r
and N such that when the polynomial
5 x 4 + 4 x 3 + 3x 2 + Mx + N is divided by the
Conjugate, Modulus and Argument of
polynomial x 2 + 1 , the remainder is 0. If M and N Complex Number
assume these unique values, then M - N is 4
14. If½z - ½ = 2, then the maximum value of|z|is
(a) -6 (b) -2 (c) 6 (d) 2 ½ z½
7. If x = a + ib is a complex number such that equal to
x 2 = 3 + 4 i and x 3 = 2 + 11 i, where i = -1, then (a) 3+1 (b) 5+1 (c) 2 (d) 2 + 2
( a + b) is equal to 2 2
15. If|z - 1|= |z| + 1, then z lies on
(a) 2 (b) 3 (c) 4 (d) 5 (a) the real axis (b) the imaginary axis
2 2 1 + iz
2 (c) a circle (d) an ellipse
8. If z (1 + a) = b + ic and a + b + c = 1, then is
1 - iz
16. If the four complex numbers z, z, z - 2 Re( z) and
equal to z - 2 Re( z) represent the vertices of a square of side
a + ib b - ic
(a) (b) 4 units in the argand plane, then|z|is equal to
1+ c 1+ a (JEE Main 2020)
a + ic
(c) (d) None of these (a) 4 2 (b) 2 (c) 2 2 (d) 4
1+ b
Complex Numbers 79

z-1 (a) equal to one (b) greater than one


17. If is a purely imaginary number ( z ¹ - 1), then
z+1 (c) zero (d) less than one
the value of|z|is 27. arg ( z) is equal to
(a) –1 (b) 1 (c) 2 (d) –2 (a) p - arg (z) (b) 2p - arg (z)
18. The complex numbers sin x + i cos 2 x and (c) p + arg (z) (d) 2p + arg (z)
cos x - i sin 2 x are conjugate to each other for 28. If arg ( z - 1) = arg ( z + 3i), then x - 1 : y is equal to
æ 1ö
(a) x = np (b) x = ç n + ÷ p (a) 3 : 1 (b) 1 : 3 (c) 3 : 2 (d) 2 : 3
è 2ø
(c) x = 0 (d) No value of x 29. z1 and z2 are two complex numbers such that
|z1 |= |z 2 |and arg ( z1) + arg ( z 2) = p, then z1 is
z1
19. If = 1 and arg ( z1z2 ) = 0, then equal to
z2
(a) 2z2 (b) z2
(a) z1 = z2 (b)|z2|2 = z1z2 (c) -z2 (d) None of these
(c) z1z2 = 1 (d) None of these 7-z
30. If f ( z) = , where z = 1 + 2 i, then| f ( z)|is
20. If z + 2 |z + 1|+ i = 0, then z equals 1 - z2
1 | z|
(a) 2 + i (b) - 2 + i (c) - +i (d) - 2 - i (a) (b) | z |
2 2
21. The complex numbers z1 and z2 are such that z1 ¹ z2 (c) 2| z | (d) None of these
-1
and|z1|= |z2|. If z1 has positive real part 31. If tan (a + i b) = x + iy, then x is equal to
æ z + z2 ö 1 æ 2a ö 1 æ 2a ö
and z2 has negative imaginary part, then ç 1 ÷ (a) tan -1 ç ÷ (b) tan -1 ç ÷
è z1 - z2 ø 2 2 2
è1 - a - b ø 2 2 2
è1 + a + b ø
may be æ 2a ö
(c) tan -1 ç ÷ (d) None of these
(a) zero (b) real and positive è1 - a 2 - b 2ø
(c) real and negative (d) purely imaginary
32. If z1, z2 and z3 are complex numbers such that
22. Let a complex number z,|z|¹ 1, satisfy
1 1 1
æ |z|+ 11 ö |z1|= |z2|= |z3|= + + = 1 , then|z1 + z2 + z3|is
log 1 ç 2÷
£ 2. Then, the largest value of|z| z1 z2 z3
2 è (|z|- 1) ø
(a) equal to 1 (b) less than 1
is equal to (JEE Main 2021) (c) greater than 3 (d) equal to 3
(a) 8 (b) 7 (c) 6 (d) 5 z-1
33. If|z|= 1 and w = (where, z ¹ - 1) , then Re ( w) is
23. If|z - 2 |= min {|z - 1|,|z - 5 |} , where z is a z+ 1
1
complex number, then (a) 0 (b) -
3 7 |z + 1|2
(a) Re (z ) = (b) Re (z ) =
2 2 z 1 2
(c) × (d)
ì 3 7ü z + 1 |z + 1|2 |z + 1|2
(c) Re (z ) Î í , ý (d) None of these
î 2 2þ
34. If z1, z2 and z3, z4 are two pairs of conjugate complex
24. If for the complex numbers z1 and z2 æz ö æ z2 ö
|1 - z1z2|2 - |z1 - z2|2 = k (1 - |z1|2 ) (1 - |z2|2 ), then k numbers, then arg ç 1 ÷ + arg ç ÷ is equal to
è z4 ø è z3 ø
is equal to
p 3p
(a) 1 (b) -1 (c) 2 (d) 4 (a) 0 (b) (c) (d) p
2 2
z-a
25. The region of the complex plane for which =1 (1 + i) 2 2
z+a 35. If a > 0 and z = , has magnitude , then z
a-i 5
[Re( a) ¹ 0 ], is
is equal to (JEE Main 2019)
(a) X-axis (b) Y -axis
(c) the straight line x = a (d) None of these 1 3 1 3 1 3 3 1
(a) - i (b) - - i (c) - + i (d) - - i
5 5 5 5 5 5 5 5
26. If|ak|< 1, l k ³ 0 for k = 1, 2, K, n and
l1 + l 2 + K + l n = 1, then the value of
|l1a1 + l 2 a2 + K + l n an |is
80 JEE Main Mathematics

36. If z and w are two non-zero complex numbers such 45. Let z be a complex number satisfying|z - 5 i|£ 1
p such that amp(z) is minimum. Then, z is equal to
that|zw|= 1 and arg ( z) - arg ( w) = , then zw is
2 2 6 24 i 24 2 6 i
(a) + (b) +
equal to 5 5 5 5
(a) 1 (b) -i (c) i (d) -1 2 6 24 i
(c) - (d) None of these
5 5
37. If z and w are two complex numbers such that
p
|zw|= 1 and arg( z) - arg( w) = , then De-Moivre’s Theorem
2 (JEE Main 2019)
4 (cos 75° + i sin 75° )
1-i 46. The value of is
(a) zw = - i (b) zw = 0. 4 (cos 30° + i sin 30° )
2
-1 + i 2 2 10 10
(c) zw = i (d) zw = (a) (1 + i ) (b) (1 - i ) (c) (1 - i ) (d) (1 + i )
2 10 10 2 2

38. If in polar form z1 = (1, a ), z 2 = (1, b), z3 = (1, g ) and 47. If (cos q + i sin q) (cos 2 q + i sin 2 q) K
z1 + z 2 + z3 = 0, then z1-1 + z-21 + z3-1 is equal to (cos nq + i sin nq ) = 1, then the value of q is
2mp 4mp mp
(a) 1 (b) 0 (a) 4mp (b) (c) (d)
n (n + 1) n (n + 1) n (n + 1)
(c) –1 (d) None of these
(1 + nz) æç1 + ö÷
c n
39. The maximum distance from the origin of 48. 1 - c2 = nc - 1 and z = eiq, then
2n è zø
coordinates to the point z satisfying the equation
1 is equal to
z+ = a is
z (a) 1 - c cos q (b) 1 + 2 c cos q
1 1 (c) 1 + c cos q (d) 1 - 2 c cos q
(a) ( a 2 + 1 + a ) (b) ( a 2 + 2 + a )
2 2 49. The real part of (1 - cos q + 2 i sin q) -1 is
1
(c) ( a 2 + 4 + a ) (d) None of these 1 1
2 (a) (b)
3 + 5 cos q 5 - 3 cos q
1 1
Triangle Inequality (c) (d)
3 - 5 cos q 5 + 3 cos q
40. For all complex numbers z1, z2 satisfying|z1|= 12 1+ a
and|z2 - 3 - 4 i |= 5, the minimum value of|z1 - z2| 50. If a = cos q + i sin q, then is equal to
1- a
is q q q
(a) 0 (b) 2 (c) 7 (d) 17 (a) cot (b) cot q (c) i cot (d) i tan
2 2 2
41. If z is a complex number, then the minimum value 3
51. If x + iy = , then x 2 + y 2 is equal to
of|z|+ |z - 1|is 2 + cos q + i sin q
(a) 1 (b) 0 (a) 3x - 4 (b) 4x - 3
1 (c) 4x + 3 (d) None of these
(c) (d) None of these
2 ra ra
52. If zr = cos 2
+ i sin 2 , where r = 1, 2 , 3, K, n, then
42. If z and w are two complex numbers such that n n
|z|£ 1 ,|w|£ 1 and|z + i w|= |z - i w|= 2, then lim z1 z 2 z3 K zn is equal to
n®¥
æ aö æ aö
z equals (a) cos a + i sin a (b) cos ç ÷ - i sin ç ÷
è2ø è2ø
(a) 1 or i (b) i or - i (c) 1 or - 1 (d) i or - 1
3
(c) eia/ 2 (d) eia
43. The maximum value of|z|where z satisfies the 6
æ 2 pk 2 pk ö
condition z +
2
= 2 , is 53. The value of å çsin
è 7
- i cos
7 ø
÷ is
z k=1

(a) 3 -1 (b) 3+1 (a) - 1 (b) 0 (c) - i (d) i


(c) 3 (d) 2+ 3
54. The imaginary part of
44. If|z1 - 1|< 1 ,|z 2 - 2 |< 2,|z3 - 3|< 3, then ( z - 1)(cos a - i sin a ) + ( z - 1) -1 ´ (cos a + i sin a ) is
|z1 + z 2 + z3 | zero, if
(a) is less than 6 (b) is more than 3 (a) |z - 1| = 2 (b) arg (z - 1) = 2 a
(c) is less than 12 (d) lies between 6 and 12 (c) arg (z - 1) = a (d) |z | = 1
Complex Numbers 81

55. If a = cos a + i sin a , b = cos b + i sin b, 64. If x = a + b, y = aa + bb and z = ab + ba, where a and
b c a b are complex cube roots of unity, then xyz is equal to
c = cos g + i sin g and + + = 1, then 2
c a b (a) a + b 2 (b) a 3 + b 3 (c) a 3 b 3 (d) a 3 - b 3
cos (b - g ) + cos ( g - a ) + cos (a - b) is equal to 65. If i = -1, then
3 3
(a) (b) - (c) 0 (d) 1 æ 1 3ö
334
æ1 3ö
365
2 2 4 + 5 ç- + i ÷ -3ç + i ÷ is equal to
è 2 2 ø è2 2 ø
56. If cos a + cos b + cos g = sin a + sin b + sin g = 0, then
(a) 1 - i 3 (b) -1 + i 3 (c) 4 3i (d) -i 3
cos 3 a + cos 3 b + cos 3 g is equal to
(a) 0 (b) cos (a + b + g ) 66. If w is a complex cube root of unity, then for positive
(c) 3 cos (a + b + g ) (d) 3 sin (a + b + g ) integral value of n, the product of w × w2 × w3. . . . . wn
1/ 4
57. The values of (16) are will be
(a) ±2 , ± 2 i (b) ±4 , ± 4 i 1-i 3 1-i 3
(a) (b) -
(c) ±1, ± i (d) None of these 2 2
2p ö (c) 1 (d) Both (b) and (c)
58. If a = cos æç æ 2 p ö , then the quadratic
÷ + i sin ç ÷
è 7 ø è 7 ø 67. If 1, w and w2 are the cube roots of unity, then
equation whose roots are a = a + a 2 + a 4 and 1 wn w2 n
b = a 3 + a 5 + a 6 , is D = wn w2 n 1 is equal to
(a) x 2 - x + 2 = 0 (b) x 2 + x - 2 = 0 w2 n 1 wn
(c) x 2 - x - 2 = 0 (d) x 2 + x + 2 = 0
(a) 0 (b) 1 (c) w (d) w2
Cube Roots of Unity 68. If a ¹ 1 is any nth root of unity, then
2p 2p
59. Let wn = cos æç ö÷ + i sin æç ö÷, i 2 = - 1, then S = 1 + 3 a + 5 a 2 + . . . upto n terms, is equal to
è n ø è n ø 2n 2n n n
(a) (b) - (c) (d) -
( x + yw3 + zw 23) (x + yw 23 + zw3) is equal to 1-a 1-a 1-a 1-a
(a) 0 69. Common roots of the equations
(b) x2+ y2+ z2 z3 + 2 z2 + 2 z + 1 = 0 and z1985 + z100 + 1 = 0 are
(c) x 2 + y 2 + z 2 - yz - zx - x y (a) w , w 2 (b) w , w 3
(d) x 2 + y 2 + z 2 + yz + zx + x y
(c) w 2, w 3 (d) None of these
30
æ -1 + i 3 ö
60. The value of ç ÷ is 70. The value of the expression
è 1- i ø (JEE Main 2020) 1 × ( 2 - w) ( 2 - w2 ) + 2 × (3 - w) (3 - w2 ) + …
15 15 15 5
(a) -2 (b) 2 i (c) -2 i (d) 6 + ( n - 1) × ( n - w) ( n - w2 ),
61. If a , b and g are the cube roots of a positive number where w is an imaginary cube root of unity, is
1 1
p, then for any real x, y and z, the expression (a) (n - 1) n (n 2 + 3n + 4) (b) (n - 1) n (n 2 + 3n + 4)
2 4
æ ax + by + gz ö 1 1
ç ÷ is equal to (c) (n + 1) n (n 2 + 3n + 4) (d) (n + 1) n (n 2 + 3n + 4)
è bx + gy + az ø 2 4
-1 - 3 i -1 + 3 i 71. The value of the expression
(a) (b)
2 æç1 + ö÷ æç1 + 2 ö÷ + 3 æç 2 + ö÷ æç 2 + 2 ö÷
2 2 1 1 1 1
1+ 3i 1- 3i è wø è w ø è wø è w ø
(c) (d)
+ 4 æç3 + ö÷ æç3 + 2 ö÷ + . . . + ( n + 1) æç n + ö÷
2 2 1 1 1
è w ø è w ø è wø
62. If i = -1, then
æ n + 1 ö,
æ 1
334 365 ç ÷
3ö æ 1 3ö è w2 ø
4 + 5 ç- + i ÷ + 3 ç- + i ÷ is equal to
è 2 2 ø è 2 2 ø where w is an imaginary cube root of unity, is
(a) 1 - i 3 (b) -1 + i 3 n (n 2 + 2) n (n 2 - 2)
(a) (b)
(c) i 3 (d) -i 3 3 3
n 2(n + 1)2
63. -1 - 1 - -1 - . . . ¥ is equal to (c) +n (d) None of these
4
2
(a) 1 (b) –1 (c) w (d) - w
82 JEE Main Mathematics

72. If 1, a1, a2 , …, an - 1 are n roots of unity, then the 80. The locus of the points z which satisfy the condition
value of (1 - a1) (1 - a2 ) (1 - a3) . . . (1 - an - 1) is æ z - 1ö p
arg ç ÷ = , is
equal to è z + 1ø 3
(a) 3 (b) 1/2
(a) a straight line (b) a circle
(c) n (d) 0
2
(c) a parabola (d) None of these
5
73. If x 2 - x + 1 = 0, then the value of å æç x n + n ö÷ is
1
è 81. Let z be a complex number and a be a real
n=1 x ø
parameter such that z2 + ax + a 2 = 0, then
(a) 8 (b) 10 (a) locus of z is a pair of straight lines
(c) 12 (d) None of these (b) locus of z is a circle
5p
Applications in Geometry (c) arg (z ) = ±
3
æ z-1 ö (d) |z | = - 2|a|
74. If Im ç ÷ = - 4, then locus of z is
è 2z + 1ø 82. If z1, z 2 , z3 and z4 are the affixes of four points in
(a) an ellipse (b) a parabola the argand plane and z is the affix of a point, such
(c) a straight line (d) a circle that|z - z1|= |z - z2|= |z - z3|= |z - z4|, then
z1, z 2 , z3 and z4 are
75. In the argand plane the complex number z = 4 - 3 i
(a) concyclic
is turned in the clockwise sense through 180° and (b) vertices of a parallelogram
stretched three times. The complex number
(c) vertices of a rhombus
represented by the new number is
(d) in a straight line
(a) 12 + 9i (b) 12 - 9i
(c) -12 - 9i (d) - 12 + 9i 83. The points represented by the complex numbers
5
76. If the complex numbers z1 and z2 and the origin 1 + i, -2 + 3 i and i on the argand diagram are
3
form an equilateral triangle, then z 12 + z 22 is equal to
(a) vertices of an equilateral triangle
(a) z1z 2 (b) z1z 2 (b) vertices of an isosceles triangle
(c) z 2z1 (d) |z1| 2 = |z 2| 2 (c) collinear
77. If the equation a|z|2 + az + az + d = 0 represents a (d) None of the above

circle where a, d are real constants then which of 84. If the area of the triangle on the complex plane
the following condition is correct? (JEE Main 2021) formed by the points z, z + iz and iz is 200, then the
2
(a) |a| - ad ¹ 0 value of |3z|must be equal to
(b) |a|2 - ad > 0 and a Î R - {0} (a) 20 (b) 40 (c) 60 (d) 80
(c) |a|2 - ad ³ 0 and a Î R 85. Let z1, z2 and z3 be three vertices of an equilateral
(d) a = 0, a , d Î R+ 1
triangle circumscribing the circle|z |= .
78. Let z1 and z 2 be the non-real roots of the equation 2
3z 2 + 3z + b = 0. If the origin together with the 1 3
If z1 = +i and z1, z2 , z3 are in anti-clockwise
points represented by z1 and z 2 form an equilateral 2 2
triangle, then the value of b is sense, then z2 is equal to
(a) 1 (b) 2 (a) 1 + i 3 (b) 1 - i 3 (c) 1 (d) -1
(c) 3 (d) None of these 86. If the points z1, z 2 and z3 are the vertices of an
79. A complex number z is said to be unimodular, if equilateral triangle in the complex plane, then the
|z|= 1. Suppose z1 and z2 are complex numbers such value of z 12 + z 22 + z 32 is equal to
z1 z 2 z3
z - 2 z2 (a) + + (b) z1z 2 + z 2z 3 + z 3 z 1
that 1 is unimodular and z2 is not z 2 z3 z1
2 - z1z2
z1 z 2 z 3
unimodular. Then, the point z1 lies on a
(JEE Main 2015)
(c) z1z 2 - z 2z 3 - z 3 z1 (d) - - -
z 2 z3 z1
(a) straight line parallel to X-axis
87. If the complex numbers iz, z and z + iz represent the
(b) straight line parallel to Y -axis
three vertices of a triangle, then the area of the
(c) circle of radius 2
triangle is (JEE Main 2021)
(d) circle of radius 2
1 1 1
(a) 1 (b) |z| 2 (c) (d) |z + iz|2
2 2 2
Complex Numbers 83

88. The area of the triangle whose vertices are 94. If z is a complex number satisfying
represented by the complex numbers 0, z, zei a , |z|2 + 2 ( z+ z) + 3 i ( z - z) + 4 = 0, then complex
(0 < a < p) is equal to number z + 3 + 2 i will lie on
1 1 (a) circle with centre (1 - 5i ), radius 4
(a) |z| 2 cos a (b) |z| 2 sin a
2 2 (b) circle with centre (1 + 5i ), radius 4
1 1
(c) |z| 2 sin a cos a (d) |z| 2 (c) circle with centre (1 + 5i ), radius 3
2 2
(d) circle with centre (1 - 5i ), radius 3
89. If A and B be two complex numbers satisfying æ z-1 ö
A B 95. If Re ç ÷ = 1, where z = x + iy, then the point
+ = 1. Then, the two points represented by A è 2z + i ø
B A
( x, y) lies on a (JEE Main 2020)
and B and the origin form the vertices of
2
(a) an equilateral triangle (a) straight line whose slope is - .
3
(b) an isosceles triangle which is not equilateral
(c) an isosceles triangle which is not right angled æ 1 3ö
(b) circle whose centre is at ç - , - ÷ .
(d) a right angled triangle è 2 2ø
3
90. Let the lines ( 2 - i) z = ( 2 + i) z and (c) straight line whose slope is .
2
( 2 + i) z + ( i - 2) z - 4 i = 0, (here i 2 = -1) be normal to
5
a circle C. If the line iz + z + 1 + i = 0 is tangent to (d) circle whose diameter is .
2
this circle C, then its radius is (JEE Main 2021)
3 3 1 96. A point z moves on the curve|z - 4 - 3 i |= 2 in an
(a) (b) 3 2 (c) (d)
2 2 2 2 2 argand plane. The maximum and minimum values
91. The equation zz + ( 2 - 3 i) z + ( 2 + 3 i) z + 4 = 0 of|z| are
represents a circle of radius (a) 2, 1 (b) 6, 5
(c) 4, 3 (d) 7, 3
(a) 2 (b) 3 (c) 4 (d) 6
z+i 97. Let z1, z 2 and z 3 be the affixes of the vertices of a
92. When is purely imaginary, the locus described
z+2 triangle having the circumcentre at the origin. If z
by the point z in the argand diagram is a is the affix of it’s orthocentre, then z is equal to
5 5 z1 + z 2 + z 3 z1 + z 2 + z 3
(a) circle of radius (b) circle of radius (a) (b)
2 4 3 2
(c) straight line (d) parabola (c) z1 + z2 + z3 (d) None of these
93. The equation of a circle whose radius and centre æ|z|2 - |z|+ 1 ö
98. If log ç ÷ < 2, then the locus of z is
are r and z0 respectively, is 3
è 2 + | z| ø
2
(a) zz - zz0 - zz0 + z0z0 = r
(a) | z | = 5 (b) | z |< 5
(b) zz + zz0 - zz0 + z0z0 = r 2
(c) | z | > 5 (d) None of these
(c) zz - zz0 + zz0 - z0z0 = r 2
(d) None of the above

ROUND II Mixed Bag


Only One Correct Option (a) | z1 | >
1
(b) | z1 | <
1
(c) | z1 | >
1
(d) | z | <
1
3 4 4 3
1. If|z - iRe ( z)|= |z - Im ( z)|(where i = -1), then z
lies on 3. The point of intersection of the curves
(a) Re (z ) = 2 3p p
arg ( z - 3 i) = and arg ( 2 z + 1 - 2 i) = ,
(b) Im (z ) = 2 4 4
(c) Re (z ) + Im (z ) = 0 (where i = -1) is
1 1
(d) None of the above (a) (3 + 9 i ) (b) (3 - 9 i )
4 4
2. If z1 is a root of the equation 1
a0n zn + a1zn - 1 + . . . + an - 1z + an = 3, where|ai |< 2 (c) ( 3 + 2 i ) (d) No point
2
for i = 0, 1, . . . , n. Then,
84 JEE Main Mathematics

4. The complex number z satisfies the condition 13. If z1 = a + ib and z 2 = c + id are complex numbers
½z - 25½ = 24 . The maximum distance from the such that|z 1|= |z 2|= 1 and Re( z1z 2 ) = 0, then the
½ z½ pair of complex numbers w1 = a + ic and w2 = b + id
origin of coordinates to the point z is satisfies
(a) 25 (b) 30 (a) |w1| = 2 (b) |w2| = 3
(c) 32 (d) None of these (c) Re|w1 w2| = 0 (d) None of these

5. The trigonometric form of z = (1 - i cot 8) 3 14. The real value of q for which the expression
1 + i cos q
(where i = -1) is is a real number, is
æ 3pö æ 3pö 1 - 2 i cos q
i ç 24 - ÷ -i ç 24 - ÷
è 2ø è 2ø
3
(a) cosec 8 × e 3
(b) cosec 8 × e p p
(a) 2np + , n ÎI (b) 2np - , n ÎI
æ pö
i ç36 - ÷
æ pö
-i ç 24 - ÷
3 3
(c) cosec 3 8 × e è 2ø
(d) cosec 2 8 × e è 2ø p p
(c) 2np ± , n Î I (d) 2np ± , n Î I
2 4
6. If the cube roots of unity are 1, w and w2 , then the
15. If|z1|= |z 2|= 1 and amp z1 + amp z 2 = 0, then
roots of the equation ( x - 1) 3 + 8 = 0, are
(a) -1, 1 + 2 w , 1 + 2 w 2 (b) -1, 1 - 2 w , 1 - 2 w 2 (a) z1z 2 = 1 (b) z1 + z 2 = 0
(c) -1, - 1, - 1 (d) -1, - 1 + 2 w , - 1 - 2 w 2 (c) z1 = - z 2 (d) None of these
7. If x = 91/ 391/ 9 91/ 27 . . . ¥, y = 41/ 34 -1/ 9 41/ 27 . . . ¥ and 16. If|z1 |= 15 and|z 2 - 3 - 4 i |= 5, then
¥
(a) | z1 - z 2| min = 15
z= å (1 + i) - r , then arg ( x + yz) is equal to
(b) | z1 - z 2| min = 10
r =1
(c) | z1 - z 2| max = 20
æ 2ö
(a) 0 (b) p - tan -1 ç ÷ (d) | z1 - z 2|max = 25
è 3 ø
æ 2ö
17. The region represented by
æ 2 ö
(c) - tan -1 ç ÷ (d) - tan -1 ç ÷ {z = x + iy Î C :|z|- Re ( z) £ 1} is also given by the
è 3 ø è 3ø
inequality (JEE Main 2020)
8. If z1 and z 2 be complex numbers such that z1 ¹ z 2 (a) y2 ³ 2(x + 1)
æ 1ö
(b) y2 £ 2ç x + ÷
and|z1 |= |z 2 |. If z1 has positive real part and z 2 è 2ø
has negative imaginary part, then 1
(c) y2 £ x + (d) y2 ³ x + 1
[( z1 + z 2 )/( z1 - z 2 )] may be 2
(a) purely imaginary (b) real and positive 18. The equation|z - i|= |z - 1|, i = -1, represents
(c) real and negative (d) None of these
(JEE Main 2019)
9. If|z1 |= |z 2 |, arg ( z1/ z 2 ) = p, then z1 + z 2 is equal to (a) a circle of radius
1
2
(a) 0 (b) purely imaginary
(b) line passing through the origin with slope 1
(c) purely real (d) None of these
(c) a circle of radius 1
10. If|z|< 2 - 1, then|z 2 + 2 z cos a |is (d) line passing through the origin with slope - 1
(a) less than 1 (b) 2 + 1 19. Let z1 and z2 be two non-real complex cube roots of
(c) 2 - 1 (d) None of these unity and| z - z1|2 + | z - z2|2 = l be the equation of
11. If A ( z1), B ( z 2 ) and C ( z3) are the vertices of the a circle with z1, z2 as ends of a diameter, then the
value of l is
DABC such that
(a) 4 (b) 3 (c) 2 (d) 2
( z1 - z 2 )/( z3 - z 2 ) = (1/ 2 ) - ( i /1 / 2) , then DABC is
(a) equilateral (b) right angled ìa + i ü
20. All the points in the set S = í : a Î Rý ( i = -1)
(c) isosceles (d) obtuse angled îa - i þ
12. If z1, z 2 and z3 are the vertices of an equilateral lie on a (JEE Main 2019)
DABC such that|z1 - i |= |z2 - i |= |z3 - i |, then (a) circle whose radius is 2
|z1 + z2 + z3 |is equal to (b) straight line whose slope is -1
1 (c) circle whose radius is 1
(a) 3 3 (b) 3 (c) 3 (d)
3 3 (d) straight line whose slope is 1
Complex Numbers 85

21. Let z1 and z2 be any two non-zero complex numbers 29. A function f is defined by f ( z) = ( 4 + i) z2 + az + g
3z1 2z for all complex numbers z, where a and g are
such that 3|z1|= 4|z2 |. If z = + 2 , then
2 z2 3z1 complex numbers. If f (1) and f ( i) are both real,
(JEE Main 2019) then the smallest possible value of|a |+ |g |equals
1 17 5 (a) 1 (b) 2 (c) 2 (d) 2 2
(a) |z| = (b) |z| =
2 2 2
30. Let z be a complex number such that|z|+ z = 3 + i
(c) Re(z) = 0 (d) None of these
(where i = - 1). Then,|z|is equal to
22. The complex number w satisfying the equation (JEE Main 2019)
w3 = 8 i and lying in the second quadrant of the 34 5 41 5
complex plane is (a) (b) (c) (d)
3 3 4 4
3 1
(a) - 3 + i (b) - + i 31. Let z = x + iy be a non-zero complex number such
2 2
(c) - 2 3 + i (d) - 3 + 2 i that z2 = i|z|2 , where i = -1, then z lies on the
(JEE Main 2020)
23. If z and w are two complex numbers simultaneously (a) line y = - x (b) imaginary axis
satisfying the equations, z3 + w5 = 0 and z2 × w 4 = 1 , (c) line y = x (d) real axis
then æ p ö
32. If zn = cos ç ÷
(a) z and w both are purely real è ( 2 n + 1) ( 2 n + 3) ø
(b) z is purely real and w is purely imaginary æ p ö
(c) w is purely real and z is purely imaginary
+ i sin ç ÷,
è ( 2 n + 1) ( 2 n + 3) ø
(d) z and w both are imaginary
z-i then lim ( z1 × z2 × z3 L zn ) is equal to
24. Let z be a complex number such that = 1 and n®¥
z + 2i p p p p
5 (a) cos + i sin (b) cos + i sin
|z|= . Then, the value of|z + 3i|is 3 3 6 6
2 (JEE Main 2020) 5p 5p 3p 3p
(c) cos + i sin (d) cos + i sin
7 15 6 6 2 2
(a) 10 (b) (c) (d) 2 3
2 4 33. If z = e2 p i / 3, then 1 + z + 3z2 + 2 z3 + 2 z4 + 3z5 is
25. If z1 and z1 represents adjacent vertices of a regular equal to
polygon of n sides with centre at the origin and (a) - 3ep i/3 (b) 3epi /3
Im ( z1) (c) 3e2 pi/3 (d) -3e2p i/3
= 2 - 1 , then the value of n is
Re ( z1)
34. sin -1 æç ( z - 1) ö÷, where z is non-real, can be the
1
(a) 8 (b) 12 (c) 16 (d) 24 èi ø
angle of a triangle, if
26. The least value of|z|where z is complex number
(a) Re (z ) = 1, Im (z ) = 2 (b) Re(z ) = 1, 0 < Im|z | < 1
which satisfies the inequality (c) Re(z ) + Im (z ) = 0 (d) None of these
æ (|z|+ 3)(|z|- 1) ö
exp ç log e 2 ÷ ³ log 2|5 7 + 9i| 35. If z be a complex number satisfying
è ||z|+ 1| ø | Re( z)|+|Im( z)|= 4, then|z|cannot be
(JEE Main 2020)
i = -1, is equal to (JEE Main 2021)
17
(a) 3 (b) 5 (c) 2 (d) 8 (a) 10 (b) 7 (c) (d) 8
2
27. If|z + 4 |£ 3, then greatest and least values of|z + 1 |
36. If z1, z2 , z3 and z4 are the vertices of a square in that
are
order, then which of the following does not hold good?
(a) 4, 1 (b) 6, 0 (c) 6, 1 (d) 4, 0
z1 - z2
(a) is a purely imaginary
28. If z1, z2 are complex numbers such that z3 - z2
Re( z1) = |z1 - 1| , Re( z2 ) = |z2 - 1| and z - z3
(b) 1 is a purely imaginary
p z2 - z4
arg( z1 - z2 ) = , then Im( z1 + z2 ) is equal to
6 z - z2
(JEE Main 2020) (c) 1 is a purely imaginary
3 1 2 z3 - z4
(a) (b) (c) (d) 2 3
2 3 3 (d) None of the above
86 JEE Main Mathematics

3 i z2
37. If z = + ( i = -1), then (1 + iz + z5 + iz8 ) 9 is 42. If z ¹ 1 and is real, then the point represented
2 2 z-1
equal to (JEE Main 2019) by the complex number z lies
(a) 1 (b) (-1 + 2i )9 (c) -1 (d) 0 (a) either on the real axis or on a circle passing through
the origin
38. If 1, a 1, a 2 , a 3, a 4 are the roots of z5 - 1 = 0, then
(b) on a circle with centre at the origin
w - a1 w-a2 w-a3 w-a4
the value of . . . is (c) either on the real axis or on a circle not passing
w2 - a 1 w2 - a 2 w2 - a 3 w2 - a 4 through the origin
(where w is imaginary cube root of unity) (d) on the imaginary axis
(a) 1 (b) w
(c) w2 (d) None of these Numerical Type Questions
m/ 2 n/ 3
39. If z is a complex number such that|z|³ 2, then the æ1 + i ö æ1 + i ö
43. If ç ÷ =ç ÷ = 1, ( m, n Î N ), then the
1 è1 - i ø è i - 1ø
minimum value of z +
2 (JEE Main 2014)
greatest common divisor of the least values of m
5 and n is ......... . (JEE Main 2020)
(a) is equal to
2 ( -1 + i 3) 21 (1 + i 3) 21
(b) lies in the interval (1, 2) 44. Let i = -1. If + = k and
(1 - i) 24 (1 + i) 24
5
(c) is strictly greater than n = [|k|] be the greatest integral part of|k|. Then,
2
n+ 5 n+ 5
(d)
3
is strictly greater than but less than
5
å j = 0 ( j + 5) 2 - å j = 0 ( j + 5) is equal to
2 2 (JEE Main 2021)

z = max {|z - 1 |,|z + 1 |}, then


40. If|| 45. Suppose A be a complex number and n Î N , such
1 that A n = ( A + 1) n = 1, then the least value of n is
(a) |z + z| = (b) z + z = 1 (c) |z + z| = 1 (d) z Î f
2 46. Let z be those complex number which satisfy
41. If z is a complex number of unit modulus and |z + 5|£ 4 and z(1 + i) + z(1 - i) ³ -10, i = -1. If the
æ1 + zö maximum value of|z + 1|2 is a + b 2, then the
argument q, then arg ç ÷ is equal to
è1 + z ø (JEE Main 2013)
value of (a + b) is (JEE Main 2021)

p 47. If a + ib, b ¸ 0 is a root of z5 = 1, then the value of


(a) -q (b) -q (c) q (d) p - q
2 4a (b 4 - a 4 ) is

Answers
Round I
1. (c) 2. (c) 3. (b) 4. (b) 5. (a) 6. (c) 7. (b) 8. (a) 9. (c) 10. (a)
11. (b) 12. (b) 13. (d) 14. (b) 15. (b) 16. (c) 17. (b) 18. (d) 19. (b) 20. (d)
21. (d) 22. (b) 23. (c) 24. (a) 25. (b) 26. (d) 27. (b) 28. (b) 29. (c) 30. (a)
31. (a) 32. (a) 33. (a) 34. (a) 35. (b) 36. (b) 37. (a) 38. (b) 39. (c) 40. (b)
41. (a) 42. (c) 43. (b) 44. (c) 45. (a) 46. (d) 47. (c) 48. (c) 49. (d) 50. (c)
51. (b) 52. (c) 53. (d) 54. (c) 55. (d) 56. (c) 57. (a) 58. (d) 59. (c) 60. (c)
61. (a) 62. (c) 63. (c) 64. (b) 65. (c) 66. (c) 67. (a) 68. (b) 69. (a) 70. (b)
71. (a) 72. (c) 73. (a) 74. (d) 75. (d) 76. (a) 77. (b) 78. (a) 79. (c) 80. (b)
81. (a) 82. (a) 83. (c) 84. (c) 85. (d) 86. (b) 87. (b) 88. (b) 89. (a) 90. (c)
91. (b) 92. (a) 93. (a) 94. (c) 95. (d) 96. (d) 97. (c) 98. (b)

Round II
1. (c) 2. (a) 3. (d) 4. (a) 5. (a) 6. (b) 7. (c) 8. (a) 9. (a) 10. (a)
11. (c) 12. (c) 13. (c) 14. (c) 15. (a) 16. (d) 17. (b) 18. (b) 19. (b) 20. (c)
21. (d) 22. (a) 23. (a) 24. (b) 25. (a) 26. (a) 27. (b) 28. (d) 29. (b) 30. (b)
31. (c) 32. (b) 33. (a) 34. (b) 35. (b) 36. (c) 37. (c) 38. (b) 39. (b) 40. (d)
41. (c) 42. (a) 43. (4) 44. (310) 45. (6) 46. (48) 47. (1)

Solutions
Solutions
Round I Þ x2 - y 2 = 0
1. (n + i )4 = (n 2 - 1 + 2ni )2 Þ x=± y
Þ x= y …(i)
= (n 2 - 1)2 - 4n 2 + 4n (n 2 - 1) i
and x=-y …(ii)
For integral solution, imaginary part must be zero.
Again,| z | = 2
\ 4n (n 2 - 1) = 0 Þ | z |2 = 4
n = 0 or n = ± 1 Þ x2 + y 2 = 4 …(iii)
Hence, number of integral solution is 3. From Eqs. (i) and (iii), we get
2n (1 + i )2n y2 + y 2 = 4 Þ 2 y2 = 4
2. Here, 2n
+
(1 + i ) 2n Þ y2 = 2 Þ y = ± 2
2n (1 + i 2 + 2i )n Therefore, from Eq. (i), we get
= 2 n
+
(1 + i + 2i ) 2n x=± 2
n n n
2 (2i ) 1 i \ z=± 2±i 2
= + n = n + i n = 2n + i n
(2i )n 2 i i On putting the value of x from Eq. (ii) in Eq. (iii),
in ì 1 ü we get
= + in = in í + 1ý or i n {(-1)n + 1}
(-1)n î (- 1 ) n
þ (- y)2 + y2 = 4 Þ 2 y2 = 4

3. Given, (3 + 2 -54 )1/ 2 - (3 - 2 -54 )1/ 2 Þ y2 = 2 Þ y = ± 2


From Eq. (ii), x=+ 2
= (3 + 2 ´ 3 6 i )1/ 2 - (3 - 2 ´ 3 6 i )1/ 2
\ z = x + iy, z = + 2 ± i 2
= (9 - 6 + 2 ´ 3 6 i )1/ 2 - (9 - 6 - 2 ´ 3 6 i )1/ 2
2
2 2
= (3 + ( 6 i ) + 2 ´ 3 ´ 6 i ) 1/ 2 6. x + 1 = 0 Þ x = ± i
2 2
- (3 + ( 6 i ) - 2 ´ 3 ´ 6 i ) 1/ 2
Q x2 + 1 is root of P (x) = 5x4 + 4x3 + 3x2 + Mx + N
2 1/ 2 2 1/ 2
= [(3 + 6 i ) ] - [(3 - 6 i ) ] Hence, x = i and -i are roots of P (x).
= ± (3 + 6 i ) m (3 - 6 i ) = ± 2 6 i Þ P (i ) = 0 and P (- i ) = 0
\ Imaginary part of (3 + 2 -54 )1/ 2 - (3 - 2 -54 )1/ 2 Þ 5(i ) + 4i3 + 3i 2 + Mi + N = 0
4

= Im (±2 6i ) = ± 2 6 and 5 (- i ) + 4 (- i )3 + 3 (i )2 + M (- i ) + N = 0
4

So, from the option -2 6 is correct. Þ 5 - 4i - 3 + Mi + N = 0


Hence, option (b) is correct. and 5 + 4i - 3 - Mi + N = 0
1 Þ (2 + N ) + i (M - 4) = 0
4. Let z =
1 - cos q + i sin q and (2 + N ) + i (4 - M ) = 0
1 On comparing real and imaginary parts to zero, we get
=
2 sin 2(q / 2) + 2i sin(q / 2) cos(q / 2) N = - 2, M = 4
1 1 and N = - 2, M = 4
=
2i sin(q / 2) [cos(q / 2) - i sin(q / 2)] Hence, M and N are unique.
cos(q / 2) + i sin(q / 2) 1 1 And M - N = 4 - (-2) = 6
= = + cot(q / 2)
2i sin(q / 2) 2 2i 7. Given, x = a + ib
1 1
= - i × cot q / 2 On squaring both sides,
2 2
1 x2 = (a 2 - b2 + 2iab)
\ Real part of z is . Þ 3 + 4i = a 2 - b2 + 2iab
2
On comparing
5. Let z = x + iy
a 2 - b2 = 3 and 2ab = 4
Þ z 2 = (x + iy)2
Þ a 2 - b2 = 3 and ab = 2
Þ z 2 = x2 + i 2 y2 + 2ixy 4
Now, a2 - 2 = 3
Þ z 2 = (x2 - y2) + i (2xy) a
Þ Re (z 2) = x2 - y2 Þ a 4 - 3a 2 - 4 = 0
Given, (z 2) = 0 Þ (a - 4) (a 2 + 1) = 0
2
88 JEE Main Mathematics

Þ a 2 = 4 or a2 = -1 [rejected] \ The condition is


\ a=±2 p(1 + p + r ) + q2 = (1 + r )(1 + p - r )
If a = 2, then b = 1 Þ p + p2 + pr + q2 = 1 + p - r + r + pr - r 2
If a = -2, then b = -1 Þ p2 + q 2 + r 2 = 1
3 3 3
x = a - ib + 3iab (a + ib)
11. Q 3 + i = (a + ib) (c + id )
When, a = 2, b = 1
= (ac - bd ) + i (ad + bc)
Then, x3 = 8 - i + 6i (2 + i )
Here, ac - bd = 3 and ad + bc = 1
Þ x3 = 8 - i + 12i - 6
æ bö æ dö æ bc + ad ö
\ x3 = 2 + 11 i Now, tan -1 ç ÷ + tan -1 ç ÷ = tan -1 ç ÷
èaø è cø è ac - bd ø
Hence, a = 2, b = 1 satisfy it.
æ 1 ö
\ a + b =2 + 1=3 = tan -1 ç ÷
è 3ø
(b + ic)
1+ i p
1 + iz 1+ a 1 + a - c + ib = np + , n Î I
8. = = 6
1 - iz 1 - i (b + ic) 1 + a + c - ib
1+ a 12. Since, x = - 5 + 4i Þ x + 5 = 4i
(1 + a - c + ib) (1 + a + c + ib) On squaring both sides, we get
=
(1 + a + c)2 + b2 x2 + 10x + 25 = - 16
a 2 + a + iab + ib Þ x2 + 10x + 41 = 0 …(i)
= 4 3 2
1 + a + ac + c Let E = x + 9x + 35x - x + 4
(a + 1 )(a + ib ) = x2[(x2 + 10x + 41 ) - x - 6] - x + 4
=
(a + 1 )(1 + c ) = x2(0) - x 3 - 6x2 - x + 4 [Q from Eq. (i)]
a + ib = - x 3 - 6 x2 - x + 4
=
1+ c = - x (x2 + 6x + 1 ) + 4
= - x (x2 + 10x + 41 - 4x - 40) + 4
9. Let (x + iy)2 = 7 + 24 i
= 4x2 + 40x + 4 [Q from Eq. (i)]
Þ x2 - y2 = 7 and 2xy = 24
= 4(x2 + 10x + 41 - 40)
Now, (x2 + y2)2 = (x2 - y2)2 + 4x2y2
= 4(0) - 160 [Q from Eq. (i)]
= 72 + 242 = 625
2 2
= - 160
Þ x + y = 25
13. Given that, z 2 + ( p + iq) z + r + i s = 0 …(i)
Þ x=±4
Let z = a (where a is real) be a root of Eq. (i), then
and y=±3
a 2 + ( p + iq) a + r + i s = 0
Hence, square root of 7 + 24 i = ± (4 + 3 i ).
q + ir Þ a 2 + pa + r + i (qa + s) = 0
10. We have, z = On equating real and imaginary parts, we get
1+ p
a 2 + pa + r = 0 …(ii)
- r + iq -s
\ iz = and qa + s = 0 Þ a =
1+ p q
By componendo and dividendo, we get On putting the value of a in Eq. (ii), we get
1 + iz 1 + p - r + iq æ -s ö
2
æ -s ö
= ç ÷ + pç ÷ + r =0
1 - iz 1 + p + r - iq èqø èqø
p + iq 1 + iz Þ s2 - pqs + q2r = 0 Þ pqs = s2 + q2r
\ = ,
1+ r 1 - iz
½æ 4 ö 4½ 4 4 4
14. |z| = ½ ç z - ÷ + ½Þ| z | < z - + Þ| z | £ 2 +
p + iq 1 + p - r + iq ½è zø z½ z | z| | z|
if =
1+ r 1 + p + r - iq 2
| z | - 2| z | - 4
Þ p(1 + p + r ) + q2 + i{ q(1 + p + r ) - pq} Þ £0
| z|
= (1 + r )(1 + p - r ) + iq(1 + r )
Since,| z | > 0
Þ p(1 + p + r ) + q2 = (1 + r )(1 + p - r )
| z |2 - 2| z | - 4 £ 0
and q(1 + p + r ) - pq = q(1 + r )
Þ [| z | - ( 5 + 1 )][| z | - (1 - 5 )] £ 0
[this is obviously true]
Þ 1 - 5 £| z| £ 5 + 1
Complex Numbers 89

15. Using the relation, if z1


19. Let z1 = r1 (cos q1 + i sin q1 ) then =1
|z1 + z2| = |z1 |+ |z2| z2
Then, arg (z1 ) = arg (z2) Þ |z1| = |z1|Þ|z1| = |z2| = r1
Since, | z 2 + (- 1 )| = |z 2| + |- 1| Now, arg (z1z2) = 0 Þ arg(z1 ) + arg (z2) = 0
Then, arg (z 2) = arg (- 1 ) Þ arg (z2) = - q1
Þ 2 arg (z ) = p [Q arg (- 1 ) = p ] Therefore, z2 = r1 [cos (- q1 ) + i sin (- q1 )]
p = r1 (cos q1 - i sin q1 ) = z1
Þ arg (z ) =
2 Þ z2 = (z1 ) = z1 Þ|z2|2 = z1z2
Þ z lies on Y -axis (imaginary axis).
20. z + 2|z + 1|+ i = 0
16. It is given that the z, z, z - 2 Re(z ) and z - 2 Re(z ) are Þ (x + iy) + 2|x + iy + 1|+ i = 0
vertices of a square of side 4 units, so
Þ (x + iy) + 2|(x + 1) + iy|+ i = 0
|z - z| = 4 …(i)
Þ (x + iy) + 2 (x + 1)2 + y2 + i = 0
Now, let z = x + iy, so z = x - iy and Re(z ) = x
Now, from Eq. (i), we have Þ x + 2(x + 1)2 + 2 y2 + i ( y + 1) = 0
2| y| = 4 Þ | y| = 2 On comparing real and imaginary parts, we get
and|z - (z - 2 Re(z ))| = 4 Þ 2|x| = 4 Þ|x| = 2 x + 2(x + 1)2 + 2 y2 = 0 and y + 1 = 0
\ |z | = x2 + y2 = 4 + 4 = 2 2 Þ x2 = 2x2 + 2 + 4x + 2 y2 and y = -1
17. Let z = x + iy Q x2 = 2x2 + 4x + 2(-1)2 + 2
z - 1 x + iy - 1 (x - 1 ) + iy (x + 1 ) - iy Þ x2 + 2 + 4 x + 2 = 0
= = ´
z + 1 x + iy + 1 (x + 1 ) + iy (x + 1 ) - iy Þ x2 + 4 x + 4 = 0
(x - 1 )(x + 1 ) - iy(x - 1 ) + iy(x + 1 ) - i 2 y2 \ (x + 2)2 = 0 Þ x = -2
= Hence, x = - 2 and y = - 1
(x + 1 )2 - i 2 y2
x2 - 1 + iy(x + 1 - x + 1 ) + y2 \ z = -2 - i
=
(x + 1 )2 + y2 21. |z1| = |z2|
2 2
z - 1 (x + y - 1 ) i (2 y) Re (z1 ) > 0, Im(z2) < 0
Þ = +
z + 1 (x + 1 ) + y
2 2
(x + 1 )2 + y2 æ z + z2 ö 1 æ z1 + z2 z1 + z2 ö
Re ç 1 ÷= ç + ÷
z -1 è z1 - z2 ø 2 è z1 - z2 z1 - z2 ø
Q is purely imaginary.
z+1 1 æ (z1 + z2) (z1 - z2) + (z1 + z2) (z1 - z2) ö
= ç ÷
æ z - 1ö 2è (z1 - z2) (z1 - z2) ø
\ Re ç ÷ =0
è z + 1ø æ z1z1 - z1z2 + z2z1 - z2z2 + z1z1 ö
ç ÷
2
x + y -1 2 1 + z z - z2z1 - z2z2
Þ =0 = ç 1 2 ÷
(x + 1 )2 + y2 2ç |(z1 - z2)|2 ÷
ç ÷
è ø
Þ x2 + y 2 - 1 = 0
Þ x2 + y2 = 1 1 æ 2|z1|2- 2|z2|2 ö
= ç ÷ =0
2 è |z1 - z2|2 ø
Þ | z 2| = 1 Þ| z | = 1
18. (sin x + i cos 2x) = (cos x - i sin 2x) æ z + z2 ö
Þ ç 1 ÷ is purely imaginary.
è z1 - z2 ø
Þ sin x + i cos 2x = cos x + i sin 2x
On comparing real and imaginary parts, we get æ |z|+11 ö
22. log 1 ç ÷ £2
sin x = cos x and cos 2x = sin 2x 2
è (|z|-1)2 ø
Þ tan x = 1 and tan 2x = 1 |z|+11 1
Þ ³
p p (|z|-1)2 2
Þ x= and 2x =
4 4 Þ 2|z|+22 ³ (|z|-1)2
p p
\ x= and x = Þ 2|z|+22 ³ |z|2+1 - 2|z|
4 8 2
Þ |z| -4|z|-21 £ 0
Q We got two different values of x, hence conjugate
Þ |z|£ 7
sin x + i cos 2x is not conjugate of (cos x - i sin 2x).
\Largest value of|z|is 7.
Þ No value of x.
90 JEE Main Mathematics

23. |z - 2| = min {|z - 1|,|z - 5|} 28. We have, arg (z - 1 ) = arg (z + 3i )


i.e. |z - 2| = |z - 1|, where|z - 1| < |z - 5| On putting z = x + iy,
3 Þ arg (x + iy - 1 ) = arg (x + iy + 3i )
Þ Re (z ) = , which satisfy|z - 1| < |z - 5|
2 Þ arg [(x - 1 ) + iy] = arg [x + i ( y + 3)]
y y+3
Also, |z - 2| = |z - 5|, where|z - 5| < |z - 1| Þ tan -1 = tan -1
x-1 x
7
Þ Re (z ) = , which satisfy|z - 5| < |z - 1| y y+3
2 Þ = Þ xy = (x - 1 )( y + 3)
x-1 x
24. We have,|1 - z1z2|2 - |z1 - z2|2
Þ xy = xy + 3x - y - 3
= (1 - z1z2)(1 - z1z2) - (z1 - z2) (z1 - z2)
Þ 0 = 3(x - 1 ) - y
(Q zz = |z 2|) x-1 1
= (1 - z1z2) (1 - z1z2) - (z1 - z2) (z1 - z2) Þ y = 3(x - 1 ) Þ =
y 3
(Q z1 - z2 = z1 - z2 and 1 = 1)
Þ (x - 1 ) : y = 1 : 3
= (1 - z1z2) (1 - z1z2) - (z1 - z2)
29. Let z1 = r1 (cos q1 + i sin q1 )
(z1 - z2) [Q (z1 ) = z1 ]
and z2 = r2 (cos q2 + i sin q2)
= 1 - z1z2 - z1z2 + z1z1z2z2 - z1z1 + z1z2 + z1z2 - z2z2
Since, | z2| = | z1 |
= 1 + |z1|2|z2|2- |z1|2 - |z2|2
\ r2 = r1
= (1 - |z1|2 ) (1 - |z22|)
Also, arg (z1 ) + arg (z2) = p
\ k =1
\ arg (z2) = p - arg (z1 )
z-a Þ arg (z2) = p - q1
25. We have, =1
z+a \ z2 = r1 {cos (p - q1 ) + i sin (p - q1 )}
Þ |z - a| = |z + a| Þ |z - a|2 = |z + a|2 = r1 (- cos q1 + i sin q1 )
Þ (z - a ) (z - a ) = (z + a ) (z + a ) = - r1 (cos q1 - i sin q1 ) = - z1
Þ (z - a ) (z - a ) = (z + a ) (z + a ) [Q (a ) = a ] Þ z1 = - z2
7 -z
Þ zz - za - az + aa = zz + za + a z + aa 30. Given, f (z ) = and z = 1 + 2i
1 - z2
Þ za + za + az + az = 0
7 - (1 + 2i )
Þ (a + a ) (z + z ) = 0 \ f (z ) =
1 - (1 + 2i )2
Þ z + z =0 [Q a + a = 2 Re(a ) ¹ 0] 6 - 2i 6 - 2i
= =
Þ 2 Re(z ) = 0 1 - (1 - 4 + 4i ) 4 - 4i
Þ 2x = 0 6 - 2i 1+ i 6 + 4i + 2
Þ x=0 = ´ =
4(1 - i ) (1 + i ) 4(12 - i 2)
Þ Y -axis
8 + 4i 1
= = (2 + i )
26. We have,|l1a1 + l 2a 2 + K + l na n | 4(2) 2
£ |l1a1| + |l 2a 2| + ... + |l na n| 4+1 5
| f (z )| = =
= |l1||a1| + K + |l n ||a n | 2 2
= l1|a1 | + K + l n |a n | < l1 + K + l n [Q each l k ³ 0] | z|
= (Q z = 1 + 2i , given Þ|z | = 5 )
2
[Q|a k| < 1 and so l k|a k|< l k for all k = 1, 2 ,..., n]
Hence, |l1a1 + l 2a 2 + K + l na n | < 1 31. We have, tan -1 (a + ib) = x + iy
Þ a + ib = tan (x + iy) …(i)
27. Let z = x + iy
Taking conjugate, we get
æ yö (a - ib ) = tan (x - iy)
\ arg (z ) = tan -1 ç ÷ …(ii)
è xø \ tan 2x = tan [(x + iy) + (x - iy)]
æ yö y (a + i b ) + (a - i b )
Then, arg (z ) = tan -1 ç - ÷ = 2p - tan -1 Þ tan 2x =
è xø x 1 - (a + i b ) (a - i b )
= 2p - arg (z ) 2a
=
1 - (a 2 + b 2)
Since, in argument of a conjugate of a complex, the real
axis is unaltered but imaginary axis be changed, hence 1 æ 2a ö
\ x = tan -1 ç ÷
it is given by 2p - arg (z ). 2 è 1 - a2 - b2 ø
Complex Numbers 91

32. |z1| = |z2| = |z3| = 1 p


36. Since, arg (z ) - arg (w) =
2 2
Now, |z1| = 1 Þ |z1| = 1
p
Þ z1z1 = 1 Þ - arg (z ) + arg (w) = -
2
Similarly, z2z2 = 1 and z3 z3 = 1 p
½ Þ arg (z ) + arg (w) = -
Again, ½1 + 1 + 1 ½
½= 1 2
½z1 z2 z3½ p
Þ arg (zw) = - …(i)
Þ |z1 + z2 + z3| = 1 2
Þ |z1 + z2 + z3| = 1 Also given,|zw| = i = | zw| …(ii)
Þ |z1 + z2 + z3| = 1 [Q|z| = |z|] \From Eq. (i) and (ii), we get
z -1 zw =| zw|ei arg ( zw )
33. Since, |z| = 1 and w = p
z+1 -i æ pö æ pö
=1 e = cos ç - ÷ + i sin
2
ç- ÷
1+ w è 2ø è 2ø
Þ z - 1 = wz + w Þ z=
1-w p
= 0 - i sin = - i
½1 + w ½ 2
|z| = ½ ½
½1 - w ½ 37. It is given that, there are two complex numbers z and w,
such that| z w| = 1
Þ ½1 + w ½
½ ½= 1 [Q|z| = 1] and arg (z ) - arg (w) = p / 2
½1 - w ½
\ |z || w| = 1 [Q| z1 z2| = | z1 || z2|]
Þ |1 + w| = |1 - w| p
and arg (z ) = + arg (w)
On squaring both sides, we get 2
1 + |w|2 + 2|w|Re(w) = 1 + |w|2 - 2|w|Re(w) 1
Let|z| = r, then | w| = …(i)
r
Q | z1 ± z2|2 = | z1|2+ | z2|2 ± 2| z1|| z2|Re (z1z2) p
and let arg (w) = q, then arg (z ) = +q … (ii)
Þ 4|w|Re(w) = 0 2
i ( p / 2 + q)
\ Re(w) = 0 So, we can assume z = re …(iii)
34. We have, z2 = z1 and z4 = z3 [Q if z = x + iy is a complex number, then it can be
written as z = reiq where, r =|z |and q = arg (z )]
Therefore, z1z2 =|z1 |2 and z3 z4 =|z3 |2 1
and w = ei q …(iv)
æz ö æz ö æzz ö r
Now, arg ç 1 ÷ + arg ç 2 ÷ = arg ç 1 2 ÷
è z4 ø è z3 ø è z4z3 ø 1
Now, z × w = re- i( p / 2 + q ) × eiq = ei( - p / 2 - q + q )
æ½z ½2ö r
æ |z |2 ö
= arg ç 1 2 ÷ = arg ç½ 1½ ÷ = 0 =e - i( p / 2)
=-i [Q e- i q = cos q - i sin q]
è|z3 | ø ç½z3½ ÷
è ø 1
and z w = re i( p / 2 + q ) × e- iq
[Q argument of positive real number is zero] r
(1 + i )2 = ei( p / 2 + q - q ) = e i ( p / 2) = i
35. The given complex number z = 38. Given, z1 = (1, a ) = cos a + i sin a = eia
a-i
z2 = (1, b ) = cos b + i sin b = eib
(1 - 1 + 2i ) (a + i )
= [Q i 2 = - 1] and z3 = (1, g ) = cos g + i sin g = eig
a2 + 1
Also, z1 + z2 + z3 = 0
2i (a + i ) -2 + 2ai
= = \ (cos a + cos b + cos g ) + i (sin a + sin b + sin g ) = 0
a2 + 1 a2 + 1
Þ cos a + cos b + cos g = 0
Q z = 2 /5 [given]
and sin a + sin b + sin g = 0 …(i)
2
4 + 4a 2 2 2 1 1 1
Þ = Þ = Let E = + + = e-ia + e-ib + e-ig
(a 2 + 1)2 5 1 + a2 5 z1 z2 z3
4 2 = (cos a + cos b + cos g ) - i (sin a + sin b + sin g )
Þ = Þ a 2 + 1 = 10
1 + a2 5 = 0 - i0 = 0 [Q from Eq. (i)]
Þ a2 = 9 Þ a = 3 [Q a > 0] 39. Let z = r (cos q + i sin q)
2
-2 + 6 i 1 3 1 3 1 1
So, z = =- + i Þ z=- - i Given that, z + =a Þ z+ = a2
10 5 5 5 5 z z
[Q if z = x + iy, then z = x - iy] 1
Þ r 2 + 2 + 2 cos 2 q = a 2 …(i)
r
92 JEE Main Mathematics

On differentiating w.r.t. q, we get 2± 4+8


dr 2 dr \ |z| £ £1 ± 3
2r - - 4 sin 2 q = 0 2
dq r3 dq Hence, maximum value of|z|is 1 + 3.
dr æ 2ö
Þ ç2r - 3 ÷ = 4 sin 2 q 44. Since, | z1 - 1| < 4
dq è r ø
dr Þ | z1 | - |1| < 1 [Q by triangle inequality]
For maximum or minimum, put = 0, we get Þ | z1 | < 2 …(i)
dq
p Similarly, | z3 - 2| < 2
Þ q = 0,
2 Þ | z2 | < 4 …(ii)
p and | z3 - 3| < 3 …(iii)
\ r is maximum for q = ,therefore from Eq. (i), we get
2 On adding Eq. (i), (ii) and (iii), we get
1 1 | z1 | + | z2| +| z3 | < 2 + 4 + 6
r + 2 - 2 = a2 Þ r - = a
2
r r Þ | z1 | + | z2| + | z3 | < 12 …(iv)
a+ a2 + 4 By using triangular inequality
Þ r 2 - ar - 1 = 0 Þ r =
2 | z1 + z2 + z3 | £ | z1 | + | z2 + | z3 |
40. Given, | z1 | = 12 …(i) Þ | z1 + z2 + z3 | < 12 [Q from Eq. (iv)]

and | z2 - 3 - 4 i | = 5 45. We have, OC = 5 and CA = 1


Þ | z2 - (3 + 4i )| = 5 Y
Now,| z2 - (3 + 4i )| ³ | z2| - |3 + 4 i |
[Q| z1 - z2| ³ | z1 | -|z2|] 1 A(z)
Þ 5 ³ | z2 | - 3 + 4 2 2 C(0, 5)

Þ 5 ³ | z2| - 25
q
Þ 5 ³ | z2 | - 5 X
O
Þ 10 ³| z2| …(ii)
Now, | z1 - z2| ³ | z1 | - | z2|
[Q| z1 - z2| ³ | z1 | - | z2|] Clearly, the point z is such that amplitude of z is
minimum to the point of contact of the circle and the
³ 12 - 10 [using Eqs. (i) and (ii)]
tangent drawn from the origin O.
| z1 - z2| ³ 2
Let q = ÐAOC. Then, ÐAOC = 90° - q
Hence, the minimum value of| z1 - z2|is 2.
1
41. We know that, |- z | = |z | From figure, sin ÐAOC =
5
and |z1 + z2| £ |z1 | + |z2| 1
Þ sin (90° - q) =
Now, |z | + |z - 1| = |z | + |1 - z | 5
³ |z + (1 - z )| = |1| = 1
1
Hence, minimum value of|z | + | z - 1|is 1. Þ cos q =
5
42. We have, 2 = | z + i w| £ | z | + | w| 24
[Q| z1 + z2| £ | z1 | + | z2|] and sin q =
5
\ | z | + | w| ³ 2 …(i)
Now, OA = OC 2 - CA 2 = 52 - 12 = 24
But it is given that| z | £ 1 and| w| £ 1
Hence, | z | + | w| £ 2 …(ii) Now, the coordinates of A are ( 24 cos q, 24 sin q)
From Eqs. (i) and (ii), we get æ 24 24 ö
i.e. ç , ÷
| z | = | w| = 1 è 5 5ø
Also, | z + i w| = | z - i w |
2 6 24
Þ | z - (- i w)| = | z - i w | Hence, z= +i
5 5
i.e. z = z , Q|z| = |z|and|z| = 1
4 (cos 75° + i sin 75° )
\ |z|= 1 46.
0.4 (cos 30° + i sin 30° )
Þ z = ±1
= 10 (cos 75° + i sin 75° ) (cos 30° - i sin 30° )
2 2
43. z + = 2 Þ |z| - £ 2 Þ|z|2 - 2|z| - 2 £ 0
z |z| = 10 e75i × e-30i = 10 e45i
This is a quadratic equation in|z|. 10
= 10 (cos 45° + i sin 45° ) = (1 + i )
2
Complex Numbers 93

47. (cos q + i sin q) (cos 2 q + i sin 2 q) 3 3 (2 + cos q - i sin q)


51. x + iy = =
…(cos nq + i sin nq) = 1 2 + cos q + i sin q (2 + cos q)2 + sin 2 q
Þ cos (q + 2 q + 3 q + K + nq) 6 + 3 cos q - 3 i sin q
=
+ i sin (q + 2 q + 3 q + K + nq) = 1 4 + cos 2 q + 4 cos q + sin 2 q
ì n (n + 1 ) ü ì n (n + 1 ) ü é 6 + 3 cos q ù é - 3 sin q ù
Þ cos í qý + i sin í qý = 1 =ê ú + i ê 5 + 4 cos q ú
î 2 þ î 2 þ ë 5 + 4 cos q û ë û
On comparing the coefficients of real and imaginary
parts both sides, we get On equating the real and imaginary parts both sides,
we get
ì n (n + 1 ) ü ì n (n + 1 ) ü
cos í qý = 1 and sin í qý = 0 3 (2 + cos q) - 3 sin q
î 2 þ î 2 þ x= , y=
5 + 4 cos q 5 + 4 cos q
n (n + 1 )
Þ q = 2mp 9
2 \ x2 + y2 = (4 + cos 2 q + 4 cos q + sin 2 q)
4mp (5 + 4 cos q)2
Þ q= , where m Î I
n (n + 1 ) 9 æ 6 + 3 cos q ö
= =4ç ÷ - 3 = 4x - 3
5 + 4 cos q è 5 + 4 cos q ø
48. Here, 1 - c2 = nc - 1
ra ra
Þ 1 - c2 = n 2c2 - 2nc + 1 52. We have, zr = cos + i sin 2
n2 n
c 1
\ = …(i) where r = 1, 2, 3, ... , n
2n 1 + n 2
a a
c æ nö 1 ì 2 æ 1 öü \ z1 = cos 2 + i sin 2 ;
or (1 + nz ) ç1 + ÷ = í1 + n + n ç z + ÷ ý n n
2n è z ø 1 + n2 î è z øþ
2a 2a
1 2 z2 = cos 2 + i sin 2
= { 1 + n + n (2 cos q )} n n
1 + n2
M M M
(1 + n 2) + 2n cos q na na
= zn = cos 2 + i sin 2
1 + n2 n n
æ 2n ö
=1 + ç ÷ cos q \ lim (z1z2z3 K zn )
è 1 + n2 ø n ®¥

æ a aöæ 2a 2 aö
= 1 + c cos q [using Eq. (i)] = lim ç cos 2 + i sin 2 ÷ ç cos 2 + i sin 2 ÷ …
n ®¥è n n ø è n n ø
49. We have, {(1 - cos q) + i × 2 sin q} -1 na na ö
æ
q q qö
-1 ç cos 2 + i sin 2 ÷
æ è n n ø
= ç2 sin 2 + i × 4 sin cos ÷
è 2 2 2ø é ìa ü

-1
q qö
-1 = lim ê cos í 2 (1 + 2 + 3 + K + n )ý
æ æ n ®¥ë î n þ
= ç2 sin ÷ çsin + i × 2 cos ÷
è 2ø è 2 2ø
ìa üù
q q + i sin í 2 (1 + 2 + 3 + K + n )ý ú
-1 - i × 2 cos sin în þû
æ qö 1 2 2
= ç2 sin ÷ × ´ a + a +
è 2ø q q q q é n (n 1 ) n (n 1 ) ù
sin + i × 2 cos sin - i × 2 cos = lim ê cos + i sin úû
2 2 2 2 n ®¥ë 2n 2 2n 2
q q q q ia
sin - i × 2 cos sin - i × 2 cos a a
= 2 2 = 2 2 = cos + i sin = e 2
qæ q qö qæ qö 2 2
2 sin çsin 2 + 4 cos 2 ÷ 2 sin ç1 + 3 cos 2 ÷ 2p 2p
2è 2 2ø 2è 2ø 53. Let z = cos + i sin , then by using De-Moivre’s
It’s real part 7 7
q theorem
sin 2 pk 2 pk
2 1 \ z k = cos + i sin …(i)
= = 7 7
qæ 2 qö æ qö
2 sin ç1 + 3 cos ÷ 2 ç1 + 3 cos 2 ÷ 6
2è 2ø è 2ø æ 2 pk 2 pk ö
Let S = å çsin - i cos ÷
1 1 è 7 7 ø
= = k =1
2 + 3 (cos q + 1 ) 5 + 3 cos q 6
é æ 2 pk 2 pk ö ù
-i q æqö
-i ç ÷ i
q q = å ê (- i ) çè cos 7 + i sin 7 ÷ø ú
ë û
è 2ø 2 cos k =1
1+ a e 2 (1 + eiq ) e + e 2 2 q
50. = = = = i cot 6
æ 2 pk 2 pk ö
q
1-a - iq
e2 iq
(1 - e ) e
æqö
-i ç ÷
è 2ø
i
-e 2
q
-2 i sin 2 = (- i ) å ç cos
è 7
+ i sin
7 ø
÷
2 k =1
94 JEE Main Mathematics

6
and c = cos g + i sin g
= (- i ) å zk [from Eq. (i)]
Therefore, a + b + c = (cos a + cos b + cos g )
k =1

= (- i ) (z + z 2 + z3 + K + z 6 ) + i (sin a + sin b + sin g )


It is a GP of which the first term is z, number of terms = 0 + i0 = 0 [from Eqs. (i) and (ii)]
is 6 and the common ratio is If a + b + c = 0, then a3 + b3 + c 3 = 3abc
2p 2p Þ (cos a + i sin a )3 + (cos b + i sin b )3 + (cos g + i sin g )3
z = cos + i sin ¹1
7 7
= 3 (cos a + i sin a ) (cos b + i sin b ) (cos g + i sin g )
z (1 - z 6 ) z - z7
\ S = (- i ) = (- i ) Þ (cos 3 a + i sin 3 a ) + (cos 3 b + i sin 3 b )
1-z 1-z
é 7 æ + (cos 3g + i sin 3g )
2p ö ù
7
2p
z -1 êQ z = ç cos + i sin ÷ ú = 3 [cos (a + b + g ) + i sin (a + b + g )]
= (- i ) =i ê è 7 7ø ú
1-z
êë = cos 2p + i sin 2p = 1úû Þ cos 3 a + cos 3 b + cos 3g = 3 cos (a + b + g )
57. We have, (16)1/ 4 = (24 )1/ 4 = 2(1 )1/ 4 = 2(cos 0 + i sin 0)1/ 4
54. Let z - 1 = r (cos q + i sin q) = reiq
1 1
1 = 2{cos (2kp + 0) + i sin (2kp + 0)}
\Given expression = re iq × e -ia + × e ia 4 4
reiq
1 -i ( q - a ) k = 0, 1, 2, 3
= rei( q - a ) +
e
r = 2 ´ 1, 2 ´ i , 2 ´ - 1, 2 ´ - i = ± 2 , ± 2 i
Since, imaginary part of given expression is zero, æ 2p ö æ 2p ö
we have 58. We have, a = cos ç ÷ + i sin ç ÷
è 7ø è 7ø
1 7
r sin (q - a ) - sin (q - a ) = 0 é æ 2p ö æ 2p ö ù
r Þ a7 = ê cos ç ÷ + i sin ç ÷ ú
ë è 7 ø è 7 øû
r2 - 1 = 0 Þ r2 = 1
= cos 2p + i sin 2p = 1 …(i)
Þ r = 1 Þ |z - 1| = 1 Let S = a + b = (a + a 2 + a 4 ) + (a3 + a5 + a 6 )
or sin (q - a ) = 0 Þ q - a = 0 (Q a = a + a 2 + a 4 , b = a3 + a5 + a 6 )
Þ q=a a (1 - a 6 )
Þ S = a + a 2 + a3 + a 4 + a5 + a 6 =
Þ arg (z - 1 ) = a 1-a
55. We have, a = cos a + i sin a, a - a7 a - 1
Þ S= = = -1 …(ii)
1-a 1-a
b = cos b + i sin b
Let P = ab = (a + a 2 + a 4 ) (a3 + a5 + a 6 )
and c = cos g + i sin g
= a 4 + a 6 + a7 + a5 + a7 + a 8 + a7 + a 9 + a10
b cos b + i sin b cos g - i sin g = a 4 + a 6 + 1 + a5 + 1 + a + 1 + a 2 + a3
Now, = ´
c cos g + i sin g cos g - i sin g [from Eq. (i)]
= cos b cos g + sin b sin g = 3 + (a + a 2 + a3 + a 4 + a5 + a 6 ) = 3 + S
+ i [sin b cos g - sin g cos b ] =3 -1 =2 [from Eq. (ii)]
b Required equation is, x2 - Sx + P = 0
Þ = cos ( b - g ) + i sin (b - g ) …(i)
c Þ x2 + x + 2 = 0
c
Similarly, = cos (g - a ) + i sin (g - a ) æ 2p ö æ 2p ö
…(ii) 59. We have, wn = cos ç ÷ + i sin ç ÷
a è nø è nø
a 2p 2p
and = cos (a - b ) + i sin (a - b ) …(iii) Þ w 3 = cos + i sin
b 3 3
On adding Eqs. (i), (ii) and (iii), we get 1 i 3
cos ( b - g ) + cos (g - a ) + cos (a - b ) =- + =w
2 2
+ i [sin ( b - g ) + sin (g - a ) + sin (a - b )] = 1 2p 2p ö
2
æ
On equating real parts to both sides, we get an w23 = ç cos + i sin ÷
è 3 3 ø
cos ( b - g ) + cos (g - a ) + cos (a - b ) = 1 4p 4p 1 i 3
= cos + i sin =- - = w2
56. We have, cos a + cos b + cos g = 0 …(i) 3 3 2 2
and sin a + sin b + sin g = 0 …(ii) \ (x + yw 3 + zw 32 ) (x + yw 32 + zw 3 )
Let a = cos a + i sin a , = (x + yw + zw2) (x + yw2 + zw)
b = cos b + i sin b = x2 + y2 + z 2 - xy - yz - zx
Complex Numbers 95

30 30
æ –1 + i 3 ö æ (–1 + i 3 )(1 + i ) ö On putting n = 1 , 2 , 3 ,... , n , we get
60. ç ÷ =ç ÷ = (1 + i )30
è 1-i ø è 2 ø S1 = w1 = w, S 2 = w3 = 1,
3 S3 = w6 = 1,... , S7 = w28 = w
é –1 + i 3 æ -1 + i 3 ö
êQ is cube root of unity, soç ÷ =1 \ We always get 1 and w.
ë 2 è 2 ø
1 wn w2n
æ –1 + i 3 ö
30 ù
Þç ÷ = 1ú 67. D = wn w2n 1
è 2 ø ú w 2n
wn
û 1
\ (1 + i )30 = [(1 + i )2]15 = (1 - 1 + 2i )15 = 215 i15 = 1 (w3 n - 1 ) - wn (w2n - w2n ) + w2n (wn - w4n )
= 215 i3 = –215 i = (1 - 1 ) - 0 + w2n [wn - (w3 )n wn ] [Q w3 n = 1]
61. Since, a , b and g be the cube roots of a positive =0 + 0 + 0 =0
number p. 68. S = 1 + 3 a + 5 a 2 + K + (2 n - 1 ) a n - 1 …(i)
\ a = p1/3 , b = wp1/3 , g = w2p1/3 2 3 n
Þ aS = a + 3 a + 5 a + K + (2 n - 1 ) a …(ii)
a x + by + gz x + wy + w2z 1 -1 - i 3
Hence, = = = w2 = On subtracting Eq. (ii) from Eq. (i), we get
bx + gy + az wx + w2y + z w 2
(1 - a ) S = 1 + 2 a + 2 a 2 + K + 2 a n - 1 - (2 n - 1 ) a n
62. Given, i = -1
= 2 (1 + a + a 2 + K + a n - 1 ) - 1 - (2 n - 1 ) a n
334 365
æ 1 i 3ö æ 1 i 3ö 2 (1 - a n )
To find 4 + 5 ç - + ÷ + 3 ç- + ÷ . = - 2 n = -2 n [Q a n = 1]
è 2 2 ø è 2 2 ø 1-a
1 i 3 -2 n
Let w = - + Þ 4 + 5 w334 + 3 w365 Þ S=
2 2 (1 - a )
= 4 + 5 w3 ´ 111 + 1 + 3 w3 ´ 121 + 2
= 4 + 5w + 3w2 69. The given equation z3 + 2z 2 + 2z + 1 = 0 can be rewritten
= 1 + 2w + (3 + 3w + 3w2) as (z + 1 ) (z 2 + z + 1 ) = 0.
=1 + 2w [Q 1 + w + w2 = 0] Its roots are -1, w and w2.
æ 1 i 3ö
= 1 + 2 ç- + ÷=i 3 Let f (z ) = z1985 + z100 + 1
è 2 2 ø
Put z = - 1, w and w2 respectively, we have
63. Let x = -1 - -1 - -1 - . . . ¥ f (-1 ) = (-1 )1985 + (-1 )100 + 1 ¹ 0
Then, x = -1 - x or x2 = - 1 - x Therefore, -1 is not a root of the equation f (z ) = 0.
or x2 + x + 1 = 0 Again, f (w ) = w1985 + w100 + 1
-1 ± 1 - 4 × 1 × 1 -1 ± -3 = (w3 )661 w2 + (w3 )33 w + 1
\ x= =
2 ×1 2 = w2 + w + 1 = 0
-1 ± 3 i 2 Therefore, w is a root of the equation f (z ) = 0.
= = w or w
2 Similarly, f (w2) = 0
64. x = a + b, y = aa + bb and z = ab + ba Hence, w and w2 are the common roots.
2 2
Now, xyz = (a + b) (aw + bw ) (aw + bw), 70. rth term of the given series
where a = w and b = w2
= r [(r + 1 ) - w] [(r + 1 ) - w2]
\ xyz = (a + b) (a 2 + abw2 + abw + b2)
= (a + b) (a 2 - ab + b2) = a3 + b3 = r [(r + 1 )2 - (w + w2) (r + 1 ) + w3 ]

æ 1 i 3ö
334
æ1 i 3 ö
365 = r [(r + 1 )2 - (-1 ) (r + 1 ) + 1]
65. 4 + 5 ç - + ÷ -3 ç + ÷
è 2 2 ø è2 2 ø = r (r 2 + 3r + 3) = r3 + 3r 2 + 3r
Thus, sum of the given series
Þ 4 + 5 (w)334 - 3 (- w2)365 Þ 4 + 5w + 3w
( n - 1)
1
Þ
2
{8 - 5 + 5i 3 - 3 + 3i 3 } = å (r3 + 3r 2 + 3r )
r =1
1
Þ {8i 3 } = 4 3i 1 1 1
2 = (n - 1 )2n 2 + 3 × (n - 1 ) (n ) (2n - 1 ) + 3 × (n - 1 ) n
4 6 2
66. We have, w1 × w2 × w3 × K × wn
1
n( n + 1 ) = (n - 1 ) n (n 2 + 3n + 4 )
1 + 2 + 3 + ... + n 4
=w = w 2 = Sn (say)
96 JEE Main Mathematics

æ 1ö æ 1ö 75. We have,
71. tn = (n + 1) ç n + ÷ ç n + 2 ÷
è wø è w ø z =4 -3i
æ 1 1 ö æ 1 1ö
= n3 + n 2 ç 2 + + 1÷ + n ç1 + 2 + ÷ + 1 \ |z| = 42 + (- 3)2 = 5
èw w ø è w wø
3 2 2 2
= n + n (w + w + 1) + n(w + w + 1) + 1 Let z1 be the new complex number obtained by rotating z
3 in the clockwise sense through 180°, therefore
=n +1
n n z1 = - 4 + 3 i
\ Sn = å tr = å (r3 + 1) Therefore, required complex number is
r =1 r =1
3 (- 4 + 3 i ) = - 12 + 9 i
n (n + 1)2
2
= +n 76. Let OA and OB be the sides of an equilateral D OAB and
4
let OA and OB represent the complex numbers
72. Q 1, a1 , a 2, ... , a n - 1 are n roots of unity. z1 and z2, respectively.
xn -1 Y B
\ = (x - a1 ) (x - a 2).... (x - a n - 1 )
x-1
p/3
Þ x n - 1 + x n - 2 + ...+ x2 + x + 1
z2 z2 – z1
= (x - a1 ) (x - a 2)... (x - a n - 1 )
Put x = 1
p/3 p/3
\ (1 - a1 ) (1 - a 2)... (1 - a n - 1 ) = 1 + 1 + ... n times = n X
O z1 A(z1)
73. Since, x2 - x + 1 = 0 (given)
Solving for x, we have From the equilateral D OAB,
x = - w and x = - w 2 AB = z2 - z1
æ z2 - z1 ö p
Case I x= -w \ arg ç ÷ = arg (z2 - z1 ) - arg z2 =
5 2 è z2 ø 3
ì 1 ü
S = å í (- w)n + ý æz ö p
n = 1î (- w)n þ and arg ç 2 ÷ = arg (z2) - arg (z1 ) =
5 è z1 ø 3
Þ S= å (wn + w2n )2
z2 - z1 z
r =1 Also, = 1 = 2 , since triangle is equilateral.
z2 z1
Þ S = (-1 )2 + (-1 )2 + 22 + (-1 )2 + (-1 )2
z - z1 z
S =8 Thus, the complex numbers 2 and 2 have same
z2 z1
Case II x = - w2 modulus and same argument, which implies that the
2
numbers are equal, that is
ïì æ 1 ö ïü
6 n
S= å íï (-w2)n + çè -w2 ÷ø ýï z2 - z1 z2
=
n =1 î þ z2 z1
5
Þ S= å ( w2n + wn )2 = 8 Þ z1z2 - z12 = z22
n =1 Þ z12 + z22 = z1z2
74. Let z = x + iy 77. azz + az + az + d = 0 ® Circle
z -1 x + iy - 1 2
\ = -a a d
2z + 1 2x + 2iy + 1 Centre = =r= -
a a a
{(x - 1 ) (2x + 1 ) + 2 y2 )} + iy (3)
= So,|a|2 - ad > 0 and a Î R - {0}.
(2x + 1 )2 + 4 y2
78. z1 + z2 = - 1
æ z -1 ö
According to the question, Im ç ÷ = -4 b
è 2z + 1 ø and z1z2 =
3
3y
\ = -4 As z1 and z2 origin form an equilateral triangle, we
(2x + 1 )2 + 4 y2 have
Þ 3 y = - 4 {(4x2 + 4x + 1 ) + 4 y2 } z12 + z22 + z32 = z1z2 + z2z3 + z3 z1
Þ 16x2 + 16 y2 + 16x + 3 y + 4 = 0 Þ z12 + z22 + 0 = z1z2 + 0 + 0
This equation represents a circle. Þ (z1 + z2)2 = 3z1z2
Hence, locus of z is a circle. Þ 1=b
Complex Numbers 97

79. Central idea If z is unimodular, then|z| = 1. æ 5ö æ -10 ö


= 1 ç3 - ÷ + 1(2) + 1 ç ÷
Also, use property of modulus i.e. z z =|z| . 2 è 3ø è 3 ø

Given, z2 is not unimodular i.e.|z2| ¹ 1 4 10


= +2-
3 3
z - 2 z2 z - 2 z2
and 1 is unimodular Þ 1 =1 4 + 6 - 10
2 - z1z2 2 - z1z2 = =0
3
Þ |z1 - 2z2|2 = |2 - z1z2|2 Hence, area of triangle is zero, therefore points are
Þ (z1 - 2z2)(z1 - 2z2) = (2 - z1z2)(2 - z1z2) [Q zz = |z|2 ] collinear.
Þ |z1|2 + 4|z2|2 - 2z1z2 - 2z1z2
84. Let z = x + iy
= 4 + |z1|2|z2|2- 2z1z2 - 2z1z2
Then, z + iz = (x + iy) + i (x + iy) = (x - y) + i (x + y)
Þ (|z2|2 - 1)(|z1|2 - 4) = 0 [Q |z2| ¹ 1]
and iz = i (x + iy) = - y + ix
\ |z1| = 2
If triangle be the area of the triangle formed by z , z + iz
Let z1 = x + iy Þ x2 + y2 = (2)2
and iz, then
\ Point z1 lies on a circle of radius 2. x y 1
1
80. Let z = x + iy D = | x - y x+ y 1 |
2
z - 1 x + iy - 1 (x2 + y2 - 1 ) + 2 iy -y x 1
\ = =
z + 1 x + iy + 1 (x + 1 )2 + y2 Applying R2 ® R2 - (R1 + R3 ),
æ z - 1ö -1 2y
\ arg ç ÷ = tan x y 1
è z + 1ø x2 + y2 - 1 1 1
2y p then D= 0 0 -1 = (x2 + y2)
Þ tan -1 2 = (given) 2
-y x 1
2
x + y2 - 1 3
2y p 1
Þ = tan = 3 = |z |2 = 200 (given)
x2 + y2 - 1 3 2
2 2 2 Þ |z|2 = 400 Þ |z | = 20
Þ x + y -1 = y
3 \ |3z | = 3|z | = 60
2
Þ x2 + y2 - y-1 =0 85. If z1 , z2, z3 are vertices of
equilateral triangle
3
1
which is an equation of a circle. circumscribing the circle |z| = i.e. z1, therefore to
2
81. z 2 + ax + a 2 = 0 Þ z = aw, aw2 2p
obtain z2 rotated through angle in anti-clockwise
(where w is a non-real root of unity) 3
Þ Locus of z is a pair of straight lines direction.
z1
and arg (z ) = arg (a ) + arg (w)
or arg (z ) = arg (a ) + arg (w2)
2p z2
Þ arg (z ) = ±
3 |z|=1/2
Also, |z | = |a || w| or |z| = |a||w2|
Þ |z| = |a| z3

82. We have, |z - z1 | = |z - z2| = |z - z3 | = |z - z4 | æ1 3 ö i 2p/3


i.e. z2 = z1 × ei 2p/3 = ç + i ÷e
Therefore, the point having affix z is equidistant from è2 2 ø
the four points having affixes z1 , z2, z3 and z4. Thus, z is
= ei p/3 × ei 2p/3 = ei p = cos p + i sin p
the affix of either the centre of a circle or the point of = -1
intersection of diagonals of a square (or rectangle).
Therefore, z1 , z2, z3 and z4 are either concyclic or 86. A(z1)
vertices of a square (or rectangle). Hence,
z1 , z2, z3 and z4 are concyclic.
p/3
5
83. Let z1 = 1 + i , z2 = - 2 + 3 i and z3 = 0 + i.
3
p/3 p/3
x1 y1 1 1 1 1
Then, x2 y2 1 = -2 3 1 B(z2) C(z3)
5
x3 y3 1 0 1 \ AC = AB eip/ 3
3
98 JEE Main Mathematics

p Þ | A | =| B| B
By rotating in clockwise sense
3
Hence, | A | =| B| =| A - B| |B| |A – B|
Þ (z3 - z1 ) = (z2 - z1 ) eip/ 3 …(i) All sides are equal.
Also, (z1 - z2) = (z3 - z2) eip/ 3 …(ii) Hence, it is an equilateral triangle. O A
|A|
On dividing Eq. (i) by Eq. (ii), we get 90. (2 - i )z = (2 + i )z
z3 - z1 z2 - z1 Þ (2 - i )(x + iy) = (2 + i )(x - iy)
=
z1 - z2 z3 - z2 Þ 2x - ix + 2 iy + y = 2x + ix - 2iy + y
Þ z12 + z22 + z32 = z1z2 + z2z3 + z3 z1 Þ 2 ix - 4iy = 0
87. Given z , iz and z + iz be the vertices of a triangle, then L1 : x - 2 y = 0
let z = x + iy Þ (2 + i )z + (i - 2)z - 4i = 0
\ iz = - y + ix, z + iz = (x - y) + i (x + y) Þ (2 + i )(x + iy) + (i - 2)(x - iy) - 4i = 0
x y 1 Þ 2x + ix + 2iy - y + ix - 2x + y + 2iy - 4i = 0
1 Þ 2 ix + 4iy - 4 i = 0
Hence, required area = -y x 1
2 L2 : x + 2 y - 2 = 0
x- y x+ y 1
1
x y 1 Solve L1 and L 2 4 y = 2 , y =
1 2
= -y x 1
2 \ x=1
0 0 -1
æ 1ö
[Q R3 ® R3 - (R1 + R2)] Centre ç1, ÷
è 2ø
é 1 2 ù L3 : iz + z + 1 + i = 0
= ê - (x + y2)ú
ë 2 û
Þ i (x + iy) + x - iy + 1 + i = 0
1
= |z |2 [Q z = x + iy,|z|2 = x2 + y2] Þ ix - y + x - iy + 1 + i = 0
2
Þ (x - y + 1) + i (x - y + 1) = 0
88. Vertices of the triangle are 0 = 0 + i0, z = x + iy æ 1ö
Radius = distance from ç1, ÷ to x - y + 1 = 0
and zeia = (x + iy) (cos a + i sin a ) è 2ø
= (x cos a - y sin a ) + i ( y cos a + x sin a ) 1
1- +1
2 3
\ Area of triangle \ r= =
2 2 2
0 0 1
1 91. Let z = x + iy, therefore given equation becomes
= x y 1
2 (x + iy) (x - iy) + (2 - 3 i ) (x + iy)
(x cos a - y sin a ) ( y cos a + x sin a ) 1
+ (2 + 3 i ) (x - iy) + 4 = 0
1
= [xy cos a + x2 sin a - xy cos a + y2 sin a ] Þ x2 + y2 + 2x + 3 y - 3 ix + 2 iy + 2x - 2 iy
2
+ 3 ix + 3 y + 4 = 0
1 1
= (x2 + y2) sin a = |z|2 sin a 2
[Q|z| = x + y ] 2
Þ x + y2 + 4x + 6 y + 4 = 0
2
2 2
Therefore, given equation represents a circle with radius
A B
89. + =1 = 22 + 32 - 4 = 4 + 9 - 4 = 9 = 3
B A
A 2 + B2 = AB 92. Let z = x + iy
B2 | B2 | z + i x + iy + i x + i ( y + 1 )
Þ =B- AÞ = | B - A| ...(i) \ = =
A | A| z + 2 x + iy + 2 (x + 2) + iy
| A - B|
Þ ½ ½=
A B A [x + i ( y + 1 )] ´ [(x + 2) - iy]
Þ =1 - =
B A ½B½ | A| [(x + 2) + iy] ´ [(x + 2) - iy]
| A 2| é x2 + 2x + y2 + y ù é ( y + 1 ) (x + 2) - xy ù
Þ =| A - B| ...(ii) =ê +iê
| B| 2 2 ú ú
ë (x + 2 ) + y û ë (x + 2)2 + y2 û
Þ | A - B| =| B - A |
Since, it is purely imaginary, therefore real part must
| A 2| | B2| be equal to zero.
Þ = [from Eqs. (i) and (ii)]
| B| | A |
x2 + y2 + 2x + y
3 3 \ =0
Þ | A | =| B | (x + 2)2 + y2
Complex Numbers 99

Þ x2 + y 2 + 2 x + y = 0
It represents the equation of circle and its radius B
(4, 3)
1 5 z C
= 1+ -0 = A
4 2
Therefore, locus of z in argand diagram is a circle of
5 O P
radius .
2
93. Equation of circle whose centre is z0 and radius is r, is \ Least or minimum value of| z | = OA
2
|z - z0| = r 2 = OC - AC = 5 - 2 = 3
Þ (z - z0 ) (z - z0 ) = r 2 [Q OC = OP 2 + CP 2 = 42 + 32 = 25 = 5]
Þ (z - z0 ) (z - z0 ) = r 2 Greatest or maximum value of z
Þ zz - zz0 - zz0 + z0z0 = r 2 OB = OC + CB = 5 + 2 = 7
94. Given equation of circle, Alternate Method
|z |2 + z (2 + 3i ) + z (2 - 3i ) + 4 = 0 Given,| z - 4 - 3i | = 2
Centre = (- 2 + 3i ), radius = 13 - 4 = 3 and | z - (4 + 3i )| ³ || z | - |4 + 3i ||
\ |z + (2 - 3i )| = 3 …(i) \ 2 ³ || z | - 5|
Let w = z + 3 + 2i = z + 2 - 3i + 1 + 5i Þ || z | - 5| £ 2 Þ - 2 £ | z | - 5 £ 2
\ w - 1 - 5i = z + 2 - 3i Þ 3 £ | z | £ 7.
Þ |w - 1 - 5i| = |z + 2 - 3i| = 3 or |w - 1 - 5i| = 3 \Maximum or minimum value of| z |is 7 and 3,
respectively.
So, w lies on a circle whose centre is (1 + 5i ) and radius
is 3. 97. Let O is orthocentre, G is centroid and C is
z -1 (x - 1) + iy circumcentre, then
95. For z = x + iy, =
2z + i 2x + (2 y + 1)i O(z) 2 G 1 C(0)
[(x - 1) + iy][2x - i (2 y + 1)]
= (on rationalization) z1 + z2 + z3
(2x)2 + (2 y + 1)2
3
æ z - 1 ö 2x(x - 1) + y(2 y + 1) z1 + z2 + z3 2 ´ 0 + 1(z )
\ Reç ÷= = Þ z = z1 + z2 + z3
è 2z + i ø 4x2 + (2 y + 1)2 3 3
Now, it is given that æ|z|2 - |z| + 1 ö
98. log ç ÷ <2
è 2 + |z| ø
3
æ z -1 ö
Re ç ÷ =1
è 2z + i ø |z|2 - |z| + 1
Þ < ( 3 )2
2x(x - 1) + y(2 y + 1) 2 + |z|
Þ =1
4x2 + (2 y + 1)2
Þ |z|2 - |z| + 1 < 3 (2 + |z|)
2 2 2 2
Þ 2x - 2x + 2 y + y = 4x + 4 y + 4 y + 1 Þ |z|2 - 4|z| - 5 < 0
Þ 2x2 + 2 y2 + 2x + 3 y + 1 = 0, is a circle Þ (|z| + 1 ) (|z| - 5) < 0
æ 1 3ö 1 9 1 Þ -1 < |z| < 5 Þ |z| < 5 as|z| > 0
whose centre is ç - , - ÷ and radius is + -
è 2 4ø 4 16 2 \ Locus of z is|z| < 5.
5 5
= , so diameter is .
4 2 Round II
96. Let z = x + iy 1. |z - i Re (z )| = |z - Im (z )|

Given, |z - 4 - 3 i |= 2 If z = x + iy,
Þ | x + iy - 4 - 3 i | = 2 then|x + iy - ix| = |x + iy - y|
Þ |(x - 4) + i ( y - 3)| = 2 x2 + ( y - x)2 = (x - y)2 + y2
Þ (x - 4)2 + ( y - 3)2 = 2 Þ x2 = y 2
or (x - 4)2 + ( y - 3)2 = 4 \ x=± y
which is an equation of circle having centre (4, 3)and Þ Re (z ) = ± Im (z )
radius 2. Þ Re (z ) + Im (z ) = 0
Thus, z lies on the boundary of the circle. and Re (z ) - Im (z ) = 0
100 JEE Main Mathematics

2. a 0z n + a1z n - 1 + a 2z n - 2 + . . . + a n - 1z + a n = 3 \ |z | - 1 ³ 0 and |z | - 25 £ 0
n
Þ|3| = | a 0z + a1z n -1
+ . . . + a n - 1z + a n | Hence, 1 £ | z | £ 25 Þ 1 £ | z - 0| £ 25
Þ 3 £ | a 0 || z | + | a1 || z |n - 1 + . . . + | a n - 1 || z | + | a n |
n 5. z = (1 - i cot 8)3
Þ 3 < 2 (| z |n + | z |n - 1 + . . . + | z | + 1 ) = cosec3 8 (sin 8 - i cos 8)3
3 3
Þ 1 + | z | + | z |2 + . . . + | z |n > ì æp ö æp öü
2 = cosec3 8 í cos ç - 8÷ - i sin ç - 8÷ý
î è 2 ø è 2 øþ
If| z | ³ 1, the inequality is clearly satisfied. For| z | < 1,
ì æ 3p ö æ 3p öü
we must have. = cosec3 8 í cos ç - 24÷ - i sin ç - 24÷ý
1 - | z |n + 1 3 î è 2 ø è 2 øþ
> æ3 p ö æ 3pö
1 -| z| 2 -i ç
è 2
- 24 ÷
ø
i ç 24 -
è
÷

Þ 2 - 2| z |n + 1 > 3 - 3| z | = cosec3 8 × e = cosec3 8 × e
Þ 2| z |n + 1 < 3| z | - 1 6. Given that, (x - 1 )3 + 8 = 0
1
Þ 3| z | - 1 > 0 Þ| z | > Þ (x - 1 )3 = (- 2 )3
3 3
Since, z1 is a root of given equation, æ x - 1ö æ x - 1ö 1/3
Þ ç ÷ =1Þç ÷ = (1 )
1 è -2 ø è -2 ø
\ | z1 | >
3 æ x - 1ö 2
\ Cube roots of ç ÷ are 1, w and w .
3p è -2 ø
3. Q arg (z - 3 i ) = arg (x + iy - 3 i ) =
4
Þ Cube roots of (x - 1 ) are - 2 , - 2 w and - 2 w2
é 3p ù
Þ x < 0, y - 3 > 0 êQ is in IInd quadrantú Þ Cube roots of x are - 1, 1 - 2 w and 1 - 2 w2.
ë 4 û
1 1 1 1/3
y-3 3p + + , + ...
= tan = -1 7. x = 93 9 27 = 91 - 1/3 = 91/ 2 = 3
x 4
1 1 1 1/3
- + + ...
Þ y= -x+ 3 …(i) y = 43 9 27 = 41 + 1/3 = 41/ 4 = 2
"x < 0 and y>3 ¥
1 1 1
and arg (2z + 1 - 2 i ) = arg [(2x + 1 ) + i (2 y - 2)] =
p z= å (1 + i )-r = + +
1 + i (1 + i )2 (1 + i )3
+ ...
r =1
4
Þ 2x + 1 > 0, 2 y - 2 > 0 1
é p ù 1+ i 1
êëQ 4 is in Ist quadrantúû = = = -i
1 i
1-
2y - 2 p 1+ i
\ = tan = 1
2x + 1 4 Let a = x + yz = 3 - 2i (fourth quadrant). Then,
Þ 2 y - 2 = 2x + 1 æ 2ö
3 -1 arg a = - tan -1 ç ÷
Þ y=x+ , "x> , y>1 …(ii) è 3 ø
2 2
From Eqs. (i) and (ii), we get 8. Let z1 = a + ib and z2 = c - id , where a > 0 and d > 0.
Y Then,| z1 | = | z2|
(0, 3) Þ a 2 + b2 = c2 + d 2 …(i)
z + z2 (a + ib) + (c - id )
Now, 1 =
z1 - z2 (a + ib) - (c - id )
[(a + c) + i (b - d )][(a - c) - i (b + d )]
=
–1/2
X [(a - c) + i (b + d )][(a - c) - i (b + d )]

It is clear from the graph that there is no point of (a 2 + b2) - (c2 + d 2) - 2(ad + bc)i
=
intersection. a 2 + c2 - 2ac + b2 + d 2 + 2bd
25½ ½ 25½ ½ 25½ - (ad + bc )i
4. ½z - ³ ½|z | - ½ Þ 24 ³ ½|z| - ½ = [using Eq. (i)]
½ z½ ½ |z|½ ½ |z|½ a 2 + b2 - ac + bd

Þ -24 £ |z| -
25
£ 24 Þ -24|z| £ |z|2 - 25 £ 24|z| Hence, (z1 + z2)/(z1 - z2) is purely imaginary. However, if
|z| ad + bc = 0, then (z1 + z2)/(z1 - z2) will be equal to zero.
\ |z |2 + 24|z | - 25 ³ 0 and |z |2 - 24|z | - 25 £ 0 According to the conditions of the equation, we can have
ad + bc = 0.
Þ (|z | + 25)(|z | - 1 ) ³ 0 and (|z | - 25)(|z | + 1 ) £ 0
Complex Numbers 101

æ z1 ö Re|w1 w2| = (ab + cd )


9. We have, arg ç ÷=p
è z2 ø = (bl ) b + c (- lc) = l(b2 - c2) = 0
Þ arg (z1 ) - arg (z2) = p 1 + i cos q (1 + i cos q)(1 + 2 i cos q)
14. We have, =
Þ arg (z1 ) = arg (z2) + p 1 - 2 i cos q (1 - 2 i cos q)(1 + 2 i cos q)
Let arg (z2) = q. Then, arg (z1 ) = p + q (1 - 2 cos 2 q) + i 3 cos q
\ z1 = | z1 |[cos (p + q) + i sin (p + q)] =
1 + 4 cos 2 q
= | z1 |(- cos q - i sin q)
(1 + i cos q)
and z2 = | z2|(cos q + i sin q) Thus, is a real number, if cos q = 0
(1 - 2 i cos q)
= | z1 |(cos q + i sin q) [Q| z1 | = | z2|] p
= - z1 Þ q = 2 np ±
2
Þ z1 + z2 = 0
where, n is an integer.
10. | z 2 + 2z cos a | £ | z 2| + |2z cos a |
15. Let z2 = (cos q2 + i sin q2)
= | z |2 + 2| z ||cos a | £ | z |2 + 2| z |
Also, arg (z1 ) + arg (z2) = 0
< ( 2 - 1 )2 + 2 ( 2 - 1 ) = 1
Þ arg (z1 ) = - arg (z2) = - q2
11. C (z3)
\ z1 = [cos (- q2) + i sin (- q2)]
= [cos q2 - i sin q2]
1
Þ z1 = z2 Þ z1 =
z2
Þ z1z2 = 1
B (z2) A (z1)
z1 - z2 1 i 16. | z2 – 3 – 4i | = 5
= - = e ip/ 4
z3 - z2 2 2 B
p
ÐCBA = A
| z1 | = 15
4
Also, | z1 - z2| = | z3 - z2| O
Hence, DABC is isosceles.
C
12. Given that,
| z1 - i | = | z2 - i | = | z3 - i | We have, | z1 | = 15,| z2 - 3 - 4i | = 5
Hence, z1 , z2 and z3 lie on the circle whose centre is i. Minimum value of| z1 - z2|is
Also, given that the triangle is equilateral. Hence, AB = OB - OA = 15 - 10 = 5 Maximum value of| z1 - z2|
centroid and cirucmcentre coincides. is
z1 + z2 + z3 CA = OA + OC = 10 + 15 = 25
\ =i
3 17. It is given for a complex number z = x + iy Î C, such that
Þ | z1 + z2 + z3 | = 3 |z | - Re(z ) £ 1 Þ x2 + y2 £ 1 + x
13. We have, z1 = a + ib and z2 = c + id
On squaring both sides, we get
\ |z1|2 = a 2 + b2 = 1 æ 1ö
x2 + y2 £ 1 + x2 + 2x Þ y2 £ 2 ç x + ÷
and |z2|2 = c2 + d 2 = 1 …(i) è 2ø
Also, Re(z1 z2) = 0 18. Let the complex number z = x + iy
Þ ac + bd = 0
Also given,| z - i | = | z - 1|
a d
Þ =- =l …(ii) Þ | x + iy - i | = | x + iy - 1|
b c
From Eqs. (i) and (ii), we get Þ x2 + ( y - 1)2 = (x - 1)2 + y2
b2l2 + b2 = c2 + l2c2 [Q| z | = (Re(z ))2 + (Im(z ))2 ]
Þ b2 = c2 and a 2 = d 2 On squaring both sides, we get
Now, 2 2
|w1| = a + c = a + b = 1 2 2
x2 + y2 - 2 y + 1 = x2 + y2 - 2x + 1
Þ y = x, which represents a line through the origin
and |w2| = b2 + d 2 = a 2 + b2 = 1
with slope 1.
102 JEE Main Mathematics

19. Given, z1 and z2 are non-real complex cube roots of æ p pö


22. Given, w3 = 8i = 8 ç cos + i sin ÷
unity. è 2 2ø
\ z1 = w, z2 = w2 é æ pö æ p öù
= 8 ê cos ç2rp + ÷ + i sin ç2rp + ÷ ú
As we know that, equation of circle when z1 and z2 ë è 2ø è 2 øû
are end parts of diameter is pö pö
æ æ
|z - z1 |2 + |z - z2 |2 = |z1 - z2|2 ç 2rp + ÷ ç 2rp + ÷
Þ w = 2 cos ç 2 ÷ + i sin ç 2 ÷,
\ l = |z1 - z2 |2 ç 3 ÷ ç 3 ÷
è ø è ø
Þ l = |w - w2|2
2 r = 0, 1, 2
-1 i 3 1 i 3
Þ l= - + - For w to be in second quadrant, r = 1
2 2 2 2
é 5p 5p ù
\ l =3 \ w = 2 ê cos + i sin =- 3+i
ë 6 6 úû
a+ i (a + i )2 (a 2 - 1) + (2a )i
20. Let x + iy = Þ x + iy = 2 = 23. Given, z3 + w5 = 0 and z 2 w 4 = 1
a-i a +1 a2 + 1
1
a 2 - 1 æ 2a ö Þ z3 = - w5 and z 2 =
= +ç ÷ i w4
a2 + 1 è a2 + 1 ø
1
On comparing real and imaginary parts, we get Þ |z |3 = |w|5 and|z |2 =
|w |4
a2 - 1 2a
x= 2 and y = 2 Þ |z |6 = |w |10 …(i)
a +1 a +1
1
2 2 and |z |6 = …(ii)
æ a2 - 1 ö æ 2a ö |w |12
Now, x2 + y2 = ç 2 ÷ +ç 2 ÷
è a + 1ø è a + 1ø
From Eq. (i) and (ii), we get
a 4 + 1 - 2a 2 + 4a 2 (a 2 + 1)2 1
= = 2 =1 |w |10 = Þ |w |22 = 1
(a 2 + 1)2 (a + 1)2 |w |12
Þ x2 + y2 = 1 Þ |w | = 1 and|z | = 1
ìa+ i ü Þ ww = 1 and z × z = 1 …(iii)
So, S = í ; a Î Rý lies on a circle with radius 1.
î a - i þ Also, z 6 = w10 and z 6 w12 = 1
3|z1| = 4|z2| 1
21. Given, z 6 = 12
w
|z1| 4
Þ = [Qz2 ¹ 0 Þ|z2| ¹ 0] 1
|z2| 3 \ w10 = 12
w
z1 z1 iq z z Þ w10 × w12 = 1
\ = e and 2 = 2 e-iq
z2 z2 z1 z1 Þ w10 × w10 × w 2 = 1 Þ w 2 = 1
iq
[Q z =|z|(cos q + i sin q) = |z| e ] Þ w = 1 or - 1 Þ w = 1, - 1
z1 4 iq z 3 If w = 1, then z3 + w5 = 0 and z 2 w 4 = 1
Þ = e and 2 = e-iq
z2 3 z1 4
Þ z3 + 1 = 0 and z 2 = 1 Þ z = - 1
3 z1 2 z2 1 -iq
Þ = 2eiq and = e If w = - 1, then z3 + w5 = 0 and z 2 w 4 = 1
2 z2 3 z1 2
Þ z - 1 = 0 and z 2 = 1
3

On adding these two, we get


Þ z =1
3 z1 2 z2 1
z= + = 2eiq + e-iq Hence, z and w both are purely real.
2 z2 3 z1 2
1 1 24. It is given for complex number z,
= 2 cos q + 2i sin q + cos q - i sin q
2 2 z-i
=1 … (i)
[Q e± iq = (cos q ± i sin q)] z + 2i
5 3
= cos q + i sin q 5
2 2 and | z| = … (ii)
2
2 2
æ5ö æ3ö 34 17 From Eq. (i), | z - i | = | z + 2i |
Þ |z| = ç ÷ + ç ÷ = =
è2ø è2ø 4 2
Þ Locus of z is a straight line and it is perpendicular
Note that z is neither purely imaginary and nor purely bisector of line joining points (0, - 2) and (0, 1), so locus
real. 1
of z is y = - , where z = x + iy.
2
Complex Numbers 103

5 28. Let the complex numbers


From Eq. (ii), x2 + y 2 =
2 z1 = x1 + iy1 and z2 = x2 + iy2
2 2
æ - 1ö æ5ö Now, it is given that
Þ x2 + ç ÷ =ç ÷
è 2 ø è2ø Re (z1 ) = |z1 - 1| Þ x12 = (x1 - 1)2 + y12
Þ
1
x2 + =
25 Þ y12 + 1 = 2x1 …(i)
4 4 and Re(z2) =|z2 - 1| Þ x22 = (x2 - 1)2 + y22
Þ x2 = 6 Þ y22 + 1 = 2x2 …(ii)
Now,| z + 3i |= | x + ( y + 3) i |= x2 + ( y + 3)2 p y1 - y2 1
and arg (z1 - z2) = Þ = …(iii)
2 6 x1 - x2 3
æ 1 ö 2
= 6 + ç - + 3÷ (Q x = 6 and y = - 1 / 2) From Eqs. (i) and (ii), we get
è 2 ø
25 24 + 25 49 7 y12 - y22 = 2(x1 - x2)
= 6+ = = = y1 - y2 1
4 4 4 2 Þ ( y1 + y2) = 2 Þ ( y1 + y2) = 2
x1 - x2 3
25. If z1 and z1 are adjacent vertices of a regular polygon of n
Þ y1 + y2 = 2 3 Þ Im (z1 + z2) = 2 3
sides with centre at origin and
Hence, option (d) is correct.
Im (z1 )
= 2 - 1 = tan [arg (z1 )].
Re (z1 ) 29. Let a = a + ib and g = c + id,
where a , b, c, d Î R. We have to minimise
A z1=r1ei q
a 2 + b2 + c2 + d 2
q Now, f (z ) = (4 + i ) z 2 + z (a + ib) + (c + id )
O q
f (1) = (4 + i ) + (a + ib) + (c + id ) is real.
B z1=r1e–i q or (4 + a + c) + i (1 + b + d ) is real.
Þ b + d + 1 =0 …(i)
If z1 = r1 ei q , then tan q = 2 - 1 = tan (p /8)
f (i ) = - (4 + i ) + i (a + ib) + (c + id ) is real.
q = p /8
= (- 4 - b + c) + i (a + d - 1) is real.
Now, rotation of z1 by 2q i.e. p /4 gives adjacent vertex
Þ a + d =1 …(ii)
2p
of polygon. i.e. there will be = 8 sides in polygon. Hence, there is no restriction on ‘c’.
p
Let c = 0
4
Hence, |a| + |g| = a 2 + b2 + d2
æ (|z|+ 3)(|z|- 1) ö
26. expç ln 2÷ ³ log 2|5 7 + 9 i| = 4 + 2ab + |d| ³ 4 + 2ab > 2
è ||z|+ 1| ø
(|z|+ 3 )(|z|-1 )
with equality if d = 0, a = 1 and b = - 1 .
Þ 2 (|z|+ 1 )
³ log (16) 30. We have,|z | + z = 3 + i
2
(|z|+ 3 )(|z|-1 ) By inspection it is clearly that imaginary part is 1.
(|z|+ 1 )
Þ 2 ³ 23 \ z = x + i for some x Î R
(|z|+ 3)(|z|-1) Þ |x + i | + x + i = 3 + i
Þ ³3
(|z|+ 1)
Þ x2 + 1 = 3 - x
Þ (|z|+ 3) (|z|- 1) ³ 3 (|z|+ 1)
Þ x 2 + 1 = 9 - 6 x + x2
Þ (|z|2+2|z|-3 ³ 3|z|+3) 4
Þ 6x = 8 Þ x =
Þ |z|2 - |z|-6 ³ 0 3
Þ (|z|-3)(|z|+2) ³ 0 Þ|z|-3 ³ 0 4
Þ z= +i
3
Þ |z|³ 3 Þ |z|min = 3
16 25 5
27. Q |z + 4| £ 3 Þ |z | = +1= Þ |z | =
9 9 3
|z + 1| = |z + 4 - 3|
31. For z = x + iy, z 2 = i | z |2
Þ ||z + 4| - |3 || £ | z + 4 - 3 | £ |z + 4| + |- 3 |
[using triangle inequality] Þ x2 - y2 + 2i xy = ix2 + iy2
Þ |z + 4 | - 3 | £ |z + 1| £ 3 + 3 Þ x2 - y2 = i (x - y)2
Þ 3 - 3 £ |z + 1| £ 6 Þ (x - y) (x + y) = i (x - y)2
0 £ |z + 1| £ 6 Þ x = y or x + y = i (x - y)
\ Greatest and least values are 6 and 0, respectively. So, the z lies on the line, y = x.
104 JEE Main Mathematics

æ p p ö æ1 ö
32. zn = cos ç + i sin ÷ 34. Q sin -1 ç (z - 1)÷ is only defined in [-1, 1]
è (2 n + 1 ) (2 n + 3 ) (2 n + 1 ) (2 n + 3 ) ø èi ø

z1 × z2 × z3 K zn which is possible, when Re (z - 1) = 0.


æ p p p ö i.e. x-1 =0
Þ cos ç + +L+ ÷
è 3 ×5 5 × 7 (2n + 1) (2n + 3) ø Þ x=1
Þ Re (z ) = 1
æ p p p ö
+ i sin ç + + L+ ÷ Im (z )
è 3 ×5 5 × 7 (2n + 1) (2n + 3) ø Also, i.e. y satisfy the condition of principal
i
é p ì æ 1 1 ö æ 1 1 öü æ 1 1 öù branch i.e. 0 < y < 1. (for angle of D)
Þ cos ê í ç - ÷ + ç - ÷ý + L + ç - ÷
ë î
2 è 3 5 ø è 5 7 ø þ è 2 n + 1 2 n + 3 ø úû Hence, sin -1 ç
æ z - 1ö
÷ is only defined when Re (z ) = 1
è i ø
é p ìæ1 1ö æ1 1 ö æ 1 1 öü ù
+ i sin ê í ç - ÷ + ç - ÷ + L + ç - ÷ý and 0 < Im (z ) < 1.
ë 2 îè3 5ø è5 7ø è 2n + 1 2n + 3 øþ úû
35. Let a complex number, z = x + iy, then|x| + | y| = 4
é p æ1 1 öù é p æ1 1 öù
= cos ê ç - ÷ ú + i sin ê ç - ÷ú Y
ë 2 è 3 2 n + 3 ø û ë 2 è 3 2 n + 3øû (0, 4)
Now, lim z1 × z2 L zn x+y=4
n® ¥

é (–4, 0)
æp p ö æp p öù X¢ X
= lim ê cos ç - ÷ + i sin ç - ÷ú O (4, 0)
n® ¥
ë è 6 2(2n + 3) ø è 6 2(2n + 3) ø û
p p
= cos + i sin
6 6 (0, –4)
2pi/3 Y¢
33. Given, z=e
So, maximum value of|z | = 4 and minimum value of
\ 1 + z + 3z 2 + 2z3 + 2z 4 + 3z5 |z | = perpendicular distance of line
= 1 + e2pi/3 + 3(e2pi/3 )2 + 2(e2pi/3 )3 |-4|
x + y = 4 from origin = =2 2
4 2
+ 2 (e2pi/3 ) + 3(e2pi/3 )5
\ |z |Î [2 2 , 4] = [ 8 , 16 ]
æ æ 2p ö æ 2p ö ö
= 1 + ç cos ç ÷ + i sin ç ÷ ÷
è è 3 ø è 3 øø So,|z |cannot be 7.
é æ 4p ö æ 4p ö ù z - z2 p
+ 3 ê cos ç ÷ + i sin ç ÷ ú 36. 1 is purely imaginary, as ÐB = .
è 3 ø è 3 øû z3 - z2 2
ë
C B
+ 2 [cos 2p + i sin 2p ] z3 z2
é 8p æ 8p ö ù
+ 2 ê cos + i sin ç ÷ ú
ë 3 è 3 øû
z4 z1
é æ 10p ö æ 10p ö ù D A
+ 3 ê cos ç ÷ + i sin ç ÷
ë è 3 ø è 3 ø úû z1 - z3
is purely imaginary as diagonals intersects at
= 1 + [cos 120° + i sin 120° ] + 3 [cos 240° z2 - z4
+ i sin 240° ] + 2 [cos 360° + i sin 360° ] right angle.
z1 - z2
+ 2 [cos 480° + i sin 480° ] is not purely imaginary as AB and CD are
z3 - z4
+ 3 [cos 600° + i sin 600° ]
parallel.
é 1 3 ù é 1 3 ù
=1 + ê- + iú + 3 ê- - iú 3 æ1ö
ë 2 2 û ë 2 2 û 37. Given, z = + ç ÷i
2 è2ø
+ 2 [1 + 0] + 2 [cos 120° + i sin 120° ] p
p p i
+ 3 [cos 120° - i sin 120° ] = cos + i sin = e 6
6 6
é 1 3 ù é 1 3 ù 9
= 3 - 2 - 3i + 2 ê - + iú + 3 ê - - iú æ i
p
i
5p 8p
i ö
ë 2 2 û ë 2 2 û so, (1 + iz + z5 + iz 8 )9 = ç1 + ie 6 + e 6 + ie 6 ÷
ç ÷
æ 5ö 3 3 3 3 è ø
= 1 - 3i + ç - ÷ - i=- - i
è 2ø 2 2 2 æ i
p
i
p
i
5p
i
p 4p
i ö
9
é p
i ù
é1 ù = ç1 + e 2 × e 6 + e 6 + e 2 × e 3 ÷ êQ i = e 2 ú
3 ç ÷ êë úû
= -3 ê + i ú = - 3epi/3 è ø
ë 2 2 û
Complex Numbers 105

2p 5p 11 p ö 9
æ i i i 40. |z | = max {|z - 1 |,|z + 1 |}
= ç1 + e 3 + e 6 + e 6 ÷
ç ÷ Case I |z - 1| > |z + 1| …(i)
è ø
é æ 2p 2p ö æ 5p 5p ö Þ |z | = |z - 1|
= ê1 + ç cos + i sin ÷ + ç cos + i sin ÷ 1
ë è 3 3 ø è 6 6 ø Þ z = , perpendicular bisector of origin and (1, 0).
9 2
æ 11p 11p ö ù
+ ç cos + i sin ÷ 1
è 6 6 ø úû But, when z =
2
9
æ 1 i 3 3 1 3 iö
= ç1 - + - + i+ - ÷
è 2 2 2 2 2 2ø O 1 /2 1
9 9
æ1 3i ö æ p pö 1
=ç + ÷ = ç cos + i sin ÷ |z - 1| =
è2 2 ø è 3 3ø 2
= cos 3p + i sin 3p [Q for any natural number ‘n’ 3
and |z + 1| =
(cos q + i sin q)n = cos(nq) + i sin(nq)] 2
= -1 \Case I does not hold.
38. We have, z5 - 1 = (z - 1) (z - a1 ) .... (z - a 4 ) …(i) Case II |z - 1| < |z + 1| …(ii)
Putting z = w in Eq. (i), we get Þ |z| = |z + 1|
(w - 1)(w - a1 )(w - a 2)(w - a3 ) 1
Þ z = - , perpendicular bisector of origin and (- 1, 0).
(w - a 4 ) = w5 - 1 2
Now, putting z = w2 in Eq. (i), we get 1
But when z = -
(w2 - 1) (w2 - a1 ) (w2 - a 2) 2
(w2 - a3 ) (w2 - a 4 ) = w10 - 1
–1 –1/2 O
(w - 1) (w - a1 ) (w - a 2) (w - a3 ) (w - a 4 ) w5 - 1
\ 2 =
(w - 1) (w2 - a1 ) (w2 - a 2) (w2 - a3 ) (w2 - a 4 ) w10 - 1 3
|z - 1| =
(w - a1 )(w - a 2)(w - a3 )(w - a 4 ) 2
Þ
(w2 - a1 )(w2 - a 2)(w2 - a3 )(w2 - a 4 ) and |z + 1 | =
1
(w2 - 1)2 2
= = (w + 1)2 = w4 = w \ Case II does not hold.
(w - 1)2
Þ z Îf
39. |z|³ 2 is the region on or outside circle whose centre is
41. Given,| z | = 1, arg z = q\z = eiq
(0, 0) and radius is 2.
1
1 But z=
Minimum z + is distance of z, which lie on the circle z
2
æ -1 ö æ ö
|z |= 2 from ç , 0÷ . ç1 + z÷
è 2 ø \ arg ç ÷ = arg (z ) = q
ç1 + 1 ÷
1 æ 1 ö è zø
\Minimum z + = Distance of ç - , 0÷ from (-2, 0)
2 è 2 ø
z2
2 42. Since, (z ¹ 1 ) is purely real.
æ 1ö 3 z -1
= ç -2 + ÷ + 0 =
è 2ø 2
z2 z2
Hence, =
Geometrically z -1 z - 1
2
1 æ -1 ö 3 Þ z 2(z - 1 ) = z 2(z - 1 )
Min z + = AD = ç + 2÷ + 0 =
2 è 2 ø 2 Þ z 2z - z 2 = z 2z - z 2
Y Þ zzz - z 2 = z 2zz - z 2
z | z |2 - z 2 = z | z |2 - z 2
Þ z | z |2 - z | z |2 = z 2 - z 2
D A
X¢ X Þ | z |2 (z - z ) = (z - z )(z + z )
(0, 0) (2, 0)
(–2, 0)
(
– 1, 0
2 ) Þ 2
| z | (z - z ) - (z - z )(z + z ) = 0
Þ (z - z ){| z |2 - (z + z )} = 0

106 JEE Main Mathematics

Either (z - z ) = 0 or {| z |2 - (z + z )} = 0 Þ a 2 + b2 = 1 ...(i)
Now, z = z Þ Locus of z is real axis and | A + 1| = 1
and {| z |2 - (z + z )} = 0 Þ (a + 1)2 + b2 = 1 ...(ii)
Þ zz - (z + z ) = 0 Þ 2
(a + 1) - a = 0 2

Locus of z is a circle passing through origin. [from Eqs. (i) and (ii)]
Alternate Method Þ a 2 + 1 + 2a - a 2 = 0
Put z = x + iy, then 1
z2 (x + iy)2 (x2 - y2) + i (2xy) a=-
= = 2
z - 1 (x + iy) - 1 (x - 1 ) + iy 3 3
From Eq. (i), b2 = Þ b = ±
(x2 - y2) + i (2xy) (x - 1 ) - iy 4 2
= ´
(x - 1 ) + iy (x - 1 ) - iy -1 ± i 3
Hence, A= Þ A = w, w2
z2 2
Since, (z ¹ 1 ) is purely real, hence its imaginary
z -1 Q ( A )n = ( A + 1)n Þ wn = (1 + w)n = (- w2)n
part should be equal to zero. Which is true when n must be even or divisible by 3.
2 2
Þ (x - y )(- y) + (2xy)(x - 1 ) = 0 For minimum value of n ¢ , n must be 6.
Þ y(x2 - y2 + 2x - 2x2) = 0 46. Given, |z + 5| £ 4
Þ y(x2 + y2 - 2x) = 0 Þ (x + 5)2 + y2 £ 16 …(i)
Þ Either y = 0 Þ x2 + y2 - 2x = 0 Also, z (1 + i ) + z (1 - i ) ³ -10
Now, y = 0 Þ Locus of z is real axis and x2 + y2 - 2x = 0. Þ x - y ³ -5 [Q z = x + iy] …(ii)
Locus of z is a circle pasing through origin. From Eqs. (i) and (ii) Locus of z is the shaded region in
Locus of z is either real axis or a circle passing through the diagram.
origin. Y

43. It is given that,


m/ 2 n/3 x–y=5
æ1 + i ö æ1 + i ö B
ç ÷ =ç ÷ = 1, (m, n Î N ) 0)
è1 - i ø è i - 1ø ,
–5 Q(–1, 0)
m/ 2 n/3 A(
Þ i = 1 = (- i ) X¢
O
X
So, the least values of ‘m’ and ‘n’ are 8 and 12
respectively. P
\ GCD {8, 12} = 4 Y¢
|z + 1|represents distance of ‘z’ from Q(-1, 0). Clearly ‘p’
Hence, answer is 4.00.
21 21
is the required position of ‘z’ when|z + 1|is maximum.
æ i 2p ö æ ip ö \ P º (-5 - 2 2 , - 2 2 )
ç2e 3 ÷ ç2e 3 ÷
ç ÷ ç ÷ [on solving Eqs. (i) and (ii)]
è ø è ø
44. +
p 24 p 24 \(PQ )2|max = 32 + 16 2
æ -i ö æ i ö
ç 2e 4 ÷ ç 2e 4 ÷ Þ a = 32
ç ÷ ç ÷
è ø è ø Þ b = 16
221 × ei14p 221 (ei7p ) Thus, a + b = 48
= 12 -i6p + 12 i6p
2 ×e 2 (e ) 47. We have, z = a + ib which is root of z5 = 1
= 29 ei( 20p ) + 29 eip and|z|5 = 1 Þ|z| = 1
9 9
= 2 + 2 (-1) = 0 Þ n = 0 1
Now, z = and a 2 + b 2 = 1
5 5 2
å ( j + 5 )2 - å ( j + 5 ) \ 4a (b 4 - a 4 ) = 4a (b 2 - a 2)|z|2
j=0 j=0 1
= - (z + z ) [(z - z )2 + (z + z )2]
= [5 + 6 + 7 + 82 + 92 + 102] - [5 + 6 + 7 + 8 + 9 + 10]
2 2 2 2
= [(12 + 22+ ...+102) - (12 + 22 + 32 + 42)] = - (z + z ) (z 2 + z 2)
æ 1 1ö 1 æ z8 - 1 ö
- [(1 + 2 + 3+ ...+10) - (1 + 2 + 3 + 4)] = - ç z3 + z + + 3 ÷ = - 3 ç 2 ÷
è z z ø z è z -1ø
= (385 - 30) - [55 - 10] = 355 - 45 = 310
z 8 - 1 1 - z3
45. Let A = a + ib =- = =1 [Q z5 = 1]
z5 - z3 1 - z3
| A | = a 2 + b2
05
Quadratic Equations

An algebraic expression of the form a0 + a1x + a2x 2 + K + an x n , where n Î N , IN THIS CHAPTER ....
is called a polynomial. It is generally denoted by p( x ), g( x ), f ( x ), q( x ), etc. Real and Complex Polynomial
Identity and Equation
Real and Complex Polynomial
Roots of an Equation
If a0 , a1 , a2 ,... , an are real numbers and x is a real variable, then
Remainder and Factor Theorem
f ( x ) = a0 + a1x + a2x 2 + .... + an x n is known as a real polynomial with real
Quadratic Equation
coefficients, if a0 , a1 , a2 ,.... , an are complex numbers and x is a varying
Nature of Roots
complex number, then f ( x ) = a0 + a1x + a2x 2 + .... + an x n is called a complex
Relation between Coefficients
polynomial with complex coefficients.
and Roots of an Equation
e.g. x 2 + 3x + 7 is a real polynomial whereas x 2 + ( 3 + 2 i ) x + ( 7 + 3 i ) is a
Formation of an Equation with
complex polynomial. Given Roots
Symmetric Function
Identity and Equation
Condition for Common Root(s)
A statement of equality of two expressions which is satisfied for each value
of variable is called an identity. Different Conditions for Roots

e.g. ( x - 4)2 + 8x = x 2 + 16 is an identity. Quadratic Expression

A statement of equality between two expressions which is satisfied for definite Possible Graph of Quadratic
values of variable, is known as an equation. Expression

e.g. x 2 - 5x + 6 = 0 is an equation, which is not satisfied for any value of x Position of Roots of a Quadratic
Equation
except 2 and 3.
Maximum and Minimum Value
of ax2 + bx + c
Roots of an Equation
The values of a variable in an equation which satisfied the given equation are
known as roots of an equation i.e. if f ( x ) = 0 is a polynomial equation and
f ( a ) = 0, then a is a root of f ( x ) = 0.
e.g. If f ( x ) = x 2 + 3x + 2 = 0
and f( -1) = ( -1) + 3 ( -1) + 2 = 1 + 2 - 3 = 0 Þ f( - 1) = 0
\ - 1 is a root of f ( x ) = 0.
108 JEE Main Mathematics

Example 1. Two non-integer roots of Example 3. Let p, q ÎR. If 2 - 3 is a root of the quadratic
æ 3x - 1 ö
4
æ 3x - 1 ö
2
equation, x 2 + px + q = 0, then (JEE Main 2019)
ç ÷ -5 ç ÷ + 4 = 0 are
è2x + 3ø è2x + 3ø 2
(a) q - 4p - 16 = 0
(a) -5 / 7, - 2 / 5 (b) - 2 / 5, 7 / 5 (b) p 2 - 4q - 12 = 0
(c) 5 / 7, 7 / 5 (d) - 2/5, 3/5 (c) p 2 - 4q + 12 = 0
æ 3x - 1 ö
2
(d) q 2 + 4p + 14 = 0
Sol. (a) Let ç ÷ =t
è 2x + 3 ø Sol. (b) Given quadratic equation is x2 + px + q = 0 , where
Then, given equation becomes p, q Î R having one root 2 - 3 , then other root is 2 + 3
t 2 - 5t + 4 = 0 Þ (t - 1) (t - 4) = 0 Þ t = 1or 4 [conjugate of 2 - 3]
3x - 1 So, sum of roots = - p = 4 Þ p = -4
When, t = 1, then = ±1
2x + 3 and product of roots = q = 4 - 3 Þ q = 1
-2 Now, from options p 2 - 4q - 12 = 16 - 4 - 12 = 0
Þ x = 4,
5
3x - 1 Nature of Roots
and when, t = 4 then = ±2
2x + 3
(i) The roots are real and distinct, iff D > 0.
Þ x = - 7,- 5 / 7 (ii) The roots are real and equal, iff D = 0.
Thus, two non-integer roots of given equation are - 2 / 5, - 5 / 7.
(iii) The roots are complex with non-zero imaginary
part, iff D < 0.
Quadratic Equation (iv) The roots are rational, iff a , b, c are rational and D is
If f ( x ) is a quadratic polynomial. Then, f ( x ) = 0 is known perfect square.
as quadratic equation. The general form of a quadratic (v) The roots are of the form p + q ( p, q Î Q ), iff a , b, c
equation is ax 2 + bx + c = 0, " a , b, c Î R or C and a ¹ 0. are rational and D is not a perfect square.
(vi) If a = 1, b, c Î I and the roots are rational numbers,
Roots of a Quadratic Equation then these roots must be integers.
Roots of the quadratic equation ax 2 + bx + c = 0 are (vii) If a quadratic equation in x has more than two roots,
then it is an identity in x that is a = b = c = 0.
- b + b2 - 4ac - b - b2 - 4ac
and , where b2 - 4ac is
2a 2a Example 4. The number of integral values of m for which
known as discriminant and it is denoted by D. equation (1 + m 2) x 2 - 2(1 + 3m) x + (1 + 8m) = 0, has no real root
If in a quadratic equation ax 2 + bx + c = 0, a , b, c are is (JEE Main 2019)
complex numbers, then we cannot assign positive or (a) 3 (b) infinitely many
negative sign to the discriminant. (c) 1 (d) 2
If one root of an equation is a + b or a + ib, then another
Sol. (b) Given quadratic equation is
root of an equation will be a - b or a - ib, respectively.
(1 + m2) x2 - 2(1 + 3m) x + (1 + 8m) = 0 …(i)
Example 2. The product of the roots of the equation Now, discriminant
9 x 2 - 18| x| + 5 = 0, is (JEE Main 2020)
D = [ -2(1 + 3m)]2 - 4(1 + m2)(1 + 8m)
5 25 5 25 = 4 [(1 + 3m) 2 - (1 + m2)(1 + 8m)]
(a) (b) (c) (d)
27 9 9 81 = 4 [1 + 9m2 + 6m - (1 + 8m + m2 + 8m3)]
Sol. (d) Given equation 9x2 - 18| x| + 5 = 0 = 4 [ -8m3 + 8m2 - 2m]
Þ 9| x|2 - 18| x| + 5 = 0 = - 8m( 4m2 - 4m + 1)
Þ 9| x|2 - 15| x| - 3| x| + 5 = 0 = - 8m(2m - 1) 2
Þ 3| x|(3| x| - 5) - 1(3| x| - 5) = 0 According to the question there is no solution of the
1 5 quadratic Eq. (i), then
Þ | x| = , D <0
3 3
1 5 \ -8m(2m - 1) 2 < 0
Þ x=± ,±
3 3 Þ m>0
-
æ öæ öæ öæ ö
1 1 5 - 5 25 So, there are infinitely many values of ‘m’ for which, there is
So, product of roots = ç ÷ ç ÷ ç ÷ ç ÷ =
è 3 ø è 3 ø è 3 ø è 3 ø 81 no solution of the given quadratic equation.
Quadratic Equations 109

Relation between Coefficients and Formation of an Equation


Roots of an Equation with Given Roots
(i) Quadratic Equation If a and b are the roots of (i) Quadratic Equation If a and b are the roots of a
the quadratic equation ax 2 + bx + c = 0 , then quadratic equation, then the equation will be
x 2 - (a + b )x + a b = 0.
b
Sum of roots, a + b = - (ii) Cubic Equation If a , b and g are the roots of a
a
cubic equation, then the equation will be
c
Product of roots, ab = x3 - (a + b + g ) x2 + (ab + bg + ga) x - abg = 0.
a
(ii) Cubic Equation If a , b and g are the roots of a Example 6. If a and b are the roots of equation
cubic equation ax3 + bx 2 + cx + d = 0, then 2 x 2 - 5 x + 7 = 0, then the equation whose roots are

Sum of roots, a + b + g = -
b
, 2 a + 3 b and 3 a + 2 b, is
a (a) 2x2 - 25x + 82 = 0 (b) 2x2 + 25x + 82 = 0
c 2
Product of two roots, ab + bg + ga = (c) x - 25x + 82 = 0 (d) None of these
a
d Sol. (a) Since, a and b be the roots of the equation
Product of three roots, abg = -
a 2x2 - 5x + 7 = 0 , then
5 7
Example 5. If the roots of the equation x 2 + px + q = 0 are a + b = and ab =
2 2
in the same ratio as those of the equation x 2 + lx + m = 0, then æ 25 ö
Now, sum of roots = (2 a + 3 b) + (3 a + 2 b) = 5( a + b) = ç ÷
2 è2ø
p m is equal to
and product of roots = (2 a + 3 b)(3 a + 2 b)
(a) lq
(b) l 2q = 6 ( a 2 + b 2) + 13 ab

(c) lq 2 = 6 [( a + b) 2 - 2 ab ] + 13 ab
(d) None of the above é æ 25 ö 91ù
= ê6 ´ ç - 7÷ + ú = 41
ë è 4 ø 2û
Sol. (b) Let the roots of each equation be in the ratio k :1.
The required equation is
Let ka and a be the roots of x2 + px + q = 0 . 25
x2 - x + 41 = 0
Then, ka + a = - p and ka × a = q 2
-p q Þ 2x2 - 25x + 82 = 0
Þ a= and a 2 =
k+1 k
æ -p ö q
2 Symmetric Function
Þ ç ÷ = A function of a and b is said to be a symmetric function,
è k + 1ø k
if it remains unchanged when a and b are interchanged.
k q
Þ 2
= 2 …(i) Let a and b be the roots of the equation ax 2 + bx + c = 0,
(k + 1) p b c
then a + b = - and a b =
Again, let kb and b be the roots of x2 + lx + m = 0 . a a
Then, kb + b = - l and kb × b = m The following results may be useful.

Þ b=-
l
and b 2 =
m (i) a 2 + b 2 = (a + b )2 - 2 ab
k+1 k (ii) a 3 + b3 = (a + b )3 - 3 ab(a + b )
2
æ -l ö m (iii) a 4 + b 4 = [(a + b )2 - 2 ab ]2 - 2 (ab )2
Þ ç ÷ =
è k + 1ø k
(iv) (a - b ) = (a + b )2 - 4 ab
k m
Þ 2
= 2 …(ii) (v) a 2 - b 2 = (a + b ) (a + b )2 - 4 ab
(k + 1) l
(vi) a 3 - b3 = (a - b )[(a + b )2 - a b ]
Thus, from Eqs. (i) and (ii), we get
q m = [ (a + b )2 - 4ab ] [(a + b )2 - ab ]
=
p2 l 2 (vii) a 4 - b 4 = (a + b )(a - b )(a 2 + b 2 )
2 2
Þ p m=l q = (a + b ) [ (a + b )2 - 4 ab ] [(a + b )2 - 2 ab ]
110 JEE Main Mathematics

Example 7. If a and b are the roots of the equation, This is the condition for roots of two quadratic
a b equations to be common.
7x 2 - 3x - 2 = 0, then the value of + is equal to
1- a 2 1- b2 (b) When two common roots In such a case, two
(JEE Main 2020) equations should be identical. The ratio of
27 1 3 27 coefficients of x 2 , x1 and x 0 must be same
(a) (b) (c) (d)
32 24 8 16 a b c
i.e., = =
Sol. (d) Given quadratic equation 7x2 – 3x – 2 = 0 has roots a and b. a ¢ b¢ c¢
3 2
So, a + b = and ab = - × Example 9. Let l ¹ 0 be in R. If a and b are the roots of
7 7
the equation, x 2 - x + 2 l = 0 and a and g are the roots of the
a b a – ab 2 + b – ba 2 bg
Now, + = equation, 3x 2 - 10 x + 27l = 0, then is equal to
1 – a 2 1 – b 2 1 – ( a 2 + b 2) + a 2b 2 l
( a + b) – ab( a + b) (JEE Main 2020)
= (a) 36 (b) 9 (c) 27 (d) 18
1 – [( a + b) 2 – 2ab ] + ( ab) 2
3 2 æ3ö Sol. (d) It is given that a is a common roots of given quadratic
+ ç ÷
7 7 è7ø equations x2 – x + 2l = 0 and 3x2 – 10 x + 27l = 0
=
éæ3ö2 4ù 4 \ 3 a 2 – 10 a + 27l = 0
1– ê ç ÷ + ú + 3 a 2 - 3 a + 6l = 0
êë è 7 ø 7 úû 49
– + –
21 + 6 27 0 – 7a + 21l = 0 Þ a = 3l
= =
49 – 37 + 4 16 1
So, 9l2 – 3l + 2l = 0 Þ l = [Q l ¹ 0 ]
Example 8. If m is chosen in the quadratic equation 9
(m 2 + 1) x 2 - 3x + (m 2 + 1) 2 = 0 such that the sum of its roots is 1
Þ a=
3
greatest, then the absolute difference of the cubes of its roots is 1
(JEE Main 2019) 2´
2l 9 =2
(a) 10 5 (b) 8 5 (c) 8 3 (d) 4 3 As ab = 2l Þ b = =
a 1/ 3 3
Sol. (b) Given quadratic equation is 1
(m2 + 1) x2 - 3x + (m2 + 1) 2 = 0 9´
…(i) 9l 9 =3
and ag = 9l Þ g = =
Let the roots of quadratic Eq. (i) are a and b, so a 1/ 3
3 2
a+b= 2 and ab = m2 + 1 ´3
m +1 bg 3
\ = = 18
According to the question, the sum of roots is greatest and it l 1/ 9
is possible only when ‘‘(m2 + 1) is min’’ and ‘‘min value of
m2 + 1 = 1, when m = 0’’. Example 10. If the equations x 2 + 2 x + 3 = 0 and
\ a + b = 3 and ab = 1, as m = 0 ax 2 + bx + c = 0; a, b, c Î R, have a common root, then a : b : c
Now, the absolute difference of the cubes of roots is equal to (JEE Main 2013)
= | a3 - b3| = | a - b || a 2 + b 2 + ab | (a) 1 : 2 : 3 (b) 3 : 2 : 1 (c) 1 : 3 : 2 (d) 3 : 1 : 2
= ( a + b) 2 - 4ab |( a + b) 2 - ab| Sol. (a) Given equations are
= 9 - 4 | 9 - 1| = 8 5 x2 + 2x + 3 = 0 …(i)
2
and ax + bx + c = 0 …(ii)
Condition for Common Roots (s) Since, Eq. (i) has imaginary roots.
Consider two equations So, Eq. (ii) will also have both roots common as Eq. (i).
ax 2 + bx + c = 0 and a ¢ x 2 + b¢ x + c¢ = 0 a b c
Thus, = =
(a) When one common root Let a be the common 1 2 3
roots of two equations, then Hence, a : b : c is 1 : 2 : 3.
aa 2 + ba + c = 0 and a ¢ a 2 + b¢ a + c¢ = 0
On solving the two equations, we get Different Conditions for Roots
a2 a 1 (i) If b is of opposite sign as compared to a and c, then
= =
bc¢ - b¢ c a ¢ c - ac¢ ab¢ - a ¢ b both roots of ax 2 + bx + c = 0 are positive.
a ¢ c - ac¢ 2 bc¢ - b¢ c
Þ a= ,a = (ii) If a , b and c are all of same sign, then both roots of
ab¢ - a ¢ b ab¢ - a ¢ b
ax 2 + bx + c = 0 are negative.
Þ ( bc¢ - b¢ c)( ab¢ - a ¢ b) = ( a ¢ c - ac¢ )2
Quadratic Equations 111

(iii) If there is no term containing coefficient of x, then Possible Graph of Quadratic Expression
both the roots of the equation ax 2 + bx + c = 0 are
Let a quadratic expression is f ( x ) = ax 2 + bx + c ×
equal in magnitude but opposite in sign.
(iv) If a and c are of opposite signs, then both the roots of Here, two cases arise.
the equation are of opposite sign. Case I When a > 0
(v) If the roots of ax 2 + bx + c are reciprocal to each Y-axis Y-axis
other, then c = a.

Example 11. If a ¹ b, then the roots of the equation


2 ( a 2 + b 2) x 2 + 2 ( a + b) x + 1 = 0 are
X-axis X-axis
(a) real and distinct
(b) real and equal D=0 D<0
f(x) = 0 has real f(x) = 0 does not
(c) imaginary and equal roots. have real roots.
(d) None of th above f(x) ³ 0 for f(x) > 0 for
all x Î R all x Î R
Sol. (c) The given equation is 2 ( a2 + b 2) x2 + 2 ( a + b) x + 1 = 0
(a) (b)

Now, D = 4 ( a + b) 2 - 8( a2 + b 2) Y-axis

= - 4 ( a2 + b 2 - 2ab) = - 4 ( a - b) 2 < 0 (Q a - b ¹ 0)
Hence, the roots of the given equation are imaginary. X-axis
x=a O x=b

Quadratic Expression D>0


Roots of f(x) = 0 are
An expression of the form ax 2 + bx + c, where a , b, c Î R real and distinct.
and a ¹ 0 is called a quadratic expression in x. So, in f(x) > 0, if x < a or x > b
general quadratic expression is represented as (c) and f(x) < 0, if a < x < b
f ( x ) = ax 2 + bx + c Case II When a < 0
2 Y-axis Y-axis
or y = ax + bx + c
X-axis O X-axis
Graph of a Quadratic Expression D<0 D=0
f (x) = 0 does not f (x) = 0 has real
We have, y = ax 2 + bx + c = f ( x ) ( a ¹ 0) have real roots. and equal roots.
éæ bö D ù
2 f (x) < 0 for f (x) £ 0 for
Þ y = a êçx + ÷ - ú all x Î R all x Î R
êë è 2a ø 4a 2 úû
2 (d) (e)
æ Dö æ bö
or çy + ÷ = a çx + ÷ Y-axis
è 4a ø è 2a ø
x=a
D b x=b
Now, let y + = Y and x + =X O
X-axis
4a 2a
\ Y = aX 2
Y
Þ X2 = D>0
a Roots of f (x) = 0 are
(i) The shape of the curve y = f ( x ) is parabolic. real and distinct.
f (x) < 0, if x < a or x > b
(ii) The axis of parabola is X = 0
and f (x) > 0, if a < x < b
b
or x + = 0, i.e. parallel to y-axis.
2a Position of Roots of a Quadratic Equation
(iii) (a) If a > 0, then the parabola opens upwards.
Let f ( x ) = ax 2 + bx + c; a , b, c Î R, a ¹ 0 and a , b be the
(b) If a < 0, then the parabola opens downwards.
roots of f ( x ) = 0. Suppose k, k1 , k2 Î R and k1 < k2. Then, the
X-axis
following hold good
a>0
a<0 (i) Conditions for a number k
X-axis [If both the roots of f ( x ) = 0 are less than k]
112 JEE Main Mathematics

b, D (a) D ³ 0 (roots may be equal)


a>0 –
2a 4a
b f (k) (b) af ( k1 ) > 0
2a k (c) af ( k2 ) > 0
X-axis X-axis
a b k a b
b b
b, 2a f (k) (d) k1 < - < k2, where a £ b and k1 < k2.
– D 2a
2a 4a a<0
(a) D ³ 0 (roots may be equal) (vi) Conditions for numbers k1 and k2
b
(b) af ( k) > 0 (c) k > - , where a £ b [If k1 and k2 lie between the roots of f ( x ) = 0]
2a
f (k1) f (k2)
(ii) Conditions for a number k a>0
k2
a k X-axis
[If both the roots of f ( x ) = 0 are greater than k] k1 k2 1 k2 b
a X-axis
a>0 b, D b
– a<0
f (k) b 2a 4a f (k1) f (k2)
2a
k
k a b
X-axis a X-axis (a) D > 0 (b) af ( k1 ) < 0
b b
b, D
f (k) 2a (c) af ( k2 ) < 0, where a < b.

2a 4a a<0
Example 12. If both the roots of the quadratic equation
-b x 2 - mx + 4 = 0 are real and distinct and they lie in the interval
(a) D ³ 0 (b) af ( k) > 0 (c) > k where a £ b
2a [1, 5] then m lies in the interval (JEE Main 2019)
(iii) Conditions for a number k (a) (4, 5) (b) ( -5, - 4) (c) (5, 6) (d) (3, 4)
[If k lies between the roots of f ( x ) = 0] Sol. (a) According to given information, we have the
b, D following graph
a>0 –
2a 4a Y
f (k )
k X-axis
X-axis a
a b k b
f (k)
b, D
– a<0
2a 4a X
O 1 5
(a) D > 0
(b) af ( k) < 0 where, a < b Now, the following conditions should satisfy
(iv) Conditions for numbers k1 and k2 (i) D > 0 Þ b 2 - 4ac > 0
[If exactly one root of f ( x ) = 0 lies in the interval Þ m2 - 4 ´ 1 ´ 4 > 0
( k1 , k2 )]
Þ m2 - 16 > 0
a>0 f (k 2 )
f (k1)
k1
X-axis Þ (m - 4) (m + 4) > 0
f (k 1 ) a k2 b Þ m Î ( - ¥ , - 4) È ( 4, ¥ )
X-axis
k1 a k2 b
a<0
(ii) The vertex of the parabola should lie
f (k2) between x = 1and x = 5
b m
\ - Î (1, 5) Þ1 < < 5
(a) D > 0 2a 2
(b) f ( k1 ) f ( k2 ) < 0, where a < b Þ m Î(2, 10)
(iii) f(1) > 0 Þ1 - m + 4 > 0 Þ m < 5 Þ m Î ( -¥ , 5)
(v) Conditions for numbers k1 and k2
(iv) f(5) > 0 Þ 25 - 5m + 4 > 0 Þ 5m < 29
[If both roots of f ( x ) = 0 lies between k1 and k2 ]
æ 29 ö
Þ m Î ç - ¥, ÷
b, D è 5ø
a>0 –
2a 4a
f (k1) f (k2) From the values of m obtained in (i), (ii), (iii) and (iv), we
k1 k2 get m Î( 4, 5).
X-axis X-axis
k1 a b k2 a b
f (k1) f (k2)
b, D –¥ ¥
– a<0 –4 2 4 5 29/5
2a 4 a
Practice Exercise
ROUND I Topically Divided Problems

Roots of Quadratic Equation 2 2


8. How many roots of the equation x - =1-
x-1 x-1
1. If one root of the equation
have?
x 2 + (1 - 3i) x - 2 (1 + i) = 0 is -1 + i, then the other
(a) One (b) Two
root is
(c) Infinite (d) None of these
-1 - i
(a) -1 - i (b) (c) i (d) 2i
2 9. The number of real solutions of the equation
|x 2 + 4 x + 3|+ 2 x + 5 = 0 are
2. If the roots of the equation ax 2 + bx + c = 0 are of the (a) 1 (b) 2 (c) 3 (d) 4
k+1 k+ 2
form and , then ( a + b + c) 2 is equal to 4
10. The roots of the equation x - 8 x - 9 = 0 are 2
k k+1
(a) ± 1, ± i (b) ± 3, ± i
(a) b2 - 4ac (b) b2 - 2ac (c) ± 2 , ± i (d) None of these
(c) 2b2 - ac (d) Sa 2
11. If the roots of the equation 8 x 3 - 14 x 2 + 7 x - 1 = 0
3. If a and b are the roots of the equation
are in GP, then the roots are
ax 2 + bx + c = 0 and Sn = a n + b n , then 1 1
(a) 1, , (b) 2 , 4, 8
aSn + 1 + bSn + cSn - 1 is equal to 2 4
(a) 0 (b) abc (c) 3, 6, 12 (d) None of these
(c) a + b + c (d) None of these
12. If 2 + i 3 is a root of the equation x 2 + px + q = 0,
2
4. If a , b are the roots of ax + bx + c = 0 ( a ¹ 0) and where p and q are real, then ( p, q) is equal to
a + h, b + h are the roots of px 2 + qx + r = 0 ( p ¹ 0), (a) (- 4, 7) (b) (4, - 7)
then the ratio of the squares of their discriminants is (c) (4, 7) (d) (- 4, - 7)
(a) a 2 : p2 (b) a : p2 13. The number of real roots of 32 x
2
- 7x + 7
= 9 is
(c) a 2 : p (d) a : 2 p
(a) 0 (b) 2 (c) 1 (d) 4
5. If one real root of the quadratic equation
14. Rational roots of the equation
81x 2 + kx + 256 = 0 is cube of the other root, then a
2 x 4 + x 3 - 11x 2 + x + 2 = 0 are
value of k is (JEE Main 2019) 1 1 1
(a) and 2 (b) ,2 , , - 2
(a) 100 (b) 144 2 2 4
(c) - 81 (d) - 300 1 1 3
(c) , 2 , 3, 4 (d) , 2 , , - 2
6. The roots of the equation 2 2 4
|2 x - 1|2 - 3|2 x - 1|+ 2 = 0 are 15. The roots of the given equation
ì 1 1ü
(a) í - , 0, ý
ì 1 3ü
(b) í - , 0, ý
( p - q) x 2 + ( q - r) x + ( r - p) = 0 are
î 2 2þ î 2 2þ p-q q-r r-p q-r
(a) , 1 (b) ,1 (c) ,1 (d) 1,
ì 3 1 ü ì 1 3ü r-p p-q p-q p-q
(c) í - , , 0, 1ý (d) í - , 0, 1, ý
î 2 2 þ î 2 2þ p
16. If 2 sin 2 is a root of the equation x 2 + ax + b = 0,
7. The number of real roots of the equation 8
where a and b are rational numbers, then a - b is
esin x - e- sin x - 4 = 0 are
equal to
(a) 1 (b) 2 5 3 1 1
(c) infinite (d) None of these (a) - (b) - (c) - (d)
2 2 2 2
114 JEE Main Mathematics

17. tan a and tan b are the roots of the equation 26. If the roots of
2
x + ax + b = 0, then the value of ( a 2 + b2 ) x 2 - 2 ( bc + ad) x + c2 + d 2 = 0 are equal,
then
sin 2 (a + b) + a sin (a + b) cos (a + b) + b cos2 (a + b) a c a b
is equal to (a) = (b) + =0
b d c d
a a b
(a) ab (b) b (c) (d) a (c) = (d) a + b = c + d
b d c
18. The coefficient of x in the equation x 2 + px + q = 0 27. Let f ( x) = x 2 + ax + b; a, b Î R. If
was taken as 17 in place of 13 its roots were found f (1) + f ( 2) + f (3) = 0, then the roots of the equation
to be - 2 and -15. The roots of the original equation f ( x) = 0
are (a) are imaginary
(a) 3, 10 (b) -3, - 10 (b) are real and equal
(c) -5, - 8 (d) None of these (c) are from the set {1, 2, 3}
(d) real and distinct
19. Two students while solving a quadratic equation in
x, one copied the constant term incorrectly and got 28. If sin a , sin b and cosa are in GP, then roots of
the roots 3 and 2. The other copied the constant x 2 + 2 x cot b + 1 = 0 are always
term coefficient of x 2 correctly as -6 and 1 (a) real (b) real and negative
respectively the correct roots are (c) greater than one (d) non-real
(a) 3, - 2 (b) - 3, 2 (c) - 6, - 1 (d) 6, - 1
29. If one root of equation x 2 + ax + 12 = 0 is 4 while
3 2
20. If a, b, g are the roots of x + 2 x - 3x - 1 = 0, then the equation x 2 + ax + b = 0 has equal roots, then
-2 -2 -2
a +b +g is equal to the value of b is
(a) 12 (b) 13 (c) 14 (d) 15 4 49 7 4
(a) (b) (c) (d)
49 4 4 7
21. If the roots of the given equation
(cos p - 1) x 2 + (cos p) x + sin p = 0 are real, then 30. The solution set of the equation
æ p pö pqx 2 - ( p + q) 2 x + ( p + q) 2 = 0 is
(a) p Î (- p , 0) (b) p Î ç - , ÷
è 2 2ø
ì p qü ì pü
(a) í , ý (b) í pq, ý
(c) p Î (0, p ) (d) p Î ( 0, 2p ) î q pþ î q þ
ìq ü ì p + q p + qü
Nature of Roots (c) í , pqý (d) í , ý
îp þ î p q þ
22. The number of all possible positive integral values
of a for which the roots of the quadratic equation, 31. If sin a and cos a are the roots of the equation
6 x 2 - 11x + a = 0 are rational numbers is ax 2 + bx + c = 0, then
(JEE Main 2019) (a) a 2 - b 2 + 2 ac = 0
(a) 5 (b) 2 (c) 4 (d) 3 (b) (a - c) 2 = b 2 + c 2
23. The number of integral values of m for which the (c) a 2 + b 2 - 2 ac = 0
(d) a 2 + b 2 + 2 ac = 0
quadratic expression, (1 + 2 m) x 2 - 2 (1 + 3m)
x + 4 (1 + m), x Î R, is always positive, is 32. If a, b are the roots of x 2 + px + 1 = 0, and c, d are
(JEE Main 2019) the roots of x 2 + qx + 1 = 0, the value of
(a) 6 (b) 8 (c) 7 (d) 3 E = ( a - c) ( b - c) ( a + d) ( b + d) is
24. For a positive integer n, if the quadratic equation, (a) p2 - q2 (b) q2 - p2
x ( x + 1) + ( x + 1) ( x + 2) + . . . (c) q2 + p2 (d) None of these
+ ( x + n - 1) ( x + n) = 10 n has two consecutive 33. If the product of the roots of the equation
integral solutions, then n is equal to (JEE Main 2016) ( a + 1) x 2 + ( 2 a + 3) x + (3a + 4) = 0 is 2, then the
(a) 12 (b) 9 (c) 10 (d) 11 sum of roots is
2 2 (a) 1 (b) -1 (c) 2 (d) -2
25. If P ( x) = ax + bx + c and Q ( x) = - ax + dx + c,
2
where ac ¹ 0, then P ( x) Q ( x) = 0 has atleast 34. If [ x ] = [ x + 2 ] , where [ x ] = the greatest integer
(a) four real roots (b) two real roots less than or equal to x, then x must be such that
(c) four imaginary roots (d) None of these (a) x = 2, - 1 (b) [-1, 0) È [2, 3)
(c) x Î [-1, 0) (d) None of these
Quadratic Equations 115

35. If a , b and g are the roots of the equation (a) both negative (b) both positive
1 1 1 (c) both real
x 3 - 7 x + 7 = 0, then + + is (d) one negative and one positive
a 4 b4 g 4
(a) 7/3 (b) 3/7 (c) 4/7 (d) 7/4 46. If a > 0, b > 0, c > 0, then both the roots of the
a b equation ax 2 + bx + c = 0
36. If the roots of the equation + = 1 be (a) are real and negative (b) have negative real part
x-a x-b
(c) are rational numbers (d) None of these
equal in magnitude but opposite in sign, then a + b
is equal to 47. If ( ax 2 + c) y + ( a ¢ x 2 + c¢ ) = 0 and x is a rational
(a) 0 (b) 1 function of y and ac is negative, then
a c
(c) 2 (d) None of these (a) ac¢ + a ¢ c = 0 (b) =
a ¢ c¢
1 1 1
37. If the roots of the equation + = are (c) a 2 + c2 = a ¢ 2 + c¢ 2 (d) aa ¢ + cc¢ = 1
x+ p x+q r
48. If roots of the equation ( a - b) x 2 + ( c - a) x + ( b - c) = 0
equal in magnitude but opposite in sign, then the are equal, then a, b and c are in
product of the roots will be (a) AP (b) HP
p2 + q 2 ( p2 + q 2 ) p2 - q 2 ( p2 - q 2 ) (c) GP (d) None of these
(a) (b) - (c) (d) -
2 2 2 2
49. If the roots of the equation ( p 2 + q2 ) x 2 - 2 q( p + r) x
38. The least value of|a|for which tan q and cot q are + ( q2 + r 2 ) = 0 be real and equal, then p, q and r will
2
roots of the equation x + ax + 1 = 0, is be in
(a) 2 (b) 1 (c) 1/2 (d) 0 (a) AP (b) GP
(c) HP (d) None of these
39. The harmonic mean of the roots of the equation
(5 + 2 ) x 2 - ( 4 + 5 ) x + 8 + 2 5 = 0 is 50. Let a and b be the roots of the equation
(a) 2 (b) 4 (c) 6 (d) 8 a
x 2 - px + r = 0 and , 2 b be the roots of the
40. If a + b + c = 0, then the roots of the equation 2
equation x 2 - qx + r = 0. Then, the value of r is
4 ax 2 + 3bx + 2 c = 0 are
2 2
(a) equal (b) imaginary (a) ( p - q)(2q - p) (b) (q - p)(2 p - q)
9 9
(c) real (d) None of these
2 2
(c) (q - 2 p)(2q - p) (d) (2 p - q)(2q - p)
41. The integer ‘k’, for which the inequality 9 9
x 2 - 2(3k - 1)x + 8k 2 - 7 > 0 is valid for every x in R is
51. Let a, b, c be in arithmetic progression. Let the
(JEE Main 2021)
(a) 3 (b) 2 (c) 4 (d) 0 centroid of the triangle with vertices ( a, c), ( 2, b) and
( a, b) be æç , ö÷. If a , b are the roots of the equation
10 7
42. If x 2 + 2 x + 2 xy + my - 3 = 0 has two rational è 3 3ø
factors, then the values of m will be ax 2 + bx + 1 = 0, then the value of a 2 + b 2 - ab is
(a) - 6, - 2 (b) - 6, 2 (c) 6, - 2 (d) 6, 2 (JEE Main 2021)
71 69 69 71
2
43. If the roots of the equation qx + px + q = 0 are (a) (b) - (c) (d) -
256 256 256 256
complex, where p and q are real, then the roots of
52. Let a and b be the roots of equation px 2 + qx + r = 0
the equation x 2 - 4 qx + p 2 = 0 are
(a) real and unequal (b) real and equal
1 1
(c) imaginary (d) None of these p ¹ 0. If p, q and r, n AP and + = 4, then the
a b
44. If a < b < c < d, then the roots of the equation
value of|a - b|is
( x - a)( x - c) + 2 ( x - b)( x - d) = 0 are 61 2 17 34 2 13
(a) real and distinct (b) real and equal (a) (b) (c) (d)
9 9 9 9
(c) imaginary (d) None of these
53. If a, b and c are the sides of DABC such that
45. If the roots of the equation x 2 + px + q = 0 are a and a ¹ b ¹ c and x 2 - 2 ( a + b + c) x + 3l
2
b and roots of the equation x - xr + s = 0 are ( ab + bc + ca) = 0 has real roots, then
a 4 and b 4 , then the roots of the equation 4 5 æ4 5ö æ1 5ö
(a) l < (b) l > (c) l Î ç , ÷ (d) l Î ç , ÷
x 2 - 4 qx + 2 q2 - r = 0 are 3 3 è3 3ø è3 3ø
116 JEE Main Mathematics

Formation of Quadratic Equation 62. If atleast one root of 2 x 2 + 3x + 5 = 0 and


54. Let a , b be the roots of x 2 - 2 x cos f + 1 = 0, then the ax 2 + bx + c = 0, a, b, c Î N is common, then the
equation whose roots are a n and b n , is maximum value of a + b + c is
(a) x 2 - 2x cos nf - 1 = 0 (b) x 2 - 2x cos nf + 1 = 0 (a) 10 (b) 0
(c) x 2 - 2x sin nf + 1 = 0 (d) x 2 + 2x sin nf - 1 = 0 (c) does not exist (d) None of these

55. If a and b be the roots of the equation 63. If the equations 2 ax 2 - 3bx + 4 c = 0 and
2 x 2 + 2 ( a + b) x + a 2 + b2 = 0, then the equation 3x 2 - 4 x + 5 = 0 have a common root, then
whose roots are (a + b) 2 and (a - b) 2 , is ( a + b)/( b + c) is equal to ( a, b, c Î R)
(a) x 2 - 2ab x - (a 2 - b 2)2 = 0 (a) 1/2 (b) 3/35 (c) 34/31 (d) 29/31
(b) x 2 - 4abx - (a 2 - b 2) 2 = 0 64. A value of b for which the equations x 2 + bx - 1 = 0,
(c) x 2 - 4abx + (a 2 - b 2) 2 = 0
x 2 + x + b = 0 have one root in common, is
(d) None of the above
(a) - 2 (b) - i 3 (c) i 5 (d) 2
56. Let a and a 2 be the roots of x 2 + x + 1 = 0, then the
equation whose roots are a 31 and a 62 , is Quadratic Expression and
(a) x 2 - x + 1 = 0 (b) x 2 + x - 1 = 0 Position of Roots
(c) x 2 + x + 1 = 0 (d) x 60 + x 30 + 1 = 0 65. The values of ‘a’ for which ( a 2 - 1) x 2 + 2 ( a - 1) x + 2
57. If a and b are the roots of the equation is positive for any x, are
2
ax + bx + c = 0, then the equation whose roots are (a) a ³ 1 (b) a £ 1
1 1 (c) a > - 3 (d) a < - 3 or a > 1
a + and b + , is
b a 66. If the roots of the equation x 2 - 2 ax + a 2 + a - 3 = 0
2 2
(a) ac x + (a + c) bx + (a + c) = 0 are real and less than 3, then
(b) ab x 2 + (a + c) bx + (a + c) 2 = 0
(a) a < 2 (b) 2 £ a £ 3
(c) ac x 2 + (a + b)cx + (a + c) 2 = 0
(c) 3 < a £ 4 (d) a > 4
(d) None of the above
67. If a and b be the roots of the quadratic equation
58. The equation whose roots are the square of the
roots of the equation 2 x 2 + 3x + 1 = 0, is ax 2 + bx + c = 0 and k be a real number, then the
condition, so that a < k < b is given by
(a) 4x2 + 5x + 1 = 0 (b) 4x2 - x + 1 = 0
(c) 4x2 - 5x - 1 = 0 (d) 4x2 - 5x + 1 = 0 (a) ac > 0 (b) ak2 + bk + c = 0
(c) ac < 0 (d) a 2k2 + abk + ac < 0
Condition for Common Roots 68. The values of a for which
59. If atleast one root of the equation 2 x 2 - 2 ( 2 a + 1) x + a ( a + 1) = 0 may have one root
3 2
x + ax + bx + c = 0 remains unchanged, when a, b less than a and other root greater than a are given
and c are decreased by one, then which one of the by
following is always a root of the given equation ? (a) 1 > a > 0 (b) - 1 < a < 0
(a) 1 (c) a ³ 0 (d) a > 0 or a < - 1
(b) -1
(c) w , an imaginary cube root of unity 69. One lies between the roots of the equation
(d) i - x 2 + ax + a = 0, a Î R if and only if a lies in the
interval
60. If the equation 2 x 2 + 3x + 5 l = 0 and x 2 + 2 x + 3l = 0 æ1 ö é 1 ö
(a) ç , ¥ ÷ (b) ê - , ¥ ÷
have a common root, then l is equal to è2 ø ë 2 ø
(a) 0 (b) -1 (c) 0, - 1 (d) 2 , - 1 æ 1ö æ 1ù
(c) ç - ¥ , ÷ (d) ç - ¥ , ú
è 2ø è 2û
61. If each pair of the equation
x 2 + ax + b = 0, x 2 + bx + c = 0 and x 2 + cx + a = 0 70. If a and b (a < b) are the roots of the equation
has a common root, then product of all common x 2 + bx + c = 0, where c < 0 < b, then
roots is (a) 0 < a < b (b) a < 0 < b < |a|
(a) abc (b) 2 abc (c) a < b < 0 (d) a < 0 < |a |< b|
(c) ab + bc + ca (d) 2 ab + bc + ca
Quadratic Equations 117

ROUND II Mixed Bag


Only One Correct Option 8. The sum of all real values of x satisfying the
2
+ 4 x - 60
1. The root of the equation equation ( x 2 - 5 x + 5) x = 1 is (JEE Main 2016)
2
2 (1 + i) x - 4 ( 2 - i) x - 5 - 3 i = 0, where i = -1, (a) 3 (b) - 4 (c) 6 (d) 5
which has greater modulus, is
3 -5i 5 -3i 3+ i 3i + 1
9. If a and b be the roots of x 2 + px + q = 0, then
(a) (b) (c) (d)
2 2 2 2 ( wa + w2b)( w2a + wb)
is equal to
2. If a and b are the roots of ax 2 + bx + c = 0, then the a 2 b2
+
equation ax 2 - bx ( x - 1) + c( x - 1) 2 = 0 has roots b a
a b 1 - a 1 -b q p
(a) , (b) , (a) - (b) a b (c) -
(d) w
1 - a 1 -b a b p q
a b a+1 b+1 (w and w2 are complex cube roots of unity)
(c) , (d) ,
a+1 b+1 a b 10. In writing an equation of the form ax 2 + bx + c = 0;
3. Let a and b be the roots of the equation the coefficient of x is written incorrectly and roots
x 2 - x - 1 = 0. If pk = (a ) k + (b) k, k ³ 1, then which are found to be equal. Again, in writing the same
one of the following statements is not true? equation the constant term is written incorrectly
(JEE Main 2020) and it is found that one root is equal to those of the
(a) p3 = p5 - p4 previous wrong equation while the other is double
(b) ( p1 + p2 + p3 + p4 + p5 ) = 26 of it. If a and b be the roots of correct equation,
(c) p5 = p2. p3 then (a - b) 2 is equal to
(d) p5 = 11 (a) 5 (b) 5 a b (c) - 4 a b (d) - 4
4. If a and b are the roots of the equation 2
11. If the equation ax + 2 bx + 3c = 0 and
2 x ( 2 x + 1) = 1, then b is equal to (JEE Main 2020) 3x 2 + 8 x + 15 = 0 have a common root, where
(a) 2a (a + 1) (b) -2a (a + 1) a, b and c are the lengths of the sides of a DABC,
(c) 2a (a - 1) (d) 2a 2 then sin 2 A + sin 2 B + sin 2 C is equal to
3
5. If a and b be two roots of the equation x 2 - 64 x (a) 1 (b) (c) 2 (d) 2
2
3 1/ 8 3 1/ 8
æa ö æb ö
+ 256 = 0. Then, value of ç 5 ÷ + ç 5÷ is 12. The sum of the solutions of the equation
èb ø èa ø
| x - 2|+ x ( x - 4) + 2 = 0 ( x > 0) is equal to
(JEE Main 2020)
(JEE Main 2019)
(a) 2 (b) 3 (c) 1 (d) 4
(a) 9 (b) 12 (c) 4 (d) 10
- 1+ i 3 100 100
6. Let a = . If a = (1 + a ) å a 2 k & b= å a 3k, 13. The number of real roots of the equation
2 k= 0 k= 0
5 + |2 x - 1|= 2 x ( 2 x - 2) is (JEE Main 2019)
then a and b are the roots of the quadratic equation (a) 1 (b) 3 (c) 4 (d) 2
(JEE Main 2020)
(a) x2 + 101x + 100 = 0 (b) x2 + 102x + 101 = 0 14. If a and b are the roots of the equation
(c) x2 - 102x + 101 = 0 (d) x2 - 101x + 100 = 0 1 1
x 2 + px + 2 = 0 and and are the roots of the
a b
7. If a and b are the roots of the quadratic equation,
equation 2 x 2 + 2 qx + 1 = 0, then
p
x + x sin q - 2 sin q = 0, q Î æç0, ö÷, then
2
æa - 1 ö æ 1ö æ 1ö æ 1ö
è 2ø ç ÷ çb - ÷ ça + ÷ çèb + ÷ø is equal to
è aø è bø è bø a
a 12 + b12 (JEE Main 2020)
-12
is equal to
(a + b -12 )(a - b) 24 (JEE Main 2019)
9 9 9 9
(a) (9 + q2) (b) (9 - q2) (c) (9 + p2) (d) (9 - p2)
4 4 4 4
212 26
(a) (b) 15. If the roots of the equation ax 2 - bx + c = 0 are
(sin q + 8)12 (sin q + 8)12
212 212 a and b, then the roots of the equation
(c) (d)
(sin q - 4)12 (sin q - 8)6 b2 cx 2 - ab2 x + a 3 = 0 are
118 JEE Main Mathematics

1 1 1 1 (a) x2 - 2x + 2 = 0 (b) x2 - 2x + 8 = 0
(a) , (b) ,
a + ab b3
3
+ ab a + ab b 2 + ab
2
(c) x2 - 2x + 136 = 0 (d) x2 - 2x + 16 = 0
1 1
(c) 4 , (d) None of these Numerical Value Types Questions
a + ab b 4 + ab
23. Let a and b be two real numbers such that a + b = 1
16. If a, b Î R, a ¹ 0 and the quadratic equation
2 and ab = - 1. Let Pn = (a ) n + (b) n , Pn - 1 = 11 and
ax - bx + 1 = 0 has imaginary roots, then ( a + b + 1)
Pn + 1 = 29 for some integer n ³ 1. Then, the value of
is
Pn2 is (JEE Main 2021)
(a) positive (b) negative
(c) zero (d) dependent on the sign of b 24. The least positive value of ‘a’ for which the equation,
2 33
17. If roots of x - ( a - 3) x + a = 0 are such that atleast 2 x 2 + ( a + 10) x + = 2 a has real roots is ……… .
2 (JEE Main 2020)
one of them is greater than 2, then
(a) a Î [7, 9] (b) a Î [7, ¥ ) 25. Let a and b be the roots of equation x 2 - 6 x - 2 = 0.
(c) a Î [9, ¥ ) (d) a Î [7, 9)
a10 - 2 a8
If an = a n - b n , for n ³ 1, then the value of
18. All the values of m for which both the roots of the 2 a9
equation x 2 - 2 mx + m 2 - 1 = 0 are greater than –2
is equal to
but less than 4, lie in the interval
(a) -2 < m < 0 (b) m > 3 26. Let a and b be the solutions of the quadratic
(c) -1 < m < 3 (d) 1 < m < 4 1 1
equation x 2 - 1154 x + 1 = 0, then a +b
19. If the roots of the quadratic equation 4 4
( 4 p - p 2 - 5) x 2 - ( 2 p - 1) x + 3 p = 0 lie on either side is equal to
of unity, then the number of integral values of p is 27. If ( a 2 - 149 + 13) x 2 + ( a + 2) x - 2 = 0 does not have
(a) 1 (b) 2 (c) 3 (d) 4 two distinct real roots, then maximum value of
20. The range of a for which the equation x 2 + ax - 4 = 0 a 2 - 15 a is k, then|k|is equal to
has its smaller root in the interval ( -1, 2) is
28. If the equation x 2 + 2 ( l + 1) x + l2 + l + 7 = 0 has
(a) (-¥ , - 3) (b) (0, 3)
only negative roots, then least value of l equals
(c) (0, ¥ ) (d) (-¥ , - 3) È (0, ¥ )
29. If a , b be the roots x 2 + px - q = 0 and g , d be the
21. For x 2 - ( a + 3)|x |+ 4 = 0 to have real solutions,
then the range of a is roots of x 2 + px + r = 0, q + r ¹ 0, then
(a - g ) (a - d)
(a) (-¥ , - 7] È [1, ¥ ) (b) (-3, ¥ ) is equal to ............ .
(b - g ) (b - d)
(c) (-¥ , - 7] (d) [1, ¥ )
22. Let p and q be two positive number such that 30. The number of solutions for the equation
p + q = 2 and p 4 + q4 = 272. Then, p and q are roots log 4 ( 2 x 2 + x + 1) - log 2 ( 2 x - 1) = 1, is ............ .
of the equation (JEE Main 2021)

Answers
Round I
1. (d) 2. (a) 3. (a) 4. (a) 5. (d) 6. (d) 7. (d) 8. (d) 9. (b) 10. (b)
11. (a) 12. (a) 13. (b) 14. (a) 15. (c) 16. (a) 17. (b) 18. (b) 19. (d) 20. (b)
21. (c) 22. (d) 23. (c) 24. (d) 25. (b) 26. (a) 27. (d) 28. (a) 29. (b) 30. (d)
31. (a) 32. (b) 33. (b) 34. (b) 35. (b) 36. (a) 37. (b) 38. (a) 39. (b) 40. (c)
41. (a) 42. (c) 43. (a) 44. (a) 45. (c) 46. (b) 47. (b) 48. (a) 49. (b) 50. (d)
51. (d) 52. (d) 53. (a) 54. (b) 55. (b) 56. (c) 57. (a) 58. (d) 59. (c) 60. (c)
61. (a) 62. (c) 63. (c) 64. (b) 65. (d) 66. (a) 67. (d) 68. (d) 69. (a) 70. (b)

Round II
1. (a) 2. (c) 3. (c) 4. (b) 5. (a) 6. (c) 7. (a) 8. (a) 9. (a) 10. (b)
11. (d) 12. (d) 13. (a) 14. (d) 15. (b) 16. (a) 17. (c) 18. (c) 19. (b) 20. (a)
21. (d) 22. (d) 23. (324) 24. (8) 25. (3) 26. (6) 27. (9) 28. (6) 29. (1) 30. (1)
Solutions
Round I b c
\a+b=- and ab =
a a
1. Let another root of equation
q r
x2 + (1 - 3i )x - 2(1 + i ) = 0. is a and a + h + b + h = - , (a + h ) (b + h ) =
p p
\ a + (-1 + i ) = - (1 - 3i )
Now, ( a + h ) - ( b + h ) = a - b
Þ a = 2i
Þ [( a + h ) - ( b + h )] 2 = ( a - b )2
k+1 k+2 b
2. We have, + =- …(i) Þ [( a + h ) + ( b + h )]2 - 4 ( a + h ) (b + h )
k k+1 a
= (a + b )2 - 4ab
k+1 k+2 c
and × = … (ii) q 2
4r b2 4c
k k+1 a Þ - = -
p2 p a2 a
From Eq. (i),
q2 - 4 pr b2 - 4ac
1 1 b Þ =
1 + +1 + =- p2 a2
k k+1 a
1 1 b b2 - 4ac a 2
Þ 2+ + =- … (iii) \ =
k k+1 a q2 - 4 pr p2

From Eq. (ii), Hence, the ratio of the squares of their discriminants
k+2 c 2 c is a 2 : p2.
= Þ1 + =
k a k a 5. Given quadratic equation is
2 c 2a 81x2 + kx + 256 = 0
Þ = -1Þk =
k a c-a Let one root be a, then other is a3 .
Now, on substituting the value of k in Eq. (iii), we get k 256
c-a 1 b Now, a + a3 = - and a × a3 =
2+ + =- 81 81
2a 2 a a b
+1 [Q for ax2 + bx + c = 0, sum of roots = -
c-a a
c-a c-a b cù
Þ 2+ + =- and product of roots = ú
2a a+c a aû
2 (2a ) (a + c) + (c - a ) (c + a ) + 2a (c - a ) b æ4ö
4
4
Þ =- Þ a4 = ç ÷ Þ a=±
2a (a + c) a è3ø 3
Þ a 2 + c2 + 6ac = - 2ab - 2bc \ k = - 81 (a + a3 ) = - 81 a (1 + a 2)
2 2 2 2
Þ a + b + c + 2ab + 2bc + 2ca = b - 4ac æ 4ö æ 16 ö
\ (a + b + c)2 = b2 - 4ac = - 81 ç ± ÷ ç1 + ÷ = ± 300
è 3ø è 9ø
3. Given, a and b are the roots of equation ax2 + bx + c = 0 6. Given equation is
and S n = a n + b n. |2x - 1|2 - 3|2x - 1| + 2 = 0
b c Let |2x - 1| = t
\ a+b=- and ab =
a a \ t2 - 3 t + 2 = 0
Now, S n + 1 = a n + 1 + b n + 1 Þ (t - 1 )(t - 2) = 0 Þ t = 1, 2
= a n + 1 + b n + 1 + a nb + b na - a nb - b na Þ |2x - 1| = 1 and |2x - 1| = 2
= a n (a + b ) + b n (a + b ) - ab (a n - 1 + b n-1 ) Þ 2x - 1 = ± 1 and 2x - 1 = ± 2
= (a + b ) (a n + b n ) - ab (a n - 1 + b n -1 ) 3 1
Þ x = 1, 0 and x = , -
b c 2 2
= - S n - S n -1
a a 7. Given equation is esin x - e- sin x - 4 = 0
- bS n - cS n - 1
Þ Sn + 1 = Let esin x = y, then given equation can be written as
a
y2 - 4 y - 1 = 0
\ aS n + 1 + bS n + cS n - 1 = 0
Þ y=2± 5
4. Given, a and b are the roots of ax2 + bx + c = 0 and But the value of y = esin x is always positive,
a + h and b + h are the roots of px2 + qx + r = 0. so we take only
120 JEE Main Mathematics

y=2 + 5 Now, sum of the roots = (2 + i 3 ) + (2 - i 3 ) = - p


Þ log e y = log e (2 + 5 ) Þ 4=- p
Þ sin x = log e (2 + 5 ) > 1
Product of roots = (2 + i 3 )(2 - i 3 ) = q
Which is impossible since sin x cannot be greater
than 1. Þ 7=q

Hence, we cannot find any real value of x which satisfies Hence, ( p, q) = (- 4, 7 )


2x 2 - 7x + 7
the given equation. 13. Given that, 3 2
= 3 Þ 2x2 - 7x + 7 = 2
2 2
8. We have, x - =1 - . If x ¹ 1 multiplying each Þ 2 x2 - 7 x + 5 = 0
x-1 x-1
term by (x - 1 ), the given equation reduces to Now, D = b2 - 4ac
x (x - 1 ) = (x - 1 ) or (x - 1 )2 = 0 or x = 1 which is not = (-7)2 - 4 ´ 2 ´ 5
possible as considering x ¹ 1.
= 49 - 40 = 9 > 0
Thus, given equation has no root.
Hence, it has two real roots.
9. We have,
| x2 + 4x + 3| + 2x + 5 = 0 14. Given equation can be reduced to a quadratic equation.
1 2
Here, two cases arise. \ 2x2 + x - 11 +
+ =0
x x2
Case I When x2 + 4x + 3 > 0 æ 1ö æ 1ö
Þ 2 ç x2 + 2 ÷ + ç x + ÷ - 11 = 0
Þ x2 + 4x + 3 + 2x + 5 = 0 è x ø è xø
Þ x2 + 6x + 8 = 0 1
Put x + = y
Þ (x + 2)(x + 4) = 0 x
Þ x = - 2 , -4 2( y2 - 2) + y - 11 = 0
2
x = - 2 is not satisfying the condition x + 4x + 3 > 0. Þ 2 y2 + y - 15 = 0
So, x = - 4 is the only solution of the given equation. 5
Þ y = - 3 and
Case II When x2 + 4x + 3 < 0 2
1 1 5
Þ - (x2 + 4x + 3) + 2x + 5 = 0 Þ x + = - 3, x + =
x x 2
Þ - x2 - 2 x + 2 = 0 Þ x2 + 3 x + 1 = 0 , 2 x 2 - 5 x + 2 = 0
2
Þ x + 2x - 2 = 0
Only 2nd equation has rational roots as D = 9 and
Þ (x + 1 + 3 )(x + 1 - 3 ) = 0 1
rootsare and 2.
Þ x = - 1 + 3, - 1 - 3 2

Hence, x = - (1 + 3 ) satisfy the given condition. 15. Given equation is


2
Since, x + 4x + 3 < 0 while x = - 1 + 3 is not satisfying ( p - q)x2 + (q - r )x + (r - p) = 0
the condition. Thus, number of real solutions are two. (r - q) ± (q - r )2 - 4(r - p)( p - q)
Þ x=
10. Q x4 - 8x2 - 9 = 0 2 ( p - q)

Þ x4 - 9x2 + x2 - 9 = 0 q2 + r 2 - 2qr - 4(rp - rq - p2 + pq)


(r - q) ±
=
2( p - q)
Þ x2(x2 - 9) + 1 (x2 - 9) = 0
(r - q) ± (q + r - 2 p)
Þ (x2 + 1 )(x2 - 9) = 0 Þ x=
2( p - q)
Þ x = ± i, ± 3 r-p
Þ x= ,1
11. Since, the roots of the equation 8x 3 - 14x2 + 7x - 1 = 0 p-q
a p p 2 -1
are in GP. Let the roots be , a , ab , b ¹ 0. Then, the 16. Q2 sin 2 = 1 - cos = 1 -
1
= (irrational root)
b 8 4 2 2
1
product of roots is a3 = 2+1
8 So, other root is .
2
1 1
Þ a = and hence, b = × 1 1
2 2 Sum of roots = - a = 1 - +1+ =2Þa = -2
2 2
1 1
So, roots are 1, , . 1 1
2 4 Product of roots = 1 - = = b
2 2
12. Since, 2 + i 3 is a root of equation x2 + px + q = 0. 1 5
So, a - b = -2 - = -
Therefore, 2 - i 3 will be other root. 2 2
Quadratic Equations 121

17. Since, tan a and tan b are the roots of the equation Now, (1 - sin p) ³ 0 for all real p and sin p > 0 for
2
x + ax + b = 0, then 0 < p < p.
a Therefore, 4 sin p (1 - sin p) ³ 0 when 0 < p < p or
tan a + tan b = - p Î (0, p ).
1
and tan a × tan b = b 22. For the roots of quadratic equation ax2 + bx + c = 0
sin a sin b a to be rational D = (b2 - 4ac) should be perfect square.
Þ + =-
cos a cos b 1 In the equation 6x2 - 11x + a = 0
sin a sin b a = 6, b = - 11 and c = a
and =b
cos a cos b \For roots to be rational
\ sin 2 (a + b ) + a sin (a + b ) cos (a + b ) + b cos 2 (a + b ) D = (- 11)2 - 4(6) (a) should be a perfect square.
= cos 2 (a + b ) [tan 2 (a + b ) + b + a tan (a + b )] Þ D(a) = 121 - 24a should be a perfect square
tan 2 (a + b ) + b + a tan (a + b ) Now, D(1) = 121 - 24 = 97 is not a perfect square.
=
1 + tan 2 (a + b ) D(2) = 121 - 24 ´ 2 = 73 is not a perfect square.
a æ a ö D(3) = 121 - 24 ´ 3 = 49 is a perfect square.
ça + ÷+b
b -1 è b - 1ø D(4) = 121 - 24 ´ 4 = 25 is a perfect square.
= =b
a2 D(5) = 121 - 24 ´ 5 = 1 is a perfect square.
1+
(b - 1 )2 and for a ³ 6, D(a) < 0, hence imaginary roots.
18. Let the equation (incorrectly written form) be \ For 3 values of a (a = 3, 4, 5), the roots are rational.
2
x + 17x + q = 0 23. The quadratic expression ax2 + bx + c, x Î R is always
Since, roots are -2 , - 15 . positive, if a > 0 and D < 0.
\ q = 30
So, the quadratic expression
So, correct equation is x2 + 13x + 30 = 0
(1 + 2m) x2 - 2 (1 + 3m)x + 4(1 + m), x Î R will be always
Þ x2 + 10x + 3x + 30 = 0
positive, if
Þ (x + 3) (x + 10) = 0 1 + 2m > 0 …(i)
Þ x = - 3, - 10 and D = 4(1 + 3m)2 - 4(2m + 1) 4(1 + m) < 0 …(ii)
2
19. Let the correct equation be ax + bx + c = 0 and the From inequality Eq. (i), we get
correct roots are a and b. Taking c wrong, the roots are 1
m>- …(iii)
3 and 2. 2
\ a+b =3+2 =5 …(i) From inequality Eq. (ii), we get
Also, a = 1 and c = - 6 1 + 9m2 + 6m - 4 (2m2 + 3m + 1) < 0
c Þ m2 - 6m - 3 < 0
\ ab = = - 6 …(ii)
a
Þ [m - (3 + 12 )][m - (3 - 12 )] < 0
On solving Eqs. (i) and (ii), the correct roots are 6 and 6 ± 36 + 12
-1. [Q m2 - 6m - 3 = 0 Þ m = = 3 ± 12]
2
20. Here, a + b + g = -2 …(i) Þ 3 - 12 < m < 3 + 12 …(iv)
ab + bg + ga = - 3 …(ii) From inequalities Eqs. (iii) and (iv), the integral
and abg = 1 …(iii) values of m are 0, 1, 2, 3, 4, 5, 6.
On squaring Eq. (ii), we get Hence, the number of integral values of m is 7.
2 2 2 2 2 2
a b + b g + g a + 2 abg (a + b + g ) = 9 24. Given, quadratic equation is
Þ a 2 b 2 + b 2 g 2 + g 2a 2 = 9 - 2 (1 ) (-2) = 13 x(x + 1) + (x + 1)(x + 2)+ ...+ (x + n - 1) (x + n ) =10n
b 2 g 2 + g 2a 2 + a 2 b 2 13 Þ (x2 + x2 + ... + x2) + [(1 + 3 + 5 + ... + (2n - 1)]x
Now, a -2 + b -2 + r -2 = = = 13
(abg )2 1 + [(1 × 2 + 2 × 3 + ... + (n - 1)n ] = 10n
2
21. Given equation is (cos p - 1 )x + (cos p)x + sin p = 0 n (n 2 - 1)
Þ nx2 + n 2x + - 10n = 0
3
Since, roots are real, its discriminant, D ³ 0
n2 - 1
\ cos 2 p - 4 (cos p - 1 )sin p ³ 0 Þ x2 + nx + - 10 = 0
3
Þ cos 2 p - 4 cos p sin p + 4 sin p ³ 0 Þ 3x2 + 3nx + n 2 - 31 = 0
Þ (cos p - 2 sin p)2 - 4 sin 2 p + 4 sin p ³ 0 Let a and b be the roots. Since, a and b are consecutive.
Þ (cos p - 2 sin p)2 + 4 sin p (1 - sin p) ³ 0 …(i) \ |a - b| = 1 Þ (a - b )2 = 1
122 JEE Main Mathematics

Again, (a - b )2 = (a + b )2 - 4ab 30. The given equation is


2
æ - 3n ö æ n 2 - 31 ö pqx2 - ( p + q)2x + ( p + q)2 = 0.
Þ 1=ç ÷ - 4ç ÷
è 3 ø è 3 ø ( p + q )2 ± ( p + q)4 - 4 pq( p + q)2
x=
4 2 2 pq
Þ 1 = n2 - (n - 31)
3 ( p + q )2 ± ( p 2 - q 2 )
Þ 3 = 3n 2 - 4n 2 + 124 x=
2 pq
Þ n 2 = 121 Þ n = ± 11
Now, taking (+ ve) sign
\ n = 11 [Q n > 0] p+ q
x=
25. Let all four roots are imaginary. Then, roots of both q
equation P (x ) = 0 and Q (x ) = 0 are imaginary. and taking (– ve) sign
Thus, b2 - 4ac < 0 ; d 2 - 4ac < 0, so b2 + d 2 < 0 which is p+ q
x=
impossible unless b = 0, d = 0. p
So, if b ¹ 0 or d ¹ 0 atleast two roots must be real, if b = 0, ì p + q p + qü
\ Solution set is í , ý.
d = 0, we have the equations î p q þ
P (x ) = ax2 + c = 0 31. Since, sin a and cos a are the roots of the equation
and Q (x ) = - ax2 + c = 0
ax2 + bx + c = 0, then
c c c c b c
or x2 = - ; x2 = as one of and - must be positive, sin a + cos a = - and sin a cos a =
a a a a a a
so two roots must be real. To eliminate a, we have
26. Since, roots are real. 1 = sin 2 a + cos 2 a
\ { 2 (bc + ad )}2 = 4(a 2 + b2) (c2 + d 2) Þ 1 = (sin a + cos a )2 - 2 sin a cos a
Þ 4b2c2 + 4a 2d 2 + 8abcd = 4a 2c2 + 4a 2d 2 + 4b2c2 + 4b2d 2 b2 2c
Þ 1= 2-
Þ 4a 2d 2 + 4b2c2 - 8abcd = 0 a a
Þ 4(ad - bc)2 = 0 Þ a 2 - b2 + 2ac = 0
Þ ad = bc 32. We have,
a c
Þ = x2 + px + 1 = (x - a ) (x - b)
b d
Thus, E = (c - a ) (c - b) (- d - a ) (- d - b)
27. Since, f (1 ) + f (2) + f (3) = 0
= (c2 + pc + 1) [(- d )2 - pd + 1]
f (1 ), f (2), f (3) all cannot be of same sign. [Q a + b = - p]
Þ Roots are real and distinct. = (c2 + pc + 1) (d 2 - pd + 1)
28. Since, sin a, sin b and cos a are in GP, then But c + qc + 1 = 0 and d 2 + qd + 1 = 0
2

sin 2 b = sin a cos a …(i) \ E = (- qc + pc) (- qd - pd )


Given equation is x2 + 2x cot b + 1 = 0. = cd (q - p) (q + p)
\ Discriminant, D = b2 - 4ac = cd (q2 - p2) = q2 - p2 [Q cd = 1]
= (2 cot b )2 - 4 = 4 (cosec2 b - 2)
33. We have, (a + 1 )x2 + (2a + 3)x + (3a + 4) = 0
= 4 (cosec a sec a - 2) [from Eq. (i)]
Let a and b be the roots of the equation.
= 4 (2 cosec 2 a - 2) ³ 0
According to the given condition,
\ Roots are real.
ab = 2
29. Since, 4 is a root of x2 + ax + 12 = 0 3a + 4
Þ =2
\ 16 + 4a + 12 = 0 a+1
Þ a=-7
Þ 3a + 4 = 2a + 2 Þ a = - 2
Let the roots of the equation x2 + ax + b = 0 be a and a. 2a + 3 -4 + 3
7 Also, a+b=- =- = -1
\ 2a = - a Þa = a+1 -2 + 1
2
2 34. [x]2 - [x] - 2 = 0
æ 7ö
and a× a = b Þ ç ÷ = b Þ ([x] - 2) ([x] + 1 ) = 0
è2ø
49 Þ [x] = 2, - 1
Þ b= Þ x Î [-1, 0) È [2, 3)
4
Quadratic Equations 123

35. Here, Sa = 0, Sab = - 7, abg = - 7 39. Given equation is (5 + 2 )x2 - (4 + 5 )x + 8 + 2 5 = 0.


1 1 1 a 4b 4 + b 4g 4 + g 4a 4 Let x1 and x2 are the roots of the equation.
\ 4
+ 4 + 4 =
a b g a 4b 4g 4 4+ 5
Þ x1 + x2 = …(i)
Sa b 4 4 5+ 2
= …(i)
a 4b 4g 4 8 + 2 5 2(4 + 5 )
and x1x2 = = = 2(x1 + x2) …(ii)
Now, Sab Sab Sab Sab = (Sab ) (Sab ) 2 2 5+ 2 5+ 2
Þ (-7)4 = [a 2 b 2 + b 2g 2 + g 2a 2 + 2abg (a + b + g )] 2x1x2 4(x1 + x2)
\ Harmonic mean = = =4
2 2 2 2 2 2
[a b + b g + g a + 2abg (a + b + g )] x1 + x2 (x1 + x2)

= (a b + b 2 g 2 + g 2a 2) (a 2 b 2 + b 2g 2 + g 2a 2)
2 2 [from Eq. (ii)]
[Q Sa = a + b + g = 0] 40. Given, a + b + c=0 …(i)
4 4 4 4 4 4 4 2 2 2 4 2 2 2 4 2
= a b + b g + g a + 2a b g + 2a b g + 2a b g and 4ax + 3bx + 2c = 0
4 4 2 2 2
= S a b + 2 a b g (a + b + g ) 2 2 2
Q D = b2 - 4ac
= S a 4b 4 + 2 a 2b 2 g 2 [(S a )2 - 2S ab ] \ D = (3b)2 - 4(4a )(2c)
= Sa 4 b 4 + 2 a 2 b 2 g 2 [0 - 2 ´ (-7 )] = 9b2 - 32ac
= Sa 4b 4 + 2 (-7 )2 (2 ´ 7 ) = 9(a + c)2 - 32ac [Q from Eq. (i)]
2
Þ 4 4
Sa b = (-7 ) + 4(-7 )4 3 = 9(a - c) + 4ac > 0
4 4 3 3 Hence, roots are real.
Þ Sa b = (-7 ) (-7 + 4) = - 3 (-7 )
On putting this value in Eq. (i), we get 41. Let f (x) = x2 - 2 (3k - 1)x + 8k2 - 7
1 1 1 -3 (-7 )3 -3 3 Given, f (x) > 0
+ + = = = \ D <0
a4 b4 g 4 (-7 )4 -7 7
Þ (2 (3k - 1))2 - 4 (8k2 - 7) < 0
a b
36. Given equation + = 1 can be rewritten as Þ 4 (9k2 - 6k + 1) - 4 (8k2 - 7) < 0
x- a x-b
Þ k2 - 6k + 8 < 0
x2 - 2 (a + b )x + 3 ab = 0
Þ (k - 4) (k - 2) < 0
Let its roots be a¢ and - a ¢.
Þ 2 < k <4
Þ a ¢ + (- a ¢ ) = 2(a + b )
Then k = 3
Þ 0 = 2 (a + b ) Þ a + b = 0
42. Given expression x2 + 2x + 2xy + my - 3 = 0 can be
1 1 1
37. Given equation + = can be rewritten as rewritten as
x+ p x+ q r
x2 + 2x (1 + y) + (my - 3) = 0.
x2 + x ( p + q - 2r ) + pq - pr - qr = 0 …(i)
Let roots are a and - a, then the product of roots But factors are rational so discriminant b2 - 4ac is a
perfect square.
- a 2 = pq - pr - qr = pq - r ( p + q) …(ii)
Now, b2 - 4ac = 4{(1 + y)2 - (my - 3)} ³ 0
and sum of roots, 0 = - ( p + q - 2r )
p+ q Þ 4{ y2 + 1 + 2 y - my + 3} ³ 0
Þ r= …(iii)
2 Þ y2 + 2 y - my + 4 ³ 0
On solving Eqs. (ii) and (iii), we get Hence, 2 y - my = ± 4 y (as it is perfect square).
p+ q Þ 2 y - my = 4 y
- a 2 = pq - ( p + q)
2 Þ m = -2
1
= - {( p + q)2 - 2 pq} Now, taking (–)ve sign, we get m = 6
2
43. The given equations are
( p2 + q 2 )
Þ a2 = - qx2 + px + q = 0 …(i)
2
and x2 - 4qx + p2 = 0 …(ii)
38. Given equation is x2 + ax + 1 = 0.
Since, root of the Eq. (i) are complex, therefore
Since, roots are tan q and cot q.
p2 - 4 q 2 < 0
\ Product of roots, tan q × cot q = a Þ a = 1
Now, discriminant of Eq. (ii) is
Again, since roots are real.
16q2 - 4 p2 = - 4( p2 - 4q2) > 0
\ a 2 - 4 ³ 0 Þ |a |³ 2
Hence, roots are real and unequal.
Thus, the least value of| a |is 2.
124 JEE Main Mathematics

44. Given equation can be rewritten as Þ (c - a )2 - 4(a - b) (b - c) = 0


3x2 - (a + c + 2b + 2d )x + ac + 2bd = 0 Þ a 2 + 4b2 + c2 + 2ac - 4ab - 4bc = 0
\ Discriminant, D Þ (a + c - 2 b)2 = 0 Þ a + c = 2 b
2
= (a + c + 2b + 2d ) - 4 × 3 (ac + 2bd ) Hence, a , b, c are in AP.
= {(a + 2d ) + (c + 2b)}2 - 12(ac + 2bd ) 49. Given equation is
= {(a + 2d ) - (c + 2b)}2 + 4(a + 2d )(c + 2b) - 12 (ac + 2bd ) ( p2 + q2)x2 - 2q( p + r )x + (q2 + r 2) = 0.
2
= {(a + 2d ) - (c + 2b)} - 8ac + 8ab + 8dc - 8bd Since, roots are real and equal, then
= {(a + 2d ) - (c + 2b)}2 + 8 (c - b)(d - a ) b2 - 4ac = 0
Which is +ve, since a < b < c < d. Þ 4 q2 ( p + r )2 - 4( p2 + q2) (q2 + r 2) = 0
Hence, roots are real and distinct. Þ q2 ( p2 + r 2 + 2 pr ) - ( p2q2 + p2r 2 + q4 + q2r 2) = 0
2
45. Since, a , b are the roots of the equation x + px + q = 0, Þ q2p2 + q2r 2 + 2 pq2r - p2q2 - p2r 2 - q4 - q2r 2 = 0
then Þ 2 pq2r - p2r 2 - q4 = 0
a + b = - p, ab = q …(i)
Þ (q2 - pr )2 = 0 Þ q2 = pr
and a , b are the roots of x2 - xr + s = 0.
4 4
\ p, q and r will be in GP.
Then, a4 + b4 = r …(ii)
50. The equation x2 - px + r = 0 has roots a , b and the
and a 4b 4 = s
a
If D is discriminant of the equation equation x2 - qx + r = 0 has roots , 2 b.
2
x2 - 4qx + 2q2 - r = 0,
Þ r = ab
then D = 16 q2 - 4 (2q2 - r ) = 8q2 + 4r
and a+b=p
= 8a 2 b 2 + 4(a 4 + b 4 ) [from Eqs. (i) and (ii)]
a
= 4 (a 2 + b 2)2 ³ 0 and + 2b = q
2
Hence, the equation x2 - 4qx + 2q2 - r = 0 has always 2q - p 2(2 p - q)
two real roots. Þ b= and a =
3 3
46. The roots of the equation ax2 + bx + c = 0 are given by 2
2
Þ ab = r = (2q - p)(2 p - q)
-b± b - 4ac 9
x=
2a 51. Given, a , b, c are in AP.
(i) Let b2 - 4ac > 0, b > 0 \ 2b = a + c
Now, if a > 0, c > 0, b2 - 4ac < b2 Centroid of triangle whose vertices
Þ The roots are negative. æ 10 7 ö
(a , c), (2, b) and (a , b) are ç , ÷.
(ii) Let b2 - 4ac < 0, then the roots are given by è 3 3ø
- b ± i (4ac - b2) 2a + 2 10 2b + c 7
x= (i = -1 ) \ = and =
2a 3 3 3 3
Which are imaginary and have negative part. (Q b > 0) 2b + c = 7ü
a = 4, ý, solving
\In each case the root have negative real part. 2b - c = 4 þ
11 3
47. Given, (ax2 + c ) y + (a ¢ x2 + c¢ ) = 0 b= ,c=
4 2
or x2 (ay + a ¢ ) + (cy + c¢ ) = 0 11
Since, x is rational, then the discriminant of the above \ Quadratic equation is 4x2 + x+ 1 =0
4
equation must be a perfect square. 121 3 71
\ The value of (a + b )2 - 3ab = - =-
\ 0 - 4 (ay + a ¢ ) (cy + c¢ ) = 0 256 4 256
Þ - acy2 - (ac¢ + a ¢ c ) y - a ¢ c¢ must be a perfect square
52. Given, a and b are roots of px2 + qx + r = 0, p ¹ 0.
Þ (ac¢ - a ¢ c)2 - 4aca ¢ c¢ = 0
-q r
Þ (ac¢ - a ¢ c)2 = 0 \ a+b= and ab = …(i)
p p
Þ ac¢ = a ¢ c Since, p, q and r are in AP.
a c
Þ = \ 2q = p + r …(ii)
a ¢ c¢ 1 1
Also, + =4
48. Since, roots of the equation a b
(a - b)x2 + (c - a )x + (b - c) = 0 are equal. a+b
Þ =4 [given]
\ Discriminant, B2 - 4 AC = 0 ab
Quadratic Equations 125

Þ a + b = 4ab Now, the required equation whose roots are (a + b )2 and


- q 4r (a - b )2 is
Þ = [from Eq. (i)]
p p x2 - {(a + b )2 + (a - b )2 }x + (a + b )2(a - b )2 = 0
Þ q = - 4r Þ x2 - {(a + b)2 - (a - b)2 } x - (a + b)2 (a - b)2 = 0
On putting the value of q in Eq. (ii), we get Þ x2 - 4abx - (a 2 - b2)2 = 0
2(- 4r ) = p + r Þ p = - 9r 56. Since, a , a 2 be the roots of the equation x2 + x + 1 = 0.
- q 4r 4r 4
Now, a + b = = = =- \ a + a 2 = -1 …(i)
p p - 9r 9 3
and a =1 …(ii)
r r 1
and ab = = = Now, a31 + a 62 = a31 (1 + a31 )
p - 9r - 9
Þ a31 + a 62 = a30 × a (1 + a30 × a )
\ (a - b )2 = (a + b )2 - 4ab
Þ a31 + a 62 = (a3 )10 × a { 1 + (a3 )10 × a }
16 4 16 + 36
= + = Þ a31 + a 62 = a (1 + a ) [from Eq. (ii)]
81 9 81
2 52 2 Þ a31 + a 62 = - 1 [from Eq. (i)]
Þ (a - b ) = Þ|a - b| = 13 Again, a31 × a 62 = a 93
81 9
Þ a31 × a 62 = [a3 ] 31 = 1
53. Since, a , b and c are the sides of a DABC.
\ Required equation is
\ |a - b| < |c|
x2 - ( a31 + a 62)x + a31 × a 62 = 0
Þ a 2 + b2 - 2ab < c2
Similarly, b2 + c2 - 2bc < a 2 Þ x2 + x + 1 = 0
and c2 + a 2 - 2ca < b2 57. Since, a and b are the roots of ax2 + bx + c = 0.
On adding, we get b c
Þ a+b = - and ab = …(i)
(c2 + b2 + c2) < 2 (ab + bc + ca ) a a
a 2 + b2 + c2 1 1
Þ <2 …(i) If roots are a + , b + , then
ab + bc + ca b a
Also, D ³ 0 Þ (a + b + c)2 - 3l (ab + bc + ca ) ³ 0 æ 1ö æ 1ö a+b
sum of roots = ç a + ÷ + çb + ÷ = (a + b ) +
a 2 + b2 + c2 è bø è aø ab
Þ ³ 3l - 2 …(ii)
ab + bc + ca -b
= (a + c) [from Eq. (i)]
From Eqs. (i) and (ii), ac
4 æ 1ö æ 1ö
3l - 2 < 2 Þ l < and product of roots = ç a + ÷ çb + ÷
3 è bø è aø
1 c a
54. The given equation is x2 - 2x cos f + 1 = 0. = ab + 1 + 1 + =2+ + [from Eq. (i)]
ab a c
2
2 cos f ± 4 cos f - 4 2ac + c2 + a 2 (a + c)2
\ x= = cos f ± i sin f = =
2 ac ac
Let a = cos f + i sin f, then b = cos f - i sin f
Hence, required equation is given by
\ a n + b n = (cos f + i sin f )n + (cos f - i sin f )n x2 - (sum of roots) x + (product of roots) = 0
= 2 cos n f b (a + c)2
n n Þ x2 + (a + c)x + =0
and a b = (cos nf + i sin n f ) (cos nf - i sin nf ) ac ac
= cos 2 nf + sin 2 nf = 1 Þ acx2 + (a + c)bx + (a + c)2 = 0
\ Required equation is 58. Let the roots of equation
x2 - 2x cos nf + 1 = 0 2x2 + 3x + 1 = 0 is a and b
55. Since, a and b are the roots of the equation 3 1
Then, a + b = - and ab =
2 2
2x2 + 2(a + b)x + a 2 + b2 = 0.
\ a 2 + b 2 = (a + b )2 - 2ab
2 2 a 2 + b2
\ (a + b ) = (a + b) and ab = æ -3 ö
2
1 9 5
2 = ç ÷ -2 ´ = -1 =
Now, (a - b )2 = (a + b )2 - 4ab è 2 ø 2 4 4
2
æ a 2 + b2 ö æ1ö 1
= (a + b)2 - 4 ç ÷ and a2 b2 = ç ÷ =
è 2 ø è2ø 4
= - (a - b)2 \ Required equation is
126 JEE Main Mathematics

x2 - (a 2 + b 2)x + a 2b 2 = 0 65. We know that, the expression ax2 + bx + c > 0 for all x, if
5 1 a > 0 and b2 < 4ac.
Þ x2 - x + = 0
4 4 \ (a 2 - 1 )x2 + 2(a - 1 )x + 2 is positive for all x, if
Þ 4 x2 - 5 x + 1 = 0 a 2 - 1 > 0 and 4(a - 1 )2 - 8(a 2 - 1 ) < 0
59. Equations x3 + ax2 + bx + c = 0 Þ a 2 - 1 > 0 and - 4(a - 1 )(a + 3) < 0
and x3 + (a - 1 )x2 + (b - 1 ) x + (c - 1 ) = 0 have atleast Þ a 2 - 1 > 0 and (a - 1 )(a + 3) > 0
one common root, let common root be a. Þ 2
a > 1 and a < - 3 or a > 1
\ a3 + aa 2 + ba + c = 0 Þ a < - 3 or a > 1
and a3 + aa 2 + ba + c - a 2 - a - 1 = 0 66. Given equation is x2 - 2ax + a 2 + a - 3 = 0.
2 2
Þ a + a + 1 = 0 Þ a = w, w If roots are real, then D ³ 0
(where, w and w2 are the cube roots of unity) Þ 4a 2 - 4 (a 2 + a - 3) ³ 0
60. Given equations are 2x2 + 3x + 5l = 0 and Þ - a + 3 ³0
x2 + 2x + 3l = 0 have a common root, if Þ a -3 £0 Þ a £3
x2 x 1 As roots are less than 3, hence f (3) > 0
= =
(9 - 10)l (5 - 6)l (4 - 3) 9 - 6a + a 2 + a - 3 > 0
Þ a 2 - 5a + 6 > 0
x2 x 1
Þ = = Þ (a - 2)(a - 3) > 0
-l -l 1
Þ Either a < 2 or a > 3.
Þ x2 = - l , x = - l or l = - 1, 0
Hence, only a < 2 satisfy.
61. Let the roots be a , b, b, g and g, a, then
67. Since, a , b are the roots of the equation ax2 + bx + c = 0.
ab = b, bg = c and ga = a
\ ax2 + bx + c = a (x - a )(x - b )
Þ abg = abc
Þ a , b be the roots of ax2 + bx + c = 0. Also, a < k < b
62. Roots of the equation 2x2 + 3x + 5 = 0 are
So, a (k - a )(k - b ) < 0
-3 ± 9 - 40
x= (imaginary roots) Also, a 2k2 + abk + ac = a (ak2 + bk + c)
6
= a 2(k - a )(k - b ) < 0
Hence, both roots coincide, so on comparing
Þ a 2k2 + abk + ac < 0
a b c
= = =k 68. The given condition suggest that a lies between the
2 3 5
roots.
Þ a = 2k, b = 3k, c = 5k
Let f (x ) = 2x2 - 2(2a + 1 )x + a (a + 1 )
Þ a + b + c = 10k
For a to lie between the roots we must have
So, maximum value does not exist.
discriminant ³ 0 and f (a ) < 0.
63. Since, the second equation has imaginary roots. Now, discriminant ³ 0
2 a -3 b 4 c Þ 4(2a + 1 )2 - 8a (a + 1 ) ³ 0
\ = = =k
3 -4 5 æ 1ö
Þ 8ç a 2 + a + ÷ ³ 0, which is always true.
3k 4k 5k è 2ø
Þ a= ,b= ,c=
2 3 4 Also, f (a ) < 0
3k 4k Þ 2a 2 - 2a (2a + 1 ) + a (a + 1 ) < 0
+
a+b 2 3 = 34 Þ - a2 - a < 0
\ =
b + c 4k + 5k 31 Þ a2 + a > 0
3 4 Þ a (1 + a ) > 0
64. We know that, if a1x2 + b1x + c1 = 0 Þ a > 0 or a < - 1
and a 2x2 + b22x + c2 = 0 have a common real root, then 69. - x2 + ax + a = 0
2
(a1c2 - a 2c1 ) = (b1c2 - b2c1 ) (a1b2 - a 2b1 ) Þ x2 - ax - a = 0
Hence, x2 + bx - 1 = 0 and x2 + x + b = 0 have a common Let f (x ) = x2 - ax - a
root f (1 ) < 0 1
2 2
Þ (1 + b) = (b + 1) (1 - b) Þ 1 - a - a <0
Þ b2 + 2b + 1 = b2 - b3 + 1 - b Þ 1 < 2a
Þ b3 + 3b = 0 Þ b(b2 + 3) = 0 1
Þ a>
Þ b = 0, ± i 3 2
Quadratic Equations 127

70. Since, a , b are the roots of the equation x2 + bx + c = 0. 4. The given quadratic equation
Here, D = b2 - 4c > 0 because c < 0 < b. So, roots are real 2x(2x + 1) = 1
and unequal. Þ 4x2 + 2x - 1 = 0 has roots a and b
Now, a + b = - b < 0 and a b = c < 0 So, a+b=-
1
Þ 2 a + 2b = -1 ...(i)
\ One root is positive and the other is negative, then the 2
negative root being numerically bigger. As, a < b , a is and 4a 2 + 2 a - 1 = 0 ...(ii)
the negative root while b is the positive root. So,|a | > b From Eqs. (i) and (ii), we get
and a < 0 < b.
4a 2 + 2a + (2a + 2b) = 0
Round II Þ 2a 2 + 2a + b = 0 Þ b = - 2a(a + 1)
1. The given equation is 5. Given quadratic equation x2 - 64x + 256 = 0 having
2
2 (1 + i )x - 4(2 - i )x - 5 - 3 i = 0 roots a and b, so
2
4(2 - i ) ± 16(2 - i ) + 8(1 + i )(5 + 3 i ) a + b = 64 and ab = 256
Þ x= 1/ 8 1/ 8
4(1 + i ) æ a3 ö æ b3 ö a+b
Now, ç 5 ÷ + ç 5÷ =
i 4 - i -1 - i 3 -5i èb ø èa ø (ab )5/ 8
=- or = or
1+ i 1+ i 2 2
64 64 64
- - = = = =2
Now, ½½ 1 i½
½= 1
+ =
1 1
(256)5/ 8 25 32
½ 2 ½ 4 4 2
-1 + i 3
½3 - 5 i½= 9 25 17 6. It is given that a = , then a 2 + a + 1 = 0 and
and ½ ½ + = 2
½ 2 ½ 4 4 2
a3 = 1.
17 1
Also, > 100 100 100
2 2 So, a = (1 + a ) å a 2k = (- a 2) å a 2k = - å a2( k + 1)
3 -5i k=0 k=0 k=0
Hence, required root is .
2 = - [a 2 + a 4 + a 6 + a 8 + ¼ + a 202]
2. We have, 2 2
ax - bx (x - 1 ) + c(x - 1 ) = 0 …(i) a 2((a 2)101 - 1)
=- (sum of GP)
2 a2 - 1
æ x ö æ x ö
Þ aç ÷ + bç ÷ + c=0 a 2(a 202 - 1) a 2(a - 1)
è 1 - xø è 1 - xø =- =- , (Q a3 = 1)
2
a -1 a2 - 1
Also, a and b be the roots of ax2 + bx + c = 0.
a2 a3
x x =- =- 2 =1
\ a= and b = a+1 a +a
1-x 1-x
100 100 100
a b
Þ x= ,x= and, b = å a3 k = å (a3 )k = å 1 = 101
a+1 b+1 k=0 k=0 k=0

a b Now, equation of quadratic equation having roots


Hence, and are the required roots.
a+1 b+1 ‘a’ and ‘b’ is
x2 - (a + b)x + ab = 0 Þ x2 - 102x + 101 = 0
3. It is given that a and b are roots of quadratic equation
Hence, option (c) is correct.
x2 - x - 1 = 0, so sum of roots = a + b = 1, and product of
roots = ab = - 1 7. Given quadratic equation is
k k
and pk = a + b , k ³ 1 æ pö
x2 + x sin q - 2 sin q = 0, q Î ç0, ÷
So, p1 = a + b = 1 è 2ø
p2 = a 2 + b 2 = (a + b )2 - 2ab = 1 + 2 = 3 and its roots are a and b.
3 3 3
p3 = a + b = (a + b ) - 3ab (a + b ) = 1 + 3 = 4 So, sum of roots = a + b = - sin q
p4 = a 4+ b 4 = (a + b )4 - 4ab (a 2 + b 2) - 6a 2 b 2 and product of roots = ab = - 2 sin q
= 1 + 12 - 6 = 7 Þ ab = 2(a + b ) …(i)
12 12
and p5 = a5 + b5 a +b
Now, the given expression is -12
= (a + b )5 - 5ab (a3 + b3 ) - 10 a 2b 2(a + b ) (a + b -12)(a - b)24
= 1 + 20 - 10 = 11 a12 + b12 a12 + b12
p3 = p5 - p4 = 4 = = 12
Q æ 1 1 ö 24 æ b + a12 ö
p1 + p2 + p3 + p4 + p5 = 1 + 3 + 4 + 7 + 11 = 26 ç 12 + 12 ÷ (a - b) ç 12 12 ÷ (a - b)24
èa b ø è a b ø
but p5 ¹ p2 × p3
128 JEE Main Mathematics

12 12
é ab ù æ ab ö = a 2 + b 2 + (w4 + w2) ab (Q w3 = 1 )
=ê 2ú
=ç ÷ = a 2 + b 2 - ab (Q w + w2 = - 1 )
ë (a - b ) û è (a + b )2 - 4ab ø
12
= (a + b )2 - 3 ab = p2 - 3q
é 2(a + b) ù a 2 b 2 a3 + b3
=ê 2 ú [from Eq. (i)] Also, + =
ë (a + b ) - 8 (a + b) û b a ab
12 12
æ 2 ö æ 2 ö (a + b )3 - 3 ab (a + b ) p(3q - p2)
=ç ÷ =ç ÷ = =
è (a + b ) - 8 ø è - sin q - 8 ø ab q
[Q a + b = - sin q] ( p2 - 3 q ) q
\ The given expression = =-
212 p(3q - p2) p
= q
(sin q + 8)12

8. Given, (x2 - 5x + 5)x


2
+ 4 x - 60
=1
10. Let the correct equation is
ax2 + bx + c = 0,
Clearly, this is possible when b c
then a+b=- and ab =
I. x2 + 4x - 60 = 0 and x2 - 5x + 5 ¹ 0 a a
or When b is written incorrectly, then the roots are equal.
2
II. x - 5x + 5 = 1 Let these are g and g.
c
or \ g × g = Þ g 2 = ab …(i)
a
2 2
III. x - 5x + 5 = - 1 and x + 4x - 60 = Even integer
When c is written incorrectly, then the roots are g and
Case I When x2 + 4x - 60 = 0 2g.
Þ x2 + 10x - 6x - 60 = 0 b
\ g + 2g = - Þ 3g = a + b
Þ x(x + 10) - 6(x + 10) = 0 a
Þ 9g = (a + b )2 Þ 9ab = (a - b )2 + 4 ab
2
Þ (x + 10) (x - 6) = 0
[from Eq. (i)]
Þ x = - 10 or x = 6 \ (a - b )2 = 5 ab
Note that, for these two values of x,
11. Discriminant of the equation 3x2 + 8x + 15 = 0 is given
x2 - 5 x + 5 ¹ 0
by
Case II When x2 - 5 x + 5 = 1
D = 64 - 180 = - 116 < 0
Þ x2 - 5 x + 4 = 0 So, its roots are imaginary and therefore roots are
Þ x2 - 4 x - x + 4 = 0 conjugate to each other. Therefore, one common root
Þ x(x - 4) - 1 (x - 4) = 0 means both the roots are common.
a 2b 3c
Þ (x - 4) (x - 1) = 0 \ = =
3 8 15
Þ x = 4 or x = 1 a b c
Case III When x2 - 5x + 5 = - 1 Þ = = =k (say), k ¹ 0
3 4 5
Þ x2 - 5x + 6 = 0 Þ a = 3k, b = 4k, c = 5k
Þ x2 - 2 x - 3 x + 6 = 0 Now, a 2 + b2 = c2
Þ x(x - 2) - 3(x - 2) = 0 Þ DABC is right angled.
Þ (x - 2) (x - 3) = 0 \ sin 2 A + sin 2 B = sin 2 C
Þ x = 2 or x = 3 Þ sin A + sin 2 B + sin 2 C = 2 sin 2 C = 2 sin 2 90° = 2
2

Now, when x = 2, x2 + 4x - 60 12. Given equation is


= 4 + 8 - 60 = - 48, which is an even integer. | x - 2| + x ( x - 4) + 2 = 0
When x = 3, x2 + 4x - 60 = 9 + 12 - 60 = - 39, Þ | x - 2| + x - 4 x + 4 = 2
which is not an even integer. Þ | x - 2| + ( x - 2)2 = 2
Thus, in this case, we get x = 2. Þ (| x - 2|)2 + | x - 2| - 2 = 0
Hence, the sum of all real values of Let| x - 2| = y, then above equation reduced to
x = - 10 + 6 + 4 + 1 + 2 = 3 y2 + y - 2 = 0
9. Since, a and b are the roots of the equation Þ y2 + 2 y - y - 2 = 0
x2 + px + q = 0, therefore Þ y( y + 2) - 1( y + 2) = 0
a + b = - p and ab = q Þ ( y + 2)( y - 1) = 0
Now, (wa + w2 b )(w2a + wb ) Þ y = 1, - 2
Quadratic Equations 129

\ y=1 [Q y = | x - 2| ³ 0] 16. D = b2 - 4a < 0 Þ a > 0


Þ | x - 2| = 1 Þ x - 2 = ± 1
Therefore, the graph is concave upwards.
Þ x = 3 or 1 Þ x = 9 or 1
f (x ) > 0, "x Î R
\ Sum of roots = 9 + 1 = 10
Þ f (-1 ) > 0 Þ a + b + 1 > 0
13. Given equation 5 + |2x - 1| = 2x (2x - 2) 2
17. x - (a - 3)x + a = 0
Case I
Now, D = (a - 3)2 - 4a
If 2x - 1 ³ 0 Þ x ³ 0 , then 5 + 2x - 1 = 2x (2x - 2)
= a 2 - 10a + 9 = (a - 1 )(a - 9)
Put 2x = t, then 5 + t - 1 = t 2 - 2t Þ t 2 - 3t - 4 = 0
Case I Both the roots are greater than 2.
Þ t 2 - 4t + t - 4 = 0 Þ t (t - 4) + 1(t - 4) = 0
B
Þ t = 4 or - 1 Þ t = 4 (Q t = 2x > 0) D ³ 0, f (2) > 0, - >2
x 2A
Þ 2 =4Þx=2 >0
a -3
Þ x = 2 is the solution. Þ (a - 1 )(a - 9) ³ 0; 4 - (a - 3)2 + a > 0; >2
2
Case II
Þ a Î (-¥ , 1] È [ 9, ¥ ); a < 10, a > 7
If 2x - 1 < 0 Þ x < 0 , then 5 + 1 - 2x = 2x (2x - 2)
Þ a ³ [9, 10] …(i)
Put 2x = y, then 6 - y = y2 - 2 y
Case II One root is greater than 2 and the other root is
Þ y2 - y - 6 = 0 Þ y2 - 3 y + 2 y - 6 = 0
less than or equal to 2. Hence,
Þ ( y + 2) ( y - 3) = 0 Þ y = 3 or - 2
f (2) £ 0
Þ y = 3 (as y = 2x > 0) Þ 2x = 3
Þ x = log 2 3 > 0 Þ 4 - (a - 3)2 + a £ 0
So, x = log 2 3 is not a solution. Þ a ³ 10 …(ii)
Therefore, number of real roots is one. From Eqs. (i) and (ii),

14. Given quadratic equations having roots a and b is a Î [9, 10) È [10, ¥ )
2
x + px + 2 = 0, so a + b = - p and ab = 2 Þ a Î [9, ¥ )
1 1 18. The given equation is
and having roots and is
a b
x2 - 2mx + m2 - 1 = 0
1 1 Þ (x - m)2 - 1 = 0
2x2 + 2qx + 1 = 0, so + = - q
a b Þ (x - m + 1 )(x - m - 1 ) = 0
æ 1ö æ 1ö æ 1ö æ 1ö Þ x = m - 1, m + 1
So, ç a - ÷ çb - ÷ ç a + ÷ çb + ÷
è aø è bø è bø è aø From given condition,
æ a b 1 öæ 1 ö m - 1 > - 2 and m + 1 < 4
= ç ab - - + ÷ ç ab + 1 + 1 + ÷
è b a ab ø è ab ø Þ m > - 1 and m > 3
æ a2 + b2 1 ö æ 1ö Hence, -1 < m < 3
= ç2 - + ÷ ç2 + 1 + 1 + ÷
è ab 2ø è 2ø 19. Note that coefficient of x2 is (4 p - p2 - 5) < 0.
é 5 (a + b )2 - 2ab ù é 9 ù Y
=ê - úê ú
ë2 2 û ë2û
9 9
= (5 - (- p)2 + 4) = (9 - p2)
4 4 X
Hence, option (d) is correct. 0 1

15. On multiplying the given equation by c /a3 , we get Therefore, the graph is concave downward.
2 2
bc 2 bc 2 According to the question,
x - 2 x+ c=0 1 must lie between the roots.
a3 a
2 Hence, f (1 ) > 0
æ bc ö æ bc ö
Þ a ç 2 x÷ - b ç 2 ÷ x + c = 0 Þ 4 p - p2 - 5 - 2 p + 1 + 3 p > 0
èa ø èa ø
bc Þ - p2 + 5 p - 4 > 0
Þ x = a, b
a2 Þ p2 - 5 p + 4 < 0
Þ (a + b )abx = a , b Þ ( p - 4)( p - 1 ) < 0
1 1 Þ 1 < p<4
Þ x= ,
(a + b )a (a + b )b Þ p Î{ 2, 3}
130 JEE Main Mathematics

20. Clearly, f (-1 ) > 0, f (2) < 0. Þ a 2- 20a + 100 - 4(33 - 4a ) ³ 0


Now, f (0) = - 4 < 0 Þ a 2 - 4a - 32 ³ 0
2
Þ f (-1) = 1 - a - 4 > 0 Þ a - 8a + 4a - 32 ³ 0
and f (2) = 4 + 2a - 4 < 0 Þ a (a - 8) + 4(a - 8) ³ 0
Þ a < - 3 and a < 0 Þ (a + 4)(a - 8) ³ 0
Þ a Î (-¥ , - 3) Þ a Î (-¥ , - 4] È [8, ¥ )
Y \The least positive value of ‘a’ for which the given
quadratic equation has real roots is 8.
Hence, answer 8 is correct.
0 2
X
25. Given, a and b are the roots of the equation
–1 x2 - 6x - 2 = 0.
Q a n = a n - b n for n ³ 1
x2 + 4 \ a10 = a10 - b10
21. a= -3
| x| a 8 = a8 - b8 Þ a 9 = a9 - b9
4 Now, consider
= | x| + -3
| x| a10 - 2a 8 a10 - b10 - 2(a 8 - b 8 )
=
4 2a 9 2(a 9 - b 9 )
³ 2 | x| -3 [Q AM ³ GM] Q a and b are the
| x| a 8 (a 2 - 2) - b 8 (b 2 - 2)
= 9 9
roots of
=4 -3 =1 2(a - b )
x2 - 6 x - 2 = 0
Þ a ³1 a 8 × 6a - b 8 × 6b or x2 = 6 x + 2
= 9 9
22. Given, p + q = 2 and p4 + q4 = 272 2(a - b ) Þ a 2 = 6a + 2
2
Þ ( p2 + q2)2 - 2 p2q2 = 272 6a 9 - 6b 9 Þ a - 2 = 6a
= and b2 = 6 b + 2
Þ (( p + q)2 - 2 pq)2 - 2 p2q2 = 272 2(a 9 - b 9 ) 2
Þ b - 2 = 6b
Þ 16 + 16 pq + 2 p2q2 = 272 6
= =3
Þ ( pq)2 - 8 pq - 128 = 0 2
8 ± 24
Þ pq = = 16, - 8 26. We have,
2
x2 - 1154x + 1 = 0
Þ pq = 16
Now, x2 - ( p + q)x + pq = 0 a + b = 1154, ab = 1
2
x - 2x + 16 = 0 Þ ( a + b )2 = a + b + 2 ab
or x2 - 2x - 8 = 0 = 1154 + 2 = 1156

23. Given, a + b = 1 , ab = - 1 Þ a + b = 1156 = 34


Again, (a1/ 4 + b1/ 4 )2 = a + b + 2(ab)1/ 4
\ Quadratic equation with roots a , b is
x2 - x - 1 = 0 Þ a 2 = a + 1 = 34 + 2 = 36
1/ 4 1/ 4
Multiplying both sides by a n - 1 a n + 1 = a n + a n - 1 …(i) \ (a +b ) = 36 = 6
Similarly, b n + 1 = b n + b n - 1 …(ii) 27. Let f (x) = (a 2 - 14a + 13)x2 + (a + 2)x - 2
Adding Eqs. (i) and (ii), we get Equation have no distinct real roots.
n+1 n+1 n n n -1 n -1
a +b = (a + b ) + (a +b ) \ Either f (x) ³ 0 or f (x) £ 0 " x Î R
Þ Pn + 1 = Pn + Pn - 1 But f (0) = - 2 < 0
Þ 29 = Pn + 1 [Given, Pn + 1 = 29, Pn - 1 = 11] \ f (0) £ 0 " x Î R
Þ Pn = 18 So, f (- 1) £ 0

\ Pn2 2
= 18 = 324 Þ (a 2 - 14a + 13) - (a + 2) - 2 £ 0
Þ a 2 - 15a + 9 £ 0
24. Since the quadratic equation
2 33 Þ a 2 - 15a £ - 9
2x + (a - 10)x + = 2a has real roots, so D ³ 0
2 So, the maximum value of a 2 - 15a = - 9
æ 33 ö \ | - 9| = 9
Þ (a - 10)2 - 4(2) ç - 2a ÷ ³ 0
è2 ø
Quadratic Equations 131

28. We have, By symmetry of the results


2 2
x + 2(l + 1)x + l + l + 7 = 0 ( b - g ) ( b - d) = q + r
Both roots are negative, then D ³ 0 Hence, the ratio is 1.
\ 4(l + 1)2 - 4(l + l + 7) ³ 0 30. log 4 (2x2 + x + 1) - log 2(2x - 1) = 1
Þ l - 6 ³ 0 Þ l Î [6, ¥ ) ...(i) log e (2x2 + x + 1) log e (2x - 1)
Sum of roots = - 2(l + 1) < 0 Þ - =1
log e (4) log e 2
Þ l Î (- 1, ¥ ) ...(ii)
2x2 + x + 1
and product of roots = l2 + l + 7 > 0 Þ log e = log e 4
(2x - 1)2
Þ l ÎR ...(iii)
Þ (2x2 + x + 1) = 4(4x2 - 4x + 1)
\ From (i), (ii) and (iii), we get
Þ 14x2 - 17x + 3 = 0
l Î [6, ¥ )
The least value of l = 6 Þ (14x - 3) (x - 1) = 0
a + b = - pü 3
Þ x= , x=1
29. Here, ý Þa + b = g + d 14
g + d = - pþ
3
Now, (a + g ) (a - d) = a 2 - a(g + d) + gd But x = does not lie in the domain of function
14
= a 2 - a(a + b ) + r Hence, x = 1 is the only solution.
= - ab + r
= - (- q) + r = q + r
06
Permutations and
Combinations
In this chapter, we study a useful counting technique in determining the IN THIS CHAPTER ....
number of different ways of arranging and selecting objects without actually
Fundamental Principles of
listing them. Firstly, examine a principle which is most fundamental to the
Counting [FPC]
learning of these techniques.
Factorial Notation

Fundamental Principles of Counting [FPC] Exponent of Prime p in n!


Permutation
Fundamental Principle of Multiplication Circular Permutation
If an operation can be performed in m different ways, following which a Combination
second operation can be performed in n different ways, then the two
Division of Objects into Groups
operations in succession can be performed in m ´ n ways. This can be
extended to any finite number of operations. Applications of Permutation and
Combination
e.g. A hall has 12 gates. After entering into the hall the man come out
through a different gate in 11 ways. Dearrangements

Hence, by the fundamental principle of multiplication, the total number of Number of Integral Solutions of
ways of man come out through different gates = 12 ´ 11 = 132 . Linear Equations

Fundamental Principle of Addition


If an operation can be performed in m different ways and another operation,
which is independent of the first operation can be performed in n different
ways. Then, either of the two operations can be performed in ( m + n )
ways. This can be extended to any finite number of mutually exclusive
operations.
e.g. There are 25 students in a class in which 15 boys and 10 girls.
The class teacher select either a boy or a girl for monitor of the class. Since,
there are 15 ways to select a boy and there are 10 ways to select a girl.
Hence, by the fundamental principle of addition, the number of ways in which
either a boy or a girl can be chosen as a monitor = 10 + 15 = 25 ways.
Permutations and Combinations 133

Example 1. If x < 4 < y and x, y Î {1, 2, 3, ..., 10 }, then find é1 x ù


\ êë 3 + 50 úû = 0 , for1 £ x £ 33
the number of ordered pairs ( x, y).
1 x 4
(a) 30 (b) 18 (c) 40 (d) 100 For 34 £ x £ 50 , 1 < + <
3 50 3
Sol. (b) We have, x < 4 < y, é1 x ù
Þ ê + úû = 1, for 34 £ x £ 50
where x, y Î {1, 2, 3, ... , 10 } ë 3 50
Þ x = {1, 2, 3} and y = {5, 6, 7, 8, 9, 10} Thus, E = 17 and p = 2
Here, x have 3 options and y have 6 options. Q 2 4 < 17 < 25
\ By multiplication rule, \ a=4
Number of ordered pairs = 3 ´ 6 = 18 é17 ù é17 ù é17 ù é17 ù
Exponent of 2 in (17) ! = ê ú + ê ú + ê ú + ê ú
ë 2 û ë 4 û ë 8 û ë16 û
Factorial Notation = 8 + 4 + 2 + 1 = 15
The product of first n natural numbers is denoted by n !
and read as ‘factorial n’.
Permutation
Thus, n ! = n ( n - 1) ( n - 2) K 3 × 2 × 1
Each of different arrangements which can be made by
e.g. 5 ! = 5 ´ 4 ´ 3 ´ 2 ´ 1 = 120 taking some or all of a number of things is called a
and 4 ! = 4 ´ 3 ! = 4 ´ 3 ´ 2 ! = 4 ´ 3 ´ 2 ´ 1 = 24 permutation.
e. g. Arrangements of objects taking 2 at a time from
Properties of Factorial Notation given 3 objects ( a , b, c) are ab, bc, ca , cb , ac, ba, then total
(i) 0 ! = 1 ! = 1 number of arrangements i.e. total number of permutation
is 6.
(ii) Factorials of negative integers and fractions are not
defined.
Meaning of n Pr
(iii) n ! = n ( n - 1)! = n ( n - 1) ( n - 2)!
Number of permutations of n distinct objects taking r at
n!
(iv) = n ( n - 1) ( n - 2) K ( r + 1) a time is denoted by n Pr .
r!
n n!
Pr = , "0 £ r £ n
Exponent of Prime p in n! ( n - r )!
Let n be a positive integer and p be a prime number. = n ( n - 1) ( n - 2) K ( n - r + 1), " n Î N and r Î W .
Then, last integer amongst 1, 2 , 3, ... , ( n - 1), n which is
énù énù Properties of n Pr
divisible by p is ê ú p, where ê ú denotes the greatest
ë pû ë pû (i) The number of permutations of n distinct object
n taken all at a time is n Pn = n !.
integer less than or equal to . n
p (ii) P0 = 1, n P1 = n and n Pn - 1 = n !
n n -1 n -1 n -1
é 12 ù é 15 ù (iii) Pr = n × Pr - 1 = r × Pr - 1 + Pr
e.g. êë 5 úû = 2 , êë 5 úû = 3 etc. n -1 n- 1
(iv) Pr = ( n - r ) Pr - 1
Let E p( n !) denotes the exponent of prime p in n ! , then
énù é n ù é n ù Important Result on Permutations
E p( n !) = ê ú + ê 2 ú + K + ê a ú
ë û ëp û
p ëp û (i) The number of permutations of n things taken all at
a time p are alike of one kind, q are alike of second
where a is a greatest positive integer such that
kind, r are alike of third kind and remaining are
pa £ n £ pa + 1. n!
distinct, is .
p! q ! r !
é1 1ù é1 2 ù é1 3 ù
Example 2. Let E = ê + + + + + +K+
ë 3 50 úû êë 3 50 úû êë 3 50 úû (ii) The number of permutations of n different things,
taken r at a time when each thing may be repeated
upto 50 terms, then the exponent of 2 in E ! is
any number of times is n r .
(a) 17 (b) 25
(iii) Number of permutations under certain conditions
(c) 15 (d) None of these
(a) Number of permutations of n different things
é1 x ù
Sol. (c) Let E = ê + úû , where x = 1, 2, K , 50
taken r at a time when a particular thing is to be
ë 3 50
always included in each arrangement is
1 1 x
For 1 £ x £ 33, < + <1 r × n - 1Pr - 1.
3 3 50
134 JEE Main Mathematics

(b) Number of permutations of n different things Now, the groups (means families) can arrange in 3 ! ways.
taken r at a time, when a particular thing is So, required number of ways is
never taken in each arrangement is n - 1 Pr . 3 ! ´ 3 ! ´ 4 ! ´ 3 ! = (3 !)3 × 4 !
(c) Number of permutations of n different things
taken all at a time, when m specified things
always come together is m ! ´ ( n - m + 1)!.
Circular Permutation
If objects are arranged along a closed curve, then
(d) Number of permutations of n different things,
permutation is known as circular permutation.
taken all at a time, when m specified things
never come together is n ! - m ! ´ ( n - m + 1)!. In other words, the permutation in a row has a
beginnings and end but there is no beginning and end in
(e) Number of permutations of n different things,
circular permutation. So, we need to consider one object
taken r at a time when p ( p < r ) particular things
is fixed and the remaining objects are arranged in ( n - 1)!
are to be always included in each arrangement is
ways.
p !{ r - ( p - 1)} n - p Pr - p.
e.g. Consider five persons A, B, C , D and E to be seated on
2n+ 1 2n- 1 the circumference of a circular table in order (which has
Example 3. If Pn-1 : Pn = 3 : 5, then the value of n is
no head). Now, shifting A, B, C , D and E one position in
equal to anti-clockwise direction, we will get arrangements as
(a) 3 (b) 4 (c) 5 (d) 6 follows
2n + 1 2n -1
Sol. (b) We have, Pn -1 : Pn = 3 : 5 we see that arrangements in all figures are same.
2n + 1
Pn -1 3 \The number of circular permutations of n different
Þ = n
2n -1
Pn 5 Pn
things taken all at a time is = ( n - 1)!, if clockwise
( 2n + 1) ! ( n - 1) ! 3 n
Þ ´ = and anti-clockwise orders are taken as different.
( n + 2 ) ! ( 2n - 1) ! 5
E D C
( 2n + 1) (2n ) ´ n ! 3
Þ =
( n + 2 ) ( n + 1)n ´ 2n ! 5 A D E C D B
( 2n + 1) ( 2n ) 3
Þ = B C A B E A
( n + 2 ) ( n + 1)n 5
Þ 10 ( 2n + 1) = 3 (n + 2) (n + 1) (i) (ii) (iii)
Þ 3n 2 - 11n - 4 = 0 B A
Þ ( n - 4 ) ( 3n + 1) = 0 Þ n = 4 C A B E

Example 4. Total number of 6-digit numbers in which only D E C D


and all the five digits 1, 3, 5, 7 and 9 appear, is
(JEE Main 2020) (iv) (v)
5 1 6
(a) 6 ! (b) (6 !) (c) (6 !) (d) 5 In a circular permutation, if the position are given by
2 2 number, then it is treated as a linear arrangement.
Sol. (b) To make 6-digit numbers from given digits 1, 3, 5, 7 and
9, we must repeat a digit and we can done the same in 5C1 ways. Important Results on Circular
Now, the arrangement of these 6-digits in which two are identical Permutation
6! (i) Number of circular permutations of n different
is .
2! things taken all at a time = ( n - 1)!. If clockwise and
6! 5 anti-clockwise orders are taken as different.
So, required numbers of 6-digit numbers = 5C1 = (6 !)
2! 2 (ii) The number of circular permutations of n different
1
Example 5. Two families with three members each and one things taken all at a time = ( n - 1) !. If clockwise
2
family with four members are to be seated in a row. In how and anti-clockwise orders are taken as not different.
many ways can they be seated so that the same family
members are not separated? (JEE Main 2020) Example 6. The number of ways in which 5 ladies and 7
(a) 2 ! 3 ! 4 ! (b) (3 !)3 × ( 4 !) gentlemen can be seated in a round table so that no two
2
(c) (3 !) × ( 4 !) (d) 3 ! ( 4 !)3 ladies sit together, is
7
(a) (720) 2 (b) 7 (360) 2
Sol. (b) Since two families has 3 members each and one family 2
with four members. So that can be seated among themself, so same (c) 7 (720) 2 (d) 720
family members are not separated in 3 !, 3 ! and 4 ! respectively.
Permutations and Combinations 135

Sol. (a) First we fix the alternate positions of 7 gentlemen in a (h) If n is odd, then the greatest value of n Cr is
round table by 6! ways. n
C n + 1 or n
Cn - 1 .
G1 2
2
G7 G2 n
(i) n Cr = × n - 1Cr - 1
r
G6 G3 n
Cr n -r +1
(j) n =
G5 G4 Cr - 1 r
n
There are seven positions between the gentlemen in which (k) C0 + n C1 + n C2 + K + n Cn = 2n
7
5 ladies can be seated in P5 ways. (l) n
C0 + n C2 + n C4 + K = n C1 + n C3 + n C5 + K = 2n - 1
\ Required number of ways (m) 2n + 1
C0 + 2n + 1
C1 + 2n + 1
C2 + K + 2n + 1
Cn = 22n
7! 7 n n +1 n +2 2n - 1 2n
= 6! ´ = (720) 2 (n) Cn + Cn + Cn + K + Cn = Cn + 1
2! 2
Important Results on Combinations
Combination (i) The number of combinations of n different things,
Each of the different groups or selections which can be taken r at a time, where p particular things occur is
made by some or all of a number of given things without n - p
Cr - p.
reference to the order of the things in each group is called
a combination. (ii) The number of combinations of n different things,
e.g. The groups made by taking 2 objects at a time taken r at a time, where p particular things never
from three objects ( a , b, c) are ab, bc, ca. Then, the occur is n - pCr .
number of groups is 3 each of which is known as (iii) The total number of combinations of n different
combination. things taken one or more at a time or the number of
Note In permutation order of objects is important whereas in ways of n different things selecting atleast one of
combination order of objects is not important. them is
n
C1 + n C2 + K + n Cn = 2n - 1
Meaning of n Cr (iv) The number of combinations of n identical things
The number of combinations of n different things taken r taking r ( r £ n ) at a time is 1.
ænö (v) The number of ways of selecting r things out of n
at a time is denoted by n Cr or C( n , r ) or ç ÷. alike things is ( n + 1), (where r = 0, 1, 2, 3, K , n ).
èr ø
(vi) If out of ( p + q + r ) things, p are alike of one kind, q
n
n n! Pr are alike of second kind and rest are alike of third
Then, Cr = = (0 £ r £ n )
r !( n - r ) ! r! kind, then the total number of combinations is
n ( n - 1) ( n - 2) K ( n - r + 1) [( p + 1) ( q + 1) ( r + 1)] - 1
=
r (r - 1) (r - 2) K 2 × 1 (vii) If out of ( p + q + r + t ) things, p are alike of one
kind, q are alike of second kind, r are alike of third
n ÎN and r Î W kind and t are different, then the total number of
If r > n , then n Cr = 0 combinations is ( p + 1)( q + 1) ( r + 1)2t - 1.

Properties of n C r Example 7. Team ‘A’ consists of 7 boys and n girls and


Team ‘B’ has 4 boys and 6 girls. If a total of 52 single matches
n
(a) Cr is a natural number. can be arranged between these two teams when a boy plays
n against a boy and a girl plays against a girl, then n is equal to
(b) C0 = n Cn = 1, n C1 = n
(JEE Mains 2021)
n n
(c) Cr = Cn - r (a) 5 (b) 2 (c) 4 (d) 6
n n +1 Sol. (c) Total matches between boys of both team
(d) Cr + n Cr - 1 = Cr
n
= 7C1 ´ 4C1 = 28
(e) Cx = n C y Û x = y or x + y = n
Total matches between girls of both team
n -1
(f) n × Cr - 1 = ( n - r + 1) n Cr - 1 = nC1 6C1 = 6n

(g) If n is even, then the greatest value of n Cr is n Cn /2. Now, 28 + 6n = 52


Þ n=4
136 JEE Main Mathematics

Example 8. A committee of 11 members is to be formed (iii) The total number of ways of dividing n identical
from 8 males and 5 females. If m is the number of ways the objects among r persons, each one of whom, receives
committee is formed with at least 6 males and n is the atleast one item is n - 1Cr - 1.
number of ways the committee is formed with atleast OR
3 females, then (JEE Main 2019)
The number of ways in which n identical things can
(a) m = n = 68 (b) m + n = 68
be divided into r groups such that blank groups are
(c) m = n = 78 (d) n = m - 8
not allowed, is n - 1Cr - 1.
Sol. (c) Since there are 8 males and 5 females. Out of these 13
members committee of 11 members is to be formed. According to
the question, m = number of ways when there is at least 6 males Example 9. The number of ways of choosing 10 objects out
= ( 8C 6 ´ 5C5) + ( 8C 7 ´5 C 4) + ( 8C 8 ´ 5 C3) of 31 objects of which 10 are identical and the remaining 21
are distinct, is (JEE Main 2019)
= (28 ´ 1) + (8 ´ 5) + (1 ´ 10) = 28 + 40 + 10 = 78
and n = number of ways when there is at least 3 females (a) 2 20 - 1 (b) 2 21
= ( 5C3 ´ 8 C 8) + ( 5C 4 ´ 8 C 7) + ( 5C5 ´ 8 C 6) (c) 2 20 (d) 2 20 + 1
= 10 ´ 1 + 5 ´ 8 + 1 ´ 28 = 78 Sol. (c) Given that, out of 31 objects 10 are identical and
So, m = n = 78 remaining 21 are distinct, so in following ways, we can choose 10
objects.
Division of Objects into Groups 0 identical + 10 distincts, number of ways = 1 ´ 21C10
21
1 identical + 9 distincts, number of ways = 1 ´ C9
Objects are Different 2 identicals + 8 distincts, number of ways = 1 ´ 21
C8
(i) The number of ways of dividing n different objects 21
10 identicals + 0 distinct, number of ways = 1 ´ C0
into 3 groups of p, q and r things ( p + q + r = n ) is
n! So, total number of ways in which we can choose 10 objects is
(a) ; p, q and r are unequal. 21 21 21 21
p! q ! r ! C10 + C9 + C8 + K + C0 = x (let) … (i)
n! Þ 21
C11 + 21
C12 + 21
C13 + K + 21
C 21 = x … (ii)
(b) ;q =r
p ! 2 !( q !)2 n
[Q C r = C n - r ]n

n!
(c) ; p= q =r On adding both Eqs. (i) and (ii), we get
3 !( p !)3 21 21 21 21
2x = C0 + C1 + C2 + K + C10
(ii) The number of ways of dividing n different objects
21 21 21
into r groups is + C11 + C12 + K + C 21
21
1 é n ær ö ær ö ær ö ù Þ 2x = 2
ê r - ç ÷ ( r - 1)n + ç ÷ ( r - 2)n - n
ç ÷ ( r - 3) +...ú Þ x = 2 20
r! ë è1ø è 2ø è 3ø û
(iii) The number of ways of dividing n different objects Arrangement in Groups
into r groups taking into account the order of the
groups and also the order of objects in each group is (a) The number of ways in which n different things can
( n + r – 1)
Pn = r ( r + 1) ( r + 2)...( r + n - 1). be arranged into r different groups is
n + r -1
Pn or n ! n - 1Cr - 1.
Objects are Identical
(b) The number of ways in which n different things can
(i) The number of ways of dividing n identical objects be distributed into r different groups is
among r persons such that each gets 1, 2, 3, ... or k
objects is the coefficient of x n - r in the expansion of r n - rC1(r - 1 )n + rC2(r - 2 )n
(1 + x + x 2 + K + x k - 1 )r . - K + ( -1 )r - 1 × rCr - 1
(ii) The number of ways of dividing n identical objects or coefficient of x n in n !( ex - 1 )r .
among r persons such that each one may get atmost Here, blank groups are not allowed.
æ n + r - 1ö
n objects is ç ÷, (c) The number of ways in which n identical things can
è r -1 ø
be distributed into r different groups is n + r - 1Cr - 1
OR
or n - 1Cr - 1, according as blank groups are or are not
The total number of ways of dividing n identical
objects into r groups, if blank groups are allowed, is admissible.
n + r -1
Cr - 1.
Permutations and Combinations 137

Applications of Permutation Geometrical Applications


(i) Out of n non-concurrent and non-parallel straight
and Combination lines, the number of point of intersection are n C2.
Functional Applications (ii) The number of straight lines passing through n
points = n C2.
(i) The number of all permutations (arrangements) of n
different objects taken r at a time, (iii) The number of straight lines passing through n
(a) when a particular object is to be always included points out of which m are collinear = n C2 - mC2 + 1.
in each arrangement is n - 1Cr - 1 ´ r ! (iv) In a polygon, the total number of diagonals out of
n points (no three points are collinear)
(b) when a particular object is never taken in each n ( n - 3)
= n C2 - n = .
arrangement is n - 1Cr ´ r! 2
(ii) If the sets A has m elements and B has n elements, (v) Number of triangles formed by joining n points is
n
then C3 .
(a) the number of functions from A to B is n m . (vi) Number of triangles formed by joining n points out
(b) the number of one-one functions from A to B is of which m are collinear are n C3 - mC3 .
n
Pm , m £ n . (vii) The number of parallelogram in two systems of
(c) the number of onto functions from A to B is parallel lines (when Ist set contains m parallel lines
and 2nd set contains n parallel lines) = n C2 ´ mC2
ænö ænö
n m - ç ÷ ( n - 1)m + ç ÷ ( n - 2)m -... , m £ n . (viii) The number of rectangles of any size in a square of
è1 ø è2 ø n

(d) the number of increasing (decreasing) functions n ´ n is å r3 and number of squares of any size is
r =1
æn ö
from A to B is ç ÷ , m £ n . n
è mø å 2
n .
r =1
(e) the number of non-decreasing (non-increasing)
functions from A to B is Example 11. There are 10 points in a plane, out of these 6
æ m + n - 1ö are collinear. If N is the number of triangles formed by joining
ç ÷ , m £ n. these points, then
è m ø
(a) N > 190 (b) N £ 100
(f) the number of bijections from A to B is n ! , (c) 100 < N £ 140 (d) 140 < N £ 190
if m = n.
(g) the number of bijections from A to A such that Sol. (b) If out of n points, m are collinear, then
Number of triangles = nC3 - mC3
f ( x ) ¹ x , " x Î A, is
é1 \ Number of triangles = 10C3 - 6C3 = 120 - 20 Þ N = 100
1 1 ( -1)m ù
m!ê - + - ...+ ú.
ë 2! 3! 4! m! û Example 12. Suppose that 20 pillars of the same height
have been erected along the boundary of a circular stadium.
Example 10. The number of functions f from {1,2,3, …,20} If the top of each pillar has been connected by beams with
onto {1,2,3,…, 20} such that f (k) is a multiple of 3, whenever the top of all its non-adjacent pillars, then the total number of
beams is (JEE Main 2019)
k is a multiple of 4, is (JEE Main 2019)
(a) 180 (b) 210 (c) 170 (d) 190
(a) (15) ! ´ 6 ! (b) 5 6 ´ 15
Sol. (c) It is given that, there are 20 pillars of the same height
(c) 5 ! ´ 6 ! (d) 65 ´ (15) !
have been erected along the boundary of a circular stadium.
Sol. (a) According to given information, Now, the top of each pillar has been connected by beams with
the top of all its non-adjacent pillars, then total number of beams
we have if k Î{4, 8, 12, 16, 20}
= number of diagonals of 20-sided polygon.
Then, f (k) Î{3, 6, 9, 12, 15, 18}
Q 20C 2 is selection of any two vertices of 20-sided polygon which
[Q codomain ( f ) = {1, 2, 3,…, 20}]
included the sides as well.
Now, we need to assign the value of f (k) for
So, required number of total beams = 20C 2 - 20
k Î{4, 8, 12, 16, 20} this can be done in
[Q the number of diagonals in a n-sided
6
C5 × 5 ! ways = 6 × 5 ! = 6 ! and remaining 15 elements can be closed polygon = nC 2 - n]
associated by 15! ways. 20 ´ 19
= - 20 = 190 - 20 = 170
\ Total number of onto functions = 15 × 6 = 15 ! 6 ! 2
138 JEE Main Mathematics

Dearrangements Number of Integral Solutions of


If n distinct objects are arranged in a row, then the number of Linear Equations
ways in which they can be dearranged so that none of them Consider the equation
occupies its original place is x1 + x2 + x3 + x4 + . . . + xr = n …(i)
ì 1 1 1 1 1ü where x1 , x2 , . . . , xr and n are non-negative
n ! í1 - + - + - K + ( -1)n ý
î 1! 2! 3! 4! n !þ integers
and it is denoted by D ( n ). objects are to be divided into r groups where a
group may contain any number of objects.
If r ( 0 £ r £ n ) objects occupy the places assigned to them i.e.,
Therefore, total number of solutions of Eq. (i),
their original places and none of the remaining ( n - r ) objects
occupies its original places, then the number of such ways is = Coefficient of x n in ( x 0 + x1 + . . . + x n )r
n + r -1 n + r -1
D ( n - r ) = n Cr × D ( n - r ) = Cr or Cn - 1

= n Cr × ( n - r )! Example 14. The total number of 3-digit numbers,


ì 1 1 1 1 ü whose sum of digits is 10, is ............ . (JEE Main 2020)
í1 - + - + K + ( -1)n - r ý
î 1 ! 2 ! 3 ! ( n - r )!þ (a) 54 (b) 56 (c) 52 (d) 50

Example 13. Ajay writes letters to his five friends and addresses the Sol. (a) Let the digits of 3-digit numbers are x, y , z
corresponding. The number of ways can the letters be placed in the such that
envelops so that atleast two of them are in the wrong envelopes are x + y + z = 10 and x, y , z Î {0, 1, 2, 3, …, 9},
but x ¹ 0
(a) 120 (b) 125 (c) 119 (d) 124
5
Now, let x = t + 1, t Î {0 , 1, 2, 3, ¼ , 8}
Sol. (c) Required number of ways = å 5
C5 - r D(r) So, t + 1 + y + z = 10
r =2 Þ t + y + z =9
5
5! ì 1 1 1 ( -1) r ü having non-negative integral solution
= å × r ! í1 - +
r ! (5 - r) ! î 1! 2 ! 3 !
- +K+
r! þ
ý
= 9 + 3 - 1C3 - 1
r =2
5
5! ì 1 1 ( -1) r ü = 11C 2 = 55
= å í -
(5 - r) ! î 2 ! 3 !
+K+
r! þ
ý
r =2 But, it include the case, when t = 9
5! ì 1 ü 5! ì 1 1 ü 5! ì 1 1 1ü Þ x = 10,
= í ý+ í - ý+ í - + ý
3 ! î 2 !þ 2 ! î 2 ! 3 !þ 1! î 2 ! 3 ! 4 !þ which is not possible, so required number of 3-digit
5! ì 1 1 1 1ü numbers
+ í - + - ý
0 ! î 2 ! 3 ! 4 ! 5 !þ = 55 - 1= 54
= 10 + 20 + (60 - 20 + 5) + (60 - 20 + 5 - 1) Hence, answer is 54.
= 10 + 20 + 45 + 44 = 119
Practice Exercise
ROUND I Topically Divided Problems
Permutation 9. The number of natural numbers less than 7000
1. The exponent of 3 in 100! is which can be formed by using the digits 0, 1, 3, 7, 9
(a) 47 (b) 48 (c) 49 (d) 50 (repitition of digits allowed) is equal to
(JEE Main 2019)
2. How many different non-digit numbers can be (a) 374 (b) 375 (c) 372 (d) 250
formed from the digits of the number 223355888 by
10. We are to form different words with the letters of
rearrangement of the digits so that the odd digits
the word INTEGER. Let m1 be the number of words
occupy even places?
(a) 16 (b) 36 (c) 60 (d) 180 in which I and N are never together and m2 be the
number of words which begin with I and end with
3. How many even numbers of 3 different digits can R, then m1 / m2 is equal to
be formed from the digits 1, 2, 3, 4, 5, 6, 7, 8, 9
(a) 30 (b) 60 (c) 90 (d) 180
(repetition of digits is not allowed)?
(a) 224 (b) 280 11. If the letters of the word KRISNA are arranged in
(c) 324 (d) None of these all possible ways and these words are written out
as in a dictionary, then the rank of the word
4. The number of seven digit integers with sum of the
KRISNA is
digits equal to 10 and formed by using the digits 1,
(a) 324 (b) 341
2 and 3 only is (JEE Main 2021)
(c) 359 (d) None of these
(a) 77 (b) 42 (c) 35 (d) 82
12. If the letters of the word MOTHER are written in
5. The number of words which can be formed out of all possible orders and these words are written out
the letters of the word ARTICLE, so that vowels
as in a dictionary, then the rank of the word
occupy the even place is
4
MOTHER is
(a) 1440 (b) 144 (c) 7! (d) C 4 ´ 3C3
(a) 240 (b) 261
6. Eight chairs are numbered 1 to 8. Two women and (c) 308 (d) 309
3 men wish to occupy one chair each. First the 13. There are 10 persons named P1, P2 , P3, . . . , P10 .
women choose the chairs from amongst the chairs 1
Out of 10 persons, 5 persons are to be arranged in a
to 4 and then men select from the remaining chairs.
line such that in each arrangement P1 must occur
Find the total number of possible arrangements.
whereas P4 and P5 do not occur. Find the number of
(a) 1440 (b) 1450
(c) 1460 (d) None of these such possible arrangements.
(a) 4210 (b) 4200
7. The number of four-digit numbers strictly greater (c) 4203 (d) 4205
than 4321 that can be formed using the digits 0, 1,
14. If a denotes the number of permutations of x + 2
2, 3, 4, 5 (repetition of digits is allowed) is
(JEE Main 2019) things taken all at a time, b the number of
(a) 306 (b) 310 (c) 360 (d) 288 permutations of x things taken 11 at a time and c
the number of permutations of x - 11 things taken
8. The number of 6 digits numbers that can be
all at a time such that a = 182 bc, then the value of
formed using the digits 0, 1, 2,5, 7 and 9 which
x is
are divisible by 11 and no digit is repeated, is
(JEE Main 2019)
(a) 15 (b) 12
(c) 10 (d) 18
(a) 60 (b) 72 (c) 48 (d) 36
140 JEE Main Mathematics

15. In a circus there are ten cages for accommodating Combination


ten animals. Out of these four cages are so small
that five out of 10 animals cannot enter into them.
23. How many numbers lying between 10 and 1000 can
be formed from the digits 1, 2, 3, 4, 5, 6, 7, 8, 9
In how many ways will it be possible to
(repetition of digits is allowed) ?
accommodate ten animals in these ten cages?
(a) 1024 (b) 810
(a) 66400 (b) 86400
(c) 2346 (d) None of these
(c) 96400 (d) None of these
16. The total number of permutations of n ( > 1) 24. A scientific committee is to formed from 6 Indians
and 8 foreigners, which includes at least 2 Indians
different things taken not more than r at a time,
and double the number of foreigners as Indians.
when each thing may be repeated any number of
Then the number of ways, the committee can be
times is
formed is (JEE Main 2021)
n (n n - 1) nr - 1
(a) (b) (a) 560 (b) 1050 (c) 1625 (d) 575
n -1 n -1
n (n r - 1) 25. How many 10-digit numbers can be written by
(c) (d) None of these
n -1 using the digits 1 and 2 ?
10
(a) C1 + 9C 2 (b) 210
17. The number of ways in which 10 candidates (c) 10
C2 (d) 10!
A1, A2 , . . . , A10 can be ranked such that A1 is
26. The number of times the digits 3 will be written
always above A10 is
1 when listing the integers from 1 to 1000 is
(a) 5! (b) 2(5!) (c) 10! (d) (10 !) (a) 269 (b) 300 (c) 271 (d) 302
2
27. The figures 4, 5, 6, 7, 8 are written in every possible
Circular Permutation order. The number of numbers greater than 56000 is
18. If eleven members of a committee sit at a round (a) 72 (b) 96 (c) 90 (d) 98
5
table so that the President and Secretary always sit 47 52 - r
together, then the number of arrangements is
28. The value of C4 + å C3 is equal to
r =1
(a) 10 ! ´ 2 (b) 10! (a) 47
C6 (b) 52C5
(c) 9 ! ´ 2 (d) None of these (c) 52
C4 (d) None of these
19. The number of ways in which seven persons can be 29. If n C3 + nC4 > n + 1C3, then
arranged at a round table, if two particular persons
(a) n > 6 (b) n > 7
may not sit together is
(c) n < 6 (d) None of these
(a) 480 (b) 120 20
(c) 80 (d) None of these 30. The value of å 50 - r C6 is equal to
r=0 (JEE Main 2020)
20. In how many ways can 15 members of a council sit 50 30 51 30
along a circular table, when the Secretary is to sit (a) C7 - C7 (b) C7 - C7
51 30 50 30
on one side of the Chairman and the Deputy (c) C7 + C7 (d) C6 - C6
Secretary on the other side? 31. There are 3 sections in a question paper and each
(a) 2 ´ 12 ! (b) 24 section contains 5 questions. A candidate has to
(c) 2 ´ 15 ! (d) None of these answer a total of 5 questions, choosing at least one
21. 20 persons are invited for a party. In how many question from each section. Then the number of ways,
different ways can they and the host be seated at in which the candidate can choose the questions, is
circular table, if the two particular persons are to (JEE Main 2020)

be seated on either side of the host? (a) 3000 (b) 1500 (c) 2255 (d) 2250
(a) 20 ! (b) 2×18 ! 32. Consider three boxes, each containing 10 balls
(c) 18 ! (d) None of these labelled 1, 2, …, 10. Suppose one ball is randomly
22. In how many ways can 5 boys and 5 girls sit in a drawn from each of the boxes. Denote by ni , the
circle so that no two boys sit together? label of the ball drawn from the ith box, ( i = 1, 2, 3).
(a) 5 ! ´ 5 ! (b) 4 ! ´ 5 !
Then, the number of ways in which the balls can be
5! ´5! chosen such that n1 < n2 < n3 is (JEE Main 2019)
(c) (d) None of these
2 (a) 82 (b) 120 (c) 240 (d) 164
Permutations and Combinations 141

33. In how many ways can a student choose a program such that there is at least one boy and at least one
of 5 courses, if 9 courses are available and girl in each team, is 1750, then n is equal to
2 specific courses are compulsory for every student? (JEE Main 2019)
(a) 34 (b) 36 (c) 35 (d) 37 (a) 28 (b) 27 (c) 25 (d) 24

34. Every body in a room shakes hands with everybody 43. Consider a class of 5 girls and 7 boys. The number
else. The total number of hand shakes is 66. The of different teams consisting of 2 girls and 3 boys
total number of persons in the room is that can be formed from this class, if there are two
(a) 11 (b) 12 (c) 13 (d) 14 specific boys A and B, who refuse to be the
members of the same team, is (JEE Main 2019)
35. The number of ways in which a team of eleven
(a) 350 (b) 500 (c) 200 (d) 300
players can be selected from 22 players always
including 2 of them and excluding 4 of them is 44. A pack of 52 cards are shuffled together. The
16 16 16 20 number of ways in which a man can be dealt 26
(a) C11 (b) C5 (c) C9 (d) C9
cards so that he does not get two cards of the same
36. A box contains two white balls, three black balls
suit and same denomination, is
and four red balls. In how many ways can three 52
(a) C 26 × 2 26 (b) 104
C 26
balls be drawn from the box, if atleast one black
52
ball is to be included in the draw? (c) 2 × C 26 (d) None of these
(a) 64 (b) 45 45. In how many ways can 21 English and 19 Hindi
(c) 46 (d) None of these books be placed in a row so that no two Hindi books
37. The number of ways in which we can choose a are together?
committee from four men and six women so that (a) 1540 (b) 1450 (c) 1504 (d) 1405
the committee includes atleast two men and exactly 46. In a football championship, there were played 153
twice as many women as men is matches. Every team played one match with each
(a) 94 (b) 126 other. The number of teams participating in the
(c) 128 (d) None of these championship is
38. A five digit number divisible by 3 is to be formed (a) 17 (b) 18 (c) 9 (d) 13
using the numbers 0, 1, 2 , 3, 4 and 5, without 47. A father with 8 children takes them 3 at a time to
repetition. The total number of ways this can be the zoological gardens, as often as he can without
done, is taking the same 3 children together more than
(a) 216 (b) 240 (c) 600 (d) 3125 once. The number of times he will go the garden, is
39. There are 10 lamps in a hall. Each one of them can (a) 336 (b) 112
be switched on independently. Find the number of (c) 56 (d) None of these
ways in which hall can be illuminated. 48. A car will hold 2 in the front seat and 1 in the rear
(a) 210 - 2 (b) 210 - 1 seat. If among 6 persons 2 can drive, then number
(c) 210 + 1 (d) None of these of ways in which the car can be filled, is
40. Find the number of positive integers greater than (a) 10 (b) 20
(c) 30 (d) None of these
6000 and less than 7000 which are divisible by 5,
provided that no digit is to be repeated. 49. Six X’s have to be placed in the square of the figure
(a) 111 (b) 112 such that each row contains atleast one ‘X’. In how
(c) 113 (d) None of these many different ways can this be done?
41. In an examination, a student has to answer
4 questions out of 5 questions; questions 1 and 2
are however compulsory. Find the number of ways
in which the student can make the choice.
(a) 3 (b) 2 (c) 4 (d) 5
42. A group of students comprises of 5 boys and n girls.
(a) 28 (b) 27
If the number of ways, in which a team of 3
(c) 26 (d) None of these
students can randomly be selected from this group
142 JEE Main Mathematics

50. A question paper is divided into two parts A and B 58. There are four balls of different colours and four
and each part contains 5 questions. The number of boxes of colours same as those of the balls. The
ways in which a candidate can answer 6 questions number of ways in which the balls, one in each box,
selecting atleast two questions from each part is could be placed such that a ball does not go to box
(a) 80 (b) 100 of its own colour, is
(c) 200 (d) None of these (a) 8 (b) 7
(c) 9 (d) None of these
51. In an examination there are three multiple choice
questions and each question has 4 choices. Number 59. Five balls of different colours are to be placed in
of ways in which a student can fail to get all three boxes of different sizes. Each box can hold all
answers correct, is five balls. In how many ways can we place the balls
(a) 11 (b) 12 (c) 27 (d) 63 so that no box remains empty?
(a) 50 (b) 100
52. There were two women participating in a chess
(c) 150 (d) 200
tournament. Every participant played two games
with the other participants. The number of games 60. The number of ways of dividing 52 cards amongst
that the men played between themselves proved to four players so that three players have 17 cards
exceed by 66 the number of games that the men each and the fourth players just one card, is
played with the women. The number of 52 !
(a) (b) 52!
participants is (17 !) 3
(a) 6 (b) 11 52 !
(c) (d) None of these
(c) 13 (d) None of these 17 !
53. A lady gives a dinner party for six guests. 61. 18 mice were placed in two experimental groups
The number of ways in which they may be selected and one control group with all group equally large.
from among ten friends, if two of the friends will In how many ways can the mice be placed into
not attend the party together, is three groups?
(a) 112 (b) 140 18 ! 18 !
(a) (b)
(c) 164 (d) None of these (6 !) 2 (6 !) 3
54. A person is permitted to select atleast one and 180
(c) (d) None of these
atmost n coins from a collection of 2 n + 1 (distinct) (6 !) 3
coins. If the total number of ways in which he can 62. A candidate is required to answer 7 questions out
select coins is 255, then n is equal to of 12 questions, which are divided into two groups,
(a) 4 (b) 8 (c) 16 (d) 32 each containing 6 questions. He is not permitted to
55. In an steamer, there are stalls for 12 animals and attempt more than 5 questions from either group.
there are horses, cows and calves (not less than 12 Find the number of different ways of doing
each) ready to be shipped in how many ways can questions.
the ship load be made? (a) 779 (b) 781
(a) 312 - 1 (b) 312 (c) (12) 3 - 1 (d) (12) 3 (c) 780 (d) 782

56. In an examination of 9 papers a candidate has to 63. In how many ways can `16 be divided into 4
pass in more papers, then the number of papers in persons when none of them get less than ` 3?
which he fails in order to be successful. The (a) 70 (b) 35
number of ways in which he can be unsuccessful, is (c) 64 (d) 192
(a) 255 (b) 256 (c) 193 (d) 319 64. A library has a copies of one book, b copies of each
of two books, c copies of each of three books and
Division of Objects into Groups single copies of d books. The total number of ways
57. Given 5 different green dyes, four different blue in which these books can be distributed, is
dyes and three different red dyes, the number of (a + b + c + d )! (a + 2 b + 3 c + d )!
(a) (b)
combinations of dyes which can be chosen taking a ! b ! c! a ! (b !) 2(c !) 3
atleast one green and one blue dye is (a + 2 b + 3 c + d )
(c) (d) None of these
(a) 3600 (b) 3720 (c) 3800 (d) 3600 a ! b ! c!
Permutations and Combinations 143

65. Eleven books consisting of 5 Mathematics, 4 72. If a polygon has 44 diagonals, then the number of
Physics and 2 Chemistry are placed on a shelf. The its sides are
number of possible ways of arranging them on the (a) 11 (b) 7
assumption that the books of the same subject are (c) 8 (d) None of these
all together, is 73. The number of diagonals in a polygon of m sides is
(a) 4! 2! (b) 11! 1 1
(a) m (m - 5) (b) m (m - 1)
(c) 5! 4! 3! 2! (d) None of these 2! 2!
1 1
66. Three boys of class X, four boys of class XI and five (c) m (m - 3) (d) m (m - 2)
2! 2!
boys of class XII sit in a row. The total number of
ways in which these boys can sit so that all the 74. The number of triangles that can be formed by
boys of same class sit together is equal to 5 points in a line and 3 points on a parallel line is
8 8
2
(a) (3!) (4!) (5!) 2
(b) (3!) (4!) (5!) (a) C3 (b) C3 - 5C3
8 5
(c) (3!) (4!) (5!) (d) (3!) (4!) (5!) 2 (c) C3 - C3 - 1 (d) None of these
75. The straight lines I1, I 2 , I 3 are parallel and lie in
Applications of Permutation and the same plane. A total numbers of m points are
Combination taken on I1, n points on I 2 , k points on I 3. The
67. The number of mappings (functions) from the set maximum number of triangles formed with vertices
A = {1, 2 , 3} into the set B = {1, 2 , 3, 4, 5, 6, 7 } such at these points is
that f ( i) £ f ( j), whenever i < j, is (a) m + n + kC3
(a) 84 (b) 90 (b) m + n + kC3 - mC3 - nC3 - kC3
(c) 88 (d) None of these (c) mC3 + nC3 + kC3
(d) None of the above
68. Let f : {1, 2, 3, 4, 5 } ® {1, 2, 3, 4, 4, 5 } that are onto
76. Six points in a plane be joined in all possible ways
and f ( x) ¹ i is equal to
by indefinite straight lines and if no two of them be
(a) 9 (b) 44 coincident or parallel and no three pass through
(c) 16 (d) None of these the same point (with the exception of the original
69. The number of triangles that are formed by 6 points). The number of distinct points or
choosing the vertices from a set of 12 points, seven intersection is equal to
of which lie on the same line is (a) 105 (b) 45
(c) 51 (d) None of these
(a) 105 (b) 15 (c) 175 (d) 185
77. The greatest possible number of points of
70. The number of parallelograms that can be formed
intersection of 8 straight lines and 4 circles is
from a set of four parallel lines intersecting another
(a) 32 (b) 64 (c) 76 (d) 104
set of three parallel lines is
(a) 6 (b) 18 (c) 12 (d) 9 78. There are n distinct points on the circumference of
a circle. The number of pentagons that can be
71. The maximum number of points of intersection of 6 formed with these points as vertices is equal to the
circles is number of possible triangles. Then, the value of n is
(a) 25 (b) 24 (c) 50 (d) 30 (a) 7 (b) 8 (c) 15 (d) 30

Only One Correct Option 2. A rectangle with sides 2 m - 1 and 2 n - 1 divided


1. In a city no two persons have identical set of teeth into squares of unit length. The number of
and there is no person without a tooth. Also, no rectangle which can be formed with sides of odd
person has more than 32 teeth. If we disregard the length is
shape and size of tooth and consider only the (a) m2n 2
positioning of the teeth, the maximum population (b) mn (m + 1) (n + 1)
of the city is (c) 4m + n - 1
(a) 232 (b) (32) 2 - 1 (c) 2 32 - 1 (d) 2 32 - 1 (d) None of the above
144 JEE Main Mathematics

3. The lock of a safe consists of five discs each of ‘vegetable dish’, three types of ‘salads’ and two
which features the digits 0, 1, 2 , . . . , 9. The safe can types of ‘sauces’?
be opened by dialing a special combination of the (a) 3360 (b) 4096
digits. The number of days sufficient enough to (c) 3000 (d) None of these
open the safe. If the work day lasts 13 h and 5 s are 12. There are three coplanar parallel lines. If any p
needed to dial one combination of digits is points are taken on each of the lines, the maximum
(a) 9 (b) 10 (c) 11 (d) 12 number of triangles with vertices on these points is
4. The interior angles of a regular polygon measure (a) 3 p2( p - 1) + 1 (b) 3 p2( p - 1)
160° each. The number of diagonals of the polygon (c) p2(4 p - 3) (d) None of these
are
13. In how many different ways can the first 12 natural
(a) 97 (b) 105 (c) 135 (d) 146
numbers be divided into three different groups
5. Let A be the set of 4-digit numbers a1 a2 a3 a4 , such that numbers in each group are in AP?
where a1 < a2 < a3 < a4 , then n ( A) is equal to (a) 1 (b) 5 (c) 6 (d) 4
(a) 84 (b) 126
14. Two packs of 52 cards are shuffled together. The
(c) 210 (d) None of these
number of ways in which a man can be dealt
6. If the total number of m elements subsets of the set 20 cards, so that he does not get two cards of the
A = { a1, a2 , a3, K, an } is l times the number of same suit and same denomination is
3 elements subsets containing a4 , then n is (a) 56C 20 ´ 2 20 (b) 104C 20
(a) (m - 1) l (b) ml (c) (m + 1) l (d) 0 (c) 2 ´ 52C 20 (d) None of these
7. Sixteen men compete with one another in running, 15. All possible numbers are formed using the
swimming and riding. How many prize lists could digits 1, 1, 2, 2, 2, 2, 3, 4, 4 taken all at a time.
be made, if there were altogether 6 prizes of The number of such numbers in which the odd
different values, one for running, 2 for swimming digits occupy even places is (JEE Main 2019)
and 3 for riding? (a) 180 (b) 175 (c) 160 (d) 162
(a) 16 ´ 15 ´ 14 (b) 163 ´ 152 ´ 14
(c) 163 ´ 15 ´ 142 (d) 162 ´ 15 ´ 14 16. There are m men and two women participating in a
chess tournament. Each participant plays two
8. The number of ways in which we can select four games with every other participant. If the number
numbers from 1 to 30 so as to exclude every of games played by the men between themselves
selection of four consecutive numbers is exceeds the number of games played between the
(a) 27378 (b) 27405
men and the women by 84, then the value of m is
(c) 27399 (d) None of these (JEE Main 2019)
9. The number of different seven digit numbers that (a) 12 (b) 11 (c) 9 (d) 7
can be written using only the three digits 1, 2 and 3 17. A man X has 7 friends, 4 of them are ladies and 3
with the condition that the digit 2 occurs twice in are men. His wife Y also has 7 friends, 3 of them
each number is are ladies and 4 are men. Assume X and Y have no
7
(a) P2 2 (b) 7C 22 5 common friends. Then, the total number of ways in
(c) 7C 252 (d) None of these which X and Y together can throw a party inviting
10. If the difference of the number of arrangements of 3 ladies and 3 men, so that 3 friends of each of X
three things from a certain number of dissimilar and Y are in this party, is (JEE Main 2017)
things and the number of selections of the same (a) 485 (b) 468
number of things from them exceeds 100, then the (c) 469 (d) 484
least number of dissimilar things is 18. If all the words (with or without meaning) having
(a) 8 (b) 6 (c) 5 (d) 7 five letters, formed using the letters of the word
11. A person always prefers to eat ‘parantha’ and SMALL and arranged as in a dictionary, then the
‘vegetable dish’ in his meal. How many ways can position of the word SMALL is (JEE Main 2016)
he make his platter in a marriage party, if (a) 46th (b) 59th
there are three types of paranthas, four types of (c) 52nd (d) 58th
Permutations and Combinations 145

19. Let A and B two sets containing 2 elements and Numerical Value Based Questions
4 elements respectively. The number of subsets of
27. If the letters of the word ‘MOTHER’ be permuted
A ´ B having 3 or more elements is
and all the words so formed (with or without
(a) 256 (b) 220 (c) 219 (d) 211
meaning) be listed as in dictionary, then the
20. The sum of all the 4-digit distinct numbers that can position of the word ‘MOTHER’ is…………
be formed with the digits 1, 2, 2 and 3 is (JEE Main 2020)
(JEE Main 2021)
(a) 26664 (b) 122664 28. The number of words (with or without meaning)
(c) 122234 (d) 22264 that can be formed from all the letters of the word
“LETTER” in which vowels never come together is
21. Assuming the balls to be identical except for
(JEE Main 2020)
difference in colours, the number of ways in which
one or more balls can be selected from 10 white, 29. The number of words, with or without meaning,
9 green and 7 black balls is that can be formed by taking 4 letters at a time
(a) 880 (b) 629 (c) 630 (d) 879 from the letters of the word ‘SYLLABUS’ such that
two letters are distinct and two letters are alike,
22. Let X = {1, 2, 3, 4, 5 }. The number of different
is …… .
ordered pairs ( Y , Z ) that can formed such that
Y Í X , Z Í X and Y Ç Z is empty, is 30. A test consists of 6 multiple choice questions, each
2 5 5 3 having 4 alternative answers of which only one is
(a) 5 (b) 3 (c) 2 (d) 5
correct. The number of ways, in which a candidate
23. There are two urns. Urn A has 3 distinct red balls
answers all six questions such that exactly four of
and urn B has 9 distinct blue balls. From each urn
the answers are correct, is ……… .
two balls are taken out at random and then
transferred to the other. The number of ways in 31. The number of 4 letter words (with or without
which this can be done, is meaning) that can be formed from the eleven letters
(a) 3 (b) 36 (c) 66 (d) 108 of the word ‘EXAMINATION’ is ..............
(JEE Main 2020)
24. From 6 different novels and 3 different dictionaries,
4 novels and 1 dictionary are to be selected and 32. The number of permutations of the word
arranged in a row on the shelf so that the ‘AUROBIND’ in which vowels appear in an
dictionary is always in the middle. Then, the alphabetical order is ……… .
number of such arrangements is 33. 5 Indian and 5 American couples meet at party and
(a) atleast 500 but less than 750 shake hand. If no wife shake hand with her own
(b) atleast 750 but less than 1000 husband and no Indian wife shakes hand with a
(c) atleast 1000 male, then the number of handshakes that take
(d) less than 500
place in the party is ……… .
25. How many different words can be formed by
34. Define a ‘Good word’ as a sequence of letters that
jumbling the letters in the word MISSISSIPPI in
consists only of the letters A, B and C and in which
which no two S are adjacent ?
A never immediately followed by B, B is never
(a) 7 × 6C 4 × 8C 4 (b) 8 × 6C 4 × 7C 4
immediately followed by A. If the number of n letter
(c) 6 × 7 × 8C 4 (d) 6 × 8 × 7C 4
good words is 384, then the value of n is ……… .
26. The set S = {1, 2, 3, . . . , 12 } is to be partitioned 35. There are four balls of different colours and four
into three sets A, B and C of equal size. boxes of colours, same as those of the balls. The
Thus, A È B È C = S, A Ç B = B Ç C = A Ç C = f. number of ways in which the balls, one each in a
The number of ways to partition S is box could be placed such that a ball does not go to a
(a) 12 ! / 3 !(4 !)3 (b) 12 ! / 3 !(3 !)4 box of its own colour, is ……… .
(c) 12 ! / (4 !)3 (d) 12 ! / (3 !)4
146 JEE Main Mathematics

Answers
Round I
1. (c) 2. (c) 3. (a) 4. (a) 5. (b) 6. (a) 7. (b) 8. (a) 9. (a) 10. (a)
11. (a) 12. (d) 13. (b) 14. (b) 15. (b) 16. (c) 17. (d) 18. (c) 19. (a) 20. (a)
21. (b) 22. (b) 23. (b) 24. (c) 25. (b) 26. (b) 27. (c) 28. (c) 29. (a) 30. (b)
31. (d) 32. (b) 33. (c) 34. (b) 35. (c) 36. (a) 37. (a) 38. (a) 39. (b) 40. (b)
41. (a) 42. (c) 43. (d) 44. (a) 45. (a) 46. (b) 47. (c) 48. (b) 49. (c) 50. (c)
51. (d) 52. (c) 53. (b) 54. (a) 55. (b) 56. (b) 57. (b) 58. (c) 59. (c) 60. (a)
61. (b) 62. (c) 63. (b) 64. (b) 65. (c) 66. (a) 67. (a) 68. (b) 69. (d) 70. (b)
71. (d) 72. (a) 73. (c) 74. (c) 75. (b) 76. (c) 77. (d) 78. (b)

Round II
1. (c) 2. (a) 3. (c) 4. (c) 5. (b) 6. (b) 7. (b) 8. (a) 9. (b) 10. (d)
11. (a) 12. (c) 13. (d) 14. (d) 15. (a) 16. (a) 17. (a) 18. (d) 19. (c) 20. (a)
21. (d) 22. (b) 23. (d) 24. (c) 25. (a) 26. (c) 27. (309) 28. (120) 29. (240) 30. (135)
31. (2454) 32. (1680) 33. (135) 34. (8) 35. (9)

Solutions
Round I 5. In a word ARTICLE, vowels are A, E, I and consonants
1. Now, 100 ! = 1 × 2 × 3 × . . . 98 × 99 × 100 are C, L, R, T.
= (1 × 2 × 4 × 5 . . . 98 × 100)(3 × 6 × 9×. . . 96 × 99) In a seven letter word, there are three even places in
which three vowels are placed in 3! way. In rest of the
= K × 333 (1 × 2 × 3 . . . 32 × 33) four places, four consonants are placed in 4! ways.
[Q let K = 1 × 2 × 4 × 5 . . . 98 × 100] \ Required number of ways = 3 ! ´ 4 ! = 6 ´ 24 = 144
= [K (1 × 2 × 4 . . . 31 × 32)] 333 × (3 × 9 × 12 . . . 30 × 33) 6. Two women occupy the chair from 1 to 4 in 4 P2 ways and
= K 1 × 333 × 311 (1 × 2 × 3 . . . 10 × 11 ) 3 men occupy the remaining chairs in 6 P3 ways.
[Q let K (1 × 2 × 4 . . . 31 × 32) = K 1] \Required number of ways = 4P2 ´ 6P3
= K 1 (1 × 2 × 4 . . . 10 × 11 )333 × 311 (3 × 6 × 9 × 12) = 12 ´ 120 = 1440
= K 2333 × 311 × 34 (1 × 2 × 3 × 4) 7. Following are the cases in which the 4-digit numbers
= K 3 × 333 × 311 × 34 × 3 [Q let K 2(1 × 2 × 3 × 4) = K 3 ] strictly greater than 4321 can be formed using digits 0,
1, 2, 3, 4, 5 (repetition of digits is allowed)
= K 3 × 349
Case I
Hence, exponent of 3 is 49. 4 3 2
2. In a nine digits number, there are four even places for 2/3/4/5 4 ways 4 numbers
the four odd digits 3, 3, 5, 5.
4! 5! Case II
\ Required number of ways = × = 60
2!2! 2!3! 4 3
3/4/5 0/1/2/3/4/5 3×6=18 numbers
3. The number will be even, if last digit is either 2, 4, 6 or 8 3 ways 6 ways
i.e. the last digit can be filled in 4 ways and remaining
two digits can be filled in 8 P2 ways. Case III
Hence, required number of numbers of three different 4
digits = 8P2 ´ 4 = 224.
4/5 0/1/2/3/4/5 2×6×6=72 numbers
2 ways 6 ways
4. Case I 1, 1, 1, 1, 1, 2, 3
7! Case IV
Number of ways = = 42
5! 5
6×6×6=216 numbers
Case II 1, 1, 1, 1, 2, 2, 2
7! 0/1/2/3/4/5
Number of ways = = 35 6 ways
4 !3 !
Total number of ways = 42 + 35 = 77 So, required total numbers = 4 + 18 + 72 + 216 = 310
Permutations and Combinations 147

8. Since, the sum of given digits 11. The number of words starting from A are = 5 ! = 120
0 + 1 + 2 + 5 + 7 + 9 = 24 The number of words starting from I are = 5 ! = 120
Let the six-digit number be abcdef and to be divisible
The number of words starting from KA are = 4 ! = 24
by 11, so the difference of sum of odd placed digits and
sum of even placed digits should be either 0 or a The number of words starting from KI are = 4! = 24
multiple of 11 means|(a + c + e) - (b + d + f )|should The number of words starting from KN are = 4 ! = 24
be either 0 or a multiple of 11.
The number of words starting from KRA are = 3 ! = 6
Hence, possible case is
a + c + e = 12 = b + d + f (only) The number of words starting from KRIA are = 2 ! = 2
Now, Case I The number of words starting from KRIN are = 2 ! = 2
set { a , c, e} = {0, 5, 7} and set { b, d , f } = {1, 2, 9} The number of words starting from KRISA are = 1 ! = 1
So, number of 6-digits numbers = (2 ´ 2 !) ´ (3 !) = 24 The number of words starting from KRISNA are
[Q a can be selected in ways only either 5 or 7] = 1! = 1
Case II Hence, rank of the word KRISNA
Set { a , c, e} = {1, 2, 9} and set { b, d , f } = {0, 5, 7} = 2 (120) + 3 (24) + 6 + 2 (2) + 2 (1 ) = 324
So, number of 6-digits numbers = 3 ! ´ 3 ! = 36 12. The number of words starting from E are = 5 ! = 120
So, total number of 6-digits numbers = 24 + 36 = 60 The number of words starting from H are = 5 ! = 120
9. Using the digits 0, 1, 3, 7, 9 The number of words starting from ME are = 4 ! = 24
number of one digit natural numbers that can be The number of words starting from MH are = 4 ! = 24
formed = 4, The number of words starting from MOE are = 3 ! = 6
number of two digit natural numbers that can be The number of words starting from MOH are = 3 ! = 6
formed = 20,
The number of words starting from MOR are = 3 ! = 6
The number of words starting from MOTE are = 2 ! = 2
The number of words starting from MOTHER are
4×5
= 1! = 1
[Q 0 can not come in Ist box]
number of three digit natural numbers that can be Hence, rank of the word MOTHER
formed = 100 = 2 (120) + 2 (24) + 3 (6) + 2 + 1 = 309
13. In out of 10 persons, P1 is always consider and P4 and P5
is not consider.
4×5× 5
and number of four digit natural numbers less than i.e. We have to select 4 persons out of 7 person and after
7000, that can be formed = 250 that they arrange it.
7 ´6 ´5
\Required number of ways = 7C 4 ´ 5 ! = ´ 120
3 ´2 ´1
2×5× 5×5 = 35 ´ 120 = 4200
[Q only 1 or 3 can come in Ist box] x+ 2
14. Hint We have, a = Px + 2 = (x + 2) !
\Total number of natural numbers formed x!
x
and b = P11 = and c = x - 11Px -11 = (x - 11 )!
= 4 + 20 + 100 + 250 = 374 (x - 11 )!
10. In the word INTEGER, we have 5 letters other than ‘I’ 15. At first we have to a accommodate those 5 animals in
and ‘N’ of which two are identical (E’s). We can arrange cages which cannot enter in 4 small cages, therefore
5! number of ways are 6 P5 and rest of the five animals
these letters in ways. In any such arrangements, ‘I’
2! arrange in 5 ! ways.
and ‘N’ can be placed in 6 available gaps in 6 P2 ways.
Total number of ways = 5 ! ´ 6P5 = 120 ´ 720 = 86400
5!
So, required number of ways = × 6P2 = m1. 16. When we arrange one things at a time, the number of
2!
possible permutations is n. When we arrange them two
Now, if word start with ‘I’ and end with ‘R’, then the
at a time the number of possible permutations are
remaining letters are 5.
5! n ´ n = n 2 and so on. Thus, the total number of
So, total number of ways = = m2 permutations are
2!
m1 5 ! 6 ! 2 ! n (n r - 1 )
\ = × × = 30 n + n 2 + ... + n r = [Q n > 1 ]
m2 2 ! 4 ! 5 ! n -1
148 JEE Main Mathematics

10
17. Two positions for A1 and A10 can be selected in C2 Again, 3 can occur in exactly two places in 3 C 2 × (9) such
ways. Rest eight students can be ranked in 8! ways. numbers.
Hence, total number of ways is 10C 2 ´ 8 ! = (1 / 2)(10 !). Lastly ,3 can occur in all the three digits in one such
18. Since, out of eleven members two members sit together, number only 333.
then the number of arrangements = 9 ! ´ 2 \The number of times 3 occurs
[Q two numbers can be sit in two ways] = 1 ´ (3 ´ 92) + 2 ´ (3 ´ 9) + 3 ´ 1 = 300
19. Q Remaining 5 can be seated in 4! ways. 27. The number forms by the figure 4, 5, 6, 7, 8 which is
Now, on cross marked five places 2 person can sit in 5 P2 greater than 56000 is in two cases.
ways. Case I Let the ten thousand digit place number be
P4 greater than 5.
The number of numbers = 3C1 ´ 4C1 ´ 3C1 ´ 2 C1 ´ 1 C1
P3
P5 = 3 ´ 4 ´ 3 ´ 2 ´ 1 = 72
Case II Let the ten thousand digit number be 5 and
thousand digit number be either 6 or greater than 6.
P2
Then, the number of numbers = 3C1 ´ 3C1 ´ 2C1 ´ 1C1
P1
= 3 ´ 3 ´ 2 ´ 1 = 18
5!
So, number of arrangements = 4 ! ´ \ Required number of ways = 72 + 18 = 90
3!
5
= 24 ´ 20 = 480 ways 47 52 - r 47 51 50 49
28. C4 + å C3 = C4 + C3 + C3 + C3
r =1 48 47
20. Since, total members are 15 but three special members + C3 + C3
constitute one member. 51 50 49 48 47 47 52
= C3 + C3 + C3 + C3 + ( C3 + C4 ) = C4
Therefore, required number of arrangements are
12 ! ´ 2 , because, chairman remains between the two 29. n
C3 + nC 4 > n+1
C3
specified persons and the person can sit in two ways. n+1 n+1 n+1
Þ C4 > C3 ( nC r + nC r + 1 = Cr + 1 )
21. There are total 20 + 1 = 21 persons. The two particular n+1
persons and the host be taken as one unit so that these C4 n -2
Þ n+1
>1 Þ >1 Þ n >6
remain 21 - 3 + 1 = 19 persons be arranged in round C3 4
table in 18! ways. But the two persons on either sides of
the host can themselves be arranged in 2! ways.
30. The value of
20
\ Required number of ways = 2 ! ´ 18 ! = 2 × 18 ! å 50 - rC 6 = 50C 6 + 49
C6 + 48
C6 + ¼ + 30
C6
r=0
22. First we fix the alternate position of girls and they 20
arrange in 4! ways and in the five places five boys can be Þ å 50 - rC 6 + 30C7
arranged in 5 P5 ways. r=0
= 30C7 + [30C 6 + 31
C6 + 32
C6 + ¼ + 50
C6 ]
\ Total number of ways = 4 ! ´ 5 P5 = 4 ! ´ 5 ! 20
50 - r 30 31 31 32 50
23. Case I When number in two digits. Þå C6 + C7 = C7 + C 6 + C 6 + ¼+ C 6
9 9 r=0 n+1
Total number of ways = C1 ´ C1 = 9 ´ 9 = 81 [as nC r + nC r + 1 = C r + 1]
20
Case II When number in three digits Similarly, å 50 - rC 6 + 30C7 = 51C7
Total number of ways = 9C1 ´ 9C1 ´ 9C1 = 9 ´ 9 ´ 9 r=0
20
= 729
\ Total number of ways = 81 + 729 = 810
Þ å 50 - rC 6 = 51C7 - 30C7
r=0
Hence, option (b) is correct.
24. (2I , 4F ) + (3I , 6F ) + (4I , 8F )
= 6C 28C 4 + 6C3 8C 6 + 6C 48C 8
31. As each section has 5 questions, so number of ways to
select 5 questions are
= 15 ´ 70 + 20 ´ 28 + 15 ´ 1 S1 S 2 S3
= 1050 + 560 + 15 = 1625
1 1 3
25. Each digit can be placed in 2 ways.
1 3 1
\ Required number of ways = 210
3 1 1
26. Any number between 1 to 999 is a 3-digit number xyz
where the digits x, y, z are any digits from 0 to 9. 1 2 2
Now, we first count the numbers in which 3 occurs once and 2 1 2
only. Since, 3 can occur at one place in 3 C1 ways, there 2 2 1
are 3 C1 × (9 ´ 9) = 3 × 92 such numbers.
Permutations and Combinations 149

\Total number of selection of 5 questions 39. Total number of ways


= 3 ´ ( 5C1 ´ 5C1 ´ 5C3 ) + 3 ´ ( 5C1 ´ 5C 2 ´ 5C 2) = 10C1 + 10
C2 + 10
C3 + 10
C4 + 10
C5 + 10
C 6 + ... + 10
C10
= 3(5 ´ 5 ´ 10) + 3(5 ´ 10 ´ 10) = 750 + 1500 = 2250 =2 10
-1 [Q C 0 + C1 + c2 + ... = 2n ]
n n n

32. Given there are three boxes, each containing 10 balls 40. Any number divisible by 5, if either 0 or 5 in unit place
labelled 1, 2, 3, … , 10.
6 0 or 5
Now, one ball is randomly drawn from each boxes, and
ni denote the label of the ball drawn from the ith box, Th H T U
(i = 1, 2, 3). In unit place, the number of ways = 2C1 = 2
Then, the number of ways in which the balls can be In thousand place, number 6 is fixed. In ten and
chosen such that n1 < n2 < n3 is same as selection of
hundred place the number of ways of selection = 8 ´ 7.
3 different numbers from numbers {1, 2, 3, … , 10}
= 10C3 = 120 \Required number of ways = 2 ´ 8 ´ 7 = 112

33. Total number of available courses = 9 41. Since, questions 1 and 2 are compulsory, so students
Out of these 5 courses have to be chosen. But it is has to select two question in out of three questions.
given that 2 courses are compulsory for every student \Required number of ways = 3C 2 = 3
i. e. , you have to choose only 3 courses instead of 5, out
of 7 instead of 9. 42. It is given that a group of students comprises of 5 boys
7 ´6 ´5 and n girls.
It can be done in 7C3 ways = = 35 ways The number of ways, in which a team of 3 students
6
34. Let total number of persons be n. can be selected from this group such that each team
consists of at least one boy and at least one girls, is
Since, total number of hand shakes = 66
n (n - 1 ) = (number of ways selecting one boy and 2 girls)
n
\ C 2 = 66 Þ = 66 + (number of ways selecting two boys and 1 girl)
2
5 5
Þ n 2 - n - 132 = 0 Þ (n - 12)(n + 11 ) = 0 = ( C1 ´ nC 2) ( C 2 ´ nC1 ) = 1750 [given]
Þ n = 12 [Q n cannot be negative] æ n (n - 1) ö æ 5 ´ 4 ö
Þ ç5 ´ ÷+ç ´ n ÷ = 1750
35. \Required number of ways = 22 - 4 - 2
C11 - 2 = 16C 9 è 2 ø è 2 ø
2
Þ n (n - 1) + 4n = ´ 1750
36. A selection of 3 balls so as to include atleast one black 5
ball, can be made in the following 3 mutually exclusive
ways Þ n 2 + 3n = 2 ´ 350
(i) The number of ways in which 1 black balls and 2 Þ n 2 + 3n - 700 = 0
others are selected = 3C1 ´ 6C 2 = 3 ´ 15 = 45 Þ 2
n + 28n - 25n - 700 = 0
(ii) The number of ways in which 2 black balls and 1 Þ n (n + 28) - 25(n + 28) = 0
other are selected = 3C 2 ´ 6C1 = 3 ´ 6 = 18
Þ n = 25 [Q n Î N ]
(iii) The number of ways in which 3 black balls and no
other are selected = 3C3 = 1 43. Number of girls in the class = 5 and number of boys in
\ Total numbers of ways = 45 + 18 + 1 = 64 the class = 7
Now, total ways of forming a team of 3 boys and 2 girls
37. The number of ways in which we can choose a
committee = Choose two men and four women = 7C3 ×5 C 2 = 350
+ Choose three men and six women But, if two specific boys are in team, then number of
= 4C 2 ´ 6C 4 + 4C3 ´ 6C 6 ways = 5C1 ×5 C 2 = 50
= 6 ´ 15 + 4 ´ 1 = 90 + 4 = 94 Required ways, i.e. the ways in which two specific boys
38. Since, a five digit number is formed using digits { 0, 1, 2, are not in the same team = 350 - 50 = 300.
3, 4 and 5} divisible by 3 i.e., only possible when sum of
Alternate Method
digits is multiple of 3 which gives two cases.
Number of ways when A is selected and B is not
Case I {using digits 0, 1, 2, 4, 5}
= 5C 2 ×5 C 2 = 100
Number of numbers = 4C1 ´ 4C1 ´ 3 C1 ´ 2C1 ´ 1C1
Number of ways when B is selected and A is not
= 4 ´ 4 ´ 3 ´ 2 ´ 1 = 96
= 5C 2 ×5 C 2 = 100
Case II {using digits 1, 2, 3, 4, 5}
Number of ways when both A and B are not selected
Number of numbers = 5C1 ´ 4C1 ´ 3C1 ´ 2 C1 ´1 C1
= 5C3 ×5 C 2 = 100
= 5 ´ 4 ´ 3 ´ 2 ´ 1 = 120
\Required ways = 100 + 100 + 100
\ Total numbers formed = 120 + 96 = 216
= 300
150 JEE Main Mathematics

44. Q 26 cards can be chosen out of 52 cards in 52C 26 ways. Þ n (n - 1 ) - 4n - 66 = 0 Þ n 2 - 5n - 66 = 0


There are two ways in which each card can be dealt Þ (n + 5) (n - 11 ) = 0 Þ n = 11
because a card can be either from the first pack or from \ Number of participants = 11 men + 2 women = 13
the second.
53. There are two cases arise
\ Total number of ways = 52C 26 × 226
Case I They do not invite the particular friend
45. First we fix the alternate position of English books. = 8C 6 = 28
Then, there are 22 vacant places for Hindi books. Case II They invite one particular friend
22 !
Hence, total number of ways = 22C19 = = 1540 = 8C5 ´ 2C1 = 112
3 ! 19 !
\ Required number of ways = 28 + 112 = 140
46. Let there are n teams. 54. Since, the person is allowed to select atmost n coins out
Each team play to every other team in nC 2 ways of (2 n + 1 ) coins, therefore in order to select one, two,
n
\ C 2 = 153 (given) three, ..., n coins. Thus, if T is the total number of ways
n! of selecting atleast one coin, then
Þ = 153
(n - 2)! 2 ! T = 2n + 1C1 + 2n + 1C 2 + ... + 2n + 1
C n = 255 …(i)
Þ n (n - 1 ) = 306 Using the binomial theorem
Þ n 2 - n - 306 = 0 Þ (n - 18) (n + 17) = 0 2n + 1
C0 + 2n + 1
C1 + 2n + 1
C 2 + ... + 2n + 1
Cn + 2n + 1
Cn + 1
Þ n = 18 [Q n is never negative] 2n + 1 2n + 1
+ Cn + 2 + K + C 2n + 1
47. The number of times he will go to the garden is same as = (1 + 1 ) 2n + 1
= 22n + 1
the number of selecting 3 children from 8. 2n + 1
Þ C 0 + 2 (2n + 1C1 + 2n + 1
C 2 + ... + 2n + 1C n )
Therefore, the required number of ways = 8C3 = 56
+ 2n + 1C 2n + 1 = 22n + 1
48. Since, 2 persons can drive the car, therefore we have to Þ 1 + 2 (T ) + 1 = 22n + 1
select 1 from these two. This can be done in 2C1 ways. 22n + 1
Now, from the remaining 5 persons we have to select 2 Þ 1+T = = 22n [from Eq. (i)]
2
which can be done in 5 C 2 ways.
Þ 1 + 255 = 22n
Therefore, the required number of ways in which the car Þ 22n = 28 Þ n = 4
can be filled = 5C 2 ´ 2C1 = 10 ´ 2 = 20
55. First stall can be filled in 3 ways, second stall can be
49. In all, we have 8 squares in which 6 ‘X’ have to be placed filled in 3 ways and so on.
and it can be done in 8C 6 = 28 ways. \ Number of ways of loading steamer
But this includes the possibility that either the top or
= 3C1 ´ 3C1 ´ . . . ´ 3C1 (12 times)
horizontal row does not have any ‘X’. Since, we want
= 3 ´ 3 ´ .... ´ 3 (12 times) = 312
each row must have atleast one ‘X’, these two
possibilities are to be excluded. 56. Q The candidate is unsuccessful, if he fails in 9 or 8 or 7
Hence, required number of ways = 28 - 2 = 26 or 6 or 5 papers.
\ Numbers of ways to be unsuccessful
50. The number of ways that the candidate may select
= 9C 9 + 9C 8 + 9C7 + 9C 6 + 9C5
(i) if 2 questions from A and 4 questions from B
= 9C 0 + 9C1 + 9C 2 + 9C3 + 9C 4
= 5C 2 ´ 5C 4 = 50
1
(ii) 3 questions from A and 3 questions from B = (9C 0 + 9C1 + ... + 9C 9 )
2
= 5C3 ´ 5C3 = 100 1
= (29 ) = 28 = 256
(iii) 4 questions from A and 2 questions from B 2
= 5C 4 ´ 5C 2 = 50 57. In each dye of chosen, there are two possibility either
Hence, total number of ways = 50 + 100 + 50 = 200 chose or reject it.
51. Each question can be answered in 4 ways and all \The total number of ways in which atleast one green
question can be answered correctly in only one way. and one blue dye is chosen
= (25 - 1 )(24 - 1 )23 = 31 ´ 15 ´ 8 = 3720
So, required number of ways = (4C1 )3 - 1 = 43 - 1 = 63
58. The number of ways in which four different balls can be
52. Let there be ‘n’ men participants.
placed in four different boxes
Then, the number of games that the men play between
= 4C1 + 3C1 + 2C1 + 1C1 = 4 + 3 + 2 + 1 = 10
themselves is 2 × nC 2 and the number of games that the
men played with the women is 2 × (2 n ). \ Required number of ways = 10 - 1 = 9
\ 2 × nC 2 - 2 × 2 n = 66 (given) [since, only one way in which the same ball
have a same box]
Permutations and Combinations 151

59. Let the boxes be marked as A , B, C. We have to ensure 64. Total number of books = a + 2 b + 3c + d
that no box remains empty and all five balls have to put \ The total number of arrangements
in. There will be two possibilities. (a + 2 b + 3c + d )!
(i) Any two box containing one ball each and 3rd box =
a !(b !)2(c !)3
containing 3 balls. Number of ways
65. Since, the books consisting of 5 Mathematics, 4 Physics
= A (1 ) B(1 ) C (3)
and 2 Chemistry can be put together of the same subject
= 5C1 ×4 C1 ×3 C3 = 5 × 4 × 1 = 20 in 5 ! 4 ! 2 ! ways.
Since, the box containing 3 balls could be any of the But these subject books can be arranged itself in 3! ways
three boxes A , B, C. Hence, the required number of
\ Required number of ways = 5 ! 4 ! 3 ! 2 !.
ways 20 ´ 3 = 60.
(ii) Any two box containing 2 balls each and 3rd 66. We can think of three packets. One consisting of three
containing 1 ball, the number of ways boys of class X, other consisting of four boys of class XI
and last one consisting of five boys of class XII. These
= A (2) B(2) C (1 ) = 5C 2 ×3 C 2 ×1 C1
packets can be arranged in 3! ways and contents of
= 10 ´ 3 ´ 1 = 30
these packets can be further arranged in 3!4! and 5!
Since, the box containing 1 ball could be any of the three ways, respectively.
boxes A , B, C. Hence, the total number of ways is 3 ! ´ 3 ! ´ 4 ! ´ 5 !.
Hence, the required number of ways = 30 ´ 3 = 90.
67. If the function is one one, then select any three from the
Hence, total number of ways = 60 + 90 = 150. set B in 7C3 ways i.e. 35 ways.
60. For the first player, distribute the cards in 52C17 ways. If the function is many one, then there are two
Now, out of 35 cards left 17 cards can be put for second possibilities. All three corresponds to same element
player in 35 C17 ways. Similarly, for third player put number of such functions = 7C1 = 7 ways.
them in 18C17 ways. One card for the last player can be Two corresponds to same element. Select any two from
put in 1C1 way. Therefore, the required number of ways the set B. The larger one corresponds to the larger and
for the proper distribution the smaller one corresponds to the smaller the third
= 52C17 ´ 35C17 ´ 18C17 ´ 1C1 may corresponds to any two.
52 ! 35 ! 18 ! 52 ! Number of such functions = 7C 2 ´ 2 = 42
= ´ ´ ´1! = So, the required number of mappings = 35 + 7 + 42 = 84
35 ! 17 ! 18 ! 17 ! 17 ! 1 ! (17 !)3
Total arrangement 18 ! 18 ! 68. Total number of functions
61. = =
Equally likely arrangement 6 ! 6 ! 6 ! (6 !)3 = Number of dearrangement of 5 objects
æ1 1 1 1ö
62. Total number of ways = 5! ç - + - ÷ = 44
è 2 ! 3 ! 4 ! 5 !ø
= (Attempt 3 from group I and 4 from group II)
+ (Attempt 4 from group I and 3 from group II) 69. We know, a triangle will be formed by taking three
points at a time.
+ (Attempt 5 from group I and 2 from group II)
\Required number of triangles = 12C3 - 7C3
+ (Attempt 2 from group I and 5 from group II) 12 ´ 11 ´ 10 7 ´ 6 ´ 5
= - = 220 - 35 = 185
= 6C3 ´ 6C 4 + 6C 4 ´ 6C3 + 6C5 ´ 6C 2 + 6C 2 ´ 6C5 3 ´2 ´1 3 ´2
= 2(6C3 ´ 6C 4 ) + 2(6C5 ´ 6C 2) 70. Total number of parallelogram formed
= 2(20 ´ 15) + 2(6 ´ 15) = 4C 2 ´ 3C 2 = 6 ´ 3 = 18
= 600 + 180 = 780 71. Two circles intersect maximum at two distinct points.
63. Required number of ways Now, two circles can be selected in 6C 2 ways.
\ Total number of points of intersection are
= Coefficient of x16 in (x3 + x4 + x5 + ... + x16 )4
6
C 2 ´ 2 = 30
= Coefficient of x16 in x12 (1 + x + x2 + ... + x12)4
72. Let n be the number of diagonals of a polygon.
= Coefficient of x4 in (1 - x13 )4 (1 - x )-4 n
Then, C 2 - n = 44
4 5
= Coefficient of x in (1 - 13x + ... ) n (n - 1 )
Þ - n = 44
é (r + 1 ) (r + 2) (r + 3) r ù 2
´ ê1 + 4x + ... + x ú 2
ë 3! û Þ n - 3n - 88 = 0
(4 + 1 ) (4 + 2) (4 + 3) Þ n = - 8 or 11
= = 35
3! \ n = 11
152 JEE Main Mathematics

73. Required number of diagonals = mC 2 - m 3. Total time required = (total number of dials required to
m(m - 1 ) sure open the lock) ´ 5 s
= -m
2! 5 500000
m = 10 ´ 5 s = days
= (m - 3) 60 ´ 60 ´ 13
2!
= 10.7 days
74. The triangle will be formed by joining any three Hence, 11 days are enough to open the safe.
non-collinear points.
\Required number of ways = 8C3 - 5C3 - 3C3 4. Let n be the number of sides of the polygon.
n × 160° = (n - 2) × 180° Þ 20° × n = 360°
= 8C3 - 5C3 - 1
\ n = 18
75. Total number of points are m + n + k, the triangles
Then number of diagonals = 18C 2 - 18 = 153 - 18 = 135
formed by these points = m + n + k C3
Joining of three points on the same line gives no 5. Required number of ways = 9C 4 = 126
triangle, the number of such triangles is
m 6. Total number of m elements subsets of A = nCm …(i)
C3 + nC3 + kC3
\ Required number of triangles and number of m elements subsets of A each containing
= m + n + k C3 - mC3 - nC3 - kC3 the element a 4 = n - 1Cm - 1
According to the question, nCm = l × n - 1Cm - 1
76. Number of lines from 6 points = 6C 2 = 15 n n -1
Þ × Cm - 1 = l × n - 1Cm - 1
Points of intersection obtained from these lines m n
= 15C 2 = 105 Þ l= Þ n = ml
m
Now, we find the number of times, the original 6 points
come. 7. Number of ways of giving one prize for running = 16
Consider one point say A1. Joining A1 to remaining 5 Number of ways of giving two prizes for swimming
points, we get 5 lines and any two lines from these 5 = 16 ´ 15
lines gives A1 as the point of intersection.
Number of ways of giving three prizes for riding
\ A1 is common in 5 C 2 = 10 times out of 105 points of
intersections. = 16 ´ 15 ´ 14
Similar is the case with other five points. \Required ways of giving prizes
\ 6 original points come 6 ´ 10 = 60 times in points of = 16 ´ 16 ´ 15 ´ 16 ´ 15 ´ 14
intersection. = 163 ´ 152 ´ 14
Hence, the number of distinct points of intersection
8. The number of ways of selecting four numbers from 1 to
= 105 - 60 + 6 = 51
30 without any restriction is 30C 4. The number of ways
77. The required number of points of selecting four consecutive [i.e., (1, 2, 3, 4), (2, 3, 4, 5),
= 8C 2 ´ 1 + 4C 2 ´ 2 + ( 8C1 ´ 4C1 ) ´ 2 …, (27, 28, 29, 30)] number is 27.
Hence, the number of ways of selecting four integers
= 28 + 12 + 32 ´ 2 = 104
which excludes consecutive four selections is
78. Since, there are n distinct points on a circle. 30 30 ´ 29 ´ 28 ´ 27
C 4 - 27 = - 27 = 27378
For making a pentagon it requires a five points. 24
According to given condition,
n 9. Other than 2, remaining five places can be filled by 1
C5 = nC3 Þ n = 8
and 3 for each place. The number of ways for five places
Round II is 2 ´ 2 ´ 2 ´ 2 ´ 2 = 25 .
For 2, selecting 2 places out of 7 is 7C 2.
1. We have, 32 places for teeth. For each place, we have
two choices either there is a tooth or there is no tooth. Hence, for required number of ways is 7C 2 ´ 25 .
n
Therefore, the number of ways to fill up these places is 10. Hint P3 - nC3 > 100
232. As there is no person without a tooth, the maximum n! n!
population is 232 - 1. Þ - > 100
(n - 3)! 3 !(n - 3)!
2. For length, number of choices is
11. The number of ways he can select atleast one parantha
(2 m - 1 ) + (2 m - 3) + ... + 3 + 1 = m2 is 23 - 1 = 7. The number of ways he can select atleast
Similarly, for breadth number of choices is one vegetable dish is 24 - 1 = 15. The number of ways be
(2 n - 1 ) + (2 n - 3 ) + ... + 3 + 1 = n 2 can select zero or more items from salads and sauces
is 25 .
Hence, required number of choices is m2n 2. Hence, the total number of ways is 7 ´ 15 ´ 32 = 3360
Permutations and Combinations 153

12. Select any three points from total 3 p points, which can m!
Þ = m ´ 2 + 42
be done in 3 pC3 ways. But this also includes selection of 2 !(m - 2)!
three collinear points.
Þ m(m - 1) = 4m + 84
Now, three collinear points from each straight line can
Þ m2 - m = 4m + 84
be selected in pC3 ways.
Þ m2 - 5m - 84 = 0
Then, the number of triangles is 2
3p Þ m - 12m + 7m - 84 = 0
C3 - 3 × pC3 = p2(4 p - 3)
Þ m(m - 12) + 7 (m - 12) = 0
13. No group of four numbers from the first 12 natural
Þ m = 12 [Q m > 0]
numbers can have the common difference 4.
If one group including 1 is selected with the common 17. Given, X has 7 friends, 4 of them are ladies and 3 are
difference 1, then the other two group can have the men while Y has 7 friends, 3 of them are ladies and 4 are
common difference 1 or 2. men.
If one group including 1 is selected with the common \Total number of required ways
difference 2, then one of the other two group can have = 3C3 ´ 4C 0 ´ 4C 0 ´ 3C3 + 3C 2 ´ 4C1 ´ 4C1 ´ 3C 2
the common difference 2 and the remaining group will + 3C1 ´ 4C 2 ´ 4C 2 ´ 3C1
have common difference 1. + 3C 0 ´ 4C3 ´ 4C3 ´ 3C 0
If one group including 1 is selected with the common = 1 + 144 + 324 + 16 = 485
difference 3, then the other two groups can have the
4!
common difference 3. 18. Clearly, number of words start with A = = 12
2!
Therefore, the required number of ways is 2 + 1 + 1 = 4.
Number of words start with L = 4 ! = 24
52
14. 20 cards can be chosen out of 52 cards in C 20 ways. 4!
There are two ways in which each card can be dealt, Number of words start with M = = 12
2!
because a card can be either from the first pack or from 3!
the second. Number of words start with SA = =3
2!
Hence, the total number of ways is 52C 20 ´ 220. Number of words start with SL = 3 ! = 6
15. Given digits are 1, 1, 2, 2, 2, 2, 3, 4, 4. Note that, next word will be “SMALL”.
According to the question, odd numbers 1, 1, 3 should Hence, the position of word “SMALL” is 58th.
occur at even places only.
19. Given, n ( A ) = 2, n (B ) = 4\ n ( A ´ B ) = 8
The number of subsets of A ´ B having 3 or more
elements
= 8C3 + 8C 4 + . . . + 8C 8
= (8C 0 + 8C1 + 8C 2 + 8C3 + . . . + 8C 8 )
Even places
- (8C 0 + 8C1 + 8C 2)
\The number of ways to arrange odd numbers at even n n n n
3! \ 2 = C 0 + C1 + . . . + C n
places are 4C3 ´ = 28 - 8C 0 - 8C1 - 8 C 2
2!
and the number of ways to arrange remaining even = 256 - 1 - 8 - 28 = 219
6! 20. Digits are 1, 2, 2, 3
numbers are .
4 !2 ! 4!
Total distinct numbers = = 12
So, total number of 9-digit numbers, that can be 2!
formed using the given digits are
Total numbers when 1 at unit place is 3.
4 3! 6!
C3 ´ ´ = 4 ´ 3 ´ 15 = 180 2 at unit place is 63 at unit place is 3.
2 ! 4 !2 !
So, sum = (3 + 12 + 9) (103 + 102 + 10 + 1)
16. Since, there are m-men and 2-women and each
= (1111) ´ 24
participant plays two games with every other
participant. = 26664
\Number of games played by the men between 21. The number of ways to choose zero or more white balls
themselves = 2 ´ mC 2 = (10 + 1 )
and the number of games played between the men and [Q all white balls are mutually identical]
the women = 2 ´ mC1 ´ 2C1 Number of ways to choose zero or more green balls
Now, according to the question, = (9 + 1 )
2 mC 2 = 2 mC1 2C1 + 84 [Q all green balls are mutually identical]
154 JEE Main Mathematics

Number of ways to choose zero or more black balls 26. \Required number of ways = 12C 4 ´ 8C 4 ´ 4C 4
= (7 + 1) 12 ! 8! 12 !
= ´ ´1 =
[Q all black balls are mutually identical] 8 !4 ! 4 !4 ! (4 !)3
Hence, number of ways to choose zero or more balls of 27. Given word in MOTHER, now alphabetical order of
any colour = (10 + 1 ) (9 + 1 ) (7 + 1 ) letters is EHMORT, so number of words start with
Also, number of ways to choose a total of zero balls = 1 letter.
Hence, the number of ways to choose atleast one ball E ------ is 5! H ------ is 5!
(irrespective of any colour) M E ------ is 4! M H ------ is 4!
= (10 + 1 ) (9 + 1 ) (7 + 1 ) - 1 = 880 - 1 = 879 M O E ------ is 3! M O H ------ is 3!
22. The number of different ordered pairs (Y , Z ) such that M O R ------ is 3! M O T E ------ is 2!
Y Í X , Z Í X and Y Ç Z = f . M O T H E R is 1
Since, Y Í X , Z Í X , hence we can only use the So, position of the word ‘MOTHER’ is
elements of X to construct sets Y and Z. 5! + 5! + 4! + 4! + 3! + 3! + 3! + 2! + 1
n(Y ) Number of ways to Number of ways to = 120 + 120 + 24 + 24 + 6 + 6 + 6 + 2 + 1 = 309
make Y make Z such that 28. Given word is LETTER, having vowels E, E and
YÇZ =f
consonants L, T, T, R.
5
0 C0 25
Now, the number of ways to arrange the consonants are
5
1 C1 24 4!
= 12.
2 5
C2 23 2!
3 5
C3 22 Now, we have five place to put vowels E, E.
5
4 C4 21 × × × ×
5 0
5 C5 2

Hence, total number of ways to construct sets Y and Z So, number of ways to arrange vowels is, 5 C 2 = 10
such that Y Ç Z = f is So, number of required words = 12 ´ 10 = 120.
5 5 5
C 0 ´ 2 + C1 ´ 2 5 -1 5
+ K + C5 ´ 2 5 -5 5
= (2 + 1 ) = 3 5 29. The given word is ‘SYLLABUS’ having letters SS, LL,
ABUY.
23.
Now, number of ways to select two like letters are 2C1.
And number of ways to select two distinct letters
3 distinct 9 distinct are 5 C 2.
red balls blue balls
And number of ways to permutate the 2 like letters
Urn A Urn B 4!
and 2 distinct letters is .
The number of ways in which two balls from urn A and 2!
two balls from urn B can be selected 4!
So, number of required words = 2C1 ´ 5C 2 ´
= 3C 2 ´ 9C 2 = 3 ´ 36 = 108 2!
24. The number of ways in which 4 novels can be selected = 2 ´ 10 ´ 12
6
= C 4 = 15 = 240
The number of ways in which 1 dictionary can be 30. Number of ways to select four questions from six
selected = 3C1 = 3 questions = 6C 4
4 novels can be arranged in 4! ways. And number of ways to answer these questions
\ The total number of ways = 15 ´ 4 ! ´ 3 = 15 ´ 24 ´ 3
correctly = 1 ´ 1 ´ 1 ´ 1 = 1
= 1080
And number of ways to answer remain two questions
25. Given word is MISSISSIPPI. wrongly = 3 ´ 3 = 9
Here, I = 4 times, S = 4 times, \ Required number of ways = 6C 4 ´ 1 ´ 9
P = 2 times, M = 1 time 6!
= ´9
_M_I_I_I_I_P_P_ 2!4!
7! 7 ´6! 6 ´5
Required number of words = 8C 4 ´ = 8C 4 ´ = ´ 9 = 135
4!2! 4!2! 2
= 7 × 8C 4 × 6C 4
Permutations and Combinations 155

31. Given word is ‘EXAMINATION’ having letters (AA), 33. Given, 5 Indian and 5 American couples meet at party
(II), (NN), EXMOT, we have to form 4 letter words, then and shake hand.
following cases are possible \ Total number of possible hand shakes = 20C 2
(I) 2 same, 2 same and number of words are Number of hand shakes, when wife shake hand with
3 4! her own husband = 10 (5 Indian and 5 American)
C2 ´ = 18
2 !2 ! Number of hand shakes when Indian wife shakes hand
(II) 2 same, 2 different and number of words are with a male = 5C1 ´ 10C1 = 50 (it include the case where
3 4! 7 ´6 4 ´3 ´2 the Indian wife shake hand with her own husband)
C1 ´ 7C 2 ´ = 3 ´ ´
2! 2 2 \ Total number of hand shakes that take place in the
= 21 ´ 36 = 756 party = 20C 2 - 5 - 50 = 135
(III) All are different and number of words are = 15 ´ 9 = 135
8 8 ´ 7 ´6 ´5 34. There are 3 choices for the first of n-letter and two
C4 ´ 4 ! = 4 ! = 56 ´ 30 = 1680
4! choices for each subsequent letters.
So, total number of 4 letter words are Hence, using fundamental principle, number of good
18 + 756 + 1680 = 2454 words = 3 × 2n-1 = 384
Hence, answer is 2454. = 2n-1 = 128 = 27
32. Let the first locate vowels in alphabetical order at any 4 \ n =8
places out of 8 and this can be done in 8C 4 ways.
35. Number of dearrangements in such problems is given
Now, we left with 4 letters R, B, N, D and 4 places that by
can be filled in 4! ways. 1 1 1 1 1ü
ì
\ Number of permutation of the word ‘AUROBIND’ in n ! í1 - + - + -...+ (-1)n ý
î 1 ! 2 ! 3 ! 4 ! n !þ
which vowels appear in an alphabetical order = 8C 4 ´ 4 !
Hence, the required number of dearrangements is
8! 8!
= ´4 ! = = 8 ´ 7 ´ 6 ´ 5 = 1680 ì 1 1 1ü
4 !´ 4 ! 4! 4 ! í - - ý = 12 - 4 + 1 = 9
î2 ! 3 ! 4 þ
07
Mathematical
Induction
The word induction means the generalization from particular cases or facts. IN THIS CHAPTER ....
In algebra or in other disciplines of mathematics, there are certain results
Peano’s Theorem
or statements that are formulated in terms of n, where n is a positive integer.
To prove such statements the well suited principle that is used based on Principle of Mathematical
specific technique, is the principle of mathematical induction. Induction
Simple Applications of
Peano’s Theorem Mathematical Induction

The statement or proposition P ( n ) is assumed to be true for all values of


n ÎN,
(a) P(1) is true
(b) P ( m ) is true Þ P( m + 1) is true

Principle of Mathematical Induction


Suppose there is a given statement P ( n ) involving the natural number n such
that
(i) The statement is true for n = 1, i. e. , P(1) is true and
(ii) If the statement is true for n = k (where, k is some positive integer), then
the statement is also true for n = k + 1, i. e. ,truth of P ( k) implies the truth
of P ( k + 1).
Then, P ( n ) is true for all natural numbers n.

Example 1. For all n Î N, 1 ´ 1! + 2 ´ 2 ! + 3 ´ 3 ! + ...+ n ´ n ! is equal to


(a) (n + 1) ! - 2 (b) (n + 1) ! (c) (n + 1) ! - 1 (d) (n + 1) ! - 3
Sol. (c) Let the statement P(n) be defined as
P(n) : 1 ´ 1! + 2 ´ 2 ! + 3 ´ 3 ! K + n ´ n ! = (n + 1) ! - 1
for all natural numbers n.
Mathematical Induction 157

Note that P(1) is true, since k2(k + 1) 2 (k + 1)3


= +
P (1) : 1 ´ 1! = 1 = 2 - 1 = 2 ! - 1 4 1
Assume that P(n) is true for some natural number k, i.e. k (k + 1) + 4(k + 1)3
2 2
=
P(k) : 1 ´ 1! + 2 ´ 2 ! + 3 ´ 3 ! + .... + k ´ k ! = (k + 1) ! - 1 …(i) 4
To prove P(k + 1) is true, we have On taking (k + 1) 2 common in numerator part,
P(k + 1) : 1 ´ 1! + 2 ´ 2 ! + 3 ´ 3 ! + ... + k ´ k ! + (k + 1) ´ (k + 1) !
= (k + 1) ! - 1 + (k + 1) ! ´ (k + 1) [by Eq. (i)] (k + 1) 2[k2 + 4(k + 1)]
=
= (k + 1 + 1)(k + 1) ! – 1 4
= (k + 2 )(k + 1) ! – 1 (k + 1) 2(k2 + 4k + 4)
=
= (k + 2 ) ! – 1 4
Thus, P(k + 1) is true, whenever P(k) is true. (k + 1) 2(k + 2) 2
Therefore, by the principle of mathematical induction,P(n) is =
4
true for all natural numbers n.
(k + 1) 2[(k + 1) + 1]2
Example 2. For all n Î N, 33n - 26 n - 1 is divisible by =
4
(a) 24 (b) 64 (c) 17 (d) 676 2
é (k + 1){(k + 1) + 1} ù
=ê úû
Sol. (d) We have, ë 2
33n - 26n - 1 = 27 n - 26n - 1 Therefore, P(k + 1) is true when P(k) is true.
3n n
Þ 3 - 26n - 1 = (1 + 26) - 26n - 1 Hence, from the principle of mathematical induction, the
3n statement is true for all natural numbers n.
Þ 3 - 26n - 1 = nC 2 ´ 26 2 + nC3 ´ 263 + K + nC n ´ 26 n
Clearly, RHS is divisible 26 2 i.e. 676. Type II : Use Divisibility Type Problems
This can be explained by given example.
Simple Applications of
Example 4. Consider the statement : ‘‘P(n) : n2 - n + 41 is
Mathematical Induction prime.’’ Then, which one of the following is true?
Mathematical induction can be used in various types of (JEE Main 2019)
problems. (a) Both P(3) and P(5) are true
(b) P(3) is false but P(5) is true
Type I : Use Identity Type Problems (c) Both P(3) and P(5) are false
This can be explained by given example. (d) P(5) is false but P(3) is true

Example 3. The identity 13 + 23 + 33 + . . . + n3 is equal to Sol. (a) Given statement is “P(n) : n 2 - n + 41 is prime”.
2 Clearly P(3) : 3 2 - 3 + 41 = 9 - 3 + 41 = 47
é n(n - 1) ù n(n + 1)
(a) ê (b)
ë 2 úû 2 which is a prime number.
2 and P(5) : 5 2 - 5 + 41 = 25 - 5 + 41 = 61,
{n(n + 1)} 2 é n(n + 1) ù
(c) (d) ê
2 ë 2 úû which is also a prime number.
\ Both P(3) and P(5) are true.
Sol. (d) Let the given statement be P(n).
é n(n + 1) ù
2 Example 5. 23n - 7n - 1 is divisible by
i.e. P(n) : 13 + 23 + 33 + . . . + n3 = ê
ë 2 úû (a) 64 (b) 36
Step I For n = 1, (c) 49 (d) 25
2 2
( + 1) ù
é11 é1 ´ 2 ù Sol. (c) Let P(n) = 23n - 7n - 1
P(1) : ê =ê = 12 = 1 = 13 , which is true.
ë 2 úû ë 2 úû Þ P(1) = 0 , P(2) = 49
Step II Let it is true for n = k, P(1) and P(2) are divisible by 49.
é k(k + 1) ù
2 Let P(k) = 23k - 7k - 1 = 49I
i.e. 13 + 23 + 33 + . . . + k 3 = ê …(i)
ë 2 úû P (k + 1) = 23k + 3 - 7k - 8
3 3 3 3
Step III For n = k + 1,(1 + 2 + 3 + 4 + . . . + k ) + (k + 1) 3 3
= 8 ( 49I + 7k + 1) - 7k - 8
2 = 49(8I) + 49k = 49l
é k(k + 1) ù
=ê + (k + 1)3 [using Eq. (i)] where, l = 8I + k, which is an integer.
ë 2 úû
158 JEE Main Mathematics

Type III : Use Inequality Type Problems Type IV : Use Recursion Type of Problems
This can be explained by given example. This can be explained by given example.

Example 6. The sum of series 1 + 2 + 3 + K n is less than Example 7. If u n + 1 = 3u n - 2u n - 1 and u 0 = 2, u1 = 3, then u n


1 1 is equal to
(a) (2n + 1) 2 (b) (2n + 1) 2
8 12 (a) 1 - 2 n (b) 2 n + 1
1
(c) (2n + 1) (d) None of these n
(c) 2 - 1 (d) 2 n + 2
8
Sol. (a) Let the given statement be P(n). Sol. (b) Q un + 1 = 3un - 2un - 1 …(i)
1
1 Step I Given, u1 = 3 = 2 + 1= 2 + 1, which is true for n = 1,
i.e. P(n) : 1 + 2 + 3 + . . . + n < (2n + 1) 2
8 Put n = 1in Eq. (i),
Step I For n = 1, then u1 + 1 = 3u1 - 2u1 - 1
1 1 9 Þ u2 = 3u1 - 2u0
1 < (2 ×1 + 1) 2 Þ1 < ´ 3 2 Þ1 <
8 8 8 = 3 ×3 - 2 ×2 = 5
which is true. = 22 + 1
Step II Let it is true for n = k, which is true for n = 2.
1
1 + 2 + 3 + . . . + k < (2k + 1) 2 …(i) Therefore, the result are true for n = 1and n = 2.
8 Step II Assume it is true for n = k, then it is also true for
Step III For n = k + 1, n = k - 1.
1 uk = 2 k + 1
(1 + 2 + 3 + . . . + k) + (k + 1) < (2k + 1) 2 + (k + 1) Then, …(ii)
8
and uk - 1 = 2 k - 1 + 1 …(iii)
[using Eq. (i)]
(2k + 1) 2 k + 1 (2k + 1) 2 + 8k + 8 Step III On putting n = k in Eq. (i), we get
= + = uk + 1 = 3uk - 2uk - 1
8 1 8
4k2 + 1 + 4k + 8k + 8 4k2 + 12k + 9 = 3(2 k + 1) - 2(2 k - 1 + 1) [from Eqs. (ii) and (iii)]
= =
8 8 = 3 × 2k + 3 - 2 × 2k - 1 - 2
(2k + 3 ) 2 (2k + 2 + 1) 2 [2(k + 1) + 1]2 = 3 × 2k + 3 - 2k - 2
= = =
8 8 8 = (3 - 1)2 k + 1
[2(k + 1) + 1]2 = 2 × 2k + 1
Þ 1 + 2 + 3 + . . . + k + (k + 1) <
8
= 2k + 1 + 1
Therefore, P(k + 1) is true when P(k) is true.
This shows that the result is true for n = k + 1, hence by the
Hence, from the principle of mathematical induction, the
principle of mathematical induction the result is true for all
statement is true for all natural numbers n.
n Î N.
Practice Exercise
ROUND I Topically Divided Problems

Principle of Mathematical Induction 8. By principle of mathematical induction


cos q cos 2 q cos 4 q K cos ( 2 n - 1q ), " n Î N is equal to
1. For n Î N , æç ö÷ n5 + æç ö÷ n3 + æç ö÷ n is
1 1 7
è5 ø è3ø è 15 ø sin 2n q cos 2n q
(a) (b)
(a) an integer 2n sin q 2n sin q
(b) a natural number sin 2n q
(c) (d) None of these
(c) a positive fraction 2n - 1 sin q
(d) None of the above
2n + 1ö Applications of
2. The value of æç1 + ö÷ æç1 + ö÷ æç1 + ö÷ . . . æç1 +
3 5 7
÷
è 1ø è 4ø è 9ø è 2
n ø Mathematical Induction
is n2 ( n + 1)
(a) (n + 1) (b) (n + 1) 2 9. If 1 + 5 + 12 + 22 + 35 + . . . to n terms = ,
2
(c) 2(n + 1) 2 (d) None of these nth term of series is
3. For all n Î N , n ( n + 1)( n + 5) is a multiple of n (4n - 1) n (3n - 1)
(a) (b)
3 2
(a) 4 (b) 3 (c) 5 (d) 7 n (3n + 1) n (4n + 1)
2 3
(c) (d)
4. The sum of series 1 × 3 + 2 × 3 + 3 × 3 + . . . + n × 3n is 2 3
(2n + 1 )3n + 1 + 3 (2n - 1 )3n + 1 + 3 10. The smallest positive integer n for which
(a) (b) n
4 4 æ n + 1ö
n! < ç ÷ holds, is
(2n - 1 )3n - 1 - 3 è 2 ø
(c) (d) None of these
4 (a) 1 (b) 2 (c) 3 (d) 4
5. If P ( n) is a statement such that P (3) is true. n -1
11. The inequality n ! > 2 is true for
Assuming P ( k) is true Þ P ( k + 1) is true for all (a) n > 2 (b) n Î N
k ³ 3, then P ( n) is true (c) n > 3 (d) None of these
(a) for all n (b) for n ³ 3
12. The greatest positive integer, which divides
(c) for n > 4 (d) None of these
( n + 2) ( n + 3) ( n + 4) ( n + 5) ( n + 6) for all n Î N , is
6. Let P ( n) : n2 + n + 1 is an even integer. If P ( k) is (a) 4 (b) 120
assumed true Þ P ( k + 1) is true. Therefore, P ( n) is (c) 240 (d) 24
true 13. If m, n are any two odd positive integer with n < m,
(a) for n > 1 (b) for all n Î N then the largest positive integers which divides all
(c) for n > 2 (d) None of these the numbers of the type m 2 - n2 is
7. By mathematical induction (a) 4 (b) 6 (c) 8 (d) 9
n
1 1 1 4 ( 2 n) !
+ +K+ is equal to 14. If < , then P ( n) is true for
1× 2 ×3 2 ×3× 4 n( n + 1 ) ( n + 2) n + 1 ( n !) 2
n (n + 1 ) n (n + 3) (a) n ³ 1 (b) n > 0
(a) (b)
4(n + 2) (n + 3) 4(n + 1 )(n + 2) (c) n < 0 (d) n ³ 2
n (n + 2) n
(c) (d) None of these 15. If an = 2 2 + 1, then for n > 1, last digit of an is
4(n + 1 ) (n + 3)
(a) 3 (b) 5 (c) 7 (d) 8
160 JEE Main Mathematics

16. For all n Î N , 41n - 14 n is a multiple of 20. x ( x n - 1 - na n - 1) + a n ( n - 1) is divisible by ( x - a ) 2


(a) 26 (b) 27 for
(c) 25 (d) None of these (a) n > 1 (b) n > 2
(c) all n Î N (d) None of these
17. By principle of mathematical induction
32 n + 2 - 8 n - 9 is divisible by 21. If x 2 n - 1 + y 2 n - 1 is divisible by x + y, if n is
(a) 9 (b) 8 (c) 7 (d) 6 (a) a positive integer
(b) an even positive integer
18. 2 3n - 7 n - 1 is divisible by
(c) an odd positive integer
(a) 64 (b) 36 (c) 49 (d) 25
(d) None of the above
19. For each n Î N , 32 n - 1 is divisible by 22. If 49 n + 16 n + k is divisible by 64 for n Î N , then the
(NCERT Exemplar)
(a) 8 (b) 16 least negative integral value of k is (NCERT Exemplar)
(c) 32 (d) None of these (a) –1 (b) –2 (c) –3 (d) –4

ROUND II Mixed Bag


Only One Correct Option 7. For all positive integers n > 1,
1. For all positive integral values of n, 3 2n
- 2 n + 1 is { x ( x n -1 - na n -1) + a n ( n - 1)} is divisible by
divisible by (a) (x - a )2 (b) x - a
(a) 2 (b) 4 (c) 8 (d) 12 (c) 2(x - a ) (d) x + a

2. If n Î N , then 11 n +2
+ 12 2n +1
is divisible by 8. Which of the following result is valid?
(a) (1 + x)n > (1 + nx), for all natural numbers n
(a) 113 (b) 123
(b) (1 + x)n ³ (1 + nx), for all natural numbers n,
(c) 133 (d) None of these
where x > - 1
3. For a positive integer n, (c) (1 + x)n £ (1 + nx), for all natural numbers n
1 1 1 1 (d) (1 + x)n < (1 + nx), for all natural numbers n
let a ( n) = 1 + + + +K+ n . Then,
2 3 4 (2 ) - 1 9. Let us consider the series Sn = 2 × 7 n + 3 × 5 n - 5.
(a) a (100) £ 100 (b) a (100) > 100
Then, Sn is divisible by the multiple of
(c) a (200) £ 100 (d) None of these
(a) 5 (b) 7
4. 10 n + 3 ( 4 n + 2 ) + 5 is divisible by ( n Î N ) (c) 24 (d) None of these
(a) 7 (b) 5 (c) 9 (d) 17 10. If n is a natural number, then
5. Let P ( n) denotes the statement that n + n is odd. 2
(a) 12 + 22 + ××× + n 2 < n3 /3 (b) 12 + 22 + ××× + n 2 = n3 /3
It is seen that P ( n) Þ P ( n + 1), P ( n) is true for all (c) 12 + 22 + ××× + n 2 > n3 (d) 12 + 22 + ××× + n 2 > n3 /3
(a) n > 1 (b) n 11. Using mathematical induction, the numbers an ’s
(c) n > 2 (d) None of these
are defined by a0 = 1, an + 1 = 3n2 + n + an , ( n ³ 0)
99
6. The remainder when 5 is divided by 13, is Then, an is equal to
(a) 6 (b) 8 (c) 9 (d) 10 (a) n3 + n 2 + 1 (b) n3 - n 2 + 1
(c) n3 - n 2 (d) n3 + n 2

Answers
Round I
1. (b) 2. (b) 3. (b) 4. (b) 5. (b) 6. (d) 7. (b) 8. (a) 9. (b) 10. (b)
11. (a) 12. (b) 13. (c) 14. (d) 15. (c) 16. (b) 17. (b) 18. (c) 19. (a) 20. (c)
21. (a) 22. (a)

Round II
1. (a) 2. (c) 3. (a) 4. (c) 5. (d) 6. (b) 7. (b) 8. (b) 9. (c) 10. (d)
11. (b)
Solutions
Round I Þ k(k2 + 5k + k + 5) = 3l
æ1ö æ1ö æ 7ö Þ k3 + 6k2 + 5k = 3l …(i)
1. Let P (n ) : ç ÷ n5 + ç ÷ n3 + ç ÷n
è5ø è3ø è 15 ø Step III For n = k + 1,
æ1ö æ1ö æ 7ö (k + 1 )(k + 1 + 1 )(k + 1 + 5)
\ P(1 ) = ç ÷ (1 )5 + ç ÷ (1 )3 + ç ÷1
è5ø è3ø è 15 ø = (k + 1 )(k + 2)(k + 6) = (k2 + 2k + k + 2)(k + 6)
1 1 7 3 + 5 + 7 15 = (k2 + 3k + 2)(k + 6) = k3 + 6k2 + 3k2 + 18k + 2k + 12
= + + = = = 1 ÎN
5 3 15 15 15 = (k3 ) + 9k2 + 20k + 12
æ1ö æ1ö æ 7ö = (3l - 6k2 - 5k) + 9k2 + 20k + 12 [using Eq. (i)]
P(2) = ç ÷ (2)5 + ç ÷ (2)3 + ç ÷2
è5ø è3ø è 15 ø 2
= 3l + 3k + 15k + 12 = 3(l + k + 5k + 4) 2

32 8 14 which is a multiple of 3.
= + +
5 3 15 Therefore, P (k + 1 ) is true when P (k) is true. Hence,
96 + 40 + 14 150 from the principle of mathematical induction, the
= = = 10 Î N
15 15 statement is true for all natural numbers n.
\By mathematical induction P (n ) is a natural number. 4. Let the given statement be P (n ). i.e.
2. Let the given statement be P (n ). i.e. (2n - 1 )3n + 1 + 3
P (n ) : 1 × 3 + 2 × 32 + 3 × 33 + . . . + n × 3n =
æ 3ö æ 5ö æ 7ö æ 2n + 1 ö 2 4
P (n ) : ç1 + ÷ ç1 + ÷ ç1 + ÷ . . . ç1 + ÷ = (n + 1 )
è 1 ø è 4 ø è 9 ø è n2 ø Step I For n = 1,
æ 3ö
(2 × 1 - 1 )31 + 1 + 3 32 + 3 9 + 3 12
Step I For n = 1, i.e., P(1 ) = (1 + 1 )2 = 22 = 4 = ç1 + ÷
è 1ø P(1 ) : = = =
4 4 4 4
which is true. Þ 3 = 1 ×3
Step II Let it is true for n = k, which is true.
æ 3ö æ 5ö æ 7ö æ 2k + 1 ö Step II Let it is true for n = k,
i.e. ç1 + ÷ ç1 + ÷ ç1 + ÷ . . . ç1 + ÷
è 1 ø è 4 ø è 9 ø è k2 ø (2k - 1 )3k + 1 + 3
i.e. 1 × 3 + 2 × 32 + 3 × 33 + . . . + k × 3k =
= (k + 1 )2 …(i) 4
Step III For n = k + 1, …(i)
ìæ 3ö æ 5ö æ 7ö æ 2k + 1 öü ì 2k + 2 + 1ü Step III For n = k + 1,
í çè1 + ÷ø çè1 + ÷ø çè1 + ÷ø . . . çè1 + 2 ÷ø ý í1 + ý (1 × 3 + 2 × 32 + 3 × 33 + . . . + k × 3k ) + (k + 1 )3k + 1
î 1 4 9 k þî (k + 1 )2 þ
ì 2k + 3 ü (2k - 1 )3k + 1 + 3
= (k + 1 )2 í1 + = + (k + 1 )3k + 1[using Eq. (i)]
ý [using Eq. (i)] 4
î (k + 1 )2 þ
(2k - 1 )3k + 1 + 3 + 4(k + 1 )3k + 1
é (k + 1 )2 + 2k + 3 ù =
= (k + 1 )2 ê 2
ú = k + 2k + 1 + 2k + 3 4
2
ë (k + 1 ) û On taking 3k + 1 common in first and last term of
= (k + 2)2 = [(k + 1 ) + 1]2 numerator part,
[Q (a + b)2 = a 2 + 2ab + b2] 3k + 1 (2k - 1 + 4k + 4) + 3 3k + 1 (6k + 3) + 3
= =
4 4
Therefore, P (k + 1 ) is true when P (k) is true. Hence,
from the principle of mathematical induction, the On taking 3 common in first term of numerator part,
statement is true for all natural numbers n. 3k + 1 × 3(2k + 1 ) + 3
=
3. Let the given statement be P (n ). 4
( k + 1) + 1
3 (2k + 2 - 1 ) + 3
i.e. P (n ) : n (n + 1 )(n + 5) is a multiple of 3. =
4
Step I For n = 1,
{ 2(k + 1 ) - 1 }3( k + 1) + 1 + 3
1 (1 + 1 )(1 + 5) = 1 ´ 2 ´ 6 = 12 = 3 ´ 4 =
4
which is a multiple of 3, that is true.
Therefore, P (k + 1 ) is true when P (k) is true.
Step II Let it is true for n = k,
Hence, from the principle of mathematical induction,
i.e. k(k + 1 )(k + 5) = 3l
the statement is true for all natural numbers n.
162 JEE Main Mathematics

5. Since, P(3) is true. \ P(1 ) is true.


Assume P (k) is true Þ P (k + 1 ) is true means, if P(3) is Step II Let P (k) is true, then
true Þ P(4) is true Þ P(5) is true and so on. So, sin 2k q
P (k) : cos q cos 2q cos 4 q K cos (2k - 1 q) =
statement is true for all n ³ 3. 2k sin q
6. Given, P (n ) : n 2 + n + 1 Step III For n = k + 1
At n = 1, P(1 ) : 3, which is not an even integer. sin 2k + 1 q
P (k + 1 ) : cos q cos 2q K cos(2k q) =
\ P(1 ) is not true (principle of induction is not 2k + 1 sin q
applicable). LHS = cos q cos 2q K cos 2( k - 1) q cos 2k q
Also, n (n + 1 ) + 1 is always an odd integer. sin (2k q)
= k × cos(2k q)
1 1 1 2 sin q
7. Let P (n ) : + +K+
1 ×2 ×3 2 ×3 ×4 n (n + 1 ) (n + 2) 2 sin (2k q) × cos (2k q)
=
n (n + 3) 2k + 1 sin q
= …(i)
4(n + 1 ) (n + 2) sin 2k + 1 q
= k+1 = RHS
Step I For n = 1, 2 sin q
1 1 1 (1 + 3) 1 \ For n = k + 1, P (n ) is true.
LHS = = and RHS = =
1 ×2 ×3 6 4 (1 + 1 ) (1 + 2) 6 Hence, by principle for mathematical induction for all
\ P(1 ) is true. n Î N , P (n ) is true.
Step II Let P (k) is true, then 9. Let P (n ) : 1 + 5 + 12 + 22 + 35 + . . . + (n terms)
1 1 1
P (k) : + + ... + n 2(n + 1 )
1 ×2 ×3 2 ×3 ×4 k (k + 1 ) (k + 2) =
2
k (k + 3)
= nth term of LHS = P (n ) - P (n - 1 )
4 (k + 1 ) (k + 2)
n 2(n + 1 ) (n - 1 )2n
Step III For n = k + 1, then Þ P (n ) - P (n - 1 ) = -
2 2
1 1 1
P (k + 1 ) : + +K+ n 2 2
1 ×2 ×3 2 ×3 ×4 k(k + 1 ) (k + 2) Þ P (n ) - P (n - 1 ) = { n + n - n + 2n - 1 }
2
1
+ \ Tn = P (n ) - P (n - 1 )
(k + 1 ) (k + 2) (k + 3)
n
(k + 1 ) (k + 4) = (3n - 1 )
= 2
4 (k + 2) (k + 3) n
æ n + 1ö
1 1 1 10. Given, n! < ç ÷
LHS = + +K+ è 2 ø
1 ×2 ×3 2 ×3 ×4 k (k + 1 ) (k + 2)
At n = 1, 1 ! </ 1
1
+ æ3ö
2
(k + 1 ) (k + 2) (k + 3) At n = 2 , 2 ! < ç ÷ Þ 2 < 2. 25 which is true.
è2ø
k (k + 3) 1
= +
4 (k + 1 ) (k + 2 ) (k + 1 ) (k + 2 ) (k + 3 ) 11. Let P (n ) º n ! > 2n - 1
[from Eq. (i)] P(3) º 6 > 4
k (k + 3)2 + 4
= Let P (k) º k ! > 2 k -1
is true.
4 (k + 1 ) (k + 2) (k + 3 )
3 2 \ P (k + 1 ) = (k + 1 )! = (k + 1 )k !
k + 6k + 9k + 4
= > (k + 1 ) 2k - 1
4 (k + 1 ) (k + 2) (k + 3)
(k + 1 )2(k + 4) (k + 1 ) (k + 4) > 2k (as k + 1 > 2)
= = = RHS
4 (k + 1 ) (k + 2) (k + 3) 4 (k + 2) (k + 3) 12. Let P (n ) : (n + 2)(n + 3)(n + 4)(n + 5)(n + 6)
Hence, P (k + 1 ) is true. \Put n = 1
Hence, by principle of mathematical induction for all P(1 ) = (1 + 2)(1 + 3)(1 + 4)(1 + 5)(1 + 6)
n Î N , P (n ) is true. = 3 ´ 4 ´ 5 ´ 6 ´ 7 = 120 ´ 21
n -1 sin 2n q Put n = 2,
8. Let P (n ) : cos q cos 2 q cos 4q ... cos (2 q) = n …(i)
2 sin q P(2) = (2 + 2)(2 + 3)(2 + 4)(2 + 5)(2 + 6)
Step I For n = 1, = 4 ´ 5 ´ 6 ´ 7 ´ 8 = 120 ´ 56
sin 2q
LHS = cos q and RHS = = cos q Hence, P (n ) is always divisible by 120.
2 sin q
Mathematical Induction 163

13. Let m = 2 k + 1, n = 2 k - 1 (k Î N ) Step I For n = 1,

\ m2 - n 2 = 4k2 + 1 + 4k - 4k2 + 4k - 1 = 8k i.e. P(1 ) : 32 ´ 1 + 2 - 8 ´ 1 - 9 = 34 - 8 - 9 = 81 - 17


= 64 = 8 ´ 8
Hence, all the numbers of the form m2 - n 2 are always which is divisible by 8.
divisible by 8.
Step II Let it is true for n = k,
4n (2 n )!
14. Let P (n ) : < i.e. 32k + 2 - 8k - 9 = 8l …(i)
n + 1 (n !)2
Step III For n = k + 1,
For n = 2 , 32( k + 1) + 2 - 8(k + 1 ) - 9
42 4! 16 24 = 32k + 2 + 2 - 8k - 8 - 9 = 32k + 2 32 - 8k - 17
P(2) : < Þ <
2 + 1 (2) 2
3 4 = (8l + 8k + 9)32 - 8k - 17 [using Eq. (i)]
which is true. = (8l + 8k + 9)9 - 8k - 17
Let for n = m ³ 2 , P (m) is true. = 72l + 72k + 81 - 8k - 17
4m (2 m)!
i.e. < …(i) = 72 l + 64k + 64 = 8(9l + 8k + 8)
m + 1 (m !)2
which is divisible by 8.
4n + 1 4m 4 (m + 1 ) (2 m )! 4 (m + 1 ) Therefore, P (k + 1 ) is true when P (k) is true.
Now, = × < ×
m+2 m+1 m+2 (m !)2 (m + 2)
Hence, from the principle of mathematical induction,
(2 m)! (2 m + 1 ) (2 m + 2 ) 4 (m + 1 ) (m + 1 )2 the statement is true for all natural numbers n.
=
(2 m + 1 ) (2 m + 2) (m ! )2 (m + 1 )2 (m + 2 ) 18. Let P (n ) = 23 n - 7n - 1
[2 (m + 1 )]! 2 (m + 1 )2 [2 (m + 1 )]! \ P(1 ) = 0, P(2) = 49
= × <
[(m + 1 )!] (2 m + 1 ) (m + 2 ) [(m + 1 )!]2
2
P(1) and P(2) are divisible by 49.
Hence, for n ³ 2 , P (n ) is true. Let P (k) º 23 k - 7k - 1 = 49I
2
15. For n = 2 , a n = 22 + 1 = 16 + 1 = 17 \ P (k + 1 ) º 23 k + 3 - 7k - 8 = 8 (49I + 7k + 1 ) - 7k - 8
k = 49(8I ) + 49k = 49I1
Assume that, a k = 22 + 1 = 10 m + 7, where k > 1 and m
Hence, by mathematical induction 23 n - 7n - 1 is
is some positive integer. divisible by 49.
k + 1 k
Now, a k + 1 = 22 + 1 = (22 )2 + 1 = (10 m + 6)2 + 1 19. Let P (n ) = 32n - 1
2
= 10 (10 m + 12 m + 3) + 7 At n = 1, P(1 ) = 8 which is divisible by 8.
Hence, last digit of a n is 7, " n > 1. \ P(1 ) is true.
16. Let the given statement be P (n ). Let P (k) is true, then P (k) º 32k - 1 = 8I
\ P (k + 1 ) º 32k + 2 - 1 = (8I + 1 ) 9 - 1 = 72 I + 8 = 8I1
i.e. P (n ) : 41n - 14n is a multiple of 27.
\ P (n ) is divisible by 8, " n Î N .
Step I For n = 1,
i.e. P(1 ) = 411 - 141 = 27 = 1 ´ 27 20. Let P (n ) = x (xn - 1 - na n - 1 ) + a n (n - 1 ) = (x - a )2 g (x )
P(1 ) º 0 is true.
which is a multiple of 27 that is true.
Let P (k) is true.
Step II Let it is true for n = k,
i.e. x (xk - 1 - ka k - 1 ) + a k (k - 1 ) = (x - a )2 g (x)
i.e. 41k - 14k = 27l …(i)
Now, P (k + 1 ) º x [xk - (k + 1 ) a k ] + a k + 1 (k)
Step III For n = k + 1, 41k+ 1 - 14k+ 1
º (x - a )2 [xg (x) + ka k - 1 ] (true)
= 41k41 - 14k 14
So, holds for all n Î N .
= (27l + 14k )41 - 14k 14 [using Eq. (i)]
= 27l ´ 41 + 14k ´ 41 - 14k ´ 14 21. Let P (n ) º x2n - 1 + y2n - 1 = l (x + y)
= 27l ´ 41 + 14k (41 - 14) P (1 ) º x + y = l1 (x + y)
k
= 27l ´ 41 + 14 ´ 27 = 27(41l + 14 ) k P (2) º x3 + y3 = l 2 (x + y)

which is multiple of 27. Hence, for " n Î N , P (n ) is true.


Therefore, P (k + 1 ) is true when P (k) is true. Hence, 22. Let P (n ) = 49n + 16n + k
from the principle of mathematical induction, the For n = 1, we get
statement is true for all natural numbers n. P (1 ) = 491 + 161 + k = 65 + k
17. Let the given statement be P (n ). As P(1 ) is divisible by 64, we take
k = -1
i.e. P (n ) : 32n + 2 - 8n - 9 is divisible by 8.
\ P(1 ) = 65 - 1 = 64, which is divisible by 64.
164 JEE Main Mathematics

Round II Consider, (1 + x)k + 1 = (1 + x)k × (1 + x)


³ (1 + kx) (1 + x) [if x > - 1]
1. On putting n = 2 in 32n - 2 n + 1, we get
2´ 2 = 1 + x + kx + kx2
3 - 2 ´ 2 + 1 = 81 - 4 + 1 = 78
which is divisible by 2. ³ 1 + x + kx [Q k > 0 and x > - 1]
= 1 + (k + 1) x
2. On putting n = 1 in 11n + 2 + 122n + 1, we get
Thus, (1 + x)k + 1 ³ 1 + (k + 1) x, if x > - 1
111 + 2 + 122 ´ 1 + 1 = 113 + 123 = 3059
which is divisible by 133. 9. Let P (n ) : 2 × 7n + 3 × 5n - 5
3. It can be proved with the help of mathematical Then, P(1 ) : 2 × 7 + 3 × 5 - 5 = 24
n Let P (m) be true.
induction that < a (n ) £ n.
2
\ P (m) : 2 × 7m + 3 × 5m - 5 = 24k, k Î N …(i)
200
\ < a(200) Þ a(200) > 100 Now, P (m + 1 ) - P (m) :
2
and a (100) £ 100 2 (7m + 1 - 7m ) + 3 (5m + 1 - 5m )
4. On putting n = 2 in 10n + 3 (4n + 2) + 5, we get = 2 × 7m (7 - 1 ) + 3 × 5m (5 - 1 ) = 12 (7m + 5m )
102 + 3 ´ 44 + 5 = 100 + 768 + 5 = 873 Since, 7m and 5m are odd integers and for all m Î N , their
sum must be even say 7m + 5m = 2l , l Î N .
which is divisible by 9.
5. P (n ) = n 2 + n \ P (m + 1 ) - P (m) : 24 l
It is always odd but square of any odd number is always Þ P (m + 1 ) : 24l + P (m)
odd and also sum of two odd number is always even. Þ P (m + 1 ) : 24 (l + k) [from Eq. (i)]
So, for no any n for which this statement is true. Here, P (m + 1 ) is divisible by 24.
6. 599 = 5 (52)49 = 5 (25)49 = 5 (26 - 1 )49 10. By taking option (d),
= 5 ´ 26 ´ (positive term) - 5 1
When n = 1 ,1 > , true;
So, when it is divided by 13, it gives the remainder - 5 3
or 8. 8
When n = 2 ,5 > , true;
7. Let P (n ) : x (xn-1 - na n-1 ) + a n (n - 1 ), where n > 1 3
P (2) : x (x - 2a ) + a 2 = (x - a )2 When n = 3, 14 > 9, true;
64
P (3) : x (x2 - 3a 2) + 2a3 = x3 - 3a 2x + 2a3 When n = 4, 30 > = 21.33, true
3
Þ P (3) = (x - a )(x2 + xa - 2a 2)
which are divisible by (x - a ) 11. Given, a 0 = 1, a n + 1 = 3n 2 + n + a n

8. Let P (n ) : (1 + x)n ³ (1 + nx) Þ a1 = 3(0) + 0 + a 0 = 1


For n = 1, (1 + x)1 = 1 + x = 1 + 1 × x ³ 1 + 1 × x Þ a 2 = 3(1)2 + 1 + a1 = 3 + 1 + 1 = 5
\ (1 + x)1 ³ 1 + 1 × x From option (b),
For n = k, let P (k) : (1 + x)k ³ (1 + kx) is true. Let P (n ) = n3 - n 2 + 1
For n = k + 1, P (k + 1) : (1 + x)k + 1 ³ {1 + (k + 1) x} we \ P (0) = 0 - 0 + 1 = 1 = a 0, P (1) = 13 - 12 + 1 = 1 = a1
shall show P (k + 1) is true. and P (2) = (2)3 - (2)2 + 1 = 5 = a 2
08
Binomial Theorem
and its Applications

In elementary algebra, the binomial theorem describes the algebraic IN THIS CHAPTER ....
expansion of powers of a binomial. According to the theorem, it is possible to
expand the power ( x + y )n into a sum involving terms of the form ax b y c , where Binomial Theorem
the exponents b and c are non-negative integers with b + c = n and the Binomial Theorem For Positive
coefficient a of each term is a specific positive integer depending on n and b. Integer Index
General Term is the Expansion of
Binomial Theorem (x + a)n
Middle Term in Binomial
An algebraic expression consisting of two terms with +ve or -ve sign between,
Expansion
then is called binomial expression.
General Term in the Expansion of
æ p qö
e.g. ( a + 2b), ç 2 - 4 ÷ (x + a)n
èx x ø
Important Expansions
Binomial theorem is the formula obtained by which the binomial raised to the Multinomial Theorem
power of n, where n can be positive or negative or fraction.
There are two types of Binomial Theorem.
(i) Binomial theorem for positive Integral Index.
(ii) Binomial theorem for negative/Rational Index.
166 JEE Main Mathematics

Binomial Theorem for Positive Example 1. The coefficient of x 20 in the expansion of


-5
Integer Index æ 1ö
(1 + x 2) 40 × ç x 2 + 2 + 2 ÷ , is
è x ø
If n is a positive integer and x , a Î R , then
( x + a )n = n C0x n + n C1x n -1 a + n C2x n - 2 a 2 + ... + n Cn a n (a) 30C10 (b) 30C 25
n (c) 1 (d) None of these
= å n Cr x n - r a r …(i)
æ 1ö
-5
r=0
Sol. (b) Given, (1 + x2) 40 × ç x2 + 2 + 2 ÷
è x ø
and ( x - a )n = n C0x n - n C1x n -1a + n C2x n - 2a 2 - ... + ( -1)n a n
-5
ïì æ 1ö ïü
n 2
= å ( -1) rn
Cr x n -r
a r
…(ii) = (1 + x ) 2 40
í çè x + ÷ø ý
r=0 îï x ïþ
- 10
Here, n C0 , n C1 , n C2 ,.. , n Cn are known as binomial æ 1ö
= (1 + x2) 40 ç x + ÷
coefficients or it can be rewritten as C0 , C1 , C2 ,... , Cn . è xø
Also, n Cr =
n! = x10 (1 + x2) 40 (1 + x2) - 10 = x10 (1 + x2)30
r !( n - r )! To find Coefficient of x20 in the expansions of
- 10
æ 1ö
Properties of Binomial Theorem (1 + x2) 20 ç x + ÷ i.e. x10 (1 + x2)30
è xø
(i) The sum of indices of x and a in each term is n. Now, (1 + x2)30 = 30C 0 + 30
C1( x2) + 30
C 2( x2) 2 + 30
C3( x2)3
(ii) The number of terms in ( x + a )n is ( n + 1). 30
+ C 4( x2) 4 + L + 30
C30( x2)30 …(i)
(iii) The coefficient of terms equidistant from the
x10 (1 + x2)30 = x10 + 30
C1x12 + 30
C 2x14 + 30
C3 x16 + 30
C 4 x18
beginning and the end are equal.
(iv) (i) ( x + a )n + ( x - a )n = 2 [n C0x n + n C2x n - 2 a 2 + ... ] + C5 x20 + ¼ + x70
30

\ Coefficient of x20 is 30C5 or 30C 25 . [Q nC r = nC n - r ]


(ii) ( x + a )n - ( x - a )n = 2[n C1x n -1a + n C3 x n -3 a3 + ... ]
(v) If n is odd, then {( x + a )n + ( x - a )n } and Example 2. The coefficient of x7 in the expression
{( x + a )n - ( x - a )n } both have the same number of
(1 + x)10 + x(1 + x)9 + x 2(1 + x) 8 + ... + x10 is
æ n + 1ö
terms equal to ç ÷ , whereas if n is even, then (JEE Main 2020)
è 2 ø (a) 420 (b) 330 (c) 210 (d) 120
æn ö
{( x + a )n + ( x - a )n } has ç + 1÷ terms and Sol. (b) Given expression (1 + x) 10
+ x(1 + x) + x (1 + x) 8 + ¼ + x10
9 2
è2 ø
n The above expression is geometric series having first term,
{( x + a )n - ( x - a )n } has terms.
2 a = (1 + x)10
(vi) In the binomial expansion of ( x + a )n , the rth term Common ratio, r =
x
and number of terms, n = 11
from the end is ( n - r + 2)th term from the beginning. 1+ x
æ æ x ö 11ö
(1 + x)10 ç1 - ç ÷
General Term in the Expansion ç è1 + x ÷ø ÷
n
a(1 - r ) è ø
of (x + a)n Q Sum of geometric series =
1- r
=
x
1-
Let us consider the binomial expansion 1+ x
( x + a )n = n C0x n + n C1x n -1a + n C2x n - 2 a 2 + ... + n Cr x n - r a r æ (1 + x)11 - x11 ö
n n (1 + x)10 ç ÷
In this expansion first term is C0 x , second term is è (1 + x)11 ø
=
n
C1x n - 1a and so on. 1+ x - x
On the basis of that we can say that General term 1+ x
(rth term) is Tr - 1 = n Cr x n - r a r . = (1 + x)11 - x11
Similarly, general term in the expansion of ( x - a )n is The coefficient of x7 in the given expression
Tr +1 = ( -1)r ×n Cr x n - r a r = coefficient of x7 in the expansion of [(1 + x)11 - x11]
n
General term in the expansion of (1 + x ) is = coefficient of x7 in the expansion of (1 + x)11
Tr +1 = n Cr x r 11 ´ 10 ´ 9 ´ 8
= 11C 7 =
General term in the expansion of (1 - x )n is 4 ´3 ´2
Tr +1 = ( -1)r n Cr x r = 11 ´ 10 ´ 3 = 330
Binomial Theorem and its Applications 167

Example 3. If the term independent of x in the expansion Example 5. If some three consecutive coefficients in the
æ3 1ö
9 binomial expansion of ( x + 1) n in powers of x are in the ratio
of ç x 2 - ÷ is k, then 18 k is equal to
è2 3x ø (JEE Main 2020)
2 : 15 : 70, then the average of these three coefficients is
(a) 964 (b) 227 (c) 232 (d) 625
(a) 5 (b) 7 (c) 9 (d) 11
Sol. (c) Given binomial is ( x + 1) n , whose general term, is
9
æ3 1ö Tr + 1 = nC r xr
Sol. (b) Given binomial is ç x2 - ÷ , So the general term,
è2 3x ø
9-r r According to the question, we have
æ3 ö æ 1ö n
C r - 1 : nC r : nC r + 1 = 2 : 15 : 70
Tr + 1 = C r ç x2÷
9
ç- ÷
è2 ø è 3x ø
n
9-r r Cr - 1 2
9 æ3ö æ 1 ö 18 - 3r Now, =
= Cr ç ÷ ç- ÷ x n
Cr 15
è2ø è 3ø
n!
If the term is independent of x, then
18 - 3r = 0 Þ r = 6 (r - 1) !(n - r + 1) ! 2 r 2
Þ = Þ =
\ (r + 1th
) term = 7th term is independent of x. n! 15 n - r + 1 15
Now, as T6 + 1 = k [given] r !(n - r) !
3
æ 3 ö æ 1ö
6 Þ 15r = 2n - 2r + 2
9
Þ C6 ç ÷ ç - ÷ = k Þ 2n - 17r + 2 = 0
è2ø è 3ø …(i)
9 ´8 ´7 1 7 n!
Þ ´ =k Þ k= Þ18k = 7 n
3 ´2 27 ´ 8 18 Cr 15 r !(n - r) ! 3
Similarly, n
= Þ =
Cr + 1 70 n! 14
Hence, option (b) is correct.
(r + 1) !(n - r - 1) !
Example 4. The positive value of l for which the r +1 3
10 Þ = Þ14r + 14 = 3n - 3r
æ lö n - r 14
coefficient of x 2 in the expression x 2 ç x + 2 ÷ is 720, is
è x ø Þ 3n - 17r - 14 = 0 …(ii)
(JEE Main 2019) On solving Eqs. (i) and (ii), we get
(a) 3 (b) 5 (c) 2 2 (d) 4 n - 16 = 0 Þ n = 16 and r = 2
16 16
Sol. (d) The general term in the expansion of binomial expression C 2 + 16C3
C1 +
Now, the average =
( a + b) n is Tr + 1 = nC r an - r b r , 3
16 + 120 + 560 696
so the general term in the expansion of binomial expression = = = 232
10 3 3
æ lö
x2ç x + 2 ÷ is
è x ø Middle Term in Binomial Expansion
æ æ lö ö
r
Tr + 1 = x2 çç 10C r ( x)10 - r ç 2 ÷ ÷÷ Consider binomial expansion
è èx ø ø
( x + a )n = n C0x n + n
C1x n -1 a + n
C2x n - 2 a 2 + ¼ + n Cn a n
10 - r
=10C r x2 × x 2 lr x-2r Or
n -1
2+
10 - r
- 2r
n n
( x - a ) = C0x - C1xn n
a + n C2x n - 2 a 2 + ¼ + ( -1)n n Cn a n
10 r
= Cr l x 2
æn ö
Now, for the coefficient of x2, (i) If n is an even number, then ç + 1÷th term is
è2 ø
10 - r
put 2+ - 2r = 2 middle term in the expansion of ( x + a )n or ( x - a )n
2
10 - r \ Tn = n Cn / 2 x n / 2a n / 2
Þ - 2r = 0 +1
2 2
Þ 10 - r = 4r Þ r = 2 or ( -1)n / 2 x n / 2 a n / 2
2 10 2
So, the coefficient of x is C 2 l = 720 [given]
æ n + 1ö æ n + 3ö
10 ! 2 (ii) If n is an odd number, then ç ÷th and ç ÷th
Þ l = 720 è 2 ø è 2 ø
2!8!
terms are middle terms in the expansion of ( x + a )n
10 × 9 × 8 ! 2
Þ l = 720 or ( x - a )n
2× 8!
n +1 n -1
Þ 45 l2 = 720 i.e. Tæ n + 1 ö = 2C n - 1 x 2 a 2
ç ÷
Þ l2 = 16 Þ l = ± 4 è 2 ø 2
168 JEE Main Mathematics

n -1 n +1
and Tæ n +3 ö = n C n +1 x 2 a 2 Greatest Binomial Coefficient
ç ÷ The greatest coefficient depends upon the value of n.
è 2 ø 2

n +1 n +1 n -1 Number of greatest
2 nC
n Greatest coefficient
or T n +1 = ( -1) n -1 (x) 2 a 2 Coefficient (s)
n
2 2 Cn/ 2
Even 1
n +3 n -1 n +1
n
and T n +3 = ( -1) 2 n C n +1 (x) 2 a 2 C n -1 and n C n +1
odd 2
2 2 2 2

Note It is clearly that, greatest binomial coefficient


● When there are two middle terms in the expansion, then their binomial compounds to the coefficient of middle term.
coefficients are equal.
Example 7. If a, b and c are the greatest values of
● Binomial coefficient of middle term is the greatest binomial coefficient.
19 20 21
Cp, C q and C r respectively, then (JEE Main 2020)
Example 6. The sum of the real values of x for which the a b c a b c
8 (a) = = (b) = =
æ x3 3 ö 10 11 42 11 22 21
middle term in the binomial expansion of ç + ÷ equals 5670 is a b c a b c
è 3 xø (c) = = (d) = =
10 11 21 11 22 42
(JEE Main 2019)
(a) 4 (b) 0 (c) 6 (d) 8 Sol. (d) Since a = maximum of 19C p = 19C 9 = 19C10 …(i)
20 20
8 b = maximum of Cq = C10 …(ii)
æ x3 3 ö
Sol. (b) In expansion of ç + ÷ , the middle term is T4 + 1. and c = maximum of 21
Cr = 21
C10 = 21
C11 …(iii)
è3 xø
19
aC10 19 !10 ! 10 1 a b
[Q here, n = 8, which is even, therefore middle term Then, = = 20
= = Þ =
bC10 9 ! 20 ! 20 2 1 2
æn + 2ö
=ç ÷ th term] 20 !
è 2 ø
b 20C10 10 !10 ! 11 b c
4 and = 21 = = Þ =
æ x3 ö æ 3 ö 4 8 × 7 × 6 × 5 8 c C11 21! 21 11 21
8
\ 5670 = C 4 ç ÷ ç ÷ = x 11!10 !
è 3 ø è xø 1× 2 × 3 × 4
a b c
é 8-r rù \ = =
æ x3 ö æ3ö ú 11 22 42
êQ Tr + 1 = 8C r ç ÷ ç ÷
ê è3ø è xø ú Hence, option (d) is correct.
ë û
Þ x8 = 3 4 Þ x = ± 3 Example 8. Numerically the greatest term in the
So, sum of all values of x i.e. + 3 and - 3 = 0 3
expansion of (2 + 3x)9 when x = , is
2
Greatest Term in the Expansion of (x +a)n (a)
7 ´ 33
(b)
7 ´ 313
If Tr and Tr +1 be the rth and (r + 1)th terms in the expansion of 2 2
313
( x + a )n , then (c) (d) None of these
2
n
Cr x n - r a r æ n - r + 1ö a æ 3x ö
9
T r +1 = =ç ÷ Sol. (b) Since, (2 + 3x ) 9 = 2 9 ç1 + ÷
n
Cr -1x n - r +1a r -1 è r ø x è 2ø
Tr
9
T r +1 æ 3 ö
Q Now, in the expansion of ç1 + x÷ , we have
Tr
³1 è 2 ø
æ 9 - r + 1ö 3
n -r +1 a Tr + 1 = ç ÷ x
\ ³ 1 Þ ( n - r + 1 ) a ³ r| x| è r ø 2
T
r x r

æ10 - r ö æ 9 ö 90 - 9r æ 3ö
(n +1) a =ç ÷ç ÷= çQ x = ÷
Þ r£ è r ø è 4ø 4r è 2ø
x + a 90 - 9r
Tr + 1
Q ³ 1 Þ 90 ³ 13r
³ 1Þ
(n + 1 ) a Tr 4r
Let =I+ f
x + a 90 12
Þ r£ Þ r £6
13 13
where, I is an integer and 0 £ f < 1.
\ Maximum value of r is 6.
Binomial Theorem and its Applications 169

So, greatest term = 2 9 × T6 + 1


6 6
Applications of Binomial Theorem
æ3 ö æ3 3ö
= 2 × C 6 ç x÷ = 2 9 × 9C3 ç ´ ÷
9 9
è2 ø è2 2ø R-f Factor Relation
12 13
9 ×8 ×7 3 7 ´3 Here, we are going to discuss problems involving
= 29 × × =
1× 2 × 3 212 2 ( A + B)n = I + f ,

Properties of Binomial Coefficients where I and n are positive integers.


n n
(i) If Cr = Cs , then either r = s or r + s = n 0 £ f £ 1, A - B2 = k and A - B < 1

(ii) n
C0 + n C1 + ¼ + n Cn = 2n Approach for these type of problems can be learnt from
n n n n n n -1
following examples.
(iii) C0 + C2 + C4 + ¼ = C1 + C3 + ¼ = 2
(iv) n
C0 - n C1 + n C2 - n C3 + ¼ ( -1 )n n Cn = 0 Example 10. Integral part of ( 7 + 4 3) n if (n Î N)
n
Cr n -r +1 (a) an even number
(v) n
= (b) an odd number
Cr - 1 r
(c) an even or an odd number depending upon the value of n
(vi) Cr + n Cr -1 = n +1Cr
n
(d) None of the above
n +1 n
(vii) n +1Cr +1 = × Cr Sol. (b) Here, " n ÎN ,(7 + 4 3) n ÏN
r +1
\ Denote (7 + 4 3) n by I + f ,
(viii) n
C1 + 2 n C2 + 3 n C3 + ¼ + n n Cn = n × 2n -1
n
where, I is an integer and f ÎR such that 0 < f < 1
(ix) C1 - 2 n C2 + 3 n C3 - ¼ = 0 Q 0 < 7 - 4 3 <1
(x) C0 + 2 n C1 + 3 n C2 + ¼ + ( n + 1 ) n Cn = ( n + 2) 2n -1
n
\We can denote(7 - 4 3) n by G where,G ÎR such that0 < G <1
( 2n )!
(xi) C0Cr + C1Cr +1 + ¼ + Cn - r Cn = Now, I + f = (7 + 4 3) n = 7 n + nC1 7 n -1 ( 4 3)
( n - r )!( n + r )!
+ nC 27 n - 2( 4 3) 2 + ¼ …(i)
( 2n )!
(xii) C02 + C12 + C22 + ¼ + Cn2 = G = (7 - 4 3) n = 7 n - nC17 n - 1( 4 3) + nC 27 n - 2( 4 3) 2… …(ii)
( n !)2
To cancel irrational terms we add Eqs. (i) and (ii), we get
(xiii) C02 - C12 + C22 - C32 + ¼ + ( -1)n × Cn2
I + f + G = 2 (7 n + nC 27 n - 2( 48) + nC 47 n - 4( 48) 2 + ¼)
ì 0 , if n is odd
=í n/ 2 n = 2k, where k is an integer
î( -1 ) Cn / 2 , if n is even
Q I is an integer.
(xiv) C12 - 2C22 + 3C32 - ¼ + ( -1 )n n × Cn2 \ f + G = 2k - I is an integer. …(iii)
n
-1 n n! Now, 0 < f <1
= ( -1 )2 × × , when n is even. and 0 <G <1
2 ænö ænö
ç ÷ !ç ÷ ! Þ 0 < f + G <2 …(iv)
è 2ø è 2ø
25
Form Eqs. (iii) and (iv), f + G =1
Example 9. If å { 50C r × 50 - rC 25 - r} = K(50C 25), then, K is Now, form Eq. (iii) I = 2k - 1
r=0 Þ Integral part of (7 + 4 3) n
equal to (JEE Main 2019) i.e., I is an odd integer.
24 25 25
(a) 2 (b) 2 -1 (c) 2 (d) (25) 2
25 Divisibility Problem
Sol. (c) Given, S {50Cr .50-r C 25-r} = K 50C25
r =0
In the expansion, (1 + a )n = 1 + n C1a + n C2a 2 + ¼ + n Cn a n .
25
(50 - r) ! ö
Þ S æç
r =0 è
50 !
´ ÷ =K
50
C 25
We can conclude that,
r !(50 - r) ! (25 - r) ! 25 ! ø (i) (1 + a )n - 1 = n C1a + n C2a 2 + ¼ + n Cn a n is divisible
25
Þ S æçè 50 !
´
25 ! ö
÷ =K
50
C 25 by a i.e., it is a multiple of a.
r =0 25 ! 25 ! r !(25 - r) ! ø
[on multiplying 25 ! in numerator and denominator.] Example 11. For all n ÎR, 9 n+1 - 8n - 9 is divisible by
25
é 50 50 ! ù (a) 64 (b) 63 (c) 62 (d) 65
Þ 50
C 25 S 25Cr = K 50C25 êQ C 25 = ú
r =0 ë 25 ! 25 ! û Sol. (a) 9 n + 1 - 8n - 9 = 9 n ´ 9 - 8n - 9
25
= (1 + 8) n ´ 9 - 8n - 9
Þ K= S 25Cr = 225 [Q nC0 + nC1 + nC2 +....+ nCn = 2n ]
r =0
= ( nC 0 + nC18 + nC 28 2 + nC383 + ¼+ nC n8 n) 9 - 8n - 9
Þ K = 2 25
170 JEE Main Mathematics

= (1 + 8n + nC 28 2 + nC383 + ¼+ nC n8 n) 9 - 8n - 9 Example 13. The coefficient of x3y 4 z 2 in the expansion of


n 2 n 3 n n
= 9 + 72n + ( C 28 + C38 + ¼+ C n8 ) 9 - 8n - 9 (2 x - 3y + 4z )9 is
2n n n n-2
= (72n - 8n) + 8 ( C 2 + C38 + ¼+ C n8 )9 (a) 12063600
= 64n + 64 ( nC 2 + nC38 + ¼+ nC n8 n - 2) 9 (b) 13063680
= 64 [n + ( nC 2 + nC38 + ¼+ nC n8 n - 2) 9] (c) 11063600
(d) None of the above
= 64 ´ Some constant numbers
= Divisible by 64 Sol. (b) The general term in the expansion of (2x - 3y + 4z) 9
9!
= × (2x) a1 ( -3y) a 2 ( 4z) a 3
Differentiability Problem a1 ! a 2 ! a3 !
9!
Sometimes to generalise the result we use the = × 2 a1 ( -3y) a 2 4 a 3 × xa1y a 2 z a 3
differentiation. a1 ! a 2 ! a3 !
\ Coefficient of x3y 4 z 2
Example 12. Series nC1 + 1× nC 2 + 2 × nC3 + ¼ + n × nC n 9!
is equal to = × 23( -3) 4 × 4 2 [Taking a1 = 3, a 2 = 4, a3 = 2]
3 ! 4 !2 !
n -1 n-1 n 3 -1
(a) n2 (b) 2 (c) n2 (d) 2 = 1260 ´ 10368
Sol. (a) Since, (1 + x) n = nC 0 + nC1x + nC 2x2 + ¼+ nC n xn = 13063680
On differentiating w.r.t., x, we get
Example 14. The coefficient of a3b 4c 5 in the expansion of
n(1 + x) n -1 = 0 + nC1 + 2 × x × nC 2 + ¼+ n × nC n xn -1
( bc + ca + ab)6 is
Put x =1, we get
(a) 6 (b) 60
n(1 + 1) n -1 = nC1 + 2 ×n C 2 + ¼+ n ×n C n
(c) 61 (d) None of these
Þ n 2 n -1 = nC1 + 2 ×n C 2 + ¼+ n ×n C n
Sol. (b) In this case, we write
a3 b 4 c5 = ( ab) x( bc) y ( ca) z
Multinomial Theorem
If n is a positive integer and a1 , a2 , ¼ , ak Î R , then Þ a3 b 4 c5 = az+ x × b x + y × cy + z
r
n ! a1r 1 × a22 ¼ akrk Þ z + x = 3, x + y = 4, y + z = 5 …(i)
( a1 + a2 + ¼ + ak )n = S On adding these equations, we get
r1 + r2 + ¼ + rk = n r1 !r2 !¼ rk !
2( x + y + z) = 12
\ x +y + z = 6 …(ii)
n +2
The number of terms in (x + y + z) n is C2. On solving Eqs. (i) and (ii), we get
The number of terms in the expansion of x = 1, y = 3, z = 2
( a1 + a2 + ¼ + ak )n is n + k-1Ck-1. Therefore, the coefficient of a3 b 4 c5 in the expansion of
( bc + ca + ab) 6
Coefficient of x n1 × y n 2 × z n3 in the expansion of
n! The coefficient of ( ab)1( bc)3( ca) 2 in the expansion of
( x + y + z )n is where, n = n1 + n 2 + n3 . Sum of
n1 ! n 2 ! n3 ! ( bc + ca + ab) 6
coefficients in the expansion of ( a + bx + cx 2 )n is =
6!
= 60
( a + b + c)n . 1! 3 ! 2 !
Practice Exercise
ROUND I Topically Divided Problems
Binomial Theorem and 9. The coefficient of t 24 in the expansion of
Its General Terms (1 + t 2)12 (1 + t12 ) (1 + t 24 ) is
12 12 12 12
(a) C 6 + 2 (b) C5 (c) C6 (d) C7
1. The total number of terms in the expansion of
53
( x + a)100 + ( x - a)100 after simplification will be 10. The coefficient of x in the following expansion
100
(a) 202 (b) 51
(c) 50 (d) None of these
å 100
Cm ( x - 3)100 - m × 2 m is
m=0
2. If (1 + ax) n = 1 + 8 x + 24 x 2 + . . ., then the values of a (a) 100C 47 (b) 100C53
and n are (c) - 100C53 (d) - 100C100
(a) 2, 4 (b) 2, 3 (c) 3, 6 (d) 1, 2 11. In the polynomial ( x - 1) ( x - 2) ( x - 3) . . . ( x - 100),
3. The sum of the coefficients of all even degree terms is the coefficient of x 99 is
3 6 3 6
x in the expansion of ( x + x - 1 ) + ( x - x - 1) , (a) 5050 (b) – 5050 (c) 100 (d) 99
( x > 1) is equal to (JEE Main 2019, 8 April) m n
12. If in the expansion of (1 + x) (1 - x) , the coefficient
(a) 29 (b) 32 (c) 26 (d) 24 of x and x 2 are 3 and - 6 respectively, then m is
4. The ratio of the coefficient of x10 in (1 - x 2 )10 and (a) 6 (b) 9 (c) 12 (d) 24
10
the term independent of x in æç x - ö÷ , is
2 13. If for positive integers r > 1, n > 2 , the coefficient of
è xø
the (3r)th and ( r + 2)th powers of x in the
(a) 1 : 16 (b) 1 : 32
expansion of (1 + x) 2 n are equal, then
(c) 1 : 64 (d) None of these
(a) n = 2 r (b) n = 3 r
5. The value of (1.002)12 upto fourth place of decimal is (c) n = 2 r + 1 (d) None of these
(a) 1.0242 (b) 1.0245
14. If the coefficients of three consecutive terms in the
(c) 1.0004 (d) 1.0254 expansion of (1 + x) n are in the ratio 1 : 7 : 42 , then
4 the value of n is
6. The coefficient of x in the expansion of
(1 + x + x 2 + x 3) n is (a) 60 (b) 70
n
(a) C 4 (c) 55 (d) None of these
(b) nC 4 + nC 2 15. The digit at the unit place in the number
(c) nC 4 + nC 2 + nC 2
19 2005 + 112005 - 9 2005 is
(d) nC 4 + nC 2 + nC1 × nC 2
(a) 2 (b) 1
7. In the expansion of the following expression (c) 0 (d) 8
1 + (1 + x) + (1 + x) 2 + . . . + (1 + x) n , the coefficient of 16. If a1, a2 , a3, a4 are the coefficients of any four
x 4 (0 £ k £ n) is
consecutive terms in the expansion of (1 + x) n , then
(a) n + 1C k + 1 (b) nC k a1 a3
(c) nC n - k - 1 (d) None of these + is equal to
a1 + a2 a3 + a4
8. The coefficient of x 5 in the expansion of a2 1 a2
(a) (b)
(1 + x) 21 + (1 + x) 22 + . . . + (1 + x) 30 is a 2 + a3 2 (a 2 + a 3 )
51 9
(a) C5 (b) C5 2a2 2 a3
31 (c) (d)
(c) C 6 - 21C 6 (d) 30
C5 + 20
C5 a 2 + a3 a 2 + a3
172 JEE Main Mathematics

17. If the coefficients of p th, ( p + 1)th and ( p + 2)th Independent, Middle and
terms in the expansion of (1 + x) n are in AP, then
(a) n 2 - 2 np + 4 p2 = 0
Greatest Term
(b) n 2 - n (4 p + 1) + 4 p2 - 2 = 0 26. If the constant term in the binomial expansion of
(c) n 2 - n (4 p + 1) + 4 p2 = 0 æ kö
10

(d) None of the above ç x - 2÷ is 405, then|k|equals


è x ø (JEE Main 2020)
18. If (1 + x - 2 x 2 ) 6 = 1 + a1x + a2 x 2 + . . . + a12 x12 , then (a) 9 (b) 1 (c) 3 (d) 2
the expression a2 + a4 + a6 + . . . +a12 has the value n
27. The term independent of x in (1 + x) m æç1 + ö÷ , is
1
(a) 32 (b) 63 è xø
(c) 64 (d) None of these m+ n m+ n
(a) Cm (b) Cn + 1
19. If (1 - x + x 2 ) n = a0 + a1x + a2 x 2 + . . . + a2 n x 2 n , then
m+ n
(c) Cm - n (d) None of these
a0 + a2 + a4 + . . . + a2 n is equal to
n
28. The middle term in the expansion of æç x 2 + + 2 ö÷ ,
n
3 +1 3 -1n 1
(a) (b) è ø
2 2 x2
1 - 3n n 1 is
(c) (d) 3 +
2 2 n! (2 n )!
(a) (b)
((n /2)!)2 ((n /2)!)2
20. If the Coefficient of second, third and fourth terms
1 × 3 × 5 ××× (2 n - 1) n 1 × 3 × 5 ××× (2 n + 1) n
in the expansion of (1 + x) 2 n are in AP, then (c) ×2 (d) ×2
n! n!
2 n2 - 9 n is equal to
(a) - 7 (b) 7 (c) 6 (d) - 6 29. If for some positive integer n, the coefficients of
21. If the fourth term in the binomial expansion of three consecutive terms in the binomial expansion
6 of (1 + x) n + 5 are in the ratio 5 : 10 : 14, then the
æ æ 1 ö 1 ö
ç çè 1+ log x ÷ø ÷ largest coefficient in this expansion is (JEE Main 2020)
ç x
10 + x12 ÷ is equal to 200, and x > 1, then
(a) 330 (b) 462 (c) 792 (d) 252
ç ÷
è ø 18
30. The coefficient of x in the product
the value of x is (JEE Main 2019)
(1 + x)(1 - x) (1 + x + x 2 ) 9 is
10
(JEE Main 2019)
(a) 100 (b) 104 (c) 10 (d) 103
(a) 84 (b) - 126 (c) - 84 (d) 126
22. The two successive terms in the expansion of
31. If the last term in the binomial expansion of
(1 + x) 24 whose coefficients are in the ratio 1:4 are n log 3 8
(a) 3rd and 4th (b) 4th and 5th æ 21/ 3 - 1 ö is æ 1 ö
ç ÷ ç 5/ 3 ÷ , then the 5th term from
(c) 5th and 6th (d) 6th and 7th è 2 ø è3 ø
the beginning is
23. If in the expansion of ( a - 2 b) n , the sum of 4th and
a (a) 210 (b) 420
5th term is zero, then the value of is (c) 105 (d) None of these
b
n -4 n -3 5 5 32. The coefficient of the term independent of x in the
(a) (b) (c) (d) 9
5 2 n -4 2(n - 4)
expansion of (1 + x + 2 x 3) æç x 2 - ö÷ is
3 1
è2 3x ø
24. If the coefficients of x 2 and x 3 are both zero, in the
expansion of the expression (1 + ax + bx 2 ) (1 - 3x)15 1 19 17 1
(a) (b) (c) (d)
in powers of x, then the ordered pair ( a, b) is equal 3 54 54 4
to (JEE Main 2019) 33. The coefficient of x in the expansion of
(a) (28, 315) (b) (- 21, 714) (1 - 3x + 7 x 2 ) (1 - x )16 is
(c) (28, 861) (d) (- 54, 315) (a) 19 (b) - 19 (c) 18 (d) - 18
25. If the third term in the binomial expansion of 34. The term independent of x in the expansion of
log 2 x 5 6
(1 + x ) equals 2560, then a possible value of x æ 1 x8 ö æ 2 3ö
ç - ÷ . ç 2 x - 2 ÷ is equal to
is (JEE Main 2019) è 60 81 ø è x ø (JEE Main 2019)
1 1
(a) 4 2 (b) (c) (d) 2 2 (a) - 72 (b) 36 (c) - 36 (d) - 108
4 8
Binomial Theorem and its Applications 173

2n
35. The middle term in the expansion of æç x +
1 ö
è
÷ is Properties of Binomial Coefficients
2x ø n n
1 r
(a)
1 × 3 × 5 ¼ (2n - 3)
(b)
1 × 3 × 5 ¼ (2n - 1) 43. If an = å n
Cr
, then å n
Cr
is equal to
n! n! r=0 r=0

1 × 3 × 5 ¼ (2n + 1) (a) (n - 1) a n (b) na n


(c) (d) None of these 1
n! (c) na n (d) None of these
10 2
36. If the middle term of æç + x sin x ö÷
1 7
is equal to 7 , n
1 + rx
èx ø 8 44. å ( -1) r ( n Cr ) 1 + nx is equal to
then value of x is r=0

p p (a) 1 (b)-1 (c) n (d) 0


(a) 2np + (b) np +
6 6 æ30 ö æ30 ö æ30 ö æ30 ö æ 30 ö æ30 ö
p p 45. ç ÷ ç ÷-ç ÷ ç ÷ + ... + ç ÷ ç ÷ is equal to
(c) np + (- 1)n (d) np + (- 1)n è0ø è10 ø è 1 ø è 11 ø è 20 ø è30 ø
6 3
30 60 30 65
(a) C11 (b) C10 (c) C10 (d) C55
37. If p is a real number and if the middle term in the n 2 n
8 46. If (1 + x) = C0 + C1x + C2 x + . . . + Cn x , then
expansion of æç + 2 ö÷ is 1120, then the value of p is
p
è2 ø C02 + C12 + C22 + C32 + . . . + Cn2 is equal to
(a) ± 3 (b) ± 1 n! (2 n )! (2 n )!
(a) (b) (c) (d) None of these
(c) ± 2 (d) None of these n! n! n!n! n!

38. If n is even, then the middle term in the expansion 47. The value of ( 21C1 - 10C1) + ( 21C2 - 10
C2 ) +
n 21 10 21 10 21 10
( C3 - C3) + ( C4 - C4 ) + . . . + ( C10 -
of æç x 2 + ö÷ is 924 x 6 , then n is equal to
1 C10 ) is
è xø (JEE Main 2017)
(a) 10 (b) 12 (a) 221 - 211 (b) 221 - 210 (c) 220 - 29 (d) 220 - 210
(c) 14 (d) None of these
48. If n Î N , n > 1, then value of
39. The largest term in the expansion of (3 + 2 x) 50 , E = a - nC1 ( a - 1) + nC2 ( a - 2) + . . .
1 + ( - 1) n ( a - n) nCn is
where x = is
5 (a) a (b) 0 (c) a 2
(d) 2n
(a) 5th (b) 3th 1n 1 n
Cn
(c) 7th (d) 6th 49. n C0 - C1 + nC2 - . . . + ( - 1) n is equal to
2 3 n+1
40. If the sum of the coefficients in the expansion of 1 1 1
(a) n (b) (c) (d)
( x - 2 y + 3z) n is 128, then the greatest coefficient in n n+1 n -1
the expansion of (1 + x) n is n

(a) 35 (b) 20
50. If (1 + x) n = å Cr x r , then
r=0
(c) 10 (d) None of these
æ C1 ö æ C2 ö æ Cn ö
41. The greatest value of the term independent of x, as a ç1 + ÷ ç1 + ÷ . . . çç1 + ÷ is equal to
è C0 ø è C1 ø è Cn - 1 ÷ø
20
æ sin a ö nn - 1 (n + 1)n - 1 (n + 1)n (n + 1)n + 1
varies over R, in the expansion of ç x cos a + ÷ (a) ! (b) (c) (d)
è x ø (n - 1) (n - 1)! n! n!
is
20 51. If Cr stands for n Cr , the sum of the given series
(a) C10 (b) 20C15
2 æç ö÷ ! æç ö÷ !
(c) 20
C19 (d) None of these n n
è2ø è2ø
42. The interval in which x must lie, so that the × [C02 - 2 C12 + 3 C22 - . . . + ( - 1) n ( n +
n!
greatest term in the expansion of (1 + x) 2 n has the 1) Cn2 ], where n is an even positive integer, is
greatest coefficient, is (a) 0 (b) (- 1)n/ 2 (n + 1)
æn -1 n ö æ n n + 1ö (c) (- 1)n (n + 2) (d) (- 1)n/ 2(n + 2)
(a) ç , ÷ (b) ç , ÷
è n n - 1ø èn + 1 n ø 52. The value of-15 C1 + 2 × 15C2 - 3 × 15C3 + . . . . - 15 × 15C15
æ n n + 2ö
(c) ç , ÷ (d) None of these +14 C1 + 14C3 + 14C5 + . . . . + 14C11 is (JEE Main 2017)
èn + 2 n ø
(a) 214 (b) 213 - 13 (c) 216-1 (d) 213 - 14
174 JEE Main Mathematics

a
Applications of Binomial Theorem (a)
2n
(b) na

and Multinomial Theorem (c) 0 (d) None of these


n 2 n
53. If (1 + x) = C0 + C1x + C2 x + . . . + Cn x , then the 60. If A = 1000 1000
and B = (1001) 999 , then
value of C0 + 2 C1 + 3 C2 + . . . + ( n + 1) Cn will be (a) A > B (b) A = B
(a) (n + 2) 2n - 1 (b) (n + 1) 2n (c) A < B (d) None of these
(c) (n + 1) 2n - 1 (d) (n + 2) 2n 61. If {p} denotes the fractional part of the number p,
54. If n > (8 + 3 7 )10 , n Î N , then the least value of n is ì 3200 ü
then í ý is equal to
(a) (8 + 3 7 )10 - (8 - 3 7 )10 î 8 þ (JEE Main 2020)
(b) (8 + 3 7 )10 + (8 - 3 7 )10 5 7 3 1
(a) (b) (c) (d)
(c) (8 + 3 7 )10 - (8 - 3 7 )10 + 1 8 8 8 8
(d) (8 + 3 7 )10 - (8 - 3 7 )10 - 1
62. Let [ x ] denote the greatest integer less than or
2n
55. Let R = ( 2 + 3) and f = R - [ R ], where [ × ] equal to x. If x = ( 3 + 1) 5 , then [ x ] is equal to
denotes the greatest integer function, then R (1 - f ) (a) 75 (b) 50 (c) 76 (d) 152
is equal to 1 1 1
63. + + + . . . is equal to
(a) 1 (b) 2 2n (c) 2 2n - 1 (d) 2n
Cn 1 ! ( n - 1) ! 3 ! ( n - 3) ! 5 ! ( n - 5) !
56. If n is the number of irrational terms in the 2n 2n - 1
1/ 4 1/ 8 (a) (b)
expansion of (3 +5 ), then ( n - 1) is divisible by n! n!
(JEE Main 2021) (c) 0 (d) None of these
(a) 26 (b) 30 (c) 8 (d) 7
2n
64. The coefficient of x n in the polynomial
57. If (1 + x + x ) = 2 n
å ar x , then
r
( x + nC0 ) ( x + 3 nC1) ( x + 5 nC2 ) . . . [ x + ( 2 n + 1) nCn ]
r=0
is
a1 - 2 a2 + 3a3 K - 2 na2 n is equal to (a) n ×2n (b) n × 2n + 1
(a) n (b) -n (c) 0 (d) 2 n (c) (n + 1) 2n (d) n × 2n + 1
15
58. If (1 + x) 2
= C0 + C1x + C2 x + . . . + C15 x15 , then 65. The coefficient of x7 in (1 + 3x - 2 x 3)10 is equal to
C2 + 2 C3 + 3 C4 + . . . + 14 C15 is equal to (a) 62640 (b) 26240
(a) 14 × 2 14 14
(b) 13 × 2 + 1 (c) 64620 (d) None of these
(c) 13 × 214 - 1 (d) None of these 2 403 k
66. If the fractional part of the number is , then
59. If a and d are two complex numbers, then the sum 15 15
to ( n + 1) terms of the following series k is equal to (JEE Main 2019)
(a) 14 (b) 6 (c) 4 (d) 8
a C 0 - (a + d ) C1 + (a + 2 d ) C 2 - ... + ... is

ROUND II Mixed Bag


Only One Correct Option 3. If n C4 , n C5 and n C6 are in AP, then n can be
1. The total number of irrational terms in the (JEE Main 2019)

binomial expansion of (7 1/ 5
-3 1/ 10 60
) is (a) 9 (b) 11 (c) 14 (d) 12
(JEE Main 2019) 4. The coefficient of x 5 in the expansion of
(a) 49 (b) 48 (c) 54 (d) 55 (1 + 2 x) 6 (1 - x)7 is
2. A ratio of the 5th term from the beginning to the (a) 192 (b) 171
5th term from the end in the binomial expansion of (c) 21 (d) None of these
10
æ 1 ö 5. The coefficient of x r [0 £ r £ ( n - 1)] in the expansion
ç 3 1 ÷
ç 2 + 1÷
is of( x + 3) n - 1 + ( x + 3) n - 2 ( x + 2)
ç ÷
è 2 (3) 3 ø + ( x + 3) n - 3 ( x + 2) 2 + . . . + ( x + 2) n - 1 is
1 1 1 1 (a) nC r (3r - 2n ) (b) nC r (3n - r - 2n - r )
(a) 1 : 2(6)3 (b) 1 : 4(16)3 (c) 4(36)3 : 1 (d) 2(36)3 : 1 (c) nC r (3r + 2n - 1 ) (d) None of these
Binomial Theorem and its Applications 175

10
6. The sum of the series æ 1 - ö
1
binomial expansion of ççax 9 + bx 6 ÷÷ is 10 k, then k
n æ1 3r 7r 15r ö
å (- 1)r nC r çè 2r + 22r + 23 r + 2 4r + ... + m terms÷ø is è ø
r=0
is equal to (JEE Main 2020)
mn mn
2 -1 2 -1 (a) 84 (b) 176 (c) 352 (d) 336
(a) (b)
2mn (2n - 1) 2n - 1 æ 1 - t6 ö
3
4
2mn + 1 17. The coefficient of t in the expansion of ç ÷ is
(c) (d) None of these è 1- t ø
2n + 1
(JEE Main 2019)
7. The value of x, for which the 6th term in the (a) 12 (b) 10 (c) 15 (d) 14
7
ì (9x - 1 + 7) 1 ü 18. In the expansion of (1 + x) n ,
expansion of í 2 log 2 + ý is
(1/ 5 ) log 2 ( 3x - 1 + 1) C1 C C C
î 2 þ + 2 2 + 3 3 +¼ + n n is equal to
84, is equal to C0 C1 C2 Cn - 1
(a) 4 (b) 3 (c) 2 (d) 5 (n + 1) n
(a) (b)
æ10 ö æ 20 ö é
m
æ pö ù 2 2
8. The sum ç ÷ç
è i ø
åè m - i
÷, ê where, ç ÷ = 0, if p <
ø èq ø

(c)
n (n + 1)
(d) None of these
i=0 ë û 2
is maximum, when m is
19. (C0 + C1)(C1 + C2 ) ¼ (Cn - 1 + Cn ) is equal to
(a) 5 (b) 10 (c) 15 (d) 20
n-1 (a) (C 0C1C 2¼C n - 1 )(n + 1)
9. If Cr = ( k2 - 3) ×n Cr + 1, then k is belongs to
(b) (C 0C1C 2 ¼ C n - 1 ) (n + 1)n
(a) (- ¥ , - 2] (b) [2 , ¥ ) (c) [- 3 , 3 ] (d) ( 3 , 2]
(C 0C1C 2 ¼ C n - 1 ) (n + 1)n
10. If (5 + 2 6 ) n = I + f ; n, I Î N and 0 £ f < 1, then I is (c)
n!
equal to (d) None of the above
1 1 1 1
(a) -f (b) - f (c) + f (d) -f
f 1+ f 1+ f 1- f 20. C0 Cr + C1 Cr + 1 + C2 Cr + 2 + . . . + Cn - r Cn is equal to
(2 n )! n!
11. The coefficients of x 2 y 2 , yzt 2 and xyzt in the (a) (b)
(n - r )! (n + r )! r ! (n + r )!
expansion of ( x + y + z + t) 4 are in the ratio n!
(a) 4 : 2 : 1 (b) 1 : 2 : 4 (c) 2 : 4 : 1 (d) 1 : 4 : 2 (c) (d) None of these
(n - r )!
12. If sum of the coefficients of the first, second and
m 21. If the number of integral terms in the expansion of
third terms of the expansion of æç x 2 + ö÷ is 46,
1
n
è xø æ 1 1
ö
ç3 2 + 5 8 ÷ is exactly 33, then the least value of n is
then the coefficient of the term that does not ç ÷
è ø (JEE Main 2020)
contain x is
(a) 84 (b) 92 (c) 98 (d) 106 (a) 264 (b) 128 (c) 256 (d) 248

13. The last digit of 33


4n
+ 1, n Î N , is 22. If a and b be the coefficients of x and x 2 4

4 8 respectively in the expansion of


(a) C3 (b) C7 (c) 8 (d) 4
( x + x 2 - 1) 6 + ( x - x 2 - 1) 6 , then (JEE Main 2020)
14. The maximum value of the term independent of ‘t’
10
æ (1 - x)1/ 10 ö (a) a + b = - 30 (b) a - b = - 132
in the expansion of ç tx1/ 5 + ÷ where (c) a + b = 60 (d) a - b = 60
è t ø 6
x Î (0, 1) is (JEE Main 2021) 23. The value of å ( 6Cr × 6C6 - r ) is equal to
10 ! 2.10 ! 10 ! 2.10 ! r=0 (JEE Main 2021)
(a) (b) (c) (d)
3 (5 !)2 3(5 !)2 3(5 !)2 3 3 (5 !)2 (a) 1124 (b) 1324
(c) 1024 (d) 924
15. The coefficient of x 4 in the expansion of
(1 + x + x 2 + x 3)11 is 24. If ( 2 + 3) n = I + f where I and n are positive
(a) 900 (b) 909 (c) 990 (d) 999 integers and 0 < f < 1, then (1 - f ) ( I + f )
16. Let a > 0, b > 0 be such that a + b = 4. If the 3 2 (a) -1 (b) 1
(c) 2 (d) None of these
maximum value of the term independent of x in the
176 JEE Main Mathematics

16
25. In the expansion of æç
x 1 ö 34. The sum of coefficients of integral powers of x in
+ ÷ , if l1 is the
è cos q x sin q ø the binomial expansion of (1 - 2 x ) 50 is
least value of the term independent of x when 1 50 1 1 50 1
p p (a) (3 + 1) (b) (350 ) (c) (3 - 1) (d) (250 + 1)
£ q £ and l2 is the least value of the term 2 2 2 2
8 4
p p 35. If the coefficients of x 3 and x 4 in the expansion of
independent of x when £ q £ , then the ratio
16 8 (1 + ax + bx 2 ) (1 - 2 x)18 in powers of x are both zero,
l2 : l1 is equal to (JEE Main 2020) then ( a, b) is equal to (JEE Main 2014)
(a) 1 : 16 (b) 1 : 8 (c) 16 : 1 (d) 8 : 1 æ 251 ö æ 251 ö æ 272 ö æ 272 ö
(a) ç16, ÷ (b) ç14, ÷ (c) ç14, ÷ (d) ç16, ÷
è 3 ø è 3 ø è 3 ø è 3 ø
26. If the fourth term in the binomial expansion of
6
æ 2 + x log 8 x ö ( x > 0) is 20 ´ 87 , then the value of x is 36. The term independent of x in expansion of
ç ÷
èx ø 10
(JEE Main 2019) æ x+1 x-1 ö
(a) 8 -2
(b) 8 3
(c) 8 (d) 8 2 ç 2/ 3 - ÷ is
èx - x + 1 x - x1/ 2 ø
1/ 3
(JEE Main 2021)
27. The smallest natural number n, such that the (a) 4 (b) 120 (c) 210 (d) 310
n
coefficient of x in the expansion of æç x 2 + 3 ö÷ is
1
2n
è x ø 37. If n is a positive integer, then ( 3 + 1) - ( 3 - 1) 2 n is
n (a) an irrational number
C23, is (JEE Main 2019)
(a) 35 (b) 23 (c) 58 (d) 38 (b) an odd positive integer
(c) an even positive integer
7
28. The coefficient of x in the expansion of (d) a rational number other than positive integers
(1 - x - x 2 + x 3) 6 is
(a) - 132 (b) - 144 (c) 132 (d) 144 Numerical Value Types Questions
n
29. The value of r for which
38. For a positive integer n, æç1 + ö÷ is expanded in
1
20
Cr 20C0 + 20Cr -1 20C1 + 20
Cr - 2 20C2 + .... + 20 C0 20Cr è xø
is maximum, is (JEE Main 2019) increasing powers of x. If three consecutive
(a) 15 (b) 10 (c) 11 (d) 20 coefficients in this expansion are in the ratio,
50 50 2 : 5 : 12, then n is equal to ……… . (JEE Main 2020)
30. Let ( x + 10) + ( x - 10)
20
a
= a0 + a1x + a2 x 2 + K + a50 x 50 , for all x Î R; then
a2 39. Let ( 2 x 2 + 3x + 4)10 = å ar x r . Then a 7 is equal to
a0 r=0 13

is equal to (JEE Main 2019) ..... . (JEE Main 2020)


(a) 12.25 (b) 12.50 (c) 12.00 (d) 12.75
40. The natural number m, for which the coefficient of
31. The sum of the series 22
x in the binomial expansion of æç x m + 2 ö÷ is 1540,
1
20 20 20 20 20
C0 - C1 + C 2 - C3 + . . . + C10 is è x ø
20 1 20 20
(a) - C10 (b) C10 (c) 0 (d) C10 is …… . (JEE Main 2020)
2
41. The coefficient of x 4 in the expansion of
32. If the expansion in powers of x of the function
1 (1 + x + x 2 + x 3) 6 in powers of x, is ……… .
is a0 + a1x + a2 x 2 + a3x 3 + . . . , then
(1 - ax)(1 - bx) 42. If sum of the coefficients of all even powers of x in
an is the product (1 + x + x 2 + ¼ + x 2 n )
a n - bn a n + 1 - bn + 1 (1 - x + x 2 - x 3 + ¼ + x 2 n ) is 61, then n is equal to
(a) (b)
b-a b-a …… . (JEE Main 2020)
bn + 1 - a n + 1 bn - a n
(c) (d) 43. The coefficient of x 4 in the expansion of (1 + x + x 2 )10
b-a b-a
is….
33. For natural numbers m, n, if 25
44. If Cr = Cr and
m n 2
(1 - y) (1 + y) = 1 + a1 y + a2 y + . . .
C0 + 5 × C1 + 9 × C2 + . . . . + (101) × C25 = 2 25 × k, then k
and a1 = a2 = 10, then ( m, n) is
is equal to... . (JEE Main 2020)
(a) 35, 20 (b) 45, 35 (c) 35, 45 (d) 20, 65
Binomial Theorem and its Applications 177

Answers
Round I
1. (b) 2. (a) 3. (d) 4. (b) 5. (a) 6. (d) 7. (a) 8. (c) 9. (a) 10. (c)
11. (b) 12. (c) 13. (c) 14. (c) 15. (b) 16. (c) 17. (b) 18. (d) 19. (a) 20. (a)
21. (c) 22. (c) 23. (b) 24. (a) 25. (b) 26. (c) 27. (a) 28. (c) 29. (b) 30. (a)
31. (a) 32. (c) 33. (b) 34. (c) 35. (b) 36. (c) 37. (c) 38. (b) 39. (d) 40. (a)
41. (a) 42. (b) 43. (c) 44. (d) 45. (c) 46. (b) 47. (d) 48. (b) 49. (c) 50. (c)
51. (d) 52. (d) 53. (a) 54. (b) 55. (a) 56. (a) 57. (b) 58. (b) 59. (c) 60. (a)
61. (d) 62. (d) 63. (b) 64. (c) 65. (a) 66. (d)

Round II
1. (c) 2. (c) 3. (c) 4. (b) 5. (b) 6. (a) 7. (c) 8. (c) 9. (d) 10. (d)
11. (b) 12. (a) 13. (d) 14. (d) 15. (c) 16. (d) 17. (c) 18. (c) 19. (c) 20. (a)
21. (c) 22. (b) 23. (d) 24. (b) 25. (c) 26. (d) 27. (d) 28. (b) 29. (d) 30. (a)
31. (b) 32. (c) 33. (c) 34. (a) 35. (d) 36. (c) 37. (a) 38. (118) 39. (8) 40. (13)
41. (120) 42. (30) 43. (615) 44. (51)

Solutions 10
Round I æ 2ö
4. Given, (1 - x2)10 and ç x - ÷
1. In (x + a )100 + (x - a )100 n is even. è xø
n 100 To find Ratio of coefficient of x10 in (1 - x2)10 and the
\ Total number of terms = +1= + 1 = 51 10
2 2 æ 2ö
term independent of x in ç x - ÷ .
è xø
2. Given that, (1 + ax )n = 1 + 8x + 24x2 + ...
n n (n - 1 ) 2 2 Now, let (r + 1)th term in (1 - x2)10 contains x10.
Þ 1 + ax + a x + ... = 1 + 8x + 24x2 + ...
1 1 ×2 \ Tr + 1 = 10C r (- x2)r = (- 1)r 10C r x2r
10
On comparing the coefficients of x, x2, we get To get x , put 2r = 10 Þ r = 5
n (n - 1 ) 2 \ T6 = (- 1)5 10
C5 x10
na = 8, a = 24
1 ×2 Thus, coefficient of x10 is - 10C5 .
10
Þ na (n - 1 ) a = 48 æ 2ö
Again, let term be independent of x in ç x - ÷ .
Þ 8 (8 - a ) = 48 è xø
r
Þ 8 -a =6 Þ a =2 Þ n =4 æ 2ö
\ Tr + 1 = 10C r (x)10 - r ç - ÷ = (- 1)r 10C r (2)r (x)10 - 2r
è xø
3. Given expression is (x + x3 - 1 )6 + (x - x3 - 1 )6
To get term independent of x, put 10 - 2r = 0 Þ r = 5
10
= 2 [6C 0x6 + 6C 2x4 ( x3 - 1 )2 æ 2ö
Thus, the term independent of x in ç x - ÷ is -10C5 (2)5 .
+ 6C 4x2( x3 - 1 )4 + 6C 6 ( x3 - 1 )6 ] è xø
- 10C5 1 1
{Q (a + b)n + (a - b)n \ Required ratio = 10 = = = 1 : 32
n-2 2 - C5 (2)5 (2)5 32
n n
= 2 [ C 0a + C 2a n
b + nC 4a n - 4b4 + …]}
= 2 [ C 0x6 + 6C 2x4 (x3 - 1) + 6C 4x2(x3 - 1)2
6 5. We have, (1.002)12 or it can be rewritten as (1 + 0.002)12
+ 6C 6 (x3 - 1)3 ] Þ (1.002)12 = 1 + 12 C1(0.002) + 12
C 2(0.002)2
The sum of the terms with even power of x + 12C3 (0.002)3 + ...
6 6 6 4 6 8 6 2
= 2 [ C 0x + C 2(- x ) + C 4x + C 4x We want the answer upto 4 decimal places and as such
+ 6C 6 (-1 - 3x6 )] we have left further expansion.
= 2 [6C 0x6 - 6C 2x4 + 6C 4x8 + 6C 4x2 - 1 - 3x6 ] 12 × 11
\ (1.002)12 = 1 + 12 (0.002) + (0.002)2
Now, the required sum of the coefficients of even 1 ×2
12 × 11 × 10
powers of x in (x + x3 - 1 )6 + (x - x3 - 1 )6 + (0.002)3 + ...
1 ×2 ×3
= 2 [ 6C 0 - 6C 2 + 6 C 4 + 6C 4 - 1 - 3] = 1 + 0.024 + 2.64 ´ 10-4 + 1.76 ´ 10-6 + . . .
= 2 [1 - 15 + 15 + 15 - 1 - 3] = 2(15 - 3) = 24 = 1.0242
178 JEE Main Mathematics

6. (1 + x + x2 + x3 )n = {(1 + x )n (1 + x2 )n } Given, m - n = 3 Þ n = m - 3
n n 2
= (1 + C1x + C 2x + ... + C nx ) n n n2 - n m2 - m
and - mn + = -6
(1 + nC1 x2 + nC 2 x4 + ¼ + nC n x2n ) 2 2
Therefore, the coefficient of x4 = nC 2 + nC 2nC1 + nC 4 (m - 3) (m - 4) m2 - m
Þ - m (m - 3) + = -6
2 2
= nC 4 + nC 2 + nC1nC 2
Þ m2 - 7m + 12 - 2 m2 + 6m + m2 - m + 12 = 0
7. The given expression is Þ - 2 m + 24 = 0 Þ m = 12
1 + (1 + x ) + (1 + x )2 + ... + (1 + x )n being in GP.
13. In the expansion of (1 + x )2n , the general term
Let S = 1 + (1 + x ) + (1 + x )2 + ... + (1 + x )n
= 2nC k xk , 0 £ k £ 2 n
(1 + x )n + 1 - 1
= = x- 1 [(1 + x )n + 1 - 1 ] As given for r > 1, n > 2 , 2nC3 r = 2nC r + 2
(1 + x ) - 1
Þ Either 3 r = r + 2 or 3 r = 2 n - (r + 2) [Q nC r = nC n - r ]
\ The coefficient of xk in S.
Þ r = 1 or n = 2 r + 1
= The coefficient of xk + 1 in [(1 + x )n+ 1 -1] = n+ 1
C k+1
21 22 30
We take the relation only
8. (1 + x ) + (1 + x ) + ... + (1 + x )
n =2r + 1 [Q r > 1]
= (1 + x )21 [1 + (1 + x )1 + ... + (1 + x )9 ]
14. Let (r + 1)th , (r + 2)th and (r + 3)th be three consecutive
é (1 + x )10 - 1 ù
= (1 + x )21 ê ú terms.
ë (1 + x ) - 1 û Then, n
C r : nC r + 1 : nC r + 2 = 1 : 7 : 42
1
= [(1 + x )31 - (1 + x )21 ] Crn
1 r+1 1
x Now, = Þ = Þ n - 8r = 7 …(i)
n
Cr + 1 7 n-r 7
\ Coefficient of x5 in the given expression
n
1 Cr + 1 7 r+2 1
= Coefficient of x5 in [(1 + x )31 - (1 + x )21 ] and = Þ = Þ n - 7r = 13 …(ii)
x n
Cr + 2 42 n - r -1 6
= Coefficient of x6 in [(1 + x )31 - (1 + x )21 ]
= 31C 6 - 21C 6 On solving Eqs. (i) and (ii), we get n = 55

9. We have, (1 + t 2)12 (1 + t12) (1 + t 24 ) 15. (19)2005 + (11 )2005 - (9)2005


= (1 + 12
C1t 2 + 12
C 2 t 4 + ... + 12
C 6t12 = (10 + 9)2005 + (10 + 1 )2005 - (9)2005

+ ... + 12
C12 t 24 + ... ) (1 + t12 + t 24 + t36 ) = {92005 + 2005
C1 (9)2004 ´ 10 + K }

\ Coefficient of t 24 in (1 + t 2)12(1 + t12) (1 + t 24 ) + ( 2005C 0 + 2005


C1 10 + K ) - (9)2005
= 12 C 6 + 12 C12 + 1 = 12C 6 + 2 = ( 2005C1 92004 ´ 10 + multiple of 10)
+ (1 + multiple of 10)
10. The given sigma expansion \ Unit digit = 1
100
å Cm (x - 3)100 - m × 2m can be written as
100 16. Let a1 , a 2, a3 , a 4 be respectively the coefficients of
m= 0 (r + 1 )th, (r + 2)th , (r + 3)th and (r + 4)th terms in the
[(x - 3) + 2] 100
= (x - 1 ) 100
= (1 - x) 100 expansion of (1 + x)n. Then,

\ Coefficient of x 53
in (1 - x ) 100
= (-1 )53 100
C53 a1 = nC r , a 2 = nC r + 1 , a3 = nC r + 2, a 4 = nC r + 3
n n
= - 100C53 a1 a3 Cr Cr + 2
Now, + = n + n
a1 + a 2 a3 + a 4 n
Cr + Cr + 1 C r + 2 + nC r + 3
11. We have, (x - 1 ) (x - 2) (x - 3) ... (x - 100)
n
Number of terms = 100 n
Cr Cr + 2
= +
\ Coefficient of x99 in (x - 1 ) (x - 2) (x - 3) ... (x - 100) n+1
Cr + 1 n+1
Cr + 3
= (- 1 - 2 - 3 - ... - 100) n+1
[Q nC r + nC r + 1 = Cr + 1 ]
100 ´ 101
= - (1 + 2 + ... + 100) = - = - 5050 Cr n Cr + 2 æ n
n
n ö
2 n -1
= + çQ C r = C r -1÷
n+1 n n+1n è r ø
ì m(m - 1 ) x2 ü Cr Cr + 2
12. (1 + x )m (1 - x )n = í1 + mx + + ...ý r+1 r+3
î 2! þ r + 1 r + 3 2 (r + 2)
é ù = + =
n (n - 1 ) 2 n+1 n+1 n+1
ê1 - nx + x - ...ú
ë 2! û n n
Cr + 1 Cr + 1 2 a2
én - n 2
(m - m) ù 2 2 =2 n+1
=2 =
= 1 + (m - n ) x + ê - mn + ú x + ... Cr + 2 n
C r + 1 + nC r + 2 a 2 + a3
ë 2 2 û
Binomial Theorem and its Applications 179

é 3 1ù
17. Coefficients of pth, ( p + 1)th and ( p + 2)th terms in the ê + ú
ë 2 (1 + log10 x ) 4 û
expansion (1 + x)n are nC p - 1 , nC p , nC p + 1, respectively. Þ 20 ´ x = 200
3 1
Since, these are in AP. +
2(1 + log10 x ) 4
Þ x = 10
\ 2 nC p = nC p - 1 + nC p + 1
n! n! é 3 1ù
Þ2 = Þ ê + ú log10 x = 1
(n - p)! p ! (n - p + 1 )! ( p - 1 )! ë 2(1 + log10 x) 4 û
n! [applying log10 both sides]
+
(n - p - 1 )! ( p + 1 )! Þ [6 + (1 + log10 x)] log10 x = 4(1 + log10 x)
2 p
Þ = Þ (7 + log10 x) log10 x = 4 + 4 log10 x
(n - p)! p ! (n - p + 1 ) (n - p )! p !
Þ t 2 + 7t = 4 + 4t [let log10 x = t]
n-p
+ Þ t 2 + 3t - 4 = 0
(n - p) ! ( p + 1 ) p !
Þ t = 1, -4 = log10 x Þ x = 10, 10-4
2 p n-p
Þ = + Since, x>1
1 (n - p + 1 ) p + 1
\ x = 10
Þ n 2 - n (4 p + 1 ) + 4 p2 - 2 = 0
22. Given expression is (1 + x )24.
18. Given that, (1 + x - 2x2)6 = 1 + a1x + a 2x2 + ... + a12x12
Let two successive terms are (r + 1)th and (r + 2 )th
On putting x = 1 and x = - 1 and adding the results, terms
we get
64 = 2 (1 + a 2 + a 4 + ... + a12) Þ Tr + 1 = 24C rxr and Tr + 2 = 24C r+ 1x r + 1
1
\ a 2 + a 4 + a 6 + ... + a12 = 31 Now, ratio of coefficients =
4
19. (1 - x + x2)n = a 0 + a1x + a 2x2 + ... + a 2nx2n 24
Cr 1 r+1 1
On putting x = 1, we get Þ = Þ =
24
C r+ 1 4 24 - r 4
(1 - 1 + 1 )n = a 0 + a1 + a 2 + ... + a 2n
Þ 4r + 4 = 24 - r Þ r = 4
Þ 1 = a 0 + a1 + a 2 + ... + a 2n …(i)
\Required terms are 5th and 6th.
Again, putting x = - 1, we get
3n = a 0 - a1 + a 2 - ... + a 2n …(ii) 23. Here, T4 = nC3 (a )n - 3 (-2 b)3
On adding Eqs. (i) and (ii), we get and T5 = nC 4 (a )n - 4 (-2 b)4
3n + 1 Given, T4 + T5 = 0
= a 0 + a 2 + a 4 + ... + a 2n
2 Þ n
C3 (a )n - 3 (-2 b)3 + nC 4 (a )n - 4 (-2 b)4 = 0
2n 2n r
20. The general term of (1 + x) is Tr + 1 = C r x Þ (a )n - 4 (-2 b)3 [a nC3 + nC 4 (-2 b)] = 0
2n 2 2n 3 2n 4
T2 = C1x , T3 = C 2x , T4 = C3 x a 2 nC 4
Þ = n
Since , coefficients are in AP. b C3
2n
Þ C1 , 2nC 2, 2nC3 , are in AP
2 × n (n - 1 )(n - 2)(n - 3) 3 ×2 ×1
Þ 2 ´ 2nC 2 = 2nC1 + 2nC3 = ´
2n 2n
4 ×3 ×2 ×1 n (n - 1 ) (n - 2)
C C
Þ 2 = 2n 1 + 2n 3 n -3
C2 C2 =
2
2 2n - 3 + 1
Þ 2= +
(2n - 2 + 1) 3 24. Given expression is (1 + ax + bx2)(1 - 3x)15 in the
2 2n - 2 expansion of binomial (1 - 3x)15 the (r + 1)th term is
Þ 2= +
2n - 1 3 Tr + 1 = 15C r (-3x)r = 15C r (-3)r xr
Þ 2
2n - 9n + 7 = 0 Now, coefficient of x2, in the expansion of
\ 2n 2 - 9n = - 7 (1 + ax + bx2)(1 - 3x)15 is
15
æ ö
6 C 2(-3)2 + a15C1 (-3)1 + b 15C 0 (-3)0 = 0 (given)
ç æ 1 ö 1 ÷ Þ (105 ´ 9) - 45 a + b = 0 Þ 45a - b = 945 …(i)
21. Given binomial is ç ççè 1 + log10 x ÷÷ø ÷
ç x + x12 ÷ 3
Similarly, the coefficient of x , in the expansion of
è ø
(1 + ax + bx2)(1 - 3x)15 is
Since, the fourth term in the given expansion is 200. 15
3 C3 (-3)3 + a 15
C 2(-3)2 + b15C1 (-3)1 = 0 (given)
æ 1 ö 2 æ 1 ö3 Þ - 12285 + 945a - 45b = 0
\ 6
C3 ç x1 + log10 x ÷ ç x12 ÷ = 200
ç ÷ çè ÷ Þ 63a - 3b = 819 Þ 21a - b = 273 …(ii)
è ø ø
180 JEE Main Mathematics

n
From Eqs. (i) and (ii), we get æ1ö 2n !
\ Tn + 1 = 2n
C n (x)2n - n ç ÷ = C n x2 n - 2 n =
2n 2n
Cn =
24a = 672 Þ a = 28 è xø n!n!
So, b = 315 Þ (a , b) = (28, 315) (2n ) (2n - 1) (2n - 2) (2n - 3) (2n - 4) L 1
=
25. The (r + 1) th term in the expansion of (a + x) n
is n!n!
{(2n - 1) (2n - 3) (2n - 5) L 1} ´ {2n × 2 (n - 1) × 2 (n - 2) L 2}
given by Tr + 1 = nC ra n - rxr =
n!n!
\ 3rd term in the expansion of (1 + xlog 2 x )5 is {(2n - 1) (2n - 3) (2n - 5) L 1} ´ {2n × n (n - 1) (n - 2) L 1}
5
C 2(1)5 - 2(xlog 2 x )2 =
n! n!
Þ 5
C 2(1)5 - 2(xlog 2 x )2 = 2560 (given) {(2n - 1) (2n - 3) (2n - 5) L 1} 2n n !
=
Þ 10 (xlog 2 x )2 = 2560 n! n!
Þ x( 2log 2 x ) = 256 [Q n ! = n (n - 1) (n - 2) L 1]
Þ log 2 x2log 2 x = log 2 256 1 × 3 × 5 L (2n - 1) 2n
=
(taking log 2 on both sides) n!
Þ 2(log 2 x)(log 2 x) = 8 [Q log 2 256 = log 2 28 = 8] 29. Let the three consecutive terms in the binomial
(log 2 x)2 = 4 expansion of (1 + x)n+ 5 are
Þ log 2 x = ± 2 C r-1 , n+ 5C r and n+ 5 C r+ 1
n+ 5

Þ log 2 x = 2 or log 2 x = - 2 Now, according to the given information


n+ 5
Þ x=4 C r-1 : n+ 5C r : n+ 5C r+ 1 = 5 : 10 : 14
1 (n + 5)! (n + 5)!
or x = 2 -2 = Þ : :
4 (r - 1)! (n - r + 6) ! r !(n - r + 5)!
26. Since, the general term in the expansion of binomial (n + 5)!
= 5 : 10 : 14
10 æ 10 - r ö (r + 1)!(n - r + 4)!
ç ÷
æ kö
ç x - 2÷ is Tr + 1 = 10C r xè 2 ø
(- k)r x- 2r Þ
1 1 1
= 5 : 10 : 14
è x ø : :
(n - r + 6)(n - r + 5) r (n - r + 5) (r + 1)r
10 - 5 r
r 5
= 10C r (- k)r x 2
So, =
n - r + 6 10
Q Term is constant, so r = 2.
10
Þ 2r = n - r + 6 Þ n + 6 = 3r …(i)
\ C 2(- k)2 = 405 r+1 5
10 ´ 9 2 and =
Þ k = 405 Þ k2 = 9 Þ | k| = 3 n-r+5 7
2 Þ 7r + 7 = 5n - 5r + 25
n
æ 1ö Þ 5n + 18 = 12r …(ii)
27. Given, (1 + x)m × ç1 + ÷
è xø From Eqs. (i) and (ii), we have n = 6.
To find Term independent of x, So, the largest coefficient in the expansion is same as
the greatest binomial coefficient
ìï æ x + 1 ö üï
n
Coefficient of x° in í (1 + x)m × ç ÷ ý = 11C5 or 11C 6
ïî è x ø ï
þ 11 ! 11 ´ 10 ´ 9 ´ 8 ´ 7
= = = 462
ì (1 + x)m + n ü 5 !6 ! 5 ´4 ´3 ´2
Þ Coefficient of x° in í ý
î xn þ 30. Given expression is (1 + x) (1 - x)10 (1 + x + x2)9
Þ Coefficient of xn in (1 + x)m + n = (1 + x) (1 - x) [(1 - x) (1 + x + x2)]9 = (1 - x2) (1 - x3 )9
Þ Coefficient of xn in {m + nC r xr } Now, coefficient of x18 in the product
Þ m + nC n , put r = n. (1 + x) (1 - x)10 (1 + x + x2)9
m+ n m+ n
\ Term independent of x = C n or Cm. = coefficient of x18 in the product (1 - x2) (1 - x3 )9
n ìï æ 2ü n
2n = coefficient of x18 in (1 - x3 )9 - coefficient of x16
æ 1 ö 1ö ï æ 1ö
28. Given, ç x2 + 2 + 2÷ Þ íç x + ÷ ý Þ ç x + ÷ in (1 - x3 )9
è x ø ïî è x ø ïþ è xø th
Since, (r + 1) term in the expansion of
Here, 2n is even. (1 - x3 )9 is 9C r (- x3 )r = 9C r (- 1)r x3 r
Therefore, total terms (2n + 1) is odd. Now, for x18, 3r = 18 Þ r = 6
Thus, only one middle term exist. 16
and for x16, 3r = 16 Þ r = ÏN.
æ 2n ö 3
and ç + 1÷ i.e. (n + 1)th term will be middle term.
è 2 ø 9! 9 ´8 ´ 7
\Required coefficient is 9C 6 = = = 84
6 !3 ! 3 ´2
Binomial Theorem and its Applications 181

n 6
æ 1 ö æ 3ö
31. Last term of ç21/3 - ÷ is 34. Let a binomial ç2x2 - ÷ , it’s (r + 1) th term
è 2ø è x2 ø
n r
æ 1 ö æ 3ö
Tn + 1 = nC n (21/3 )n - n ç - ÷ = Tr + 1 = 6C r (2x2)6 - r ç - 2 ÷
è 2ø è x ø
1 (- 1 )n = 6C r (- 3)r (2)6 - r x12 - 2r - 2r
= nC n (- 1 )n n/ 2 = n/ 2
2 2 = 6C r (-3)r (2)6 - r x12 - 4r …(i)
Also, we have
Now, the term independent of x in the expansion of
log 3 8
æ 1 ö - (5 /3 ) log 3 23 -5
æ1 6
ç 5/3 ÷ =3 =2 x8 ö æ 2 3 ö
è3 ø ç - ÷ ç2x - 2 ÷ = the term independent of x in
è 60 81 ø è x ø
(- 1 )n
Thus, = 2- 5 1 æ 2 3ö
6
2n/ 2 the expansion of ç2x - 2 ÷
(- 1 )n (- 1 )10 60 è x ø
Þ =
2n/ 2 25 + the term independent of x in the
6
n x8 æ 2 3 ö
Þ = 5 Þ n = 10 expansion of - ç2x - 2 ÷
2 81 è x ø
4
æ 1 ö 6
C3
Now, T5 = T4 + 1 = 10C 4 (21/3 )10 - 4 ç - ÷ = (- 3)3 (2)6 - 3 x12 - 4 (3 ) [put r = 3]
è 2ø 60
10 ! 1/3 6 æ 1ö 6
= (2 ) (-1 )4 (2- 1/ 2)4 + ç- ÷ C (-3) (2)
5 6 - 5 12 - 4 (5 ) 8
x x
4 !6 ! è 81 ø 5
= 210 (2)2 (1 ) (2- 2) = 210 [put r = 5]
9
æ3 1ö 1 35 ´ 2(6)
32. The general term in the expansion of ç x2 - ÷ is = (- 3)3 23 +
è2 3x ø 3 81
9-r r
æ3 ö æ 1ö = 36 - 72 = - 36
Tr + 1 = 9C r ç x2÷ ç- ÷
è2 ø è 3x ø 2n
æ 1ö
9 -r r 35. Given expression is ç x + ÷ .
æ3ö æ 1 ö 18 - 3 r è 2x ø
= 9C r ç ÷ ç- ÷ x …(i)
è2ø è 3ø n
2n æ 1 ö
Now, the coefficients of the terms x , x 0 -1
and x -3
in \ Middle term = C n (x )n ç ÷
è 2 xø
9
æ3 2 1 ö 2 n! 1 × 3 × 5 ... (2 n - 1 )
ç x - ÷ is = =
è2 3x ø n ! n ! 2n n!
For x0, 18 - 3r = 0 Þ r = 6 10
æ1 ö
For x-1, there exists no integer value of r 36. ç + x sin x÷
èx ø
For x-3 , 18 - 3r = - 3 Þ r = 7
Here, n = 10 (even)
Now, the coefficient of the term independent of x in the
æ 10 ö
9 Þ Middle term = ç + 1÷ th = 6th
æ3 1ö è2 ø
expansion of (1 + x + 2x3 ) ç x2 - ÷
è2 3x ø 10 -5
æ1ö
9-6 6 T6 = 10C5 ç ÷ (x sin x )5
æ3ö æ1ö è xø
= 1 × 9C 6 (- 1 )6 ç ÷ ç ÷ +0
è2ø è3ø 63 1
Þ 252 (sin x )5 = Þ (sin x )5 =
9 -7 7 8 32
9 7æ3ö æ1ö
+ 2 × C7 (- 1 ) ç ÷ ç ÷ 1
è2ø è3ø Þ sin x = Þ sin x = sin p / 6
3 2
2
9 ×8 × 7 3 1 9 ×8 3 1 p
= × × + 2× (- 1 ) 2 × 7 \ x = n p + (-1 )n
1 × 2 × 3 23 36 1 ×2 2 3 6
7 2 17 8
= - = æp ö
18 27 54 37. Given expression is ç + 2÷
è2 ø
33. (1 - 3x + 7x2)(1 - x )16 Here, n = 8 (even)
æ8 ö
= (1 - 3x + 7x2)(16C 0 - 16C1x + 16C 2x2 + ¼ ) Þ Middle term = ç + 1÷ th = 5th
è2 ø
After multiplying, the term containing x is
T5 = 8C 4 ( p / 2)8- 4 (2)4
-16C1x - 3x16C 0x 8 ´ 7 ´6 ´5 p 4
Þ ´ ´ 24 = 1120 Þ P 4 = 16; p = ± 2
\ Coefficient of x = - 16 - 3 = - 19 4 ´ 3 ´ 2 ´ 1 24
182 JEE Main Mathematics

æn ö n
n-r
38. Since, n is even, therefore ç
è2
+ 1÷ th term is the middle
ø = na n - å n
Cn - r
[Q nC r = nC n - r ]
r=0
term.
n/ 2
= n an - b
æ1ö n
\ Tn = nC n/ 2 (x2)n/ 2 ç ÷ = 924 x6 (given) Þ 2 b = na n Þ b = an
+1 è xø 2
2

Þ x n/2 = x6 Þ n = 12
n
æ 1 + rx ö
50
44. Let E= å (-1 )r nC r çè 1 + nx ÷ø
r=0
æ 2 xö
39. Q (3 + 2x) 50 = 350 ç1 + ÷
è 3 ø æ 1 ö n

r
=ç ÷
è 1 + nx ø
å (-1 )r nC r (1 + rx)
æ 2 xö r=0
Here, Tr + 1 = 350 50C r ç ÷
è 3 ø æ 1 ö ïì
n n ü
r n ï
í å (-1 ) × C r + x å r (-1 ) C rý
r n
r -1
=ç ÷
æ 2 xö è 1 + nx ø ïî r = 0 r=0 ïþ
and Tr = 350 50C r - 1 ç ÷
è 3 ø æ 1 ö
1 =ç ÷ (0 + 0) = 0
But x= (given) è 1 + nx ø
5 [Q nC 0 - nC1 + nC 2 - nC3 + ¼ (-1 )n nC n = 0 ]
Tr + 1 50
C 2 1
\ ³ 1 Þ 50 r × ³1 æ30ö æ30ö æ30ö æ30ö æ30ö æ30ö æ30ö æ30ö
Tr Cr - 1 3 5 45. Let A= ç ÷ ç ÷ - ç ÷ ç ÷ + ç ÷ ç ÷ - ... + ç ÷ ç ÷
è 0 ø è10ø è 1 ø è11ø è 2 ø è12ø è20ø è30ø
Þ 102 - 2 r ³ 15 r Þ r £ 6
or A = 30C 0 × 30C10 - 30C1 × 30C11
40. Sum of the coefficients in the expansion of + 30C 2 × 30C12 - ... + 30C 20 × 30C30
n n n
(x - 2 y + 3z ) is (1 - 2 + 3) = 2 (put x = y = z = 1 ) = Coefficient of x20 in (1 + x )30 (1 - x )30
\ 2n = 128 Þ n = 7 = Coefficient of x20 in (1 - x2 )30
Therefore, the greatest coefficient in the expansion of 30

(1 + x )7 is 7C3 or 7C 4 because both are equal to 35.


= Coefficient of x20 in å (- 1 )r 30C r (x2 )r
r=0
20
æ sin a ö = (- 1 )10 30C10 [for coefficient of x20 , let r = 10]
41. The general term in the expansion of ç x cos a + ÷
è x ø
r
= 30C10
æ sin a ö
is 20
C r (x cos a )20 - r ç ÷ 46. We have, (1 + x )n = C 0 + C1x + C 2x2 + ... + C nxn …(i)
è x ø
n 2 n
= 20
C rx20 - 2r (cos a )20 - r (sin a )r æ 1ö 1 æ1ö æ1ö
and ç1 + ÷ = C 0 + C1 + C 2ç ÷ + ... + C n ç ÷ …(ii)
è xø x è xø è xø
For this term to be independent of x, we get
On multiplying Eqs. (i) and (ii) and taking the
20 - 2 r = 0 Þ r = 10
coefficient of constant terms in right hand side
Let b = Term independent of x
= C 02 + C12 + C 22 + ... + C n2
20
= C10 (cos a)10 (sin a )10 n
æ 1ö 1
æ sin 2 a ö
10 In right hand side (1 + x) n ç1 + ÷ or in n (1 + x)2n or
= 20
C10 (cos a sin a )10 = 20
C10 ç ÷ è xø x
è 2 ø
term containing xn in (1 + x )2n.
10
20 æ1ö Clearly, the coefficient of x n in (1 + x)2n is equal to
Thus, the greatest possible value of b is C10 ç ÷ .
è2ø
2n (2 n )!
2n
Cn = ×
42. Here, the greatest coefficient is C n. n!n!
n
C n + 1x n - 1 Þ x >
21
\ 2n
Cn x n > 2n
47. ( C1 - 10C1 ) + (21C 2 - 10C 2) + (21C3 - 10C3 )
n+1
+ ... + (21C10 - 10C10 )
n+1
and 2n
C nx n > 2nC n - 1x n + 1 Þ x < =( 21
C1 + 21
C 2 + ... + 21
C10 ) - (10C1
n
+ 10C 2 + ... + 10C10 )
æ n n + 1ö
\ x must lie in the interval ç , ÷ 1 21
èn+1 n ø = ( C1 + 21C 2 + ... + 21C 20 ) - (210 - 1)
2
n n
r n - (n - r ) 1
43. Let b = å n
Cr
= å n
Cr
= (21C1 + 21C 2 + ... + 21C 21 - 1) - (210 - 1)
2
r=0 r=0
1
n
1 n
n-r = (221 - 2) - (210 - 1) = 220 - 1 - 210 + 1
=n å n
Cr
- å n
Cr
2
r=0 r=0 = 220 - 210
Binomial Theorem and its Applications 183

48. Given equation can be rewritten as (1 + x )n + nx (1 + x )n - 1 = C 0 + 2 C1x + 3 C 2x2


E = a [nC 0 - nC1 + nC 2 - ... + (- 1 )n nC n ] + K + (n + 1 ) C nxn
+ [nC1 - (2) (nC 2) + (3) (nC3 ) - ... + (- 1 )n (n ) (nC n )] Put x = 1, we get

ÞE =0 + 0 =0 [by properties] C 0 + 2 C1 + 3 C 2 + K + (n + 1 ) C n
= 2n + n 2n - 1 = 2n - 1 (n + 2 )
49. We know, (1 - x)n = nC 0 - nC1x + nC 2x2 K (-1)n nC nxn
On integrating limit 0 to 1, we get 54. Let f = (8 - 3 7 )10, here 0 < f < 1
1 1
\ (8 + 3 7 )10 + (8 - 3 7 )10 is an integer, hence this is
ò0 (1 - x)n dx = ò (nC 0 - nC1x + nC 2x2 K (-1)n nC nxn ) dx
0 the value of n.
1
é - (1 - x)n + 1 ù 55. Given that, R = (2 + 3 )2n and f = R - [R]
Þ ê ú
ë n+1 û0 As 0 < 2 - 3 < 1, we get 0 < F = (2 - 3 )2n < 1
é n - 1 ù1
n
C1x2 nC 2x3 (-1)n nC nx We have, R + F = (2 + 3 )2n + (2 - 3 )2n
= ê nC 0x - + K ú
ë 2 3 n+1 û0 = 2 [2nC 022n + 2n
C 2 22n - 2( 3 )2
n n nn 2n - 4
1 C1 C2 (-1) C n + 2n
C 4 (2 ) ( 3 )4 + ¼ + 2n
C 2n ( 3 )2n ]
Þ0 + = nC 0 - + -K
n+1 2 3 n+1 Þ R + F is an even integer.
æ C1 ö æ C ö æ Cn ö Þ [R] + f + F is an even integer.
50. We have, ç1 + ÷ ç1 + 2 ÷ ... çç1 + ÷
è C0 ø è C1 ø è C n - 1 ÷ø Þ f + F is an integer.
But 0 £ f < 1 and 0 < F < 1 Þ 0 < f + F < 2
é n (n - 1 ) ù
æ nö ê 2! ú æ 1ö But the only integer between 0 and 2 is 1.
= ç1 + ÷ ê1 + ú ... çè1 + ÷ø
è 1ø ê n n Thus, f + F = 1 Þ 1 - f = F
ú
ë û Now, R (1 - f ) = RF = (2 + 3 )2n (2 - 3 )2n
(1 + n ) (1 + n ) (1 + n ) (1 + n ) (n + 1 )n = (4 - 3)2n = 12n = 1
= × × ... =
1 2 3 n n!
56. Given, (31/ 4 + 51/ 8 )60
51. We have, C 02 - 2 C12 + 3 C 22 n
- ... + (- 1 ) (n + 1 ) C n2 60
C r (31/ 4 )60- r × (51/ 8 )r
= [C 02 - C12 + C 22 - ... + (- 1 ) C n2 ]
n 60 - r r
60
C r (3) 4 × 58
- [C12 - 2 C 22 + 3 C32
- ... + (- 1 )n n C n2 ]
n! 1 n! For rational terms
= (- 1 )n/ 2 × - (-1 )n/ 2 - 1 × n r
æ nö æ nö 2 æ nö æ nö = k; 0 £ r £ 60
ç ÷ !ç ÷! ç ÷!ç ÷! 8
è2ø è2ø è2ø è2ø
0 £ 8k £ 60
n! æ nö
= (-1 )n/ 2 × × ç1 + ÷ 60
æ nö æ nö è 2ø 0£k£
ç ÷!ç ÷! 8
è2ø è2ø
0 £ k £ 7. 5
Therefore, the value of the given expression is
k = 0, 1, 2, 3, 4, 5, 6, 7
æ nö æ nö
2 ç ÷ !ç ÷ ! 60 - 8k
è2ø è2ø (n )! æ nö is always divisible by 4 for all value of k total
´ (-1 )n/ 2 × ç1 + ÷
n! æ nö æ nö è 2ø 4
ç ÷!ç ÷! rational terms = 8 total terms = 61 irrational terms = 53
è2ø è2ø
n - 1 = 53 - 1 = 52
= (-1)n/ 2 (2 + n )
52 is divisible by 26.
52. S1 = - 15C1 + 2 × 15C 2 - ...... - 15 15C15
57. We have,
= å15 r
r =1 (-1 ) × r
15
C 2 = 15 å15
r =1 (-1 )
r 14
C r-1
(1 + x + x2)n = a 0 + a1x + a 2x2 + a3 x3 + K + a 2nx2n
= 15(- 14C 0 + 14C1 - ...... - 14C14 ) = 15(0) = 0
On differentiating both sides, we get
S 2 = 14C1 + 14C3 + ...... + 14C11
= ( 14C1 + 14C3 + ...... + 14C11 + 14C13 ) - 14C13 n (1 + x + x2)n - 1 (1 + 2x ) = a1 + 2a 2x + 3a3 x2
= 213 - 14 + K + 2na 2nx2n - 1
= S1 + S 2 = 213 - 14 On putting x = - 1, we get

53. Since, x (1 + x )n = x C 0 + C1x2 + C 2x3 + K + C nxn + 1 n (1 - 1 + 1 )n - 1 (1 - 2) = a1 - 2a 2 + 3a3 - K - 2na 2n


Þ a1 - 2a 2 + 3a3 - K - 2na 2n = - n
On differentiating w.r.t. x, we get
184 JEE Main Mathematics

58. We have, (1 + x )15 = C 0 + C1x + C 2x2 + ... + C15 x15 1 1 1


63. Given that, + + + ...
15 1 ! (n - 1 )! 3 ! (n - 3)! 5 ! (n - 5)!
(1 + x ) - 1
Þ = C1 + C 2x + ... + C15 x14
x 1 é n! n! n! ù
= ê + + + ...ú
On differentiating both sides w.r.t. x, we get n ! ë 1 !(n - 1 )! 3 ! (n - 3)! 5 ! (n - 5)! û
x × 15 (1 + x )14 - (1 + x )15 + 1 1 n
= C 2 + 2 C3 x + ... + 14 C15 x13 = ( C1 + nC3 + nC5 + ... )
x2 n!
On putting x = 1, we get
2n - 1
C 2 + 2 C3 + ... + 14 C15 = 15 × 214 - 215 + 1 = 13 × 214 + 1 =
n!
59. We can write, 64. There are total (n + 1 ) factors, let P (x ) = 0
aC 0 - (a + d ) C1 + (a + 2 d ) C 2 - ... upto (n + 1 ) terms Let (x + nC 0 ) (x + 3 nC1 ) (x + 5 nC 2) ... [x + (2 n + 1 ) nC n ]
= a (C 0 - C1 + C 2 - ... ) + d (- C1 + 2 C 2 - 3 C3 + ... ) …(i) = a nxn + a n - 1xn - 1 + ... + a1x + a 0
We know, Clearly, a n = 1 and roots of the equation P (x ) = 0 are
(1 - x )n = C 0 - C1x + C 2x2 - ... + (- 1)n C n xn …(ii) - nC 0 , - 3 nC1 , ...
On differentiating Eq. (ii) w.r.t. x, we get Sum of roots = - a n -1 /a n = - nC 0 - 3 nC1 - 5 nC 2...
- n (1 - x)n - 1 = - C1 + 2 C 2x - ... + (- 1 )nC n nxn - 1 …(iii) Þ a n - 1 = (n + 1 ) 2n
On putting x = 1 in Eqs. (ii) and (iii), we get
65. The coefficient of x7 in (1 + 3x - 2x3 )10
C 0 - C1 + C 2 - ... + (- 1 )nC n = 0 ...(iv)
and - C1 + 2 C 2 - ... + (- 1)n n C n = 0 ...(v) =
å10! (1 )n1 (3 )n2 (-2 )n3 ,
n1 ! n2 ! n3 !
From Eq. (i),
where n1 + n2 + n3 = 10, n2 + 3n3 = 7
a C 0 – (a + d ) C1 + (a + 2d ) C 2 - ... upto (n + 1) terms
Different possibilities are as follow
= a × 0 + d × 0 = 0 [from Eqs. (iv) and (v)] n1 n2 n3
n
æ 1ö 3 7 0
60. Since, ç1 + ÷ < 3 for " n Î N
è nø 5 4 1
1000
(1001 )999 1 æ 1001 ö 7 1 2
Now, = ×ç ÷
(1000 )1000
1001 è 1000 ø \Coefficient of x7 =
10 !
(1 )3 (3)7 (-2)0
1000 3!7!
1 æ 1 ö 1
= ç1 + ÷ < ×3 < 1 10 !
1001 è 1000 ø 1001 + (1 )5 (3)4 (-2)1
5 !4 !1 !
(1001 )999 < (1000)1000 10 !
+ (1 )7 (3)1 (-2)2
\ B< A 7 !1 !2 !
200 100 = 62640
61. Q 3 = (1 + 8) = 1 + 100C1 (8) + 100C 2(8)2 + …
66. Consider,
3200 1 100
\ = + C1 + 100C 2(8) + 100C3 (8)2 + … 2403 = 2400 + 3 = 8 × 2400 = 8 × (24 )100 = 8 (16)100
8 8
1 = 8(1 + 15)100
= +n [where, n is a natural number]
8 = 8 (1 + 100C1 (15) + 100C 2(15)2 +¼ + 100C100 (15)100 )
ì 3200 ü ì 1 ü 1 [By binomial theorem,
\ í ý = í + ný =
8 î 8 þ 8 (1 + x)n = nC 0 + nC1x + nC 2x2 + ¼ nC nxn , n Î N ]
î þ
= 8 + 8 (100C1 (15) + 100
C 2(15)2 + ¼
62. x = ( 3 + 1 )5 = ( 3 )5 + 5C1 ( 3 )4 + 5C 2( 3 )3 100
+ C100 (15)100 )
5 2 5 5
+ C3 ( 3 ) + C 4 ( 3 ) + C5 = 8 + 8 ´ 15l
= 9 3 + 45 + 30 3 + 30 + 5 3 + 1 where l =100 C1 +......+ 100C100 (15)99 Î N
= 76 + 44 3 2403 8 + 8 ´ 15l 8
5
\ = = 8l +
\ [x] = [( 3 + 1 ) ] 15 15 15
= (76 + 44 3 ) ì 2403 ü 8
Þ í ý=
= (76) + (44 ´ 1.732) î 15 þ 15
= 76 + (76.2) [where {×} is the fractional part function]
= 76 + 76 = 152 \ k =8
Binomial Theorem and its Applications 185

Alternate Method 3. If nC 4, nC5 and nC 6 are in AP,


403 400 100
2 = 8 ×2 = 8(16) then 2 ×n C5 = nC 4 + nC 6
Note that, when 16 is divided by 15, gives remainder 1. [if a , b, c are in AP , then 2b = a + c]
\When (16)100 is divided by 15, gives remainder 1100 = 1 n! n! n!
Þ 2 = +
and when 8(16)100 is divided by 15, gives remainder 8. 5 !(n - 5)! 4 !(n - 4)! 6 !(n - 6)!
ì 2403 ü 8 é n n! ù
\ í ý= . êQ C r = r !(n - r )! ú
î 15 þ 15 ë û
[where {×} is the fractional part function] 2
Þ
Þ k =8 5 × 4 !(n - 5) (n - 6)!
1 1
Round II = +
4 !(n - 4) (n - 5) (n - 6)! 6 × 5 × 4 ! (n - 6)!
1. The general term in the binomial expansion of (a + b)n 2 1 1
Þ = +
is Tr + 1 = nC r a n - rbr. 5(n - 5) (n - 4) (n - 5) 30
So, the general term in the binomial expansion of 2 30 + (n - 4) (n - 5)
Þ =
(71/5
-31/10 60
) is 5(n - 5) 30 (n - 4) (n - 5)
Tr + 1 = 60
C r (71/5 )60 - r (-31/10 )r Þ 12 (n - 4) = 30 + n 2 - 9n + 20
60 - r r
60 Þ n 2 - 21n + 98 = 0
= Cr 7 5 (-1 )r 310
r r
Þ n 2 - 14n - 7n + 98 = 0
12 -
= (-1)r 60
Cr 7 5 310 Þ n (n - 14) - 7(n - 14) = 0
The possible non-negative integral values of ‘r’ for Þ (n - 7) (n - 14) = 0
r r Þ n = 7 or 14
which and are integer, where r £ 60, are
5 10
4. (1 + 2x)6 (1 - x)7 = [1 + 6C1 (2x) + 6C 2(2x)2 + 6C3 (2x)3
r = 0, 10, 20, 30, 40, 50, 60. 6
+ C 4 (2x)4 + 6C5 (2x)5 + 6C 6 (2x)6 ]
\There are 7 rational terms in the binomial expansion
and remaining 61 - 7 = 54 terms are irrational terms. ´ [1 - 7C1x + 7
C 2x2 - 7C3 x3 + 7
C 4x4... ]
= (1 + 12x + 60x2 + 160x3 + 240x4 + 192x5 + . . . )
2. Since, rth term from the end in the expansion
of a binomial (x + a )n is same as the (n - r + 2)th ´ (1 - 7x + 21x2 - 35x3 + 35x4 - 21x5 + ¼ )
term from the beginning in the expansion of same \ Coefficient of x5 in the expansion of (1 + 2x)6 (1 - x)7
binomial. = 1 ´ (- 21) + 12 ´ 35 + 60 ´ ( - 35)
T5
\Required ratio = + 160 ´ 21 + 240 ´ (- 7) + 192
T10 - 5 + 2
= - 21 + 420 - 2100 + 3360 - 1680 + 192
T T4 + 1
= 5 = = 171
T7 T6 + 1
5. We have,
4
æ 1 ö (x + 3)n - 1 + (x + 3)n - 2(x + 2) + (x + 3)n -3 (x + 2)2
10
C 4 (21/3 )10- 4 ç ÷
T5 è 2(3)1/3 ø + ... + (x + 2)n - 1
Þ = 6
T10 - 5 + 2 æ 1 ö n n
10
C 6 (21/3 )10- 6 ç ÷ (x + 3) - (x + 2)
è 2(3)1/3 ø =
(x + 3) - (x + 2)
[Q Tr + 1 = nC rxn - ra r ] = (x + 3)n - (x + 2)n
6/3 1/3 6
2 (2(3) ) æ x n - an n -1ö
10 10
= 4/3 [Q C 4 = C6 ]
2 (2(3)1/3 )4 çQ = x n -1 + xn - 2a1 + xn - 3 a 2 + ... + a ÷
è x-a ø
6/3 - 4/3 1/3 6 - 4
=2 (2(3) )
23
/
Therefore, the coefficient of x r in the given expression
=2 × 2 × 323
2 /
= coefficient of x r in [(x + 3)n - (x + 2)n ]
= 4(6)23
/
= nC r 3n - r - nC r 2n - r
= 4(36)1/3
= nC r (3n - r - 2n - r )
So, the required ratio is 4(36)1/3 : 1.
186 JEE Main Mathematics

n ì1 3r 7r ü Þ 3+
1
£ k2 £ 4
6. å (- 1 )r nC r í 2r + 22r + 23 r + ... upto m termsý n
r=0 î þ
n n r When n ® ¥ , 3 £ k2 £ 4 or k Î [-2 , - 3 ] È ( 3 , 2]
1 3
= å (- 1 ) r n
Cr × r +
2
å (- 1 ) × C r 22r
r n
10. QI + f + f ¢ = (5 + 2 6 )n + (5 - 2 6 )n = 2k [even integer]
r=0 r=0
n
7r \ f + f¢ =1
+ å (- 1 )r nC r 23 r + ... Now, (I + f ) f ¢ = (5 + 2 6 )n (5 - 2 6 )n = (1 )n = 1
r=0
1
æ 1ö æ
n
3ö æ 7ö
n n Þ (I + f ) (1 - f ) = 1 or I = -f
= ç1 - ÷ + ç1 - ÷ + ç1 - ÷ + ... upto m terms (1 - f )
è 2ø è 4ø è 8ø
4!
1 1 1 11. Coefficient of x2 y2 in (x + y + z + t )4 = = 6 and
= n + 2n + 3 n ... upto m terms 2 !2 !
2 2 2 4!
coefficient of yzt 2 in (x + y + z + t )4 = = 12
1 ìï æ 1 ö üï
m
- 1 !1 !1 !2 !
í ç ÷
è 2n ø ýï
1
2n ïî mn
þ = 2 -1 4!
= Also, coefficient of xyzt in (x + y + z + t )4 = = 24
æ 1ö 2mn (2n - 1 ) 1 !1 !1 !1 !
ç1 - n ÷
è 2 ø \Required ratio is 6 : 12 : 24 = 1 : 2 : 4
é 9x - 1 + 7 1 ù
7
12. Given, mC 0 + mC1 + mC 2 = 46
7. We have, ê2log 2 + x-1 ú
ë 2(1/5 ) log 2(3 + 1)
û Þ 2m + m (m - 1 ) = 90
é 1 ù
7 Þ m2 + m - 90 = 0 Þ m = 9 as m > 0
= ê 9x - 1 + 7 + x - 1 1/5 ú
m
(3 + 1 ) æ 1ö
ë û Now, (r + 1 )th term of ç x2 + ÷ is
5
è xø
é 1 ù
\ T6 = 7C5 ( 9x - 1 + 7 )7 - 5 ê x - 1 1/5 ú æ1ö
r

ë (3 + 1 ) û
m
C r (x2)m - r ç ÷ = mC r x2m - 3 r
è xø
7 x -1 1
= C5 (9 + 7) x - 1 For this to be independent of x put
(3 + 1)
x -1 2m - 3r = 0 Þ r = 6
(9 + 7)
Þ 84 = 7C5 x -1 \ Coefficient of the term independent of x is 9C 6 = 84.
(3 + 1)
13. Q 34n = 81n = (1 + 80)n = 1 + 80l, l Î N
Þ 9x - 1 + 7 = 4 (3x - 1 + 1 )
4n
3 2x æ 3x ö \ 33 = 31 + 80l = 3 × 380l = 3 × (9)40l = 3(10 - 1 )40l
Þ + 7 =4 ç + 1÷
9 è 3 ø = 3(1 + 10m ) = 3 + 30 m
4n
Þ 3 2x x
- 12 (3 ) + 27 = 0 \Last digit of 33 + 1 is 4.
Þ y2 - 12 y + 27 = 0 (put y = 3x ) é (1 - x)1/10 ù
r

Þ ( y - 3) ( y - 9) = 0 14. Tr+ 1 = 10C r (tx1/5 )10- r ê ú


ë t û
Þ y = 3, 9 Þ 3x = 3, 9 Þ x = 1, 2
10 - r r
m m
æ10ö æ 20 ö = 10C rt(10- 2r) ´ x 5 ´ (1 - x)10
8. Q å ç ÷ç ÷ = å Ci × Cm - 1
è i ø èm - i ø i = 0
10 20

i=0 Þ 10 - 2r = 0 Þ r = 5
= Coefficient of x m in the expansion of T6 = 10C5 x 1 - x
(1 + x )10 (1 + x ) 20 = 30Cm dT6 10 é x ù
= C5 ê 1 - x - =0
It is maximum, when dx ë 2 1 - x úû
30 = 1 - x = x / 2 Þ 3x = 2
m= = 15
2 Þ x = 2 /3
n -1 n -1
2 Cr Cr æ r + 1ö 10 ! 2
9. Q (k - 3) = = =ç ÷ …(i) T6|max = ´
n
Cr + 1 æ n ö n -1 è n ø 5 !5 ! 3 3
ç ÷ C r
è r + 1ø 15. Let E = (1 + x + x2 + x3 )11
Q 0 £ r £ n -1 = [(1 + x)(1 + x2)]11
1 r+1
Þ 1£r+1£n Þ £ £1 = (1 + x)11 (1 + x2)11
n n
\ General term of E is,
1
Þ £ k2 - 3 £ 1 [from Eq. (i)] T (r , k) = [11C rxr )(11C k (x2)k ]
n
Binomial Theorem and its Applications 187

=11 C r ×11 C k xr + 2k 19. (C 0 + C1 ) (C1 + C 2) ¼ (C n - 1 + C n )


For taking coefficient of x4 , we take
æ C ö æ C ö æ Cn ö
r + 2k = 4 = C 0 ç1 + 1 ÷ C1 ç1 + 2 ÷ ¼ C n - 1 çç1 + ÷
è C0 ø è C1 ø è C n - 1 ÷ø
r 0 1 2 3 4 5
æ C öæ C ö æ Cn ö
4 -r = {C 0C1C 2 ¼ C n - 1 } ç1 + 1 ÷ ç1 + 2 ÷ ... çç1 + ÷
k= 2 x 1 x 0 x è C0 ø è C1 ø è C n - 1 ÷ø
2
æ n -1ö æ 1ö
\ Coefficient of x4 = T ( 0 , 2) + T ( 2 , 1 ) + T ( 4 , 0) = {C 0C1C 2 ¼ C n - 1 } (n + 1) ç1 + ÷ ¼ ç1 + ÷
è 2 ø è nø
= 11C 011C 2 + 11
C 211C1 + C 411C 0
11
æ n + 1ö æ n + 1ö æ n + 1ö
= {C 0C1C 2 ¼ C n - 1 } (n + 1) ç ÷ç ÷ ¼ç ÷
= 55 + 55 ´ 11 + 330 è 2 øè 3 ø è n ø
= 55 + 605 + 330 = 990 {C 0C1C 2 ¼ C n - 1 } (n + 1)n
=
16. Given binomial is (ax1/ 9 + bx-1/ 6 )10 n!
Q The general term in the expansion of binomial is 20. We know,
Tr + 1 = 10C r (a x1/ 9 )10- r (bx-1/ 6 )r (1 + x )n = C 0 + C1x + C 2x2 + ... + C rx r + ... …(i)
æ 10 - r r ö n
ç - ÷ æ 1ö 1 1 1
10 - r r è 9 6ø
= 10C r a bx and ç1 + ÷ = C 0 + C1 + C 2 2 + ... + C r r
è xø x x x
æ 20 - 2r - 3 r ö
ç ÷ 1 1 1
= 10C r a10 - r b rxè 18 ø
+ C r + 1 r + 1 + C r + 2 r + 2 ... C n n …(ii)
x x x
For the term independent of x,
On multiplying Eqs. (i) and (ii), equating coefficient of
20 - 2r - 3r = 0 Þ r = 4
1
So, the value of the term independent of x is xr in n (1 + x) 2n or the coefficient of x n + r in (1 + x) 2n,
10
x
C 4 a 6b 4 = 210 a 6b 4
Now, it is given that a > 0, b > 0, such that a3 + b 2 = 4,
so according to AM-GM in equality, we have we get the value of required expression which is
(2 n )!
a3 + b 2 4 2n
Cn + r =
³ a3b 2 Þ ³ a3b 2 Þ a 6b 4 £ 16 (n - r )! (n + r )!
2 2
It is given that, the maximum value of 21. Given binomial is (31/ 2 + 51/ 8 )n having general term
6 4 n-r
210 a b = 10 k
Tr + 1 = nC r 3 2 5r/ 8, where 0 £ r £ n.
Þ 210 ´ 16 = 10 k Þ k = 336
3 r n-r
æ 1 - t6 ö Now, for integral terms Î I and Î I.
17. Clearly, ç ÷ = (1 - t 6 )3 (1 - t )- 3 8 2
è1-t ø
Þ r = 0, 8, 16, 24, .....
\ Coefficient of t 4 in (1 - t 6 )3 (1 - t )-3 Q There are 33 integral terms, so maximum value of
= Coefficient of t 4 in (1 - t18 - 3t 6 + 3t12) (1 - t )- 3 r = 256
= Coefficient of t 4 in (1 - t )- 3 \The least value of n is 256.
= 3 + 4 - 1C 4 = 6C 4 = 15 Hence, option (c) is correct.
[Q coefficient of xr in (1 - x)- n = n + r -1
C r] 22. Given expression is (x + x2 - 1 )6 + (x - x2 - 1 )6
n
Cr n-r+1
18. Since, we know that n
= = 2 [6C 0x6 + 6C 2 x4 (x2 - 1) + 6C 4x2(x2 - 1)2
Cr - 1 r
+ 6C 6 (x2 - 1)3 ]
C1 C n - 1 C3 n - 2 The coefficients of x4 in above expansion
\ = n, 2 = , = ,…
C0 C1 2 C2 3 a = 2 [6C 2(-1) + 6C 42C1 (-1) + 6C 63C1 (-1)]
C1 C C n -1 = 2 [-15 - (15 ´ 2) - (1 ´ 3)] = - 96
Thus, + 2 2 + 3 3 + ¼ = n + 2×
C0 C1 C2 2 and the coefficient of x2 in the expansion
n -2 1
+ 3× +¼+ n × b = 2 [6C 44C 0 + 6C 63C 2]
3 n
= 2 [15 + 3] = 36
= [n + (n - 1) + (n - 2) +¼ + 1 ]
n (n + 1) \ a - b = - 96 - 36 = - 132
= Hence, option (b) is correct.
2
188 JEE Main Mathematics

6 6
23. å 6C r × 6C 6 - r æ2 log 8 x ö
26. Given binomial is ç + x ÷
r= 0 èx ø
= 6C 0 × 6C 6 + 6C1 × 6C5 + .... + 6C 6 × 6C 0 Since, general term in the expansion of (x + a )n is
Now, (1 + x)6 (1 + x)6 Tr+ 1 = nC r xn- ra r
6 6 6 2 6 6
= ( C 0 + C1x + C 2x + .... + C 6x ) 6 -3
æ2ö
6 6 6 2 6 6 \ T4 = T3 + 1 = 6 C3 ç ÷ (xlog 8 x )3 = 20 ´ 87 [given]
( C 0 + C1x + C 2x + .... + C 6x ) è xø
Comparing coefficient of x6 both sides 3
æ2ö
6
C 0 × 6C 6 + 6C1 × 6C5 + .... + 6C 6 × 6C 0 = 12C 6 = 924 Þ 20 ç ÷ x3 log 8 x = 20 ´ 87 [Q 6C3 = 20]
è xø
24. I + f = (2 + 3 )n= 2n + nC1 2n-1 3 + nC 2 2n- 2( 3 )2 æ3 ö
ç log 2 x -3 ÷
n
+ C3 2 n-3 3
( 3 ) + . . . … (i) Þ 23 x[3(log 8 x )-3 ] = (23 )7 Þ xè 3 ø
= (23 )6
Now, 0 <2 - 3 <1 é 1 ù
Þ 0 < (2 - 3 )n < 1 êëQ log an (x) = n log a x for x > 0; a > 0, ¹ 1úû
Let (2 - 3 )n = f ¢ where 0 < f ¢ < 1 ( log x - 3 )
Þ x 2 = 218
Þ f ¢ = 2n - nC12n-1 3 + nC 22n- 2( 3 )2 On taking log 2 x both sides, we get
- nC3 2n-3 ( 3 )3 + . . . … (ii) (log 2 x - 3) log 2 x = 18
On adding Eqs. (i) and (ii), we get
Þ (log 2 x)2 - 3 log 2 x - 18 = 0
I + f + f ¢ = 2[2n + nC 22n- 2 × 3 + ¼ ] …(iii)
Þ (log 2 x)2 - 6 log 2 x + 3 log 2 x - 18 = 0
Þ I + f + f ¢ = even integer
Þ log 2 x(log 2 x - 6) + 3 (log 2 x - 6) = 0
Now, 0 < f <1
Þ (log 2 x - 6) (log 2 x + 3) = 0
and 0 < f ¢ <1 Þ 0 < f + f ¢ <2
Hence, from Eq. (iii) we conclude that f + f ¢ is an Þ log 2 x = -3, 6
integer between 0 and 2. Þ x = 2 -3 , 2 6
Þ f + f ¢ =1 Þ f ¢ =1 - f …(iv) Þ
1
x = , 82
\ I + f = (2 + 3 )n , f ¢ = 1 - f = (2 - 3 )n 8
n
Þ (I + f ) (1 - f ) = [(2 + 3 ) (2 - 3 )]n = (4 - 3)n = 1 æ 1ö
27. Given binomial is ç x2 + th
÷ , its (r + 1) term, is
Þ (I + f ) (1 - f ) = 1 è x3 ø
16 r
æ x 1 ö æ1ö 1
25. The general term in the expansion of ç + ÷ Tr + 1 = nC r (x2)n - r ç 3 ÷ = nC rx2n - 2r 3 r
è cos q x sin q ø èx ø x
16 - r r
æ x ö
is Tr + 1 = 16C r ç
æ 1 ö = nC rx2n - 2r - 3 r = nC rx2n - 5 r
÷ ç ÷
è cos q ø è x sin q ø For the coefficient of x ,
16 - r
æ 1 ö
r
æ 1 ö 16 - 2r 2n - 5r = 1
= 16C r ç ÷ ç ÷ x
è cos q ø è sin q ø Þ 2n = 5r + 1 …(i)
If the term is independent of ‘x’, then As coefficient of x is given as nC 23 , then either r = 23 or
n - r = 23 .
16 - 2r = 0 Þ r = 8
If r = 23, then from Eq. (i), we get
So, the term independent of x is
2n = 5(23) + 1
1 28 1 6C 8
T9 = 16C 8 8
= Þ 2n = 115 + 1 Þ 2n = 116 Þ n = 58.
(sin q cos q) sin 8 (2 q) If n - r = 23, then from Eq. (i) on replacing the value of
p p ‘r’, we get 2n = 5(n - 23) + 1
Now, the least value of T9, when £ q £ is
8 4 Þ 2n = 5n - 115 + 1 Þ 3n = 114 Þ n = 38
28 16C 8 æ pö So, the required smallest natural number n = 38.
I1 = = 28 (16C 8 ) çat q = ÷
(1)8 è 4ø 28. Here, (1 - x - x2 + x 3 )6 = {(1 - x) - x2 (1 - x )}6
p p
and the least value of T9, when £ q £ is = {(1 - x ) (1 - x2 )}6
16 8
= (1 - x)6 × (1 - x2 )6
28 16C 8 æ pö
I2 = = 212(16C 8 ) çat q = ÷ ìï 6 üï ìï 6 üï
æ 1 ö
8 è 8ø = í å (- 1 )r 6C r × xrý í å (- 1 )s 6C s × x2sý
ç ÷ ïî r = 0 ïþ ïî s = 0 ïþ
è 2ø
6 6
\ I 2 : I1 = 16 : 1 = å å (- 1 )r + s ×6 C r ×6 C s × x r + 2s
r=0 s=0
Hence, option (c) is correct .
Binomial Theorem and its Applications 189

For coefficient of x7 , + r + 2s = 7 Þ 20
C10 = 2(20C 0 - 20
C1 + . . . + 20
C10 )
i.e. (s = 1, r = 5) or (s = 2, r = 3) or (s = 3, r = 1 ) Þ 20
C0 - 20
C1 + . . . + 20
C10
\Coefficient of x7 is 1 20
{( - 1 ) 5 + 1×6 C5 ×6 C1 } + {(- 1 ) 3 + 2×6 C3 ×6 C 2} = C10
2
+ {(- 1) 1 + 3 ×6 C1 ×6 C3 }
32. Q (1 - ax )-1 ( 1 - bx )-1
= (36) - (20) (15) + 6(20)
= 36 - 300 + 120 = - 144 = (1 + ax + a 2x2 + . . . )(1 + bx + b2x2 + . . . )

29. We know that, \ a n = coefficient of x n in (1 - ax)-1 (1 - bx )-1

(1 + x)20 = 20
C 0 + 20C1x + 20C 2x2 + ... + = a 0bn + abn - 1 + . . . + a nb0
20
C r - 1xr - 1 + 20C rxr + ... + 20
C 20x20 é a æaö
2 ù
= a 0bn ê1 + + ç ÷ + . . .ú
\ (1 + x)20 × (1 + x)20 = (20C 0 + 20
C1x + êë b è b ø úû
20 2 20 r -1 20 20
C 2x + ... + C r - 1x + c20x20 )
C rxr + ... + é æ a ön + 1 ù
20 20 20 20 r -1 êç ÷ - 1ú
´ ( C0 + C1x + ...+ C r - 1x + C r xr è bø
+ ....+ C 20x20 )
20 = a 0bn ê ú
ê a ú
ê - 1 ú
Þ (1 + x)40 = (20C 0 . 20
C r + 20C1 20C r - 1 ... b
ë û
20
C r20C 0 ) xr + ...
bn (a n + 1 - bn + 1 ) b
r
= × n+1
On comparing the coefficient of x of both sides, we get a-b b
20
C 020C r + 20
C120C r - 1 + ... + 20
Cr 20
C0 = 40
Cr a n + 1 - bn + 1
=
The maximum value of 40
C r is possible only when a-b
n
r = 20 [Q C n/2 is maximum when n is even] 33. (1 - y)m (1 + y)n = 1 + a1 y + a 2y2 + a3 y3 + . . .
Thus, required value of r is 20. On differentiating w.r.t. y, we get
30. We have, (x + 10) 50
+ (x - 10) 50
= a 0 + a1x + a 2x 2 -m(1 - y)m - 1 (1 + y)n + (1 - y)m n (1 + y)n - 1
+ ¼ + a50x50 = a1 + 2a 2y + 3a3 y2 + . . . …(i)
\ a 0 + a1x + a 2x + ¼ + a50x 2 50
On putting y = 0 in Eq. (i), we get
-m + n = a1 = 10 [Q a1 = 10, given] …(ii)
= [(50C 0x50 + 50
C1x49 10 + 50
C 2 x48 × 102
50
On again differentiating Eq. (i), we get
+ ¼+ C50 1050 )
-m[- (m - 1 )(1 - y)m - 2(1 + y)n + (1 - y)m - 1 n (1 + y)n - 1 ]
50 50 50 49 50 48 2
+ ( C 0x - C1x 10 + C 2 x 10
+ n [-m(1 - y)m - 1 (1 + y)n - 1 + (1 - y)m (n - 1)(1 + y)n - 2]
50 50
-¼+ C50 10 )]
= 2a 2 + 6a3 y + . . . …(iii)
50 50 50 48 2 50
= 2 [ C0 x + C 2x × 10 + C 4x × 104
46
On putting y = 0 in Eq. (iii), we get
50
+ ¼+ C50 × 1050 ] -m[- (m - 1 ) + n ] + n [-m + (n - 1 )] = 2a 2 = 20
By comparing coefficients, we get Þ m(m - 1 ) - mn - mn + n (n - 1 ) = 20
a 2 = 2 50C 48 (10)48 ; a 0 = 2 50C50 (10)50 = 2(10)50 Þ m2 + n 2 - m - n - 2 mn = 20
a 2 2(50C 2)(10)48 50 × 49 (10)48 Þ (m - n )2 - (m + n ) = 20
\ = =2 Þ 100 - (m + n ) = 20
a0 2 (10)50
1 × 2 2 × (10)50
Þ m + n = 80 …(iv)
50
[Q C 48 = 50C 2]
On solving Eqs. (ii) and (iv), we get
50 ´ 49 5 ´ 49 245
= = = = 12 .25 m = 35and n = 45
2 × (10 ´ 10) 20 20
34. Let Tr+1 be the general term in the expansion of
31. We know that,
(1 - 2 x )50
(1 + x) 20 = 20C 0 + 20
C1x + . . . + 20
C10x10 + . . . + 20
C 20x20
\ Tr+ 1 = 50C r (1)50- r (-2x1/ 2)r = 50C r2r xr/ 2(-1)r
On putting x = - 1, we get
For the integral power of x, r should be even integer.
20 20 20 20 20 20
0= C0 - C1 + . . . - C9 + C10 - C11 + . . . + C 20 25

Þ0 = 20
C0 - 20
C1 + . . . - 20
C9 + 20
C10 - 20
C9 + . . . + 20
C0
\ Sum of coefficients = å 50C 2r (2)2r
r= 0
n n
[Q C r = C n- r ] 1 1
= [(1 + 2)50 + (1 - 2)50 ] = [350 + 1]
Þ 0 = 2(20C 0 - 20
C1 + . . . - 20
C9 ) + 20
C10 2 2
190 JEE Main Mathematics

Aliter We have, For independent of x, put


(1 - 2 x ) 50
= C 0 - C12 x + C 2(2 x ) 2 10 - r r
- =0
+ ...+ C50 (2 x )50…(i) 3 2
(1 + 2 x )50 = C 0 + C12 x + C 2(2 x )2 Þ 20 - 2 r - 3 r = 0
+ ... + C50 (2 x )50…(ii) Þ 20 = 5 r Þ r = 4
On adding Eqs. (i) and (ii), we get 10 ´ 9 ´ 8 ´ 7
\ T5 = 10C 4 = = 210
(1 - 2 x )50 + (1 + 2 x )50 = 2[C 0 + C 2(2 x )2 4 ´3 ´2 ´1
+ ... + C50 (2 x )50 ] 37. ( 3 + 1 )2n = 2nC 0 ( 3 )2n + C1 ( 3 )2n - 1 +
2n 2n
C 2 ( 3 )2n - 2
50 50
(1 - 2 x ) + (1 + 2 x ) + ... + 2n
C 2n ( 3 )2n - 2n
Þ = C 0 + C 2(2 x )2
2
and ( 3 - 1 )2n = 2n
C 0 ( 3 )2n (- 1 )0 + 2n
C1 ( 3 )2n - 1 (- 1 )1
+ ... + C50 (2 x )50
On putting x = 1, we get + C 2 ( 3 )2n - 2 (- 1 )2 + ....+
2n 2n
C 2n ( 3 )2n - 2n (- 1 )2n
(1 - 2 1 )50 + (1 + 2 1 )50 \ ( 3 + 1 )2n - ( 3 - 1 )2n = 2 [2nC1 ( 3 )2n - 1
= C 0 + C 2(2)2
2 + 2n
C3 ( 3 )2n - 3 + C5 ( 3 )2n - 5
2n

+ ... + C50 (2)50


50 50 + ... + 2n
C 2n - 1 ( 3 )2n - ( 2n - 1) ]
(-1) + (3)
Þ = C 0 + C 2(2)2 + ... + C50 (2)50 which is most certainly an irrational number because of
2
1 + 350 odd powers of 3 in each of the terms.
Þ = C 0 + C 2(2)2 + ... + C50 (2)50
2 38. Let the three consecutive coefficients nC r-1 , nC r and
2 18
35. In the expansion of (1 + ax + bx ) (1 - 2x) , n æ 1ö
n
C r+1are in ratio 2 : 5 : 12 in the expansion of ç1 + ÷ .
3
Coefficient of x in (1 + ax + bx ) (1 - 2x) 2 18 è xø
= Coefficient of x3 in (1 - 2x)18 \ n
C r-1 : nC r : nC r+ 1 = 2 : 5 : 12
2 18
+ Coefficient of x in a (1 - 2x) n!
n
+Coefficient of x in b (1 - 2x)18 C r-1 2 (r - 1)! (n - r + 1)! 2
So, n
= Þ =
= - 18C3 × 23 + a18C 2 × 22 - b18C1 × 2 Cr 5 n! 5
Q - 18C3 × 23 + a18C 2 × 22 - b18C1 × 2 = 0 r !(n - r )
18 ´ 17 ´ 16 18 ´ 17 2 Þ
r 2
= Þ 2n - 7r + 2 = 0
Þ ×8 + a× × 2 - b × 18 × 2 = 0 …(i)
3 ´2 2 n - r+1 5
34 ´ 16 n!
Þ 17a - b = …(i) n
3 Cr 5 r !(n - r )! 5
and = Þ =
Similarly, coefficient of x4 n
C r+ 1 12 n! 12
18
C 4 × 24 - a × 18C3 23 + b × 18C 2 × 22 = 0 (r + 1)! (n - r - 1)!
\ 32a - 32b = 240 …(ii) r+1 5
Þ =
On solving Eqs. (i) and (ii), we get n - r 12
272 Þ 5n - 17r - 12 = 0 ...(ii)
a = 16 and b =
3 On the elimination of ‘r’ from Eqs. (i) and (ii), we get
10
é x+1 (x - 1 ) ù 35n - 119r - 84 = 0
36. ê 2/ 3
- ú 34n - 119r + 34 = 0
ëx - x + 1 x - x1/ 2 û
1/3

10 - + -
é (x1/3 )3 + 1 3 {( x )2 - 1 } ù
= ê 2/ 3 1/3
- ú n - 118 = 0 Þ n = 118
ëx -x +1 x( x - 1 )û
10 Hence, answer is 118.00.
é (x1/3 + 1) (x23
/
+ 1 - x1/3 ) {( x )2 - 1} ù 20
=ê - ú
ë x 2/ 3 1/3
-x +1 x( x - 1 )û 39. It is given that, (2x2 + 3x + 4)10 = å a r xr
r=0
10
é ( x + 1 )ù -1/ 2 10 Now, the general term in the expansion of trinomial
= ê (x1/ 3 + 1 ) - ú
1/3
= (x -x )
ë x û 10 !
(2x2 + 3x + 4)10 is 2n1 3n24n3 x2n1 + n2
\The general term is n1 ! n2 ! n3 !
Tr + 1 = 10C r (x1/3 )10 - r (- x-1/ 2)r such that n1 + n2 + n3 = 10 and n1 , n2, n3 ³ 0
10 - r r
- For a7, 2n1 + n2 = 7
= 10C r (-1 ) r x 3 2
Binomial Theorem and its Applications 191

10 ! 0 7 3 10 ! 1 5 4 41. The coefficient of x4 in the expansion of (1 + x + x2 + x3 )6


\ a7 = 2 34 + 23 4 6
0 ! 7 !3 ! 1 !5 !4 ! æ 1 – x4 ö
10 ! 2 3 5 10 ! 3 1 6 = coefficient of x4 in ç ÷
+ 23 4 + 2 34 è 1– x ø
2 !3 !5 ! 3 !1 !6 !
= coefficient of x4 in (1 – 6x4 ) (1 – x)–6
n1 n2 n3 = coefficient of x4 in (1 – 6x4 )[1 + 6C1x + 7C 2x2 + .... ]
0 7 3 9 ´8 ´ 7 ´6
= 9C 4 – 6 = – 6 = 126 – 6 = 120
1 5 4 4 ´3 ´2
2 3 5 42. Let (1 + x + x2 +¼ + x2n )(1 - x + x2 - x3 + ¼ + x2n )
3 1 6 = a 0 + a1x + a 2x2 + ¼ + a 4nx4n
On putting x = 1, we get
and, for a13 , 2n1 + n2 = 13 (1 + 1 + 1 + ¼ + 1)(1 - 1 + 1 - 1 + ¼ + 1)
10 ! 3 7 10 ! 4 5 1 (2n + 1) terms (2n + 1) terms
\ a13 = 23 + 234
3 ! 7 !0 ! 4 !5 !1 ! = a 0 + a1 + a 2 + ¼ + a 4n
+
10 ! 5 3 2
234 +
10 ! 6 1 3
2 34 Þ (2n + 1) = a 0 + a1 + a 2 + ¼ + a 4n …(i)
5 !3 !2 ! 6 !1 !3 ! Similarly, on putting x = - 1, we get
n1 n2 n3
(1 - 1 + 1 - 1 + ¼ + 1)(1 + 1 + 1 + ¼ + 1)
(2n + 1) terms (2n + 1) terms
0 13 Not possible = a 0 - a1 + a 2 - a3 + ¼ + a 4n
1 11 Not possible Þ (2n + 1) = a 0 - a1 + a 2 - a3 ¼ + a 4n …(ii)
2 9 Not possible On adding Eqs. (i) and (ii), we get
3 7 0 2(a 0 + a 2 + a 4 + ¼ + a 4n ) = 2(2n + 1)
4 5 1 Þ a 0 + a 2 + a 4 + ¼ + a 4n = 2n + 1
5 3 2
Þ Sum of coefficients of all even powers of x in the
product (1 + x + x2 + ¼ + x2n ) (1 - x + x2 - x3 + ¼ + x2n )
6 1 3
= 2n + 1 = 61 (given)
10 ! 7 3 10 ! 1 5 4 Þ 2n = 60 Þ n = 30
34 + 23 4
7 !3 ! 5 !4 !
43. The expression (1 + x + x2)10 have general term in the
10 ! 2 3 5 10 ! 3 1 6
+ 23 4 + 2 34 10 !
\
a7
= 2 ! 3 !5 ! 3 !6 ! expansion is (1)n1 xn2 (x2)n3 ,
a13 10 ! 3 7 10 ! 4 5 1 n1 ! n2 ! n3 !
23 + 234 10 !
3!7! 4 !5 ! where n1 + n2 + n3 = 10 = xn2 + 2n3
10 ! 5 3 2 10 ! 6 1 3 n1 ! n2 ! n3 !
+ 234 + 2 34
5 !3 !2 ! 6 !3 ! Now, table for n1, n2 and n3 such that n2 + 2n3 = 4
é 10 ! 3 7 10 ! 6 5 10 ! 9 3 10 ! 12 1 ù n1 n2 n3
23 + 23 + 23 + 2 3 ú
3 ê 7 !3 ! 4 ! 5 ! 2 ! 3 !5 ! 6 !3 ! 6 4 0
=2 ê
10 ! 3 7 10 ! 6 5 10 ! 9 3 10 ! 12 1 ú 7 2 1
ê 23 + 23 + 23 + 2 3 ú
ë 3!7! 4 !5 ! 2 !3 !5 ! 6 !3 ! û 8 0 2
= 23 = 8 So, coefficient of x4 in the expansion of (1 + x + x2)10 is
Hence, answer is 8.00. 10 ! 10 ! 10 !
+ + = 210 + 360 + 45 = 615
40. The general term (i.e. (r + 1)th term) in the expansion of 6 !4 !0 ! 7 !2 !1 ! 8 !0 !2 !
22 Hence, answer is 615.
æ 1ö
binomial ç xm + 2 ÷ is
è x ø 44. As it is given C r = 25C r, so
Tr + 1 = 22
C r (xm )22 - r x-2r C 0 + 5 × C1 + 9 × C 2 + K + (101) × C 25
22 25 25 25
Q The coefficient C r = 1540 [given] = S (4r + 1) 25C r = 4 S r 25
Cr + S 25
Cr
r =0 r=0 r =0
Þ r = 3 or 19 25
æ 25 ö 24
25
25 n n-1
and 22m - mr - 2r = 1 = 4 S rç ÷ C r-1 + S C r [as nC r= C r-1]
r =1 è r ø r =0 r
at r = 3, 22m - 3m - 6 = 1 Þ 18m = 7 25 25
24 25
7 = 100 S C r -1 + S Cr
Þ m= ÏN r =1 r =0
16
= 100(24 ) + 225 [Q nC 0 + nC1 + nC 2 + K + nC n = 2n]
Now, at r = 19, 22m - 19m - 38 = 1
= (50 + 1)225 = (51) × 225 + 225 × k (given)
Þ 3m = 39 Þ m = 13 Î N
So, k = 51
So, the natural value of m = 13
Hence answer is 51.
09
Trigonometric Function
and Equations

Measure of Angles IN THIS CHAPTER ....


The measure of angle is the amount of rotation from the direction of one ray of Measure of Angles
the angle to the other. The initial and final positions of the revolving ray are Trigonometric Ratios
respectively called the initial and terminal sides (arms).
Trigonometric Functions
Q
Graph of Trigonometric Functions
Trigonometric Identities
Complementary and
Supplementary Angles
O P Variation of Values of
Trigonometric Ratios in Different
If the rotation is in clockwise sense, the angle measured is negative and if the
Quadrants
rotation is in anti-clockwise sense, the angle measured is positive.
Trigonometrical Ratios of Some
Useful Angles between 0° and 90°
System of Measurement of Angles
Trigonometric Ratios of
There are two systems of measurement
Compound Angles
Sexagesimal System Maximum and Minimum
In this system each angle is divided into 90 equal parts and each part is Greatest and Less Value of
known as a degree. Thus, a right angle is equal to 90 degrees. One degree is a sin q + b cos q
denoted as 1°. Trigonometric Equations
Each degree is divided into 60 equal parts each of which is known as one Solution of Trigonometric
minute. One minute is denoted as 1¢. Each minute is consist of 60 parts, each Equation of the Form
part is known as a second. One second is denoted by 1¢ ¢. a cos q + b sin q = c
Hence, 1 right angle = 90° ( 90 degree)
1 degree = 1° = 60 ¢ ( 60 minute)
1 minute = 1 ¢ = 60 ¢ ¢ (60 second)
Trigonometric Function and Equations 193

Circular System Let ÐAOP = q


If the angle subtended by an arc of é arc AP q c lù
length l to the centre of circle of r
l êQ ÐAOP = radius OP = 1 = q , using q = r ú
ë û
l
radius r is q, then q = . q
Now, the six trigonometric functions may be defined as
r r
under
If the length of arc is equal to the
radius of the circle, then the angle OM PM
(i) cos q = =x (ii) sin q = =y
subtended at the centre of the circle OP OP
will be one radian. One radian is denoted by 1c. OP 1 OP 1
The ratio of the circumference of the circle to the (iii) sec q = = ,x ¹ 0 (iv) cosec q = = ,y¹ 0
OM x PM y
diameter of the circle is denoted by a greek letter p and it
is a constant quantity. PM y OM x
(v) tan q = = ,x ¹ 0 (vi) cot q = = ,y¹0
Circumference of circle OM x PM y
\ =p
Diameter of circle
Graph of Trigonometric Functions
Relation between Degree and Radian
The number of radians of an angle subtended by an arc of Graph of sin x
a circle at the centre is equal to the ratios of arc and Y
radius.
180° = p c
(– 3p2 , 1) ( p2, 1)
and 1 radian = 57°17 ¢ 44 . 8¢¢ y=1
B D
If measure of an angle is given in degree, then to convert h1 h1
p X¢ O
X
it into radians, multiply the measure of an angle by A p C
180°
–2p – 3p –p –p p 3p
2p
2 2 2 2
and if the measure of an angle is given in radians, then y=–1
to convert it into degree, write 180° at the place of p . ( – p , –1
2 ) ( 3p , –1
2 )

Example 1. Given that the side length of rhombus the
geometric mean of the lengths of its diagonals. The radian Facts Related to sin x
measure of the acute angle of the rhombus is (a) Domain = R
p p p p
(a) (b) (c) (d) (b) Range = [-1, 1]
12 6 4 3
(c) Period = 2p
Sol. (b) Let side of rhombus is x units and D C (d) Graph of sin x is continuous for all real values of x.
diagonal of rhombus are a and b units.
b p/2
Given, x2 = ab Graph of cos x
1 2 ab a Y
Area of rhombus = x sin q = q
2 2 A B
x
1 (– 2p, 1) (0, 1) (2p, 1)
Þ sin q = = sin 30 º y=1
2 D
Þ q = 30 º O
X¢ X
p –2p – 3p –p –p
p p 3p 2p
In radian 180° ´ p = radius. 2 2 2 2
6 y = –1
(– p, –1) (p, –1)
Trigonometric Functions Y¢

Let X ¢ OX and YOY ¢ be the Y


Facts Related to cos x
coordinate axes. Taking O B
P(x, y) (a) Domain = R
as the centre and a unit q
1 y (b) Range = [-1, 1]
radius, draw a circle, q A

cutting the coordinate A¢ O x M
X (c) Period = 2p
axes at A, B, A¢ and B¢ , (d) Graph of cos x is continuous for all real values of x
as shown in the figure.


194 JEE Main Mathematics

Graph of tan x Facts Related to sec x


p
Y (a) Domain = R - ( 2n + 1) ,n ÎI
2
(b) Range = ( - ¥ , - 1] È [1, ¥ )
1
(c) Period = 2p
mp
X¢ –p p X (d) Graph of sec x is discontinuous at x = , where m
– 3p –p –p O p p 3p 2
2 2 4 4 2 2
–1 is an odd integer.

Graph of cosec x

Y
Facts Related to tan x
p
(a) Domain = R - ( 2n + 1) ,n ÎI y = cosec x
2
(b) Range = ( - ¥ , ¥ )
(p2 ,1)
(c) Period = p
y=1
mp 1
(d) Graph of tan x is discontinuous at x = , where m y = sin x
2 X¢ –2p –p 3p X
–p/2 O p p 2p
– 3p
is an odd integer. 2 2 2
–1 y = –1
Graph of cot x 3p ,_1
– p,–1
(2 ) (
2 )
Y


X¢ X Facts Related to cosec x
–2p – 3p –p –p O p p 3p 2p
asymptotes

2 2 2 2
(a) Domain = R - np, n Î I
(b) Range = ( -¥ , - 1] È [1, ¥ )
(c) Period = 2p

(d) Graph of cosec x is discontinuous at x = mp, where
Facts Related to cot x m is an integer.
(a) Domain = R - np, n Î I
(b) Range = ( - ¥ , ¥ ) Trigonometric Identities
(c) Period = p A trigonometric equation is an identity, if it is true for all
(d) Graph of cot x is discontinuous at x = mp, where m is values of the angle or angles involved.
an integer. Some Important Identities are Given Below
Graph of sec x (i) cos2 q + sin2 q = 1
Y or cos2 q = 1 - sin2 q
y = sec x
(–2p, 1) (2p, 1) or sin2 q = 1 - cos2 q
(ii) sec2 q - tan2 q = 1
y=1 or sec2 q = 1 + tan2 q
1

–2p – 3p –p O p p 3p 2p
X
y = cos x or tan2 q = sec2 q - 1
–p
2 2 2 2
–1 O y = –1 (iii) cosec2 q - cot2 q = 1
or cosec2 q = 1 + cot2 q
(–p, –1) Y¢ (p, –1) or cot2 q = cosec2 q - 1
Trigonometric Function and Equations 195

Transformation of One Trigonometric Ratio to Another Trigonometric Ratios


sin q cos q tan q cot q sec q cosec q
tan q 1 2
(sec q - 1) 1
sin q sinq (1 - cos 2 q) 2
(1 + tan q) 2
(1 + cot q) sec q cosec q
1 cot q 1 (cosec 2 q - 1)
cos q (1 - sin2 q) cos q 2
(1 + tan q) 2
(1 + cot q) sec q cosec q
sin q (1 - cos 2 q) 1 1
tan q tanq (sec 2 q - 1)
(1 - sin2 q) cos q cot q (cosec 2 q - 1)
cos q 1
(1 - sin2 q) 1
cot q cot q (cosec 2 q - 1)
sin q (1 - cos 2 q) tanq (sec 2 q - 1)
1 1 cosec q
(1 + cot 2 q)
sec q (1 + tan2 q) sec q
(1 - sin2 q) cos q cot q (cosec 2 q - 1)
1 sec q
1 (1 + tan2 q)
cosec q 2 (1 + cot 2 q) cosec q
sinq (1 - cos q) tan q (sec 2 q - 1)

Note Above table is applicable only when q Î(0° , 90° ).

Example 2. If sec x + sec2 x = 1, then the value of sin 4 x cos4 x 1


Sol. (b) We have, + =
tan8 x - tan4 x - 2 tan2 x + 1 equal to 2 3 5
sin 4 x (1 - sin 2 x) 2 1
(a) 0 (b) 1 (c) 2 (d) 3 + =
2 3 5
2
Sol. (c) We have, sec x + sec x = 1 4 2 2 6
Þ 3 sin x + 2(1 - sin x) =
sec x = - (sec2 x - 1) = - tan 2 x 5
Þ sec2 x = tan 4 x Þ1 + tan 2 x = tan 4 x Þ 25 sin 4 x - 20 sin 2 x + 4 = 0
Þ 2 2
(1 + tan x) = tan x 8 2
Þ sin 2 x =
5
Þ 1 + 2 tan 2 x + tan 4 x = tan 8 x 2
Þ cos x = 3 / 5
Þ tan 8 x - tan 4 x - 2 tan 2 x = 1
Þ tan 2 x = 2 / 3
Þ tan 8 x - tan 4 x - 2 tan 2 x + 1 = 2
sin 8 x cos8 x (2 / 5) 4 (3 / 5) 4
\ + = +
x2 + y 2 + 1 8 27 8 27
Example 3. If sin2 q = , then x must be 2 3
2x = +
625 625
(a) - 3 (b) 2 (c) 1 (d) 0
5 1
2 2 = =
x + y +1 625 125
Sol. (c) We have, sin 2 q =
2x
0 £ sin 2 q £ 1 Complementary and
x2 + y 2 + 1
\ 0£
2x
£1 Supplementary Angles
Þ 0 £ x2 + y 2 + 1 £ 2x If the sum of two angles is equal to a right angle, then
Þ 0 £ x2 + y 2 - 2x + 1 £ 0 these angles are known as complementary angles of each
Þ x2 - 2x + 1 + y 2 = 0 other. Thus, q and 90° - q are complementary angles of
Þ ( x - 1) 2 + y 2 = 0 each other.
\ x - 1 = 0 and y = 0 Now, if the sum of two angles is equal to two right
x = 1and y = 0 angles, then these angles are known as supplementary
angles of each other.
sin4 x cos4 x 1
Example 4. If + = , then Thus, q and 180° - q are supplementary angles of each
2 3 5
other.
4 sin 8 x cos8 x 1
(a) tan 2 x + (b) + = e. g. , 23° and 67° are complementary angles of each other
3 8 27 125
1 sin 8 x cos8 x 2 while 167° and 13° are supplementary angles of each
(c) tan 2 x = (d) + =
3 8 27 125 other.
196 JEE Main Mathematics

Trigonometric Ratios of Complementary and Supplementary Angles


a sin a cos a tan a cot a sec a cosec a
-q -sinq cos q - tanq -cot q sec q -cosec q
90° - q cos q sinq cot q tanq cosec q sec q
90° + q cos q - sinq -cot q - tanq -cosec q sec q
180° - q sin q -cos q - tanq - cot q - sec q cosec q
180° + q -sin q -cos q tanq cot q - sec q -cosec q

Note • sin {np + (-1)n q} = sin q, n Î I • cos (2np ± q) = cos q, n Î I • tan (np + q) = tan q, n Î I

Variation of Values of Trigonometrical Ratios in Different Quadrants


Y
II I
(i) Except sin and cosec, all ratios are negative. All trigonometric ratios are positive.
X¢ III O IV
Except tan and cot, all ratios are negative. Except cos and sec, all ratios are negative. X

Y
II I
sine decreases from 1 to 0 sine increases from 0 to 1
cosine decreases from 0 to -1 cosine decreases from 1 to 0
tangent increases from -¥ to 0 tangent increases from 0 to ¥
cotangent decreases from 0 to -¥ cotangent decreases from ¥ to 0
secant increases from -¥ to -1 secant increases from 1 to ¥
cosecant increases from 1 to ¥ cosecant decreases from ¥ to 1
(ii) X¢ III O IV
sine increases from -1 to 0
X
sine decreases from 0 to –1
cosine increases from -1 to 0 cosine increases from 0 to 1
tangent increases from 0 to ¥ tangent
increases from -¥ to 0
cotangent decreases from ¥ to 0 cotangent
decreases from 0 to -¥
secant decreases from -1 to -¥ secant
decreases from ¥ to 1
cosecant increases from -¥ to -1 cosecant
decreases from -1 to -¥

Trigonometrical Ratios of Some Useful Angles between 0° and 90°


Angle

p p p p p p 3p p 3p 2p 5p p
0°/0 15°/ 18°/ 22.5°/ 30°/ 36°/ 45°/ 54°/ 60°/ 67.5°/ 72°/ 75°/ 90°/
12 10 8 6 5 4 10 3 8 5 12 2
sin q

3 -1 5 -1 2- 2 1 10 - 2 5 1 5+1 3 2+ 2 10 + 2 5 3+1
0 1
2 2 4 2 2 4 2 4 2 2 4 2 2
cos q

3+1 10 + 2 5 2+ 2 3 5+1 1 10 - 2 5 1 2- 2 5 -1 3 -1
1 0
2 2 4 2 2 4 2 4 2 2 4 2 2

5 -1 5+1 5+1
tan q

3 -1 1 10 + 2 5 3+1
0 2 -1 1 3 2 +1 ¥
3+1 10 + 2 5 3 10 - 2 5 10 - 2 5 5 -1 3 -1

3+1 5 -1
cot q

10 + 2 5 10 - 2 5 10 - 2 5 1 3 -1
¥ 2 +1 3 1 2 -1 0
3 -1 5 -1 5+1 5+1 3 10 + 2 5 3+1
sec q

2 2 4 2 4 2 2
1 4-2 2 5 -1 2 2 4+2 2 5+1 ¥
3+1 10 + 2 5 3 10 - 2 5 3 -1
cosec q

2 2 4+2 2 4 2 4 2 2
¥ 5+1 2 2 5 -1 4-2 2 1
3 -1 10 - 2 5 3 10 + 2 5 3+1
Trigonometric Function and Equations 197

Example 5. The value of (ix) sin( A + B) sin ( A - B) = sin2 A - sin2 B


æp ö æ p ö = cos2 B - cos2 A
2 cos3 ç + x ÷ cot(3p + x) sec( x - 3p) cosecç3 - x ÷
è2 ø è 2 ø (x) cos( A + B) cos ( A - B) = cos2 A - sin2 B
2
is equal to
cot x tan ( x - p) sin( x - 2 p)
= cos2 B - sin2 A
(a) 0 (b) - 1 (c) - 2 (d) 1
(xi) sin( A + B + C ) = sin A cos B cos C + cos A sin B cos C
Sol. (c) We have, + cos A cos B sin C - sin A sin B sin C
æp ö æ p ö
2 cos3 ç + x÷ cot(3p + x) sec( x - 3p ) cosec ç3 - x÷ Also, sin( A + B + C ) = sin A cos B cos C
è2 ø è 2 ø
(tan A + tan B + tan C )
cot x tan 2( x - p ) sin( x - 2p )
- tan A tan B tan C
2( - sin3 x) (cot x) ( - sec x) ( - sec x)
= (xii) cos( A + B + C ) = cos A cos B cos C - sin A sin B cos C
cot x tan 2 x sin x
- sin A cos B sin C - cos A sin B sin C
= -2
Also, cos( A + B + C ) = cos A cos B cos C
Example 6. The value of (1 - tan A tan B - tan B tan C - tan C tan A)
p 3p 5p 7p 3 5
cos2 + cos2 + cos2 + cos2 is Example 7. If cos(a + b) = , sin(a - b) = and
16 16 16 16 5 13
(a) 0 (b) 1 (c) - 1 (d) 2 p
0 < a, b < , then tan(2a ) is equal to
4 (JEE Main 2019)
Sol. (d) We have,
63 63 21 33
p 3p 5p 7p (a) (b) (c) (d)
cos2 + cos2 + cos2 + cos2 52 16 16 52
16 16 16 16
p 3p æ p 3p ö 5
2æ p pö Sol. (b) Given, sin( a - b) =
= cos2 + cos2 + cos2ç - ÷ + cos ç - ÷ 13
16 16 è 2 16 ø è 2 16 ø
p 3p 3p p 3 æ pö
= cos2 + cos2 + sin 2 + sin 2 and cos( a + b) = , where a , b Î ç0 , ÷
5 è 4ø
16 16 16 16
æ p pö æ 3p 3p ö 5
= ç cos2 + sin 2 ÷ + ç cos2 + sin 2 ÷ Now, sin( a - b) =
è 16 16 ø è 16 16 ø 13
5
=1+ 1= 2 Þ tan( a - b) = …(i)
12
3
Trigonometric Ratios of and cos( a + b) =
5
Compound Angles Þ tan ( a + b) =
4
…(ii)
3
An angle made up of the algebraic sum of two or more
Now, tan(2a) = tan[( a + b) + ( a - b)]
angles is called compound angles.
tan( a + b) + tan( a - b)
=
Sum and Difference Formulae 1 - tan( a + b) tan( a - b)

(i) sin( A + B) = sin A cos B + cos A sin B 4 5


+
= 3 12 [from Eqs. (i) and (ii)]
(ii) sin( A - B) = sin A cos B - cos A sin B 4 5
1- ´
(iii) cos( A + B) = cos A cos B - sin A sin B 3 12
(iv) cos( A - B) = cos A cos B + sin A sin B 48 + 15 63
= =
tan A + tan B 36 - 20 16
(v) tan( A + B) =
1 - tan A tan B
tan A - tan B Product into Sum and Difference Formulae
(vi) tan( A - B) =
1 + tan A tan B (i) 2 sin A cos B = sin ( A + B) + sin ( A - B)ü
cot A cot B - 1 ý
(vii) cot( A + B) = (ii) 2 cos A sin B = sin ( A + B) - sin ( A - B)þ
cot A + cot B
(iii) 2 cos A cos B = cos ( A + B) + cos ( A - B)
cot A cot B + 1
(viii) cot( A - B) = (iv) 2 sin A sin B = cos ( A - B) - cos ( A + B)
cot B - cot A
198 JEE Main Mathematics

p 2p 4p 8p Sum and Difference into Product Formulae


Example 8. The value of cos cos cos cos is
15 15 15 15
1 1 æC + D ö æC - D ö
(a) (b) - (c) 1 (d) 0 (i) sin C + sin D = 2 sin ç ÷ cos ç ÷
16 16 è 2 ø è 2 ø
p 2p 4p 8p æC + D ö æC - D ö
Sol. (b) cos cos cos cos (ii) sin C - sin D = 2 cos ç ÷ sin ç ÷
15 15 15 15 è 2 ø è 2 ø
1æ 4p pöæ 8p 2p ö æC + D ö æC - D ö
= ç2 cos cos ÷ ç2 cos cos ÷ (iii) cos C + cos D = 2 cos ç
4è 15 15 ø è 15 15 ø ÷ cos ç ÷
è 2 ø è 2 ø
1æ p pö æ 2p 2p ö
= ç cos + cos ÷ ç cos + cos ÷ æC + D ö æD - Cö
4è 3 5ø è 3 5 ø (iv) cos C - cos D = 2 sin ç ÷ sin ç ÷
è 2 ø è 2 ø
1 æ1 5 + 1ö æ 1 5 - 1ö
= ç + ÷ ç- + ÷
4 è2 4 øè 2 4 ø Example 10. sin 47° + sin 61°- sin 11° - sin 25 ° is equal to
1 é 1 1 æ 5 -1 5 + 1ö 5 - 1ù (a) sin 7° (b) cos 7°
= ê- + ç - ÷+ ú
4 êë 4 2 è 4 4 ø 16 ûú (c) sin 36° (d) cos 36°
1 Sol. (b) sin 47° - sin 25° + sin 61° - sin 11°
=-
16
= 2 cos 36° sin 11° + 2 cos 36° sin 25°
pö æpö2æ = 2 cos 36° [sin 11° + sin 25° ]
Example 9. If L = sin ç ÷ - sin2 ç ÷
è16 ø è8 ø é æ 25° + 11° ö æ 25° - 11° ö ù
= 2 cos 36° ê2 sin ç ÷ cos ç ÷ú
æpö æpö ë è 2 ø è 2 øû
and M = cos2 ç ÷ - sin2 ç ÷, then
è16 ø è8 ø (JEE Main 2020) æ 5 + 1ö æ 5 - 1ö
= 4ç ÷ç ÷ cos 7°
1 1 p 1 1 p è 4 øè 4 ø
(a) L = - + cos (b) L = - cos
2 2 2 8 4 2 4 8 5 -1
= cos 7°
1 1 p 1 1 p 4
(c) M = + cos (d) M = + cos
4 2 4 8 2 2 2 8 = cos 7°
p p Trigonometric Ratios of Multiples of an Angle
Sol. (d) Given, L = sin 2 – sin 2
16 8
æ p pö æ p pö 2 tan q
= sinç + ÷ sinç – ÷ (i) sin 2 q = 2 sin q cos q =
è16 8 ø è16 8 ø 1 + tan2 q
3p æ pö (ii) cos 2 q = cos2 q - sin2 q = 1 - 2 sin2 q
= sin sinç – ÷
16 è 16 ø
1 - tan2 q
1é 3p æ p öù = 2 cos2 q - 1 =
= ê2 sin sinç – ÷ ú 1 + tan2 q
2ë 16 è 16 ø û
1é p 2p ù 1 1
= cos – cos (iii) cos2 q = (1 + cos 2 q ), sin2 q = (1 - cos 2 q )
2 êë 4 16 úû 2 2
1 1 p 2 tan q
= – cos . (iv) tan 2 q =
2 2 2 8 1 - tan2 q
2 p 2p cot2 q - 1
and, M = cos – sin (v) cot 2 q =
16 8 2 cot q
æ p p ö æ p pö
= cosç + ÷ cosç – ÷ (vi) sin 3 q = 3 sin q - 4 sin3 q
è16 8 ø è16 8 ø
3p æ pö (vii) cos 3 q = 4 cos3 q - 3 cos q
= cos cosç – ÷
16 è 16 ø 3 tan q - tan3 q
(viii) tan 3 q =
1é 3p pù 1 - 3 tan2 q
= 2 cos cos ú
2 êë 16 16 û
cot3 q - 3 cot q
1é p 2p ù (ix) cot 3 q =
= ê cos + cos 3 cot2 q - 1
2ë 4 16 úû
1 1 p 3 cot q - cot3 q
= + cos . =
2 2 2 8 1 - 3 cot2 q
Trigonometric Function and Equations 199

Example 11. 3 cosec 20° - sec 20° is equal to Sol. (b) Given trigonometric expression
æpö æ 3p ö æpö 3p
(a) 1 (b) 2 (c) 3 (d) 4 cos3 ç ÷ cosç ÷ + sin3 ç ÷ sin
è8ø è 8 ø è8ø 8
Sol. (d) 3 cosec 20° - sec 20°
3æ p ö æp pö 3æ p ö æp pö
cos ç ÷ cosç - ÷ + sin ç ÷ sinç - ÷
3 1 3 cos 20° - sin 20° è8ø è2 8ø è8ø è2 8ø
= - =
sin 20° cos 20° sin 20°× cos 20° é p p 3p ù
æ 3 1 ö êëQ 2 - 8 = 8 úû
4ç cos 20° - sin 20° ÷
è 2 2 ø æpö æpö æpö æpö
= = cos3 ç ÷ sinç ÷ + sin3 ç ÷ cosç ÷
2 sin 20°× cos 20° è8ø è8ø è8ø è8ø
4 (sin 60° cos 20° - cos 60° sin 20° ) æpö æpö é 2æ p ö 2æ p ö ù
= = sinç ÷ cosç ÷ ê cos çè 8 ÷ø + sin çè 8 ÷ø ú
sin 40° è8ø è8ø ë û
4 sin 40°
= =4 1 æpö æpö 1 p 1
sin 40° = ´ 2 sinç ÷ cosç ÷ = sin =
2 è8ø è8ø 2 4 2 2
Trigonometric Ratios of Submultiple Angles Hence, option (b) is correct.
1 Maximum and Minimum (Greatest and Least)
2 tan q
1 1
(i) sin q = 2 sin q cos q = 2 Value of a sin q + b cos q
2 2 1
1 + tan2 q Let a = r cosa and b = r sina
2
\ a sin q + b cos q = r (cos a sin q + sin a cos q )
1 1 1
(ii) cos q = cos2 q - sin2 q = 1 - 2 sin2 q = r sin (q + a )
2 2 2
æ bö
2 1 where, r = a 2 + b2 and a = tan-1 ç ÷
1 - tan q èaø
1 2
= 2 cos2 q - 1 = Q Maximum and minimum values of sin (q + a ) are 1 and
2 2 1
1 + tan q -1 respectively. Therefore,
2
1 1 (i) Maximum value of a sin q + b cos q = a 2 + b2
2 tan q cot2 q - 1
(iii) tan q = 2 (iv) cot q = 2 (ii) Minimum value of a sin q + b cos q = - a 2 + b2
2 1 1
1 - tan q 2 cot q
2 2 Example 13. The maximum value of
æ pö
Note 3 cos q + 5 sin çq - ÷ for any real value of q is
è 6ø (JEE Main 2019)
A A
• 1 - cos A = 2 sin2 • 1 + cos A = 2 cos 2 79
2 2 (a) (b) 34 (c) 31 (d) 19
2
• sin(a) + sin (a + b ) + sin (a + 2 b ) + ... + sin[a + (n - 1)b ]
æ pö
ì b ü nb Sol. (d) Given expression 3 cos q + 5 sinç q - ÷
sin ía + (n - 1) æç ö÷ ý sin æç ö÷ è 6ø
î è 2 ø þ è 2 ø
= æ p p ö
æ bö = 3 cos q + 5 ç sin q cos - sin cos q÷
sin ç ÷ è 6 6 ø
è2 ø
æ 3 1 ö
• cos (a) + cos (a + b ) + cos (a + 2 b ) + ... + cos [a + (n - 1)b ] = 3 cos q + 5 ç sin q - cos q÷
è 2 2 ø
b nb
cos ìía + (n - 1) üý sin 5 5 3
= î 2 þ 2 = 3 cos q -cos q + sin q
b 2 2
sin
2 1 5 3
= cos q + sin q
2 2
Example 12. The value of
Q The maximum value of a cos q + b sin q is a2 + b 2
æpö æ 3p ö æpö æ 3p ö 1 5 3
cos3 ç ÷ × cosç ÷ + sin3 ç ÷ × sinç ÷ is So, maximum value of cos q + sin q is
è8ø è8 ø è8 ø è8 ø 2 2
(JEE Main 2020) 2 2
æ 1ö æ5 3 ö
1 1 = ç ÷ +ç ÷
(a) (b) è2ø è 2 ø
4 2 2
1 1 1 75 76
(c) (d) = + = = 19.
2 2 4 4 4
200 JEE Main Mathematics

p
Trigonometric Equations ● If q is an even multiple of , i.e. when q = n p , n Î I ,
2
An equation involving one or more trigonometrical ratios then cosec q and cot q are not defined.
of unknown angle is called a trigonometric equation. ● The general solution of equations containing
1 cosec q , sec q and cot q is equivalent to that of the
e.g. cos2 q - sin q = , tan m q = cot n q
2 equation involving sin q , cos q , tan q.
etc., are trigonometric equations. ● If value of q which satisfies the two trigonometric
equations, is least positive value a , then
Solutions of a Trigonometric Equation q = 2np + a , n Î I . e.g., sin q = sin a and
A value of the unknown angle which satisfies cos q = cos a , then q = 2np + a , n Î I
the given equation, is called a solution of the equation. æ 1ö
When cos q = 0, then sin q = 1 or –1 Þ q = ç n + ÷ p
The trigonometric equation may have infinite number of è 2ø
solutions and can be classified as If sin q = 1, then n is even and if sin q = -1, then n is odd.
Similarly, when sin q = 0, then cos q = 1 or –1 Þ q = np
Principal Solution
The least value of unknown angle which satisfies the If cos q = 1, then n is even and if cos q = - 1, then n is odd.
given equation, is called a principal solution of
trigonometric equation. Example 14. Let S = {q Î [ -2 p, 2 p] : 2 cos2 q + 3 sin q = 0},
then the sum of the elements of S is (JEE Main 2019)
General Solution 5p 13p
(a) 2p (b) p (c) (d)
The solution consisting of all possible solutions of a 3 6
trigonometric equation is called its general solution. Sol. (a) We have, q Î [ -2p , 2p ]
Note A function f( x) is said to be a periodic function, if a least and 2 cos2 q + 3 sin q = 0
positive real number T is such that f ( x + T ) = f( x), then T is known as
Þ 2 (1 - sin 2 q) + 3 sin q = 0
period of function f( x).
Þ 2 - 2 sin 2 q + 3 sin q = 0
General Solutions of Trigonometric Equations Þ 2 sin 2 q - 3 sin q - 2 = 0
(i) sin q = 0 q = np, n Î I Þ 2 sin 2 q - 4 sin q + sin q - 2 = 0
p Þ 2 sin q (sin q - 2) + 1(sin q - 2) = 0
(ii) cos q = 0 q = (2 n + 1) , n Î I
2 Þ (sin q - 2) (2 sin q + 1) = 0
(iii) tan q = 0 q = np, n Î I -1
Þ sin q = [Q (sin q - 2) ¹ 0]
(iv) q = np + (-1)n a, 2
p p
sin q = sin a a Î éê - , ùú , n Î I p p p p
ë 2 2û Þ q = 2p – , - p + , - , p + [Q q Î [ -2p , 2p ]]
6 6 6 6
cos q = cos a q = 2np ± a, p p p p
(v) Now, sum of all solutions = 2p - - p + - + p + = 2p
a Î[0, p ], n Î I
6 6 6 6
(vi) q = np + a,
tan q = tan a p p
æ
a Î ç - , ö÷ , n Î I Example 15. The solution of
è 2 2ø
tan x + tan 2x + tan 3x = tan x tan 2x tan 3x is
sin2 q = sin2 a ü np
ï (a) x = np , n Î I (b) x = , n ÎI
(vii) cos 2 q = cos 2 aý q = np ± a, n Î I 2
tan2 q = tan2 a ï (c) x =
np
, n ÎI
þ (d) None of these
3
(viii) p
sin q = 1 q = (4n + 1) , n Î I tan x + tan 2x
2 Sol. (c) We know that, tan 3x = tan ( x + 2x ) =
1 - tan x tan 2x
(ix) cos q = 1 q = 2np, n Î I
Þ tan 3x - tan x tan 2x tan 3x = tan x + tan 2x
(x) cos q = - 1 q = (2 n + 1)p, n Î I
Þ tan x tan 2x tan 3x = tan 3x - tan x - tan 2x …(i)
(xi) sin x = sin a and cos x = cos a x = 2 np + a
Now, tan x + tan 2x + tan 3x = tan x tan 2x tan 3x
Þ tan x + tan 2x + tan 3x = tan 3x - tan x - tan 2x
Important Points
p [from Eq. (i)]
● If q is an odd multiple of i.e., when Þ 2 tan x = - 2 tan 2x Þ tan 2x = tan ( - x )
2
p 2x = np + ( - x ) Þ 3x = np
q = ( 2n + 1) , n Î I, then sec q and tan q are not np
2 Þ x= , where n Î I
defined. 3
Trigonometric Function and Equations 201

Example 16. The number of solutions of the equation Example 17. The solution of the equation
4 é 5p 5p ù
2 k cos x - 3 sin x = k + 1is possible only if
1 + sin x = cos 3x, x Î ê - , is
ë 2 2 úû (JEE Main 2019) (a) k Î ( -¥ , 4]
(a) 3 (b) 5 (c) 7 (d) 4 (b) k Î ( -¥ , ¥)
4 2
(c) k Î [ 4, ¥)
Sol. (b) Given equation is1 + sin x = cos (3x) (d) None of the above
Since, range of (1 + sin 4 x) = [1, 2]
Sol. (a) Here, k cos x - 3 sin x = k + 1could be rewritten as
and range of cos2(3x) = [0 , 1]
k 3 k+1
So, the given equation holds if cos x - sin x =
2 2
k +9 k +9 k2 + 9
1 + sin 4 x = 1 = cos2(3x)
k+1
Þ sin 4 x = 0 and cos2 3x = 1 Þ cos ( x + f) =
k2 + 9
é 5p 5p ù
Since, x Îê- ,
ë 2 2 úû
k
where, cos f =
\ x = - 2p, - p, 0, p, 2 p. k2 + 9
Thus, there are five different values of x is possible. 3
and sin f =
2
k +9
Solution of Trigonometric Equation of which posses solution only, if
the Form a cos q + b sin q = c -1 £
k+1
£1
Let the equation is k2 + 9
a cos q + b sin q = c ½ k+1 ½
On dividing by a 2 + b2 both sides, we get
Þ ½ 2 ½£ 1
½ k + 9½
a b c
cos q + sin q = …(i) Þ (k + 1) 2 £ k2 + 9
2 2 2 2
a +b a +b a + b2
2
Þ k2 + 2k + 1 £ k2 + 9
b
Let tan a = Þ k£4
a
Hence, the interval in which k cos x - 3 sin x = k + 1admits
b
\ sin a = solution for k is ( -¥ , 4 ] .
a + b2
2

a Example 18. The number of integral values of n so that


and cos a = sin x(sin x + cos x) = n has at least one solution is
a 2 + b2
(a) 0 (b) 1
From Eq. (i), we get (c) 2 (d) 3
c
cos q cos a + sin q sin a = Sol. (c) We have,
a + b2
2
sin x (sin x + cos x) = n
c Þ sin 2 x + sin x cos x = n
Þ cos (q - a ) =
a 2 + b2 1 - cos 2x sin 2x
Þ + =n
2 2 2 2
If |c| > a + b , then equation a cos q + b sin q = c has
Þ 1 - cos 2x + sin 2x = 2n
no solution and if,
Þ sin 2x - cos 2x = 2n - 1
|c| £ a 2 + b2 ,
Þ - 2 £ 2n - 1 £ 2
|c|
then let cos f = [Q sin 2x - cos 2x Î [ - 2 , 2 ] ]
a 2 + b2
- 2 +1 2 +1
Þ £n <
Þ cos (q - a ) = cos f 2 2
Þ q - a = 2 np ± f \ n = 0 ,1
Þ q = 2 np ± f + a , n Î I Two integral value of n i.e., 0 and 1.
Practice Exercise
ROUND I Topically Divided Problems
Trigonometric Functions and Identities (a)
1
(b) x
x
1. The value of the expression
(c) 1 - x (d) None of these
sin 2 y 1 + cos y sin y
1- + - is equal to 9. If sin q + cos q = m and sec q + cosec q = n, then
1 + cos y sin y 1 - cos y
n ( m + 1) ( m - 1) is equal to
(a) 0 (b) 1 (c) sin y (d) cos y
(a) m (b) n (c) 2 m (d) 2 n
2. If q lies in the second quadrant, then the value of x y z
10. If = = , then x + y + z
1 - sin q 1 + sin q cos q cos æq - 2 p ö cos æq + 2 p ö
+ is equal to ç ÷ ç ÷
1 + sin q 1 - sin q è 3 ø è 3 ø
(a) 2 sec q (b) - 2 sec q is equal to
(c) 2 cosec q (d) None of these (a) 1 (b) 0
(c) -1 (d) None of these
3p
3. If < a < p, then cosec 2 a + 2 cot a is equal to
4 11. If x lies in IInd quadrant, then
(a) 1 + cot a (b) 1 - cot a 1 + sin x + 1 - sin x
(c) -1 - cot a (d) -1 + cot a
is equal to
1 + sin x - 1 - sin x
4. If tan A + sin A = m and tan A - sin A = n, then x x
(a) sin (b) tan
( m 2 - n2 ) 2 2 2
is equal to x x
mn (c) sec (d) cosec
2 2
(a) 4 (b) 3 (c) 16 (d) 9
12. If tan q , 2 tan q + 2 and 3 tan q + 3 are in GP, then
5. The value of 7 - 5 cot q
the value of is
6 (sin 6 q + cos6 q ) - 9 (sin 4 q + cos4 q ) + 4 is 9 - 4 sec 2 q - 1
(a) - 3 (b) 0 (c) 1 (d) 3
12 33
p p (a) (b) -
6. For any q Î æç , ö÷, the expression 5 28
è4 2ø 33 12
(c) (d)
3 (sin q - cos q ) 4 + 6 (sin q + cos q ) 2 + 4 sin 6 q equals 100 13
(JEE Main 2019)
13. If cos x = tan y, cot y = tan z and cot z = tan x, then
(a) 13 - 4 cos 4 q + 2 sin 2 q cos 2 q
sin x is equal to
(b) 13 - 4 cos 2 q + 6 cos 4 q 5+1 5 -1 5+1 5 -1
(a) (b) (c) (d)
(c) 13 - 4 cos 2 q + 6 sin 2 q cos 2 q 4 4 2 2
(d) 13 - 4 cos 6 q 14. If sin A - 6 cos A = 7 cos A, then
1 cos A + 6 sin A is equal to
7. If fk ( x) = (sin k x + cosk x), where x Î R, k ³ 1, then
k (a) 6 sin A (b) - 7 sin A
f4 ( x) - f6 ( x) is equal to (JEE Main 2014) (c) 6 cos A (d) 7 cos A
1 1 1 1
(a) (b) (c) (d) 15. If A, B, C and D are the angles of a cyclic
6 3 4 12
quadrilateral, then cos A + cos B + cos C + cos D is
2 sin a 1 - cos a - sin a equal to
8. If = x, then is
1 + cos a + sin a cos a (a) 2 (cos A + cos C ) (b) 2 (cos A + cos B)
equal to (c) 2 (cos A + cos D ) (d) 0
Trigonometric Function and Equations 203

sin A 3 cos A 5 p 25. The expression


16. If = and = ,0 < A, B < , then
sin B 2 cos B 2 2 cos 2( A - B) + cos 2 B - 2 cos ( A - B) cos A cos B is
3 5 (a) dependent on B
(a) tan A = (b) tan A =
5 3 (b) dependent on A and B
(c) tan A = 2 (d) tan B = 2 (c) dependent on A
3 2
17. If a cos a + 3a cos a sin a = m and (d) independent of A and B
a sin 3 a + 3a cos2 a sin a = n, then sin A - sin C
26. If = cot B, then A, B and C are in
( m + n) 2 / 3 + ( m - n) 2 / 3 is equal to cos C - cos A
(a) AP (b) GP
(a) 2 a 2 (b) 2 a1/ 3 (c) 2 a 2/ 3 (d) 2 a3
(c) HP (d) None of these

ç x + ö÷, then æç x 2 + 2 ö÷ is equal to
1 1 1
18. If cos q = 4 5
2è xø 2è x ø 27. If cos (a + b) =
, sin (a - b) = and a , b lies
5 13
(a) sin 2 q (b) cos 2 q p
between 0 and , then tan 2 a is equal to
(c) tan 2 q (d) None of these 4
3p 16 56
é ù
19. The value of 3 sin 4 æç - a ö÷ + sin 4 (3p + a ) ú (a) (b)
êë è 2 ø û
63 33
28
é æp ö ù (c) (d) None of these
- 2 êsin 6 ç + a ÷ + sin 6 (5p - a )ú is equal to 33
ë è2 ø û n n
æ cos A + cos B ö æ sin A + sin B ö
(a) 0 (b) 1 28. The value of ç ÷ +ç ÷
(c) 3 (d) sin 4a + sin 6a è sin A - sin B ø è cos A - cos B ø
20. If tan x + cot x = 2 , then sin 2 n x + cos2 n x is equal to (where, n is an even) is
1 æ A - Bö æ A - Bö
(a) 2n (b) - (a) 2 tan n ç ÷ (b) 2 cotn ç ÷
2 è 2 ø è 2 ø
1 (c) 0 (d) None of these
(c) (d) None of these
2 p 3p 15 12
29. If < a < p, p < b < ; sin a = and tan b = ,
21. If 15 sin 4 a + 10 cos4 a = 6, for some a Î R, then the 2 2 17 5
value of 27 sec 6 a + 8 cosec6a is equal to then the value of sin(b - a ) is
(JEE Main 2021) 171 21 21 171
(a) - (b) - (c) (d)
(a) 350 (b) 500 221 221 221 221
(c) 400 (d) 250
30. If cos x + cos y + cos a = 0 and
x + yö
sin x + sin y + sin a = 0, then cot æç
22. Let n be a positive integer such that
÷ is equal to
p p n è 2 ø
sin + cos = . Then,
2n 2n 2 æ x + yö
(a) sin a (b) cos a (c) cot a (d) sin ç ÷
(a) n = 6 (b) n = 1, 2 , 3, . . . , 8 è 2 ø
(c) n = 5 (d) None of these 31. tan 81° - tan 63° - tan 27° + tan 9° equals
(a) 6 (b) 0 (c) 2 (d) 4
Trigonometric Ratios of Sum and
p 3 4
Difference of Two and Three Angles 32. If 0 < b < a < , cos (a + b) = and cos (a - b) = ,
4 5 5
23. If m sin q = n sin (q + 2a ), then tan (q + a ) cot a is then sin 2 a is equal to
equal to (a) 0 (b) 1
m-n
(a) (m + n )(m - n ) (b) (c) 2 (d) None of these
m+ n
m+ n 33. If tan a = (1 + 2 - x ) -1, tan b = (1 + 2 x +1) -1, then a + b
(c) (d) 2(m + n )(m - n ) equals
m-n
p p p p
sin ( B + A) + cos ( B - A) (a) (b) (c) (d)
24. The value of is equal to 6 4 3 2
sin ( B - A) + cos ( B + A) A+B B- A
cos B + sin B cos A + sin A 34. If cos A = m cos B and cot = l tan ,
(a) (b) 2 2
cos B - sin B cos A - sin A then l is
cos A - sin A m m+1 m+1
(c) (d) None of these (a) (b) (c) (d) None of these
cos A + sin A m -1 m m -1
204 JEE Main Mathematics

35. If cos (q - a ) = a and cos (q - b) = b, then 46. If x and y are acute angles, such that
sin 2 (a - b) + 2 ab cos (a - b) is equal to 3 3
cos x + cos y = and sin x + sin y = , then
(a) a 2 + b2 (b) a 2 - b2 (c) b2 - a 2 (d) - a 2 - b2 2 4
sin ( x + y) equals
36. If sin q = 3 sin (q + 2 a ), then the value of 2 3 3 4
tan (q + a ) + 2 tan a is (a) (b) (c) (d)
5 4 5 5
(a) 3 (b) 2 (c) - 1 (d) 0 p sin (a + b + g )
p 47. If a , b, g Î æç0, ö÷, then the value of
37. If a + b = and b + g = a , then the value of tan a is è 2ø sin a + sin b + sin g
2
is
(a) tan b + tan g (b) 2 (tan b + tan g )
(a) < 1 (b) > 1
(c) tan b + 2 tan g (d) 2 tan b + tan g
(c) 1 (d) None of these
38. The value of cos 12° + cos 84° + cos 156° + cos 132° is
1 1 1 48. If angle q be divided into two parts such that the
(a) (b) 1 (c) - (d)
2 2 8 tangent of one part is k times the tangent of the
p 3p other and f is their difference, then sin q is equal to
39. The value of æç1 + cos ö÷ æç1 + cos ö÷ k+1 k -1
è 8ø è 8 ø (a) sin f (b) sin f
k -1 k+1
æ 5p ö æ 7p ö 2 k -1
ç1 + cos ÷ ç1 + cos ÷ is equal to (c) sin f (d) None of these
è 8 øè 8 ø 2k+1
1 1 1 1
(a) (b) (c) (d) 49. If A + B + C = p and cos A = cos B cos C, then
2 4 8 16
tan B tan C is equal to
40. If cos a + cos b = 0 = sin a + sin b, then 1 1
(a) (b) 2 (c) 1 (d) -
cos 2a + cos 2 b is equal to 2 2
(a) 2 cos (a + b ) (b) -2 cos (a + b )
50. The value of sin 12° sin 48° sin 54° is equal to
(c) 3 cos (a + b ) (d) None of these 1 1 1 1
3 (a) (b) (c) (d)
41. If 0 < x, y < p and cos x + cos y - cos( x + y) = then 16 32 8 4
2
51. The value of 1 + cos 56° + cos 58° - cos 66° is equal to
sin x + cos y is equal to (JEE Main 2021)
(a) 2 cos 28° cos 29° cos 33° (b) 4 cos 28° cos 29° sin 33°
1 1+ 3 3 1- 3 (c) 4 cos 28° cos 29° cos 33° (d) 2 cos 28° cos 29° sin 33°
(a) (b) (c) (d)
2 2 2 2 2p p 2p p
52. tan - tan - 3 tan tan is equal to
Trigonometric Ratios of Multiple and 5 15 5 15
1
(a) - 3 (b) (c) 1 (d) 3
Submultiple Angles 3
42. The value of sin 20° sin 40° sin 60° sin 80° is equal to 53. The sum S = sin q + sin 2 q + K + sin nq equals
3 5 3 5 1 nq q
(a) - (b) (c) (d) - (a) sin (n + 1) q sin /sin
16 16 16 16 2 2 2
1 nq q
p 2p 4p 8p (b) cos (n + 1) q sin /sin
43. The value of cos cos cos cos is equal to 2 2 2
5 5 5 5 1 nq q
(c) sin (n + 1) q cos /sin
1 1 1 2 2 2
(a) (b) 0 (c) - (d) -
16 8 16 1 nq q
(d) cos (n + 1) q cos /sin
2p 4p 6p 2 2 2
44. cos + cos + cos n
7 7 7 54. If sin 3 x sin 3x = å cm cos mx, where c0 , c1, c2 , . . . , cn
(a) is equal to zero m=0
(b) lies between 0 and 3 are constants and cn ¹ 0, then the value of n is
(c) is a negative number
(a) 15 (b) 6 (c) 1 (d) 0
(d) lies between 3 and 6
2p ö 4p ö
q +aö æ q - a ö is 55. If x cos q = y cos æçq + æ
÷ = z cos çq + ÷, then the
45. If cos q = cos a cos b, then tan æç ÷ tan ç ÷ è 3 ø è 3 ø
è 2 ø è 2 ø 1 1 1
equal to value of + + is equal to
a b q b x y z
(a) tan 2 (b) tan 2 (c) tan 2 (d) cot2
2 2 2 2 (a) 1 (b) 2 (c) 0 (d) 3 cos q
Trigonometric Function and Equations 205

3p 66. If ( a - b) sin (q + f) = ( a + b) sin (q - f) and


56. If A + B + C = , then cos 2 A + cos 2 B + cos 2 C is
2 q f
a tan - b tan = c, then the value of sin f is equal
equal to 2 2
(a) 1 - 4 cos A cos B cos C (b) 4 sin A sin B sin C to
(c) 1 + 2 cos A cos B cos C (d) 1 - 4 sin A sin B sin C 2 ab 2 bc
(a) (b)
a - b 2 - c2
2
a - b 2 - c2
2
57. If a + b - g = p, then sin 2 a + sin 2 b - sin 2 g is equal
to 2 bc 2 ab
(c) (d)
(a) 2 sin a sin b cos g (b) 2 cos a cos b cos g a 2 - b 2 + c2 a 2 - b 2 + c2
(c) 2 sin a sin b sin g (d) None of these 67. The value of 2 cos x - cos 3x - cos 5 x is equal to
58. If a + b + g = 2 p, then (a) 16 cos3 x sin 2 x (b) 16 sin3 x cos 2 x
a b g a b g (c) 4 cos3 x sin 2 x (d) 4 sin3 x cos 2 x
(a) tan + tan + tan = tan tan tan
2 2 2 2 2 2 p 2p 4p 32 p
68. The value of cos cos cos . . . cos is
a b b g g a 65 65 65 65
(b) tan tan + tan tan + tan tan = 1
2 2 2 2 2 2 1 1 1 1
(a) (b) (c) - (d) -
a b g a b g 32 64 32 64
(c) tan + tan + tan = - tan tan tan
2 2 2 2 2 2 p 3p 5p 7p
69. sin 4 + sin 4 + sin 4 + sin 4 is equal to
(d) None of the above 8 8 8 8
cos 12° - sin 12° sin 147° (a) 1 (b) 3/2 (c) 2 (d) 1/4
59. The value of + is equal to
cos 12° + sin 12° cos 147° 70. If a cos 2 q + b sin 2 q = c has a and b as its roots,
(a) 1 (b) -1 then tan a + tan b is equal to
2b 2b
(c) 0 (d) None of these (a) - (b)
a+c a+c
sin x 1 cos x 3 p
60. If = , = , where x, y Î æç0, ö÷ , then the (c)
3b
(d) None of these
sin y 2 cos y 2 è 2ø a+c
value of tan ( x + y) is equal to
71. The value of cos2 A (3 - 4 cos2 A) 2 + sin 2
(a) 13 (b) 14 (c) 17 (d) 15
A (3 - 4 sin 2 A) 2 is equal to
61. If sin q + cos q = x, then (a) cos 4 A (b) sin 4 A
1 (c) 1 (d) None of these
sin 6 q + cos6 q = [ 4 - 3 ( x 2 - 1) 2 ] for
4
72. If 4na = p, then the value of
(a) all real x (b) x2 £ 2
(c) x2 > 2 (d) None of these tan a × tan 2 a × tan 3 a × tan 4 a ... tan ( 2 n - 2)a
tan ( 2 n - 1) a is
62. The value of tan 20° tan 40° tan 60° tan 80° is equal (a) 0 (b) 1
to (c) -1 (d) None of these
3
(a) 1 (b) 2 (c) 3 (d)
2
Maximum and Minimum Value of
Trigonometric Equation
63. The value ofsin 36° sin 72° sin 108° sin 144° is equal to
1 1 3 5 73. If in a D ABC, ÐC = 90°, then the maximum value
(a) (b) (c) (d) of sin A sin B is
4 16 4 16
1
64. The value of sin 10° sin 30° sin 50° sin 70° is (a) (b) 1
2
(JEE Main 2019)
(c) 2 (d) None of these
1 1 1 1
(a) (b) (c) (d) 74. If A = sin 2 q + cos4 q , then for all real values of q
36 32 16 18
3
b a+b a-b (a) 1 £ A £ 2 (b) £ A £1
65. If tan x = , then find the value of + . 4
a a-b a+b 13 3 13
(c) £ A £1 (d) £ A £
2 cos x cos x 16 4 16
(a) (b)
cos 2x cos 2x 75. If y = 1 + 4 sin 2 x cos2 x, then
2 cos 2x (a) 1 £ y £ 2 (b) -1 £ y £ 1
(c) (d) None of these
cos x (c) - 3 £ y £ 3 (d) None of these
206 JEE Main Mathematics

p p
76. The maximum value of cos2 æç - x ö÷ - cos2 æç + x ö÷ is 86. The number of values of x in the interval [0, 5p ]
è3 ø è3 ø satisfying the equation 3 sin 2 x - 7 sin x + 2 = 0 is
3 1 3 3 (a) 0 (b) 5 (c) 6 (d) 10
(a) - (b) (c) (d)
2 2 2 2
87. The number of solutions of the equation
p p x
77. The maximum value of sin æç x + ö÷ + cos æç x + ö÷ in 1 + sin x sin 2 = 0 in [ - p, p ] is
è 6 ø è 6ø 2
p
the interval æç0, ö÷ is attained at
(a) zero (b) one (c) two (d) three
è 2ø
88. The number of values of x lying in the interval
p p p p
(a) x = (b) x = (c) x = (d) x = ( - p, p) which satisfy the equation
12 6 3 2 (1 + cos x + cos 2 x + cos 3 x + . . . . ¥)
8 = 43 , is
78. The greatest and least value of sin x cos x are
respectively (a) 3 (b) 4 (c) 5 (d) 6
1 1 1 1 p
(a) 1 , - 1 (b) , - (c) , - (d) 2 , - 2 89. If 0 £ x < , then the number of values of x for
2 2 4 4 2
79. The maximum value of which sin x - sin 2 x + sin 3x = 0, is (JEE Main 2019)
x x (a) 2 (b) 3 (c) 1 (d) 4
4 sin 2 x + 3 cos2 x + sin + cos is
2 2
90. If sum of all the solutions of the equation
(a) 4 + 2 (b) 3 + 2 (c) 9 (d) 4
æ p p 1ö
80. cos 2 q + 2 cos q is always 8 cos x × ç cosæç + x ö÷ × cosæç - x ö÷ - ÷ = 1 in [0, p ] is
è è6 ø è6 ø 2ø
3
(a) greater than - kp, then k is equal to (JEE Main 2018)
2
3 2 13 8 20
(b) less than or equal to (a) (b) (c) (d)
2 3 9 9 9
3
(c) greater than or equal to - and less than or equal to 3 91. The equation esin x - e- sin x - 4 = 0 has
2
(d) None of the above (a) no solution (b) two solutions
(c) three solutions (d) None of these
81. The maximum value of (cos a 1) (cos a 2 ) K (cos a n )
p 92. The number of solutions of the equation
under the restriction 0 £ a 1, a 2 , . . . , a n £ and
2 p
x + 2 tan x = in the interval [0, 2p ] is
(cot a 1) (cot a 2 ) K (cot a n ) = 1, is 2 (JEE Main 2021)
1 1 1 (a) 3 (b) 4 (c) 2 (d) 5
(a) (b) (c) (d) 1
2n/ 2 2n 2n
93. The number of ordered pairs ( x, y) satisfying
Solution of Trigonometric Equations y = 2 sin x and y = 5 x 2 + 2 x + 3 is
(a) 0 (b) 1 (c) 2 (d) ¥
82. If sin q = 3 cos q , - p < q < 0, then q is equal to
5p 4p 4p 5p 94. The most general solutions of the equation
(a) - (b) - (c) (d) sec x - 1 = ( 2 - 1) tan x are given by
6 6 6 6
p p
83. Total number of solutions of cos x = 1 - sin 2 x in (a) np + (b) 2 np , 2 np +
8 4
[0, 2p ] is equal to (c) 2 np (d) None of these
(a) 2 (b) 3
(c) 5 (d) None of these 95. The solution set of (5 + 4 cos q ) ( 2 cos q + 1) = 0 in
the interval [0, 2p] is
84. Total number of solutions of the equation
5p ì p 2p ü ìp ü
3x + 2 tan x = in x Î[0, 2p ], is equal to (a) í , ý (b) í , p ý
2 î3 3 þ î3 þ
(a) 1 (b) 2 (c) 3 (d) 4 ì 2p 4p ü ì 2p 5p ü
(c) í , ý (d) í , ý
î 3 3þ î 3 3þ
85. Find the general solution of the equation
( 3 - 1) cos q + ( 3 + 1) sin q = 2 96. If 4 cos q - 3 sec q = 2 tan q , then q is equal to
p 5p p 5p p p
(a) 2np ± - (b) 2np ± + (a) np + (- 1)n (b) np + (- 1)n
4 12 4 12 10 6
3p 3p
(c) 2np ± p - (d) None of these (c) np + (- 1)n (d) np
11 10
Trigonometric Function and Equations 207

3x x p
97. If the equation cos 3x + cos 2 x = sin + sin , 106. The sum of all values of q Î æç0, ö÷ satisfying
2 2 è 2ø
3
0 £ x £ 2 p, then the number of values of x is sin 2 2q + cos4 2q = is
4 (JEE Main 2019)
(a) 6 (b) 7 (c) 4 (d) 5 3p 5p p
(a) (b) (c) (d) p
98. The most general values of q satisfying 8 4 2
3p
tan q + tan æç + q ö÷ = 2 is/are 107. If 0 £ x < 2 p, then the number of real values of x,
è 4 ø which satisfy the equation
p p cos x + cos 2 x + cos 3x + cos 4 x = 0, is (JEE Main 2016)
(a) np ± , n ÎI (b) 2 np + , n ÎI
3 3 (a) 3 (b) 5 (c) 7 (d) 9
p p
(c) 2 np ± , n Î I (d) 2 np + (-1)n , n Î I 108. If tan q + tan 2 q + 3 tan q tan 2 q = 3, then
3 3
(6n + 1)p (6n + 1)p
99. The general value of q satisfying sin 2 q + sin q = 2 is (a) q = , "n Î I (b) q = , "n Î I
18 9
p p
(a) np + (-1)n (b) 2 np + (3n + 1)p
6 4 (c) q = , "n Î I (d) None of these
9
n p p
(c) np + (-1) (d) np + (-1)n
2 3 109. Total number of solutions of
sin 4 x + cos4 x = sin x × cos x in [0, 2p ] is equal to
100. The general solution of sin 2 q sec q + 3 tan q = 0 is
p (a) 2 (b) 4 (c) 6 (d) 8
(a) q = np + (-1)n + 1 , q = np , n Î I
3 1
(b) q = np , n Î I 110. The general value of x for which cos 2 x, and
p 2
(c) q = np + (-1)n + 1 , n ÎI sin 2x are in AP, are given by
3
np p p
(d) q = , n ÎI (a) np , np + (b) np , np +
2 2 4
p 3 np
101. The general solution of sin x - 3 sin 2 x + sin 3x (c) np + , (d) None of these
= cos x - 3 cos 2 x + cos 3x is 4 4
p np p 111. Set of values of x lying in [0, 2p ] satisfying the
(a) np + (b) +
8 2 8 inequality|sin x |> 2 sin 2 x contains
n np p 3
(c) (-1) + (d) 2 np + cos -1 æ p ö æ 7p ö æ 7p ö
2 8 2 (a) ç0, ÷ È ç p , ÷ (b) ç0, ÷
è 6ø è 6 ø è 6 ø
102. The equation 3 sin 2 x + 10 cos x - 6 = 0 is satisfied, if p
æ1ö æ1ö (c) (d) None of these
(a) x = np ± cos -1 ç ÷ (b) x = 2 np ± cos -1 ç ÷ 6
è3ø è3ø
112. The solution of the equation sin10 2 x = 1 + cos10 x is
æ1ö æ1ö p
(c) x = np ± cos -1 ç ÷ (d) x = 2 np ± cos -1 ç ÷ (a) x = (2 n + 1) (b) x = np
è6ø è6ø 2
p
103. The smallest positive root of the equation (c) x = (2 n + 1) (d) None of these
4
tan x - x = 0 is in
113. The equation sin x + sin y + sin z = - 3 for
æ pö æ 3p ö æp ö æ 3p ö
(a) ç0, ÷ (b) ç p , ÷ (c) ç , p ÷ (d) ç , 2p ÷ 0 £ x £ 2 p, 0 £ y £ 2 p, 0 £ z £ 2 p has
è 2ø è 2 ø è2 ø è 2 ø
(a) one solution (b) two sets of solutions
104. The most general value of q satisfying the equation (c) four sets of solutions (d) no solution
(1 + 2 sin q ) 2 + ( 3 tan q - 1) 2 = 0 are given by 114. The number of solutions of the equation
p 7p 1 1
(a) np ± (b) np + (-1)n sin 5 x - cos5 x = - (sin x ¹ cos x) is
6 6 cos x sin x
7p 11p
(c) 2 np + (d) 2 np + (a) 0 (b) 1
6 6 (c) infinite (d) None of these
105. If the equation cos 3x cos3 x + sin 3x sin 3 x = 0, then 115. The number of ordered pairs ( x, y) , where
x is equal to x, y Î [0, 10 ] satisfying
p p
(a) (2 n + 1) (b) (2 n - 1) æ 2 1 ö sec 2 y
4 4 ç sin x - sin x + ÷ × 2 £ 1 is
np è 2ø
(c) (d) None of these
4 (a) 0 (b) 16 (c) infinite (d) 12
208 JEE Main Mathematics

x p 117. The general solution of e-1/ 2


( esin x + ecos x ) = 2 is
116. The equation 2 cos2 sin 2 x = x 2 + x -2 , x < has
2 9 (4m + 1)p
(a) x = mp (b) x =
(a) no real solution 4
(b) one real solution (4m + 1)p
(c) x = (d) None of these
(c) more than one real solutions 2
(d) None of the above

ROUND II Mixed Bag


Only One Correct Option 8. The value of cos2 10° - cos 10° cos 50° + cos2 50° is
(JEE Main 2019)
1. If tan a , tan b and tan g are the roots of the 3 3
equation x 3 - px 2 - r = 0, then the value of (a) (1 + cos 20° ) (b) + cos 20°
2 4
(1 + tan 2 a ) (1 + tan 2 b) (1 + tan 2 g ) is equal to (c) 3 / 2 (d) 3 / 4
(a) ( p - r ) 2 (b) 1 + ( p - r ) 2
(c) 1 - ( p - r ) 2 (d) None of these 9. If the equation cos4 q + sin 4 q + l = 0 has real
solutions for q, then l lies in the interval
2. If a , b, g and d are four solutions of the equation (JEE Main 2020)
p
tan æçq + ö÷ = 3 tan 3 q , then tan a tan b tan g tan d æ 5 ö
(a) ç - , - 1÷
é 1ù
(b) ê - 1, - ú
è 4ø è 4 ø ë 2û
equals æ 1 1ù é 3 5ù
(c) ç - , - ú (d) ê - , - ú
1 è 2 4û ë 2 4û
(a) 3 (b) 1/3 (c) - (d) None of these
3
10. If tan a / 2 and tan b/ 2 are the roots of the equation
3. The least value of cosec2 x + 25 sec 2 x is 8 x 2 - 26 x + 15 = 0, then cos (a + b) is equal to
(a) 0 (b) 26 (c) 28 (d) 36 627 627
(a) - (b)
4. If x sin a + y sin 2 a + z sin 3 a = sin 4 a 725 725
x sin b + y sin 2 b + z sin 3 b = sin 4 b (c) - 1 (d) None of these
n
x sin c + y sin 2 c + z sin 3 c = sin 4 c 11. Let n be an odd integer. If sin nq = å br sin r q for
Then, the roots of the equation all real q, then r=0

y + 2ö æ z - x ö = 0 , a , b , c ¹ np, are (a) b0 = 1, b1 = 3 (b) b0 = 0, b1 = n


t 3 - æç ö÷ t 2 - æç
z
÷t+ç ÷ (c) b0 = - 1, b1 = n (d) b0 = 0, b1 = n 2 - 3n - 3
è2ø è 4 ø è 8 ø
(a) sin a , sin b, sin c (b) cos a , cos b , cos c 1
12. If q is an acute angle and tan q = , then the
(c) sin 2 a , sin 2 b, sin 2 c (d) cos 2 a , cos 2 b, cos 2 c 7
cosec 2 q - sec 2 q
5. If 0° < q < 180° , then value of is
cosec 2 q + sec 2 q
2 + 2 + 2 + K + 2 (1 + cos q ), then being n
(a) 3/4 (b) 1/2 (c) 2 (d) 5/4
number of 2’s, is equal to p p
13. The number of values of q in the interval æç - , ö÷
æ qö æ q ö è 2 2ø
(a) 2 cos ç n ÷ (b) 2 cos ç n - 1 ÷
è2 ø è2 ø 2
q
satisfying the equation ( 3) sec = tan 4 q + 2 tan 2 q
æ q ö
(c) 2 cos ç n + 1 ÷ (d) None of these is
è2 ø
¥ ¥
(a) 1 (b) 2
6. If x = å ( -1) n tan 2 n q and y = å cos2 n q, for (c) 3 (d) None of these
n=0 n=0 p
14. If sin 4 x + cos4 y + 2 = 4 sin x cos y and 0 £ x, y £ ,
p 2
0 < q < , then
4 (JEE Main 2020) then sin x + cos y is equal to
(a) y(1 + x) = 1 (b) y(1 - x) = 1 3
(a) –2 (b) 0 (c) 2 (d)
(c) x(1 + y) = 1 (d) x(1 - y) = 1 2

7. If sin 4 a + 4 cos4 b + 2 = 4 2 sin a cos b; 15. The equation cos4 x - ( l + 2) cos2 x - ( l + 3) = 0


a, b Î[0, p ], then cos(a + b) - cos(a - b) is equal to possesses a solution, if
(JEE Main 2019) (a) l > - 3 (b) l < - 2
(a) - 1 (b) 2 (c) - 2 (d) 0 (c) -3 £ l £ - 2 (d) l is any positive integer
Trigonometric Function and Equations 209

16. The arithmetic mean of the roots of the equation 26. The minimum value of 27 cos 2 x + 81sin x is equal to
4 cos3 x - 4 cos2 x - cos (315 p + x) = 1 in the 2 1
(a) (b)
interval (0, 315) is equal to 3 3 3 3
2
(a) 50p (b) 51p (c) 100p (d) 315p (c) (d) None of these
sin q 9 3
17. If , cos q and tan q are in GP, then the general 1 3
6 27. If sin a = and sin b = , then b - a lies in the
value of q is 5 5
p p interval
(a) 2 np ± , n ÎI (b) 2 np ± , n ÎI
3 6 æ p ö æ 3p ö é p 3p ù
(a) ç0, ÷ È ç , p÷ (b) ê ,
p
(c) 2 np + (-1)n , n Î I
p
(d) n p + , n Î I
è 4ø è 4 ø ë 2 4 úû
3 3 æp pö æ 5p ù
(c) ç , ÷ (d) ç p ,
18. Set a , b Î [ -p, p ] be such that cos ( a - b) = 1 and
è4 2ø è 4 úû
1 tan A cot A
cos ( a + b) = . The number of pairs of a and b 28. The expression + can be written
e 1 - cot A 1 - tan A
satisfying the above system of equations is
as (JEE Main 2013)
(a) 0 (b) 1 (c) 2 (d) 4
(a) sin A cos A + 1 (b) sec A cosec A + 1
19. The number of solutions of the equation (c) tan A + cot A (d) sec A + cosec A
x 3 + x 2 + 4 x + 2 sin x = 0 in 0 £ x £ 2 p is
29. In a DPQR, if 3 sin P + 4 cos Q = 6 and
(a) zero (b) one (c) two (d) four
4 sin Q + 3 cos P = 1, then the ÐR is equal to
20. Let a and b be any two positive values of x for 5p p p 3p
(a) (b) (c) (d)
which 2 cos x,|cos x |and 1 - 3 cos2 x are in GP. The 6 6 4 4
minimum value of|a - b |is 30. If A = sin 2 x + cos4 x, then for all real x
p p 13
(a) (b) (a) £ A £1 (b) 1 £ A £ 2
3 4 16
p 3 13 3
(c) (d) None of these (c) £ A £
2 (d) £ A £1
4 16 4
21. If 5 (tan 2 x - cos2 x) = 2 cos 2 x + 9, then the value of 4 5
31. Let cos (a + b) = and let sin (a - b) = , where
cos 4x is (JEE Main 2017) 5 13
3 1 2 7 p
(a) - (b) (c) (d) - 0 £ a, b £ . Then, tan 2a is equal to
5 3 9 9 4
p p p p 25 56 19 20
22. The value of cos × cos 3 . . . . . . . cos 10 × sin 10 is (a) (b) (c) (d)
16 33 12 7
22 2 2 2
(JEE Main 2019) 32. The number of values of x in [0, 3p ] such that
1 1 1 1 2 sin 2 x + 5 sin x - 3 = 0 is
(a) (b) (c) (d)
1024 2 512 256 (a) 1 (b) 2 (c) 4 (d) 6
23. The equation 2
y = sin x sin( x + 2) - sin ( x + 1) 1
33. If 0 < x < p and cos x + sin x = , then tan x is equal to
represents a straight line lying in (JEE Main 2019) 2
(a) second and third quadrants only - (4 + 7 ) 1+ 7 1- 7 4- 7
(a) (b) (c) (d)
(b) first, second and fourth quadrants 3 4 4 3
p
. If tan æç ö÷ and tan æç ö÷ are the
(c) first, third and fourth quadrants P Q
34. In a D PQR, ÐR =
(d) third and fourth quadrants only 2 è2ø è2ø
24. Let S be the set of all a Î R such that the equation, roots of ax 2 + bx + c = 0, a ¹ 0, then
cos 2 x + a sin x = 2a - 7 has a solution. Then, S is (a) b = a + c (b) b = c (c) c = a + b (d) a = b + c
equal to (JEE Main 2019) 35. Let a and b such that p < a - b < 3p.
21 27
(a) R (b) [1, 4] (c) [3, 7] (d) [2, 6] If sin a + sin b = - , cos a + cos b = - , then
65 65
9p 11p 13p æa - b ö
25. sin × sin sin is equal to cos ç ÷ is
14 14 14 è 2 ø
1 1 1 1 3 3 6 6
(a) (b) - (c) (d) - (a) - (b) (c) (d) -
64 64 8 8 130 130 65 65
210 JEE Main Mathematics

Numerical Value Types Questions 41. The number of solutions of equation


[sin x ] = [1 + sin x ] + [1 - cos x ], 0 £ x £ 2 p, is _____ .
2 sin a 1 1 - cos 2b 1
36. If = and = , 2 sin 2q
1 + cos 2a 7 2 10 42. Given that, f ( nq ) = , and
cos 2q - cos 3nq
p
a , b Î æç0, ö÷, then tan(a + 2b) is equal to .............. . sin lq
è 2ø (JEE Main 2020) f (q ) + f ( 2q ) + f (3q ) + ¼ + f ( nq ) = , then
sin q sin mq
37. The number of distinct solutions of the equation, the value of m - l, is ____ .
log1/ 2|sin x|= 2 - log1/ 2|cos x|in
p
the interval [0, 2 p ], is (JEE Main 2020) 3 - tan 2
43. If 7 = l cos p , then l is ____.
q f p 7
38. If a + b tan = a - b tan , then ( b - sec q ) 1 - tan 2
2 2 7
( b + a cos f) has maximum value l, then l is _____.
44. If log|sin x ||cos x |+ log|cos x ||sin x |= 2, then|tan x |,
39. The number of integral values of a such that is equal to _____ .
sin x cos 3x - a cos 3x sin 3x = 0 does not have any
p 45. For any real value of q ¹ p, the value of the
real root other than ( 2 n + 1) , n Î1 for any real
2 cos2 q - 1
value of x, is _____ . expression y = , y Î R - [ a, b], then
cos2 q + cos q
40. The maximum value of the expression ( b - a) is ______ .
1
is _____ .
sin q + 3 sin q cos q + 5 cos2 q
2

Answers
Round I
1. (d) 2. (b) 3. (c) 4. (c) 5. (c) 6. (d) 7. (d) 8. (d) 9. (c) 10. (b)
11. (b) 12. (c) 13. (d) 14. (b) 15. (d) 16. (a) 17. (c) 18. (b) 19. (b) 20. (d)
21. (d) 22. (a) 23. (c) 24. (b) 25. (c) 26. (a) 27. (b) 28. (b) 29. (d) 30. (c)
31. (b) 32. (b) 33. (a) 34. (a) 35. (d) 36. (c) 37. (c) 38. (c) 39. (c) 40. (b)
41. (b) 42. (c) 43. (d) 44. (c) 45. (b) 46. (d) 47. (a) 48. (a) 49. (b) 50. (c)
51. (c) 52. (d) 53. (a) 54. (b) 55. (c) 56. (d) 57. (a) 58. (a) 59. (c) 60. (d)
61. (b) 62. (c) 63. (d) 64. (c) 65. (a) 66. (b) 67. (a) 68. (b) 69. (b) 70. (b)
71. (c) 72. (b) 73. (a) 74. (b) 75. (a) 76. (c) 77. (a) 78. (b) 79. (a) 80. (c)
81. (a) 82. (b) 83. (a) 84. (c) 85. (a) 86. (c) 87. (a) 88. (b) 89. (a) 90. (b)
91. (a) 92. (a) 93. (a) 94. (b) 95. (c) 96. (a) 97. (d) 98. (a) 99. (c) 100. (b)
101. (b) 102. (b) 103. (b) 104. (c) 105. (a) 106. (c) 107. (c) 108. (c) 109. (a) 110. (b)
111. (a) 112. (d) 113. (a) 114. (a) 115. (b) 116. (a) 117. (c)

Round II
1. (b) 2. (c) 3. (d) 4. (b) 5. (a) 6. (b) 7. (c) 8. (d) 9. (b) 10. (a)
11. (b) 12. (a) 13. (b) 14. (c) 15. (c) 16. (b) 17. (a) 18. (d) 19. (a) 20. (d)
21. (d) 22. (c) 23. (d) 24. (d) 25. (c) 26. (c) 27. (a) 28. (b) 29. (b) 30. (d)
31. (b) 32. (c) 33. (a) 34. (c) 35. (a) 36. (1) 37. (8) 38. (0) 39. (3) 40. (2)
41. (- 2) 42. (1) 43. (4) 44. (1) 45. (2)
Solutions
Round I = 13 + 3 sin 2 2 q - 4 cos 6 q - 12 cos 2 q + 12 cos 4 q
1. The given expression can be written as = 13 + 3(2 sin q cos q)2 - 4 cos 6 q - 12 cos 2 q(1 - cos 2 q)
1 + cos y - sin 2 y (1 - cos 2 y) - sin 2 y = 13 + 12 sin 2 q cos 2 q - 4 cos 6 q - 12 cos 2 q sin 2 q
+
1 + cos y sin y (1 - cos y) = 13 - 4 cos 6 q
cos y (1 + cos y) 1
= + 0 = cos y 7. fk (x) = (sin k x + cos k x), where x Î R and k ³ 1
1 + cos y k
1 - sin q 1 + sin q 1 - sin q + 1 + sin q Now, f4 (x) - f6 (x)
2. + = 1 1
1 + sin q 1 - sin q 1 - sin 2 q = (sin 4 x + cos 4 x) - (sin 6 x + cos 6 x)
4 6
2 2
= = 1 1
2 |cos q| = (1 - 2 sin x × cos x) - (1 - 3 sin 2 x × cos 2 x)
2 2
cos q 4 6
2 é p ù 1 1 1
= = - 2 sec q êëQ 2 < q < p úû = - =
- cos q 4 6 12
3. We have, cosec2 a + 2 cot a 2 sin a
8. Given, =x
1 + cos a + sin a
= 1 + cot2 a + 2 cot a = |1 + cot a|
2 sin a (1 - cos a - sin a )
3p Þ =x
But <a<p (1 + cos a + sin a ) (1 - cos a - sin a )
4
2 sin a (1 - cos a - sin a )
Þ cot a < - 1 Þ 1 + cot a < 0 Þ =x
Hence, |1 + cot a| = - (1 + cot a ) 1 - sin 2 a - cos 2 a - 2 sin a cos a
1 - cos a - sin a
4. Since, tan A + sin A = m Þ = -x
cos a
and tan A - sin A = n
9. Given that, sin q + cos q = m …(i)
\ m + n = 2 tan A
and sec q + cosec q = n …(ii)
and m - n = 2 sin A
Now, n (m + 1 ) (m - 1 ) = n (m2 - 1 )
Also, mn = (tan A + sin A ) (tan A - sin A )
= (sec q + cosec q) 2 sin q cos q
= tan 2 A - sin 2 A (Q m2 = 1 + 2 sin q cos q )
2 2 2
(m - n ) (m + n )2(m - n )2 sin q + cos q
Now, = = 2 sin q cos q
mn mn sin q cos q
(2 tan A )2(2 sin A )2 = 2m [from Eq. (i)]
=
tan 2 A - sin 2 A x y z
10. We have, = =
16 tan 2 A sin 2 A cos q æ 2p ö æ 2p ö
= = 16 cos ç q - ÷ cos ç q + ÷
è 3 ø è 3 ø
sin 2 A tan 2 A
Therefore, each ratio is equal to
5. 6 (sin 6 q + cos 6 q ) - 9 (sin 4 q + cos 4 q ) + 4 x+ y+ z
= 6 [(sin 2 q + cos 2 q )3 - 3 sin 2 q cos 2 q (sin 2 q + cos 2 q )] æ 2p ö æ 2p ö
cos q + cos ç q - ÷ + cos ç q + ÷
è 3 ø è 3 ø
- 9 [(sin 2 q + cos 2 q )2 - 2 sin 2 q cos 2 q] + 4
x+ y+ z x+ y+ z
= 6 [1 - 3 sin q cos 2 q] - 9 (1 - 2 sin 2 q cos 2 q) + 4
2
= =
2p 0
=6 -9 + 4 =1 cos q + 2 cos q cos
3
6. Given expression Þ x+ y+ z =0
= 3(sin q - cos q)4 + 6(sin q + cos q)2 + 4 sin 6 q 1 + sin x + 1 - sin x
= 3((sin q - cos q)2)2 + 6(sin q + cos q)2 + 4(sin 2 q)3 11. We have,
1 + sin x - 1 - sin x
= 3(1 - sin 2 q)2 + 6(1 + sin 2 q) + 4(1 - cos 2 q)3 2 2
[Q1 + sin 2 q = (cos q + sin q)2 æ x xö æ x xö
çsin + cos ÷ + çsin - cos ÷
è 2 2ø è 2 2ø
and 1 - sin 2 q = (cos q - sin q)2] =
2 2
= 3(12 + sin 2 2 q - 2 sin 2 q) + 6(1 + sin 2 q) æ x xö æ x xö
çsin + cos ÷ - çsin - cos ÷
+ 4(1 - cos 6 q - 3 cos 2 q + 3 cos 4 q) è 2 2ø è 2 2ø
[Q (a - b)2 = a 2 + b2 - 2ab x x x x
cos + sin + sin - cos
and (a - b) = a3 - b3 - 3a 2b + 3ab2]
3 2 2 2 2 x
= = tan
2 x x x x 2
= 3 + 3 sin 2 q - 6 sin 2 q + 6 + 6 sin 2 q + 4 cos + sin - sin + cos
2 2 2 2
- 4 cos 6 q - 12 cos 2 q + 12 cos 4 q
212 JEE Main Mathematics

12. Since, (2 tan q + 2)2 = tan q (3 tan q + 3) \ (m + n ) = a cos3 a + 3a cos a sin 2 a


+ 3a cos 2 a sin a + a sin3 a
Þ 4 tan 2 q + 8 tan q + 4 = 3 tan 2 q + 3 tan q
= a (cos a + sin a )3
Þ tan 2 q + 5 tan q + 4 = 0
and similarly, (m - n ) = a (cos a - sin a )3
Þ (tan q + 4) (tan q + 1 ) = 0
Þ tan q = - 4 (Q tan q ¹ - 1 ) \ (m + n )2/ 3 + (m - n )2/ 3
5 = a 2/ 3 {(cos a + sin a )2 + (cos a - sin a )2}
7+
7 - 5 cot q 4 = 33 = a 2/ 3 { 2 (cos 2 a + sin 2 a )} = 2 a 23
/
\ =
9 - 4 tan q 2 9 - 4 (- 4) 100 1æ 1ö
18. Given that, cos q = ç x + ÷
2è xø
13. Given, cos x = tan y, cot y = tan z and cot z = tan x 1
1 Þ x + = 2 cos q …(i)
\ cos x = tan y Þ cos x = x
tan z We know that,
2
Þ cos x = cot z Þ cos x = tan x 1 æ 1ö
x2 + 2 = ç x + ÷ - 2
Þ cos x = sin x/cos x Þ cos 2 x = sin x x è xø
Þ 1 - sin 2 x = sin x Þ sin 2 x + sin x - 1 = 0 = (2 cos q )2 - 2 = 4 cos 2 q - 2
- 1 ± 1 - 4 ´ (- 1) -1 ± 5 = 2 cos 2 q [from Eq. (i)]
\ sin x = = 1æ 2 1ö 1
2 ´1 2 \ ç x + 2 ÷ = ´ 2 cos 2 q = cos 2 q
2è x ø 2
5 -1 é -1 - 5 ù
Þ sin x = < -1 ú é 3p ù
2
êQ
ë 2 û 19. We have, 3 êsin 4 æç ö
- a ÷ + sin 4 (3p + a )ú
ë è 2 ø û
14. Given, sin A - 6 cos A = 7 cos A é æp ö ù
- 2 êsin 6 ç + a ÷ + sin 6 (5p - a )ú
On squaring given equation, we get ë è 2 ø û
sin 2 A + 6 cos 2 A - 2 6 sin A cos A = 7 cos 2 A = 3 [(- cos a )4 + (- sin a )4 ] - 2 [cos 6 a + sin 6 a ]
Þ sin 2 A + 6 (1 - sin 2 A ) = cos 2 A + 6 cos 2 A = 3 [(cos 2 a + sin 2 a )2 - 2 sin 2 a cos 2 a ]
+ 2 6 sin A cos A - 2 [(cos 2 a + sin 2 a )3 - 3 cos 2 a sin 2 a (cos 2 a + sin 2 a )]
Þ sin A - 6 cos A + 6 = cos A + 6 sin 2 A
2 2 2
= 3 - 6 sin 2 a cos 2 a - 2 + 6 sin 2 a cos 2 a = 3 - 2 = 1
+ 2 6 sin A cos A 1 p
20. Since, tan x + = 2 Þ tan x = 1 Þ x =
Þ 7 sin 2 A = (cos A + 6 sin A )2 tan x 4
Þ ± 7 sin A = cos A + 6 sin A 1 1
\ sin x = and cos x =
2 2
15. Given that, ABCD is a cyclic quadrilateral.
1 1 1
So, A + C = 180° Þ A = 180° - C Hence, sin 2n x + cos 2n x = n + n = n - 1
Þ cos A = cos (180° - C ) = - cos C 2 2 2
Þ cos A + cos C = 0 …(i) 21. 15 sin 4 a + 10 cos 4 a = 6
Similarly, cos B + cos D = 0 …(ii)
On adding Eqs. (i) and (ii), we get 15 sin 4 a + 10 cos 4 a = 6 (sin 2 a + cos 2 a )2
cos A + cos B + cos C + cos D = 0 (3 sin 2 a - 2 cos 2 a )2 = 0
2 3
16. Given that, 2 sin A = 3 sin B tan 2 a = , cot2 a = Þ 27 sec6 a + 8 cosec6a
3 2
Þ 2 5 sin A = 15 sin B …(i)
= 27 (sec2 a )3 + 8 (cosec2a )3
and 2 cos A = 5 cos B
= 27 (1 + tan 2 a )3 + 8 (1 + cot2 a )3 = 250
Þ 2 3 cos A = 15 cos B …(ii)
æ p ö æ p ö n
On squaring and adding Eqs. (i) and (ii), we get 22. Qsin ç ÷ + cos ç ÷=
è2 nø è2 nø 2
20 sin 2 A + 12 cos 2 A = 15
3 On squaring both sides, we get
Þ 8 sin 2 A = 3 Þ sin 2 A = æ p ö
8 2æ p ö æpö n
sin 2 ç ÷ + cos ç ÷ + sin ç ÷=
2 5 è2 nø è2 nø è nø 4
Þ cos A =
8
æpö n
2
sin A 3 3 Þ sin ç ÷ = - 1
\ = Þ tan A = è nø 4
cos 2 A 5 5
æpö n -4
17. Given, a cos3 a + 3a cos a sin 2 a = m Þ sin ç ÷ =
è nø 4
and a sin3 a + 3a cos 2 a sin a = n Þ n = 6 only
Trigonometric Function and Equations 213

23. Given, m sin q = n sin (q + 2a ) 56


=
m sin(q + 2a ) 33
Þ = 56
n sin q Q tan 2a =
33
Applying componendo and dividendo rule, we get n n
m + n sin(q + 2a ) + sin q æ cos A + cos B ö æ sin A + sin B ö
= 28. ç ÷ +ç ÷
m - n sin (q + 2a ) - sin q è sin A - sin B ø è cos A - cos B ø
n
2 sin(q + a ) cos a é æ A + Bö æ A - Bö ù
=
2 cos (q + a ) sin a ê 2 cos çè 2 ÷ø cos çè 2 ÷ø ú
=ê ú
= tan (q + a ) cot a ê 2 cos æç A + B ö÷ sin çæ A - B ö÷ ú
sin (B + A ) + cos (B - A ) êë è 2 ø è 2 ø úû
24.
sin (B - A ) + cos (B + A ) n
é æ A + Bö æ A - Bö ù
sin (B + A ) + sin (90° - B - A ) ê 2 sin çè 2 ÷ø cos çè 2 ÷ø ú
= +ê ú
sin (B - A ) + sin (90° - A + B ) ê 2 sin æç A + B ö÷ sin çæ B - A ö÷ ú
2 sin ( A + 45° ) cos (45° - B ) êë è 2 ø è 2 ø úû
=
2 sin (45° - A ) cos (45° - B )
æ A - Bö n æ B - Aö
1 1 = cotn ç ÷ + cot ç ÷
sin A + cos A è 2 ø è 2 ø
sin ( A + 45° )
= = 2 2
æ A - Bö æ A - Bö
sin (45° - A ) 1
cos A -
1
sin A = cotn ç n
÷ + (-1 ) cot ç ÷
è 2 ø è 2 ø
2 2
cos A + sin A æ A - Bö
= = 2 cotn ç ÷ (Q n is even)
cos A - sin A è 2 ø
15 12
25. We have, 29. Given, sin a = , tan b =
17 5
cos 2( A - B ) + cos 2 B - 2 cos ( A - B ) cos A cos B
p 3p
= cos 2( A - B ) + cos 2 B - cos ( A - B ) Since, < a < p, p < b <
2 2
´ [cos ( A - B ) + cos ( A + B )] 8 12
2 \ cos a = - , sin b = -
= cos B - cos ( A - B ) cos ( A + B ) 17 13
2 2 2
= cos B - (cos A - sin B ) 5
and cos b = -
= 1 - cos 2 A = sin 2 A 13
Hence, it depends on A . Now, sin ( b - a ) = sin b cos a - cos b sin a
sin A - sin C 12 æ - 8 ö æ - 5 ö æ 15 ö
26. Given, = cot B =- ç ÷-ç ÷ç ÷
cos C - cos A 13 è 17 ø è 13 ø è 17 ø
æ A + Cö æ A -Cö =
96
+
75
2 cos ç ÷ sin ç ÷
è 2 ø è 2 ø 221 221
Þ = cot B
æ A + Cö æ A -Cö 171
2 sin ç ÷ sin ç ÷ =
è 2 ø è 2 ø 221
æ A + Cö A+C 30. Given equations may be written as
Þ cot ç ÷ = cot B Þ B =
è 2 ø 2 cos x + cos y = - cos a
Hence, A , B and C will be in AP. and sin x + sin y = - sin a
4 5 æ x + yö æ x - yö
27. We have, cos (a + b) = and sin (a - b ) = Þ 2 cos ç ÷ cos ç ÷ = - cos a …(i)
5 13 è 2 ø è 2 ø
3 12 æ x + yö æ x - yö
Þ sin (a + b ) = and cos (a - b ) = and 2 sin ç ÷ cos ç ÷ = - sin a …(ii)
5 13 è 2 ø è 2 ø
3 5 From Eqs. (i) and (ii), we get
Þ Now, tan (a + b ) = and tan (a - b ) =
4 12 æ x + yö æ x - yö
2 cos ç ÷ cos ç ÷
tan (a + b ) + tan (a - b ) è 2 ø è 2 ø cos a
tan 2a = tan (a + b + a - b ) = =
1 - tan (a + b ) tan (a - b ) æ x + yö æ x - y ö sin a
2 sin ç ÷ cos ç ÷
è 2 ø è 2 ø
3 / 4 + 5 / 12
= æ x + yö
5 Þ cot ç ÷ = cot a
1 - 3 /4 ´ è 2 ø
12
214 JEE Main Mathematics

31. tan 81° - tan 63° - tan 27° + tan 9° A+B B- A


2 cos cos
2 2 m+1
= (tan 81° + tan 9° ) - [tan (63° ) + tan (27° )] Þ =
A+B B - A m -1
2 sin sin
é sin 81° sin 9° ù é sin 63° sin 27° ù 2 2
=ê + ú-ê + ú
ë cos 81° cos 9° û ë cos 63° cos 27° û A + B æm + 1ö B- A
Þ cot =ç ÷ tan
é sin 81° cos 9° + sin 9° cos 81° ù 2 èm - 1ø 2
=ê ú
ë cos 9° cos 81° û A+B B- A
But cot = l tan
é sin 63° cos 27° + sin 27° cos 63° ù 2 2
-ê ú m+1
ë cos 27° cos 63° û \ l=
m -1
[sin (81° + 9° )] [sin (63° + 27° )]
= -
cos 9° cos (90° - 9° ) cos 27° cos (90° - 27° ) 35. Qa - b = (q - b) - (q - a ) …(i)
sin 90° sin 90° \ cos (a - b ) = cos (q - b ) cos (q - a )
= -
cos 9° sin 9° cos 27° sin 27° + sin (q - b ) sin (q - a )
1 2 1 2 and sin (a - b ) = sin (q - b ) cos (q - a )
= ´ - ´
sin 9° cos 9° 2 sin 27° cos 27° 2 - sin (q - a ) cos (q - b )
2 2 Þ cos (a - b ) = b × a + 1 - a 2
1 - b2
= -
sin 18° sin 54°
and sin (a - b ) = (a 1 - b2 - b 1 - a 2 )
é sin 54° - sin 18° ù é 2 cos 36° sin 18° ù
=2 ê ú =2 ê ú =4
ë sin 18° sin 54° û ë sin 18° cos 36° û Now, sin 2 (a - b ) = (a 1 - b2 )2 + (b 1 - a 2 )2

32. sin 2 a = sin {(a + b) + (a - b)} - 2ab 1 - a 2 1 - b2

= sin (a + b ) cos (a - b ) + cos (a + b ) sin (a - b )…(i) Þ sin 2 (a - b ) = a 2(1 - b2) + b2(1 - a 2)


Now, sin (a + b ) = 1 - cos 2(a + b ) - 2ab{cos (a - b ) - ab}
2
Þ sin (a - b ) + 2ab cos (a - b )
9 4 é pù
= 1- = êëQ 0 < a + b < 2 úû = a 2 - a 2b2 + b2 - b2a 2 + 2a 2b2
25 5
Þ sin 2 (a - b ) + 2ab cos (a - b ) = a 2 + b2
and sin (a - b ) = 1 - cos 2(a - b )
sin q 3
36. Q =
16 3 é pù sin (q + 2 a ) 1
= 1- = êëQ 0 < a - b < 4 úû
25 5 On applying componendo and dividendo rule, we get
From Eq. (i), sin q + sin (q + 2 a ) 3 + 1
4 4 3 3 16 9 25 =
× + × =
sin 2 a = + = =1 sin q - sin (q + 2 a ) 3 - 1
5 5 5 5 25 25 25
2 sin (q + a ) cos a
tan a + tan b Þ - =2
33. We have, tan (a + b) = 2 cos (q + a )sin a
1 - tan a tan b
Þ - tan (q + a ) cot a = 2
1 1
Q tan a = -x
and tan b = Þ tan (q + a ) + 2 tan a = 0
1+2 1 + 2x + 1
p
1 1 37. Given, a + b = and b + g = a
+ 2
1+ x
1 1 + 2x + 1
p
\ tan (a + b ) = 2 Þ b = - a and b + g = a
1 1 2
1- ×
1 1 + 2x + 1 æp ö
1+ x Þ tan b = tan ç - a ÷
2 è2 ø
2x + 2 × 22x + 2x + 1 Þ tan b = cot a …(i)
Þ tan (a + b ) =
1 + 2x + 2 × 2x + 2 × 22x - 2x and tan (b + g ) = tan a
p tan b + tan g
Þ tan (a + b ) = 1 Þ a + b = Þ = tan a
4 1 - tan b tan g
34. We have, cos A = m cos B tan b + tan g tan a
Þ = [from Eq. (i)]
cos A m 1 - cot a tan g 1
Þ =
cos B 1 Þ tan b + tan g = tan a - tan g
cos A + cos B m + 1 Þ tan a = tan b + 2 tan g
Þ =
cos A - cos B m - 1
Trigonometric Function and Equations 215

38. cos 12° + cos 84° + cos 156° + cos 132° æ x + yö æ x - yö 2 æ x + yö


Þ 4 cos ç ÷ cos ç ÷ - 4 cos ç ÷ =1
è 2 ø è 2 ø è 2 ø
= cos 156° + cos 84° + cos 132° + cos 12°
æ 156° + 84° ö æ 156° - 84° ö æ x - yö 2 æ x - yö
= 2 cos ç ÷ cos ç ÷ = cos 2 ç ÷ + sin ç ÷
è 2 ø è 2 ø è 2 ø è 2 ø

æ 132° + 12° ö æ 132° - 12° ö æ x + yö 2 æ x - yö æ x + yö æ x - yö


+ 2 cos ç ÷ cos ç ÷ Þ 4 cos 2 ç ÷ cos ç ÷ - 4 cos ç ÷ cos ç ÷
è 2 ø è 2 ø è 2 ø è 2 ø è 2 ø è 2 ø

= 2 cos 120° cos 36° + 2 cos 72° cos 60° æ x - yö


+ sin 2 ç ÷ =0
è 2 ø
æ 1ö 1
= 2ç - ÷ cos 36° + 2 cos 72° ´ 2
è 2ø 2 æ æ x - yö æ x + yö ö 2 æ x - yö
Þ ç cos ç ÷ - 2 cos ç ÷ ÷ + sin ç ÷ =0
= - cos 36° + cos 72° è è 2 ø è 2 øø è 2 ø
5+1 5 -1 æ x - yö
=- + Þ sin ç ÷ =0Þx= y
4 4 è 2 ø
1
=- æ x - yö æ x + yö
2 Þ cos ç ÷ = 2 cos ç ÷ [Now, put x = y]
è 2 ø è 2 ø
39. Given equation can be written as 1
Þ cos x = = cos y
æ pö æ 7p ö æ 3p ö æ 5p ö 2
ç1 + cos ÷ ç1 + cos ÷ ç1 + cos ÷ ç1 + cos ÷
è 8ø è 8 øè 8 øè 8 ø 3 1 3+1
\ sin x + cos y = + =
æ p 7p p 7p ö 2 2 2
= ç1 + cos + cos + cos cos ÷
è 8 8 8 8 ø
42. sin 20° sin 40° sin 60° sin 80°
æ 3p 5p 3p 5p ö 1
ç1 + cos + cos + cos cos ÷ = sin 20° sin 60° (2 sin 40° sin 80° )
è 8 8 8 8 ø 2
æ p p p 7p ö 1
= ç1 + cos - cos + cos cos ÷ = sin 20° sin 60° (cos 40° - cos 120° )
è 8 8 8 8 ø 2
æ 5p 5p 3p 5p ö 1 3 æ 1ö
ç1 - cos + cos + cos cos ÷ = × sin 20° ç1 - 2 sin 2 20° + ÷
è 8 8 8 8 ø 2 2 è 2ø
æ p 7p ö æ 3p 5p ö 3 æ3 ö
= ç1 + cos cos ÷ ç1 + cos cos ÷ = sin 20° ç - 2 sin 2 20° ÷
è 8 8 ø è 8 8 ø 4 è2 ø
1æ p 7p ö æ 3p 5p ö 3
= ç2 + 2 cos cos ÷ ç2 + 2 cos cos ÷ = (3 sin 20° - 4 sin3 20° )
4 è 8 8 ø è 8 8 ø 8
1æ 3p öæ p ö 3 3 3 3
= ç2 + cos + cos p ÷ ç2 + cos + cos p ÷ = sin 60° = × =
4è 4 øè 4 ø 8 8 2 16
1æ 3p ö æ pö 1 æ pö æ pö p
= ç1 + cos ÷ ç1 + cos ÷ = ç1 - cos ÷ ç1 + cos ÷ sin 24
4è 4 øè 4ø 4 è 4ø è 4ø p 2p 4p 8p 5
43. cos cos cos cos =
p
2 pö
1æ 1æ 1ö 1 5 5 5 5 24 sin
= ç1 - cos ÷ = ç1 - ÷ = 5
4 è 4 ø 4 è 2ø 8
æ sin 2n x ö
40. Given, cos a + cos b = 0 and sin a + sin b = 0 çQ cos x cos 2x cos 4xK cos 2 nx = n ÷
è 2 sin x ø
On squaring and subtracting both the equations,
we get 16p sin æ3p + p ö
sin ç ÷
(cos a + cos b )2 - (sin a + sin b )2 = 0 5 = è 5ø
=
Þ (cos 2 a + cos 2 b + 2 cos a cos b ) p p
16 sin 16 sin
5 5
- (sin 2 a + sin 2 b + 2 sin a sin b ) = 0
p
Þ (cos a - sin a ) + (cos 2 b - sin 2 b )
2 2
- sin
= 5 =- 1
+ 2 [cos a cos b - sin a sin b ] = 0 p
16 sin 16
Þ cos 2a + cos 2b + 2 cos (a + b ) = 0
5
Þ cos 2a + cos 2b = - 2 cos(a + b ) 2p
x + yö 44. Let = q Þ 2p = 3 q + 4q Þ 4q = 2p - 3 q
æ x - yö é 2 æ x + yö ù 3
41. 2 cos æç ÷ cos ç ÷ - 2 cos ç ÷ - 1ú = 7
è 2 ø è 2 ø êë è 2 ø û 2 Þ sin 4q = sin (2p - 3 q) Þ sin 4 q = - sin 3 q
æ x + yö æ x - yö 2 æ x + yö 1 Þ 2 sin 2 q cos 2 q = - [3 sin q - 4 sin3 q]
Þ 2 cos ç ÷ cos ç ÷ - 2 cos ç ÷=
è 2 ø è 2 ø è 2 ø 2 Þ 2 ´ 2 sin q cos q (2 cos 2 q - 1) = - 3 sin q + 4 sin3 q
216 JEE Main Mathematics

Þ sin q [8 cos3 q - 4 cos q + 3 - 4 (1 - cos 2 q)] = 0 48. Let A + B = q and A - B = f


Þ 8 cos3 q + 4 cos 2 q - 4 cos q - 1 = 0 …(i) Then, tan A = k tan B
2p 4p 6p tan A k k sin A cos B
Thus, cos , cos and cos are the roots of Eq. (i). Þ = Þ =
7 7 7 tan B 1 1 cos A sin B
2p 4p 6p 1 Applying componendo and dividendo rule, we get
\ cos + cos + cos =-
7 7 7 2 k + 1 sin A cos B + cos A sin B
=
q a k - 1 sin A cos B - cos A sin B
tan 2 - tan 2
æ q + aö æ q - aö 2 2 sin ( A + B ) sin q
45. tan ç ÷ × tan ç ÷= = =
è 2 ø è 2 ø q a sin ( A - B ) sin f
1 - tan 2 tan 2
2 2 k+1
Þ sin q = sin f
q a a q k -1
sin 2 cos 2 - sin 2 cos 2
= 2 2 2 2
a q a q 49. Since, A+ B+C=p
cos 2 cos 2 - sin 2 sin 2
2 2 2 2 Þ A = p - (B + C )
a q a q a q We have, cos A = cos B cos C
cos 2 - cos 2 cos 2 - cos 2 + cos 2 cos 2
= 2 2 2 2 2 2 Þ cos [p - (B + C )] = cos B cos C
q a q a a q
cos 2 - sin 2 cos 2 - sin 2 + sin 2 cos 2 Þ - cos (B + C ) = cos B cos C
2 2 2 2 2 2
Þ - [cos B cos C - sin B sin C ] = cos B cos C
a q
cos 2 - cos 2 Þ sin B sin C = 2 cos B cos C
2 2 (cos a - cos q)
= = Þ tan B tan C = 2
2 q 2 a (cos a + cos q)
cos - sin
2 2 50. Now, sin 12° sin 48° sin 54°
cos a (1 - cos b ) b 1
= = tan 2 = (cos 36° - cos 60° ) cos 36°
cos a (1 + cos b ) 2 2
3 1 é 5 + 1 1ù é 5 + 1ù
46. Given, cos x + cos y = = ê - úê ú
2 2ë 4 2û ë 4 û
æ x + yö æ x - yö 3 1 é 5 - 1ù é 5 + 1ù
Þ 2 cos ç ÷ cos ç ÷= …(i) = ê úê ú
è 2 ø è 2 ø 2 2ë 4 û ë 4 û
3 5 -1 4 1
and sin x + sin y = = = =
4 32 32 8
æ x + yö æ x - yö 3 51. 1 + cos 56° + cos 58° - cos 66°
Þ 2 sin ç ÷ cos ç ÷= …(ii)
è 2 ø è 2 ø 4
= 2 cos 2 28° + 2 sin 62° sin 4°
On dividing Eq. (ii) by Eq. (i), we get = 2 cos 2 28° + 2 cos 28° cos 86°
æx+ yö æ x - yö = 2 cos 28° (cos 28° + cos 86° )
2 sin ç ÷ cos ç ÷
è 2 ø è 2 ø 3 /4 æx+ yö 1 = 2 cos 28° (2 cos 57° cos 29° )
= Þ tan ç ÷=
æx+ yö æ x - y ö 3 /2 è 2 ø 2 = 4 cos 28° cos 29° sin 33°
2 cos ç ÷ cos ç ÷
è 2 ø è 2 ø 6p p
tan - tan
æ x + yö p æ 6p pö 15 15
2 tan ç ÷ 2´
1 52. Now, tan = tan ç - ÷=
è 2 ø 3 è 15 15 ø 6p p
\ sin (x + y) = = 2 = 4 =4 1 + tan tan
æ x + y ö 2
4+1 5 15 15
÷ 1 + æç ö÷
1 + tan 2ç 1
è 2 ø 6p p 6p p
è2ø Þ tan - tan = 3 + 3 tan tan
15 15 15 15
47. sin (a + b + g ) = sin a cos b cos g 6p p 6p p
Þ tan - tan - 3 tan tan = 3
15 15 15 15
+ cos a sin b cos g + cos a cos b sin g - sin a sin b sin g
2p p 2p p
Þ sin (a + b + g ) - sin a - sin b - sin g Þ tan - tan - 3 tan tan = 3
5 15 5 15
= sin a (cos b cos g - 1 ) + sin b (cos a cos g - 1 ) 53. We have, S = sin q + sin 2 q + sin 3 q + K + sin nq
+ sin g (cos a cos b - 1 ) - sin a sin b sin g We know that,
Þ sin (a + b + g ) - sin a - sin b - sin g < 0 sin q + sin (q + b ) + sin (q + 2 b ) + K n terms
nb
Þ sin (a + b + g ) < sin a + sin b + sin g sin
= 2 sin é q + q + (n - 1 ) b ù
sin (a + b + g ) b êë úû
Þ <1 sin 2
sin a + sin b + sin g 2
Trigonometric Function and Equations 217

Put b = q, = sin a [sin (b - g ) + sin (b + g )]


nq (n + 1 ) q
sin × sin = sin a [2 sin b cos g ]
\ S= 2 2
q = 2 sin a sin b cos g
sin
2 58. We have, a + b + g = 2p
n
a b g
54. We have, sin3 x sin 3x = å cm cos mx Þ + + =p
2 2 2
m= 0
1 a b g
Now, sin3 x sin 3x = (3 sin x - sin 3x )sin 3x Þ + =p-
4 2 2 2
3 1 æ a bö æ gö
= × 2 sin x sin 3x - × 2 sin 2 3x Þ tan ç + ÷ = tan ç p - ÷
8 8 è 2 2ø è 2ø
3 1 a b
= (cos 2x - cos 4x ) - (1 - cos 6x ) tan + tan
8 8 Þ 2 2 = - tan g
1 3 3 1 a b 2
= - + cos 2 x - cos 4x + cos 6x …(i) 1 - tan tan
8 8 8 8 2 2
n a b g a b g
RHS = å cm cos mx = c0 + c1 cos x + c2 cos 2x Þ tan + tan + tan = tan tan tan
2 2 2 2 2 2
m= 0
cos 12° - sin 12° sin 147° 1 - tan 12°
+ c3 cos 3x + . .. + cn cos n x …(ii) 59. + = + tan 147°
On comparing Eqs. (i) and (ii), we get cos 12° + sin 12° cos 147° 1 + tan 12°
n =6 = tan (45° - 12° ) + tan (180° - 33° )
2p ö
55. We have, x cos q = y cos æç q + ÷
= tan 33° - tan 33° = 0
è 3 ø sin x 1 cos x 3
60. Given, = , =
æ 4p ö sin y 2 cos y 2
= z cos ç q + ÷=k (say)
è 3 ø tan x 1
Þ =
k æ 2p ö k æ 4p ö k tan y 3
Þ cos q =, cos ç q + ÷ = and cos ç q + ÷=
x è 3 ø y è 3 ø z tan x + tan y 4 tan x
tan (x + y) = =
k k k æ 2p ö æ 4p ö 1 - tan x tan y 1 - 3 tan 2 x
\ + + = cos q + cos ç q + ÷ + cos ç q + ÷
x y z è 3 ø è 3 ø 2
Also, sin y = 2 sin x, cos y = cos x
æp ö æp ö 3
= cos q - cos ç - q÷ - cos ç + q÷ 2
è3 ø è3 ø 4 cos x
Þ sin 2 y + cos 2 y = 4 sin 2 x + =1
p 9
= cos q - 2 cos cos q = 0
3 Þ 36 tan x + 4 = 9 sec x = 9 (1 + tan 2 x)
2 2

1 1 1 Þ 27 tan 2 x = 5
Þ + + =0 5
x y z Þ tan x =
3p 3 3
56. Since, A+ B+C= 4 5
2
\ cos 2 A + cos 2 B + cos 2 C Þ tan (x + y) = 3 3 = 15
15
= 2 cos ( A + B) cos ( A - B) + cos 2 C 1-
27
æ 3p ö
= 2 cos ç - C ÷ cos ( A - B ) + 1 - 2 sin 2 C 61. Since, sin q + cos q = x …(i)
è 2 ø
1
é æ 3p öù and sin 6 q + cos 6 q = [4 - 3 (x2 - 1 )2]
= 1 - 2 sin C ê cos ( A - B ) + sin ç - ( A + B )÷ ú 4
ë è 2 øû
On squaring Eq. (i), we get
= 1 - 2 sin C [cos ( A - B ) - cos ( A + B )] sin 2 q = x2 - 1 £ 1 (Q sin 2 q £ 1)
2
= 1 - 4 sin A sin B sin C Þ x £2 Þ - 2 £ x£ 2
Now, sin 6 q + cos 6 q = (sin 2 q + cos 2 q )3
57. We have, a+b-g=p
- 3 sin 2 q cos 2 q (sin 2 q + cos 2 q )
Now, sin a + sin 2 b - sin 2 g
2
3
= 1 - 3 sin 2 q cos 2 q = 1 - sin 2 2 q
= sin 2 a + sin (b - g ) sin (b + g ) 4
= sin 2 a + sin (p - a ) sin (b + g ) (Q a + b - g = p ) 3 2 1
= 1 - (x - 1 ) = [4 - 3 (x2 - 1 )2]
2

= sin 2 a + sin a sin (b + g ) 4 4


= sin a [sin a + sin (b + g )] Thus, the given result will hold true only when x2 £ 2
= sin a [sin(p - (b - g )) + sin (b + g )] and not for all real values of x .
218 JEE Main Mathematics

62. We have, tan 20° tan 40° tan 60° tan 80° a+b a-b
65. +
sin 20° sin 40° sin 80° a-b a+b
= tan 60°
cos 20° cos 40° cos 80° b b b b
1+ 1- 1+ +1-
Here, numerator = (sin 20° sin 40° sin 80° ) = a + a = a a
sin 20° b b æ bö æ bö
= (2 sin 40° sin 80° ) 1- 1+ ç1 - ÷ ç1 + ÷
2 a a è aø è aø
sin 20° [dividing by a in numerator and denominator
= (cos 40° - cos 120° )
2 of both the terms]
1 æ 1ö 2 2 2 æ bö
= sin 20° ç1 - 2 sin 2 20° + ÷ = = = çQ tan x = ÷
2 è 2ø b 2
1 - tan x2
sin x2 è a ø
1- 2 1-
1 æ3 ö a cos x2
= sin 20° ç - 2 sin 2 20° ÷
2 è2 ø 2 cos x 2 cos x
= = [Q cos 2 x - sin 2 x = cos 2x]
1 cos 2 x - sin 2 x cos 2x
= [ 3 sin 20° - 4 sin3 20° ]
4
sin 60° 3 66. Since, (a - b) sin (q + f ) = (a + b) sin (q - f )
= = Þ a {sin (q + f ) - sin (q - f )} = b {sin (q - f ) + sin (q + f )}
4 8
Now, denominator = cos 20° cos 40° cos 80° Þ 2 a sin f cos q = 2 b sin q cos f Þ a tan f = b tan q
f q
sin 2320° sin 160° 2 a tan 2 b tan
= 3 = 2 2
2 sin 20° 8 sin 20° Þ = …(i)
f q
sin 20° 1 1 - tan 2 1 - tan 2
= = 2 2
8 sin 20° 8 q f
3 Since, a tan - b tan = c (given)
2 2
Hence, tan 20° tan 40° tan 80° = 8 = 3 f
1 b tan + c
q 2
8 Þ tan = …(ii)
2 a
Þ tan 20° tan 40° tan 60° tan 80° = 3 × 3 = 3 From Eqs. (i) and (ii), we get
æ f ö
63. sin 36° sin 72° sin 108° sin 144° ç b tan + c÷
f è 2 ø
= sin 2 36° sin 2 72° a tan b
2 = a
1 f 2
= [(2 sin 2 36° ) (2 sin 2 72° )] 1 - tan 2 æ f ö
4 2 ç b tan + c ÷
è 2 ø
1 1-
= [(1 - cos 72° ) (1 - cos 144° )] a2
4
f æ fö
1
= [(1 - sin 18° ) (1 + cos 36° )] Þ tan (a 2 - b2 - c2) = bc ç1 + tan 2 ÷
2 è 2ø
4
f
1 éæ 5 - 1ö æ 5 + 1öù 2 tan
= ê ç1 - ÷ ç1 + ÷ú 2 2 bc
4 ëè 4 øè 4 øû Now, sin f = =
f2 a - b2 - c2
2
1 + tan
1é æ 5 + 1ö æ 5 - 1ö æ 4 öù 2
= ê1 + ç ÷-ç ÷ - ç ÷ú
4ë è 4 ø è 4 ø è 16 ø û 67. 2 cos x - cos 3x - cos 5x = 2 cos x - 2 cos x cos 4x
1é 1 1ù 5 = 2 cos x (1 - cos 4x ) = 2 cos x 2 sin 2 2x
= ê1 + - ú =
4ë 2 4 û 16 = 4 cos x (2 sin x cos x )2 = 16 sin 2 x cos3 x

64. We have, sin 10° sin 30° sin 50° sin 70° p 2p 32p p 2p 25 p
68. cos cos K cos = cos × cos K cos
= sin(30° )[sin(10° )sin(50° )sin(70° )] 65 65 65 65 65 65
1 26 p 64p
= [sin(10° )sin(60° - 10° )sin(60° + 10° )] sin sin
2 65 65
= =
1 é1 ù 6 p p
= ê sin(3(10° ))ú 2 sin 64 sin
2 ë4 û 65 65
1 æ pö
[Qsin q sin(60° - q)sin(60° + q) = sin 3 q] sin ç p - ÷
4 è 65 ø 1
= =
1 1 1 1 p 64
= sin 30° = ´ = 64 sin
8 8 2 16 65
Trigonometric Function and Equations 219

p 3p 5p 7p Again, sin 2 q + cos 4 q = 1 - cos 2 q + cos 4 q


69. sin 4 + sin 4 + sin 4 + sin 4
8 8 8 8 = cos 4 q - cos 2 q + 1
1 éæ
2 2ù
p ö æ 3 p ö æ 1ö
2
3 3
= ê ç2 sin 2 ÷ + ç2 sin 2 ÷ ú = ç cos 2 q - ÷ + ³
4 êë è 8ø è 8 ø úû è 2ø 4 4
éæ 1 2 pö
2
æ 2 3p ö
2ù 3
+ ÷ + ç2 sin Hence, £ A £1
ê ç2 sin ÷ ú 4
êë è 4 8ø è 8 ø úû
75. Since, y = 1 + 4 sin 2 x cos 2 x
1 éæ 3p ö ù
2 2
pö æ
= ê ç1 - cos ÷ + ç1 - cos ÷ ú Þ y = 1 + sin 2 2x
4 êë è 4 ø è 4 ø úû
We know that, 0 £ sin 2 2x £ 1
1 éæ 3p ö ù
2 2
pö æ Þ 1 £ 1 + sin 2 2x £ 2
+ ê ç1 - cos ÷ + ç1 - cos ÷ ú
4 êë è 4ø è 4 ø úû Þ 1 £ y£2
p p
1 éæ 1 ö
2
æ 1 ö ù 1 éæ
2
1 ö
2
æ 1 ö ù
2
76. cos 2 æç - xö÷ - cos 2 æç + xö÷
= ê ç1 - ÷ + ç1 + ÷ ú + ê ç1 - ÷ + ç1 + ÷ ú è3 ø è3 ø
4 êë è 2ø è 2 ø úû 4 êë è 2ø è 2 ø úû
é æp ö æp öù é æp ö æp öù
1 1 3 = ê cos ç - x÷ + cos ç + x÷ ú ê cos ç - x÷ - cos ç + x÷ ú
= (3) + (3) = ë è 3 ø è 3 ø ûë è 3 ø è 3 øû
4 4 2
æ p öæ p ö
= ç2 cos cos x÷ ç2 sin sin x÷
70. Given, a cos 2q + b sin 2q = c è 3 øè 3 ø
æ 1 - tan 2 q ö æ 2 tan q ö 2p 3
\ aç 2
÷ + bç 2 ÷
=c = sin sin 2x = sin 2x
è 1 + tan q ø è 1 + tan q ø 3 2
Þ a - a tan 2 q + 2b tan q = c (1 + tan 2 q ) 3
Hence, maximum value of given expression is .
Þ tan 2 q (a + c) - 2b tan q + c - a = 0 2
Since, a and b are the roots of the above equation. pö pö
2b 77. We have, sin æç x + æ
÷ + cos ç x + ÷
\ tan a + tan b = è 6ø è 6ø
a+c
é 1 æ pö 1 æ p öù
= 2ê sin ç x + ÷+ cos ç x + ÷ ú
2 2 2 2
71. cos A (3 - 4 cos A ) + sin A (3 - 4 sin A ) 2 2
ë 2 è 6 ø 2 è 6 øû
= (3 cos A - 4 cos3 A )2 + (3 sin A - 4 sin3 A )2 é p pù
= 2 cos ê x + -
= (- cos 3 A )2 + (sin 3 A )2 = 1 ë 6 4 úû
72. tan a tan 2 a K tan (2 n - 2) a tan (2 n - 1) a æ pö
= 2 cos ç x - ÷
è 12 ø
= {tan a tan (2 n - 1) a} {tan 2 a tan (2 n - 2) a}
p
K {tan (n - 1) a tan (n + 1) a} tan na Hence, maximum value will be at x = .
12
ì æ p öü ì æp öü p 1
= í tan a tan ç - a ÷ý í tan 2 a tan ç - 2 a ÷ý …tan 78. Let f (x) = sin x cos x = sin 2x
î è 2 ø þî è 2 ø þ 4 2
æ pö 1 1 1
çQ na = ÷ We know that, - 1 £ sin 2 x £ 1 Þ -
£ sin 2x £
è 4ø 2 2 2
=1 1 1
Thus, the greatest and least value of f (x) are and -
1 2 2
73. Q sin A sin B = ´ 2 sin A sin B
2 respectively.
1 79. Maximum value of 4 sin 2 x + 3 cos 2 x i. e. , sin 2 x + 3 is 4
= [cos ( A - B ) - cos ( A + B )]
2 x x 1 1
and that of sin + cos is + = 2, both attained
1 2 2 2 2
= [cos ( A - B ) - cos 90° ]
2 p
at x = . Hence, the given function has maximum value
[Q A + B + C = 180° and ÐC = 90° , given] 2
1 1 4 + 2.
= cos ( A - B ) £
2 2 80. cos 2 q + 2 cos q = 2 cos 2 q - 1 + 2 cos q
1
\ Maximum value of sin A sin B = æ 1ö
2
3
2 = 2 ç cos q + ÷ -
è 2ø 2
74. We have, A = sin 2 q + cos 4 q
3 é æ 1ö
2 ù
= sin 2 q + cos 2 q cos 2 q ³- êQ 2 ç cos q + ÷ ³ 0, " qú
2 êë è 2ø úû
£ sin 2 q + cos 2 q (since, cos 2 q £ 1 )
2 4
Þ sin q + cos q £ 1 Þ A £ 1 Then, maximum value of cos 2 q + 2 cos q is 3.
220 JEE Main Mathematics

81. Since, (cot a1 ) (cot a 2) K (cot a n ) = 1 5p 3x


Let - =y
4 2
Þ (cos a1 ) (cos a 2) K (cos a n )
= (sin a1 ) (sin a 2) K (sin a n ) and y = tan x
5p 3x
Þ cos 2 a1 cos 2 a 2 ... cos 2 a n \ Graph of y = - and y = tan x meet exactly three
4 2
sin 2 a1 sin 2 a 2 K sin 2 a n
= times in [0, 2p ]. Thus, the number of solutions of given
2n equation is 3.
Þ cos a1 cos a 2 K cos a n
85. Given ( 3 - 1 ) cos q + ( 3 + 1 ) sin q = 2 …(i)
1/ 2
æ sin 2 a1 sin 2 a 2 K sin 2 a n ö
=ç ÷ Let ( 3 - 1 ) = r cos a and ( 3 + 1 ) = r sin a
è 2n ø
\ r 2(cos 2 + sin 2 a ) = ( 3 - 1 )2 + ( 3 + 1 )2
Since, maximum value of sin a = 1
Þ cos 2 a r 2 = 3 + 1 - 2 3 + 3 + 1 + 2 3
1
\ Maximum value of cos a1 K cos a n = Þ r2 = 8 Þ r = 2 2
2n/ 2
r sin a 3+1 3+1
82. Given that, And = =-
r cos a 3 -1 (1 - 3 )
p p
tan q = 3 = tan Þ q = np + æp pö
3 3 = - tan ç + ÷
è6 4ø
For -p < q < 0 put n = - 1, we get
p -2 p -4p æ 7p ö
q = -p + = or Þ tan a = - tan ç ÷
è 12 ø
3 3 6
5p
83. Q cos x = 1 - sin 2x Þ a=-
12
Þ cos x = |sin x - cos x|
\From Eq. (i) becomes,
There are two cases arise. r(cos a cos q + sin a sin q) = 2
Case I sin x £ cos x
Þ 2 2 cos(a - q) = 2
Þ cos x = cos x - sin x 1
Þ sin x = 0 Þ cos(q - a ) =
2
é p ö æ 5p ù p
where, x Î ê0, ÷ È ç , 2p ú Þ q - a = 2 np ±
ë 4ø è 4 û 4
p 5p
Þ x = 2p, neglecting x = p Þ q = 2 np ± -
4 12
Case II sin x > cos x 86. 3 sin 2 x - 7 sin x + 2 = 0
Þ tan x = 2 Þ 3 sin 2 x - 6 sin x - sin x + 2 = 0
æ p 5p ö Þ 3 sin x (sin x - 2) - 1 (sin x - 2) = 0
where, x Îç , ÷
è4 4 ø Þ (3 sin x - 1 ) (sin x - 2) = 0
Q tan x = 2 1
Þ sin x = or 2
Þ x = tan -1 (2) 3
Thus, the given equation has two solutions. 1
Þ sin x = ( Q sin x ¹ 2)
5p 3
84. We have, 3x + 2 tan x = 1 p
2 Let sin -1 = a ,0 < a <
3 2
Y
Then, a , p - a , 2p + a , 3 p - a , 4p + a , 5p - a are the
y = 5p – 3x y = tan x solutions in [0, 5p ].
4 2
\ Required number of solutions = 6
X¢ X x
O p p 3p 2p 87. Since, 1 + sin x sin 2 =0
2 2 2
æ 1 - cos x ö
\ 1 + sin x ç ÷ =0
è 2 ø
Þ 2 + sin x - sin x cos x = 0

Þ sin 2x - 2 sin x = 4
5p 3x
Þ tan x = - which is not possible for any x in [-p , p ].
4 2
Trigonometric Function and Equations 221

1 91. Given, esin x - e- sin x - 4 = 0


88. Now, 1 + |cos x| + cos 2 x + |cos3 x| + . . . ¥ =
1 - |cos x|
1 Þ e2sin x - 4esin x - 1 = 0
1 - | cos x |
\ 8 = 43 4 ± 16 + 4
3 Þ esin x = =2 + 5
1 - |cos x | 2
Þ 2 = 26
Þ 1 = 2 - 2|cos x|
Þ sin x = log (2 + 5 ) [Q log (2 - 5 ) is not defined]
1
Þ |cos x| = Since, 2 + 5 > e Þ log (2 + 5 ) > 1
2
1 Þ sin x > 1, which is not possible.
Þ cos x = ±
2 Hence, no solution exist.
p p 2p 2p
Þ x= ,- , ,- p
3 3 3 3 92. x + 2 tan x =
2
\ Number of solutions = 4
p
Þ 2 tan x = - x
89. We have, sin x - sin 2x + sin 3x = 0 2
Þ (sin x + sin 3x) - sin 2x = 0 1 p
Þ tan x = - x +
æ x + 3x ö æ x - 3x ö 2 4
Þ 2 sin ç ÷ cos ç ÷ - sin 2x = 0
è 2 ø è 2 ø Y y=tan x

é æC + Dö æC - Döù
ê Q sin C + sin D = 2 sin çè 2 ÷ø cos çè 2 ÷ø ú
ë û
Þ 2 sin 2x cos x - sin 2x = 0 [Q cos (- q) = cos q] X
Þ sin 2x(2 cos x - 1) = 0
Þ sin 2x = 0 or 2 cos x - 1 = 0
1
Þ 2x = 0, p, ... or cos x = x=2p
2 p x= p 3p
x= x= 1 p
p p 2 2 y= - x -
Þ x = 0, ... or x = 2 4
2 3
Number of solutions of the given equation is ‘3’.
é pö
In the interval ê0, ÷ only two values satisfy, namely
ë 2ø 93. 2 sin x = 5x2 + 2x + 3
p
x = 0 and x = . Þ 2 sin x = 4x2 + (x + 1)2 + 2
3
p But 2 sin x £ 2
æ æp ö 1ö
90. We have, 8 cos xç cos æç ö
+ x÷ cos ç - x÷ - ÷ = 1
è è6 ø è6 ø 2ø and 4x + (x + 1 )2 + 2 > 2 , so it has no solution.
2

æ p 1ö 94. Q sec x - 1 = ( 2 - 1 ) tan x


Þ 8 cos xç cos 2 - sin 2 x - ÷ = 1
è 6 2ø Þ 1 - cos x = ( 2 - 1 ) sin x
æ3 1ö xì x xü
Þ 8 cos xç - sin 2 x - ÷ = 1 Þ sin ísin - ( 2 - 1 ) cos ý = 0
è4 2ø 2î 2 2þ
æ3 1 ö x x p
Þ 8 cos xç - - 1 + cos 2 x÷ = 1 Þ sin = 0 or tan = 2 - 1 = tan
è4 2 ø 2 2 8
æ -3 + 4 cos 2 x ö x x p
Þ 8 cos xç ÷ =1 Þ = np or = np +
è 4 ø 2 2 8
p
Þ 2(4 cos3 x - 3 cos x) = 1 \ x = 2 np , 2 np +
4
1
Þ 2 cos 3x = 1 Þ cos 3x = 95. We have, (5 + 4 cos q )(2 cos q + 1 ) = 0
2
p 5p 7p 5
Þ 3x = , , [0 £ 3x £ 3p] Þ cos q = - which is not possible.
3 3 3 4
p 5p 7p 1
Þ x= , , \ 2 cos q + 1 = 0 Þ cos q = -
9 9 9 2
p 5p 7p 13p 13p 2p 4p
Sum = + + = Þ kp = Þ q=
,
9 9 9 9 9 3 3
k=
13 ì 2p 4p ü
Hence, \ Solution set is í , ý Î [0, 2p ].
9 î 3 3þ
222 JEE Main Mathematics

96. Since, 4 cos q - 3 sec q = 2 tan q But sin q ¹ -2


3 sin q p
Þ 4 cos q - =2 \ sin q = 1 = sin
cos q cos q 2
p
Þ 4 cos 2 q - 3 = 2 sin q Þ q = np + (-1 )n
2
Þ 4 - 4 sin 2 q - 3 = 2 sin q
100. The given equation can be rewritten as
Þ 4 sin 2 q + 2 sin q - 1 = 0
tan q (sin q + 3 ) = 0
-1 ± 5 Þ tan q = 0 but sin q + 3 ¹ 0 Þ tan q = 0
Þ sin q =
4
Þ q = np , n Î I
-1 + 5 -1 - 5
Either sin q = or sin q =
4 4 101. We have,
p æ 3p ö sin x - 3 sin 2x + sin 3x = cos x - 3 cos 2x + cos 3x
Þ sin q = sin or sin q = sin ç - ÷
10 è 10 ø Þ sin x + sin 3x - 3 sin 2x = cos x + cos 3x - 3 cos 2x
p æ 3p ö Þ 2 sin 2x cos x - 3 sin 2x - 2 cos 2x cos x + 3 cos 2x = 0
Þ q = np + (-1 )n or q = np + (-1 )n ç - ÷
10 è 10 ø Þ sin 2x(2 cos x - 3) - cos 2x(2 cos x - 3) = 0
3x x
97. We have, cos 3x + cos 2x = sin + sin Þ (sin 2x - cos 2x)(2 cos x - 3) = 0
2 2
æ 3ö
5x x x Þ sin 2 x = cos 2x çQ cos x ¹ ÷
Þ 2 cos cos = 2 sin x cos è 2ø
2 2 2
x x p æp ö
Either cos = 0 Þ = (2 n + 1 ) Þ 2x = 2 np ± ç - 2x÷
2 2 2 è2 ø
5x np p
Þ x = (2 n + 1 )p or cos = sin x Taking +ve sign x= +
2 2 8
5x æp ö 3 sin 2 x + 10 cos x - 6 = 0
Þ cos = cos ç - x÷ 102. We have,
2 è2 ø
Þ (cos x - 3)(3 cos x - 1) = 0
5x æp ö
Þ = 2 np ± ç - x÷ 1
2 è2 ø Þ cos x ¹ 3 or cos x =
3
7x p 4np p
Taking the +ve sign = 2 np + Þ x = + -1 æ 1 ö
2 2 7 7 Þ x = 2 np ± cos ç ÷
è3ø
Taking –ve sign
3x p 4 np p Y y = tan x
= 2 np - Þ x= - 103. y=x
2 2 3 3
For 0 £ x £ 2p
p 5p 9p 13p
x= , , , ,p
7 7 7 7
Thus, number of solutions = 5 –p/2 -p/4 p/2
X¢ p X
–p 3p/2
æ 3p ö p/4
98. Since, tan q + tan ç + q÷ = 2
è 4 ø
-1 + tan q
\ tan q + =2
1 + tan q
Þ tan q + tan 2 q - 1 + tan q = 2 + 2 tan q

Þ tan 2 q = 3
It is clear from the figure that, the smallest positive root
2 2 p 2 æ 3pö
Þ tan q = ( 3 ) = tan of a given equation is in ç p , ÷.
3 è 2 ø
p
Þ q = np ± , n Î I 104. (1 + 2 sin q )2 + ( 3 tan q - 1 )2 = 0
3
Þ 1 + 2 sin q = 0
99. We have, sin 2 q + sin q - 2 = 0
and 3 tan q - 1 = 0
Þ (sin q - 1 )(sin q + 2) = 0 1 æ pö
\ sin q = - Þ q = mp + (-1)m ç - ÷
Þ sin q = 1, sin q = - 2 2 è 6ø
Trigonometric Function and Equations 223

1 p 5x x
and tan q = Þ q = mp + Þ cos x = 0 or cos = 0 or cos = 0
3 6 2 2
For common values, m must be odd. p 3p
Now, cos x = 0 Þ x = , [Q 0 £ x < 2p]
7p 2 2
i.e., m = 2 n + 1 Þ q = 2 np + 5x 5x p 3p 5p 7p 9p 11p
6 cos =0Þ = , , , , ...,
2 2 2 2 2 2 2 2
105. Q sin 3x = 3 sin x - 4 sin3 x
p 3p 7p 9p
1 Þ x= , , p, , [Q 0 £ x < 2p]
\ sin3 x = (3 sin x - sin 3x ) 5 5 5 5
4 x x p 3p 5p
and cos = 0 Þ = , , , ...
and cos 3x = 4 cos3 x - 3 cos x 2 2 2 2 2
1 Þ x=p [Q 0 £ x < 2p]
Þ cos3 x = (cos 3x + 3 cos x )
4 p 3p p 3p 7p 9p
Hence, x= , , p, , , ,
\ cos 3x cos3 x + sin 3x sin3 x 2 2 5 5 5 5
1 108. Q tan q + tan 2 q + 3 tan q tan 2 q = 3
=[cos 2 3x + 3 cos x cos 3x + 3 sin x sin 3x - sin 2 3x]
4
Þ tan q + tan 2 q = 3 (1 - tan q tan 2 q)
1
= [3 cos 2x + cos 6x] = cos3 2x tan q + tan 2 q p
4 Þ = 3 Þ tan 3 q = tan
p 1 - tan q tan 2 q 3
Þ cos 2x = 0 Þ 2x = (2 n + 1 ) p (3n + 1 )p
2 Þ 3 q = np + Þ q= , n ÎI
p 3 9
Þ x = (2 n + 1 )
4 109. Given, sin 4 x + cos 4 x = sin x × cos x
3
106. Given, sin 2 2 q + cos 4 2 q = Þ (sin 2 x + cos 2 x)2 - 2 sin 2 x × cos 2 x = sin x × cos x
4
sin 2 2x sin 2x
2 4 3 Þ 1- =
Þ (1 - cos 2 q) + cos 2 q = (Q sin 2 x = 1 - cos 2 x) 2 2
4
Þ sin 2 2x + sin 2x - 2 = 0
Þ 4 cos 4 2 q - 4 cos 2 2 q + 1 = 0
Þ (2 cos 2 2 q - 1)2 = 0 Þ 2 cos 2 2 q - 1 = 0 Þ (sin 2x + 2)(sin 2x - 1 ) = 0
1 1 Þ sin 2x = 1 (Q sin 2x ³ - 1)
Þ cos 2 2 q = Þ cos 2 q = ±
2 2 p
\ 2x = (4n + 1 )
æ pö 2
If q Î ç0, ÷ , then 2 q Î (0, p ) p p 5p
è 2ø Þ x = (4n + 1) Þ x = ,
4 4 4
1
\ cos 2 q = ± Hence, two solutions exist.
2
1
p 3p 110. Since, cos 2x, , sin 2x are in AP.
Þ 2q= , , 2
4 4
Þ cos 2x + sin 2x = 1
é æ 3p ö æ pö p 1 ù
êQ cos çè 4 ÷ø = cos çè p - 4 ÷ø = - cos 4 = - 2 ú Þ sin 2x = 1 - cos 2x = 2 sin 2 x
ë û
p 3p Þ 2 sin x (cos x - sin x) = 0
Þ q= ,
8 8 Þ sin x = 0 or cos x - sin x = 0
p 3p p Þ x = np or tan x = 1
Sum of values of q = + =
8 8 2 p
Þ x = np or x = np +
107. Given equation is 4
cos x + cos 2x + cos 3x + cos 4x = 0 p
Thus, required values of x are np and np + .
Þ (cos x + cos 3x) + (cos 2x + cos 4x) = 0 4
Þ 2 cos 2x cos x + 2 cos 3x cos x = 0 111. |sin x|> 2 sin 2 x
Þ 2 cos x (cos 2x + cos 3x) = 0 Þ |sin x|(2 |sin x| - 1) < 0
æ 5x xö 1
Þ 2 cos x ç2 cos cos ÷ = 0 Þ 0 <|sin x| <
è 2 2ø 2
5x x æ p ö æ 5p ö æ 7p ö æ 11p ö
Þ cos x × cos × cos = 0 Þ x Î ç0, ÷ È ç , p÷ È çp, ÷ Èç , 2p ÷
2 2 è 6ø è 6 ø è 6 ø è 6 ø
224 JEE Main Mathematics

112. We have, sin10 2x = 1 + cos10 x Þ tan x = 1


Minimum value of RHS = 1 p
Þ x = mp +
and maximum value of LHS = 1. 4
Therefore, solution is possible only when sin10 2x = 1 p
Þ x = (4m + 1 )
and cos10 x = 0. But this is not possible. 4
Therefore, it has no solution.
Round II
113. Given equation, sin x + sin y + sin z = - 3 is satisfied
1. From the given equations, we have S tan a = p
3p
only when x = y = z = , for x, y, z Î [0, 2p ] . S tan a tan b = 0
2
sin x - cos x and tan a tan b tan g = r
114. Q sin5 x - cos5 x =
sin x cos x \ (1 + tan 2 a ) (1 + tan 2 b ) (1 + tan 2 g )
é sin5 x - cos5 x ù = 1 + S tan 2 a + S tan 2 a tan 2 b + tan 2 a tan 2 b tan 2 g
Þ sin x cos x ê ú=1 = 1 + (S tan a )2 - 2S tan a tan b + (S tan a tan b )2
ë sin x - cos x û
1 - 2 tan a tan b tan g S tan a + tan 2 a tan 2 b tan 2 g
Þ sin 2x [sin 4 x + sin3 x cos x + sin 2 x cos 2 x
2 = 1 + p2 - 2 pr + r 2 = 1 + ( p - r )2
+ sin x cos 2 x + cos 4 x] = 1 1 + tan q æ 3 tan q - tan3 q ö
2 2 2 2 2 2. =3 ç ÷
Þ sin 2x [(sin x + cos x ) - 2 sin x cos x 1 - tan q 2
è 1 - 3 tan q ø
+ sin x cos x (sin 2 x + cos 2 x) + sin 2 x cos 2 x] = 2
On simplification, we get
Þ sin 2x [1 - sin 2 x cos 2 x + sin x cos x] = 2
3 tan 4 q - 6 tan 2 q + 8 tan q - 1 = 0
Þ sin3 2x - 2 sin 2 2x - 4 sin 2x + 8 = 0 1
\ Product of roots = tan a × tan b × tan g × tan d = -
Þ (sin 2x - 2)2(sin 2x + 2) = 0 3
Þ sin 2x = ± 2 , which is not possible for any x . 3. cosec2x + 25 sec2 x = 26 + cot2 x + 25 tan 2 x
2
1 æ 1ö 1 1 = 26 + 10 + (cot x - 5 tan x )2 ³ 36
115. sin 2 x - sin x + = çsin x - ÷ + ³ , "x
2 è 2ø 4 2 4. Equation first can be written as
2 sec2 y
and sec y ³ 1," y, so 2 ³2. x sin a + y ´ 2 sin a cos a + z ´ sin a (3 - 4 sin 2 a )
Hence, the above inequality holds only for those values = 2 ´ 2 sin a cos a cos 2 a
1 2
of x and y for which sin x = and sec2 y = 1. Þ x + 2 y cos a + z (3 + 4 cos a - 4)
2 = 4 cos a (2 cos 2 a - 1 ) as sin a ¹ 0
p 5p 13p 17p
Hence, x = , , , Þ 8 cos3 a - 4z cos 2 a - (2 y + 4) cos a + (z - x) = 0
6 6 6 6
æ zö æ y + 2ö æ z - xö
and y = 0, p , 2p , 3p . Þ cos3 a - ç ÷ cos 2 a - ç ÷ cos a + ç ÷ =0
è2ø è 4 ø è 8 ø
Hence, required number of ordered pairs are 16.
which shows that cos a is a root of the equation
x
116. Since, 2 cos 2 sin 2 x< 2 æ zö æ y + 2ö æ z - xö
2 t3 - ç ÷ t 2 - ç ÷t+ ç ÷ =0
è2ø è 4 ø è 8 ø
1
But x2 + 2 ³ 2
x Similarly, from second and third equation we can verify
Thus, the equation has no solution. that cos b and cos c are the roots of the given equation.

117. esin x + ecos x = 2e1/ 2


…(i) 5. 2 + 2 + 2 + K + 2 (1 + cos q) [n numbers of 2’s]
sin x cos x
Þ e >0 and e >0
æ qö
= 2 + 2 + 2 + K + ç2 + 2 cos ÷
\ esin x + ecos x ³ 2 esin x + cos x [Q AM ³ GM] è 2ø

Þ esin x + ecos x ³ 2 e1/ 2


…(ii) [(n - 1 ) numbers of 2’s]
… … … … …
Since, equality holds … … … … …
Þ esin x = ecos x = 2 + 2 cos (q / 2n - 1 )
Þ sin x = cos x = 2 { 1 + 2 cos 2(q / 2n ) - 1 } = 2 cos(q / 2n )
Trigonometric Function and Equations 225

¥
6. It is given that x = S (-1)n tan 2x q é1 1 ù é 1ù
= êë2 + cos 20º + cos 100º - 2 - cos 40º úû êëQ cos 60º = 2 úû
n =` 0 2
= 1 - tan 2 q + tan 4 q - tan 6 q + ....+ upto ¥ 1 é3 ù
= ê + (cos 20º - cos 40º ) + cos 100º ú
1 æ pö 2 ë2 û
= {Q q Î ç0, ÷
1 + tan 2 q è 4ø 1 é3 20º + 40º 20°- 40° ù
= - 2 sin sin + cos 100° ú
Þ 2
tan q Î (0, 1) } 2 êë 2 2 2 û
x = cos 2 q …(i) é C+D C - Dù
¥ êëQ cos C - cos D = -2 sin 2 sin 2 úû
and y = S cos 2n q
n =0 é3
1 ù
= êë 2 - 2 sin 30º sin(-10º ) + cos(90º +10º )úû
2 4 6 2
= 1 + cos q + cos q + cos q + K + upto ¥
1 æ pö 1 é3
= ê + sin 10º - sin 10º ú
ù
[Q cos (90º + q) = - sin q]
= {Q q Î ç0, ÷
1 - cos 2 q è 4ø 2 ë2 û
æ 1ö 1 3 3
Þ cos 2 q Î ç0, ÷ } = ´ =
è 2ø 2 2 4
1 1 9. The expression, cos 4 q + sin 4 q
= Þ sin 2 q = …(ii)
sin 2 q y = (cos 2 q + sin 2 q)2 - 2 sin 2 q cos 2 q
On adding Eqs. (i) and (ii), we get 1
1 = 1 - sin 2(2 q)
1 =x+ 2
y Q sin 2(2 q) Î [0, 1]
Þ y(1 - x) = 1 1 é 1 ù
Þ - sin 2(2 q) Î ê - , 0ú
Hence, option (b) is correct. 2 ë 2 û
7. By applying AM ³ GM inequality, on the numbers 1 é1 ù
Þ 1- sin 2(2 q) Î ê , 1ú
sin 4 a , 4 cos 4 b , 1 and 1, we get 2 ë2 û
sin 4 a + 4 cos 4 b + 2 Now, as cos 4 q + sin 4 q + l = 0
³ ((sin 4 a ) (4 cos 4 b ) × 1 × 1)1/ 4
4 Þ l = - (cos 4 q + sin 4 q)
4 4
Þ sin a + 4 cos b + 2 ³ 4 2 sin a cos b Þ l Î[-1, - 1 / 2] for real solution of the given equation
But, it is given that cos 4 q + sin 4 q + l = 0 for q.
sin 4 a + 4 cos 4 b + 2 = 4 2 sin a cos b Hence, option (b) is correct.
So, sin 4 a = 4 cos 4 b = 1 26 13
10. Q tan a/ 2 + tan b/ 2 = =
[Q in AM ³ GM , equality holds when all given positive 8 4
quantities are equal.] 15
1 and tan a/ 2 tan b/ 2 =
Þ sin a = 1 and sin b = …(i) 8
2 æ a + bö tan a/ 2 + tan b/ 2
\ tan ç ÷=
[Q a , b Î [0, p ]] è 2 ø 1 - tan a/ 2 tan b/ 2
Now, cos (a + b ) - cos (a - b ) 13
= -2 sin a sin b 26
= 4 =-
é C+D D -C ù 15 7
êëQ cos C - cos D = 2 sin 2 sin 2 úû 1-
8

= -2 ´ 1 ´
1 æ a + bö
[from Eq. (i)] 1 - tan 2ç ÷
2 è 2 ø
Now, cos (a + b ) =
=- 2 æ a + bö
1 + tan 2ç ÷
è 2 ø
8. We have, cos 2 10º - cos 10º cos 50º + cos 2 50º
2
1 æ 26 ö
= [2 cos 2 10º -2 cos 10º cos 50º +2 cos 2 50º ] 1 - ç- ÷
2 è 7ø
= 2
1 æ 26 ö
= [1 + cos 20º - (cos 60º + cos 40º ) + 1 + cos 100º ] 1 + ç- ÷
2 è 7ø
[Q 2 cos 2 A = 1 + cos 2 A and 49 - 676 627
= =-
2 cos A cos B = cos( A + B) + cos( A - B)] 49 + 676 725
226 JEE Main Mathematics

n
Þ (sin 2 x - 1 )2 + (cos 2 y - 1 )2 + 2 (sin x - cos y)2 = 0
11. Given, sin n q = å br sin r q = b0 + b1 sin q + b2 sin 2 q
r=0 which is possible only when
n
+ K + bn sin q …(i) sin 2 x - 1 = 0, cos 2 y - 1 = 0, sin x - cos y = 0
Putting q = 0 in Eq. (i), we get 0 = b0
n Þ sin 2 x = 1, cos 2 y = 1, sin x = cos y
Again, Eq. (i) can be written as sin n q = å br sin r q
As 0 £ x, y £
p
r=0 2
n
sin n q
sin q
= å br sin r - 1 q We get sin x = cos y = 1
r =1 \ sin x + cos y = 1 + 1 = 2
On taking limit as q ® 0, we get
sin n q 15. cos 4 x - (l + 2) cos 2 x - (l + 3) = 0
lim = b1
q ® 0 sin q Þ (cos 2 x )2 - (l + 2) cos 2 x - (l + 3) = 0
æ sin nq ö æ q ö
Þ lim n ç ÷ç ÷ = b1 (l + 2) ± (l + 2)2 + 4(l + 3)
q®0 è nq ø è sin q ø \ cos 2 x =
2
Þ n = b1 (l + 2) ± (l + 4)
= = l + 3, - 1
Hence, b0 = 0; b1 = n 2
1 Þ cos 2 x = l + 3 (Q cos 2 x ¹ - 1 )
12. Given, tan q =
7 But 0 £ cos 2 x £ 1
cosec2 q - sec2 q (1 + cot2 q ) - (1 + tan 2 q ) Þ 0 £ l + 3 £ 1 Þ -3 £ l £ - 2
\ =
cosec2 q + sec2 q (1 + cot2 q ) + (1 + tan 2 q )
16. Q cos (315 p + x) = (-1 )315 cos x = - cos x
(cot2 q - tan 2 q)
= \ 4 cos3 x - 4 cos 2 x - cos (315p + x ) = 1
2 + tan 2 q + cot2 q
Þ 4 cos3 x - 4 cos 2 x + cos x - 1 = 0
1
7- Þ (4 cos 2 x + 1 )(cos x - 1 ) = 0
= 7 = 48 = 3
1 Þ cos x = 1, 4 cos 2 x + 1 ¹ 0
2 + + 7 64 4
7 Þ cos x = cos 0 Þ x = 2 np , n Î I
2
q
13. ( 3 )sec = (tan 2 q + 1 )2 - 1 \ x = 2p , 4p , 6p , 8p , . . . , 100p (Q 0 < x < 315)
2
= (sec q ) - 1 2
(i.e. 100p < 315 < 101p )
2
Put sec q = x (Q x ³ 1 ) Required arithmetic mean
Then, ( 3 ) x = x2 - 1 2p + 4p + 6p + 8p + . . . + 100p
=
Let y = ( 3 )x = (x2 - 1 ) (x > 1 ) 50
50
x
Now, graphs of y = ( 3 ) and y = x - 1 intersect at one 2 2p × × 51
2p (1 + 2 + 3 + 4 + . . . + 50) 2
point = = = 51p
50 50
Y y = ( 3)x 1
17. Since, cos 2 q = sin q × tan q
6
1 y = x2 – 1 Þ 6 cos3 q + cos 2 q - 1 = 0
1
X¢ X As cos q = satisfied the equation.
O 1 2

_1
\ (2 cos q - 1 )(3 cos 2 q + 2 cos q + 1 ) = 0
1
Þ cos q = (other values of cos q are imaginary)
Y¢ 2
i.e., x = 2, then y = 3 p
\ q = 2 np ± , n Î I
Then, sec2 q = 2 3
Þ sec q = ± 2 18. Given, cos (a - b) = cos 0
æ p pö a - b = 2np , n Î I Þ a - b = - 2p , 0, 2p
Therefore, there are two values of q in ç - , ÷ . \
è 2 2ø
\ a + b = 2p + 2 a , 2 a , 2a - 2p
14. The given equation is 1
sin 4 x + cos 4 y + 2 = 4 sin x cos y Þ cos (a + b) = cos 2a , cos 2a , cos 2a =
e
Þ (sin 2 x - 1 )2 + (cos 2 y - 1 )2 + 2 sin 2 x 1
y = cos 2a =
+ 2 cos 2 y - 4 sin x cos y = 0 e
Trigonometric Function and Equations 227

Y p p p p
\ cos × cos 3 …cos 10 × sin 10
y = 1/e 22 2 2 2
ì æ p 9öü

O
X ïï sin çè 210 2 ÷ø ïï p p
=í ý sin 10 [ Q here, a = 10 and n = 9 ]
p
y = cos 2a ï 29 sin æç öï
÷
2 2
ïî è 210 ø ïþ
Y¢ 1 æpö 1 1
= 9 sin ç ÷ = 9 =
Hence, number of solutions is 4. 2 è2ø 2 512
19. Q x3 + x2 + 4x + 2 sin x = 0 23. Given equation is y = sin x sin(x + 2) - sin 2(x + 1)
Þ 3 2
x + (x + 2) + 2 sin x = 4 1 1
= [cos 2 - cos(2x + 2)] - [1 - cos(2x + 2)]
x = 0, satisfies this equation. 2 2
Now, in 0 < x £ p , x3 + (x + 2)2 + 2 sin x > 4 [Q2 sin A sin B = cos( A - B) - cos( A + B) and
cos 2 q = 1 - 2 sin 2 q
and in p<x £ 2p , x3 + (x + 2)2 + 2 sin x > 27 + 25 - 2 = 50 Þ 2 sin 2 q = 1 - cos 2 q]
Hence, x = 0 is the only solution. 1 1 1 1
= cos 2 - cos(2x + 2) - + cos(2x + 2)
20. Q 2 cos x ,|cos x|, 1 - 3 cos 2 x are in GP. 2 2 2 2
1 1
\ cos 2 x = 2 cos x × (1 - 3 cos 2 x ) = (cos(2) - 1) = - (2 sin 2(1))
2 2
Þ 6 cos3 x + cos 2 x - 2 cos x = 0 = - sin 2(1) < 0 Þ y < 0
1 2 and as we know that y < 0, is in third and fourth
\ cos x = 0,
,-
2 3 quadrants only.
p p -1 æ 2 ö 24. The given trigonometric equation is
\ x = , , cos ç - ÷ (Q a , b are positive)
2 3 è 3ø cos 2x + a sin x = 2a - 7
p p Þ 1 - 2 sin 2x + a sin x = 2a - 7
If a = , b =
2 3 [Q cos 2x = 1 - 2 sin 2 x]
p Þ 2 sin 2 x - a sin x + 2a - 8 = 0
Then, |a - b | =
6 Þ 2(sin 2 x - 4) - a (sin x - 2) = 0
21. Given, 5 (tan 2 x - cos 2 x) = 2 cos 2x + 9 Þ 2 (sin 2 x - 2) (sin x + 2) - a (sin x - 2) = 0
æ 1 - cos 2x 1 + cos 2x ö Þ (sin x -2) (2 sin x + 4 - a ) = 0
Þ 5ç - ÷ = 2 cos 2x + 9
è 1 + cos 2x 2 ø Þ 2 sin x + 4 - a = 0 [Q sin x + 2 ¹ 0]
a -4
Put cos 2x = y, we have Þ sin x = …(i)
2
æ 1 - y 1 + yö
5ç - ÷ = 2y + 9 Now, as we know -1 £ sin x £ 1
è1 + y 2 ø a -4
\ -1 £ £1 [from Eq. (i)]
Þ 5 (2 - 2 y - 1 - y2 - 2 y) = 2(1 + y)(2 y + 9) 2
Þ 5(1 - 4 y - y2) = 2(2 y + 9 + 2 y2 + 9 y) Þ -2 £ a -4 £2
Þ 5 - 20 y - 5 y2 = 22 y + 18 + 4 y2 Þ 2 £ a £ 6 Þ a Î [2, 6]
Þ 9 y2 + 42 y + 13 = 0 9p 11p 13p 5p 3p p
25. sin sin sin = sin sin sin
2
Þ 9 y + 3 y + 39 y + 13 = 0 14 14 14 14 14 14
Þ 3 y (3 y + 1) + 13(3 y + 1) = 0 8p
sin
1 13 p 2p 3p p 2p 4p 7 =1
Þ (3 y + 1)(3 y + 13) = 0 Þ y = - , - = cos cos cos = cos cos cos =
3 3 7 7 7 7 7 7 8 sin p 8
1 13 7
\ cos 2x = - , -
3 3 Hence, (c) is the correct answer.
1 é 13 ù 26. 27cos x + 81sin x = 33 cos x + 34sin x ³ 2 × 33 cos x × 34sin x
Þ cos 2x = - êëQ cos 2x ¹ - 3 úû
3
2
= 2 × 31/ 2(3 cos x + 4sin x ) ³ 2 × 31/ 2( - 5 )
2 æ 1ö 2 7 = 2 × 3- 5/ 2
Now, cos 4x = 2 cos 2x - 1 = 2 ç - ÷ - 1 = - 1 = -
è 3ø 9 9 2
= 2 × 31/ 2 × 3- 1/ 2 =
22. We know that, 9 3
sin (2n a ) 2
cos a × cos (2a ) cos(22a )…cos (2n-1 a ) = Thus, minimum value of given expression is .
2n sin a 9 3
228 JEE Main Mathematics

1 3 1 3
27. Given that, sin a = and sin b = 30. A = sin 2 x + cos 4 x = cos 4 x - cos 2 x + +
5 5 4 4
2 4 2
Þ cos a = and cos b = æ 1ö 3
= ç cos 2 x - ÷ +
5 5 è 2ø 4
\ sin(b - a ) = sin b cos a - cos b sin a æ
2
1ö 1
æ3ö æ 2 ö æ4ö æ 1 ö 2 where, 0 £ ç cos 2 x - ÷ £
=ç ÷ç ÷-ç ÷ç ÷= è 2ø 4
è5ø è 5 ø è5ø è 5 ø 5 5
3
1 \ £ A £1
Clearly, 0 < sin(b - a ) < 4
2 4
p 3p 31. cos (a + b) = Þ a + b Î 1st quadrant
Þ 0 < (b - a) < or < (b - a ) < p 5
4 4 5
æ pö æ 3p ö sin (a - b ) = Þ (a - b ) Î 1st quadrant
Þ ( b - a ) Î ç0, ÷ or (b - a) Î ç , p÷ 13
è 4ø è 4 ø
Þ 2a = (a + b ) + (a - b )
28. Given expression is tan (a + b ) + tan (a - b )
tan A cot A sin A sin A \ tan 2a =
+ = ´ 1 - tan (a + b ) tan (a - b )
1 - cot A 1 - tan A cos A sin A - cos A
3 5
cos A cos A +
+ ´ 56
sin A cos A - sin A = 4 12 =
3 5 33
1- ×
1 ì sin3 A - cos3 A ü 4 12
= í ý
sin A - cos A î cos A sin A þ 32. (2 sin x - 1 )(sin x + 3) = 0
2 2
sin A + sin A cos A + cos A 1
= Þ sin x = (Q sin x ¹ -3)
sin A cos A 2
1 + sin A cos A p 5p 13p 17p
= = 1 + sec A cosec A \ x= , , ,
sin A cos A 6 6 6 6
æ pö 1
29. Given, 3 sin P + 4 cos Q = 6 …(i) 33. Since, cos ç x - ÷ =
è 4ø 2 2
and 4 sin Q + 3 cos P = 1 …(ii) æ pö
Þ tan ç x - ÷ = 7
On squaring and adding the Eqs. (i) and (ii), we get è 4ø
and 9 (sin 2 P + cos 2 P ) + 16(sin 2 Q + cos 2 Q ) tan x - 1
Þ = 7
+ 2 ´ 3 ´ 4 (sin P cos Q + sin Q cos P ) = 37 tan x + 1
Þ 24 {sin (P + Q )} = 37 - 25 7 + 1 - (4 + 7 )
1 \ tan x = =
\ sin (P + Q ) = 1- 7 3
2
P Q b
Since, P and Q are angles of DPQR , hence 34. Since, tan + tan = -
2 2 a
0° < P , Q < 180°
P Q c
\ P + Q = 30° or 150° and tan tan =
2 2 a
Þ R = 150° or 30° (respectively) P Q R p
Also, + + =
Hence, two cases arise here. 2 2 2 2
Case I When R = 150° P+Q p æ pö
Þ = çQ Ð R = ÷
P + Q = 30° Þ 0 < P , Q < 30° 2 4 è 2ø
1 P Q
Þ sin P < , cos Q < 1 tan + tan
2 Þ 2 2 =1
3 P Q
Þ 3 sin P + 4 cos Q < + 4 1 - tan tan
2 2 2
11 b
Þ 3 sin P + 4 cos Q < <6 -
2 Þ a =1
c
\3 sin P + 4 cos Q = 6 is not possible. 1-
a
Case II When R = 30°
\ c= a + b
Hence, R = 30° is the only possibility.
Trigonometric Function and Equations 229

a + bö æ a - bö 1
35. 2 sin æç
21
÷ cos ç ÷=- solutions and 2x Î [2p , 4p ], sin 2x = ± has four more
è 2 ø è 2 ø 65 2
solutions.
æ a + bö æ a - bö 27
2 cos ç ÷ × cos ç ÷=- \ Total number of solutions are 8.
è 2 ø è 2 ø 65
Hence, answer is 8.
On squaring and adding, we get
38. Given that,
æ a - b ö 1170
4 cos 2 ç ÷= q f
è 2 ø (65)2 a + b tan = a - b sin Þ a ³ b … (i)
2 2
æ a - b ö - 3 130 2q
Þ cos ç ÷= 1 - tan
è 2 ø 130 2 = a cos f + b
Þ cos q =
æ a - bö -3 æ p a - b 3p ö 2q a + b cos f
\ cos ç ÷= çQ < < ÷ 1 + tan
è 2 ø 130 è 2 2 2 ø 2
p Þ (b - a sec q) (b + a cos f ) = b2 - a 2
36. It is given that, for a , b Î æç0, ö÷ Þ (b - a sec q) (b + a cos f ) £ 0
è 2ø
Þ Maximum value is 0.
2 sin a 1
= \ l =0
1 + cos 2a 7
39. sin x cos 3x - a cos x sin 3x = 0 …(i)
2 sin a 1
Þ = sin x cos 3x
2|cos a| 7 Þ a=
cos x sin 3x
1 æ pö tan x
Þ tan a = Q a Î ç0, ÷ …(i) a=
7 è 2ø tan 3x
1 - cos 2 b 1 tan x(1 - 3 tan 2 x)
and = a=
2 10 3 tan x - tan 2 x
1 1 - 3 tan 2 x
Þ |sin b| = a= … (ii)
10 3 - tan 2 x
1 æ pö
Þ sin b = Q b Î ç0, ÷ For real value of x, RHS of 2nd never lies between
10 è 2ø
æ1 ö
1 ç , 3÷
\ tan b = …(ii) è3 ø
3
2 Þ Number of integral points [0, 1, 2]
2 tan b 6 3 3 Þ 3 points.
\ tan 2 b = = = =
1 - tan 2 b 1 - 1 8 4 1
40. Let f (q) =
9 sin 2 q + 3 sin q cos q + 5 cos 2 q
tan a + tan 2 b
Now, tan(a + 2 b ) = Again let g(q) = sin 2 q + 3 sin q cos q + 5 cos 2 q
1 - tan a tan 2 b
1 - cos 2q æ 1 + cos 2q ö 3
1 3 = + 5ç ÷ + sin 2q
+ 2 è 2 ø 2
7 4 4 + 21 25
= = = =1 3
æ 1 3 ö 28 - 3 25 = 3 + 2 cos 2q + sin 2q
1-ç ´ ÷ 2
è 7 4ø
9 5 1
Hence, answer is 1. \ g(q)min = - 4 + = 3 - =
4 2 2
37. Given equation, log1/ 2|sin x| = 2 - log1/ 2|cos x| 1
\ f (q) = =2
Þ - log 2|sin x| = 2 + log 2|cos x| g (q)min
Þ log 2|sin x| + log 2|cos x| + log 2 4 = 0 41. [sin x] = [1 + sin x] + [1 - cos x]
Þ log 2(4|sin x||cos x|) = 0
Þ 0 = 2 + [- cos x]
Þ 4|sin x||cos x| = 1
Þ [- cos x] = - 2
1
Þ sin 2x = ± Since, - 1 £ cos x £ 1
2
Þ - 1 £ - cos x £ 1
Q x Î [0, 2p ] Þ 2x Î [0, 4p ]
1 Þ [- cos x] ¹ - 2
\ For 2x Î [0, 2p ], sin 2x = ± has four Hence, [- cos x] = - 2 has no real solution.
2
230 JEE Main Mathematics

42. Given that, Þ 4 cos q(2 cos 2 q - 1) = 4 cos 2 q - 1


2 sin 2q 4 cos 2 q - 1 3 - tan 2 q
f (nq) = Þ 4 cos q = =
cos 2q - cos 4nq 2 cos 2 q - 1 1 - tan 2 q
2 sin 2q p
= 3 - tan 2
2 sin(2n + 1) q sin(2n - 1)q p 7
Þ 4 cos =
sin(2n + 1) q - (2n - 1)q 7 1 - tan 2 p
= 7
sin(2n + 1)q sin(2n - 1)q
Hence, l = 4
sin(2n + 1)q cos(2n - 1)q - cos(2n + 1)q sin(2n - 1)q
= 1
sin(2n + 1)q sin(2n - 1)q 44. Here, log|sin x ||cos x| + =2
log|sin x ||cos x|
= cot(2n - 1)q - cot(2n + 1)q
1
Þ f (q) + f (2q) + f (3q) + ¼ + f (nq) Þ y+ = 2, where y = log|sin x ||cos x|
y
= (cot q - cot 3q) + (cot 3q - cot 5q) + (cot 5q - cot 7q)
Þ y = 1 Þ log|sin x ||cos x| = 1
+ ¼ + cot(2n - 1)q - cot(2n + 1)q
Þ |sin x| = |cos x|
cos q cos(2n + 1)q
= cot q - cot(2n + 1)q = - Þ |tan x| = 1
sin q sin(2n + 1)q
cos 2 q - 1
sin 2nq 45. Given that, =
= cos 2 q + cos q
sin q sin(2n + 1)q
Þ ( y - 1) cos 2 q + y cos q + 1 = 0
Þ m = 2n + 1 Þ l = 2n
1
Þ m - l =1 Þ cos q + - 1 and cos q =
1- y
p
43. Let q = Þ 3q = p - 4q 1
7 Þ -1 < <1
1- y
Þ sin 3q = sin 4q
Þ 3 sin q - 4 sin3 q = 4 sin q cos q cos 2q Þ y < 0 and y > 2
Þ 3 - 4 sin 2 q = 4 cos q(2 cos 2 q - 1) Þ y Î R - [0, 2]
Þ 8 cos3 q - cos q = 4 cos 2 q - 1 \ b - a =2
10
Properties of Triangles,
Heights and Distances

Properties of triangle is a part of trigonometry which is a branch of IN THIS CHAPTER ....


Mathematics. In which we study about triangles and relationship between
Relation between the Sides and
their sides and angles. Height and distance is an application of trigonometry.
Angles of Triangle
Here with the help of trigonometry, we find the height of any object or
distance of an object from any point/object which cannot measured directly. Trigonometric Ratios of Half
Angles of a Triangle

Relation between the Sides and Area of Triangles


A
Different Types of Circle
Angles of Triangle Connected with Triangle
In a D ABC, angles are denoted by A, B and C and the
Regular Polygon
lengths of corresponding sides opposite to these angles are c b
denoted by a , b and c, respectively. Area and perimeter of a Angle of Elevation
triangle are denoted by D and 2 s, respectively. Angle of Depression
a+ b+ c Bearing
Also, Semi-perimeter of the triangle is s = . B a C
2
The sum of all angles of a triangle is 180° i.e. ÐA + ÐB + ÐC = 180°.
The longest side of a triangle have corresponding largest angle and vice-versa,
while the smallest side of a triangle have corresponding smallest angle and
vice-versa.

Sine Rule
In any D ABC, the sines of the angles are proportional to the lengths of the
opposite sides,
sin A sin B sin C
i.e. = =
a b c
It can also be written as
a b c
= = = k (say)
sin A sin B sin C
Then, a = k sin A, b = k sin B, c = k sin C
232 JEE Main Mathematics

1 7 19 25
Cosine Rule Thus, cos A = = , cos B = , cos C =
5 35 35 35
In any D ABC, cosine of an angle can express in terms of cos A cos B cos C 1
sides. = = =
7 19 25 35
b2 + c2 - a 2 c2 + a 2 - b2 Þ ( a , b , g) = (7, 19, 25)
cos A = , cos B =
2 bc 2 ac
Example 2. If the angles A, B and C of a triangle are in an
a 2 + b2 - c2
and cos C = arithmetic progression and if a, b and c denote the lengths of
2 ab the sides opposite to A, B and C respectively, then the value of
a c
the expression sin 2 C + sin 2 A is
Projection Rule c a (JEE Main 2010)
In any DABC, 1 3
(a) (b) (c) 1 (d) 3
a = b cos C + c cos B 2 2
b = a cos C + c cos A Sol. (d) Since, A, B, C are in AP.
c = a cos B + b cos A Þ 2 B = A + C i.e. Ð B = 60 º
a c
Napier’s Analogy \
c
(2 sin C cos C) + (2 sin A cos A)
a
In any D ABC, = 2 k ( a cos C + c cos A)
C- A c-a B A-B a-b C é a b c 1ù
tan = cot , tan = cot
2 c+a 2 2 a+b 2 êëusing, sin A = sin B = sin C = k úû
B -C b - c A = 2 k ( b)
and tan = cot
2 b+ c 2 = 2 sin B [using b = a cos C + c cos A]
= 3
b +c c + a a+ b
Example 1. Given, = = for a DABC with
11 12 13 Example 3. With the usual notation, in DABC, if
cos A cos B cos C ÐA + ÐB = 120 °, a = 3 + 1and b = 3 - 1, then the ratio
usual notation. If = = , then the ordered triad
a b g ÐA : ÐB, is (JEE Main 2019)
( a, b, g) has a value (JEE Main 2019) (a) 7 : 1 (b) 3 : 1 (c) 9 : 7 (d) 5 : 3
(a) (19, 7, 25) (b) (3, 4, 5) Sol. (a) For a DABC, it is given that a = 3 + 1,
(c) (5, 12, 13) (d) (7, 19, 25)
b = 3 - 1and ÐA + ÐB = 120 º
b+ c c+ a a+ b
Sol. (d) Given, = = = l (say) A
11 12 13
A
c b

c b B a C

Clearly, ÐC = 60 º [Q ÐA + ÐB + ÐC = 180 º ]
Now, by tangent law, we have
B a C A -B a - b C
b + c = 11l, c + a = 12l and a + b = 13l ...(i) tan = cot
2 a+ b 2
Þ 2( a + b + c) = 36l ( 3 + 1) - ( 3 - 1) æ 60 º ö
Þ a + b + c = 18l ...(ii) = cot ç ÷
( 3 + 1) + ( 3 - 1) è 2 ø
From Eqs. (i) and (ii), we get
2 1
a = 7l, b = 6l, c = 5l = cot (30 º) = ´ 3 =1
2 3 3
b 2 + c 2 - a2
Now, cos A = æ A - Bö
2bc Þ tanç ÷ = 1= tan 45º
è 2 ø
l2[36 + 25 - 49] 12 1 A -B
= = = Þ = 45º Þ ÐA - ÐB = 90 º
60 l2 60 5 2
a2 + c2 - b 2 l2[ 49 + 25 - 36] 19 On solving ÐA - ÐB = 90 º and ÐA + ÐB = 120 º , we get
cos B = = =
2ac 70 l2 35 ÐA = 105º and ÐB = 15º
a2 + b 2 - c2 l2[ 49 + 36 - 25] 60 5 So, ÐA : ÐB = 7 : 1
and cos C = = = =
2ab 84l2 84 7
Properties of Triangles, Heights and Distances 233

Trigonometric Ratios of Half Angles Area of Triangles


of a Triangle In a D ABC, if the sides of the triangle are a , b, c and
In a D ABC, if the sides of the triangle are a , b, c and corresponding angles are A, B and C respectively, then
area of triangle
corresponding angles are A, B and C respectively and s is
the semi-perimeter, then (i) When two sides and angle between them is known
1 1 1
A ( s - b)( s - c) B ( s - c)( s - a ) D= ab sin C, D = bc sin A, D = ca sin B
(i) sin = (ii) sin = 2 2 2
2 bc 2 ca
(ii) When one side and corresponding angles are known
C ( s - a )( s - b) A s( s - a ) c 2 sin A sin B a 2 sin B sin C
(iii) sin = (iv) cos = D= , D=
2 ab 2 bc 2 sin C 2 sin A
B s( s - b) C s( s - c) b2 sin C sin A
(v) cos = (vi) cos = D=
2 ca 2 ab 2 sin B
A ( s - b)( s - c) B ( s - a )( s - c)
(vii) tan = (viii) tan = (iii) When all the three sides are known
2 s( s - a ) 2 s( s - b)
D= s (s - a) (s - b) (s - c )
C ( s - a )( s - b)
(ix) tan = It is known as Hero’s formula.
2 s( s - c)
Example 5. The angles A, B and C of a DABC are in AP
7
Example 4. If PQR is a triangle of area D with a = 2, b = and a : b =1: 3. If c = 4 cm, then the area (in sq cm) of this
2
5 triangle is (JEE Main 2019)
and c = , where a, b and c are the lengths of the sides of the
2 2 4
(a) (b) 4 3 (c) 2 3 (d)
triangle opposite to the angles at P, Q and R, respectively. 3 3
2 sin P - sin 2P Sol. (c) It is given that angles of a DABC are in AP.
Then, equals
2 sin P + sin 2P (JEE Main 2012) So, ÐA + ÐB + ÐC = 180 º
2 5 Þ ÐB - d + ÐB + ÐB + d = 180 º
3 45 æ 3 ö æ 45 ö
(a) (b) (c) çç ÷÷ (d) çç ÷÷ [if ÐA, ÐB and ÐC are in AP, then it taken as
4D 4D è 4 Dø è 4 Dø ÐB - d, ÐB, ÐB + d respectively,
where d is common difference of AP]
Sol. (c) If DABC has sides a, b, c.
A Þ 3ÐB = 180 º Þ ÐB = 60 º …(i)
( s - b) ( s - c) a 1
Then, tan ( A /2) = and = [given]
s ( s - a) c b
b 3
a+ b+ c B C sin A 1
where, s= a Þ =
2 sin B 3
7 5 é sin A sin B sin C ù
2+ +
Þ s= 2 2 =4 êëby sine rule a = b = c úû
2
2 sin P - sin 2 P 2 sin P (1 - cos P) sin A 1 é 3ù
\ = Þ = êQ sin B = sin 60 ° = ú
2 sin P + sin 2 P 2 sin P (1 + cos P) 3 3 ë 2 û
2
2 sin 2 (P /2)
= = tan 2 (P /2) 1
2 cos2 (P /2) Þ sin A = Þ ÐA = 30 º
2
P So, ÐC = 90 º
a b c
\ From sine rule, = =
c = 5/2 b = 7/2 sin A sin B sin C
a b 4
Þ = = [Q c = 4 cm]
Q R 1 3 1
a=2
2 2
( s - b) ( s - c) ( s - b) ( s - c)
Þ ´ Þ a = 2 cm, b = 2 3 cm
s ( s - a) ( s - b) ( s - c)
1 1
æ 7ö æ 5ö
2 2 \ Area of DABC = ab sin C = ´ 2 ´ 2 3 ´ 1
2 2ç4 - ÷ ç4 - ÷ 2
2 2
[( s - b) ( s - c) ] è 2ø è 2ø æ 3 ö = 2 3 sq. cm
= = = ç ÷
D2 D2 è 4D ø
234 JEE Main Mathematics

1
Angle of Elevation Þ x=
3
y …(i)

If ‘O’ be the observer’s eye and OX be the horizontal line and in DTSA, we have
through O. If object P is at a higher level than eye, then x + 30
tan 45° =
Ð POX is called the angle of elevation. y
P Þ y = x + 30 …(ii)
On the elimination of quantity x from
ht Eqs. (i) and (ii), we get
s ig
of 1
e y= y + 30
Lin 3
q æ 1 ö
Þ y ç1 - ÷ = 30
O X è 3ø
Horizontal line
30 3 30 3 ( 3 + 1)
Þ y= =
3 -1 3 -1
Angle of Depression 30
If ‘O’ be the observer’s eye and OX is a horizontal line. If = 3 ( 3 + 1)
2
object P is at a lower level than O, then the ÐPOX is
called the angle of depression. = 15 (3 + 3)
Horizontal line
O X Example 7. Two poles standing on a horizontal ground
q
Lin are of heights 5 m and 10 m, respectively. The line joining
eo
fs their tops makes an angle of 15º with ground. Then, the
igh
t distance (in m) between the poles, is (JEE Main 2019)
P 5
(a) 5( 3 + 1) (b) (2 + 3)
● Angle of elevation and depression are always acute 2
angle. (c) 10( 3 - 1) (d) 5(2 + 3)
● Angle of elevation of an object from an observer is same
as angle of depression of an observer from the object. Sol. (d) Given heights of two poles are 5 m and 10 m.
A
Example 6. The angle of elevation of the top of a vertical
tower standing on a horizontal plane is observed to be 45° 5m
15° E
from a point A on the plane. Let B be the point 30 m vertically B
10 m d
above the point A. If the angle of elevation of the top of the
tower from B be 30°, then the distance (in m) of the foot of 5m
the tower from the point A is (JEE Main 2019)
15° F
(a) 15 (3 + 3) (b) 15 (5 - 3) C d D
(c) 15 (3 - 3) (d) 15 (1 + 3)
i.e. from figure, AC = 10 m, DE = 5 m
Sol. (a) According to the question, we have the following figure. \ AB = AC - DE = 10 - 5 = 5 m
T Let d be the distance between two poles.
Clearly, DABE ~ DACF [by AA-similarity criterion]
\ ÐAEB = 15°
xm
In DABE, we have
AB 3 -1 5 é 3 - 1ù
30° B tan15° = Þ = êQ tan15° = ú
P BE 3 +1 d ë 3 + 1û
30 m 5( 3 + 1)
Þ d=
45° ( 3 - 1)
S ym A
3 +1 3 +1
Þ d =5 ´
Now, let distance of foot of the tower from the point A is y m. 3 -1 3 +1
Draw BP ^ ST such that PT = x m. 5(3 + 2 3 + 1 5(2 3 + 4)
Then, in DTPB, we have = =
3 -1 2
x
tan 30 ° = 2 ´ 5( 3 + 2)
y = = 5(2 + 3) m
2
Properties of Triangles, Heights and Distances 235

(v) In a right angled DABC, the mid-point D of


Bearing hypotenuse AC is equidistant from its vertices A, B
If the observer and the object are O and P N and C.
P
be on the same level respectively, then
A
bearing is defined. To measure the q
bearing the four standard directions W E D
O
East, West, North and South are taken
as the cardinal directions. Angle between B C
the line of observations. i. e. ,OP and any S
i.e., AD = BD = CD
one standard direction is measured.
(vi) m-n Theorem
Thus, Ð POE is called the bearing of the point P with In a DABC, D divides BC in the ratio m : n, then
respect to O measured from East to North.
A
In other words the bearing of P as seen from O is the
direction in which P is seen from O. a b
Note North-East means equally inclined to North and East. ENE
means equally inclined to East and North-East. B q C
B m D n C
Some Important Facts (a) ( m + n ) cot q = n cot B - m cot C
(i) In an isosceles triangle, the median is perpendicular (b) ( m + n ) cot q = m cot a - n cot b
to the base. D ABC is isosceles and AD^ BC. (vii) Appolonius Theorem
A If in D ABC, AD is median, then
AB2 + AC 2 = 2 ( AD 2 + BD 2 )
A

B D C

(ii) The exterior angle of a triangle is equal to the sum of C B


D
interior opposite angles.
A (viii) Angles in the same segment of a circle are equal.
q2 (ix) The angle subtended at the centre by an arc is
double of the angle subtended at any point of the
q q1 remaining arc by the same arc.
X C B (x) Ptolemy’s Theorem
In cyclic quadrilateral ABCD,
In DABC, q = q1 + q2
AB × CD + AD × BC = AC × BD
(iii) In a triangle, the internal bisector of an angle
D
divides the opposite side in the ratio of the arms of
A
the angle.
A
B C
A A
2 2
c b Example 8. A man find that at a point due South of a
p
verticle tower the angle of elevation of the tower is . He
B a D C 3
In DABC, AD is bisector of Ð A. then walks due West 10 6 m on the horizontal plane and find
BD c p
\ = the angle of elevation of the tower to be . Original distance
DC b 6
of the man from the tower is
(iv) In a DABC, if DE|| AB, then
(a) 5 3 m (b) 15 3 m
AB BC AC
= = (c) 10 3 m (d) None of these
DE DC EC
A Sol. (a) Let PQ be the tower and A, B are points of observations.
E We have,
p p p
ÐPAB = , ÐQAP = and ÐQBP = .
C D B 2 3 6
236 JEE Main Mathematics

p PQ 1
Now, AP = PQ × cot = Clearly, length of BD = 2 a2 + 2 c 2 - b 2 ,
3 3 2
Q
[using Appollonius theorem]
where, c = AB = 7, a = BC = 5
and b = CA = 6
1
\ BD = 2 ´ 25 + 2 ´ 49 - 36
A p/3 2
1 1
= 112 = 4 7 = 2 7
2 2
p/6 Now, let ED = h be the height of the lamp post.
B P
p E
BP = PQ × cot = PQ 3
6
In right angled DPAB, h
BP 2 = AP 2 + AB2
30°
PQ 2 B D
Þ 3PQ 2 = + 600
3 h
8 600 × 3 Then, in DBDE, tan 30 ° =
Þ PQ 2 = 600 Þ PQ 2 = BD
3 8
1 h 2 7 2
Þ PQ = 15 m Þ = Þh = = 21 m
3 2 7 3 3
PQ 15
Now, AP = = m =5 3 m
3 3
Hence, (a) is the correct answer. Example 10. A train travelling on one of two intersecting
railway lines, subtends at a certain station on the other line,
Example 9. Consider a triangular plot ABC with sides
an angle a when the front of the carriage reaches the junction
AB = 7 m, BC = 5 m and CA = 6 m. A vertical lamp-post at the
mid-point D of AC subtends an angle 30° at B. The height (in and an angle b when the end of the carriage reaches it. Then,
m) of the lamp-post is (JEE Main 2019) the two lines are inclined to each other at an angle q
2 determined by 2 cot q is equal to
(a) 21 (b) 2 21
3 (a) tan a - tan b (b) cot b - cot a
3 (c) cot a - cot b (d) cot a + cot b
(c) 7 3 (d) 21
2 Sol. (c) l1 and l2 are two intersecting lines.
Sol. (a) According to given information, we have the following l2 l1
A
figure.
E x
A
q
x
a
C b
c=7

B
b=6
D
In DABC, by m - n theorem, we have
( x + x ) cot q = x cot a - x cot b Þ 2 cot q = cot a - cot b
30°
( x + x) cot q = x cot b - x cot a
B a=5 C
2 cot q = cot b - cot a
Practice Exercise
ROUND I Topically Divided Problems
Relation between the Sides and 8. In a triangle, the lengths of the two larger sides are
Angles of a Triangle 10 cm and 9 cm, respectively. If the angles of the
triangle are in AP, then the length of the third side
sin 3B
1. If in a D ABC, 2 b2 = a 2 + c2 , then is equal to in cm can be
sin B (a) 5 - 6 only
c2 - a 2 c2 - a 2 (b) 5 + 6 only
(a) (b) (c) 5 - 6 or 5 + 6
2 ca ca
2 2 (d) neither 5 - 6 nor 5 + 6
æ c 2 - a 2ö æ c 2 - a 2ö
(c) ç ÷ (d) ç ÷ 9. Points D, E are taken on the side BC of a DABC
è ca ø è 2 ca ø
such that BD = DE = EC. If ÐBAD = x, ÐDAE = y,
2. sin A : sin C = sin ( A - B) : sin ( B - C), then sin ( x + y) sin ( y + z)
ÐEAC = z, then the value of
a 2 , b2 and c2 are in sin x sin z
(a) AP (b) GP is equal to
(c) HP (d) None of these (a) 1 (b) 2
3. If a , b and c are sides of a triangle, then (c) 4 (d) None of these
(a) a + b> c 10. In a DPQR as shown in figure given that
(b) a - b > c (if c is smallest) x : y : z : : 2 : 3 : 6, then the value of ÐQPR is
p
(c) a + b < c (a) R y
6 S
(d) None of the above
p
4. In a DABC, if 2 s = a + b + c and (b) x
4 a z
2 A p
( s - b) ( s - c) = x sin , then the value of x is (c)
b
Q
2 3 P
(a) bc (b) ca (c) ab (d) abc
(d) None of the above
5. If the angles of a triangle are in the ratio 4 : 1 : 1, 11. In a triangle, the lengths of two larger sides are 10
then the ratio of the longest side to the perimeter is
cm and 9 cm. If the angles of the triangle are in AP,
(a) 3 : (2 + 3 ) (b) 1 : 6
then the length of the third side is
(c) 1 : (2 + 3 ) (d) 2 : 3
(a) 5 – 6 (b) 5 + 6
1 1 (c) 5 ± 6 (d) 5 ± 6
6. In a DABC, ÐC = 60°, then + is equal to
a+ c b+ c p
1 2 12. Let ABC be a triangle such that ÐACB = and let a,
(a) (b) 6
a+ b+ c a+ b+ c
b and c denote the lengths of the sides opposite to
3
(c) (d) None of these A, B and C respectively. The value(s) of x for which
a+ b+ c a = x 2 + x + 1, b = x 2 - 1 and c = 2 x + 1 is (are)
7. In a DABC, if a , b and c are in AP, then the value of (a) - (2 + 3 ) (b) 1 + 3
A C (c) 2 + 3 (d) 4 3
sin sin
2 2 is A B C
B
13. In DABC, if sin 2 , sin 2 and sin 2 are in HP.
sin 2 2 2
2 Then, a, b and c will be in
1 (a) AP (b) GP
(a) 1 (b) (c) 2 (d) - 1
2 (c) HP (d) None of these
238 JEE Main Mathematics

14. In DABC, AD is median and ÐA = 60°, then 4 AD 2 a 2 + b 2 + c2 a 2 + b 2 + c2


(a) (b)
is equal to 2R R
A a 2 + b 2 + c2 2 (a 2 + b 2 + c 2)
(c) (d)
4R R
°
60 C C
22. In DABC, ( a - b) 2 cos2 + ( a + b) 2 sin 2 is equal to
2 2
(a) a 2 (b) b 2
60° (c) c 2 (d) None of these
B D C
2 2 23. If two adjacent sides of a cyclic quadrilateral are 2
(a) b + c - bc (b) 2 b + c2 - 2 bc
2
and 5 and the angle between them is 60°. If the
(c) b2 + c2 + 2 bc (d) None of these third side is 3, then the remaining fourth side is
(a) 2 (b) 3 (c) 4 (d) 5
15. If cos2 A + cos2 C = sin 2 B, then DABC is
(a) equilateral (b) right angled Heights and Distances
(c) isosceles (d) None of these
24. A flag staff 20 m long standing on a wall 10 m high
b+ c c+ a a+ b
16. If in a DABC, = = , then cos A is subtends an angle whose tangent is 0.5 at a point
11 12 13 on the ground. If q is the angle subtended by the
equal to wall at this point, then
1 5
(a) (b) (a) tan q = 1 (b) tan q = 3
5 7 1
19 (c) tan q = (d) None of these
(c) (d) None of these 2
35
cos A cos B cos C 1 25. A ladder rests against a wall making an angle a
17. In a DABC, = = . If a = , then the with the horizontal. The foot of the ladder is pulled
a b c 6
away from the wall through a distance x, so that it
area (in sq units) of the triangle is
1 1 slides a distance y down the wall making an angle b
(a) 1/24 (b) 3 / 24 (c) (d) with the horizontal. The correct relation is
8 3
æ a + bö æ a + bö
18. In a D ABC, a, c, A are given and b1, b2 are two (a) x = y tan ç ÷ (b) y = x tan ç ÷
è 2 ø è 2 ø
values of the third side b such that b2 = 2 b1, then (c) x = y tan (a + b ) (d) y = x tan (a + b )
sin A is equal to
26. At a distance 2 h m from the foot of a tower of height
9 a 2 - c2 9a 2 - c2
(a) 2
(b) h m the top of the tower and a pole at the top of the
8a 8 c2
tower subtend equal angles. Height of the pole
9a 2 + c2 should be
(c) (d) None of these
8 a2 5h 4h 7h 3h
(a) m (b) m (c) m (d) m
3 3 3 5 2
19. In a DABC, a = 5, b = 7 and sin A = , then the
4 27. Two vertical poles of heights, 20 m and
number of possible triangles are 80 m stand apart on a horizontal plane. The height
(a) 1 (b) 0 (c) 2 (d) infinite (in m) of the point of intersection of the lines
20. If p 1, p 2 and p 3 are altitudes of a DABC drawn joining the top of each pole to the foot of the other,
from the vertices A, B and C and D the area of the from this horizontal plane is (JEE Main 2019)

triangle, then p -1 2 + p -2 2 + p -3 2 is equal to (a) 15 (b) 16 (c) 12 (d) 18


2 2 2
a+ b+ c a +b +c 28. The angle of elevation of the summit of a mountain
(a) (b)
D 4 D2 from a point on the ground is 45°. After climbing up
2
a +b +c 2 2 one km towards the summit at an inclination of 30°
(c) (d) None of these from the ground, the angle of elevation of the
D2
summit is found to be 60°. Then, the height (in km)
21. If x, y and z are perpendicular drawn from the of the summit from the ground is (JEE Main 2020)
vertices of triangle having sides a , b and c, then
3 -1 3 +1 1 1
bx cy az (a) (b) (c) (d)
the value of + + will be 3 +1 3 -1 3 -1 3 +1
c a b
Properties of Triangles, Heights and Distances 239

29. A house of height 100 m subtends a right angle at (Ignore man’s height). After sailing for 20 seconds
the window of an opposite house. If the height of towards the base of the tower (which is at the level
the window be 64 m, then the distance between the of water), the boat has reached a pint B, where the
two houses is angle of depression is 45°. Then the time taken (in
(a) 48 m (b) 36 m (c) 54 m (d) 72 m seconds) by the boat from B to reach the base of the
30. A pole stands vertically inside a triangular park tower is (JEE Main 2021)

ABC. Let the angle of elevation of the top of the (a) 10( 3 - 1) (b) 10 3
p (c) 10 (d) 10( 3 + 1)
pole from each corner of the park be . If the radius
3
37. At a point on the ground the angle of elevation of a
of the circumcircle of DABC is 2, then the height of
3
the pole is equal to (JEE Main 2021) tower is such that its cotangent is . On walking
5
2 3 1
(a) (b) 2 3 (c) 3 (d) 32 m towards the tower the cotangent of the angle
3 3 2
of elevation is . The height of the tower is
31. An observer on the top of tree, finds the angle of 5
depression of a car moving towards the tree to be (a) 160 m (b) 120 m
30°. After 3 min this angle becomes 60°. After how (c) 64 m (d) None of these
much more time, the car will reach the tree ?
38. From the bottom of a pole of height h the angle of
(a) 4 min (b) 4.5 min (c) 1.5 min (d) 2 min
elevation of the top of a tower is a and the pole
32. The horizontal distance between two towers is 60 m subtends an angle b at the top of the tower. The
and the angle of depression of the top of the first height of the tower is
tower as seen from the top of the second is 30°. If h tan (a - b ) h cot (a - b )
(a) (b)
the height of the second tower be 150 m, then the tan (a - b) - tan a cot (a - b) - cot a
height of the first tower is cot (a - b )
(c) (d) None of these
(a) (150 - 60 3 ) m (b) 90 m cot (a - b) - cot a
(c) (150 - 20 3 ) m (d) None of these
39. The top of a hill observed from the top and bottom
33. The angle of elevation of a cloud C from a point P,
of a building h is at angles of elevation p and q,
200 m above a still lake is 30°. If the angle of
respectively. The height of hill is
depression of the image of C in the lake from the
h cot q h cot p
point P is 60°, then PC (in m) is equal to (a) (b)
cot q - cot p cot p - cot q
(JEE Main 2020)
(a) 100 (b) 400 (c) 200 3 (d) 400 3 h tan p
(c) (d) None of these
tan p - tan q
34. PQR is a triangular park with PQ = PR = 200 m. A
40. From the top of a cliff of height a , the angle of
TV tower stands at the mid-point of QR. If the
depression of the foot of a certain tower is found to
angles of elevation of the top of the tower at P , Q
be double the angle of elevation of the top of the
and R are respectively 45°, 30° and 30°, then the
tower of height h. If q be the angle of elevation,
height of the tower (in m) is (JEE Main 2018)
then its value is
(a) 100 (b) 50 (c) 100 3 (d) 50 2
2h 2h
(a) cos -1 (b) sin -1
35. An aeroplane flying horizontally 1 km above the a a
ground is observed at an elevation of 60° and after a 2h
(c) sin - 1 (d) tan - 1 3 -
10 s the elevation is observed to be 30°. The 2-h a
uniform speed of the aeroplane (in km/h) is
(a) 240 (b) 240 3 41. From a point a metres above a lake the angle of
(c) 60 3 (d) None of these elevation of a cloud is a and the angle of depression
of its reflection is b. The height of the cloud is
36. A man is observing, from the top of a tower, a sin(a + b ) a sin(a + b )
a boat speeding towards the tower from a certain (a) m (b) m
sin(a - b ) sin (b - a)
point A, with uniform speed. At that point, angle
a sin (b - a)
of depression of the boat with the man’s eye is 30° (c) (d) None of these
sin (a + b )
240 JEE Main Mathematics

42. A flag staff of 5 m high stands on a building of 25 m 48. ABC is a triangular park with AB = AC = 100 m.
high. At an observer at a height of 30 m, the flag staff A clock tower is situated at the mid-point of BC.
and the building subtend equal angles. The distance The angles of elevation of the top of the tower at A
of the observer from the top of the flag staff is and B are cot - 1 3.2 and cosec -1 2.6, respectively.
5 3 3 The height of the tower is
(a) m (b) 5 m
2 2 (a) 50 m (b) 25 m (c) 40 m (d) None of these
2 49. The angle of elevation of the top of a tower from a
(c) 5 m (d) None of these
3 point A due South of the tower is a and from a
43. The angles of depression of the top and the foot of a point B due East of the tower is b. If AB = d, then
chimney as seen from the top of a second chimney, the height of the tower is
d d
which is 150 m high and standing on the same level (a) (b)
2 2
as the first are q and f respectively, then the tan a - tan b tan a + tan 2 b
2

4 d d
distance between their tops when tan q = and (c) (d)
3 cot 2 a + cot 2 b cot 2 a - cot 2 b
5
tan f = , is
2 50. ABCD is a square plot. The angle of elevation of
150 the top of a pole standing at D from A or C is 30°
(a) m (b) 100 3 m (c) 150 m (d) 100 m
3 and that from B is q, then tan q is equal to
(a) 6 (b) 1/ 6 (c) 3/ 2 (d) 2 /3
44. From the tower 60 m high angles of depression of
the top and bottom of a house are a and b, 51. A vertical tower stands on a declivity which is
respectively. If the height of the house is inclined at 15° to the horizon. From the foot of the
60 sin(b - a ) tower a man ascends the declivity for 80 ft and
, then x is equal to
x then, finds that the tower subtends an angle of 30°.
(a) sin a sin b (b) cos a cos b The height of tower is
(c) sin a cos b (d) cos a sin b (a) 20( 6 - 2 ) ft (b) 40( 6 - 2 ) ft
(c) 40( 6 + 2 ) ft (d) None of these
45. The angle of elevation of the top of the tower
observed from each of the three points A, B, C on 52. If A and B are two points on one bank of a straight
the ground forming a triangle is the same Ða. If R river and C, D are two other points on the
is the circumradius of the DABC, then the height of other bank of river. If direction from A to B is same
the tower is as that from C to D and AB = a, Ð CAD = a,
(a) Rsin a (b) R cos a (c) R cot a (d) R tan a Ð DAB = b, Ð CBA = g, then CD is equal to
a sin b sin g a sin a sin g
46. A spherical balloon of radius r subtends an Ða at (a) (b)
sin a sin (a + b + g ) sin b sin (a + b + g )
the eye of an observer. If the angle of elevation of
the centre of the balloon be b, then height of the a sin a sin b
(c) (d) None of these
centre of the balloon is sin g sin (a + b + g )
æ aö æbö
(a) r cosec ç ÷ sin b (b) r cosec a sin ç ÷ 53. The angle of elevation of the top of a hill from a
è2ø è2ø
æ aö æbö point is a. After walking b m towards the top up a
(c) r sin ç ÷ cosec b (d) r sin a cosec ç ÷ slope inclined at an Ðb to the horizon, the angle of
è2ø è2ø
elevation of the top becomes g. Then, the height of
47. A ladder leaves again a wall at an Ða to the the hill is
horizontal. Its foot is pulled away through a b sin a sin(g - b ) b sin a sin(g - a )
distance a1, so that it slides a distance b1 down the (a) (b)
sin (g - a ) sin (g - b )
wall and rests inclined at Ðb with the horizontal. It b sin (g - b ) sin(g - b )
foot is further pulled aways through a 2 , so that it (c) (d)
sin (g - a ) b sin a sin (g - a )
slides a further distance b 2 down the wall and is
now, inclined at an Ðg. If a1 a 2 = b1b 2 , then 54. The angle of elevation of a cloud at a point 2500 m
(a) a + b + g is greater than p high above a lake is 15° and the angle of depression
(b) a + b + g is equal to p of its image in the lake is 45°, the height of the
(c) a + b + g is less than p cloud above the surface of the lake is
(d) nothing can be said about a + b + g (a) 2500 3 m (b) 2500 m (c) 500 3 m (d) 500 m
Properties of Triangles, Heights and Distances 241

ROUND II Mixed Bag


Only One Correct Option 9. Two vertical poles AB = 15 m and CD = 10 m are
1. In an ambiguous case of solving a triangle when standing apart on a horizontal ground with points
p A and C on the ground. If P is the point of
a = 5, b = 2 , Ð A = , the two possible values of
6 intersection of BC and AD, then the height of P
third side are c1 and c2 , then (in meters) above the line AC is (JEE Main 2020)
(a) c1 - c2 = 2 6 (b) c1 - c2 = 4 6 (a) 20/3 (b) 6 (c) 10/3 (d) 5
(c) c1 - c2 = 4 (d) c1 - c2 = 6
10. ABC is a triangular park with AB = AC = 100 m.
2. In an isosceles D ABC, AB = AC. If vertical ÐA is A vertical tower is situated at the mid-point of BC.
20°, then a 3 + b3 is equal to If the angles of elevation of the top of the tower at A
(a) 3a 2b (b) 3b 2c and B are cot -1 (3 2 ) and cosec-1 ( 2 2 )
(c) 3c 2a (d) abc respectively, then the height of the tower(JEE(in m)2019)
Main is
3. If A, B, C, D are the angles of a quadrilateral, then 100
(a) 25 (b) 20 (c) 10 5 (d)
S tan A 3 3
is equal to
S cot A 11. If PQ be a vertical tower subtending angles a , b and
(a) P tan A (b) P cot A 2
(c) S tan A (d) S cot A 2 g at the points A, B and C respectively on the line
in the horizontal plane through the foot D of tower
4. In a D ABC, ( a + b + c)( b + c - a) = kbc, if
and on the same side of it, then
(a) k < 0 (b) k > 6
BC cot a - CA cot b + AB cot g is equal to
(c) 0 < k < 4 (d) k > 4
(a) 0 (b) 1
5. In a triangle, the sum of lengths of two sides is x (c) 2 (d) None of these
and the product of the lengths of the same two
12. A flag staff stands in the centre of a rectangular
sides is y. If x 2 - c2 = y, where c is the length of the
field whose diagonal is 1200 m and subtends angles
third side of the triangle, then the circumradius of
15° and 45° at the mid-points of the sides of the
the triangle is (JEE Main 2019)
c c 3 y field. The height of the flag staff is
(a) (b) (c) y (d) (a) 200 m (b) 300 2 + 3 m
3 3 2 3
(c) 300 2 - 3 m (d) 400 m
6. ABCD is a trapezium such that AB and CD are
parallel and BC ^ CD. If ÐADB = q, BC = p and 13. At each end of a horizontal line of length 2a, the
CD = q, then AB is equal to (JEE Main 2013) angular elevation of the peak of a vertical tower is q
( p2 + q2) sin q p2 + q2 cos q and that at its middle point it is f. The height of
(a) (b)
p cos q + q sin q p cos q + q sin q the peak is
p2 + q 2 ( p2 + q2) sin q a sin q sin f
(c) (d) (a) a sin q sin f (b)
sin ( q + f ) sin (f - q)
p2 cos q + q2 sin q ( p cos q + q sin q)2
a cos q cos f
p 5p (c) (d) None of these
7. In a DABC, B = and C = . The altitude from A cos (f + q) cos (f - q)
8 8
to the side BC is 14. In a cubical hall ABCDPQRS with each side 10 m,
a G is the centre of the wall BCRQ and T is the
(a) (b) 2a
2 mid-point of the side AB. The angle of elevation of
1 G at the point T is
(c) (b + c) (d) None of these
2 (a) sin -1 (1/ 3 ) (b) cos -1 (1/ 3 )
8. If the sides of a triangle are in GP and the largest (c) cot-1 (1/ 3 ) (d) None of these
angle is twice the smallest angle, then the common 15. Two vertical poles 20 m and 80 m stands apart on a
ratio, which is greater than 1, lies in the interval horizontal plane. The height of the point of
(a) (1, 3 ) (b) (1, 4 3 ) intersection of the lines joining the top of each pole
æ 5 + 1ö to the foot of the other is
(c) ç1, ÷ (d) None of these (a) 15 m (b) 16 m (c) 18 m (d) 50 m
è 2 ø
242 JEE Main Mathematics

16. A bird is sitting on the top of a vertical pole 20 m top of the tower from A or B is 30°. The height of
high and its elevation from a point O on the ground the tower is
is 45°. It flies off horizontally straight away from 2a a
(a) (b) 2a 3 (c) (d) 3
the point O. After 1s, the elevation of the bird from 3 3
O is reduced to 30°. Then, the speed (in m/s) of the
Numerical Value Type Questions
bird is (JEE Main 2014)
(a) 40 ( 2 - 1) (b) 40( 3 - 2 )
21. In a D ABC', medians AD andCE are drawn. If AD = 5,
(c) 20 2 (d) 20( 3 - 1) ÐDAC = p /8 and ÐACE = p / 4, then the area of the
5a
17. Let a vertical tower AB have its end A on the level D ABC is equal to , then a + b is equal to ............ .
b
ground. Let C be the mid-point of AB and P be a
point on the ground such that AP = 2 AB. If
22. In DABC, the lengths of sides AC and AB are 12 cm
and 5 cm, respectively. If the area of DABC is 30
ÐBPC = b, then tan b is equal to (JEE Main 2017)
cm 2 and R and r are respectively the radii of
6 1 2 4
(a) (b) (c) (d) circumcircle and incircle of DABC then the value of
7 4 9 9
2 R + r (in cm) is equal to ………
18. If the angles of elevation of the top of a tower from
23. Let ABC and ABC¢ be two non-congruent triangles
three collinear points A, B and C on a line leading
with sides AB = 4, AC = AC ¢ = 2 2 and angle
to the foot of the tower are 30°, 45° and 60°
B = 30°. The absolute value of the difference
respectively, then the ratio AB : BC is (JEE Main 2015)
between the areas of these triangles is ............ .
(a) 3 : 1 (b) 3 : 2 (c) 1 : 3 (d) 2 : 3
24. The angle of elevation of the top of a hill from a
19. AB is a vertical pole with B at the ground level and point on the horizontal plane passing through the
A at the top. A man finds that the angle of
foot of the hill is found to be 45°. After walking a
elevation of the point A from a certain point C on
distance of 80 m towards the top, up a slope
the ground is 60°. He moves away from the pole
inclined at an angle of 30° to the horizontal plane,
along the line BC to a point D such that CD = 7m.
the angle of elevation of the top of the hill becomes
From D the angle of elevation of the point A is 45°.
75°. Then, the height of the hill (in meters) is ........ .
Then, the height of the pole is (JEE Main 2020)
7 3 æ 1 ö 7 3 æ 1 ö
(a) ç ÷m (b) ç ÷m 25. A man is walking towards a vertical pillar in a
2 è 3 + 1ø 2 è 3 - 1ø
straight path, at a uniform speed. At a certain
7 3 7 3 point A on the path, he observes that the angle of
(c) ( 3 + 1) m (d) ( 3 - 1) m
2 2 elevation of the top of the pillar is 30°. After walking
20. A tower stands at the centre of a circular park. A for 10 min from A in the same direction, at a point
and B are two points on the boundary of the park B, he observes that the angle of elevation of the top
such that AB ( = a) subtends an angle of 60° at the of the pillar is 60°. Then, the time taken (in minutes)
foot of the tower and the angle of elevation of the by him, from B to reach the pillar, is ............ .

Answers
Round I
1. (d) 2. (a) 3. (a) 4. (a) 5. (a) 6. (c) 7. (b) 8. (c) 9. (c) 10. (b)
11. (d) 12. (b) 13. (c) 14. (d) 15. (b) 16. (a) 17. (b) 18. (b) 19. (b) 20. (b)
21. (a) 22. (c) 23. (a) 24. (a) 25. (a) 26. (a) 27. (b) 28. (c) 29. (a) 30. (b)
31. (c) 32. (d) 33. (b) 34. (a) 35. (b) 36. (d) 37. (a) 38. (b) 39. (b) 40. (d)
41. (b) 42. (b) 43. (d) 44. (d) 45. (d) 46. (a) 47. (c) 48. (b) 49. (c) 50. (b)
51. (b) 52. (b) 53. (a) 54. (a)

Round II
1. (c) 2. (c) 3. (a) 4. (c) 5. (b) 6. (a) 7. (a) 8. (b) 9. (b) 10. (b)
11. (a) 12. (c) 13. (b) 14. (a) 15. (b) 16. (d) 17. (c) 18. (a) 19. (c) 20. (c)
21. (8) 22. (15) 23. (4) 24. (80) 25. (5)
Solutions
Round I Thus, the ratio of longest side to the perimeter
sin 3B 3 sin B - 4 sin B 3 a
1. = = 3 - 4 sin 2 B =
sin B sin B a+ b+ c

= 3 - 4(1 - cos 2 B ) Let b=c=x


4(a 2 + c2 - b2)2 \ a 2 = b2 + c2 - 2 bc cos A
= -1 +
4(ac)2 Þ a 2 = 2 x 2 - 2 x 2 cos A = 2 x 2 (1 - cos A )
2 A
æ a 2 + c2 ö Þ a 2 = 4x2 sin 2
ç ÷ 2
è 2 ø A
= -1 + Þ a = 2 x sin
(ac)2 2
(a 2 + c2)2 Þ a = 2 x sin 60° = 3x
= -1 + [Q 2 b2 = a 2 + c2 (given)] Thus, required ratio is
4(ac)2
2 a 3x 3
(a 2 + c2)2 - 4a 2c2 æ c2 - a 2 ö = =
= =ç ÷ a + b + c x + x + 3x 2 + 3
4(ac)2 è 2 ac ø
a 2 + b2 - c2
sin A sin ( A - B ) 6. We have, cos C =
2. Q = 2 ab
sin C sin (B - C )
sin A sin A cos B - cos A sin B a 2 + b2 - c2
Þ = Þ cos 60° =
sin C sin B cos C - cos B sin C 2 ab
a a cos B - b cos A Þ a 2 + b2 - c2 = ab
Þ =
c b cos C - c cos B Þ b2 + bc + a 2 + ac = ab + ac + bc + c2
Þ ab cos C + bc cos A = 2 ac cos B Þ b(b + c) + a (a + c) = (a + c)(b + c)
a + b2 - c2 b2 + c2 - a 2
2
c2 + a 2 - b2 On dividing by (a + c)(b + c) and add 2 on both sides,
Þ + =2
2 2 2 we get
Þ a 2 + c2 = 2 b2 b a
1+ +1+ =3
Þ a 2, b2 and c2 are in AP. a+c b+ c
3. Q ( a + b + c )( a + b - c ) 1 1 3
Þ + =
a+ c b+ c a+ b+ c
=( a + b )2 - c = a + b - c + 2 ab > 0
\ a + b> c 7. Since a , b and c are in AP.
A (s - b)(s - c) Þ 2b = a + c
4. We have, =
sin
2 bc Þ 3b =2s
A 3b
Þ bc sin 2 = (s - b)(s - c) Þ s=
2 2
A A C
On comparing with x sin 2 = (s - b)(s - c), we get sin sin
2 Now, 2 2 = ac (s - b)(s - c)(s - b)(s - a )
x = bc sin
B (s - a )(s - c) ´ bc ´ ab
2
5. Here, ratio of angles are 4 : 1 : 1. 3b
-b
Þ 4x + x + x = 180° s-b 1
= = 2 =
Þ x = 30° b b 2
\ Ð A = 120° , ÐB = ÐC = 30°
8. We know that, in triangle larger side has an larger
C angle opposite to it. Since, ÐA , ÐB and ÐC are in AP.
Þ 2B = A + C
30°
a Q A+ B+C=p
b Þ B = 60°
120° 30° a 2 + c2 - b2
\ cos B =
A c B 2 ac
244 JEE Main Mathematics

1 100 + a 2 - 81 Now, in DABC ,


Þ cos 60° = =
2 20a a 2 + c2 - b2
2 cos B =
Þ a + 19 = 10a 2ac
Þ 2
a - 10a + 19 = 0 (10)2 + x2 - 92
Þ cos 60° =
10 ± 100 - 76 2 ´ 10 ´ x
\ a= =5 ± 6
2 1 100 + x2 - 92
Þ =
sin ( y + z ) sin C 2 20x
9. From D ADC , = A
DC AD Þ x2 - 10x + 19 = 0
sin x sin B x y z
From D ABD, = 10 ± 100 - 76
BD AD \ x=
2
sin z sin C
From D AEC , = 10 ± 24
EC AE = =5 ± 6
2
sin (x + y) sin B B D E C
From D ABE , = a 2 + b2 - c2
BE AE 12. Using, cos C =
BE sin B DC sin C 2ab
´
sin (x + y) sin ( y + z ) AE AD
A
\ =
sin x sin z BD sin B EC sin C
´
AD AE

–1

c=
BE DC AD AE

x2
= ´ ´ ´

2x
b=

+
AE AD BD EC

1
2BD ´ 2EC
= =4
BD ´ EC 30°
C B
tan a + tan b a = x2 + x + 1
10. tan (a + b) = R y
1 - tan a tan b 3 (x2 + x + 1)2 + (x2 - 1)2 - (2x + 1)2
Þ =
y x
+
S 2 2(x2 + x + 1)(x2 - 1)
= z z x
y x Þ (x + 2)(x + 1)(x - 1) x + (x2 - 1)2
z
1- ´ a = 3 (x2 + x + 1)(x2 - 1)
z z b
3 2 P Q Þ x + 2 x + (x - 1) = 3 (x2 + x + 1)
2 2
+
Þ (2 - 3 ) x2 + (2 - 3 ) x - ( 3 + 1) = 0
= 6 6 =1
3 2 Þ x = - (2 + 3 ) and x = 1 + 3
1- ´
6 6 If x = - (2 + 3 ), then c is negative.
p \ x = 1 + 3 is the only solution.
Þ a + b = ÐQPR =
4 1 1 1
13. Here, , , are in AP.
11. Let in the figure, 2 A 2 B C
sin sin sin 2
ÐA > ÐB > ÐC 2 2 2
C 1 1 1 1
Þ - = -
2C 2 B 2 B A
sin sin sin sin 2
2 2 2 2
b=9 a=10 ab ac
Þ -
(s - a )(s - b) (s - a )(s - c)
ac bc
= -
A c=x B (s - a )(s - c) (s - b)(s - c)

Since, angles are in AP. æ a ö é b(s - c) - c (s - b) ù


Þç ÷
\ ÐC = q - d , ÐB = q è s - a ø êë (s - b)(s - c) úû
and ÐA = q + d æ c ö é a (s - b) - b(s - a ) ù
=ç ÷
As ÐA + ÐB + ÐC = 180° è s - c ø êë (s - a )(s - b) úû
q + d + q + q - d = 180° 1 1 2
Þ ab + bc = 2ac Þ + =
Þ 3q = 180° c a b
Þ q = 60° Hence, a , b and c are in HP.
Properties of Triangles, Heights and Distances 245

b2 + c2 - a 2 Þ
3
=
sin B
14. Q Ð A = 60°, cos 60° =
2 bc 4 ´5 7
21
Þ b2 + c2 - a 2 = bc Þ sin B =
20
Now, AB2 + AC 2 = 2 ( AD 2 + BD 2)
2
which is not possible because its value is greater than
2 2 æaö
2 one.
Þ c + b = 2 AD + 2 ç ÷
è2ø 1 1 1
20. We have, ap1 = D, bp2 = D, cp3 = D
Þ 2 b2 + 2 c2 - a 2 = 4 AD 2 2 2 2
Þ b2 + c2 + bc = 4 AD 2 (Q b2 + c2 - a 2 = bc) where a , b and c are the sides of a triangle.
2 2 2 2D 2D 2D
15. Given that, cos A + cos C = sin B Þ p1 = , p2 = , p3 =
a b c
Obviously it is not an equilateral triangle because 1 1 1 a2 b2 c2 a 2 + b2 + c2
A = B = C = 60° does not satisfy the given condition. But \ + + = + + =
B = 90°, then sin 2 B = 1 and p12 p22 p32 4 D2 4 D2 4 D2 4 D2
æp ö 21. Let area of triangle be D,then according to the question
cos 2 A + cos 2 C = cos 2 A + cos 2 ç - A ÷
è2 ø 1 1 1
D= ax = by = cz
= cos 2 A + sin 2 A = 1 2 2 2
Hence, this satisfies the condition, so it is a right angled bx cy az b æ 2 D ö c æ 2 D ö a æ2 Dö
\ + + = ç ÷+ ç ÷+ ç ÷
triangle but not necessarily isosceles triangle. c a b cè a ø a è b ø b è c ø
b+ c a+ c a+ b 2 D(b2 + c2 + a 2)
16. Given, = = =k [say] =
11 12 13 abc
Þ b + c = 11k, c + a = 12k, a + b = 13k 2 (a 2 + b2 + c2) abc æ abc ö
= × çQ D = ÷
Þ 2 (a + b + c) = 36k abc 4R è 4R ø
Þ a + b + c = 18k a 2 + b2 + c2
=
Þ a = 7k, b = 6k, c = 5k 2R
b2 + c2 - a 2 36 + 25 - 49 1 C C
\ cos A = = = 22. (a - b)2 cos 2 + (a + b)2 sin 2
2bc 2 ´6 ´5 5 2 2
cos A cos B cos C 2 2 2C C
17. Given, = = = (a + b - 2 ab) cos + (a 2 + b2 + 2 ab) sin 2
k sin A k sin B k sin C 2 2
æ C C ö
Þ cot A = cot B = cot C Þ A = B = C = 60° = a 2 + b2 + 2 ab çsin 2 - cos 2 ÷
è 2 2ø
So, DABC is an equilateral triangle.
= a 2 + b2 - 2 ab cos C
3 2 3 1 3
\ D= a = ´ = sq unit = a 2 + b2 - (a 2 + b2 - c2)
4 4 6 24
æ a 2 + b2 - c2 ö
b2 + c2 - a 2 = c2 çQ cos C = ÷
18. Since, cos A = è 2 ab ø
2bc
Þ b2 - 2bc cos A + (c2 - a 2) = 0 23. In D ABD,
It is given that b1 and b2 are the roots of this equation. D C
2 2
Therefore, b1 + b2 = 2c cos A and b1b2 = c - a 120°
Þ 3b1 = 2c cos A and 2b12 = c2 - a 2 3
2
[Q b2 = 2b1]
2
æ 2c ö 60°
Þ 2 ç cos A ÷ = c2 - a 2
è3 ø A 5 B
2 2 2 2
Þ 8c (1 - sin A ) = 9c - 9a 22 + 52 - BD 2
2 2 cos 60° =
9a - c 2 (2)(5)
\ sin A =
8c2 Þ BD 2 = 19
3 CD 2 + 9 - 19
19. Given that, a = 5, b = 7 and sin A = Now, in D BCD, cos 120° =
4 (2)(3)(CD )
sin A sin B
As we know, = Þ CD 2 + 3 CD - 10 = 0 Þ CD = - 5, 2
a b
Þ CD = 2 (Q CD ¹ - 5)
246 JEE Main Mathematics

1 h
24. Given, tan f = 0.5 = D 26. In DABD, tan a =
2 2h
10 20 m C
In DABC, tan q =
AB
C p
10
Þ AB = D
tan q f 10 m
30 q
In DABD, tan(q + f ) = A B h
AB a
a
tan q + tan f 30 tan q
Þ = A 2h B
1 - tan q tan f 10
1
1 3 Þ tan a = …(i)
Þ tan q + = 3 tan q - tan 2 q 2
2 2 h+ p
In DABC, tan 2 a =
Þ 3 tan 2 q - 4 tan q + 1 = 0 2h
Þ (3 tan q -1) (tan q -1) = 0 2 tan a h+ p
Þ =
1 1 - tan 2 a 2h
Þ tan q = ,1
3 æ1ö
2ç ÷
\ tan q = 1 è2ø h+ p 4 h+ p
Þ 2
= Þ =
æ1ö 2h 3 2h
25. Let l be the length of the ladder 1-ç ÷
è2ø
i.e., BP = CQ
Þ 8h = 3h + 3 p
PA AB
In DPAB, cos a = and sin a = 5h
PB PB Þ 5h = 3 p Þ p = m
3
Þ PA = l cos a and AB = lsin a …(i)
27. (b) Let a first pole AB having height 20 m and second
AQ
In DQAC, cos b = Þ AQ = l cos b pole PQ having height 80 m
QC
and ÐPBQ = a, ÐAQB = b
and AC = lsin b …(ii) P
B

C 80 m
l
A
M
b a
Q x P A 20 m
h
Now, CB = AB - CA a b
B N Q
= l sin a - l sin b[from Eqs. (i) and (ii)] x y
= l (sin a - sin b ) and MN = hm is the height of intersection point from
and QP = AQ - PA the horizontal plane
= l cos b - l cos a h 80
Q tan a = =
= l (cos b - cos a ) x x+ y
CB l (sin a - sin b ) [in DMNB and DPQB] …(i)
\ =
QP l (cos b - cos a ) h 20
and tan b = =
æ a -bö æ a + bö y x+ y
2 sin ç ÷ cos ç ÷
y è 2 ø è 2 ø [in DMNQ and DABQ] …(ii)
Þ =
x æ a + bö æ a -bö From Eqs. (i) and (ii), we get
2 sin ç ÷ sin ç ÷
è 2 ø è 2 ø y
= 4 Þ y = 4x …(iii)
y æ a + bö x
Þ = cot ç ÷
x è 2 ø From Eqs. (i) and (iii), we get
æ a + bö h
=
80
Þ h=
80
= 16 m
Þ x = y tan ç ÷
è 2 ø x x + 4x 5
Properties of Triangles, Heights and Distances 247

28. From the diagram as ÐPRQ = 45° h


31. In DABC, tan 60° =
BC
\ PQ = RQ
Þ BC = h cot 60° …(i)
P h
In DABD, tan 30° =
BD
A
30°
60°
S 60º
T h
15º
30º
R U Q 30° 60°
D d C B
and ÐSPT = 30° and ÐRPQ = 45°
\ ÐPRS = ÐSPR Þ SR = SP = 1 km Þ BD = h cot 30°
SU 1 Þ BC + CD = h cot 30°
In DRSU , sin 30° = Þ SU = km
RS 2 Þ CD = h cot 30° - BC
\ Height of mountain = PT + TQ = PT + SU Þ d = h cot 30° - h cot 60°[from Eq. (i)]
3 1 3+1 1 \ Speed of car =
Distance from D to C
= + = km or km
2 2 2 3 -1 Time taken
d h cot 30°- h cot 60°
29. In DDAB, tan q =
64 = =
d 3 3
distance
C \ Time taken from C to B =
speed
1
´3
h cot 60° 3 3
= = = = 1.5 min
h (cot 30°- cot 60° ) æ 1 ö 2
ç 3- ÷
100 m

D 90° – q E 3 è 3ø
q
d BC
32. In DABC, tan 30° =
64 m AC
q 1 h - 150 60
Þ = Þ h - 150 =
A d B 3 60 3
Þ d = 64 cot q …(i) B
(100 - 64) 30°
In DCDE, tan (90°- q) =
d A 30° C
Þ d = 36 tan q …(ii) h
150 m
From Eqs. (i) and (ii), we get
d 2 = 36 ´ 64 Þ d = 48 m O 60 D
30. Let PD = h, R = 2 Þ h = (150 + 20 3 ) m
As angle of elevation of top of pole from A , B, C are
33. According to given information in question, from the
equal.
diagram, where PQ = MN = 200m and let CM = l m. So,
So, D must be circumcentre of DABC NC ¢ = (l + 200) m
P C
l
A M 30° P
p 60°
200 m 200 m
R
N
Q
R
R D
(l + 200) m
B C
æ p ö PD h æpö
tan ç ÷ = = Þ h = R tan ç ÷ = 2 3
è3ø R R è3ø C¢ (Image of C)
248 JEE Main Mathematics

So, in right angled triangle CMP, we have 36.


Main (P)
l
tan 30° = Þ p = 3l …(i)
p
and in right angle triangled PMC ¢, we have h tower
200 + l + 200
tan 60° =
p
30º 45º
Þ 3 p = 400 + l …(ii) A x B y Foot (Q)
From Eqs. (i) and (ii), we get
h
2l = 400 Þ l = 200m and p = 200 3m = tan 30°
x+ y
\ PC = l2 + p2 = 400 m.
x + y = 3h …(i)
34. Let height of tower TM be h. Also
P h
= tan 45°
y

45°
h=y
200 m 200 m
Put in Eq. (i), we get,
T
x + y = 3h
30° 90° 30° x = ( 3 - 1) y
Q M R x
= ‘v’ speed
TM 20
In DPMT, tan 45° =
PM \time taken to reach foot from B
h y
Þ 1= Þ PM = h Þ =
PM V
h x é x ù
In DTQM, tan 30° = ; QM = 3h = ´ 20
QM êQ y = ú
( 3 - 1)x ë 3 - 1û
In DPMQ, PM 2 + QM 2 = PQ 2
= 10( 3 + 1)
h 2 + ( 3h )2 = (200)2
37. Given that,
Þ 4h 2 = (200)2 Þ h = 100 m
3 2
1 cot a = and cot b =
35. In DADF, tan 60° = 5 5
AF
D
E D d C

h
1 km

60° a b
30° A 32 m B C
A F B
1 h
Þ AF = cot 60° = …(i) In DBCD, tan b =
3 BC
1 Þ BC = h cot b
In DABC, tan 30° =
AB
2h
Þ AB = cot 30° Þ BC = …(i)
5
Þ AF + FB = 3 h
1 2 and in DACD, tan a =
Þ d= 3- = [using Eq. (i)] 32 + BC
3 3
æ 2 hö 5
Distance from D to C Þ h = ç32 + ÷ [using Eq. (i)]
\ Speed of aeroplane = è 5 ø3
Time taken
2 Þ 3h = 160 + 2 h
Þ h = 160 m
= 3 ´ 60 ´ 60 = 240 3 km/h
10
Properties of Triangles, Heights and Distances 249

H D
38. In DABD, tan a =
d
D
q
b
H–h h E 2q
C

E a–b
C a
d H
h 2q
A B
a
A d B a a
In DABC, tan 2 q = = (QCE = AB )
AB CE
Þ d = H cot a …(i)
2 tan q a
H -h Þ = [from Eq. (i)]
In D ECD, tan (a - b ) = 1 - tan 2 q h - a
d tan q
H -h 2 tan q
Þ tan (a - b ) = [from Eq. (i)] Þ a tan q = (h - a ) ´
H cot a 1 - tan 2 q
Þ H [1 - cot a tan(a - b )] = h Þ a tan q (1 - tan 2 q) - 2 (h - a ) tan q = 0
h cot (a - b ) Þ tan q( a - a tan 2 q - 2 h + 2 a ) = 0
Þ H =
cot (a - b ) - cot a
Q tan q ¹ 0
39. Let AD be the building of height h and BP be the hill. -2 h + 3 a
\ tan 2 q =
Then, a
h+x -2 h + 3 a
tan q = …(i) Þ q = tan - 1
y a
x 2h
and tan p = = tan - 1 3 -
y a
Þ y = x cot p …(ii) 41. In DPMC,
P h-a
tan a =
PM
x
C
D p
C
h–a
h h h
q a
P M
A y B b
a
Q O
From Eqs. (i) and (ii), we get
h+x
tan q = h
x cot p
Þ x cot p = (h + x ) cot q
C'
h cot q
Þ x= Þ PM = (h - a ) cot a …(i)
cot p - cot q
h cot q h+a
Þ h+x= +h In DPMC ¢, tan b =
cot p - cot q PM
h cot p Þ h + a = PM tan b
\ Height of hill =
cot p - cot q Þ h = (h - a ) cot a tan b - a
DE
40. In D DEC, tan q = Þ h (1 - cot a tan b ) = - a (1 + cot a tan b )
CE
h-a a (sin a cos b + cos a sin b )
Þ tan q = Þ h=
CE sin b cos a - sin a cos b
h-a a sin (a + b )
Þ CE = …(i) = m
tan q sin (b - a )
250 JEE Main Mathematics

5 60
42. In DDBC, tan a = …(i) 44. In DABD, tan b =
x d
and in DDAC, D
30 a
tan 2 a =
x b

D x C 60
a E a C
a 5m
B d
h
30 m

b
25 m
A d B

E A Þ d = 60 cot b …(i)
2 tan a 30 DC
Þ = In DDEC, tan a =
1 - tan 2 a x EC
æ5ö Þ DC = d tan a
2ç ÷
è x ø 30 Þ 60 - h = d tan a (Q BC = EA = h )
Þ =
25 x
1- 2 Þ 60 - h = 60 cot b tan a [from Eq. (i)]
x
10x 30 æ cos b sin a ö
Þ = Þ h = 60 ç1 - × ÷
x2 - 25 x è sin b cos a ø
Þ 10x2 = 30x2 - 750 60 sin (b - a )
Þ h=
Þ 2
20x = 750 cos a sin b
75 60 sin (b - a ) 60 sin (b - a )
Þ x2 = Þ = (given)
2 x cos a sin b
3 x = cos a sin b
Þ x=5 m Þ
2
45. Let OP be the tower. Since, the tower make equal angles
43. Given that,
at the vertices of the triangle, therefore foot of tower is
4 5
tan q = and tan f = …(i) at the circumcentre.
3 2
A P
E
a
x h R
D q
d C150 m
O
a
a
R R
f B C
A d B
150 OP
In DABE, tan f = In DOAP, tan a =
d OA
2 OP = OA tan a
Þ d = 150 cot f = 150 ´ = 60 m …(ii) Þ
5
h Þ OP = R tan a (QOA = R, given)
In DDCE, tan q =
d 46. Since, Ð QPC = a
4 h a
Þ = [from Eq. (i)] \ Ð QPB = Ð BPC =
3 d 2
4 a r
Þ h = (60) [from Eq. (ii)] In DPQB, sin =
3 2 l
Þ h = 80 m a
Þ l = r sec …(i)
Now, in DDCE, DE 2 = DC 2 + CE 2 2
Þ x2 = 602 + 802 = 10000 h
and in DPOB, sin b =
l
Þ x = 100 m
Properties of Triangles, Heights and Distances 251

In DPAD and DPBD,


Q r
AD = h cot a = 3.2 h and BD = h cot b = 2.4 h
B
In DABD, AB2 = AD 2 + BD 2
l Þ 1002 = [(3. 2)2 + (2 .4)2] h 2 = 16 h 2
Ch 100
a Þ h= Þ h = 25 m
4
b 49. Let OP be the tower whose height is h metres.
P O
Þ h = lsin b P
a h
Þ h = r cosec sin b [from Eq. (i)]
2 b
B
a1 æ a + bö O
47. Q = tan ç ÷ …(i)
b1 è 2 ø
a d
a2 æb + g ö
and = tan ç ÷ …(ii) A
b2 è 2 ø
G OP
In DOAP, tan a =
OA
b1
F Þ OA = h cot a …(i)
OP
b2 In DOBP, tan b =
E OB
Þ OB = h cot b …(ii)
Now, in DOAB, AB2 = OA 2 + OB2
g b a Þ d 2 = h 2 (cot2 a + cot2 b )
A B C D [from Eqs. (i) and (ii)]
a2 a1
d
Þ h=
Since, a1a 2 = b1b2 cot2 a + cot2 b
a1 b2
Þ = 50. Let PD be a pole.
b1 a 2 P
DP
æ a + bö 1 In DDAP, tan 30° =
Þ tan ç ÷= AD
è 2 ø æb + g ö a
tan ç ÷ Þ DP =
è 2 ø 3
D
30°
æ a + bö æb + g ö DP 30°
Þ tan ç ÷ tan ç ÷=1 In DPDB, tan q = a a
è 2 ø è 2 ø BD A C
a/ 3 1 q
a+b b+ g p Þ tan q = = a a
\ + = 2a 6
2 2 2 B

Þ a + b + g = p -b < p 51. Let BC be the declivity and BA be the tower.


\ In DABC, on applying sine rule A
48. Let DP is clock tower standing at the middle point D BC AB Tower
of BC. = 75°
sin 75° sin 30°
30° C
Let Ð PAD = a = cot-1 3.2 Þ cot a = 3.2 80 sin 30° t
Þ AB = 75° 8 0f
P sin 75° 15°
C
40 ´ 2 2 B
= = 40 ( 6 - 2 ) ft
3 +1
h
10 0 m

D
52. Q Ð CDA = b, Ð DCB = g,
90° 90° and Ð ACB = p - (a + b + g)
a b C D
90°
g b
A 100 m B
-1
and Ð PBD = b = cosec 2.6 Þ cosecb = 2.6
a
\ cot b = (cosec2 b - 1 ) = 5.76 = 2.4 b g
A a B
252 JEE Main Mathematics

In DABC, on applying sine rule, we get Also, in DAMC, we have


AB CA MC
= tan 15° =
sin [p - (a + b + g )] sin g AM
a sin g Þ MC = AM tan 15°
Þ CA =
sin (a + b + g ) On substituting respective values, we get
æ 3 - 1ö
Now, in DCAD, on applying sine rule (h - 2500) = (h + 2500)ç ÷
CA CD è 3 + 1ø
=
sin b sin a h - 2500 3 -1
Þ =
CA × sin a h + 2500 3+1
Þ CD =
sin b Using componendo and dividendo, we have
a sin g sin a
= h 3
sin b sin (a + b + g ) =
- 2500 - 1
BE
53. In DABE, sin b = Þ h = 2500 3 m
b
D Round II
a 1. We have,
g- h2
b2 + c2 - a 2
g cos A =
2 bc
b F
a-b E
or c2 - 2 bc cos A + b2 - a 2 = 0
a h1
b
A 3
B C \ c1 + c2 = 2 b cos A = 2 ´ 2 ´ =2 3
2
Þ BE = h1 = b sin b
and c1c2 = b2 - a 2 = 4 - 5 = - 1
Using sine rule, in D AED
\ |c1 - c2| = (c1 + c2)2 - 4c1c2
sin (a - b ) sin (g - a ) b sin (a - b )
= Þ ED = = 12 + 4 = 16 = 4
ED b sin (g - a )
h2 b sin (a - b )sin g 2. Given, ÐA = 20°
Now, in DFED, sin g = Þ h2 =
ED sin (g - a ) \ ÐB = ÐC = 80°
\ Total height of the hill = CD = h1 + h2 Then, b=c
b sin (a - b )sin g a b c
= b sin b + \ = =
sin (g - a ) sin 20° sin 80° sin 80°

b [sin b sin(g - a ) + sin(a - b )sin g ] b sin a sin (g - b ) a b


= = Þ =
sin (g - a ) sin (g - a ) sin 20° cos 10°
Þ a = 2 b sin 10° …(i)
54. Let h be the height of the cloud above the surface of the
3 3 3 3 3
lake. The arrangement(s) given in the problem is/are \ a + b = 8b sin 10° + b
shown in the figure. = b3 {2 (4 sin3 10° ) + 1 }
C = b3 {2 (3 sin 10° - sin 30° ) + 1 }
A 15° M = b3 { 6 sin 10° } = 3b2(2 b sin 10° )
2500 m

2500 m

45° = 3b2a [from Eq.(i)]


h = 3ac2 (Q b = c )
3. Q A + B + C + D =2p
B O
Þ tan ( A + B + C + D ) = 0
C¢ h S tan A - S tan A tan B tan C
Þ =0
1 - S tan A tan B + tan A tan B tan C tan D
Now, in DAMC ¢, we have Þ S tan A - S tan A tan B tan C = 0
AM Þ S tan A = tan A tan B tan C tan D S cot A
tan 45° =
MC ¢ S tan A
Þ = Õ tan A
Þ AM = MC ¢ Þ AM = (h + 2500) …(i) S cot A
Properties of Triangles, Heights and Distances 253

4. (a + b + c)(b + c - a ) = kbc 1 p
7. a = area = bc × sin
2 4
Þ 2 s(2 s - 2 a ) = kbc
s(s - a ) k bc
æ Aö k p=
Þ = Þ cos 2 ç ÷ = a 2
bc 4 è2ø 4
a b c
æ A ö Also, = =
Q 0 < cos 2 ç ÷ < 1 p p 5p
è2ø sin sin sin
k 4 8 8
\ 0 < < 1 Þ0 < k < 4 p 5p
4 b = 2a sin × c = 2 a sin
8 8
a b c
5. We know that, = = = 2R p 5 p
sin A sin B sin C 2a 2 × sin × sin
p= 8 8 = a æç cos 4p - cos 6p ö÷
and given that, a + b = x, ab = y and x2 - c2 = y
a 2 2 è 8 8 ø
A
a æ 1 ö a
p= ç ÷=
2 è 2ø 2
8. b = ar , c = ar 2, where r > 1
c b
O
From the question,
R C = 2A
So, B = p - A - C = p - 3 A
B a C
a b c
\ = =
sin A sin B sin C
\ (a + b)2 - c2 = ab sin 3 A
\ r 2 = 2 cos A and r = = 3 - 4 sin 2 A = 4 cos 2 A - 1
Þ a 2 + b2 - c2 = - 2ab + ab sin A
Þ a 2 + b2 - c2 = - ab \ r = r 4 - 1 or r 4 = 1 + r
a + b - c2 - ab
2 2
1 5+1
Þ = =- Among 3 , 4 3 , we find 4 3 is the smallest
2ab 2ab 2 2
1 because
\ cos C = - Þ C = 120° 4 2
2 æ 5 + 1ö æ3 + 5 ö
( 3 )4 = 9, (4 3 )4 = 3, ç ÷ =ç ÷
a 2 + b2 - c2 è 2 ø è 2 ø
[using cosine rule, cos C = ]
2ab 14 + 6 5
c 1 c c 2 = >3
Now, = 2R Þ R = = 4
sin C 2 sin (120° ) 2 3
c 9. The diagram of given situation is
\ R= B
3
æ a b c ö
6. Applying sine rule in DABD, ç = = ÷ D
è sin A sin B sin C ø 15 m P
q 10 m
C h
D a
q A C
p2 Q
+ p
q2 Let height of P above the line AC is h m andAQ = x m
and CQ = y m, so
p–(q+a)
A 10 h
B = …(i)
x+ y x
2 2
AB p +q AB p2 + q 2 15 h
= Þ = and = …(ii)
sin q sin { p - (q + a )} sin q sin (q + a ) x+ y y
p2+ q2 sin q ( p2 + q2) sin q From Eqs. (i) and (ii), we get
Þ AB = = 10 y
sin q cos a + cos q sin a q sin q + p cos q = Þ 2x = 3 y …(iii)
15 x
é q p ù
êQ cos a = and sin a = ú Now, from Eqs. (i) and (iii), we get
êë p + q2
2
p2 + q2 úû 10 h 10
= Þ = hÞh =6m
( p2 + q2) sin q 2x x 5
= x+
p cos q + q sin q 3 3
254 JEE Main Mathematics

10. Given ABC is a triangular park with AB = AC = 100 m. 12. Let OP be the flag staff of height h standing at the
A vertical tower is situated at the mid-point of BC. Let centre O of the field.
the height of the tower is h m. P
Now, according to given information, we have the 45° 45°
following figure. D h C
F
Q
90° 15°
90° E
O
C
A B
h
In DOEP, OE = h cot 15° = h (2 + 3 )
and in DOFP,OF = h cot 45° = h
100 P \ EF = h 1 + (2 + 3 ) 2 = 2 h 2 + 3
l a
Since, AC = 1200
b
B
Þ 2 EF = 4h (2 + 3 )
10 0
A 300
From the figure and given information, we have Þ h= = 300 2 - 3 m
2+ 3
-1
b = cot (3 2 )
and a = cosec-1 (2 2 ) 13. Let the height of the vertical tower PQ = h , C is the
l middle point of line segment AB. Since, PQ is
Now, in DQPA, cot b = perpendicular to the plane QAB.
h
P
Þ l = (3 2 )h …(i) q B
and in DBPQ, h a
h BP f
tan a = Þ cot a = Q C
BP h q a
(100)2 - l2
= A
h
\ ÐPQA = ÐPQC = ÐPQB = 90°, we get
[Q p is mid-point of isosceles DABC, AP ^ BC]
PQ
Þ 2 2
h cot a = (100) - l 2 2 = tan q Þ QA = h cot q
QA
Þ h (cosec2a - 1) = (100)2 - (3 2h )2 [from Eq. (i)]
2
Similarly, QB = h cot q and QC = h cot f
Þ h 2(8 - 1) = (100)2 - 18h 2
Since, QA = QB, the DQAB is an isosceles triangle.
Þ 25h 2 = (100)2
2 Here, QCA is a right angled triangle in which
æ 100 ö Ð QCA = 90°.
Þ h2 = ç ÷ Þ h = 20 m
è 5 ø
\ QC 2 + AC 2 = QA 2
11. Let h be the height of the tower, then Þ h cot2 f + a 2 = h 2 cot2 q
2

h = AQ tan a = BQ tan b = CQ tan g Þ h (cot2 q - cot2 f ) = a 2


2

a2
P or h2 =
cot q - cot2 f
2

a2
h Þ h2 =
(cosec q - 1 ) - (cosec2 f - 1 )
2

a b g a2 a2
A Q = =
B C cosec q - cosec2 f
2 1
-
1
Þ BC = BQ - CQ = h (cot b - cot g ) sin q sin 2 f
2

Þ CA = h (cot a - cot g ) a 2 sin 2 q sin 2 f


Þ h2 = = sin(q + f )
and AB = h (cot a - cot b ) sin 2 f - sin 2 q
Now, BC cot a - CA cot b + AB cot g Hence, the required height h of the peak
= h [cot a (cot b - cot g ) - cot b (cot a - cot g ) a sin q sin f
=
+ cot g (cot a - cot b )] sin (f + q) sin (f - q)
=0
Properties of Triangles, Heights and Distances 255

14. Let H be the mid-point of BC. h


17. Let AB = h, then AP = 2h and AC = BC =
S R 2
Since, Ð TBH = 90° , then
Again, let ÐCPA = a
TH 2 = BT 2 + BH 2 = 52 + 52 = 50 P Q B
Also since, G
D h/2
Ð THG = 90° , TG 2 = TH 2 + GH 2 C
q H h C
= 50 + 25 = 75
A T B h/2 b
Let q be the required angle of
a
elevation of G at T. A P
GH 5 1 2h
Then, sin q = = =
TG 5 3 3 AB h 1
Now, in DABP, tan (a + b ) = = =
-1 AP 2h 2
Þ q = sin (1 / 3 )
h
15. Let PQ and RS be the poles of height 20 m and 80 m
AC 1
subtending angles a and b at R and P, respectively. Also, in DACP, tan a = = 2 =
AP 2h 4
Leth be the height of the point T, the intersection of QR
Now, tan b = tan[(a + b ) - a ]
and PS.
1 1 1
PR = h cot a + h cot b -
Then, tan(a + b ) - tan a 2 4 2
= = =4 =
= 20 cot a = 80 cot b 1 + tan(a + b ) tan a 1 + 1 ´ 1 9 9
cot a 2 4 8
Þ cot a = 4 cot b Þ =4
cot b 18. According to the given information, the figure should be
S as follows.
Let the height of tower = h
E
80 m Q
T
20 m
a h b
R h
V P
Again, h cot a + h cot b = 20 cot a
Þ (h - 20) cot a = - h cot b 30° 45° 60°
cot a h A B C D
Þ = =4
cot b 20 - h ED
In DEDA, tan 30° =
Þ h = 80 - 4h Þ h = 16 m AD
1 ED h
16. In DOA1 B1 , = =
3 AD AD
AB1 20
tan 45° = Þ = 1 Þ OB1 = 20 Þ AD = h 3
OB1 OB1
h
20 In DEDB, tan 45° = Þ BD = h
In DOA2 B2, tan 30° = BD
OB2
In DEDC,
Þ OB2 = 20 3 h h
tan 60° = Þ CD =
Þ B1B2 + OB1 = 20 3 CD 3
Þ B1B2 = 20 3 - 20 AB AD - BD
Now , =
A1 A2 BC BD - CD
AB h 3 - h
Þ =
BC h
20 m 20 m h-
45°
3
30° AB 3 -1
O 20 B1 B2 Þ = ´ 3
BC ( 3 - 1)
Þ B1B2 = 20 ( 3 - 1) m AB 3
Distance 20 ( 3 - 1) Þ =
\ Speed = = BC 1
Time 1 \ AB : BC = 3 : 1
= 20 ( 3 - 1) m /s
256 JEE Main Mathematics

19. In DABC , BC = h cot 60° 25 25 5a


\ Area of DABC = 3 × = =
9 3 b
A
Þ a = 5 and b = 3
\ a + b =5 + 3 =8
h
22.
A
45° 60° 5 12
D B
7m C

and in DABD, BD = h cot 45° B C

Since, BD - BC = DC 1
D= × 5 × 12 sin A = 30
Þ h cot 45° - h cot 60° = 7 2
7 7 sin A = 1
h= =
cot 45° - cot 60° æ 1 ö A = 90°
ç1 - ÷
è 3ø Þ BC = 13
7 3 3+1 BC = 2R = 13
= ´
3 -1 3+1 D
r=
7 3 S
= ( 3 + 1)m 30
2 = =2
15
20. Let h be the height of a tower
2R + r = 15
Since, ÐAOB = 60°
\DOAB is an equilateral. 23. (4) In DABC , by sine rule,
\ OA = OB = AB = a a 2 2 4
= =
C sin A sin 30° sin C
h Þ C = 45° , C ¢ = 135°
O 90° When, C = 45° Þ A = 180° - (45° + 30° ) = 105°
a 60° a
30° When, C ¢ = 135° Þ A = 180° - (135° + 30° ) = 15°
A
a A
B
h a 15°
In DOAC , tan 30° = Þ h= 2Ö2 4
2Ö2
a 3
45° 135°
21. (8) Let O be the point of intersection of the medians of C
30°
B
triangle ABC. Then, the area of DABC is three times C¢
that of DAOC. 1
Area of DABC = AB ´ AC sin A
2 10 2
Now, in DAOC, AO = AD = .
3 3 1
= ´ 4 ´ 2 2 sin (105° )
Therefore, applying the sine rule to DAOC, we get 2
OC AO 3+1
= =4 2 ´
sin (p / 8) sin (p / 4) 2 2
10 sin (p / 8) = 2 ( 3 + 1) sq units
Þ OC = × 1
3 sin (p / 4) Area of DABC ¢ = AB ´ AC ¢ sin A
1 2
Area of DAOC = × AO × OC × sin ÐAOC 1
2 = ´ 4 ´ 2 2 sin (15° )
1 10 10 sin (p / 8) 2
æp pö
= × × × × sin ç + ÷ = 2 ( 3 - 1) sq units
2 3 3 sin (p / 4) è2 8ø
50 sin (p / 8) cos (p / 8) Difference of areas of triangle
= × = |2 ( 3 + 1) - 2 ( 3 - 1)|
9 sin (p / 4)
50 25 = 4 sq units
= =
18 9
Properties of Triangles, Heights and Distances 257

24. (80) According to the given situation if we draw the 25. (5) According to given information, we have the
diagram. following figure
A D

h–x
Pillar h
h

E 75º 60º
F 30º
m
80 A x B y C
45º x
30º
C
y D B Now, from Right DACD and Right DBCD, we have
h h
tan 30° =
x+ y
Q ÐACB = 45°
h
\ AB = BC = h [let] and tan 60° =
y
Now, from the diagram
x+ y
sin 30° =
x
(in DCDE) Þ h= ...(i)
80 3
Þ x = 40 m and h= 3 y ...(ii)
y From Eqs. (i) and (ii),
and cos 30° = x+ y
80 = 3 y
Þ y = 40 3 m 3
Now, in DAEF, Þ x +y = 3y
h-x Þ x - 2y = 0
tan 75° =
h-y Þ y=
x
3+1 h - 40 2
Þ = Q Speed is uniform.
3 - 1 h - 40 3
\ Distance y will be cover in 5 min.
Þ 3h + h - 120 - 40 3
Q Distance x covered in 10 min.
= 3h - h - 40 3 + 40 x
\ Distance will be cover in 5 min.
Þ 2h = 160 Þ h = 80 m 2
11
Cartesian
Coordinate System
Cartesian System of Rectangular Axes IN THIS CHAPTER ....
The position of a point in a plane is determined with reference to two Cartesian System of
intersecting straight lines called the coordinate axes and their point of Rectangular Axes
intersection is called the origin. If these two axes of reference (generally, Cartesian Coordinates in
we call them X and Y -axes) cut each other at right angle, then they are called a Plane Point
rectangular axes otherwise they are called oblique axes. Distance Formula
The horizontal axes, OX and OX ¢ are called positive and negative directions Area of Some
respectively of X-axis and OY andOY ¢ are called positive and negative Geometrical Figures
directions respectively of Y -axis.
Different Points Related to
Positive

Y a Triangle
Translate of Axes
Negative Positive
X¢ X
O
Negative

Quadrants
Let X ¢ OX and Y ¢ OY be the coordinate axes. We observe that the two axes
divide the Euclidean plane into four regions called quadrants. The regions
XOY , X ¢ OY , X ¢ OY ¢ and XOY ¢ are respectively known as first, second, third
and fourth quadrants. The ray OX is taken as positive X-axis, OX ¢ as negative
X-axis. Also, the ray OY is called positive Y -axis and OY ¢ as negative Y -axis.
Y
(–, +) (+, +)

X¢ X
O
(–, –) (+, –)

Cartesian Coordinate System 259

Sign of Coordinates in a Quadrant Example 1. ABC is an isosceles triangle. If the


Quadrant x y coordinates of the base are B (1, 3) and C ( - 2 , 7), the
I. XOY >0 >0 coordinates of vertex A, is
II. X ¢OY <0 >0 æ 1 ö
(a) ç - , - 5÷ (b) (1,6)
III. X ¢OY ¢ <0 <0 è 2 ø
IV. XOY ¢ >0 <0 æ5 ö
(c) ç , 6÷ (d) None of these
è6 ø
Coordinates of Any Point on Coordinate Axes
Sol. (c) Let coordinates of vertex A be ( x, y).
Let X ¢ OX and Y ¢ OY be two perpendicular axes in the
plane paper intersecting at O and P be any point in the A (x, y)
plane.
Y

P(x, y)
B (1, 3) C (–2, 7)
y
\ AB = ( x - 1) 2 + (y - 3) 2
X¢ X [using distance formula]
O x M
AC = ( x + 2) 2 + (y - 7) 2

Since, DABC is isosceles, therefore
Draw PM perpendicular to OX. Let OM = x , PM = y. AB2 = AC 2
Then, the position of the point P in the plane with respect
to coordinate axes is represented by the ordered pair ( x , y ). Þ ( x - 1) 2+ (y - 3) 2 = ( x + 2) 2 + (y - 7) 2
The order pair ( x , y ) is called the coordinates of point P, Þ x + 1 - 2x + y 2 + 9 - 6y = x2 + 4 + 4x + y 2 + 49 - 14y
2

i.e. OM = x = x-coordinate or abscissa of the point P. Þ 6x - 8y + 43 = 0 …(i)


and PM = y = y-coordinate or ordinate of the point P. Only option (c) satisfy Eq. (i),
5
Note as 6 ´ - 8 ´ 6 + 43 = 5 - 48 + 43 = 0
6
• The ordinate of every point on X-axis is 0.
Hence, option (c) is correct.
• The abscissa of every point on Y-axis is 0.
• The abscissa and ordinate of the origin O are both zero. Example 2. If the points (0 , 0), (2 , 2 3) and ( a, b) are
the vertices of an equilateral triangle, then ( a, b) is
Distance Formula (a) (0 , - 4) (b) (0 , 4) (c) ( 4 , 0) (d) ( - 4 , 0)
The distance between any two points in the plane is the Sol. (c) Let O (0, 0), A(2, 2 3) and B ( a, b) be the vertices of an
length of the line segment joining them. equilateral triangle, then
Y OA = (2) 2 + (2 3) 2 = 4 + 12 = 16 = 4
AB = ( a - 2) 2 + ( b - 2 3) 2
Q(x2, y2)
= a2 + 4 - 4a + b 2 + 12 - 4 3b
P(x1, y1) = a2 + b 2 - 4a - 4 3 b + 16

X¢ X and OB = a2 + b 2
O
Y¢ Since, D ABC is equilateral.
\ OA = OB = AB
D = ( x2 - x1 )2 + ( y2 - y1 )2 On taking first two equalities, we have
OA = OB
(difference of abscissa)2
or D= 4 = a2 + b 2 Þ a2 + b 2 = 16 …(i)
+ (difference of ordinates)2
On taking last two equalities, we have
If O is the origin and P ( x1 , y1 ) is any point, then from the OB = AB
distance formula, we have
Þ a2 + b 2 = a2 + b 2 - 4a - 4 3 b + 16
2 2 2 2
OP = ( x - 0) + ( y - 0) = x + y Þ a2 + b 2 = a2 + b 2 - 4a - 4 3 b + 16
If the line PQ is parallel to Y -axis, then PQ =| y2 - y1|. Þ 4a + 4 3 b - 16 = 0
If the line PQ is parallel to X-axis, then PQ =| x2 - x1|. Þ a+ 3 b-4 =0 …(ii)
260 JEE Main Mathematics

On using Eq. (ii) in Eq. (i), we have Example 4. Suppose A(1, 1) and B(2, - 3) are two points
( 4 - 3 b) 2 + b 2 = 16 and D is a point on AB produced such that AD = 3AB. Then,
Þ 2
16 + 3 b - 8 3 b + b = 16 2 coordinates of D are
Þ 4 b2 - 8 3 b = 0 (a) (3, - 12) (b) ( 4, - 11) (c) (11, 4) (d) ( -11, 7)
Þ 4 b ( b - 2 3) = 0 Sol. (b) Given, AD = 3AB
Þ b = 0 or b = 2 3
A(1, 1) B(2, –3) D
Now, when b = 0 Þ a = 4 and when b = 2 3 Þ a = -2
\ Coordinates may be (4, 0) or ( -2, 2 3). \ BD = 2AB
Thus, D divides AB externally in the ratio AD : BD = 3 : 2.
Section Formula é 3(2) - 2(1) 3( -3) - 2(1) ù
\The coordinates of D are ê , ú
If a point P ( x , y ) divides the line segment joining two ë 3 -2 3 -2 û
points A ( x1 , y1 ) and B ( x2 , y2 ) in the ratio m : n, then æ 6 - 2 -9 - 2 ö
=ç , ÷ = ( 4, - 11)
coordinates of point P are è 1 1 ø
(i) when it divides AB internally
n
B Area of Some Geometrical Figures
m P (x 2, y 2)
A (x, y) Area of Triangle
(x 1, y 1) Let A( x1 , y1 ), B ( x2 , y2 ) and C( x3 , y3 ) be the coordinates of
mx2 + nx1 my2 + ny1
x= ,y= the vertices of DABC.
m+n m+n
A(x1, y1)
(ii) when it divides AB externally
n
P
m B (x, y)
(x 2, y 2)
A
(x 1, y 1)
mx2 - nx1 my2 - ny1
x= ,y= B(x2, y2) C(x3, y3)
m-n m-n
Note 1
\ Area of D ABC = |[x1( y2 - y3 ) + x2( y3 - y1 ) + x3 ( y1 - y2 )]|
• The coordinates of the points of trisection P and Q of the line 2
2 x + x2 2 y1 + y2 ö
segment joining A( x1, y1 ) and B( x2 , y2 ) are æç 1 , ÷ Condition of Collinearity
è 3 3 ø
æ x1 + 2 x2 y1 + 2 y2 ö Three points are said to be collinear, if area of triangle is
and ç , ÷ , respectively.
è 3 3 ø zero.
• X-axis divides the line joining ( x1, y1 ), ( x2 , y2 ) in the ratio y1 : y2 and 1
i.e. |[x1 ( y2 - y3 ) + x2 ( y3 - y1 ) + x3 ( y1 - y2 )]| = 0
Y-axis divides the same line segment in the ratio x1 : x2 . 2
The mid-point of the line joining P ( x1, y1 ) and Q ( x2 , y2 ) is
æ x1 + x2 , y1 + y2 ö . Area of Trapezium
ç ÷
è 2 2 ø If A( x1 , y1 ), B ( x2 , y2 ), C( x3 , y3 ) and D( x4 , y4 ) are the
vertices of a trapezium, then
Example 3. The ratio in which y - x + 2 = 0 divides the
A(x1, y1) B(x2, y2)
line joining (3, - 1) and (8, 9) is
(a) 2 : 3 (externally) (b) 2 : 3 (internally)
(c) 3 : 2 (externally) (d) 3 : 2 (internally)
Sol. (b) Let the required ratio be l :1, then point of division is
æ 8l + 3 9l - 1ö C(x3, y3) D(x4, y4)
ç , ÷ . Now, this point must satisfy the equation
è l + 1 l + 1ø
1
of the line y - x + 2 = 0. Area of trapezium = (Sum of parallel sides)
æ 9l - 1ö æ 8l + 3 ö 2 2
Þ ç ÷-ç ÷ + 2 =0 Þ l = ´ (Distance between parallel sides)
è l + 1ø è l + 1 ø 3
1
2
So, the required ratio is : 1 i. e. ,2 : 3 (internally) since, l is = |[( y1 + y2 ) ( x1 - x2 ) + ( y2 + y3 ) ( x2 - x3 )
3 2
positive. + ( y3 + y4 ) ( x3 - x4 ) + ( y4 + y1 ) ( x4 - x1 )]|
Cartesian Coordinate System 261

|7x + 7y - 14|
Area of Quadrilateral =
| 42 + 15 - 8 |
If A( x1 , y1 ), B ( x2 , y2 ), C( x3 , y3 ) and D( x4 , y4 ) are the 7| x + y - 2|
vertices of a quadrilateral ABCD, then =
| 49 |
x1 y1 x+ y -2
=
x2 y2 7
1
Area of quadrilateral = x3 y3
2 Example 6. The area of the pentagon whose vertices are
x4 y4 A (1, 1), B ( 7, 21), C ( 7, - 3) , D (12 , 2) and E (0, - 3) , is
x1 y1 137 137
(a) sq units (b) sq units
1 2 3
= |{( x1 y2 + x2 y3 + x3 y4 + x4 y1 ) 137
2 (c) sq units (d) None of these
4
- ( y1x2 + y2x3 + y3 x4 + y4x1 )}|
x1 y1 1 1
B(x2, y2) x2 y 2 7 21
A(x1, y1)
1 x3 y3 1 7 -3
Sol. (a) The required area = =
2 x4 y 4 2 12 2
x5 y5 0 -3
x1 y1 1 1
C(x3, y3) D(x4, y4)
1
= |[(21 - 21 + 14 - 36 + 0) - (7 + 147 - 36 + 0 - 3)]|
2
Area of Polygon 137
= sq units
The area of polygon whose vertices are ( x1 , y1 ), 2
( x2 , y2 ), ( x3 , y3 ), K , ( xn , yn ) is given by
Example 7. Let A (1, 2), B (3 , 4) be two points and C ( x, y)
x1 y1
be a point such that ( x -1) ( x - 3) + (y - 2) (y - 4) = 0. If area of
x2 y2 D ABC is 1 sq unit, then the maximum number of positions of
1 x3 y3 = 1 |{( x1 y2 + x2 y3 + K + xn y1 ) C in the XY-plane, is
2 M M 2 (a) 2 (b) 4
- ( y1x2 + y2x3 + K + yn x1 )}| (c) 8 (d) None of these
xn yn
x1 y1 Sol. (b) Given, vertices of triangle are A (1, 2), B (3, 4) and C ( x, y).
\ Area of triangle = 1sq unit
Example 5. The coordinates of A, B and C are (6, 3), 1
\ D = |[ x1 (y 2 - y3) + x2 (y3 - y1) + x3 (y1 - y 2)]| = 0
( -3, 5) and ( 4, - 2) respectively and P is any point ( x, y). 2
1
Then, the ratio of the area of DPBC and DABC is Þ |1( 4 - y) + 3 (y - 2) + x (2 - 4)| = 1
2
x+y +2 x+ y -2 1
(a) (b) Þ |4 - y - 6 + 2x + 3y - 4x| = 1
7 49 2
x+ y -2 1
(c) (d) None of these Þ |- 2x + 2y - 2 | = 1
7 2
Þ -2x + 2y - 2 = ± 2
Sol. (c) We have, Þ x - y = - 2 or x - y = 0
1 Case I When x = (y - 2), then
|[ x (5 + 2) - 3( -2 - y) + 4( y - 5)]|
Area of D PBC 2 ( x - 1)( x - 3) + (y - 2) (y - 4) = 0
=
Area of D ABC 1 |[6(5 + 2) - 3( - 2 - 3) + 4(3 - 5)]| Þ (y - 2 - 1) (y - 2 - 3) + (y - 2) (y - 4) = 0
2 Þ (y - 3) (y - 5) + (y - 2) (y - 4) = 0
A (6, 3) Þ y 2 - 8y + 15 + y 2 - 6y + 8 = 0
Þ 2y 2 - 14y + 23 = 0
P (x, y)
7± 3
\ y=
2
Hence, coordinates of point C are
æ3 + 3 7 + 3 ö æ7 - 3 3 - 3 ö
B (–3, 5) C (4, –2) ç , ÷ or ç , ÷.
è 2 2 ø è 2 2 ø
262 JEE Main Mathematics

Case II When x - y = 0, then Circumcentre of a Triangle


( x - 1) ( x - 3) + (y - 2) (y - 4) = 0
Þ ( x - 1) ( x - 3) + ( x - 2) ( x - 4) = 0
The circumcentre of a triangle is the point of intersection
Þ x2 - 4x + 3 + x2 - 6x + 8 = 0 of the perpendicular bisectors of the sides of a triangle.
The distance between the circumcentre and any vertex of
Þ 2x2 - 10 x + 11 = 0
triangle is called the circumradius of the circumscribed
5± 3 circle.
\ x=
2
A(x1, y1)
Hence, coordinates of point C is
æ5 + 3 5 + 3 ö æ5 - 3 5 - 3 ö
ç , ÷ or ç , ÷.
è 2 2 ø è 2 2 ø
O
Thus, there are four positions of point C.
B(x2, y2) C(x3, y3)
Different Points Related to a Triangle
Let A( x1 , y1 ), B( x2 , y2 ) and C( x3 , y3 ) be the vertices of
Centroid of a Triangle D ABC, then the coordinates of circumcentre of triangle is
The point of intersection of the medians of a triangle is given by using OB = OA = OC
called the centroid of the triangle.
Let A( x1 , y1 ), B( x2 , y2 ) and C( x3 , y3 ) are the vertices of a æ ax cos A + bx2 cos B + cx3 cos C
Cç 1 ,
D ABC, then coordinates of centroid are given by è a cos A + b cos B + c cos C
æ x1 + x2 + x3 y1 + y2 + y3 ö ay1 cos A + by2 cos B + cy3 cos C ö
ç , ÷. ÷
è 3 3 ø
a cos A + b cos B + c cos C ø
A(x1, y1)
æ x sin 2 A + x2 sin 2B + x3 sin 2C
or C ç 1 ,
è sin 2 A + sin 2B + sin 2C
y1 sin 2 A + y2 sin 2B + y3 sin 2C ö
÷
sin 2 A + sin 2B + sin 2C ø
where a , b and c are the sides of a triangle.
B(x2, y2) (x3, y3)C
Orthocentre of a Triangle
Incentre of a Triangle The orthocentre of a triangle is the point of intersection of
The point of intersection of internal angle bisectors of altitudes (i.e., the lines through the vertices and
triangle is called the incentre of the triangle. If we draw perpendicular to opposite sides. The coordinates of
a perpendicular on each side of a triangle through the orthocentre of a triangle whose vertices are
incentre I, then they all will be of equal lengths, these A( x1 , y1 ), B( x2 , y2 ) and C( x3 , y3 ) are given as
are called in radius of inscribed circle.
æ x tan A + x2 tan B + x3 tan C
A(x1, y1) Oç 1 ,
è tan A + tan B + tan C
y1 tan A + y2 tan B + y3 tan C ö
A/2
A/2

÷
c F E b tan A + tan B + tan C ø
l A(x1, y1)
B/2 C/2
B/2 C/2
B(x2, y2) D (x3, y3)C
a M

Let A( x1 , y1 ), B( x2 , y2 ) and C( x3 , y3 ) be the vertices of O


D ABC, then coordinates of incentre of triangle are
given by P
B(x2, y2) (x3, y3)C
æ ax1 + bx2 + cx3 ay1 + by2 + cy3 ö
ç , ÷ If equation of lines are given, then firstly determine the
è a+ b+ c a+ b+ c ø
two altitudes of a triangle
where a , b and c are the sides of the triangle.
Cartesian Coordinate System 263

Equation for AP Sol. (b) Given vertices of triangle ABC are A(1, 0), B(6, 2)
a1x + b1 y + c1 a2x + b2 y + c2 æ3 ö
= and C ç , 6÷ and the centroid of triangle G divides the
a1a3 + b1b3 a2a3 + b2b3 è2 ø
and for a BM triangle such that area of triangles AGB, BGC and CGA are
a1x + b1 y + c1 a3 x + b3 y + c3 equal, so point P is the centroid of triangle ‘G’.
=
a1a2 + b1b2 a2a3 + b2b3 æ 1 + 6 + 3 /2 0 + 2 + 6 ö æ17 8 ö
\ Coordinate of P ç , ÷ = Pç , ÷
After solving equations for AP and BM, we get the è 3 3 ø è 6 3ø
orthocentre. æ 7 1ö
and given point Q ç - , - ÷ .
è 6 3ø
A So, length of the line segment PQ
2 2
a2

æ17 7 ö æ 8 1ö
=0

= ç + ÷ +ç + ÷
cx

è6 6ø è3 3ø
c1

+b

M
y+

2
cy

2 2
b1

æ 24 ö æ9ö
+

= ç ÷ +ç ÷
x+

c2

è6ø è3ø
a1

=0

B P C = 4 2 + 3 2 = 16 + 9 = 25 = 5
a3x + b3y + c3 = 0

Example 9. The x-coordinate of the incentre of the


Excentre of a Triangle triangle that has the coordinates of mid-points of its sides as
Let A( x1 , y1 ), B( x2 , y2 ) and C( x3 , y3 ) be the vertices of a (0, 1), (1, 1) and (1, 0) is (JEE Main 2013)
DABC. Let a , b, c be the lengths of the sidesBC , CA (a) 2 + 2 (b) 2 - 2 (c) 1 + 2 (d) 1 - 2
and AB or a , b and c be the lengths of the sides opposite Sol. (b) Given, mid-points of a triangle are (0, 1), (1, 1) and (1, 0).
to ÐA, ÐB and ÐC respectively, then the circle which
Plotting these points on a graph paper and make a triangle.
touches the side BC and two sides AB and AC produced
is called the escribed circle opposite to the ÐA. The So, the sides of a triangle will be 2, 2 and 2 2 + 2 2 = 2 2.
bisector of the external angles B and C meet at E1 (which Y
is the centre of the escribed circle) such that
C (0,2)
æ - ax1 + bx2 + cx3 - ay1 + by2 + cy3 ö
E1 ç , ÷
è -a + b + c -a + b + c ø (1, 1)
2 (0, 1)
æ ax - bx2 + cx3 ay1 - by2 + cy3 ö
Similarly, E2 ç 1 , ÷
è a- b+ c a- b+ c ø X¢
B (1, 0) A (2, 0)
X
(0, 0) 2
æ ax + bx2 - cx3 ay1 + by2 - cy3 ö
and E3 ç 1 , ÷ Y¢
è a+ b- c a+ b- c ø 2 ´ 0 + 2 2 ×0 + 2 ×2
x-coordinate of incentre =
2+2+2 2
E2 2 2- 2
A = ´ =2 - 2
E3 2+ 2 2- 2

C B
Example 10. If G be the centroid and I be the incentre of
the triangle with vertices A( -36, 7 ), B(20, 7 ) and C(0, - 8) and
25
GI = 205 l , then the value of l is
E1 3
(a) 1/25 (b) 1/5
(c) 1/125 (d) None of these
æ3 ö Sol. (a) Coordinates of centroid are
Example 8. Let A(1, 0), B(6, 2) and C ç , 6 ÷ be the vertices æ -36 + 20 + 0 7 + 7 - 8 ö
è2 ø Gºç , ÷
è 3 3 ø
of a D ABC. If P is a point inside the D ABC such that the
triangles APC, APB and BPC have equal areas, then the length æ 16 ö
i. e. , G º ç- , 2÷
æ 7 1ö è 3 ø
of the line segment PQ, where Q is the point ç - , - ÷, is
è 6 3ø and a = BC = (0 - 20) 2 + ( - 8 - 7) 2 = 25
(JEE Main 2020)
b = AC = (0 + 36) 2 + ( - 8 - 7) 2 = 39
(a) 2 (b) 5 (c) 7 (d) 4
264 JEE Main Mathematics

and c = BA = ( -36 - 20) 2 + (7 - 7) 2 = 56 Example 12. IfA ( a, 0) and B ( - a, 0) are two fixed points
Therefore, the coordinates of I are and a point P moves such that ÐAPB = 90 °, then locus of P is
é 25 ´ ( -36) + 39 ´ 20 + 56 ´ 0 25 ´ 7 + 39 ´ 7 + 56 ´ ( - 8) ù (a) x2 + y 2 = 2a2 (b) x2 + y 2 = a2
ê , ú
ë 25 + 39 + 56 25 + 39 + 56 û 2 2
(c) x + y + 2a = 0 2
(d) None of these
\ I º ( -1, 0)
Sol. (b) Let coordinates of point P be (h, k).
2
æ 16 ö 2 Given, ÐAPB = 90°
GI = ç -1 + ÷ + (0 - 2)
è 3ø Y
1 P (h, k)
= 205
3
25
But given, GI = 205 l
3 B A X

1 25 (–a, 0) O (a, 0)
\ 205 = 205 l
3 3
1
Þ l=
25 Y¢

By Pythagoras theorem, we have


Locus AB2 = PA2 + PB2
The curve described by any point which moves under
Þ (2a) 2 = (h - a) 2 + k2 + (h + a) 2 + k2
given conditions is called locus of the point.
Þ 4a2 = h 2 + a2 - 2ah + h 2 + a2 + 2ah + 2k2
Equation of Locus
Þ 2a2 = 2h 2 + 2k2
A relation f ( x , y ) = 0 between x and y which is satisfied
by each point on the locus and such that each point Þ h + k2 = a2
2

satisfying the equation is on the locus is called the \ x2 + y 2 = a2


equation of the locus. which is the required locus.
Example 11. Let O(0, 0) and A(0, 1) be two fixed points,
then the locus of a point P such that the perimeter of DAOP is Translation of Axes
4, is (JEE Main 2019)
Shifting of Origin
(a) 8x2 - 9y 2 + 9y = 18 (b) 9x2 - 8y 2 + 8y = 16
(c) 9x2 + 8y 2 - 8y = 16 (d) 8x2 + 9y 2 - 9y = 18 Y¢
Y
P
Sol. (c) Given vertices of DAOP are O(0 , 0) and A(0 , 1.
) x¢ (x¢, y ¢)
P
Let the coordinates of point P are ( x, y).
y¢ (x, y )
Clearly, perimeter = OA + AP + OP = 4 (given)
h

Þ (0 - 0) 2 + (0 - 1) 2 + (0 - x) 2 + (1 - y) 2 + x2 + y 2 = 4 y O¢ (h, k)

Þ 1+ x2 + (y - 1) 2 + x2 + y 2 = 4 k
2 2 2 2
Þ x + y - 2y + 1 + x + y =3 O x X
2 2 2 2 (0, 0)
Þ x + y - 2y + 1 = 3 - x + y
Þ x + y - 2y + 1 = 9 + x2 + y 2 - 6 x2 + y 2
2 2 Let P ( x , y ) be any point with respect to the origin O.
[squaring both sides] Now, the origin O ( 0, 0) is shifted to a new point say
O ¢ ( h , k), then the coordinates of same point P with
Þ 1 - 2y = 9 - 6 x2 + y 2
respect to the new origin O¢ be P ¢ ( x ¢ , y ¢ ) in such a way
Þ 6 x2 + y 2 = 2y + 8 that
Þ 3 x2 + y 2 = y + 4 x = x¢ + h
Þ 2 2
9( x + y ) = (y + 4) 2
[squaring both sides] and y = y¢ + k
Þ 2 2 2
9x + 9y = y + 8y + 16 Hence, if the origin is shifted to ( h , k) without rotation of
axes, then the new equation of the curve can be obtained
Þ 9x2 + 8y 2 - 8y = 16
by putting ( x + h ) and ( y + k) in place of x and y,
Thus, the locus of point P( x, y) is
respectively.
9x2 + 8y 2 - 8y = 16
Cartesian Coordinate System 265

Rotation of Axes without Changing Origin Double Translation


Let OX and OY be the axes. Let OX ¢ and OY ¢ be the new (Origin Shifted and Axes Rotated)
axes obtained by rotating the original axes through an If origin is shifted to the point O ¢ ( h , k) and at the same
angle q. time the directions of axes are rotated through an angle q
Y in the anti-clockwise sense such that new coordinates of
Y¢ P ( x , y ) become ( X , Y ).
P
Y
Y Y¢
q X¢
yQ P (x, y)
N (X, Y)
q X
q X¢
q X
O x M L q

Let P be a point in the plane whose coordinates are ( x , y ) X
O
and ( X , Y ) referred to old and new axes, respectively.
Now, x = OL - ML = X cos q - Y sin q Then, x = h + X cos q - Y sin q
and y = PQ + NL = Y cos q + X sin q and y = k + X sin q + Y cos q
or above can be written in matrix form as
If it is required to shift the new coordinates axes back,
then replace X by x cos q + y sin q and Y by é x ù é cos q - sin q ù é X ù é h ù
ê y ú = ê sin q cos q ú êY ú + ê k ú
- x sin q + y cos q. ë û ë ûë û ë û
These relations can also obtained from the table If it is required to shift to coordinate axes back to their
original positions, then replace X by
x¯ y¯ ( x - h ) cos q + ( y - k) sin q and Y by
X® cos q sin q - ( x - h ) sin q + ( y - k) cos q .

Y ® - sin q cos q Example 14. If the axes are shifted to the point ( -2 , - 3)
and then they are rotated through an angle of 45° in
Example 13. If the axes be turned through an angle anti-clockwise sense, then the equation
2 x 2 + 4 x y - 5 y 2 + 20 x - 22 y - 24 = 0 becomes
tan-1 2 . What does the equation 4 xy - 3x 2 = a 2 become ?
(a) X 2 - 14XY - 7Y 2 - 12 = 0 (b) X 2 - 10 XY - 2Y 2 - 12 = 0
(a) X 2 - 4Y 2 = a2
(c) X 2 + 14XY + 7Y 2 - 12 = 0 (d) None of these
(b) X 2 + 4Y 2 = a2
Sol. (a) Let ( X , Y ) be the coordinates of a point with respect to the
(c) X 2 + 4Y 2 = - a2
new axes. Then,
(d) None of the above
p p X -Y
x = - 2 + X cos - Y sin = -2 +
Sol. (a) q = tan -1 2 4 4 2
Þ tan q = 2 p p X+Y
and y = - 3 + X sin + Y cos = -3 +
1 2 4 4 2
So, cos q = , sin q =
5 5 On substituting these value of x and y in the equation, we get
X - 2Y 2x2 + 4xy - 5y 2 + 20 x - 22y - 24 = 0
x = X cos q - Y sin q =
5 2
2X + Y ì X - Yü ìæ X -Yö æ X + Y öü
and y = X sin q + Y cos q = 2 í -2 + ý + 4 í çè -2 + ÷ ç -3 + ÷ý
î 2 þ î 2 ø è 2 øþ
5
2
The equation 4xy - 3x2 = a2 reduces to ì X + Yü ì X - Yü
- 5 í -3 + ý + 20 í -2 + ý
2 î 2 þ î 2 þ
4 ( X - 2Y) (2X + Y) æ X - 2Y ö 2
× -3 ç ÷ =a ì X + Yü
5 5 è 5 ø - 22 í -3 + ý - 24 = 0
î 2 þ
Þ 4 (2X 2 - 2Y 2 - 3XY) - 3 ( X 2 - 4XY + 4Y 2) = 5a2
Þ X 2 - 14XY - 7Y 2 - 12 = 0
Þ 5X 2 - 20Y 2 = 5a2 Hence, the given equation reduces to
Þ X 2 - 4Y 2 = a2 X 2 - 14XY - 7Y 2 - 12 = 0
Practice Exercise
ROUND I Topically Divided Problems
System of Coordinates, Distance between æ kx1 + lx 2 + mx 3 ky1 + ly 2 + my 3 ö
(a) ç , ÷
Two Points and Section Formula è k+ l+m k+ l+m ø
æ 1 mx 2 + kx 3
lx + ly1 + my 2 + ky 3 ö
1. The distance of the point ( a cos a , a sin a ) from the (b) ç , ÷
è l+m+ k l+m+ k ø
origin is independent of
æ mx1 + kx 2 + lx 3 my1 + ky2 + ly3 ö
(a) a (b) a (c) ç , ÷
è m + k+ l m+ k+ l ø
(c) a, a (d) None of these
(d) None of the above
2. The distance between the points ( a cos a , a sin a )
8. The coordinates of point on the line joining the
and ( a cos b, a sin b), where a > 0 is
points P (3, - 4) and Q( -2 , 5) that is twice as far
½ æ a - bö ½
½ ½ æ a - bö ½
½
(a) 2a ½ sin ç ÷ (b) a ½ sin ç ÷ from P as from Q, are
½ è 2 ø½ ½ è 2 ø½
(a) (7, 14) (b) (-7, 14)
½ æ a - bö ½
(c) 2½ sin ç ÷½ (d) None of these (c) (14, 7) (d) (7, 15)
½ è 2 ø½
9. Let AB be divided internally and externally at
3. Distance between foot of perpendicular drawn from P and Q in the same ratio. Then, AP , AB and AQ
a point ( - 3 , 4) on both axes, is are is
(a) 5 units (b) 2 units (a) AP (b) GP
(c) 4 units (d) 1 unit (c) HP (d) None of these
4. The line joining A ( b cos a , b sin a ) and 10. The points whose coordinates are x = x1 + t ( x 2 - x1),
B ( a cos b, a sin b) is produced to the point M ( x, y), y = y1 + t ( y2 - y1) divides the join of ( x1, y1) and
so that AM : MB = b : a , then ( x2 , y2 ) in the ratio
t (1 + t ) t (1 - t )
æa + b ö æa + b ö (a) (b) (c) (d)
x cos ç ÷ + y sin ç ÷ is (1 + t ) t (1 - t ) t
è 2 ø è 2 ø
(a) - 1 (b) 0 (c) 1 (d) a 2 + b 2 11. The straight lines x = y, x - 2 y = 3 and x + 2 y = - 3
form a triangle, which is
5. P , Q, R and S are the points on the line joining the
(a) isosceles (b) equilateral
points P ( a , x) and T ( b, y) such that (c) right angled (d) None of these
æ 5 a + 3b 5 x + 3 y ö
PQ = QR = RS = ST, then ç , ÷ is the 12. Length of the median from B on AC, where
è 8 8 ø
A( -1, 3), B(1, - 1), C(5, 1) is
mid-point of
(a) 18 (b) 10 (c) 2 3 (d) 4
(a) PQ (b) QR (c) RS (d) ST
13. The centre of a circle which passes through points
6. Three vertices of a parallelogram taken in order are
(1, 1), (2, 3) and (–2, 2) is
( - 1, - 6), ( 2 , - 5) and (7, 2). The fourth vertex is
æ 1 39 ö
(a) (1, 4) (b) (4, 1) (a) (3, 4) (b) ç - ,- ÷
è 14 14 ø
(c) (1, 1) (d) (4, 4)
æ 1 39 ö æ 39 1 ö
7. The coordinates of points A, B and C are (c) ç - , ÷ (d) ç - , ÷
è 14 14 ø è 14 14 ø
( x1, y1), ( x 2, y2 ) and ( x3, y3) and point D divides AB
in the ratio l : k. If P divides line DC in the ratio 14. The points A ( 2 a , 4 a), B ( 2 a , 6 a) and
m : ( k + l ), coordinates of P are C ( 2 a + 3a, 5 a) , a > 0 are the vertices of
Cartesian Coordinate System 267

(a) an isosceles triangle 1 1 1 1


(a) + =0 (b) + =1
(b) a right angled triangle p q p q
(c) an acute angled triangle 1 1 1 3
(c) + = 3 (d) + =1
(d) None of the above p q p q

15. Two opposite vertices of a rectangle are (1, 3) and 24. The area of the pentagon whose vertices are A (1, 1),
(5, 1). If the rest two vertices lie on the line B (7, 21), C (12, 2), D (7, - 3) and E(0, - 3) is
y - x + l = 0, then l is equal to (a) 146 sq units (b) 174 sq units
(a) 1 (b) –1 (c) 2 (d) 3 (c) 150 sq units (d) 131 sq units

16. If an equilateral triangle has one vertex at the 25. If the area of the triangle with vertices ( x, 0), (1, 1)
point (0, 0) and another at (3, 3), then the and (0, 2) is 4 sq units, then the value of x is
coordinates of the third vertex is (a) - 2 (b) - 4 (c) - 6 (d) 8
(a) (0, 2 3 ) (b) (0, - 2 3 ) 26. A (6 , 3), B ( - 3 , 5), C ( 4 , - 2) and D ( x, 3x) are four
(c) (-1, 2 3 ) (d) None of these
points. If the area of D DBC and D ABC are in the
17. Let the opposite angular points of a square be (3, 4) ratio 1 : 2, then x is equal to
and (1, –1). Then, the coordinates of the remaining 11 8
(a) (b)
angular points are 8 11
æ9 1ö æ 1 5ö æ9 1ö æ 1 5ö (c) 3 (d) None of these
(a) ç , ÷ and ç - , ÷ (b) ç , - ÷ and ç - , ÷
è2 2ø è 2 2ø è2 2ø è 2 2ø
27. The length of altitude through A of the D ABC,
æ 9 1ö æ 1 5ö
(c) ç - , ÷ and ç - , ÷ (d) None of these where A º ( - 3, 0), B º ( 4, - 1), C º (5, 2), is
è 2 2ø è 2 2ø
2 4 11 22
(a) (b) (c) (d)
18. If C is the reflection of A ( 2, 4) in X-axis and B is 10 10 10 10
the reflection of C in Y -axis, then|AB|is equal to
28. Area of a triangle is 5 sq units and two of its
(a) 20 (b) 2 5 (c) 4 5 (d) 4
vertices are ( 2 , 1) and (3 , - 2). If its third vertex is
19. If point P (3, 2) divides the line segment AB on the line y = x + 3, then it is
internally in the ratio 3 : 2 and point Q( -2, 3) æ -3 3ö æ 13 19 ö
(a) (1, 4) (b) (10, 13) (c) ç , ÷ (d) ç , ÷
divides AB externally in the ratio 4 :3, then the è 2 2ø è2 2ø
coordinates of points A and B are 29. If the vertices of a triangle have integral
æ 66 31 ö æ 41 36 ö æ 3 11 ö æ 21 25 ö
(a) ç , ÷, ç , ÷ (b) ç , ÷, ç , ÷ coordinates, the triangle cannot be
è 17 17 ø è 17 17 ø è 17 17 ø è 17 17 ø
(a) an equilateral triangle
æ 11 31 ö æ 13 37 ö æ 61 73 ö
(c) ç , ÷, ç , ÷ (d) ç , ÷ (b) a right angled triangle
è 17 17 ø è 17 17 ø è 17 17 ø
(c) an isosceles triangle
(d) None of the above
Area of Some Geometrical Figures
20. If A (6, - 3), B ( -3, 5), C( 4, - 2), P (a , b), then the Different Points Related to a Triangle
ratio of the areas of the DPBC, DABC is 30. The x-coordinate of the incentre of the triangle
(a) | a + b | (b) | a - b | where the mid-point of the sides are (0, 1), (1, 1)
(c) | a + b + 2| (d) | a + b - 2| and (1, 0), is
21. The coordinates of two points A and B are (3, 4) (a) 2 + 2 (b) 1 + 2 (c) 2 - 2 (d) 1 - 2
and (5, - 2) respectively. The coordinates of any P, 31. The orthocentre of the triangle formed by (0, 0), (8,
if PA = PB and area of DAPB is 10 0) and (4, 6) is
(a) (7, 2) or (1, 0) (b) (5, 2) or (3, 2) æ 8ö
(c) (4, 5) or (3, 8) (d) (6, 4) or (7, 1) (a) ç4, ÷ (b) (3, 4) (c) (4, 3) (d) (- 3, 4)
è 3ø
22. Area of quadrilateral whose vertices are (2, 3),
32. If orthocentre and circumcentre of triangle are
(3, 4), (4, 5) and (5, 6), is equal to
respectively (1, 1) and (3, 2), then the coordinates of
(a) 0 (b) 4
its centroid are
(c) 6 (d) None of these
æ 7 5ö æ5 7ö
23. If the three points ( 3q, 0 ), ( 0, 3 p ) and (1, 1) are (a) ç , ÷ (b) ç , ÷
è 3 3ø è3 3 ø
collinear, then which one is correct? (c) (7, 5) (d) None of these
268 JEE Main Mathematics

33. The incentre of the triangle formed by lines x = 0, Locus of a Point


y = 0 and 3x + 4 y = 12 , is at
42. The locus of a point P which moves such that
æ1 1ö æ 1ö æ1 ö
(a) ç , ÷ (b) (1, 1) (c) ç1, ÷ (d) ç , 1÷ 2 PA = 3 PB, where coordinates of points A and B
è2 2ø è 2ø è2 ø
are (0, 0) and ( 4, - 3), is
34. If t1 + t2 + t3 = - t1t2 t3, then orthocentre of the (a) 5x 2 - 5 y 2 - 72x + 54 y + 225 = 0
triangle formed by the points (b) 5x 2 + 5 y 2 - 72x + 54 y + 225 = 0
[ at1t2 , a ( t1 + t2 )], [ at2 t3, a ( t2 + t3)] (c) 5x 2 + 5 y 2 + 72x - 54 y + 225 = 0
and [ at3t1, a ( t3 + t1)], lies on (d) 5x 2 + 5 y 2 - 72x - 54 y - 225 = 0
(a) (a , 0) (b) (- a , 0) (c) (0, a ) (d) (0, - a ) 43. The locus of a point whose difference of distance
35. The coordinates of the circumcentre of the triangle from points (3, 0) and ( - 3, 0) is 4, is
with vertices (8, 6), (8, - 2) and ( 2 , - 2) are x2 y2 x2 y2
(a) - =1 (b) - =1
æ 2ö 4 5 5 4
(a) ç6, ÷ (b) (8, 2) (c) (5, - 2) (d) (5, 2)
è 3ø x2 y2 x2 y2
(c) - =1 (d) - =1
2 3 3 2
36. If two vertices of a triangle are ( - 2 , 3) and (5, - 1).
Orthocentre lies at the origin and centroid on the 44. What is the equation of the locus of a point which
line x + y = 7, then the third vertex lies at moves such that 4 times its distance from the
(a) (7, 4) (b) (8, 14) X-axis is the square of its distance from the origin?
(c) (12, 21) (d) None of these (a) x 2 + y 2 - 4 y = 0 (b) x 2 + y 2 - 4| y|= 0
37. The vertices of a triangle are ( pq, 1/ ( pq)), ( qr, 1/ ( qr)) (c) x 2 + y 2 - 4x = 0 (d) x 2 + y 2 - 4|x| = 0
and ( rq, 1/ ( rp)), where p, q and r are the roots of the 45. A rod of length l slides with its ends on two
equation y 3 - 3 y 2 + 6 y + 1 = 0. The coordinates of its perpendicular lines. The locus of a point which
centroid are divides it in the ratio 1 : 2, is
(a) (1, 2) (b) (2, - 1) (a) 36x 2 + 9 y 2 = 4l 2 (b) 36x 2 + 9 y 2 = l 2
(c) (1, - 1) (d) (2, 3) (c) 9x 2 + 36 y 2 = 4l 2 (d) 9x2 - 36 y2 = 4l2
38. If the vertices P , Q, R of a DPQR are rational points, 46. A point moves in such a way that the sum of its
which of the following points of the DPQR is (are) distances from two fixed points ( ae, 0) and ( -ae, 0)
always rational points ? is 2a. Then, the locus of the points is
(a) Centroid (b) Incentre x2 y2 x2 y2
(a) + 2 =1 (b) - 2 =1
(c) Circumcentre (d) Orthocentre a 2
a (1 - e 2) a 2
a (1 - e2)
(A rational point is a point both of whose x2 y2
coordinates are rational numbers) (c) 2 2
+ 2 =1 (d) None of these
a (1 - e ) a
39. ABC is a triangle with vertices A ( - 1, 4), B (6, - 2)
47. Coordinates of two points are A(1, 0) and B( -1, 0)
and C ( - 2 , 4). D, E and F are the points which and Q is a point which satisfies AQ - BQ = ± 1. The
divide each AB, BC and CA respectively in the ratio locus of the point Q is
3 : 1 internally. Then, the centroid of DDEF is (a) 12x 2 + 4 y 2 = 3 (b) 12x 2 - 4 y 2 = 3
(a) (3, 6) (b) (1, 2) (c) 12x 2 - 4 y 2 + 3 = 0 (d) 12x 2 + 4 y 2 + 3 = 0
(c) (4, 8) (d) (- 3, 6)
48. A point moves is such a way that the sum of
40. If (0, 1) is the orthocentre and (2, 3) is the centroid squares of its distances from A ( 2, 0) and B ( -2, 0) is
of a triangle. Then, its circumcentre is always eaual to the square of the distance between
(a) (3, 2) (b) (1, 0) A and B, then the locus of point P is
(c) (4, 3) (d) (3, 4) (a) x 2 + y 2 - 2 = 0 (b) x 2 + y 2 + 2 = 0
41. A triangle has a vertex at (1, 2) and the mid-points (c) x 2 + y 2 + 4 = 0 (d) x 2 + y 2 - 4 = 0
of the two sides through it are ( -1, 1) and ( 2, 3). 49. If the equation of the locus of a point equidistant
Then, the centroid of this triangle is (JEE Main 2019) from points ( a1, b1) and ( a2 , b2 ) is
æ 7ö æ1 ö ( a1 - a2 ) x + ( b1 - b2 ) y + c = 0, then c is equal to
(a) ç1, ÷ (b) ç , 2÷
è 3ø è3 ø 1 2
(a) a12 + b12 - a 22 - b22 (b) (a 2 + a12 - b12 - b22)
æ1 ö æ1 5ö 2
(c) ç , 1÷ (d) ç , ÷ 1 2
è3 ø è3 3ø (c) (a 2 + b22 - a12 - b12) (d) None of these
2
Cartesian Coordinate System 269

50. If A(cos a , sin a ), B(sin a , - cos a ), C(1, 2) are the æ5 2ö æ 5 2ö


(a) ç , - ÷ (b) ç - , ÷
vertices of a DABC, then the locus of centroid of triangle is è3 3 ø è 3 3 ø
(a) x 2 + y 2 - 2x - 4 y + 1 = 0 æ5 3 ö æ 5 3ö
(c) ç , ÷ (d) ç - , ÷
(b) 3(x 2 + y 2) - 2x - 4 y + 1 = 0 è2 2 ø è 2 2 ø
(c) x 2 + y 2 - 2x - 4 y + 3 = 0
(d) None of the above 55. Without change of axes the origin is shifted to
( h, k), then from the equation
Translation of Axes x 2 + y 2 - 4 x + 6 y - 7 = 0 the terms containing
51. The coordinate axes rotated through an angle 135°. linear powers are missing. Then, point ( h, k) is
If the coordinates of a point P in the new system (a) (3, 2) (b) (–3, 2)
(c) (2, –3) (d) (–2, –3)
are known to be ( 4, - 3), then the coordinates of P in
the original system are 56. A line L has intercepts a and b on the coordinate
æ 1 7 ö æ 1 7 ö axes. Keeping the origin fixed, the axes are rotated
(a) ç , ÷ (b) ç ,- ÷
è 2 2ø è 2 2ø through a fixed angle. Now, the same line has
æ 1 7 ö æ 1 7 ö intercepts p and q on the new axes too many
(c) ç - ,- ÷ (d) ç - , ÷
è 2 2ø è 2 2ø (a) a 2 + p 2 = b 2 + q 2 (b) a 2 + b 2 = p 2 + q 2
1 1 1 1 1 1 1 1
52. Without changing the direction of coordinate axes, (c) 2 + 2 = 2 - 2 (d) 2 + 2 = 2 + 2
a b p q a p b q
to which point should origin be transferred, so that
the equation x 2 + y 2 - 4 x + 6 y - 7 = 0 is changed to 57. If q is an angle by which axes are rotated about
an equation which contains no term of first degree? origin and equation ax 2 + 2 hxy + by 2 = 0 does not
(a) (3, 2) (b) (2 , - 3) contain xy term in the new system, then tan 2 q is
(c) (- 2 , 3) (d) None of these h h2
(a) (b)
53. If origin is shifted to (7, - 4), then point (4, 5) shifted a+b (a - b)2
to 2h 3h
(c) (d)
(a) (- 3, 9) (b) (3, 9) a-b (a + b)2
(c) (11, 1) (d) None of these
58. By rotating the axes through 180° the equation
54. If the axes are rotated through an angle 60°, the x - 2 y + 3 = 0 changes to
coordinates of a point in the new system are (a) x + 2 y - 3 = 0 (b) x - 2 y + 3 = 0
( 2, - 3), then its original coordinates are (c) x - 2 y - 3 = 0 (d) None of these

ROUND II Mixed Bag


Only One Correct Option circumcentre are (1, 1) and (2, 0) respectively, then
1. A straight line with negative slope passing through radius of the circumcircle of DABC is
the point (1, 4) meets the coordinate axes at A and (a) 3 (b) 10
B. The minimum value of OA + OB is equal to (c) 2 2 (d) None of these
(a) 5 (b) 6 4. The coordinates of circumcentre of the triangle
(c) 9 (d) 8
whose vertices are ( -2 , - 3), ( -1, 0), (7, - 6), are
2. The equation (a) (3, 3) (b) (3, - 3)
2 2 2
( x - 2) + ( y - 1) + ( x + 2) + ( y - 4) = 52 (c) (-3, 3) (d) None of these
represents 5. Without change of axes the origin is shifted to
(a) circle ( h, k), then from the equation
(b) ellipse
(c) line segment x2 + y2 - 4 x + 6 y - 7 = 0
(d) None of the above the terms containing linear powers are missing.
The point ( h, k) is
3. Let equation of the side BC of a D ABC be
(a) (3, 2) (b) (-3, 2)
x + y + 2 = 0. If coordinates of its orthocentre and (c) (2 , - 3) (d) (-2 , - 3)
270 JEE Main Mathematics

6. If P = (1, 0), Q = ( -1, 0) and R = ( 2 , 0) are three 14. A line joining A ( 2 , 0) and B (3, 1) is rotated about
given points, then the locus of the point S ( x, y) A in anti-clockwise direction through 15°. Find the
satisfying the relation SQ 2 + SR 2 = 2 SP 2 is equation of the line in the new position. If B goes to
(a) a straight line parallel to X-axis C in the new position, then coordinates of C are
(b) a circle through the origin æ 1 3ö æ 1 3ö
(a) ç2 + , ÷ (b) ç2 - , ÷
(c) a circle with centre at the origin è 2 2ø è 2 2ø
(d) a straight line parallel to Y -axis æ 1 3ö
(c) ç2 + , ÷ (d) None of these
7. Let A ( 1, k ), B ( 1, 1 ) and C ( 2, 1 ) be the vertices of a è 2 2 ø
right angled triangle with AC as its hypotenuse. If
the area of the triangle is 1, then the set of values
15. Two points P ( a , 0) and Q ( - a, 0) are given, R is a
variable point on one side of the line PQ such that
which ‘ k’ can take is given by
Ð RPQ - Ð RQP is 2 a, then
(a) (1, 3) (b) (0, 2)
(c) ( - 1, 3) (d) ( - 3, - 2) (a) locus of R is x 2 - y 2 + 2xy cot 2 a - a 2 = 0
(b) locus of R is x 2 + y 2 + 2xy cot a - a 2 = 0
8. Given points are A (0, 4) and B (0, - 4), the locus of p
(c) locus of R is a hyperbola, if a =
P ( x, y) such that|AP - BP |= 6, is 4
p
(a) 9x 2 - 7 y 2 + 63 = 0 (b) 9x 2 + 7 y 2 - 63 = 0 (d) locus of R is a circle, if a =
4
(c) 9x 2 + 7 y 2 + 63 = 0 (d) None of these
9. The transformed equation of 3x 2 + 3 y 2 + 2 xy = 2 , 16. The area of a triangle is 5 sq units. Two of its vertices
are (2, 1) and (3, –2). The third vertex lies on
when the coordinate axes are rotated through an
y = x + 3. The coordinates of the third vertex can be
angle of 45°, is
(a) (–3/2, 3/2) (b) (3/4, –3/2)
(a) x2 + 2 y2 = 1 (b) 2 x2 + y2 = 1
(c) (9/2, 11/2) (d) (–1/4, 11/4)
(c) x2 + y2 = 1 (d) x2 + 3 y2 = 1
17. The point ( p + 1, 1), ( 2 p + 1, 3) and ( 2 p + 2, 2 p)
10. The area of the region bounded by the lines
collinear, if
y = |x - 2|, x = 1, x = 3 and the X-axis is
1
(a) 1 (b) 2 (c) 3 (d) 4 (a) p = - 1 (b) p = 1 /2 (c) p = 4 (d) p = -
2
11. If P is the point (1, 0) and Q be the point on
18. If the distance of any point P from the points
y 2 = 8 x. Then, the locus of mid-point of PQ is
A ( a + b, a - b ) and B ( a - b, a + b ) are equal, then
(a) x2 - 4 y + 2 = 0 (b) x2 + 4 y + 2 = 0
the locus of P is
(c) y2 + 4x + 2 = 0 (d) y2 - 4x + 2 = 0
(a) ax + by = 0 (b) x - y = 0
12. ABC is an isosceles triangle, if the coordinates of (c) x + y = 0 (d) bx - ay = 0
the base are B (1, 3) and C ( -2 , 7), the coordinates of
19. Let the orthocentre and centroid of a triangle be
vertex A can be
A( -3, 5) and B(3, 3), respectively. If C is the
æ 1 ö
(a) (1, 6) (b) ç - , 5÷ circumcentre of this triangle, then the radius of the
è 2 ø
circle having line segment AC as diameter, is
æ5 ö æ 1ö
(c) ç , 6÷ (d) ç -8 , ÷ (JEE Main 2018)
è6 ø è 8ø 5 3 5
p (a) 10 (b) 2 10 (c) 3 (d)
13. Let 0 < a < be a fixed angle. If P = (cos q, sin q) 2 2
2
20. The shortest distance between the point æç , 0 ö÷ and
3
and Q = {cos (a - q ), sin (a - q )}. Then, Q is obtained è2 ø
from P by
the curve y = x , ( x > 0), is (JEE Main 2019)
(a) clockwise rotation around the origin through
angle a 3 5 3 5
(a) (b) (c) (d)
(b) anti-clockwise rotation around origin through 2 4 2 2
angle a 21. The origin is shifted to (1, 2). The equation
(c) reflection in the line through the origin with slope
tan a
y 2 - 8 x - 4 y + 12 = 0 changes to y 2 = 4 a x, then a is
(d) reflection in the line through the origin with slope
equal to
a (a) 1 (b) 2 (c) -2 (d) -1
tan
2
Cartesian Coordinate System 271

22. The centroid of the triangle is (3, 3) and the 28. Three distinct points A, B and C given in the
orthocentre is (–3, 5), then its circumcentre is 2-dimensional coordinate plane such that the ratio
(a) (0, 4) (b) (0, 8) (c) (6, 2) (d) (6, –2) of the distance of any one of them from the point
23. If the line 3x + 4 y - 24 = 0 intersects the X-axis at (1, 0) to the distance from the point (–1, 0) is equal
the point A and the Y -axis at the point B, then the 1
to . Then, the circumcentre of the DABC is at the
incentre of the triangle OAB, where O is the origin, 3
is (JEE Main 2019) point
(a) (4, 3) (b) (3, 4) (c) (4, 4) (d) (2, 2) æ5 ö æ5 ö
(a) ç , 0÷ (b) ç , 0÷
è4 ø è2 ø
24. A rectangle is inscribed in a circle with a diameter æ5 ö
(c) ç , 0÷ (d) (0, 0)
lying along the line 3 y = x + 7. If the two adjacent è3 ø
vertices of the rectangle are (– 8, 5) and (6, 5), then
29. Let A ( h, k), B (1, 1) and C ( 2, 1) be the vertices of a
the area of the rectangle (in sq units) is
(JEE Main 2019) right angled triangle with AC as its hypotenuse.
(a) 72 (b) 84 (c) 98 (d) 56 If the area of the triangle is 1, then the set of
values which k can take is given by
25. If the line 2x + y = k passes through the point
(a) {1, 3} (b) {0, 2}
which divides the line segment joining the points (c) {–1, 3} (d) {–3, –2}
(1, 1) and (2, 4) in the ratio 3 : 2, then k is equal to
29 11 30. If a vertex of a triangle is (1, 1) and the mid-point of
(a) (b) 5 (c) 6 (d)
5 5 two sides of a triangle through this vertex are (–1, 2)
and (3, 2), then the centroid of the triangle is
26. The x-coordinate of the incentre of the triangle
æ 1 7ö æ 7ö
that has the coordinates of mid-points of its sides (a) ç - , ÷ (b) ç - 1, ÷
è 3 3ø è 3ø
as (0, 0), (1, 1) and (1, 0) is
æ1 7ö æ 7ö
(a) 2 + 2 (b) 2 - 2 (c) ç , ÷ (d) ç1, ÷
è3 3 ø è 3ø
(c) 1 + 2 (d) 1 - 2
27. If A( 2, - 3) and B( -2,1) are two vertices of a triangle 31. Let A( 2, - 3) and B( -2, 1) be the vertices of a DABC.
and third vertex moves on the line 2 x + 3 y = 9, then If the centroid of this triangle moves on the line
the locus of the centroid of the triangle is 2 x + 3 y = 1, then the locus of the vertex C is the line
(a) 2x - 3 y = 1 (b) x - y = 1 (a) 2x + 3 y = 9 (b) 2x - 3 y = 7
(c) 2x + 3 y = 1 (d) 2x + 3 y = 3 (c) 3x + 2 y = 5 (d) 3x - 2 y = 3

Answers
Round I
1. (a) 2. (a) 3. (a) 4. (b) 5. (b) 6. (b) 7. (a) 8. (b) 9. (a) 10. (c)
11. (d) 12. (b) 13. (c) 14. (c) 15. (a) 16. (a) 17. (a) 18. (c) 19. (a) 20. (d)
21. (a) 22. (a) 23. (c) 24. (a) 25. (c) 26. (a) 27. (d) 28. (c) 29. (a) 30. (c)
31. (a) 32. (a) 33. (b) 34. (b) 35. (d) 36. (d) 37. (b) 38. (a) 39. (b) 40. (d)
41. (b) 42. (b) 43. (a) 44. (b) 45. (a) 46. (a) 47. (b) 48. (d) 49. (c) 50. (b)
51. (d) 52. (b) 53. (a) 54. (c) 55. (c) 56. (b) 57. (c) 58. (c)

Round II
1. (c) 2. (c) 3. (b) 4. (b) 5. (c) 6. (d) 7. (c) 8. (a) 9. (b) 10. (a)
11. (d) 12. (c) 13. (d) 14. (a) 15. (a) 16. (a) 17. (d) 18. (b) 19. (c) 20. (d)
21. (b) 22. (c) 23. (d) 24. (b) 25. (c) 26. (b) 27. (c) 28. (a) 29. (c) 30. (d)
31. (a)
Solutions
Round I æ a + bö æ a -bö
2 sin ç ÷ sin ç ÷
x è 2 ø è 2 ø
1. Let P º (a cos a, a sin a) and O º (0, 0), Þ =
y æ a + bö æb - aö
then |OP | = (a cos a - 0) + (a sin a - 0) 2 2 2 cos ç ÷ sin ç ÷
è 2 ø è 2 ø
= (a 2 cos 2 a + a 2 sin 2 a ) æ a + bö æ a + bö
Þ x cos ç ÷ + y sin ç ÷ =0
= a 2(cos 2 a + sin 2 a ) = a 2 è 2 ø è 2 ø
=| a |, which is independent of a. 5. R is mid-point of PT.
2. Let P º (a cos a, a sin a) and Q º (a cos b, a sin b) P(a, x) T(b, y)
2 2
Q R S
Then, | PQ | = (a cos a - a cos b ) + (a sin a - a sin b )
æ 5a + 3b 5x + 3 y ö
= a 2{(cos a - cos b )2 + (sin a - sin b )2 } Point ç , ÷ divides PT in ratio 3 : 5 and
è 8 8 ø
a 2{(cos 2 a + cos 2 b - 2 cos a cos b that is mid-point of QR.
=
2 2
+ (sin a + sin b - 2 sin a sin b )} 6. Let the fourth vertex be D (x, y).
2
= a { 1 + 1 - 2(cos a cos b + sin a sin b )} D C
(x, y) (7, 2)
= a 2[2 - 2 cos (a - b )]
= 2a 2[1 - cos (a - b )]
é æ a - böù 2 æ a - bö
= ê2a 2 × 2 sin 2 ç 2
÷ = 4a sin ç ÷
ë è 2 ø úû è 2 ø A B
(–1, –6) (2, – 5)
½ æ a - bö ½
= ½ 2a sin ç ÷½
½ è 2 ø½ We know that, two diagonals of a parallelogram are
½ æ a -bö ½
½ bisect each other.
= 2a ½ sin ç ÷ (Q a > 0)
½ è 2 ø½ -1 + 7 2 + x
\ = Þ x=4
2 2
3. From the figure, M and N are the foot of perpendicular
-6 + 2 -5 + y
of point (-3, 4) on Y and X-axes, respectively. and = Þ y =1
2 2
Y
(–3, 4) \ Fourth vertex of D is (4, 1).
(0, 4)
M 7. Coordinates of point D are
æ lx + kx1 ly2 + ky1 ö
N Dç 2 , ÷
X¢ X è k+ l k+ l ø
(–3, 0)
l D k P
A(x1, y1) B(x2, y2) S C(x3, y3)

Y¢ m (k + l)

Now, distance MN = (0 + 3)2 + (4 - 0)2 Coordinates of point P are


æ lx + kx1 + mx3 ly2 + ky1 + my3 ö
= (3)2 + (4)2 = 9 + 16 Pç 2 , ÷
è k+ l+m k+ l+m ø
= 25 = 5 units
8. Let A (x, y) be the required point,
4. Given, M divides AB in the ratio b : a (externally).
\ PA = 2 PQ (given)
ba cos b - ab cos a
\ x= Thus, A divides PQ internally or externally in the ratio
b-a
2 : 1.
ab sin b - ab sin a
and y= If A divides PQ internally in the ratio 2 : 1, then
b-a coordinates of A are
x cos b - cos a 2 ´ (-2) + 1 ´ 3 2 ´ 5 + 1 ´ (- 4)
Þ = x= and y =
y sin b - sin a 2+1 2+1
Cartesian Coordinate System 273

1 11. The lines of a triangle are x = y, x - 2 y = 3 and x + 2 y = -3


Þ x=- and y = 2
3 intersection points at sides are A (- 3, - 3), B (- 1, - 1 )
æ 1 ö
Þ Coordinates of A are ç - , 2÷ . æ 3ö
è 3 ø and C ç0, - ÷
è 2ø
Again, if A divides PQ externally in the ratio 2 : 1, then \ AB = 4 + 4 = 2 2
coordinates of A are,
2 ´ (-2) - 1 ´ 3 2 ´ 5 - 1 ´ (- 4) 9 3 5
x= and y = AC = 9 + =
2 -1 2 -1 4 2
1 5
Þ x = - 7 and y = 14 and BC = 1 + =
4 2
\ Coordinates of A are (-7, 14). 2
æ 5ö 5 37
Now, ç ÷ + (2 2 )2 = + 8 =
9. Let P and Q divides the join of AB in the ratio of l : 1 both è 2 ø 4 4
internally and externally respectively as shown below.
Hence, option (d) is correct.
Let coordinates of points A and B are (x1 , y1 ) and (x2, y2)
12. Let BE be the median from B on AC, where E is the
A P B Q
mid-point of AC.
l
1 According to mid-point formula, coordinates of E are
1
l
æ5 - 1 1 + 3ö
ç , ÷ i.e., (2, 2).
æ lx + x1 ly2 + y1 ö è 2 2 ø
\ Pç 2 , ÷
è l+1 l+1 ø
\Length of median from B =| BE |.
æ lx - x1 ly2 - y1 ö
and Qç 2 , ÷ Þ | BE | = (2 - 1 )2 + (2 + 1 )2 = 10
è l -1 l -1 ø
2 2 13. Let C (x, y) be centre of the circle passing through the
æ lx + x1 ö æ ly + y1 ö
Now, AP = ç x1 - 2 ÷ + ç y1 - 2 ÷ points P (1, 1 ), Q (2, 3), R(-2, 2)
è l+1 ø è l+1 ø
\ | PC | = |QC | = | RC |
2 2
æ lx - lx2 ö æ ly - ly2 ö Case I | PC | = |QC |
Þ AP = ç 1 ÷ +ç 1 ÷
è l+1 ø è l+1 ø (x - 1 )2 + ( y - 1 )2 = (x - 2)2 + ( y - 3)2
æ l ö 2 2 Þ 2x + 4 y = 11 …(i)
Þ AP = ç ÷ (x2 - x1 ) + ( y2 - y1 )
è l + 1ø Case II |QC | = | RC |
Also, 2
AB = (x2 - x1 ) + ( y2 - y1 ) 2 (x - 2)2 + ( y - 3)2 = (x + 2)2 + ( y - 2)2
Þ 8x + 2 y = 5 …(ii)
2 2
æ lx - x1 ö æ l y2 - y1 ö
and AQ = ç x1 - 2 ÷ + ç y1 - ÷ On solving Eqs. (i) and (ii) for x and y, we have the
è l -1 ø è l -1 ø coordinates of centre of circle are
2 2 æ 1 39 ö
æ lx - x1 - lx2 + x1 ö æ ly - y1 - ly2 + y1 ö C (x, y) = ç - , ÷
Þ AQ = ç 1 ÷ +ç 1 ÷ è 14 14 ø
è l -1 ø è l -1 ø
14. Given vertices are A (2a , 4a ), B (2a , 6a ) and
æ l ö 2 2
Þ AQ = ç ÷ (x2 - x)1 + ( y2 - y1 ) C (2a + 3 a , 5a ), a > 0
è l - 1ø
Now, AB = (2a - 2a )2 + (4a - 6a )2 = 2a
Clearly, AP , AB and AQ are in AP.
BC = ( 3a )2 + a 2 = 2a
10. Since, x = x1 + t (x2 - x1 )
Þ x = x1 + t x2 - t x1 and CA = ( 3a )2 + (- a )2 = 2a
Þ x = (1 - t ) x1 + tx2 \ AB = BC = CA
(1 - t )x1 + tx2 Hence, triangle is an equilateral triangle, therefore it is
Þ x=
(1 - t ) + t an acute angled triangle.
(1 - t ) y1 + ty2 15. The mid-point of (1, 3) and (5, 1) satisfies
Similarly, y=
(1 - t ) + t y- x+ l =0 …(i)
Þ The point (x, y) divides the join of (x1 , y1 ) and (x2, y2) in Here, mid-point of given points, is (3, 2), which is
the ratio of t : (1 - t ). satisfied the given Eq. (i)
\ 2 - (3) + l = 0 Þ l = 1
274 JEE Main Mathematics

16. Let O º (0, 0) and A º (3, 3 ) be the given points and let Also in DABC, ( AB)2 + (BC )2 = ( AC )2
B º (x, y) be the required point. Then, OA = OB = AB Þ (x - 3)2 + ( y - 4 )2 + (x - 1 )2 + ( y + 1 )2
Y = (3 - 1 )2 + (4 + 1 )2
2 2
D(0, y) Þ x + y - 4x - 3 y - 1 = 0 …(ii)
A(3, Ö3) On substituting the value of x from Eq. (i) into Eq. (ii),
we get
2
æ 23 - 10 y ö 2 æ 23 - 10 y ö
X¢ X ç ÷ + y -4 ç ÷ - 3y - 1 = 0
O è 4 ø è 4 ø
(0, 0)
Y¢ B(x, y) Þ 4 y2 - 12 y + 5 = 0 or (2 y - 1 ) (2 y - 5) = 0
1 5
Þ (OA )2 = (OB )2 = ( AB )2 \ y = or
2 2
Þ (3 - 0) + ( 3 - 0)2 = (x - 0)2 + ( y - 0)2
2
1 9
On putting y = in Eq. (i), we get x = ,
= (x - 3)2 + ( y - 3 )2 2 2
Þ 12 = x + y = x2 + y2 - 6x - 2 3 y + 12
2 2
5 1
and putting y = , in Eq. (i) we get x = -
Taking first two members, then 2 2
x2 + y2 = 12 …(i) æ9 1ö
Hence, the required vertices of the square are ç , ÷
and taking last two members, then è2 2ø
6x + 2 3 y = 12 or y = 3 (2 - x ) æ 1 5ö
…(ii) and ç - , ÷ .
è 2 2ø
From Eqs. (i) and (ii), we get
x2 + 3(2 - x )2 = 12 Þ 4x2 - 12x = 0 18. Here, coordinates are A (2, 4), B(-2, - 4) and C (2, - 4).
Þ x = 0, 3 Y
On putting x = 0, 3 respectively in Eq. (ii), we get A (2, 4)
y = 2 3, - 3
Hence, the coordinates of the third vertex B are (0, 2 3 )
or (3, - 3 ). X¢ X

Alternate Method
æ x1 + x2 + 3 ( y2 - y1 ) y1 + y2 ± 3 (x2 - x1 ) ö B (–2, –4) C (2, –4)
ç , ÷
è 2 2 ø Y¢
æ 0 + 3 + 3 ( 3 - 0) 0 + 3 ± 3 (3 - 0) ö Now,| AB| = (-2 - 2)2 + (- 4 - 4)2
i.e., ç , ÷
è 2 2 ø
= 16 + 64 = 80 = 4 5
æ3 + 3 3 ± 3 3 ö
Þ =ç , ÷ 19. Let coordinates of A and B are (a , b) and (c, d )
è 2 2 ø respectively.
Þ = (0, 2 3 ) or (3, - 3 ) Since, P(3, 2) divides AB internally in the ratio 3 : 2
17. Let A(3, 4 ) and C (1, - 1 ) be the given angular points of a æ 3c + 2a 3d + 2b ö
\ (3, 2) = ç , ÷
square ABCD and let B(x, y) be the unknown vertex. è 3+2 3+2 ø
A(3, 4) Þ 3c + 2a = 15 …(i)
and 3d + 2b = 10 …(ii)
D
Also, (- 2, 3) divides AB externally in the ratio 4 : 3.
O æ 4c - 3a 4d - 3b ö
\ (- 2, 3) = ç , ÷
è 4 -3 4 -3 ø
B(x, y)
Þ 4c - 3a = - 2 …(iii)
C(1, –1) and 4d - 3b = 3 …(iv)
Then, AB = BC 66 41
On solving Eqs. (i) and (iii), we get a = ,c=
Þ ( AB)2 = (BC )2 17 17
Þ (x - 3)2 + ( y - 4)2 = (x - 1 )2 + ( y + 1 )2 31 36
and solving Eqs. (ii) and (iv), we get b = ,d =
Þ 4x + 10 y - 23 = 0 17 17
æ 23 - 10 y ö æ 66 31 ö æ 41 36 ö
Þ x=ç ÷ …(i) \ Coordinates of A º ç , ÷ and B º ç , ÷.
è 4 ø è 17 17 ø è 17 17 ø
Cartesian Coordinate System 275

1 24. The required area is


20. QArea of DPBC = |[a (5 + 2) + (-3) (- 2 - b) + 4 (b - 5)]|
2 1 1
1
= | 7a + 7b - 14| 7 21
2
1 12 2 1
Also, Area of = |(21 + 14 - 36 - 21) - (7 + 252 + 14 - 3)|
2 7 -3 2
1
DABC = |[6 (5 + 2) + (-3) (-2 + 3) + 4 (-3 - 5)]| 0 -3
2
1 1
1 7
= |42 - 21 - 14| =
2 2 = 146 sq units
7 25. Given that, x1 = x, x2 = 1, x 3 = 0
| a + b - 2|
Area of DPBC 2
\ = and y1 = 0, y2 = 1, y3 = 2
Area of DABC 7
\ Area of triangle
2
1
=| a + b - 2| = [x1 ( y2 - y3 ) + x2( y3 - y1 ) + x3 ( y1 - y2)]
2
21. Let coordinates of P be (h , k). 1
= [x (1 - 2) + 1 (2 - 0) + 0(0 - 1 )]
2
PA = PB Þ PA 2 = PB2
1 1
Þ (h - 3)2 + (k - 4)2 = (h - 5)2 + (k + 2)2 = [- x + 2 + 0] = (2 - x )
2 2
Þ (h - 3)2 - (h - 5)2 + (k - 4)2 - (k + 2)2 = 0 But area of triangle is 4 sq units
Þ (2h - 8) (2) + (2k - 2) (- 6) = 0 1
\ (2 - x ) = 4
Þ (h - 4) - 3 (k - 1) = 0 2
Þ h - 3k - 1 = 0 …(i) Þ 2 - x = 8 Þ x = -6
h k 1 26. (a) Given, points A (6, 3), B (-3, 5), C (4, - 2) and D (x, 3x).
1
Now, area of DPAB = 3 4 1 = 10 ar (D DBC ) 1
2 = …(i)
5 -2 1 ar (D ABC ) 2
1
Þ 6h + 2k - 26 = ± 20 Now, area of D DBC = |x (5 + 2) - 3x (-3 - 4) + 1(6 - 20)|
2
Þ 6h + 2k - 46 = 0 or 6h + 2k - 6 = 0
1 1
Þ 3h + k - 23 = 0 or 3h + k - 3 = 0 = |7x + 21 x - 14 |= |28x - 14 |…(ii)
2 2
On solving h - 3k - 1 = 0 and 3h + k - 23 = 0, we get 1
and ar (D ABC ) = |6 (5 + 2) - 3 (-3 - 4) + 1 (6 - 20)|
h = 7, k = 2 2
Also, solving h - 3k - 1 = 0 and 3h + k - 3 = 0, we get 1
= |42 + 21 - 14 |
h = 1, k = 0 2
Hence, the coordinate of P are (7, 2) or (1, 0). 1 49
ar (D ABC ) = |49 | = …(iii)
2 2
22. The vertices of quadrilateral ABCD is A (2, 3), B (3, 4),
C(4, 5) and D (5, 6). Using Eqs. (ii) and (iii) in Eq. (i), we get
1 1
Area of quadrilateral = |[x1 y2 + x2 y3 + x3 y4 + x4 y1 ] |28x - 14 |
2 2 1
=
- [ y1x2 + y2 x3 + y3 x4 + y4x1 ]| 1 2
|49 |
1 2
= |[2 ´ 4 + 3 ´ 5 + 4 ´ 6 + 5 ´ 3]| 49 49
2 Þ |28x - 14 |= Þ 28x - 14 = ±
- [3 ´ 3 + 4 ´ 4 + 5 ´ 5 + 6 ´ 2] 2 2
1 On taking positive sign, we have
= |[8 + 15 + 24 + 15]
2 49 77
- [9 + 16 + 25 + 12]| 28x = + 14 Þ 28x =
2 2
1
= |[62 - (62)]| = 0 77 11
2 \ x= =
2 ´ 28 8
23. Since, points (3q, 0),(0, 3p) and (1, 1) are collinear.
On taking negative sign, we have
1
[3q (3 p - 1) + 0 (1 - 0) + 1 (0 - 3 p)] = 0 49
2 28x = - + 14
2
Þ 3 q ( 3 p - 1 ) + 1 ( 0 - 3 p) = 0 -21 -3
1 1 \ x= =
Þ 9 pq = 3 p + 3q Þ + =3 2 ´ 28 8
p q
276 JEE Main Mathematics

27. In DABC the vertices are A (- 3, 0), B (4, - 1 ) and C (5, 2). 29. Let A (x1 , y1 ), B (x2, y2) and C (x3 , y3 ) be the vertices of a
A (–3, 0) D ABC, where x1 , x2, x3 and y1 , y2, y3 all are integers,
then the area of D ABC is given by
1
D = [x1 ( y2 - y3 ) + x2 ( y3 - y1 ) + x3 ( y1 - y2)]
2
= an integer [Q all xi , yi , i = 1, 2, 3, are integers]
A(x1, y1)
(4, –1) B L C (5, 2)

\ BC = (5 - 4)2 + (2 + 1)2
= 1 + 9 = 10
1 B(x2, y2) C(x3, y3)
Area of DABC = [x1 ( y2 - y3 ) + x2( y3 - y1 ) + x3 ( y1 - y2)]
2 If possible, let the D ABC be an equilateral triangle,
1
= [-3(- 1 - 2) + 4 (2 - 0) + 5(0 + 1 )] then its area is given by
2
3 3
1 D= (Side)2 = ( AB)2 [Q AB = BC = CA ]
= [9 + 8 + 5 ] = 11 4 4
2
1 3
As we know that, area of triangle = ´ BC ´ AL = (an integer)
2 4
1 [Q vertices are integer, \ AB2 is an integer]
Þ 11 = ´ 10 ´ AL
2 = An irrational number
2 ´ 11 22
Þ AL = = This contradict the fact that the area is an integer
10 10 number. Hence, the triangle cannot be equilateral.
28. Let third vertex of triangle be (h , k). 30. Since, (0, 1), (1, 1) and (1, 0) are mid-points of sides AB,
(h, k) BC and CA, respectively.
A (0, 0)

c b
(2, 1) (3, –2)
Q Area of triangle = 5 sq units [given]
1
[h (1 + 2) + 2 (-2 - k) + 3 (k - 1)] = ± 10 (0, 2) B a C (2, 0)
2
1 \ Coordinates of A, B and C are (0, 0), (0, 2) and (2, 0),
Þ [h (1 + 2) - k (2 - 3) + 1 (-4 - 3)] = ± 10
2 respectively.
Þ 3 h + k - 7 = ± 10 Now, AB = 2 , BC = 2 2, CA = 2
On taking positive sign, we have \ x-coordinate of incentre
3 h + k = 17 …(i) 0 + 0 + 2 ×2 æ ax1 + bx2 + cx3 ö
= çQ x = ÷
2+2 2 +2 è a + b+ c ø
On taking negative sign, we have
3 h + k = -3 …(ii) 2
= =2- 2
Also, vertex (h , k) lies on the line y = x + 3 2+ 2
Þ k=h+3 31. Line perpendicular to OA passing through B is x = 4.
Þ h - k = -3 …(iii) Y
On solving Eqs. (i) and (iii), we get
B (4, 6)
7 13
h = ,k=
2 2
æ 7 13 ö
Hence, third vertex is ç , ÷ .
è2 2 ø X
O (0, 0) A (8, 0)
On solving Eqs. (ii) and (iii), we get
-3 3 3
h= and k = Slope of AB = -
2 2 2
æ -3 3 ö Line perpendicular to AB through origin is y =
2
x.
Hence, third vertex is ç , ÷.
è 2 2ø 3
Cartesian Coordinate System 277

2 36. Let O(0, 0) be the orthocentre, A (h , k) be the third vertex


\ The point of intersection of a line x = 4 and y = x is
3 and B (- 2 , 3) and C (5, - 1) the other two vertices. Then,
æ 8ö k
ç4, ÷. the slope of the line through A and O is , while the line
è 3ø
h
(-1 - 3) 4
32. As we know that orthocentre, centroid and through B and C has the slope = - . By the
circumcentre are collinear and the centroid divides the (5 + 2) 7
line segment joining orthocentre and circumcentre in poperty of the orthocentre, these two lines must be
the ratio 2 : 1. If the coordinates of orthocentre and perpendicular, so we have
æ kö æ 4ö
circumcentre are (1, 1) and (3, 2) respectively, the ç ÷ ç- ÷ = -1
coordinate of centroid is è hø è 7ø

æ 2 ×3 + 1 ×1 2 ×2 + 1 ×1ö æ 7 5ö k 7
ç , ÷ =ç , ÷ Þ = …(i)
è 2+1 2 + 1 ø è 3 3ø h 4
5 - 2 + h -1 + 3 + k
33. Given equation of lines are Also, + =7
3 3
x = 0, y = 0 and 3x + 4 y = 12 Þ h + k = 16 …(ii)
Incentre is on the line y = x (Angle bisector of OA and OB )
which is not satisfied by the points given in the options
Y
(a), (b) or (c).
(0, 3) B
3x
+
37. Given, p, q, r are the roots of equation
4y
= y3 - 3 y2 + 6 y + 1 = 0.
12
So, p + q + r = 3, pq + qr + rp = 6 and pqr = - 1.
X
O (4, 0)A Now, the centroid of the triangle is
Angle bisector of y = 0 and 3x + 4 y = 12 is æ 1 1 1 ö
ç pq + qr + rp + + ÷
± 5 y = 3x + 4 y - 12 ç pq qr rp ÷
,
3x + 9 y = 12 and 3x - y = 12 ç 3 3 ÷
Þ ç ÷
è ø
Here, 3x + 9 y = 12 internal bisector.
æ pq + qr + rp p + q + r ö æ 6 3 ö
So, intersection point of y = x and 3x + 9 y = 12 is (1, 1). i.e., ç , ÷ºç , ÷ or (2, - 1)
è 3 3 pqr ø è 3 - 3 ø
\The required point of the incentre of triangle is (1, 1).
34. Let A [at1t2, a (t1 + t2)], B [at2 t3 , a (t2 + t3 )], 38. Since, coordinates of the centroid are
C [at3 t1 , a (t3 + t1 )] æ x1 + x2 + x3 y1 + y2 + y3 ö
ç , ÷ , the centroid is always a
a (t3 - t1 ) 1 è 3 3 ø
Slope of AB (mAB ) = =
at2 (t3 - t1 ) t2 rational point.
Equation of line through C perpendicular to AB is 39. Let (x1 , y1 ), (x2, y2) and (x3 , y3 ) are coordinates of the
y - a (t3 + t1 ) = - t2 (x - at3 t1 ) points D, E and F, which divide each AB, BC and CA
Þ y - a (t3 + t1 ) = - t2x + at1 t2 t3 …(i) respectively in the ratio 3 : 1 (internally).
3 ´ 6 - 1 ´ 1 17
Similarly, equation of line through B perpendicular to \ x1 = =
CA is 4 4
-2 ´3 + 4 ´1 2 1
y - a (t2 + t3 ) = - t1 (x - at2 t3 ) y1 = =- =-
4 4 2
Þ y - a (t2 + t3 ) = - t1x + at1t2 t3 …(ii)
A (–1, 4)
Using t1t2 t3 = - (t1 + t2 + t3 ) in Eqs. (i) and (ii), we get
y = - t2 x - at2 and y = - t1x - at1
Þ t2 (x + a ) = t1 (x + a ) Þ x = - a , y = 0 (x1, y1) D F (x3, y3)
35. Triangle is right angled triangle. In right angled
triangle mid-point of hypotenuse is circumcentre.
So, coordinates of the circumcentre are (5, 2). (6, –2) B E (x2, y2) C (–2, 4)
Y (8, 6) 5 5
Similarly, x2 = 0, y2 = and x3 = - , y3 = 4
2 4
Let (x, y) be the coordinates of centroid of DDEF
X¢ X
O 1 æ 17 5ö 1æ 1 5 ö
\ x= ç + 0 - ÷ = 1 and y = ç - + + 4÷ = 2
3è 4 4ø 3è 2 2 ø
(2, –2) (8, –2)
Y¢ \ Coordinates of centroid are (1, 2).
278 JEE Main Mathematics

40. Given orthocentre O(0, 1 ), centroid G(2, 3). Let the Þ (h - 3)2 + k2 = 4 + (h + 3)2 + k2
circumcentre be (x, y).
On squaring both sides, we get
Since, we know that centroid divide orthocentre and
circumcentre in the ratio 2 : 1. (h - 3)2 + k2 = 16 + (h + 3)2 + k2 + 8 (h + 3)2 + k2
é 2(x) + 1(0 ) 2( y) + 1(1 ) ù Þ h 2 + 9 - 6h + k2 = 16 + h 2 + 9 + 6h + k2 + 8 (h + 3)2 + k2
\ G (2, 3) = G ê , ú
ë 2+1 2+1 û Þ - 6h = 16 + 6h + 8 (h + 3)2 + k2
æ 2x 2 y + 1 ö Þ - 8 (h + 3)2 + k2 = 12 h + 16
Þ G (2, 3) = G ç , ÷
è3 3 ø Again, squaring both sides, we get
2x 2y + 1 64 {(h + 3)2 + k2} = (12 h + 16)2
\ 2= and 3 =
3 3 Þ 64 (h 2 + 9 + 6h + k2) = 144h 2 + 256 + 2 × 16 × 12h
Þ x = 3 and y = 4 Þ 64 (h 2 + 9 + 6h + k2) = 16 (9h 2 + 16 + 24h )
41. Let a DABC is such that vertices A (1, 2), B(x1 , y1 ) and Þ 4 (h 2 + 9 + 6h + k2) = 9h 2 + 16 + 24h
C (x2, y2).
Þ 4h + 36 + 24h + 4k2 = 9h 2 + 16 + 24h
2
A(1,2)
h 2 k2
Þ 5h 2 - 4k2 = 20 Þ - =1
4 5
x 2 y2
Hence, the locus of point P is - = 1.
B(x1,y1) C(x2,y2) 4 5

It is given that mid-point of side AB is (- 1, 1). 44. Let (h , k ) be the point.


x +1 y +2 According to the question,
So, 1 = - 1 and 1 =1
2 2
4 (h - h )2 + k2 = h 2 + k2 Þ 4|k|= h 2 + k2
Þ x1 = - 3 and y1 = 0
Locus of the point is
So, point B is (- 3, 0)
Also, it is given that mid-point of side AC is 4| y|= x2 + y2 Þ x2 + y2 - 4| y|= 0
x +1 y +2 1 (a ) + 2 × 0
(2, 3), so 2 = 2 and 2 =3 45. \ h= Y
2 2 2+1
Þ x2 = 3 and y2 = 4 1 ×0 + 2 × b B(0, b)
and k= 1
So, point C is (3, 4). 2+1
P(h, k)
Now, centroid of DABC is a 2b l
Þ h= and k = 2
æ 1 + (- 3) + 3 2 + 0 + 4 ö æ1 ö 3 3 X¢
Gç , ÷ = G ç , 2÷ O
X
è 3 3 ø è3 ø 3k A(a, 0)
Þ a = 3h and b =
2 Y¢
42. Let P (h , k) be the required point, then 2
9 k
4PA 2 = 9PB2 \ 9h 2 + = l2 Þ 36h 2 + 9k2 = 4l2
4
2 2 2 2
Þ 4 (h + k ) = 9 (h - 4) + 9 (k + 3) Hence, locus of a point is 36x2 + 9 y2 = 4l2
Þ 4h 2 + 4k2 = 9 (h 2 + 16 - 8h ) + 9 (k2 + 9 + 6k) 46. Since, A (ae, 0) and B (-ae, 0) be the given points and let
2 2
Þ 5h + 5k - 72h + 54k + 225 = 0 P (h , k) be any point whose distance from A and B is
constant i.e., 2a.
\ Required locus of P (h , k) is
i.e., | PA | + | PB| = 2a
5x2 + 5 y2 - 72x + 54 y + 225 = 0
Þ (h - ae)2 + k2 + (h + ae)2 + k2 = 2a …(i)
43. Let the point be P (h , k).
It is given that difference of the distance from points Let us assume
A(3, 0) and B (- 3, 0) is 4 i.e., PA - PB = 4
{(h - ae)2 + k2 } - {(h + ae)2 + k2 } = - 4aeh …(ii)
2 2 2 2
Þ (h - 3) + k - (h + 3) + k = 4
On dividing Eq. (ii) by Eq. (i), we have
P (h, k)
{(h - ae )2 + k2 } - {(h + ae)2 + k2 } 4aeh
=
2
(h - ae) + k + 2 2
(h + ae) + k 2 2a

Þ (h - ae)2 + k2 - (h + ae)2 + k2 = - 2eh …(iii)


B (–3, 0) A (3, 0) {Q a - b = ( a + b )( a - b )}
Cartesian Coordinate System 279

On adding Eqs. (i) and (iii), we have sin a - cos a + 2


and k= (by definition)
2 2 3
2 (b - ae) + k = 2a - 2eh
Þ 3h - 1 = cos a + sin a
Þ 2 (h - ae)2 + k2 = 2(a - eh ) and 3k - 2 = sin a - cos a
On squaring both sides, we have Þ (3h - 1 )2 + (3k - 2)2 = 2
(h - ae)2 + k2 = (a - eh )2 (squaring and adding)
Þ h + a e - 2aeh + k2 = a 2 + e2h 2 - 2aeh
2 2 2 Þ 9(h 2 + k2) - 6h - 12k + 3 = 0
h 2 - e2h 2 + k2 = a 2 - a 2e2 Þ 3(h 2 + k2) - 2h - 4k + 1 = 0
Þ h 2(1 - e2) + k2 = a 2(1 - e2) \ Locus of centroid G (h , k ) is
Replacing h by x and k by y, we get the locus of point 3(x2 + y2) - 2x - 4 y + 1 = 0.
P (h , k) which is a circle.
x2 y2 51. We know, if coordinate axes are rotated, then
2
+ 2 =1
a a (1 - e2) p = (x cos q - y sin q, x sin q + y cos q)
47. Let Q (x, y) be a point. It is rotated at an angle 135° i.e., q = 135° and the new
point be
Such that, AQ - BQ = ± 1 p = [4 cos (90° + 45° ) + 3 sin (90° + 45° ),
Þ (x - 1 ) + y2 - (x + 1 )2 + y2 = ± 1
2
4 sin (90° + 45° ) - 3 cos (90° + 45° )]
Assuming [(x - 1 )2 + y2] + [(x + 1 )2 + y2] = [ - 4 sin 45° + 3 cos 45° , 4 cos 45° + 3 sin 45° ]
2
-2 (x - 1 ) + y 2 2 2
(x + 1 ) + y = 1 é æ -1ö 1 1 1 ù æ 1 7 ö
= ê4 × ç ÷ + 3× ,4 × + 3× ú = ç- , ÷
2 2
Þ 2x + 2 y + 1 = 2 (x - 1 ) + y 2 2 2
(x + 1 ) + y 2 ë è 2ø 2 2 2û è 2 2ø

On squaring, we have 52. Let origin be shifted at point (h , k) without changing the
2
12x - 4 y = 3 2 direction of coordinate axes.
Then, we replace x by (x + h ) and y by ( y + k) in the
48. Let P (h , k ) be the point such that equation of given curve, then the transformed
| PA |2 + | PB|2 = | AB|2 equation is
Þ (h - 2)2 + k2 + (h + 2)2 + k2 = 42 + 0 (x + h )2 + ( y + k)2 - 4(x + h ) + 6( y + k) - 7 = 0
Þ 2h 2 + 8 + 2k2 = 16 Þ x2 + h 2 + 2hx + y2 + k2 + 2ky - 4x - 4h
Þ h 2 + k2 = 4 + 6 y + 6k - 7 = 0
\ Locus of P is x2 + y2 = 4. Þ x2 + y2 + x(2h - 4) + y(2k + 6)
+ h 2 + k2 - 4h + 6k - 7 = 0
49. Let P (h , k) be the point which is equidistant from
Since, this equation is required to be free from the
A (a1 , b1 ) and B(a 2, b2). terms of first degree.
PA = PB Þ PA 2 = PB2 \ 2h - 4 = 0 and 2k + 6 = 0
(h - a1 ) + (k - b1 )2 = (h - a 2)2 + (k - b2)2
2
Þ h = 2 and k = - 3
Þ h 2 + a12 - 2a1h + k2 + b12 - 2kb1 Hence, the point to which the origin be shifted is (2, - 3).
= h 2 + a 22 - 2a 2h + k2 + b22 - 2kb2
53. Let X = x - h, Y = y - k
Þ 2(a 2 - a1 )h + 2(b2 - b1 ) k + (a12 + b12 - a 22 - b22) = 0
Þ 0 = 7 - h, 0 = - 4 - k
Thus, the equation of locus is
1 Þ h = 7, k = -4
(a 2 - a1 )x + (b2 - b1 ) y + (a12 + b12 - a 22 - b22) = 0
2 Hence, X = x - 7 and Y = y + 4, then the point (4, 5)
But the given equation is shifted to (- 3, 9).
(a 2 - a1 )x + (b2 - b1 ) y - c = 0 54. Let P (x¢ , y¢ ) be the coordinates of the point obtained by
1 1 rotating the axes through an angle of 60°.
\ c = - (a12 + b12 - a 22 - b22) = (a 22 + b22 - a12 - b12) \The transformation matrix can be written as
2 2
é x¢ ù é cos q sin qù é x ù
50. Let G (h , k ) be the centroid of the triangle having ê y¢ú = ê - sin q cos qú ê yú
ë û ë ûë û
vertices
é 1 3ù
A(cos a , sin a ), B (sin a , - cos a ), C (1, 2) é 2 ù ê 2 ú é xù
Þ 2 ú
cos a + sin a + 1 ê- 3ú = ê ê yú
Then, h= ë û ê- 3 1 ú ë û
3 êë 2 2 úû
280 JEE Main Mathematics

é x 3y ù = F (X , Y ) (say)
é 2 ù ê 2+ 2 ú Now, in F (X , Y ), we require that the coefficient of the
Þ ê- 3ú = ê ú
ë û ê - 3x + y ú XY -term is zero.
êë 2 2 úû \ 2 [(b - a ) cos q sin q + h (cos 2 q - sin 2 q)] = 0
Þ x + 3 y = 4 and 3x - y = 2 3 Þ (a - b) sin 2q = 2h cos 2q
On solving the above equations, we have 2h
Þ tan 2q =
æ5 3 ö a-b
(x, y) º ç , ÷
è2 2 ø
58. Let P (x¢ , y¢ ) be the coordinates of the point P (x, y) after
55. Let the new coordinates of point P (x, y) after shifting the rotation of axes at an angle of 180°.
origin to (h , k ) is P (x¢ , y¢ ). é x¢ ù é cos q sin qù é x ù
Þ ê y¢ú = ê - sin q cos qú ê yú
Þ x¢ = x - h and y¢ = y - k ë û ë ûë û
Þ x = x¢ + h and y = y¢ + k Since, here q = 180°
Since, (x, y) satisfy the equation, we have é x¢ ù é -1 0 ù é x ù
Þ ê y¢ú = ê 0 -1ú ê yú
(x¢ + h )2 + ( y¢ + k )2 - 4(x¢ + h ) + 6( y¢ + k ) - 7 = 0 ë û ë ûë û
Þ (x ¢ )2 + ( y¢ )2 + 2(h - 2)x¢ + 2(k + 3) y¢ é x¢ ù é - x ù
Þ ê y¢ú = ê - yú
+ (h 2 + k2 - 4h + 6k - 7 ) = 0 ë û ë û
Since, the terms containing linear powers are missing
\ x = - x¢ and y = - y¢
\ h - 2 = 0 and k + 3 = 0
Hence, the new equation of curve,
Þ (h , k) º (2, - 3)
x - 2 y + 3 = 0 is
56. Since, the line L makes intercepts a and b along axes. (- x¢ ) - 2 (- y¢ ) + 3 = 0
Þ The line L passes through (a , 0) and (0, b). Þ - x¢ + 2 y¢ + 3 = 0
Similarly, the same line passes through ( p, 0) and (0, q) Þ x¢ - 2 y¢ - 3 = 0
after rotation
or x - 2 y - 3 = 0 in general
é pù é cos q sin qù é a ù
\ ê 0 ú = ê - sin q cos qú ê 0 ú
ë û ë ûë û Round II
é pù é a cos q ù 1. Let Ð OAB = q
Þ ê 0 ú = ê - a sin qú
ë û ë û Y
Þ p = a cos q or - a sin q = 0 …(i)
é 0 ù é cos q sin qù é 0 ù B
Also, ê qú = ê - sin q cos qú ê bú
ë û ë ûë û
é 0 ù é - b sin qù (1, 4)
Þ ê qú = ê b cos q ú
ë û ë û
q
Þ - b sin q = 0 or b cos q = q …(ii) A X
O
From Eqs. (i) and (ii), we get it is clear that
Then, OA + AB = 1 + 4 cot q + 4 + tan q
sin q = 0 Þ q = np
= 5 + 4 cot q + tan q ³ 5 + 4 = 9
Þ p = a (±1 ) or q = b (±1 ) [using Eqs. (i) and (ii)]
(using AM ³ GM)
On squaring and adding, we have
a 2 + b 2 = p2 + q 2 2. Let A (2 , 1 ), B (-2 , 4)\ AB = 5
57. Clearly, h ¹ 0 Hence, the locus is the line segment AB.
Rotating the axes through an angle q, we have 3. Since, reflection of the orthocentre of DABC in base BC
x = X cos q - Y sin q will always lie on the circumcircle of the DABC ,
y = X sin q + Y cos q therefore coordinate of a point lying on the circumcircle
æ 1 ´4 1 ´ 4ö
\ f (x, y) = ax2 + 2hxy + by2 is ç1 - ,1 - ÷ i. e. , (-1, - 1 ) and coordinates of
è 2 2 ø
= a (X cos q - Y sin q)2 + 2h (X cos q - Y sin q)
the circumcentre is (2, 0).
´ (X sin q + Y cos q) + b (X sin q + Y cos q)2
\Radius of the circumcircle of
= (a cos 2 q + 2h cos q sin q + b sin 2 q)X 2
+ 2 [(b - a ) cos q sin q + h (cos 2 q - sin 2 q))XY DABC = (2 + 1 )2 + (1 )2
+ (a sin 2 q - 2h cos q sin q + b cos 2 q)Y 2 = 10
Cartesian Coordinate System 281

4. Let the vertices of triangle are P º (-2 , - 3), Q º (-1, 0) Þ 2 = ( 1 - k )2 Þ 2 = ( k - 1 )2


and R º (7, - 6).
On squaring both sides, we get
P (–2, –3)
4 = k2 + 1 - 2k
Þ k2 - 2k - 3 = 0 Þ k = -1, 3
Thus, the set of values of k is (-1, 3).
A (x, y)
(–1, 0) Q R (7, –6) 8. BP - AP = ± 6 or BP = AP ± 6
Let A (x, y) be the circumcentre of DPQR. Þ x2 + ( y + 4 ) 2 = x 2 + ( y - 4 ) 2 ± 6
\ AP 2 = AQ 2 On squaring and simplifying, we get
2 2 2 2
Þ (x + 2) + ( y + 3) = (x + 1 ) + y 4 y - 9 = ± 3 x2 + ( y - 4 ) 2
Þ 4x + 6 y + 13 = 2x + 1 Again squaring, we get
Þ 2x + 6 y = - 12 9x 2 - 7 y2 + 63 = 0
Þ x + 3y = -6 …(i) 9. Since, the axes are rotated through an angle 45°, then
Similarly, AP = AR 2 2 we replace (x, y) by
Þ (x + 2)2 + ( y + 3)2 = (x - 7)2 + ( y + 6)2 (x cos 45° - y sin 45° , x sin 45° + y cos 45° )
4x + 6 y + 13 = - 14x + 12 y + 85 æ x y x y ö
Þ i.e. ç - , + ÷ in the given equation
è 2 2 2 2ø
Þ 18x - 6 y = 72 Þ 3x - y = 12 …(ii)
3x2 + 3 y2 + 2xy = 2.
From Eqs. (i) and (ii), we get 2 2
(x, y) º (3, - 3) æ x - yö æ x + yö æ x - yö æ x + yö
\ 3 ç ÷ +3ç ÷ +2 ç ÷ç ÷ =2
Hence, circumcentre is A(3, - 3). è 2 ø è 2 ø è 2 øè 2 ø
3 2 3 2
5. Let the new coordinates be P (x ¢ , y¢ ) after shifting origin Þ (x + y2 - 2 xy) + (x2 + y2 + 2 xy) + (x2 - y2) = 2
2 2 2
to P (x ¢ , y¢ ) i. e. , x = x ¢ + h and y = y¢ + k
Þ 4 x2 + 2 y 2 = 2 Þ 2 x2 + y2 = 1
\ (x ¢ + h )2 + ( y¢ + k)2 - 4 (x ¢ + h ) + 6 ( y¢ + k) - 7 = 0
Þ (x ¢ )2 + ( y¢ )2 + 2 (h - 2) x ¢ + 2 (k + 3) y¢ 10. y =|x - 2| Þ y = x - 2 and y = 2 - x
+ (h 2 + k2 - 4h + 6k - 7) = 0 Y
According to the question,
h - 2 = 0 and k + 3 = 0
Þ (h , k) = (2 , - 3)
6. Given, SQ 2 + SR2 = 2SP 2
Using distance formula, we get X
2 2 2 2 2 2
(x + 1) + y + (x - 2) + y = 2 [(x - 1) + y ] 1
Area of shaded region = 2 ABC = 2 × × 1 × 1 = 1
Þ 2 x2 + 2 y2 + 2 x - 4 x + 5 = 2 x 2 + 2 y 2 - 4 x + 2 2
Þ 2x + 3 = 0 11. (d) Since, the coordinates of P are (1, 0).
which is a straight line parallel to Y -axis. Let any point Q on y2 = 8x is (2t 2, 4t ).
7. Since, A ( 1, k ), B(1,1) and C(2, 1) are the vertices of a Again, let mid-point of PQ is (h , k), so
right angled DABC. éQ h = 1 ù 2t 2 + 1
h= Þ 2h = 2t 2 + 1 …(i)
ê ÐB = 90° ú 2
ë û
4t + 0 k
Y
A (1, k) and k= Þ t= …(ii)
2 2
On putting the value of t from Eq. (ii) in Eq. (i), we get
2k2
2h = + 1 Þ 4h = k2 + 2
C (2,1) 4
X' B (1,1)
X
O Y' Hence, locus of (h , k) is y2 - 4x + 2 = 0 .
Now, given that area of the triangle is 1. 12. Let the vertex A (x, y) is equidistant from B and C.
1
Then, area (D ABC ) = ´ AB ´ BC \ (x - 1 )2 + ( y - 3)2 = (x + 2)2 + ( y - 7)2
2
1 Þ 6x - 8 y + 43 = 0
Þ 1 = ´ ( 1 - 1 )2 + ( 1 - k )2 ´ 1 Only the option (c) satisfy it.
2
282 JEE Main Mathematics

13. OP is inclined at angle q with X-axis and OQ is inclined Again now, 2a = q- f


at angle a with X-axis. tan q - tan f
\ tan 2 a = tan (q - f ) =
1 + tan q tan f
P(cos q, sin q)
k (a + h ) - k (a - h )
Y =
a 2 - h 2 + k2
L (Line of reflection)
2q – a q – a Þ a 2 - h 2 + k2 = 2 hk cot 2 a
2
q Hence, the locus of R is x2 - y2 + 2xy cot 2 a - a 2 = 0
X¢ X
O –q 16. As the third vertex lies on the line y = x + 3,its coordinates
a are of the form (x, x + 3). The area of the triangle with
vertices (2, 1 ), (3, - 2) and (x, x + 3) is given by
Q (cos (a – q), sin (a – q)) 1
Y¢ Þ [2 (-2 - x - 3) + 3 (x + 3 - 1) + x (1 + 2)] = 5
2
(cos(– q), sin(– q)) Þ |2x - 2| = 5 (given)
From the above figure, \ 2x - 2 = ± 5 Þ x = 3/2, 7/2
æ aö a Thus, the coordinates of the third vertex are (7/2, 13/2)
LOX = q - ç q - ÷ =
è 2ø 2 or (–3/2, 3/2).
\ Q is obtained from P by reflection in the line through 1
a 17. [( p + 1) (3 - 2 p) + (2 p + 1) (2 p - 1)
the origin with slope tan . 2
2
+ (2 p + 2) (1 - 3)] = 0
14. Since, AB = AC = 2 1
Þ [3 p - 2 p + 3 - 2 p + 4 p2 - 1 - 4 p - 4] = 0
2
C 2
1
B (3, 1) Þ [2 p2 - 3 p - 2] = 0
2
Þ 2 p2 - 3 p - 2 = 0
15°
Þ (2 p + 1) ( p - 2) = 0
45° 1
A Þ p=- ,2
(2, 0) 2
The slope AB is 1. Hence, AB is inclined at 45° with the 18. (b) Let P be (x, y) and we have
X-axis and AC is inclined at 60° with the X-axis. A = ( a + b, a - b ), B = ( a - b, a + b )
Equation of AC is
Here, PA = PB Þ PA 2 = PB2
y = 3 (x - 2). Þ [x - ( a + b )] 2 + [ y - ( a - b )] 2
The coordinates of C is (2 + 2 cos 60° , 0 + 2 sin 60° ) = [x - (a - b)] 2 + [ y - (a + b)] 2
æ 1 3ö Þ [x - (a + b)] 2 - [x - (a - b)] 2
or ç2 + , ÷
è 2 2ø = [ y - (a + b)] 2 - [ y - (a - b)] 2
Þ [x - (a + b) + x - (a - b)] [x - (a + b) - x + a - b]
15. Let Ð RPQ = q and Ð RQP = f
= [ y - (a + b) + y - (a - b)] [ y - (a + b) - y + a - b]
Y
Þ (2x - 2a ) (-2b) = (2 y - 2a ) (-2b)
Þ x- y =0
19. (c) We have orthocentre and centroid of a triangle be
A(-3, 5) and B(3, 3) respectively and C circumcentre.

X X¢ A(–3, 5) B(3,3) C
M
Clearly, AB = (3 + 3)2 + (3 - 5)2 = 36 + 4 = 2 10
Y¢ We know that, AB : BC = 2 : 1
\ q- f =2a Þ BC = 10
Let RM ^ PQ , so that RM = k, Now, AC = AB + BC = 2 10 + 10 = 3 10
MP = a - h and MQ = a + h Since, AC is a diameter of circle.
RM k RM k AC 3 10 5
Then, tan q = = and tan f = = \ r= Þ r= =3
MP a - h MQ a + h 2 2 2
Cartesian Coordinate System 283

20. Let P (x1 , y1 ) be any point on the curve y = x . Let AB = c = 82 + 62 = 10


Clearly,y1 = x1 Þ x1 = y12 [Q (x1 , y1 ) lies on y = x] OB = a = 6 and OA = b = 8
\ The point is P ( y12, y1 ) Also, let incentre is (h k), then
æ3 ö ax + bx2 + cx3
Now, let the given point be A ç , 0÷ , then h= 1 (here, x1 = 8, x2 = 0, x3 = 0)
è2 ø
a+ b+ c
2
æ 3ö 9 6 ´ 8 + 8 ´ 0 + 10 ´ 0 48
PA = ç y12 - ÷ + y12 = y14 - 3 y12 + + y12 = = =2
è 2ø 4 6 + 8 + 10 24
9 5 ay1 + by2 + cy3
= y14 - 2 y12 +
= ( y12 - 1)2 + and k= (here, y1 = 0, y2 = 6, y3 = 0)
4 4 a+ b+ c
Clearly, PA will be least when y12 - 1 = 0.
6 ´ 0 + 8 ´ 6 + 10 ´ 0 48
5 5 = = =2
Þ PA min = 0 + = 6 + 8 + 10 24
4 2
\ Incentre is (2, 2)
21. (b) Let P (x, y) be the original position of the point w.r.t.
the original axes. Let us move the origin at new position 24. Given points are (-8, 5) and (6, 5) in which
to (h , k ). Hence, the position of the same point P in the y-coordinate is same, i.e. these points lie on horizontal
new system is line y = 5.
x¢ = x - h 3y=x+7
y¢ = y - k
Here, (h , k ) = (1, 2)
(–8, b) (6, b)
\ x ¢ = (x - 1 ), y¢ = ( y - 2)
According to the question,
y2 - 8x - 4 y + 12 º ( y - 2)2 - 4 a (x - 1 )
Þ y2 - 8x - 4 y + 12 º y2 - 4 y + 4 - 4a x + 4 a
On comparing respective coefficients, we get (–8, 5) (6, 5)

4a = 8 Þ a = 2
22. Since, the centroid divides the join of orthocentre and
circumcentre in the ratio 2 : 1. Let (-8, b) and (6, b ) are the coordinates of the other
vertices of rectangle as shown in the figure.
Let the circumcentre of triangle is (a , b ).
Since, the mid-point of line joining points (-8, 5) and
æ 2a - 3 2b + 5 ö
\ G (3, 3) = G ç , ÷ (6, b ) lies on the line 3 y = x + 7.
è 2+1 2+1ø æ 5 + b ö -8 + 6
\ 3ç ÷= +7
æ 2a - 3 2b + 5 ö è 2 ø 2
Þ G (3, 3) = G ç , ÷
è 3 3 ø Þ 15 + 3 b = - 2 + 14
2a - 3 2b + 5 Þ 3b = -3
Þ = 3 and =3
3 3 Þ b = -1
Þ 2a - 3 = 9 and 2b + 5 = 9 Now, area of rectangle = |-8 - 6 | ´ |b - 5|
Þ 2a = 12 and 2b = 4 = 14 ´ 6 = 84
Þ a = 6 and b = 2
25. Using section formula, the coordinates of the point P,
23. Given equation of line is 3x + 4 y - 24 = 0 which divides AB internally in the ratio 3 : 2 are
For intersection with X-axis put y = 0 æ3 ´ 2 + 2 ´ 1 3 ´ 4 + 2 ´ 1ö æ 8 14 ö
Þ 3x - 24 = 0 Þ x = 8 Pç , ÷ºPç , ÷
è 3+2 3+2 ø è5 5 ø
For intersection with Y -axis, put x = 0
Þ 4 y - 24 = 0 Þ y = 6 Also, since the line L passes through P, hence
\ A(8, 0) and B (0, 6) æ 8 14 ö
substituting the coordinates of P ç , ÷ in the
è5 5 ø
B(0,6)
equation of L : 2x + y = k, we get
æ 8 ö æ 14 ö
Þ 2ç ÷ + ç ÷ =k
è5ø è 5 ø
Þ k =6
O A(8,0)
284 JEE Main Mathematics

26. Given mid-points of a triangle are (0, 1), (1, 1) and (1, 0). 28. Let (x, y) denotes the coordinates in A , B and C plane.
Plotting these points on a graph paper and make a
(x - 1 )2 + y2 1
triangle. Then, =
Y (x + 1 )2 + y2 9
Þ 9x2 + 9 y2 - 18x + 9 = x2 + y2 + 2x + 1
C(0, 2)
Þ 8x2 + 8 y2 - 20x + 8 = 0
5
2 (0, 1) (1, 1) Þ x2 + y2 - x + 1 = 0
2
\ A , B and C lie on a circle with C (5 /4, 0).
X¢ X
B (1, 0) A(2, 0) 29. Now, AB = (1 - h )2 + (1 - k)2
(0, 0)
2
Y¢ BC = (2 - 1 )2 + (1 - 1 )2 = 1
So, the sides of a triangle will be 2, 2 and 22 + 22 and CA = (h - 2)2 + (k - 1 )2
i.e., 2 2 Since, AC 2 = AB2 + BC 2
2 ´ 0 + 2 2 ×0 + 2 ×2 Þ h 2 + 4 - 4h + k2 + 1 - 2k
x-coordinate of incentre =
2+2+2 2 = 1 + h 2 - 2h + k2 + 1 - 2k
2 2- 2 Þ h =1 …(i)
= ´
2+ 2 2- 2 1
\Area of DABC = ´ AB ´ BC
=2 - 2 2
1
27. The third vertex lies on 2x + 3 y = 9 Þ 1 = ´ (1 - h )2 + (1 - k )2 ´ 1
2
æ 9 - 2x ö
i.e., ç x, ÷ Þ 2 = (k - 1 )2 [from Eq. (i)]
è 3 ø
\Locus of centroid is Þ 2
4 = k + 1 - 2k Þ (k - 3)(k + 1 ) = 0
æ 9 - 2x ö Þ k = - 1, 3
ç 2 - 2 + x -3 + + 1÷
ç , 3 ÷ = (h , k )
ç 3 3 ÷ 30. Let ABC be the triangle with A(1, 1 ). The mid-point of
è ø AB is (-1, 2 ).
A (2, –3) Þ B (-3, 3)
The mid-point of AC is (3, 2)
Þ C (5, 3)
æ 7ö
Hence, centroid is ç1, ÷ .
è 3ø
B(2, –1) C x, 4 – 2x 31. Let the third vertex be C (x1 , y1 ).
3
x æ 2 - 2 + x1 -3 + 1 + y1 ö
The centroid is ç , ÷.
\ h= è 3 3 ø
3
3 - 2x æ x1 y1 - 2 ö
k= i.e., ç , ÷
9 è3 3 ø
Þ 9k = 3 - 2(3h ) It lies on the line 2x + 3 y = 1,
Þ 9k = 3 - 6h
2 æ y - 2ö
Þ 2h + 3k = 1 \ (x1 ) + 3 ç 1 ÷ =1
3 è 3 ø
Hence, locus of a point is
Hence, the locus of C is 2x + 3 y = 9.
Þ 2x + 3 y = 1
12
Straight Lines

A straight line is the locus of all those points which are collinear with two IN THIS CHAPTER ....
given points. One and only one line can be drawn from any two given points.
Straight Line
Let A( x1 , y1 ) and B( x2 , y2 ) be two given fixed points. Let P ( x , y ) be a point such
that the points A, B and P always lie in the same straight line, then the locus Slope of Gradient of a Line
of all such points will be the straight line through AB. Equations of Lines Parallel
to Axes
P(x, y)
Various Forms of Equations of
B(x2, y2) a Line

A(x1, y1) Angle between Two Lines


Equation of a Line Parallel to a
Given Line
Slope or Gradient of a Line
Equation of a Line Perpendicular
If a line AB makes an angle q with the positive X-axis, then m or tan q is to a Given Line
called the slope or gradient of the line.
Area of Parallelogram
i.e. m = tan q
Distance of a Point from a Line
Y
Equation of Internal and
External Bisectors of Angles
B
q between Two Lines
X¢ X
O A Foot of Perpendicular
Image of a Point with Respect to
a Line
Y¢ Family of Lines
If 0° < q < 90°, then q is an acute angle.
If q = 0°, either the line is X-axis or it is parallel to X-axis.
If 90° < q < 180°, then q is an obtuse angle.
286 JEE Main Mathematics

Slope of a Line when Coordinates If a line parallel to X-axis is at a distance b and below
X-axis, then equation is y = - b
of Any Two Points are Given
Equation of X-axis is y = 0.
If a line passes through A ( x1 , y1 ) and B ( x2 , y2 ), then the
y - y1
slope of line AB is given by m = tan q = 2 Equation of a Line Parallel to Y -axis
x2 - x1
Let AB be a straight line parallel to Y -axis and at a
Y
distance a from it. Then, the abscissa of every point on
Q B (x2, y2) AB is a. So, it can be treated as the locus of a point at
(x1, y1)
A q a distance a from Y -axis, then x = a is an equation of
line AB.
Y
q A
X¢ X
O
a

X¢ X
O

x=a
● If a angle q is measured from X-axis in anti-clockwise
or clockwise direction, then sign will be taken as
positive or negative. B

● Slope of any line parallel to X-axis is zero.
● Slope of any line parallel to Y -axis i.e. perpendicular to If a line parallel to Y -axis is at a distance a and to the
X-axis is infinite. left of Y -axis, then its equation is x = - a.
● If three points A, B and C are collinear, then slope of Equation of Y -axis is x = 0
AB = slope of BC = slope of CA
e.g. Find the angle between the X-axis and the line
Various Forms of Equations of a Line
joining the points (3, – 1) and (4, – 2) is Slope intercept Form
Given, ( x1 , y1 ) = ( 3, - 1 ) Y The equation of a line with slope m and making an
and ( x2 , y2 ) = ( 4, - 2) intercept c on Y -axis is
Slope of line y = mx + c
O
y - y1 - 2 + 1 X¢ X
m= 2 = –1 1 2 3 4 Y
x2 - x1 4-3 –2 (3,–1)
(4,–2)
-1
tan q = Y¢
P(x, y)
1
Þ tan q = - 1 c
X¢ q X
Þ tan q = - tan 45°(Q1 = tan 45°) O L

Þ tan q = tan (180° - 45° ) Y¢


[Q tan (180° - q ) = - tan q]
Þ tan q = tan 135° If the line passes through the origin, then c = 0, then
equation of the line will become y = mx.
Þ q = 135°
Point slope Form
Equations of Lines Parallel to Axes The equation of a line which passes through a point
Equation of a Line Parallel to X-axis P ( x1 , y1 ) and has the slope m, is
Let AB be a straight line parallel to X-axis at a distance b y - y1 = m ( x - x1 ).
from it. Then, the ordinate of every point on AB is b. So, Y
it can be treated as the locus of a point at a distance b P(x, y)
from X-axis, then y = b is an equation of line AB.
Y
P1(x1, y1)
y=b
A B X¢ X
b O
X¢ X
O


Straight Lines 287

Two point Form where, r is the directed distance between the points
( x , y ) (any point on the line) and ( x1 , y1 ).
The equation of a line passing through two points A( x1 , y1 )
and B( x2 , y2 ) is given by Note
y - y1 x - x1 • x = x1 + r cos q, y = y1 + r sin q are called parametric
= equations of straight line AB.
y2 - y1 x2 - x1
• If point P above Q, then r is positive and if P below Q, then r is
Y
negative.
P(x, y) P2(x2, y2)
General Form
P1(x1, y1) General equation of first degree in x and y is
X¢ X ax + by + c = 0, which represents an equation of line.
O Slope of this line is given by
Y¢ (coefficient of x ) a
- =-
(coefficient of y ) b
Intercept Form
Given equation of a line can be written as
If a line makes intercept a and b on X-axis and Y -axis
respectively, then the equation of line is - ax æ - c ö
y= +ç ÷
x y b è b ø
+ = 1.
a b x y
The intercept form of the line is + = -1
Y c/ a c/ b

B(0, b) Example 1. If a DABC has vertices A ( - 1, 7), B ( - 7, 1)


b and C(5, - 5), then its orthocentre has coordinates
(JEE Main 2020)
A(a, 0)
X (a) ( 3, - 3) (b) ( - 3, 3)
O a
æ 3 3ö æ3 3ö
(c) ç - , ÷ (d) ç , - ÷
Intercept may be -ve but length of intercept is always +ve. è 5 5ø è5 5ø

Normal (Perpendicular) Form Sol. (b) The vertices of given DABC are A( -1 , 7), B( -7, 1)
If perpendicular distance of the origin from the line is p and and C(5, - 5).
perpendicular drawn from origin to the line is making an 1 - ( -5) 6 1
Since, slope of line BC is m1 = = =-
angle a from the positive direction of X-axis, then equation -7 - 5 -12 2
of line is x cos a + y sin a = p, 0 £ a £ p
A (–1, 7)
Y

p B (–7, 1) C (5, –5)


a
X¢ X So, slope of altitude through vertex A is
O
1
m1¢ = - = 2
Y¢ m
\Equation of altitude through vertex A is
Parametric Form or the Distance Form or y - 7 = 2( x + 1) Þ 2x - y + 9 = 0 …(i)
Symmetric Form Similarly, the slope of line CA is
The equation of the straight line passing through ( x1 , y1 ) and 7 - ( -5) 12
m2 = = = -2
making an angle q with the positive direction of X-axis is -1 - 5 -6
x - x1 y - y1 1
= =r So, slope of altitude through vertex B is m2¢ =
cos q sin q 2
Y
\ Equation of altitude through vertex B is
P(x, y) B
1
y - 1 = ( x + 7) Þ x - 2y + 9 = 0 …(ii)
r 2
(x1, y1) Q q
As, the point of intersection of altitudes (i) and (ii) is the
q orthocentre, so from lines (i) and (ii), the orthocentre is ( -3, 3).
X
O A Hence, option (b) is correct.
288 JEE Main Mathematics

Example 2. A ray of light coming from the point (2, 2 3) is On putting the value of a and b in Eq. (i), we get
x y
incident at an angle 30° on the line x =1at the point A. The ray + = 1 Þ 8x - 6y = -48
gets reflected on the line x =1and meets X-axis at the point B. -6 8
Þ 4x - 3y + 24 = 0
Then, the line AB passes through the point (JEE Main 2020)

æ 1 ö æ 3ö Example 4. A straight line L at a distance of


(a) ç3, - ÷ (b) çç 4, - ÷ (c) (3, - 3) (d) ( 4, - 3)
è 3ø è 2 ÷ø 4 units from the origin makes positive intercepts on the
coordinate axes and the perpendicular from the origin to this
Sol. (c) According to the given situation, if we draw the diagram. line makes an angle of 60° with the line x + y = 0. Then, an
Y equation of the line L is (JEE Main 2019)
P¢ (0, 2 Ö3) P (2, 2 Ö3)
(a) x + 3y = 8 (b) ( 3 + 1) x + ( 3 - 1)y = 8 2
30º 30º (c) 3x + y = 8 (d) ( 3 - 1) x + ( 3 + 1)y = 8 2
A Sol. (d) According to the question, we have the following figure.
120º
X
O B (0,b)
x=1
M

60°
a (a,0)
Q The line x = 1is equidistance, from point P(2, 2 3) and the
O x+
Y-axis. y=
0 x y
— + — =1
a b
So, P ¢ (0 , 2 3) and according to the concept of reflection line
passes through P ¢(0 , 2 3). Let q be the inclination of the line x + y = 0. Then,
As the angle ÐP ¢ BX = ÐABX = 120 ° tan q = - 1 = tan (180 ° - 45° )
\Slope of line AB = tan120 ° = - 3 Þ tan q = tan 135°
\Equation of line AB is Þ q = 135°
y - 2 3 = - 3( x - 0) Þ a + 60 ° = 135° Þ a = 75°
Since, line L having perpendicular distance OM = 4.
Þ 3x + y = 2 3 …(i)
So, equation of the line ‘L’ is
From the options, the line (i) passes through the point (3, - 3). x cos a + y sin a = 4
Þ x cos 75° + y sin 75° = 4
Example 3. If a straight line passing through the point
Þ x cos ( 45° + 30 ° ) + y sin ( 45° + 30 ° ) = 4
P( - 3, 4) is such that its intercepted portion between the
ì 3 1 ü ì 3 1 ü
coordinate axes is bisected at P, then its equation is Þ xí - ý+ y í + ý= 4
(JEE Main 2019) î2 2 2 2 þ î2 2 2 2 þ
(a) x - y + 7 = 0 (b) 4x - 3y + 24 = 0 Þ ( 3 - 1) x + y ( 3 + 1) = 8 2
(c) 3x - 4y + 25 = 0 (d) 4x + 3y = 0
Example 5. If the perpendicular bisector of the line
Sol. (b) Let the equation of required line having intercepts a and b segment joining the points P(1, 4) and Q (k, 3) has y-intercept
with the axes is
x y equal to -4, then a value of k is (JEE Main 2020)
+ =1 …(i)
a b (a) 15 (b) -4 (c) 14 (d) -2
Y Sol. (b) The mid-point of line joining point P(1, 4) and Q(k,3) is
B (0,b)
æ1+ k 4 + 3 ö
Mç , ÷ and slope of line perpendicular to line PQ is
è 2 2 ø
b P (–3,4) 1- k
– = k - 1. [Q m1 × m2 = - 1]
4 -3
A (a,0)
\ Equation of perpendicular, bisector of the line segment
X joining the points P and Q is
O a
7 æ 1+ k ö
y – = (k – 1) ç x – ÷ …(i)
Now, according to given information, 2 è 2 ø
P is the mid-point of AB and line (i) passes through (0 ,–4), so
æ a bö 7 æ 1+ k ö 2
\ P = ç , ÷ = ( -3, 4) [given] –4 – = (k – 1) ç – ÷ Þ 15 = k – 1Þ k = ±4
è2 2 ø 2 è 2 ø
Þ ( a, b) = ( -6, 8)
Straight Lines 289

Angle between Two Lines Example 7. Suppose that the points (h, k), (1, 2) and ( -3, 4)
lie on the line L1. If a line L2 passing through the points (h, k)
Let the equations of two lines be y = m1x + c1 and and (4, 3) is perpendicular to L1, then k / h equals (JEE Main 2019)
y = m2x + c2, then angle between these lines is given by
1 1
æ m - m2 ö (a) - (b) (c) 3 (d) 0
q = tan- 1 çç 1 ÷÷ 7 3
è 1 + m1m2 ø Sol. (c) Given, points (1, 2), (-3, 4) and (h, k) are lies on line L1,
so slope of line L1 is
4 -2 k-2
m1 = =
q -3 - 1 h - 1
-1 k - 2
q1 q2
Þ m1 = = …(i)
2 h -1
Þ 2(k - 2) = - 1 (h - 1)
If q is the angle between the lines a1x + b1 y + c1 = 0 and Þ 2k - 4 = - h + 1
a2x + b2 y + c2 = 0, then Þ h + 2k = 5 …(ii)
a b - a1b2 and slope of line L2 joining points (h, k) and
q = tan- 1 2 1
a1a2 + b1b2 3 -k
(4, 3), is m2 = …(iii)
4 -h
If two lines are parallel to each other, then
a b Since, lines L1 and L2 are perpendicular to each other.
m1 = m2 , a2b1 - a1b2 = 0 or 1 = 1 \ m1m2 = - 1
a2 b2
æ 1 ö æç 3 - k ö÷
If two lines are perpendicular to each other, then Þ ç- ÷ ç = -1 [from Eqs. (i) and (iii)]
m1m2 = - 1 or a1a2 + b1b2 = 0 è 2 ø è 4 - h ÷ø
Þ 3 - k = 8 - 2h Þ 2h - k = 5 …(iv)
Note Coordinates of point of intersection of two lines
a1 x + b1 y + c1 = 0 and a2 x + b2 y + c 2 = 0 are On solving Eqs. (ii) and (iv), we get
(h, k) = (3, 1)
ìïæ b1c 2 - b2c1 ö æ a2c1 - a1c 2 ö üï
íçç ÷, ç ÷ý k 3
ïîè a1b2 - a2 b1 ÷ø çè a1b2 - a2 b1 ÷ø ïþ So, = =3
h 1
Example 6. The equation of one of the straight lines
which passes through the point (1, 3) and makes an angles Condition of Concurrency
tan-1 ( 2 ) with the straight line, y + 1 = 3 2 x is (JEE Main 2021) of Three Lines
(a) 4 2x + 5y - (15 + 4 2) = 0 (b) 5 2x + 4y - (15 + 4 2) = 0 Let the equation of three lines be L1 º a1x + b1 y + c1 = 0,
L2 º a2x + b2 y + c2 = 0 and L3 º a3 x + b3 y + c3 = 0, then
(c) 4 2x + 5y - 4 2 = 0 (d) 4 2x - 5y - (5 + 4 2) = 0
a1 b1 c1
Sol. (a) Given line passes through point (1, 3).
three lines will be concurrent, if a2 b2 c2 = 0
So, y = mx + c
3 =m+ c a3 b3 c3
m-3 2 Another condition Three lines L1 , L2 and L3 are
2=
1 + 3 2m concurrent iff there exists constants l1 , l2 and l3 not all
Taking positive sign, zero such that l1L1 + l2L2 + l3 L3 = 0
2 (1 + 3 2m) = m - 3 2 i.e. l1( a1x + b1 y + c1 ) + l2( a2x + b2 y + c2 )
Þ 2 + 6m = m - 3 2 + l3 ( a3 x + b3 y + c3 ) = 0
-4 2 Example 8. The lines a1x + b1y + c 1 = 0, a2x + b 2y + c 2 = 0
Þ 5m = - 4 2 Þ m =
5 and a3x = 0 are concurrent, if b1c 2 - b 2c 1 is equal to
Taking negative sign, (a) 10 (b) 1
2 (1 + 3 2m) = - m + 3 2 (c) 0 (d) None of these
Þ 2 + 6m = - m + 3 2 Sol. (c) Let the lines a1x + b1y + c1 = 0 , a2x + b2y + c2 = 0 and a3 x = 0
2 2 are concurrent.
Þ 7m = 2 2 Þ m = a1 b1 c1
7
-4 2 4 2 \ a2 b2 c2 = 0
If m= , then c = 3 + a3 0 0
5 5
-4 2 15 + 4 2 Þ a3 ( b1 c2 - b2 c1)
So, y = x+ Þ 4 2x + 5y - (15 + 4 2) = 0
5 5 Þ b1c2 - b2c1 = 0 (Q a3 ¹ 0)
290 JEE Main Mathematics

Equation of a Line Parallel The length of the perpendicular from the origin to the
|c|
to a Given Line line ax + by + c = 0 is
a 2 + b2
Let ax + by + c = 0 be any straight line, then the equation
of a line parallel to given line is ax + by + l = 0, where l is ● If the length of perpendicular drawn from the origin to
an arbitrary constant, which can be determined by some the line x/ a + y/ b = 1 is p, then
given conditions. 1 1 1
2
= 2 + 2
e.g. Find equation of the line parallel to the line p a b
3x - 4 y + 2 = 0 and passing through the point ( - 2, 3) is ● The equation of a line which makes a triangle of
Given equation of a line is centroid ( x1 , y1 ) with coordinates axes, is
3x - 4 y + 2 = 0 …(i) x y
+ =1
Equation of line parallel to Eq. (i) is 3x1 3 y1
3x - 4 y + k = 0 …(ii)
Example 9. If p and p¢ be perpendiculars from the origin
Q Line (ii) passes through the point ( - 2, 3) i.e. put
x = - 2, y = 3 in Eq. (ii). upon the straight lines x sec q + y cosec q = a and
x cos q - y sin q = a cos 2 q respectively, then the value of the
\ 3( - 2) - 4 ´ 3 + k = 0 expression 4p 2 + p¢ 2 is
Þ - 6 - 12 + k = 0 Þ k = 18
(a) a2 (b) 3a2 (c) 2a2 (d) 4a2
Put k = 18 in Eq. (ii), we get
3x - 4 y + 18 = 0 Sol. (a) Since, p = length of the perpendicular from (0,0) on
x sec q + y cosec q = a
Equation of a Line Perpendicular \ p=
a
=
a sin 2q
to a Given Line 2
sec q + cosec q2 2
Let ax + by + c = 0 be an equation of straight line, then Þ 2p = a sin 2q …(i)
equation of straight line perpendicular to given line is Also, p¢ = length of perpendicular from (0,0) on
bx - ay + l = 0, where l is an arbitrary constant. x cos q - y sin q = a cos 2 q
e.g. The equation of the line perpendicular to the line a cos 2q
\ p¢ = = a cos 2q …(ii)
x - 7 y + 5 = 0 and having x-intercept 3 is cos2 q + sin2 q
Given equation of line is On squaring and adding Eqs. (i), (ii), we get
x - 7y + 5 = 0 …(i) 4 p 2 + p ¢ 2 = a2
Equation of line perpendicular to Eq. (i) is
7x + y + k = 0 …(ii) Example 10. Slope of a line passing through P (2, 3) and
Q Line (ii) passes through the point (3, 0) i.e., put intersecting the line, x + y = 7 at a distance of 4 units from P,
x = 3, y = 0 in Eq. (i) is (JEE Main 2019)

7 ´ 3 + 0 + k = 0 Þ k = - 21 1- 5 7 -1 1- 7 5 -1
(a) (b) (c) (d)
Put k = - 21 in Eq. (ii), we get 1+ 5 7 +1 1+ 7 5 +1
7x + y - 21 = 0 Sol. (c) Let the slope of line is m, which is passing through P(2, 3).
Y
Distance of a Point from a Line
Let ax + by + c = 0 be any equation of line and P ( x1 , y1 ) be 8
any point in space, then the perpendicular distance d of 7
point P from the line is given by 6
ax1 + by1 + c 5
d=
a 2 + b2 4 R
d
3 P(2,
P(x1, y1) 3)
2 q
4 Q
d 1
X¢ X
1 2 3 4 5 6 7 8 9
A Y¢ x+y=7
ax + by + c = 0
Straight Lines 291

Since, the distance of a point ( x1, y1) from the line Y


ax1 + by1 + c
ax + by + c = 0 is d = .
a2 + b 2 Q(x2, y2)

\The distance of a point P(2, 3) from the line x + y - 7 = 0 , is


P(x1, y1)
|2 + 3 - 7| 2 X¢ X
d= = = 2 O
1+ 1 2 ax + by + c = 0
Now, in DPRQ, Y¢
2
QR = 16 - d = 16 - 2 = 14 (ii) If ax1 + by1 + c and ax2 + by2 + c both are of opposite
ax + by1 + c
d 2 1 m +1 é m2 - m1 ù signs to each other and hence, 1 < 0, then
\ tan q = = = = êQ tan q = ú ax2 + by2 + c
QR 14 7 1- m ë 1 + m1m2 û
the points P ( x1 , y1 ) and Q( x2 , y2 ) lie on opposite side
m +1 1
Þ =± of the line ax + by + c = 0
1- m 7
m +1 1 m +1 1 Y
Þ = or =-
1- m 7 1- m 7 Q(x2, y2)
1- 7 -1 - 7
Þ m= or m =
1+ 7 7 -1 P(x1, y1)
X¢ X
O
Distance between Two Parallel Lines

Distance between two parallel lines a1x + b1 y + c1 = 0 and
a1x + b1 y + c2 = 0 is given by (iii) If origin lie on line, then the line is known as origin
side.
c1 - c2
d=
a12 + b12 Example 12. The points (2, 1) and ( -3, 5) are lies along the
line 3x - 2y + 1 = 0
Example 11. The distance between the lines (a) same side (b) opposite side
5 x - 12y + 65 = 0 and 10 x - 24y - 39 = 0 is (c) can’t say (d) None of these
169 169
(a) (b) Sol. (b) Let l = 3x - 2y + 1
5 2
Then, the value of l at (2, 1) is
(c) 169 (d) None of these
3 ´ 2 - 2 ´1 + 1 = l1 (say)
Sol. (b) Given lines are 5x - 12y + 65 = 0 Þ l1 = 5
39
and 10 x - 24y - 39 = 0 Þ 5x - 12y - =0 and the value of l at ( - 3, 5) is
2
Since, given lines are parallel. 3 ´ ( - 3) - 2 ´ 5 + 1 = l 2 (say)
\ Distance between two lines Þ l 2 = - 18
39 169 Since, l1 and l 2 are of opposite signs, therefore, the two
- - 65 -
c2 - c1 2 2 points are on the opposite sides of the given line.
= = =
a2 + b 2 5 2 + ( - 12) 2 25 + 144
169 Equation of Internal and External
169
= 2 = Bisectors of Angles between Two Lines
169 2
The equation of bisectors of the angle between the lines
a1x + b1 y + c1 = 0 and a2x + b2 y + c2 = 0 is given by
Position of Two Points (x1 , y1 ) and
a1x + b1 y + c1 a x + b2 y + c2
(x2 , y2 ) Relative to a Given Line 2 2
=± 2
Let the equation of line be ax + by + c = 0 and P ( x1 , y1 ) a1 + b1 a22 + b22
and Q( x2 , y2 ) are two given points.
(i) If ax1 + by1 + c and ax2 + by2 + c both are of the same a 1x + b 1 y + c 1 = 0
ax1 + by1 + c
sign and hence, > 0, then the points
ax2 + by2 + c
P ( x1 , y1 ) and Q( x2 , y2 ) lie on same side of line
a 2 x + b 2y + c 2 = 0
ax + by + c = 0
292 JEE Main Mathematics

(i) If a1a2 + b1b2 > 0, then the bisector for positive sign P(x1, y1)
gives the obtuse angle bisector and for negative sign ax + by + c = 0
gives the acute angle bisector.
(ii) If a1a2 + b1b2 < 0, then bisector corresponding to
positive and negative give the acute and obtuse
angle bisectors respectively. Q(x, y)

Example 13. The equation of line bisecting the obtuse Example 14. Let L denote the line in the xy-plane with x
angle between y - x = 2 and 2y + x = 5 is and y intercepts as 3 and 1 respectively. Then, the image of
y - x - 2 x + 2y - 5 the point ( -1, - 4) in this line is (JEE Main 2020)
= , where n is
2 n æ11 28 ö æ 29 8 ö
(a) ç , ÷ (b) ç , ÷
(a) 5 è5 5 ø è 5 5ø
(b) 2 æ 8 29 ö æ 29 11ö
(c) ç , ÷ (d) ç , ÷
(c) 5 è5 5 ø è 5 5ø
(d) None of the above
Sol. (a) It is given that the line ‘L’ made x and y intercepts 3 and 1
Sol. (c) Equations of given lines are respectively, so equation of line L is
-x + y = 2 …(i) x y
+ =1
and x + 2y = 5 …(ii) 3 1
Since, a1a2 + b1b2 = - 1 + 2 = 1 > 0 Þ x + 3y = 3 ...(i)
\Equation of bisector of the obtuse angle is Now, let image of point ( - 1, - 4) with respect to line (i) is
x - y + 2 - x - 2y + 5 ( x1, y1), then
=
2 5 x1 + 1 y1 + 4
= = -2
y - x - 2 x + 2y - 5 1 3
Þ = æ - 1 - 12 - 3 ö 16
2 5 çç ÷÷ =
Hence, n= 5 è 1+ 9 ø 5
é ù
êSince, image (h, k) or ( x , y ) w. r. t. the lineú
Foot of Perpendicular ê 1 1 ú
The foot M of perpendicular drawn from a point P ( x1 , y1 ) êmirror ax + by + c = 0 is given by ú
êh - x k - y - 2( ax + by + c ) ú
to the line ax + by + c = 0 is given by ê 1
= 1
= 1 1 ú
2 2
x - x1 y - y1 êë a b a +b úû
=
a b 16 11
Þ x1 = -1=
( ax1 + by1 + c) 5 5
=-
a 2 + b2 æ 16 ö
and y1 = ç ´ 3÷ - 4
P (x1, y1)
è5 ø
28
=
5
æ11 28 ö
\ ( x1, y1) = ç , ÷
M è5 5 ø
ax + by + c = 0

Image of a Point with Respect to a Line Family of Lines


If a1x + b1 y + c1 = 0 and a2x + b2 y + c2 = 0 are two
The image of a point ( x1 , y1 ) with respect to the line
non-parallel lines, then
mirror ax + by + c = 0 be Q( x , y ) is given by
( a1x + b1 y + c1 ) + l( a2x + b2 y + c2 ) = 0
x - x1 y - y1
= represents the family of all those lines, which passes
a b
2( ax1 + by1 + c) through the point of intersection of given lines.
=-
a 2 + b2
Straight Lines 293

If algebraic sum of the perpendiculars drawn upon a x + y -1= 0 …(iii)


straight line from any numbers of fixed points is zero, and 2x + 3y - 1 = 0 …(iv)
then the line always passes through a fixed point. On solving Eqs. (iii) and (iv), we get
a 1x + b 1y + c 1 = 0 x = 2, y = -1

Example 16. Let C be the centroid of the triangle


with vertices (3, -1), (1, 3) and (2, 4). Let P be the point of
intersection of the lines x + 3y - 1 = 0 and 3x - y + 1 = 0.
a 2x + b 2y + c 2 = 0
Then, the line passing through the points C and P also passes
If there are three points, such that algebraic sum of through the point (JEE Main 2020)
perpendicular drawn from them or a line is zero, then fix (a) ( -9, - 7) (b) ( -9, - 6)
point is centroid of D. (c) (7, 6) (d) (9, 7)

Example 15. The straight lines x( a + 2 b) + y( a + 3b) = a + b, Sol. (b) The centroid of the triangle having vertices (3, - 1), (1, 3) and
æ3 + 1+ 2 -1+ 3 + 4ö
for different values of a and b pass through a fixed point. (2, 4) is C ç , ÷ = (2, 2) and equation of line
è 3 3 ø
(a) (2, 1) (b) (–2, 1) passes through point of intersection P of lines
(c) (2, –1) (d) (–2, –1) x + 3y - 1 = 0 and 3x - y + 1 = 0 is
Interpret (c) Given equation can be rewritten as ( x + 3y - 1) + l(3x - y + 1) = 0 … (i)
a( x + y - 1) + b(2x + 3y - 1) = 0 …(i) Q Line (i) passes through point C(2, 2), so
Both a and b cannot be simultaneously zero, therefore (2 + 6 - 1) + l(6 - 2 + 1) = 0
7
atleast one of a and b is non-zero. Þ 7 + 5l = 0 Þ l = -
Let a ¹ 0. Now, from Eq. (i) can be rewritten as 5
b \ Equation of line passes through points C and P is
x + y - 1 + (2x + 3y - 1) = 0 7
a ( x + 3y - 1) - (3x - y + 1) = 0
5
Þ x + y - 1 + k (2x + 3y - 1) = 0 Þ 5x + 15y - 5 - 21x + 7y - 7 = 0
b
where k= …(ii) Þ 16x - 22y + 12 = 0
a Þ 8x - 11y + 6 = 0 … (ii)
From Eq.(ii), it is clear that lines (ii) pass through the point of From the options, point ( - 9, - 6) satisfy the line.
intersection of lines Hence, option (b) is correct.
Practice Exercise
ROUND I Topically Divided Problems
Equation of Straight Line in 7. The line PQ whose equation x - y = 2 cuts the
Standard Form X-axis at P and Q is (4, 2). The line PQ is rotated
about P through 45° in anti-clockwise direction.
1. If the point (3 , - 4) divides the intercept of a line
The equation of the line PQ in the new position is
between the coordinate axes in the ratio 2 : 3. Then, (a) y = - 2 (b) y = 2
its equation is (c) x = 2 (d) x = - 2
(a) 2x + 3 y + 1 = 0 (b) 3x - 4 y + 1 = 0
8. The number of integral values of m so that the
(c) 2x - 3 y = 5 (d) 2x - y = 10
abscissa of point of intersection of lines 3x + 4 y = 9
2. The equation of lines on which the perpendicular and y = mx + 1 is also integer, is (JEE Main 2021)
from the origin makes 30° angle with X-axis and (a) 1 (b) 2 (c) 3 (d) 0
50
which form a triangle of area with axes, are 9. The straight line whose sum of the intercepts on
3
the axes is equal to half of the product of the
(a) x ± 3 y - 10 = 0 (b) 3x + y ± 10 = 0
intercepts, passes through the point whose
(c) x + 3 y ± 10 = 0 (d) None of these
coordinates are
3. If PS is the median of the triangle with vertices (a) (1, 1) (b) (2, 2) (c) (3, 3) (d) (4, 4)
P ( 2, 2), Q (6, - 1) and R (7, 3), then equation of the
10. If the x-intercept of the line y = mx + 2 is greater
line passing through (1, – 1) and parallel to PS is
(JEE Main 2014)
1
than , then the gradient of the line lies in the
(a) 4x - 7 y - 11 = 0 (b) 2x + 9 y + 7 = 0 2
(c) 4x + 7 y + 3 = 0 (d) 2x - 9 y - 11 = 0 interval
æ -1 ö
4. The equations of the lines which pass through the (a) (-1 , 0) (b) ç , 0÷ (c) (-¥ , - 4) (d) (-4 , 0)
è 4 ø
point (3, –2) and are inclined at 60° to the line
3x + y = 1 is 11. In a D ABC, side AB has the equation 2 x + 3 y = 29
(a) y + 2 = 0, 3x - y - 2 - 3 3 = 0 and the side AC has the equation, x + 2 y = 6. If the
(b) x - 2 = 0, 3x - y + 2 + 3 3 = 0 mid-point of BC is (5, 6), then the equation of BC is
(c) 3x - y - 2 - 3 3 = 0 (a) x - y = - 1 (b) 5x - 2 y = 13
(d) None of the above (c) 21x + 31 y = 291 (d) 3x - 4 y = - 9
5. Equation to the straight line cutting off an 12. For which values of a and b, intercepts on axes by
intercept 2 from the negative direction of the axis line ax + by + 8 = 0 are equal and opposite in sign of
of Y and inclined at 30° to the positive direction of intercepts on axis by line 2 x - 3 y + 6 = 0
X-axis, is 8 -8
(a) a = ,b = -4 (b) a = ,b = -4
(a) y + x - 3 = 0 (b) y - x + 2 = 0 3 3
(c) y - 3x - 2 = 0 (d) 3 y - x + 2 3 = 0 8 -8
(c) a = , b = 4 (d) a = ,b =4
3 3
6. If a line passes through the point P (1 , 2) makes an
angle of 45° with the X-axis and meets the line 13. The points (2, 5) and (5, 1) are two opposite vertices
x + 2 y - 7 = 0 at Q, then PQ equals of a rectangle. If other two vertices are points on
the straight line y = 2 x + k, then the value of k is
2 2 3 2
(a) (b) (c) 3 (d) 2 (a) 4 (b) 3 (c) - 4 (d) - 3
3 3
Straight Lines 295

14. The line parallel to the X-axis and passing through 21. The equation of line with slope 2 and the length of
the intersection of the lines ax + 2 by + 3 b = 0 and the perpendicular from the origin equal to 5, are
bx - 2 ay - 3 a = 0 is, [where, ( a, b) ¹ (0 , 0)] (a) y = 2x ± 5 (b) x = 2 y ± 5
3 (c) y = x ± 5 (d) None of these
(a) below the X-axis at a distance of from it
2 22. Let A (1 , 1) and B (3 , 2) be two points. If C is a point
2
(b) below the X-axis at a distance of from it on X-axis such that AC + BC is minimum, then the
3
coordinates are
3
(c) above the X-axis at a distance of from it æ5 ö æ1 ö
2 (a) ç , 0÷ (b) ç , 0÷
è3 ø è3 ø
2
(d) above the X-axis at a distance of from it (c) (3, 0) (d) None of these
3
15. Find the equation to the straight line which passes 23. Find the equation of the line passing through the
point of intersection of 2 x + y = 5 and x + 3 y + 8 = 0
through the point ( - 4, 3) and is such that the
and parallel to the line 3x + 4 y = 7.
portion of it between the axes is divided by the
(a) 4x - 3 y + 3 = 0 (b) 3x + 4 y + 3 = 0
point in the ratio 5 : 3, is
(c) 4x + 3 y - 3 = 0 (d) 3x - 4 y - 3 = 0
(a) 9x - 20 y + 96 = 0 (b) 2x - y + 11 = 0
(c) 2x + y + 5 = 0 (d) 3x - 2 y + 7 = 0 24. Find the equations of the lines through the point of
intersection of the lines x - y + 1 = 0 and
16. Line AB passes through point (2, 3) and intersects
2 x - 3 y + 5 = 0 and whose distance from the point
the positive X and Y -axes at A ( a , 0) and B (0 , b), 7
respectively. If the area of D AOB is 11, the (3, 2) is .
5
numerical value of 4 b2 + 9 a 2 , is (a) 3x - 4 y + 6 = 0 and 4x - 3 y + 1 = 0
(a) 220 (b) 240 (c) 248 (d) 284 (b) 3x + 4 y + 6 = 0 and 4x + 3 y + 1 = 0
17. Let PS be the median of the triangle with vertices (c) 3x - 4 - 6 = 0 and 4x + 3 y + 1 = 0
P ( 2 , 2), Q (6, - 1) and R (7, 3). The equation of the (d) None of the above
line passing through (1, - 1) and parallel to PS is 25. A point on the straight line, 3x + 5 y = 15 which is
(a) 2x - 9 y - 7 = 0 (b) 2x - 9 y - 11 = 0 equidistant from the coordinate axes will lie only in
(c) 2x + 9 y - 11 = 0 (d) 2x + 9 y + 7 = 0 (JEE Main 2019)
(a) IV quadrant (b) I quadrant
18. Find the equation of one of the sides of an isosceles (c) I and II quadrants (d) I, II and IV quadrants
right angled triangle whose hypotenuse is given by
3x + 4 y = 4 and the opposite vertex of the
26. Consider the set of all lines px + qy + r = 0 such
hypotenuse is (2, 2). that 3 p + 2 q + 4 r = 0. Which one of the following
(a) x - 7 y + 12 = 0, 7x + y - 16 = 0 statements is true? (JEE Main 2019)

(b) x - 7 y + 6 = 0, 7x + y - 16 = 0 (a) Each line passes through the origin.


(c) x - 7 y + 12 = 0, 7x + y + 16 = 0 æ3 1ö
(b) The lines are concurrent at the point ç , ÷
(d) None of the above è4 2ø
(c) The lines are all parallel
19. The equations of the lines passing through the
(d) The lines are not concurrent
3
point (1, 0) and at a distance from the origin, 27. Two sides of a rhombus are along the lines,
2
x - y + 1 = 0 and 7 x - y - 5 = 0. If its diagonals
are
intersect at (- 1, - 2), then which one of the
(a) 3 x + y - 3 = 0, 3 x - y - 3 = 0
following is a vertex of this rhombus? (JEE Main 2016)
(b) 3x + y + 3 = 0, 3x - y + 3 = 0
æ1 8ö æ 10 7ö
(c) x + 3 y - 3 = 0, x - 3 y - 3 = 0 (a) (- 3, - 9) (b) (- 3, - 8) (c) ç , - ÷ (d) ç - ,- ÷
è3 3ø è 3 3ø
(d) None of the above
p
28. The line passing through æç -1, ö÷ and perpendicular
20. The equation of straight line cutting off an è 2ø
intercept - 2 from Y -axis and being equally inclined 4
to 3 sin q + 2 cos q = , is
to the axes are r
(a) y = x + 2 , y = x - 2 (b) y = - x - 2 , y = x - 2 (a) 2 = 3r cos q - 2r sin q (b) 5 = - 2 3r sin q + 4r cos q
(c) y = x - 2 , y = x - 2 (d) None of these (c) 2 = 3r cos q + 2r sin q (d) 5 = 2 3r sin q - 4r cos q
296 JEE Main Mathematics

29. A and B are any two points on the positive X and 38. The inclination of the straight line passing through
Y -axes, respectively satisfying 2 (OA) + 3 (OB) = 10. the point ( -3, 6) and the mid-point of the line
If P is the middle point of AB, then the locus of P is joining the points ( 4, - 5) and ( - 2 , 9) is
(a) 2x + 3 y = 5 (b) 2x + 3 y = 10 p p p 3p
(a) (b) (c) (d)
(c) 3x + 2 y = 10 (d) 3x + 2 y = 10 4 6 3 4

30. A line AB makes zero intercepts on X-axis and 39. In what direction should a line be drawn through
Y -axis and it is perpendicular to another line CD the point (1, 2) so that its point of intersection with
6
which is 3x + 4 y + 6 = 0. The equation of line AB is the line x + y = 4 is at a distance of from the
3
(a) y = 4 (b) 4x - 3 y + 8 = 0
given point?
(c) 4x - 3 y = 0 (d) 4x - 3 y + 6 = 0
(a) 75° (b) 60° (c) 90° (d) 45°
31. Equation of a line passing through the line of
intersection of lines 2 x - 3 y + 4 = 0, 3x + 4 y - 5 = 0 Angle between Two Lines
and perpendicular to 6 x - 7 y + 3 = 0, is 40. Find the angle between the lines y = ( 2 - 3)( x + 5)
(a) 119x + 102 y + 125 = 0 (b) 119x + 102 y = 125 and y = ( 2 + 3)( x - 7).
(c) 119x - 102 y = 125 (d) None of these p 2p p 2p
(a) - or (b) or
32. If the lines x = a + m, y = - 2 and y = mx are 3 3 3 3
p 2p
concurrent, then least value of|a|is (c) or - (d) None of these
3 3
(a) 0 (b) 2
(c) 2 2 (d) None of these
41. If the two lines x + ( a - 1) y = 1 and 2 x + a 2 y = 1,
( a Î R - {0, 1}) are perpendicular, then the distance
33. The equation of straight line through the of their point of intersection from the origin is
intersection of the lines x - 2 y = 1 and x + 3 y = 2 (JEE Main 2019)
and parallel to 3x + 4 y = 0, is 2 2 2 2
(a) (b) (c) (d)
(a) 3x + 4 y + 5 = 0 (b) 3x + 4 y - 10 = 0 5 5 5 5
(c) 3x + 4 y - 5 = 0 (d) 3x + 4 y + 6 = 0 42. If the straight line, 2 x - 3 y + 17 = 0 is perpendicular
34. The lines ax + by + c = 0, bx + cy + a = 0 and to the line passing through the points (7, 17) and
cx + ay + b = 0 ( a ¹ b ¹ c) are concurrent, if (15,b), then b equals (JEE Main 2019)
35 35
(a) a3 + b3 + c3 + 3abc = 0 (a) (b) - 5 (c) - (d) 5
3 3
(b) a 2 + b2 + c2 - 3abc = 0
(c) a + b + c = 0 43. One side of a rectangle lies along the line
(d) None of the above 4 x + 7 y + 5 = 0. Two of its vertices are ( - 3, 1) and
x y (1, 1). Then, the equations of other sides are
35. The line - = 1 cuts the X-axis at P. The
a b (a) 7x - 4 y + 25 = 0, 7x + 4 y - 3 = 0, 7x + 4 y + 25 = 0
equation of the line through P perpendicular to the (b) 7x + 4 y + 25 = 0, 4x + 7 y = 11, 7x - 4 y + 25 = 0
given line is (c) 7x - 4 y + 25 = 0, 7x - 4 y - 3 = 0, 4x + 7 y = 11
(a) x + y = ab (b) x + y = a + b (d) None of the above
(c) ax + by = a 2 (d) bx + ay = b 2 44. The equation of the sides of a triangle are x - 3 y = 0,
36. Let A ( -1, 1), B (3, 4) and C ( 2, 0) be given three 4 x + 3 y = 5 and 3x + y = 0. The line 3x - 4 y = 0
points. A line y = mx, m > 0, intersects lines AC and passes through the
BC at point P and Q respectively. Let A1 and A2 be (a) incentre
the areas of DABC and DPQC respectively, such (b) centroid
that A1 = 3 A2 , then the value of m is equal to (c) orthocentre
(JEE Main 2021) (d) circumcentre
4 45. Two lines are drawn through (3, 4) each of which
(a) (b) 1 (c) 2 (d) 3
15 makes angle of 45° with line x - y = 2 , then area of
37. The value of l for which the lines 3x + 4 y = 5, the triangle formed by these lines is
5 x + 4 y = 4 and lx + 4 y = 6 meet at a point is (a) 9 sq units (b) 9/2 sq units
(a) 2 (b) 1 (c) 4 (d) 3 (c) 2 sq units (d) 2/9 sq unit
Straight Lines 297

46. The triangle formed by the lines x + y = 0, Bisector of Lines


3x + y = 4, x + 3 y = 4 is
(a) isosceles (b) equilateral
55. P is a point on either of the two lines y - 3|x|= 2
(c) right angled (d) None of these
at a distance 5 units from their point of
intersection. The coordinates of the foot of the
Distance between Two Lines and perpendicular from P on the bisector of the angle
Distance of a Point from a Line between them are
47. Distance between the lines 5 x + 3 y - 7 = 0 and æ 4+5 3ö æ 4 -5 3 ö
(a) çç0, ÷ or ç0,
÷ ç
÷ depending on which the
15 x + 9 y + 14 = 0 is è 2 ø è 2 ÷ø
35 1 35 35 point P is taken
(a) (b) (c) (d)
34 3 34 3 34 2 34 æ 4+5 3ö
(b) çç0, ÷
48. A point equidistant from the lines 4 x + 3 y + 10 = 0, è 2 ÷ø
æ 4 -5 3 ö
5 x - 12 y + 26 = 0 and 7 x + 24 y - 50 = 0 is (c) çç0, ÷
(a) (1, –1) (b) (1, 1) (c) (0, 0) (d) (0, 1) è 2 ÷ø
æ5 5 3 ö
49. The length of perpendicular from the point (d) çç , ÷
÷
( a cos a, a sin a) upon the straight line è2 2 ø
y = x tan a + c, c > 0, is
56. The equations of perpendicular bisectors of sides
(a) c (b) csin 2 a (c) c cos a (d) csec2 a
AB and AC of a D ABC are x - y + 5 = 0 and
50. The distance of the point (3, 5) from the line x + 2 y = 0 respectively. If the coordinates of
2 x + 3 y - 14 = 0 measured parallel to line x - 2 y = 1, vertex A are (1, - 2), then equation of BC is
is
7 7 (a) 23x + 14 y - 40 = 0 (b) 14x - 23 y + 40 = 0
(a) (b) (c) 5 (d) 13
5 13 (c) 23x - 14 y + 40 = 0 (d) 14x + 23 y - 40 = 0

57. The equation of the bisector of the acute angle


Foot of Perpendicular
between the lines 3x - 4 y + 7 = 0 and
51. The coordinates of the foot of perpendicular from 12 x + 5 y - 2 = 0 is
the point (2, 3) on the line y = 3x + 4 is given by
(a) 99x - 27 y - 81 = 0 (b) 11x - 3 y + 9 = 0
æ 37 -1 ö æ -1 37 ö
(a) ç , ÷ (b) ç , ÷ (c) 21x + 77 y - 101 = 0 (d) 21x + 77 y + 101 = 0
è 10 10 ø è 10 10 ø
æ 10 ö æ2 1ö 58. The equation of bisectors of the angles between the
(c) ç , - 10÷ (d) ç , - ÷
è 37 ø è 3 3ø lines|x|=|y|are
1 1
52. The coordinates of the foot of the perpendicular (a) y = ± x and x = 0 (b) x = and y =
2 2
drawn from the point (3, 4) on the line 2 x + y - 7 = 0 are
(c) y = 0 and x = 0 (d) None of these
æ 9 17 ö
(a) ç , ÷ (b) (1, 5)
è5 5 ø Family of Lines
(c) (- 5, 1) (d) (1, - 5)
59. Equation of straight line belonging to families of
53. If the foot of the perpendicular from the origin to a straight lines ( x + 2 y) + l(3x + 2 y + 1) = 0
straight line is at the point (3, - 4). Then, the
and ( x - 2 y) + m( x - y + 1) = 0 is
equation of the line is
(a) 6x + 5 y = 2 (b) 5x - 6 y + 4 = 0
(a) 3x - 4 y = 25 (b) 3x - 4 y + 25 = 0
(c) 5x + 6 y = 4 (d) None of these
(c) 4x + 3 y - 25 = 0 (d) 4x - 3 y + 25 = 0
60. A variable straight line drawn through the point of
54. Let the tangent to the circle x 2 + y 2 = 25 at the x y x y
point R (3, 4) meet X-axis and Y -axis at point P and intersection of the lines+ = 1 and + = 1
a b b a
Q, respectively. If r is the radius of the circle meets the coordinates axes at A and B, the locus of
passing through the origin O and having centre at the mid-point of AB is
the incentre of the DOPQ, then r 2 is equal to (a) 2xy (a + b) = ab (x + y)
(JEE Main 2021)
(b) 2xy (a - b) = ab (x - y)
529 125 625 585
(a) (b) (c) (d) (c) 2xy (a + b) = ab (x - y)
64 72 72 66
(d) None of the above
298 JEE Main Mathematics

61. Locus of the image of the point (2, 3) in the line 63. Orthocentre of triangle with vertices (0, 0), (3, 4)
( x - 2 y + 3) + l( 2 x - 3 y + 4) = 0 is ( l Î R) and (4, 0) is
2 2
(a) x + y - 3x - 4 y - 4 = 0 (a) (3, 5/4) (b) (3, 12) (c) (3, 3/4) (d) (3, 9)
2 2
(b) 2x + 3 y + 2x + 4 y - 7 = 0 64. Orthocentre of the triangle formed by the lines
(c) x 2 + y 2 - 2x - 4 y + 4 = 0 x + y = 1 and xy = 0 is
(d) None of the above (a) (0, 0) (b) (0, 1)
(c) (1, 0) (d) (- 1, 1)
Applications of Straight Line 65. If t1 and t2 are roots of the equation t 2 + lt + 1 = 0,
in Geometry where l is an arbitrary constant. Then, the
62. Two vertices of a triangle are (5, - 1) and ( - 2 , 3). If line joining the points ( at12 , 2 at1) and ( at22 , 2 at2 )
the orthocentre of the triangle is the origin, then always passes through a fixed point whose
coordinates of third vertex are coordinates are
(a) (4, 7) (b) (- 4, - 7) (a) (a , 0) (b) (- a , 0)
(c) (- 4, 7) (d) None of these (c) (0, a ) (d) (0, - a )

ROUND II Mixed Bag


Only One Correct Option 5. If 5 a + 4 b + 20 c = t, then the value of t for which the
1. A system of lines is given as y = mi x + ci , where mi line ax + by + c - 1 = 0 always passes through a
can take any value out of 0, 1, -1 and when mi is fixed point is
positive, then ci can be 1 or -1, when mi equal 0, ci (a) 0 (b) 20
(c) 30 (d) None of these
can be 0 or 1 and when mi equals to -1, ci can take
0 or 2. Then, the area enclosed by all these straight 6. Consider the family of lines
line is ( x + y - 1) + l ( 2 x + 3 y - 5) = 0
3 3 and (3x + 2 y - 4) + m ( x + 2 y - 6) = 0,
(a) ( 2 - 1) sq units (b) sq units
2 2 equation of a straight line that belongs to both the
3 families is
(c) sq units (d) None of these
2 (a) x -2y -8 = 0
2. The number of points on the line 3x + 4 y = 5, which (b) x -2y + 8 = 0
are at a distance of sec 2 q + 2 cosec 2 q, q Î R, from (c) 2x + y - 8 = 0
the point (1, 3) is (d) 2x - y - 8 = 0
(a) 1 (b) 2 (c) 3 (d) infinite 7. If the distance of any point ( x, y) from the origin is
3. The base BC of D ABC is bisected at ( p, q) and defined as d ( x, y) = max{|x|,|y|}, d ( x, y) = a,
equation of sides AB and AC are px + qy = 1 and non-zero constant, then the locus is a
qx + py = 1 respectively. Then, the equation of the (a) circle (b) straight line
median through A is (c) square (d) triangle
(a) (2 pq - 1)( px + qy - 1) = ( p2 + q 2 - 1)(qx + py - 1) 8. The straight line ax + by + c = 0 where abc ¹ 0 will
(b) ( px + qy - 1)(qx + py - 1) = 0 pass through the first quadrant if
(c) ( px + qy - 1)(qx - py - 1) = 0
(a) ac > 0, bc > 0
(d) None of the above
(b) c > 0 and bc < 0
4. A beam of light is sent along the line x - y = 1. (c) bc > 0 and/or ac > 0
Which after refracting from the X-axis enters the (d) ac < 0 and/or bc < 0
opposite side by turning through 30° towards the
9. The lines y = m1x, y = m2 x and y = m3x make equal
normal at the point of incidence on the X-axis. intercepts on the line x + y = 1, then
Then, the equation of the refracted ray is (a) 2(1 + m1 )(1 + m3 ) = (1 + m2)(2 + m1 + m3 )
(a) (2 - 3 ) x - y = 2 + 3 (b) (1 + m1 )(1 + m3 ) = (1 + m2)(1 + m1 + m3 )
(b) (2 + 3 ) x - y = 2 + 3 (c) (1 + m1 )(1 + m2) = (1 + m3 )(2 + m1 + m3 )
(c) (2 - 3 ) x + y = 2 + 3 (d) None of the above
(d) None of the above
Straight Lines 299

10. One diagonal of a square is along the line 18. Locus of the image of the point (2, 3) in the line
8 x - 15 y = 0 and one of its vertex is (1, 2). Then, the ( 2 x - 3 y + 4) + k( x - 2 y + 3) = 0, k Î R, is a
equations of the sides of the square passing (JEE Main 2015)

through this vertex are (a) straight line parallel to X-axis


(a) 23x + 7 y = 9, 7x + 23 y = 53 (b) straight line parallel to Y -axis
(b) 23x - 7 y + 9 = 0, 7x + 23 y + 53 = 0 (c) circle of radius 2
(c) 23x - 7 y - 9 = 0, 7x + 23 y - 53 = 0 (d) circle of radius 3
(d) None of the above 19. The lines x + y =|a |and ax - y = 1 intersect each
11. A light ray coming along the line 3x + 4 y = 5 gets other in the first quadrant. Then, the set of all
reflected from the line ax + by = 1 and goes along possible values of a in the interval
the line 5 x - 12 y = 10. Then, (a) (–1, 1] (b) (0, ¥ ) (c) [1, ¥ ) (d) (-1, ¥ )
(a) a = 64/115, b = 112/15 x y
(b) a = 14/15, b = - 8/115
20. The line L given by + = 1 passes through the
5 b
(c) a = 64/115, b = - 8/115 point (13, 32). The line K is parallel to L and has
(d) a = 64/15, b = 14/15 x y
the equation + = 1. Then, the distance between
12. The straight lines 4 ax + 3by + c = 0, where c 3
a + b + c = 0, are concurrent at the point L and K is
23 17 23
(a) (4, 3) (b) (1/4, 1/3) (a) (b) 17 (c) (d)
(c) (1/2, 1/3) (d) None of these 15 15 17

13. Two sides of a triangle are the lines 21. The bisector of acute angle between the lines
( a + b) x + ( a - b) y - 2 ab = 0 and x + y - 3 = 0 and 7 x - y + 5 = 0 is
( a - b) x + ( a + b) y - 2 ab = 0. If the triangle is (a) 6x + 2 y - 5 = 0 (b) 3x - 10 y + 6 = 0
isosceles and the third side passes through point (c) 4x + 5 y + 3 = 0 (d) 7x + y - 4 = 0
( b - a, a - b), then the equation of third side can be 22. The perpendicular bisector of the line segment
(a) x + y = 4 (b) x = y + 2 (b - a ) joining P (1, 4) and Q ( k, 3) has y-intercept –4.
(c) x - b + a = 0 (d) y - a + b = 0 Then, a possible value of k is
14. If bx + cy = a, where a , b, c are of the same sign, be a (a) 4 (b) 1 (c) 2 (d) –2

line such that the area enclosed by the line and 23. Let P = ( -1, 0), Q = (0, 0) and R = (3, 3 3) be three
1 points. The equation of the bisector of the ÐPQR is
the axes of reference is sq unit, then
8 3
(a) x+ y=0 (b) x + 3 y = 0
(a) b, a , c are in GP (b) b, 2a , c are in GP 2
a 3
(c) b, , c are in AP (d) b, - 3 a , c are in GP (c) 3x + y = 0 (d) x + y=0
2 2

15. Consider the straight lines x + 2 y + 4 = 0 and 24. A straight line through the point A(3, 4) is such
4 x + 2 y - 1 = 0. The line 6 x + 6 y + 7 = 0 is that is intercept between the axis is bisected at A.
(a) bisector of the angle not including origin Its equation is
(b) bisector of acute angle (a) 4x + 3 y = 24 (b) 3x + 4 y = 25
(c) bisector of obtuse angle (c) x + y = 7 (d) 3x - 4 y + 7 = 0
(d) None of the above 25. If ( a, a 2 ) falls inside the angle made by the lines,
16. Two sides of a parallelogram are along the lines, x
y= , x > 0 and y = 3x, x > 0, then a Î
x + y = 3 and x - y + 3 = 0. If its diagonals intersect at 2
(2, 4), then one of its vertex is (JEE Main 2019) æ1 ö æ 1ö æ 1ö
(a) ç , 3÷ (b) ç -3, - ÷ (c) ç0, ÷ (d) (3, ¥ )
(a) (3, 6) (b) (2, 6) (c) (2, 1) (d) (3, 5) è2 ø è 2 ø è 2ø

17. Two vertices of a triangle are (0, 2) and 26. If non-zero numbers a, b and c are in HP, then the
(4, 3). If its orthocentre is at the origin, then its x y 1
straight line + + = 0 always passes through a
third vertex lies in which quadrant? (JEE Main 2019) a b c
(a) IV quadrant (b) III quadrant fixed point. That point is
(c) II quadrant (d) I quadrant æ 1ö
(a) ç1, - ÷ (b) (1, –2) (c) (–1, –2) (d) (–1, 2)
è 2ø
300 JEE Main Mathematics

27. The line parallel to the X-axis and passing through Numerical Value Type Questions
the intersection of the lines ax + 2 by + 3b = 0 and 1 2
bx - 2 ay - 3a = 0, where ( a , b) ¹ (0, 0), is 30. If the line, 2 x - y + 3 = 0 is at a distance and
5 5
(a) above the X-axis at a distance of (2/3) from it
from the lines 4 x - 2 y + a = 0 and 6 x - 3 y + b = 0,
(b) above the X-axis at a distance of (3/2) from it
respectively, then the sum of all posible values of a
(c) below the X-axis at a distance of (2/3) from it
(d) below the X-axis at a distance of (3/2) from it and b is ............ . (JEE Main 2020)

28. Let A( 2 , - 3) and B ( - 2 , 1) be vertices of a DABC. 31. The orthocentre of the triangle formed by the lines
2 x + 3 y - 1 = 0, x + 2 y - 1 = 0, ax + by - 1 = 0 is at
If the centroid of this triangle moves on the
origin, then ( a + b) is ........... .
line 2 x + 3 y = 1, then the locus of the vertex C is
( 2 n - 1) p
the line 32. Let tan a, tan b and tan g; a, b, g ¹ ,
(a) 2x + 3 y = 9 2
(b) 2x - 3 y = 7 n Î N be the slopes of three line segments
(c) 3x + 2 y = 5 OA, OB and OC, respectively, where O is origin.
(d) 3x - 2 y = 3 If circumcentre of DABC coincides with origin
and its orthocentre lies on Y -axis, then the value
29. The equation of the straight line passing through 2
the point (4, 3) and making intercepts on the æ cos 3a + cos 3b + cos 3g ö
of çç ÷÷ is equal to
coordinate axes whose sum is –1, is è cos a cos b cos g ø
(JEE Main 2021)
x y x y
(a) + = - 1, + = -1
2 3 -2 1 33. For which value of 3 ( a + b), intercepts on axes by
x y x y line ax + by + 8 = 0 are equal and opposite in sign of
(b) - = - 1, + = -1
2 3 -2 1 intercepts on axis by line 2 x - 3 y + 6 = 0
x y x y
(c) + = 1, + =1
2 3 -2 1 34. The image of the point A (1, 2) by the line mirror y = x
x y x y is the point B and the image of B by the mirror y = 0
(d) - = 1, + =1
2 3 -2 1 is the point ( a, b), then ( a - b) is ........... .

Answers
Round I
1. (d) 2. (b) 3. (b) 4. (a) 5. (d) 6. (a) 7. (c) 8. (b) 9. (b) 10. (d)
11. (c) 12. (d) 13. (c) 14. (a) 15. (a) 16. (a) 17. (d) 18. (a) 19. (a) 20. (b)
21. (a) 22. (a) 23. (b) 24. (a) 25. (c) 26. (b) 27. (c) 28. (a) 29. (a) 30. (c)
31. (b) 32. (c) 33. (c) 34. (c) 35. (c) 36. (a) 37. (b) 38. (d) 39. (a) 40. (b)
41. (d) 42. (d) 43. (c) 44. (c) 45. (b) 46. (a) 47. (c) 48. (c) 49. (c) 50. (c)
51. (b) 52. (a) 53. (a) 54. (a) 55. (b) 56. (d) 57. (c) 58. (c) 59. (b) 60. (a)
61. (c) 62. (b) 63. (c) 64. (a) 65. (b)

Round II
1. (c) 2. (b) 3. (a) 4. (d) 5. (b) 6. (b) 7. (b) 8. (d) 9. (d) 10. (c)
11. (c) 12. (b) 13. (b) 14. (b) 15. (b) 16. (a) 17. (c) 18. (c) 19. (c) 20. (d)
21. (a) 22. (a) 23. (c) 24. (a) 25. (a) 26. (b) 27. (d) 28. (a) 29. (c) 30. (30)
31. (0) 32. (144) 33. (4) 34. (3)
Solutions
Round I 1 50
Þ ´ OA ´ OB =
1. Let the line intersect X-axis at (x1 , 0) andY -axis at (0, y1 ). 2 3
1 2p 50
Y Þ ´ ´2p = [Q ON = p]
2 3 3
Þ p2 = 25
Þ p=±5 ……(ii)
X¢ X From Eq. (i) and (ii), we have
O 2 (x1, 0)
3x + y = ± 10 Þ 3x + y ± 10 = 0
(3, –4) 7 + 6 3 - 1 ö æ 13 ö
3
3. Coordinate of S = æç , ÷=ç , 1÷
(0, y1) è 2 2 ø è2 ø
Y¢ P (2, 2)

3x1
\ = 3 Þ x1 = 5
5
2 y1
and = - 4 Þ y1 = - 10 Q R
5 S
(6, –1) (7, 3)
\Equation of line
x y -2
+ =1 Slope of the line PS is .
5 -10 9
x y Required equation passes through (1, - 1) is
or - =1
5 10 -2
y+1 = (x - 1)
or 2x - y = 10 9
Þ 2x + 9 y + 7 = 0
2. Let the equation of line in perpendicular form is
æ m ± tan a ö
Y 4. Equation of a line is y - y1 = çç ÷÷ (x - x1 )
è 1 + m tan a ø
B (0, b)
æ - 3 ± tan 60° ö
N Þ y + 2 = çç ÷ (x - 3)
÷
p è 1 + (- 3 ) tan 60° ø
30° A X - 3± 3
O (a, 0) Þ y+2= (x - 3)
1 + (-3)
- 3+ 3
Þ y+2= (x - 3) and
1 - (-3)
x cos a + y sin a = p - 3- 3
y+2= (x - 3)
Þ x cos 30° + y sin 30° = p 1 + (-3)
3x y Þ y+ 2 =0
+ =p
2 2
and 3x - y - 2 - 3 3 = 0
Þ 3x + y = 2 p …(i)
Again, let OA = a and OB = b 5. Let the equation of line is
Now, In D ONA y = mx + c
p 1
cos 30° = Q m = tan 30° =
a 3
2p and c = -2
Þ a=
3 [Q it is intercepted in negative axis of Y
p with an angle of 30°]
In D ONB, cos 60° =
b \ The equation of required line is
Þ b = 2p x
y= -2
50 3
Area of D ONA = (given)
3 Þ 3y - x + 2 3 = 0
302 JEE Main Mathematics

6. Slope of line, m = tan 45° = 1 10. Given, equation of line,


\ Equation of line having slope ‘1’ and passes through y = mx + 2
P (1, 2), is x y
Þ + =1
2 2
y - 2 = 1 × (x - 1) Þ y-2 = x-1 -
Þ x- y+ 1 =0 …(i) m
It is given that x-intercept of the above line is greater
Line (i) meets with line
1
x + 2y - 7 = 0 …(ii) than .
2
at point Q. 2 1
æ5 8ö \ - >
\ Point of intersection of lines (i) and (ii) is Q ç , ÷ . m 2
è3 3ø 2 1 æ 2 1ö
2 2
Þ - - > 0Þ - ç + ÷ > 0
æ5 ö æ8 ö 4 4 8 2 2 m 2 èm 2ø
\PQ = ç - 1÷ + ç - 2÷ = + = =
è3 ø è3 ø 9 9 9 3 æ 4 + m ö 4 +m
Þ -ç ÷ >0 Þ <0
è 2m ø 2m
7. Given equation of line,
Þ Either 4 + m > 0 and 2m < 0
x- y=2 …(i)
or 4 + m < 0 and 2m > 0
Since, the line (i) intersects X-axis at P.
Þ m > - 4 and m < 0
\Coordinates of P are (2, 0).
\ m Î (-4, 0)
Now, slope Eq. (i), m =1
x1 + x2 y + y2
Let q be the angle subtended by line (i) with positive 11. Here, = 5, 1 =6 …(i)
direction of X-axis. 2 2
Also, x1 + 2 y1 = 6 …(ii)
\ m = tan q
2x2 + 3 y2 = 29 …(iii)
Þ tan q = 1 Þ q = 45°
From Eq. (ii) and (iii), we get
Since, the line (i) rotated through 45° in anti-clockwise
direction. 2 (x1 + x2) + 4 y1 + 3 y2 = 41
\ Slope of the line in new position, Þ 4 y1 + 3 y2 = 21 …(iv)
From Eqs. (i) and (iv), we get
m1 = tan 90° = ¥
A
Equation of line PQ in new position,
y-0
2x+

= ¥ Þ x=2
6

x-2
y=

3y=
2
x+

29

8. 3x + 4 y = 9
y = 3x + 1 B C
D
Þ 3 + 4mx + 4 = 9 (x1, y1) (5, 6) (x2, y2)
Þ (3 + 4m)x = 5 y1 = - 15 …(v)
Þ x will be an integer when From Eq. (ii) and (v), we get
3x + 4m = 5, - 5, 1, - 1 x1 = 36 …(iv)
1 1
Þ m = , - 2, - , - 1 \ B (36, - 15) and D (5, 6)
2 2
Hence, equation of BC is 21x + 31 y = 291
So, number of integral values of m is 2.
12. ax + by + 8 = 0
9. Let a and b the intercepts made by the straight line on
Þ ax + by = -8
the axes.
x y
ab Þ + =1 (intercept form)
Given that, a+ b= 8 8
2 - -
2a + 2b a b
Þ =1 Also, 2x - 3 y = - 6
ab
x y
2 2 Þ - + =1
Þ + =1 3 2
a b
According to given condition, we have
x y
On comparing with + = 1, we get x = 2 , y = 2 8 8
a b - = - (- 3) and - = - 2
a b
\ Required point is (2 , 2). 8
Þ a = - and b = 4
So, the straight line passes through the point (2 , 2). 3
Straight Lines 303

13. Now, mid-point of A (2, 5) and B (5, 1) is 9


y-3 = (x + 4)
20
æ2 + 5 5 + 1ö æ7 ö
Cç , ÷ or C ç , 3÷ . 20 ( y - 3) = 9 (x + 4)
è 2 2 ø è2 ø
y = 2x + k Þ 9x - 20 y + 96 = 0
A (2, 5)
16. Equation of line AB,
C
x y
+ =1 …(i)
a b

B (5, 1) B (0, b)

We know that the diagonals of a rectangle bisect (2, 3)


each other.
æ7 ö A (a, 0)
Since, the mid-point C ç , 3÷ lies on a given line. O
è2 ø
7
\ 3 =2 ´ + k Þ 3 = 7 + k Þ k = -4
2
14. Equation of line passing through the intersection of
lines ax + 2by + 3b = 0 and bx - 2ay - 3a = 0 is Since, point (2, 3) lies on line (i).
2 3
(ax + 2by + 3b) + l (bx - 2ay - 3a ) = 0 …(i) \ + =1
a b
Now, this line is parallel to X-axis, so coefficient of x
should be zero. 3a + 2b
Þ =1
a ab
i.e. a + lb = 0 Þ l = -
b Þ 3a + 2b = ab
On putting this value in Eq. (i), we get Þ (3a + 2b)2 = a 2b2
b (ax + 2by + 3b) - a (bx - 2ay - 3a ) = 0 Þ 9a 2 + 4b2 = a 2b2 - 12ab …(ii)
Þ 2b2y + 3b2 + 2a 2y + 3a 2 = 0 Now, area of D AOB = 11
Þ 2 (b2 + a 2) y + 3 (b2 + a 2) = 0 1
3 Þ ab = 11
y=- 2
2
Þ ab = 22 …(iii)
The negative sign shows that the line is below X-axis
From Eq. (ii) and (iii), we get
3
and at a distance from it. 4b2 + 9a 2 = (22)2 - 12 ´ 22
2
-32 = 22 (22 - 12)
15. Intercept made by the line on X-axis is =
3 = 22 ´ 10 = 220
24
and on Y -axis = 17. Since, S is mid-point of QR. P (2, 2)
5
\ Coordinate of S are
Y
24 æ 6 + 7 -1 + 3 ö æ 13 ö
(0, –– ) ç , ÷=ç , 1÷
(–4, 3) 5 è 2 2 ø è2 ø
5:3 2 -1 2
X¢ X \ Slope of PS = =-
O 13 9
–32
( –—, 0)
2- Q S R
3 2 (6, –1) (7, 3)
The required equation which is
Y¢ 2
passing through (1, -1 ) and slope - , is
Equation of line, 9
24 2
y + 1 = - (x - 1 )
y - 3 = 5 (x + 4) 9
32
Þ 9 y + 9 = - 2x + 2
3
72 Þ 2x + 9 y + 7 = 0
y-3 = (x + 4)
160
304 JEE Main Mathematics

18. Let ABC be an isosceles right angled triangle at A with 20. Since, the required line is equally inclined with the
AB = AC. coordinate axes, therefore it makes an angle of 45° or
135° with the X-axis.
Þ ÐABC = ÐACB = 45°
Y
Now, we have to determine the angle between AC and A C
BC.
The slope of line BC i. e. ,3x + 4 y = 4 is 135° 45°
X¢ X
3 O
m2 = - B
4 (0, –2)
Let the slope of the line AC is m1 = m. Y¢
Q Angle between two lines is So, its slope is either m = tan 45° or m = tan 135°
æ -3 ö i.e., m = 1 or - 1
m-ç ÷
m - m2 è 4 ø It is given that c= -2
tan q = 1 Þ tan 45° =
1 + m1m2 æ -3 ö Hence, the equation of the lines are
1+mç ÷
è 4 ø y = x - 2 or y = - x - 2
3
m+ 21. Let c be the intercept onY -axis, then the equation of the
Þ 1= 4 Þ ± 1 = 4m + 3 line is
3m 4 - 3m
1- y = 2x + c …(i)
4
Þ -2x + y = c
4m + 3
Taking positive sign, 1 = C 2x y c
4 - 3m Þ- + =
(-2)2 + 12 2
(-2) + 1 2
(-2)2 + 12
Þ 4 - 3m = 4m + 3 45º
2 y c
3x

Þ - x+ = …(ii)
+4

Þ 4 - 3 = 4m + 3m
y=

5 5 5
4

Þ 7m = 1
This is the normal form, therefore RHS denotes the
1 90º 45º length of the perpendicular from the origin. But the
Þ m=
7 A B length of the perpendicular from the origin is 5.
4m + 3 c
Taking negative sign, - 1 = \ = 5 Þ |c|= 5
4 - 3m 5
Þ - 4 + 3m = 4m + 3 On putting c = ± 5 in Eq. (i), we get the equation of line
as y = 2x ± 5
Þ - 4 - 3 = mÞ m = - 7
22. Clearly, AC + BC will be minimum when points B, C
Hence, equation of line AB and line AC are
and D (image of A in X-axis) are collinear. In that case,
1 C is the intersection of BD with X-axis.
y - 2 = (x - 2) and y - 2 = - 7 (x - 2)
7 Y
Þ 7 y - 14 = x - 2 and y - 2 = - 7x + 14 B(3, 2)
A(1, 1)
Þ x - 7 y + 12 = 0 and 7x + y - 16 = 0
19. Let slope of a line be m. X¢ X
O C
Now, the equation of a line passing through (1, 0) is
y - 0 = m(x - 1 ) Þ mx - y - m = 0
D (1, –1)
3
Distance from origin =
2 Y¢
| - m| 3 The equation of BD is
Þ = Þ 4m2 = 3(1 + m2)
1+m 2 2 2+1
y+1= (x - 1)
3 -1
Þ m2 = 3
Þ 3x - 2 y - 5 = 0
Þ m=± 3
æ5 ö
\Equations are It cut X-axis at ç , 0÷ .
è3 ø
3x - y - 3 = 0 and - 3x - y + 3 = 0
æ5 ö
Þ 3x - y - 3 = 0 and 3x + y - 3 = 0 Hence, the coordinates of the required point are ç , 0÷ .
è3 ø
Straight Lines 305

23. Equation of a line passing through the intersection of 26. Given, px + qy + r = 0 is the equation of line such that
given lines is
3 p + 2q + 4r = 0
(2x + y - 5) + l (x + 3 y + 8) = 0
Consider, 3 p + 2q + 4r = 0
Þ x (2 + l ) + y(1 + 3l ) - 5 + 8l = 0 …(i) 3 p 2q
Þ + + r =0
It is parallel to 3x + 4 y = 7 4 4
- (2 + l ) -3 (dividing the equation by 4)
Þ =
1 + 3l 4 æ3ö æ1ö
Þ pç ÷ + q ç ÷ + r = 0
Þ l =1 è4ø è2ø
Put in Eq. (i), æ 3 1 ö
Þ ç , ÷ satisfy px + qy + r = 0
3x + 4 y + 3 = 0 è4 2ø
æ3 1ö
24. Equation of a line passing through the point of So, the lines always passes through the point ç , ÷ .
intersection of lines is è4 2ø

x - y + 1 + l(2x - 3 y + 5) = 0 27. As the given lines x - y + 1 = 0 and 7x - y - 5 = 0 are not


Þ x (1 + 2l ) + y(-1 - 3l ) + 1 + 5l = 0 …(i) parallel, therefore they represent the adjacent sides of
7 the rhombus.
Its distance from point (3, 2) =
5 On solving x - y + 1 = 0 and 7x - y - 5 = 0, we get x = 1
|3(1 + 2l ) + 2(-1 - 3l ) + 1 + 5l | 7 and y = 2. Thus, one of the vertex is A(1, 2).
Þ = D C (x, y)
(1 + 2l )2 + (-1 - 3l )2 5

=0
|2 + 5l | 7

y–5
Þ = (–1, –2)
13l2 + 10l + 2 5

On squaring, we get 7x –
A x – y+1=0 B
25(4 + 25l2 + 20l ) = 49(13l2 + 10l + 2) (1, 2)
1 Let the coordinate of point C be (x, y).
Þ 6l2 - 5l - 1 = 0 Þ l = 1, -
6 x+1 y+2
1 Then, -1 = and - 2 =
On putting l = 1, - in Eq. (i) respectively, we get 2 2
6
Þ x + 1 = - 2 and y = - 4 - 2
3x - 4 y + 6 = 0 and 4x - 3 y + 1 = 0
Þ x = - 3 and y = - 6
25. Given equation of line is 3x + 5 y = 15 …(i) Hence, coordinates of C = (- 3, - 6)
Clearly, a point on the line (i), which is equidistance Note that, vertices B and D will satisfy x - y + 1 = 0
from X and Y -axes will lie on the line either y = x or and 7x - y - 5 = 0, respectively.
y = - x.
Since, option (c) satisfies 7x - y - 5 = 0, therefore
Y æ1 - 8ö
coordinate of vertex D is ç , ÷.
B
è3 3 ø
y=x
4
28. Any line which is perpendicular to 3 sin q + 2 cos q =
(0, 3) r
y=–x A is
æp ö æp ö k
(5, 0) 3 sin ç + q÷ + 2 cos ç + q÷ = …(i)
X è 2 ø è 2 ø r
0
3x+5y=15 æ p ö
Since, it is passing through ç -1, ÷ .
è 2ø
In the above figure, points A and B are k
\ 3 sin p + 2 cos p = Þ k =2
on the line (i) and are equidistance from the coordinate -1
axes. On putting k = 2 in Eq. (i), we get
æ 15 15 ö 2
On solving line (i) and y = x, we get A ç , ÷. 3 cos q - 2 sin q =
è8 8ø r
Similarly, on solving line (i) and y = - x, we get Þ 2 = 3r cos q - 2 r sin q
æ 15 15 ö
B ç- , ÷. 29. Given,
è 2 2ø
2 (OA ) + 3 (OB) = 10
So, the required points lie only in I and II quadrants.
Þ 2x1 + 3 y1 = 10 …(i)
306 JEE Main Mathematics

3
\The slope of required line = -
4
B (0, y1)
æ 7 1ö
P (d, k) \Equation of required line which passes through ç , ÷
è 5 5ø
A 3
and having slope - , is
O (x1, 10) 4
1 -3 æ 7ö
y- = çx - ÷
5 4 è 5ø
3x 21 1
Þ + y= +
4 20 5
Let P (h , k) be the mid-point of AB
3x + 4 y 21 + 4
x1 Þ =
\ = h Þ x1 = 2h 4 20
2
y1 Þ 3x + 4 y = 5 Þ 3x + 4 y - 5 = 0
and =k Þ y1 = 2k
2 34. Since, the given lines are concurrent.
Substituting the values of x1 and y1 in Eq. (i), we get a b c
2 (2h ) + 3 (2k) = 10 \ b c a = 0 Þ a3 + b3 + c3 - 3abc = 0
Þ 4h + 6k = 10 c a b
Þ 2h + 3k = 5
Þ (a + b + c) (a 2 + b2 + c2 - ab - bc - ca ) = 0
\Required locus is 2x + 3 y = 5
(a + b + c)
30. Given line AB makes 0 intercepts on X-axis and Y -axis Þ {(a - b)2 + (b - c)2 + (c - a )2 } = 0
2
or (x1 , y1 ) º (0, 0) and the line is perpendicular to line
CD, 3x + 4 y + 6 = 0. Þ a + b + c=0 (as a ¹ b ¹ c)
\ Slope of required line which is perpendicular to 35. Line perpendicular to the given Y
3x + 4 y + 6 = 0 is 4/3. x y x y
line - = 1 is – =1
a b
\ Required line which is passing through origin and a b
having slope 4/3, is 1 1 X
x+ y+ l =0 …(i) O P(a, 0)
4 b a
y - 0 = (x - 0) Þ 4x - 3 y = 0 Q (0, –b)
3 According to the question, line (i)
is passing through the point P (a , 0).
31. The point of intersection of lines 2x - 3 y + 4 = 0 and
a a
æ 2 22 ö \ + 0 + l =0Þl = -
3x + 4 y - 5 = 0 is ç - , ÷. b b
è 34 17 ø
On putting the value of l in Eq. (i), we get
The slope of required line which is perpendicular to
7 x y a
6x - 7 y + 3 = 0 is - . + - = 0 Þ ax + by = a 2
6 b a b
\ Equation of required line 36. B(3, 4)
22 7æ 2ö (–1, 1)A P
y- = - çx + ÷ Q
17 6è 34 ø C(2, 0)
6 (17 y - 22) 7(34x + 2)
Þ =- P º (x1 , mx1 )
17 34
Þ 119x + 102 y = 125 Q º (x2, mx2)
3 4 1
32. From the given equations, we get 1 13
A1 = 2 0 1=
2 2
m2 + am + 2 = 0 -1 1 1
Since, m is real, a 2 ³ 8 Þ| a | ³ 2 2 x1 mx1 1
So, least value of| a |is 2 2. 1
A2 = x2 mx2 1
2
33. The intersection point of lines 2 0 1
æ 7 1ö 1
x - 2 y = 1 and x + 3 y = 2 is ç , ÷ . A2 = |2 (mx1 - mx2)| = m|x1 - x2|
è 5 5ø 2
13
Q Required line is parallel to 3x + 4 y = 0. A1 = 3 A2 Þ = 3m|x1 - x2|
2
Straight Lines 307

AC : x + 3 y = 2 x-1 6 y-2 6
Þ = , =
BC : y = 4x - 8 cos q 3 sin q 3
2 6 6
P : x + 3 y = 2 and y = mx Þ x1 = x= cos q + 1, y = sin q + 2
1 + 3m 3 3
8 It satisfies x + y = 4
Q : y = 4x - 8 and y = mx Þ x2 =
4 -m 6 6
\ cos q + 1 + sin q + 2 = 4
2 8 3 3
|x1 - x2| = -
1 + 3m 4 - m 6 6
Þ cos q + sin q = 1
- 26m 26m 3 3
= =
(1 + 3m) (4 - m) (3m + 1)|m - 4| Þ 6 cos q + 6 sin q = 3

=
26m Þ 6 cos q + 6 sin 2 q + 2 6 6 cos q sin q = 9
2

(3m + 1) (4 - m) Þ 6 (cos 2 q + sin 2 q) + 2 ´ 6 cos q sin q = 9


13
|x1 - x2| = Þ 6 + 6 sin 2q = 9
6m
Þ 6 sin 2q = 9 - 6
26m 13
= 3
(3m + 1) (4 - m) 6m Þ sin 2q =
2
6
Þ 12m = - (3m + 1) (m - 4) 1
Þ 12m2 = - (3m2 - 11m - 4) Þ sin 2q = = sin 30°
2
2
Þ 15m - 11m - 4 = 0
Þ 2q = 30° Þ q = 15°
Þ 15m2 - 15m + 4m - 4 = 0
or sin 2q = sin (180° - 30° )
Þ (15m + 4) (m - 1) = 0
Þ 2q = 150° Þ q = 75°
Þ m =1
40. y = (2 - 3 )(x + 5) and y = (2 + 3 )(x - 7 )
37. Given lines are 3x + 4 y = 5, 5x + 4 y = 4 and lx + 4 y = 6
These three lines meet at a point, if the point of Slopes are,
intersection of first two lines lies on the third line. m1 = 2 - 3 ,m2 = 2 + 3
Now, point of intersection of line 3x + 4 y = 5 and Angle between two lines,
æ 1 13 ö ½ 2 + 3 - (2 - 3 ) ½
5x + 4 y = 4 is ç - , ÷ . tan q = ½ ½
è 2 8ø ½1 + (2 + 3 )(2 - 3 )½
æ 1 13 ö Þ tan q = | 3 |
The line lx + 4 y = 6 passes through the point ç - , ÷ .
è 2 8ø i.e., tan q = 3 and tan q = - 3
æ 1ö æ 13 ö p 2p
\ l ç - ÷ + 4 ç ÷ = 6 Þ - l + 13 = 12 \ q = or
è 2ø è8ø 3 3
Þ l =1 41. Given, lines x + (a - 1) y = 1
38. Mid-point of the line joining the points (4, - 5) and (-2 , 9) and 2x + a 2y = 1, where a Î R - {0, 1}
æ 4 - 2 -5 + 9 ö are perpendicular to each other
is ç , ÷ i. e. , (1, 2). æ
è 2 2 ø 1 ö æ 2ö
\ çç - ÷ ´ ç- ÷ = -1
è a - 1 ÷ø è a 2 ø
\ Inclination of straight line passing through point
(-3, 6) and mid-point (1, 2) is [Q if lines are perpendicular,
then product of their slopes is -1]
2 - 6 -4
m= = = -1 Þ a 2(a - 1) = - 2
1+3 4
3p Þ a - a2 + 2 = 0
3

\ tan q = - 1 Þ q= Þ (a + 1)(a 2 - 2a + 2) = 0 Þ a = - 1
4
\Equation of lines are
39. Let equation of line is
x - 2y = 1 …(i)
x - x1 y - y1
= =r and 2x + y = 1 …(ii)
cos q sin q
On solving Eq. (i) and Eq. (ii), we get
x-1 y-2 6
\ = = 3 1
cos q sin q 3 x = and y = -
5 5
308 JEE Main Mathematics

\ Point of intersection of the lines (i) and (ii) is 1±1


\ y-4 = (x - 3)
æ3 1ö 1m1
P ç , - ÷.
è5 5ø
Þ y = 4 or x=3
æ3 1ö
Now, required distance of the point P ç , - ÷ from Hence, the lines which make the triangle are x - y = 2 ,
è 5 5ø
x = 3 and y = 4.
origin
The intersection points of these lines are (6, 4), (3, 1) and
9 1 10 2
= + = = (3, 4).
25 25 25 5
1
\ Area of triangle = |6(1 - 4) + 3(4 - 4) + 3(4 - 1 )|
42. Slope of the line 2x - 3 y + 17 = 0 is 2
2 1
= m1, (let) and the slope of line joining the points = |6(-3) + 3(0) + 3(3)|
3 2
b - 17 b - 17 1 9
(7, 17) and (15, b ) is = = m2 (let) = | - 18 + 0 + 9| = sq units
15 - 7 8 2 2
According to the question, m1m2 = - 1 46. The vertices of triangle are the intersection points of the
2 b - 17 lines
Þ ´ = -1
3 8 x+ y=0 …(i)
Þ b - 17 = - 12 Þ b = 5 3x + y = 4 …(ii)
43. Since, BA ^ OA C B (1, 1) and x + 3y = 4 …(iii)
1 On solving Eqs. (i) and (ii), (ii) and (iii), (iii) and (i), we
\ Slope of AB = -
Slope DA get the vertices of triangles are A(-2 , 2), B(1, 1 ) and
4x+7y+5=0 C (2 , - 2).
1 7 A
=- = D(–3, 1)
4 4 Now, AB = (1 + 2)2 + (1 - 2)2 = 9 + 1 = 10
-
7
BC = (2 - 1 )2 + (-2 - 1 )2
7
Equation of AB, y - 1 = (x - 1)
4 = 12 + (-3)2 = 10
Þ 4 y - 4 = 7x - 7 AC = (2 + 2)2 + (-2 - 2)2 = 16 + 16 = 4 2
7x - 4 y - 3 = 0
Q AB = BC
Since, CD || AB.
So, triangle is isosceles.
7
\Equation of CD, y - 1 = (x + 3) 47. Given equation of lines are
4
4 y - 4 = 7x + 21 5x + 3 y - 7 = 0 …(i)
14
7x - 4 y + 25 = 0 and 15x + 9 y + 14 = 0 or 5x + 3 y + =0 …(ii)
3
Now, since BC || AD.
Q Lines (i) and (ii) are parallel and c1 and c2 are of
Equation of BD, 4x + 7 y = l
opposite signs, therefore these lines are on opposite
This is passes through (1, 1). sides of the origin.
\ 4 ´1 + 7 ´1 = l So, the distance between them is
Þ l = 11 c1 c2 7 14
+ = - +
Now, equation of BC ,4x + 7 y = 11 a12 + b12 a 22 + b22 2
5 +3 2
3 52 + 32
44. Two sides x - 3 y = 0 and 3x + y = 0 are perpendicular to
7 14 35
each other. Therefore, its orthocentre is the point of = - + =
intersection of 34 3 34 3 34

x - 3y = 0 48. Lines are


and 3x + y = 0 i. e. , (0, 0) L1 : 4x + 3 y + 10 = 0, L 2 : 5x - 12 y + 26 = 0
So, the line 3x - 4 y = 0 passes through the orthocentre of L3 : 7x + 24 y - 50 = 0
triangle. By option method,
45. The equation of lines are Distance of lines from (0, 0) are equal.
m ± m2 0 + 0 + 10½ ½0 + 0 + 26½ ½0 + 0 - 50½
y - y1 = 1 (x - x1 ) i.e., ½ = =
1 m m1m2 ½ 5 ½ ½ 13 ½ ½ 25 ½
Since, m1 = 1, m2 = 1 Þ 2 =2 =2
Straight Lines 309

49. The given line is x tan a - y + c = 0 53. Let P(3, - 4) be the foot of the perpendicular from the
origin O on the required line.
or x sin a - y cos a + c cos a = 0
-4 - 0 -4
\ Length of perpendicular from (a cos a , a sin a ) Then, the slope of OP = =
3 -0 3
a cos a sin a - a sin a cos a + c cos a
= Therefore, the slope of the required line is 3 / 4.
sin 2 a + cos 2 a 3
Hence, its equation is y + 4 = (x - 3)
c cos a 4
= = c cos a
1 Þ 3x - 4 y - 9 - 16 = 0 Þ 3x - 4 y = 25
50. Let the equation of line parallel to x - 2 y = 1 is 54. Tangent to circle 3x + 4 y = 25
x - 2y + l = 0
0 , 25 Q
Since, it is passes through (3, 5). 4
\ 3 - 10 + l = 0 125
Þ l=7 12
\ Line is x - 2 y + 7 = 0
The point of intersection 2x + 3 y - 14 = 0 and P
x - 2 y + 7 = 0 is (1, 4). O 25 , 0
\ The distance between (3, 5) and (1, 4) 3
= (3 - 1 )2 + (5 - 4)2 = 4 + 1 = 5 OP + OQ + OR = 25
51. Let the foot be A(a , b). æ 25 25 25 25 ö
ç ´ ´ ÷
P(2, 3) Incentre = ç 4 3 , 4 3 ÷
ç 25 25 ÷
ç ÷
è ø
æ 25 25 ö
=ç , ÷
è 12 12 ø
y = 3x + 4 A(a, b) 2
æ 25 ö 625 625
\ r2 = 2 ç ÷ = 2 ´ =
b = 3a + 4 …(i) è 12 ø 144 72
and m1 ´ m2 = - 1 55. Given that, y = 3 | x| + 2
b -3 5 3
´3 = - 1 and PQ = 5, so QR =
a -2 2
Þ 3b = - a + 11 …(ii) Y
On solving Eqs. (i) and (ii), we get
R P
1 37
a=- ,b = 30° 5
10 10
52. Here, (x1 , y1 ) = (3, 4) Q(0, 2)
(3,4) X
O

æ 5 3ö æ 4 + 5 3ö
So, coordinates of R are çç0, 2 + ÷ or ç0,
÷ ç
÷.
÷
P 2x + y – 7= 0 è 2 ø è 2 ø
(h , k )
Given equation is 2x + y - 7 = 0 56. Let B(x1 , y1 ) and C (x2, y2) are the vertices of a triangle.
On comparing with ax + by + c = 0 , we get æ x + 1 y1 - 2 ö
Pç 1 , ÷ lies on the line x - y + 5 = 0.
a = 2, b = 1 and c = -7 è 2 2 ø
x1 + 1 y1 - 2
Let (h , k) be the coordinates of the foot. \ - = -5
h - 3 k - 4 - (2 ´ 3 + 1 ´ 4 - 7) - 3 2 2
Then, = = = Þ x1 - y1 = - 13 …(i)
2 1 22 + 12 5
h - 3 -3 k - 4 -3 A(1, – 2)
Þ = and =
2 5 1 5
P
-6 -3
Þ h= + 3 and k = +4 N Q
5 5
9 17
Þ h= and k = B(x1, y1) M C(x2, y2)
5 5
310 JEE Main Mathematics

Also, PN ^ AB 60. The intersection of given lines is


y1 + 2
\ = -1 x y æx y ö
x1 - 1 + - 1 + l ç + - 1÷ = 0
a b è b a ø
Þ y1 + 2 = - x1 + 1 meets the coordinate axes at
Þ x1 + y1 = - 1 …(ii) é ù é ù
On solving Eqs. (i) and (ii), we get ê1+ l ú ê 1+ lú
Aê , 0 and B ê0 ,
1 l ú 1 lú
x1 = - 7 and y1 = 6 ê + ú ê + ú
ëa b û ë b aû
\Coordinates of B are (-7, 6).
The mid-point of AB is given by
æ 11 2 ö
Similarly, the coordinates of C are ç , ÷. 1+ l 1+ l
è 5 5ø 2x = ,2y =
1 l 1 l
\Equation of BC is + +
a b b a
2
-6 é1 1 ù
( y - 6) = 5 (x + 7 ) Þ (1 + l ) ê + ú
11 ë x yû
+7
5 é 1 lù é1 lù
=2 ê + ú + 2 ê + ú
Þ 14x + 23 y - 40 = 0 ë a bû ëb aû
57. Equation of the straight lines are é 1 1ù
= 2 (1 + l ) ê + ú
3x - 4 y + 7 = 0 …(i) ë a bû
and 12x + 5 y - 2 = 0 …(ii) \ (x + y) ab = 2xy (a + b)
The equation of bisectors of the acute angle between 61. The family of lines (x - 2 y + 3) + l(2x - 3 y + 4) = 0 are
these lines are concurrent at point P(1, 2).
3x - 4 y + 7 12x + 5 y - 2
= If image of point A(2, 3) in the above variable line is
32 + 42 122 + 52 B(h , k ), then AP = BP
3x - 4 y + 7 12x + 5 y - 2 Þ (h - 1 )2 + (k - 2)2 = (2 - 1 )2 + (3 - 2)2
Þ =
5 13 Hence, locus of point P is
Þ 39x - 52 y + 91 = 60x + 25 y - 10
x2 + y2 - 2x - 4 y + 4 = 0
Þ 21x + 77 y - 101 = 0
62. Let the coordinates of the third vertex A be (h , k ).
58. The equation of lines are
Also, AD ^ BC
x + y = 0 and x - y = 0.
k -0 æ 4 ö
\The equation of bisectors of the angles between these \ ´ ç ÷ = -1
h - 0 è -7 ø
lines are
x+ y x- y A(h, k)

1+1 1+1
Þ x + y = ± (x - y)
Taking positive sign, x + y = (x - y) Þ y = 0
O(0, 0)
Taking negative sign, x + y = - (x - y) Þ x = 0
Hence, the equation of bisectors are x = 0 and y = 0. B(5, –1) D C(–2, 3)

59. Equation belonging to both families will pass through Þ 7h = 4k …(i)


two fixed points. First intersection point of lines
and OB ^ AC
æ 1 1ö
x + 2 y = 0 and 3x + 2 y + 1 = 0, is ç - , ÷ and second k - 3 æ 1ö
è 2 4ø Þ ´ ç- ÷ = -1
h + 2 è 5ø
intersection point of lines x - 2 y = 0 and x - y + 1 = 0, is
(-2 , - 1 ). Þ 5h - k + 13 = 0 …(ii)
æ 1 1ö
Line passing through ç - , ÷ and (-2, - 1 ) is On solving Eqs. (i) and (ii), we get
è 2 4ø
h = - 4,
1
-1 - k = -7
1 4 æç x + 1 ö÷ Þ 5x - 6 y + 4 = 0
y- =
4 -2 + 1 è 2ø Hence, the coordinates of third vertex are (-4, - 7 ).
2
Straight Lines 311

63. Now, we take BD ^ AC and AE ^ BC. Round II


A(0, 0)
1. Lines are y = 1, y = 0
y = - x, y =-x+2
y = x + 1, y = x -1
D
X
y = x–1

B C G
B(4, 0) E C (3, 4)
3 A D
Slope of BD = -
4 Y
-3 O F
Equation of BD, y - 0 = (x - 4) E
4 y = –x+2
Þ 4 y = - 3x + 12
Þ 3x + 4 y - 12 = 0 …(i) Area of OABCDE = Area of OBGF
1 3 3
and slope of AE = = ´ 1 = sq units
4 2 2
1
\ Equation of AE, y - 0 = (x - 0) Þ x - 4 y = 0 …(ii) 2. The perpendicular distance of (1, 3) from the line
4
On solving Eqs. (i) and (ii), we get 3x + 4 y = 5 is 2 units while
3 sec2 q + 2 cosec2 q ³ 3 (as sec2 q, cosec2 q ³ 1 )
x = 3, y =
4 So, there will be two such points on the line.
æ 3ö
\Orthocentre of the triangle is ç3, ÷ .
è 4ø 3. Any line through A is given by
64. We have, x+ y=1 …(i) ( px + qy - 1 ) + l (qx + py - 1 ) = 0
and xy = 0 …(ii) which is passing through ( p, q).
Y ( p2 + q 2 - 1 )
Hence, l=-
Q(0, 1) 2 pq - 1

Thus, the required line is


P(1, 0) ( p2 + q 2 - 1 )
X ( px + qy - 1 ) - × (qx + py - 1 ) = 0
O (2 pq - 1 )
On putting x = 1 - y from Eq. (i) into Eq. (ii), we get Þ (2 pq - 1 )( px + qy - 1 ) - ( p2 + q2 - 1 )(qx + py - 1 ) = 0
(1 - y) y = 0 Þ y = 0, 1
4. From figure refracted ray makes an angle of 75° with
At y = 0 Þ x = 1 and at y = 1 Þ x = 0 positive direction of X-axis and passes through the
So, coordinates of the vertices of a triangle are (0, 0), point (1, 0).
(1, 0) and (0, 1). \ Its equation is
Since, the triangle is right angled at O(0, 0). ( y - 0) = tan(45°-30° ) (x - 1 )
So, point (0, 0) is its orthocentre.
Y
65. Since, t1, t2 are the roots of the equation t 2 + lt + 1 = 0.
\ t1 + t2 = - l, t1t2 = 1 °
30
The equation of a line passing through (at12, 2 at1 ) and 45°
X
(at22, 2 at2) is O A(1, 0)
2 2
y - 2 at2 = (x - at22) Þ y - 2 at2 = - (x - at22)
t1 + t2 l
Þ ly - 2 alt2 = - 2 x + 2 at22 Þ ly + 2 x = 2 a (lt2 + t22) B (0, –1)

Þ ly + 2 x = 2 a (-1 ) Þ 2 (x + a ) + ly = 0
or y = (2 - 3 ) (x - 1)
So, fixed point is (-a , 0).
312 JEE Main Mathematics

ax by 9. x+y=1
5. Equation of line + + 1 = 0 has two independent
c-1 c-1
A y = m 1x
parameters.
It can pass through a fixed point, if it contains only one
independent parameter. O(0, 0) B
a y = m 2x
Now, there must be one relation between and
c-1 C
b a
independent of a , b and c so that can be
c-1 c-1
b y = m 3x
expressed in terms of and straight line contains
c-1 On solving the given lines, we get
only one independent parameter. Now, that given æ 1 m1 ö æ 1 m3 ö
5a 4b t - 20c A = çç , ÷÷ , C = çç , ÷
relation can be expressed as + = è 1 + m1 1 + m1ø è 1 + m3 1 + m3 ÷ø
c-1 c-1 c-1
If AB = BC , then mid-point of AC lies on
RHS in independent of c, if t = 20
y = m2x
6. The family of lines
m m3 é 1 1 ù
(x + y - 1 ) + l (2x + 3 y - 5) = 0 + ê1 + m + 1 + m ú
1 + m1 1 + m3 1 3
passes through a point such that Þ = m2 ê ú
2 ê 2 ú
x + y -1 = 0 ëê úû
2x + 3 y - 5 = 0
Þ m1 (1 + m3 ) + m3 (1 + m1 ) = m2 [1 + m3 + 1 + m1 ]
i.e., (- 2 , 3) and family of lines
Þ (1 + m1 ) (m3 - m2) = (1 + m2) ( m2 - m1 )
(3x + 2 y - 4) + m( x + 2 y - 6) = 0
10. Slope of BD is 8/15 and angle made by A B
passes through a point such that BD with DC and BC is 45°. So, let
0
3x + 2 y - 4 = 0 slope of DC be m. Then, =
y
and x + 2y -6 = 0
8 15
m- –
tan 45° = ± 15 8x
i.e., (- 1, 7/2) 8 45°
1+ m 45°
\ Equation of the straight line that belongs to both the 15
D C(1, 2)
families passes through (- 2 , 3) and (- 1, 7/2) is Þ (15 + 8m) = ± (15m - 8)
7 23 7
-3 Þ m= and -
y -3 = 2 (x + 2) 7 23
-1 + 2
Hence, the equations of DC and BC are
x+2 23
Þ y -3 = y-2 = (x - 1 ) Þ 23x - 7 y - 9 = 0
2 7
Þ x -2y + 8 = 0 7
and y-2 = - (x - 1 ) Þ 7x + 23 y - 53 = 0
7. d (x, y) = max {|x|| y|} …(i) 23

but d (x, y) = a …(ii) 11. ax + by = 1 will be one of the bisectors of the given line.
Equation of bisectors of the given lines are
From Eq. (i) and (ii), a = max {|x|, | y|}
3x + 4 y - 5 æ 5x - 12 y - 10 ö
if|x|>| y|, then a =|x| =± ç ÷
5 è 13 ø
\ x=±a Þ 64x - 8 y = 115 or 14x + 112 y = 15
and if| y|>|x|, then a =| y| 64 8 14 12
Þ a= ,b = - or a = ,b =
\ y=±a 115 115 15 115
Therefore, locus represents a straight line. 12. The set of lines is 4ax + 3by + c = 0, where a + b + c = 0.
8. If the line meets the X and Y -axes at A and B, then Eliminating c, we get
A º (- c/a , 0), B º (0, - c/b). The line will pass through 4ax + 3by - (a + b) = 0
the first quadrant, if Þ a (4x - 1 ) + b(3 y - 1 ) = 0
- c /a > 0 and /or - c /b > 0 This passes through the intersection of the lines
Þ ac < 0 and /or bc < 0 4x - 1 = 0 and 3 y - 1 = 0, i.e. x = 1/4, y = 1/3, i.e.
(1/4, 1/3).
Straight Lines 313

13. As (a + b)x + (a - b) y - 2ab = 0 Clearly, point A is point of intersection of lines


and (a - b)x + (a + b) y - 2ab = 0 x+ y =3 …(i)
The equations of the angle bisectors are and x - y = -3 …(ii)
(a + b)x + (a - b) y - 2ab = ± {(a - b)x + (a + b) y - 2ab} So, A = (0, 3)[solving Eqs. (i) and (ii)]
Þ 2bx - 2by = 0 i.e., x = y …(i) Now, as point M (2, 4) is mid-point of line joining the
and 2ax + 2ay - 4ab = 0 i.e., x + y = 2b …(ii) points A and C, so
\Equation of third side as given by Eq. (i) is æ 0 + x2 3 + y2 ö
(2, 4) = ç , ÷
x - y = k satisfying the point (b - a , a - b) è 2 2 ø
\ k = 2b - 2a é æ x1 + x2 y1 + y2 ö ù
êQ mid-point = ç 2 , 2 ÷ ú
\The line is x - y = 2 (b - a ) ë è øû
From Eq. (ii), x + y - 2b = k is passing through the point 0 + x2 3 + y2
Þ 2= ;4 =
(b - a , a - b) 2 2
\ k = - 2b Þ x2 = 4 and y2 = 5
\The line is x + y = 0. \ Thus, C º (4, 5)
x y Now, equation of line BC is given by
14. bx + cy = a Þ + =1
(a /b) (a /c) ( y - y1 ) = m (x - x1 )
1 y - 5 = 1(x - 4)
Area of D OAB = (given)
8 [line BC is parallel to x - y + 3 = 0 and slope
Y (-1)
of x - y + 3 = 0 is = 1]
(-1)
(0, a/c)
B Þ y = x+1 …(iii)
and equation of line DC is
y - 5 = -1 (x - 4)
(a/b, 0) [line DC is parallel to x + y = 3 and
X -1
O A slope of x + y = 3 is = -1]
1
1 a a 1
Þ × × =± Þ x+ y =9 …(iv)
2 b c 8
On solving Eqs. (i) and (iii), we get B (1, 2) and on
a2 1 solving Eqs. (ii) and (iv), we get D (3, 6)
Þ =±
bc 4
17. Let ABC be a given triangle with vertices B(0, 2), C (4, 3)
Þ (2 a )2 = ± bc
and let third vertex be A (a , b)
So, b, ± 2 a , c are in GP.
A (a, b)
15. Given lines are x + 2 y + 4 = 0 and 4x + 2 y - 1 = 0
Þ x + 2 y + 4 = 0 and - 4x - 2 y + 1 = 0 E
F
Here, (1 ) (- 4) + (2) (-2) = - 8 < 0 (0,0)
So, bisector of the angle is acute angle bisector including
origin.
x + 2 y + 4 (-4x - 2 y + 1 ) (0, 2) B C (4,3)
Its equation is = D
5 2 5
Also, let D , E and F are the foot of perpendiculars
Þ 6x + 6 y + 7 = 0 drawn from A , B and C respectively.
16. According to given information, we have the following b -0 3 -2
Then, AD^ BC Þ ´ = -1
figure a -0 4 -0
[if two lines having slopes m1 and m2, are
D C (x2 , y2 ) perpendicular then m1m2 = -1]
M
Þ b + 4a = 0 …(i)
x–y+3=0 (2,4) and CF^ AB
b -2 3 -0
B Þ ´ = -1
a -0 4 -0
A x+y=3
Þ 3 b - 6 = -4 a
[Note that given lines are perpendicular to each Þ 4a + 3b = 6 …(ii)
other as m1 ´ m2 = -1]
From Eqs. (i) and (ii), we get
314 JEE Main Mathematics

-b + 3b = 6 Þ 2b = 6 20. Since, the line L is passing through the point (13, 32).
3
Þ b = 3 and a = - [from Eq. (i)] Therefore,
4 13 32 32 8
æ 3 ö + - 1Þ = Þ b = - 20
So, the third vertex (a , b) º ç - , 3÷ , which lies in 5 b b 5
è 4 ø The line K is parallel to the line L, its equation must
II quadrant. be
18. Given line is x y x y
- = a or - =1
(2x - 3 y + 4) + k(x - 2 y + 3) = 0, k Î R …(i) 5 20 5a 20a
x y
This line will pass through the point of intersection of On comparing with + = 1, we get
c 3
the lines
3
2x - 3 y + 4 = 0 …(ii) 20a = - 3, c = 5a = -
4
and x - 2y + 3 = 0 …(iii)
Hence, the distance between lines
On solving Eqs. (ii) and (iii), we get
| a - 1| -3 17 23
x = 1, y = 2 = = ½ - 1½ =
1 1 ½20 ½ 400 17
\Point of intersection of lines (ii) and (iii) is (1, 2). +
25 400
Let M be the mid-point of PP ¢, then AM is
perpendicular bisector of PP ¢ (where, A is the point of 21. Firstly, make the constant terms (c1 , c2) positive, then
intersection of given lines). - x - y + 3 = 0 and 7x - y + 5 = 0
P (2, 3)
0

Q a1a 2 + b1b2 = (-1) (7) + (-1) (-1)


4=
y+

= - 7 + 1 = -6
–3

a1a 2 + b1b2 < 0


2x

Þ
(1, 2)
A M \ ‘ ‘ + ’ ’ sign gives the acute bisector
-x- y+3 7x - y + 5
x=

\ Acute bisector is =+
2

2 2 5 2
y+

(-1) + (-1)
P¢ (h, k)
3=

Þ - 5x - 5 y + 15 = 7x - y + 5
0

\ AP = AP ¢ Þ 12x + 4 y - 10 = 0
Þ (2 - 1)2 + (3 - 2)2 = (h - 1)2 + (k - 2)2 Þ 6x + 2 y - 5 = 0
Þ 2 = h 2 + k2 - 2h - 4k + 1 + 4 4 -3 1
22. Since, slope of PQ = =
2 2 1-k 1-k
Þ 2 = h + k - 2h - 4k + 5
A
Þ h 2 + k2 - 2h - 4k + 5 = 2
Þ h 2 + k2 - 2h - 4k + 3 = 0
Thus, the required locus is
x2 + y2 - 2x - 4 y + 3 = 0
which is a equation of circle with P Q (k, 3)
(1, 4) M
radius = 1 + 4 - 3 = 2 k + 1, 7
2 2
19. As x + y =| a |and ax - y = 1, intersect in first quadrant.
\x and y- intercepts are positive. Slope of AM = (k - 1 )
1 + |a | \ Equation of AM is
\ x= ³0
1+ a 7 é æ k + 1öù
y - = (k - 1 ) ê x - ç ÷ú
a |a |- 1 2 ë è 2 øû
and y= ³0
a+1 For y-intercept, x = 0, y = - 4
Þ 1 + a ³ 0 and a | a |- 1 ³ 0 7 æ k + 1ö
-4 - = - (k - 1 ) ç ÷
Þ a ³ - 1 and a | a |³ 1 …(i) 2 è 2 ø
If -1 £ a < 0 15 k2 - 1
Þ =
2 2
Þ -a 2 > 1 (not possible)
If a ³ 0 Þ a2 ³ 1 Þ k2 - 1 = 15
Þ a ³1 Þ k2 = 16
\ a > 1 or a Î [1, ¥ ) …(ii) Þ k = ±4
Straight Lines 315

3 3 -0 p 27. Equation of a line passing through the intersection of


23. Now, slope of QR = = 3 = tan qÞ q =
3 -0 3 lines a x + 2 by + 3b = 0 and bx - 2 ay - 3a = 0 is
Y (ax + 2 by + 3b) + l (bx - 2 ay - 3a ) = 0 …(i)
M R(3, 3Ö3) Now, this line is parallel to X-axis, so coefficient
of x = 0
P(–1, 0) a
p/3 Þ a + lb = 0 Þ l = -
X¢ X
O (0, 0)Q 2p/3 b
On putting this value in Eq. (i), we get
b (ax + 2 by + 3 b) - a (bx - 2 ay - 3a ) = 0
Y¢ Þ 2 b2 y + 3 b2 + 2 a 2 y + 3a 2 = 0
3
2p Þ 2 (b2 + a 2) y + 3 (b2 + a 2) = 0 Þ y = -
\The angle between ÐPQR is , so the line QM makes 2
3
2p Therefore, the required line is below X-axis, at a
an angle from positive direction of X-axis. Slope of 3
3 distance from it.
2p 2
the line QM = tan =- 3
3 28. Let the coordinates of vertex C is (x, y) and the
Hence, equation of line QM is y = - 3x
coordinates of centroid of the triangle is (x1 , y1 ).
Þ 3x + y = 0 x + 2 -2 y -3 + 1
\ x1 = and y1 =
24. Since, A is the mid-point of line PQ. 3 3
x y -2
Y Þ x1 = and y1 =
3 3
P(0, b)
Since, the centroid lies on the line 2x + 3 y = 1.
A(3, 4) x ( y - 2)
\ 2x1 + 3 y1 = 1 Þ 2 + 3 =1
3 3

O
X Þ 2x + 3 y - 6 = 3 Þ 2x + 3 y = 9
Q(a, 0)
Y¢ 29. Let the line intercept the X-axis at a and Y -axis at b
distance.
a+0
\ 3= Þ a =6 Since, a + b = - 1 Þ b = - (a + 1 )
2
x y
0+ b \Equation of line is + =1
and 4= Þ b =8 a a+1
2
4 3 4a + 4 - 3a
Hence, the equation of line is \ + = 1Þ =1
a a+1 a (a + 1 )
x y
+ = 1 Þ 4x + 3 y = 24 Þ a + 4 = a2 + a Þ a = ± 2
6 8
Hence, equation of lines are
25. The point (a , a 2) lies in the sector bounded by the lines x y x y
+ = 1 and + =1
x - 2 y = 0 and 3x - y = 0 2 3 -2 1
\ (a - 2a 2)(3a - a 2) < 0 30. It is given that distance between lines 2x - y + 3 = 0 and
é 1ù æ1 ö 1
4x - 2 y + a = 0 is
Þ êë a - 2 úû (a - 3) < 0 \a Î çè 2 , 3÷ø 5
|6 - a| 1
26. Since, a , b and c are in HP. \ = Þ |a - 6| = 2 Þ a = 8, 4
16 + 4 5
1 1 1
Then, , and are in AP. And it is also given that distance between lines
a b c 2
2 1 1 2x - y + 3 = 0 and 6x - 3 y + b = 0 is .
\ = + 5
b a c
|b - 9| 2
1 2 1 \ =
Þ - + =0 36 + 9 15
a b c
x y 1 Þ |b - 9| = 3
Hence, straight line + + = 0 is always passes Þ b = 12, 6
a b c
through a fixed point (1, –2). So, sum of all possible values of a and b is
4 + 8 + 6 + 12 = 30.
316 JEE Main Mathematics

31. The equation of a line through A. i.e. the point of cos 3a + cos 3b + cos 3g
\
intersection of AB and AC is cos a cos b cos g
Y 4 (cos3 a + cos3 b + cos3 g ) - 3 (cos a + cos b + cos g )
=
B cos a cos b cos g
= (12)2 = 144
33. ax + by + 8 = 0
Þ ax + by = -8
x y
Þ + =1 (intercept form)
8 8
- -
X¢ X a b
O D
ax+by–1=0 x y
C Also, 2x - 3 y = - 6 Þ - + = 1
3 2
E
A According to given condition, we have
8 8
Y¢ - = - (- 3) and - = - 2
a b
(2x + 3 y - 1) + l (ax + by - 1) = 0 …(i) 8
Þ a = - and b = 4
It passes through O (0, 0), then - 1 - l = 0 3
\ l = -1 8
\ 3|a + b| = 3 - + 4
From Eq. (i), we get 3
2x + 3 y - 1 - ax - by + 1 = 0 = 3 (4 / 3)
Þ (2 - a )x + (3 - b) y = 0 =4
Since, AD ^ BC 34. Let (x1 , y1 ) be the image of the point (1, 2) about the line
(2 - a ) æ 1 ö y = x.
\ - ´ ç- ÷ = -1
3 - b è 2ø Then, x1 = 2, y1 = 1 …(i)
Þ 2 - a = - 6 + 2b Y A (1, 2)
y=x
a + 2b = 8 …(ii)
Similarly, BE ^ AC
B (x1, x2)
Þ a + b =0 …(iii) O
X¢ X
Solving Eqs. (ii) and (iii), we get
b = 8, a = - 8 (a, b)
\ a + b =0 Y¢

32. Since orthocentre and circumcentre both lies on Y -axis Also, given image of (x1 , y1 ) by the mirror line y = 0 is
(a , b ). Then, a = x1 = 2
Þ Centroid also lies on Y -axis
and b = - y1 = - 1 [from Eq. (i)]
Þ S cos a = 0
\ a = 2, b = - 1
cos a + cos b + cos g = 0
3 3 3 \ |a - b| = 2 + 1 = 3
Þ cos a + cos b + cos g = 3 cos a cos b cos g
13
Circles
A circle is defined as the locus of all such points in a plane, which remains at IN THIS CHAPTER ....
constant distance from a fixed point.
Standard Form of Equation
Y of Circle
N
P Cyclic Quadrilateral
Position of a Point with Respect to
C r X
M a Circle
Tangent to a Circle
Normal to a Circle
Here, the fixed point (C ) is called the centre of the circle and the constant
distance is called its radius. Equation of Pair of Tangents
Director Circle
Different Form of Equation of Circle Chord of Contact

Equation of Circle in Standard Form Equation of the Chord Bisected at


a Given Point
Let C( h , k) be the centre of circle and CP ( = r ) be the radius of circle, then
Diameter of a Circle
equation of circle is
Pole and Polar
( x - h )2 + ( y - k)2 = r 2 …(i)
Family of Circles
The above equation is known as the central form of the equation of a circle.
Common Tangents to Two Circles
Y
Angle of Intersection of
P(x, y) Two Circles
r Radical Axis

C(h, k) Coaxial System of Circles


Image of the Circle by the
Line Mirror
X
O

Now, if origin (0, 0) is the centre of circle, then Eq. (i) becomes,
x2 + y2 = r 2 …(ii)
318 JEE Main Mathematics

Equation of Circle in General Form Example 1. The lines 2 x - 3y = 5 and 3x - 4y = 7 are


diameters of a circle of area 154 sq units. Then, the
The general equation of second degree may represents a circle.
equation of the circle is
If the coefficient of x 2 and coefficient of y 2 are identical and (a) x2 + y 2 - 2x + 2y - 47 = 0
the coefficient of xy becomes zero.
(b) x2 + y 2 + 2x - 2y - 47 = 0
i.e., ax 2 + by 2 + 2hxy + 2gx + 2 fy + c = 0 …(i)
(c) x2 + y 2 - 2x + 2y - 47 = 0
represents a circle, if
(d) x2 + y 2 - 2x - 2y - 47 = 0
(a) a = b, i. e. , coefficient of x 2 = coefficient of y 2
(b) h = 0, i. e. , coefficient of xy = 0, then Eq. (i) reduces Sol. (a) If two diameters intersect each other, then
intersecting point is called centre of the circle i.e. (1, -1).
as, x 2 + y 2 + 2gx + 2 fy + c = 0, whose centre and radius
Given, area of circle = 154 units
are ( - g, - f ) and g2 + f 2 - c, respectively. Þ p (Radius) 2 = 154
7
Nature of the Circle Þ (Radius) 2 = 154 ´ = (7) 2
22
If g2 + f 2 - c > 0, then the radius of circle will be real and real Þ Radius = 7
circle is possible. \ Equation of circle is
If g2 + f 2 - c = 0, then the radius of circle is zero and the circle ( x - 1) 2 + (y + 1) 2 = 49
is called a point circle. Þ x2 + y 2 - 2x + 2y - 47 = 0
2 2
If g + f - c < 0, then the radius of circle will be imaginary, so
in this case, no real circle is possible. Example 2. Circle of radius 8 is passing through
origin and the point (1, 0). If the centre lies on the line
Equation of Circle in Diameter Form y = x, then the equation of the circle is
Let A( x1 , y1 ) and B( x2 , y2 ) be the end points of a diameter of (a) ( x - 2) 2 + (y - 2) 2 = 8 (b) ( x + 2) 2 + (y + 2) 2 = 8
the given circle and let P ( x , y ) be any point on the circle. (c) (y - 3) 2 + (y - 3) 2 = 8 (d) ( x + 3) 2 + (y + 3) 2 = 8
Then equation of circle is
Sol. (a) Let the equation of circle be
( x - x1 )( x - x2 ) + ( y - y1 )( y - y2 ) = 0
x2 + y 2 + 2gx + 2fy + c = 0 , whose centre is ( - g , - f ).
Equation of Circle in Parametric Form Since, the circle passing through (0, 0) and (4, 0).
A general point P ( x , y ) on the circle in terms of parameters \ 0 2 + 0 2 + 2g (0) + 2f (0) + c = 0 Þ c = 0
r and q can be expressed as and 4 2 + 0 2 + 2g ( 4) + 2f (0) + 0 = 0 Þ 16 + 8g = 0
x = h + r cos q and y = k + r sin q, 0 £ q £ 2p g = -2
Þ x - h = r cos q and y - k = r sin q Also, the centre ( - g , - f ) lies on the line y = x.
On squaring and then adding, we get Þ -f =-gÞf =g
\ f = -2
( x - h )2 + ( y - k)2 = r 2(cos2 q + sin2 q ) = r 2
Now, the equation of circle having centre (2, 2) and
which is the central equation of the circle satisfied by each radius 8 is
fixed value of r (i. e. , radius) and varying q. ( x - 2) 2 + (y - 2) 2 = ( 8) 2
If the circle is centered at origin, then the coordinates of Þ ( x - 2) 2 + (y - 2) 2 = 8
centre are C( 0, 0), then the parametric coordinates are
x = r cos q and y = r sin q.
Some Particular Cases of the
Equation of Circle through Three Central Form
Non-collinear Points Case I When the circle passes through the origin
Let A( x1 , y1 ), B( x2 , y2 ) and C( x3 , y3 ) be three non-collinear (0, 0)
points, then the equation of circle through these three x 2 + y 2 - 2hx - 2ky = 0
non-collinear points is given by
Y
x2 + y2 x2 y2 1
x12 + y12 x12 y12 1 C(h, k)
=0
x22 + y22 x22 y22 1 r k
x32 + y32 x32 y32 1 h
(0, 0)O X
M
Circles 319

Case II When the circle touches abscissa (X-axis) Case VI When the circle passes through O (0, 0) and
Let the centre of circle be C( h , k) and it touches X-axis at centre lies on ordinate (Y-axis) In this case h = 0, then
point P, then the radius of circle is CP = k. equation of circle is
Y
( x - 0)2 + ( y - k)2 = k2
Þ x 2 + y 2 - 2 ky = 0
C(h, k)
Y

O X
P
\Equation of circle is C (0, k)

( x - h )2 + ( y - k)2 = (CP )2 = k2
X¢ X
Þ x 2 + y 2 - 2hx - 2ky + h 2 = 0 (0, 0) O

Case III When the circle touches ordinate (Y-axis) Y¢


Let the centre of circle be C( h , k) and it touches Y -axis at Case VII When the circle touches X-axis and cut off
point P with radius CP = h an intercepts on Y-axis of length 2l.
Y
In DCMQ, k2 = h 2 + l 2
h
P Þ h = k2 - l 2
C(h, k)
Y
O X

\Equation of circle is
( x - h )2 + ( y - k)2 = (CP )2 = h 2 l h
M C(h, k)
Þ x 2 + y 2 - 2 hx - 2 ky + k2 = 0 lk k
Q
Case IV When the circle touches both abscissa and O h N
X
ordinate (X-axis and Y-axis) In this case h = k = a.
Y \ Equation of circle is
[x - ( k2 - l 2 )]2 + ( y - k)2 = k2
h C(h, k) Case VIII When the circle touches Y-axis and cut
M
k off an intercept on X-axis of length 2 m.
O X
P
In DCPQ, h 2 = k2 + m 2
\Equation of circle is
( x - a )2 + ( y - a )2 = a 2 Þ k = h 2 - m2
Þ x 2 + y 2 - 2 ax - 2 ay + a 2 = 0 Y
Case V When the circle passes through O (0, 0) and
centre lies on abscissa (X-axis) In this case k = 0, then
equation of circle is h C (h , k )
Y N
h k
m m
X
OQ P

X¢ X \Equation of circle is
(0, 0) O C(h, 0)
( x - h )2 + [ y - ( h 2 - m 2 )]2 = h 2

Note The length of intercepts cut by the circle



x2 + y2 + 2 gx + 2 fy + c = 0 on X-axis is 2 g 2 - c
( x - h )2 + ( y - 0)2 = h 2
and on Y-axis is 2 f 2 - c .
Þ x 2 + y 2 - 2hx = 0
320 JEE Main Mathematics

Example 3. A circle touching the X-axis at (3, 0) and Position of a Point with Respect
making a intercept of length 8 on the Y-axis passes through
the point (JEE Main 2019)
to a Circle
(a) (3, 10) (b) (3, 5) (c) (2, 3) (d) (1, 5) Let S º x 2 + y 2 + 2gx + 2 fy + c = 0 be the equation of the
circle and P ( x1 , y1 ) be any point in the plane of the circle,
Sol. (a) It is given that the circle touches the X-axis at (3, 0) and
then S1 º x12 + y12 + 2gx1 + 2 fy1 + c = 0
making an intercept of 8 on the Y-axis.
Y Case I If S1 > 0, then the point lies outside the circle.
Case II If S1 = 0 , then the point lies on the circle.
B
Case III If S1 < 0, then the point lies inside the circle.

8 M C Maximum and Minimum Distance of a


r r Point from a Circle
A
O (3,0)
X Let any point P ( x1 , y1 ) and circle x 2 + y 2 + 2gx + 2 fy + c = 0
The centre and radius of the circle are ( -g, - f ) and
Let the radius of the circle is ‘r’, then the coordinates of
g2 + f 2 - c, respectively.
centre of circle are (3, r).
P(x1, y1)
From the figure, we have A
C
CM = 3, CA = radius = r B
AB
and AM = BM = =4
2
The maximum and minimum distance from P ( x1 , y1 ) to
Then, r 2 = CM 2 + AM 2 the given circle are
= 9 + 16 = 25 PB = CB + PC = r + PC
Þ r=±5 and PA =|CP - CA| =| PC - r|(P inside or outside)
Now, the equation of circle having centre (3, ± 5) and radius where r = ( g2 + f 2 - c )
= 5 is
If PC < r, then P inside, and if PC > r, then P outside.
( x - 3) 2 + (y ± 5) 2 = 25
A P(x1, y1)
Now, from the options (3, 10) satisfy the equation of circle P
( x - 3) 2 + (y - 5) 2 = 25
,–f )
C (–g
B
Example 4. The circle passing through (1, - 2) and touching
the X-axis at (3, 0), also passes through the point Example 5. The least and the greatest distances of the
(JEE Mains 2013) point (10, 7) from the circle x 2 + y 2 - 4x - 2y = 20 = 0 are
(a) ( - 5, 2) (b) (2, - 5) (c) (5, - 2) (d) ( - 2, 5)
(a) 10, 5 (b) 15, 20 (c) 12, 16 (d) 5, 15
Sol. (c) Let the equation of circle be
Sol. (d) Given equation of circle is
( x - 3) 2 + (y - 0) 2 + ly = 0
x2 + y 2 - 4x - 2y - 20 = 0 …(i)
Y
Let S = x2 + y 2 - 4x - 2y - 20 = 0
Put ( x, y) = (10 , 7) in Eq. (i), we get
A (3, 0)
X¢ X 10 2 + 7 2 - 4 ´ 10 - 2 ´ 7 - 20 = 75 > 0
P
(1, –2) So, the point P (10, 7) lies outside the circle.

C (2, 1) P (10, 7)
As, it passes through (1, - 2). B
\ (1 - 3) 2 + ( - 2) 2 + l( - 2) = 0
Þ 4 + 4 - 2l = 0
Also, centre of given circle is C (2, 1) and r = 5.
Þ l=4
\ Equation of circle is \ PC = (10 - 2) 2 + (7 - 1) 2 = 10
( x - 3) 2 + y 2 + 4y = 0 and BC = 5 [QB is a point on circle and line PC]
By hit and trial method, we see that, point (5, – 2) satisfies \ Greatest distance = 10 + 5 = 15
equation of circle. Least distance = 10 - 5 = 5
Circles 321

Intersection of a Line and a Circle Example 6. The circle x 2 + y 2 = 4x + 8y = 5 intersects


the line 3x - 4y = m at two distinct points, if
Let y = mx + c and x 2 + y 2 = r 2 be the equation of a line and
(a) - 85 < m < - 35 (b) - 35 < m < 15
a circle, respectively.
(c) 15 < m < 65 (d) 35 < m < 85
\ x 2 + ( mx + c) 2 = r 2 [Q y = mx + c]
Þ (1 + m 2 )x 2 + 2mcx + ( c 2 - r 2 ) = 0 …(i) Sol. (b) Given, centre of circle is (2, 4) and radius is 5.
The line will intersect the circle at two distinct points,
Which is quadratic in x, then three cases arises. if the distance of (2, 4) from 3x - 4y = m is less than radius
Case I Roots of Eq. (i) are real and distinct, of the circle.
if y = mx + c
|6 - 16 - m|
i.e. <5
D = r 2(1 + m 2 ) - c 2 > 0 B 5
c Þ - 25 < 10 + m < 25
i.e., if r> A \ - 35 < m < 15
1+ m2
Hence, the line meets the circle at two Example 7. If the equation of tangent to the circle
distinct points. x 2 + y 2 - 2 x + 6y - 6 = 0 and parallel to 3x - 4y + 7 = 0 is
y = mx + c
Case II Roots of Eq. (i) are coincident, A 3x - 4y + k = 0, then the value of k are
if (a) 5, - 35 (b) - 5, 35 (c) 7, - 32 (d) - 7, 32
D = r 2 (1 + m 2 ) - c 2 = 0, Sol. (a) The centre and radius of given circle are (1, - 3) and 4,
c respectively.
i.e., if r=
1 + m2 So, the length of perpendicualr from centre (1, - 3) to
c
x+

3x - 4y + k = 0 is equal to radius 4.
m

Hence, the line touches the circle.


y=

|3 + 12 + k|
Case III Roots of Eq. (i) are imaginary, Þ =4
9 + 16
if
D = r 2(1 + m 2 ) - c 2 < 0, Þ 15 + k = ± 20
c Þ k = 5, - 35
i.e., if r<
1 + m2
Hence, the line will not intersect the circle at all.
Tangent to a Circle
A straight line is a tangent to a circle, if the distance of
c the centre from the line equals the radius.
Note Length of perpendicular from (0, 0) on y = mx + c is
1 + m2 T
)
y1

Condition of Tangency
1,
x
P(

(i) The line y = mx + c touches the circle x 2 + y 2 = r 2, iff C(0, 0)

c = ± r 1 + m2
(ii) The line lx + my + n = 0 will touch the circle
x 2 + y 2 + 2gx + 2 fy + c = 0, iff Different Forms of the Equation
( l 2 + m 2 )( g2 + f 2 - c) = ( lg + mf - n )2 of Tangents
(iii) If y = mx + c is the tangent to the circle x 2 + y 2 = r 2, Point Form
æ mr 2 r 2 ö The equation of tangent at the point P ( x1 , y1 ) to the
then the coordinates of point of contact are ç - , ÷.
ç c c ÷ø
è circle x 2 + y 2 = r 2 is
(iv) If the line ax + by + c = 0 is the tangent to the circle T º xx1 + yy1 = r 2
x 2 + y 2 = r 2, then the coordinates of point of contact
æ ar 2 Also, the equation of tangent to the circle
br 2 ö÷
are ç - ,- . x 2 + y 2 + 2gx + 2 fy + c = 0 at the point P ( x1 , y1 ) is
ç c c ÷ø
è
(v) The length of the intercept cut off from the line T º xx1 + yy1 + g( x + x1 ) + f ( y + y1 ) + c = 0
y = mx + c by the circle x 2 + y 2 = a 2 is Note When we convert a cartesian equation into a tangent
2 2 2 x + x1 y + y1
a (1 + m ) - c equation, we replace. x2 by xx1, y2 by yy1, x by , y by
2 2 2
1 + m2
322 JEE Main Mathematics

Slope Form Sol. (a) Given, y = mx - b 1 + m2 touches both the circles.


The equation of a tangent of slope m to the circle \ Distance from centre = Radius of both the circle
2 2 2
x + y = r is T º y = mx ± r (1 + m ) and the 2 | ma - 0 - b 1 + m2 |
Þ =b
coordinates of the point of contact are m2 + 1
æ rm r ö | - b 1 + m2 |
ç± ,-
+ ÷. and =b
ç 1+ m 2 2 ÷
1+ m ø m2 + 1
è
Þ | ma - b 1 + m2 | = | - b 1 + m2 |
Parametric Form
The equation of tangent to the circle x 2 + y 2 = r 2 at the Þ m2a2 - 2abm 1 + m2 + b 2(1 + m2) = b 2(1 + m2)
point (r cos q , r sin q ) is T º x cos q + y sin q = r Þ ma - 2b 1 + m2 = 0 Þ m2a2 = 4b 2(1 + m2)
T = 0 is Possible Only 2b
Þ m=
(i) if points is on the curve ® Tangent a - 4b2
2

(ii) if point is outside ® Chord of contact


(iii) if point is outside/inside ® Pole and polar Normal to a Circle
The normal to a circle at any point is a straight line
Example 8. If a line, y = mx + c is a tangent to the circle, which is perpendicular to the tangent at that point and
( x - 3) 2 + y 2 = 1 and it is perpendicular to a line L1, where L1 is always passes through the centre of the circle.
æ 1 1 ö T
the tangent to the circle, x 2 + y 2 = 1 at the point ç , ÷; , y 1)
P(x 1
è 2 2ø
then (JEE Main 2020)
(a) c2 + 7c + 6 = 0 (b) c2 - 6c + 7 = 0 C(0, 0)
2 2
(c) c - 7c + 6 = 0 (d) c + 6c + 7 = 0

Sol. (d) Since equation of tangent to the circle x2 + y 2 = 1 at point


Different Forms of the Equation
æ 1 1 ö
ç , ÷ is T = 0. of Normals
è 2 2ø
x y Point Form
Þ + =1
2 2 The equation of normal at the point P ( x1 , y1 ) to the circle
Þ x+y = 2 …(i) x 2 + y 2 = r 2 is
Q Line L1 : x + y = 2 is perpendicular to line y = mx + c, so x y
=
m = 1, since line y = mx + c is tangent to the circle x1 y1
( x - 3) 2 + y 2 = 1, where m = 1, then y = x + c is equivalent to OR
y = 1( x - 3) ± 1 1 + 12. The equation of normal at the point P ( x1 , y1 ) to the circle
So, c = -3 ± 2 x 2 + y 2 + 2gx + 2 fy + c = 0 is
Þ c+3=± 2 x - x1 y - y1
=
Þ ( c + 3) 2 = 2 x1 + g y1 + f
Þ c2 + 6 c + 9 = 2 Slope Form
Þ c2 + 6 c + 7 = 0 The equation of a normal of slope m to the circle
Hence, option (d) is correct. x 2 + y 2 = r 2 is

Example 9. If a > 2 b > 0, then positive value of m for my = - x ± r 1 + m 2

which y = mx - b 1 + m 2 is a common tangent to x 2 + y 2 = b 2 Parametric Form


2 2
and ( x - a) + y = b , is 2 The equation of normal at the point P (r cos q , r sin q ) to
the circle x 2 + y 2 = r 2 is
2b a2 - 4 b 2
(a) (b) x y
a2 - 4 b 2 2b = Þ y = x tan q
r cos q r sin q
2b b
(c) (d) Note Normal always passes through the centre of the circle.
a-2b a-2b
Circles 323

Example 10. The equation of normal to the circle Sol. (a) Any line through the point ( -5, - 4) is
2 2 y + 4 = m( x + 5)
x + y - 5 x + 2y - 48 = 0 at the point (5, 6) is
(a) 14x - 5y + 40 = 0 (b) 14x + 5y + 40 = 0 Þ mx - y + (5m - 4) = 0 ...(i)
(c) 14x - 5y - 40 = 0 (d) 14x + 5y - 40 = 0 If it is a tangent, then perpendicular from centre ( -2, - 3) is
equal to radius.
Sol. (c) Equation of the normal to the given circle at (5, 6) is
\ Radius = (2) 2 + (3) 2 - 8 = 4 + 9 - 8 = 5
x-5 y -6
= m( -2) - ( -3) + (5m - 4)
5 6 +1 Þ = 5
5-
2 m2 + 1
x-5 y -6
Þ = Þ -2m + 3 + 5m - 4 = 5 m2 + 1
5 7
2 Þ (3m - 1) 2 = 5(m2 + 1)
2x - 10 y - 6 Þ 9m2 + 1 - 6m = 5m2 + 5
Þ =
5 7 Þ 4m2 - 6m - 4 = 0
Þ 14x - 70 = 5y - 30
Þ 2m2 - 3m - 2 = 0
Þ 14x - 5y - 40 = 0 1
Þ m=- ,2
2
Equation of Pair of Tangents \ Required equations are
Let P ( x1 , y1 ) be any point lying outside the circle 2x - y + 6 = 0 and x + 2y + 13 = 0.
x 2 + y 2 + 2gx + 2 fy + c = 0, then the equation of pair of
Example 12. Length of the tangents from the point (1, 2) to
tangents represented by PA and PB is SS1 = T 2
the circles x 2 + y 2 + x + y - 4 = 0 and 3x 2 + 3y 2 - x - y - k = 0
P(x1, y1)
A are in the ratio 4 : 3, then k is equal to
(a) 37/2 (b) 4/37
(c) 12 (d) 39/4

B Sol. (d) The length of tangent from the point (1, 2) to the circle
x2 + y 2 + x + y - 4 = 0 is 1 + 4 + 1 + 2 - 4 , i.e. 2
where, S = x 2 + y 2 + 2gx + 2 fy + c
and the length of tangent from the point (1, 2) to the circle
S1 = x12 + y12 + 2gx1 + 2 fy1 + c
3x2 + 3y 2 - x - y - k = 0 is 3 + 12 - 1 - 2 - k i.e. 12 - k.
and T = xx1 + yy1 + g( x + x1 ) + f ( y + y1 ) + c
2 4
\ =
Similarly, equation of pair of tangents at the point ( x1 , y1 ) 12 - k 3
to the circle x 2 + y 2 = r 2 is 3
Þ = 12 - k
SS1 = T 2 2
9
where, S = x2 + y2 - r 2 Þ = 12 - k
4
S1 = x12 + y12 - r 2 39
Þ k=
4
and T = xx1 + yy1 - r 2

Length of the Tangent Director Circle


The length of the tangent drawn from the point ( x1 , y1 ) to The locus of the point of intersection of two perpendicular
tangents to a given circle is known as director circle.
the circle x 2 + y 2 + 2gx + 2 fy + c = 0 is
If the equation of circle is x 2 + y 2 = a 2 , then the equation
x12 + y12 + 2gx1 + 2 fy1 + c; S1 of the director circle to this circle is x 2 + y 2 = 2a 2
P
Example 11. The equation of the two tangents from
( - 5, - 4) to the circle x 2 + y 2 + 4 x + 6y + 8 = 0 are A
a
O B
(a) x + 2y + 13 = 0 ,2 x - y + 6 = 0
2a
(b) 2 x + y + 13 = 0 , x - 2y = 6
Circle
(c) 3 x + 2y + 23 = 0 ,2 x - 3y + 4 = 0
rc l
e

Dire
(d) x - 7y = 23,6 x + 13y = 4 ctor c i
324 JEE Main Mathematics

Chord of Contact of Tangents Equation of the Chord Bisected


From any external point, two tangents can be drawn at a Given Point
to a given circle. The chord joining the points of contact Let P ( x1 , y1 ) be the mid-point of chord AB of the circle
of the two tangents is called the chord of contact of x 2 + y 2 + 2gx + 2 fy + c = 0, then the equation of the chord
tangents. of circle bisected at the point P ( x1 , y1 ) is
P(x1, y1)
B T = S1
ac f where, T = xx1 + yy1 + g( x + x1 ) + f ( y + y1 ) + c
nt o
t
S1 = x12 + y12 + 2gx1 + 2 fy1 + c
co hord

and
C

A
B

The equation of the chord of contact of tangents drawn


from the point ( x1 , y1 ) to the circle x 2 + y 2 = r 2 is P(x1, y1)

xx1 + yy1 = r 2 and to the circle


A
x 2 + y 2 + 2gx + 2 fy + c = 0 is
xx1 + yy1 + g( x + x1 ) + f ( y + y1 ) + c = 0 Similarly, for the circle x 2 + y 2 = r 2 , equation of chord is
Example 13. Let the tangents drawn from the origin to the T = S1
circle, x 2 + y 2 - 8 x - 4y + 16 = 0 touch it at the points A and B.
where, T = xx1 + yy1 - r 2 and S1 = x12 + y12 - r 2
2
The ( AB) is equal to (JEE Main 2020)
56 52 Example 14. The middle point of the chord intercepted on
(a) (b)
5 5 the line lx + my + n = 0 by the circle x 2 + y 2 = a 2 is
64 32 æ
(c) (d) nl nm ö æ nl nm ö
5 5 (a) çç - 2 ,- 2 ÷ (b) çç 2 , 2 ÷
è l +m
2
l + m2 ÷ø 2
èl + m l + m ø

Sol. (c) The equation of chord of contact AB to circle æ l m ö


x2 + y 2 - 8x - 4y + 16 = 0 w.r.t. point origin (0 , 0) is T = 0 (c) çç - 2 2
, 2 ÷

(d) None of these
è l +m l +m ø
A
Sol. (a) Let ( x1, y1) be the middle point of the chord intercepted by
2
the circle x2 + y 2 = a2 on the line lx + my + n = 0.
C (4, 2) M F (0, 0)
2 Then, equation of the chord of the circle x2 + y 2 = a2,
whose middle point is ( x1, y1), is
B
xx1 + yy1 - a2 = x12 + y12 - a2
x +y2–8x–4y+16=0
2
Þ xx1 + yy1 = x12 + y12 …(i)
Þ 0 x + 0y - 4( x + 0) - 2(y + 0) + 16 = 0
Clearly, lx + my + n = 0 and Eq. (i) represent the same line.
Þ 2x + y - 8 = 0 …(i)
x1 y1 x2 + y12
In right angled triangle AMC, \ = =- 1 =l (say)
l m n
|2( 4) + 2 - 8| 2
CM = = x1 = ll ü
2 2 + 12 5 Þ ý …(ii)
y1 = ml þ
Q AM 2 = CA2 - CM 2
and x12 + y12 = - nl
2
æ 2 ö
= 22 - ç ÷ Þ l 2l2 + m2l2 = - nl [from Eq. (ii)]
è 5ø
n
4 16 \ l=-
=4- = l 2 + m2
5 5
4 \ From Eq. (ii), we get
Þ AM =
5 nl mn
x1 = - ,y1 = - 2
Q AB = (2AM) 2
2 l 2 + m2 l + m2
2 æ
æ2 ´ 4ö 64 nl nm ö
=ç ÷ = Hence, the required point is çç - 2 ,- 2 ÷.
è 5 ø 5 è l +m
2
l + m2 ÷ø
Circles 325

Family of Circles Example 15. The circle passing through the intersection of
the circles, x 2 + y 2 - 6 x = 0 and x 2 + y 2 - 4y = 0, having its
(i) Family of circles passing through the point of
intersection of line P º lx + my + n = 0 and circle centre on the line, 2 x - 3y + 12 = 0, also passes through the
S º x 2 + y 2 + 2gx + 2 fy + c = 0 is S + lP = 0 point (JEE Main 2020)
(a) ( -1, 3) (b) ( -3, 1) (c) (1, - 3) (d) ( -3, 6)

S + Pl = 0
Sol. (d) Equation of circle passing through the intersection of the
circles x2 + y 2 - 6x = 0 and x2 + y 2 - 4y = 0 , is
S=0 ( x2 + y 2 - 6x) + l( x2 + y 2 - 4y) = 0 , ( l ¹ -1)
P=0 Þ (1 + l) x2 + (1 + l)y 2 - 6x - 4ly = 0 , having centre
(ii) Family of circles passing through the points of æ 3 2l ö
Cç , ÷ and the centre ‘C ’ lies on the lies on the line
intersection of two given circles è 1 + l 1 + lø
S1 º x 2 + y 2 + 2g1x + 2 f1 y + c1 = 0 2x - 3y + 12 = 0 , so
2 2 6 6l
and S 2 º x + y + 2g2x + 2 f2 y + c2 = 0 - + 12 = 0
1+ l 1+ l
is S1 + lS 2 = 0 Þ 1- l + 2l + 2 = 0
Þ l = -3.
S1 + lS2 = 0

So, equation of required circle is


2x2 + 2y 2 + 6x - 12y = 0
S1 Þ x2 + y 2 + 3x - 6y = 0 …(i)
=
0 S2 = 0 From the given options circle (i) passes through the point
( -3,6).
(iii) Equation of family of circles passing through two
points ( x1 , y1 ) and ( x2 , y2 ) is Example 16. If the circles x 2 + y 2 + 5Kx + 2y + K = 0 and
2 ( x 2 + y 2) + 2Kx + 3y -1 = 0, (K Î R), intersect at the points P
P (x1, y1) and Q, then the line 4x + 5y - K = 0 passes through P and Q,
for (JEE Main 2019)

Q (x2, y2) (a) no values of K (b) exactly one value of K


(c) exactly two values of K (d) infinitely many values of K

( x - x1 )( x - x2 ) + ( y - y1 )( y - y2 ) Sol. (a) Equation of given circles


x y 1 x2 + y 2 + 5Kx + 2y + K = 0 …(i)

+ l x1 y1 1 = 0 and 2( x2 + y 2) + 2Kx + 3y - 1 = 0
3 1
x2 y2 1 Þ x2 + y 2 + Kx + y - = 0 …(ii)
2 2
(iv) The equation of the family of circles touching the On subtracting Eq. (ii) from Eq. (i), we get
circle 1 1
4Kx + y + K + = 0
2 2
S=0 T=0
Tangent Þ 8Kx + y + (2K + 1) = 0 …(iii)
[Q if S1 = 0 and S 2 = 0 be two circles, then their
P common chord is given by S1 - S 2 = 0 .]
(x1, y1) Eq. (iii) represents equation of common chord as it is given
that circles (i) and (ii) intersects each other at points P and Q.
Since, line 4x + 5y - K = 0 passes through point P and Q.
S º x 2 + y 2 + 2gx + 2 fy + c = 0 8K 1 2K + 1 1
\ = = ÞK =
4 5 -K 10
at point P ( x1 , y1 ) is x 2 + y 2 + 2gx + 2 fy [equating first and second terms]
+ c + l{xx1 + yy1 + g( x + x1 ) + f ( y + y1 ) + c} = 0 and -K = 10K + 5 [equating second and third terms]
or 5
Þ 11K + 5 = 0 Þ K = -
S + lT = 0 11
1 5
where, T = 0 is the equation of the tangent to S = 0 Q ¹ - , so there is no such value of K, for which line
10 11
at ( x1 , y1 ) and l Î R.
4x + 5y - K = 0 passes through points P and Q.
326 JEE Main Mathematics

Concentric Circles Mathematically, |C1C2| = r1 + r2


Two circles having the same centre but different radii Direct common tengents
r1 and r2 respectively, are called concentric circles. Thus, D
the circles x 2 + y 2 + 2gx + 2 fy + c = 0 and
x 2 + y 2 + 2gx + 2 fy + m = 0 are concentric circles.
r2
Therefore, the equations of concentric circles differ only in r1 P C2
constant term. C1
T Transverse common
tangent
Example 17. If (–3, 2) lies on the circle
x 2 + y 2 + 2 gx + 2 fy + c = 0 which is concentric with the circle
x 2 + y 2 + 6 x + 8y - 5 = 0, then c is equal to In such a case, there are 3 common tangents to two
(a) 11 (b) –11 (c) 24 (d) 100 touching circles.

Sol. (b) Equation of family of concentric circles to the circle Circles Intersecting Each Other
x2 + y 2 + 6x + 8y - 5 = 0 If the distance between centres is less than the sum of
2 2 their radii, then geometrically, the two circles intersect
x + y + 6x + 8y + l = 0
each other.
which is similar to x2 + y 2 + 2gx + 2fy + c = 0
Mathematically,|C1C2| < r1 + r2
Thus, the point (– 3, 2) lies on the circle
Direct common tangents
x2 + y 2 + 6x + 8y + c = 0
2 2
\ ( - 3) + (2) + 6( - 3) + 8(2) + c = 0
Þ 9 + 4 - 18 + 16 + c = 0 \ c = - 11
C1
Common Tangents to Two Circles C2

Circles Neither Touching Nor Intersecting


If the distance between the centres is more than the sum In such a case, there are two direct common tangents.
of their radii, then geometrically the two circles neither
touch nor intersect each other. Circles Touching Each Other Internally
Mathematically,|C1C2| > r1 + r2 If the distance between centres equals the difference of
Direct common tangents the radii, then geometrically, the two circles touch each
D other internally. The point of contact P divides the line
Q joining the centres externally in the ratio of their radii
r2 C1P r1
C i.e. =
P P' T r2 2 C2P r2
r1 Q1
r1 Mathematically, |C1C2| =|r1 - r2|
C1 Tangent at the
point of contact
Transverse common tangents r2 P

In such a case there are two direct common tangents and C2


two transverse common tangents. C1
r1

Circles Touching Each Other Externally


If the distance between the centres equals the sum of
their radii, then geometrically the two circles touch each In such a case, there is one common tangent to two circles.
other externally. The point P where the two circles touch
each other is called the point of contact. P divides the line One Circle Lies Inside Another
joining centres internally in the ratio of their radii i.e., If distance between centres is less than the difference of
C1P r1 the radii, then geometrically, one circle lies inside the
= another.
C2P r2
Circles 327

Mathematically, |C1C2| <|r1 - r2| Sol. (c) x2 + y 2 - 10 x - 10y + 41 = 0


A (5, 5), R1 = 3
r1 x2 + y 2 - 22x - 10y + 137 = 0
C1 r2 B(11, 5), R2 = 3
C2 AB = 6 = R1 + R2
Touch each other externally
Þ Circles have only one meeting point.
In such a case, there is no common tangent.
Note Length of an external common tangent and internal common Angle of Intersection of Two Circles
tangent to two circles is given by length of external common tangent The angle of intersection of two intersecting circles is the
Lex = d 2 - (r1 - r2 )2 angle between their tangents at the point of intersection,
and length of internal common tangent. ½r 2 + r22 - d 2½
cos a =½ 1 ½
r1 – r2 ½ 2r1r2 ½
A Lex where, d is distance between centres of the circles.
B
r1 B¢ R
r2
C1 d r1 r2
C2
P a Q
L in
r1
r1 +

q d
L in T T¢
r2

r2

A'
2 2
Lin = d - (r1 + r2 ) , where d is the common distance between Condition of Orthogonality
centres of two circles and r1 and r2 are the radii of two circles, where If the angle between the circles is 90°, then the circles are
|C1C 2|= d . said to be orthogonal circle.
Example 18. The common tangent to the circles Let S1 = 0 and S 2 = 0 be any two circles, then condition of
x 2 + y 2 = 4 and x 2 + y 2 + 6 x + 8y - 24 = 0 also passes through orthogonality is 2 ( g1g2 + f1 f2 ) = c1 + c2
the point (JEE Main 2019)
Example 20. The angle between the circles
(a) (6, - 2) (b) ( 4, - 2) (c) ( -6, 4) (d) ( -4, 6)
S : x2 + y 2 - 4x + 6y + 11 = 0
Sol. (a) Given circles are x2 + y 2 = 4, centreC1(0 , 0) and radiusr1 = 2 2 2
and S ¢ : x + y - 2x + 8y + 13 = 0 is
and x2 + y 2 + 6x + 8y - 24 = 0 , centre C 2( -3, - 4) and radius
(a) 45° (b) 90°
r2 = 7
(c) 60° (d) None of these
Q C1C 2 = 9 + 16 = 5 and|r1 - r2| = 5
Q C1C 2 = |r1 - r2| = 5 Sol. (a) Centres and radii of circles S and S ¢ are C1(2 , - 3),
\ circle x2 + y 2 = 4 touches the circle r1 = 2; C 2( 1, - 4),r2 = 2 .
x2 + y 2 + 6x + 8y - 24 = 0 internally. Distance between centres,
So, equation of common tangent is d =|C1C 2 | = (2 - 1) 2 + ( -3 + 4) 2 = 2
S1 - S 2 = 0 If angle between the circles is q, then
Þ 6x + 8y - 20 = 0 ½2 + 4 - 2½
½ 1
cos q = ½ =
Þ 3x + 4y = 10 …(i) ½ 2 2 ×2 ½ 2
The common tangent passes through the point (6, - 2), from \ q = 45°
the given options.
Example 21. Let C1 and C 2 be the centres of the circles
Example 19. Choose the correct statement about two x 2 + y 2 - 2 x - 2y - 2 = 0 and x 2 + y 2 - 6 x - 6y + 14 = 0
circles whose equations are given below respectively. If P and Q are the points of intersection of these
x 2 + y 2 - 10 x - 10y + 41 = 0 circles, then the area (in sq units) of the quadrilateral PC1QC 2
x 2 + y 2 - 22 x - 10y + 137 = 0 is (JEE Main 2019)
(JEE Main 2021)
(a) 8 (b) 4 (c) 6 (d) 9
(a) circles have same centre
2 2
(b) circles have no meeting point Sol. (b) Given circles, x + y - 2x - 2y - 2 = 0 … (i)
2 2
(c) circles have only one meeting point and x + y - 6x - 6y + 14 = 0 … (ii)
(d) circles have two meeting point
328 JEE Main Mathematics

are intersecting each other orthogonally, Length of Common Chord


because 2(1)(3) + 2(1)(3) = 14 - 2
Length of common chord = PQ = 2( PM )
[Q two circles are intersected
orthogonally if 2g 1g 2 + 2f1f2 = c1 + c2] = 2 {(C1P )2 - (C1M )2 } where,
So, area of quadrilateral C1P = radius of circle S1 = 0 and C1M = length of
PC1QC 2 = 2 ´ ar (DPC1C 2) perpendicular from C1 on common chord PQ.
æ1 ö
= 2 ´ ç ´ 2 ´ 2÷ = 4 sq units Example 22. The length of the common chord of the two
è2 ø
P circles ( x - a) 2 + (y - b) 2 = c 2 and ( x - b) 2 + (y - a) 2 = c 2 is

2 2 (a) 3c2 + ( a - b) 2 (b) 4c2 - 2 ( a - b) 2


C1 (3,3)
C2 (c) c2 + ( a - b) 2 (d) 2c2 + ( a + b) 2
(1,1)
Sol. (b) The equation of circles are
Q S1 = ( x - a) 2 + (y - b) 2 - c2 = 0 …(i)
and S 2 = ( x - b) 2 + (y - b) 2 - c2 = 0 …(ii)
Common Chord of Two Circles P
The common chord joining the point of intersection of two
given circles is called their common chord. The equation
of the common chord of two circles C1(a, b) m C2(a, b)
2 2
S1 º x + y + 2g1x + 2 f1 y + c1 = 0
Q
and S 2 º x 2 + y 2 + 2g2x + 2 f2 y + c2 = 0 Then, equation of common chord is S1 - S 2 = 0
is 2 ( g1 - g2 )x + 2 ( f1 - f2 ) y + c1 - c2 = 0 Þ ( x - a) 2 - ( x - b) 2 + (y - b) 2 - (y - a) 2 = 0
i.e., S1 - S 2 = 0 Þ (2x - a - b) ( - a + b) + (2y - b - a) ( - b + a) = 0
Þ 2x - a - b - 2y + b + a = 0
Þ x-y =0
P Now, C1m = Length of perpendicular from C1( a, b) on
M | a - b|
C1 C2 PQ( x - y = 0) = and C1P = Radius of circle (i) = c
Q 2
S1 = 0
( a - b) 2
S2 = 0 \ In DPC1m, Pm = (PC1) 2 - (C1m) 2 = c2 -
S1 – S2 = 0 2
2 ( a - b) 2
\ PQ = 2Pm = 2 c - = 4c - 2 ( a - b) 2
2
2
Practice Exercise
ROUND I Topically Divided Problems

Different Form of Equation of Circle (a) x2 + y2 - 4 y + 3 = 0


(b) x2 + y2 - 2 y - 2 = 0
1. If 2 ( x 2 + y 2 ) + 4 lx + l2 = 0 represents a circle of
(c) x2 + y2 - 2 x - 2 y + 2 = 0
meaningful radius, then the range of real values of
(d) x2 + y2 - 2 x - 2 y + 1 = 0
l is
(a) R 8. The diameters of a circle are along 2 x + y - 7 = 0
(b) (0, + ¥ ) and x + 3 y - 11 = 0. Then, the equation of this circle,
(c) (-¥ , 0)
which also passes through (5, 7) , is
(d) None of the above
(a) x2 + y2 - 4x - 6 y - 16 = 0
2. If one end of a diameter of the circle (b) x2 + y2 - 4x - 6 y - 20 = 0
x 2 + y 2 - 4 x - 6 y + 11 = 0 is (3, 4), then find the (c) x2 + y2 - 4x - 6 y - 12 = 0
coordinate of the other end of the diameter. (d) x2 + y2 + 4x + 6 y - 12 = 0
(a) (2, 1) (b) (1, 2) 9. The equation of circle which passes through the
(c) (1, 1) (d) None of these
origin and cuts off intercepts 5 and 6 from the
3. The centre of a circle is (2, – 3) and the positive parts of the axes respectively is
circumference is 10p. Then, the equation of the 2
æ x - 5 ö + ( y - 3) 2 = l, where l is
circle is ç ÷
è 2ø
(a) x2 + y 2 + 4x + 6 y + 12 = 0 61 6 1
(b) x2 + y 2 - 4x + 6 y + 12 = 0 (a) (b) (c) (d) 0
4 4 4
(c) x2+ y 2 - 4x + 6 y - 12 = 0
(d) x2 + y2 - 4x - 6 y - 12 = 0 10. If one of the diameters of the circle, given by the
equation, x 2 + y 2 - 4 x + 6 y - 12 = 0, is a chord of a
4. Find the equation of the circle which passes
circle S, whose centre is at ( -3, 2), then the radius
through the points (2, 3) and (4, 5) and the centre
of S is (JEE Main 2016)
lies on the straight line y - 4 x + 3 = 0.
(a) 5 2 (b) 5 3 (c) 5 (d) 10
(a) x2 + y2 - 4x - 10 y + 25 = 0
(b) x2 + y2 - 4x - 10 y - 25 = 0 11. If the length of the chord of the circle,
(c) x2 + y2 - 4x + 10 y - 25 = 0 x 2 + y 2 = r 2 ( r > 0) along the line, y - 2 x = 3 is r,
(d) None of the above then r 2 is equal to (JEE Main 2020)
5. The circle passing through (1, - 2) and touching the 9 24 12
(a) (b) 12 (c) (d)
axis of x at (3, 0) also passes through the point 5 5 5
(JEE Main 2013)
12. The locus of the mid-point of the chord of the circle
(a) (- 5, 2) (b) (2, - 5)
x 2 + y 2 - 2 x - 2 y - 2 = 0, which makes an angle of
(c) (5, - 2) (d) (- 2, 5)
120° at the centre, is
6. The sum of the minimum distance and the (a) x 2 + y 2 - 2x - 2 y + 1 = 0
maximum distance from the point ( 4, - 3) to the (b) x 2 + y2 + x + y -1 =0
circle x 2 + y 2 + 4 x - 10 y - 7 = 0 is (c) x 2 + y 2 - 2x - 2 y - 1 = 0
(a) 20 (b) 12 (d) None of the above
(c) 10 (d) 16
13. If the area of the circle 4 x 2 + 4 y 2 - 8 x + 16 y + k = 0
7. The equation of circle which touches Xand Y - axes is 9p sq units, then the value of k is
at the points (1, 0) and (0, 1) respectively is (a) 4 (b) 16 (c) - 16 (d) ± 16
330 JEE Main Mathematics

æ 1 ö x y 1 1 1
14. If çç mi , ÷, i = 1, 2 , 3, 4 are concyclic points, then 22. If the line + = 1 moves such that 2 + 2 = 2 ,
è mi ÷ø a b a b c
where c is a constant, then the locus of the foot of
the value of m1 m2 m3 m4 is
the perpendicular from the origin to the line is
(a) 1 (b) - 1
(a) straight line (b) circle
(c) 0 (d) None of these
(c) parabola (d) ellipse
15. If the abscissae and ordinates of two points P and Q
23. If the straight line y = mx lies outside the circle
are roots of the equations x 2 + 2 ax - b2 = 0 and
x 2 + y 2 - 20 y + 90 = 0, then the value of m will
y 2 + 2 py - q2 = 0 respectively, then the equation of satisfy
the circle with PQ as diameter, is (a) m < 3 (b)|m| < 3 (c) m > 3 (d)|m| > 3
(a) x2 + y2 + 2ax + 2 py - b2 - q2 = 0
(b) x2 + y2 - 2ax - 2 py + b2 + q2 = 0 24. If the line y = 7 x - 25 meets the circle x 2 + y 2 = 25
(c) x2 + y2 - 2ax - 2 py - b2 - q2 = 0 in the points A, B, then the distance between A and
(d) x2 + y2 + 2ax + 2 py + b2 + q2 = 0 B is
(a) 10 (b) 10 (c) 5 2 (d) 5
16. The range of a, for which the point ( a, a) lies inside
25. If the line y cos a = x sin a + a cos a is a tangent to
the region bounded by the curves y = 1 - x 2 and
the circle x 2 + y 2 = a 2 , then
x + y = 1 is (a) sin 2 a = 1 (b) cos 2 a = 1
1 1 1 1 (c) sin 2 a = a 2 (d) cos 2 a = a 2
(a) < a < (b) < a <
2 2 2 3
1 1 1 1 26. Find the equation of a circle which touches both the
(c) < a < (d) < a <
3 3 4 2 axes and the line 3x - 4 y + 8 = 0 and lies in the
third quadrant.
17. Let L1 be a straight line passing through the origin (a) x 2 + y 2 + 4x + 4 y - 4 = 0
and L 2 be the straight line x + y = 1. If the (b) x 2 + y 2 - 4x - 4 y + 4 = 0
intercepts made by the circle x 2 + y 2 - x + 3 y = 0 on (c) x 2 + y 2 + 4x + 4 y + 4 = 0
L1 and L2 are equal, then L1 can be represented by (d) None of the above
(a) x + y = 0 (b) x - y = 0
(c) 7x + y = 0 (d) x - 7 y = 0
Tangent to the Circle
27. The equation of the tangents to the circle
18. Let AB be a chord of the circle x 2 + y 2 = r 2 x 2 + y 2 = 4, which are parallel to x + 2 y + 3 = 0, are
subtending a right angle at the centre. Then, the (a) x - 2 y = 2 (b) x + 2 y = ± 2 3
locus of the centroid of the D PAB as P moves on the (c) x + 2 y = ± 2 5 (d) x - 2 y = ± 2 5
circle is 28. Three circles of radii a, b, c( a < b < c) touch each
(a) a parabola (b) a circle
other externally. If they have X-axis as a common
(c) an ellipse (d) None of these
tangent, then (JEE Main 2019)
Intersection of Line and Circle (a) a , b, c are in AP (b)
1
=
1
+
1
a b c
19. If the line 3x - 4 y - k = 0, ( k > 0) touches the circle 1 1 1
(c) a , b , c are in AP (d) = +
x 2 + y 2 - 4 x - 8 y - 5 = 0 at ( a , b) , then k + a + b is b a c
equal to
29. The equation of the tangent from the point (0, 1) to
(a) 20 (b) 22 (c) -30 (d) - 28
the circle x 2 + y 2 - 2 x - 6 y + 6 = 0, is
20. The line 3x - 2 y = k meets the circle x 2 + y 2 = 4 r 2 at (a) y - 1 = 0 (b) 4x + 3 y + 3 = 0
only one point, if k2 is (c) 4x + 3 y - 3 = 0 (d) y + 1 = 0
52 2 20 2
(a) 20r 2 (b) 52 r 2 (c) r (d) r 30. The tangent at (1, 7) to the curve x 2 = y - 6 touches
9 9
the circle x 2 + y 2 + 16 x + 12 y + c = 0 at
2 2
21. A line through (0, 0) cuts the circle x + y - 2 ax = 0 (a) (6, 7) (b) (- 6, 7)
at A and B, then locus of the centre of the circle (c) (6, - 7) (d) (- 6, - 7)
drawn AB as diameter is 31. The square of the length of the tangent from (3, - 4)
(a) x2 + y2 - 2ay = 0 (b) x2 + y2 + ay = 0 to the circle x 2 + y 2 - 4 x - 6 y + 3 = 0, is
2 2 2 2
(c) x + y + ax = 0 (d) x + y - ax = 0 (a) 20 (b) 30 (c) 40 (d) 50
Circles 331

32. From a point on the circle x 2 + y 2 = a 2 , two (c) x2 + y2 - 2x + 2 y + 12 = 0


(d) x2 + y2 - 2x - 2 y + 14 = 0
tangents are drawn to the circle x 2 + y 2 = a 2 sin 2 a.
The angle between them is 40. If line ax + by = 0 touches x 2 + y 2 + 2 x + 4 y = 0 and
a is a normal to the circle x 2 + y 2 - 4 x + 2 y - 3 = 0,
(a) a (b)
2 then value of ( a, b) will be
(c) 2 a (d) None of these (a) (2,1) (b) (1, –2) (c) (1, 2) (d) (–1, 2)

33. The locus of a point which moves so that the ratio 41. The area of the triangle formed by the positive
of the length of the tangents to the circles X-axis and the normal and tangent to the circle
x 2 + y 2 + 4 x + 3 = 0 and x 2 + y 2 - 6 x + 5 = 0 is 2 : 3, is x 2 + y 2 = 4 at (1, 3) is
(a) 5x 2 + 5 y 2 + 60x - 7 = 0 (a) 2 3 sq units (b) 3 2 sq units
(b) 5x 2 + 5 y 2 - 60x - 7 = 0 (c) 6 sq unit (d) None of these
(c) 5x 2 + 5 y 2 + 60x + 7 = 0
(d) 5x 2 + 5 y 2 + 60x + 12 = 0 Chord of Contact and Diameter
34. P ( a, b) be any point such that the length of 42. The condition that the chord
x cos a + y sin a - p = 0 of x 2 + y 2 - a 2 = 0 may
tangents from P to both the circles
subtend a right angle at the centre of circle, is
x 2 + y 2 - 6 x - 8 y = 0 and
(a) a 2 = 2 p 2 (b) p 2 = 2 a 2
x 2 + y 2 - 12 x - 16 y + 12 = 0 are equal, then (c) a = 2 p (d) p = 2 a
(a) 3a + 4b - 6 = 0 (b) 3a - 4b + 6 = 0
(c) 6a - 8b + 12 = 0 (d) 4a - 3b + 7 = 0 43. Which of the following is a point on the common
chord of the circles x 2 + y 2 + 2 x - 3 y + 6 = 0 and
35. If the tangent at the point P on the circle
x 2 + y 2 + x - 8 y - 13 = 0 ?
x 2 + y 2 + 6 x + 6 y = 2 meets the straight line
5 x - 2 y + 6 = 0 at a point Q on the Y -axis, then the (a) (1, - 2) (b) (1, 4) (c) (1, 2) (d) (1, - 4)
length of PQ is 44. If two distinct chords drawn from the point ( p, q) on
(a) 4 (b) 2 5 (c) 5 (d) 3 5 the circle x 2 + y 2 = px + qy (where, pq ¹ 0) are
36. A line meets the coordinate axes in A and B. A bisected by the X-axis, then
circle is circumscribed about the DOAB. The (a) p 2 = q 2 (b) p 2 = 8q 2
distances from the points A and B of the side AB to (c) p 2 < 8q 2 (d) p 2 > 8q 2
the tangent at O are equal to m and n respectively. 45. If the chord of contact of tangents drawn from a
Then, the diameter of the circle is point on the circle x 2 + y 2 = a 2 to the circle
(a) m(m + n ) (b) n (m + n )
x 2 + y 2 = b2 touches the circle x 2 + y 2 = c2 , then
(c) m - n (d) None of these
a , b and c are in
37. If the area of the quadrilateral formed by the (a) AP (b) GP
tangent from the origin to the circle (c) HP (d) None of these
x 2 + y 2 + 6 x - 10 y + c = 0 and the pair of radii at
46. The length of the common chord of the two circles
the points of contact of these tangents to the circle
x 2 + y 2 - 4 y = 0 and x 2 + y 2 - 8 x - 4 y + 11 = 0 is
is 8 sq units, then c is a root of the equation
145 11 135
(a) c 2 - 32c + 64 = 0 (b) c 2 - 34c + 64 = 0 (a) cm (b) cm (c) 135 cm (d) cm
4 2 4
(c) c 2 + 2c - 64 = 0 (d) c 2 + 34c - 64 = 0
47. If the angle of intersection at a point where the two
Normal to the Circle circles with radii 5 cm and 12 cm intersect is 90°,
38. The equation of normal to the circle then the length (in cm) of their common chord is
(JEE Main 2019)
2 x 2 + 2 y 2 - 2 x - 5 y + 3 = 0 at (1, 1) is
13 120 60 13
(a) 2x + y = 3 (b) x - 2 y = 3 (a) (b) (c) (d)
5 13 13 2
(c) x + 2 y = 3 (d) None of these
39. The normal at the point (3, 4) on a circle cuts the Family of Circles
circle at the point (–1, –2). Then, the equation of 48. If the circle x 2 + y 2 + 4 x + 22 y + c = 0 bisects the
the circle is circumference of the circle x 2 + y 2 - 2 x + 8 y - d = 0,
2 2
(a) x + y + 2x - 2 y - 13 = 0 then c + d is equal to
(b) x2 + y2 - 2x - 2 y - 11 = 0 (a) 60 (b) 50 (c) 40 (d) 30
332 JEE Main Mathematics

49. The equation of the circle passing through (1, 1) (a) x2 + y2 + 4x - 6 y + 19 = 0


and the points of intersection of (b) x2 + y2 - 4x - 10 y + 19 = 0
x 2 + y 2 + 13x - 3 y = 0 and (c) x2 + y2 - 2x + 6 y - 29 = 0
(d) x2 + y2 - 6x - 4 y + 19 = 0
2 x 2 + 2 y 2 + 4 x - 7 y - 25 = 0 is
(a) 4x 2 + 4 y 2 - 30x - 10 y = 25 58. If two circles, each of radius 5 units, touch each
(b) 4x 2 + 4 y 2 + 30x - 13 y - 25 = 0 other at (1, 2) and the equation of their common
(c) 4x 2 + 4 y 2 - 17x - 10 y + 25 = 0 tangent is 4 x + 3 y = 10, then equation of the circle a
(d) None of the above portion of which lies in all the quadrants, is
(a) x2+ y 2 - 10x - 10 y + 25 = 0
50. If ( - 3, 2) lies on the circle x 2 + y 2 + 2 gx + 2 fy + c = 0
(b) x2+ y 2 + 6x + 2 y - 15 = 0
which is concentric with the circle
(c) x2+ y 2 + 2x + 6 y - 15 = 0
x 2 + y 2 + 6 x + 8 y - 5 = 0, then c is equal to (d) x2+ y 2 + 10x + 10 y + 25 = 0
(a) 11 (b) - 11 (c) 24 (d) 100
59. C1 and C2 are circles of unit radius with centres at
51. Find the equation of a circle concentric with the
circle x 2 + y 2 - 6 x + 12 y + 15 = 0 and has double of (0, 0) and (1, 0) respectively. C3 is a circle of unit
its area. radius, passes through the centres of the circles
(a) x2 + y2 - 6x + 12 y - 15 = 0 C1 and C2 and have its centre above X-axis.
(b) x2 + y2 - 6x - 12 y + 15 = 0 Equation of the common tangent to C1 and C2 which
(c) x2 + y2 - 6x + 12 y + 15 = 0 does not pass through C2 , is
(d) None of the above (a) x - 3 y + 2 = 0 (b) 3x - y + 2 = 0
52. The line x = y touches a circle at the point (1, 1). (c) 3x - y - 2 = 0 (d) x + 3 y + 2 = 0
If the circle also passes through the point (1, - 3), 60. The locus of centre of a circle
then its radius is (JEE Main 2019)
x 2 + y 2 - 2 x - 2 y + 1 = 0, which rolls outside the
(a) 3 2 (b) 2 2 (c) 2 (d) 3 circle x 2 + y 2 - 6 x + 8 y = 0, is
53. The straight line x + y - 1 = 0 meets the circle (a) x2 + y2 - 2x - 2 y - 34 = 0
2 2
x + y - 6 x - 8 y = 0 at A and B. Then, the (b) x2 + y2 - 6x - 8 y + 11 = 0
equation of the circle of which AB is a diameter, is (c) x2 + y 2 - 6x + 8 y - 11 = 0
(a) x2 + y2 - 2 y - 6 = 0 (b) x2 + y2 + 2 y - 6 = 0 (d) None of the above
(c) 2 (x2 + y2) + 2 y - 6 = 0 (d) 3 (x2 + y2) + 2 y - 6 = 0 61. The locus of the centre of a circle which cuts
54. If a circle C passing through the point (4, 0) touches orthogonally the circle x 2 + y 2 - 20 x + 4 = 0 and
2 2
the circle x + y + 4 x - 6 y = 12 externally at the which touches x = 2 , is
point (1, - 1), then the radius of C is (a) y2 = 16x + 4 (b) x2 = 16 y
(JEE Main 2019) (c) x2 = 16 y + 4 (d) y2 = 16x
(a) 5 (b) 2 5 (c) 57 (d) 4
62. The locus of centres of family of circle passing
55. If P is a point such that the ratio of the squares of through the origin and cutting the circle
the lengths of the tangents from P to the circles
x 2 + y 2 + 4 x - 6 y - 13 = 0 orthogonally, is
x 2 + y 2 + 2 x - 4 y - 20 = 0 and
(a) 4x + 6 y + 13 = 0 (b) 4x - 6 y + 13 = 0
x 2 + y 2 - 4 x + 2 y - 44 = 0 is 2 : 3, then the locus of
(c) 4x + 6 y - 13 = 0 (d) 4x - 6 y - 13 = 0
P is a circle with centre
(a) (7, - 8) (b) (- 7, 8) (c) (7, 8) (d) (- 7, - 8) 63. The locus of the centre of circle which cuts the
56. Equation of the circle passing through the point circles x 2 + y 2 + 4 x - 6 y + 9 = 0 and
(3, 4) and concentric with the circle x 2 + y 2 - 4 x + 6 y + 4 = 0 orthogonally, is
x 2 + y 2 - 2 x - 4 y + 1 = 0 is (a) 12x + 8 y + 5 = 0 (b) 8x + 12 y + 5 = 0
(a) x2 + y 2 - 2 x - 4 y = 0 (c) 8x - 12 y + 5 = 0 (d) None of these
(b) x2 + y 2 - 2 x - 4 y + 3 = 0
(c) x2 + y2 - 2x - 4 y - 3 = 0 64. If the circles x 2 + y 2 + 2 x + 2 ky + 6 = 0 and
(d) None of the above x 2 + y 2 + 2 ky + k = 0 intersect orthogonally, then k is
57. Tangents drawn from the point P (1, 8) to the circle 3 3
(a) 2 or - (b) - 2 or
2 2 2 2
x + y - 6 x - 11 = 0 touch the circle at the points
A and B. The equation of the circumcircle of the 3 3
(c) 2 or (d) - 2 or
D PAB is 2 2
Circles 333

65. The circles x 2 + y 2 + 2 g1x - a 2 = 0 and 3


slope equal to , the coordinates of the centre of S2
4
x 2 + y 2 + 2 g 2 x - a 2 = 0 cut each other orthogonally.
are
If p1 and p2 are perpendiculars from (0, a) and
æ 9 12 ö æ 9 12 ö
(0, - a) on a common tangent of these circles, then (a) ç - , ÷ , ç , - ÷
è 5 5 ø è5 5ø
p1 p2 is equal to æ 9 12 ö æ 9 12 ö
a2 (b) ç - , - ÷ , ç , ÷
(a) (b) a 2 è 5 5 ø è5 5 ø
2 æ 12 9 ö æ 12 9 ö
(c) 2a 2 (d) a 2 + 2 (c) ç , ÷, ç- , ÷
è 5 5ø è 5 5ø
66. If the circle S1 : x 2 + y 2 = 16 intersects another (d) None of the above
circle S1 of radius 5 in such a manner that the 67. Circle x 2 + y 2 - 2 x + lx - 1 = 0 passes through two
common chord is of maximum length and has a fixed points, coordinates of the points are
(a) (0, ± 1)

ROUND II Mixed Bag


Only One Correct Option 6. If a tangent to the circle x 2 + y 2 = 1 intersects the
1. The set of values of c so that the equations coordinate axes at distinct points P and Q, then the
2 2
y =|x |+ c and x + y - 8 |x |- 9 = 0 have no locus of the mid-point of PQ is (JEE Main 2019)

solution, is (a) x2 + y2 - 2x2y2 = 0 (b) x2 + y2 - 2xy = 0


(c) x2 + y2 - 4x2y2 = 0 (d) x2 + y2 - 16x2y2 = 0
(a) (-¥ , - 3) È (3, ¥ ) (b) (-3, 3)
(c) (-¥ , 5 2 ) È (5 2 , ¥ ) (d) (5 2 - 4, ¥ ) 7. The locus of the centres of the circles, which touch
2. Two rods of lengths a and b slide along the X-axis the circle, x 2 + y 2 = 1 externally, also touch the
Y-axis and lie in the first quadrant, is (JEE Main 2019)
and Y -axis respectively in such a manner that their
(a) y = 1 + 2x, x ³ 0 (b) y = 1 + 4x, x ³ 0
ends are concyclic. The locus of the centre of the (c) x = 1 + 2 y, y ³ 0 (d) x = 1 + 4 y, y ³ 0
circle passing through the end points is
(a) 4(x 2 + y 2) = a 2 + b 2 8. The line 2 x - y + 1 = 0 is a tangent to the circle at
(b) x2 + y2 = a 2 + b2 the point (2, 5) and the centre of the circle lies on
(c) 4(x 2 - y 2) = a 2 - b 2 x - 2 y = 4. Then, the radius of the circle is
(JEE Main 2021)
(d) x2 - y2 = a 2 - b2
(a) 3 5 (b) 5 3 (c) 5 4 (d) 4 5
3. The range of values of a such that the angle q
between the pair of tangents drawn from ( a , 0) to
9. A pair of tangents are drawn to a unit circle with
p centre at the origin and these tangents intersect at
the circle x 2 + y 2 = 1 satisfies < q < p, is
2 A enclosing an angle of 60°. The area enclosed by
(a) (1, 2) (b) (1, 2 ) these tangents and the arc of the circle is
2 p p
(c) (- 2 , - 1) (d) (- 2 , - 1) È (1, 2 ) (a) - (b) 3-
3 6 3
4. If OA and OB are equal perpendicular chord of the p 3 æ pö
2 2
(c) - (d) 3 ç1 - ÷
circles x + y - 2 x + 4 y = 0, then equations of 3 6 è 6 ø
OA and OB are (where, O is origin)
10. Let the lengths of intercepts on X-axis and Y -axis
(a) 3x + y = 0 and 3x - y = 0
made by the circle x 2 + y 2 + ax + 2 ay + c = 0 ( a < 0)
(b) 3x + y = 0 and 3 y - x = 0
(c) x + 3 y = 0 and y - 3x = 0 be 2 2 and 2 5, respectively. Then, the shortest
(d) x + y = 0 or x - y = 0 distance from origin to a tangent to this circle
5. Equation of chord of the circle which is perpendicular to the line x + 2 y = 0, is
x 2 + y 2 - 3x - 4 y - 4 = 0, which passes through the equal to (JEE Main 2021)

origin such that the origin divides it in the ratio (a) 11 (b) 7 (c) 6 (d) 10
4 : 1, is 11. Two circles with radii a and b touch each other
(a) x = 0 (b) 24x + y = 0 externally such that q is the angle between the
(c) 7x + 24 y = 0 (d) 7x - 24 y = 0 direct common tangents ( a > b ³ 2), then
334 JEE Main Mathematics

æ a -bö æ a + bö 20. Choose the incorrect statement about the two


(a) q = 2 cos -1 çç ÷÷ (b) q = 2 tan -1 çç ÷÷
è a + bø è a - bø circles whose equations are given below
æ a + bö æ a -bö x 2 + y 2 - 10 x - 10 y + 41 = 0
(c) q = 2 sin -1 çç ÷÷ (d) q = 2 sin -1 çç ÷÷
è a - bø è a + bø and x 2 + y 2 - 16 x - 10 y + 80 = 0 (JEE Main 2021)
12. The equation of a line inclined at an angle p/4 to (a) Distance between two centres is the average of
the X-axis, such that the two circles x 2 + y 2 = 4, radii of both the circles
x 2 + y 2 - 10 x - 14 y + 65 = 0 intercept equal lengths (b) Both circles centres lie inside region of one
another
on it, is
(c) Both circles pass through the centre of each other
(a) 2x - 2 y - 3 = 0 (b) 2x - 2 y + 3 = 0
(d) Circles have two intersection points
(c) x - y + 6 = 0 (d) x - y - 6 = 0
13. The sum of the squares of the lengths of the chords 21. Let C be the circle with centre (0, 0) and radius 3.
2 2
intercepted on the circle, x + y = 16, by the lines, The equation of the locus of the mid-points of the
x + y = n, n Î N , where N is the set of all natural chords of the circle C that subtend an angle 2 p/3 at
numbers, is (JEE Main 2019)
its centre is
27 9
(a) 320 (b) 105 (c) 160 (d) 210 (a) x 2 + y 2 = (b) x 2 + y 2 =
4 4
14. A square is inscribed in the circle 2 2 3
(c) x + y = (d) x 2 + y 2 = 1
2 2
x + y - 6 x + 8 y - 103 = 0 with its sides parallel to 2
the coordinate axes. Then, the distance of the vertex 22. If the lines 3x - 4 y - 7 = 0 and 2 x - 3 y - 5 = 0 are
of this square which is nearest to the origin is two diameters of a circle of area 49p sq units, the
(JEE Main 2019) equation of the circle is
(a) 6 (b) 13 (c) 41 (d) 137 (a) x2 + y 2 + 2x - 2 y - 62 = 0
15. A circle touches the Y -axis at the point (b) x2 + y 2 - 2x + 2 y - 62 = 0
(0, 4) and passes through the point (2, 0). Which of (c) x2 + y 2 - 2x + 2 y - 47 = 0
the following lines is not a tangent to this circle? (d) x2 + y 2 + 2x - 2 y - 47 = 0
(JEE Main 2020)
(a) 4x - 3 y + 17 = 0 (b) 3x + 4 y - 6 = 0 Numerical Value Type Questions
(c) 4x + 3 y - 8 = 0 (d) 3x - 4 y - 24 = 0 23. Let PQ be a diameter of the circle x 2 + y 2 = 9. If a
16. The length of the diameter of the circle which and b are the lengths of the perpendiculars from P
touches the X-axis at the point (1, 0) and passes and Q on the straight line, x + y = 2 respectively,
through the point (2, 3) is (AIEEE 2012) then the maximum value of ab is ……… .
(JEE Main 2020)
10 3 6 5
(a) (b) (c) (d)
3 5 5 3 24. If the curves, x 2 - 6 x + y 2 + 8 = 0 and
x 2 - 8 y + y 2 + 16 - k = 0, ( k > 0) touch each other at
17. The two circles x 2 + y 2 = ax and x 2 + y 2 = c2 , ( c > 0)
a point, then the largest value of k is ........ .
touch each other, if (JEE Main 2020)
(a) |a|= c (b) a = 2c (c) |a|= 2c (d) 2|a|= c
25. A circle passes through the points (0, 0) and (0, 1)
18. If P and Q are the points of intersection of the and also touches the circle x 2 + y 2 = 16. The radius
circles x 2 + y 2 + 3x + 7 y + 2 p - 5 = 0 and of the circle is ............
x 2 + y 2 + 2 x + 2 y - p 2 = 0, then there is a circle
passing through P, Q and (1, 1) and 26. If the length of the tangent from any point on the
(a) all values of p circle ( x - 3) 2 + ( y + 2) 2 = 5 r 2 to the circle
(b) all except one value of p ( x - 3) 2 + ( y + 2) 2 = r 2 is 16 units, then the area
(c) all except two values of p between the two circles (in sq units) is kp, then k is
(d) exactly one value of p equal to ............
19. Consider a family of circles which are passing 27. Let ABCD be a square of side of unit length.
through the point ( - 1, 1) and are tangent to X-axis. Let a circle C1 centered at A with unit radius is
If ( h, k) is the centre of circle, then drawn. Another circle C2 which touches C1 and the
(a) k ³ 1/2 (b) - 1/ 2 £ k £ 1/2
lines AD and AB are tangent to it, is also drawn.
(c) k £ 1/2 (d) 0 < k < 1/2
Let a tangent line from the point C to the circle
Circles 335

C2 meet the side AB at E. If the length of EB is 29. The value of|a|for which the common chord of the
a + 3b, where a, b are integers, then a + b is circles x 2 + y 2 = 8 and ( x - a) 2 + y 2 = 8 subtends a
equal to ............ . right angle at the origin are ............
28. The length of the common chord of two circles of 30. If q be the angle between two tangents which are
k drawn to the circle x 2 + y 2 - 6 3x - 6 y + 27 = 0
radii 3 and 4 unit which intersect orthogonally is ,
5 from the origin, then 2 3 tan q equals ............
then k equals ............

Answers
Round I
1. (a) 2. (b) 3. (c) 4. (a) 5. (c) 6. (a) 7. (b) 8. (c) 9. (a) 10. (b)
11. (d) 12. (a) 13. (c) 14. (a) 15. (a) 16. (a) 17. (b) 18. (b) 19. (a) 20. (b)
21. (d) 22. (b) 23. (b) 24. (c) 25. (b) 26. (c) 27. (c) 28. (b) 29. (c) 30. (d)
31. (c) 32. (c) 33. (c) 34. (a) 35. (c) 36. (d) 37. (b) 38. (c) 39. (b) 40. (c)
41. (a) 42. (a) 43. (d) 44. (d) 45. (b) 46. (d) 47. (b) 48. (b) 49. (b) 50. (b)
51. (a) 52. (b) 53. (a) 54. (a) 55. (b) 56. (c) 57. (b) 58. (b) 59. (b) 60. (c)
61. (d) 62. (d) 63. (c) 64. (a) 65. (b) 66. (a) 67. (a)

Round II
1. (d) 2. (c) 3. (d) 4. (c) 5. (b) 6. (c) 7. (a) 8. (c) 9. (b) 10. (c)
11. (d) 12. (a) 13. (d) 14. (c) 15. (c) 16. (a) 17. (a) 18. (c) 19. (a) 20. (b)
21. (b) 22. (c) 23. (7) 24. (36) 25. (2) 26. (256) 27. (1) 28. (24) 29. (4) 30. (6)

Solutions
Round I The equation of circle, if centre is (2, –3) and radius is 5,
2 2 l2 is
1. Since, x + y + 2lx + = 0 is a circle representing
2 (x - 2)2 + ( y + 3)2 = 52
meaningful radius. Þ x + y2 - 4x + 6 y + 13 = 25
2

l2 æ 2ö Þ x2 + y2 - 4x + 6 y - 12 = 0
Þ l2 - ³0 çQ r = l2 - l ÷
2 ç 2÷ 4. Let the equation of circle be
è ø
Þ l2 ³ 0 x2 + y2 + 2 gx + 2 fg + c = 0 …(i)
which is always true, " l Î R. Since, it is passing through the points (2, 3) and (4, 5).

2. The centre of the given circle x2 + y2 - 4x - 6 y + 11 = 0 is \ 22 + 32 + 2 g (2) + 2 f (3) + c = 0


(2, 3). Þ 4 g + 6 f + c + 13 = 0 …(ii)
Let the other end of diameter is (x1 , y1 ). and 42 + 52 + 2 g (4) + 2 f (5) + c = 0
æ x + 3 y1 + 4 ö Þ 8 g + 10 f + c + 41 = 0 …(iii)
Since, centre = ç 1 , ÷
è 2 2 ø On subtracting Eq. (ii) from Eq. (iii), we get
æ x + 3 y1 + 4 ö 4 g + 4 f + 28 = 0
\ (2, 3) = ç 1 , ÷
è 2 2 ø Þ g + f + 7 =0 …(iv)
x +3 Also, centre (- g , - f ) lies on line y - 4x + 3 = 0
Þ 2= 1
2 \ - f + 4g + 3 = 0 …(v)
y1 + 4 On solving Eqs. (iv) and (v), we get
3= Þ x1 = 1, y1 = 2
2
g = - 2, f = - 5
\ Coordinates of other end of the diameter is (1, 2).
On putting g = - 2, f = - 5 in Eq. (ii), we get
3. It is given, centre is (2, –3) and circumference of circle 4 (- 2) + 6 (- 5) + c + 13 = 0 Þ c = 25
= 10p
\ From Eq. (i), x2 + y2 - 4x - 10 y + 25 = 0 C2
Þ 2pr = 10p Þ r = 5
336 JEE Main Mathematics

5. Let the equation of circle be 9. From figure, we have


(x - 3)2 + ( y - 0)2 + ly = 0 OP = 5, OQ = 6
Y 5
and OM = , CM = 3
2
\ In DOMC , OC 2 = OM 2 + MC 2
A (3, 0) 2
X¢ X æ5ö 61
Þ OC 2 = ç ÷ + (3)2 Þ OC =
P è2ø 2
(1, –2)
Y

As it passes through (1, - 2). Q

(1 - 3)2+ (- 2)2 + l (- 2) = 0
\
6
C (52 , 3)
Þ 4 + 4 - 2l = 0 Þ l = 4
\ Equation of circle is (x - 3)2 + y2 + 4 y = 0. M
O P X
By hit and trial method, we see that point (5, - 2)
5
satisfies equation of circle.
6. Given, equation of the circle is æ5 ö
Thus, the required circle has its centre ç , 3÷ and
x2 + y2 + 4x - 10 y - 7 = 0 è2 ø
61
whose centre is (- 2, 5) and radius (r ) = 4 + 25 + 7 = 6 radius .
2
Distance of point (4, - 3) from centre (- 2, 5) is 2
æ 5ö æ 61 ö
(- 2 - 4)2 + (5 + 3)2 = 100 = 10 Hence, its equation is ç x - ÷ + ( y - 3)2 = ç ÷ .
è 2ø è4ø
Also, we check whether the point lies inside the circle 61
or not. Hence, l=
4
(4)2 + (- 3)2 + 4 × 4 - 10 × (- 3) - 7 = 64 > 0
10. Given equation of circle is x2 + y2 - 4x + 6 y - 12 = 0, whose
So, minimum distance = 10 - 6 = 4
centre is (2, - 3) and radius
and maximum distance = 10 + 6 = 16
= 2 2 + (- 3) 2 + 12 = 4 + 9 + 12 = 5
\ Sum of the minimum and maximum distances
= 4 + 16 = 20 Now, according to given information, we have the
following figure.
7. Since, circle touches X-axis and Y -axis at points (1, 0)
and (0, 1), respectively. S
So, centre of circle is (1, 1) and radius is 1.
A (–3, 2) C
Y

O (2,–3)
B
(0, 1)
(1,1)
x2 + y2 – 4x + 6y – 12 = 0
X¢ X
(1,0)
Clearly, AO ^ BC, as O is mid-point of the chord.
Y¢ Now, in DAOB, we have
OA = (- 3 - 2) 2 + (2 + 3) 2
Hence, equation of circle is
(x - 1)2 + ( y - 1)2 = 12 = 25 + 25 = 50 = 5 2
Þ 2 2
x + y - 2 x - 2y + 1 = 0 and OB = 5
8. The intersection point of diameter lines is (2, 3) which is \ AB = OA 2 + OB2
the centre of circle. = 50 + 25 = 75 = 5 3
Now, radius = (5 - 2)2 + (7 - 3)2 = 9 + 16 = 5 11. Equation of given circle
Hence, required equation of circle is x2 + y 2 = r 2 …(i)
2 2 2
(x - 2) + ( y - 3) = 5 and the chord
Þ x2 + y2 - 4x - 6 y - 12 = 0 y = 2x + 3 …(ii)
Circles 337

Also, y1 and y2 are roots of the equation


2 2
x +y =r 2 y2 + 2 py - q2 = 0.
C (0, 0) \ y1 + y2 = - 2 p
r d
and y1 y2 = - q2
The equation of the circle with P (x1 , y1 ) and Q (x2, y2) as
A r/2 M r/2 B y=2x+3
the end points of diameter is
From the figure (x - x1 )(x - x2) + ( y - y1 )( y - y2) = 0
2 2
AB = r [given] Þ x + y - x (x1 + x2) - y( y1 + y2) + x1x2 + y1 y2 = 0
r 3 2
3 Þ x2 + y2 + 2 ax + 2 py - b2 - q2 = 0
2
d= r – = r and d =
4 2 1+4 16. The point should lies on the opposite side of the origin of
3 3 12 the line x + y - 1 = 0
\ = r Þ r2 = Y
5 2 5
12. Now, the coordinates of the centre (a, a)
A M(h, k) B
and radius of the given circle are
(1, 1) and 2 respectively. 2 120°
X
O
Let AB be the chord subtending an O
æ 2p ö (1, 1) Then, a + a -1 >0
angle of ç ÷ at the centre.
è3ø 1
Þ 2a >1 Þ a> …(i)
2
Also, M (h , k) be the mid-point
Also, (a 2 + a 2) < 1
of AB.
p æ 3ö æ 1 ö æ 1 ö
\In DOAM , AM = OA × sin = 2 çç ÷= 3 Þ ç- ÷<a<ç ÷ …(ii)
3 ÷ è 2 ø è 2ø
è 2 ø
From Eqs. (i) and (ii), we get
\ OM = OA - AM = 4 - ( 3 )2 = 1
2 2 2
1 1
But OM 2 = (h - 1 )2 + (k - 1 )2 <a<
2 2
Þ (h - 1 )2 + (k - 1 )2 = 1
\The locus of (h , k) is (x - 1 )2 + ( y - 1 )2 = 1 17. Let the equation of L1 be y = mx. Since, the intercepts
Þ x2 + y2 - 2x - 2 y + 1 = 0 made by the circle on L1 and L 2 are equal, their
13. Given equation of circle can be rewritten as distances from the centre of the circle are also equal.
k æ1 3ö
x2 + y 2 - 2 x + 4 y +
=0 The centre of the given circle is ç , - ÷ .
4 è2 2ø
k k ½1 - 3 - 1½ ½m ´ 1 + 3½
\ Radius of circle = 1 + 4 - = 5 - ½2 2 ½ ½
4 4 \ = 2 2½
½
½ 1 + 1 ½ ½
½ ½ m2 + 1 ½½
Area of circle = 9p (given)
kö ½ ½ ½ ½
æ
Þ p ç5 - ÷ = 9p 2 |m + 3|
è 4 ø Þ =
k 2 2 m2 + 1
Þ 5 -9 = Þ k = - 16
4 Þ 8(m2 + 1) = (m + 3)2
14. Let the equation of circle is Þ 7m2 - 6m - 1 = 0
x2 + y2 + 2 gx + 2 fy + c = 0 Þ ( m - 1 )( 7m + 1 ) = 0
æ 1ö 1
Since, çm, ÷ lies on this circle. Þ m = 1 or m = -
è mø 7
1 2f So, the equations representing L1 are
\ m2 + 2 + 2 gm + + c=0
m m æ 1ö
y = x or y = ç- ÷ x
Þ m4 + 2 gm3 + cm2 + 2 fm + 1 = 0 è 7ø
Þ m1m2 m3m4 = 1
Þ x - y = 0 or x + 7y = 0
15. Let x1 and x2 are the roots of the equation
18. Given equation of circle is x2 + y2 = r 2. Let any point on
x2 + 2 ax - b2 = 0. the circle be P (r cos q, r sin q) and let the coordinates of
\ x1 + x2 = - 2 a centroid of the triangle be (a , b ).
and x1x2 = - b2
338 JEE Main Mathematics

Y
21. Let AB is a chord and its equation is y = mx. …(i)
Y
B(0, r )
B
P(r cos q, r sin q)
x 2 + y 2 – 2ax = 0
A(r, 0) M
O X (0, 0)
X
A C(a, 0)
r + r cos q
Then, a=
3
r r
Þ cos q = a - Equation of CM which is perpendicular to AB, is
3 3
r + r sin q x + my = l
and b=
3 It passes through the centre (a , 0).
r r
Þ sin q = b - Þ x + my = a …(ii)
3 3

2

2
r2 On eliminating m from Eqs. (i) and (ii), we get
æ æ
Now, ç a - ÷ + çb - ÷ = x2 + y2 = ax
è 3ø è 3ø 9
2 2 2 2 2
æ rö æ rö æ rö Þ x + y - ax = 0 is the locus of the centre of the
\ The locus is ç x - ÷ + ç y - ÷ = ç ÷ which is a required circle.
è 3 ø è 3 ø è3ø
circle. x y
22. Equation of line is + =1 …(i)
a b
19. Since, the given line touches the given circle, the length
of the perpendicular from the centre (2, 4) of the circle to Let P be the foot of the perpendicular drawn from the
the line 3x - 4 y - k = 0 is equal to the radius origin to the line whose coordinates are (x1 , y1 ).
4 + 16 + 5 = 5 of the circle. Since, OP ^ AB.
3 ´2 - 4 ´4 - k Y
\ = ± 5 Þ k = 15 (Q k > 0)
9 + 16
B(0, b)
Now, equation of the tangent at (a , b) to the given circle is
xa + yb - 2(x + a ) - 4( y + b) - 5 = 0 P(x1, y1)
Þ (a - 2)x + (b - 4) y - (2 a + 4b + 5) = 0 A(a, 0)
X
If it represents the given line 3x - 4 y - k = 0 (0, 0)O
a - 2 b - 4 2a + 4b + 5 \ Slope of OP ´ Slope of AB = - 1
Then, = = =l (say)
3 -4 k æ y1 ö æ b ö
Þ çç ÷÷ ç ÷ = -1
Þ a = 3l + 2 , b = 4 - 4l è x1 ø è -a ø
and 2a + 4b + 5 = kl
Þ by1 = ax1 …(ii)
Þ 2(3l + 2) + 4(4 - 4l ) + 5 = 15l (Q k = 15)
Since, P lies on the line AB, then
Þ l = 1 Þ a = 5, b = 0 x1 y
\ k + a + b = 15 + 5 + 0 = 20 + 1 =1
a b
20. Given equation of line is Þ bx1 + ay1 = ab …(iii)
3x - 2 y = k …(i) From Eqs. (ii) and (iii), we get
and equation of circle is ab2
x1 = 2
x2 + y2 = 4r 2 …(ii) a + b2
3 k a 2b
Eq. (i) can be rewritten as y = x - and y1 = 2
2 2 a + b2
3 k 2 2
Þ m= , c=- æ ab2 ö æ a 2b ö
2 2 Now, x12 + y12 = çç 2 ÷ +ç

÷
ç a 2 + b2 ÷
èa + b ø è ø
The line will meet the circle in one point, if c = a 1 + m2
2 2 a 2b4 a 4b2
2 Þ x1 + y1 = 2 +
k æ3ö (a + b2)2 (a 2 + b2)2
Þ - = (2 r ) 1 + ç ÷
2 è2ø a 2b2(a 2 + b2)
Þ x12 + y12 =
On squaring, we get (a 2 + b2)2
k2 13 a 2b2
Þ = 4r 2 ´ Þ k2 = 52 r 2 Þ x12 + y12 = 2
4 4 (a + b2)
Circles 339

1 27. Centre of circle is (0, 0).


Þ x12 + y12 =
1 1
+ Equation of tangent which is (0, 0)
a 2 b2 parallel to x + 2 y + 3 = 0 is
1 1 1
But 2
+ 2= 2 (given) x + 2y + l = 0 …(i)
a b c
\ x12 + y12 = c2 As we know perpendicular distance x + 2y + l = 0
from centre (0, 0) to x + 2 y + l = 0
Thus, the locus of P (x1 , y1 ) is
should be equal to radius.
x2 + y2 = c2 0 + 2 ´0 + l
which is the equation of circle. \ = ±2 Þ l = ±2 5
12 + 22
23. The intersection of line and circle is
On putting the value of l in Eq. (i), we get
x2 + m2x2 - 20mx + 90 = 0
x + 2y = ± 2 5
Þ x2 (1 + m2) - 20 mx + 90 = 0
which represents the required equation of tangents.
Now, D <0
28. According to given information, we have the following
[since the line does not intersect the circle]
figure.
Þ 400m2 - 4 ´ 90 (1 + m2) < 0
Þ 40m2 < 360
\ |m| < 3
C
24. The intersection point of line y = 7x - 25 and circle
2 2 2 2 B
x + y = 25 is x + (7x - 25) = 25. F
2 c
Þ 50x - 350x + 600 = 0 b E
Þ (x - 3) (x - 4) = 0 D a
A
Þ x = 3, x = 4 Þ y = -4, 3
where A , B, C are the centres of the circles
So, coordinates are A (3, - 4) and B(4, 3).
Clearly, AB = a + b (sum of radii) and BD = b - a
\ Distance between A and B = (4 - 3)2 + (3 + 4)2
\ AD = (a + b)2 - (b - a )2
=5 2
[using Pythagoras theorem in DABD]
Alternate Method = 2 ab
a 2(1 + m2) - c2 Similarly, AC = a + c and CE = c - a
Required distance = 2
1 + m2 \In DACE, AE = (a + c)2 - (c - a )2 = 2 ac
25(1 + 49) - 625 Similarly, BC = b + c and CF = c - b
=2 =5 2
1 + 49 \In DBCF, BF = (b + c)2 - (c - b)2 = 2 bc
Q AD + AE = BF
25. Given, y = x tan a + a
\ 2 ab + 2 ac = 2 bc
Condition for tangency is a 2 = a 2(1 + tan 2 a ) 1 1 1
Þ + =
[Q c2 = a 2(1 + m2)] c b a
Þ sec2a = 1 Þ cos 2 a = 1
29. Given equation of circle is
26. Let a be the radius of the circle and centre of circle will x2 + y2 - 2x - 6 y + 6 = 0 …(i)
be (- a , - a ).
Its centre is (1, 3) and radius = 1 + 9 - 6 = 2
Since, the perpendicular distance from centre to the Equation of any line through (0, 1) is
tangent is equal to the radius of the circle.
y - 1 = m (x - 0) Þ mx - y + 1 = 0 …(ii)
3(- a ) - 4(- a ) + 8
\ a= If it touches the circle (i), then the length of
32 + (-4)2 perpendicular from centre (1, 3) to the circle is equal to
a+8 radius 2.
Þ a= m -3 + 1
5 \ =±2
Þ 4a = 8 Þ a = 2 m2 + 1
\Equation of circle is Þ (m - 2)2 = 4 (m2 + 1) Þ 3m2 + 4m = 0
(x + 2)2 + ( y + 2)2 = 22 4
\ m = 0, -
3
Þ x 2 + 4 x + 4 + y2 + 4 y + 4 = 4
On substituting these values of m in Eq. (ii), we get the
Þ x2 + y2 + 4x + 4 y + 4 = 0 required tangents are y - 1 = 0 and 4x + 3 y - 3 = 0.
340 JEE Main Mathematics

30. The tangent at (1, 7) to the parabola x2 = y - 6 is \ According to question,


1 x12 + y12 + 4x1 + 3 2
( y + 7) - 6
x= =
2 x12 + y12 - 6x1 + 5 3
Þ 2x = y + 7 - 12 Þ y = 2x + 5
Þ 9x12
+ 9 y12
+ 36x1 + 27 - 4x12 - 4 y12 + 24x1 - 20 = 0
which is also tangent to the circle
Þ 5x12 + 5 y12 + 60x1 + 7 = 0
x2 + y2 + 16x + 12 y + c = 0 2 2
\ Locus of point is 5x + 5 y + 60x + 7 = 0.
2 2
\ x + (2x + 5) + 16x + 12(2x + 5) + c = 0
34. Since, S1 : x2 + y2 - 6x - 8 y = 0
Þ 5x2 + 60x + 85 + c = 0
S 2 : x2 + y2 - 12x - 16 y + 12 = 0
must have equal roots.
Now, according to given condition, we have
Let a and b are the roots of the equation.
| PT1 | = | PT2|
Þ a + b = - 12 Þ a = -6 (Q a = b )
Þ a 2 + b2 - 6a - 8b = a 2 + b2 - 12a - 16b + 12
\ x = -6
Þ a 2 + b2 - 6a - 8b = a 2 + b2 - 12a - 16b + 12
and y = 2x + 5 = - 7
\ 6a + 8b = 12 Þ 3a + 4b - 6 = 0
Þ Point of contact is (– 6, – 7).
35. Let any point P (x1 , y1 ) outside the circle. Then, equation
31. Required length of tangent from the point (3, – 4) to the
of tangent to the circle x2 + y2 + 6x + 6 y = 2 at the point
circle x2 + y2 - 4x - 6 y + 3 = 0
P is
= 32 + 42 - 4(3) - 6(-4) + 3 = 40 xx1 + yy1 + 3(x + x1 ) + 3( y + y1 ) - 2 = 0 …(i)

\ Square of length of tangent = 40 The Eq. (i) and the line 5x - 2 y + 6 = 0 intersect at a
point Q on Y -axis i.e., x = 0.
32. Let PQ and RP be the two tangents and P be the point Þ 5 (0) - 2 y + 6 = 0 Þ y = 3
on the circle x2 + y2 = a 2 whose coordinates are \Coordinates of Q are (0, 3).
(a cos t , a sin t ) and ÐOPQ = q Point Q satisfies Eq. (i).
\ 3x1 + 6 y1 + 7 = 0 ...(ii)
Q Distance between P and Q is given by
q PQ 2 = x12 + ( y1 - 3)2
O P
= x12 + y12 - 6 y1 + 9 = 11 - 6x1 - 12 y1
(Q x12 + y12 + 6x1 + 6 y1 - 2 = 0)
R
= 11 - 2(3x1 + 6 y1 )
= 11 - 2 (-7 ) = 25 [from Eq. (ii)]
Now, PQ = Length of tangent from P on the circle \ PQ = 5
2 2 2 2
x + y = a sin a 36. Let the equation of circle be
\ PQ = a 2 cos 2 t + a 2 sin 2 t - a 2 sin 2 a x2 + y2 + 2 gx + 2 fgy = 0 (passing through origin)
2 2
= a (cos t + sin t ) - a sin a2 2 2 Radius = g2 + f 2

= a cos a (Q cos 2 t + sin 2 t = 1 ) Y

and OQ = Radius of the circle (x2 + y2 = a 2 sin 2 a ) B


n
Þ OQ = a sin a
OQ
\ tan q = X
PQ O A
a sin a m
= = tan a Þ q = a
a cos a Now, equation of tangent at O(0, 0) is
\ Angle between tangents = ÐQPR = 2 q = 2 a x(0) + y(0) + g (x) + f ( y) = 0 Þ gx + fy = 0
33. Let P (x1 , y1 ) be any point outside the circle. Length of 2 g2
Distance from A (2 g , 0) = =m
tangent to the circle x2 + y2 + 4x + 3 = 0 is g2 + f 2
x12 + y12 + 4x1 + 3 2f 2
and distance from B (0, 2 f ) = =n
2
and length of tangent of the circle x + y - 6x + 5 = 0 is2 g2 + f 2

x12 + y12 - 6x1 + 5 2 r2


Þ =m + n Þ 2r =m + n
r
Circles 341

37. Let OA and OB be the tangents from the origin to the 41. Y
given circle with centre C (-3, 5) and radius P(1, Ö3)
9 + 25 - c = 34 - c . A(4, 0)
O X
Then, area of the quadrilateral,
æ1ö
OACB = 2 ´ Area of the DOAC = 2 ´ ç ÷ ´ OA ´ AC
è2ø
Now, OA = Length of the tangent from the origin to the Equation of tangent to x2 + y2 = 4 at (1, 3 ) on the circle
given circle = c is
and AC = Radius of the circle = 34 - c x + 3y = 4 …(i)
So, that c 34 - c = 8 (given) Now, any line perpendicular to Eq. (i) and passing
Þ c (34 - c) = 64 through (1, 3 ) or centre (0, 0) is normal to circle at
Þ 2
c - 34c + 64 = 0 (1, 3 ).
38. The equation of tangent to the given circle \Equation of normal is
2x2 + 2 y2 - 2x - 5 y + 3 = 0 at point (1, 1) is 3x - y + l = 0
5 l =0
2x + 2 y - (x + 1 ) - ( y + 1 ) + 3 = 0
2 Since, this line passes through origin.
1 1 \ 3 (0) - (0) + l = 0
Þ x- y- =0
2 2 Þ l =0
Þ 2x - y - 1 = 0 Þ Equation of normal is
Þ y = 2x - 1 3x - y = 0
1 Intercept of tangent on X-axis A(4, 0)
Slope of tangent = 2 , therefore slope of normal = - .
2 1
Now, ar (DOAP ) = ( AP )(OP )
Hence, equation of normal at point (1, 1) and having 2
æ 1ö 1
slope ç - ÷ is \ ar (DOAP ) = ´ 2 12 = 2 3 sq units
è 2ø 2
1
y - 1 = - (x - 1 ) Þ 2 y - 2 = - x + 1 Þ x + 2 y = 3 42. The combined equation of the lines joining the origin to
2 the points of intersection of x cos a + y sin a = p and
39. Since, normal passes through the centre of the circle. x2 + y2 - a 2 = 0 is a homogeneous equation of second
degree given by
\The required circle is the circle with ends of diameter 2
as (3, 4) and (– 1, – 2). æ x cos a + y sin a ö
x2 + y2 - a 2 ç ÷ =0
\It’s equation is è p ø
(x - 3)(x + 1 ) + ( y - 4)( y + 2) = 0 Þ [x2( p2 - a 2 cos 2 a ) + y2( p2 - a 2 sin 2 a )
Þ x 2 - 2 x - 3 + y2 - 2 y - 8 = 0 - 2xya 2 sin a cos a = 0]
Þ x 2 + y2 - 2x - 2 y - 11 = 0 The lines given by this equation are at right angle, if

40. Since, the line ax + by = 0 touches the circle ( p2 - a 2 cos 2 a ) + ( p2 - a 2 sin 2 a ) = 0


2 2
x + y + 2x + 4 y = 0, distance of the centre (– 1, – 2) Þ 2 p2 = a 2(sin 2 a + cos 2 a )
from the line = radius Þ a 2 = 2 p2
- a - 2b 43. Let the equation of circles are
\ = (- 1)2 + (- 22)
2
a +b 2 S1 º x2 + y2 + 2x - 3 y + 6 = 0 …(i)
S 2 º x2 + y2 + x - 8 y - 13 = 0 …(ii)
Þ (a + 2b)2 = 5(a 2 + b2)
\Equation of common chord is
Þ a + 4b2 + 4ab = 5a 2 + 5b2
2
S1 - S 2 = 0
Þ (2a - b)2 = 0 Þ b = 2a
Also, ax + by = 0 is normal to the circle Þ (x + y + 2x - 3 y + 6) - (x2 + y2 + x - 8 y - 13) = 0
2 2

x2 + y2 - 4x + 2 y - 3 = 0, the centre (2, – 1) should lie on Þ x + 5 y + 19 = 0 …(iii)


ax + by = 0 In the given option only the point (1, – 4) satisfied the
\ 2a - b = 0 Þ b = 2a Eq. (iii).
Hence, a = 1, b = 2
342 JEE Main Mathematics

44. Suppose AB is a chord of the circle Y 47. Let, the length of common chord = AB = 2 AM = 2x
through A ( p, q) having M (h , 0) as
its mid-point. The coordinates of B A
are (- p + 2h , - q). 12 5
) A (p , q )
As, B lies on the circle 0 C2
( h, C1
M
x2 + y2 = px + qy, we have M X 12 5
O
B
(- p + 2 h )2 + (- q)2 B
= p(- p + 2 h ) + q(- q)
Þ 2 p2 + 2q2 - 6 ph + 4h 2 = 0 Now, C1C 2 = AC12 + AC 22
Þ 2 h 2 - 3 ph + p2 + q2 = 0 …(i) [Q circles intersect each other at 90º] … (i)
As, there are two distinct chords from A ( p, q) which are and C1C 2 = C1M + MC 2
bisected on X-axis, there must be two distinct values of Þ C1C 2 = 12 - AM 2 + 52 - AM 2
2
… (ii)
h satisfying Eq. (i).
From Eqs. (i) and (ii), we get
D = 9 p2 - (4)(2)( p2 + q2) > 0 Þ p2 > 8q2
AC12 + AC 22 = 144 - AM 2 + 25 - AM 2
45. Let the point on x2 + y2 = a 2 is (a cos q, a sin q)
Þ 144 + 25 = 144 - x2 + 25 - x2
Equation of chord of contact is
ax cos q + ay sin q = b2 Þ 13 = 144 - x2 + 25 - x2
It touches circle x2 + y2 = c2 On squaring both sides, we get
½ -b2 ½ 169 = 144 - x2 + 25 - x2 + 2 144 - x2 25 - x2
\ ½ 2 2 2 2
½= c
½ a cos q + a sin q½ Þ x2 = 144 - x2 25 - x2
Þ b2 = ac Again, on squaring both sides, we get
\ a , b and c are in GP. x4 = (144 - x2) (25 - x2)
46. Given, equation of circles are = (144 ´ 25) - (25 + 144)x2 + x4
144 ´ 25 12 ´ 5 60
x2 + y2 - 4 y = 0 Þ x2 = Þx= = cm
2 2 169 13 13
and x + y - 8x - 4 y + 11 = 0
120
\ Equation of chord is Now, length of common chord 2x = cm
13
x2 + y2 - 4 y - (x2 + y2 - 8x - 4 y + 11) = 0 Þ 8x - 11 = 0
48. Given that, circle S1 º x2 + y2 + 4x + 22 y + c = 0 bisects
the circumference of the circle
P
S 2 º x2 + y2 - 2x + 8 y - d = 0.
The common chord of the given circles is
O M O¢
S1 - S 2 = 0
Þ x2 + y2 + 4x + 22 y + c - x2 - y2 + 2x - 8 y + d = 0
Q Þ 6x + 14 y + c + d = 0 …(i)
So, Eq. (i) passes through the centre of the second circle,
So, centre and radius of first circle are O(0, 2) and
i.e., (1, – 4).
OP = r = 2.
Now, perpendicular distance from O(0, 2) to the line \ 6 - 56 + c + d = 0 Þ c + d = 50
8x - 11 is 49. The required equation of circle is
|8 ´ 0 - 11| 11 1 25 ö
d = OM = = æ
8 (x2 + y2 + 13x - 3 y) + l ç11x + y + ÷ = 0 …(i)
82 è 2 2ø
In DOMP, PM = OP 2 - OM 2 It passes through (1, 1).
2 1
æ 11 ö 121 \ 12 + l(24) = 0 Þ l = -
= 22 - ç ÷ = 4- 2
è8ø 64
On putting in Eq. (i), we get
256 - 121 135
= = 11 1 25
64 8 x2 + y2 + 13x - 3 y - x- y- =0
2 4 4
135 135 Þ 4x2 + 4 y2 + 52x - 12 y - 22x - y - 25 = 0
\ Length of chord PQ = 2PM = 2 ´ = cm
8 4
Þ 4x2 + 4 y2 + 30x - 13 y - 25 = 0
Circles 343

50. Let the equation of circles be 54. Required equation of circle is


S1 º x2 + y2 + 13x - 3 y = 0 …(i) x2 + y2 - 6x - 8 y + l (x + y - 1) = 0
and S 2 º 2x2 + 2 y2 + 4x - 7 y - 25 = 0 …(ii) or x2 + y2 - (6 - l ) x - (8 - l ) y - l = 0,
The equation of intersecting circle is lS1 + S 2 = 0.
æ l lö
æ 7 y 25 ö whose centre is ç3 - , 4 - ÷ and
Þ l(x2 + y2 + 13x - 3 y) + ç x2 + y2 + 2x - - ÷ =0 è 2 2ø
è 2 2ø
é æ7 ö 25 ù which lies on the line x + y - 1 = 0.
Þ ê x2(1 + l ) + y2(1 + l ) + x (2 + 13l ) - y ç + 3l ÷ - ú l l
ë è 2 ø 2û Þ 3 - + 4 - -1 =0
2 2
=0 …(iii)
Þ l =6
é (2 + 13l ) ( 7 / 2) + 3l ù
\ Centre = ê - , Hence, required equation is
ë 2 (1 + l ) 2 (1 + l ) úû
x2 + y 2 - 6 x - 8 y + 6 x + 6 y - 6 = 0
Q Centre lies on 13x + 30 y = 0.
Þ x2 + y 2 - 2 y - 6 = 0
æ 2 + 13l ö é (7/ 2) + 3l ù
Þ -13 ç ÷ + 30 ê úû = 0 55. Equation of tangent to the circle
è 2 ø ë 2
x2 + y2 + 4x - 6 y - 12 = 0 at (1, - 1) is given by
Þ -26 - 169l + 105 + 90l = 0 Þ l = 1
xx1 + yy1 + 2 (x + x1 ) - 3 ( y + y1 ) - 12 = 0,
Hence, putting the value of x in Eq. (iii), then required
equation of circle is where x1 = 1 and y1 = - 1
4x2 + 4 y2 + 30x - 13 y - 25 = 0 Þ x - y + 2 (x + 1) - 3( y - 1) - 12 = 0
Þ 3x - 4 y - 7 = 0
51. Equation of family of concentric circles to the circle
x2 + y2 + 6x + 8 y - 5 = 0 is x2 + y2 + 6x + 8 y + l = 0. This will also a tangent to the required circle.
Now, equation of family of circles touching the line
which is similar to x2 + y2 + 2 gx + 2 fy + c = 0. Since, it is
equation of concentric circle to the circle 3x - 4 y - 7 = 0 at point (1, - 1) is given by
x2 + y2 + 6x + 8 y - 5 = 0. Thus, the point (–3, 2) lies on (x - 1)2 + ( y + 1)2 + l (3x - 4 y - 7) = 0
the circle x2 + y2 + 6x + 8 y + c = 0. So, the equation of required circle will be
Þ (-3)2 + (2)2 + 6(-3) + 8(2) + c = 0 (x - 1)2 + ( y + 1)2 + l (3x - 4 y - 7) = 0,
Þ 9 + 4 - 18 + 16 + c = 0 for some l Î R …(i)
Þ c = - 11 Q The required circle passes through (4, 0)
52. Centre of given circle x2 + y2 - 6x + 12 y + 15 = 0 is (3, - 6). \(4 - 1)2 + (0 + 1)2 + l (3 ´ 4 - 4 ´ 0 - 7) = 0
Þ 9 + 1 + l (5) = 0 Þ l = - 2
\ Radius = (3)2 + (-6)2 - 15 = 30
Substituting l = - 2 in Eq. (i), we get
Area of circle = pr 2 = p ( 30 )2 = 30p (x - 1)2 + ( y + 1)2 - 2 (3x - 4 y - 7) = 0
Area of require circle = 2 (Area of given circle) Þ x2 + y2 - 8x + 10 y + 16 = 0
\ pR2 = 2 ´ 30p = 60p On comparing it with
Þ R2 = 60 Þ R = 2 15 x2 + y2 + 2 gx + 2 fy + c = 0, we get
\Equation of required circle is g = - 4, f = 5, c = 16
(x - 3)2 + ( y + 6)2 = (2 15 )2 \ Radius = g2 + f 2 - c
2 2
Þ x + 9 - 6x + y + 36 + 12 y = 60
= 16 + 25 - 16 = 5
Þ x2 + y2 - 6x + 12 y - 15 = 0
56. Let P (x1 , y1 ) be the point outside the circle. From the
53. Since, the equation of a family of circles touching line
given condition
L =0 at their point of contact(x1 , y1 ) is
(x - x1 )2 + ( y - y1 )2 + l L = 0, where l Î R. x12 + y12 + 2x1 - 4 y1 - 20 2
=
\ Equation of circle, touches the x = y at point (1, 1) is x12 + y12 - 4x1 + 2 y1 - 44 3
(x - 1)2 + ( y - 1)2 + l (x - y) = 0
Þ 3x12 + 3 y12 + 6x1 - 12 y1 - 60
Þ x2 + y2 + (l - 2)x + (- l - 2) y + 2 = 0 …(i)
= 2x12 + 2 y12 - 8x1 + 4 y1 - 88
Q Circle (i) passes through point (1, - 3).
\ 1 + 9 + (l - 2) + 3(l + 2) + 2 = 0 Þ x12 + y12 + 14x1 - 16 y1 + 28 = 0
Þ 4l + 16 = 0 Þ l = - 4 Thus, the locus of point is
So, equation of circle (i) at l = - 4 , is x2 + y2 + 14x - 16 y + 28 = 0
x2 + y2 - 6x + 2 y + 2 = 0
\Coordinates of centre of circle are (–7, 8).
Now, radius of the circle = 9 + 1 - 2 = 2 2.
344 JEE Main Mathematics

57. Let the equation of the concentric circles be Y


x2 + y2 - 2x - 4 y + l = 0, it passes through (3, 4).
\ 32 + 42 - 2(3) - 4(4) + l = 0 Þ l = - 3
C3
Thus, the equation of concentric circle is
x2 + y2 - 2x - 4 y - 3 = 0. C1 C2
X

58. For required circle, P(1, 8) and O(3, 2) will be the end (0, 0) (1, 0)
points of its diameter.

A So, let the equation of a common tangent be


3x - y + k = 0
(1, 8) P O(3, 2)
½ k ½
It will touch C1 , if ½ ½ = 1 Þ k = ±2
B
½ 3 + 1½

\ (x - 1 ) (x - 3) + ( y - 8)( y - 2) = 0 From the figure, we observe that the required tangent


2 2 makes positive intercept on the Y -axis and negative on
Þ x + y - 4x - 10 y + 19 = 0
the X-axis and hence its equation is 3x - y + 2 = 0.
59. The centres of the two circles will lie on the line which is the equation of required locus.
through P(1, 2) perpendicular to the common tangent
4x + 3 y = 10. If C1 and C 2 are the centres of these 61. Centre of required circle = (3, - 4)
circles, then PC1 = 5 = r1 , PC 2 = - 5 = r2. Radius of required circle = 5 + 1 = 6
x-1 y-2 \Locus of circle is
Also, C1 , C 2 lie on the line = = r, where
cos q sin q (x - 3)2 + ( y + 4)2 = 36
3
tan q = . When r = r1 the coordinates of C1 are Þ x2 - 6x + 9 + y2 + 16 + 8 y = 36
4
4 3 Þ x2 + y2 - 6x + 8 y - 11 = 0
(5 cos q + 1, 5 sin q + 2) or (5, 5) as cos q = , sin q = .
5 5 62. Let the general equation of circle be
When r = r2, the coordinates of C 2 are (–3, –1). x2 + y2 + 2 gx + 2 fy + c = 0 …(i)
The circle with centre C1 (5, 5) and radius 5 touches It cuts the circle x2 + y2 - 20x + 4 = 0 orthogonally, then
both the coordinate axes and hence lies completely in by the condition, 2( g1 g2 + f1 f2) = c1 + c2
the first quadrant.
2(-10 g + 0 ´ f ) = c + 4 Þ -20 g = c + 4 …(ii)
Therefore, the required circle is with centre (–3, –1)
and radius 5, so its equation is Q Circle (i) touches the line x = 2 or x + 0 y - 2 = 0
(x + 3)2 + ( y + 1 )2 = 52 \ Perpendicular distance from centre to the tangent =
Þ x + y2 + 6x + 2 y - 15 = 0
2
radius.
Since, the origin lies inside the circle, a portion of the ½- g + 0 - 2½
circle lies in all the quadrants. Þ ½ 2 ½= g2 + f 2 - c
2
60. Equation of any circle through (0, 0) and (1, 0) is ½ 1 +0 ½
x y 1 Þ ( g + 2 )2 = g 2 + f 2 - c
(x - 0 )(x - 1 ) + ( y - 0)( y - 0) + l 0 0 1 = 0 Þ g2 + 4 + 4 g = g2 + f 2 - c
1 0 1
Þ 4g + 4 = f 2 - c …(iii)
2 2
Þ x + y - x + ly = 0
On eliminating c from Eqs. (ii) and (iii), we get
If it represents C3 , its radius = 1
2 -16 g + 4 = f 2 + 4 Þ f 2 + 16 g = 0
æ1ö æ l ö Hence, the locus of (- g , - f ) is y2 - 16x = 0.
Þ 1 = ç ÷ + çç ÷÷
è4ø è 4 ø (replacing - f and - g by x and y)
Þ l=± 3 63. Let the family of circles passing through origin be
As, the centre of C3 , lies above the X-axis, we take x2 + y2 + 2 gx + 2 fy = 0
l = - 3 and thus an equation of C3 is
x2 + y2 - x - 3 y = 0. They intersect circle x2 + y2 + 4x - 6 y - 13 = 0
orthogonally.
Since, C1 and C3 intersect and are of unit radius, their
common tangents are parallel to the line joining their So, 2 g (2) - 2 f (3) = - 13
æ1 3 ö Hence, locus of (- g , - f ) is - 4x + 6 y + 13 = 0
centres (0, 0) and çç , ÷.
÷ Þ 4x - 6 y - 13 = 0
è2 2 ø
Circles 345

64. Let the equation of circle be Þ 2hx + 2ky = h 2 + k2 - 9 …(i)


2 2 Let p be the length of the perpendicular from the centre
x + y + 2 hx + 2 ky + c = 0 …(i)
(0, 0) of S1 to common chord
The locus of whose centre is to be obtained, since the
circle cuts ½h 2 + k2 - 9½
\ p =½ ½
x2 + y2 + 4x - 6 y + 9 = 0 …(ii) 2
½ 4h + 4k ½
2

and x2 + y2 - 4x + 6 y + 4 = 0 …(iii)
Now, the length of the comon chord = 2 42 - p2
Orthogonally, then
It will be of maximum length if p = 0
2 h (2) + 2 k(-3) = c + 9
Þ h 2 + k2 - 9 = 0 …(ii)
Þ 4h - 6k = c + 9 …(iv)
3
and 2 h (-2) + 2 k(3) = c + 4 The slope of Eq. (i) is (given)
4
Þ - 4h + 6k = c + 4 …(v)
h 3 4h
On solving Eqs. (iv) and (v), we get \ - = Þ k=- …(iii)
k 4 3
c + 9 = - c - 4 Þ 2c = - 13 …(vi)
On substituting the respective value of k in Eq. (ii),
On putting the value of c in Eq. (iv), we get we have
Þ 8h - 12k = 5 …(vii) 9 12
h=± and k = ± [using Eq. (iii)]
Centre of the given circle is (- h , - k). 5 5
\ Locus of (- h , - k) from Eq. (vii) is æ9 12 ö æ 9 12 ö
\The centres of circle C 2 are C ç , - ÷ or C ç - , ÷
8 (- x) - 12 (- y) = 5 Þ 8x - 12 y + 5 = 0 è 5 5 ø è 5 5ø
65. Given equations of circles are x2 + y2 + 2x + 2ky + 6 = 0 68. (x2 + y2 - 2x - 1 ) + lx = 0, they pass through
and x2 + y2 + 2ky + k = 0. They intersect each other intersection points of line x = 0 and circle
orthogonally. x2 + y2 - 2x - 1 = 0.
\ 2 × 1 × 0 + 2 × k × k = 6 + k Þ 2k2 - k - 6 = 0 Þ y= ±1
3 \ Required points are (0 , ± 1).
Þ (2k + 3)(k - 2) = 0 Þ k = 2 ,-
2
66. Since, the given circles cut each other orthogonally.
Round II
\ g1 g2 + a = 02
…(i)
1. Since, y = |x|+ c and x2 + y2 - 8| x| - 9 = 0 both are
If lx + my = 1 is a common tangent of these circles, then symmetrical aboutY -axis for x > 0, y = x + c.Equation of
- lg1 - 1 tangent to circle x2 + y2 - 8x - 9 = 0 which is parallel to
= ± g12 + a 2
l2 + m 2 y = x + c is y = (x - 4) + 5 1 + 1
Þ (lg1 + 1)2 = (l2 + m2)( g12 + a 2) Þ y = x + (5 2 - 4)
Þ m g1 - 2lg1 + a 2(l2 + m2) - 1 = 0
2 2 For no solution c > 5 2 - 4,
Similarly, m2g22 - 2lg2 + a 2(l2 + m2) - 1 = 0 \ c Î (5 2 - 4, ¥ )
So that g1 and g2 are the roots of the equation 2. Let C (h , k) be the centre of the circle passing through
m2g 2 - 2lg + a 2(l2 + m2) - 1 = 0 the end points of the rod AB and PQ of lengths a and b
a 2(l2 + m2) - 1 respectively. CL and CM be perpendiculars drawn from
Þ g1 g2 = = - a2 [from Eq. (i)]
m2 C on AB and PQ respectively. Then, CA = CP (radii of
the same circle)
Þ a 2(l2 + m2) = 1 - a 2m2 …(ii)
|ma - 1| |-ma - 1|
Now, p1 p2 = × Q
l2 + m 2 l2 + m 2
M C(h, k)
|1 - m2a 2|
= = a2 [from Eq. (ii)]
l2 + m 2 P
A B
67. Let P (h , k) be the coordinates of the centre of circle S 2. L
Then, its equation is a2 b2 æ a bö
(x - h )2 + ( y - k)2 = 52 Þ k + = h2 + çQ AL = and MP = ÷
4 4 è 2 2ø
The equation of S1 is x2 + y2 = 42 and so the equation of
Þ 4(h 2 - k2) = a 2 - b2
the common chord of S1 and S 2 is
Hence, locus of (h , k) is 4(x2 - y2) = a 2 - b2.
S1 - S 2 = 0
346 JEE Main Mathematics

3. Equation of pair of tangents is Since, (0, 0) divides the chord in the ratio 1 : 4.
2 2 2 2 Therefore, x2 = - 4x1
(a - 1 ) y - x + 2ax - a = 0
3 + 4m -4
If q be the angle between the tangents, then \ -3x1 = 2
and 4x12 = -
1+m 1 + m2
2 (h 2 - ab)
tan q = \ 9 + 9m2 = 9 + 16m2 + 24m
a+b
24
2 - (a 2 - 1 )(-1 ) 2 a2 - 1 i.e., m = 0, -
= = 7
2
a -2 a2 - 2 Therefore, the lines are y = 0 and y + 24x = 0.
Q q lies in II quadrant, then tan q < 0 6. Equation of given circle is x2 + y2 = 1, then equation of
2
2 a -1 tangent at the point (cos q,sin q) on the given circle is
\ 2
<0
a -2 x cos q + y sin q = 1 …(i)
Þ 2
a - 1 > 0 and a - 2 < 0 2 [Q equation of tangent at the point
Þ 1 < a2 < 2 P(cos q, sin q) to the circle x2 + y2 = r 2
Þ a Î (- 2 , - 1 ) È (1, 2 ) is x cos q + y sin q = r]
4. Let the equation of the chord OA of the circle Now, the point of intersection with coordinate axes
are P(sec q, 0) and Q(0, cosec q).
x2 + y2 - 2x + 4 y = 0 …(i)
Q Mid-point of line joining points P and Q is
by y = mx …(ii)
æ sec q cosec q ö
Y Mç , ÷ = (h , k) (let)
è 2 2 ø
A 1 1
So, cos q = and sin q =
C 2h 2k
Q sin 2 q + cos 2 q = 1
B 1 1 1 1
X \ + = 1Þ 2 + 2 = 4
O 4h 2 4k2 h k
On solving Eqs. (i) and (ii), we get Now, locus of mid-point M is
x2 + m2x2 - 2x + 4mx = 0 1 1
Þ + = 4 Þ x2 + y 2 - 4 x 2 y 2 = 0
Þ (1 + m2)x2 - (2 - 4m)x = 0 x2 y 2
2 - 4m So, correct option is (c).
Þ x = 0 and x =
1 + m2 7. Let (h , k) be the centre of the circle and radius r = h, as
Hence, the points of intersection are circle touch theY -axis and other circle x2 + y2 = 1 whose
æ 2 - 4m m(2 - 4m) ö centre (0, 0) and radius is 1.
(0, 0) and A çç , ÷.
è1 + m
2
1 + m2 ÷ø Y
2
æ 2 - 4m2 ö (2 - 4m)2
Þ OA 2 = çç ÷ (1 + m2) =
2 ÷
(h,k)
è1+m ø 1 + m2 C r=h>O
r
k >O
Since, OAB is an isosceles right angled triangle
for first
1
OA 2 = AB2, quadrant
2 X
O
where AB is a diameter of the given circle OA 2 = 10
x2+y2=1
(2 - 4m)2
Þ = 10
1 + m2
\ OC = r + 1
Þ 4 - 16m + 16m2 = 10(1 + m)2
[Q if circles touch each other externally,
Þ 3m2 - 8m - 3 = 0
then C1C 2 = r1 + r2]
1
Þ m = 3 or - Þ h 2 + k2 = h + 1, h > 0
3
Hence, the required equation are y = 3x or x + 3 y = 0. (x1, y1)
y = 2x + 5
5. y = mx be the chord. Then, point of intersection are
given by x2(1 + m2) - x (3 + 4m) - 4 = 0.
3 + 4m -4 (–8, –6)
Therefore, x1 + x2 = 2
and x1x2 =
1+m 1 + m2
Circles 347

and k > 0, for first quadrant. 11. C


Þ h 2 + k2 = h 2 + 2h + 1 B
N
Þ k2 = 2h + 1 b
a a
Þ k = 1 + 2h , as k > 0 A
M L
Now, on taking locus of centre (h , k), we get D
y = 1 + 2x, x ³ 0 E
8. 2x – y + 1 = 0 a-b
A(2, 5) From DMLN , sin a =
a+b
æ a - bö
\ a = sin -1 çç ÷÷
O x – 2y = 4 è a + bø
h, h – 4 m1m2 = –1 æ a - bö
2 \ Angle between AB and AD, q = 2a = 2 sin -1 çç ÷÷ .
è a + bø
æ (h - 4) ö 12.
çh - ÷
ç 2 ÷ (2) = - 1 P(5, 7)
p2
çç 2 - h ÷÷ 3 D
è ø M
O(0, 0) d2
C
h =8 2 p1 B
N
center (8, 2) A d
1
Radius = (8 - 2)2 + (2 - 5)2 = 3 5
Let equation of line be y = x - c or y - x = c …(i)
9. r = 1; L = 3 ½- c½
½= c
Perpendicular from (0, 0) on line (i) is ½
Area of quadrilateral = 3 ½ 2½ 2
1 2p p 2
Sector = ×1 × = æ c ö
2 3 3 In DAON , 22 - ç ÷ = AN
p è 2ø
Shaded region = 3 -
3 2
æ 2 - cö
2 2 and in DCPM , 32 - ç ÷ = CM
10. x + y + ax + 2ay + c = 0 è 2 ø
a2 Given, AN = CM
2 g2 - c = 2 - c =2 2
4 c2 (2 - c)2 1
Þ 4- =9 - Þ c=-
a2 2 2 2
Þ - c=2 …(i)
4 3
Therefore, equation of the line is y = x - or
2 f 2 - c = 2 a2 - c = 2 5 2
2x - 2 y - 3 = 0.
Þ a2 - c = 5
From Eqs. (i) and (ii), we get 13. Given equation of line is x + y = n,n Î N …(i)
3a 2 and equation of circle is x + y = 162 2
…(ii)
= 3 Þ a = - 2 (a < 0)
4 Now, for intercept, made by circle (ii) with line (i)
\ c= -1
Circle Þ x2 + y2 - 2x - 4 y - 1 = 0 x2+y2=16
Þ (x - 1)2 + ( y - 2)2 = 6 (0, 0)
1 4
Given, x + 2 y = 0 Þ m = - d
2
mtangent = 2 A B x+y=n, n 0 N
Equation of tangent n
d < 4Þ <4
Þ ( y - 2) = 2 (x - 1) ± 6 1 + 4 2
Þ 2x - y ± 30 = 0 [Q d = perpendicular distance from (0, 0) to the line
± 30 |0 + 0 - n | n ù
Perpendicular distance from (0, 0) = = 6 x + y = n and it equal to = ú
4+1 12 + 12 2û
348 JEE Main Mathematics

Þ n <4 2 …(iii) Q Circle (i) passes through point (2, 0), so


Q n Î N , so n = 1, 2, 3, 4, 5
4 + 16 + 2l = 0 Þ l = - 10
Clearly, length of chord AB = 2 42 - d 2 Therefore, equation of the circle is
n2 é n ù x2 + ( y - 4)2 - 10x = 0
= 2 16 - êëQ d = 2 úû
2 Þ (x - 5)2 + ( y - 4)2 = 25 … (ii)
\Sum of square of all possible lengths of chords (for Now, equation of tangent to the circle (ii) having slope
n = 1, 2, 3, 4, 5) ‘m’ is
é 1 ù y - 4 = m(x - 5) ± 5 1 + m2
= 4 ê (16 ´ 5) - (12 + 22 + 32 + 42 + 52)ú
ë 2 û Þ y = mx + (4 - 5m ± 5 1 + m2 ) … (iii)
5(6)(11)
= 320 - 2 = 320 - 110 = 210 From the option, on taking slope of line, we have
6
4
14. Given equation of circle is when m = , equation of tangent by Eq. (iii), we get
3
x2 + y2 - 6x + 8 y - 103 = 0, which can be written as
4 æ 20 16 ö
(x - 3)2 + ( y + 4)2 = 128 = (8 2 )2 y = x + çç4 - ±5 1+ ÷
3 è 3 9 ÷ø
\ Centre = (3, - 4) and radius = 8 2
Now, according to given information, we have the Þ 3 y = 4x + (12 - 20 ± 25)
following figure. Þ 4x - 3 y - 8 ± 25 = 0
Y Þ 4x - 3 y + 17 = 0 and 4x - 3 y - 33 = 0
Similarly, when m = - 3 / 4, equation of tangent is
D C 3 æ 15 9 ö
O X y = - x + çç4 + ± 5 1+ ÷
÷
4 è 4 16 ø
45°
G Þ 4 y = - 3x + 31 ± 25
(3,–4)
Þ 3x + 4 y - 56 = 0 and 3x + 4 y - 6 = 0
4
A B Similarly, when m = - , equation of tangent is
3
4 æ 20 25 ö
y = - x + ç4 + ± ÷
For the coordinates of A and C. 3 è 3 3ø
x-3 y+ 4 Þ 4x + 3 y - 57 = 0 and 4x + 3 y - 7 = 0
Consider, = =±8 2
1 1
\ The line 4x + 3 y - 8 = 0 is not a tangent to the circle.
2 2
Hence, option (c) is correct.
[using distance (parametric) form of line,
x - x1 y - y1 16. Let us assume that the coordinates of the centre of the
= = r] circle are C (h , k) and its radius is r.
cos q sin q
Since, the circle touches X-axis at (1, 0), hence its radius
Þ x = 3 ± 8, y = - 4 ± 8
should be equal to ordinate of centre.
\ A(- 5, - 12) and C (11, 4)
Þ r=k
Similarly, for the coordinates of B and D, consider
Hence, the equation of the circle is
x-3 y+ 4
= = ±8 2 (x - h )2 + ( y - k)2 = k2
1 1
-
2 2 Also, given that the circle passes through the points
[in this case, q = 135°] (1, 0) and (2, 3). Hence, substituting them in the
Þ x = 3 m 8, y = - 4 ± 8 equation of the circle, we get
\ B (11, - 12) and D (- 5, 4) (1 - h )2 + (0 - k)2 = k2 …(i)
2 2 2
Now, OA = 25 + 144 = 169 = 13; (2 - h ) + (3 - k) = k …(ii)
OB = 121 + 144 = 265 From Eq. (i), we get
OC = 121 + 16 = 137 h =1
and OD = 25 + 16 = 41 On substituting in Eq. (ii), we get
5
15. Since, equation of a circle which touches the Y-axis at (2 - 1 )2 + (3 - k)2 = k2 Þ k =
point (0, 4) is 3
10
x2 + ( y - 4)2 + lx = 0 … (i) \The diameter of the circle is 2k = ×
3
Circles 349

17. x2 + y2 - ax = 0 and x2 + y2 = c2 Y
22. The given equations of diameters are
touch each other. 3x - 4 y - 7 = 0 …(i)
(i) If circles touch internally, C
and 2x - 3 y - 5 = 0 …(ii)
a a a/2 On solving Eqs. (i) and (ii), we get
c- = X¢ X
2 2 O a/2 C x = 1 and y = - 1
a a \Centre of circle is (1, –1).
Þ c - = Þ c = a, c > 0
2 2 Let r be the radius of circle, then
\ |a|= c pr 2 = 49p Þ r = 7 unit

(ii) If circles touches externally, \Equation of required circle is
Y (x - 1 )2 + ( y + 1 )2 = 49
Þ x2 + y2 - 2x + 2 y + 1 + 1 = 49
Þ x2 + y2 - 2x + 2 y - 47 = 0
a/2 23. (7) Equation of given circle is x2 + y2 = 9
X¢ C
X
(0, 0) C a/2
So, let end point of diameter PQ are P(3 cos q, 3 sin q)
and Q (– 3 cos q, - 3 sin q).
Now, the lengths of the perpendiculars from P and Q
Y¢ on the straight line, x + y = 2 are respectively.
a a a a |3 cos q + 3 sin q – 2| |–3 cos q – 3 sin q – 2|
c+ = Þ c+ = a= and b =
2 2 2 2 2 2
\ c = 0, i. e. , not possible as c > 0 |(3 cos q + 3 sin q)2 – 22|
\ ab =
\ The circles should touch internally and|a|= c. 2
|9 – 4 + 9 sin 2 q| |5 + 9 sin 2 q|
18. Let S º x 2 + y2 + 3 x + 7 y + 2 p - 5 = 0 = =
2 2
and S ¢ º x2 + y2 + 2x + 2 y - p2 = 0 Since, maximum value of ab, if sin 2 q = 1
Equation of the required circle is S + lS ¢ = 0 5+9
\ a b|max = =7
As, it passes through (1, 1), the value of 2
l = - (7 + 2 p) / (6 - p2)
24. (36) As the given curves, x2 + y2 - 6x + 8 = 0
Here, l is not defined at p = ± 6
and x2 + y2 - 8 y + 16 - k = 0 , (k > 0) touches each other
Hence, it is true for all except two values of p. at a point, then
19. Circle with centre (h , k) and touching X-axis is Distance between centre of circles
2 2 2
x + y - 2hx - 2ky + h = 0 = Sum of radii or difference of radii
Since, (-1, 1 ) lies on it. Þ C1C 2 = |r1 ± r2|
Þ 2 + 2h - 2k + h 2 = 0 Þ (3 - 0)2 + (0 - 4)2 =| 9 - 8 ± 16 - 16 + k|
For real circles, D ³ 0 Þ 5 = |1 ± k|
1
Þ (2)2 - 4 (-2k + 2) ³ 0 Þ k ³ Þ 1± k = ±5
2
Þ ± k = 4, - 6
20. r1 = 3, c1 (5, 5)
Þ k = 16 or 36
r2 = 3, c2(8, 5) \ Largest value of k is 36.
C1C 2 = 3, r1 = 3, r2 = 3
25. (2) Let S º x2 + y2 = 16
At point (0, 0), S º 0 + 0 - 16 = - 16 < 0

C1 C2

C1 C (0, 0)

21. The locus is the circle with centre (0, 0)


p 3
and radius 3 cos =
3 2
3 3 At point (0, 1), S º 0 + 1 - 16 = - 15 < 0
9
Its equation is x2 + y2 = × Thus, both points lie inside the circle.
4 P
It means required circle touch inside the given circle.
350 JEE Main Mathematics

So, the centre and radius of given circle are C (0, 0) and Þ x-1 = 3 -1
r = 4. EB = 1 - x = 1 - ( 3 - 1)
Since, the required circle is passing through centre (0, 0). EB = 2 - 3
\ Diameter of required circle = 4
4 28. (24) Let length of common chord be 2a, then
and radius of required circle = = 2
2 9 - a 2 + 16 - a 2 = 5
26. (256) Let point P (x1 , y1 ) be any point on the circle, 16 - a 2 = 5 - 9 - a 2
therefore it satisfy the circle
16 - a 2 = 25 + 9 - a 2 - 10 9 - a 2
(x1 - 3)2 + ( y1 + 2)2 = 5r 2 …(i)
The length of the tangent drawn from point P (x1 , y1 ) to
the circle (x - 3)2 + ( y + 2)2 = r 2 is
3 4
(x1 - 3)2 + ( y1 + 2)2 - r 2 = 5r 2 - r 2 [from Eq. (i)]
Þ 16 = 2r Þ r =8
\The area between two circles
= p × 5r 2 - pr 2 = 4pr 2
= 4p ´ 82 = 256p sq units 10 9 - a 2 = 18
\ k = 256 Þ 100 (9 - a 2) = 324
27. (1) i.e. 100a 2 = 576
Y-axis
576 24
D(0, 1) C(1, 1) Þ a= =
100 10
24 k
D Þ 2a = = Þ k = 24
5 5
(r, r)
O 29. (4) Common chord is given by
(x2 + y2 - 8) - {(x - a )2 + y2 - 8} = 0
Þ 2ax - a 2 = 0
X-axis
A(0, 0) 2x
E B(1, 0) Þ =1
a
2
æ 2x ö
Here, AO + OD = 1 or ( 2 + 1) r = 1 Þ r = 2 - 1 Now, x2 + y2 - 8 ç ÷ = 0 gives the combined equation
èaø
Equation of circle (x - r )2 + ( y - r )2 = r 2
of the straight lines joining the end points of this
Equation of CE common chord and origin.
y - 1 = m (x - 1) If this common chord subtends a right angle at origin,
mx - y + 1 - M = 0 then
It is tangent to circle 32
1 - 2 + 1 =0
mr - r + 1 - m a
\ =r Þ a 2 = 16
m2 + 1
a=±4
(m - 1)r + 1 - m \ |a| = 4
Þ =r
2 2 2
m +1 30. (6) x + y - 6 3x - 6 y + 27 = 0
2 2
(m - 1) (r - 1) Equation of the pair of tangent is given by
Þ = r2
m2 + 1 (-3 3x - 3 y + 27)2 = 27 (x2 + y2 - 6 3x - 6 y + 27)
Put r = 2 - 1 27x2 + 9 y2 + 272 + 18 3xy - 6 ´ 27 3x - 6 ´ 27 y
On solving m = 2 - 3 , 2 + 3 = 27x2 + 27 y2 - 6 ´ 27 3x - 6 ´ 27 y + 272
Taking greater slope of CE as 2 + 3 18 y2 - 18 3xy = 0
y - 1 = (2 + 3 ) (x - 1) y( y - 3x) = 0
Put y = 0 \ The tangents are y = 0, y = 3x
- 1 = (2 + 3 ) (x - 1) p
\ Angle between the triangles is .
-1 æ2 - 3 ö 3
Þ ´ç ÷ = x-1
2 + 3 çè 2 - 3 ÷ø \ 2 3 tan q = 2 3 ´ 3 = 6
14
Parabola
A parabola is a two dimensional, minor symmetrical curve, which is IN THIS CHAPTER ....
approximately U-shaped.
Conic Section
A parabola is as a conic section, created from the intersection of a right
circular conical surface and a plane, which is parallel to a straight line on the Equation of Conic Section
conical surface and perpendicular to another plane which includes both the Recognisation of Conics
axis of the cone and also the same straight line on its surface. Parabola
Terms Related to All Parabolas
Conic Section (in Standard Form)
The locus of a point P which moves in a plane such that its distance from a Position of a Point with respect to
fixed point is always in a constant ratio to its perpendicular distance from a the Parabola
fixed straight line, is known as conic section. Equation of Chord
Intersection of Line and Parabola
P
M
Condition of Tangency
Tangent to the Parabola
Directrix

Normal to the Parabola


S (Focus)
Conormal Points
Pair of Tangents

The fixed point is called the focus of the conic section and this fixed line is Chord of Contact
called the directrix of the conic section. Also, this constant ratio is called the
eccentricity of the conic section and is denoted by e. Thus,
PS
= Constant = e Þ PS = ePM
PM

Eccentricity Shape
e =0 Circle
0<e <1 Ellipse
e =1 Parabola
e >1 Hyperbola
e =¥ Pair of straight lines
352 JEE Main Mathematics

Equation of Conic Section Condition Nature of Conic

If the focus is (a, b ) and the directrix is ax + by + c = 0, D = 0 and ab - h2 = 0 A pair of straight lines or empty set
then the equation of the conic section whose eccentricity D = 0 and ab - h2 ¹ 0 A pair of intersecting straight lines
is e, is D = 0 and ab - h2 < 0 Real or imaginary pair of straight lines
D = 0 and ab - h2 > 0 Point
P(x, y)
M
Case II D = abc + 2 fgh - af 2 - bg2 - ch2 ¹ 0
ax+by+c=0

In this case Eq. (i) represents the non-degenerate conic.


S (a, b)
Condition Nature of Conic
D ¹ 0, h = 0, a = b ¹ 0 A circle
2
D ¹ 0, ab - h = 0 A parabola
D ¹ 0, ab - h2 > 0 An ellipse or empty set
|ax + by + c|
( x - a )2 + ( y - b )2 = e D ¹ 0, ab - h2 < 0 A hyperbola
a 2 + b2 D ¹ 0, ab - h2 < 0 and a + b = 0 A rectangular hyperbola
( ax + by + c)2
Þ ( x - a )2 + ( y - b )2 = e2
a 2 + b2

Some Important Definitions


(i) Centre The point which bisects every chord of the
conic passing through it, is called the centre of the
conic section.
(ii) Axis The straight line passing through the focus
and perpendicular to the directrix, is called axis of (a) (b) (c) (d)
Circle : Ellipse : plane not Parabola : Hyperbola :
the conic section. plane perpendicular to plane plane
(iii) Vertex The points of intersection of the conic perpendicular axis not parallel parallel parallel to
to cone axis to side of cone to side of cone cone axis
section and the axis, are called the vertices of the
conic section.
Parabola
(iv) Latusrectum The chord passing through the A parabola is the locus of a point which moves in a plane
focus and perpendicular to the axis is called the such that its distance from a fixed point (i.e. focus) is
latusrectum of the conic section. always equal to its distance from a fixed straight line
(v) Focal Chord A chord of a conic passing through (i.e. directrix).
the focus is called a focal chord. |PS |
Mathematically, = e=1
(vi) Double Ordinate A straight line drawn |PM |
perpendicular to the axis and terminated at both where, e is called eccentricity.
ends of the curve is a double ordinate of the conic Focal chord
Y
section. M
P

Focus
Recognisation of Conics A S
X
The equation of conics represented by general equation of Vertex
second degree Double
ax 2 + 2 hxy + by 2 + 2gx + 2 fy + c = 0 …(i) Directrix Latusrectum
ordinate

can be recognised easily by the condition given below. If the vertex of the parabola is ( h , k), then generalised
Case I When D = abc + 2 fgh - af 2 - bg2 - ch2 = 0 form of the parabola is ( y - k)2 = 4a( x - h )2. Its focus is at
In this case Eq. (i) represents the degenerate conic. ( a + h , k) equation of directrix is x + a - h = 0 and
parametric equations are x = h + at 2 , y = k + 2at.
Parabola 353

Terms Related to all Parabolas (in Standard Form)


(a) Equation of parabola y 2 = 4a x y 2 = - 4a x x 2 = 4 ay x 2 = - 4ay
(b) Graphs
S Z

Z A S S A Z A
A
Z S

(c) Eccentricity e =1 e =1 e =1 e =1
(d) Focus S ( a, 0) S ( - a, 0) S ( 0, a ) S ( 0, - a )
(e) Equation of directrix x+ a = 0 x-a = 0 y+ a = 0 y- a = 0
(f) Equation of axis y=0 y=0 x=0 x=0
(g) Vertex A(0, 0) A(0, 0) A(0, 0) A( 0, 0)
(h) Extremities of latusrectum ( a, ± 2 a ) ( - a, ± 2 a ) ( ±2 a, a ) ( ± 2 a, - a )
(i) Length of latusrectum 4a 4a 4a 4a
(j) Equation of tangent at vertex x=0 x=0 y=0 y=0
ì x = at 2 ì x = - at 2 ì x = 2 at ì x = 2 at
(k) Parametric equations í í í 2 í 2
î y = 2 at î y = 2 at î y = at î y = - at
(l) Parametric coordinates of any point on P( at 2,2 at ) P( - at 2,2 at ) P(2 at , at 2 ) P(2 at , - at 2 )
parabola
(m) Focal distance of any point P( h, k ) on the h+ a h- a k+ a k-a
parabola
(n) Equation of latusrectum x=a x+ a = 0 y=a y+ a = 0

Example 1. If the three normals drawn to the parabola, Y


2
y = 2 x pass through the point ( a, 0) a ¹ 0, then ‘a’ must be
greater than (JEE Main 2021)
k)
1 1
h,
(a) (b) - Q (4t,2t2)
1: P (

2 2 3
(c) - 1 (d) 1
X¢ X
(0, 0) O
Sol. (d) For standard parabola for more than 3 normals (on axis)
L
x > (where L is length of LR) Y¢
2
For y 2 = 2x Let P(h, k) be the point which divides the line segment joining
(0,0) and ( 4t , 2t 2) in the ratio 1 : 3.
LR = 2
For (a, 0) 1 ´ 4t + 3 ´ 0
\ h= Þ h =t
LR 4
a> Þ a >1
2 1 ´ 2t 2 + 3 ´ 0 t2 1
and k= Þ k= Þ k = h 2 [Q t = h]
4 2 2
Example 2. Let O be the vertex and Q be any point on the Þ 2k = h 2 Þ 2y = x2,
parabola x 2 = 8y. If the point P divides the line segment OQ
which is required locus.
internally in the ratio 1: 3, then the locus of P is (JEE Main 2015)
(a) x2 = y (b) y 2 = x Example 3. Axis of a parabola lies along X-axis. If its vertex
2
(c) y = 2x 2
(d) x = 2y and focus are at distances 2 and 4 respectively from the
origin, on the positive X-axis, then which of the following
Sol. (d) Equation of parabola is points does not lie on it? (JEE Main 2019)
x2 = 8y …(i) (a) (4, -4) (b) (6, 4 2)
Let any point Q on the parabola (i) is ( 4t , 2t 2). (c) (8, 6) (d) (5, 2 6)
354 JEE Main Mathematics

Sol. (c) According to given information, we have the following Equation of the Chord
figure.
Y Let P ( at12 , 2 a t1 ) and Q( at22 , 2 a t2 ) be any two points on the
parabola y 2 = 4ax such that PQ is a focal chord.
X Then, the equation of chord is
(2, 0) (4, 0)
y ( t1 + t2 ) = 2x + 2at1t2
Y P(at12 ,2at1)
Now, if the origin is shifted to (2, 0) and ( X , Y) are the
coordinates with respect to new origin, then equation of
parabola is Y 2 = 4aX , X
A
where, X = x - 2 and Y = y and a = 4 - 2 = 2
Q(at 2
\ y 2 = 8( x - 2) 2 ,2at
2)
Note that (8, 6) is the only point which does not satisfy the
equation. If the chord PQ is a focal chord of the parabola, then ( a , 0)
must satisfy this equation
Example 4. The area (in sq. units) of an equilateral triangle 0 = 2a + 2at1t2 Þ t1t2 = - 1
inscribed in the parabola y 2 = 8 x, with one of its vertices on
Thus, if t is the parameter for one end of a focal chord,
the vertex of this parabola, is (JEE Main 2020) -1
(a) 64 3 (b) 256 3
then parameter for other end is and the coordinates of
t
(c) 192 3 (d) 128 3 the end points of a focal chord PQ of the parabola
Sol. (c) Equation of given parabola is y 2 = 8x and the equilateral æ a -2a ö
y 2 = 4ax can be taken as P ( at 2 , 2at ) and Q ç 2 , ÷.
triangle inscribed in the given parabola having one of its èt t ø
vertices on the vertex of the parabola, so according to
symmetry other two vertices of the equilateral triangle, Length of the Focal Chord
we can take as, A (2t 2,4t ) and B (2t 2, 4t )
1 3 Let P ( at 2 , 2at ) be the one end of a focal chord PQ of the
\ Area of D OAB is D = (2t 2) (8t ) = (8t ) 2
2 4 parabola y 2 = 4ax. Then, the lengths of focal chord is
2
Y A(2t2, 4t) y2=8x æ 1ö
a ç t + ÷ , where t is the parameter for one end of the
è tø
chord.
8t
O
X If l1 and l2 are two lengths of focal segments.
l1l2
Then, 4a =
l1 + l2
B(2t2, –4t)
i.e. Length of latusrectum
Þ t =2 3 = 2 (Harmonic mean of the focal segments)
3 Note The length of focal chord having parameters t1 and t 2 for its
\Area of equilateral DOAB = (8 ´ 2 3) 2
4 end points is a (t 2 - t1 )2 .
= 192 3 sq unit
Example 5. The length of focal chord of parabola y 2 = 4ax
Position of a Point with respect makes an angle a with the X-axis, is
to the Parabola (a) 4a cosec a (b) 4 cosec2a
2
Let S º y 2 - 4ax be the equation of the parabola and (c) 4a cosec a (d) None of these
P ( x1 , y1 ), be any point in the region of the parabola, then Sol. (c) Let P( at12, 2at1) and Q( at 22, 2at 2) be the end points of a
S1 º y12 - 4ax1 focal chord PQ which makes an angle a with the axis of the
If S1 > 0, then P lies outside the parabola. parabola. Then,
If S1 = 0, then P lies on the parabola. PQ = a (t 2 - t1) 2 = a [(t 2 + t1) 2 - 4t1t 2]
If S1 < 0, then P lies inside the parabola. = a [(t 2 + t1) 2 + 4] [Qt1t 2 = - 1 ] …(i)
Parabola 355

2at 2 - 2at1 (ii) If D > 0 i.e. a = cm, then line intersect the parabola
Q tan a = Slope of PQ =
at 22 - at12 at two coincident points i.e. at one point.
2 y 2 = 4ax
Þ tan a =
t 2 + t1
Þ t 2 + t1 = 2 cot a …(ii)

y=
On substituting the value of t 2 + t1 from Eq. (ii) in Eq. (i),

mx
we get

+
c
PQ = a ( 4 cot 2 a + 4) = 4a cosec2a
(iii) If D < 0 i.e. a < cm, then line neither touch nor
Example 6. If one end of a focal chord of the parabola, intersect the parabola.
y 2 = 16 x is at (1, 4), then the length of this focal chord is y 2 = 4ax
(JEE Main 2019)
(a) 22 (b) 25

y=
(c) 24 (d) 20

mx
Sol. (b) Equation of given parabola is y 2 = 16x, its focus is ( 4, 0).

+c
Since, slope of line passing through ( x1, y1) and ( x2, y 2) is given
y -y
by m = tan q = 2 1.
x2 - x1 Example 7. If the line y - 3x + 3 = 0 cuts the parabola
\ Slope of focal chord having one end point is (1, 4) is y 2 = x + 2 at A and B, then PA × PB is equal to
4 -0 4 [where, P = ( 3, 0)]
m = tan a = =-
1- 4 3 4( 3 + 2) 4(2 - 3)
(a) (b)
[where, ‘a’ is the inclination of focal chord with X-axis] 3 3
Since, the length of focal chord = 4a cosec2a 4 3 2( 3 + 2)
(c) (d)
3 3
\ The required length of the focal chord = 16 [1 + cot 2 a ]
Sol. (a) Given, P = ( 3 , 0)
[Q a = 4 and cosec2 a = 1 + cot 2 a ]

é 9ù é 1 3ù B
= 16 ê1 + ú = 25 units êëQ cot a = tan a = - 4 úû
ë 16 û
A
Intersection of a Line and a Parabola 60°

Let the parabola be y 2 = 4ax …(i) \Equation of line AB is


and the given line be y = mx + c …(ii) x- 3 y -0
= = r (say)
cos 60° sin 60°
On eliminating x from Eqs. (i) and (ii), we get
r
æ y - cö Þ x= 3 +
y 2 = 4a ç ÷ 2
è m ø
2 r 3
Þ my - 4ay + 4ac = 0 …(iii) and y=
3
It is a quadratic equation in y. æ r r 3ö
Discriminant, D = ( -4a )2 - 4( 4ac)m
\Point ç 3 + , ÷ lie on y 2 = x + 2.
è 2 2 ø
= 16a( a - cm ) 3r 2 r
\ = 3+ +2
(i) If D > 0 i.e. a > cm, then line intersect the parabola 4 2
at two distinct points. 3r 2 r
Þ - - (2 + 3) = 0
4 2
y 2 = 4ax
Let the roots be r1 and r2.
½-(2 + 3)½
y=

\Product of roots = PA × PB = rr
1 2 =½ ½
mx

½ 3/4 ½
+c

4(2 + 3)
=
3
356 JEE Main Mathematics

Example 8. The length of the chord of the parabola x 2 = 4y Since, the line is a tangent to the parabola.
having equation x - 2y + 4 2 = 0 is (JEE Main 2019)
\ D=0
4[( mc - 2a )2 - 4m 2c2 = 0 [ [Q D = B2 - 4 AC ]
(a) 8 2 (b) 2 11 (c) 3 2 (d) 6 3
Þ 4[m 2c2 + 4a 2 - 4amc] - 4m 2c2 = 0
2
Sol. (d) Given, equation of parabola is x = 4y … (i)
Þ 4[4a 2 - 4amc] = 0
and the chord is x - 2y + 4 2 = 0 … (ii)
Þ 4a 2 = 4amc
From Eqs. (i) and (ii), we have
[ 2(y - 4)]2 = 4y Þ 4a( a - mc) = 0
Þ 2
2(y - 4) = 4y Þ mc = a
a
Þ (y - 4) 2 = 2y Þ c=
2 m
Þ y - 8y + 16 = 2y
Þ y 2 - 10y +16 = 0 …(iii) Example 9. If y = mx + 4 is a tangent to both the parabolas,
Y y 2 = 4x and x 2 = 2 by, then b is equal to (JEE Main 2020)

x– Ö 2 y+4 Ö 2 =0 (a) -32 (b) -128 (c) -64 (d) 128


A
(x1 , y1 )
Sol. (b) As we know, equation of tangent to the parabola y 2 = 4x,
B (x2 , y2 )
X having slope ‘m’ is
O
1
y = mx + …(i)
m
On comparing the Eq. (i) with the equation of given tangent
Let the roots of Eq. (iii) be y1 and y 2.
y = mx + 4, we get
Then, y1 + y 2 = 10 and y1y 2 = 16 … (iv)
1 1
Again from Eqs. (i) and (ii), we have = 4 Þm =
m 4
é x ù 1
x2 = 4 ê + 4ú \Equation of the tangent is y = x + 4, now it is tangent to
ë 2 û 4
Þ x2 - 2 2x - 16 = 0 … (v) the parabola x2 = 2by, so on solving the equation of parabola
1
Let the roots of Eq. (v) be x1 and x2 x2 = 2by and tangent y = x + 4, we must get only a
Then, x1 + x2 = 2 2 and x1x2 = -16 … (vi) 4
common point, so
Clearly, length of the chord AB æ1 ö
x2 = 2bç x + 4÷ Þ 2x2 - bx - 16b = 0
= ( x1 - x2) 2 + (y1 - y 2) 2 è4 ø
= ( x1 + x2) 2 - 4x1x2 + (y1 + y 2) 2 - 4y1y 2 is a quadratic equation having one solution.
2 2
[Q ( a - b) = ( a + b) - 4ab] So, D = 0 Þ b 2 + 4(2)(16b) = 0

= 8 + 64 + 100 - 64 Þ b = -128 [Q b cannot be zero]

= 108 [from Eqs. (iv) and (vi)]


=6 3
Tangent to the Parabola
A line which intersect the parabola at only one point is
called the tangent to the parabola.
Condition of Tangency
Let the line y = mx + c is tangent to the parabola Equation of Tangent in Different Forms
y 2 = 4ax.
Point Form
y2 = 4ax The equation of the tangent to the parabola y 2 = 4ax at a
point ( x1 , y1 ) is yy1 = 2a ( x + x1 ). The equation of tangents
y=m

to all standard forms of parabola at point ( x1 , y1 ) are


x+c

given below.
Equation of parabola Equation of tangent
\ ( mx + c)2 = 4ax 2
y = - 4ax yy1 = - 2 a( x + x1 )
Þ 2 2 2
m x + c + 2mcx = 4ax x 2 = 4ay xx1 = 2 a ( y + y1 )
x 2 = - 4ay xx1 = - 2 a ( y + y1 )
Þ m 2x 2 + 2( mc - 2a )x + c2 = 0
Parabola 357

The equation of tangent at ( x1 , y1 ) to any second degree Thus, the coordinates of the point of intersection of
curve can also be obtained by replacing x 2 by xx1 , y 2 by tangents at ( at12 , 2at1 ) and ( at22 , 2at2 ) are [at1t2 , a ( t1 + t2 )].
x + x1 y + y1 xy + x1 y
yy1 , x by , y by and xy by 1 and )
2 2 2 Y 2 at 1
2
without changing the constant (if any) in the equation of a t 1,
P(
the curve.
X¢ X
O
Slope Form R
(at1t2, a(t1 + t2)) Q(at 2
The equation of a tangent to the parabola y 2 = 4ax in Y¢ 2 , 2 at
2)
a
slope form is y = mx + , where m is the slope of the
m
Note
tangent. The equation of all tangent in slope form are
given below. • x-coordinate, at1 t 2 = GM of at12 , at 22
• y-coordinate, a(t1 + t 2 ) = AM of 2 at1, 2 at 2
Equation of
Point of contact Equation of tangent
parabola Important Results on Tangents
2
y = - 4ax æ - a , -2 a ö a (i) The tangent at any point on a parabola bisects the
ç 2 ÷ y = mx -
èm m ø m angle between the focal distance of the point and
the perpendicular on the directrix from the point.
x 2 = 4ay (2 am, am2 ) y = mx - am2
(ii) The tangents at the extremities of any focal chord
x 2 = - 4ay ( -2 am, - am2 ) y = mx + am2 of a parabola intersect at right angles on the
directrix.
The equation of tangent of slope m to the parabola (iii) The portion of the tangent to a parabola cut-off
a
( y - k)2 = 4a ( x - h ) is given by y - k = m( x - h ) + and between the directrix and the curve subtends a
m right angle at the focus.
æ a 2a ö
coordinates of point of contact are ç h + 2 , k + ÷. (iv) The perpendicular drawn from the focus on any
è m mø tangent to a parabola intersect it at the point,
where it cuts the tangent at the vertex.
Parametric Form (v) The orthocentre of any triangle formed by three
The equation of tangent at the point ( at 2 , 2at ) or t to the tangents to a parabola lies on the directrix.
parabola y 2 = 4ax is ty = x + at 2. The parametric equation (vi) The tangent at any point of a parabola is equally
inclined to the focal distance of the point and the
of tangents to all standard forms of parabola are given
axis of the parabola.
below.
Example 10. If one end of a focal chord AB of the parabola
Equation of
Point of contact Equation of tangent æ1 ö
parabola y 2 = 8 x is at A ç , - 2 ÷, then the equation of the tangent to it
è2 ø
y 2 = - 4ax ( -at 2, 2 at ) ty = - x + at 2
at B is (JEE Main 2020)
x 2 = 4ay (2 at , at 2 ) tx = y + at 2
(a) x - 2y + 8 = 0 (b) x + 2y + 8 = 0
x 2 = - 4ay (2 at , - at 2 ) tx = - y + at 2
(c) 2x + y - 24 = 0 (d) 2x - y - 24 = 0
Sol. (a) Equation of given parabola y 2 = 8x and one end of a focal
Point of Intersection of Tangents at
æ1 ö
Any Two Points on the Parabola chord AB is Aç , - 2÷ . As, we know, if one end of a focal
è2 ø
Let the parabola be y 2 = 4ax. Let two points on the chord of parabola y = 4ax is ( at 2, 2at ), then other end will be
2

æ ö
parabola be P º ( at12 ,2at1 ) and Q º ( at22 ,2at2 ). ç ÷
æ a -2 a ö ç 2 , -4 ÷ = B(8, 8)
Equation of tangents at P ( at12 , 2at1 ) and Q ( at22 , 2at2 ) are ç 2 , ÷ , so other end point B
ç æ 1 ö 2 -1 ÷
èt t ø
ç ç- ÷ 2 ÷ø
t1 y = x + at12 è è 2ø
and t2 y = x + at22 Now, equation of tangent of parabola y 2 = 8x at point B(8, 8) is
On solving these equations, we get T =0
Þ 8y = 4( x + 8)
x = at1t2 and y = a ( t1 + t2 )
Þ x - 2y + 8 = 0
358 JEE Main Mathematics

x y The line y = mx + c will be a normal to the parabola


Example 11. If the line + = 1touches the parabola
l m y 2 = 4ax, if c = - 2am - am3 and point of contact are
y 2 = 4a( x + b), then m 2(l + b) is equal to ( am 2 , - 2am ).
2 2 2
(a) al (b) - al (c) l (d) - a
Parametric Form
Sol. (b) The given parabola is
y 2 = 4a ( x + b) …(i)
The equation of normal at the point ( at 2 ,2 at ) to the
parabola y 2 = 4ax is y + tx = 2 at + at3 . The parametric
Vertex of this parabola is ( - b, 0).
equation of normal to all standard form of parabola are
Now, shifting (0, 0) at ( - b, 0), then
given below.
x = X + ( - b) and y = Y + 0
Þ x + b = X and y = Y …(ii) Equation of
Point of contact Equation of normal
From Eq. (i), Y 2 = 4aX …(iii) parabola
x y y 2 = - 4ax ( -at 2, 2 at ) y = tx + 2 at + at 3
and the line + =1
l m 2 2
x = 4ay (2 at , at ) x = - ty + 2 at + at 3
X -b Y æ X - bö
reduces to + = 1 Þ Y = m ç1 - ÷ x 2 = - 4ay (2 at , - at 2 ) x = ty + 2 at + at 3
l m è l ø
æ mö æ bö
Þ Y = ç - ÷ X + m ç1 + ÷
è lø è
…(iv) Point of Intersection of Normals Drawn

Any Two Points on the Parabola
The line (iv) will touch the parabola (iii), if
æ bö a If two normals are drawn to the parabola y 2 = 4ax at the
m ç1+ ÷ =
è l ø æ mö points P ( at12 , 2 at1 ) and Q( at22 , 2 at2 ) intersect at a point R,
ç- ÷
è lø then coordinates of the point R are
2
m æ bö 2 2 [2a + a( t12 + t22 + t1t2 ), - at1t2 ( t1 + t2 )].
Þ ç1+ ÷ = - a Þ m (l + b) + al = 0
l è lø
Y
2 , 2at 1)
Þ m2 (l + b) = - al 2 P(at 1

R
Normal to the Parabola X¢
A X

A line which is perpendicular to the tangent of the


Q(at 2
parabola is called the normal to the parabola. Y¢ 2 , 2at
2)

Equation of Normal in Different Forms Important Results on Normals


Point Form (i) If the normal drawn at the point P ( at12 , 2at1 ) to the
The equation of normal at the point ( x1 , y1 ) to the parabola y 2 = 4ax meets the parabola again at
parabola y 2 = 4ax is 2
y Q( at22 , 2at2 ), then t2 = - t1 - .
( y - y1 ) = - 1 ( x - x1 ) t1
2a
(ii) The tangent at one extremity of the focal chord of a
Slope Form parabola is parallel to the normal at the other
The equation of normal at the point ( am 2 , - 2am ) to the extremity.
parabola y 2 = 4ax in the slope form is (iii) If the normals at point P ( at12 , 2at1 ) and Q( at22 , 2at2 )
y = mx - 2am - am3 on the parabola y 2 = 4ax meet on the parabola, then
The equation of normals to various standard forms of the t1t2 = 2 .
parabola in terms of the slope of the normal are as given (iv) If the normal chord at a point P ( at 2 , 2at ) to the
below. parabola y 2 = 4ax subtends a right angle at the
Equation of parabola Equation of normal Point of contact vertex of the parabola, then t 2 = 2 .
2
y = - 4ax y = mx + 2 am + am 3
( -am2, 2 am) (v) The normal chord of a parabola at the point whose
ordinate is equal to the abscissa, subtends a right
y = mx + 2 a +
a æ - 2a, a ö
x 2 = 4ay ç ÷ angle at the focus.
m2 è m m2 ø
(vi) The tangent and normal at any point of a parabola
y = mx - 2 a -
a æ2 a , - a ö
x 2 = - 4ay ç ÷ are equally inclined to the focal distance to the point
m2 è m m2 ø
and the axis of the parabola.
Parabola 359

Example 12. Tangent and normal are drawn at P(16, 16) on Results on Conormal Points
the parabola y 2 = 16 x, which intersect the axis of the parabola (i) The algebraic sum of the slopes of the normals at
at A and B, respectively. If C is the centre of the circle through conormal points is zero.
the points P, A and B and ÐCPB = q, then a value of tanq is (ii) The sum of the ordinates of the conormal points is
(JEE Main 2018) zero.
1 4 (iii) The centroid of the triangle formed by the conormal
(a) (b) 2 (c) 3 (d)
2 3 points on a parabola lies on the axis of the parabola.
Sol. (b) Equation of tangent and normal to the curve y 2 = 16x at Example 13. Find the locus of the point through which pass
(16, 16) is x - 2y + 16 = 0 and 2x + y - 48 = 0 , respectively. three normals to the parabola y 2 = 4ax such that two of them
Y make angles a and b respectively with the axis such that
P tan a tan b = 2.
(16, 16)
2x
=0 q +
y–
(a) x2 - 4ay = 0 (b) y 2 - 4ax = 0
6
y +1 48 (c) x2 + 4ay = 0 (d) y 2 + 4ax = 0
x–2 =
0
X¢ X
A(–16, 0) C(4, 0) B(24, 0) Sol. (b) Let (h, k) be the point of intersection of three normals to
the parabola y 2 = 4ax. The equation of any normal to y 2 = 4ax is
y = mx - 2am - am3
If it passes through (h, k), then k = mh - 2am - am3

Þ am3 + m(2a - h) + k = 0 …(i)
A = ( -16, 0) ; B = (24, 0)
C is the centre of circle passing through PAB i.e. C = ( 4, 0) Let roots of Eq. (i) be m1, m2, m3 , then from Eq. (i),
16 - 0 16 4 k
Slope of PC = = = = m1 m1m2 m3 = - …(ii)
16 - 4 12 3 a
16 - 0 16 Also, m1 = tan a , m2 = tan b and tan a tan b = 2 …(iii)
Slope of PB = = = - 2 = m2 \ m1m2 = 2
16 - 24 -8
k
½m - m2½ From Eqs. (ii) and (iii), 2m3 = -
tan q = ½ 1 ½ a
½1 + m1m2½ k
Þ m3 = -
½ 4 ½ 2a
½ 3 +2 ½ Which being a root of Eq. (i) must satisfy it
Þ tan q = ½
4 ½
½1 - æç ö÷(2)½ i.e., am33 + m3(2a - h) + k = 0
½ è3ø ½ æ kö
3
k
Þ tan q = 2 Þ aç- ÷ - (2a - h) + k = 0
è 2a ø 2a
k3 kh
Conormal Points Þ - -k+ + k=0
8 a2 2a
The points on the parabola through which normals Þ k2 - 4ah = 0
drawn are concurrent i.e. pass through the same point
\Required locus of (h, k) is y 2 - 4ax = 0
are called conormal points. The conormal points are also
called the feet of the normals. Points A, B, C in which the
Example 14. If the parabolas y 2 = 4b( x - c ) and y 2 = 8 ax
three normals from P ( h , k) meet the parabola are called
conormal points. have a common normal, then which one of the following is a
valid choice for the ordered triad ( a, b, c ) ? (JEE Main 2019)
æ1 ö
(a) ç , 2, 0 ÷ (b) (1, 1, 0)
A è2 ø
B æ1 ö
(c) (1, 1, 3) (d) ç , 2, 3÷
è2 ø
C P
Sol. (c) Normal to parabola y 2 = 4ax is given by
y = mx - 2am - am3
360 JEE Main Mathematics

\ Normal to parabola y 2 = 4b( x - c) is Since, lines (i) and (ii) pass through ( h , k), then
3
y = m( x - c) - 2bm - bm ky1 = 2 a ( h + x1 ) …(iii)
[replacing a by b and x by x - c] and ky2 = 2 a ( h + x2 ) …(iv)
= mx - (2b + c)m - bm3 … (i)
Y (x1, y1)
and normal to parabola y 2 = 8 ax is
Q
y = mx - 4am - 2am3 …(ii) Chord of
contact
[replacing a by 2a] X¢' X
A
For common normal, we should have P(h, k)
mx - 4am - 2am3 = mx - (2b + c)m - bm3 R
Y¢ (x2, y2)
[using Eqs. (i) and (ii)]
4am + 2am3 = (2b + c)m + bm3 Hence, it is clear Q ( x1 , y1 ) and R ( x2 , y2 ) lie on
Þ (2a - b)m3 + ( 4a - 2b - c)m = 0 yk = 2a ( x + h ) which is chord of contact QR.
Þ m((2a - b)m2 + ( 4a - 2b - c)) = 0
Þ m=0 Equation of the Chord Bisected
2b + c - 4a
2 c at a Given Point
or m = = -2
2a - b 2a - b The equation of the chord of the parabola y 2 = 4ax which
c is bisected at ( x1 , y1 ) is
As, m2 > 0, therefore >2
2a - b yy1 - 2a( x + x1 ) = y12 - 4ax1
Note that if m = 0, then all options satisfy (Q y = 0 is a
or T = S1
common normal) and if common normal is other than the
axis, then only option (c) satisfies. where, T = yy1 - 2a( x + x1 ) and S1 = y12 - 4ax1
é c 3 ù
êQ for option (c), = = 3 > 2ú Example 15. The tangents to the curve y = ( x - 2) 2 - 1 at its
ë 2a - b 2 - 1 û
points of intersection with the line x - y = 3, intersect at the
point (JEE Main 2019)
Pair of Tangents æ5 ö æ 5 ö
(a) ç , 1÷ (b) ç - , - 1÷
Let P ( x1 , y1 ) be any point outside the parabola è2 ø è 2 ø
S = y 2 - 4ax, then the equation of the pair of tangents æ5 ö æ 5 ö
drawn from the point P to the parabola is given by (c) ç , - 1÷ (d) ç - , 1÷
è2 ø è 2 ø
Y
Sol. (c) Given equation of parabola is
Q
P(x1, y1) y = ( x - 2) 2 - 1

A S
X Þ y = x2 - 4x + 3 …(i)
R Now, let (x1, y1) be the point of intersection of tangents of
parabola (i) and line x - y = 3, then
Y¢ Equation of chord of contact of point ( x1, y1) w.r.t. parabola (i)
SS1 = T 2 is
T =0
where, S = y 2 - 4ax, S1 = y12 - 4ax1 1
Þ (y + y1) = xx1 - 2( x + x1) + 3
and T = yy1 - 2a ( x + x1 ) 2
Þ y + y1 = 2x ( x1 -2) - 4x1 + 6
Chord of Contact
Þ 2x( x1 - 2) - y = 4 x1 + y1 - 6,
Let PQ and PR be tangents to the parabola y 2 = 4ax this equation represent the line x - y = 3 only, so on
drawn from any external point P ( h , k), then QR is called comparing, we get
chord of contact of the parabola y 2 = 4ax.
2( x1 - 2) - 1 4x1 + y1 - 6
= =
Let Q º ( x1 , y1 ) and R º ( x2 , y2 ) 1 -1 3
Equation of the tangent PQ is 5
Þ x1 = and y1 = - 1
yy1 = 2 a ( x + x1 ) …(i) 2
and equation of the tangent PR is æ5 ö
So, the required point is ç , - 1÷ .
è2 ø
yy2 = 2 a ( x + x2 ) …(ii)
Parabola 361

Practice Exercise
ROUND I Topically Divided Problems
Basic Terms of Parabola and 8. The length of the latusrectum of the parabola
Equation of Parabola 2{( x - a) 2 + ( y - a) 2 } = ( x + y) 2 is
(a) 2a (b) 2 2a (c) 4a (d) 2a
1. The equation of latusrectum of a parabola is
2
x + y = 8 and the equation of the tangent at the 9. AB is a focal chord of x - 2 x + y - 2 = 0 whose focus
vertex is x + y = 12 , then length of the latusrectum is ‘S’. If AS = l1 then BS is equal to
is 4l1 l1
(a) 4 2 (b) 2 2 (c) 8 (d) 8 2 (a) (b)
4l1 - 1 4l1 - l
2. The locus of the mid-point of the line segment 2l1
joining the focus to a moving point on the parabola (c) (d) None of these
4l1 - 1
y 2 = 4 ax is another parabola with the directrix
(a) x = - a (b) x = -
a 10. The latusrectum of the parabola y 2 = 4 ax whose
2 focal chord is PSQ such that SP = 3 and SQ = 2 , is
a
(c) x = 0 (d) x = given by
2 24 12 6 1
(a) (b) (c) (d)
3. The locus of a point which divides the line segment 5 5 5 5
joining the point (0, - 1) and a point on the 11. If a focal chord of the parabola y 2 = ax is
parabola, x 2 = 4 y, internally in the ratio 1 : 2, is 2 x - y - 8 = 0, then the equation of the directrix is
(JEE Main 2020)
(a) x + 4 = 0 (b) x - 4 = 0
(a) 4x2 - 3 y = 2 (b) x2 - 3 y = 2
(c) y - 4 = 0 (d) y + 4 = 0
(c) 9x2 - 12 y = 8 (d) 9x2 - 3 y = 2
12. The length of the chord of the parabola x 2 = 4 y
4. If the vertex = ( 2, 0) and the extremities of the
passing through the vertex and having slope cot a
latusrectum are (3, 2) and (3, - 2), then the
is
equation of the parabola is
(a) y2 = 2x - 4 (b) x2 = 2 y - 8 (a) 4 cos a cosec2a (b) 4 tan a sec a
(c) y2 = 4x - 8 (d) None of these (c) 4 sin a sec2 a (d) None of these

5. The vertex of the parabola y 2 = 8 x is at the centre 13. The line x - 1 = 0 is the directrix of the parabola
of a circle and the parabola cuts the circle at the y 2 - kx + 8 = 0. Then, one of the value of k is
ends of its latusrectum. Then, the equation of the 1 1
(a) (b) 8 (c) 4 (d)
circle is 8 4
(a) x2 + y2 = 4 (b) x2 + y2 = 20
Intersection of a Parabola
(c) x2 + y2 = 80 (d) None of these
and a Curves
6. The length of the latusrectum of the parabola
14. The angle made by a double ordinate of length 8a
169 {( x - 1) 2 + ( y - 3) 2 } = (5 x - 12 y + 17) 2 is
(a) 14/13 (b) 12/13
at the vertex of the parabola y 2 = 4 ax is
p p p p
(c) 28/13 (d) None of these (a) (b) (c) (d)
3 2 4 6
7. The parametric equation of a parabola is x = t 2 + 1,
y = 2 t + 1. The cartesian equation of its directrix is 15. The angle of intersection between the curves
(a) x = 0 (b) x + 1 = 0 x 2 = 4 ( y + 1) and x 2 = - 4 ( y + 1) is
(c) y = 0 (d) None of these p p p
(a) (b) (c) 0 (d)
6 4 2
362 JEE Main Mathematics

16. The parabola y 2 = kx makes an intercept of length 25. Equation of a common tangent to the circle,
4 on the line x - 2 y = 1. Then, k is equal to x 2 + y 2 - 6 x = 0 and the parabola, y 2 = 4 x, is
(JEE Main 2019)
105 - 5 5 - 105
(a) (b) (a) 3 y = 3x + 1 (b) 2 3 y = 12x + 1
10 10
(c) 3 y = x + 3 (d) 2 3 y = - x - 12
5 + 105
(c) (d) None of these
10 26. If tangents at A and B on the parabola y 2 = 4 ax
17. The chord AB of the parabola y 2 = 4 ax cuts the axis intersect at point C, then ordinates of A, C and B
of the parabola at C. If A = ( at12 ,
2 at1), are
2 (a) always in AP (b) always in GP
B = ( at2 , 2 at2 ) and AC : AB = 1 : 3, then
(c) always in HP (d) None of these
(a) t2 = 2t1 (b) t2 + 2t1 = 0
(c) t1 + 2t2 = 0 (d) None of these 27. The number of real tangents that can be drawn to
18. An equilateral DSAB is inscribed in the parabola the curve y 2 + 2 xy + x 2 + 2 x + 3 y + 1 = 0 from the
y 2 = 4 ax having its focus at S. If chord AB lies point (1, - 2) is
(a) one (b) two
towards the left of S, then side length of this
(c) zero (d) None of these
triangle is
(a) 2a (2 - 3 ) (b) 4a (2 - 3 ) 28. Number of common tangents to the parabola
(c) a(2 - 3 ) (d) 8a(2 - 3 ) y 2 = 4 ax and x 2 = 4 by is
(a) 4 (b) 3 (c) 2 (d) 1
Tangent to the Parabola 2
29. If the tangent to the parabola y = ax makes an
19. If the line x cos a + y sin a = p touches the parabola
angle of 45° with X-axis, then the point of contact is
y 2 = 4 ax, then point of contact is
æa aö æa aö æa aö æa aö
(a) (a tan 2 a , - 2a tan a ) (b) (a tan 2 a , 2a tan a ) (a) ç , ÷ (b) ç , ÷ (c) ç , ÷ (d) ç , ÷
è2 2ø è4 4ø è2 4ø è4 2ø
(c) (a tan a , 2a tan a ) (d) None of these
20. Two perpendicular tangents to y 2 = 4 ax always 30. Tangent at the vertex divides the distance between
directrix and latusrectum in the ratio
intersect on the line, if
(a) 1:1
(a) x = a (b) x + a = 0
(b) 1:2
(c) x + 2a = 0 (d) x + 4a = 0
(c) depends on directrix and focus
21. The equation of the tangent to the parabola y 2 = 9 x (d) None of the above
which goes through the point (4, 10), is
31. The tangent drawn at any point P to the parabola
(a) x + 4 y + 1 = 0 (b) 9x + 4 y + 4 = 0
y 2 = 4 ax meets the directrix at the point K , then
(c) x + 4 y + 36 = 0 (d) 9x - 4 y + 4 = 0
the angle which KP subtends at its focus is
22. The equation of a tangent to the parabola, x 2 = 8 y, (a) 30° (b) 45° (c) 60° (d) 90°
which makes an angle q with the positive direction
32. If the tangents at P and Q on the parabola meet in
of X-axis, is (JEE Main 2019)
T, then SP, ST and SQ are in
(a) y = x tan q - 2 cot q (b) x = y cot q + 2 tan q
(c) y = x tan q + 2 cot q (d) x = y cot q - 2 tan q (a) AP (b) GP
(c) HP (d) None of these
23. If the line ax + by + c = 0 is a tangent to the
33. AB, AC are tangents to a parabola y 2 = 4 ax;
parabola y 2 - 4 y - 8 x + 32 = 0, then
p1, p 2 , p3 are the lengths of the perpendiculars
(a) 4b2 = a (7a + 2c + 4b)
from A, B, C on any tangent to the curve, then
(b) 4b2 = a (7a + c - 4b)
(c) 4b2 = a (7a + 2c + b)
p 2 , p1, p3 are in
(d) 4b2 = a (7a + 2c - b) (a) AP (b) GP
(c) HP (d) None of these
24. If two tangents drawn from the point (a, b) to the
parabola y 2 = 4 x is such that the slope of one Normal to the Parabola
tangent is double of the other, then 34. The normal to the parabola y 2 = 8 x at the point
2 2 (2, 4) meets the parabola again at the point
(a) b = a 2 (b) a = b 2
9 9 (a) (- 18, - 12) (b) (- 18, 12)
(c) 2a = 9 b 2 (d) None of these (c) (18, 12) (d) (18, - 12)
Parabola 363

43. The shortest distance between the point æç , 0 ö÷ and


35. Three distinct normals to the parabola y 2 = x are 3
è2 ø
drawn through a point ( c, 0), then
1 1 the curve y = x , ( x > 0), is (JEE Main 2019)
(a) c = (b) c =
4 2 3 5
(a) (b)
1 2 4
(c) c > (d) None of these
2 3 5
(c) (d)
36. If the normals at two points P and Q of a parabola 2 2
y 2 = 4 ax intersect at a third point R on the curve, 44. Find the length of the line segment joining the
then the product of ordinates of P and Q is vertex of the parabola y 2 = 4 ax and a point on the
(a) 4a 2 (b) 2a 2 parabola where the line segment makes an angle q
(c) - 4a 2 (d) 8a 2 to the X-axis.
2a cos q 4a cos q
37. If the tangent and normal at any point P of a (a) (b)
sin 2 q sin 2 q
parabola meet the axes in T and G respectively,
4a cos q
then (c) (d) None of these
3 sin 2 q
(a) ST ¹ SG = SP (b) ST - SG ¹ SP
(c) ST = SG = SP (d) ST = SG × SP 45. The equation of the parabola whose focus is the
2
38. If two different tangents of y = 4 x are the normals point (0, 0) and the tangent at the vertex is
x - y + 1 = 0, is
to x 2 = 4 by, then
1 1 (a) x2 + y2 - 2xy - 4x + 4 y - 4 = 0
(a) |b|> (b) |b|< (b) x2 + y2 - 2xy + 4x - 4 y - 4 = 0
2 2 2 2
1 1 (c) x2 + y2 + 2xy - 4x + 4 y - 4 = 0
(c) |b|> (d) |b|< (d) x2 + y2 + 2xy - 4x - 4 y + 4 = 0
2 2
46. The area (in sq units) of the smaller of the two
Chord of Contact circles that touch the parabola, y 2 = 4 x at the point
39. Equation of chord of the parabola y 2 = 16 x whose (1, 2) and the X-axis is (JEE Main 2019)

mid-point is (1, 1), is (a) 8p(3 - 2 2 ) (b) 4p (3 + 2 )


(a) x + y = 2 (b) x - y = 0 (c) 8p (2 - 2 ) (d) 4p (2 - 2 )
(c) 8x + y = 9 (d) 8x - y = 7 47. The equation of a common tangent to the curves,
40. The area of the triangle formed by the tangents y 2 = 16 x and xy = - 4, is (JEE Main 2019)
2
drawn from ( x1, y1) to y = 4 ax and their chord of (a) x - y + 4 = 0 (b) x + y + 4 = 0
contact is (c) x - 2 y + 16 = 0 (d) 2x - y + 2 = 0
( y1 - 4ax1 )3/ 2 ( y1 + x1 )3/ 2 48. On the parabola y = x 2 , the point atleast distance
(a) (b)
2a a from the straight line y = 2 x - 4 is
(2 y1 - 4ax1 )3/ 2 ( y1 + 2x1 )3/ 2
(c) (d) (a) (1, 1) (b) (1, 0)
3a 4a (c) (1, - 1) (d) (0, 0)
41. AB is a chord of the parabola y 2 = 4 ax with vertex 49. The shortest distance between the parabola y 2 = 4 x
A, BC is drawn perpendicular to AB meeting the and the circle x 2 + y 2 + 6 x - 12 y + 20 = 0 is
axis at C. The projection of BC on the axis of the (a) 4 2 - 5 (b) 0
parabola is (c) 3 2 + 5 (d) 1
(a) a (b) 2a
(c) 4a (d) 8a 50. The ends of a line segment are P(1, 3) and Q(1, 1).
R is a point on the line segment PQ such that
42. The focal chord to y 2 = 16 x is tangent to PR : QR = 1 : l. If R is an interior point of the
( x - 6) 2 + y 2 = 2 , then the possible values of the parabola y 2 = 4 x, then
slope of this chord are æ 3 ö
(a) l Î(0, 1) (b) l Î ç - , 1÷
(a) { - 1, 1} (b) { - 2 , 2} è 5 ø
ì 1ü ì 1ü æ1 3ö
(c) í - 2 , ý (d) í 2 , - ý (c) l Î ç , ÷ (d) None of these
î 2þ î 2þ è2 5ø
364 JEE Main Mathematics

ROUND II Mixed Bag


Only One Correct Option 9. The DPQR of area A is inscribed in the parabola
1. The shortest distance between the parabolas y 2 = 4 ax such that the vertex P lies at the vertex of
y 2 = 4 x and y 2 = 2 x - 6 is the parabola and the base QR is a focal chord. The
(a) 2 (b) 5 modules of the difference of the ordinates of the
(c) 3 (d) None of these points Q and R is
A A 2A 4A
2. A parabola is drawn with focus at (3, 4) and vertex (a) (b) (c) (d)
2a a a a
at the focus of the parabola y 2 - 12 x - 4 y + 4 = 0.
The equation of the parabola is
10. Let P be a point on the parabola, y 2 = 12 x and N be
(a) x 2 - 6x - 8 y + 25 = 0 (b) y 2 - 8x - 6 y + 25 = 0 the foot of the perpendicular drawn from P on the
(c) x 2 - 6x + 8 y - 25 = 0 (d) x 2 + 6x - 8 y - 25 = 0 axis of the parabola. A line is now drawn through
3. If the tangent at the point P ( 2, 4) to the parabola the mid-point M and PN, parallel to its axis which
meets the parabola at Q. If the y-intercept of the
y 2 = 8 x meets the parabola y 2 = 8 x + 5 at Q and R, 4
then the mid-point of QR is line NQ is , then
3 (JEE Main 2020)
(a) (3, 5) (b) (2, 4) 1
(c) (1, 4) (d) (4, 3) (a) PN = 4 (b) MQ =
3
4. Axis of a parabola is y = x and vertex and focus are 1
(c) MQ = (d) PN = 3
at a distance 2 and 2 2 respectively from the 4
origin. Then, equation of the parabola is 11. The shortest distance between the line y = x and
(a) (x - y) 2 = 8 (x + y - 2) curve y 2 = x - 2 is (JEE Main 2019)
(b) (x + y) 2 = 2 (x + y - 2) 7 7 11
(c) (x - y) 2 = 4 (x + y - 2) (a) 2 (b) (c) (d)
8 4 2 4 2
(d) (x + y) 2 = 2 (x - y + 2)
12. The tangent to the parabola y 2 = 4 x at the point
5. If the common tangent of the parabolas, y 2 = 4 x
where it intersects the circle x 2 + y 2 = 5 in the first
2 2 2 2
and x = 4 y also touches the circle, x + y = c , quadrant, passes through the point (JEE Main 2019)
then c is equal to (JEE Main 2020)
æ1 3ö æ3 7ö
1 1 1 1 (a) ç , ÷ (b) ç , ÷
(a) (b) (c) (d) è4 4ø è4 4 ø
2 4 2 2 2 æ 1 4ö æ 1 1ö
(c) ç - , ÷ (d) ç - , ÷
6. The centre of the circle passing through the point è 3 3ø è 4 2ø
(0, 1) and touching the parabola y = x 2 at the point 13. If the area of the triangle whose one vertex is at the
(2, 4) is (JEE Main 2020)
vertex of the parabola, y 2 + 4 ( x - a 2 ) = 0 and the
æ -53 16 ö æ 6 53 ö other two vertices are the points of intersection of
(a) ç , ÷ (b) ç , ÷
è 10 5 ø è 5 10 ø
the parabola and Y -axis, is 250 sq units, then a
æ 3 16 ö æ 16 53 ö
(c) ç , ÷ (d) ç - , ÷ value of ‘a’ is (JEE Main 2019)
è 10 5 ø è 5 10 ø
(a) 5 5 (b) 5 (c) 5(21/3 ) (d) (10)2/3
7. The locus of a point on the variable parabola 14. Equation of a common tangent to the parabola
y 2 = 4 ax, whose distance from focus is always equal
y 2 = 4 x and the hyperbola xy = 2 is (JEE Main 2019)
to k, is equal to (a is parameter)
(a) x + 2 y + 4 = 0 (b) x - 2 y + 4 = 0
(a) 4x 2 + y 2 - 4kx = 0 (b) x 2 + y 2 - 4kx = 0
(c) 4x + 2 y + 1 = 0 (d) x + y + 1 = 0
(c) 2x 2 + 4 y 2 - 8kx = 0 (d) 4x 2 - y 2 + 4kx = 0
15. If two tangents drawn from a point P to the
8. The length of the chord of the parabola y 2 = x
parabola y 2 = 4 x are at right angles, then the locus
which is bisected at the point (2, 1) is
of P is
(a) 2 3 (b) 4 3 (a) x = 1 (b) 2x + 1 = 0
(c) 3 2 (d) 2 5 (c) x = - 1 (d) 2x - 1 = 0
Parabola 365

16. A parabola has the origin as its focus and the line 20. If a ¹ 0 and the line 2 bx + 3cy + 4 d = 0 passes
x = 2 as the directrix. Then, the vertex of the through the points of intersection of the parabolas
parabola is at y 2 = 4 ax and x 2 = 4 ay, then
(a) (2, 0) (b) (0, 2) (a) d 2 + (2b + 3c) 2 = 0 (b) d 2 + (3b + 2c) 2 = 0
(c) (1, 0) (d) (0, 1) (c) d 2 + (2b - 3c) 2 = 0 (d) d 2 + (3b + 2c) 2 = 0

17. The equation of the tangent to the parabola y 2 = 8 x Numerical Value Type Questions
is y = x + 2. The point on this line from which the
21. Let a line y = mx ( m > 0) intersect the parabola,
other tangent to the parabola is perpendicular to
y 2 = x at a point P, other than the origin. Let the
the given tangent is
tangent to it at P meet the X-axis at the point Q.
(a) (0, 2) (b) (2, 4)
If area ( DOPQ) = 4 sq. units, then m is equal to
(c) (-2, 0) (d) (-1, 1)
........ (JEE Main 2020)
18. The locus of the vertices of the family of parabolas
a 3x 2 a 2 x 22. If the normals of the parabola y 2 = 4 x drawn at the
y= + - 2 a is end points of its latusrectum are tangents to the
3 2
35 64 circle ( x - 3) 2 + ( y + 2) 2 = r 2 , then the value of r 2 is
(a) xy = (b) xy =
36 105 ……… .

(c) xy =
105
(d) xy =
3 23. Let S be the focus of the parabola y 2 = 8 x and PQ
64 4 be the common chord of the circle
19. Let P be the point (1, 0) and Q a point on the locus x 2 + y 2 - 2 x - 4 y = 0 and the given parabola. The
y 2 = 8 x. The locus of mid-point of PQ is area of DPQS is ……… .
(a) x 2 - 4 y + 2 = 0 24. If x + y = k is normal to y 2 = 12 x, then k is ……… .
2
(b) x + 4 y + 2 = 0
25. If the tangent at (1, 7) to the curve x 2 = y - 6
(c) y 2 + 4x + 2 = 0
touches the circle x 2 + y 2 + 16 x + 12 y + c = 0, then
(d) y 2 - 4x + 2 = 0
the value of c is ……… (JEE Main 2018)

Answers
Round I
1. (d) 2. (c) 3. (c) 4. (c) 5. (b) 6. (c) 7. (a) 8. (b) 9. (b) 10. (a)
11. (a) 12. (a) 13. (c) 14. (b) 15. (c) 16. (a) 17. (b) 18. (b) 19. (a) 20. (b)
21. (d) 22. (b) 23. (a) 24. (b) 25. (c) 26. (a) 27. (c) 28. (d) 29. (d) 30. (a)
31. (d) 32. (b) 33. (b) 34. (d) 35. (c) 36. (d) 37. (c) 38. (b) 39. (d) 40. (a)
41. (c) 42. (a) 43. (d) 44. (b) 45. (c) 46. (a) 47. (a) 48. (a) 49. (a) 50. (a)

Round II
1. (b) 2. (a) 3. (b) 4. (a) 5. (a) 6. (d) 7. (a) 8. (d) 9. (c) 10. (c)
11. (c) 12. (b) 13. (b) 14. (a) 15. (c) 16. (c) 17. (c) 18. (c) 19. (d) 20. (a)
21. (0.5) 22. (2) 23. (4) 24. (9) 25. (95)
366 JEE Main Mathematics

Round I Clearly, the directrix is perpendicular to VS whose


1. Since, the equation of latusrectum and equation of equation is y = 0.
tangent both are parallel and they lie in the same side of So, the directrix is x = k which passes through M (1. 0).
the origin Therefore, we get x = 1.
- 8 + 12 4 \The equation of the parabola is
\ a= = =2 2 2
2
1 +1 2 2 æ x - 1ö
(x - 3)2 + ( y - 0)2 = ç ÷ Þ y2 = 4 x - 8
\ Length of latusrectum = 4a = 4 (2 2 ) = 8 2 è 1 ø

2. Let any point on the parabola be (at 2, 2 at ). 5. Vertex = (0, 0)

If the equation of parabola is y2 = 4ax, then focus is The ends of latusrectum are (2, 4) and (2, - 4).
(a , 0). \Centre of circle is (0, 0)
Let the locus of a point be (a, b), if it is a mid-point. and radius of circle = 22 + 42 = 20
at 2 + a 2at + 0 \Equation of circle is x2 + y2 = 20
\ a= , b=
2 2 6. Given equation of parabola is rewritten as
Þ 2 a = at 2 + a , b = at
ïì æ 5x - 12 y + 17 ö ïü
2

æbö
2 169 {(x - 1 )2 + ( y - 3 ) 2} = (13)2í ç ÷ ý
2a = a ç ÷ + a è ø ï
\
èaø îï 13 þ
2
Þ 2 aa = b 2 + a 2 Þ b 2 = - a 2 + 2aa æ 5x - 12 y + 17 ö
Þ (x - 1 )2 + ( y - 3)2 = ç ÷
è 13 ø
4a æ aö
Þ b2 = ça - ÷
2 è 2ø Þ SP = PM
4a æ aö \ Focus is (1, 3) and equation of directrix is
\ The locus is y2 = çx - ÷ ×
2 è 2ø 5x - 12 y + 17 = 0
a a a 5 - 36 + 17
The directrix is X = - Þ x- = - Þ x=0 The distance of the focus from directrix =
2 2 2 25 + 144
3. Let a point on parabola x2 = 4 y is A (x1 , y1 ), so 14
=
13
x12 = 4 y1 …(i) 14 28
\ Length of latusrectum = 2 ´ =
and an another point P (h , k) which divides the line 13 13
segment joining the point B(0, - 1) and A (x1 , y1 ) 7. Given parametric equation of parabola is
internally in the ratio 1 : 2.
(2 ´ 0) + (1 ´ x1 ) 2 ´ (-1) + (1 ´ y1 ) x = t 2 + 1, y=2t+1
So, h = and k = 2
2+1 2+1 æ y -1ö
Þ x=ç ÷ +1
è 2 ø
Þ x1 = 3h and y1 = 3k + 2 …(ii)
Now, on the elimination of (x1 , y1 ) with the help of Þ ( y - 1 )2 = 4 (x - 1 ) Þ Y 2 = 4X
Eq. (i) and relation (ii), we get Vertex is (1, 1), length of latusrectum = 4
9h 2 = 4(3k + 2) Clearly, equation of directrix is
On taking locus of point P (h , k), we have X = -1 Þ x -1 = -1 Þ x = 0
9x2 = 12 y + 8 Þ 9x2 - 12 y = 8 8. We have, 2{(x - a )2 + ( y - a )2} = (x + y)2
æ3 + 3 2 - 2ö 1
4. The focus º ç , ÷ º (3, 0). The vertex º (2, 0) Þ (x - a )2 + ( y - a )2 = =| x + y|
è 2 2 ø 2
x+ y
Þ (x - a )2 + ( y - a )2 =
2
V S Clearly, the equation represents a parabola having the
M
focus at (a , a ) and directrix x + y = 0.
(2, 0) (3, 0)
\LR = 2 (Distance between focus and directrix)
directrix a+a
Þ LR = 2 = 2 2a
As, MV = VS, M º (1, 0). 1+1
Parabola 367

9. x2 - 2x + y - 2 = 0 Þ x2 - 2x + 1 = 3 - y So, Ð AOM = 45°


Þ (x - 1)2 = - ( y - 3) \ The ÐAOB is 90°.
Length of its latusrectum is 1 unit. 15. The point of intersection between the curves
1
Since, AS, , BS are in HP, thus x2 = 4 ( y + 1 ) and x2 = - 4 ( y + 1 ) is (0, - 1 ).
2 The slopes of curve first and curve second at the point
1 2 × AS × BS l1 (0, - 1 ) are respectively
= Þ BS =
2 AS + BS (4l1 - 1) 2x -2 x
m1 = = 0 and m2 = =0
10. Since, the semi-latusrectum of a parabola is HM of 4 4
segments of a focal chord. m - m2
\ tan q = 1 = 0 Þ q = 0°
2 SP × SQ 2 ´ 3 ´ 2 12 1 + m1m2
\ Semi-latusrectum = = =
SP + SQ 3+2 5 16. On solving x - 2 y = 1 and y2 = k x, we get
Þ Latusrectum of the parabola = 2 ´ Semi-latusrectum y2 = k(1 + 2 y)
24
= Þ y2 - 2ky - k = 0
5
\ y1 + y2 = 2k, y1 × y2 = - k
11. Since, focal chord of parabola y2 = ax is 2x - y - 8 = 0
\ 16 = (x1 - x2)2 + ( y1 - y2)2
æa ö 2
Q This chord passes through focus i.e. ç , 0÷ æ y2 y2 ö
è4 ø = ç 1 - 2 ÷ + ( y1 - y2)2
a è k kø
\ 2 × - 0 - 8 = 0 Þ a = 16
4
ì ( y + y )2 ü
\ Directrix is x = - 4 Þ x + 4 = 0 = ( y1 - y2)2 × í 1 2 2 + 1ý
î k þ
12. Let A be the vertex of the parabola and AP is chord of
parabola such that slope of AP is cot a. ì 4k2 ü
= {( y1 + y2)2 - 4 y1 y2}í 2 + 1ý
Let coordinates of P be (2 t , t 2) which is a point on the î k þ
parabola. = 5 {4 k2 + 4 k} = 20k2 + 20k
t t \ 5 k2 + 5k - 4 = 0
\ Slope of AP = Þ cot a = Þ t = 2 cot a
2 2
105 - 5
In DAPB, AP = 4 t 2 + t 4 = t 4 + t 2 Þ k=
10
\ AP = 2 cot a 4(1 + cot2 a ) - 5 ± 25 + 80
k=
10
= 2 cot a 4 cosec2a = 4 cot a cosec a
- 5 + 105
cos a =
=4 cosec a = 4 cos a cosec2 a 10
sin a
105 - 5
13. y2 - kx + 8 = 0 Þ y2 = k (x - 8 /k) =
10
\ Directrix is x - 8 / k = - k / 4
æ 2at12 + at22 4 at1 + 2at2 ö
Þ x = 8 / k - k /4 = 1 17. Here, C º ç , ÷
è 3 3 ø
Þ k2 + 4k - 32 = 0 Þ k = - 8 or 4
\ One value of k is 4. It lies on y = 0.
2
14. Given equation of parabola is y = 4ax. Since, AB = 8a, it t 1)
t21, 2a
means ordinate of A and B respectively 4a and - 4a. A (a
General point on this parabola is (at 2, 2at ) Þ t = ± 2 1
Y V C
X¢ X
A(4a, 4a)
2

X
O M 2
B (at 2 , 2at2)

B(4a, – 4a) 4 at1 + 2at2


\ =0
2 3
So, at = 4a
Þ 2 t1 + t2 = 0
\ OM = 4a, AM = 4a
368 JEE Main Mathematics

18. Let A º (at12, 2at1 ), B º (at22, 2at2) 22. Given parabola is x2= 8 y ...(i)
æ 5p ö 2at1 1 Now, slope of tangent at any point (x, y) on the
We have, mAS = tan ç ÷ Þ =-
è 6 ø 2
at1 - a 3 parabola (i) is
dy x
Y = = tan q
A dx 4
p
[Q tangent is making an angle q with the positive
6
direction of X-axis]
X¢ X
O S (a, 0) So, x = 4 tan q Þ 8 y = (4 tan q)2
[on putting x = 4 tan q in Eq. (i)]
B
Y¢ Þ y = 2 tan 2q
Þ t12 + 2 3 t1 - 1 = 0 Now, equation of required tangent is
Þ t1 = - 3 ± 2 y - 2 tan 2q = tan q( x - 4 tan q)
Clearly, t1 = - 3 - 2 is rejected. Þ y = 2 tan q - 2 tan 2 q
Thus, t1 = (2 - 3 ) Þ x = y cot q + 2 tan q
8 (- by - c)
Hence, AB = 4 at1 = 4 a (2 - 3 ) 23. Line will touch the parabola, if y2 - 4 y + 32 =
a
19. Given equation of line is has equal roots.
x cos a + y sin a = p or y = - x cot a + p cosec a Þ 4b2 = 7a 2 + 2a (c + 2b) Þ 4b2 = a (7a + 2c + 4b)
Here, m = - cot a 1
24. Any tangent to the parabola y2 = 4x is y = mx + .
æ a 2a ö m
We know that, the point of contact is ç 2 , ÷
èm m ø It passes through (a , b ), if
é a 2a ù b = ma +
1
Þ a m 2 - bm + 1 = 0
=ê , ú m
ë (- cot a ) 2
- cot aû
It will have roots m1 and 2m1, if
= (a tan 2 a , - 2a tan a )
b 1
m1 + 2m1 = and m1 × 2m1 =
20. Since, the tangent to the parabola at point t1 and t2 are a a
2
t1 y = x + at12 and t2 y = x + at22 æ b ö 1
Þ 2× ç ÷ =
Also, tangents are perpendicular to the parabola è3 aø a
therefore,
2 b2 1
1 1 Þ =
× = - 1 or t1 t2 = - 1 9 a2 a
t1 t2
2 2
We also know that their point of intersection is Þ 2b 2 = 9a Þ a = b
9
[at1t2, a (t1 + t2)] or [- a , a (t1 + t2)]
25. We know that, equation of tangent to parabola y2 = 4ax
\ Point of intersection lie on directrix x = - a a
is y = mx + .
or x+ a =0 m
21. Given that equation of parabola is y2 = 9x \Equation of tangent to the parabola y2 = 4x is
9 1
On comparing with y2 = 4ax, we get a = y = mx + [Q a = 1]
4 m
Now, equation of tangent to the parabola y2 = 9x is Þ m2x - my + 1 = 0 ...(i)
94
/ Now, let line (i) is also a tangent to the circle.
y = mx + …(i)
m Equation of circle x2 + y2 - 6x = 0
If this tangent passing through the point (4, 10), then Clearly, centre of given circle is (3, 0) and radius = 3
9 [Q for the circle x2+ y2 + 2 gx + 2 fy + c = 0,
10 = 4m +
4m centre = (- g , - f ) and radius = g 2 + f 2 - c]
Þ 16m2 - 40m + 9 = 0
\The perpendicular distance of (3, 0) from the line (i)
1 9 is 3.
Þ (4m - 9) (4m - 1 ) = 0 Þ m = ,
4 4 [Q radius is perpendicular to the tangent of circle]
On putting the values of m in Eq. (i), |m2× 3 - m × 0 + 1|
4 y = x + 36 and 4 y = 9x + 4 Þ =3
(m2)2 + (- m)2
Þ x - 4 y + 36 = 0 and 9x - 4 y + 4 = 0
Parabola 369

The length of perpendicular from a point (x1 , y1 ) to the 30. As we know that distance from vertex to the parabola is
ax1 + by1 + c equal to the focus and directrix.
line ax + by + c = 0 is .
2
a +b 2 Y

3m2 + 1
Þ =3 S
m4 + m2 a O
X
(a, 0)
Þ 9m4 + 6m2 + 1 = 9(m4 + m2)
1
Þ m » ¥ or m = ± x = –a
3
\ The tangent at the vertex divide in the ratio 1 : 1.
é 1 ù
ê 3+ 2 ú 31. Let P (at 2, 2at ) any point on the parabola and focus is
3m2 + 1 m = 3ú
êQ lim = lim (a , 0).
ê m ® ¥ m4 + m2 m ® ¥ 1 ú Y
êë 1+ 2 úû (at 2, 2at)
m
P
\Equation of common tangents are x = 0, K
X
x -x é 1ù S(a, 0)
y= + 3 and y = - 3 ê using y = mx + ú
3 3 ë mû
i.e. x = 0, 3 y = x + 3 and 3 y = - x - 3 x = –a
26. Let A º (at12, 2at1 ), B º (at22, 2 at2). The equation of tangent at P is yt = x + at 2.
Since, it meets the directrix x = - a at K.
Tangents at A and B will intersect at the point C, whose
coordinate is given by { at1t2, a (t1 + t2)}. æ at 2 - a ö
Then, the coordinate of K is ç - a , ÷.
è t ø
Clearly, ordinates of A , C and B are always in AP.
2 at
27. Given equation of curve is y2 + 2xy + x2 + 2x + 3 y + 1 = 0. Slope of SP = m1 =
2 a (t 2 - 1 )
Here, h = ab, therefore the given curve is a parabola.
The position of the point (1, - 2) with respect to the a (t 2 - 1 )
Slope of SK = m2 =
parabola is obtained as -2 at
(-2)2 + 2 (1 ) (-2) + (1 )2 + 2 (1 ) + 3 (- 2) + 1 = - 2 < 0 2 at a (t 2 - 1 )
\ m1m2 = × = -1
Since, point is inside the parabola therefore no tangent a (t - 1 ) (- 2 at )
2

can be drawn to the parabola.


\ Ð PSK = 90°
28. It is clear from the figure, that only one common
tangent is possible. 32. Since, tangent at P and Q on the parabola meet in T.
Y If the coordinates of P and Q are (at12, 2at1 ) and (at22, 2at2)
x 2 = 4by
respectively, then coordinates of T are { at1t2, a (t1 + t2)}.
\ SP = a (1 + t12)
X SQ = a (1 + t22)
ST 2 = a 2(1 - t1t2)2 + a 2 (t1 + t2)2
y 2 = 4ax
= a 2 (1 + t12 + t22 + t12 t22)
2
29. Given parabola is y = ax = a (1 + t12) a (1 + t22) = SP × SQ
æaö Thus, SP , ST , SQ are in GP.
i.e. y2 = 4 ç ÷ x …(i)
è4ø 33. Let third tangent is tangent at vertices, then
Let point of contact is (x1 , y1 ), then equation of tangent is p1 =|at1t2|, p2 = at12, p3 = at22 , clearly p2, p1 , p3 are in GP.
a
yy1 = (x + x1 ) Y
2
a a a t1
Here, m = = tan 45° Þ = 1 Þ y1 = 2
B (at1, 2at1)
2 y1 2 y1 2
X¢ X
a O
From Eq. (i), x1 = A 2
C (at2, 2at2)
4 [at1t2, a(t1 + t2)] t2
æa aö
\ Point of contact is ç , ÷ . Y¢
è4 2ø
370 JEE Main Mathematics

34. Given that equation of parabola is y2 = 8x. 1


38. Let y = mx + is a tangent to y2 = 4ax.
m
Þ a =2
Equation of normal to the parabola x2 = 4by at (x1 , y1 ) is
We know, if the normal at point (at12, 2 at1 ) is passing 2b
through the point on the parabola (at22, 2 at2), then y - y1 = - (x - x1 ) and x12 = 4by1
x1
2
t2 = - t1 - x12 2b
t1 Þ y- =- (x - x1 )
4b x1
Given point is (2, 4). 2b x2
Þ at12 = 2 Þ y=- x+ 1 +2b
x1 4b
Þ t1 = 1 1
2 On comparing with y = mx + , we get
\ t2 = - 1 -
= -3 m
1 2b
The other end will be (at22, 2at2) i.e., (18, - 12) m=- …(i)
x1
m m3 æ 1ö x12 1
35. Equation of normal is y = mx - - ç a = ÷ . It passes +2b = …(ii)
2 4 è 4ø 4b m
through (c, 0) .
From Eqs. (i) and (ii), we get
m m3 4b2
\ 0 = cm - - Þ m =0 +2b =
1
2 2 2
m
m 4b
m2 1 1
and = c- Þ c> Þ b + 2 bm2 = m Þ 2 bm2 - m + b = 0
4 2 2
For real values of m, D > 0
Then, all values of m are real. 1 1
Þ 1 - 8b2 > 0 Þ b2 < Þ |b|<
36. Let the coordinates of P and Q are (at12, 2at1 ) and 8 2 2
(at22, 2at2) respectively. 39. Given equation of parabola is y2 = 16x.
Then, the coordinates of R are If (1, 1) is the mid-point of the chord, then its equation of
{2a + a (t12 + t22 + t1t2), - at1t2 (t1 + t2)} chord is
Since, R lies on the parabola. T = S1
\ a 2t12 t22 (t1 + t2)2 = 4a [2a + a{(t1 + t2)2 - t1 t2 }] \ y(1) - 8(x + 1) = 1 - 16
Þ y - 8x - 8 = - 15 Þ 8x - y = 7
Þ (t1 + t2)2 { t12 t22 - 4} + 4 (t1t2 - 2) = 0
Þ t1t2 = 2 40. Equation of QR (chord of contact) is
Þ y1 y2 = (2at1 ) (2at2) = 4a t1 t2 2 yy1 = 2a (x + x1 )
\ y1 y2 = 8a 2 Þ yy1 = 2a (x + x1 ) = 0
Q PM = Length of perpendicular from P (x1 , y1 ) on QR.
37. Let P (at 2, 2at ) be any point on the parabola y2 = 4ax, | y y - 2a (x1 + x1 )| |( y12 - 4ax1 )|
then equation of tangent and normal at P (at 2, 2at ) are = 1 1 =
( y12 + 4a 2) ( y12 + 4a 2)
ty = x + at 2 and y = - t x + 2 at + at3 , respectively.
Since, tangent and normal meet its axis at T and G.
Q
\ Coordinates of T and G are (- at 2, 0) and (2a + at 2, 0)
respectively.
Y P (x
1, y1 ) M
M P(at 2, 2at)
x+a=0 R
G
X
T O V S(a, 0)
1
Now, area of DPQR = QR × PM
2
1 1 y2 - 4ax1
= ( y12 - 4ax1 ) ( y12 + 4a 2) 1
From definition of parabola 2 |a| y12 + 4a 2
SP = PM = a + at 2
[since, P (x1 , y1 ) lies outside the parabola
Now, SG = VG - VS = 2a + at 2 - a = a + at 2
\ y12 - 4ax1 > 0]
and ST = VS + VT = a + at 2
( y2 - 4ax1 )3/ 2
Hence, SP = SG = ST = 1 , if a > 0
2a
Parabola 371

41. Given equation of parabola is y2 = 4ax. i.e. k2 = 4ah …(i)


Let the coordinates of B are (at 2, 2at ). Let a line OP makes an angle q from the X-axis.
2 Y
Slope of AB = P(h, k)
t
Since, BC is perpendicular to AB. l k
q
Y X¢ X
B(at 2, 2at) O h A

90°
X
(0, 0)A D C Y¢
PA k
\ In D OAP, sin q = Þ sin q =
OP l
t OA
So, slope of BC = - Þ k = l sin q and cos q =
2 OP
t h
Equation of BC is y - 2at = - (x - at 2). Þ cos q = Þ h = l cos q
2 l
This line meets to the x-axis at point C. Hence, from Eq. (i), we get
Put y = 0 Þ x = 4a + at 2 l2 sin 2 q = 4a ´ l cos q [put k = l sin q, h = l cos q]
4a cos q
So, distance CD = 4a + at 2 - at 2 = 4a Þ l=
2 2 2
sin 2 q
42. Given equation of circle is (x - 6) + y = ( 2 )
45. Given that focus is S(0, 0).
Y
Let A is the vertex of parabola.
2B Take any point Z on the directrix such that AS = AZ .
2
q X
(0, 0) S Since, the given tangent x - y + 1 = 0 is parallel to the
(4, 0)D C(6, 0)
directrix.
Equation of directrix is x - y + l = 0.
Q A is the mid-point of SZ.
BC = Radius = 2
The length of the tangent from S to B. Y

\ SB = (4 - 6)2 + 0 - 2 = 22 - 2 = 2 M x–y+1=0
From figure, DCBS is an isosceles triangle. Z A
P(x, y)
Þ q = 45° Þ m =1 [Q BC = BS ] S( X
0,
Similarly, for DCSD, m = - 1 0)

43. Let P (x1 , y1 ) be any point on the curve y = x .


Clearly,y1 = x1 Þ x1 = y12 [Q (x1 , y1 ) lies on y = x] \ SZ = 2 SA
\ The point is P ( y12, y1 ) |0 - 0 + l| |0 - 0 + 1|
Þ =2 ´
æ3 ö 2
1 +1 2
12 + 12
Now, let the given point be A ç , 0÷ , then
è2 ø Þ |l|= 2 Þ l = 2
2
æ 3ö 9 \ Equation of directrix is x - y + 2 = 0.
PA = ç y12 - ÷ + y12 = y14 - 3 y12 + + y12
è 2ø 4 Now, P be any point on the parabola.
9 5 \ SP = PM Þ SP 2 = PM 2
= y14 - 2 y12 + = ( y12 - 1)2 + 2
4 4 æ|x - y + 2|ö
Þ (x - 0)2 + ( y - 0)2 = ç ÷
Clearly, PA will be least when y12 - 1 = 0 è 2 ø
5 5 Þ x2 + y2 + 2xy - 4x + 4 y - 4 = 0
Þ PA min = 0 + =
4 2
46. Given parabola y2 = 4x …(i)
44. Let any point (h , k ) will satisfy
So, equation of tangent to parabola (i) at point (1, 2) is
y2 = 4ax 2 y = 2(x + 1).
372 JEE Main Mathematics

[Q equation of the tangent to the parabola x2 - 2 x + 4 = 0


y2 = 4ax at a point (x1 , y1 ) is given by Let z = x2 - 2 x + 4
yy1 = 2a (x + x1 )] \ z ¢ = 2x - 2
Þ y=x+1 …(ii)
For least value, z¢ = 0 Þ 2x - 2 = 0 Þ x = 1
Now, equation of circle, touch the parabola at point
(1, 2) is z¢¢ is positive at x = 1.
2 2
(x - 1) + ( y - 2) + l (x - y + 1) = 0 \ It is minimum, putting x = 1 in Eq. (i), we get
2 2
Þ x + y + (l - 2)x + (-4 - l ) y + (5 + l ) = 0 …(iii) y=1
Also, Circle (iii) touches the X-axis, so g 2 = c So, the required point at the least distance from the line
2 is (1, 1).
æ l - 2ö
Þ ç ÷ =5 + l
è 2 ø 49. Normal at a point (m2, - 2 m) on the parabola y2 = 4x is
Þ l2 - 4l + 4 = 4l + 20 given by y = mx - 2 m - m3 . If this is normal to the circle
also, then it will passes through centre (- 3, 6) of the
8 ± 64 + 64
Þ l2 - 8l - 16 = 0 Þ l = circle.
2
\ 6 = - 3m - 2 m - m3 Þ m = -1
Þ l = 4 ± 32 = 4 ± 4 2
Since, shortest distance between parabola and circle
Now, radius of circle is r = g2 + f 2 - c will occurs along common normal.
\ Shortest distance = distance between (m2, - 2 m)
Þ r =| f | [Q g 2 = c]
and centre (-3, 6) - radius of circle = 4 2 - 5
=½½l + 4½
½ = 8 + 4 2 or 8 - 4 2
½ 2 ½ 2 2 50. Let any point on the line segment PQ is R (a, b), then
8 -4 2 l (1 ) + 1
For least area r = = 4 - 2 2 units a= =1
2 l+1
So, area = pr 2 = p (16 + 8 - 16 2 ) 3l + 1
and b= [Q l > 0 as R is on segment AB ]
= 8p (3 - 2 2 ) sq unit l+1
A point is inside parabola y2 = 4x, if
47. Given equation of curves are
y2 - 4x < 0
y2 = 16x (parabola) …(i) 2
æ 3l + 1 ö
and xy = - 4 (rectangular hyperbola) …(ii) Þ ç ÷ - 4(1 ) < 0
è l+1ø
Clearly, equation of tangent having slope ‘m’ to
4 æ 3l + 1 ö æ 3l + 1 ö
parabola (i) is y = mx + …(iii) Þ ç + 2÷ ç - 2÷ < 0
m è l+1 ø è l+1 ø
3
Now, eliminating y from Eqs. (ii) and (iii), we get Þ (5l + 3) (l - 1 ) < 0 Þ - < l <1
5
æ 4ö 4
x çmx + ÷ = - 4 Þ mx2 + x + 4 = 0, So, 0 < l <1 [but l > 0]
è m ø m
which will give the points of intersection of tangent Round II
and rectangular hyperbola.
1. Shortest distance between two curves occured along the
4
Since, line y = mx + is also a tangent to the common normal.
m
rectangular hyperbola. Normal to y2 = 4x at (m2, 2 m) is
\Discriminant of quadratic equation y + mx - 2 m - m3 = 0
4 æ m2 ö
mx2 + x + 4 = 0, should be zero. Normal to y2 = 2 (x - 3) at ç + 3 , m÷ is
m è 2 ø
[Q there will be only one point of intersection] m3
2 y + m (x - 3) - m - =0
æ4ö 2
Þ D = ç ÷ - 4 (m) (4) = 0
èmø 1
Both normals are same, if -2 m - m3 = - 4m - m3
Þ 3
m =1 Þm =1 2
So, equation of required tangent is y = x + 4. Þ m = 0, ± 2
2 So, points will be (4, 4) and (5, 2) or (4, - 4) and (5, - 2).
48. Given equation of parabola is y = x …(i)
Hence, shortest distance will be
Equation of straight line is y = 2x - 4 …(ii)
(1 + 4) = 5
On solving Eqs. (i) and (ii), we get
Parabola 373

2. Q y2 - 12x - 4 y + 4 = 0 Þ ( y - 2)2 = 12x (x - y)2 æ x + y -2ö


Þ = 4 2ç ÷
2 è 2 ø
Y
Þ (x - y)2 = 8 (x + y - 2)
5. Equation of tangent to the parabola y2 = 4x having slope
O
‘m’ is
X¢ X 1
y = mx + …(i)
m
Q Line (i) is also tangent to the parabola x2 = 4 y
Y¢ 4
So, the discriminant of x2 - 4mx - = 0 must be zero,
m
Its vertex is (0, 2) and a = 3 and its focus is (3, 2). 16
so 16m2 + = 0 Þ m = -1
Hence, for the required parabola, focus is (3, 4), m
vertex is (3, 2) and a = 2 Therefore, the equation of common tangent to given
Hence, the equation of the parabola is parabolas is
(x - 3)2 = 4(2) ( y - 2) or x2 - 6x - 8 y + 25 = 0 x+ y+ 1 =0 …(ii)
It is given that line (ii) is tangent to the circle
3. The equation of the tangent to parabola y2 = 8x at P (2, 4)
is 4 y = 4 (x + 2). x2 + y2 = c2
1
Þ x- y+ 2 =0 …(i) \ c=
2
Since, this chord meets the parabola y2 = 8x + 5
at Q and R. 6. It is given that circle passes through the points A(0, 1)
and B (2, 4), so the centre of the circle is point of
Now, let (x1 , y1 ) be the mid-point of chord QR.
intersection of perpendicular bisector of AB and the
Then, equation of QR is normal to the parabola y = x2 at point B (2, 4), as circle
yy1 - 4 (x + x1 ) - 5 = y12 - 8x - 5 [Q T = S1 ] touches the parabola at point B (2, 4) itself.
Þ 4x - yy1 - 4x1 + y12 = 0 …(ii) Now, equation of normal to the parabola y = x2
Here, Eqs. (i) and (ii) represents the same line. at point B (2, 4) is
4 y - 4x1 + y12 1
=- 1 = y - 4 = - (x - 2) Þ x + 4 y = 18 ...(i)
1 -1 2 4
Þ y1 = 4 and 8 = - 4x1 + y12 and equation of perpendicular bisector of points A(0, 1)
and B (2, 4) is
Þ y1 = 4 and x1 = 2 5 2
\ Mid-point of QR is (2, 4). y - = - (x - 1) Þ 4x + 6 y = 19 ...(ii)
2 3
4. Since, distance of vertex and focus from origin are 2 From Eqs. (i) and (ii), the required centre will get
and 2 2 , respectively. 4x + 16 y = 72
\ V (1, 1) and F (2, 2) (i.e. lying on y = x ) 4x + 6 y = 19
where, length of latusrectum = 4a = 4 2 [Q a = 2 ] - - -
\ By definition of parabola 53
10 y = 53 Þ y=
Y y=x 10
106 16
\ x+ = 18 Þ x = -
P 5 5
æ 16 53 ö
F (2, 2) \Coordinate of centre is ç - , ÷.
N
M è 5 10 ø
(1, 1)
V
7. Let the point be P (at 2, 2at ).
x+y–2=0
X¢ X Then, according to question, SP = at 2 + a = k …(i)
O
Let (a , b ) be the moving point, then a = at 2 , b = 2at
a t
\ =
Y¢ b 2
b2
PM 2 = (4a )(PN ) and a= [Q point (a , b ) lies on y2 = 4ax ]
4a
where, PN is length of perpendicular upon x + y - 2 = 0
On putting these values in Eq. (i), we get
(i.e., tangent at vertex).
374 JEE Main Mathematics

b2 æ 4a 2 ö (i.e. X-axis) so coordinate of N is (3 t 2,0) and M is


ç 1 + 2 ÷ = k Þ b 2 + 4 a 2 = 4 ka mid-point of PN , so coordinate of M is (3 t 2, 3 t ). Now,
4a è b ø
equation of the parallel to the axis of parabola through
Hence, locus of a point is point M is
4x2 + y2 - 4kx = 0 y=3t … (ii)
x-2 y-1 So point of intersection of parabola (i) and
8. Chord through (2, 1) is = =r …(i)
cos q sin q æ 3 t2 ö
line (ii) is Q ç , 3 t ÷, so equation of line NQ is
è 4 ø
Y A
3t
y-0 = (x - 3 t 2) … (iii)
3 t2 2
(2, 1) -3 t
X¢ X 4
O
For y-intercept made by line (iii) put x = 0 then
1 12 t 4
Y¢ B
y= (- 3 t ) = = [given]
(1 /4) - 1 3 3
1
On solving Eq. (i) with parabola y2 = x, we have Þ t=
3
(1 + r sin q)2 = 2 + r cos q
æ1 ö æ1 ö æ1 ö æ 1 ö
Þ 2 2
sin q r + (2 sin q - cos q)r - 1 = 0 \ P ç , 2÷ , N ç , 0÷ , M ç , 1÷ , Q ç , 1÷
è3 ø è3 ø è 3 ø è 12 ø
This equation has two roots r1 = AC and r2 = - BC.
1 1 1
Then, sum of roots r1 + r = 0 \ PN = 2, and MQ = - =
3 12 4
1
Þ 2 sin q - cos q = 0 Þ tan q = 11. Given equation of curve is
2
AB = r1 - r2 y2 = x - 2 …(i)
and the equation of line is
= (r1 + r2)2 - 4r1r2 y=x …(ii)
Y y=x
1 y2=x–2
= 4 =2 5
sin 2 q M
P(t2+2, t)
é 1 1 ù
êëQ tan q = 2 Þ sin q = 5 úû X
O (2, 0)
9. Difference of the ordinate,
2a 1
d = 2at + = 2a t +
t t
Consider a point P (t 2 + 2, t ) on parabola (i).
Y R(at 2, 2at)
For the shortest distance between curve (i) and line
(ii), the line PM should be perpendicular to line (ii)
and parabola (i), i.e. tangent at P should be parallel to
X¢ X y = x.
P S
(0, 0) dy
Q \ = Slope of tangent at point P to curve (i)
dx at point P
Y¢ a , 2a
t2 t =1 [Q tangent is parallel to line y = x]
1
Þ =1
at 2 2at 1 2yP
1 a 2a æ 1ö
Now area, A = - 1 = a2 çt + ÷ dy
2 t2 t è tø [differentiating the curve (i), we get 2 y =1]
0 0 1 dx
1 1
æ 1ö 2A Þ =1Þt = [Q P (x, y) = P (t 2 + 2, t )]
Þ 2a ç t + ÷ = 2t 2
è tø a
æ9 1ö
10. Equation of given parabola is So, the point P is ç , ÷ .
è4 2ø
y2 = 12x … (i)
½9 - 1½
½ ½
Now, let a point P (3 t 2, 6 t ) on the parabola, since point
Now, minimum distance = PM = ½
4 2½
N is the foot of perpendicular drawn from P on the
2
axis of the parabola
Parabola 375

[Q distance of a point P (x1 , y1 ) from a line æ 1ö


x çmx + ÷ = 2
|ax1 + by1 + c |ù è mø
ax + by + c = 0 is ú
a 2 + b2 úû Þ m2x2 + x - 2m = 0
7 Note that tangent touch the curve exactly at one point,
= units therefore both roots of above equations are equal.
4 2
Þ D = 0 Þ 1 = 4(m2) (- 2m)
12. Given equations of the parabola y2 = 4x …(i) 3
æ 1ö 1
and circle 2
x + y =5 2
…(ii) Þ m3 = ç - ÷ Þ m=-
è 2ø 2
So, for point of intersection of curves (i) and (ii), put \Required equation of tangent is
y2 = 4x in Eq. (ii), we get x
x2 + 4 x - 5 = 0 y = - - 2 Þ 2y = - x - 4
2
Þ x2 + 5 x - x - 5 = 0 Þ x + 2y + 4 = 0
Þ (x - 1)(x + 5) = 0 Þ x = 1, - 5 15. We know that the locus of point P from which two
For first quadrant x = 1 , so y = 2 . perpendicular tangents are drawn to the parabola, is
the directrix of the parabola.
Now, equation of tangent of parabola (i) at point (1, 2) is
T =0 Hence, the required locus is x = - 1.
Þ 2 y = 2(x + 1) Þ x - y + 1 = 0 16. From the figure, it is clear that vertex of the parabola at
æ3 7ö (1, 0).
The point ç , ÷ satisfies, the equation of line Y
è4 4 ø
x- y+ 1 =0 (1, 0)
2 2
13. Vertex of parabola y = - 4(x - a ) is (a , 0). 2 X¢ X
(0, 0) (2, 0)
For point of intersection with Y-axis, put x = 0 in the
given equation of parabola. Y¢ x=2
2 2
This gives, y = 4a 17. Perpendicular tangents can be drawn only from a point
Þ y = ± 2a on the directrix, x = - 2.
Thus, the point of intersection are (0, 2a ) and (0, - 2a ). \The point is (– 2, 0).
Y 2
3y 3x 6 æ 3 ö 105
18. = x2 + - = çx + ÷ -
B (0, 2a) a3 2a a 2 è 4a ø 16a 2
2
æ 3 ö 3 æ 35a ö
Þ çx + ÷ = 3 çy+ ÷
O A è 4a ø a è 16 ø
X
(a2, 0) 3 35a
If (a , b ) is the vertex, then a = - ,b = -
4a 16
C(0, –2a) 105
Hence, the locus of (a , b ) is xy =
64
From the given condition, we have 19. The coordinates of P are (1, 0). A general point Q on
Area of DABC = 250 y2 = 8x is (2t 2, 4t ). Mid-point of PQ is (h , k ).
\
1
(BC )(OA ) = 250
1
[Q area = ´ base ´ height] \ 2h = 2t 2 + 1 …(i)
2 2 and 2k = 4t Þ t = k /2
1 Then, from Eq. (i), we get
Þ (4a )a = 250 Þ a = 125 = 53
2 3
2 2k2
\ a =5 2h = + 1 Þ 4h = k2 + 2
4
a
14. We know that, y = mx + is the equation of tangent to Hence, locus is y2 - 4x + 2 = 0
m
the parabola y2 = 4ax. 20. Solving y2 = 4ax and x2 = 4ay (a ¹ 0)
1 Point of intersection are (0, 0) and (4a , 4a ).
\ y = mx + is a tangent to the parabola
m Both points lie on the line
y2 = 4 x [Q a = 1]
2bx + 3cy + 4d = 0
Let, this tangent is also a tangent to the hyperbola
xy = 2. Þ d =0
1 and 2b + 3c = 0 [Q a ¹ 0]
Now, on substituting y = mx + in xy = 2, we get 2 2
m \ d + (2b + 3c) = 0
376 JEE Main Mathematics

21. It is given that line y = mx intersect the parabola y2 = x, 23. Parametric coordinates for y2 = 4ax are (at 2 2at).
2
so on solving the line y = mx and parabola y = x, we get
1 P
m2x2 = x Þ x = 0, 2
m
æ 1 1ö
\ Point of intersections are (0, 0) and ç 2 , ÷ .
èm mø
æ 1 1ö Q
\ Point P is ç 2 , ÷
èm mø
Now equation of tangent to parabola y2 = x at As the circle intersects the parabola at P and Q. Thus,
points P and Q should satisfy circle.
æ 1 1ö 1 1æ 1 ö
P ç 2 , ÷ is T = 0 Þ y = çx + 2÷ …(i) P (2t 2, 4t ) should lie on x2 + y2 - 2x - 4 y = 0
èm mø m 2è m ø
Þ 4t 4 + 16t 2 - 4t 2 - 16t = 0
Q Tangent (i) meet the X-axis at point Q, so point Q is
æ 1 ö Þ 4t 4 + 12t 2 - 16t = 0
ç - 2 , 0÷ .
è m ø Þ 4t (t3 + 3t - 4) = 0
½ ½ Þ 4t (t - 1) (t 2 + t + 4) = 0
½ 0 0 1½ \ t = 0, 1
1½ 1 ½ 1½ 1 ½
1½|= ½
1 ½= 4
Now, area of DOPQ = |½ 2 Þ P(2, 4) and PQ is the diameter of circle.
2 m m 2½m3½ 1 1
½ 1 ½ Thus, area of DPQS = × OS ´ PQ = × (2) × (4) = 4
½- 0 1½ 2 2
½ m2 ½
1 24. If y = mx + c is normal to the parabola y2 = 4ax, then
Þ m = = 0.5
2 c = - 2am - am3 .
22. End points of latusrectum are (a ± 2a ) i.e. (1, ± 2). From given condition, y2 = 12x
y - y1 y Þ y2 = 4 × 3 × x
Equation of normal at (x1 , y1 ) is = - 1. Þ a =3
x - x1 2a
y-2 2 y+2 2 And x+ y =k
i.e. = - and =
x-1 2 x-1 2 Þ y = (- 1)x + k
Þ m = - 1 and c = k
Þ x + y = 3 and x - y = 3
\c = k = - 2(3) (- 1) - 3(- 1)3 = 9
which is tangent to (x - 3)2 + ( y + 2)2 = r 2.
Y 25. Tangent to the curve x2 = y - 6 at (1, 7) is
y+ 7
x= -6
L (1, 2) 2
Þ 2x - y + 5 = 0 …(i)
Normal
Equation of circle is x2 + y2 + 16x + 12 y + c = 0
X¢ F X
O (1, 0) Centre (-8, - 6)
Normal r = 82 + 62 - c = 100 - c
L¢ (1, –2) Since, line 2x - y + 5 = 0 also touches the circle.
½2(-8) - (-6) + 5½
Y¢ \ 100 - c = ½ ½
½ 22 + 12 ½
\Length of perpendicular from centre = Radius
Þ
|3 - 2 - 3|
=r Þ ½-16 + 6 + 5½
100 - c = ½ ½
12 + 12 ½ 5 ½
Þ 100 - c = |- 5|
\ r2 = 2
Þ 100 - c = 5 Þ c = 95
15
Ellipse
An ellipse is the set of points in a plane whose distances from two fixed IN THIS CHAPTER ....
points in the plane have a constant sum. The two fixed points are the foci of
Ellipse
the ellipse.
Terms Related to an Ellipse
Position of a Point with respect to
P(x, y) an Ellipse
Auxiliary Circle
F1 F2 Intersection of a Line and an
Ellipse
Condition of Tangency
OR
Tangent to the Ellipse
The locus of a point in a plane which moves in the plane in such a way that
the ratio of its distance from a fixed point (i.e. focus) in the same plane to its Normal to the Ellipse
distance from a fixed straight line (i.e. directrix) is always constant, which is Director Circle
always less than unity. Number of Normal and
Y Conormal Points
Minor axis
Pair of Tangents
L B P(x, y) Chord of Contact
M¢ Q M
Major Some Useful Properties of an
X¢ axis
X Ellipse
Z¢ A¢ S¢ C Foci S A Z
e
ntr
Ce

Q¢ Directrix
Directrix
L¢ B'
Vertex
Double Vertex
Ordinate Y¢
Latus rectum

SP
Mathematically, = e, where e is called eccentricity and 0 < e < 1.
PM
378 JEE Main Mathematics

Standard Equations of an Ellipse Latusrectum


If the centre of the ellipse is origin and foci are on the The double ordinate passing through the focus is called
latusrectum.
X-axis or Y -axis, then that types of equation are called
standard equation of an ellipse. The two such possible Important Points
orientations are shown below.
● The second degree equation ax 2 + 2hxy + by 2 + 2gx
Y Y
A + 2 fy + c = 0 represents an ellipse, if
P(x, y) abc + 2 fgh - af 2 - bg2 - ch 2 ¹ 0 and h 2 - ab < 0.
S(0, be) ● The vertex divides the ling joining focus and point of
X¢ X X¢ X intersection of directrix with axis internally and
S2(–ae, 0) S1(ae, 0) O
externally in the ratio of e : 1.
S¢(0, – be)
Example 1. The equation of an ellipse whose focus is ( -1, 1),

1
Y¢ Y¢ eccentricity is and the directrix is x - y + 3 = 0, is
2
x2 y2 x2 y2 (a) 7x2 + 7y 2 + 2xy + 10 x + 10y + 7 = 0
(i) 2
+ 2
= 1, a > b (ii) 2
+ 2
= 1, a < b
a b a b (b) 7x2 + 7y 2 + 2xy + 10 x - 10y + 7 = 0
(c) 7x2 + 7y 2 + 2xy - 10 x + 10y + 7 = 0
Terms Related to an Ellipse (d) None of the above
Vertices Sol. (b) Let P( x, y) be any point on the ellipse whose focus is
The point A and A¢ , where the curve meets the line
S( -1, 1) and the directrix is x - y + 3 = 0.
joining the foci S and S ¢, are called the vertices of the
ellipse. Draw PM perpendicular from P( x, y) on the directrix
x - y + 3 = 0.
Major and Minor Axes P(x, y)
M
Major axis is the one which lie along the line passing
x–y+3=0

through focus and perpendicular to directrix and minor


axis is the one which is perpendicular to major axis and
passes through the mid-point of the foci.
S(–1, 1)
Centre Then, by definition
Since, all chords passing through C are bisected at C. SP = ePM Þ ( SP) 2 = e2(PM) 2
Hence, C( 0, 0) is the centre of an ellipse. 2
1 ì x - y + 3ü
Þ ( x + 1) 2 + ( y - 1) 2 = í ý
Ordinate and Double Ordinate 4î 2 þ
Let P be a point on the ellipse and let PN be Þ 8 ( x2 + y 2 + 2x - 2y + 2) = x2 + y 2 + 9 - 2xy + 6x - 6y
perpendicular to the major axis AA¢ such that PN Þ 7x2 + 7y 2 + 2xy + 10 x - 10y + 7 = 0
produced meets the ellipse at P ¢. Then, PN is called the
ordinate of P and is PNP ¢ the double ordinate of P. which is the required equation of an ellipse.

Terms Related to an Ellipse


Fundamental Terms Ellipse (Horizontal ellipse) Conjugate Ellipse (Vertical ellipse)
2 2
x y x2 y2
(a) Equation 2
+ 2
= 1( a > b ) 2
+ = 1( a < b )
a b a b2 Y
Z
Y
A
B
S
X¢ X
Z¢ A¢ S' C S A Z X¢ B C B¢ X
(b) Graph
B¢ S'




Ellipse 379

Fundamental Terms Ellipse (Horizontal ellipse) Conjugate Ellipse (Vertical ellipse)


(c) Centre C( 0, 0) C( 0, 0)
(d) Vertices ( ± a, 0) ( 0, ± b )
(e) Length of major axis 2a 2b
(f) Length of minor axis 2b 2a
(g) Foci ( ± ae, 0) ( 0, ± be )
x = ± æç ö÷ y = ± æç ö÷
a b
(h) Equations of directrices èe ø èe ø
2
b2
e = 1 - æç ö÷
a
(i) Eccentricity e = 1-
a 2 èbø
2
2b 2 a2
( j) Length of latusrectum
a b
æ b2 ö æ a2 ö
(k) Ends of latusrectum ç ± ae, ± ÷ ç± , ± be ÷
è aø è b ø
ì x = a cos a ì x = a cos a
(l) Parametric equations í í
î y = b sin a î y = b sin a
(m) Parametric coordinates ( a cos a, b sin a) ( a cos a, b sin a)
(n) Focal distance or radii | SP | = ( a - ex1 ) and| S ¢ P | = ( a + ex1 ) | SP | = ( b - ey1 ) and| S ¢ P | = ( b + ey1 )
(o) Sum of focal radii| SP | + | S ¢ P | 2a 2b
(p) Distance between foci 2 ae 2 be

Example 2. If the coordinates of two points A and B are 1 b2 b2 1


Þ e2 = = 1 - 2 Þ 2 = Þ b2 = 9
( 7, 0) and ( - 7, 0) respectively and P is any point on the 2 a a 2
conic, 9 x 2 + 16y 2 = 144, then PA + PB is equal to Now, as length of latus rectum is
(JEE Main 2020) 2b 2 2(9) 18
= = = 18 = 3 2
(a) 16 (b) 8 (c) 6 (d) 9 a 18 18
2 2
Sol. (b) Equation of given conic 9x + 16y = 144 Example 4. An ellipse, with foci at (0, 2) and (0, - 2) and
2 2
x y minor axis of length 4, passes through which of the following
Þ + = 1 is an ellipse,
16 9 points? (JEE Main 2019)
9 7 (a) ( 2 , 2) (b) (2, 2) (c) (2, 2 2) (d) (1, 2 2)
so it’s eccentricity, e = 1 - =
16 4
Sol. (a) Let the equation of ellipse be
\Coordinate of foci are F1( 7 ,0) = A
x2 y 2
and F2( - 7 , 0) = B + =1 …(i)
a2 b 2
As, point P on the ellipse itself, Since, foci are at (0, 2) and (0 , - 2), major axis is along the
so PA + PB = 2 ´ 4 = 8 according to definition of ellipse. Y-axis.
Example 3. If the distance between the foci of an ellipse is So, be = 2 …(ii)
6 and the distance between its directrices is 12, then the [where, e is the eccentricity of ellipse]
length of its latus rectum is (JEE Main 2020) and 2a = length of minor axis = 4 [given]
3 Þ a=2 …(iii)
(a) 3 2 (b) 3 (c) 2 3 (d) a 2
2 Q e2 = 1 - 2
b
x2 y 2 æ2ö
2
4 é 2ù
Sol. (a) Let the equation of ellipse is + = 1 , ( a > b) \ ç ÷ =1- 2 Qe= ú
a2 b 2 è bø b ê
ë bû
According to given information, 8 2
Þ =1Þ b = 8
Distance between foci = 2ae = 6 b2
2a Thus, equation of required ellipse is
Distance between directrices = = 12
e x2 y 2
+ =1
æ 2a ö 4 8
So, (2ae) ç ÷ = 6 ´ 12
è eø x2 y 2
Now, from the option the ellipse + =1
2ae 6 4 8
Þ 4a2 = 72 Þ a2 = 18 and =
2a / e 12 passes through the point ( 2 , 2).
380 JEE Main Mathematics

x2 y2 Case I When S1 > 0, point ( x1 , y1 ) lies outside the ellipse.


Example 5. Let + = 1( a > b) be a given ellipse, length
a b 2 2 Case II S1 = 0, point ( x1 , y1 ) lies on the ellipse.
of whose latus rectum is 10. If its eccentricity is the maximum Case III S1 < 0, point ( x1 , y1 ) lies inside the ellipse.
5
value of the function, f(t) = + t - t 2 , then a 2 + b 2 is equal e.g. The position of the point (1, - 2) relative to an ellipse
12
5x 2 + 7 y 2 = 120 is
to (JEE Main 2020)
(a) 145 (b) 116 (c) 126 (d) 135 The equation of an ellipse is 5x 2 + 7 y 2 = 120.

Sol. (c) Equation of given ellipse is S1 = 5(1 )2 + 7( -2)2 - 120


x2 y 2 = 5 + 28 - 120 = - 87 < 0
+ = 1, ( a > b)
a2 b 2 So, the point (1, - 2) lies inside the ellipse.
2b2
\Length of latus rectum = = 10 [given]
a Auxiliary Circle
Þ b2 = 5a …(i) The circle described on the major axis of an ellipse as
5 diameter is called an auxiliary circle of the ellipse.
Now, given function, f(t ) = + t - t 2 for maximum,
12 Y
f¢ (t ) = 0
1 Q (a cos f, a sin f)
Þ 1 - 2t = 0 Þ t =
2 B P
(a cos f, b sin f)
According to given information eccentricity f
æ 1ö 5 1 1 X¢ X
e = fç ÷ = + - A¢ C M A
è 2 ø 12 2 4
2
5 + 6 -3 2 x2 + y = 1
= = a 2
b2
12 3 x2 + y2 = a2
b 2 2 Y¢
\ = 1-
a2 3 x2 y2
On squaring both sides, we get If the equation of an ellipse is + = 1, then its
a2 b2
b2 4
1- 2 = auxiliary circle is x 2 + y 2 = a 2.
a 9
2
b 5 5
Þ = Þ b 2 = a2 …(ii) Eccentric Angle of a Point
a2 9 9
Now, from Eqs. (i) and (ii), we get x2 y2
5 2 Let P be any point on the ellipse = 1. Draw PM +
a = 5a Þ a = 9, so b 2 = 45 a 2 b2
9 perpendicular from P on the major axis of the ellipse and
\ a2 + b 2 = 81 + 45 = 126 produce MP to meet the auxiliary circle in Q Join CQ.
Hence, option (c) is correct. The Ð ACQ = f is called the eccentric angle of the point
P on the ellipse.
Position of a Point with respect to
Example 6. Which of the following points lies on the locus
an Ellipse of the foot of perpendicular drawn upon any tangent to the
2 2
x y x2 y 2
Let 2
+ 2
= 1 be the equation of an ellipse and P ( x1 , y1 ) ellipse, + = 1 from any of its foci?
a b 4 2 (JEE Main 2020)
be any point in the plane of an ellipse, then (a) ( -2, 3) (b) ( -1, 2)
x2 y2 (c) ( -1, 3)
S1 = 12 - 12 - 1 (d) (1, 2)
a b
Y Sol. (c) As we know that locus of the foot of perpendicular drawn
P(outside) x2 y 2
B upon any tangent to the ellipse + = 1 from any of its foci
P(on) 4 2
P(inside) is the auxilliary circle and its equation is
X¢ X
A¢ A x2 + y 2 = 4 …(i)
B¢ So, from the option the auxilliary circle (i) passes through the
Y¢ point ( -1, 3).
Ellipse 381

Sol. (a) Given equation of line can be rewritten as


Intersection of a Line -2x 12 -2x
y= + = +4
and an Ellipse 3 3 3
2
x2 y2 Here, m=- ,c=4
Let the ellipse be +
=1 …(i) 3
a 2 b2
Also, equation of ellipse can be rewritten as
and the given line be y = mx + c …(ii) x2 y 2
+ =1
On eliminating y from Eqs. (i) and (ii), we get 18 8
2 2
x2 ( mx + c)2 Here, a = 18, b = 8
+ =1
a2 b2 æ a2m b 2 ö é 18 ´ ( -2 / 3) 8ù
\Point of contact is ç m ,± ÷= m ,± ú
Þ ( a 2m 2 + b2 )x 2 + 2mca 2x 2 + c2a 2 - a 2b2 = 0 è c c ø êë 4 4û

It is a quadratic equation in x which gives two values of x. = ( ±3, 2)


Discriminant, D = 4m 2c2a 2 - 4( a 2m 2 + b2 )( c2a 2 - a 2b2 )
Example 8. The length of the minor axis (along Y-axis) of
= - a 2b2c2 + a 4b2m 2 + a 2b4 4
an ellipse in the standard form is . If this ellipse touches
2 2 2
= - a b (c - a m - b ) 2 2 2 3
the line, x + 6y = 8 ; then its eccentricity is (JEE Main 2020)
(i) If D > 0, i.e. c2 > a 2m 2 + b2, then line intersect the
ellipse at two points. 5 1 11 1 11 1 5
(a) (b) (c) (d)
y = mx + c 6 2 3 3 3 2 3

Interpret (b) According to given information,


x2 3y 2
Let the equation of ellipse + =1 …(i)
a2 4
Q Ellipse (i) touches the line x + 6y = 8, then
(ii) If D = 0, i. e. , c2 = a 2m 2 + b2 , then the line intersect 2 2
æ8ö 2æ 1 ö 4
the ellipse at one point. ç ÷ =a ç ÷ +
è6ø è6ø 3
y = mx + c
16 a2 4 a2 4
Þ = + Þ =
9 36 3 36 9
2
Þ a = 16
b2 4 1 11
Now, eccentricity e = 1 - = 1- =
(iii) If D < 0 i.e., c2 < a 2m 2 + b2, then line intersect the a2 3 ´ 16 2 3
ellipse at an imaginary point i.e., the line neither Hence, option (b) is correct.
touches nor intersects the ellipse.
y = mx + c
Tangent to the Ellipse
A line which intersect the ellipse at only one point is
called the tangent to the ellipse.

Equation of Tangent in Different Forms


Condition of Tangency (i) Point Form The equation of tangent at the point
The line y = mx + c will be a tangent to the ellipse x2 y2 xx yy
( x1 , y1 ) to the ellipse 2 + 2 = 1 is 21 + 21 = 1, if
x2 y2 a b a b
+ = 1 at any point on the ellipse, if
a 2 b2 point lies on the curve.
c2 = a 2m 2 + b2 (ii) Slope Form The equation of tangents of slope m to
æ a 2m b2 ö x2 y2
and point of contact is ç m ,± ÷. the ellipse 2 + 2 = 1 in the slope form are
è c cø a b
y = mx ± a 2m 2 + b2 and the coordinates of the
Example 7. The line 2 x + 3y = 12 touches the ellipse points of contact are
x2 y 2 æ ö
+ = 2 at the point. a 2m b2
9 4 çm ,± ÷
ç a 2m 2 + b2 a 2m 2 + b2 ÷ø
(a) (3, 2) (b) (2, 3) (c) (–3, 2) (d) (3, 4) è
382 JEE Main Mathematics

(iii) Parametric Form The equation of tangent to the Sol. (d) Given equation of ellipse is x2 + 2y 2 = 2 , which can be
x2 y2 written as
ellipse 2 + 2 = 1 at the point ( a cos q , b sin q ) is
a b x2 y 2
+ =1
x y 2 1
cos q + sin q = 1.
a b Let P be a point on the ellipse, other than its four vertices.
Then, the parametric coordinates of P be ( 2 cos q, sin q)
Note The intersection point of two tangents of the ellipse at
Y
points (a cos q1, b sin q1 ) and (a cos q2 , b sin q2 ) is
B
æ æ q1 + q2 ö æ q + q2 ö ö
ç a cos çè ÷ b sin ç 1 ÷
2 ø è 2 ø÷
ç , ÷. P (Ö2 cos q, sin q)
ç æ q1 - q2 ö æ q1 - q2 ö ÷
ç ç ÷ ç ÷
è 2 ø ÷ø
cos co s
è è 2 ø X
A

Example 9. If 3x + 4y = 12 2 is a tangent to the ellipse


x2 y2 Now, the equation of tangent at P is
2
+ = 1 for some a Î R, then the distance between the foci x 2 cos q y sin q
a 9 + =1
of the ellipse is (JEE Main 2020) 2 1
[Q equation of tangent at ( x1, y1) is given by T = 0
(a) 2 7 (b) 4 (c) 2 2 (d) 2 5
xx yy ù
Þ 21 + 21 = 1ú
Sol. (a) Since line 3x + 4y = 12 2 is a tangent to the ellipse a b û
x2 y 2 x y
+ = 1 for some a Î R, then the equation of tangent to ellipse Þ + =1
a2 9 2 sec q cosec q
having slope ‘m’ is \A ( 2 sec q, 0) and B (0 , cosec q)
y = mx ± a2m2 + 9 …(i) Let mid-point of AB be R(h, k), then
Q Slope of line (i), m = slope of line 3x + 4y = 12 2 2 sec q cosec q
h= and k =
3 2 2
Þ m=-
4 2h = 2 sec q and 2k = cosec q
On putting the value of m in Eq. (i), we get 1 1
Þ cos q = and sin q =
3 9 2 2h 2k
y =- x± a +9
4 16 We know that, cos2 q + sin 2 q = 1
Þ 3x + 4y = ± 3 a2 + 16 …(ii) 1 1
\ 2
+ 2 =1
Since tangent 3x + 4y = 12 2 represented by Eq. (ii) for some 2h 4k
a Î R, therefore 1 1
So, locus of (h, k) is 2 + =1
12 2 = ± 3 a2 + 16 2x 4y 2

Þ 4 2 = ± a2 + 16
Þ 32 = a2 + 16 [on squaring both sides]
Normal to the Ellipse
A line which is perpendicular to the tangent of the ellipse
Þ a2 = 16 Þ a = ± 4
is called the normal to the ellipse.
b2 9 7 7
\ Eccentricity e = 1 - = 1- = =
a2 16 16 4 Equation of Normal in Different forms
7 Point Form
\ Distance between the foci =|2ae| = 2 ´ 4 ´ =2 7
4 x2 y2
The equation of normal to the ellipse 2
+ = 1 at the
Example 10. If tangents are drawn to the ellipse x 2+ 2y 2 = 2 a b2
a 2x b2 y
at all points on the ellipse other than its four vertices, then the point ( x1 , y1 ) is - = a 2 - b2
mid-points of the tangents intercepted between the coordinate x1 y1
axes lie on the curve (JEE Main 2019)
Slope Form
x2 y 2 1 1
(a) + =1 (b) + =1 The equation of normal of slope m to the ellipse
4 2 4x2 2y 2
x2 y2 m( a 2 - b2 )
x2 y 2 1 1 + = 1 is y = mx m
(c) + =1 (d) + =1 a2 b2 a 2 + b2m 2
2 4 2x2 4y 2
Ellipse 383

and points of contact are Now, let point P(2 cos q, 3 sin q ) , so equation of tangent to
æ a2 mb2 ö ellipse (i) at point P is
ç± ,± ÷. x cos q y sin q
ç a 2 + b2m 2 a 2 + b2m 2 ÷ø
+ =1 …(ii)
è 2 3
x2 y2 Since, tangent (ii) passes through point Q( 4, 4)
If a line y = mx + c be a normal to an ellipse + = 1, 4
a2 b2 \ 2 cos q + sin q = 1 …(iii)
then 3
and equation of normal to ellipse (i) at point P is
m 2( a 2 - b2 )2
c2 = 4x 3y
( a 2 + b2m 2 ) - = 4 -3
2 cos q 3 sin q
Þ 2x sin q - 3 cos qy = sin q cos q …(iv)
Parametric Form
Since, normal (iv) is parallel to line, 2x + y = 4
The equation of normal at the point ( a cos q , b sin q ) to
\ Slope of normal (iv) = slope of line, 2x + y = 4
x2 y2 2
the ellipse 2
+ = 1 is ax sec q - by cosec q = a 2 - b2 Þ tan q = - 2
a b2 3
Þ tan q = - 3 Þ q = 120 º
Example 11. Let x = 4 be a directrix to an ellipse whose
æ 3 1ö
1 Þ (sin q, cos q ) = ç ,- ÷
centre is at origin and its eccentricity is . If P(1, b), b > 0 is a è 2 2ø
2
point on this ellipse, then the equation of the normal to it at æ 3ö
Hence, point P ç - 1, ÷
P is (JEE Main 2020) è 2ø
(a) 8x - 2y = 5 (b) 4x - 3y = 2 æ 3ö
2
Now, PQ = ( 4 + 1) 2 + ç 4 - ÷
(c) 7x - 4y = 1 (d) 4x - 2y = 1 è 2ø
Sol. (d) According to the given information we can assume ellipse [given cordinates of Q ºº ( 4, 4)]
x2 y 2 25 5 5
as 2 + 2 = 1, ( a > b) = 25 + =
a b 4 2
a ì 1ü
\ = 4 Þ a = 4e Þ a = 2 ías e = ý
e î 2þ Director Circle
æ 1ö The locus of the point of intersection of perpendicular
\ b 2 = a2(1 – e2) = 4 ç1 – ÷ = 3
è 4ø tangents to an ellipse is known as its director circle.
x2 y 2 The equation of the director circle is x 2 + y 2 = a 2 + b2.
So, required ellipse is + =1
4 3 Clearly, it is a circle concentric to the ellipse and its
Now, as point P(1, b), b > 0 is a point on this ellipse, so
radius is equal to a 2 + b2 .
1 b2 9 3
+ = 1 Þ b2 = Þ b = (as b > 0) Y
4 3 4 2
æ 3ö
\ Equation of normal at point P ç1 , ÷ is
è 2ø (0, b)B 90
°
x –1 y – 3 / 2 3 P(h, k)
= Þ 2x – 2 = y –
1 3 /2 2 X¢ X
4 3 A¢(–a, 0) A(a, 0)
Þ 4x – 4 = 2y – 3 Þ 4x – 2y = 1.
B' (0, –b)
2 2
Example 12. If the normal to the ellipse 3x + 4y = 12 at a

point P on it is parallel to the line, 2 x + y = 4 and the tangent to
the ellipse at P passes through Q( 4, 4) then PQ is equal to The tangents drawn from any point on the director circle
(JEE Main 2019) of a given ellipse are always at right angle.
5 5 61 221 157 Example 13. The locus of the point of intersection of the
(a) (b) (c) (d)
2 2 2 2 x2 y 2
perpendicular tangents to ellipse + = 1 is
Sol. (a) Equation of given ellipse is 3x2 + 4y 2 = 12 9 4
x2 y 2 (a) x2 + y 2 = 4 (b) x2 + y 2 = 9
Þ + =1 …(i) (c) x2 + y 2 = 5 (d) x2 + y 2 = 13
4 3
384 JEE Main Mathematics

Sol. (d) The locus of the point of intersection of the perpendicular Sol. (a) If S be the point ( -ae, 0), we have
2 2
x y SA = a + ex, SB = a + ex2, SC = a + ex3 and SD = a + ex4
tangents to ellipse + = 1 is a director circle and whose
a2 b 2 \ SA × SB × SC × SD = ( a + ex1)( a + ex2)( a + ex3)( a + ex4)
equation is given by x2 + y 2 = a2 + b 2
= a4 + a3 e S x1 + a2e2 Sx1x2 + ae3 S x1x2x3 + e4 x1x2x3 x4
\The required equation of director circle is
b2
x2 + y 2 = 9 + 4 Þ x2 + y 2 = 13 = {(h + ae) 2 + k} (on substitution and simplification)
e2
b2
Number of Normal and = × SO 2
e2
Conormal Points
At a point on a given ellipse exactly one normal can be Pair of Tangents
drawn. If there is a point not lying on a given ellipse,
infinitely many lines can be drawn which intersect the The equation of pair of tangents drawn from an external
ellipse. Out of these lines there are atmost four lines which x2 y2
point P ( x1 , y1 ) to the ellipse 2 + 2 = 1, is
are normal to the ellipse at the points where they cut the a b
ellipse. Such points on the ellipse are called conormal SS1 = T 2
points. Some points about the conormals points and
x2 y2
various relation between their eccentric angle are as given where, S = 2
+ - 1, P(x1, y1)
below. a b2
(i) The four normals can be drawn from a point to an x12 y12
S1 = -1
2
+
ellipse. b2
a
(ii) Conormal point The point on the ellipse, the xx1
yy
normal at which to the ellipse pass through a given and T = 2 + 21 - 1
a b
point is called conormal point.
(iii) Properties of eccentric angles of conormal points Example 15. The equation of the pair of tangents drawn
(a) The sum of the eccentric angles of the conormal from the point (1, 2) to the ellipse 3x 2 + 2y 2 = 5 is
x2 y2 (a) 9x2 - 4y 2 - 24xy + 40y + 30 x - 55 = 0
points on the ellipse 2 + 2 = 1 is equal to odd
a b (b) 9x2 - 4y 2 - 24xy + 40y - 30 x + 55 = 0
multiple of p. (c) 9x2 + 4y 2 + 24xy + 40y + 30 x - 55 = 0
(b) If q 1 , q 2 , q 3 and q 4 be eccentric angles of four (d) None of the above
points on the ellipse, the normals at which are
concurrent, then Sol. (a) The combined equation of the pair of tangents drawn
from (1, 2) to the ellipse 3x2 + 2y 2 = 5 is
● S cos (q 1 + q 2 ) = 0 ● S sin (q 1 + q 2 ) = 0
(3x2 + 2y 2 - 5)(3 + 8 - 5) = (3x + 4y - 5) 2 [Q SS1 = T 2]
(c) If q 1 , q 2 and q 3 are the eccentric angles of three
x2 y2 Þ (3x2 + 2y 2 - 5)(6) = 9x2 + 16y 2 + 25 + 24xy - 40y - 30 x
points on the ellipse 2 + 2 = 1 such that
a b Þ 9x2 - 4y 2 - 24xy + 40y + 30 x - 55 = 0
sin (q 1 + q 2 ) + sin (q 2 + q 3 ) + sin (q 3 + q 1 ) = 0,
then the normals at these points are concurrent. Chord of Contact
(d) If the normals at four points P ( x1 , y1 ), Q( x2 , y2 ), x2 y2
Let PQ and PR be tangents to the ellipse =1 +
x2 y2 a 2 b2
R( x3 , y3 ) and S ( x4 , y4 ) on the ellipse 2 + 2 = 1
a b drawn from any external point P ( h , k). Then, QR is called
are concurrent, then x2 y2
chord of contact of the ellipse 2 + 2 = 1, whose equation
æ1 1 1 1ö a b
( x1 + x2 + x3 + x4 ) ç + + + ÷=4
è x1 x2 x3 x4 ø xx1 yy1
is 2 + 2 = 1 or T = 0.
a b
Example 14. If the normals at the points A, B, C and D of , y 1)
Q(x 1
an ellipse meet in a point O, then SA × SB × SC × SD is equal to
(where, S is one of the foci) P(h
,k Chord of co
) ntact
b2 b3 2b2 b2
(a) 2 × SO 2 (b) 3 × SO3 (c) 2 × SO 2 (d) 2 × 2SO 2 R(x
2,
e e e 3e y2 )
Ellipse 385

a2 Example 17. The locus of the middle points of chords of an


Example 16. If tan q 1 tan q 2 = - , then the chord joining
b2 ellipse, which passes through a fixed point, is
x2 y 2 (a) an ellipse (b) a circle
two points q 1 and q 2 on the ellipse 2 + 2 = 1 will subtend a
a b (c) a hyperbola (d) a parabola
right angle at Sol. (a) Let P( x1, y1) be the middle point of any chord AB of an
(a) focus
x2 y 2
(b) centre ellipse + = 1, then equation of chord AB is
a2 b 2
(c) end of the major axis
T = S1
(d) end of minor axis
xx1 yy1 x2 y 2
Sol. (b) Let P( a cos q1, b sin q1) and Q( a cos q2, b sin q2) be two Þ 2
+ 2 - 1 = 12 + 12 - 1
a b a b
points on the ellipse.
Y xx1 yy1 x12 y12
Þ + 2 = 2+ 2 …(i)
P(q1) a2 b a b
But it passes through a fixed point Q(h, k), so its coordinates
must satisfy Eq. (i).
X¢ X
Q A
P (x1, y1)
Q(q2)
B

Then, Q(h, k)
b hx1 ky1 x12 y12
m1 = slope of OP = tan q1 \ + 2 = 2+ 2
a a2 b a b
b This can be rewritten as
and m2 = slope of OQ = tan q2
a æ hö
2
æ kö
2

æ a 2ö ç x1 - ÷ çy1 - ÷
b 2
b 2
è 2ø è 2ø 1 æ h 2 k2 ö
\ m1m2 =
tan q1 tan q2 = 2 ´ ç - 2 ÷ = - 1 + = ç + ÷
a2 a è b ø a 2
b 2
4 è a2 b 2 ø
p Hence, locus of P( x1, y1) is
\ ÐPOQ =
2 2 2
æ hö æ kö
Hence, PQ makes a right angle at the centre of the ellipse. çx - ÷ çy - ÷
è 2ø è 2ø 1 æ h 2 k2 ö
+ = ç 2 + 2÷
a2 b 2
4 èa b ø
Equation of Chord of the Ellipse
æh kö
whose Mid-point is (x1 , y1 ) Its obviously an ellipse with centre at ç , ÷ and axes
è2 2ø
x2 y2 parallel to coordinates axes.
The equation of a chord of the ellipse 2
+ = 1 whose
a b2
mid-point is ( x1 , y1 ) is T = S1 Some Useful Properties of an Ellipse
(i) The tangent and normal at any point of an ellipse
xx1 yy1 x12 y12
where, T = 2
+ 2
- 1 and S1 = 2
+ 2
-1 bisect the external and internal angles between the
a b a b focal radii to the point.
Note Y B(0, b)
• Equation of chord of the ellipse whose eccentric angles are q T
P(x1, y1)

and f, is T'
x æ q + fö y æ q + fö æ q - fö
cos ç ÷ + sin ç ÷ = cos ç ÷
a è 2 ø b è 2 ø è 2 ø N' M
• If the chord passes through (ae, 0), then X¢ N
G X
A' S' C S A (a, 0)
f f e -1 (–ae, 0) (ae, 0)
tan 1 tan 2 = (–a, 0)
2 2 e +1
B' (0,–b)
• If the chord passes through (- ae , 0 ), then
æf ö æf ö e +1
tan ç 1 ÷ tan ç 2 ÷ =
è2ø è 2 ø e -1 Y¢
386 JEE Main Mathematics

(ii) If SM and S ¢ M ¢ are perpendiculars from the foci (a) a circle


upon the tangent at any point of the ellipse, (b) a parabola
then SM × S ¢ M ¢ = b2 and M , M ¢ lie on the auxiliary (c) an ellipse
circle. (d) a hyperbola
(iii) If the tangent at any point P on the ellipse Sol. (a) Let the two lines be taken as the X-axis and Y-axis,
x2 y2
+ = 1 meets the major axis in T and minor respectively.
a 2 b2 Let C( a , b) be the centre of the ellipse.
axis in T ¢, then Let a and b be the semi-major and semi-minor axes of the
CN × CT = a 2 , CN ¢× CT ¢ = b2 , ellipse.
where, N and N ¢ are the feet of the perpendiculars Let S( x1, y1) and S ¢ ( x2, y 2) be the foci of the ellipse.
from P on the respective axes. Then, x1 + x2 = 2 a …(i)
and y1 + y 2 = 2 b …(ii)
(iv) If SM and S ¢ M ¢ be perpendiculars from the foci
Again, since X-axis and Y-axis are the tangents to the ellipse,
S and S ¢ respectively upon a tangent to the ellipse,
\ y1y 2 = b 2 …(iii)
then CM and CM ¢ are parallel to S ¢ P and SP,
respectively. and x1x2 = b 2 …(iv)
(v) The common chord of an ellipse and a circle are Also, S ¢ S 2 = 4a2e2
equally inclined to the axes of the ellipse. Þ ( x1 - x2) 2 + (y1 - y 2) 2 = 4a2e2 …(v)
We have, to eliminate x1, y1, x2, y 2 from the Eqs. (i), (ii), (iii),
Example 18. An ellipse slides between two lines at right (iv) and (v).
angle to one another. Then, the locus of its centre is Now, {( x1 + x2) 2 + ( y1 + y 2) 2} - {( x1 - x2) 2 + ( y1 - y 2) 2}
Y = 4 ( x1x2 + y1y 2)
Þ 4a 2 + 4b 2 - 4a2e2 = 4( b 2 + b 2) = 8b 2
)
,y2 Þ a 2 + b 2 - a2e2 = 2b 2
(x 2

y
)
1 b) Þ a + b 2 = 2b 2 + a2e2 = 2b 2 + a2 - b 2 = a2 + b 2
2
, a,
(x 1 C(
Hence, the locus of ( a , b) is
S
x2 + y 2 = a2 + b 2.
X
O which is a circle.
Practice Exercise
ROUND I Topically Divided Problems
Basic Terms and Equation of Ellipse 7. Let S(5 , 12) and S ¢ ( -12 , 5) be the foci of an ellipse
1. In an ellipse, with centre at the origin, if the passing through the origin. The eccentricity of
difference of the lengths of major axis and minor ellipse is
axis is 10 and one of the foci is at (0, 5 3), then the (a) 1/2 (b) 1 / 3
length of its latus rectum is (JEE Main 2019) (c) 1 / 2 (d) 2 /3
(a) 5 (b) 10 (c) 8 (d) 6 8. Let S and S¢ be the foci of an ellipse and B be any
2. The equation of the ellipse whose focus is (1, –1), the one of the extremities of its minor axis. If DS ¢ BS is
1 a right angled triangle with right angle at B and
directrix of line x - y - 3 = 0 and eccentricity is
2 area ( DS ¢ BS) = 8 sq units, then the length of a
(a) 7x 2 + 2xy + 7 y 2 - 10x + 10 y + 7 = 0 latus rectum of the ellipse is (JEE Main 2019)

(b) 7x 2 + 2xy + 7 y 2 + 7 = 0 (a) 2 2 (b) 4 2


2 2 (c) 2 (d) 4
(c) 7x + 2xy + 7 y + 10x - 10 y - 7 = 0
(d) None of the above 9. The eccentricity of an ellipse with centre at the
origin and axes along the coordinate axes is 1/2.
3. Let the length of the latus rectum of an ellipse with
If one of the directrices is x = 4, then the ellipse is
its major axis along X-axis and centre at the origin,
(a) 4x2 + 3 y2 = 1 (b) 3x2 + 4 y2 = 12
be 8. If the distance between the foci of this ellipse
(c) 4x2 + 3 y2 = 12 (d) 3x2 + 4 y2 = 1
is equal to the length of its minor axis, then which
one of the following points lies on it? (JEE Main 2019) 10. If the focal distance of an end of the minor axis of
(a) (4 2 , 2 3 ) (b) (4 3 , 2 2 ) an ellipse (referred to its axes as the X and Y-axes
(c) (4 2 , 2 2 ) (d) (4 3 , 2 3 ) respectively) is k and the distance between its foci
is 2h, then its equation is
4. The eccentricity of ellipse whose axes are parallel x2 y2 x2 y2
to the coordinate axes having its centre at the point (a) 2
+ 2 =1 (b) 2
+ 2 =1
k h k k - h2
( 2 , - 3), one focus at (3 , - 3) and one vertex is
x2 y2 x2 y2
( 4 , - 3) , is (c) + 2 =1 (d) + 2 =1
2
(a) 3 /4 (b) 1 / 3 (c) 1 /2 (d) 1 / 2 k h - k2 k 2
k + h2

5. The eccentricity of ellipse 11. The foci of the ellipse 25 ( x + 1) 2 + 9 ( y + 2) 2 = 225


4 ( x - 2 y + 1) 2 + 9 ( 2 x + y + 2) 2 = 180 is are
(a) 5 /3 (b) 3 /4 (c) 1 /2 (d) 1/4 (a) (-1, 2) and (-1, - 6) (b) (1, 2) and (1, 6)
(c) (1, 2) and (-1, - 6) (d) (-1, 2) and (1, 6)
( x - h) 2 ( y - k) 2
6. If the ellipse + = 1 has major axis 12. The focus of an ellipse is ( -1, - 1) and the
M N
on the line y = 2 , minor axis on the line x = - 1. corresponding directrix is x - y + 3 = 0. If the
Major axis has length 10 and minor axis has length eccentricity of ellipse is 1/2, then the coordinates of
4, then the values of h, k, N , M respectively are centre of the ellipse is
(a) -1 , 2 , 5 , 2 æ1 3ö æ 1 3ö
(a) ç , ÷ (b) ç - , ÷
(b) -1 , 2 , 10 , 4 è2 2ø è 2 2ø
(c) 1, - 2 , 25 , 4 æ 1 3ö
(c) ç - , - ÷ (d) None of thses
(d) -1 , 2 , 25 , 4 è 2 2ø
388 JEE Main Mathematics

13. The curve represented by the equation x2 y2


20. If the eccentricity of the two ellipse + = 1 and
169 25
4 x 2 + 16 y 2 - 24 x - 32 y - 12 = 0 is 2 2
x y a
(a) a parabola 2
+ 2 = 1 are equal, then the value of is
a b b
(b) a pair of straight lines
5 6 13 13
1 (a) (b) (c) (d)
(c) an ellipse with eccentricity 13 13 5 6
2
3 21. The radius of the circle passing through the foci of the
(d) an ellipse with eccentricity x2 4 2
+ y = 1 and having its centre at æç , 2 ö÷, is
2 1
ellipse
4 7 è 2 ø
x2 y2
14. Let E be the ellipse + = 1 and C be the circle 7
9 4 (a) 5 (b) 3 (c) 12 (d)
2
x 2 + y 2 = 9. Let P and Q be the points (1, 2) and
(2, 1), respectively. Then, 22. A man running round a race course notes that the
(a) Q lies inside C but outside E
sum of the distances of two flag posts from him is
always 10 m and the distance between the flag posts
(b) Q lies outside both C and E
is 8 m. The area of the path he encloses (in square
(c) P lies inside both Cand E
metre) is
(d) P lies inside C but outside E
(a) 15 p (b) 12 p (c) 18 p (d) 8 p
15. The length of the axes of the conic
9 x 2 + 4 y 2 - 6 x + 4 y + 1 = 0, are Intersection of a Ellipse and a Curve
1 2 2 23. The ellipse 4 x 2 + 9 y 2 = 36 and the straight line
(a) ,9 (b) 3 , (c) 1 , (d) 3, 2
2 5 3
y = mx + c intersect in real points only if
16. In an ellipse length of minor axis is 8 and (a) 9m2 £ c2 - 4 (b) 9m2 > c2 - 4
5 (c) 9m2 ³ c2 - 4 (d) None of these
eccentricity is . The length of major axis is
3 x2 y2
24. If the point of intersections of the ellipse + 2 =1
(a) 6 (b) 12 (c) 10 (d) 16 16 b
17. If P ( x, y), F1 (3, 0), F2 ( -3, 0) and 16 x + 25 y 2 = 400,
2 and the circle x 2 + y 2 = 4 b, b > 4 lie on the curve

then PF1 + PF2 equals y 2 = 3x 2 , then b is equal to (JEE Main 2021)


(a) 12 (b) 5 (c) 6 (d) 10
(a) 8 (b) 6 (c) 10 (d) 12
x2 y2
18. The eccentric angle of a point on the ellipse 25. The line lx + my + n = 0 cuts the ellipse 2
+ 2 =1
a b
x2 y2 p
+ = 1, whose distance from the centre of the in points whose eccentric angles differ by , if
6 2 2
ellipse is 2, is a 2 l 2 + b2 m 2 is equal to
p 3p 5p 7p (a) n 2 (b) 2n 2 (c) 2n (d) n
(a) (b) (c) (d)
4 2 3 6 26. Equation of the circle passing through the
ì y2 x2 ü x2 y2 x2 y2
19. Let S = í ( x, y) Î R 2 : - = 1ý, intersection of ellipses + = 1 and + = 1, is
î 1 + r 1 - r þ a 2 b2 b2 a 2
where r ¹ ± 1. Then, S represents (JEE Main 2019) (a) x 2 + y 2 = a 2 (b) x 2 + y 2 = b 2
a 2b 2 2a 2b 2
2 (c) x 2 + y 2 = 2 (d) x 2 + y 2 = 2
(a) a hyperbola whose eccentricity is , a + b2 a + b2
1-r
when 0 < r < 1. Tangent to the Ellipse
2
(b) a hyperbola whose eccentricity is , 27. The equation of tangents to the ellipse
r+1 3x 2 + 4 y 2 = 5, which are inclined at 30° to the
when 0 < r < 1. X-axis, are
2 5 1 5
(c) an ellipse whose eccentricity is , when r > 1. (a) y = 3x ± (b) y = x±
r+1 2 3 2
1
1 (c) y = x±1 (d) None of these
(d) an ellipse whose eccentricity is , when r > 1. 3
r+1
Ellipse 389

28. The angle between the pair of tangents drawn from 36. If tangents are drawn to the ellipse x 2 + 2 y 2 = 2 ,
2 2
the point (1, 2) to the ellipse 3x + 2 y = 5, is then the locus of the mid-point of the intercept made
(a) tan -1 (12 / 5) (b) tan -1 (6/ 5 ) by the tangents between the coordinate axes is
(c) tan -1 (12 / 5 ) (d) tan -1 (6 / 5) 1 1 1 1
(a) + =1 (b) + =1
29. If x cos a + y sin a = p is a tangent to the ellipse, 2x 2 4 y 2 4x 2 2 y 2
x 2 y2 x 2 y2
then (c) + =1 (d) + =1
2 4 4 2
(a) a 2 sin 2 a + b 2 cos 2 a = p2
(b) a 2 + b 2 sin 2 a = p2 cosec 2 a x2
37. Tangent is drawn to the ellipse + y 2 = 1 at
(c) a 2 cos 2 a + b 2 sin 2 a = p 2 27
é p ù
(3 3 cos q, sin q) ê where, q Î æç0, ö÷ ú. Then, the value
(d) None of the above
ë è 2 øû
30. If the tangents on the ellipse 4 x 2 + y 2 = 8 at the
of q such that sum of intercepts on axes made by
points (1, 2) and ( a, b) are perpendicular to each
this tangent is minimum, is
other, then a 2 is equal to (JEE Main 2019)
128 64 4 2 (a) p/3 (b) p /6 (c) p/8 (d) p/4
(a) (b) (c) (d)
17 17 17 17 x2 y2
38. If ( 3) bx + ay = 2 ab touches the ellipse 2
+ 2 = 1,
a b
31. The distance of the centre of ellipse x 2 + 2 y 2 - 2 = 0
then eccentric angle f is
to those tangents of the ellipse which are equally p p p p
inclined from both the axes, is (a) (b) (c) (d)
6 4 3 2
3 3 2 3
(a) (b) (c) (d) 39. If p is the length of the perpendicular from a focus
2 2 3 2
upon the tangent at any point P of the ellipse
32. If the line x - 2 y = 12 is tangent to the ellipse
x2 y2
x2 y2 -9 ö + 2 = 1 and r is the distance of P from the
+ 2 = 1 at the point æç3,
2
÷, then the length of a b
a 2
b è 2 ø 2 a b2
focus, then - 2 is equal to
the latusrectum of the ellipse is (JEE Main 2019) r p
(a) 8 3 (b) 9 (a) - 1 (b) 0 (c) 1 (d) 2
(c) 5 (d) 12 2
40. The eccentricity of an ellipse whose centre is at the
x2 y2
33. Equation of tangents to the ellipse + = 1, origin is 1/2. If one of its directrices is x = - 4, then
9 4
the equation of the normal to it at æç1, ö÷ is
3
which are perpendicular to the line 3x + 4 y = 7, are è 2ø
(a) 4x - 3 y = ± 20 (b) 4x - 3 y = ± 12 (JEE Main 2017)
(c) 4x - 3 y = ± 2 (d) 4x - 3 y = ± 1 (a) 2 y - x = 2 (b) 4x - 2 y = 1
(c) 4x + 2 y = 7 (d) x + 2 y = 4
34. On the ellipse 2 x 2 + 3 y 2 = 1 the points at which the
41. If the tangent at the point æç 4 cos f, sin f ö÷ to the
16
tangent is parallel to 4 x = 3 y + 4, are è ø
11
æ 2 1 ö æ 2 1 ö
(a) ç , ÷ or ç - ,- ÷ ellipse 16 x 2 + 11 y 2 = 256 is also a tangent to the
è 11 11 ø è 11 11 ø
circle x 2 + y 2 - 2 x = 15, then the value of f is
æ 2 1 ö æ 2 1 ö
(b) ç - , ÷ or ç ,- ÷ (a) ± p/2 (b) ± p /4 (c) ± p/3 (d) ± p / 6
è 11 11 ø è 11 11 ø
æ 2 1ö
42. A tangent at any point to the ellipse 4 x 2 + 9 y 2 = 36
(c) ç - , - ÷
è 5 5ø is cut by the tangent at the extremities of the major
æ 3 2ö æ 3 2ö axis at T and T ¢. The circle on T T ¢ as diameter
(d) ç - ,- ÷ or ç , ÷
è 5 5ø è 5 5ø passes through the point
(a) (0, 5 ) (b) ( 5 , 0)
35. The locus of point of intersection of perpendicular (c) (2, 1) (d) (0, - 5 )
x2 y2 x2 y2
tangents to + = 1 and + = 1 is 43. Product of the perpendicular from the foci upon any
a 2 b2 a+ l b+ l x2 y2
2 2 2 2 2 2 tangent to the ellipse + 2 = 1 ( a < b) is equal to
(a) x + y = a + l (b) x + y = b + l a 2
b
2 2 2
(c) x + y = a + b + l 2
(d) x2 + y2 = a 2 + b2 (a) 2a (b) a 2 (c) b 2 (d) ab 2
390 JEE Main Mathematics

44. Let d be the perpendicular distance from the 51. If the line x cos a + y sin a = p be normal to the
x2 y2 x2 y2
centre of ellipse 2 + 2 = 1 to the tangent drawn ellipse + = 1, then
a b a 2 b2
at a point P on the ellipse. If F1 and F2 are two foci (a) p2 (a 2 cos 2 a + b 2 sin 2 a ) = a 2 - b 2
æ (b) p2 (a 2 cos 2 a + b 2 sin 2 a ) = (a 2 - b 2) 2
b2 ö
of the ellipse, where ( PF1 - PF2 ) 2 = la 2 ç1 - 2 ÷, (c) p2 (a 2 sec 2 a + b 2 cosec 2 a ) = a 2 - b 2
è d ø
(d) p2 (a 2 sec 2 a + b 2 cosec 2 a ) = (a 2 - b 2) 2
then l is equal to
(a) 2 (b) 3 (c) 4 (d) 5 52. The equation of the normal at the point (2, 3) on
the ellipse 9 x 2 + 16 y 2 = 180 is
45. The sum of the squares of the perpendiculars on
(a) 3 y = 8x - 10 (b) 3 y - 8x + 7 = 0
x2 y2 (c) 8 y + 3x + 7 = 0 (d) 3x + 2 y + 7 = 0
any tangent to the ellipse 2 + 2 = 1 from two
a b
53. Equation of the normal to the ellipse
points on the minor axis each at a distance
4 ( x - 1) 2 + 9 ( y - 2) 2 = 36, which is parallel to the
a 2 - b2 from the centre is
line 3x - y = 1, is
(a) 2 a 2 (b) 2 b 2 (c) a 2 + b 2 (d) a 2 - b 2
(a) 3x - y = 5 (b) 3x - y = 5 - 3
46. The equations of tangents to the ellipse (c) 3x - y = 5 + 2 (d) 3x - y = 5 ( 5 + 1)
4 x 2 + 3 y 2 = 5 , which are inclined at an angle of 60°
to the X-axis is 54. If the normal at the point P (q) to the ellipse
65 x2 y2
(a) y = 3x ± (b) y = 3x ± 5 + = 1 intersect it again at the point Q (2 q),
12 14 5
1 65 1 then cosq is equal to
(c) y = x± (d) y = x±5 2 2 1 1
3 12 3 (a) (b) - (c) (d) -
3 3 3 3
47. A common tangent to circle x 2 + y 2 = 16 and an
55. In the normal at the end of latusrectum of the
x2 y2 x2 y2
ellipse + = 1 is ellipse + = 1 with eccentricity e, passes
49 4 a 2 b2
(a) y = x + 4 5 (b) y = x + 53 through one end of the minor axis, then
2 4 4 (a) e 2 (1 + e 2) = 0 (b) e 2 (1 + e 2) = 1
(c) y = x+ (d) None of these
11 11 (c) e 2 (1 + e 2) = - 1 (d) e 2 (1 + e 2) = 2

48. Let L be a tangent line to the parabola y 2 = 4 x - 20 ( a 2 - b2 ) m


56. If y = mx - is normal to the ellipse
at (6, 2). If L is also a tangent to the ellipse a 2 + b2 m 2
x2 y2 x2 y2
+ = 1, then the value of b is equal to 2
+ 2 = 1 for all values of m belonging to
2 b (JEE Main 2021) a b
(a) 11 (b) 14 (c) 16 (d) 20 (a) (0, 1) (b) (0, ¥)
2 2 (c) R (d) None of these
49. The point on the ellipse 16 x + 11 y = 256 where
the common tangent to it and the circle 57. If w is one of the angles between the normals to the
x 2 + y 2 - 2 x = 15 touch are x2 y2
ellipse + = 1 at the points whose eccentric
æ 8 3ö a 2 b2
(a) ç2, ± ÷ (b) (± 4, 5) p
è 11 ø angles are q and + q , then
2
æ 7 5 ö æ 3ö 2 cot w e2 3 cot w e
(c) ç ± , 2÷ (d) ç1, ± ÷ (a) = (b) =
è 2 ø è 5ø sin 2 q 1 - e2 sin 2 q 1 + e2
3 cot w 2e2 2 cot w e
Normal to the Ellipse (c) = (d) =
sin 2 q 1 + e sin 2 q 1-e
x2 y2
50. If the normal at point P on the ellipse 2
+ 2 =1 58. If the normal at one end of the latusrectum of an
a b
meets the axes in R and S respectively, then x2 y2
ellipse 2
+ 2 = 1 passes through the one end of
PR : RS is equal to a b
(a) a : b (b) a 2 : b 2 (c) b 2 : a 2 (d) b : a the minor axis, then (JEE Main 2020)
Ellipse 391

(a) e4 - e 2 + 1 = 0 (b) e 2 - e + 1 = 0 a2 b2 a4 b4
(a) (b) - (c) - (d) -
(c) e 2 + e + 1 = 0 (d) e4 + e 2 - 1 = 0 b2 a2 b4 a4

Chord of Contact of an Ellipse 62. Tangents are drawn from the point P (3, 4) to the
x2 y2
59. Locus of the mid-points of the chord of ellipse ellipse + = 1 touching the ellipse at point
x2 y2 9 4
+ = 1, so that chord is always touching the A and B. The equation of the locus of the point
a 2 b2
circle x 2 + y 2 = c2 , ( c < a , c < b) is whose distance from the point P and the line AB
(a) (b 2x 2 + a 2y 2) 2 = c 2(b 4x 2 + a 4 y 2)
are equal is
(a) 9x2 + 9 y2 - 6xy - 54xy - 62 y - 241 = 0
(b) (a 2x 2 + b 2y 2) 2 = c 2(a 4x 2 + b4 y 2)
(b) x2 + y2 - 2xy + 27x + 31 y - 120 = 0
(c) (b 2x 2 + a 2y 2) 2 = c 2(b 2x 4 + a 2y 4 ) (c) 9x2 + y2 - 6xy - 54x - 62 y + 241 = 0
(d) None of the above (d) x2 + 9 y2 + 6xy - 54x + 62 y - 241 = 0
60. Tangents are drawn from the point on the line x2 y2
2 2 63. Equation of chord of an ellipse + = 1, whose
x - y - 5 = 0 to x + 4 y = 4, then all the chords of 25 9
contact pass through a fixed point, whose mid-point is (1, 1), is
coordinates are (a) 25x + 9 y = 36 (b) 9x + 25 y = 34
æ1 4ö æ4 1ö (c) 9x - 25 y = 34 (d) None of these
(a) ç , ÷ (b) ç , ÷
è5 5ø è5 5ø
64. Equation of the chord of contact of pair of tangents
æ 4 1ö æ4 1ö
(c) ç - , - ÷ (d) ç , - ÷ drawn to the ellipse 4 x 2 + 9 y 2 = 36 from the point
è 5 5ø è5 5ø
( m, n), where mn = m + n, m and n being non-zero
61. If the chords of contact of tangents from two points positive integers is
x2 y2 (a) 2x + 9 y = 18 (b) 2x + 2 y = 1
( x1, y1) and ( x2 , y2 ) to the ellipse + = 1 are at
a 2 b2 (c) 4x + 9 y = 18 (d) None of these
x1x2
right angles, then is equal to
y1 y2

Only One Correct Option (b) is constant


1. The area (in sq units) of the quadrilateral formed (c) depends upon the tangent
by the tangents at the end points of the latera recta (d) None of the above
x2 y2 5. Tangents are drawn to the ellipse
to the ellipse + = 1 is
9 5 (JEE Main 2015) x2 y2
27 27 + = 1 ( a > b) and the circle x 2 + y 2 = a 2 at
(a) (b) 18 (c) (d) 27 a2 b2
4 2
the points, where a common ordinate cuts them (on
2. The locus of the foot of perpendicular drawn from the same side of the X-axis). Then, the greatest
the centre of the ellipse x 2 + 3 y 2 = 6 on any tangent
acute angle between these tangents is given by
to it, is (JEE Main 2014)
æ a - bö æ a + bö
(a) (x2 - y2)2 = 6x2 + 2 y2 (b) (x2 - y2)2 = 6x2 - 2 y2 (a) tan -1 ç ÷ (b) tan -1 ç ÷
è 2 ab ø è 2 ab ø
(c) (x2 + y2)2 = 6x2 + 2 y2 (d) (x2 + y2)2 = 6x2 - 2 y2
æ 2ab ö æ 2ab ö
3. Let (a , b) be a point from which two perpendicular (c) tan -1 ç ÷ (d) tan -1 ç ÷
è a - bø è a + bø
tangents can be drawn to the ellipse
4 x 2 + 5 y 2 = 20. If F = 4 a + 3 b, then 6. Minimum area of the triangle by any tangent to the
(a) -15 £ F £ 15 (b) F ³ 0 x2 y2
ellipse + = 1 with the coordinate axes is
(c) -5 £ F £ 20 (d) F £ - 5 5 or F ³ 5 5 a 2 b2
4. The product of the perpendiculars drawn from the (a + b)2
(a) a 2 + b2 (b)
foci upon any tangent to an ellipse 2
(a - b)2
(a) depends upon foci (c) ab (d)
2
392 JEE Main Mathematics

7. The locus of the points of intersection of the 14. If the tangent at the point P (q) to the ellipse
tangents at the extremities of the chords of the 16 x 2 + 11 y 2 = 256 is also a tangent to the circle
ellipse x 2 + 2 y 2 = 6 which touches the ellipse x 2 + y 2 - 2 x = 15, then q is equal to
x 2 + 4 y 2 = 4, is 2p 4p 7p p
2 2 2 2
(a) (b) (c) (d)
(a) x + y = 4 (b) x + y = 6 3 3 3 3
(c) x2 + y2 = 9 (d) None of these
15. Let the line y = mx and the ellipse 2 x 2 + y 2 = 1
8. If a number of ellipses be described having the intersect at a point P in the first quadrant. If the
same major axis, but a variable minor axis, such normal to this ellipse at P meets the co-ordinate
that each one of the tangents at the ends of their
axes at æç - , 0 ö÷ and (0, b), then b is equal to
1
latusrectum passes through the fixed point, then è 3 2 ø
(JEE Main 2020)
the point of concurrency is
2 2 2 2 2
(a) (1, 1) (b) (0, 1) (a) (b) (c) (d)
3 3 3 3
(c) (1, 0) (d) None of these
x2 y2 16. If the tangent to the parabola y 2 = x at a point
9. If the ellipse + = 1 is inscribed in a
2
a - 7 13 - 5 a (a , b), (b > 0) is also a tangent to the ellipse,
square of side length 2a, then a is equal to x 2 + 2 y 2 = 1, then a is equal to (JEE Main 2019)

(a) 6/5 (b) (- ¥ , - 7 ) È ( 7 , 13 /5) (a) 2 + 1 (b) 2 - 1


(c) (- ¥ , - 7 ) È (13 /5, 7 ) (d) No such a exists (c) 2 2 + 1 (d) 2 2 - 1
x2 y2 17. The equation of the circle passing through the foci
10. If the ellipse + 2 = 1 is inscribed in a rectangle
a 2
b x2 y2
of the ellipse + = 1 and having centre at (0, 3)
whose length to breadth ratio is 2 : 1, then the area 16 9
of the rectangle is is (JEE Main 2013)
2 2 2 2
a +b2 2
a +b2 2
(a) x + y - 6 y - 7 = 0 (b) x + y - 6 y + 7 = 0
(a) 4 (b) 4
7 3 (c) x 2 + y 2 - 6 y - 5 = 0 (d) x 2 + y 2 - 6 y + 5 = 0
a 2 + b2 a 2 + b2 18. An ellipse is drawn by taking a diameter of the
(c) 12 (d) 8
5 5 circle ( x - 1) 2 + y 2 = 1 as its semi-minor axis and a
11. A parabola is drawn with focus is at one of the foci diameter of the circle x 2 + ( y - 2) 2 = 4 is
x2 y2 semi-major axis. If the centre of the ellipse is at the
of the ellipse 2
+ 2 = 1 (where, a > b) and
a b origin and its axes are the coordinate axes, then the
directrix passing through the other focus and equation of the ellipse is
perpendicular to the major axes of the ellipse. If (a) 4x 2 + y 2 = 4 (b) x 2 + 4 y 2 = 8
latusrectum of the ellipse and the parabola are (c) 4x 2 + y 2 = 8 (d) x 2 + 4 y 2 = 16
same, then the eccentricity of the ellipse is 19. Equation of the ellipse whose axes are the axes of
1
(a) 1 - (b) 2 2 - 2 coordinates and which passes through the point
2
(-3, 1) and has eccentricity 2 /5 is
(c) 2 - 1 (d) None of these (a) 5x 2 + 3 y 2 - 48 = 0 (b) 3x 2 + 5 y 2 - 15 = 0
12. The points, where the normals to the ellipse (c) 5x 2 + 3 y 2 - 32 = 0 (d) 3x 2 + 5 y 2 - 32 = 0
x 2 + 3 y 2 = 37 be parallel to the line 6 x - 5 y + 7 = 0 20. The ellipse x 2 + 4 y 2 = 4 is inscribed in a rectangle
is/are
aligned with the coordinate axes, which is turn in
(a) (5, 2) (b) (2, 5)
inscribed in another ellipse that passes through the
(c) (1, 3) (d) (6, 5) point (5, 0). Then, the equation of the ellipse is
13. Let F1 and F2 be two foci of the ellipse and PT and (a) x 2 + 12 y 2 = 16 (b) 4x 2 + 48 y 2 = 48
(c) 4x 2 + 64 y 2 = 48 (d) x 2 + 16 y 2 = 16
PN be the tangent and the normal respectively to
the ellipse at point P. Then, 21. A focus of an ellipse is at the origin. The directrix is
(a) PN bisects Ð F1PF2 the line x = 4 and the eccentricity is 1/2, then
(b) PT bisects Ð F1PF2 length of semi-major axis is
(c) PT bisects angle (180° + Ð F1PF2) 5 8 2 4
(a) (b) (c) (d)
(d) None of the above 3 3 3 3
Ellipse 393

22. If the angle between the lines joining the end x - y + 3 = 0 and eccentricity is 1/2, is
p lx 2 + 7 y 2 + 2 xy + 10 x - 10 y + m = 0, then the value
points of minor axis of an ellipse with its foci is ,
2 of|l - m|is ……… .
then the eccentricity of the ellipse is
(a) 1/2 (b) 1/ 2 (c) 3/2 (d) 1/2 2
26. A man running on the race course notes that the
sum of distance of two flag-posts from him is
23. Let d be the perpendicular distance from the centre always 10 m and the distance between the
of the ellipse to any tangent to the ellipse. If F1 and flag-posts is 8 m. If the area of path he encloses (in
F2 are the two foci of the ellipse, then sq m) is (3lp), then the value of l is ……… .
æ b2 ö
(a) (PF1 + PF2)2 = a 2 ç1 + 2 ÷ 27. The length of latusrectum of the ellipse
è d ø
(10 x - 5) 2 + (10 y - 5) 2 = (3x + 4 y - 1) 2 is l, then the
4a 2b2
(b) (PF1 + PF2)2 = value of (162l) is ……… .
d2
æ b2 ö 28. The slope of common tangent to the ellipses
(c) (PF1 - PF2)2 = 4a 2 ç1 - 2 ÷
è d ø x2 y2 x2 y2
+ = 1 and + = 1 is m, then the value of
æ d2 ö 6 1 5 6
(d) (PF1 - PF2)2 = a 2 ç1 + 2 ÷
è b ø m 2 is ……… .
29. The shortest distance from the ellipse x 2 + 2 y 2 = 2
24. The eccentricity of an ellipse, with centre at the
origin, is 1/2 If one directrix is x = 4, the equation of k-1
and the circle x 2 + y 2 - 3x - 2 2 y + 4 = 0 is ,
the ellipse is (AIEEE 2004) 2
(a) 3x 2 + 4 y 2 = 1 (b) 3x 2 + 4 y 2 = 12 then the value of k is ……… .
(c) 4x 2 + 3 y 2 = 1 (d) 4x 2 + 3 y 2 = 12
30. If the normal at the point P (q) to the ellipse
Numerical Value Type Questions x2 y2
+ = 1 intersect it again at the point Q( 2 q ) ,
25. The equation of ellipse whose focus is the point 14 5
( -1 , 1) whose directrix is the straight line then the value of 3 cosq + 2 is ……… .

Answers
Round I
1. (a) 2. (a) 3. (b) 4. (c) 5. (a) 6. (d) 7. (c) 8. (d) 9. (b) 10. (b)
11. (a) 12. (c) 13. (d) 14. (d) 15. (c) 16. (b) 17. (c) 18. (a) 19. (c) 20. (c)
21. (a) 22. (a) 23. (c) 24. (a) 25. (b) 26. (d) 27. (d) 28. (c) 29. (c) 30. (d)
31. (d) 32. (b) 33. (a) 34. (a) 35. (c) 36. (a) 37. (b) 38. (a) 39. (c) 40. (b)
41. (c) 42. (b) 43. (b) 44. (c) 45. (a) 46. (a) 47. (d) 48. (b) 49. (a) 50. (c)
51. (d) 52. (b) 53. (d) 54. (b) 55. (b) 56. (c) 57. (a) 58. (d) 59. (a) 60. (d)
61. (c) 62. (c) 63. (b) 64. (c)

Round II
1. (d) 2. (c) 3. (a) 4. (b) 5. (a) 6. (c) 7. (c) 8. (b) 9. (d) 10. (d)
11. (c) 12. (a) 13. (d) 14. (d) 15. (b) 16. (a) 17. (a) 18. (d) 19. (d) 20. (a)
21. (b) 22. (b) 23. (c) 24. (b) 25. (0) 26. (5) 27. (81) 28. (5) 29. (3) 30. (0)
Solutions
Round I 1
Þ 2e2 = 1 Þ e = …(ii)
2 2 2
x y
1. One of the focus of ellipse + = 1 is on Y-axis
a 2 b2 From Eqs. (i) and (ii), we get
(0, 5 3 ). b =4 2
\ be = 5 3 …(i) b2 32
Now, a2 = = = 64
[where, e is eccentricity of ellipse] 1-e 2 1
1-
According to the question, 2
2b - 2a = 10 x2 y2
\Equation of ellipse be + =1
Þ b - a =5 …(ii) 64 32
On squaring Eq. (i) both sides, we get Now, check all the options.
b2e2 = 75 Only (4 3 , 2 2 ), satisfy the above equation.
æ a2ö é 2 a2ù 4. Let 2a and 2b be the major and minor axes of the ellipse.
Þ b2ç1 - 2 ÷ = 75 êQ e = 1 - 2 ú
è b ø ë b û Then, its equation will be
Þ 2
b - a = 75 2 (x - 2)2 ( y + 3)2
+ =1
Þ (b + a )(b - a ) = 75 a2 b2
Þ b + a = 15 [from Eq. (ii)] …(iii) We have, a = (4 - 2)2 + (-3 + 3)2 = 2
On solving Eqs. (ii) and (iii), we get Since, the distance between the focus and the centre of
b = 10 and a = 5 ellipse is equal to ae, therefore
2a 2 2 ´ 25 ae = (2 - 3)2 + (- 3 + 3)2
So, length of latusrectum is = =5
b 10
Þ ae = 1
2. By definition of ellipse SP = ePM , where P is any point, 1 1
Þ e= Þ e=
S is focus and M is point on directrix. a 2
1 x- y-3 5. We have, 4(x - 2 y + 1)2 + 9 (2x + y + 2)2 = 180
\ (x - 1 )2 + ( y + 1 )2 =
2 12 + 12 2 2
æ x - 2y + 1ö æ 2x + y + 2 ö
1 ç ÷ ç ÷
Þ (x - 1 )2 + ( y + 1 )2 = x- y-3 è 1+4 ø è 4 +1 ø
2 2 Þ + =1
2 2 2 2
32 22
Þ 8(x + 1 - 2x + y + 1 + 2 y) = x + y + 9 - 2xy + 6 y - 6x
X2 Y 2
Þ 7x2 + 7 y2 + 2xy - 10x + 10 y + 7 = 0 Þ + 2 =1
32 2
x2 y2 x - 2y + 1 2x + y + 2
3. Let the equation of ellipse be + = 1. where, X = and Y =
a 2 b2 5 5
Then, according the problem, we have This is of the form
2b2
= 8 and 2ae = 2b X2 Y 2
a + 2 =1
a2 b
2b2
[length of latus rectum = and On comparing, we get
a
a = 3 and b = 2
length of minor axis = 2b]
Let e be the eccentricity of ellipse.
æ bö b
Þ b ç ÷ = 4 and = e Þ b(e) = 4
èaø a b2
Then, e= 1-
Þ b = 4×
1
…(i) a2
e 4 5
Also, we know that b2 = a 2(1 - e2) Þ e= 1- =
9 3
b2
Þ = 1 - e2 (x - h )2 ( y - k)2
a2 6. + = 1 has,
M N
é b ù
Þ e2 = 1 - e2 êëQ a = eúû Major axis on the line y = 2 Þ k = 2
Ellipse 395

Minor axis on the line æ1ö æ1ö


b2 = a 2ç ÷ = 16ç ÷ [using Eq. (ii)]
x= -1 Þ h = -1 è2ø è2ø
Major axis has length Þ b2 = 8
2b = 10 Þ b = 5 2b2 2 ´8
\ N = b2 = 25 Now, length of latus rectum = = = 4 units
a 4
Minor axis has length 1
9. We have, e = and equation of directrix is x = 4.
2a = 4 Þ a =2 2
2
\ M = a =4 We know that, equation of directrix is x = a /e.
7. If two foci are S(5, 12) and S¢ (- 12, 5) and the ellipse \ a /e = 4
passes through the origin O, then Þ a = 4e
SO = 25 + 144 = 13 1
Þ a =4 ´ =2
S ¢ O = 144 + 25 = 13 2
and SS¢ = 338 b2
Also, e= 1-
If the conic is an ellipse, then a2
SO + S ¢O = 2a and SS ¢ = 2ae 1 b2
Þ =1 - Þ b2 = 3
SS ¢ 338 1 4 4
\ e= = =
SO + S ¢O 26 2 \Equation of ellipse is
x2 y2 x2 y2
8. Let the ellipse be + 2 = 1. + = 1 or 3x2 + 4 y2 = 12
2 4 3
a b
x2 y2
Then, according to given information, we have the 10. Let the equation of ellipse be + = 1 and e be the
following figure. a 2 b2
eccentricity of ellipse.
Y
It is given that distance between foci = 2h
(0,b)
B \ 2ae = 2h Þ ae = h …(i)
Focal distance of the one end of minor axis, say (0, b)
X is k.
S¢(–ae,o)O S(ae,0)
\ a + e (0) = k Þ a = k …(ii)
From Eqs. (i) and (ii), we have
b2 = a 2(1 - e2) = a 2 - (ae)2 = k2 - h 2
b x2 y2
Clearly, slope of line SB = Hence, the equation of ellipse is 2 + 2 = 1.
- ae k k - h2
b
and slope of line S ¢B = 11. The given equation of an ellipse can be rewritten as
ae
(x + 1 )2 ( y + 2)2
Q Lines SB and S ¢ B are perpendicular, so + =1
9 25
æ b ö æbö
ç ÷ . ç ÷ = -1 whose centre is (-1, - 2) and semi-major and
è - ae ø è ae ø
semi-minor axes are 5 and 3 respectively.
[Q product of slopes of two perpendicular lines is (-1)]
i.e. b = 5 and a = 3
Þ b2 = a 2e2 …(i)
\The eccentricity of the ellipse is given by
Also, it is given that area of DS ¢ BS = 8
a 2 = b2(1 - e2)
1 2
\ a =8 4
2 Þ 9 = 25 (1 - e2) Þ e =
5
[QS ¢ B = SB = a because S ¢ B + SB = 2a and S ¢ B = SB]
On shifting the origin at (-1, - 2) the given equation
Þ a 2 = 16 Þ a = 4 …(ii)
reduces to
b2
Q e2 = 1 - 2 = 1 - e2 [from Eq. (i)] X2 Y 2
a + =1 …(i)
9 25
2 2 1
Þ 2e = 1 Þ e = …(iii) where, x = X - 1 and y = Y - 2 …(ii)
2
\ Coordinates of the foci are (X = 0, Y = ± be)
From Eqs. (i) and (iii), we get
i.e. (-1, 2) and (-1, - 6)
396 JEE Main Mathematics

12. The major axis passes through S (- 1, - 1) and is x2 y2


16. Let the standard equation of ellipse is + =1
perpendicular to the directrix x - y + 3 = 0. a 2 b2
So, the equation of major axis is x + y + 2 = 0. (a > b).
Minor axis = 2b = 8 Þ b = 4
5
P and eccentricity = e =
M 3
C Now, b2 = a 2 (1 - e2)
Z A S (–1, –1) S¢ A¢ Z¢
æ 5ö æ4ö
Þ (4)2 = a 2 ç1 - ÷ Þ 16 = a 2 ç ÷
è 9ø è9ø
Þ a 2 = 36 Þ a = 6
æ 5 11 ö Length of major axis = 2a = 12
The axis meets the directrix at Z. Then, Z is ç - , ÷ .
è 2 2ø
17. We have, 16x2 + 25 y2 = 400
1
A and A ¢ divide ZS in the ratio 1 : e, i.e. 1 : or 2 : 1 x2 y2
2 Þ + =1
internally and externally, respectively. 25 16
æ 3 1ö æ1 5ö Þ a 2 = 25 and b2 = 16
Therefore, we have A ç - , - ÷ and A ¢ ç , - ÷ .
è 2 2ø è2 2ø This equation represents an ellipse with eccentricity e
æ 1 3ö given by
Thus, centre C (mid-point of AA¢) = ç - , - ÷ .
è 2 2ø b2 16 3
e2 = 1 - 2 = 1 - Þ e=
a 25 5
13. The given equation can be rewritten as
So, the coordinates of foci are (± ae, 0), i.e. (3, 0) and
4x2 - 24x + 36 + 16 y2 - 32 y + 16 - 36 - 16 - 12 = 0
(- 3, 0). Thus, F1 and F2 are the foci of the ellipse.
Þ (2x - 6)2 + (4 y - 4)2 = 64
Since, the sum of the focal distance of a point on an
(x - 3)2 ( y - 1)2 ellipse is equal to its major axis.
Þ + =1
16 4 \ PF1 + PF2 = 2a = 10
This represents an ellipse. 18. Let point is ( 6 cos q, 2 sin q) and let it’s distance d from
Here, a 2 = 16, b2 = 4 origin.
4 3 \ d = 6 cos 2 q + 2 sin 2 q
\ e= 1- =
16 2
Þ 2 = 2 + 4 cos 2 q Þ 2 + 4 cos 2 q = 4
14. The position of the points (1, 2) and (2, 1) with respect to 1 1 p
the circle x2 + y2 = 9 is given by 12 + 22 = 5 < 9 and Þ cos 2 q = Þ cos q = ± Þ q=
2 2 4
22 + 12 = 5 < 9.
ì y2 x2 ü
Thus, both P and Q lie inside C. 19. Given, S = í (x, y) Î R2: - = 1ý
î 1 + r 1 - r þ
The position of the points (1, 2) and (2, 1) with respect to
x2 y2 ì y 2
x 2 ü
the ellipse + = 1 is given by = í (x, y) Î R2 : + = 1ý
9 4 î 1 + r r -1 þ
12 22 1 22 1 16 + 9 25 y2 x2
+ = + 1 > 1 and + = = < 1, For r > 1, + = 1, represents a vertical ellipse.
9 4 9 9 4 36 36 1 + r r -1
P lies outside E and Q lies inside E. [Q for r > 1, r - 1 < r + 1 and r - 1 > 0]
Thus, P lies inside C but outside E. r -1
Now, eccentricity (e) = 1 -
15. Given equation is 9x2 + 4 y2 - 6x + 4 y + 1 = 0 r+1
æ 2 1ö æ 1ö é x2 y2 a2 ù
Þ 9 ç x2 - x + 2 ÷ + 4 ç y2 + y + ÷ + 1 - 1 - 1 = 0 êQ for 2 + 2 = 1, a < b, e = 1 - 2 ú
è 3 3 ø è 4ø a b b úû
ë
2 2
æ 1ö æ 1ö
çx - ÷ çy+ ÷ (r + 1) - (r - 1) 2
è 3ø è 2ø = =
Þ 2
+ 2
=1 (here, a < b) r+1 r+1
æ ö
1 æ ö
1
ç ÷ ç ÷ x2 y2 x2 y2
è3ø è2ø 20. Equations of the ellipse are + = 1 and + =1
132 52 a 2 b2
æ1ö
Length of major axis = 2 b = 2 ç ÷ = 1 and their eccentricity are
è2ø
25 b2
æ1ö 2 e= 1- and e¢ = 1 - 2
Length of minor axis = 2 a = 2 ç ÷ = 169 a
è3ø 3
Ellipse 397

According to given condition, e¢ = e la cos q + mb sin q + n = 0


æb 2ö
æ 25 ö and - la sin q + mb cos q + n = 0
Þ 1 - ç 2÷ = 1 - ç ÷
èa ø è 169 ø Þ la cos q + mb sin q = - n

b 5 and - la sin q + mb cos q = - n


Þ = (Q a > 0, b > 0) Þ (la cos q + mb sin q)2 + (- la sin q + mb cos q)2
a 13
a 13 = n2 + n2
Þ =
b 5 Þ l2a 2 + m2b2 = 2n 2
x2 y2 26. Given equations of ellipse are
21. Given equation of ellipse is + = 1.
4 7
x2 y2 x2 y2
4 2
+ 2 = 1 and 2 + 2 = 1
a b b a
7
Here, a 2 = 4 and b2 = Y
4 2
7 x2 + y = 1
\ b2 = a 2(1 - e2) Þ = 4(1 - e2) b 2 a2
4
P
7 9 3 r
Þ e2 = 1 - = Þ e=
16 16 4 X¢ X
O
2
æ 3 ö 2
x +y =1
Thus, the foci are ç ± , 0÷ .
è 2 ø a 2
b 2

2
æ3 1ö
The radius of required circle = ç - ÷ + (2 - 0)2 Y¢
è2 2ø
The point of the intersection of these ellipse are
= 1+4= 5
æ ab ab ö
22. Let P is the position of man and S , S ¢ are position of ç± ,± ÷
ç a 2 + b2 a 2 + b2 ÷ø
flags, then è
SP + S ¢ P = 10 = 2a Þ a =5 æ ö
ab ab
4 i.e. Pç , ÷.
\ SS ¢ = 2ae = 8 Þ e = ç a 2 + b2 a 2 + b2 ÷ø
è
5
b2 16 b2 \The distance between OP = r
Now, e2 = 1 - Þ =1 - 2 2
a2 25 25 æ ö æ ö
ab ab
b2 = 9 Þ b = 3 = ç - 0÷ + ç - 0÷
Þ ç a 2 + b2 ÷ ç a 2 + b2 ÷
è ø è ø
Area of ellipse = pab = 15p sq m
ab
23. Solving 4x2 + 9 y2 = 36 and y = mx + c, we get = 2
a + b2
2

4x2 + 9(mx + c)2 - 36 = 0


\Equation of circle is
Þ (9m + 4)x2 + 18cmx + 9c2 - 36 = 0
2

2a 2b2
Roots are real, so x2 + y 2 = r 2 Þ x2 + y 2 =
18 ´ 18c2m2 - 4 (9m2 + 4) (9c2 - 36) ³ 0 a 2 + b2
Þ 9m2 - c2 + 4 ³ 0 Þ 9m2 ³ c2 - 4 27. Given equation of ellipse is
24. y2 = 3x2 and x2 + y2 = 4b x2 y2
+ =1
Solving both the equation, we get 5 5
So, x2 = b 3 4
2 The equation of tangents in slope form is
x 3 x2 b 3
+ 2 =1Þ + =1 5 2 5
16 b 16 b y = mx ± m +
3 4
Þ b2 - 16b + 48 = 0 Þ (b - 12) (b - 4) = 0
1 1
b = 12, b > 4 Slope of tangents are or -
3 3
25. Suppose the line lx + my + n = 0 cuts the ellipse at
1 5 5
ì æp ö æp öü \ y=± x± +
P (a cos q, b sin q) and Q í a cos ç + q÷ , b sin ç + q÷ý. 3 9 4
î è 2 ø è 2 øþ
1 65
Then, these two points lie on the line Þ y=± x±
3 6
398 JEE Main Mathematics

28. The combined equation of the pair of tangents drawn x2 y2


2 2 2 31. Equation of ellipse is + = 1. General equation of
from (1, 2) to the ellipse 3x + 2 y = 5 is SS ¢ = T 2 1
Þ (3x2 + 2 y2 - 5)[3 (1)2 + 2 (2)2 - 5 ] = [3x (1 ) + 2 y(2) - 5 ]2 tangent to the ellipse of slope m is

Þ (3x2 + 2 y2 - 5) (3 + 8 - 5) = (3x + 4 y - 5)2 y = mx ± 2 m2 + 1

Þ 9x2 - 24xy - 4 y2 + 40 y + 30x - 55 = 0 Since, this is equally inclined to axes, so m = ± 1.


This is the equation of pair of straight lines, Thus, tangents are
where, a = 9, h = - 12 , b = - 4 y=±x± 2+1 =±x± 3
|0 + 0 ± 3 | 3
The angle between these lines is given by Distance of any tangent from origin = =
2
1 +1 2 2
2 h 2 - ab
tan q =
a+b 32. Equation of given ellipse is
2 144 + 36 2 180 12 x2 y2
Þ tan q = = = + =1 …(i)
9 -4 5 5 a 2 b2
-1 æ 12 ö æ 9ö
Þ q = tan ç ÷ Now, equation of tangent at the point ç3, - ÷ on the
è 5ø è 2ø
ellipse (i) is
29. We have, x cos a + y sin a = p
3x 9 y
- =1 …(ii)
Þ y = - x cot a + p cosec a a 2 2b2
Since, above line is tangent to the ellipse. x2 y2
[Q the equation of the tangent to the ellipse 2 + 2 = 1
\ c2 = a 2m2 + b2 a b
xx yy
Þ p2 cosec2 a = a 2 cot2 a + b2 at the point (x1 , y1 ) is 21 + 21 = 1 ]
a b
Þ a 2 cos 2 a + b2 sin 2 a = p2
Q Tangent (ii) represent the line x - 2 y = 12, so
30. Equation of given ellipse is 1 2 12
= =
4x2 + y2 = 8 …(i) 3 9 1
x2 y 2 x2 y2 a 2 2b2
Þ + =1Þ 2
+ =1 Þ a 2 = 36 and b2 = 27
2 8 ( 2) (2 2 )2
2b2 2 ´ 27
Now, equation of tangent at point (1, 2) is Now, Length of latusrectum = = = 9 units
a 6
2x + y = 4 ...(ii)
x2 y2 33. Let the equation of tangent, which is perpendicular to
[Q equation of tangent to the ellipse 2 + 2 = 1 at the line 3x + 4 y = 7 is 4x - 3 y = l.
a b
xx yy Since, it is a tangent to the ellipse.
(x1 , y1 ) is 21 + 21 = 1 ]
a b \ l2 = a 2m2 + b2
and equation of another tangent at point (a , b) is 4
Here, a 2 = 9, b2 = 4 and m =
4ax + by = 8 …(iii) 3
2
Since, lines (ii) and (iii) are perpendicular to each æ4ö
\ l2 = 9 ´ ç ÷ + 4 = 16 + 4
other. è3ø
æ 2 ö æ 4a ö
\ ç- ÷ ´ ç- ÷ = -1 Þ l2 = 20 Þ l = ± 20
è 1ø è b ø
[if lines a1x + b1 y + c1 = 0 and a 2x + b2y + c2 = 0 \Equation of tangents are 4x - 3 y = ± 20.
æ a öæ a ö 1 1 4
are perpendicular, then ç - 1 ÷ ç - 2 ÷ = - 1 ] 34. We have, a 2 = , b2 = , m =
è b1 ø è b2 ø 2 3 3
æ 2
am b2 ö
Þ b = - 8a …(iv) The required points are ç ± ,m ÷
ç a m + b2
2 2
a 2m2 + b2 ÷ø
Also, the point (a , b) lies on the ellipse (i), so è
4a 2 + b2 = 8 æ 1 4 1 ö
Þ 4a + 64a 2 = 8
2
[from Eq.(iv)] ç ´ ÷
= ç± 2 3 ,m 3 ÷
8 ç 1 16 1 1 16 1 ÷
Þ 68a 2 = 8 Þ a 2 = ç ´ + ´ + ÷
68 è 2 9 3 2 9 3ø
2
Þ a2 = æ 2 1 ö
17 = ç± ,m ÷
è 11 11 ø
Ellipse 399

x2 y2 1 1 p
35. Equation of any tangent to + = 1 is Þ tan3 q = Þ tan q = Þ q=
a 2 b2 3 3 3 6
p
y = mx + a 2m2 + b2 …(i) At q = , f ¢ ¢ (q ) > 0
3
x2 y2 p
The equation of a tangent to 2 + 2 =1 \ f (q) is minimum at q = .
a +l b +l 6
perpendicular to Eq. (i) is x 3 y1
38. Given equation of tangent is + = 1 and
1 1 a 2 b2
y = - x + (a 2 + l ) 2 + (b2 + l )
m m equation of tangent at the point (a cos f , b sin f ) on the
x y
Þ my = - x + (a 2 + l ) + (b2 + l ) m2 …(ii) ellipse is cos f + sin f = 1.
a b
Let P (h , k) be the point of intersection of Eqs. (i) Both are same.
and (ii). Then, 3 1 p
\ cos f = , sin f = Þ f=
(k - mh )2 = a 2m2 + b2 2 2 6
and (mk + h )2 = (a 2 + l ) + (b2 + l ) m2
39. The equation of the tangent at P (a cos q, b sin q) to the
Þ (k - ml )2 + (mk + h )2 = (a 2 + b2 + l ) (m2 + 1)
x2 y2 x y
Þ h 2 + k2 = a 2 + b2 + l. ellipse 2
+ 2 = 1 is cos q + sin q = 1.
a b a b
Hence, the locus of (h , k) is x2 + y2 = a 2 + b2 + l.
Length of perpendicular from the focus (ae, 0) on the
x2 y2
36. Let the point be P( 2 cos q, sin q) on + = 1.
2 1 e cos q - 1
Y ellipse =
cos 2 q sin 2 q
+
(0, cosec q)B P( Ö2 cos q, sin q) a2 b2
X¢ X ab(e cos q - 1 ) ab(e cos q - 1 )
O A = =
( Ö2 sec q, 0) 2 2 2
b cos q + a (1 - cos q) 2
a 2 - a 2e2 cos 2 q

1 - e cos q b2 1 + e cos q
Y¢ \ p=b Þ 2=
x 2 1 + e cos q p 1 - e cos q
\ Equation of tangent at P is cos q + y sin q = 1
2 Now, r 2 = (ae - a cos q)2 + b2 sin 2 q
whose intercept on coordinate axes are A( 2 sec q, 0)
= a 2[(e - cos q)2 + (1 - e2) sin 2 q]
and B(0, cosec q).
\ Mid-point of its intercept between axes is = a 2 [e2 cos 2 q - 2e cos q + 1] = a 2 (1 - e cos q)2
æ 2 1 ö Þ r = a (1 - e cos q)
ç sec q, cosec q÷ = (h , k )
è 2 2 ø 2a b2 2 1 + e cos q
\ - 2= - =1
1 1 r p 1 - e cos q 1 - e cos q
Þ cos q = and sin q =
2h 2k 1 a
1 1 40. We have, e = and = 4
Now, cos 2 q + sin 2 q = 1 Þ + =1 2 e
2
2h 4k2 \ a =2
1 1
The locus of mid-point M is 2 + = 1. é æ1ö ù
2
2x 4 y2 æ 1ö
Now, b2 = a 2(1 - e2) = (2)2ê1 - ç ÷ ú = 4ç1 - ÷ = 3
êë è ø
2 úû è 4ø
37. Equation of tangent at (3 3 cos q, sin q) to the ellipse
x2 x cos q Þ b= 3
+ y2 = 1 is + y sin q = 1.
27 3 3 x2 y2
\Equation of the ellipse is 2
+ =1
It cuts intercepts on the coordinate axes. (2) ( 3 )2
\Sum of intercepts on axes is x2 y2
Þ + =1
3 3 sec q + cosec q = f (q) (say) 4 3
On differentiating w.r.t. q, we get æ 3ö
Now, the equation of normal at ç1, ÷ is
è 2ø
3 3 sin3 q - cos3 q
f ¢ (q) =
sin 2 q cos 2 q a 2x b2y
- = a 2 - b2
x1 y1
For maxima and minima, put f ¢ (q ) = 0
4x 3y
Þ 3 3 sin3 q - cos3 q = 0 Þ - = 4 - 3 Þ 4x - 2 y = 1
1 (3 /2)
400 JEE Main Mathematics

æ 16 ö 44. Tangent at P (a cos q, b sin q) is


41. The equation of the tangent at ç4 cos f , sin f ÷ to
è 11 ø
x y
the ellipse 16x2 + 11 y2 = 256 is cos q + sin q = 1
a b
æ 16 ö Now, PF1 = a + ex, PF2 = a - ex
16(4 cos f ) x + 11 ç sin f ÷ y = 256
è 11 ø
Þ PF1 - PF2 = 2ex = 2e × a cos q
Þ 4x cos f + 11 y sin f = 16
\ (PF1 - PF2)2 = 4a 2e2 cos 2 q …(i)
2 2 2
This, touches the circle (x - 1 ) + y = 4 . x y
We know tangent at P(q) is cos q + sin q = 1
4 cos f - 16 a b
\ =4 1
16 cos 2 f + 11 sin 2 f \ d=
cos q sin 2 q
2
Þ (cos f - 4)2 = 16 cos 2 f + 11 sin 2 f +
a2 2b2
Þ 4 cos 2 f + 8 cos f - 5 = 0
1 cos 2 q sin 2 q
\ = +
Þ (2 cos f - 1 ) (2 cos f + 5) = 0 d 2
a2 b2
1 p é 5ù b2 b2
Þ cos f = Þf=± êëQ cos f ¹ 2 úû or = cos 2 q + sin 2 q
2 3 d2 a 2
42. Any point on the ellipse is P(3 cos q, 2 sin q). b2 b2 æ b2 ö
\ 1- 2
= 1 - 2 cos 2 q - sin 2 q = cos 2 q ç1 - 2 ÷
x y d a è a ø
Equation of the tangent at P is cos q + sin q = 1,
3 2 = cos 2 q × e2
which meets the tangents x = 3 and x = - 3 at the æ b2 ö
Þ 4a 2ç1 - 2 ÷ = 4a 2 cos 2 q × e2 …(ii)
ì 2 (1 - cos q)ü è d ø
extremities of the major axis at T í3, ý and
î sin q þ On comparing Eqs. (i) and (ii), we get
ì 2 (1 + cos q)ü (PF1 - PF2)2 = 4a 2(1 - b2 /d 2)
T ¢ í -3 , ý.
î sin q þ Hence, l =4
Equation of circle on TT ¢ as diameter is b2
45. Q e2 = 1 -
ì 2 (1 - cos q)ü ì 2 (1 + cos q)ü a2
(x - 3)(x + 3) + í y - ýíy - ý =0
î sin q þ î sin q þ \ a 2 - b2 = a 2e2
4 So, the points on the minor axis at a distance a 2 - b2
Þ x2 + y2 - y-5 =0
sin q
from the centre (0, 0) of the ellipse are (0, ± ae).
which passes through ( 5 , 0). The equation of tangent at any point (a cos q, b sin q) on
43. Given equation of ellipse is the ellipse is
x2 y2 x y
+ 2 = 1 (a < b) cos q + sin q = 1
2 a b
a b
It is a vertical ellipse with foci (0, ± be). \ Required sum
2 2
Equation of any tangent line to the above ellipse is é ae sin q ù é - ae sin q ù
ê -1 ú ê -1 ú
y = mx + a 2m2 + b2
=ê b ú +ê b ú
ê cos 2 q sin 2 q ú ê cos 2 q sin 2 q ú
\Required product ê + ú ê + ú
½- be + a 2m2 + b2½½be + a 2m2 + b2½ ë a2 b2 û ë a2 b2 û
=½ ½½ ½
m2 + 1 m2 + 1 (ae sin q - b)2 + (ae sin q + b)2
½ ½½ ½ = ´ a2
(b2 cos 2 q + a 2 sin 2 q)
½a 2m2 + b2 - b2e2½
=½ ½ 2a 2 [(a 2 - b2) sin 2 q + b2]
½ m2 + 1 ½ = = 2a 2
b2 cos 2 q + a 2 sin 2 q
½a 2m2 + b2 (1 - e2)½
=½ ½ x2 y 2
½ m2 + 1 ½ 46. The equation of tangents to + =1
a 2 b2
½a 2m2 + a 2½ 2
=½ 2
½= a [Q a 2 = b2(1 - e2)] are given by y = mx ± a 2m2 + b2, where m is the slope
½ m +1 ½
of the tangent.
Ellipse 401

For the ellipse 4x2 + 3 y2 = 5, we have 50. The equation of normal at point P (a cos q, b sin q) is
5 5 ax sec q - by cosec q = a 2 - b2
a= ,b= ,m = tan 60° = 3 [given]
2 3 The point of intersection with coordinate axes are
So, the equations of tangents are æ a 2 - b2 ö æ a 2 - b2 ö
5 5 Rç cos q, 0÷ and S ç0, - sin q÷.
y = 3x ± ´3 + è a ø è a ø
4 3 2
65 2
é æ a 2 - b2 ö ù
Þ y = 3x ± Now, RP = ê a cos q - ç ÷ cos qú + b2 sin 2 q
12 êë è a ø úû
2
47. Let the equation of tangent to the circle x2 + y2 = 16 is b
= 2 (b2 cos 2 q + a 2 sin 2 q)
a
y = mx + 4 1 + m2 [Q y = mx + a 1 + m2 ]
a2
and RS 2 = 2 (b2 cos 2 q + a 2 sin 2 q)
x2 y2 b
and let the equation of tangent to the ellipse + =1
49 4 \ RP 2 : RS 2 = b4 : a 4 Þ RP : PS = b2 : a 2
is
x2 y2
2
y = mx + 49m + 4 [Q y = mx + 2 2
am +b ] 2 51. The equation of any normal to + = 1 is
a 2 b2
For common tangent, ax sec f - by cosec f = a 2 - b2 …(i)
2
4 1 + m = 49m + 4 2 The straight line x cos a + y sin a = p will be a normal
x2 y2
Þ 2
16 + 16m = 49m + 4 2
to the ellipse 2 + 2 = 1, then Eq. (i) and
a b
Þ 12 = 33m2 x cos a + y sin a = p will represent the same line.
12 2
Þ m2 = Þm = a sec f - b cosec f a 2 - b2
33 11 \ = =
cos a sin a p
2 4
\ y= x+4 1+ ap
11 11 Þ cos f =
(a - b2) cos a
2
2 15
= x+4 - bp
11 11 and sin f = 2
(a - b2) sin a
48. Tangent to parabola 2 y = 2 (x + 6) - 20
Q sin 2 f + cos 2 f = 1
Þ y= x-4
b 2 p2 a 2 p2
Condition of tangency for ellipse, Þ + 2 =1
2
(a - b ) sin a (a - b2)2 cos 2 a
2 2 2 2
16 = 2 (1) + b Þ b = 14
x2 y2 Þ p2(b2 cosec2a + a 2 sec2 a ) = (a 2 - b2)2
49. The given ellipse is + =1
16 (256 / 11) x2 y2
52. The given equation can be rewritten as + =1
æ 16 ö 20 45
Tangent to it at point P ç4 cos q, sin q÷ is
è 11 ø 4
x y On comparing the given equation with the standard
cos q + sin q = 1 equation, we get
4 (16 / 11 )
45
It also touches the circles (x - 1)2 + y2 = 16 a 2 = 20, b2 =
4
1 \ The equation of normal at the point (2, 3) is
cos q - 1
\
4
=4 x-2 y-3
=
cos 2 q 11 sin 2 q 2 æ 12 ö
+ ç ÷
16 256 20 è 45 ø
|cos q - 4| Þ 40(x - 2) = 15 ( y - 3)
Þ =1
16 cos 2 q + 11 sin 2 q Þ 8x - 3 y = 7 Þ 3 y - 8x + 7 = 0
2
Þ cos q - 8 cos q + 16 = 11 + 5 cos q 2
(x - 1 )2 ( y - 2)2
2
53. Equation of given ellipse is + = 1.
Þ 4 cos q + 8 cos q - 5 = 0 Þ (2 cos q - 1) (2 cos q + 5) = 0 9 4
1 p X2 Y 2
Þ cos q = Þ q = ± Equation of normal to ellipse + = 1 is
2 3 9 4
æ 8 3 ö 3X sec q - 2Y cosec q = 5.
\ Points are ç2, ± ÷
è 11 ø
402 JEE Main Mathematics

3 56. The equation of normal to the given ellipse at


Slope of normal is tan q,
2 P (a cos q, b sin q) is
3 a x sec q - by cosec q - a 2 = b2
which is parallel to 3x - y = 1, then tan q = 3
2 æa ö a 2 - b2
Þ y = ç tan q÷ x - sin q …(i)
Þ tan q = 2 èb ø b
2 1 a bm
\ sin q = , cos q = Let tan q = m, then sin q =
5 5 b a + b2m2
2

So, equation of normal is 3 5X - 5Y = 5. \ From Eq. (i), we get


Q X = x - 1, Y = y - 2 (a 2 - b2)m
y = mx -
\ 3 5 (x - 1 ) - 5 ( y - 2) = 5 a 2 + b2m2
Þ 5 (3x - y) = 5 ( 5 + 1 ) Q
a
tan q Î R Þ m Î R
b
Þ 3x - y = 5 ( 5 + 1 )
x2 y2
x2 y2 57. The normal to the ellipse + = 1 at the points
54. Equation of ellipse is + = 1. a 2 b2
14 5 p
whose eccentric angles are qand + q, are respectively.
Any point on the ellipse is ( 14 cos q, 5 sin q). 2
\Equation of normal at ( 14 cos q, 5 sin q) is ax sec q - by cos ec q = a 2 - b2
14x sec q - 5 y cosec q = 9. and - ax cosec q - by sec q = a 2 - b2
It passes through (a cos 2 q, b sin 2 q ). Since, w is the angle between these two normals,
\ 14 14 cos 2 q sec q - 5 5 sin 2 q cosec q = 9 a a
tan q + cot q
cos 2 q sin 2 q \ tan w = b b
Þ 14 -5 =9 a2
cos q sin q 1- 2
b
Þ 14(2 cos 2 q - 1 ) - 10 cos 2 q = 9 cos q
ab (tan q + cot q) 2ab
Þ 18 cos 2 q - 9 cos q - 14 = 0 = 2 2
=
b -a (sin 2q) (b2 - a 2)
Þ 18 cos 2 q - 21 cos q + 12 cos q - 14 = 0
2ab 2a 2 1 - e2
Þ (3 cos q + 2) (6 cos q - 7 ) = 0 = 2 2
= 22
(a - b ) sin 2 q a e sin 2 q
2 7
Þ cos q = - , cos q ¹ - 2 cot w e2
3 6 Þ =
sin 2 q 1 - e2
æ b2 ö x2 y2
55. Normal at ç ae, ÷ of ellipse 2 + 2 = 1 is 58. The coordinates of the extremity of the latusrectum
è aø a b
æ b2 ö
b2 which lies in the first quadrant are ç ae, ÷. The
y- è aø
x - ae a
= equation of the normal at (x1 , y1 ) is
ae æ b2 2ö
ç /b ÷ a 2x b2 y
a2 èa ø - = (a 2 - b2)
x1 y1
Since, it passes through (0, - b), then
æ b2 ö
b2 \The equation of the normal at ç ae, ÷ is
-b - è aø
0 - ae a
=
ae 1 a 2x b2 y
a2 a - = a 2 - b2
ae b2 / a
æ b2 ö
Þ - a2 = - a çb + ÷ Þ ax - aey = e (a 2 - b2)
è aø
Þ ax - aey = ea 2e2
2 2
Þ a = ab + b Þ x - ey = ae3
Þ a 2 = ab + a 2 - a 2e2 [Q b2 = a 2 - a 2e2] This passes through the extremity of the minor axis
Þ b = ae2 i.e. (0, - b)
\ 0 + eb - ae3 = 0
Þ b2 = a 2e4
Þ b = ae2 Þ b2 = a 2e4
Þ a 2(1 - e2) = a 2e4
Þ a 2(1 - e2) = a 2e4
2 4 2 2
Þ 1 - e = e Þ e (e + 1 ) = 1 \ e4 + e2 - 1 = 0
Ellipse 403

59. Let mid-point of the chord be (h , k ), then equation of the 64. Given, m(n - 1 ) = n
chords be n is divisible by n - 1.
hx2 ky2 h 2 k2 Þ n = 2 Þm = 2
2
+ 2 -1 = 2 + 2 -1
a b a b Hence, chord of contact of tangents drawn from (2, 2)
b2 h æ h 2 k2 ö b2 x2 y 2
Þ y=- × x+ ç 2 + 2÷ …(i) to + = 1 is
a2 k èa b ø k 9 4
2x 2 y
Since, line (i) is touching the circle x2 + y2 = c2. + = 1 Þ 4x + 9 y = 18
9 4
æ h 2 k2 ö b4 æ b4h 2 ö
\ ç 2 + 2 ÷ 2 = c2 ç1 + 4 2 ÷
èa b ø k è a k ø Round II
2 2 2 2 2 2 4 2
Hence, locus is (b x + a y ) = c (b x + a y ). 4 2 1. Given equation of ellipse is
x2 y2
60. Any point on the line x - y - 5 = 0 will be of the form + =1 …(i)
9 5
(t , t - 5). Chord of contact of this point with respect to
\ a 2 = 9, b2 = 5 Þ a = 3, b = 5
curve x2 + 4 y2 = 4 is
b2 5 2
tx + 4(t - 5) y - 4 = 0 Now, e= 1- = 1- =
a2 9 3
Þ (-20 y - 4) + t (x + 4 y) = 0 foci = (± ae, 0) = (± 2, 0)
which is a family of straight lines, each member of this b2 5
and =
family pass through point of intersection of straight a 3
æ4 1ö Y
lines -20 y - 4 = 0 and x + 4 y = 0 which is ç , - ÷ .
è5 5ø P
(0,3)

61. Chords of contact are (–2, 5/3)


xx1 yy M L (2, 5/3)
+ 21 = 1
a2 b Q
xx2 yy2 X¢ X
and + 2 =1 (–2,0) O (2,0) (9/2, 0)
a2 b

Product of slopes = - 1 M¢
(2, –5/3)
(–2, –5/3)
æ x1 b2 ö æ x2 b2 ö
Þ ç- 2 × ÷ ç- 2 × ÷ = -1
è a y1 ø è a y2 ø

x1x2 a4
Þ =- 4 \Extremities of one of latusrectum are
y1 y2 b æ 5ö æ -5 ö
ç2, ÷ and ç2, ÷.
62. AB being the chord of contact of the ellipse from P (3, 4) è 3ø è 3 ø
has its equation æ 5ö
\Equation of tangent at ç2, ÷ is,
3x 4 y è 3ø
+ = 1 Þ x + 3y = 3
9 4 x(2) y(5 / 3)
+ =1
If (h , k) is any point on the locus, then 9 5
h + 3k - 3 or 2x + 3 y = 9 …(ii)
(h - 3)2 + (k - 4)2 = æ9 ö
1+9 Eq.(ii) intersects X and Y-axes at ç , 0÷ and (0, 3),
è2 ø
respectively.
Þ 10 (h 2 + k2 - 6h - 8k + 25) = (h + 3k - 3)2
\ Area of quadrilateral = 4 ´ Area of DPOQ
\ Locus of (h , k) is
æ1 9 ö
9x2 + y2 - 6xy - 54x - 62 y + 241 = 0 = 4 ´ ç ´ ´ 3÷ = 27 sq units
è2 2 ø
63. We know that, if (x1 , y1 ) is the mid-point of the chord,
then equation of chord is 2. Equation of ellipse is x2 + 3 y2 = 6.
x cos q y sin q
T = S1 Equation of the tangent is + = 1.
a b
xx1 yy1 x12 y2
Þ + = + 1 Let (h , k) be any point on the locus, then
25 9 25 9
h k
x y 1 1 cos q + sin q = 1 …(i)
Q Point is (1, 1), then + = + Þ 9x + 25 y = 34 a b
25 9 25 9
404 JEE Main Mathematics

-b -1
Slope of the tangent line is cot q. SM × S ¢ N = (e2 cos 2 q - 1)
a cos 2 q sin 2 q
+
Slope of perpendicular drawn from centre (0, 0) to (h , k) a2 b2
k Þ SM × S ¢ N = b2 = a constant.
is .
h
5. Y
Since, both the lines are perpendicular.
æ kö æ b ö Q
\ ç ÷ ´ ç - cot q÷ = -1
è hø è a ø P q
cos q sin q X¢ X
Þ = =a [say]
ha kb
Þ cos q = aha , sin q = akb
From Eq. (i), Y¢
h k
(aha ) + (akb) = 1 Tangent to the ellipse at P (a cos a , b sin a ) is
a b x y
cos a + sin a = 1 …(i)
Þ h 2a + k2a = 1 a b
1 Tangent to the circle at Q (a cos a , a sin a ) is
Þ a=
h + k2
2
cos ax + sin ay = a …(ii)
Also, sin 2 q + cos 2 q = 1 Now, angle between tangents is q,
Þ (akb)2 + (aha )2 = 1 ½ b ½
Þ a 2k2b2 + a 2h 2a 2 = 1 ½ - a cot a - (- cot a ) ½
then tan q = ½ ½
½1 + æç - cot a ö÷ (- cot a )½
kb2 2
h 2a 2 b
Þ + 2 =1 è a ø
2 2 2
(h + k ) (h + k2)2 ½ ½
2k2 6h 2 ½ æ b ö½
Þ + 2 = 1 [Q a 2 = 6 and b2 = 2] cot a ç1 - ÷
22 2 ½ è a ø½ ½ a-b ½
(h + k ) (h + k2)2 =½ ½= ½ ½
b
½ 1 + cot a ½ ½
2 a tan a + b cot a½
Þ 6x2 + 2 y2 = (x2 + y2)2 a
½ ½
[replacing k by y and h by x ]
½ a-b ½
3. (a , b) lies on the director circle of the ellipse i.e. on =½ 2 ½
½( a tan a - b cot a ) + 2 ab½
x2 + y2 = 9.
Now, the greatest value of the above expression is
So, we can assume
a = 3 cos q, b = 3 sin q ½a - b½
½ ½when a tan a = b cot a
½2 ab½
\ F = 12 cos q + 9 sin q = 3 (4 cos q + 3 sin q)
-15 £ F £ 15 æ a - bö
Þ Þ qmax = tan -1 ç ÷
è 2 ab ø
4. Let equation of an ellipse is
6. Equation of tangent at (a cos q, b sin q) is
x2 y2
2
+ 2 =1 …(i) Y
a b
Y Q (a cos q, b sin q)
N B
M X¢ X
O P

X¢ X
A¢ S'(–ae, 0) S(ae, 0) A

B¢ x y
cos q + sin q = 1
a b

æ a ö æ b ö
Its foci are S (ae, 0) and S ¢ (- ae, 0). The equation of \ Coordinates of P and Q are ç , 0÷ and ç0, ÷
tangent at any point (a cos q, b sin q) to ellipse (i) is è cos q ø è sin q ø
x y respectively.
cos q + sin q = 1 …(ii)
a b 1½ a b ½ ab
\ Area of DOPQ = ½ ´ ½=
Let the perpendicular from S and S¢ upon Eq. (ii) be 2 ½cos q sin q½ |sin 2 q|
SM and S ¢ N , then \ (Area)minimum = ab
Ellipse 405

7. The given equation of second ellipse can be rewritten as Since, sides of the square are tangent and perpendicular
x 2
y 2 to each other, so two vertices lie on director circle.
+ =1 Þ x2 + y2 = (a 2 - 7 ) + (13 - 5a )
4 1
Equation of tangent to this ellipse is = a 2 ( 2a is side of the square)
x 6
cos q + y sin q = 1 …(i) Þ (a 2 - 7 ) + (13 - 5a ) = a 2 Þ a =
2 5
Equation of the first ellipse can be rewritten as But for an ellipse to exist a 2 - 7 > 0 and 13 - 5a > 0
x2 y2 Þ a Î (-¥ , - 7 )
+ =1 …(ii) 6
6 3 Hence, a¹
5
Let ellipse (i) meets the first ellipse at P and Q and the
tangents at P and Q to the second ellipse intersected at Hence, no such a exists.
(h , k), then Eq. (i) is the chord of contact of (h , k) with 10. Since, mutually perpendicular tangents can be drawn
respect to the ellipse (ii) and thus, its equation is from vertices of rectangle.
hx ky
+ =1 …(iii) So, all the vertices of rectangle should lie on director
6 3
circle x2 + y2 = a 2 + b2.
Since, Eqs. (i) and (iii) represent the same line.
h /6 k /3
\ = =1
cos q/2 sin q C A
Öa2 + b2
l
Þ h = 3 cos q and k = 3 sin q O 2l
Hence, locus is x2 + y2 = 9. D B
8. Let the equation of the ellipse be
x2 y2 Let breadth = 2l and length = 4l, then
2
+ 2 = 1.
a b 8
area of rectangle = 2l ´ 4l = 8l2 = (a 2 + b2)
Since, major axis is fixed. 5
\ a is variable. [\In DOAB, a 2 + b2 = l2 + 4l2 Þ 5l2 = a 2 + b2]
Hence, e is variable. Now, the ends of the latusrectum x2 y2
æ 11. Equation of the ellipse is + 2 = 1.
b2 ö a 2
b
are ç ± ae, ± ÷ .
è aø
Equation of the parabola with focus S (ae, 0) and
Hence, the tangents at these points are directrix x + ae = 0 is y2 = 4aex.
xx1 yy Y
+ 21 = 1
a2 b
b2
± y×
± x × ae a =1 D
Þ + X¢ O X
a2 b 2
S¢(–ae, 0) S (ae, 0)
xe y
Þ ± ± =1
a a
Þ ± xe ± y = a Y¢
Þ ± ex ± y - a = 0 2b2
Now, length of latusrectum of the ellipse is and that
Hence, there are 4 tangents. a
Since, e is a variable. of the parabola is 4ae.
\ Each of the tangents passes through the point (0, 1) For the two latusrectum to be equal, we get
which is fixed. 2b2
= 4ae
9. Y a
A
2a 2(1 - e2)
Þ = 4ae
a
D B
X¢ O X Þ 1 - e2 = 2e Þ e2 + 2e - 1 = 0
2± 8
Therefore, e=- = -1 ± 2
2
C
Y¢ Hence, e= 2 -1
406 JEE Main Mathematics

12. Let (x1 , y1 ) be a point, then x y


+ =1 …(i)
-
1 b
+ x12
= 37 3 y12 …(i)
3 2
Equation of tangent at (x1 , y1 ) is
Now, let point P (x1 , y1 ), so equation of normal to ellipse
xx1 + 3 yy1 = 37 x2 y2
x1 + = 1 at point P is
Slope of tangent = - 1 1
3 y1 2
3 y1 6 x y 1 x y 1
Then, slope of normal = = (given) Þ - = -1 Þ + = …(ii)
x1 5 2x1 y1 2 -2x1 y1 2
5 y1 Q Eqs. (i) and (ii) represents same line
Þ x1 = …(ii)
2 2x1 y 2 1
\ - = 1 = Þ x1 = and y1 = 2 b
From Eqs. (i) and (ii), we get -
1 b 1 3 2
25 y12 3 2
+ 3 y12 = 37 Þ y12 = 4
4 Q Point P (x1 , y1 ) lies on ellipse, so
\ y1 = ± 2 æ 1 ö 2 2 2 2
2ç ÷ + 4b = 1 Þ b = Þ b = ±
è9 ´ 2ø 9 3
From Eq. (ii), x1 = ± 5
\ Required points are (5, 2) and (-5, - 2). Q Point P (x1 , y1 ) is in first quadrant so y1 = 2 b > 0
PF1 NF1 2
13. Q = \ b=
PF2 NF2 3
16. Since the point (a , b) is on the parabola y2 = x, so
Y
a = b2 …(i)
B P Now, equation of tangent at point (a , b ) to the parabola
y2 = x, is T = 0
X¢ X 1
A¢ F2 C F1 A T Þ yb = (x + a )
N 2
B¢ [Q equation of the tangent to the parabola
y2 = 4ax at a point (x1 , y1 ) is given byyy1 = 2a (x + x1 )]

Þ 2 yb = x + b 2 [from Eq. (i)]
\ PN bisects the Ð F1PF2. x b
Þ y= + …(ii)
Q Bisectors are perpendicular to each other. 2b 2
\ PT bisects the angle (180° - Ð F1PF2). Since, line (ii) is also a tangent of the ellipse
14. Given equation of ellipse is 16x2 + 11 y2 = 256. x2 + 2 y2 = 1
2 2
æbö 2æ 1 ö 1
æ 16 ö \ ç ÷ = (1) ç ÷ +
Equation of tangent at ç4 cos q, sin q÷ is è2ø è2bø 2
è 11 ø
æ 16 ö [Q condition of tangency of line y = mx + c to ellipse
16x (4 cos q) + 11 y ç sin q÷ = 256 x2 y2
è 11 ø 2
+ 2 = 1 is c2 = a 2m2 + b2,
a b
It touches (x - 1 )2 + y2 = 42, if 1 1 bù
here, m = , a = 1, b = and c = ú
½ 4 cos q - 16 ½ 2b 2 2û
½ ½= 4
2 2
½ 16 cos q + 11 sin q½ b2 1 1
Þ = +
4 4 b2 2
Þ (cos q - 4)2 = 16 cos 2 q + 11 sin 2 q
1 Þ b4 = 1 + 2 b2
Þ 4 cos 2 q + 8 cos q - 5 = 0 Þ cos q =
2 Þ b4 - 2 b2 - 1 = 0
p 5p 2± 4+4 2±2 2
\ q= , Þ b2 = = =1 ± 2
3 3 2 2
15. Since equation of normal to the ellipse at P meets the Þ b2 = 1 + 2 [Qb 2 > 0]
æ 1 ö
co-ordinate axes at ç - , 0÷ and (0, b ) is a = b2 = 1 + 2
è 3 2 ø Q
Ellipse 407

x2 y2 Y
21. Since,
a
- ae = 4 and e =
1
17. Given equation of ellipse is + =1
16 9 e 2
Y
9 7 r
Here,a = 4, b = 3, e = 1 - = X¢ X
16 4
æ 7 ö X¢ X
\Foci is (± ae, 0) = ç ± 4 ´ , 0÷ = (± 7, 0)
è 4 ø Y¢
\Radius of the circle, Y¢ x = 4
a
2 2
r = (ae) + b = 7 + 9 = 16 = 4 \ 2a - = 4
2
Now, equation of circle is 3a 8
Þ =4 Þ a =
(x - 0)2 + ( y - 3)2 = 16 2 3
\ x2 + y 2 - 6 y - 7 = 0 x2 y2
22. Let the equation of ellipse be + = 1 (a > b)
a 2 b2
18. Diameter of circle (x - 1 )2 + y2 = 1 is 2 units and that of
p
circle x2 + ( y - 2)2 = 4 is 4 units. Q ÐFBF ¢ =
2
Semi-minor axis of ellipse, b = 2 units and semi-major p
axis of ellipse, a = 4 units. \ ÐFBC =
4
Hence, the equation of the ellipse is Y
x2 y2 x2 y 2 B
2
+ 2 =1 Þ = =1
a b 16 4 X¢ X
F' CF
Þ x2 + 4 y2 = 16
x2 y2
19. Given, 2
+ 2 =1 Y Y¢
a b p
P(–3, 1) \ÐCFB is also an angle of .
2 4
Passes through P(-3, 1 ) and e =
5 X¢ X
O Þ BC = CF Þ b = ae
9 1 b2 Þ b2 = a 2e2 …(i)
\ 2
+ 2 = 1 and e2 = 1 - 2
a b a Y¢ We know, b2 = a 2(1 - e2)
9 5 2 b2 \ a e = a 2(1 - e2)
2 2
[from Eq. (i)]
Þ + = 1 and = 1 -
a 2 3a 2 5 a2 Þ e2 = 1 - e2 Þ 2e2 = 1
27 + 5 b 2
3 1
Þ = 1 and 2 = Þ e=
3a 2 a 5 2
2 32 2 32 23. Equation of the tangent at point P (a cos q, b sin q) on the
Þ a = and b =
3 5 x2 y2
\Equation of ellipse is ellipse 2
+ 2 = 1 is
3 x2 5 y2 a b
+ = 1 Þ 3x2 + 5 y2 = 32 x y
32 32 cos q + sin q = 1 …(i)
a b
x2 y2
20. Let the equation of the required ellipse be + 2 = 1. The perpendicular distance of Eq. (i) from the centre
16 b (0, 0) of the ellipse is given by
But the ellipse passes through the point (2, 1). 1 ab
Y x2 y 2
d= =
+ =1
1 1
cos 2 q + 2 cos 2 q b cos q + a 2 sin 2 q
2 2
4 1 a 2
b
A(2, 1)
æ b2 ö ì b2 cos 2 q + a 2 sin 2 qü
(0, 1) Þ 4 a 2 ç 1 - 2 ÷ = 4 a 2 í1 - ý
X¢ X è d ø î d2 þ
(2, 0) (4, 0)
= 4 (a 2 - b2) cos 2 q = 4a 2e2 cos 2 q …(ii)
\ The foci are F1 (ae, 0) and F2 (- ae, 0)
Y¢ Þ PF1 = e(1 - e cos q) and PF2 = a (1 + e cos q)
1 1 1 3 4 \ (PF1 - PF2)2 = 4a 2e2 cos 2 q …(iii)
Þ + = 1 Þ 2 = Þ b2 =
4 b2 b 4 3 From Eqs. (ii) and (iii), we get
x 2
3y 2 æ b2 ö
Hence, equation is + = 1 Þ x2 + 12 y2 = 16 (PF1 - PF2)2 = 4a 2 ç1 - 2 ÷
16 4 è d ø
408 JEE Main Mathematics

a 1
24. Since, = 4 Þ 2a = 4 Þ a = 2 \ e=
e 2
æ 1ö Length of latusrectum
Now, b2 = a 2(1 - e2) = 4 ç1 - ÷ = 3
è 4ø æ1 1ö
= 2(e) ´ Distance of ç , ÷ from the line
x2 y2 è2 2ø
There fore, equation is + = 1 or 3x2 + 4 y2 = 12
4 3 3x + 4 y - 1 = 0
25. Let S(- 1, 1) be the focus and ZZ ¢ be the directrix, and 1 1
3´ + 4 ´ -1
P (x, y) be any point on the ellipse and PM be 1 2 2 1
=2 ´ ´ =
perpendicular from P on the directrix. 2 5 2
Then, by definition, we have
1
SP = ePM Hence, l =
2
Þ SP 2 = e2PM 2
1
æ1ö
2
x- y+3 é 1ù Þ 162 l = 162 ´ = 81
2
Þ (x + 1) + ( y - 1) = ç ÷ 2 2
è2ø 1 +1 êëQ e = 2 úû
x2 y 2
28. Equation of tangent to + =1
Z P(x, y) 6 1
M
Þ y = mx ± 6m2 + 1 …(i)
x–y+3=0

(–1, 1)
S 2 2
x y
If (i) is also a tangent to the ellipse + = 1, then
5 6
Z¢ (6m2 + 1) = 5m2 + 6
Þ 7x2 + 7 y2 + 2xy + 10x - 10 y + 7 = 0 Þ m2 = 5
On comparing it with given equation, we get 29. The normal at ( 2 cos q, sin q) on the ellipse is
l = 7, m = 7 2 x sec q - y cosec q =1
\ | l - u | =| 7 - 7| = 0 æ3 ö
It passes through the centre ç , 2 ÷ of the circle
è2 ø
26. The distance between two flag-posts = 10
Þ 2a = 10 3
\ sec q - 2 cosec q = 1
Þ a =5 2
p
and distance between the flag-posts = 8 Solving, q =
4
Þ 2ae = 8
æ 1 ö
Þ ae = 4 The nearest point on the ellipse is ç1, ÷.
è 2ø
4
Þ e= æ 3ö
5 Its distance from the centre of circle is ç ÷,
Also, b2 = a 2 (1 - e2) è 2 ø
æ 16 ö
æ3ö
2
Þ b2 = 25ç1 - ÷ and Radius of circle = ç ÷ + 2 - 4 =
1
è 25 ø è2ø 2
Þ b2 = 9
3 -1
Þ b =3 \ Shortest distance =
2
Now, area of ellipse = pab Þ k =3
Þ p ´ 5 ´ 3 = 3lp 2
x y2
\ l =5 30. We have, + =1 …(i)
14 5
27. Given equation is The equation of normal to this ellipse at
(10x - 5)2 + (10 y - 5)2 = (3x + 4 y - 1)2 P ( 14 cos q, 5 sin q) is
2 2 2
æ 1ö æ 1ö æ 3x + 4 y - 1 ö 14 sec q x - 5 cosec q y = 9
Þ çx - ÷ + ç y - ÷ = ç ÷
è 2ø è 2ø è 10 ø
This meets the ellipse (i) at Q ( 14 cos 2q, 5 sin 2q).
2 2
æ 1ö æ 1ö 3x + 4 y - 1 \ 14 sec cos 2q - 5 cosec sin 2q = 9
Þ çx - ÷ + ç y - ÷ =
è 2ø è 2ø 10 Þ 18 cos 2 q - 9 cos q - 14 = 0
2 2
æ 1ö æ 1ö 1 3x + 4 y - 1 Þ (6 cos q - 7) (3 cos q +2) = 0
Þ çx - ÷ + ç y - ÷ =
è 2ø è 2ø 2 5 Þ 3 cos q + 2 = 0
16
Hyperbola
A hyperbola is the set of points in a plane whose distances from two fixed IN THIS CHAPTER ....
points in the plane have constant difference. The two fixed points are the foci
of the hyperbola. Hyperbola

OR Conjugate Hyperbola
Auxiliary Circle
A hyperbola is the locus of a point in a plane which moves in the plane in such
a way that the ratio of its distance from a fixed point (i.e. focus) in the same Position of a Point with respect to
plane to its distance from a fixed line (i.e. directrix) is always constant which a Hyperbola
is always greater than unity. Intersection of a Line and a
Hyperbola
M P Tangent to the Hyperbola
Normal to the Hyperbola
Number of Normals and
Conormal Points
Directrix S(Focus) Equation of Pair of Tangents
Chord of Contact
SP Director Circle
Mathematically, = e, where e > 1 is eccentricity of the hyperbola.
PM Asymptotes
Rectangular Hyperbola
Standard Equation of Hyperbola
If the centre of the hyperbola is at the origin and foci are on the X-axis or
Y-axis, then that types of equation are called standard equation of an ellipse.
The two such possible orientations are shown below.
Y
Y (0, be)

X¢ X X¢ X
(–ae,0) O (a,0) (ae, 0) O

(0, –be)


x2 y2 y2 x2
(i) - =1 (ii) - =1
a2 b2 a2 b2
410 JEE Main Mathematics

Terms Related to Hyperbola Example 1. The equation of the hyperbola whose


directrix is 2 x + y = 1, focus is (1, 2) and eccentricity is 3 ,
Y Directrix
Directrix Q is 7x 2 - 2y 2 + 12 xy - 2 x + 14y + k = 0 , then k is equal to
M P

(a) 0
Latusrectum (b) 22
X¢ X
S¢ (–ae, 0) A¢ C A (c) –22
Centre
S (ae, 0)
(a, 0)
Latusrectum (–a, 0) Double (d) None of the above
ordinate
Q' Sol. (c) We know that, SP 2 = e2PM 2
x = –a/e x = a/e

Þ 5 [( x - 1) 2 + ( y - 2) 2] = 3(2x + y - 1) 2
Vertices Þ 5x2 + 5y 2 - 10 x - 20y + 25
The point A and A¢, where the curve meets the line joining = 3( 4x2 + y 2 + 1 + 4xy - 4x - 2y)
the foci S and S ¢, are called the vertices of the hyperbola.
Þ 7x2 - 2y 2 + 12xy - 2x + 14y - 22 = 0
Transverse and Conjugate Axes Thus, k = -22
Transverse axis is the one which lie along the line passing
through the foci and perpendicular to the directrices and Conjugate Hyperbola
conjugate axis is the one which is perpendicular to the
The hyperbola whose transverse and conjugate axes
transverse axis and passes through the mid-point of the foci
(i.e. centre). are respectively the conjugate and transverse axes of a
given hyperbola is called the conjugate hyperbola of
Centre the given hyperbola.
The middle point C of AA¢ bisects every chord of the The hyperbola conjugate of the hyperbola
hyperbola passing through it and is called the centre of
x2 y2
the hyperbola. 2
- =1
a b2
Focal Chord x2 y2
A chord of a hyperbola which is passes through the focus is is - + =1
a2 b2
called a focal chord of the hyperbola.
Y
Latusrectum

Directries
A line which is perpendicular to the axis and it lies between
a
centre and vertex. The equations of directries are x = ± . (0, be)
Directrix

e
S
Double Ordinates
B(0, b)
If Q be a point on the hyperbola draw QN perpendicular to
Z
the axis of the hyperbola and produced to meet the curve y = b/e
again at Q¢. Then, QQ¢ is called a double ordinate of Q. C Centre
X¢ X
Latusrectum Z'
Directrix

y = –b/e
The double ordinate passing through focus is called
B'(0, –b)
latusrectum.
Note
• The vertex divides the join of focus and the point of intersection of S'
(0, –be)
Latusrectum

directrix with axis internally and externally in the ratio e : 1.


x2 y2
• Domain and range of a hyperbola - = 1 are x £ - a or x ³ a
a2 b2
and y Î R respectively. Y¢
Hyperbola 411

Detailed Description of Hyperbola


Fundamental Terms Hyperbola Conjugate Hyperbola
2 2
x y x2 y2
(a) Equation 2
- 2
=1 - 2
+ =1
a b a b2
Y

Y S
A
X¢ Z X
(b) Graph X¢ X
S¢ A¢ Z¢ Z A S Z¢

Y¢ S'

(c) Centre C( 0, 0) C( 0, 0)
(d) Vertices ( ± a, 0) ( 0, ± b )
(e) Length of transverse axis 2a 2b
(f) Length of conjugate axis 2b 2a
(g) Foci ( ± ae, 0) ( 0, ± be )

x = ± æç ö÷ y = ± æç ö÷
a b
(h) Equation of directrices èe ø èe ø

b2 a2
(i) Eccentricity e = 1+ e = 1+
a 2 b2
2 2
2b 2a
(j) Length of latusrectum
a b
æ b2 ö æ a2 ö
(k) Ends of latusrectum ç ± ae, ± ÷ ç ± , ± be ÷
è aø è b ø
ì x = a sec q ì x = b sec q
(l) Parametric equations í í
î y = b tan q î y = a tan q
(m) Parametric coordinates ( a sec q, b tan q), 0 £ q £ 2 p ( b sec q, a tan q), 0 £ q £ 2 p
(n) Focal radi |SP | = (ex1 - a ) and|S ¢ P | = (ex1 + a ) |SP | = (ey1 - b ) and|S ¢ P | = (ey1 + b )
(o) Difference of focal radi =|SP | -|S ¢ P | 2a 2b
(p) Distance between foci 2 ae 2 be
(q) Tangents at vertices x = a and x = - a x = b and y = - b

Example 2. A hyperbola has its centre at the origin, passes 4 2


Þ b2 =Þb =
through the point (4, 2) and has transverse axis of length 3 3
4 along the X-axis. Then the eccentricity of the hyperbola is Now, eccentricity,
(JEE Main 2019) b2
e = 1+
2 3 a2
(a) 2 (b) (c) (d) 3
3 2
4
2
x y2 1 2
Sol. (b) Equation of hyperbola is given by - =1 = 1+ 3 = 1+ =
a2 b 2 4 3 3
Q Length of transverse axis = 2a = 4
\ a=2 Example 3. If the vertices of a hyperbola be at ( -2, 0) and
x2 y 2 (2, 0) and one of its foci be at ( -3, 0), then which one of the
Thus, - = 1 is the equation of hyperbola following points does not lie on this hyperbola?
4 b2
(JEE Main 2019)
Q It passes through ( 4, 2).
(a) (2 6 , 5) (b) (6, 5 2)
16 4 4
\ - 2 = 1Þ 4 - 2 = 1 (c) ( 4, 15) (d) ( - 6, 2 10 )
4 b b
412 JEE Main Mathematics

Sol. (b) The vertices of hyperbola are given as ( ± 2, 0) and one of æ 5 ö æ5 ö


(a) ç - , 0 ÷ (b) ( - 5, 0) (c) ç , 0 ÷ (d) (5, 0)
its foci is at ( - 3, 0). è 3 ø è3 ø
\ ( a, 0) = (2, 0) and ( - ae, 0), = ( - 3, 0)
Sol. (b) Equation of given hyperbola is16x2 - 9y 2 = 144
On comparing x-coordinates both sides, we get
x2 y 2
Þ a = 2 and - ae = - 3 Þ - =1 …(i)
3 9 16
Þ 2e = 3 Þ e = So, the eccentricity of Eq. (i)
2
9 b 2 é b2 ù 16 5
=1+ Þ b2 = 5 2 e = 1+ =
Also, êQ e = 1 + 2 ú 9 3
4 4 ë a û
x2 y 2
So, equation of the hyperbola is [Q the eccentricity (e) of the hyperbola - = 1 is
a2 b 2
x2 y 2
- =1 … (i) 1 + ( b / a) 2]
4 5
The point (6, 5 2) from the given options does not satisfy the and given directrix is 5x + 9 = 0
above equation of hyperbola. Þ x = - 9 /5
æ æ5ö ö
So, corresponding focus is ç - 3 ç ÷ , 0 ÷ = ( - 5, 0)
Example 4. Let e1 and e2 be the eccentricities of the è è3ø ø
x2 y 2 x2 y 2
ellipse, + 2 = 1( b < 5) and the hyperbola, - =1 Example 6. A hyperbola having the transverse axis of
25 b 16 b 2
length 2 has the same foci as that of the ellipse
respectively satisfying e1e2 = 1. If a and b are the distances
between the foci of the ellipse and the foci of the hyperbola 3x 2 + 4y 2 = 12, then this hyperbola does not pass through
respectively, then the ordered pair (a , b) is equal to which of the following points? (JEE Main 2020)
(JEE Main 2020) æ 1 ö æ 3 ö
(a) ç ,0 ÷ (b) ç - ,1÷
æ 20 ö æ 24 ö è 2 ø è 2 ø
(a) (8, 12) (b) (8, 10) (c) ç , 12÷ (d) ç ,10 ÷
è 3 ø è5 ø
æ 1 ö æ 3 1 ö
2 2 (c) ç1, - ÷ (d) ç , ÷
x y è 2ø è 2 2ø
Sol. (b) For the given ellipse, + = 1 , ( b < 5),
25 b 2
Sol. (d) Equation of given ellipse is
b2
the eccentricity e1 = 1 - x2 y 2
25 3x2 + 4y 2 = 12 Þ + =1 … (i)
2 2 4 3
x y
and for the given hyperbola - = 1, 3 1
16 b 2 \Eccentricity of ellipse (i) is e1 = 1 - =
4 2
b2
the eccentricity e2 = 1 + \Coordinate of foci is ( ± 1, 0).
16
Now, it is given that length of the transverse axis of
b2 b2 2
Since, e1 e2 = 1 Þ 1 - 1+ =1 hyperbola is 2, so e2 = 1,
25 16 2
æ b2 ö æ b2 ö b2 b2 b4 where e2 is the eccentricity of the hyperbola.
Þ ç1 - ÷ ç1 + ÷ =1 Þ 1- + - =1
è 25 ø è 16 ø 25 16 400 So, e2 = 2
Þ b 2 = 25 - 16 Þ b 2 = 9 \Given hyperbola is rectangular hyperbola and it’s equation
4 5 is
\ e1 = and e2 = x2 y2 1
5 4 2
- 2
= 1 Þ x2 - y 2 = … (ii)
Now, as a = distance between the foci of the ellipse æ 1 ö æ 1 ö 2
ç ÷ ç ÷
4 è 2ø è 2ø
Þ a =2 ´5 ´ Þa =8
5 æ 3 1 ö
So, from the given options point ç , ÷ does not pass
and b = distance between the foci of the hyperbola è 2 2ø
5
Þ b = 2 ´ 4 ´ = 10 through the hyperbola (ii). Hence, option (d) is correct.
4
\The order pair ( a , b) = (8, 10)
Hence, option (b) is correct.
Auxiliary Circle
x2 y2
Example 5. If 5 x + 9 = 0 is the directrix of the hyperbola Let 2
-= 1 be the hyperbola, then equation of the
a b2
16 x 2 - 9y 2 = 144, then its corresponding focus is auxiliary circle is x 2 + y 2 = a 2
(JEE Main 2019)
Hyperbola 413

Let ÐQCN = f. Sol. (c) The equation of hyperbola is 9x2 - y 2 = 1.


Here, P and Q are the corresponding points on the At point (5, – 4), 9(5) 2 - ( -4) 2 - 1 = 225 - 16 - 1 = 208 > 0
hyperbola and the auxiliary circle ( 0 £ f < 2p ). So, the point (5, – 4) lies inside the hyperbola 9x2 - y 2 = 1.
Y
Intersection of a Line and a Hyperbola
P(x, y)
Q x2 y2
Let the hyperbola be 2
-
=1 …(i)
90° a b2
f N

(–a, 0)A' (0, 0)C A(a, 0)
X and the line be y = mx + c …(ii)
On eliminating y from Eqs. (i) and (ii), we get
x 2 ( mx + c)2
- =1
a2 b2

Þ b2x 2 - a 2( mx + c)2 = a 2b2
x2 y2 Þ ( a 2m 2 - b2 )x 2 + 2mca 2x + a 2( b2 + c2 ) = 0
e.g. If the equation of hyperbola is - = 1 , then …(iii)
9 16
It is a quadratic equation in x which gives two values of x.
equation of auxiliary circle is x 2 + y 2 = 9.
\ D = 4m 2c2a 4 - 4( a 2m 2 - b2 )a 2( b2 + c2 )
Parametric Equations = - a 2m 2 + b2 + c2
The equations x = a sec f and y = b tan f are known as (i) If D > 0 i.e. c2 > a 2m 2 - b2, then line intersect the
x2 y2 hyperbola at two points.
the parametric equations of the hyperbola - =1.
a2 b2
( a sec f , b tan f ) lies on the hyperbola for all values of f.
X¢ X
c
Position of a Point mx+
y=
with respect to a Hyperbola
x2 y2 (ii) If D = 0 i.e. c2 = a 2m 2 - b2, then line touches the
Let the equation of a hyperbola be - = 1 and hyperbola.
a2 b2
coordinates of a point P be ( h , k). y = mx + c

Then, point P lies outside, if


X¢ X
h2 k2
- -1< 0
a2 b2
Y P(outside)
B P(on) (iii) If D < 0 i.e. c2 < a 2m 2 - b2, then line neither touches
P(inside) nor intersect the hyperbola.
X¢ X y = mx + c
A' O A

B'
Y¢ X¢ X

h2 k2
Point P lies on the hyperbola, if - =1
a2 b2
h2 k2 Condition of Tangency
and point P lies inside, if - -1> 0
a2 b2 x2 y2
The line y = mx + c touches the hyperbola - = 1, if
a2 b2
Example 7. The position of the point (5, - 4) relative to the c2 = a 2m 2 - b2
2 2
hyperbola 9 x - y = 1, is and the coordinates of the point of contact are
(a) outside (b) on the hyperbola æ a 2m b2 ö
ç± ,± ÷
(c) inside (d) None of these ç a 2m 2 - b2 a 2m 2 - b2 ÷ø
è
414 JEE Main Mathematics

Example 8. The line x cos a + y sin a = p touches the æ bö æ bö


(a) sin -1ç ÷ (b) sin -1ç ÷
2 2 è mø è aø
x y
hyperbola - = 1, if a 2 cos2 a - b 2 sin2 a is equal to
2
b2 æ b ö
a (c) sin -1ç ÷ (d) None of these
2 2 2 è ma ø
(a) p (b) p (c) - p (d) 2p
Sol. (c) Since, the point ( a sec q, b tan q) lies on the line
Sol. (b) The given line is x cos a + y sin a = p
or y = - x cot a + p cosec a y = mx + a2m2 - b 2
On comparing with y = mx + c, we get
m = - cot a, x = p cosec a \ b tan q = am sec q + a2m2 - b 2
Since, the given line touches the hyperbola Þ ( b tan q - am sec q ) 2 = a2m2 - b 2
x2 y 2
- = 1, then c2 = a2m2 - b 2 Þ b 2 tan 2 q + a2m2 sec2 q -2abm tan q sec q = a2m2 - b 2
a2 b 2
\ p 2 cosec2 a = a2 cot 2 a - b 2 Þ a2m2 tan 2 q - 2abm tan q sec q + b 2 sec2 q = 0
Þ a2 cos2 a - b 2 sin 2 a = p 2 Þ a2m2 sin 2 q - 2abm sin q + b 2 = 0 [Q cos q ¹ 0 ]

2 abm ± 4a2b 2m2 - 4a2b 2m2 æ b ö


\ sin q = =ç ÷
Tangent to a Hyperbola 2 a2m2 è am ø
A line which intersect the hyperbola at only one point is æ b ö
\ q = sin -1ç ÷
called the tangent to a hyperbola. è am ø

Equation of Tangent in Different Forms Example 10. If the eccentricity of the standard hyperbola
passing through the point ( 4, 6) is 2, then the equation of the
Point Form tangent to the hyperbola at ( 4, 6) is (JEE Main 2019)
x2 y2
The equation of tangent to the hyperbola - = 1 at (a) 3x - 2y = 0 (b) x - 2y + 8 = 0
2
a b2 (c) 2x - y - 2 = 0 (d) 2x - 3y + 10 = 0
xx1 yy1
( x1 , y1 ) is - = 1.
a2 b2 Sol. (c) Let the equation of standard hyperbola is
x2 y 2
- =1 …(i)
Slope Form a2 b 2
The equations of tangents of slope m to the hyperbola Now, eccentricity of hyperbola is
x2 y2 b2
- = 1 are given by y = mx ± a 2m 2 - b2. 1+ =2 (given)
a 2 b2 a2
Þ a2 + b 2 = 4 a2
Parametric Form
x2 y2 Þ b 2 = 3 a2 …(ii)
The equation of the tangent to the hyperbola 2
- 2
=1 Since, hyperbola (i) passes through the point (4, 6)
a b
x y 16 36
at ( a sec q , b tan q ) is secq - tan q = 1 \ - =1 …(iii)
a b a2 b 2
On solving Eqs. (ii) and (iii), we get
Note
a2 = 4 and b 2 = 12 …(iv)
• The tangents at the points P (a sec q1, b tan q1 ) and
Now, equation of tangent to hyperbola (i) at point ( 4, 6), is
Q(a sec q2 , b tan q2 ) intersect at the point 4x 6y
ì æ q1 - q2 ö æ q1 - q2 ö ü - =1
ïï a cos çè 2 ÷ø b sin çè 2 ÷ø ïï a2 b 2
Rí , ý. 4x 6y
Þ - =1 [from Eq. (iv)]
ï cos æç q1 + q2 ö÷ cos æç q1 + q2 ö÷ ï 4 12
îï è 2 ø è 2 ø þï y
Þ x - =1
• To a hyperbola, two tangents can be drawn from a point 2
outside the hyperbola. Þ 2x - y - 2 = 0

Example 9. If the line y = mx + a 2m 2 - b 2 touches the Example 11. The equation of a tangent to the hyperbola
x 2
y 2 4x 2 - 5y 2 = 20 parallel to the line x - y = 2 is (JEE Main 2019)
hyperbola - = 1 at the point ( a sec q , b tan q ), then q is
a2 b2 (a) x - y - 3 = 0 (b) x - y + 9 = 0
equal to (c) x - y + 1 = 0 (d) x - y + 7 = 0
Hyperbola 415

Sol. (c) Given equation of hyperbola is 4x2 - 5y 2 = 20 Þ ax cos f + by cot f - ( a2 + b 2) = 0 ...(i)


x2 y 2 The straight line lx + my - n = 0 will be a normal to the
which can be rewritten as - =1
5 4 x2 y 2
hyperbola 2 - 2 = 1, then Eq. (i) and l x + my - n = 0
The line x - y = 2 has slope, m = 1 a b
\ Slope of tangent parallel to this line = 1 represent the same line.
x2 y 2 a cos f b cot f ( a2 + b 2)
We know equation of tangent to hyperbola 2 - 2 = 1 \ = =
a b l m n
having slope m is given by y = mx ± a2m2 - b 2 na
Þ sec f = 2
l( a + b 2)
Here, a2 = 5, b 2 = 4 and m = 1
nb
\Required equation of tangent is and tan f =
m( a2 + b 2)
Þ y = x ± 5 - 4 Þ y = x± 1 Þ x - y ± 1=0
\ sec2 f - tan 2 f = 1
n 2a2 n 2b 2 a2 b 2 ( a2 + b 2) 2
Normal to a Hyperbola Þ 2 2 22
- 2 2
l (a + b ) m (a + b ) 22
=1 Þ 2 - 2 =
l m n2
A line which is perpendicular to the tangent of the
hyperbola is called the normal to a hyperbola. Thus, k = n2

Example 13. If a hyperbola passes through the point


Equation of Normal in Different Forms P(10, 16) and it has vertices at ( ± 6, 0), then the equation of
Point Form the normal to it at P is (JEE Main 2020)
The equation of normal at the point ( x1 , y1 ) to the (a) 3x + 4y = 94 (b) x + 2y = 42
x2 y2 a 2x b2 y (c) 2x + 5y = 100 (d) x + 3y = 58
hyperbola 2 - 2 = 1 is + = a 2 + b2
a b x1 y1 Sol. (c) Since, the vertices of hyperbola on X-axis at ( ±6, 0), so
equation of hyperbola we can assume as
Slope Form x2 y 2
The equation of the normals of slope m to the hyperbola - = 1 and| a| = 6
a2 b 2
x2 y2 m( a 2 + b2 ) and hyperbola passes through point P(10 , 16) so
2
- 2 = 1 are given by y = mx m at the points
a b a 2 - b2m 2 (10) 2 (16) 2
- 2 =1
æ ö 62 b
ç± a2 b2m ÷
,m (16) 2 100 - 36 64
ç a 2 - b2m 2 a 2 - b2m 2 ÷ø Þ = =
è b2 36 36
2 36 ´ 256
Note The line y = mx + c will be a normal to the hyperbola Þ b = = 144
64
x2 y2 m2 (a2 + b2 )2
- 2 = 1, if c 2 = 2 x2 y2
a2
b a - b2 m2 So, equation of hyperbola is - = 0 , and the equation
36 144
of normal to hyperbola at point P is
Parametric Form 36x 144y
x2 y2 + = 36 + 144
The equation of the normal to the hyperbola 2
- = 1 at 10 16
a b2 18
( a sec q , b tan q ) is ax cosq + by cotq = a 2 + b2 Þ x + 9y = 180
5
Note Four normals can be drawn to the hyperbola from any point Þ 2x + 5y = 100
outside to the hyperbola. Hence, option (c) is correct.

Example 12. If the line lx + my - n = 0 will be a normal to Example 14. If the line y = mx + 7 3 is normal to the
a 2
b 2 2 2 2
(a + b ) x2 y 2
the hyperbola, then - = , where k is equal to hyperbola - = 1, then a value of m is (JEE Main 2019)
l2 m2 k 24 18
(a) n (b) n 2 3 15 2 5
(a) (b) (c) (d)
(c) n3 (d) None of these 5 2 5 2
x2 y 2 Sol. (c) Given equation of hyperbola, is
Sol. (b) The equation of any normal to - = 1 is
a2 b 2 x2 y 2
- =1 …(i)
ax cos f + by cot f = a2 + b 2 24 18
416 JEE Main Mathematics

Since, the equation of the normals of slope m to the


x2 y 2
Equation of Pair of Tangents
hyperbola 2 - 2 = 1, are given by
a b x2 y2
Let P ( x1 , y1 ) be a point outside the hyperbola - = 1.
m( a2 + b 2) a2 b2
y = mx m Y
a2 - b 2m2 P(x1, y1)
A
\ Equation of normals of slope m, to the hyperbola (i), are
m(24 + 18)
y = mx ± …(ii) X¢ X
24 - m2(18)
Q Line y = mx + 7 3 is normal to hyperbola (i).
\On comparing with Eq. (ii), we get Y¢
m( 42) 6m Then, the equation of pair of tangents drawn from
± =7 3 Þ± = 3
2
24 - 18m 24 - 18m2 external point P to the hyperbola is given by
36m2 SS1 = T 2
Þ =3 [squaring both sides]
24 - 18m2 x2 y2 x12 y12
where, S= - - 1, S1 = - -1
Þ 12m2 = 24 - 18m2 Þ 30m2 = 24 a2 b2 a2 b2
2 xx1 yy1
Þ 5m2 = 4 Þ m = ± and T = - -1
5 a2 b2

Number of Normals and Chord of Contact


Conormal Points Let PA and PB be any two tangents to the hyperbola from
a point P ( x1 , y1 ), then AB is known as chord of contact
There are exactly four lines passing through a given
and its equation is given by
point such that they are normals to the hyperbola at the
points where they intersect the hyperbola. Such points on Y
A
the hyperbola are known as the conormal points.
P(x1, y1)
Properties of Eccentric Angles X¢ X
of Conormal Points C Chord of
contact
(i) The sum of the eccentric angles of conormal points is B

an odd multiple of p. xx1 yy1
T =0 Þ - -1= 0
(ii) If q 1 , q 2 , q 3 and q 4 are eccentric angles of four points a2 b2
x2 y2
on the hyperbola 2 - 2 = 1, the normals at which Equation of the Chord Bisected
a b
at a Given Point
are concurrent, then
x2 y2
The equation of the chord of the hyperbola - = 1,
(a) S cos (q 1 + q 2 ) = 0 a2 b2
(b) S sin (q 1 + q 2 ) = 0 bisected at the point ( x1 , y1 ) is
xx1 yy1 x2 y2
(iii) If q 1, q 2 and q 3 are the eccentric angle of three points 2
- 2 - 1 = 12 - 12 - 1 or T = S1
a b a b
x2 y2
on the hyperbola 2 - 2 = 1 such that where T and S1 have their usual meanings.
a b
sin (q 1 + q 2 ) + sin (q 2 + q 3 ) + sin (q 3 + q 1 ) = 0, Example 15. From the points on the circle x 2 + y 2 = a 2
then the normals at these points are concurrent. tangents are drawn to the hyperbola x 2 - y 2 = a 2, then the
locus of the middle points of the chords of contact is the
(iv) If the normals at four points P ( x1 , y1 ), Q( x2 , y2 ),
curve ( x 2 - y 2) 2 = k( x 2 + y 2), where k is equal to
x2 y2
R( x3 , y3 ) and S ( x4 , y4 ) on the hyperbola 2 - 2 = 1 (a) a (b) a3
a b
(c) a2 (d) None of these
are concurrent, then
Sol. (c) Since, any point of the circle x2 + y 2 = a2is ( a cos q, a sin q),
æ1 1 1 1ö
( x1 + x2 + x3 + x4 ) ç + + + ÷=4 chord of contact of this point w.r.t. hyperbola x2 - y 2 = a2 is
è x1 x2 x3 x4 ø
x ( a cos q) - y ( a sin q) = a2
Hyperbola 417

or x cos q - y sin q = a ...(i) The equations of two asymptotes of the hyperbola


If its mid-point be (h, k), then it is same as x2 y2 b
T = S1 - = 1 are y = ± x.
a 2 b2 a
Þ hx - ky - a2 = h 2 - k2 - a2 Y
Þ hx - ky = h 2 - k2 ...(ii)
2
On comparing Eqs. (i) and (ii), we get x2 – y = 1
2
cos q sin q a a b2
= = 2 2 X¢ X
h k (h - k ) A' A
C
2 2
\ (h - k ) cos q = ah ...(iii)
2 2
and (h - k ) sin q = ak ...(iv)
On squaring and adding Eqs. (iii) and (iv), we get Y¢
2 22 2 2 2 2 2 22 2 2 2 Note
(h - k ) = a h + a k Þ (h - k ) = a (h + k )
• A hyperbola and its conjugate hyperbola have the same
Hence, the required locus is
asymptotes.
( x2 - y 2) 2 = a2( x2 + y 2)
x2 y2
• The angle between the asymptotes of 2 - 2 = 1 is
a b
Director Circle -1 æ b ö
2 tan ç ÷.
The locus of the point of intersection of the tangents to è aø
x2 y2 • The asymptotes pass through the centre of the hyperbola.
the hyperbola 2 - 2 = 1, which are perpendicular to • The bisectors of the angles between the asymptotes are the
a b
each other is called director circle. The equation of the coordinate axes.
• The product of the perpendiculars from any point on the
x2 y2
director circle of the hyperbola 2 - 2 = 1 is x2 y2 a2 b2
a b hyperbola - = 1 to its asymptotes is equal to .
a2 b2 a2 + b2
x 2 + y 2 = a 2 - b2.
Y
Example 17. The asymptotes of a hyperbola having centre
90° at the point (1, 2) are parallel to the lines 2 x + 3y = 0 and
3x + 2y = 0. If the hyperbola passes through the point (5, 3), its
X¢ X equation is (2 x + 3y - 8)(3x + 2y - 7) = k, where k is equal to
A' C A
(a) 15 (b) 154
(c) 0 (d) None of these

Sol. (b) Let the asymptotes be 2x + 3y + l = 0 and 3x + 2y + m = 0 .
x2 y2
Note For director circle of 2 - 2 = 1, a must be greater than b. If Since, asymptotes passes through (1, 2), then
a b l = -8 and m = -7
a < b, then director circle x 2 + y 2 = a2 - b2 does not exist.
Thus, the equations of asymptotes are
2x + 3y - 8 = 0 and 3x + 2y - 7 = 0
Example 16. The equation of the director circle of the
Let the equation of hyperbola be
hyperbola 9 x 2 - 16y 2 = 144 is (2x + 3y - 8)(3x + 2y - 7) + v = 0 …(i)
(a) x2 + y 2 = 7 (b) x2 + y 2 = 9 It passes through (5, 3), then
2 2
(c) x + y = 16 (d) x2 + y 2 = 25 (10 + 9 - 8)(15 + 6 - 7) + v = 0 Þ 11 ´ 14 + v = 0
x2 y 2 \ v = -154
Sol. (b) Given equation can be rewritten as - =1 On putting the value of v in Eq. (i), we get
16 9
\Required equation of director circle is (2x + 3y - 8)(3x + 2y - 7) - 154 = 0
x2 + y 2 = 16 - 9 This is the equation of required hyperbola.

\ x2 + y 2 = 9 Rectangular Hyperbola
A hyperbola whose asymptotes are at right angles to each
Asymptotes other, is said to be a rectangular hyperbola.
An asymptote of any hyperbola is a straight line which Or
touches in it two points at infinity. In other words
If the length of transverse and conjugate axes of any
asymptotes are the lines which are tangents to the curve
hyperbola are equal, then hyperbola is known as
at infinity.
rectangular hyperbola.
418 JEE Main Mathematics

The equation of rectangular hyperbola to the hyperbola (f) Directrices, x + y = ± 2c


x2 y2 (g) Latusrectum = 2 2c
- = 1 is x 2 - y 2 = a 2 and equation of the asymptotes
a 2 b2 (h) Point form, x = x1 , y = y1
are y = ± x i.e. y = x and y = - x. Clearly, each of these two
x y
asymptotes is inclined at 45° to the transverse axis. Equation of tangent, xy1 + yx1 = 2c2 Þ + =2
x1 y1
When the centre of any rectangular hyperbola is at the
origin and its asymptotes coincide with the coordinate Equation of normal, xx1 - yy1 = x12 - y12
axes, then the equation of rectangular hyperbola is xy = c2. c
(i) Parametric form : x = ct , y = where t Î R - { 0 }
Its eccentricity is 2. t
Rectangular Hyperbola of the Form Equation of tangent, x + yt 2 = 2ct
æ 1ö
x 2 - y2 = a2 Equation of normal, t 2x - y = c ç t3 - ÷
è tø
(a) Asymptotes are perpendicular lines i.e., x ± y = 0
Y Properties of Rectangular Hyperbola xy = c 2
L¢ L (a) Equation of the chord joining tl and t2 is
x + yt1t2 - c( t1 + t2 ) = 0.
A1 A (b) Equation of tangent at ( x1 , y1 ) is xy1 + x1 y = 2c2.
X¢ X
S1 S
O æ cö x
(c) Equation of tangent at ç ct , ÷ is + yt = 2 c.
è tø t
I¢ I
L¢1 Y¢ L1 (d) Point of intersection of tangents at t1 and t2 is
æ 2 ct1t2 2c ö
(b) Eccentricity, e = 2. (c) Centre, (0, 0) ç , ÷.
è t1 + t2 t1 + t2 ø
a
(d) Foci ( ± 2 a , 0) (e) Directrices, x = ± æ cö
2 (e) Equation of normal at ç ct , ÷ is xt3 - yt - ct 4 + c = 0.
è tø
(f) Latusrectum = 2a
(f) Equation of normal to rectangular hyperbola at
(g) Point form, x = x1 , y = y1 ( x1 , y1 ) is x x1 - yy1 = x12 - y12.
Equation of tangent, xx1 - yy1 = a 2
x1 y1 Example 18. Number of common tangent with finite slope
Equation of normal, + = 2. to the curve xy = c 2 and y 2 = 4ax is
x y
(a) 0 (b) 1 (c) 2 (d) 4
(h) Parametric form, x = a sec q , y = a tan q
Equation of tangent, x sec q - y tan q = a Sol. (b) Let the point on parabola P ( at12, 2at1) and on rectangular
x y æ cö
Equation of normal, + = 2a hyperbola q ç ct 2, ÷
sec q tan q è t2ø
Equation of tangent at P ( at12, 2at1) is
2
Rectangular Hyperbola of the Form xy = c Yt1 - x = at12 …(i)
(a) Asymptotes are perpendicular lines i.e., x = 0 and æ cö
y=0 and equation of tangent at Q ç ct 2, ÷ is
è t2ø
(b) Eccentricity, e = 2 (c) Centre,(0, 0)
x
(d) Foci are S ( 2c, 2c), S1( - 2c, - 2c) + yt 2 = 2c …(ii)
t2
(e) Vertices, V1( c, c), V 2( - c, - c) Now, Eqs. (i) and (ii) are identical.
Y t at 2
\ - t2 = 1 = 1
t 2 2c
S2 a(t 2) 4
Þ = - t2
V1 2c
X¢ X -2 c
Þ (t 2)3 =
a
V2
Þ only one real value of t 2 exists.
S1
Hence, only one common tangent of the curve
Y¢ xy = c2 and y 2 = 4ax
Hyperbola 419

Practice Exercise
ROUND I Topically Divided Problems

Basic Terms of Hyperbola, Equation of 7. The length of transverse axis of the hyperbola
Hyperbola and Intersection of Two Curves 3x 2 - 4 y 2 = 32 is
1. If equation (10 x - 5) 2 + (10 y - 4) 2 = (a)
8 2
(b)
16 2
2 2 3 3
l (3x + 4 y - 1) represents a hyperbola, then
3 64
(a) -2 < l < 2 (b) l > 2 (c) (d)
32 3
(c) l < -2 or l > 2 (d) 0 < l < 2
2. If a directrix of a hyperbola centred at the origin 8. The difference between the length 2a of the
and passing through the point ( 4, - 2 3) is transverse axis of a hyperbola of eccentricity e and
5 x = 4 5 and its eccentricity is e, then (JEE Main 2019) the length of its latusrectum is
(a) 2a (3 - e 2) (b) 2a |2 - e 2|
(a) 4e4 - 12e2 - 27 = 0 (b) 4e4 - 24e2 + 27 = 0
(c) 2a (e 2 - 1) (d) a (2e 2 - 1)
(c) 4e4 + 8e2 - 35 = 0 (d) 4e4 - 24e2 + 35 = 0
x2 y2
3. If a hyperbola has length of its conjugate axis equal 9. If eccentricity of hyperbola - = 1 is e and e¢ is
a 2 b2
to 5 and the distance between its foci is 13, then the the eccentricity of its conjugate hyperbola, then
eccentricity of the hyperbola is (JEE Main 2019)
(a) e = e¢ (b) ee¢ = 1
13 1 1
(a) (b) 2 (c) 2 + =1 (d) None of these
12 e (e¢)2
13 13
(c) (d)
8 6 10. The equation of the transverse axis of the
x2 y2 hyperbola ( x - 3) 2 + ( y + 1 ) 2 = ( 4 x + 3 y) 2 is
4. A general point on the hyperbola 2 - 2 = 1 is
a b (a) x + 3 y = 0 (b) 4x + 3 y = 9
(a) (a sin q, b cos q) (where, q is parameter) (c) 3x - 4 y = 13 (d) 4x + 3 y = 0
(b) (a tan q, b sec q) (where, q is parameter) 11. The equation of the hyperbola whose eccentricity is
æ et + e- t et - e- t ö 2 and the distance between the foci is 16, taking
(c) ç a ,b ÷ (where, t is parameter)
è 2 2 ø transverse and conjugate axes of the hyperbola as x
(d) None of the above and y axes respectively, is
(a) x2 - y2 = 0 (b) x2 - y2 = 32
5. If a point ( x, y) = (tan q + sin q, tan q - sin q), then
(c) x2 - y2 = 2 (d) None of these
locus of ( x, y) is
(a) (x 2y) 2/ 3 + (xy 2) 2/ 3 = 1 12. The equation of the hyperbola whose foci are (6, 4)
2 2
(b) x - y = 4xy
and ( -4, 4) and eccentricity 2, is
(c) (x2 - y2)2 = 16xy 4(x - 1) 2 4( y - 4) 2 4(x + 1) 2 4( y + 4) 2
(a) + = 1 (b) + =1
(d) x2 - y2 = 6xy 25 25 25 75
2 2 2 2
4(x - 1) 4( y - 4) 4(x - 1) 4( y - 4)
6. The equation 16 x 2 - 3 y 2 - 32 x + 12 y - 44 = 0 (c) - = 1 (d) - =1
75 25 25 75
represents a hyperbola
(a) the length of whose transverse axis is 4 3
13. The distance between the foci of a hyperbola is 16
(b) the length of whose conjugate axis is 4 and its eccentricity is 2. Its equation is
(c) whose centre is (-1, 2) x 2 y2
(a) x2 - y2 = 32 (b) - =1
19 4 9
(d) whose eccentricity is (c) 2x - 3 y2 = 7 (d) None of these
3
420 JEE Main Mathematics

3 æ9 ö æ3 ö æ9 ö
14. Equation of the hyperbola with eccentricity and (a) ç , 3÷ (b) ç , 2÷ (c) ç , 2÷ (d) (9, 3)
2 è2 ø è2 ø è2 ø
foci at ( ± 2, 0) is
22. If a hyperbola passes through the point P ( 2 , 3)
x2 y2 4 x2 y2 4
(a) - = (b) - = and has foci at ( ± 2, 0), then the tangent to this
4 5 9 9 9 9
x2 y2 hyperbola at P also passes through the point
(c) - =1 (d) None of these (JEE Main 2017)
4 9
(a) (3 2 , 2 3 ) (b) (2 2 , 3 3 )
15. The vertices of a hyperbola are at (0, 0) and (10, 0) (c) ( 3 , 2 ) (d) (- 2 , - 3 )
and one of its foci is at (18, 0). The equation of
hyperbola is 23. The point of intersection of two tangents to the
2 2
(x - 5) 2 2 x2 y2
(a)
x
-
y
=1 (b) -
y
=1 hyperbola 2
- 2 = 1, the product of whose slope is
25 144 25 144 a b
2
x2 ( y - 5)2 (x - 5)2 ( y - 5)2 c , lies on the curve
(c) - =1 (d) - =1 (a) y 2 - b 2 = c 2(x 2 + a 2) (b) y 2 + a 2 = c 2(x 2 - b 2)
5 144 25 144
(c) y 2 + b 2 = c 2(x 2 - a 2) (d) y 2 - a 2 = c 2(x 2 + b 2)
16. The points of intersection of curves whose
parametric equations are x = t 2 + 1, y = 2 t and 24. Equation of the normal to the hyperbola
2 x2 y2
x = 2 S , y = is - = 1 perpendicular to the line 2 x + y = 1 is
S 25 16
(a) (1, -3) (b) (2, 2) (c) (-2, 4) (d) (1, 2) (a) 21 (x - 2 y) = 41 (b) x - 2 y = 1
(c) 41 (x - 2 y) = 41 (d) 21 (x - 2 y) = 21
17. The equation 9 x 2 - 16 y 2 - 18 x + 32 y - 151 = 0
25. The locus of point of intersection of two lengths of
represents a hyperbola
x2 y2
(a) the length of the transverse axes is 4 the hyperbola - = 1 , if the product of their
a 2 b2
(b) the length of latusrectum is 9
21 11 slopes is c2 , will be
(c) the equation of directrix is x = and x = -
5 5 (a) y2 - b2 = c2 (x2 + a 2) (b) y2 + b2 = c2 (x2 - a 2)
(d) None of the above (c) y2 + a 2 = c2(x2 - b2) (d) y2 - a 2 = c2(x2 + b2)

18. The equations of transverse and conjugate axes of a 26. The equation of common tangents to the two
hyperbola are respectively x + 2 y - 3 = 0, x2 y2 y2 x2
hyperbolas 2
- 2 = 1 and 2 - 2 = 1 are
2 x - y + 4 = 0 and their respectively lengths are 2 a b a b
and 2 / 3. The equation of the hyperbola is (a) y = ± x ± b2 - a 2 (b) y = ± x ± a 2 - b2
2 3
(a) (x + 2 y - 3)2 - (2x - y + 4)2 = 1 (c) y = ± x ± (a 2 - b2) (d) y = ± x ± a 2 + b2
5 5
2 2 3
(b) (2x - y + 4) - (x + 2 y - 3)2 = 1 27. Let P ( a sec q, b tan q) and Q ( a sec f, b tan f), where
5 5 p
(c) 2 (2x - y + 4)2 - 3 (x + 2 y - 3)2 = 1 q+f= be two points on the hyperbola
2
(d) 2 (2x - y + 4)2 - (x + 2 y - 3)2 = 1 x2 y2
- = 1. If ( h, k) is the point of intersection of
Tangent and Normal to the Hyperbola a 2 b2
normals at P and Q, then k is equal to
19. The straight line x + y = 2 p will touch the
a 2+ b2 é a 2 + b 2ù
2
hyperbola 4 x - 9 y = 36, if2 (a) (b) - ê ú
a ë a û
(a) p2 = 2 (b) p2 = 5 (c) 5 p2 = 2 (d) 2 p2 = 5
a 2+ b2 é a 2 + b 2ù
20. The product of the perpendicular from two foci on (c) (d) - ê ú
b ë b û
x2 y2
any tangent to the hyperbola - 2 = 1, is
a 2
b 25 3
28. The value of m, for which the line y = mx + is
(a) a 2 (b) b 2 (c) -a 2
(d) -b 2 3
x2 y2
x2 y2 a normal to the conic - = 1, is
21. Let P (3, 3) be a point on the hyperbola, 2 - 2 = 1. 16 9
a b 2
(a) ± (b) 3
If the normal to it at P intersects the X-axis at ( 9, 0) 3
3
and e is its eccentricity, then the ordered pair (c) - (d) None of these
2
( a 2 ,e2 ) is equal to (JEE Main 2020)
Hyperbola 421

Chord of Contact of a Hyperbola 35. The product of the lengths of perpendiculars drawn
from any point on the hyperbola x 2 - 2 y 2 - 2 = 0 to
29. If the chords of contact of tangents from two points
its asymptotes, is
( x1, y1) and ( x 2 , y2 ) to the hyperbola
x1 x2 (a) 1 /2 (b) 2 /3 (c) 3 /2 (d) 2
4 x 2 - 9 y 2 - 36 = 0 are at right angles, then is
y1 y2 36. A hyperbola has the asymptotes x + 2 y = 3 and
equal to x - y = 0 and passes through (2, 1). Its centre is
9 9 81 81 (a) (1, 2) (b) (2, 2)
(a) (b) - (c) (d) -
4 4 16 16 (c) (1, 1) (d) (2, 1)
30. If x = 9 is the chord of contact of the hyperbola 37. The angle between the asymptotes of the hyperbola
x 2 - y 2 = 9, then the equation of the corresponding x 2 + 2 xy - 3 y 2 + x + 7 y + 9 = 0 is
pair of tangents is (a) tan -1 (±2) (b) tan -1 (± 3 )
(a) 9x 2 - 8 y 2 + 18x - 9 = 0 æ 1 ö æ1ö
(c) tan -1 ç ÷ (d) tan -1 ç ÷
(b) 9x 2 - 8 y 2 - 18x + 9 = 0 è 3ø è2ø
(c) 9x 2 - 8 y 2 - 18x - 9 = 0
(d) 9x 2 - 8 y 2 + 18x + 9 = 0 38. The angle between the two asymptotes of the
x2 y2
31. If chords of the hyperbola x 2 - y 2 = a 2 touch the hyperbola - = 1 is
16 9
parabola y 2 = 4 ax. Then, the locus of the middle æ3ö æ3ö
(a) p - 2 tan -1 ç ÷ (b) p - 2 tan -1 ç ÷
points of these chords is è4ø è2ø
(a) y 2 = (x - a )x 3 (b) y 2 (x - a ) = x 3 æ3ö æ4ö
(c) 2 tan -1 ç ÷ (d) p - 2 tan -1 ç ÷
(c) x 2 (x - a ) = x 3 (d) None of these è4ø è3ø
32. The locus of middle points of chords of hyperbola 39. If tangent and normal to a rectangular hyperbola
3x 2 - 2 y 2 + 4 x - 6 y = 0 parallel to y = 2 x is
xy = c2 cut off intercepts a1 and a2 on one axis and
(a) 3x - 4 y = 4 (b) 3 y - 4x + 4 = 0
b1, b2 on the other, then
(c) 4x - 3 y = 3 (d) 3x - 4 y = 2
(a) a1 = b1 (b) a 2 = b2
Rectangular Hyperbola a
(c) 1 = 1
b
(d) a1a 2 + b1b2 = 0
a 2 b2
33. The equation of a line passing through the centre of
a rectangular hyperbola is x - y - 1 = 0. If one of the 40. The length of the transverse axis of the rectangular
asymptotes is 3x - 4 y - 6 = 0, the equation of other hyperbola xy = 18 is
asymptote is (a) 6 (b) 12
(a) 4x - 3 y + 17 = 0 (b) -4x - 3 y + 17 = 0 (c) 18 (d) 9
(c) -4x + 3 y + 1 = 0 (d) 4x + 3 y + 17 = 0
41. The equation of the common tangent to the curves
34. The asymptotes of the hyperbola xy = hx + ky are y 2 = 8 x and xy = -1 is
(a) x = k, y = h (b) x = h , y = k (a) 3 y = 9x + 2 (b) y = 2x + 1
(c) x = h , y = h (d) x = k, y = k (c) 2 y = y + 8 (d) y = x + 2

ROUND II Mixed Bag


Only One Correct Option 2. The normal at P to a hyperbola of eccentricity e,
1. If PQ is a double ordinate of the hyperbola intersects its transverse and conjugate axes at
x 2
y 2 L and M respectively. If locus of the mid-point of LM
- = 1 such that OPQ is an equilateral is hyperbola, then eccentricity of the hyperbola is
a 2 b2
triangle, O being the centre of the hyperbola. Then, æ e + 1ö
(a) ç ÷
the eccentricity e of the hyperbola satisfies è e - 1ø
2 2 e
(a) 1 < e < (b) e = (b)
2
3 3 (e - 1)
3 2 (c) e
(c) e = (d) e >
2 3 (d) None of the above
422 JEE Main Mathematics

3. The eccentricity of the hyperbola whose length of 10. If the tangent and normal to x 2 - y 2 = 4 at a point
the latusrectum is equal to 8 and the length of its cut off intercepts a1, a2 on the X-axis respectively
conjugate axis is equal to half of the distance and b1, b2 on Y -axis respectively, then the value of
between its foci, is (JEE Main 2016) a1a2 + b1b2 is
4 4 2 (a) 1 (b) -1 (c) 0 (d) 4
(a) (b) (c) (d) 3
3 3 3
11. The locus of the mid-points of the chord of the
x2 y2
4. With one focus of the hyperbola - = 1 as the circle, x 2 + y 2 = 25 which is tangent to the
9 16 x2 y2
centre, a circle is drawn which is tangent to the hyperbola, - = 1 is
9 16 (JEE Main 2021)
hyperbola with no part of the circle being outside (a) (x2 + y2)2 - 16x2 + 9 y2 = 0
the hyperbola. The radius of the circle is (b) (x2 + y2)2 - 9x2 + 144 y2 = 0
(a) less than 2 (b) 2 (c) (x2 + y2)2 - 9x2 - 16 y2 = 0
(c) 1/3 (d) None of these (d) (x2 + y2)2 - 9x2 + 16 y2 = 0
p
5. Let 0 < q < . If the eccentricity of the hyperbola 12. Let P be the point of intersection of the common
2
x2 y2 tangents to the parabola y 2 = 12 x and the
- = 1 is greater than 2, then the length hyperbola 8 x 2 - y 2 = 8. If S and S¢ denotes the foci
cos2 q sin 2 q
of its latus rectum lies in the interval (JEE Main 2019) of the hyperbola where S lies on the positive X-axis
3 then P divides SS¢ in a ratio (JEE Main 2019)
(a) (1, ] (b) (3, ¥)
2 (a) 13 : 11 (b) 14 : 13 (c) 5 : 4 (d) 2 : 1
3
(c) ( ,2] (d) (2, 3] 13. A straight line touches the rectangular hyperbola
2
9 x 2 - 9 y 2 = 8 and the parabola y 2 = 32 x. The
2 2
6. The tangents to the hyperbola x - y = 3 are equation of the line is
parallel to the straight line 2 x + y + 8 = 0, at which (a) 9x + 3 y - 12 = 0 (b) 9x - 3 y + 11 = 0
of the following points? (c) 9x + 3 y + 8 = 0 (d) 9x - 3 y - 7 = 0
(a) (2, 1) (b) (2, - 1) x2 y2
(c) (-2, - 1) (d) (-2, 1)
14. If the normal at q on the hyperbola - =1
a2 b2
x2 y2 meets the transverse axis at G, then AG × A ¢G is
7. P is a point on the hyperbola 2
- 2 = 1, N is the
a b equal to (where, A and A ¢ are the vertices of
foot of the perpendicular from P on the transverse hyperbola)
axis. The tangent to the hyperbola at P meets the (a) a 2 (e4 sec2 q - 1) (b) a 2 (e4 tan 2 q - 1)
transverse axis at T. If O is the centre of the (c) b2(e4 sec2 q - 1) (d) b2(e4 sec2 q + 1)
hyperbola, then OT ´ ON is equal to 15. Equation of a common tangent to the parabola
(a) e 2 (b) a 2 y 2 = 4 x and the hyperbola xy = 2 is (JEE Main 2019)
2 b2
(c) b (d) 2 (a) x + 2 y + 4 = 0 (b) x - 2 y + 4 = 0
a
(c) 4x + 2 y + 1 = 0 (d) x + y + 1 = 0
8. Consider a hyperbola H : x 2 - 2 y 2 = 4. Let the 16. If normal to the rectangular hyperbola xy = c2 at
tangent at a point P ( 4, 6 ) meet the X-axis at Q
the point t on it intersects the hyperbola at t1 , then
and latus rectum at R ( x1, y1), x1 > 0. If F is a focus
t 3t1 is equal to
of H which is nearer to the point P, then the area of
(a) 1 (b) 2 (c) -1 (d) -2
DQFR is equal to (JEE Main 2021)
7 17. The equation of the hyperbola whose foci are (–2, 0)
(a) 4 6 (b) 6 - 1 (c) -2 (d) 4 6 - 1
6 and (2, 0) and eccentricity is 2 is given by
x 2
y 2 (a) - 3x 2 + y 2 = 3 (b) x 2 - 3 y 2 = 3
9. Number of points on the hyperbola - =3 (c) 3x 2 - y 2 = 3 (d) - x 2 + 3 y 2 = 3
a 2 b2
p
from which mutually perpendicular tangents can 18. For some q Î æç0, ö÷, if the eccentric of the
è 2ø
be drawn to the circle x 2 + y 2 = a 2 , is/are
(a) 0 (b) 2 (c) 3 (d) 4 hyperbola, x - y 2 sec 2 q = 10 is 5 times the
2
Hyperbola 423

eccentricity of the ellipse, x 2 sec 2 q + y 2 = 5 , then 22. If (5, 12) and (24, 7) are foci of a hyperbola passing
the length of the latus rectum of the ellipse, is 12 e
through the origin, then the value of is ………
(JEE Main 2020) 386
2 5 4 5
(a) 2 6 (b) 30 (c) (d) (where, e is the eccentricity of hyperbola)
3 3
x2 y2
19. If e1 and e2 are the eccentricities of the ellipse, 23. PQ is double ordinate of hyperbola - = 1 such
a 2 b2
x2 y2 x2 y2
+ = 1 and the hyperbola, - =1 that OPQ is an equilateral triangle, O being the
18 4 9 4
respectively and ( e1, e2 ) is a point on the ellipse, centre of hyperbola, where the eccentricity of
15 x 2 + 3 y 2 = k, then k is equal to (JEE Main 2020) hyperbola e satisfy, 3 e > k, then the value of k is
(a) 14 (b) 15 (c) 17 (d) 16 ……… .
2 2
x y 24. If the tangent at point P ( h, k) on the hyperbola
20. Tangents at any point on the hyperbola 2
- 2 =1
a b x2 y2
- = 1 cuts the circle x 2 + y 2 = a 2 at the point
cut the axes at A and B respectively. If the a 2 b2
rectangle OAPB, where O is the origin is completed, 1 1 l
P1 ( x1, y1) and P2 ( x2 , y2 ), where + = , then
then locus of point P is given by y1 y2 k
a 2 b2 a 2 b2 the value of l is ……… .
(a) - =1 (b) + 2 =1
x2 y2 x2 y x2 y2
a 2 b2 25. If the tangent drawn to the hyperbola - =1
(c) 2 - 2 = 1 (d) None of these a 2 b2
y x
at any point P meets the coordinate axes at the
Numerical Value Type Questions points A and B respectively. If the rectangle
21. The equation of one of the latusrectum of the OACB (O being the origin) is completed, where C
2 2
hyperbola (10 x - 5) + (10 y - 2) = 9 (3x + 4 y - 7) 2
a 2 b2
lies on 2 - 2 = l. Then, the value of l is
is ax + by + c = 0, then the value of|b + 2 c|is x y
……… .
Answers
Round I
1. (c) 2. (d) 3. (a) 4. (c) 5. (c) 6. (d) 7. (a) 8. (b) 9. (c) 10. (c)
11. (b) 12. (d) 13. (a) 14. (a) 15. (b) 16. (b) 17. (c) 18. (b) 19. (d) 20. (b)
21. (a) 22. (b) 23. (c) 24. (a) 25. (b) 26. (b) 27. (d) 28. (a) 29. (d) 30. (b)
31. (d) 32. (a) 33. (d) 34. (a) 35. (b) 36. (c) 37. (a) 38. (c) 39. (d) 40. (b)
41. (d)

Round II
1. (d) 2. (b) 3. (c) 4. (b) 5. (b) 6. (b) 7. (b) 8. (c) 9. (a) 10. (c)
11. (d) 12. (c) 13. (c) 14. (a) 15. (a) 16. (c) 17. (c) 18. (d) 19. (d) 20. (a)
21. (6) 22. (1) 23. (2) 24. (2) 25. (1)
Solutions
Round I Þ 4(5 e2 - e4 - 5 + e2) = 15
1. Given equation of hyperbola is Þ 4 e4 - 24 e2 + 35 = 0
(10x - 5)2 + (10 y - 4)2 = l2(3x + 4 y - 1)2 x2 y2
3. We know that in - = 1, where b2 = a 2(e2 - 1), the
can be rewritten as a 2 b2
2 2 length of conjugate axis is 2b and distance between the
æ 1ö æ 2ö
çx - ÷ + ç y - ÷ foci is 2ae.
è 2ø è 5ø l
= \ According the problem, 2b = 5 and 2ae = 13
3x + 4 y - 1 2
Now, b2 = a 2(e2 - 1)
5 2
PS æ5ö 2 2 2
This is of the form of =e Þ ç ÷ =a e -a
è2ø
PM
where, P is any point on the hyperbola and S is a focus 25 (2ae)2
Þ = - a2
and M is the point of directrix. 4 4
l 25 169
Here, > 1 Þ |l|> 2 (Q e > 1) = - a2 [Q 2ae = 13]
2 4 4
169 - 25 144
Þ l < -2 or l >2 Þ a2 = = = 36 Þ a = 6
4 4
2. Let the equation of hyperbola be 13
Now, 2ae = 13 Þ 2 ´ 6 ´ e = 13 Þ e =
x2 y 2 12
- =1 …(i)
a 2 b2 4. Now, taking option (c).
Since, equation of given directrix is 5x = 4 5 et + e- t 2x
æaö a Let x=a Þ = et + e- t …(i)
So, 5ç ÷ = 4 5 [Q equation of directrix is x = ] 2 a
è eø e 2y
a 4 and = et - e-t …(ii)
Þ = …(ii) a
e 5 On squaring and subtracting Eq. (ii) from Eq. (i), we
and hyperbola (i) passes through point (4, - 2 3 ) get
so,
16 12
- =1 …(iii) 4 x2 4 y2 x2 y 2
- 2 =4 Þ - =1
a 2 b2 a 2
b a 2 b2
b2 5. Put the value of (x, y) º (tan q + sin q, tan q - sin q) in the
The eccentricity e = 1 +
a2 given option, we get the required result.
b2 On putting the value of x and y in option (c), we get
Þ e2 = 1 + 2 Þ a 2e2 - a 2 = b2 …(iv)
a [(tan q + sin q)2 - (tan q - sin q)2]2
From Eqs. (ii) and (iv), we get = 16(tan q + sin q) ´ (tan q - sin q)
16 4 16 2
e - e = b2 …(v) Þ [tan 2 q + sin 2 q - tan 2 q - sin 2 q + 4 tan q sin q]2
5 5
= 16(tan 2 q - sin 2 q)
From Eqs. (ii) and (iii), we get
16 12 5 12 Þ (4 tan q × sin q) = 16 ( tan q - sin 2 q)
2 2
- =1 Þ 2 - 2 =1
16 2 b2 e b Þ 16 tan 2 q sin 2 q = 16 tan 2 q(1 - cos 2 q)
e
5 Þ 16 tan 2 q sin 2 q = 16 tan 2 q sin 2 q
12 5 12 5 - e2 Hence, the option (c) satisfies.
Þ = - 1 Þ =
b2 e2 b2 e2
2
6. We have, 16(x2 - 2x) - 3( y2 - 4 y) = 44
12e
Þ b2 = …(vi) Þ 16(x - 1)2 - 3( y - 2)2 = 48
5 - e2
(x - 1 )2 ( y - 2)2
From Eqs. (v) and (vi), we get Þ - =1
3 16
æ 12e2 ö
16 e4 - 16 e2 = 5ç ÷ This equation represents a hyperbola with eccentricity
è 5 - e2 ø
16 19
e= 1+ =
Þ 16 (e2 - 1)(5 - e2) = 60 3 3
Hyperbola 425

7. The given equation may be written as 12. Given, S (6,4) and S ¢ (-4,4) and eccentricity, e = 2
x2 y 2 x2 y2 \ SS¢ = (6 + 4)2 + (4 - 4)2 = 10
- =1 Þ 2
- =1
32 8 æ4 2 ö (2 2 )2
3 ç ÷ But SS ¢ = 2 ae
è 3 ø 5
\ 2 a ´ 2 = 10 Þ a =
On comparing the given equation with the standard 2
x2 y2 and we know that,
equation 2 - 2 = 1, we get
a b 25 75
b2 = a 2(e2 - 1) Þ b2 = (4 - 1) =
2 4 4
æ4 2 ö
a2 = ç ÷ and b2 = (2 2 )2 é 6 + (-4) 4 + 4 ù
è 3 ø Centre of hyperbola is ê , = (1,4)
ë 2 2 úû
\ Length of transverse axis of a hyperbola (x - 1 )2 ( y - 4)2
4 2 8 2 \ Equation of hyperbola is - =1
=2a =2´ = 25 75
3 3 4 4
x2 y2 4(x - 1)2 4( y - 4)2
8. Let equation of hyperbola be - =1 Þ - =1
a 2 b2 25 75
Length of transverse axis is 2a and x2 y2
13. Let equation of hyperbola be - =1 …(i)
2b2 a 2 b2
Length of latusrectum is .
a According to the given condition,
2b2 2 2ae = 16 and e = 2
Now, difference E = 2a - = 2a 2 - a 2e2|
a a 8
Þ 2a( 2 ) = 16 Þ a = Þ a 2 = 32
\ Difference = 2a |2 - e2| 2
x2 y2 Q b2 = a 2(e2 - 1 ) = 32(2 - 1) = 32
9. Given equation of hyperbola is - = 1 and equation
a 2 b2 On putting the values of a 2 and b2 in Eq. (i), we get
2 2
x y
of conjugate hyperbola is 2 - 2 = 1 x2 y 2
- =1
b a 32 32
Since, e and e¢ are the eccentricities of the respective Þ x2 - y2 = 32
hyperbola, then
14. Let the equation of hyperbola be
b2 a2
e2 = 1 +, (e¢ ) 2
= 1 + x2 y2
a2 b2 - =1 …(i)
2 2 a 2 b2
1 1 a b
\ + = + =1 3
e2 e¢2 a 2 + b2 a 2 + b2 Given, e =and foci = (± ae, 0) = (±2, 0)
2
10. (x - 3)2 + ( y + 1)2 = (4x + 3 y)2 3
\ e = and ae = 2
2 2
æ 4x + 3 y ö
Þ (x - 3)2 + ( y + 1)2 = 25 ç ÷ 3 16
è 5 ø Þ a ´ = 2 Þ a2 =
2 9
Þ PS = 5PM
Q b2 = a 2(e2 - 1 )
Þ Directrix is 4x + 3 y = 0 and focus (3, - 1). 16 æ 9 ö 16 5 20
3 \ b2 = ç - 1÷ = ´ =
So, equation of transverse axis is y + 1 = (x - 3) 9 è4 ø 9 4 9
4
On putting the values of a 2 and b2 in Eq. (i), we get
Þ 3x - 4 y = 13
x2 y2 x2 y2 4
x2 y 2 - =1 Þ - =
11. Let the equation of the hyperbola be - =1 16 / 9 20 / 9 4 5 9
a 2 b2
The coordinates of the foci are (ae, 0) and (-ae, 0). 15. The centre of hyperbola is the mid-point of vertices.
\ 2ae = 16 Þ 2a 2 = 16 Þ a = 4 2 æ 0 + 10 0 + 0 ö
So, coordinates of centre are ç , ÷ , i.e. (5, 0).
è 2 2 ø
Also, b2 = a 2(e2 - 1) = 32(2 - 1) = 32
Thus, a 2 = 32 and b2 = 32 One of the foci is (18, 0).
Hence, the required equation is \ Other foci of hyperbola is (-8, 0).
x2 y2 [Q centre of hyperbola is also mid-point of foci of
- = 1 Þ x2 - y2 = 32
32 32 hyperbola]
426 JEE Main Mathematics

Let 2a and 2b be the lengths of transverse and 18. We have, transverse axis = x + 2 y - 3 = 0 and
conjugate axes and e be the eccentricity. conjugate axis = 2x - y + 4 = 0 both are perpendicular,
2
Then, the equation of hyperbola is and 2a = 2 and 2b =
3
(x - 5)2 y2
- 2 =1 1 1
a2 b Þ a= and b =
2 3
Distance between two vertices = 2a
i.e. 2a = (10 - 0)2 + (0 - 0)2 = 10 N P
Þ a =5
Now, distance between two foci = 2ae
O M
i.e. 2ae = (18 + 8)2 + (0 - 0)2 = 26
26
Þ ae = = 13
2
In hyperbola, (ae)2 = a 2 + b2 We know that,
Þ b2 = (ae)2 - a 2 Equation of the hyperbola referred to two
Þ b2 = (13)2 - (5)2 = 144 perpendicular lines,
PN 2 PM 2
\ Equation of hyperbola is i.e. - =1
a2 b2
(x - 5)2 y2
- =1 2 2
25 144 æ 2x - y + 4 ö æ x + 2y - 3ö
ç ÷ ç ÷
è 5 ø è 5 ø
16. We have, x = t 2 + 1 and y = 2t Þ - =1
1 1
On eliminating t, we get 2 3
y2 = 4x - 4 \
2 3
(2x - y + 4) - (x + 2 y - 3)2 = 1
2
2 5 5
and x = 2s and y =
s x2 y2
2 19. Given equation of hyperbola is - =1.
On substituting this y = 4x - 4, we get 9 4
2s3 - s2 - 1 = 0 Here, a 2 = 9, b2 = 4
2
Þ (s - 1) (2s + s + 1) = 0 and equation of line is y = - x + 2 p ...(i)
Þ s=1 If the line y = mx + c touches the hyperbola
2 x2 y2
Putting s = 1 in x = 2s, y = , we get - = 1, then c2 = a 2m2 - b2 …(ii)
s
a 2 b2
x = 2, y = 2
From Eq. (i), we get
So, point is (2, 2).
m = -1 , c = 2 p
17. We have, 9x2 - 16 y2 - 18x + 32 y - 151 = 0 On putting these values in Eq. (ii), we get
Þ 9 (x2 - 2x + 1) - 16 ( y2 - 2 y + 1) = 151 + 9 - 16 ( 2 p)2 = 9(1) - 4 Þ 2 p2 = 5
Þ 9 (x - 1)2 - 16 ( y - 1)2 = 144 x2 y 2
20. Let equation of tangent to hyperbola - = 1 is
(x - 1)2 ( y - 1)2 a 2 b2
Þ - =1
16 9 y = mx + a 2m2 - b2
(a) Length of transverse axes = 2a = 2 ´ 4 = 8
i.e. mx - y + a 2m2 - b2 = 0
2b2 2 ´ 9 9
(b) Length of latusrectum = = =
a 4 2 \Required product
a
(c) Equation of directrix is x = ± + 1 ½mae + a 2m2 - b2½½- mae + a 2m2 - b2½
e =½ ½½ ½
½ m2 + 1 ½½ m2 + 1 ½
4 é a 2 + b2 5 ù
Þ x = ± ´ 4 + 1 êQ e = = ú ½a 2m2 - b2 - m2a 2e2½
5 êë a2 4ú
û =½ ½
½ m2 + 1 ½
21
\ x= ½m2a 2 (1 - e2) - b2½ ½- m2b2 - b2½
5 =½ ½= ½ ½ [Q b2 = a 2(e2 - 1)]
2 2
11 ½ m + 1 ½ ½ m + 1 ½
and x=-
5 = b2
Hyperbola 427

21. Equation of given hyperbola is On squaring and subtracting Eq. (ii) from Eq. (i), we get
x 2
y 2 ( y - cmx)2 - (my - cx)2 = a 2c2m2 - b2 - a 2c2 + b2m2
- 2 =1 …(i)
a 2
b Þ (1 - m2) ( y2 - c2x2) = - (1 - m2)(a 2c2 + b2)
Since, P(3, 3), be a point on Eq. (i). Þ y2 + b2 = c2(x2 - a 2)
9 9 24. Equation of normal to the hyperbola at the point
So, - =1 …(ii)
a 2 b2 (5 sec q,4 tan q) is
Now, equation of normal at point P to the hyperbola is 5x cos q + 4 y cot q = 25 + 16 ...(i)
a 2x b2y This line is perpendicular to the line 2 x + y = 1
+ = a 2 + b2 …(iii)
3 3
\ m1m2 = -1
The normal Eq. (iii) intersect the X-axis at (9, 0), so
æ -5 cos q ö 2
3a 2 = a 2 + b2 Þ b2 = 2a 2 …(iv) Þ ç ÷ (-2) = -1 Þ sin q = -
è 4 cot q ø 5
Q The eccentricity (e) of hyperbola (i) is
4 21 21
b2 \ cos q = 1 - =m and cot q = m
e= 1+ = 1+2= 3 Þ e2 = 3 25 5 2
a2
From Eq. (i),
From Eqs. (ii) and (iv), we get
21 4 y 21
9 9 9 5x - = 41 Þ 21 (x - 2 y) = 41
- = 1 Þ 18 - 9 = 2a 2 Þ a 2 = 5 2
a 2 2a 2 2
æ 9 ö 25. Let P (h , k) be the point of intersection of tangents
\The ordered pair (a 2, e2) = ç , 3÷ .
è2 ø
Þ y = mx ± a 2m2 - b2 passes through (h , k)
x2 y 2
22. Let the equation of hyperbola be 2 - 2 = 1. Þ k - mh = ± a 2m2 - b2
a b
\ ae = 2 Þ a 2e2 = 4 Þ (k - mh )2 = a 2m2 - b2
Þ a + b2 = 4 Þ b2 = 4 - a 2
2 Þ k2 + m2h 2 - 2khm = a 2m2 - b2
x2 y2 Þ m2(h 2 - a 2) - 2khm + (k2 + b2) = 0
\ - =1
a 2
4 - a2 Given, m1m2 = c2
k2 + b2
Since, ( 2 , 3 ) lie on hyperbola. Þ = c2
2 3 h2 - a 2
\ - =1
a2 4 - a2 Þ k2 + b2 = c2(h 2 - a 2)
Þ 8 - 2a 2 - 3a 2 = a 2(4 - a 2) \Locus of (h , k) is
y2 + b2 = c2(x2 - a 2)
Þ 8 - 5a 2 = 4a 2 - a 4
Þ a 4 - 9a 2 + 8 = 0 26. Tangent at (a sec f , b tan f ) on the I st hyperbola is
Þ 4 4
(a - 8)(a - 1) = 0 x y
sec f - tan f = 1 ...(i)
4 4 a b
Þ a = 8, a = 1
Similarly, tangent at any point (b tan q, a sec q) on
\ a =1
IInd hyperbola is
x 2 y2 y x
Now, equation of hyperbola is - = 1. sec q - tan q = 1 ...(ii)
1 3 a b
\ Equation of tangent at ( 2 , 3 ) is given by Since, Eqs. (i) and (ii) are common tangents, then they
should be identical.
3y y sec q tan f
2x - =1 Þ 2x - =1 Þ =-
3 3 a b
which passes through the point (2 2 , 3 3 ). a
Þ sec q = - tan f ...(iii)
23. Let the slopes of the two tangents to the hyperbola b
tan q sec f
x2 y 2 c Þ - =
- = 1 be cm and b a
a 2 b2 m b
Þ tan q = - sec f ...(iv)
The equations of tangents are a
y = cmx + a 2c2m2 - b2 ...(i) Q sec2 q - tan 2 q = 1
a2 b2
and my - cx = a 2c2 - b2m2 ...(ii) Þ 2 tan 2 f - 2 sec2 f = 1 [from Eqs. (iii) and (iv)]
b a
428 JEE Main Mathematics

a2 2 b2 The equation of the chords of contact of tangents from


Þ tan f - (1 + tan 2 f ) = 1
b2 a2 (x1 , y1 ) and (x2, y2) to the given hyperbola are
æ a 2 b2 ö b2 x x1 y y1
Þ ç 2 - 2 ÷ tan 2 f = 1 + 2 - =1 ...(ii)
èb a ø a 9 4
x x2 y y2
b2 and - =1 ...(iii)
Þ tan 2 f = 9 4
a - b2
2

Lines (ii) and (iii) are at right angles.


2 a2
2
\ sec f = 1 + tan f = 2 9 x1 4 x2
a - b2 × ´ × = -1
4 y1 9 y2
Now, substituting sec f and tan f in Eq. (i), we get 2
x y x1x2 æ9ö 81
± =1 Þ = -ç ÷ = -
m
y1 y2 è4ø 16
a 2 - b2 a 2 - b2

\ xm y=± a 2 - b2 or y = ± x ± a 2 - b2 30. Let (h , k) be the point whose chord of contact w.r.t.


hyperbola x2 - y2 = 9 is x = 9. We know that chord of (h , k)
27. Equation of the tangents at P (a sec q, b tan q) is w.r.t. hyperbola x2 - y2 = 9 is T = 0
x y Þ hx - ky - 9 = 0
sec q - tan q = 1
a b
But it is the equation of line x = 9. This is possible only
\ Equation of the normal at P is when h = 1, k = 0.
ax + b cosec q y = (a 2 + b2)sec q ...(i) Again, equation of pair of tangents is
Similarly, the equation of normal at Q (a sec f , b tan f ) T 2 = SS1
is
Þ (x - 9)2 = (x2 - y2 - 9)(1 - 9)
ax + b cosec f y = (a 2 + b2)sec f ...(ii)
Þ x - 18x + 81 = (x2 - y2 - 9)(-8)
2
On subtracting Eq. (ii) from Eq. (i), we get
Þ 9x2 - 8 y2 - 18x + 9 = 0
a 2 + b2 sec q - sec f
y= × 31. Equation of chord of hyperbola x2 - y2 = a 2 with
b cosec q - cosec f
mid-point as (h , k) is given by
æp ö xh - yk = h 2 - k2
sec q - sec ç - q÷
a 2 + b2 è2 ø h (h 2 - k2)
So, k= y= × Þ y = x-
b æp ö
cosec q - cosec ç - q÷ k k
è2 ø
This will touch the parabola y2 = 4ax, if
a 2 + b2 sec q - cosec q æ h 2 - k2 ö
= × a
b cosec q - sec q -ç ÷=
è k ø h /k
é a 2 + b2 ù Þ ak2 = - h3 + k2h
= -ê ú
ë b û \ Locus of the mid-point is x 3 = y2(x - a )
25 3 32. Let (h , k) is mid-point of chord.
28. Given that, y = mx + ...(i)
3 Then, its equation is
2 2
x y 3hx - 2 ky + 2(x + h ) - 3( y + k) = 3h 2 - 2 k2 + 4h - 6k
and - =1 ...(ii)
16 9 Þ x(3h + 2) + y(-2 k - 3) = 3h 2 - 2 k2 + 2 h - 3k
Here, Eq. (i) is normal to Eq. (ii), then Since, this line is parallel to y = 2x.
(a 2 + b2)2 a 2 b2 3h + 2
= 2- \ =2
c2 m 1 2k+3
(16 + 9)2 ´ 9 16 9 Þ 3h + 2 = 4k + 6
Þ = 2-
625 ´ 3 m 1 Þ 3h - 4k = 4
16 2 Thus, locus of mid-point is 3x - 4 y = 4
Þ 2
= 12 Þ m = ±
m 3 33. We know that asymptotes of rectangular hyperbola are
mutually perpendicular, thus other asymptote should
29. The equation of hyperbola is be 4x + 3 y + l = 0
4x2 - 9 y2 = 36
Also, intersection point of asymptotes is also the centre
x2 y2 of the hyperbola.
Þ - =1 ...(i)
9 4
Hyperbola 429

Hence, intersection point of 4x + 3 y + l = 0 and 39. Q Tangent and normal are at 90°.
æ 18 - 4l -12l - 96 ö
3x - 4 y - 6 = 0 is ç , ÷ and it should lie Y
è 25 100 ø
on the line x - y - 1 = 0 (0, b1)
18 - 4l -12l - 96
\ - - 1 = 0 Þ l = 17
25 100
X¢ X
Hence, equation of other asymptote is 4x + 3 y + 17 = 0 O (a2, 0) (a1, 0)
34. Given that, xy = hx + ky Þ (x - k)( y - h ) = hk
On shifting origin to (k, h ) the above equation reduces
to (0, –b2)
XY = hk = c2 (say) Y¢
where, x = X + k and y = Y + h \ Product of slopes is -1.
Then, the equation of the asymptotes are X = 0 andY = 0 æ b1 ö æ b2 ö
Þ ç - ÷ ç - ÷ = -1 Þ a1a 2 + b1b2 = 0
i.e. x = k, y = h è a1 ø è a 2 ø

35. Given equation is x2 - 2 y2 - 2 = 0, it can be rewritten as 40. The given equation of rectangular hyperbola is
x 2
y 2
xy = 18 ...(i)
- =1
2 1 On comparing Eq. (i), with general equation of
Here, a 2 = 2 , b2 = 1 a2
rectangular hyperbola xy = , we get
x2 y 2 2
We know that equation of hyperbola is - = 1, then a2
a 2 b2 = 18 Þ a 2 = 36 Þ a = 6
the product of length of perpendicular drawn from any 2
point on the hyperbola to the asymptotes is \Length of the transverse axis of rectangular hyperbola
2 2
a b 2 (1) 2 is 2 a = 2 ´ 6 = 12
= =
a 2 + b2 2 + 1 3 41. Any point on parabola y2 = 8x is (2 t 2,4t ). The equation of
36. Equation of asymptotes are tangent at that point is
x + 2y = 3 ...(i) yt = x + 2 t 2 ...(i)
Given that, xy = -1 ...(ii)
and x- y =0 ...(ii)
On solving Eqs. (i) and (ii), we get
On solving Eqs. (i) and (ii), we get
y( yt - 2 t 2) = -1 Þ ty2 - 2 t 2y + 1 = 0
x = 1, y = 1
Q It is common tangent. It means they are intersect only
\ Centre of hyperbola is (1, 1) because asymptotes at one point and the value of discriminant is equal to
passes through the centre of the hyperbola. zero.
37. Equation of asymptotes of the hyperbola are i.e. 4t 4 - 4t = 0 Þ t = 0, 1
x2 + 2 xy - 3 y2 = 0 \ The common tangent is y = x + 2 , (when t = 0, it is x = 0
which can touch xy = -1 at infinity only)
The angle between asymptotes is
é 1 - 1(-3) ù Round II
q = tan -1 ê
ë 1 - 3 úû 1. Q PQ is the double ordinate. Let MP = MQ = l.
-1 æ 1 + 3 ö -1
= tan ç ÷ = tan (±2) Y a Öb2 + l 2, l
è -2 ø
b
P
38. We know that angle between two asymptotes of the
x2 y 2 æ bö
hyperbola - 2 = 1 is 2 tan -1 ç ÷ . X¢ O M X
a 2
b èaø

x2 y 2 Q
Equation of given hyperbola is - = 1. a Öb2 + l 2, –l
16 9 Y¢
b
Here, a = 4 and b = 3
Given that DOPQ is an equilateral, then OP = OQ = PQ
-1 æ 3 ö Þ (OP )2 = (OQ )2 = (PQ )2
\ Required angle = 2 tan ç ÷
è4ø
430 JEE Main Mathematics

a2 2 2 2 a2 2 2 4. Given hyperbola is
Þ (b + l ) + l = (b + l ) + l2 = 4l2
b2 b2 Y
a2 2 2
Þ (b + l ) = 3l2
b2 A
æ a2ö a 2b2 X¢
(5, 0)
X
Þ a 2 = l2 ç 3 - 2 ÷ Þ l2 = >0 O (3, 0)
è b ø (3b2 - a 2)
\ 3b2 - a 2 > 0 Þ 3b2 > a 2
Þ 3a 2(e2 - 1) > a 2 Þ e2 > 4 /3 Y¢
2 x2 y2
\ e> - =1 …(i)
3 9 16
2. Equation of normal at P (a sec f , b tan f ) is 16 25 5
\ e2 = 1 + = Þ e=
9 9 3
a x cos f + by cot f = a 2 + b2.
Hence, its foci are (±5, 0).
Then, coordinates of L and M are
The equation of the circle with (5, 0) as centre is
æ a 2 + b2 ö æ a 2 + b2 ö
ç × sec f ,0÷ and ç0, tan f ÷ respectively. (x - 5)2 + y2 = r 2 …(ii)
è a ø è b ø
On solving Eqs. (i) and (ii), we have
Let mid-point of ML be Q (h , k), then 16x2 - 9[r 2 - (x - 5)2] = 144
(a 2 + b2) Þ 25x2 - 90x - 9r 2 + 81 = 0
h= sec f
2a Since, the circle touches the hyperbola, above equation
2 ah must have equal roots. Hence,
\ sec f = ...(i)
(a 2 + b2) 902 - 4(25)(81 - 9r 2) = 0
2 2
(a + b ) Þ 9 - (9 - r 2) = 0
and k= tan f
2b Þ r = 0, which is not possible.
2 bk Hence, the circle cannot touch at two points.
\ tan f = ...(ii)
(a 2 + b2) It can only be tangent at the vertex. Hence,
From Eqs. (i) and (ii), we get r =5 -3 =2
4a 2h 2 4b2k2 x2 y 2 b2
sec2 f - tan 2 f = 2 - 2 5. For the hyperbola - 2 = 1, e = 1 + 2
(a + b ) (a + b2)2
2 2
a 2
b a
Hence, required locus is sin 2 q
x2 y2 \ For the given hyperbola, e = 1 + >2
- =1 cos 2 q
2 2
æ a 2 + b2 ö æ a 2 + b2 ö [Q a 2 = cos 2 q and b2 = sin 2 q]
ç ÷ ç ÷
è 2a ø è 2b ø Þ 2
1 + tan q > 4
Let eccentricity of this curve be e1. Þ tan 2 q > 3
æ a 2 + b2 ö
2
æ a 2 + b2 ö 2
2
Þ tan q Î (- ¥ , - 3 ) È ( 3 , ¥ )
Þ ç ÷ =ç ÷ (e1 - 1) [x 2
> 3 Þ|x| > 3 Þ x Î (-¥ , - 3 ) È ( 3 , ¥ )]
è 2b ø è 2a ø
æ pö æp pö
Þ a 2 = b2 (e12 - 1) But q Î ç0, ÷ Þ tan q Î ( 3 , ¥ ) Þ q Î ç , ÷
è 2ø è3 2ø
Þ a 2 = a 2 (e2 - 1)(e12 - 1) [Q b2 = a 2(e2 - 1 ) ]
2b2 sin 2 q
Þ e2e12 - e2 - e12 + 1 =1 Now, length of latusrectum = =2
a cos q
e
Þ e12(e2 - 1) = e2 Þ e1 = = 2 sin q tan q
2
(e - 1)
Since, both sin q and tan q are increasing functions in
2b2 æp pö
3. We have, = 8 and 2b = ae ç , ÷.
a è3 2ø
Þ b2 = 4a and 2b = ae \ Least value of latus rectum is
Consider, 2b = ae Þ 4b2 = a 2e2 Þ 4a 2(e2 - 1) = a 2e2 p p 3 æ pö
> 2 sin × tan = 2× × 3 =3 çat q = ÷
Þ 4e2 - 4 = e2 [Q a ¹ 0] 3 3 2 è 3ø

Þ 2
3e =4 Þ e =
2
[Q e > 0] and greatest value of latusrectum is < ¥
3 Hence, latusrectum length Î (3, ¥ ).
Hyperbola 431

6. We know that, the equation of tangent to the hyperbola æ 2 ö


and latus rectum x = 6 at R ç 6 , ( 6 - 1)÷
x 2
y 2 è 6 ø
- = 1 in slope form is
a 2 b2 1
\ Area of DQFR = ( 6 - 1) ×
2
( 6 - 1)
2 6
y = mx ± a 2m2 - b2
7
Q Equation of tangent to the hyperbola x2 - y2 = 3 is = -2
6
y = mx ± 3m2 - 3, since this tangent is parallel to the
9. Director circle of circle
line 2x + y + 8 = 0.
x2 + y2 = a 2 is x2 + y2 = 2a 2
Q m = - 2 Þ y = - 2x ± 3
The semi-transverse axis is 3a.
or 2x + y ± 3 = 0 ...(i)
Radius of the circle is 2a.
Let (x1 , y1 ) be the point of contact
Hence, director circle and hyperbola do not intersect.
xx1 - yy1 = 3 ...(ii)
Since, Eqs. (i) and (ii) are identical, then
10. Let point on the hyperbola x2 - y2 = 4 be (2 sec q, 2 tan q).
x1 y Equation of tangent at (2 sec q, 2 tan q)
=- 1 =±1
2 1 x y
sec q - tan q = 1
Þ x1 = ± 2 and y1 = ± 1 2 2
\ Point of contact = (2, - 1) X-intercept, i.e. a1 = 2 cos q
Y-intercept, i.e. b1 = - 2 cot q
7. Tangent at point P is
x y Now, the equation of normal at (2 sec q, 2 tan q) is
sec q - tan q = 1.
a b 2x cos q + 2 y cot q = a 2 + b2
Y a 2 + b2
X-intercept, i.e. a2 =
2 cos q
P(a sec q, b tan q)
a 2 + b2
T N Y-intercept, i.e. b2 =
X¢ X 2 cot q
O
a 2 + b2 æ a 2 + b2 ö
\ a1a 2 + b1b2 = 2 cos q ´ + (-2 cot q) ç ÷
2 cos q è 2 cot q ø
Y¢ = a 2 + b2 - a 2 - b2 = 0
It meets the X-axis at point T (a cos q, 0) and foot of
11.
perpendicular from P to X-axis is N (a sec q, 0). x2 + y2 = 25
From the diagram, we have
OT = a cos q
and ON = a sec q Þ OT ´ ON = a 2 (0, 0)

8. Y
(h, k)
P(4, Ö6)

R Equation of chord
h
O X y-k=- (x - h )
Q k
F(Ö6, 0)
Þ ky - k2 = - hx + h 2
x = Ö6 Þ hx + ky = h 2 + k2
hx h 2 + k2
x2 y2 Þ y=- +
- =1 k k
4 2
x2 y2
b 2
3 Tangent to - =1
e= 1+ = 9 16
a2 2
c2 = a 2m2 - b2
Focus F (ae, 0) Þ F ( 6 , 0) æ h 2 + k2 ö 2
æ hö
Equation of tangent at P to the hyperbola is Þ ç ÷ = 9 ç - ÷ - 16
è k ø è kø
2x - y 6 = 2
Tangent meet X-axis at Q (1, 0) Þ (x2 + y2)2 = 9x2 - 16 y2
432 JEE Main Mathematics

12. Equation of given parabola y2 = 12x … (i) Þ 9x + 3 y - 8 = 0


and 9x + 3 y + 8 = 0
and hyperbola 8x2 - y2 = 8 … (ii)
x2 y2
Now, equation of tangent to parabola y2 = 12x having 14. Given equation ot hyperbola is 2 - 2 = 1.
a b
3
slope ‘m’ is y = mx + … (iii) Its vertices are A (a , 0) and A ¢ = (- a , 0)
m
Normal at any point q i.e. (a sec q, b tan q) is
and equation of tangent to hyperbola
ax cos q + by cot q = a 2 + b2
x 2 y2
- = 1 having slope ‘m’ is Now, G is it intersection on transverse axis i.e. X-axis.
1 8
æ (a 2 + b2) sec q ö
y = mx ± 12m2 - 8 …(iv) \ G=ç , 0÷
è a ø
Since, tangents (iii) and (iv) represent the same line
2 (a 2 + b2) sec q
2 æ3ö Þ AG = a -
\ m -8 = ç ÷ a
èmø
(a 2 + b2) sec q
Þ m4 - 8m2 - 9 = 0 Þ A¢ G = - a -
a
Þ (m2 - 9) (m2 + 1) = 0 (a + b2)2 × sec2 q
2

Þ m=±3 \ AG ´ A ¢ G = a 2 -
a2
Now, equation of common tangents to the parabola (i) a e × sec2 q
4 4
and hyperbola (ii) are = a2 - [Q a 2 + b2 = a 2e2]
a2
y = 3x + 1 and y = - 3x - 1
= a 2 - a 2e4 × sec2 q
Q Point ‘P’ is point of intersection of above common
tangents, = a 2(1 - e4 sec2 q)
\ P(- 1 / 3, 0) = a 2(e4 sec2 q - 1)
a
and focus of hyperbola S(3, 0) and S¢ (- 3, 0). 15. We know that, y = mx + is the equation of tangent to
m
PS 3 + 1 / 3 10 5 the parabola y2 = 4ax.
Thus, the required ratio = = = =
PS ¢ 3 - 1 / 3 8 4 1
\ y = mx + is a tangent to the parabola
13. The equation of tangent in terms of slope of y2 = 32 x is m
8 y2 = 4 x [Q a = 1]
y = mx + ...(i)
m Let, this tangent is also a tangent to the hyperbola
xy = 2
which is also tangent of the hyperbola 9x2 - 9 y2 = 8
1
8 Now, on substituting y = mx + in xy = 2, we get
i.e., x 2 - y2 = m
9
æ 1ö
æ8ö
2
8 2 8 x çmx + ÷ = 2 Þ m2x2 + x - 2m = 0
Then, ç ÷ = m - è mø
èmø 9 9
Note that tangent touch the curve exactly at one point,
8 m2 1 therefore both roots of above equations are equal.
Þ = -
m2 9 9 Þ D = 0 Þ 1 = 4(m2) (- 2m)
3
Þ 72 = m4 - m2 æ 1ö 1
Þ m3 = ç - ÷ Þ m=-
è 2ø 2
Þ m4 - m2 - 72 = 0
\Required equation of tangent is
Þ (m2 - 9)(m2 + 8) = 0
x
Þ m2 = 9, [Qm2 + 8 =/ 0] y = - - 2 Þ 2y = - x - 4 Þ x + 2y + 4 = 0
2
Þ m = ±3
16. The equation of normal at æç ct , ö÷ to the hyperbola
c
From Eq. (i), we get è tø
8 2
xy = c is
y = ± 3x ±
3
xt3 - yt - ct 4 + c = 0 ...(i)
Þ 3 y = ±9 x ± 8 æ cö
Þ ±9 x - 3 y ± 8 = 0 Since, Eq. (i) intersect at hyperbola ç ct1 , ÷ .
è t1 ø
Þ 9x - 3 y + 8 = 0, 9x - 3 y - 8 = 0
c
-9 x - 3 y + 8 = 0 , -9 x - 3 y - 8 = 0 ct1t3 - t - ct 4 + c = 0
t1
or 9x - 3 y + 8 = 0, 9x - 3 y - 8 = 0
Hyperbola 433

t 19. It is given that e1 is the eccentricity of the ellipse,


Þ + t4 - 1
t3 t1 =
t1
x2 y2 4 14 7
Þ t3 t12 - t - t 4 t1 + t1 = 0 + = 1,so e1 = 1 – = =
18 4 18 18 9
Þ (t3 t12 - t 4 t1 ) + (t1 - t ) = 0 and e2 is the eccentricity of the hyperbola,
Þ t3 t1 (t1 - t ) + 1(t1 - t ) = 0 x2 y2 4 13
3 - = 1, so e2 = 1 + =
Þ (t1 - t ) (t t1 + 1) = 0 9 4 9 9
Þ t1 ¹ t æ 7 13 ö
Now, as (e1 , e2) = ç , ÷ is a point on the ellipse,
or 3
t t1 + 1 = 0 è 9 9 ø
\ t3 t1 = - 1 15x2 + 3 y2 = k, then
æ 7ö æ 13 ö 105 + 39
17. Let equation of hyperbola be 15 ç ÷ + 3 ç ÷ = k Þ =k
è9ø è9ø 9
x2 y 2 144
- =1 Þ = k Þ k = 16
a 2 b2 9
where, 2ae = 4 and e = 2 x y
20. The equation of tangent is sec q - tan q = 1
Y a b
\Coordinates of A and B are (a cos q, 0) and (0, - b cot q)
respectively.
X¢ X Let coordinates of P be (h , k).
(–2, 0) O (2, 0)
\ h = a cos q, k = - b cot q
k b bh
Þ =- Þ sin q = -
Y¢ h a sin q ak
Þ a =1 b2h 2 b 2 2
h h 2
Þ = sin 2 q Þ 2 2 + 2 = 1
a 2k2 a k a
a 2e2 = a 2 + b2
b2 a2 a 2 b2
Þ 4 = 1 + b2 Þ +1 = 2 Þ 2 - 2 =1
k2 h h k
\ b2 = 3 a 2
b 2
Hence, the locus of P is 2 - 2 = 1
Thus, equation of hyperbola is x y
x2 y2
- =1 21. We have,
1 3
(10x - 5)2 + (10 y - 2)2 = 9 (3x + 4 y - 7)2
Þ 3 x2 - y 2 = 3 2 2
æ 1ö æ 1ö 9 (3x + 4 y - 7)2 ´ 25
18. Equation of given çx - ÷ + ç y - ÷ =
è 2 ø è 5 ø 100 25
x2 y2 2 2 2 2
Hyperbola : x2 - y2 sec 2 q = 10 Þ - = 1 ...(i) æ 1ö æ 1ö æ 3 ö æ 3x + 4 y - 7 ö
10 10 cos 2 q çx - ÷ + ç y - ÷ = ç ÷ ç ÷
è 2ø è 5ø è2ø è 5 ø
\Eccentricity of hyperbola (i) is
æ1 1ö
From this equation, foci of hyperbola is ç , ÷
e1 = 1 + cos 2 q è2 5ø
x2 y2 3
and Ellipse : x2 sec2 q + y2 = 5 = 2
+ =1 …(ii) eccentricity of hyperbola =
5 cos q 5 2
\Eccentricity of ellipse (ii) is e2 = 1 - cos 2 q equation of directrix is 3x + 4 y - 7 = 0
equation of latusrectum is 3x + 4 y = l.
It is given that
[Q latusrectum is parallel to directrix]
e1 = 5 e2
Since, latusrectum is passes through the foci
Þ 1 + cos 2 q = 5(1 - cos 2 q) 3 4
2 i.e. + =l
Þ 6 cos 2 q = 4 Þ cos 2 q = 2 5
3 23
Þ l=
2 (5 cos 2 q) 10
\Length of latus rectum of ellipse (i) =
5 \ Equation of latusrectum is
æ2ö 4 5 30x + 40 y - 23 = 0
= 2 5ç ÷ =
è3ø 3 Now, | b + 2c| = |40 - 46| = 6
434 JEE Main Mathematics

22. Foci of the hyperbola are (5, 12) and (24, 7). (a 2b2 + yka 2)
Þ x=
Let P (x, y) be any point on the hyperbola and S and S¢ b2h
be the foci of parabola. This tangent cuts the circle x + y2 = a 2
2

By definition, Putting the value of x in equation of circle, we get


2
| PS - PS ¢| = Length of transverse axis æ a 2b2 + yka 2 ö
ç ÷ + y2 = a 2
\ | (x - 5)2 + ( y - 12)2 - (x - 24)2 + ( y - 7)2 | = 2a è b2h ø
Since, hyperbola is passing through the origin. a 4b4 + 2 yka 4b4 + y2k2a 4 + y2b4h 2 = a 2b4h 2
\ 2a = | 25 + 144 - 5 + 6 + 49 | Þ (k a + b h ) y2 + 2 y ka 4b2 + a 4b4 - a 2b4h 2 = 0
2 4 4 2

= |13 - 25|= 12 Now, sum of roots, i.e.


Now, distance between two foci = 2ae - 2ka 4b2
y1 + y2 = 2 4
i.e. (5 - 24)2 + (12 - 7)2 = 2ae k a + b4h 2

192 + 52 = 2ae and product of roots i.e.


a 4b4 - a 2b4h 2
Þ 368 = 12e y1 y2 = 2 4
k a + b4h 2
12e
\ =1 1 1 y + y2
368 Now, + = 1
y1 y2 y1 y2
23. Let the length of each side of the equilateral DOPQ be l
- 2ka 4b2 - 2ka 4b2
æ 3 l lö = =
units. Then, the coordinates of P are ç , ÷. a 4b4 - a 2b4h 2 a 2b4 (a 2 - h 2)
è 2 2ø
2ka 2 - 2k
=- =
æ 3 l lö x y 2 2
b (a 2 - h 2)
2
æ h2 ö
Point P ç , ÷ lies on the hyperbola 2 - 2 = 1 b2ç1 - 2 ÷
è 2 2ø a b è a ø
3 l2 l2 2k 2 é h 2 k2 ù
\ - =1 =- = êQ 2 - 2 = 1ú
4a 2 4b2 é k ù k ë a
2
a û
b2ê - 2 ú
Þ (3b2 - a 2)l2 = 4a 2b2
ë b û
Þ (3 e2 - 4)l2 = 4a 2(e2 - 1)
\ l =2
e2 - 1
Þ l = 2a 25. Let the tangent of point P (a sec q, b tan q) on the
3 e2 - 4
x2 y2 x sec q y tan q
Since, l is real and e > 1. hyperbola - = 1 be - = 1.
a 2 b2 a b
4
\ e2 > This line meets the coordinate axes at A and B
3
2 \Coordinates of point A (a cos q, 0) and B(0, - b cot q).
Þ e> = 3 e>2
3 Since, OACB is formed a rectangle.
Þ k =2 \ C is (a cos q, - b cot q)
a 2 b2
24. Equation of tangent at point (h , k) on the hyperbola is Now, C lies on - =l
x2 y2
x2 y 2
- = 1 is a2 b2
a 2 b2 Then, - =l
a 2 cos 2 q b2 cot2 q
xh yk
- =1 Þ sec2 q - tan 2 q = l
a 2 b2
\ l =1
17
Introduction to Three
Dimensional Geometry

In geometry, we can define a three-dimensional shape as a solid figure or an IN THIS CHAPTER ....
object or shape having three-dimensions. They have length, width and height.
Coordinate Axes and Coordinate
Always remember that unlike two dimensional shapes, three-dimensional
Planes in Three Dimensional
shapes consists of thickness or depth. In three dimensional space, three
Space
parameters are required to find the exact location of a point.
Distance between Two Points

Coordinate Axes and Coordinate Planes Section Formulae

in Three Dimensional Space Area of a Triangle

Let X ¢OX ,Y ¢OY and Z ¢OZ be three mutually perpendicular lines intersecting Circles Associated with a Triangle
at point O. O is the origin and the lines X ¢OX , Y ¢OY and ZOZ ¢ are called
X-axis, Y-axis and Z-axis, respectively. These three lines are also called the
rectangular axes of coordinates. The planes containing these three lines in
pairs, determine three mutually perpendicular planes XOY ,YOZ and ZOX.
Z
y=0

Y¢ x=0
z=0 O
Origin (0, 0)

Y
X


The three planes divide space into eight cells called octants.
436 JEE Main Mathematics

The following table show the signs of coordinates of Sol. (b)


points in various octants. Point Octant
Octant coordinate Octant x y z (1, 2, 3) I (all the coordinates are positive)
( 4,-2, 3) IV ( y-coordinate is negative)
OXYZ I + + +
( 4,-2,-5) VIII ( y and z-coordinates are negative)
OX ¢ YZ II – + + ( 4,2, - 5) V (z-coordinate is negative)
OX ¢ Y ¢ Z III – – +
OXY ¢ Z IV + – + Distance between Two Points
OXYZ ¢ V + + – The distance between two points P ( x1 , y1 , z1 ) and
OX ¢ YZ ¢ VI - + – Q( x2 , y2 , z 2 ) is given by
OX ¢ Y ¢ Z ¢ VII - – –
PQ = ( x2 - x1 )2 + ( y2 - y1 )2 + ( z 2 - z1 )2
OXY ¢ Z ¢ VIII + – –
z
Coordinates of a Point in Space P(x1, y1, z1)

The cartesian coordinates ( x , y , z ) of a point P in a space Q(x2, y2, z2)


are the numbers at which the planes through P is y
O
perpendicular to the axes.
The coordinates of a point on X-axis are ( x , 0, 0), on Y-axis x
are ( 0, y , 0) and on Z-axis are ( 0, 0, z ) .
Important Points
The standard equations of xy-plane, yz-plane and
zx-plane are z = 0, x = 0 and y = 0, respectively. ● The distance of a point P ( x , y , z ) from the origin is
Y x2 + y2 + z 2 .
● The distance of a point P ( x , y , z ) from X-axis is y 2 + z 2 .
(0, y, 0)
(x, y, 0) ● The distance of a point P ( x , y , z )from Y-axis is x 2 + z 2 .
(0, y, z) P
(x, y, z) ● The distance of a point P ( x , y , z ) from Z-axis is x 2 + y 2 .
● If P ( x , y , z ) is a point in a space, then distance from yz,
X
O (x, 0, 0) zx and xy-planes are x, y and z, respectively.
(0, 0, z) (x, 0, z)
Z Example 3. Find the coordinates of a point on Y-axis which
are at a distance of 5 2 from the point P(3, - 2, 5).
Consider a point P in space whose position is given by
(a) (0 ,-6,0) or (0 , 2, 0) (b) (0 ,6,0) or (0 ,-2, 0)
(x,y,z) where, x, y,z are perpendicular distance from
(c) (0 ,-6, 0) or (1, 2, 0) (d) None of these
YZ-plane, ZX-plane and XY-plane respectively.
If we assume, $i, $j , k$ unit vectors along OX, OY, OZ Sol. (a) Let any point on the Y-axis is A(0 , y , 0).
respectively, then position vector of point P is x i$ + y $j + zk$ Given, distance between P a n d A, PA = 5 2
or simply (x, y, z). Þ (3 - 0) 2 + ( - 2 - y) 2 + (5 - 0) 2 = 5 2
Example 1. A plane is parallel to yz-plane so it is On squaring both sides, we get
perpendicular to (3 - 0) 2 + ( - 2 - y) 2 + (5 - 0) 2 = 50
(a) X -axis (b) Y-axis Þ 9 + 4 + y 2 + 4y + 25 = 50
(c) Z-axis (d) None of these Þ y 2 + 4y + 38 - 50 = 0
Sol. (a) If any plane is parallel to yz-plane, then it is perpendicular Þ y 2 + 4y - 12 = 0
to X-axis. Þ (y + 6) (y - 2) = 0
Þ y = - 6, 2
Example 2. In which of the following point lies in a fourth Hence, the points on Y-axis are (0 , - 6, 0) or (0 , 2, 0).
octant?
(1, 2, 3), (4, -2, 3), ( 4,-2,-5) and ( 4, 2, - 5) Example 4. Which of the following pairs of points have a
(a) (1, 2,3) (b) ( 4,-2, 3) distance 43?
(c) ( 4,-2,-5) (d) ( 4, 2,-5) (a) (2, 3, 5) and (4, 3, 1) (b) ( -3, 7, 2) and (2, 4, - 1)
(c) ( -1, 3, - 4 ) and (1, - 3, 4 ) (d) (2, - 1, 3) and ( -2, 1, 3 )
Introduction to Three Dimensional Geometry 437

Sol. (b) (a) Let the given points are A(2, 3, 5) and B( 4, 3, 1). Section Formulae
\ x1 = 2, y1 = 3, z1 = 5
and x2 = 4, y 2 = 3, z 2 = 1 Section Formulae for Internal Division
\ Required distance Let P ( x1 , y1 , z1 ) and Q ( x2 , y2 , z 2 ) be two points.
2 2 2
AB = ( x2 - x1) + (y 2 - y1) + ( z 2 - z1) Let R be a point on the line segment joining P and Q such
that it divides the join of P and Q internally in the ratio
Þ AB = ( 4 - 2) 2 + (3 - 3) 2 + (1 - 5) 2 m : n.
= 4 + 0 + 16 = 20 = 2 5 Then, the coordinates of R are
(b) Let the given points are A( - 3, 7, 2 ) and B(2, 4, - 1). æ mx2 + nx1 my2 + ny1 mz 2 + nz1 ö
ç , , ÷
Here, x1 = - 3, y1 = 7, z1 = 2 è m+n m+n m+n ø
x2 = 2, y 2 = 4, z 2 = - 1
m R n
\ Required distance AB = ( x2 - x1) 2 + (y 2 - y1) 2 + ( z 2 - z1) 2 P Q
(x1, y1, z1) (x2, y2, z2)
2 2 2
= [2 - ( - 3)] + ( 4 - 7) + ( - 1 - 2)
= (2 + 3) 2 + ( 4 - 7 ) 2 + ( - 1 - 2) 2 Section Formulae for External Division
= 25 + 9 + 9 = 43 If P and Q are such that R divides the join of P and Q
(c) Let the given points are A( - 1, 3, - 4) and B(1, - 3, 4).
externally in the ratio m : n. Then, the coordinates of R
are
\ x1 = - 1,y1 = 3, z1 = - 4
æ mx2 - nx1 my2 - ny1 mz 2 - nz1 ö
x2 = 1,y 2 = - 3, z 2 = 4 ç ; , ÷
è m-n m-n m-n ø
\ Required distance, AB = (1 + 1) 2 + ( - 3 - 3) 2 + ( 4 + 4) 2
m n
= 4 + 36 + 64 = 104 = 2 26
P Q R
(d) Let the given points are A(2, - 1, 3) and B( - 2, 1, 3). (x1, y1, z1) (x2, y2, z2)
\ x1 = 2, y1 = - 1, z1 = 3
x2 = - 2, y 2 = 1, z 2 = 3 Mid-point
\Required distance
If A ( x1 , y1 , z1 ) and B ( x2 , y2 , z 2 ) are two points, then the
AB = ( - 2 - 2) 2 + (1 + 1) 2 + (3 - 3) 2 = 4 2 + 2 2 + 0
æ x + x2 y1 + y2 z1 + z 2 ö
mid-point of AB is ç 1 ; , ÷.
= 16 + 4 = 20 = 2 5 è 2 2 2 ø

Example 5. The equation of the set of points which are


equidistant from the points (1, 2, 3) and (3, 2, - 1) is Example 6. A point R with x-coordinate 4 lies on the line
segment joining the points P(2, - 3, 4) and Q(8, 0, 10). Find the
(a) x - 3z = 0 (b) x - 2z = 0
(c) x - 4z = 0 (d) x + 2z = 0 coordinates of the point R.
(a) (4, –2, –6) (b) (4, 2, 6)
Sol. (b) Let the given points are A and B.
(c) (4, –2, 6) (d) None of these
Let P( x, y , z) be any point equidistant from A and B.
P Sol. (c) Let the point R ( x, y , z) divides PQ in the ratio k :1.
k 1
P R Q
(2, –3, 4) (4, y, z) (8, 0, 10)

k ´ 8 + 1´ 2
Þ =4 (given)
A B k+1
\ PA = PB, 8k + 2
Þ =4
i.e. Distance between P and A = distance between P and B k+1
Þ ( x - 1) 2 + (y - 2 ) 2 + ( z - 3) 2 = ( x - 3) 2 + (y - 2) 2 + ( z + 1) 2 Þ 8k + 2 = 4k + 4
2 2 2 2
Þ ( x - 1) + (y - 2) + ( z - 3) = ( x - 3) + (y - 2) + ( z + 1) 2 2 Þ 8k - 4k = 4 - 2
Þ x2 + 1 - 2x + y 2 + 4 - 4y + z 2 + 9 - 6z Þ 4k = 2
1
= x2 + 9 - 6x + y 2 + 4 - 4y + z 2 + 1 + 2z Þ k=
2
Þ 4x - 8z = 0 Þ k : 1 = 1: 2
Þ x - 2z = 0
Hence, the point R divides PQ internally in the ratio 1 : 2.
which is the required equation.
438 JEE Main Mathematics

æ1 ´ 0 + 2 ´ ( -3) ö -6 Sol. (b) Given, points are A(2, 3, 4), B( -1, 2, - 3) and C( -4, 1, - 10).
Therefore, y-coordinate of R = ç ÷= = -2
è 1+ 2 ø 3 Now, determine area of triangle.
æ1 ´ 10 + 2 ´ 4 ö 10 + 8 18 \ Area of triangle, D = D2xy + D2yz + D2zx
and z-coordinate of R = ç ÷= = = 6.
è 1+ 2 ø 3 3 x1 y1 1 y1 z1 1
1 1
Hence, coordinate of R are (4, –2, 6). where D xy = x2 y 2 1 , D yz = y2 z2 1
2 2
x3 y3 1 y3 z3 1
Area of a Triangle z1 x1 1
Let the vertices of a DABC be A( x1 , y1 , z1 ), B( x2 , y2 , z 2 ) and 1
and D zx = z 2 x2 1
C( x3 , y3 , z3 ). 2
z3 x3 1
If Ayz , Azx , Axy be the projection of an area A on the
2 3 1
coordinate planes yz , zx and xy respectively, then 1
Now,D xy = -1 2 1
Area, D= A2yz + Azx
2 2
+ Axy 2
-4 1 1
y1 z1 1 1
1 = |[2(2 - 1) - 3( -1 + 4)] + 1( -1 + 8)]|
where Ayz = y2 z2 1 , 2
2 1
y3 z3 1 = |[2 - 9 + 7]| = 0
2
z1 x1 1 x1 y1 1
1 1 3 4 1
Azx = z2 x2 1 and Axy = x2 y2 1 1
2 2 D yz = 2 -3 1
z3 x3 1 x3 y3 1 2
1 -10 1
$i $j k$ 1
1 = |[3( -3 + 10) - 4(2 - 1) + 1( -20 + 3)]|
Note Area of DABC = x2 - x1 y2 - y1 z2 - z1 2
2
x3 - x1 y3 - y1 z3 - z1 1
= |21 - 4 - 17| = 0
2
Conditions for Collinearity 4 2 1
1
If A, B and C are collinear, then area of triangle should D zx = -3 -1 1
2
be zero -10 -4 1
i.e. D=0 1
= |[ 4( -1 + 4) - 2 ( -3 + 10) + 1(12 - 10)]|
Example 7. If there are three points A(2, 3, 4), B( -1, 2, - 3) 2
and C( -4, 1, - 10) in a space, then they are 1
= |[12 - 14 + 2]| = 0
(a) collinear 2
(b) non-collinear \ D = 02 + 02 + 02 = 0
(c) Do not say anything
(d) None of the above Hence, given points are collinear.
Practice Exercise
ROUND I Topically Divided Problems

Cartesian Coordinates 9. If A and B be the points (3, 4, 5) and (– 1, 3, – 7)


in Three Dimensions respectively, find the equation of the set of points P
1. A plane is parallel xy-plane, so it is perpendicular to such that ( PA) 2 + ( PB) 2 = K 2 , where K is a
constant.
(a) Z-axis (b) Y-axis
(c) X-axis (d) None of these (a) 2 (x 2 + y 2 + z 2) + 4 x + 14 y + 4 z + 109 - K 2 = 0
(b) 2 (x 2 + y 2 + z 2) - 4x - 14 y + 4z + 109 - K 2 = 0
2. The locus of a point for which y = 0, z = 0 is
(c) x 2 + y 2 + z 2 + 4x + 14 y + 4 z + 109 - K 2 = 0
(a) equation of x-axis (d) None of the above
(b) equation of y-axis
(c) equation of z-axis 10. Distance of the point (1, 2, 3) from the coordinate
(d) None of the above axes are
(a) 13, 10, 5 (b) 13, 10 , 5
3. The point ( - 2, - 3, - 4) lies in the 1 1 1
(c) 5, 13 , 10 (d) , ,
(a) first octant (b) seventh octant 13 10 5
(c) second octant (d) eighth octant
11. If the sum of the squares of the distance of a point
4. L is the foot of the perpendicular drawn from a from the three coordinate axes be 36, then its
point P (3, 4, 5) on the xy-plane. The coordinates of distance from the origin is
point L are (a) 6 (b) 3 2
(a) (3, 0, 0) (b) (0, 4, 5) (c) 2 3 (d) None of these
(c) (3, 0, 5) (d) None of these
12. The coordinates of a point which is equidistant
from the points (0, 0, 0), ( a, 0, 0), (0, b, 0), (0, 0, c) are
Distance between Two Points given by
5. The distance of point P (3, 4, 5) from the yz-plane is æ a b cö æ a b cö
(a) ç , , ÷ (b) ç - , - , ÷
(a) 3 units (b) 4 units è 2 2 2ø è 2 2 2ø
(c) 5 units (d) 550 units æa b cö æ a b cö
(c) ç , - , - ÷ (d) ç - , , - ÷
è2 2 2ø è 2 2 2ø
6. What is the length of foot of perpendicular drawn
from the point P (3, 4, 5) on Y-axis? 13. If x 2 + y 2 = 1, then the distance from the point
(a) 41 (b) 34 ( x, y, 1 - x 2 - y 2 ) to the origin is
(c) 5 (d) None of these
(a) 1 (b) – 1 (c) 0 (d) 2
7. If the distance between the points ( a, 0, 1) and
14. If a parallelopiped is formed by planes drawn
(0, 1, 2) is 27, then the value of a is
through the points (5, 8, 10) and (3, 6, 8) parallel to
(a) 5 (b) ± 5
the coordinate planes, then the length of diagonal
(c) – 5 (d) None of these
of the parallelopiped is
8. If x-coordinate of a point P of line joining the points (a) 2 3 (b) 3 2 (c) 2 (d) 3
Q ( 2, 2, 1) and R(5, 2, - 2) is 4, then the z-coordinate
15. The points A (5, - 1, 1), B (7, - 4, 7), C (1, - 6, 10) and
of P is
(a) –2 (b) –1 D( - 1, - 3, 4) are vertices of a
(c) 1 (d) 2 (a) square (b) rhombus
(c) rectangle (d) None of these
440 JEE Main Mathematics

16. The points (5, – 4, 2),(4,– 3, 1), (7, - 6, 4) and 23. Find the centroid of a triangle, the mid-point of
(8, – 7, 5) are the vertices of whose sides are D (1, 2, - 3), E (3, 0, 1) and
(a) a rectangle F ( - 1, 1, - 4).
(b) a square (a) (1, 1, 2) (b) (1, 1, – 2)
(c) a parallelogram (c) (– 1, –1, –2) (d) (1, –1, –2)
(d) None of the above
24. The mid-points of the sides of a triangle are
17. If the coordinates of the vertices of a DABC are
(5, 7, 11), (0, 8, 5) and (2, 3, – 1). Then, the vertices
A ( - 1, 3, 2), B ( 2 , 3, 5) and C(3, 5, - 2), then Ð A is
are
equal to
(a) (7, 2, 5), (3, 12, 17), (– 3, 4, – 7)
(a) 45° (b) 60° (b) (7, 2, 5), (3, 12, 17), ( 3, 4, 7)
(c) 90° (d) 30° (c) (7, 2, 5), (– 3, 12, 17), (– 3, – 4, – 7)
(d) None of the above
Section Formlae
18. Three vertices of a parallelogram ABCD are Area of Triangle and Volume
A (1, 2, 3), B ( - 1, - 2, - 1) and C( 2, 3, 2). Find the of Tetrahedran
fourth vertex D. 25. The area of the triangle, whose vertices are at the
(a) (– 4, – 7, – 6) (b) (4, 7, 6) points (2, 1, 1), (3, 1, 2) and (– 4, 0, 1) is
(c) (4, 7, – 6) (d) None of these 1
(a) 19 (b) 19
19. Find the coordinates of the point which divides the 2
1 1
line segment joining the points (– 2, 3, 5) and (c) 38 (d) 57
2 2
(1, - 4, 6) in the ratio 2 : 3 externally.
(a) (– 8, – 17, 3) 26. If vertices of a triangle are A (1, - 1, 2), B ( 2 , 0, - 1)
(b) (– 8, 17, 3) and C(0, 2 , 1), then the area of a triangle is
(c) (8, – 17, 3) (a) 6 (b) 2 6 (c) 3 6 (d) 4 6
(d) None of the above
27. The triangle formed by the points
20. Find the ratio in which the yz-plane divides the line
(0, 7, 10), ( - 1, 6, 6), ( - 4, 9, 6) is
segment formed by joining the points (– 2, 4, 7) and
(a) equilateral
(3, – 5, 8).
(b) isosceles
(a) externally 2 : 3 (b) internally 2 : 3
(c) right angled
(c) internally 3 : 2 (d) externally 3 : 2
(d) right angled isosceles
21. Find the length of the medians of the triangle with
28. The points (5, 2, 4), (6, - 1, 2) and (8, - 7, k) are
vertices A (0, 0, 6), B (0, 4, 0) and C(6, 0, 0).
(a) 7, 7, 34 (b) 7, 8, 34 collinear, if k is equal to
(c) 7, 9, 34 (d) None of these (a) - 2 (b) 2
(c) 3 (d) - 1
22. Find the coordinates of the points which trisect the
line segment joining the points P ( 4 , 2 , - 6) and 29. The point A(1, - 1, 3), B( 2, - 4, 5) and C(5, - 13, 11)
Q (10, - 16, 6). are
(a) (6, – 4, – 2), (8, – 10, 2) (a) collinear
(b) (6, 4, – 2), (8, – 10, 2) (b) non-collinear
(c) (6, – 4, – 2), (8, 10, 2) (c) Do not say anything
(d) None of the above (d) None of the above

Answers
1. (a) 2. (a) 3. (b) 4. (d) 5. (a) 6. (b) 7. (b) 8. (b) 9. (b) 10. (b)
11. (b) 12. (a) 13. (a) 14. (a) 15. (b) 16. (c) 17. (c) 18. (b) 19. (b) 20. (b)
21. (a) 22. (a) 23. (b) 24. (a) 25. (c) 26. (b) 27. (d) 28. (a) 29. (a)
Solutions
1. If a plane is parallel to xy-plane, then it is perpendicular Þ 2x2 + 2 y2 + 2z 2 - 4x - 14 y + 4z + 109 - K 2 = 0
to Z-axis. Þ 2 (x2 + y2 + z 2) - 4x - 14 y + 4z + 109 - K 2 = 0
2. In X-axis, the coordinates of y and z are zero. which is the required equation.
3. In a given point (-2, - 3, - 4) all coordinates are 10. Let point be P (x, y, z ) = (1, 2, 3)
negative. Hence, it lies in seventh octant.
\The distance from x-axis = y2 + z 2
4. Since, in xy-plane, z-coordinate is zero.
Hence, the coordinate of the foot of the point L are = 4 + 9 = 13
(3, 4, 0).
The distance from y-axis = x2 + z 2 = 12 + 32
5. When we draw a perpendicular line from P(3, 4, 5) on
the yz-plane the x-coordinate will be zero and the other = 1 + 9 = 10
coordinate y and z will be 4 and 5, i.e. coordinate on
The distance from z-axis = x2 + y2 = 12 + 22
yz-plane be Q(0, 4, 5).
\Distance between P and Q = PQ = 1+4= 5
2 2 2
= (3 - 0) + (4 - 4) + (5 - 5) 11. Let P (x, y, z ) be any point in the plane.
2 2
= 3 + 0 + 0 = 3 units 2 According to the given condition,
(distance from P to X-axis) 2 + (distance from P to
6. When we drawn perpendicular from the point P(3, 4, 5)
Y-axis) 2 + (distance from P to Z-axis) 2 = 36
on Y-axis, the x and z-coordinates will be zero and
y-coordinate will be 4. i.e., coordinate on Y-axisQ(0, 4, 0). Þ ( y2 + z 2 )2 + ( x2 + z 2 )2 + ( x2 + y2 )2 = 36
\Distance between P and Q = PQ
Þ ( y2 + z 2) + (x2 + z 2) + (x2 + y2) = 36
= (3 - 0)2 + (4 - 4)2 + (5 - 0)2 Þ 2(x2 + y2 + z 2) = 36
= 32 + 02 + 52 = 9 + 25 = 34 units Þ x2 + y2 + z 2 = 18 …(i)
\The distance from origin to the point (x, y, z ) is
7. Given, distance between points (a , 0, 1 ) and (0, 1, 2) is
27. = x2 + y2 + z 2 = 18 [Q from Eq. (i)]
\ (a - 0)2 + (0 - 1 )2 + (1 - 2 )2 = 27 =3 2

Þ a 2 + 12 + 12 = 27 12. Let P be any point in a space.


Þ a 2 + 2 = 27 According to the given condition,
[Q squaring both sides] (x - 0)2 + ( y - 0)2 + (z - 0)2 = (x - a )2 + y2 + z 2
Þ a 2 = 25 Þ a = ± 5 = x2 + ( y - b)2 + z 2 = x2 + y2 + (z - c)2
8. Suppose P divides QR in the ratio l : 1. Then, coordinate On squaring both sides, we get
æ 5 l + 2 2 l + 2 -2 l + 1 ö x2 + y2 + z 2 = (x - a )2 + y2 + z 2 = x2 + ( y - b)2 + z 2
of P are ç , , ÷.
è l+1 l+1 l+1 ø = x2 + y2 + (z - c)2
Since, the x-coordinate of P is 4. Þ x2 + y2 + z 2 = x2 + a 2 - 2ax + y2 + z 2
5l + 2 Þ a 2 - 2ax = 0
i.e. =4
l+1 Þ x=
a
Þ l =2 2
-2 l + 1 -4 + 1 Similarly, we can solve the other values, we get
So, z-coordinate of P is = = -1 b c
l+1 2+1 y= ,z=
2 2
9. Let the point P is (x, y, z ).
13. \Distance from point (x, y, 1 - x2 - y2 ) to the origin.
2 2 2
Given, (PA ) + (PB ) = K
Þ (x - 3)2 + ( y - 4)2 + (z - 5)2 = (x - 0)2 + ( y - 0)2 + (1 - x2 - y2 - 0)2

+ (x + 1 )2 + ( y - 3)2 + (z + 7 )2 = K 2 = x2 + y2 + 1 - x2 - y2
Þ x + 9 - 6x + y + 16 - 8 y + z 2 + 25 - 10z
2 2
= 1 + 1 - (1 ) [Q x2 + y2 = 1 given]
2 2 2 2
+ x + 1 + 2x + y + 9 - 6 y + z + 49 + 14z = K = 1 =1
442 JEE Main Mathematics

14. The edge of a parallelopiped = (3 - 5, 6 - 8, 8 - 10) CA = 16 + 4 + 16 = 6


= (-2, - 2, - 2) and BC = 1 + 4 + 49 = 54
2
\Length of diagonal = (-2) + (-2) + (-2) 2 2 Q AB2 + CA 2 = BC 2
D ABC is right angled triangle at A.
= 4 + 4 + 4 =2 3
\ ÐA = 90°
15. Given that, A(5, - 1, 1 ), B(7, - 4, 7 ), C (1, - 6, 10) and
D(-1, - 3, 4). 18. Let the fourth vertex be (x, y, z ).

Now, AB = (7 - 5)2 + (-4 + 1 )2 + (7 - 1 )2 We know that diagonals of a parallelogram are bisecting


to each other.
= 4 + 9 + 36 = 7 i.e. mid-point of a diagonals are coinciding.
2 2 2
BC = (1 - 7 ) + (-6 + 4) + (10 - 7) \Mid-point of diagonal AC = Mid-point of diagonal BD
= 36 + 4 + 9 = 7 æ 1 + 2 2 + 3 3 + 2 ö æ -1 + x -2 + y -1 + z ö
\ ç , , ÷=ç , , ÷
è 2 2 2 ø è 2 2 2 ø
CD = (-1 - 1 )2 + (-3 + 6)2 + (4 - 10)2
æ 3 5 5 ö æ -1 + x -2 + y -1 + z ö
Þ ç , , ÷=ç , , ÷
= 4 + 9 + 36 = 7 è2 2 2ø è 2 2 2 ø
DA = (5 + 1 )2 + (-1 + 3)2 + (1 - 4)2 3 -1 + x 5 -2 + y 5 -1 + z
Þ = , = , =
2 2 2 2 2 2
= 36 + 4 + 9 = 7
Þ x = 4, y = 7, z = 6
\ AB = BC = CD = DA = 7,
Hence, required point is (4, 7, 6).
Also, AC = (1 - 5)2 + (-6 + 1 )2 + (10 - 1 )2
19. Let the point C divides the line externally in the ratio
= 16 + 25 + 81 = 122 2 : 3.
3
BD = (-1 - 7 )2 + (-4 + 3)2 + (7 - 4)2 2

= 64 + 1 + 9 = 74 C A B
(–2, 3, 5) (1, –4, 6)
\Diagonal AC ¹ BD
Here, the ratio is 2 : 3.
Hence, ABCD is not a square, so it is a rhombus.
\ m = 2, n = 3
16. Let A(5, - 4, 2), B(4, - 3, 1 ), C (7, - 6, 4) and D(8, - 7, 5). The coordinates of point C
Then, AB = (4 - 5)2 + (-3 + 4)2 + (1 - 2)2 é æ m x2 - nx1 ö æ my2 - ny1 ö æ mz2 - nz1 ö ù
= êç ÷, ç ÷, ç ÷ú
= 1+1+1= 3 ëè m - n ø è m - n ø è m - n øû
é 2 ´ ( 1 ) - 3 ´ (-2) 2 ´ (-4) - 3 ´ 3 2 ´ 6 - 3 ´ 5 ù
BC = (7 - 4)2 + (-6 + 3)2 + (4 - 1 )2 ÞC = ê , , ú
ë (2 - 3) (2 - 3) (2 - 3) û
= 9 + 9 + 9 =3 3
é 2 + 6 - 8 - 9 12 - 15 ù
CD = (8 - 7 )2 + (-7 + 6)2 + (5 - 4)2 =ê , , ú = (- 8, 17, 3)
ë (- 1 ) (- 1 ) (- 1 ) û
= 1+1+1= 3
20. The given points are A(- 2, 4, 7 ) and B(3, - 5, 8).
AD = (8 - 5)2 + (-7 + 4)2 + (5 - 2)2
Let the point P (0, y, z ) in yz-plane divides AB in the
= 9 + 9 + 9 =3 3 ratio k : 1, then
AC = (7 - 5)2 + (-6 + 4)2 + (4 - 2)2 P
A k 1 B
= 4 + 4 + 4 =2 3
(–2, 4, 7) (3, –5, 8)
BD = (8 - 4)2 + (-7 + 3)2 + (5 - 1 )2
mx2 + nx1
x-coordinate of point P =
= 16 + 16 + 16 = 4 3 m+ n
Here, we see that opposite sides AB = CD , BC = AD are k ´ 3 + 1 ´ (-2)
equal and diagonals AC ¹ BD. = 0 [Q x-coordinate of P is zero]
k+1
Hence, given points are the vertices of a 2
parallelogram. Þ 3k - 2 = 0 Þ k =
3
17. Q Vertices of D ABC are A (-1, 3, 2), B(2 , 3, 5) and Þ k :1 = 2 :3
C (3, 5, - 2). \ yz-plane divides the segment internally in the ratio
Þ AB = 9 + 0 + 9 = 18 2 : 3.
Introduction to Three Dimensional Geometry 443

21. ABC is a triangle with vertices A (0,0,6) 24. Let the vertices of a triangle are A (x1 , y1 , z1 ), B(x2, y2, z2)
A(0, 0, 6), B (0, 4, 0) and C (6, 0, 0). and C (x3 , y3 , z3 ).
Let points D , E and F are the F E A(x1, y1, z1)
mid-points of BC , AC and AB,
respectively. So, AD , BE and CF
will be the medians of the B E D(5, 7, 11)
C (0, 8, 5)
triangle. (0,4,0) D (6,0,0)
æ0 + 6 4 + 0 0 + 0ö
Þ Coordinates of point D = ç , , ÷ = (3, 2, 0) B(x2, y2, z2) C(x3, y3, z3)
è 2 2 2 ø F
(2, 3, –1)
æ0 + 6 0 + 0 6 + 0ö
Coordinates of point E = ç , , ÷ = (3, 0, 3)
è 2 2 2 ø Since, D , E and F are the mid-points of
AC , BC and AB.
æ0 + 0 0 + 4 6 + 0ö
and coordinates of point F = ç , , ÷ æ x1 + x2 y1 + y2 z1 + z2 ö
è 2 2 2 ø \ ç , , ÷ = (0, 8, 5)
è 2 2 2 ø
= (0, 2, 3)
Þ x1 + x2 = 0, y1 + y2 = 16, z1 + z2 = 10 …(i)
Now, length of median AD = Distance between A and D
æ x2 + x3 y2 + y3 z2 + z3 ö
ç , , ÷ = (2, 3, - 1 )
AD = (0 - 3)2 + (0 - 2)2 + (6 - 0)2 è 2 2 2 ø
= 9 + 4 + 36 = 49 = 7 Þ x2 + x3 = 4, y2 + y3 = 6, z2 + z3 = - 2 …(ii)
æ x1 + x3 y1 + y3 z1 + z3 ö
Similarly, BE = (0 - 3)2 + (4 - 0)2 + (0 - 3)2 and ç , , ÷ = (5, 7, 11 )
è 2 2 2 ø
= 9 + 16 + 9 = 34 Þ x1 + x3 = 10, y1 + y3 = 14, z1 + z3 = 22 …(iii)
2 2 2
and CF = (6 - 0) + (0 - 2) + (0 - 3) On adding Eqs. (i), (ii) and (iii), we get
2(x1 + x2 + x3 ) = 14,2( y1 + y2 + y3 ) = 36, 2(z1 + z2 + z3 ) = 30
= 36 + 4 + 9 = 49 = 7
Þ x1 + x2 + x3 = 7, y1 + y2 + y3 = 18, z1 + z2 + z3 = 15 …(iv)
22. Let the point R1 trisects the line PQ i.e., it divides the
On solving Eqs. (i), (ii), (iii) and (iv), we get
line in the ratio 1 : 2.
x3 = 7, x1 = 3, x2 = - 3;
1 2
y3 = 2, y1 = 12, y2 = 4
P R1 Q
(4, 2, –6) (10, –16, 6) and z3 = 5, z1 = 17, z2 = -7
Hence, vertices of a triangle are
é 1 ´ 10 + 2 ´ 4 1 ´ (-16) + 2 ´ 2 1 ´ 6 + 2 ´ (-6) ù
Þ R1 = ê , , ú (7, 2, 5), (3, 12, 17), (–3, 4, –7).
ë 1+2 1+2 1+2 û y1 z1 1 z1 x1 1
æ 10 + 8 - 16 + 4 6 - 12 ö æ 18 -12 -6 ö 25. Q D yz =
1
y2 z2 1 , D zx =
1
=ç , , ÷=ç , , ÷ z2 x2 1
è 3 3 3 ø è3 3 3 ø 2 2
y3 z3 1 z3 x3 1
= (6, - 4, - 2)
Again, let the point R2 divides PQ internally in the ratio x1 y1 1
1
2 : 1. Then, D xy = x2 y2 1
2
2 1 x3 y3 1
P R2 Q 1 1 1
(4, 2, –6) (10, –16, 6) 1
\ D yz = 1 2 1
é 2 ´ 10 + 1 ´ 4 2 ´ (-16) + 1 ´ 2 2 ´ 6 + 1 ´ (-6) ù 2
Þ R2 = ê , , 0 1 1
ú
ë 2+1 2+1 1+2 û 1
= |[1 (2 - 1 ) - 1 (1 - 0) + 1 (1 - 0)]|
æ 20 + 4 - 32 + 2 12 - 6 ö æ 24 -30 6 ö 2
=ç , , ÷=ç , , ÷
è 3 3 3 ø è3 3 3ø 1 1
= |[1 - 1 + 1]| =
= (8, - 10, 2) 2 2
Hence, required points are (6, - 4, - 2) and (8, - 10, 2). 1 2 1
1
D zx = 2 3 1
23. Given, mid-point of sides of a triangle are D(1, 2, - 3), 2
1 -4 1
E (3, 0, 1 ) and F (-1, 1, - 4).
æ 1 + 3 - 1 2 + 0 + 1 -3 + 1 - 4 ö 1
\Centroid of triangle = ç , , ÷ = |[1(3 + 4) - 2(2 - 1 ) + 1 (-8 - 3 )]|
è 3 3 3 ø 2
æ 3 3 - 6 ö 1
=ç , , = |[7 - 2 - 11]| = 3
÷ = (1, 1, - 2) 2
è3 3 3 ø
444 JEE Main Mathematics

2 1 1 1
1 = |[30 + 4 - 34]| = 0
D xy = 3 1 1 2
2 2 4 1
-4 0 1 1
1 1 D yz = -1 2 1
= |[2(1 - 0) - 1 (3 + 4 ) + 1 (0 + 4 )]| = 2
2 2 -7 k 1
\ Area of triangle, D = D2yz + D2zx + D2xy 1
= |[2(2 - k ) - 4 (-1 + 7 ) + 1 (- k + 14 ) ]|
2 2
2
æ1ö æ1ö 1 1 3k + 6
= ç ÷ + (3)2 + ç ÷ = |[4 - 2k - 24 - k + 14]|= |[-3k - 6]| =
è2ø è2ø 2 2 2
1 1 38 4 5 1
= +9+ = sq unit 1
4 4 2 D zx = 2 6 1
2
1 -1 1 k 8 1
1
26. Now, D xy = 2 0 1 1
2 = |[4(6 - 8) - 5(2 - k) + 1(16 - 6k)]|
0 2 1 2
1 1
= |1 (0 - 2) + 1(2 - 0) + 1 (4 - 0)| = |[-8 - 10 + 5k + 16 - 6k]|
2 2
1 1 k+2
= |-2 + 2 + 4| = 2 = |[-2 - k]|=
2 2 2
-1 2 1 \For collinear, D =0
1 2 2
D yz = 0 -1 1 æ 3k + 6 ö æ k + 2ö
2 \ 02 + ç ÷ +ç ÷ =0
2 1 1 è 2 ø è 2 ø
1 1
= |[-1 (-1 - 1 ) - 2(0 - 2) + 1(0 + 2)]| Þ [(3k + 6)2 + (k + 2)2] = 0
2 4
1
= |[2 + 4 + 2]| = 4 Þ 9k2 + 36 + 36k + k2 + 4 + 4k = 0
2 Þ 10k2 + 40k + 40 = 0 Þ k2 + 4k + 4 = 0
2 1 1
1 Þ (k + 2)2 = 0 Þ k = - 2
D zx = -1 2 1
2 1 -1 1
1 0 1 1
29. Now, D xy = 2 -4 1
1 2
= |[2(2 - 0) - 1 (-1 - 1 ) + 1(0 - 2)]| 5 -13 1
2
1 1
= |[4 + 2 - 2]| = 2 = |[1 (-4 + 13 ) + 1 (2 - 5) + 1 (-26 + 20 )]|
2 2
1
\Area of triangle, D = D2xy + D2yz + D2zx = |[9 - 3 - 6]| = 0
2
= 22 + 42 + 22 = 24 = 2 6 -1 3 1
1
D yz = -4 5 1
27. Let A(0, 7, 10), B(-1, 6, 6) and C (-4, 9, 6). 2
-13 11 1
Then, AB = (-1 - 0)2 + (6 - 7 )2 + (6 - 10)2
1
= 1 + 1 + 16 = 18 = 3 2 = |[-1 (5 - 11 ) - 3 (-4 + 13 ) + 1 (-44 + 65 )]|
2
BC = (-4 + 1 )2 + (9 - 6)2 + (6 - 6)2 1
= |[6 - 27 + 21]| = 0
= 9 + 9 + 0 = 18 = 3 2 2
3 1 1
AC = (-4 - 0)2 + (9 - 7 )2 + (6 - 10)2 1
D zx = 5 2 1
= 16 + 4 + 16 = 36 = 6 2
11 5 1
Clearly, AC 2 = AB2 + BC 2
1
Hence, triangle is right angled. Also, AB = BC = |[3 (2 - 5) - 1 (5 - 11 ) + 1 (25 - 22)]|
2
\ Triangle is right angled isosceles.
1
5 2 1 = |[-9 + 6 + 3]| = 0
1 2
28. Now, D xy = 6 -1 1
2 \Area of triangle = D2xy + D2yz + D2zx
8 -7 1
1 = 0 + 0 + 0 =0
= |[5(-1 + 7 ) - 2(6 - 8) + 1(-42 + 8)]| Hence, given points are collinear.
2
18
Limits and
Derivatives
Limits IN THIS CHAPTER ....
Let y = f ( x ) be a given function defined in the neighbourhood of x = a but not Limits
necessarily at the point x = a. The limiting behaviour of the function in the Indeterminate Forms
neighbourhood of x = a when| x - a| is small, is called the limit of the function
when x approaches a and we write this as lim f ( x ). Fundamental Theorems on
x ®a Limits
Let lim f ( x ) = l. It would simply mean that when we approach the point Evaluation of Algebraic Limit
x ®a
Squeeze Play Theorem
x = a from the values which are just greater than or just smaller than x = a , (Sandwich Theorem)
f ( x ) would have a tendency to move closer to the value l.
Some Useful Expansions
Formal Approach of Limit Evaluations of Trigonometric
Limits
Let a be a real number and let d be a positive real number. Then, the set of
all real numbers lying between a - d and a + d is called the neighbourhood of Exponential Limits
a of radius d and is denoted by N d ( a ). L’Hospital’s Rule
Thus, N d ( a ) = ( a - d, a + d) = { x Î R|a - d < x < a + d|} Derivative
The set N d ( a ) - { a } is called nbd of a of radius d. Derivative from First Principle
The set ( a - d, a ) is called the left nbd of a and the set ( a , a + d) is know as the Differentiation of Some Important
right nbd of a. If d is very small and x lies in the interval ( a - d, a ), then x is Functions
said to approach to a from the left and we write x ® a - . If x Î ( a , a + d), then x
Algebra of Derivative of Functions
is said to approach to a from the right which is denoted by x ® a + .
Consider the statement|x - a| < d. We have
|x - a| < d Û - d < x - a < d Û a - d < x < a + d Û x Î N d ( a )
Thus,|x - a| < d mean that x lies in the nbd of ‘a’ of radius as shown in figure
a–d a a+d
446 JEE Main Mathematics

Let f ( x ) be a function with domain D and let ‘a’ be a point 2


Þ 2m - 3 = Þ 2m2 - 3m - 2 = 0
such that every nbd of a contains infinitely many points m
of D. A real number l is limit of f ( x ) as x tends to a, if for 1
\ m = - ,2
every nbd of l, there exists a nbd of ‘a’ such that images 2
of all points in the deleted nbd of a are in the nbd of l.
x -2
Example 2. lim is equal to
Right Hand Limit x ® 2| x - 2|
A function f is said to approach l as x approaches a from (a) 1 (b) –1
right, if corresponding to an arbitrary positive number e, (c) 0 (d) does not exist
there exists a positive number d, such that| f ( x ) - l| < e
whenever a < x < a + d. ìx - 2 , if x ³ 2
Sol. (d) Q | x - 2| = í
It is written as lim f ( x ) = l or f ( a + 0) = l. î-( x - 2), if x < 2
x ® a+ æ x -2ö
Now, RHL = lim ç ÷ = lim+ 1 = 1
+ x-2
The working rule for finding the right hand limit is ‘put x ®2 è ø x ®2
a + h for x in f ( x ), where h is positive and very-very small æ x-2 ö
and make h approach zero. and LHL = lim ç ÷
x ® 2- è -( x - 2) ø
i.e. f ( a + 0) = lim f ( a + h ) æ x - 2ö
h ®0 = - lim çç ÷÷ = -1
x ® 2 è x - 2ø
-

Left Hand Limit Since, lim f( x ) ¹ lim f ( x )


A function f is said to approach l as x approaches a from x ® 2+ x ® 2-
the left, if corresponding to an arbitrary positive number Therefore, the limit does not exist.
e, there exists a positive number d, such that
Uniqueness of Limit If lim f ( x ) exists, then it is unique.
| f ( x ) - l| < e, whenever a - d < x < a. x ®a
There cannot be two distinct numbers l1 and l2 such that
It is written as lim f ( x ) = l or f ( a - 0) = l . when x tends to a, the function f ( x ) tends to both l1 and l2.
x ® a-

The working rule for finding the left hand limit is put Indeterminate Forms
a - h for x in f ( x ), where h is positive and very-very small
and make h approach zero. If a function f ( x ) takes any of the following forms at
0 ¥
i.e. f ( a - 0) = lim f ( a - h ) x = a ; , , ¥ - ¥ , 0 ´ ¥ , 00 , ¥ 0 , 1¥ , then f ( x ) is said to be
h ®0 0 ¥
Existence of Limit indeterminate at x = a and these forms are called
If both right hand limit and left hand limit exist and are indeterminate forms.
equal, then their common value, evidently will be the
limit of f as x ® a Fundamental Theorems on Limits
i.e. If lim f ( x ) = lim f ( x ) = l , then lim f ( x ) = l , If f ( x ) and g( x ) be two functions of x such that lim f ( x )
x ® a+ x ® a- x ®a x ®a
and lim g( x ) both exist, then
x ®a
If however, either both of these limits do not exist or both
these limits exist but are not equal in value, then (i) lim [ f ( x ) ± g( x )] = lim f ( x ) ± lim g( x )
lim f ( x ) does not exist. x ®a x ®a x ®a
x ®a (sum and difference rule)
Example 1. For what values of m does the lim f ( x) exist, (ii) lim [kf ( x )] = k lim f ( x ), where k is a fixed real
x®2 x ®a x ®a
number. (constant multiple rule)
ì mx - 3, when x <2
ï
when f ( x) = í x (iii) lim [ f ( x ) g( x )] = lim f ( x ) lim g( x ) (product rule)
ïî m , when x ³2 x ®a x ®a x ®a

lim f ( x )
1 1 f(x) x ® a
(a) ,1 (b) - ,2 (iv) lim = , lim g( x ) ¹ 0 (quotient rule)
2 2 x ®a g( x ) lim g( x ) x ® a
x ®a
(c) -1, 1 (d) None of these
lim g ( x ) lim { f ( x ) - 1} g ( x )
(v) lim [ f ( x )] g ( x ) = é lim f ( x )ù
x ®a
Sol. (b) lim f ( x) = lim (mx - 3) = 2m - 3; = ex ® a
x ® 2- x ® 2- x ®a ëêx ® a ûú
x 2
lim f ( x) = lim = (vi) lim g[ f ( x )] = g é lim f ( x )ù , provided g is continuous
x ® 2+ x ® 2+ m m x ®a êëx ® a úû
lim f ( x) exists when lim f ( x) = lim f ( x) function at x = lim f ( x )
x ®2 x ® 2- x ® 2+
x ®a
Limits and Derivatives 447

(vii) lim log f ( x ) = log é lim f ( x )ù , provided lim f ( x ) > 0 x4 - 1 x3 - k3


Example 3. If lim = lim 2 , then k is
x ®a ëêx ® a ûú x ®a x ®1 x -1 x ® k x - k2
lim f ( x )
(JEE Main 2019)
(viii) lim e f ( x ) = ex ® a (a)
4
(b)
3
(c)
3
(d)
8
x ®a
3 8 2 3
(ix) If f ( x ) £ g( x ) for every x excluding a, then
x4 - 1 x3 - k3
lim f ( x ) £ lim g( x ). Sol. (d) Given, lim = lim 2
x ®1 x - 1 x ® k x - k2
x ®a x ®a
( x - 1)( x + 1)( x2 + 1) ( x - k)( x2 + k2 + xk)
(x) If f ( x ) £ g( x ) £ h( x ) for every x excluding a, then Þ lim = lim
x ®1 x -1 x ®k ( x - k)( x + k)
lim f ( x ) = l = lim h( x ), then lim g( x ) = l.
x ®a x ®a n® a 3k2
Þ 2 ´2 =
(xi) lim | f ( x )| =| lim f ( x )| 2k
x ®a x ®a
8
1 Þ k=
(xii) If lim f ( x ) = + ¥ or - ¥, then lim =0 3
x ®a x ®a f(x)
(xiii) lim ( mx + c) = ma + c
Method of Rationalisation
x ®a Rationalisation method is used when, we have radical
n
(xiv) lim [ f ( x )] = [ lim f ( x )] n 1 1
signs in an expression (like , etc.,) and there exists a
x ®a x ®a 2 3
n negative sign between two terms of an algebraic
(xv) lim x =n a
x ®a expression.
After rationalisation, the terms are factorised which on
Evaluation of Algebraic Limits cancellation gives the required result.
Let f ( x ) be an algebraic function (polynomial or rational) 3x - a - x + a
Example 4. The value of lim is
and ‘a’ be a real number, then lim f ( x ) is known as an x® a x-a
x ®a
algebraic limit. 1 1 1 1
(a) - (b) (c) (d)
x3 - 1 1+ x - 1-x 2a 2a 3a 2
e.g. lim , lim etc.
x ®1 x - 1 x ® 0 x 3x - a - x + a
Sol. (b) lim
The limit of algebraic functions can be evaluated by the x ®a x-a
following methods. 3x - a - x + a 3x - a + x + a
= lim ´
x ®a x-a 3x - a + x + a
Method of Direct Substitution
(3x - a) - ( x + a)
lim f ( x ) can be evaluated by method of direct = lim
x ®a x ®a ( x - a)( 3 x - a + x + a)

substitution, if f ( x ) exists finitely for x = a [rationalising the numerator]


2 2 2( x - a)
x + 2x + 3 (1) + 2(1) + 3 6 = lim
e.g. lim = = =2 x ® a ( x - a)( 3 x - a + x + a)
x ®1 x2 + x + 1 (1)2 + 1 + 1 3
2
=
Method of Factorisation 3a - a + a + a
2 1
f(x) = =
Consider lim . If by putting x = a the rational 2 2a 2a
x ®a g( x )
f( x) 0 ¥
function takes the form , etc., then ( x - a ) is a 1 + 1 + y4 - 2
g( x ) 0 ¥ Example 5. lim
y ®0 y4 (JEE Main 2019)
factor of both f ( x ) and g( x ). In such a case we factorise 1
the numerator and denominator and then cancel out (a) exists and equals
the common factor ( x - a ). After cancelling out the 4 2
common factor ( x - a ), we again put x = a in the given (b) does not exist
1
expression and see whether we get a meaningful (c) exists and equals
number or not. This process is repeated till we get 2 2
a meaningful number. 1
(d) exists and equals
2 2( 2 + 1)
448 JEE Main Mathematics

1 + 1 + y4 - 2 If f ( x ) £ g ( x ) £ h( x ), " x Î ( a , b) - { a }
Sol. (a) Clearly, lim
y ®0 y4 and lim f ( x ) = l = lim h(x)
x ®a x ®a
1 + 1 + y4 - 2 1 + 1 + y4 + 2 then, lim g ( x ) = l , where a Î( a, b).
= lim ´ x ®a
y ®0 y4 1 + 1 + y4 + 2 [r] + [2r] + K + [nr]
[rationalising the numerator]
Example 7. The value of lim , where r is
n2n®¥

(1 + 1 + y 4 ) - 2 non-zero real number and [r] denotes the greatest integer less
= lim than or equal to r, is equal to (JEE Main 2021)
y ®0
y 4( 1 + 1 + y 4 + 2)
r
[Q( a + b) ( a - b) = a2 - b 2] (a) (b) r (c) 2r (d) 0
2
1 + y4 - 1 1 + y4 + 1 Sol. (a) r £ [r ] < r + 1
= lim ´
y ®0 4
y ( 1 + 1 + y + 2) 4 1 + y4 + 1 2r £ [2r ] < 2r + 1
3r £ [3r ] < 3r + 1
[again, rationalising the numerator]
M M M
y4
= lim nr £ [nr ] < nr + 1
y ®0
y 4( 1 + 1 + y 4 + 2) ( 1 + y 4 + 1)
r + 2r + K + nr £ [r ] + [2r ] + K + [nr ] < (r + 2r + K + nr) + n
1 n (n + 1) n (n + 1) r
= ×r +n
2 2 ´2 2 [r ] + [2r ] + K + [nr ] 2
£ <
[by cancelling y 4 and then by direct substitution] n2 n2 n2
1 [Q divide by n 2 ]
=
4 2 n (n + 1) r r
Now, lim =
n ®¥ 2n 2 2
Evaluation of Limits Using Standard Results n (n + 1) r
+n
xn - an 2 r
lim = lim ( x n - 1 + x n - 2a + x n - 3 a + . . . + a n - 1 ) and lim 2
=
x ®a x - a x ®a n ®¥ n 2
[using expansion] So, by Sandwich theorem, we can conclude that
xn - an n -1 n -1 n -1 [r ] + [2r ] + K + [nr ] r
Þ lim =a +a +a + . . . + an - 1 lim =
x ®a x - a n ®¥ n2 2
xn - an
\ lim = na n - 1
x ®a x - a Some Useful Expansions
Many limits can be evaluated very easily by applying
( x + 2) 5/3 - ( a + 2) 5/3 expansion series. Some of the standard expansions are
Example 6. lim is equal to
x®a x-a
x 2 x3
5 3 3 5 (i) ex = 1 + x + + +K
(a) ( a + 2) 2/3 (b) ( a + 3)5 /3 (c) ( a + 3)1/ 2 (d) ( a + 2)3 / 2 2! 3!
3 5 5 3
x2 x3
( x + 2)5 /3 - ( a + 2)5 /3 (ii) a x = 1 + x log a + (loge a )2 + (loge a )3 + K
Sol. (a) lim 2! 3!
x ®a x- a
( x + 2)5 /3 - ( a + 2)5 /3 y5 /3 - b5 /3
x 2 x3 x 4
= lim = lim , (iii) log(1 + x ) = x - + - + ... ,|x| < 1
x ®a ( x + 2) - ( a + 2) x ®a y-b 2 3 4
n( n - 1) 2
where x + 2 = y and a + 2 = b (iv) (1 + x )n = 1 + nx + x +K
2!
5 5 5
= b5 /3 -1 = b 2/3 = ( a + 2) 2/3 x3 x5
3 3 3 (v) sin x = x - + - ... , x Î R
3! 5!
Squeeze Play Theorem x2 x4
(vi) cos x = 1 - + - ... , x Î R
(Sandwich Theorem) 2! 4!
x3 2x5
(vii) tan x = x + + + ...
y = h(x) 3 15
y = g(x)
12 3 12 × 32 5 12 × 32 × 52 7
y = f(x) (viii) sin-1 x = x + x + x + x + K,
3! 5! 7!
| x| < 1
-1 x3
x=a x=a x=b (ix) cos x = x - + K ;| x| < 1
6
Limits and Derivatives 449

x3 x5 Example 8. Let [ x] denote the greatest integer less than or


(x) tan-1 x = x - + - K ;| x| < 1
3 5 tan( p sin2 x) + (| x| - sin( x[ x])) 2
n n equal to x. Then, lim
x -a x2
(xi) = x n - 1 + x n - 2a + x n - 3 a 2 + ... + a n - 1 x® 0
x-a (JEE Main 2019)
1/ xì x 11 2 ü (a) equals p (b) equals p + 1
(xii) (1 + x ) = e í 1 - + x -K ¥ý
î 2 24 þ (c) equals 0 (d) does not exist
x2 5 4 61 6 p tan( p sin 2 x) + (| x| - sin( x [ x])) 2
(xiii) sec x = 1 + + x + x + K ¥ ;| x| < Sol. (d) At x = 0, RHL = lim
2 12 720 2 x ® 0+ x2
1 x 7 3 31 5 tan( p sin 2 x) + ( x - sin( x × 0)) 2
(xiv) cosec x = + + x + x + K ¥; = lim
x 6 360 15120 x ®0 +
x2
0 <| x| < p
1 x x3 2 5 é Q| x| = x for x > 0 ù
(xv) cot x = - - - x - K ¥ ; 0 <| x| < p êand [ x] = 0 for 0 < x < 1ú
x 3 45 945 ë û
p 1 1 1 1 tan ( p sin 2 x) + x2
(xvi) tan-1 x = ± - + 3 - 5 + 7 - K ¥ ; = lim
2 x 3x 5x 7x x ® 0+ x2
ì + if x ³ 1 æ tan( p sin 2 x) p sin 2 x ö
í = lim çç . + 1÷÷
î - if x £ - 1 x ® 0+ è p sin 2 x x2 ø
tan ( p sin 2 x) sin 2 x
Evaluation of Trigonometric Limits = p lim
x ® 0+ p sin 2 x
. lim
x ® 0+ x2
+1
To evaluate trigonometric limits, reduce the terms of the tan x
é ù
function in terms of sin q and cos q. Remove the positive êQ xlim ®0 x
=1 ú
and negative signs in between two terms i.e., express the = p +1 ê ú
function in product form. Arrange terms and take help of êand lim sin x = 1 ú
the following standard results. êë x ®0 x úû
2 2
tan ( p sin x) + (| x| - sin ( x [ x])
Trigonometric Limits and LHL = lim
x ® 0- x2
sin x x tan ( p sin 2 x) + ( - x - sin( x ( - 1)) 2
(i) lim = 1 = lim = lim
x ®0 x x ® 0 sin x
x ®0 -
x2
tan x x
(ii) lim = 1 = lim é Q| x| = - x for x < 0 ù
x ®0 x x ® 0 tan x êand [ x] = - 1 for - 1 < x < 0 ú
ë û
sin-1 x x
(iii) lim = 1 = lim tan( p sin 2 x) + ( x + sin( - x)) 2
x ®0 x x ® 0 sin-1 x = lim
x ®0 -
x2
tan-1 x x tan( p sin x) + ( x - sin x) 2
2
(iv) lim = 1 = lim = lim
x ®0 x x ® 0 tan-1 x x ® 0- x2
[Q sin ( - q) = - sin q]
sin x ° p
(v) lim = æ tan( p sin 2 x) + x2 + sin 2 x - 2x sin x ö
x ®0 x 180 = lim çç ÷
-
x ®0 è x2 ÷
(vi) lim cos x = 1 ø
x ®0
æ tan( p sin 2 x) sin 2 x 2x sin x ö
sin( x - a ) = lim çç + 1+ - ÷
(vii) lim =1 x ® 0- è x2 x2 x2 ÷ø
x ®a x-a
æ tan ( p sin 2 x) p sin 2 x
tan( x - a ) = lim çç . +1
(viii) lim =1 -
x ®0 è p sin 2 x x2
x ®a x-a
sin 2 x sin x ö
(ix) lim sin-1 x = sin-1 a ,|a| £ 1 -2
+ ÷
x ®a x2
x ÷ø
(x) lim cos-1 x = cos-1 a ,|a| £ 1 tan( p sin 2 x) p sin 2 x
x ®a = lim . lim
x ® 0- p sin 2 x x ® 0- x2
(xi) lim tan-1 x = tan-1 a , - ¥ < a < ¥
x ®a sin 2 x sin x
+1 + lim 2
- 2 lim
sin x cos x x ®0 -
x x ®0 - x
(xii) lim = lim =0
x ®¥ x x ®¥ x = p + 1+ 1- 2 = p
sin 1/ x Q RHL ¹ LHL
(xiii) lim =1
x ®¥ 1/ x \ Limit does not exist.
450 JEE Main Mathematics

Example 9. If a is the positive root of the equation, æ x3 ö 1æ 2 ö 1


= 1 + çç x + + K÷÷ + ç x2 + x4 + K÷ + ( x3 + K )
1 - cos( p( x)) è 3 ø 2è 3 ø 6
p( x) = x 2 - x - 2 = 0, then lim is equal to
x® a + x+ a -4 x2 æ 1 1 ö
(JEE Main 2020) =1+ x + + x3 ç + ÷ + K
1 1 3 3 2 è3 6ø
(a) (b) (c) (d) x2 1 3
2 2 2 2 =1+ x + + x +K
2 2
Sol. (c) Given equation p( x) = x2 - x - 2 = ( x - 2)( x + 1) having a
x2 x3
positive root a, so a = 2. and ex = 1 + x + + +K
p( x) ö½ 2! 3!
½ 2 sinæç
½
÷½
1 - cos( p( x)) æ 1 1ö
Now, lim = lim ½
è 2 ø½ \ etan x - ex = x3 ç - ÷ + K
x ®a + x+ a -4 x ®2+ x-2 è2 3ø
x3
é æ ( x - 2)( x + 1) ö ù = + Higher powers of x
ê sinç ÷ú 6
è 2 ø ú [Q sin q > 0, if q ® 0 + ]
= lim ê 2 x3 2 5
x ®2+ ê x-2 ú Similarly, tan x - x = + x + Higher powers of x
êë úû 3 15
3
x
( x 2 )( x 1) + Higher powers of x
é æ - + ö ù etan x - ex 1
ê sinç ÷ ú \ lim = lim 63 =
= lim ê 2 è 2 ø ´ æ x + 1ö ú = 2 æ 3 ö = 3 x ® 0 tan x - x x ®0 x 2
ç ÷ ç ÷ + Higher powers of x
+
x ®2 ê ( x - 2)( x + 1) è 2 øú è2ø 2 3
êë 2 úû
Logarithmic Limits
Exponential Limits x 2 x3
2 3 We use the series log(1 + x ) = x - + - … ¥,
x x 2 3
We use the series ex = 1 + x + + + …¥
2! 3! where -1 < x £ 1 and expansion is true only, if base is e.
ex - 1 ax - 1 (i) lim
log(1 + x )
=1 (ii) lim loge x = 1
(i) lim =1 (ii) lim = loge a
x ®0 x x ®0 x x ®0 x x®e

e lx - 1 loge (1 - x )
(iii) lim = l, where ( l ¹ 0) (iii) lim = -1
x ®0 x x ®0 x
loga (1 + x )
( 1 + x2 + x4 - 1)/ x (iv) lim = loga e, a > 0, ¹ 1
x( e - 1) x ®0 x
Example 10. lim
x® 0 2 4 (JEE Main 2020) (v) If lim f ( x ) exists and positive , then
1+ x + x -1
x ®a
(a) is equal to e (b) is equal to 1 lim f ( x ) log f ( x )
lim [ f ( x )]f ( x ) = ex ® a
(c) is equal to 0 (d) does not exist x ®a
2 4 2 4
x( e( 1+ x + x –1)/ x
– 1) e( 1+ x + x –1)/ x
–1 ln(cos 2 x) sin2 2 x
Sol. (b) lim
x ®0 = lim
x ®0 =1 Example 12. Let p = lim , q = lim and
1 + x2+ x4 – 1 ( 1 + x2+ x4 – 1) / x x® 0 3x 2 x® 0 x(1 - e x)
é (1 + x2 + x4)1/ 2 – 1 ey – 1 ù x -x
iml
= 0 and lim
= 1ú r = lim . Then p, q, r satisfy
êQ x ®0
x
y ®0
y x®1 ln x
ë û
(a) p < q < r (b) q < r < p (c) p < r < q (d) q < p < r
e tan x - e x ln(1 + cos 2x - 1)
Example 11. lim is equal to Sol. (d) Clearly, p = lim
x ® 0 tan x - x x ®0 3x2
1 1 ln(1 + cos 2x - 1) cos 2x - 1 2
(a) 1 (b) (c) (d) 0 = lim × 2
=-
2 3 x ®0 (cos 2x - 1) 3x 3
tan 2 x sin 2 2x 4x2
Sol. (b) Since, etan x = 1 + tan x + +K q = lim × = -4
2! x ®0 4x2 x (1 - ex)
x3 2 5 x-x x(1 - x)
But we know that, tan x = x + + x +K and r = lim = lim
3 15 x - 1) x®1 æ1 + x - 1ö
x ®1 ln(1 +
é ì 2 lnç ÷ × ( x - 1)
x3 ü 1ì x3 ü è x -1 ø
\ etan x = ê1 + íx + + Ký + íx + + Ký
ê î 3 þ 2î 3 þ x (1 - x ) 1
ë = lim =-
3 ù x ®1 æ 1 + ( x - 1) ö 2
1ì x3 ü lnç ÷ × ( x - 1)(1 + x)
+ í x + + Ký + ...ú è x -1 ø
6î 3 þ ú
û Hence, q < p < r.
Limits and Derivatives 451

1 æç 3 x 2 + 2 - 7 x 2 - 2 ö÷
Exponential Limits of the Form 1 ¥ lim
x ®0 x 2 çè 7x2 + 2 ÷
ø
Þ P=e
To evaluate the exponential limits of the form 1¥ , we use
1 æç 4 x 2 ö÷ -4
the following results, lim - lim
x ®0 x 2 çè 7 x 2 + 2 ÷ø x ®0 7 x 2 + 2
=e =e
If lim f ( x ) = lim g( x ) = 0,
x ®a x ®a On applying limit, we get
f (x ) 1
lim P = e- 4 / 2 = e- 2 =
then lim { 1 + f ( x )}1/ g ( x ) = e x ®a g ( x ) e2
x ®a

or If lim f ( x ) = 1 and
x ®a
lim g( x ) = ¥
x ®a
L’Hospital’s Rule
Then, lim { f ( x )} g ( x ) = lim { 1 + f ( x ) - 1} g( x¢ ) If f ( x ) and g( x ) be two functions of x such that
x ®a x ®a
(i) lim f ( x ) = lim g( x ) = 0
lim { f ( x ) - 1} g ( x ) x ®a x ®a
x®a
=e (ii) both are continuous at x = a
1 x
æ 1ö (iii) both are differentiable at x = a
(i) lim (1 + x ) x = e (ii) lim ç 1 + ÷ = e
x ®0 x ®¥ è xø (iv) f ¢ ( x ) and g¢ ( x ) are continuous at the point x = a ,then
1 x f(x) f¢ (x)
æ lö lim = lim provided that g( a ) ¹ 0
(iii) lim (1 + lx ) x = el (iv) lim ç 1 + ÷ = el x ® a g( x ) x ® a g¢ ( x )
x ®0 x ®¥ è xø
ì 0, 0 £ a <1 Above rule is also applicable, if lim f ( x ) = ¥ and
x ®a
ï 1, a =1 lim g( x ) = ¥.
ï
(v) lim a x = í x ®a

ï ¥, a >1
x ®¥ f¢ (x) 0 ¥
If lim assumes the indeterminate form or and
ïî does not exist, a<0 x ®a g¢ ( x ) 0 ¥
1/ x f ¢ ( x ), g¢ ( x ) satisfy all the conditions embodied in
æ æp öö L’Hospital’s rule, we can repeat the application of this
Example 13. lim ç tanç + x÷÷ is equal to
x ® 0è è4 øø (JEE Main 2020)
f¢ (x) f¢ (x) f¢ ¢ (x)
(a) e (b) 2 (c) 1 (d) e2
rule on to get lim = lim .
g¢ ( x ) x ® a g¢ ( x ) x ® a g¢ ¢ ( x )
1/ x
æ æp öö
Sol. (d) Given, limç tanç + x÷ ÷ [Having 1¥ form] Example 15. Let f : R ® R be a differentiable function
x ®0 è è4 øø
1
lim ì1 é æp ö ù ü
= ex ®0í ê tanç + x÷ - 1ú ý æ 1 + f (3 + x) - f (3) ö x
satisfying f ¢ (3) + f ¢ (2) = 0. Then, limçç ÷ is
îx ë è 4 ø û þ x® 0è 1 + f (2 - x) - f (2) ÷
ø
lim ì1 æ 1 + tan x öü
= ex ®0í ç - 1÷ ý equal to (JEE Main 2019)
î x è 1 - tan x ø þ (a) e (b) e-1 (c) e2 (d) 1
tan A + tan B lim æ 1 2 tan x ö 1
As, tan( A + B) = = ex ®0 çç ´ ÷÷ æ1 + f (3 + x) - f (3) ö x
1 - tan A tan B è x 1 - tan x ø Sol. (d) Let l = lim çç ÷ [1¥ form]
x ®0 è 1 + f (2 - x) - f (2) ÷
ø
é tan x ù
= e2 êëQ xlim = 1ú 1 æç 1 + f (3 + x ) - f (3 ) ö
®0 x û lim ç - 1 ÷÷
x ®0 x è 1 + f ( 2 - x ) - f ( 2) ø
Þ l=e
1/ x2 é 1 + f (3 + x ) - f (3 ) - 1 - f ( 2 - x ) + f ( 2) ù
æ 3x 2 + 2 ö lim ê ú
Example 14. lim çç 2 ÷ is equal to (JEE Main 2020) =e x ®0 ë x(1 + f ( 2 - x ) - f ( 2)) û
x ® 0 7x + 2 ÷
è ø é f (3 + x ) - f ( 2 - x ) - f (3 ) + f ( 2) ù
lim ê ú
1 1 x(1 + f ( 2 - x ) - f ( 2))
(a) e2 (b) e (c) (d) =e x ®0 ë û
e2 e
On applying L’Hopital rule, we get
1/ x 2
æ 3x2 + 2 ö é f ¢( 2 - x ) + f ¢(3 + x ) ù
Sol. (c) Given limit limçç 2 ÷ = P (let), having1¥ form, lim ê
x ®0 ë 1 - xf ¢( 2 - x ) + f ( 2 - x ) - f ( 2) û
ú
x ®0 7 x + 2 ÷ l=e
è ø
1 æ 3x2 + 2 ö
On applying limit, we get
lim ç - 1÷
x ®0 x 2 çè 7 x 2 + 2 ÷
æ
\ P=e ø
ç f ¢( 2) + f ¢(3 ) ö÷
ç 1 - 0 + f ( 2) - f ( 2) ÷
[Q If lim( f ( x)) g( x )
have indeterminant form1 , ¥ l =eè ø
= e0 = 1
x ®a 1
lim g( x )( f ( x ) - 1)
then lim( f ( x)) g( x )
=e x ®a ] æ1 + f (3 + x) - f (3) ö x
So, lim çç ÷ =1
x ®a x ®0 è 1 + f (2 - x) - f (2) ÷
ø
452 JEE Main Mathematics

log log (1 - x 2)
Example 16. lim
x ® 0 log log cos x
is equal to Derivative
Suppose, f is a real valued function, the function defined
(a) 0 (b) 1 f( x + h ) - f( x)
1 by lim , wherever the limit exists is
(c) (d) ¥ h ®0 h
2
defined to be the derivative of f at x and is denoted by
Sol. (b) We have,
f ¢ ( x ). This definition of derivative is also called the first
log log (1 - x2) principle of derivative.
lim (form ¥ / ¥ )
x ® 0 log log cos x
f( x + h ) - f( x)
1 1 Hence, f ¢ ( x ) = lim
× × ( -2x) h
h ®0
log 1 - x ) 1 - x2
2
d
= lim Sometimes f ¢ ( x ) is denoted by [ f ( x )] or if y = f ( x ), it is
x ®0 1 1
× × ( - sin x) dx
log cos x cos x dy
denoted by . This is referred to as derivative of f ( x ) or
x cos x log cos x dx
= 2 lim
x ®0 sin x × (1 - x2) log (1 - x2) y w.r.t. x. It is also denoted by D [ f ( x )]. Further the
x cos x log cos x derivative of f at x = a is denoted by
= 2 lim × lim × lim æ df ö æ df ö
sin x x ® 0 1 - x2 x ® 0 log (1 - x2)
x ®0
ç ÷ or ç ÷
log cos x è dx øa è dx øx = a
= 2 ´ 1 ´ 1 ´ lim (from 0/0)
x ® 0 log (1 - x2)

1 Derivative from First Principle


× ( - sin x)
cos x Let f ( x ) be a function finitely differentiable at every point
= 2 lim
x ®0 1 on the real number line. Then, its derivative is given by
× ( -2x)
1 - x2 the first principle is
f(x + h ) - f(x)
1 æ sin x 1 - x2 ö f ¢ ( x ) = lim .
=2 ´ × lim çç × ÷ =1 h ®0 h
2 x ®0 è x cos x ÷ø x +1
Example 18. Find the derivative of a function f ( x) = by
x -1
Example 17. The value of the constants a and b such that
the first principle.
æ x2 + 1 ö
lim çç - ax - b ÷÷ = 0 are, respectively 2 -2 -2 2
(a) (b) (c) (d)
x®¥
è x +1 ø ( x - 1) 2 ( x - 1) 2 x -1 (1 - x) 2
(a) (1, 1) (b) ( - 1, 1) x+1
(c) (1, - 1) (d) (0 , 1) Sol. (b) Let f ( x ) =
x -1
æ x2 + 1 ö f ( x + h) - f ( x)
Sol. (c) Given, lim çç - ax - b ÷÷ = 0 We have, f ¢ ( x) = lim
h ®0 h
x ®¥
è x+1 ø
x+ h +1 x+1
æ x + 1 - a ( x2 + x ) - b( x + 1) ö
2
-
Þ lim çç ÷ =0
÷ x + h -1 x -1 é x + 1ù
x ®¥
è x+ 1 ø Þ f ¢ ( x) = lim êQ f ( x) = ú
h ®0 h ë x - 1û
æ 2x - a (2x + 1) - b(1) ö
Þ lim ç ÷ =0 ( x + h + 1)( x - 1) - ( x + 1)( x + h - 1)
x ®¥
è 1 ø = lim
h ®0 ( x + h - 1)( x - 1)h
[using L’Hospital’s rule]
( x2 - x + hx - h + x - 1) - ( x2 + hx - x + x + h - 1)
If this limit is zero, then the function = lim
h ®0 ( x + h - 1)( x - 1)h
2x - a (2x + 1) - b = 0
Þ x(2 - 2a ) - ( a + b) = 0 x2 + hx - h - 1 - x2 - hx - h + 1
= lim
Equating the coefficient of x and constant terms, we get h ®0 ( x + h - 1)( x - 1)h
2 - 2a = 0 - 2h
= lim
h ®0 ( x + h - 1)( x - 1)h
and a + b =0
Þ a = 1, b = - 1 -2 -2
= =
( x + 0 - 1)( x - 1) ( x - 1) 2
Limits and Derivatives 453

Differentiation of Some Algebra of Derivative of Functions


Important Functions Let f and g be two functions such that their derivatives are defined in
d a common domain. Then,
(i) ( c) = 0, c is independent of x
dx (i) Derivative of sum of two functions is sum of the derivatives of the
functions.
d
(ii) ( x n ) = nx n - 1 i.e.
d
[ f ( x ) + g( x )] =
d
f(x) +
d
g( x )
dx dx dx dx
d (ii) Derivative of difference of two functions is the difference of the
(iii) (sin x ) = cos x
dx derivatives of two functions.
d d d d
(iv) (cos x ) = - sin x i.e. [ f ( x ) - g( x )] = f(x) - g( x )
dx dx dx dx
(iii) Derivative of product of two functions is given by the following
d ì pü
(v) (tan x ) = sec2 x , í x ¹ np + ý product rule.
dx î 2þ d d d
[ f ( x ) × g( x )] = g( x ) f(x) + f(x) × g( x )
d dx dx dx
(vi) (cot x ) = - cosec2 x , { x ¹ np}
dx (iv) Derivative of quotient of two functions is given by the following
d ì pü quotient rule (whenever the denominator is non-zero)
(vii) ( sec x ) = sec x tan x , í x ¹ np + ý d d
dx î 2þ g( x ) × f(x) - f(x) × g( x )
d é f(x)ù dx dx
\ =
(viii)
d
( cosec x ) = - cosec x cot x , { x ¹ np} dx êë g( x ) úû [g( x )]2
dx
Example 19. Find the derivative of (3 cot x + 4 cosec x).
d
(ix) ( a x ) = a x loge a (a) - 3 cosec2 x - 4 cosec x cot x (b) 3 cosec2 x - 5 cosec x cot x
dx
2
(c) - 3 cosec x + 5 cosec x cot x (d) None of these
d x
(x) ( e ) = ex
dx Sol. (a) Let y = 3 cot x + 4cosec x
d 1 On differentiating w.r.t. x, we get
(xi) (loge x ) = , ( x > 0) d d
dx x (y) = (3 cot x + 4 cosec ( x )
dx dx
d 1 3d d
(xii) (loga x ) = = (cot x ) + 4 ( cosec x)
dx x loge a dx dx

= - 3 cosec2 x - 4 cosec x cot x


454 JEE Main Mathematics

Practice Exercise
ROUND I Topically Divided Problems

Limits of Algebraic Function 8. Find lim f ( x) and lim f ( x), where


x®0 x ®1
1. Let [ t ] denote the greatest integer £ t. If for some ì 2 x + 3, x £ 0
f ( x) = í .
1 - x + |x| î 3( x + 1), x > 0
l Î R - {0, 1}, lim = L, then L is equal to
x®0 l - x + [x] (a) 3, 5 (b) 3, 6 (c) 4, 7 (d) 3, –6
(JEE Main 2020)
æ1 - 1 ö æ1 - 1 ö . . . æ1 - 1 ö
1 ç ÷ç ÷ ç ÷
(a) 1 (b) 2 (c) (d) 0
9. lim è 22 ø è 32 ø è n2 ø is equal to
2
n®0 æ1 - 1 ö æ1 - 1 öKæ1 - 1 ö
( a + 2 x)1/ 3 - (3x)1/ 3 ç ÷ç ÷ ç ÷
2. lim ( a ¹ 0) is equal to è 2 øè 3ø è nø
x®a (3a + x)1/ 3 - ( 4 x)1/ 3 1 1
(JEE Main 2020) (a) - (b) (c) 2 (d) - 2
2 2
4/3 4/3
æ2ö æ2ö ì 1 2 n ü
(a) ç ÷ (b) ç ÷ 10. lim í
è3ø è9ø 2
+ 2
+K+ ý is equal to
n®¥
î1 - n 1-n 1 - n2 þ
1/3 1/3
æ2ö æ2ö æ2ö æ2ö 1 1
(c) ç ÷ ç ÷ (d) ç ÷ ç ÷ (a) 0 (b) - (c) (d) 1
è9ø è3ø è3ø è9ø 2 2
[ x ] + [ 2 x ] + [3x ] + K + [ nx ]
3. lim , where [×] denotes 3 7 + x2 - 3 + x2
n®¥ n2 11. The value of lim is
x ®1 x -1
the greatest integer function, is equal to
x 1 3 1 1
(a) (b) x (a) – (b) (c) - (d)
2 3 2 4 4
(c) 2x (d) None of these x-2 + x - 2
12. The value of lim is
éx + 1 3 ù x®2 x2 - 4
4. If lim ê 2 - ( ax + b) ú = 2 , then
x®¥ x + 1 1 1
ë û (a) 1 (b) 2 (c) (d)
2 4
(a) a = 1 and b = 1 (b) a = 1 and b = - 1
(c) a = 1 and b = - 2 (d) a = 1 and b = 2
Limits of Trigonometric Function
[x] - 3
5. lim , is equal to 1 - cos x
x ® 3 ( x - 3) 13. lim is equal to
x®0 x
(a) 0 (b) 2
1 1
(c) 3 (d) does not exist (a) - (b)
2 2
( x + 1)10 + ( x + 2)10 + K + ( x + 100)10 (c) 0 (d) does not exist
6. lim is equal to
x®¥ x10 + 1010 (1 - cos 2 x) sin 5 x
14. The value of lim is
(a) 0 (b) 1 (c) 10 (d) 100 x®0 x 2 sin 3x
4
x -4 10 3 6 5
7. The value of lim is (a) (b) (c) (d)
2 3 10 5 6
x® 2 x + 3 2x - 8
1
(a) -
8
(b)
7 x 4 × sin æç ö÷ + x 2
5 5 15. lim èxø equals
8 x®¥ 1 + |x |3
(c) (d) None of these
5 (a) 0 (b) - 1 (c) 2 (d) 1
Limits and Derivatives 455

æ 1 - tan x ö ( 1 - sin x) tan ( p cos2 q)


ç ÷ 26. The value of the limit lim is equal to
16. lim è 2ø is equal to q®0 sin ( 2 p sin 2 q)

p æ xö 3 (JEE Main 2021)
2 ç 1 + tan ÷ ( p - 2 x) 1 1 1
è 2ø (a) - (b) - (c) 0 (d)
1 1 2 4 4
(a) (b) 0 (c) (d) ¥
8 32
Limits of Exponential and
sin 2 x
17. lim
x ®0 2 - 1 + cos x
equals Logarithmic Function
(JEE Main 2019)
27. Let p = lim+ (1 + tan 2 x )1/ 2 x , then log p is equal to
(a) 4 2 (b) 2 (c) 2 2 (d) 4 x®0 (JEE Main 2016)
(1 - cos 2 x) (3 + cos x) 1 1
18. lim is equal to (a) 2 (b) 1 (c) (d)
x®0 x tan 4 x (JEE Main 2013) 2 4
1 1 1/ x 1/ x
(a) - (b) (c) 1 (d) 2 é f ( x) ù é f ( x) ù
28. If lim ê1 + x + = e3, then lim ê1 + is
4 2 x ®0
ë x úû x ®0 ë x úû
x cot( 4 x) equal to
19. lim is equal to
x®0 sin 2 x cot 2 ( 2 x) (JEE Main 2019) (a) e (b) e2
(a) 0 (b) 1 (c) 4 (d) 2 (c) e3 (d) None of these
2
sin( cx + bx + a) 29. lim (log 2 2 x) log x 5 is equal to
20. The value of lim (where a and b x ®1
1 xa - 1

a (a) log 2 5 (b) elog 2 5 (c) e (d) 0
2 m
are the roots of ax + bx + c = 0) is x ö
30. lim æç cos ÷ is equal to
æb - a ö æb - a ö 1 c æ 1 1ö m ®¥ è mø
(a) c çç ÷÷ (b) ç ÷ (c) (d) çç - ÷÷
è ab ø è a ø a a è a bø 1
(a) 0 (b) e (c) (d) 1
e
x tan 2 x - 2 x tan x
21. lim is equal to n ( n - 1)
x®0 (1 - cos 2 x) 2 æ n2 - n + 1 ö
31. lim çç 2 ÷ is equal to
1 1 2 n®¥ n - n -1÷
(a) (b) (c) (d) 0 è ø
2 4 3 (a) e (b) e2 (c) e-1 (d) 1
cot x - cos x sin [log e (1 + x)]
22. lim is equal to 32. The value of lim is

p ( p - 2 x) 3 x ® 0 log (1 + sin x)
2
p 1 1 1
(a) 1 (b) (c) (d) 0 (a) 0 (b) (c) (d) 1
2 16 2 4
2
ì 4 æ1ö 2ü 2
33. The value of lim ìí 3 (tan x - sin x) üý x is
2
ïï x sin çè x ÷ø + x ïï x®0 îx
23. lim í þ
ý is
x®¥
ï 1 + |x |3 ï 1
(a) e2 (b) e (c) (d) -1
ïî ïþ e
æ px ö
(a) 2 (b) 1 l
l tan ç ÷
1
(c) –1 (d) does not exist 34. If lim æç 2 - ö÷ è 2l ø
= , then l is equal to
x®lè xø e
ì sin [ x ] p 2
, [x] ¹ 0
24. If f ( x) = ïí [ x ] , where [ x ] denotes the (a) -p (b) p (c)
2
(d) -
p
ïî 0, [x] = 0 1/ x
nx
1 æ + 21/ x + × × × + n1/ x ö÷
greatest integer less than or equal to x, then 35. The value of lim ç is
x®¥ç n ÷
è ø
lim f ( x) equals (a) en (b) n !
x®0
(c) e1/ n (d) None of these
(a) 1 (b) 0 cosec x
(c) –1 (d) does not exist æ 1 + tan x ö
36. The value of lim çç ÷ is
x + 2 sin x x ® 0 1 + sin x ÷
25. lim is è ø
x®0 x 2 + 2 sin x + 1 - sin 2 x - x + 1 1
(a) e (b)
(JEE Main 2019) e
(a) 6 (b) 2 (c) 3 (d) 1 (c) 1 (d) None of these
456 JEE Main Mathematics

sin ( ex - 2 - 1) é 1
sin x ù
37. The value of lim is 47. For x > 0, lim ê(sin x)1/ x + æç ö÷ ú is equal to
x ® 2 log ( x - 1) x®0 èxø
e êë úû
(a) 0 (b) -1 (c) 2 (d) 1 (a) 0 (b) –1 (c) 1 (d) 2
é 1 ù
38. The value of lim êx - x 2 × log æç 1 + ö÷ú is Simple Differentiation
x®¥ë è x øû
3
1 3 1 1ö
(a) (b) (c) (d) 1 48. The derivative of æç x + ÷ is
2 2 3 è xø
pö 3 3
39. If a n Î éê0 , ÷ is a root of equation tan a + cot a = n, (a) 3x 2 + 4 + 3 + 2
ë 4ø x x
3 3
n ³ 2, then lim (sin a n + cos a n ) n is (b) 3x 2 - 4 + 3 - 2
n®¥ x x
(a) 0 (b) 1 (c) 2 (d) e 3 3
(c) 3x 2 + 4 + 3 _ 2
x x x
æ x + 1ö
40. Let l = lim çç ÷ , then { l } (where, { x } denotes the (d) None of the above
÷
x®¥ x -1
è ø
49. The derivative of (3x + 5)(1 + tan x) is
fractional part of x), is
(a) 3x sec 2 x - 6 sec 2 x - 3 + 3 tan x
(a) 8 - e2 (b) 7 - e2
(c) e2 - 6 (d) e2 - 7 (b) 3x sec 2 x - 5 sec 2 x - 3 - 3 tan x
(c) 3x sec 2 x + 5 sec 2 x + 3 + 3 tan x
41. The value of lim (1 + log 2 x cos x) 2 is (d) None of the above
x®0 cos
2
x 5 - cos x
(a) 4 (b) 9 50. The derivative of is
sin x
(c) 289 (d) non-existent
px 5x4 sin x - 1 - x 5 cos x
px ö
tan (a)
42. The value of lim æç tan sin 2 x
2
÷ is
x ®1 è 4 ø 5x4 sin x + 1 - x 5 cos x
-2 -1 (b)
(a) e (b) e (c) e (d) 1 sin 2 x
cosec 2 x cosec 2 x cosec 2 x sin 2 x 5x 4 sin x - 1 - x 3 cos x
43. lim (1 +2 +K+ n ) is
(c)
x®0
sin 2 x
1
(a) 1 (b) (d) None of the above
n
(c) n (d) 0 51. The derivative of ( ax 2 + cot x)( p + q cos x) is
2/ x
æ a x + bx + cx ö (a) ax 2 cot x (- q sin x) + ( p + q cos x)(2ax - cosec 2 x)
44. The value of lim çç ÷
÷
; ( a , b , c > 0) is (b) ax 2 cot x (q sin x) + ( p + q cos x)(2ax - cosec 2 x)
x®0
è 3 ø (c) ax 2 cot x (- q sin x) + ( p - q cos x)(2ax + cosec 2 x)
(a) (abc)3 (b) abc (d) None of the above
(c) (abc)1/ 3 (d) None of these
52. The derivative of ( x 2 sin x + cos 2 x) is
5 3 2 2
45. If f ( x) = ( x - 1) ( x + 1), g ( x) = ( x - 1)( x - x + 1) (a) x 2 cos x + 2x sin x - sin 2x
and h( x) be such that f ( x) = g ( x) h( x), then lim h( x) (b) x 2 cos x + 2x sin x - 2 sin 2x
x ®1
is equal to (c) x 2 cos x + x sin x - 2 sin 2x
(a) 0 (b) 1 (c) 4 (d) 5 (d) x 2 cos x + 2x sin x + 2 sin 2x

46. If l1 = lim+ ( x + [ x ]), l2 = lim- ( 2 x - [ x ]) and 53. The derivative of sin 3 x cos3 x is
x®2 x®2
3 3
cos x (a) sin 2 2x cos 2x (b) sin 2 2x cos 2x
l3 = lim , then 5 4
x ® p / 2 ( x - p / 2) 3 3
(c) sin 2x cos 2 2x (d) sin 2 2x cos 2x
(a) l1 < l2 < l3 4 2
(b) l2 < l3 < l1 1 dy
54. If y = x + , then at x = 1 is
(c) l3 < l2 < l1 x dx
(d) l1 < l3 < l2 1 1
(a) 1 (b) (c) (d) 0
2 2
Limits and Derivatives 457

ROUND II Mixed Bag


Only One Correct Option ( 4 x - 1) 3
9. The value of lim is
x 2 - ax + b x®0 æ xö æ x2 ö
1. If lim = 5, then a + b is equal to sin çç ÷÷ × log ç 1 + ÷
pø ç 3 ÷ø
x ®1 x -1 (JEE Main 2019) è è
(a) 9 p (log e 4) (b) 3 p (log e 4)3
(a) - 4 (b) 1 (c) - 7 (d) 5
(c) 12 p (log e 4)3 (d) None of these
2. Let f ( x) = 5 -|x - 2|and g ( x) =|x + 1,
| x Î R. If f ( x) sin (6 x 2 )
attains maximum value at a and g ( x) attains
10. The value of lim is
x®0 log e [cos ( 2 x 2 - x)]
( x - 1) ( x 2 - 5 x + 6)
minimum value of b, then lim (a) 12 (b) -12 (c) 6 (d) -6
x ® - ab x2 - 6 x + 8
is equal to (JEE Main 2019)
p - 2 sin - 1 x
11. lim- is equal to
(a) 1/2 (b) - 3 / 2 (c) - 1 / 2 (d) 3/2 x ®1 1-x (JEE Main 2019)

3. For each x Î R, let [ x ] be the greatest integer less p 2 1


(a) (b) (c) p (d)
x ([ x ] + |x |) sin [ x ] 2 p 2p
than or equal to x. Then, lim is 1
x ® 0- |x | x 3 × sin æç ö÷ + x + 1
12. The value of lim èxø is
equal to (JEE Main 2019)
x®¥ x2 + x + 1
(a) 0 (b) sin 1
1
(c) - sin 1 (d) 1 (a) 0 (b) (c) 1 (d) None of these
2
4. For each t Î R, let [ t ] be the greatest integer less
13. Let a and b be the distinct roots of ax 2 + bx + c = 0,
than or equal to t. Then,
p 1 - cos ( ax 2 + bx + c)
(1 -|x|+ sin|1 - x|) sinæç [1 - x ] ö÷ then the value of lim
x®a ( x - a) 2
is
lim è2 ø
2
x ® 1+ |1 - x|[1 - x ] a
(a) (a - b )2 (b) 0
(JEE Main 2019) 2
(a) equals 0 (b) does not exist a2 1
(c) (a + b )2 (d) (a - b )2
(c) equals - 1 (d) equals 1 2 2
cot 3 x - tan x (1 - cos 2 x) (3 + cos x)
5. lim is 14. The value of lim is
p p x®0 x tan 4 x

4 cos æç x + ö÷ 1 1
è 4ø (JEE Main 2019) (a) – (b) (c) 1 (d) 2
4 2
(a) 4 2 (b) 4 (c) 8 (d) 8 2
15. Let f : R ® R be such that f (1) = 3 and f ¢ (1) = 6,
tan [ e2 ] x 2 - tan [ - e2 ] x 2 1
6. lim is equal to
x®0 sin 2 x ì f (1 + x) ü x
then lim í ý equals
(a) 15 (b) 14 (c) 13 (d) 12 x®0
î f (1) þ
2 1
f ( 2 h + 2 + h ) - f ( 2)
7. lim [given that f ¢ ( 2) = 6 and (a) 1 (b) e2 (c) e2 (d) e3
h®0 f ( h - h2 + 1) - f (1)
16. For each t Î R, let [ t ] be the greatest integer less
f ¢ (1) = 4 ]
than or equal to t.
3
(a) does not exist (b) is equal to - æ 1 2 15 ö
2 Then, lim xç éê ùú + éê ùú + ¼ + éê ùú ÷
3 x ®0 + è ë x û x
ë û ë x ûø [JEE Main 2018]
(c) is equal to (d) is equal to 3
2 (a) is equal to 0 (b) is equal to 15
1/ x
æ ex log ( 2 x (c) is equal to 120 (d) does not exist (in R)
- 1)
- ( 2 x - 1) x sin x ö÷
8. The value of lim ç is cot x - cos x
x®0 ç ex log x ÷ 17. lim equals
è ø x ®p/ 2 ( p - 2 x) 3 (JEE Main 2017)
1
(a) e (b) log 2 1 1 1 1
e (a) (b) (c) (d)
(c) e log 2 (d) None of these 24 16 8 4
458 JEE Main Mathematics

18. The value of lim (1 + [ x ]) log(tan x )


(where, [×] Numerical Value Type Questions
x ®p/4
x + x 2 + x 3 +. . . . + x n - n
denotes greatest integer function) is 27. If lim = 820, ( nÎ N)
x ®1 x -1
1
(a) 0 (b) 1 (c) e (d) then the value of n is equal to ………
e (JEE Main 2020)
ì tan 2 { x }
ï 2 2
, x >0 28. If lim
ï x - [x]
x®0
19. If f ( x) = í 1 , x =0 ïì 1æ x2 x2 x2 x 2 öïü
í 8ç 1 - cos - cos + cos cos ÷ý = 2 - k, then
ï ç
ïî x 2 4 2 4 ÷øïþ
ï { x } cot{ x } , x < 0 è
î the value of k is .......... . (JEE Main 2020)
where, [ x ] is the integral part of x and { x } is the
3x + 33 - x - 12
fractional part of x, then 29. lim is equal to ……… .
(a) lim f (x) = 1 (b) lim f (x) = cot 1
x ®2 3- x / 2 - 31 - x (JEE Main 2020)
x ®0 x ® 0-
n
æ 1 ö
S æç x - 1 ö÷ × çç x -
-1
(c) cot [ lim f (x)]2 = 1 (d) None of these 30. If f ( x) = ÷, then the value of
x ® 0-
lø è
l=1 è l + 1 ÷ø
æ n sin q ù é n tan q ù ö
20. The value of lim ç éê + ÷ , where [ × ] lim f (0) is ……… .
q®0 èë q úû êë q úû ø n®¥
31. If lim f ( x) exists, is finite, non-zero and
is greatest integer function and n Î N , is x®¥

(a) 2n (b) 2n + 1 é 3 f ( x) - 1 ù
lim ê f ( x) + ú = 3, then the value of
(c) 2n - 1 (d) Does not exist x®¥
ë f 2 ( x) û
[12 x ] + [ 2 2 x ] + × × × + [ n2 x ] lim f ( x) is ……… .
21. The value of lim is x®¥
n®¥ n3 ì x 3 + a, x < -1
x x x ï
(a) x (b) (c) (d) 32. If limit of f ( x) = í bx + x, -1 £ x < 1 exits at x = 1 ,
2 3 6 ï cx 2 ,
î x ³1
22. The value of lim+ log æxö
sin x is equal to
x®0 sin ç ÷ x = - 1 and both are equal, then the value of
è2ø
1 a + b + c is ……… .
(a) 1 (b) 0 (c) 4 (d)
4 2 2 - (cos x + sin x) 3
1 33. If f ( x) = , then the value of
x 5 tan æç 2 ö÷ + 3|x |2 + 7 1 - sin 2 x
23. lim è px ø is equal to é ù
x®- ¥ |x |3 + 7 |x |+ 8 2 ê lim f ( x) ú is ……… .
ë x ® p / 4 û
1
(a) - (b) 0 (c) ¥ (d) does not exist 34. If lim ( x -3 sin 3x + ax -2 + b) exists and is equal to
p x®0
x-4
24. If f ( x) = , then f ¢ (1) is zero, then the value of| 2 ( a + b)|is ……… .
2 x
5 4 c
1 æ 5b ö
(a) (b) (c) 1 (d) 0 35. If lim (1 + ax + bx 2 ) x -1 = e3, then ç ÷
4 5 x ®1 17 ç a + 4 bc ÷ is
è ø
sin ( x + 9) dy
25. If y = , then at x = 0 is equal to ……… .
cos x dx 1
(a) cos 9 (b) sin 9 (c) 0 (d) 1 36. If lim [cos x + a 3 sin ( b6 x)] x = e512 , then the value of
x®0
x2 x100
26. If f ( x) = 1 + x + +... + , then f ¢ (1) is equal to ab2 is ……… .
2 100
1 é tan x ù
(a) (b) 100 37. The value of lim êlog (1 + x) × ú (where, [×]
100 x®0
ë |x| û
(c) does not exist (d) 0
denotes the greatest integer function) is ……… .
Limits and Derivatives 459

Answers
Round I
1. (c) 2. (d) 3. (a) 4. (c) 5. (d) 6. (d) 7. (c) 8. (b) 9. (b) 10. (b)
11. (a) 12. (c) 13. (d) 14. (a) 15. (d) 16. (c) 17. (a) 18. (d) 19. (b) 20. (d)
21. (a) 22. (c) 23. (c) 24. (d) 25. (b) 26. (a) 27. (c) 28. (b) 29. (b) 30. (d)
31. (b) 32. (d) 33. (b) 34. (c) 35. (b) 36. (c) 37. (d) 38. (a) 39. (d) 40. (d)
41. (c) 42. (b) 43. (c) 44. (d) 45. (d) 46. (c) 47. (c) 48. (b) 49. (c) 50. (b)
51. (a) 52. (b) 53. (b) 54. (d)

Round II
1. (c) 2. (a) 3. (c) 4. (a) 5. (c) 6. (a) 7. (d) 8. (b) 9. (b) 10. (a)
11. (b) 12. (c) 13. (a) 14. (d) 15. (c) 16. (c) 17. (b) 18. (b) 19. (d) 20. (c)
21. (c) 22. (a) 23. (a) 24. (a) 25. (a) 26. (b) 27. (40) 28. (8) 29. (36) 30. (1)
31. (1) 32. (0) 33. (3) 34. (3) 35. (1) 36. (8) 37. (0)

1
Round I 1/3 1- 1/3
æ3ö 4 4 3 42/3 24/3 2 æ 2 ö
1. For some l Î R - {0, 1} =ç ÷ 2
= 1
= 5/3 = 5/3 = ç ÷
è4ø (3) 2- 3 3 3 è9ø
3 3
1 - x + |x|
lim =L
x ®0 l - x + [x] 3. We know that, x - 1 < [x] £ x
n
1 -x- x 1-x+ x
Þ L = lim = lim Þ x + 2x + K + nx - n < å [rx] £ x + 2x + K + nx
x ® 0- l -1 x ® 0+ l -0 r =1
n
1 1 x × n (n + 1) x × n (n + 1)
Þ L= = Þ - n < å [rx] £
|l - 1| |l| 2 r =1 2
1 n
Q |l - 1| = |l|Þ = xæ 1ö 1 1 xæ 1ö
2 Þ ç1 + ÷ - < 2
2è nø n n
å [rx] £ ç1 + ÷ .
2è nø
\ L =2 r =1
1/3
(a + 2x) - (3x) 1/3
é0 ù æ 1ö x
x
2. lim , (a ¹ 0) Now, lim
ç1 + ÷ =
x ®a (3a + x)1/3 - (4x)1/3 ê 0 formú n®¥ 2
è nø 2
ë û
xæ 1ö 1 x
Put x = a + h and lim ç1 + ÷ - =
n®¥ 2 è nø n 2
(a + 2a + 2h )1/3 - (3a + 3h )1/3
So, lim Using Sandwich theorem, we find that
h®0 (3 a + a + h )1/3 - (4 a + 4 h )1/3
[x + [2x] + K + [nx]] x
éæ 1/3 1/3 ù lim =
2h ö æ 3h ö x ®¥ n2 2
(3a )1/3 ê ç1 + ÷ - ç1 + ÷ ú
êë è 3a ø è 3h ø úû é x3 + 1 ù
= lim 1/3 1/3 ù 4. Given that, lim ê 2 - (ax + b)ú = 2
h®0 é æ hö æ 4h ö x ®¥ x + 1
(4a )1/3 ê ç1 + ÷ - ç1 + ÷ ú ë û
êë è 4a ø è 4a ø úû
é x3 (1 - a ) - bx2 - ax + (1 - b) ù
Þ lim ê ú =2
é 2h 3h ù x ®¥ x2 + 1
ê 1+ -1 - + higher ú ë û
9a 9a
1/3 ê ú é a (1 - b ) ù
æ3ö
= limç ÷ ê
degree terms ú ê x(1 - a ) - b - x + x2 ú
h®0 è 4 ø ê h 4h ú Þ lim ê ú =2
ê 1 + - 1 - + higher ú x ®¥
ê 1 ú
12 a 12 a 1+ 2
ê ú êë x úû
êë degree terms úû
This limit will exist, if
æ 2 3 ö æ -1 ö 1 - a = 0 and b = -2
1/3 ç - ÷ 1/3 ç ÷
æ3ö ç 9 9 ÷ = æç 3 ö÷ ç 9 ÷
=ç ÷ Þ a = 1 and b = -2
è4ø ç 1 - 4 ÷ è 4 ø ç -3 ÷
ç ÷ ç ÷
è 12 12 ø è 12 ø
460 JEE Main Mathematics

[x] - 3 æ 1öæ 1ö æ 1ö
5. lim ç1 - 2 ÷ ç1 - 2 ÷ ... ç1 - 2 ÷
x ®3 x-3 è 2 ø è 3 ø è n ø
9. lim
Towards the right of x = 3, [x] = 3 n ®0 æ 1ö æ 1ö æ 1ö
ç1 - ÷ ç1 - ÷ ... ç1 - ÷
Þ [x] - 3 = 0, in the right neighbourhood of x = 3 è 2ø è 3ø è nø
[x] - 3 æ 1ö æ 1ö æ 1ö æ 1ö æ 1ö æ 1ö
Þ lim =0 ç1 + ÷ ç1 + ÷ ... ç1 + ÷ ç1 - ÷ ç1 - ÷... ç1 - ÷
2ø è 3ø è nø è 2ø è 3ø è nø
x ®3 + 0 x - 3 = lim è
n ®0 æ 1ö æ 1ö æ 1ö
Towards the left of x = 3, [x] = 2 ç1 - ÷ ç1 - ÷... ç1 - ÷
è 2ø è 3ø è nø
Þ [x] - 3 = - 1, in the left neighbourhood of x = 3
3 4 n+1 n+1 1
[x] - 3 -1 = lim ´ ´ ... ´ = lim =
Þ lim = lim =¥ n ®0 2 3 n n ®0 2 2
x ®3 - 0 x - 3 x ®3 - 0 x - 3
æ 1 2 n ö
[x] - 3 10. lim çç + +K+ ÷
Thus, lim
x ®3 x-3
does not exist. n ® ¥ 1 - n2
è 1 - n2 1 - n 2 ÷ø
1+2+3+K+ n
(x + 1)10 + (x + 2)10 + ... + (x + 100)10 = lim
6. lim n®¥ 1 - n2
x ®¥ x10 + 1010 1
1+
10 10 10 Sn n (n + 1) n 1
éæ 1ö æ 2ö æ 100 ö ù = lim = lim = lim =-
x10 × ê ç1 + ÷ + ç1 + ÷ + ... + ç1 + ÷ ú n ® ¥ 1 - n2 n ® ¥ 2 (1 - n 2 ) n®¥ é 1 ù 2
êë è xø è xø è x ø úû 2 ê 2 - 1ú
= lim ën û
x® ¥ é 10 ù
10
x10 ê1 + 10 ú 3 7 + x2 - 3 + x 2
ë x û 11. lim
x ®1 x-1
éæ 10 10 10
1ö æ 2ö æ 100 ö ù æ0ö
ê ç1 + ÷ + ç1 + ÷ + ... + ç1 + ÷ ú It is an indeterminate ç ÷ form.
êè xø è xø è x ø úû è0ø
= lim ë 10 ù
= 100
x® ¥ é 10 On applying L’ Hospital’s rule, we get
ê1 + 10 ú -2 -1
ë x û 1 1
(7 + x2) 3 × 2x - (3 + x2) 2 × 2x
x4 - 4 (x + 2 )(x - 2 )(x2 + 2) lim 3 2
7. lim = lim x ®1 1 -0
x® 2 x2 + 3 2 x - 8 x ® 2 (x - 2 )(x + 4 2 )
-2 -1
(x + 2 )(x2 + 2) 1 2 2 2
= lim = (8) 3 × 2 - (4) 2 = (2)-2 - (2)-1
x® 2 (x + 4 2 ) 3 2 3 2
2 é1ù 1 1
( 2 + 2 )(2 + 2) 8 2 8 = ê ú- =-
= = = 3 ë4û 2 3
( 2 + 4 2) 5 2 5
x-2 + x - 2
ì 2x + 3, x £ 0 12. lim
8. We have, f (x) = í x ®2
x2 - 4
î3(x + 1), x > 0
æ0ö
At x = 0, RHL = lim f (x ) = lim f (0 + h ) At x ® 2, it is an indeterminate ç ÷ form.
x ®0 + h®0 è0ø
= lim 3(0 + h + 1) On applying L’ Hospital’s rule, we get
h®0
1 1 1é 1 1 ù
= 3 (0 + 0 + 1) = 3 ´ 1 = 3 ×1 + -0 ê + ú
2 x-2 2 x 2 ê x-2 x úû
LHL = lim f (x ) = lim f (0 - h ) lim = lim ë
h ® 0- h®0 x ®2 1 x ®2 x
× 2x
2 x - 2 x+2
= lim 2(0 - h ) + 3 2 x -4
h®0
= 2(0 - 0) + 3 = 3 1 é x + x -2ù
ê ú
Þ RHL = LHL = 3 2 êë x x - 2 úû
= lim
At x = 1, RHL = lim f (x ) = lim f (1 + h ) x ®2 x
+ h®0
x ®1 x-2 x+ 2
= lim 3(1 + h + 1) [putting x = 1 + h]
h®0 1 é x + x -2ù
= lim
ê ú x+2
= 3 (1 + 0 + 1) = 6 x ®2 2
êë x x úû
LHL = lim f (x ) = lim f (1 - h )
x ® 1- h®0 1 é 2 ù
[ putting x = 1 - h] = ê ú 2+2
2 ë2 2 û
= lim 3(1 - h + 1) = 6
h®0 1 é1ù 1
= × ê ú ×2 =
\ LHL = RHL = 6 2 ë2û 2
Limits and Derivatives 461

ì x sin 2 x é0 ù
ïï- 2 sin 2 , x<0 17. Given limit is lim ê 0 formú
13. 1 - cos x = í x ®0 2 - 1 + cos x ë û
ï 2 sin x , x³0 sin 2 x é 2 xù
ïî 2 = lim êQ 1 + cos x = 2 cos 2 ú
x ®0 x ë û
x 2 - 2 cos
- 2 sin 2
1 - cos x 2 1
Therefore, lim = lim =- . sin 2 x sin 2 x
x ® 0- x x ® 0- x 2 = lim = lim
x ®0 æ x ö x ®0 æ xö
2 sin
x 2 ç1 - cos ÷ 2 ´ 2 sin 2ç ÷
1 - cos x 2 1 è 2 ø è4ø
lim = lim = x
x ® 0+ x x ® 0+ x 2 é 2 xù
êQ 1 - cos 2 = 2 sin 4 ú
1 - cos x 1 - cos x ë û
Since, lim ¹ lim , x2 16
x ® 0- x x ® 0+ x = lim = =4 2 [lim sin x = lim x]
2
1 - cos x x ®0 æ xö 2 2 x ®0 x ®0
lim does not exist. 2 2ç ÷
x ®0 x è4ø
(1 - cos 2x)sin 5x 2 sin 2 x sin 5x (1 - cos 2x) (3 + cos x) x
14. lim = lim 18. Let I = lim × ×
x ®0 2
x sin 3x x ®0 x2 sin 3x x ®0 x2 1 tan 4x
æ sin 5x ö 2 sin 2 x 3 + cos x x
ç ÷ = lim × ×
æ 2 sin 2 x ö è x ø x ®0 x2 1 tan 4x
= lim ç ÷
x ®0 ç 2 ÷ æ sin 3x ö 2
è x øç ÷ æ sin x ö 4x
è x ø = 2 lim ç ÷ × lim (3 + cos x) × lim
x ®0 è x ø x ®0 x ® 0 4 tan 4 x

æ sin 5x ö sin q q
2 5 lim ç ÷ Q lim = 1 and lim =1
æ sin x ö x ® 0 è 5x ø
q®0 q q ® 0 tan q
= lim 2 ç ÷ ´
x ®0 è x ø æ sin 3x ö 1
3 lim ç ÷
x ® 0 è 3x ø Þ I = 2 × (1)2 × (3 + cos 0° ) × (1)
4
2 ´ 5 10 é sin x ù 1 1
= = êQ xlim = 1ú = 2 × 1 × (3 + 1) × = 2 × 4 × = 2
3 3 ë ® 0 x û 4 4
æ1ö é æ1ö 1 ù x cot 4x x 1 tan 2 2x
x4 × sin ç ÷ + x2 19. lim = lim .
ê x sin ç x ÷ + x ú 2 2
x ®0 sin x × cot 2 x x ®0 tan 4 x sin 2 x 1
15. lim è xø = lim ê è ø ú 2 2
x ®¥ 1 + | x|3 x ®¥ ê 1 | x|3 ú = lim
1 4x x
.
tan 2x
êë x3 + úû
x3 x ®0 4 (tan 4 x) sin 2 x x2
2 2
[dividing numerator and denominator by x3 ] 1 4x æ x ö æ tan 2x ö 4
= lim ç ÷ .ç ÷ .
æ1ö x ®0 4 (tan 4 x) è sin x ø è 2x ø 1
sin ç ÷
lim è x ø + lim 1 1 4 é x tan x ù
= ×1 ×1 ×1 × êQ lim = 1 = lim
x ®¥ 1 x ®¥ x 4 1 ë x ®0 sin x x ®0 x úû
= x =1
1 | x|3
lim 3 + lim 3 20. Q Roots of ax2 + bx + c = 0 are a and b,
x ®¥ x x ®¥ x
1 -0 1 1
= =1 so roots of cx2 + bx + a = 0 are and .
0+1 a b
æ 1öæ 1ö
16. Put x =
p p
- h as x ® , h ® 0 \ cx2 + bx + a = c ç x - ÷ çç x - ÷÷
2 2 è a øè b ø
æ p hö é æ 1ö æ 1öù
1 - tan ç + ÷ (1 - cos h ) sin ê cç x - ÷ çç x - ÷÷ ú
è4 2ø ë è aø è bøû æ 1ö
Given, limit = lim Now, lim × cçç x - ÷÷
h®0 æ p hö 1 æ 1 öæ 1 ö è bø
1 + tan ç + ÷ (2h )3 x®
a acç x - ÷ çç x - ÷÷
è2 2ø è aø è bø
æ hö
2 × sin 2ç ÷ é ì æ 1 öæ 1 ö üù
æ hö è2ø ê sin íc ç x - ÷ çç x - ÷÷ ýú
= lim tan ç ÷ lim c î è a øè b ø þú æ 1ö
h®0 2
è ø h ® 0 8 h3 = lim ê × lim ç x - ÷÷
a x® 1 ê æ 1öæ 1 ö ú x ® 1 çè bø
æ hö æ hö
2
a ê c ç x - ÷ çç x - ÷÷ ú a
tan ÷ ç sin ÷ êë è a ø è b ø úû
1 çç 2 ÷ ´ lim ç 2 ÷ ´1 = 1
= lim c æ 1 1ö
h®0 4 ç h ÷ h®0 ç h ÷ 4 32 = çç - ÷÷
ç2´ ÷ ç ÷ a è a bø
è 2 ø è 2 ø
462 JEE Main Mathematics

x tan 2x - 2x tan x Q lim f (x) ¹ lim f (x)


21. lim x ® 0- x ® 0+
x ®0 (1 - cos 2x)2
\ lim f (x) does not exist.
ì 8x3 64x5 ü ì x3 2x5 ü x ®0
x í2x + + + Ký - 2x íx + + + Ký
3 15 3 15 x + 2 sin x é0 ù
= lim î þ î þ 25. Let P = lim
x ®0 2 ê 0 formú 2
x ®0 4 sin 4 x x + 2 sin x + 1 - sin x - x + 1 ë û
ìæ 8 2 ö ü On rationalization, we get
x4íç - ÷ + terms containing positive power of xý
î è3 3ø þ (x + 2 sin x)
= lim P = lim 2
x ® 0 x + 2 sin x + 1 - sin 2 x + x - 1
x ®0 4 sin 4 x

=
1
×2 ´
1
=
1 1 1
´ = ( x2 + 2 sin x + 1 + sin 2 x - x + 1 )
4
4 æ sin xö 2 1 2
lim ç ÷ = lim ( x2 + 2 sin x + 1 + sin 2 x - x + 1 )
x ®0 è x ø x ®0
x + 2 sin x
cot x - cos x ´ lim
22. lim x2 - sin 2 x + 2 sin x + x
x ®0

p (p - 2x)3
2 x + 2 sin x é0 ù
= 2 ´ lim 2 ê 0 formú
p p x ® 0 x - sin 2 x + 2 sin x + x ë û
Let x = t + . If x ® Þ t®0
2 2 Now applying the L¢ Hopital’s rule, we get
t3 t3 1 + 2 cos x
t- +K-t- K P = 2 ´ lim
sin t - tan t 1 3 x ® 0 2 x - sin 2 x +2 cos x + 1
lim = lim
t ®0 -8 t3 t ®0 -8 t3 (1 + 2)
=2 [on applying limit]
t3 0 -0 + 2 + 1
-
= lim 2 = 1 3
t ® 0 -8 t3 16 =2 ´ =2
3
x + 2 sin x
é 4 æ1ö 2ù Þ lim =2
ê x sin ç x ÷ + x ú x ®0
x2 + 2 sin x + 1 - sin 2 x - x + 1
23. lim ê è ø ú
x ®¥ ê (1 + | x|3 ) ú tan (p (1 - sin 2 q))
êë úû 26. lim
q ®0 sin (2p sin 2 q)
x3 é æ1ö 1ù - tan (p sin 2 q)
Let L = lim 3 ê
x sin ç ÷+ ú = lim
x ®- ¥ 1 + | x| ë è xø xû q ® 0 sin (2 p sin 2 q)

é ù æ tan (p sin 2 q) ö æ 2p sin 2 q ö 1


x ê 3
1 ú = lim - çç ÷ ç ÷´
é æ1ö 1ù 2 ÷ ç sin (2p sin 2 q) ÷ 2
= lim 3
ê ú ê x sin ç x ÷ + x ú …(i) q ®0
è p sin q ø è ø
x ® - ¥ | x| ê 1 ú ë è ø û
1+ -1
êë | x| x úû =
2
x3 x3 1
Now, lim = lim = - 1 [Q | x| = - x for x < 0]
x ® - ¥ | x| 3 x ® - ¥ - x3 27. Given, p = lim (1 + tan 2 x ) 2x (1¥ form)
x ® 0+
1 tan 2 x 1
2
lim =0 lim lim
æ tan x ö
ç ÷ 1
x ® - ¥ | x| x x ® 0+ 2x 2 x ® 0+ ç x ÷ø
=e =e è = e2
æ1ö sin y é 1ù 1
and lim x sin ç ÷ = lim =1 ê where y = x ú 1
x ®- ¥ è x ø y ® 0- y ë û \ log p = log e2 =
2
æ 1 ö 1/ x
\ From Eq. (i), L = - 1 çç ÷ (1 + 0) = - 1 é f (x) ù
÷
è1 + 0ø 28. We have, limê1 + x + = e3
x ®0 ë x úû
24. The given function can be restarted as ì f ( x) ü 1
lim í 1 + x + -1 ý
ìsin [x] Þ ex ®0î x þx = e3
ï , if x Î (-¥ , 0) È [1, ¥ ] é f ( x)ù
lim ê1 +
f (x) = í [x] 2 ú f (x)
ï 0, Þ ex ®0 ë x û = e3 Þ lim =2
î if x Î [0, 1] x ®0 x2
sin (- h ) sin (-1) 1/ x é f ( x) ù1
f (x) ù lim ê1 + -1 ú
\ lim f (x) = lim = lim = sin 1 é x ûx
- [- h ] (-1) Now, limê1 + = ex ®0 ë
x úû
x ®0 h®0 h®0
x ®0 ë
and lim f (x) = lim 0 = 0 lim
f ( x)
x ® 0+ h®0 2
= ex ®0 x = e2
Limits and Derivatives 463

é æ x5 öù
29. lim (log 2 2x)log x 5 = lim (log 2 2 + log 2 x)log x 5 2 ê2 ç + x7 and higher powers of x ÷ ú
x ®1 x ®1 ç
êë è 8 ÷ú
øû
log 2 x log x 5 lim
é 1 ù
= ex ® 0 x5
= lim ê (1 + log 2 x) log 2 x ú = e1/ 2 = e
x ®1 ê ú
ë û æ px ö
lim log 2 5 l tan ç ÷
x ®1 log 2 5 æ lö è 2l ø 1
=e =e 34. lim ç2 – ÷ =
x ®l è xø e
m m
æ xö é æ x öù æ px ö é px ù é l ù
30. lim ç cos ÷ = lim ê1 + ç cos - 1÷ ú é lù
l tan ç ÷
è 2l ø
lim l tan ê ú ê 2 - - 1 ú
m®¥ è mø m®¥ ë è m øû = lim ê2 – ú = ex ® l ë 2l û ë x û
m x ®l ë xû
é æ x öù é lù
= lim ê1 - ç1 - cos ÷ ú l ê1 - ú
m®¥ ë è møû lim
ë xû
m é px ù é lù x®l é px ù
lim l tan ê ú ê1 - ú cot ê ú
é x ù ë 2l û ë xû ë 2l û
= lim ê1 - 2 sin 2 ú = ex ® l =e
m®¥
ë 2mû
é px ù
é x ù sin 2 ê ú
lim m ê - 2sin 2 l2 / x 2 æ l2 ö ë 2l û
m ®¥ ë 2múû lim lim ç ÷ ( -1 )
=e x®l é px ù p
- cosec 2 ê ú ×
x ® l çè x 2 ÷
ø æ p ö
ç ÷
2
æ x ö ë 2l û 2l è 2l ø
ç sin ÷ æ x2 ö =e =e
lim - 2ç 2m ÷ ç ÷m
m ®¥ ç x/ 2m ÷ ç 4m2 ÷ é px ù é px ù
ç ÷ è ø - l2 sin 2 ê ú sin 2 ê ú
= e è ø ë 2l û æ –2l ö ë 2l û
lim lim ç ÷
x ® l æ 2l ö æ p ö 2 x®lè p ø 2
x2 2 æ px ö
ç ÷ç ÷ x ç ÷
-2 lim è p ø è 2l ø è 2l ø
m ®¥ 4m =e =e
=e = e0 = 1 -2l
n ( n - 1)
æ n2 - n + 1 ö = e p= e-1
31. lim çç 2 ÷
n ®¥ n - n - 1 ÷ 2
æ ö
è ø Þ - l× ç ÷ = -1
n( n - 1 ) è pø
æ n (n - 1) + 1 ö
= lim çç ÷ p
n ®¥ è n (n - 1 ) - 1 ÷
ø \ l=
n( n - 1 )
2
é 1 ù nx
[n (n - 1)]n( n - 1) ê1 + ú é 11/ x + 21/ x + ××× + n1/ x ù
ë n (n - 1 ) û 35. lim ê ú
= lim n( n - 1 )
x ®¥
ë n û
n ®¥ é ù
1
[n (n - 1)]n( n - 1) ê1 - ú 1
ë n (n - 1) û Let y = , then
n( n - 1 ) x
æ 1 ö n/ y
ç1 + ÷ é 1y + 2y + ××× + n y ù
ç n (n - 1) ÷ø e é æ 1ö
n ù = lim ê
= lim è = = e 2
lim 1 + = eú ú
n( n - 1 ) - 1 ê Q ç ÷ y ®0
ë n û
n ®¥ æ
1 ö e êë n ®¥ è nø úû
ç1 - ÷ n é 1 y + 2y + 3 y + ××× + ny ù
ç n (n - 1) ø÷ lim ê - 1ú
è y ® 0 y êë n úû
=e
sin [log e (1 + x)]
32. lim é 1 y + 2y + 3 y + L + ny - n ù
x ® 0 log (1 + sin x) lim ê ú
y ® 0 êë y úû
æ0ö =e
It is an indeterminate ç ÷ form. é æ 1 y - 1 ö æ 2y - 1 ö æ 3 y - 1 ö æ ny - 1 ö ù
è0ø lim ê ç ÷+ ç ÷+ ç ÷ + ××× + ç ÷ú
1 y ® 0 ê çè y ÷ø çè y ÷ø çè y ÷
ø
ç y ÷ú
è øû
=e ë
cos [log e (1 + x)] × × (1)
(1 + x)
= lim = e(log 1 + log 2+ log 3 + ××× + log n) = e log(1×23× ××× n) = n !
x ®0 1
(0 + cos x) cos x ù 1/cos x
(1 + sin x) é
[using L’ Hospital’s rule] ê æç1 + sin x ö÷ sin x ú
1/sin x êè cos x ø ú
cos (log e 1) × (1) cos 0 1 æ 1 + tan x ö ê úû
= = = =1 36. lim çç ÷ = lim ë
1 × cos 0 cos 0 1 x ® 0 1 + sin x ÷ x ®0 (1 + sin x)1/sin x
è ø
2/ x 2 1/ cos x
ì 2 ü é cos x sin x ù
33. lim í1 + (tan x – sin x) – 1ý lim ×
x ® 0 ëê sin x cos x ûú
x ®0 î x3 þ e
[ 2(tan x –sin x ) – x 3 ] ×2
= 1
lim lim sin x ×sin x
=e x®0 x5 x®0
e
é ìï æ x 3 2x 5 ö æ x3 x5 ö üï ù 1
2 ê 2í ç x – + – L÷ – ç x – + L – x3 ÷ýú lim
êë ïî çè 3 15 ÷ ç 3! 5! ÷ïú
lim
ø è øþû ex ® 0 cos x e1
x®0 x5 = lim 1
= =1
=e e1
ex ® 0
464 JEE Main Mathematics

sin (ex - 2 - 1) 2
41. Consider, lim (1 + log cos x/ 2 cos x)
2
37. lim x ®0
x ® 2 log e (x - 1 )

0 é 2ù 2
It is a indeterminate form. æ log e cos x ö
0 = lim ê1 + ç ÷ ú …(i)
x ®0 ê ç log cos x / 2 ÷ ú
ë è e ø û
cos (ex - 2 - 1) × ex - 2 × 1
Þ lim [using L’ Hospital’s rule]
x ®2 1 Now, consider
× (1 - 0)
(x - 1) 1
(- sin x)
x-2
e x-2
- 1) (x - 1)
cos (e log e cos x cos x
= lim lim = lim
x ® 0 log e cos x /2 x ®0 1 1
x ®2 1 (- sin x/2) ´
cos x/ 2 2
= e cos (e0 - 1) (2 - 1) = 1 cos 0 × (1) = 1
0
[using L’ Hospital’s rule]
é æ 1öù
38. lim ê x - x2 × log ç1 + ÷ ú 2 tan x é0 ù
x ®¥ ë x è
øû = lim ê 0 formú
x ® 0 tan x /2 ë û
1
Let y = , then sec2 x
x = 2 lim
1 1 x ®0 x 1
= lim - 2 log (1 + y) sec2 ×
y ®0 y y 2 2
y - log (1 + y) é0 ù [again using L’ Hospital’s rule]
= lim ê 0 formú
y ®0 y2 ë û sec2 x æ1ö
= 4 lim =4ç ÷ =4
1 x ® 0 sec2 x /2 è1ø
1-
1+ y From Eq. (i), we get
= lim [using L’ Hospital’s rule]
y ®0 2y 2 2 2 2
lim (1 + log cos x/ 2 cos x) = (1 + 4 )
y 1 x ®0
= lim = = (1 + 16)2 = 17 = 289
y ® 0 2 y (1 + y) 2 px
tan
æ px ö 2
39. Here, tan a + cot a = n 42. lim ç tan ÷ [1¥ form]
1 2
x ®0 è 4ø
Þ = n Þ sin 2a = …(i) æ px px ö px px
sin a cos a n ç sin
4
- cos
4 ÷
÷ 2 sin
4
cos
4
lim ç
2 æ
lim ç tan
px ö
- 1 ÷ tan
px x ®1 ç px ÷ px
\ sin 2a n = , as a n is root 4 2 ç cos ÷ cos
n = ex ® 1 è ø
= e è 4 ø 2

lim (sin a n + cos a n )n = lim [(sin a n + cos a n )2]n/ 2 lim 2 sin 2 px


- 2 sin
px
cos
px æ px px ö
n®¥ n®¥ ç 1 - cos - sin ÷
x ®1 4 4 4 2 2 ÷
lim ç
= lim [1 + sin 2a n ]n/ 2 cos
px x ®1 ç
ç cos
px ÷
÷
n®¥ 2 è 2 ø
n/ 2 =e = e
é 2ù
= lim ê1 + ú px p px p
n®¥ ë nû 2 sin 2 ´ - cos ´
lim 2 2 2 2
2 n x ®1 px p
lim × - sin ´
=e n ®¥ n 2
=e = e 2 2 = e-1
x
æ x + 1ö 43. Let y = cosec2 x;
40. We have, l = lim çç ÷
x ®¥ x - 1÷
è ø Required limit = lim (1y + 2y + K n y )1/ y [0 / ¥ form]
x x y ®¥
æ x - 1 + 1 + 1ö é (x - 1) + 2 ù 1/y
= lim çç ÷
÷ = lim ê ú éæ 1 öy æ 2 öy æ n - 1ö
y ù
x ®¥
è x-1 ø x ®¥
ë (x - 1) û = lim (n y )1/ y ê ç ÷ + ç ÷ + K + ç ÷ + 1ú
x
y ®¥ êë è n ø è nø è n ø úû
é 2 ù
= lim ê1 + ú éæ 1 öy æ 2 öy y ù
1/y
x ®¥
ë (x – 1) û æ n - 1ö
= lim n ê ç ÷ + ç ÷ + K + ç ÷ + 1ú
Clearly, it is of the form 1¥ and we know if lim f (x) = 1 y ®¥ êë è n ø è nø è n ø úû
x ®a
lim {f ( x ) - 1 }g( x ) = n × 10 = n × 1 = n
g( x ) x®a
and lim g (x) = ¥, then lim [ f (x)] =e 2/ x
x ®a x ®a æ a x + bx + cx ö
44. Let y = lim ç ÷
ìæ 2 ö ü x ®0 ç 3 ÷
lim í çç 1 + ÷ - 1ý x è ø
x ®¥îè ( x - 1 ) ÷ø þ
\We have, l=e æ a x + bx + cx ö
2
2x Þ log y = lim log ç ÷
lim lim 2 x ®0 x ç 3 ÷
x ® ¥ x -1
e = ex ® ¥ [using L’ Hospital’s rule] è ø
=e 2 log(a x + bx + cx ) - log 3
= 2 lim
Now, { l} = { e2} = {7.389} = 0.389 = e2 - 7 x ®0 x
Limits and Derivatives 465

Apply L’Hospital’s rule, d æ 3 1 3ö


= ç x + 3 + 3x + ÷
a x log a + bx log a + cx log a dx è x xø
a x + bx + cx 3 3
= 2 lim = 3x2 - 4 + 3 - 2
x ®0 1 x x
log y = log (abc) 23
/
49. Let y = (3x + 5)(1 + tan x)
Þ y = (abc) 23
/
On differentiating w.r.t. x, we get
45. Given, f (x) = (x5 - 1) (x3 + 1) dy d d
= (3x + 5) (1 + tan x ) + (1 + tan x ) (3x + 5)
and g (x) = (x2 - 1) (x2 - x + 1) dx dx dx
Q f (x) = g (x) h (x) = (3x + 5)(0 + sec2 x ) + (1 + tan x )(3)
f (x) = 3x sec2 x + 5 sec2 x + 3 + 3 tan x
\ h (x) =
g (x) x 5 - cos x
5 3 50. Let y =
(x - 1) (x + 1) sin x
lim h (x) = lim
x ®1 x ®1 (x2 - 1) (x2 - x + 1) On differentiating w.r.t. x, we get
5
(x - 1) (x + 1) (x - x + 1) 2 d d
= lim sin x (x 5 - cos x) - (x 5 - cos x ) (sin x )
dy dx dx
x ®1 (x - 1) (x + 1) (x2 - x + 1) = 2
dx sin x
(x5 - 1) é0 ù
= lim ê 0 formú sin x (5x4 + sin x ) - (x 5 - cos x )(cos x )
x ®1 (x - 1) ë û =
sin 2 x
5 x4
= lim [using L’Hospital’s rule] 5x4 sin x + sin 2 x - x 5 cos x + cos 2 x
x ®1 1 =
sin 2 x
5(1)4
= =5 5x 4 sin x + 1 - x5 cos x
1 =
sin 2 x
46. l1 = lim x + [x] = lim 2 + h + [2 + h ] = 4
x ® 2+ h®0 51. Let y = (ax2 + cot x )( p + q cos x )
l2 = lim (2x - [x]) = lim {2(2 - h ) - (2 - h )} dy d
= (ax2 + cot x ) ( p + q cos x )
x ® 2- h®0
dx dx
d
= lim {2(2 - h ) - 1} = 3 + ( p + q cos x ) (ax2 + cot x )
h®0 dx
l3 = lim
cos x
= lim - sin x = - 1 = (ax2 + cot x )(0 - q sin x ) + ( p + q cos x )
p p x®p (2ax - cosec2 x )

2
x- 2
2 [using L’Hospital’s rule] 2
= (ax cot x )(- q sin x ) + ( p + q cos x )
Thus, l3 < l2 < l1 (2ax - cosec2 x )
sin x
æ1ö 2
æ1ö
sin x log ç ÷ 52. Let y = x sin x + cos 2x
47. Here, lim (sin x)1/ x + lim ç ÷ = 0 + lim e èxø
dy d 2 d
x ®0 x ® 0 è xø x ®0
= (x sin x ) + (cos 2x )
dx dx dx
é ù
êëQ xlim (sin x)1/ x ® 0 as 0 < sin x < 1ú d d 2
®0 û = x2 (sin x ) + sin x (x ) + (-2 sin 2x)
dx dx
æ 1 ö
log (1/ x )
x çç -
è x2 ø
÷÷
= x2 cos x + 2x sin x - 2 sin 2x
lim lim
=e x ® 0 cosec x
=e x ® 0 - cosec x cot x 8 (sin 2x )3
53. Let y = sin3 x cos3 x ´ =
8 8
[using L’Hospital’s rule]
dy 1 2 d
sin x
lim tan x
= 3(sin 2x) (sin 2x )
x®0 x dx 8 dx
=e = e0 = 1
3 3
3 = (sin 2 2x ) ´ cos 2x (2) = sin 2 2x cos 2x
æ 1ö 8 4
48. Let y = ç x + ÷
è xø 1
54. Given, y = x +
On differentiating w.r.t. x, we get x
3 dy 1 1
dy d æ 1ö On differentiating w.r.t. x, we get = -
= çx + ÷ dx 2 x 2x3/ 2
dx dx è xø
dy 1 1 1 1
d ì 3 1 æ 1öü At x = 1, = - = - =0
= íx + 3 + 3 ç x + ÷ ý dx 2 1 2(1 )3/ 2 2 2
dx î x è xø þ
466 JEE Main Mathematics

æp ö
Round II (1 - |h + 1| + sin|- h|) sin ç [- h ]÷
2 è 2 ø
x - ax + b = lim
1. It is given that lim =5 …(i) h ® 0+ |- h|[- h ]
x ®1 x-1
æp ö
Since, limit exist and equal to 5 and denominator is zero (1 - (h + 1) + sin h ) sin ç [- h ]÷
è 2 ø
at x = 1 , so numerator x2 - ax + b should be zero at x = 1, = lim
h ® 0+ h [- h ]
So 1 - a + b =0 Þ a =1 + b …(ii)
[Q |- h| = h and|h + 1 | = h + 1 as h > 0]
On putting the value of ‘a’ from Eq. (ii) in Eq. (i), we get
æp ö
x2 - (1 + b) x + b ( - h + sin h ) sin ç (- 1)÷
lim =5 è 2 ø
x ®1 x-1 = lim
h ® 0+ h (- 1)
(x2 - x) - b(x - 1) [Q [x] = - 1 for - 1 < x < 0 and h ® 0+ Þ - h ® 0- ]
Þ lim =5
x ®1 x-1 (- h + sinh) æ - pö
(x - 1) (x - b) = lim sin ç ÷
Þ lim =5 h®0 + -h è 2 ø
x ®1 x-1 (- h + sin h ) sin h - h
= lim (-1) = lim
Þ lim (x - b) = 5 h ® 0+ -h h ® 0+ h
x ®1
æ sin h ö æ hö
Þ 1 - b =5 Þ b = -4 …(iii) = lim ç ÷ - lim+ ç ÷ = 1 - 1 = 0
h ® 0+ è h ø h®0 è hø
On putting value of ‘b’ from Eq. (iii) to Eq. (ii), we get é sin h ù
a = -3 êQ lim+ = 1ú
ë h®0 h û
So, a+ b=-7
cot3 x - tan x
2. Given functions are f (x) = 5 - |x - 2| 5. Given, limit = Lt
x ® p/ 4 æ pö
cos ç x + ÷
and g (x) = | x + 1|, where x Î R. è 4ø
Clearly, maximum of f (x) occurred at x = 2, so a = 2. 1 - tan 4 x 1 é 1 ù
and minimum of g (x) occurred at x = - 1, so b = - 1. = Lt
x ® p/ 4 1
´
tan3
x êQ cot x = tan x ú
(cos x - sin x) ë û
Þ ab = - 2 2
(x - 1) (x2 - 5x + 6) (1 - tan 2 x) 2 (1 + tan 2 x)
Now, lim = Lt ´
x ® - ab x2 - 6 x + 8 tan3 x
x ® cos x - sin x
p
(x - 1) (x - 3) (x - 2) 4
= lim [Q ab = - 2]
(x - 4) (x -2) cos 2 x - sin 2 x 2 (sec2 x)
x ®2
= Lt ´ [Q 1 + tan 2 x = sec2 x]
(x - 1) (x - 3) x®
p cos x - sin x cos 2 x tan3 x
= lim 4
x ®2 (x - 4) (cos x - sin x) (cos x + sin x) 2 sec4 x
= Lt ´
(2 - 1) (2 - 3) 1 ´ (- 1) 1 x®
p (cos x - sin x) tan3 x
= = = 4
(2 - 4) (- 2) 2
[Q (a 2 - b2) = (a - b) (a + b)]
x([x] + | x|) sin [x] x([x] - x) sin [x] 4
2 sec x
3. lim = lim = Lt (cos x + sin x)
x ®0 - | x | x ®0 - -x

p tan3 x
4
[Q | x| = - x, if x < 0]
x(- 1 - x) sin (- 1) 2 ( 2 )4 æ 1 1 ö
= lim (Q lim [x] = - 1) = ç + ÷ [on applying limit]
x ® 0- -x x ® 0- (1)3 è 2 2ø
- x(x + 1) sin(- 1)
= lim = lim (x + 1)sin(- 1) æ 2 ö
x ®0 - -x x ® 0-
=4 2 ç ÷ =8
è 2ø
= (0 + 1) sin (- 1) (by direct substitution)
tan [ e2] x2 - tan [- e2] x2
= - sin 1 [Q sin(- q) = - sin q] 6. lim
x ®0 sin 2 x
æp ö
(1 - |x| + sin|1 - x|) sin ç [1 - x]÷ tan 7 x2 + tan 8 x2
è2 ø = lim (Q 7 < e2 < 8)
4. Given, lim x ®0 sin 2 x
x ® 1+ |1 - x|[1 - x]
Put x = 1 + h , then tan 7x2 tan 8x2
2
7+ 8
x ® 1+ Þ h ® 0+ = lim 7x 8x2
x ®0 2
æp ö æ sin x ö
(1 - |x| + sin|1 - x|) sin ç [1 - x]÷ ç ÷
è 2 ø è x ø
\ lim 7+8
x ® 1+ |1 - x|[1 - x] = = 15
1
Limits and Derivatives 467

f (2 h + 2 + h 2) - f (2) lim p - 2 sin -1 x


7. lim 11. Let L = , then
h ®0 f (h - h 2 + 1) - f (1) x® 1 -
1-x
f (2 h + 2 + h 2) - f (2) lim p - 2 sin -1 x p + 2 sin -1 x
´ (2 + 2 h ) L= ´
(2 + 2 h ) x® 1 -
= lim 1-x p + 2 sin -1 x
h ® 0 f (h - h 2 + 1 ) - f (1 )
´ (1 - 2 h ) [on rationalization]
(1 - 2 h )
lim p - 2 sin -1 x 1
f ¢ (2 h + 2 + h 2)(2 + 2 h ) = ´
= lim x ® 1- 1-x p + 2 sin -1 x
h ®0 f ¢ (h - h 2 + 1)(1 - 2 h )
æp ö
f ¢ (2) 2 6 ´ 2 p - 2ç - cos -1 x÷
= × = =3 lim è2 ø ´ 1
f ¢ (1) 1 4 ´ 1 =
x ® 1- 1-x p + 2 sin -1 x
1/ x
ìï ex log( 2x -1) - (2x - 1)x sin x üï pù
8. lim í é -1 -1
x®0 ex log x
ý êQ sin x + cos x = 2 ú
ïî ïþ ë û
1/ x lim 2 cos -1 x lim 1
ì (2x - 1)x - (2x - 1)x sin x ü = ´
= lim í ý x ® 1- 1-x x ® 1- p + 2 sin -1 x
x®0
î xx þ
1 lim 2 cos -1 x é lim -1 pù
2x - 1 = - êQ - sin x= ú
= lim lim (1 - sin x)1/ x 2 p x® 1 1-x ë x® 1 2û
x®0 x x®0
log 2 Put x = cos q, then as x ® 1- , therefore q ® 0+
= log 2 × e-1 = lim
e 1 2q
Now, L = +
(4x - 1)3 2 p q® 0 1 - cos q
9. lim
x ®0 æ xö æ x2 ö 1 lim 2q é 2 qù
sin çç ÷÷ × log ç1 + ÷ = + êQ 1 - cos q = 2 sin 2 ú
è pø ç 3 ÷ø 2 p q® 0 æ qö ë û
è 2 sin ç ÷
æ0ö è2ø
It is an indeterminate ç ÷ form.
è0ø æ qö
2× ç ÷
3 1 è2ø
æ 4x - 1 ö = × 2 lim
lim ç ÷ × x3 2 p q®0 + æ qö
x ®0 ç x ÷ sin ç ÷
= è ø è2ø
æ xö æ x2 ö
sin çç ÷÷ log ç1 + ÷÷ 1 2 é lim q ù
ç 3 ø æ x2 ö = 2 2 = = 1ú
lim è p ø × æç x ö÷ × lim è × çç ÷ 2 p p
êQ x ® 0+
sin q
ë û
x ®0 æ x ö çè p ÷ø x ® 0 x2 ÷
è3ø
çç ÷÷
è pø 3 æ1ö
x3 × sin ç ÷ + x + 1
è xø 1
x3 (log e 4)3 12. We have, lim Put = y
= lim = 3 p (log e 4)3 y ®¥ x2 + x + 1 x
x ®0 x3
1 1 1 é sin y ù
3p × sin( y) + + 1 ê + y + y2ú
y3 y y2 ë y û
sin (6x2) Þ lim = lim
10. lim y ®0 1 1 x ®0 1 é 2 ù
x ®0 log e [cos (2x2 - x)] + + 1 1 + y + y
y2 y y2 ëê ûú
cos (6x2) × 12x sin y
= lim
x ®0 1 + y + y2
[- sin (2x2 - x)] × (4x - 1) y
= lim =1
cos (2x2 - x) y ®0 1 + y + y2
12 [x cos (6x2)]
= lim 1 - cos (ax2 + bx + c)
x ®0 sin (2x2- x) 13. lim
(1 - 4x) x ®a (x - a )2
cos (2x2 - x)
As a and b are two distinct roots.
12x cos (6x2)
= lim \ ax2 + bx + c = a (x - a ) (x - b )
x ®0 sin (2x2 - x) (2x2 - x)
2
× (1 - 4x)
(2x - x) cos (2x2 - x) -b ± b2 - 4ac
i.e. a,b =
12 cos 6x2 2a
= lim = 12
x ®0 (2x - 1) 1 - cos [(x - a ) (x - b ) a ]
(1 - 4 x) = lim
cos(2x2 - x) x ®a (x - a )2
468 JEE Main Mathematics

[(x - a ) (x - b ) a ] cot x - cos x 1 cos x(1 - sin x)


2 sin 2 17. lim = lim ×
= lim 2 (p - 2x)3
x ® p/ 2 x ® p/ 2 8 æp ö
3

x ®a (x - a )2 sin xç - x÷
è2 ø
[(x - a ) (x - b )a ] æp öé æp öù
2 sin 2 2 cos ç - h ÷ ê1 - sin ç - h ÷ ú
2 2 æ x - bö 1
= lim a ç ÷ è2 øë è2 øû
x ®a 2
è 4 ø = lim ×
é (x - a ) (x - b ) a ù h®0 8 æp ö æp p ö
3
ê
ë 2 ú
û sin ç - h ÷ ç - + h ÷
è2 ø è2 2 ø
2 2 1 sin h (1 - cos h )
= lim a (x - b )2 = lim
x ®a 4 8 h®0 cos h × h3
a2 æ hö
sin h ç2 sin 2 ÷
æ hö
sin h × sin 2ç ÷
= (a - b )2
2 1 è 2ø 1 è2ø
= lim = lim
(1 - cos 2x) (3 + cos x) 8 h®0 cos h × h3 4 h®0 h3 cos h
14. lim 2
x ®0 x tan 4x æ hö
1 ç sin ÷
é [1 - (1 - 2 sin 2 x)] (3 + cos x) ù æ sin h ö ç 2 ÷ × 1 ×1 = 1 ´1 = 1
= lim ç ÷
= lim ê ú 4 h ® 0è h ø ç h ÷ cos h 4 4 4 16
x ®0
ë x tan 4x û ç ÷
è 2 ø
2 1
æ sin x ö
2 2ç ÷ (3 + cos x) log (tan x )
(2 sin x) (3 + cos x) x ø 18. We have, lim (1 + [x])
= lim = lim è x®
p
x ®0 æ tan 4x ö x ®0 æ tan 4x ö 4
xç ÷ × 4x 4ç ÷ 1
è 4x ø è 4x ø æ ép ùö é æp öù
Now, lim f (x) = lim çç1 + log ê tan ç - h ÷ ú
æ sin x ö
2
p- p ê 4 - h ú ÷÷ ë è4 øû
lim ç ÷ × lim (3 + cos x) x® x® -hè ë ûø
4
2 x ®0 è x ø x ®0 2 4
= = ×4 = 2 1
4 tan 4x 4 é æp ö ù
lim = lim (1 + 0) 0 êQ ç 4 - h ÷ = 0ú
x ®0 4x h®0 ë è ø û
1/ x
lim í
1 ì f (1 + x ) ü
- 1ý
= 1¥ = 1
ì f (1 + x) ü x®0 xî f (1 ) þ 1
15. lim í ý =e æ
x ® 0 î f (1 ) þ ép ùö æp ö
log tan ç + h ÷
and lim f (x) = lim çç1 + ê + h ú ÷÷ è4 ø
ì f (1 + x ) - f (1 ) ü p+ x ®
p
+ h è ë 4 ûø
lim í x® 4
ý 4
x ® 0î x f (1 ) þ
=e é æp ö ù
= lim 1¥ = 1 êQ ç 4 + h ÷ = 0ú
ì f (1 + x ) - f (1 ) 1 ü h®0
lim í × ý ë è ø û
x ® 0î x f (1 ) þ
= e \ lim f (x) = 1
ì f (1 + x ) - f (1 ) ü 1 x ®p/4
lim í ý× tan 2 { x} tan 2 x
=e x ® 0î x þ f (1 ) 19. We have, lim f (x) = lim 2 2
= lim =1
x ® 0+ x ® 0 + x - [x] x ® 0+ x2
= ef ¢(1)/ f (1) = e6/3 = e2 [Q x ® 0+ ; [x] = 0 Þ { x} = x]
æé1ù é2ù é 15 ù ö Also, lim f (x) = lim { x} cot { x}
16. We have, lim xçç ê ú + ê ú + ¼+ ê ú ÷÷ x ® 0- x ® 0-
x ®0 + è ë x û ë xû ë x ûø éQ { x} = x - [x] ù
= cot 1 ê ú
We know, [x] = x - { x} -
ëÞ x ® 0 Þ { x} = 1 – h = 1}û
é 1 ù 1 ì1 ü 2
\ ê xú = x - íxý é ù
ë û î þ Also, cot- 1 ê lim f (x)ú = cot- 1 ( cot 1 )2
-
ëx ®0 û
é n ù n ìn ü
Similarly, ê xú = x -íxý = cot- 1 (cot 1) = 1
ë û î þ
æ é n sin q ù é n tan q ù ö
æ 1 ì1 ü 2 ì2 ü 15 ì15 üö 20. We have, lim çç ê + ÷
\Given limit = lim xçç - í ý + - í ý + … - í ý÷÷ q®0 èë q úû êë q úû ÷ø
x ®0 + è x îxþ x îxþ x î x þø
sin q é sin q ù
æì1 ü ì2 ü ì15 üö We know that, lim =1 êQ q < 1ú
= lim (1 + 2 + 3+ ...+15)- x ççí ý + í ý + ... + í ý÷÷ q®0 q ë û
x ®0 + èî x þ î x þ î x þø é n sin q ù
Þ lim ú = n-1
é ìn ü ù q®0 ê ë q û
êQ 0 £ í x ý < 1, therefore ú æ n tan q ö é tan q ù
ê î þ ú Similarly, lim ç
= 120 - 0 = 120
ê ú q®0 è q ø
÷=n êQ q > 1ú
ìn ü ìn ü ë û
ê 0 £ xí ý < x Þ lim xí ý = 0 ú
ë î x þ x ®0 +
î x þ û æ é n sin q ù é n tan q ù ö
\ lim çç ê + ÷ = n - 1 + n = 2n - 1
q®0 èë q úû êë q úû ÷ø
Limits and Derivatives 469

[12 x] + [22 x] + ××× + [n 2x] ì 1 ü


21. lim tan í 2 ý
3 1
n®¥ n × î (px ) þ + 3 + 7
n 2x - 1 < [n 2x] £ n 2x, p 1 x x3
2
Putting n = 1, 2, 3, L , n, we get (px )
= lim
x ®- ¥ 7 8
x Sn 2 - n < S [n 2x] £ xSn 2 -1 - 2 + 3
x x
Sn 2 1 S [n 2x] Sn 2
x 3 - 2< £ x …(i) 1
n n n3 n3 ×1 + 0 + 0
1
ì Sn 2
1ü Sn 2
1 = p =-
Þ lim íx × 3 - 2 ý = x × lim 3 - lim 2 -1 -0 + 0 p
n®¥
î n n þ n ® ¥ n n ® ¥ n
é 1 tan q ù
= x × lim
n (n + 1) (2n + 1)
-0 êQ x ® - ¥ Þ px2 ® 0 and qlim
®0 q
= 1ú
ë û
n®¥ 6n3
x-4
æ 1ö æ 1ö 24. Given, f (x) =
ç1 + ÷ ç2 + ÷ 2 x
n nø
= x × lim è øè
On differentiating w.r.t. x, we get
n®¥ 6
1
2 x x (1 ) - (x - 4)
= x× = 2 x
6 3 f ¢ (x ) =
2 ( x )2
Sn 2 x
and lim x = 1
n ® ¥ n3 3 1 (1 ) - (1 - 4)
x 2 1
As two limits are equal i.e. . \ f ¢ (1 ) =
3 2 ( 1 )2
æ xö 3
log sin ç ÷ 1+
22. lim log æ x ö (sin x) = lim è2ø = 2 =5
x ® 0+ sin ç ÷ x ® 0+ log sin x 2 4
è 2ø
sin (x + 9)
æ hö 25. Given, y =
log sin ç ÷ cos x
= lim è2ø
h ® 0 log sin h
On differentiating w.r.t. x, we get
dy cos x cos (x + 9) - sin (x + 9) sin x
1 æ hö 1 =
cos ç ÷ × dx (cos x)2
æ hö è2ø 2
sin ç ÷ cos (x + 9 + x )
è 2ø =
= lim (cos x)2
h®0 1
× cos h
sin h cos (2x + 9)
=
h (cos x)2
2 æ hö dy cos 9
cos ç ÷ At x = 0, = = cos 9
æ hö è2ø dx (cos 0)2
sin ç ÷
2
è ø
= lim =1 x2 x100
h®0 æ h ö 26. Given, f (x) = 1 + x + + ... +
ç ÷ cos h 2 100
è sin h ø
On differentiating w.r.t. x, we get
æ 1 ö 100x99
x5 tan ç 2 ÷ + 3|x|2 + 7 f ¢ (x) = 0 + 1 +
2x
+ ... +
23. We have, lim è px ø 2 100
x ®- ¥ |x|3 + 7|x| + 8 = 1 + x + x2 + .. . + x99
æ 1 ö \ f ¢ (x) = 1 + 1 + 12 + . . . + 199 = 100
x5 tan ç 2 ÷ + 3x2 + 7
= lim è px ø x + x2 + x3 + K + xn - n
x ®- ¥ - x3 - 7x + 8 27. Given, lim = 820, (n Î N )
x ®1 x-1
[Q x < 0 = |x| = - x]
ì x - 1 x2 - 1 x3 - 1 xn - 1 ü
æ 1 ö 3 7 Þ lim í + + + ... + ý = 820
x2 tan ç 2 ÷ + + 3 x ®1 x - 1
î x-1 x-1 x-1 þ
è px ø x x
= lim n (n + 1)
x ®- ¥ 7 8 Þ 1 + 2 + 3 + ... + n = 820 Þ = 820
-1 - 2 + 3
x x 2
On dividing the numerator and denominator by x3 , Þ n (n + 1) = 1640 = 40 ´ 41 Þ n = 40
470 JEE Main Mathematics

ìï 1 æ 2 2 2 2öü é 3 f (x) - 1 ù
28. Given, lim í ç1 - cos x - cos x + cos x cos x ÷ ïý = 2-k 31. If lim f (x) = l, then lim ê f (x) + ú =3
x ® 0 ï x8 ç
è 2 4 2 4 ÷ø ïþ x ®¥ x ®¥ ë f 2(x) û
î
2ö 2ö 3l - 1
1 æ æ l+ =3
Þ lim ç1 - cos x ÷ ç1 - cos x ÷ = 2- k Þ
l2
x ®0 x8ç 2 ø çè
÷ 4 ÷ø
è
Þ l3 + 3l - 1 = 3l2
æ x2 ö x2
2 sin 2ç ÷÷ . 2 sin 2 Þ l - 3 l2 + 3 l - 1 = 0
3
ç4 8
Þ lim è ø = 2-k Þ (l - 1)3 = 0
8
x ®0 x Þ l=1
ìé 2 8
ïê sin x
2
x2 ù x üï \ lim f (x) = 1
sin ú 2 x ®¥
Þ
ï
lim 4 íê 2 4 ´ 8 ú (32) ï = 2- k 32. Since, lim f (x) = lim f (x)
2 8 ý x ®1 x ® -1
x ®0
ïê x / 4 x ú x ï
ïêë 8 úû ï Here, lim f (x) exists.
î þ x ®1
4 Þ lim f (x) = lim f (x)
Þ = 2-k x ® 1+ x ® 1-
210 Þ c=b+1 …(i)
Þ 2 - k = 2 -8 Also, lim f (x) exists.
x ® -1
Þ k =8 Þ lim f (x) = lim f (x)
3x + 33 - x - 12 x ®-1 + x ®-1 -
æ0 ö
29. lim ç form÷ Þ -b - 1 = a - 1 …(ii)
x ®2 3 - x/ 2 - 31 - x è0 ø
From Eqs. (i) and (ii), if both limits are equal, then
Put x = 2 + h as x ® 2 Þ h ® 0
b + 1 =0
32 + h + 31 - h - 12 9 × 3h + 3 × 3- h - 12 Þ c = 0, 1 - a = 0
= lim h
= lim
h®0 h®0 1 - h/ 2
3
-1 -
2 - 3 -1 - h (3 - 3-h ) \ a = 1, b = - 1, c = 0
3
Þ a + b + c=0
9(3(3h - 1) + (3- h - 1)) 2 2 - (cos x + sin x)3
= lim 33. Given, f (x) =
h®0 3- h (3h/ 2 - 1) 1 - sin 2x
é æ 3h - 1 ö æ -h ö ù
ê 3çç ÷ h + ç 3 - 1 ÷ (- h ) ú Now, lim f (x) = lim
2 2 - (cos x + sin x)3 é0 ù
h ø ÷ ç
è -h ø
÷
1 - sin 2x ê 0 formú
= lim 9 × 3h êê è ú p p ë û
x® x®
h®0 æ h /2 - 1 ö h ú 4 4
ê ç h/ 2 ÷ ú -3 (cos x + sin x)2 (- sin x + cos x)
êë è 3 ø2 úû = lim

p -2 cos 2x
h
é æ3 - 1ö æ3 - 1ö ù - h 4
ê 3çç ÷-ç ÷ú
-3 (cos x + sin x) (cos 2 x - sin 2 x)
hê è h ÷ø çè - h ÷ø ú = lim
= lim 9 × 3 ê ú p -2 cos 2x
h®0 1 æç 3h/ 2 - 1 ö÷ x®
4
ê ú
êë 2 çè h /2 ÷ø úû -3 (cos x + sin x) cos 2x
= lim

p -2 cos 2x
é ù 4
ê 3 log e 3 - log e 3 ú æ3 - 1ö 3 (cos x + sin x)
= 9 ´1 ê ú = 9ç ÷ = 36 = lim
1 è 1 /2 ø p 2
ê log e 3 ú x®
ë 2 û 4
3 æ 1 1 ö 3
ah - 1 ×ç =+ ÷=
[Q lim 3h = 30 = 1 and lim = log e a] 2 è 2 2ø 2
h®0 h®0 h
n 3
æ 1ö æ 1 ö Then, 2 ´ lim f (x) = 2 × =3
30. We have, f (x) = S
ç x - ÷ çç x - ÷ x®
p 2
l =1 è lø è l + 1 ÷ø 4
n
1 1 1 1 sin 3x + ax + bx3
\ f (0) = S = + + ××× + 34. Given, lim =0
l =1 l (l + 1) 1 ´ 2 2 ´ 3 n (n + 1) x ®0 x3
æ 1ö æ1 1ö æ1 1 ö By using L’ Hospital’s rule, we get
= ç1 - ÷ + ç - ÷ + ××× + ç - ÷
2ø è2 3ø ç n n + 1÷
è è ø 3 cos 3x + a + 3bx2 é0 ù
1 lim =0 ê 0 formú
=1 - x ®0 3 x2 ë û
n+1
\ 3 + a =0
Thus, lim f (0) = 1
n®¥
Limits and Derivatives 471

Again, using L’ Hospital’s rule, we get 36. Given, lim [1 + cos x + a3 × sin (b6x) - 1]1/ x = e512
x ®0
-9 sin 3x + 6bx
Þ lim =0 a3 × sin (b 6x ) + cos x - 1
lim
x ®0 6x x
Þ ex®0
= e512
æ 9 sin 3x ö
Þ lim ç - × + b÷ = 0 a3 × sin (b6x) + cos x - 1
x ®0 è 2 3x ø Þ lim = 512
9
x ®0 x
Þ - + b =0 a3 cos (b6x) × b6 - sin x
2 Þ lim = 512
9 x ®0 1
\ b= and a = - 3
2 Þ a3 × b6 = 512
Þ |2 (a + b)| = 3 \ ab2 = 8
c é tan x ù
35. Given, lim (1 + ax + bx ) 2 x -1
= e3 37. We have, lim ê log (1 + x) × ú
x ®1
x ®0
ë | x| û
2
( ax + bx ) × c tan x
lim Let f (x) = log (1 + x) ×
x ®1 ( x - 1) 3 | x|
Þ e =e
2 tan (- h )
c × (ax + bx ) lim f (x) = lim log (1 - h ) ×
Þ lim =3 x ® 0– x ® ( 0 - h) |- h|
x ®1 x-1
(- tan h )
For limit to exists, c (a + b) = 0 …(i) = lim log (1 - h ) × =0
h®0 h
c (a + 2bx) tan h
Þ lim =3 Þ lim f (x) = lim log (1 + h ) ×
x ®1 1 x ® 0+ x ® ( 0 + h) |h|
[using L’ Hospital’s rule] tan h
= lim log (1 + h ) × =0
Þ c (a + 2b) = 3 …(ii) h®0 h
From Eqs. (i) and (ii), we get é tan x ù
Q lim ê log (1 + x) × ú = [0] = 0
b = - a and bc = 3 x ®0
ë | x| û
1 æ 5b ö 1
\ ç + 4bc÷÷ = (5 + 12) = 1
17 çè a ø 17
19
Mathematical
Reasoning
Statement (Proposition) IN THIS CHAPTER ....
We convey our daily views in the form of sentence which is a collection of Sentence
words. This collection of words is called statements, if it has some sense.
Statement
Therefore,
Open Statement
‘A declarative sentence, whose truth or falsity can be decided is called a
statement of logical sentence but the sentence should not be imperative, Truth Table
interrogative and exclamatory.’ Logical Connective or Sentencial
Statements are denoted by p, q , r ,.. etc. Connectives

The working nature of statement in logic is same as nature of switch in Elementary Operations of Logic
circuit. Converse, Inverse and
ON (1) True (T) Contrapositive of an Implication
i.e. Switch and Statement Tautology
OFF (0) False (F) Contradiction

e.g. ‘Delhi is the capital of India’ is a statement, while ‘Do your work’, is not a
statement. There are following types of statements.

Simple Statement
A statement, if cannot be broken into two or more sentences, is a simple
statement. The truth value of the simple statement does not explicitly depend
on any other statement. Generally, small letters p, q , r, . . . denote simple
statements.

Compound Statement
A statement formed by two or more simple statement by the words such as
‘and’, ‘or’, ‘not’, ‘if then’, ‘if and only if ’, then the resulting statement is called
a compound statement. This is also called Logical connectives.
Note Two statements can be connected either by ‘Ù’ or ‘Ú’.
Mathematical Reasoning 473

Substatements (iii) For three statements p, q and r,


Simple statements which when combined form a Number of Rows = 23 = 8
compound statement are called substatements, also p q r
called components.
T F F
Note T F T
• A true statement is known as a valid statement. T T F
• A false statement is known as an invalid statement. T T T
• Imperative, exclamatory, interrogative, optative sentences are not F F F
statements. F F T
• Mathematical identities are considered to be statements because F T F
they can either be true or false but not both. F T T
e.g. Identify the following sentence is/are not statement? If a compound statement has simply n substatements,
(a) The sum of 5 and 7 is greater then 10. then there are 2n rows representing logical possibilities.
(b) Answer this question e.g. If there are 6 simple statements, then for making a
(c) The product of (– 1) and 8 is 8. table, the number of rows are
(d) All real numbers are complex numbers. We know that, if compound statements has
Here, n substatements, then there are 2n rows in a table.
(a) It is true that sum of 5 and 7 is greater than 10. Here, n = 6
Hence, it is a statement. \ Total number of rows = 26 = 64
(b) It is an order. Hence, it is not a statement.
(c) It is true that sum of interior angles of a triangle is Logical Connectives or
180°. Hence, it is a statement. Sentencial Connectives
(d) It is true that all real numbers are complex Two or more statements are combined to form a
numbers. All real number can be expressed as a + ib. compound statement by using symbols. These symbols
Hence, it is a statement. are called logical connectives.
Logical connectives are given below.
Open Statement Words Symbols
A sentence which contains one or more variables such (i) Ù
that when certain values are given to the variable it and
becomes a statement, is called an open statement. (ii) or Ú
(iii) implies that (if ..., then) Þ
eg, “He is a great man” is an open sentence because in
(iv) If and only if (implies and is implied by) Û
this sentence “He” can be replaced by any person.

Truth Table Elementary Operations of Logic


A table that shows the relationship between the truth Formation of compound sentences from simple sentence
value of compound statement, S ( p, q , r , . . . ) and the truth using logical connectives are termed as elementary
values of its substatements p, q , r , . . . etc., is called the operation of logic. There are five such operations
truth table of statement S. discussed below
(i) For a single statement p, number of rows = 21 = 2
Negation (Inversion) of Statement
p A statement which is formed by changing the truth value
T
of a given statement by using word like ‘no’ or ‘not’ is
called negation of a given statement. It is representalley
F
the symbol ‘~’
(ii) For two statements p and q, number of rows = 22 = 4 e.g. Let p : the number 2 is greater than 7.
Then, ~ p : the number 2 is not greater than 7.
p q If p is statement, then negation of p is denoted by ‘ ~ p’.
T T The truth table for NOT is given by
T F p ~p
F T
T F
F F F T
474 JEE Main Mathematics

Conjunction The above disjunction can be explained through circuit.


A compound sentence formed by two simple sentences Here, denote I » T , O » F
p and q using connective ‘and’ is called the conjunction of A1
p and q and is represented by p Ù q.
e.g. Let p : Ramesh is a student A2
and q : Ramesh belongs to allahabad. Then,
p Ù q º Ramesh is a student and he belongs to Allahabad.
The truth table for operation ‘and’ is given by Result
A1 A2 p q pÚq
(Bulb)
p q p Ùq
1 0 1 T F T
T T T 1 1 1 ¾¾® T T T
T F F 0 1 1 F T T
0 0 0 F F F
F T F
F F F Note
• The statement p Ú q is true, if atleast one of p and q or both are
Above conjunction can be explained through circuit. true.
Here, denote 1 » T , 0 » F • The statement p Ú q is false, if both p and q are false.
A1 A2
Implication (Conditional)
A compound sentence formed by two simple sentences p
and q using connective ‘if ... then ...’ is called the
implication of p and q and represented by p Þ q
which is read as ‘ p implies q’.
pÚ q
Here, p is called antecedent or hypothesis and q is called
consequent or conclusion.
A1 A2 Result p q pÙq e.g. Let p : Train reaches in time.
(Bulb)
and q = I can attend the meeting. Then,
1 0 0 T F F
p Þ q º if train reaches in time, then I can attend the
1 1 1 ¾¾® T T T meeting.
0 1 0 F T F
The truth table for if … then is given by
0 0 0 F F F
p q pÞq ~p ~p Ú q
Note
• The statement p Ù q is true, if both p and q are true. T T T F T
• The statement p Ù q is false, if atleast one of p and q or both are T F F F F
false.
F T T T T
Disjunction (Alternation) F F T T T
A compound sentence formed by two simple sentences
It is clear from the truth table that column III is equal to
p and q using connective ‘or’ is called the disjunction of column V
p and q and is represented by p Ú q.
i.e. Statement p Þ q is equivalent to ~ p Ú q.
e.g. Let p : Bus left early and q : My watch is going slow.
Then, p Ú q = Bus left early or my watch is going slow. Biconditional Statement
The truth table for ‘operation’ OR is given by Two simple sentences connected by the phrase ‘if and
only if,’ form a biconditional statement. It is represented
p q p Úq by the symbol ‘Û’.
T T T e.g. Let p : DABC is an isosceles triangle
T F T and q : two sides of a triangle are equal.
F T T Then, p Û q : DABC is an isosceles triangle if and only if
F F F two sides of a triangle are equal.
Mathematical Reasoning 475

The truth table for if and only if is given by ( p Ù q) Ù ( ~ p Ú q) º ( p Ù q)

p q p Û q ~p ~ q ~p Ú q p Ú~ q (~ p Ú q ) Ù (p Ú ~ q)
and
T T T F F T T T
T F F F T F T F [ take Ú » È and Ù » Ç]
F T F T F T F F (b) Consider, Å = Ù and × = Ù.
F F T T T T T T In that case, we get
( p Ù q) Ù ( ~ p Ù q) º null set
Note
• It is clear form the truth table that column III is equal to column VIII. and
i.e., statement p Û q is equivalent to (~ p ~ Úq ) Ù( pÚ ~ q )
• The statement p Û q has true, if either both are true or both are false.
(c) Consider, Å = Ú and × = Ù.
• The statement p Û q has false, if exactly one of them is false.
In that case, we get
( p Ú q) Ù ( ~ p Ù q) º ( ~ p Ù q)
Table for Basic Logical Connections
p q ~p pÙq qÚq pÞq pÛq
and
T T F T T T T
T F F F T F F
(d) Consider, Å = Ú and u = Ú.
F T T F T T F In that case, we get
F F T F F T T ( p Ú q) Ù ( ~ p Ú q) º q

Example 1. The logical statement ( p Þ q) Ù (q Þ ~ p) is and


equivalent to (JEE Main 2020)
(a) ~p (b) q (c) p (d) -q
Example 4. Consider the following three statements:
Sol. (a) From the truth table
P : 5 is a prime number.
p q pÞq ~p q Þ~ p (p Þ q) Ù (q Þ ~ p) Q : 7 is a factor of 192.
T T T F F F R : LCM of 5 and 7 is 35.
Then, the truth value of which one of the following
T F F F T F
statements is true ? (JEE Main 2019)
F T T T T T
(a) (P Ù Q) Ú (~R) (b) P Ú (~Q Ù R)
F F T T T T (c) (~P) Ú (Q Ù R) (d) (~P) Ù (~Q Ù R)
( p Þ q) Ù (q Þ ~ p) is equivalent to~p. Sol. (b) Since, the statements
P : 5 is a prime number, is true statement.
Example 2. If p ® ( p Ù~ q) is false, then the truth values
Q : 7 is a factor of 192, is false statement
of p and q are respectively (JEE Main 2020)
and R : LCM of 5 and 7 is 35, is true statement.
(a) F, T (b) T, F (c) F, F (d) T, T So, truth value of P is T, Q is F, R is T
Sol. (d) The statement p ® ( p Ù ( ~q)) is false, if p is true and Now, let us check all the options.
p Ù (~q) is false and the statement p Ù (~q) is false,
P Q R ~P ~Q ~R P ÙQ QÙR ~Q Ù R
if (~q) is false as p is true, so q must be true.
T F T F T F F F T
So, the truth values of p and q are T, T respectively.
Hence, option (d) is correct. ( P Ù Q ) Ú (~ R ) P Ú (~Q Ù R ) (~ P) Ú (Q Ù R ) (~ P) Ù (~Q Ù R )

Example 3. If the Boolean expression ( p Å q) Ù (~ p × q) is F T F F


equivalent to p Ù q, where Å, × Î{Ù, Ú}, then the ordered Clearly, the truth value of P Ú (~Q Ù R) is T.
pair(Å, ×) is (JEE Main 2019)
Example 5. The Boolean expression
(a) (Ù, Ú) (b) (Ù, Ù) (c) (Ú, Ù) (d) (Ú, Ú)
(( p Ù q) Ú ( p Ú~ q)) Ù (~ p Ù~ q) is equivalent to
Sol. (a) Let us check all the options (JEE Main 2019)
(a) Consider, Å = Ù and × = Ú. (a) p Ù q (b) p Ú (~q)
In that case, we get (c) p Ù (~q) (d) (~ p) Ù (~q)
476 JEE Main Mathematics

Sol. (d) Let the given Boolean expression


Tautology
(( p Ù q) Ú ( pÚ ~q)) Ù (~ pÙ ~q) º r
Now, let us construct the following truth table A compound statement is called a tautology, if it has
truth value T whatever may be the truth value of its
p q ~ p ~ q p Ù q pÚ ~ q ~ pÙ ~ q (p Ù q) Ú (pÚ ~ q) r compounds.
T T F F T T F T F e.g. Statement ( p Þ q ) Ù p Þ q is a tautology.
T F F T F T F T F
The truth table is prepared as follows
F T T F F F F F F
F F T T F T T T T p q pÞq pÞq Ù p (p Þ q) Ùp Þ q
Clearly, r º~ pÙ ~q T T T T T
T F F F T
Converse, Inverse and F T T F T
Contrapositive of an Implication F F T F T
If p and q are two statements, then here is a quick
definition of their converse, inverse and contrapositive Contradiction (Fallacy)
Implied Statement : if p then q, Converse : if q then p,
A compound statement is called contradiction, if its truth
Inverse : if not p then not q , Contrapositive : if not q,
then not p. value is F whatever may be the truth value of its
components. e.g. Statement ~ p Ù p is a contradiction.
Note
• The converse of p Þ q is q Þ p
The truth table is prepared as follows
• The contrapositive of p Þ q is ~ q Þ ~ p p ~p ~p Ùp
• The inverse of p Þ q is ~ p Þ ~ q
T F F
e.g. If 2 + 2 = 4, then Jawahar Lal Nehru is the first F T F
prime minister of India.
Let p º 2 + 2 = 4 Note A statement which is neither a tautology nor a contradiction is
and q º Jawahar Lal Nehru is the first prime minister of a contigency.
India. Bicondintial statement p Þ q
(i) The converse of p Þ q is q Þ p Example 7. Which of the following is a tautology?
(JEE Main 2020)
i.e. If Jawahar Lal Nehru is the first prime (a) (~p) Ù ( p Ú q) ® q (b) (q ® p) Ú ~( p ® q)
minister of India, then 2 + 2 = 4
(c) (~q) Ú ( p Ù q) ® q (d) ( p ® q) Ù (q ® p)
(ii) The contrapositive of p Þ q is ~ q Þ ~ p
i.e. If Jawahar Lal Nehru is not the first prime Sol. (a) By truth table
minister of India, then 2 + 2 = 4, (~p)Ù (~p) Ù
p q ~p ~q p Ú q p Ù q p ® q q ® p (p Ú q) (p v q) ®q
(iii) The inverse of p Þ q is ~ p Þ ~ q i.e. if 2 + 2 ¹ 4,
then Jawahar Lal Nehru is not the first prime T T F F T T T T F T
minister of India. T F F T T F F T F T
Example 6. The contrapositive of the statement F T T F T F T F T T
“If I reach the station in time, then I will catch the train” is F F T T F F T T F T
(JEE Main 2020)
\ The statement (~p) Ù (p Ú q) ® q is a tautology.
(a) If I do not reach the station in time, then I will catch the
Hence, option (a) is correct.
train
(b) If I will not catch the train, then I do not reach the station
Example 8. Given the following two statements:
in time
(c) If I do not reach the station in time, then I will not catch (S1) : (q Ú p) ® ( p «~ q) is a tautology.
the train (S 2) :~ q Ù (~ p « q) is a fallacy. Then, (JEE Main 2020)
(d) If I will catch the train, then I reach the station in time (a) both ( S1) and ( S 2) are not correct
Sol. (b) Let the statements (b) only ( S1) is correct
p : I reach the station in time. (c) both ( S1) and ( S 2) are correct
q : I will catch the train. (d) only ( S 2) is correct
As we know contrapositive of p Þ q º (~q) Þ (~p) Sol. (a) Given statement
So, contrapositive of given statement, “If I reach the station in S1 : (q Ú p) ® ( p « (~q)) is a tautology.
time, then I will catch the train” is “If I will not catch the train,
For S1:
then I do not reach the station in time”.
Mathematical Reasoning 477

p q ~q qÚp p « (~ q) (q Ú p) ® (p « (~ q)) (v) Involution Laws


T T F T F F For any statement p, we have ~ (~ p) º p
T F T T T T (vi) De-morgan’s Laws
F T F T T T If p and q are any two statement, then
F F T F F T (a) ~ ( p Ù q ) = ~ p Ú ~ q
Þ S1 is not a tautology, so S1 is not correct.
(b) ~ ( p Ú q ) = ~ p Ù ~ q
Now, S 2 : (~q) Ù ((~ p) « q) is a fallacy. (vii) Contrapositive Law
For S 2 : For any two statements p and q , we have
p q ~p ~q (~ p) « q (~ q) Ù ((~ p) « (q) p Þ q º (~ p) Þ (~ q )
(viii) Identity Law
T T F F F F
T F F T T T
For any statement p, we have
F T T F T F p Ù T º p; where, T is a true statement.
F F T T F F Example 10. The proposition p ® ~ ( p Ù~ q) is
Þ S 2 is not a fallacy, so S 2 is not correct. equivalent to (JEE Main 2020)
Hence, option (a) is correct. (a) q (b) (~ p) Ú q
(c) (~p) Ù q (d) (~p) Ú (~q)
Example 9. The statement ( p ® (q ® p)) ® ( p ® ( p Ú q)) is
(JEE Main 2020)
Sol. (b) Given proposition p Þ ~( p Ù (~q))
(a) equivalent to ( p Ù q) Ú (~q) = p Þ (~ p Ú q)
(b) a contradiction = (~p) Ú ((~p) Ú q) [by De-Morgan Law]
(c) equivalent to ( p Ú q) Ù (~ p) = (~p) Ú q
(d) a tautology Hence, option (b) is correct.

Sol. (d) The truth table Example 11. The negation of the boolean expression
(p ® (q ® p))® ~ s Ú (~ r Ù s) is equivalent to (JEE Main 2019)
p q p Ú q q ® p p ® (q ® p) p ® (p Ú q)
(p ® (p Ú q)) (a) s Ù r (b) ~ s Ù ~r (c) s Ú r (d) r
T T T T T T T Sol. (a) The given boolean expression is~ s Ú ((~r) Ù s)
T F T T T T T Now, the negation of given boolean expression is
F T T F T T T ~(~ s Ú ((~r) Ù s))
F F F T T T T = s Ù (~((~r) Ù s)) [Q ~( p Ù q) = ~ pÚ ~q ]
= s Ù (r Ú (~ s)) [Q ~( p Ú q) = ~ pÙ ~q ]
\The statement p ® (q ® p) ® ( p ® ( p Ú q)) is a tautology.
= ( s Ù r) Ú ( s Ù (~ s)) [Q p Ù (q Ú r) º ( p Ù q) Ú ( p Ù r)]
= ( s Ù r) [Q pÙ ~ p º F]
Laws of Algebra of Statement
(i) Idempotent Laws Example 12. The logical statement
[~ (~ p Ú q) Ú ( p Ù r)] Ù (~ q Ù r) is equivalent to
For any statement p, we have (JEE Main 2019)
(a) p Ù p º p (b) p Ú p º p (a) ~p Ú r (b) ( pÙ ~q) Ú r
(ii) Commutative Laws (c) ( p Ù r) Ù ~q (d) (~ pÙ ~q) Ù r
For any two statements p and q, we have Sol. (c) Clearly, [~(~ p Ú q) Ú ( p Ù r)] Ù (~q Ù r)
(a) p Ù q º q Ù p (b) p Ú q º q Ú p º [( p Ù ~q) Ú ( p Ù r)] Ù (~q Ù r)
(iii) Associative Laws [Q~(~ p Ú q) º ~(~ p) Ù ~q º p Ù ~q
For any three statements p, q and r, we have by De-Morgan’s law]
(a) ( p Ù q ) Ù r º p Ù ( q Ù r ) º [p Ù (~q Ú r)] Ù (~q Ù r)] [distributive law]
(b) ( p Ú q ) Ú r º p Ú ( q Ú r ) º p Ù [(~q Ú r) Ù (~q Ù r)] [associative law]
(iv) Distributive Laws º p Ù [(~q Ù r) Ù (~q Ú r)] [commutative law]
For any three statements p, q and r, we have º p Ù [{(~q Ù r) Ù (~q )} Ú {(~q Ù r) Ù r ] [distributive law]
º p Ù [(~q Ù r) Ú (~q Ù r)] [idempotent law]
(a) p Ú ( q Ù r ) º ( p Ú q ) Ù ( p Ú r )
º p Ù [~q Ù r ] [idempotent law]
(b) p Ù ( q Ú r ) º ( p Ù q ) Ú ( p Ù r )
º p Ù ~q Ù r º ( p Ù r) Ù (~q) [associative law]
Practice Exercise
ROUND I Topically Divided Problems
Statements and Elementary 5. The negation of the ‘statement, if a quadrilateral is
Operations of Logic a square, then it is a rhombus’ is
(a) if a quadrilateral is not a square, then it is a rhombus
1. The conditional statement of
(b) if a quadrilateral is a square, then it is not a rhombus
‘You will get a sweet dish after the dinner’ is
(c) a quadrilateral is a square and it is not a rhombus
(a) If you take the dinner, then you will get a sweet dish
(d) a quadrilateral is not a square and it is a rhombus
(b) If you take the dinner, you will get a sweet dish
(c) You get a sweet dish if and only if you take the 6. In the truth table for the statement
dinner (~ p Þ ~ q) Ù (~ q Þ ~ p), the last column has the
(d) None of the above truth value in the following order
2. Contrapositive of the statement (a) TTTF (b) FTTF (c) TFFT (d) TTTT

‘If a function f is differentiable at a, then it is also 7. If p, q and r are simple propositions with truth
continuous at a’, is (JEE Main 2020) value true, false and true respectively, then the
(a) If a function f is not continuous at a, then it is truth value of ((~ p Ú q) Ù ~ r) Þ p
differentiable at a (a) true (b) false
(b) If a function f is continuous at a, then it is (c) true, if r is false (d) true, if q is true
differentiable at a
8. The converse of the statement
(c) If a function f is continuous at a, then it is not
differentiable at a ‘If sun is not shining, then sky is filled with clouds’
(d) If a function f is not continuous at a, then it is not is
differentiable at a (a) if sky is filled with clouds, then the sun is not shining
(b) if sun is shining, then sky is filled with clouds
3. Which of the following is not logically equivalent to (c) if sky is clear, then sun is shining
the following proposition? (d) if sun is not shining, then sky is not filled with clouds
‘A real number is either rational or irrational’ 9. Suppose p : A natural number n is odd
(a) If a number is neither rational nor irrational, then it and q : natural number n is not divisible by 2, then
is not real
the biconditional statement p Û q is
(b) If a number is not a rational or not an irrational,
(a) A natural number n is odd, if and only if it is divisible
then it is not real
by 2
(c) If a number is not real, then it is neither rational or
(b) A natural number n is odd, if and only if it is not
irrational
divisible by 2
(d) If a number is real, then it is rational or irrational
(c) If a natural number n is odd, then it is not divisible
4. Consider the statement ‘For an integer n, if n3 - 1 is by 2
even, then n is odd’. The contrapositive statement (d) None of the above
of this statement is (JEE Main 2020) 10. Let p be the proposition Mathematics is interesting
(a) For an integer n, if n is even, then n3 - 1 is odd and let q be the proposition that Mathematics is
3
(b) For an integer n, if n - 1 is not even, then n is not difficult, then the symbol p Ù q means
odd (a) Mathematics is interesting implies that
(c) For an integer n, if n is even, then n3 - 1 is even Mathematics is difficult.
(d) For an integer n, if n is odd, then n3 - 1 is even (b) Mathematics is interesting implies and is implied by
Mathematics is difficult.
Mathematical Reasoning 479

(c) Mathematics is interesting and Mathematics is 17. Which of the following is true for the statements
difficult. p and q?
(d) Mathematics is interesting or Mathematics is
(a) p Ù q is true when atleast one of p and q is true
difficult.
(b) p ® q is true when p is true and q is false
11. If p : A man is happy. (c) p « q is true only when both p and q are true
q : A man is rich. (d) ~ ( p Ú q) is false only when both p and q are false

Then, the statement, ‘If a man is not happy, then 18. If p Þ ( q Ú r) is false, then the truth values of
he is not rich’ is written as p, q, r are respectively,
(a) ~ p ® ~ q (b) ~ q ® p (a) T, F, F (b) F, F, F
(c) ~ q ® ~ p (d) q ® ~ p (c) F, T, T (d) T, T, F
12. Which of the following is the inverse of the 19. Let truth values of p be F and q be T. Then, truth
proposition ‘If a number is a prime, then it is odd.’ value of ~ (~ p Ú q) is
(a) If a number is not a prime, then it is odd
(a) T (b) F
(b) If a number is not a prime, then it is not odd
(c) either T or F (d) neither T nor F
(c) If a number is not odd then it is not a prime
(d) If a number is not odd, then it is a prime 20. If statements p and r are false and q is true, then
truth value of ~ p Þ ( q Ù r) Ú r is
13. If p and q are two statements such that
(a) T (b) F
p : the questions paper is easy
(c) either T or F (d) neither T nor F
q : we shall pass,
then the symbolic statement ~ p ® ~ q means 21. If p Þ ( q Ú r) is false, then the truth values of
(a) If the question paper is easy, then we shall pass p, q and r are respectively.
(b) If the question paper is not easy, then we shall not (a) TFF (b) FFF
pass (c) FTT (d) TTF
(c) The question paper is easy and we shall pass 22. The logically equivalent proposition of p Û q is
(d) The question paper is easy or we shall pass
(a) ( p Ù q) Ú ( p Ù q) (b) ( p Þ q) Ù (q Þ p)
14. For the following three statements (c) ( p Ù q) Ú (q Þ p) (d) ( p Ù q) Þ ( p Ú q)
p : 2 is an even number
23. The statement p Þ ( q Þ p) is equivalent to
q : 2 is a prime number.
(a) p Þ ( p Þ q) (b) p Þ ( p Ú q)
r : Sum of two prime numbers is always even, then
(c) p Þ ( p Ù q) (d) p Þ ( p Û q)
the symbolic statement ( p Ù q) ® ~ r means
(a) 2 is an even and prime number and the sum of two 24. If p and q are simple propositions, then p Þ ~ q is
prime numbers is always even true when
(b) 2 is an even and prime number and the sum of two (a) both p and q are true
prime numbers is not always even (b) both p and q are false
(c) 2 is an even and prime number, then the sum of two (c) p is false and q is true
prime numbers is not always even (d) None of the above
(d) 2 is an even and prime number, then the sum of two
prime numbers is always even 25. Which of the following is logically equivalent to
~ (~ p Þ q)?
15. For two statements p and q (a) p Ù q (b) p Ù ~ q
p : A quadrilateral is a parallelogram, (c) ~ p Ù q (d) ~ p Ù ~ q
q : The opposite sides are parallel.
26. ~ ( p Ú q) Ú (~ p Ù q) is logically equivalent to
Then, the compound proposition, ‘A quadrilateral is
(a) ~ p (b) p (c) q (d) ~ q
a parallelogram if and only if the opposite sides are
parallel’ is represented by 27. Which of the following is equivalent to ( p Ù q)?
(a) p Ú q (b) p ® q (c) p Ù q (d) p « q (a) p ® ~ q (b) ~ (~ p Ù ~ q)
16. If p Þ ( q Ú r) is false, then truth values of p, q, r (c) ~ ( p ® ~ q) (d) None of these

are respectively (JEE Main 2019)


28. If the compound statement p ® (~ p Ú q) is false,
(a) T, T, F (b) T, F, F
then the truth value of p and q are respectively
(c) F, F, F (d) F, T, T (a) T, T, (b) T, F (c) F, T (d) F, F
480 JEE Main Mathematics

Tautology and Contradiction 39. The proposition ( p Þ ~ p) Ù (~ p Þ p) is


(a) contigency
29. Which one of the following is a tautology? (b) neither Tautology nor Contradiction
(JEE Main 2020)
(c) contradiction
(a) (P Ù (P ® Q )) ® Q (b) P Ù (P Ú Q ) (d) tautology
(c) P Ú (P Ù Q ) (d) Q ® (P Ù (P ® Q )
40. If p and q are two statements, then
30. Which of the following statements is a tautology?
(JEE Main 2020)
( p Þ q) Û (~ q Þ ~ p) is a
(a) ~ ( pÙ ~ q) ® p Ú q (b) ~ ( p Ú ~ q) ® p Ù q (a) contradiction
(c) p Ú (~ q) ® p Ù q (d) ~ ( p Ú ~ q) ® p Ú q (b) tautology
(c) neither (a) nor (b)
31. Which of the following Boolean expression is a (d) None of the above
tautology? (JEE Main 2021)
(a) ( p Ù q) Ú ( p Ú q) (b) ( p Ù q) Ú ( p ® q) 41. The proposition S : ( p Þ q) Û (~ p Ú q) is
(c) ( p Ù q) Ù ( p ® q) (d) ( p Ù q) ® ( p ® q) (a) a tautology
(b) a contradiction
32. If the Boolean expression ( p Ù q) Ç ( p Ä q) is a
(c) either (a) or (b)
tautology, then Ç and Ä are respectively given by
(d) neither (a) nor (b)
(JEE Main 2021)
(a) ®, ® (b) Ù , Ú 42. The false statement in the following is
(c) Ú , ® (d) Ù , ® (a) p Ù (~ p) is a contradiction
33. If P and Q are two statements, then which of the (b) ( p Þ q) Û (~ q Þ ~ p) is a contradiction
following compound statement is tautology? (c) ~ (~ p) Û p is a tautology
(JEE Main 2021) (d) p Ú (~ p) is a tautology
(a) ((P Þ Q ) Ù ~ Q) Þ Q 43. ( p Ù ~ q) Ù (~ p Ù q) is
(b) ((P Þ Q ) Ù ~ Q) Þ ~ P
(a) a tautology
(c) ((P Þ Q ) Ù ~ Q) Þ P (b) a contradiction
(d) ((P Þ Q ) Ù ~ Q) Þ (P Ù Q ) (c) both a tautology and a contradiction
34. The statement ( p Þ q) Û (~ p Ù q) is a (d) neither a tautology nor a contradiction

(a) tautology (b) contradiction 44. The propositions ( p Þ ~ p) Ù (~ p Þ p) is


(c) neither (a) nor (b) (d) None of these (a) Tautology and contradiction
(b) Neither tautology nor contradiction
35. The proposition ~ ( p Þ q) Þ (~ p Ú ~ q) is (c) Contradiction
(a) a Tautology (b) a Contradiction (d) Tautology
(c) either (a) or (b) (d) neither (a) nor (b) 45. Negation of the statement :
36. The negation of the compound proposition is 5 is an integer or 5 is an irrational is
p Ú (~ p Ú q) (JEE Main 2020)
(a) ( p Ù ~ q) Ù ~ p (b) ( p Ú ~ q) Ú ~ p (a) 5 is irrational or 5 is an integer
(c) ( p Ù ~ q) Ú ~ p (d) None of these (b) 5 is not an integer or 5 is not irrational
(c) 5 is an integer and 5 is irrational
37. If p and q are two statements, then (d) 5 is not an integer and 5 is not irrational
~ ( p Ù q) Ú ~ ( q Û p) is
46. Which one of the following statements is not a
(a) tautology
tautology? (JEE Main 2019)
(b) contradiction
(c) neither tautology nor contradiction (a) ( p Ù q) ® (~ p) Ú q
(d) either tautology or contradiction (b) ( p Ù q) ® p
(c) p ® ( p Ú q)
38. Let p and q be two statements. (d) ( p Ú q) ® ( p Ú (~ q))
Then, (~ p Ú q) Ù (~ p Ù ~ q) is a
47. The false statement in the following is
(a) tautology
(a) p Ù (~ p) is contradiction
(b) contradiction
(b) ( p Þ q) Û (~ q Þ ~ p) is a contradiction
(c) neither tautology nor contradiction
(c) ~ (~ p) Û p is a tautology
(d) both tautology and contradiction
(d) p Ú (~ p) Û is a tautology
Mathematical Reasoning 481

(a) true (b) false


Laws of Algebra of Statement
(c) false for any p (d) None of these
48. If p, q, r and s are four simple statements, such as
54. Which of the following is always true?
p : The school bus will come (a) (~ p Ú ~ q) º ( p Ù q)
q : I go to school
(b) ( p ® q) º (~ q ® ~ p)
r : I shall meet my friends
s : I shall go out for a movie, (c) ~ ( p ® ~ q) º ( p Ù ~ q)
(d) ~ ( p « q) º ( p ® q) ® (q ® p)
then the compound statement, ‘If the school bus
does not come or I will not go to school, then I shall 55. The negation of the compound statement ( p Ú q) Ù r
meet my friend and go out for a movie.’ is is
represented by (a) (~ p Ú ~ q) Ù ~ r (b) (~ p Ù ~ q) Ú ~ r
(a) ~ ( p Ù q) Þ (r Ù s) (b) ~ ( p Ù ~ q) Þ (r Ù s) (c) ~ ( p Ú q) Þ r (d) p Ù q
(c) ~ ( p Ù q) Þ (r Ú s) (d) None of these
56. The negation of (~ p Ù q) Ú ( p Ù ~ q) is
49. Negation of the statement p ® ( q Ù r) is (a) ( p Ú ~ q) Ú (~ p Ú q) (b) ( p Ú ~ q) Ù (~ p Ú q)
(a) ~ p ® ~ (q Ú r ) (b) ~ p ® ~ (q Ù r ) (c) ( p Ù ~ q) Ù (~ p Ú q) (d) ( p Ú ~ q) Ù ( p Ú ~ q)
(c) (q Ù r ) ® p (d) p Ù (~ q Ú ~ r )
57. The negation of ( p Ú q) Ù ( p Ú ~ r) is
50. The statement (~ p Ù q) Ú ~ q is
(a) (~ p Ù ~ q) Ú ( p Ù ~ r ) (b) (~ p Ù ~ q) Ù (~ p Ù r )
(a) p Ú q (b) p Ù q (c) ~ ( p Ú q) (d) ~ ( p Ù q)
(c) (~ p Ù ~ q) Ú (~ p Ù r ) (d) None of these
51. Which of the following is true? 58. If q is false and p Ù q ¬® r is true, then which one
(a) p Þ q º ~ p Þ ~ q
of the following statements is a tautology?
(b) ~ ( p Þ~ q) º ~ p Ù q
(JEE Main 2019, 11 January)
(b) ~ ( p Þ~ q) º ~ p Ù q
(a) p Ú r (b) ( p Ù r ) ® ( p Ú r )
(c) ~ (~ p Þ~ q) º ~ p Ù q
(c) ( p Ú r ) ® ( p Ù r ) (d) p Ù r
(d) ~ (~ p Û q) º [~ ( p Þ q)Ù ~ (q Þ p)]
59. The boolean expression ~ ( p Ú q) Ú (~ p Ù q) is
52. Negation of the statement ( p Ù r) ® ( r Ú q) is
(a) ~ ( p Ù r ) ® ~ (r Ú q) (b) (~ p Ú ~ r ) Ú (r Ú q) equivalent to (JEE Main 2018)

(c) ( p Ù r ) Ù (r Ù q) (d) ( p Ù r ) Ù (~ r Ù ~ q) (a) ~ p (b) p (c) q (d) ~ q

53. If p, q are true statements and r, s are false 60. For any two statements p and q, the negation of
statements, then the truth value of the expression p Ú (~ p Ù q) is (JEE Main 2019, 9 April)
~ (( p Ù ~ r) Ú (~ q Ú s)) is (a) ~ p Ù ~ q (b) ~ p Ú ~ q

ROUND II Mixed Bag


Only One Option Correct 3. The negation of the Boolean expression x «~ y is
equivalent to (JEE Main 2020)
1. The contrapositive of the statement ‘If you are born
in India, then you are a citizen of India’, is (a) (~ x Ù y) Ú (~ xÙ ~ y)
(JEE Main 2019) (b) (x Ù y) Ú (~ xÙ ~ y)
(a) If you are not a citizen of India, then you are not (c) (xÙ ~ y) Ú (~ x Ù y)
born in India. (d) (x Ù y) Ù (~ xÚ ~ y)
(b) If you are a citizen of India, then you are born in 4. The negation of the Boolean expression
India.
p Ú (~ p Ù q) is equivalent to (JEE Main 2020)
(c) If you are born in India, then you are not a citizen of
(a) p Ù ~ q (b) ~ p Ù ~ q
India.
(c) ~ p Ú ~ q (d) ~ p Ú q
(d) If you are not born in India, then you are not a
citizen of India. 5. Statement ( p Ù ~ q) Ù (~ p Ú q) is
2. Let p, q, r be three statements such that the truth (a) a tautology
value of ( p Ù q) ® (~ q Ú r) is F. Then, the truth (b) a contradiction
values of p, q, r are respectively. (JEE Main 2020) (c) both a tautology and a contradiction
(a) T, F, T (b) F, T, F (c) T, T, F (d) T, T, T (d) neither a tautology nor a contradiction
482 JEE Main Mathematics

6. Which of the following is wrong statement? 16. Contrapositive of the statement ‘If two numbers are
(a) p ® q is logically equivalent to ~ p Ú q not equal, then their squares are not equal’ is
(b) If the truth values of p, q, r are T, F, T respectively, (JEE Main 2019)
then the truth value of ( p Ú q) Ù (q Ú r ) is T (a) If the squares of two numbers are not equal, then the
(c) ~ (Ú q Ú q Ú r ) @ ~ p Ù ~ q Ù ~ r numbers are not equal.
(d) The truth value of p Ù ~ ( p Ú q) is always T (b) If the squares of two numbers are equal, then the
numbers are equal.
7. Which of the following statement is a (c) If the squares of two numbers are not equal, then the
contradiction? numbers are equal.
(a) (~ p Ú ~ q) Ú ( p Ú ~ q) (b) ( p ® q) Ú ( p Ù ~ q) (d) If the squares of two numbers are equal, then the
(c) (~ p Ù q) Ù (~ q) (d) (~ p Ù q) Ú (~ q) numbers are not equal.
8. Which of the following is wrong? 17. The expression ~ (~ p ® q) is logically equivalent to
(a) p Ú ~ p is a tautology (JEE Main 2019)
(b) ~ (~ p) « p is a tautology (a) p Ù ~ q (b) p Ù q
(c) p Ù ~ p is a contradiction
(c) ~ p Ù q (d) ~ p Ù ~ q
(d) (( p Ù q) ® q) ® p is a tautology
18. The statement ( p ® q) ® [(~ p ® q) ® q] is
9. If p, q and r are simple propositions, then
(JEE Main 2017)
( p Ù q) Ù ( q Ù r) is true, then
(a) a tautology (b) equivalent to ~ p ® q
(a) p, q and r are true (c) equivalent to p ® ~ q (d) a fallacy
(b) p, q are true and r is false
(c) p is true and q, r are false 19. The Boolean expression ( p Ù ~ q) Ú q Ú (~ p Ù q) is
(d) p, q and r are false
equivalent to (JEE Main 2016)
10. If ( q Ù - r) Þ ( q Ú r) is false, then p is (a) ~ p Ù q (b) p Ù q
(a) True (b) False (c) p Ú q (d) p Ú ~ q
(c) May be true and false (d) None of these 20. The negation of ~ s Ú (~ r Ù s) is equivalent to
11. If p and q are simple proposition, then (JEE Main 2015)
(a) s Ù ~ r (b) s Ù (r Ù ~ s)
(~ p Ù q) Ú (~ q Ù p) is true when p and q are
(c) s Ú (r Ú ~ s) (d) s Ù r
respectively
(a) T, T (b) T, F (c) F, F (d) F, T 21. The statement ~ ( p « ~ q ) is (JEE Main 2014)

12. If p, q and r are simple proposition, then (~ p Ú q) (a) equivalent to p « q


(b) equivalent to ~ p « q
Þ r is true, when p, q and r are, respectively
(c) a tautology
(a) T, F, T (b) F, T, T
(d) a fallacy
(c) T, F, F (d) F, F, F
22. Consider the following statements
13. Which one of the following Boolean expressions is a
P : Suman is brilliant.
tautology? (JEE Main 2019)
Q : Suman is rich.
(a) ( p Ú q) Ú ( p Ú ~ q)
R : Suman is honest.
(b) ( p Ù q) Ú ( p Ù ~ q)
(c) ( p Ú q) Ù ( p Ú ~ q) The nagative of the statement. ‘Suman is brilliant
(d) ( p Ú q) Ù (~ p Ú ~ q) and dishonest, if and only if Suman is rich.’ can be
14. If the truth value of the statement p ® (~ q Ú r) is expressed as
false (F), then the truth values of the statements p, q (a) ~ (Q « (P Ù ~ R)) (b) ~ Q « P Ù R
and r are respectively (JEE Main 2019)
(c) ~ (P Ù ~ R) « Q (d) ~ P Ù (Q « ~ R)
(a) T, T and F (b) T, F and F 23. The only statement among the following that is a
(c) T, F and T (d) F, T and T tautology is
15. The boolean expression ~ ( p Þ (~ q)) is equivalent (a) B ® [ A Ù ( A ® B)]
(b) A Ù ( A Ú B)
to (c) A Ú ( A Ù B)
(JEE Main 2019)
(d) [ A Ù ( A ® B)] ® B
(a) p Ù q (b) q Þ ~ p
(c) p Ú q (d) (~ p) Þ q
Mathematical Reasoning 483

Answers
Round I
1. (a) 2. (d) 3. (b) 4. (a) 5. (c) 6. (c) 7. (a) 8. (a) 9. (b) 10. (c)
11. (a) 12. (b) 13. (b) 14. (c) 15. (d) 16. (b) 17. (d) 18. (a) 19. (b) 20. (b)
21. (a) 22. (b) 23. (b) 24. (c) 25. (d) 26. (a) 27. (b) 28. (b) 29. (a) 30. (d)
31. (d) 32. (a) 33. (b) 34. (c) 35. (a) 36. (a) 37. (c) 38. (c) 39. (c) 40. (b)
41. (a) 42. (d) 43. (b) 44. (c) 45. (d) 46. (d) 47. (b) 48. (a) 49. (d) 50. (d)
51. (c) 52. (d) 53. (b) 54. (b) 55. (b) 56. (b) 57. (c) 58. (b) 59. (a) 60. (a)

Round II
1. (a) 2. (c) 3. (b) 4. (b) 5. (b) 6. (d) 7. (c) 8. (d) 9. (a) 10. (a)
11. (d) 12. (a) 13. (a) 14. (a) 15. (a) 16. (b) 17. (d) 18. (a) 19. (c) 20. (d)
21. (a) 22. (a) 23. (d)

Solutions
Round I 9. Given, p : A natural number n is odd and q : natural
1. The conditional statement of given statement is ‘If you number n is not divisible by 2
take the dinner, then you will get a sweet dish’. The biconditional statement p Û q i. e. , ‘A natural
2. As the contrapositive of statement number n is odd if and only if it not divisible by 2’.
p ® q is ~ q ® ~ p. 10. Given, p : Mathematics is interesting
\ Contrapositive of statement ‘If a function f is q : Mathematics is difficult
differentiable at a, then it is also continuous at a’ is if \ p Ù q. Mathematics is interesting and Mathematics is
a function f is not continuous at a, then it is not difficult.
differentiable at a.
11. Q p : A man is happy
3. Given proposition is not logically equivalent to
‘If a number is not a rational or not an irrational, then it and q : A man is rich.
is not real’. ‘If a man is not happy, then he is not rich’ is written as
~ p ® ~ q.
4. The contrapositive of statement
p ® q is ~ q ® ~ p. 12. p : A number is a prime
For an integer n, if n is even, then n3 - 1 is odd. q : It is odd.
We have, p Þ q
5. Let p and q be the propositions as given below
The inverse of p Þ q is ~ p Þ ~ q
p : A quadrilateral is a square i.e. if a number is not a prime, then it is not odd.
q : A quadrilateral is a rhombus
13. Given, p : The question paper is easy and q : We shall
The given proposition is p ® q
pass given.
Now, ~ ( p ® q) @ p Ù ~ q
The symbolic representation of given option are
Therefore, the negation of the given proposition is
“A quadrilateral is a square and it is not a rhombus” (a) : p ® q
(b) : ~ p ® ~ q
6. Let S : (~ p Þ ~ q) Ù (~ q Þ~ p) (c) : p Ù q
~p ~q ~p Þ~q ~q Þ~p S (d) : p Ú q

T T T T T 14. Given, p : 2 is an even number


T F F T F q : 2 is a prime number
F T T F F r : sum of two prime numbers is always even.
F F T T T
The symbolic representation of given option (s) are
7. Let S : ((~ p Ú q) Ù ~ r ) Þ p (a) : p Ù q Ù r
Þ S : ((~ T Ú F ) Ù ~ T ) Þ T (b) : p Ù q Ù ~ r
(c) : p Ù q Þ ~ r
8. The converse of the given statement is “If sky is filled (d) : p Ù q Þ r
with clouds, then the sun is not shining”.
484 JEE Main Mathematics

15. Given, p : A quadrilateral is a parallelogram. 22. p q pÞq qÞp pÞq (p Þ q) Ù (q Þ p)


q : The opposite sides are parallel.
‘A quadrilateral is a parallelogram if and only if the T T T T T T
opposite sides are parallel’ is represented by p « q. T F F T F F
16. Given statement p Þ (q Ú r ) is false. F T T F F F
F F T T T T
Q p ® (q Ú r ) = (~ p) Ú (q Ú r )
Now, by trial and error method, if truth value of p is T, 23.
q is F and r is F, then truth value of (q Ú r ) is F.
p q qÞp p Þ (q Þ p) pÚq p Þ (p Ú q)
So, truth value of [(~ p) Ú (q Ú r )] is false.
Thus, if truth value of p, q, r are T, F, F, then the T T T T T T
statement p ® (q Ú r ) is false. T F T T T T
17. (a) We know that, p Ù q is true when both p and q are F T F T T T
true. F F T T F T
(b) We know that, p ® q is false when p is true Hence, p Þ ( p Þ q) º p Þ ( p Ú q)
and q is false.
24.
(c) We know that, p « q is true when either both
p and q are true or both are false. p q ~q p Þ~q
(d) If p and q both are false, then p Ú q is false T T F F
Þ ~ ( p Ú q) is true. T F T T
Hence, option (d) is true. F T F T
F F T F
18. p q r qÚr p Þ (q Ú r )
25.
T F F F F
p q ~p ~q ~p Þ q ~(~ p Þ q) ~ p Ù ~ q
T F T T T
T T F T T T T F F T F F
T T T T T T F F T T F F
F F F F T F T T F T F F
F F T T T F F T T F T T
F T F T T Hence, ~ (~ p Þ q) º ~ p Ù ~ q
F T T T T
26.
Hence, truth values of p, q and r are respectively p q ~ p p Ú q ~(p Ú q) ~ p Ù q ~(p Ú q) Ú(~ p Ù q)
T, F, F.
T T F T F F F
19. p q ~p ~p Ú q ~(~ p Ú q) T F F T F F F
F T T T F T T
F T T T F
F F T F T F T

\ ~ ( p Ú q) Ú (~ p Ù q) º ~ p
20. p q r ~p q Ùr (q Ù r ) Ú r ~ p Þ (q Ù r ) Ú r

27. p q pÙq ~p ~q p ®~q ~( p ® ~ q)


F T F T F F F
T T T F F F T
T F F F T T F
21. p q r q Úr p Þ (q Ú r )
F T F T F T F
T F F F F F F F T T T F
T F T T T
28. We know that, p ® q is false only when p is true and q is
T T F T T
false.
T T T T T
So, p ® (~ p Ú q) is false only when p is true and (~ p Ú q)
F F F F T
is false.
F F T T T But (~ p Ú q) is false if q is false because ~ p is false.
F T F T T Hence, p ® (~ p Ú q) is false when truth value of p and q
F T T T T are T and F respectively.
Mathematical Reasoning 485

29. From the truth table = ( p Ù q) Ù (~ p Ú q)


PÙ PÙ PÚ = p Ù q (Not a tautology)
P Q P ÚQ P ÙQ P ®Q
(P ® Q ) (P Ú Q ) (P Ù Q ) 33. ((P ® Q ) Ù ~ Q )
T T T T T T T T º (~ P Ú Q ) Ù ~ Q
T F T F F F T T
º (~ P Ù ~ Q ) Ú (Q Ù ~ Q )
F T T F T F F F
º ~ P Ù ~Q
F F F F T F F F
For option (a),
P Ù (P ® Q ) ® Q Q ® (P Ù (P ® Q )) (P Ù ~ Q ) ® Q
T T º ~ (~ P Ù ~ Q ) Ú Q
T T º (P Ú Q ) Ú Q ¹ Tautology
T F For option (b),
T T
(~ P Ù ~ Q ) ® ~ P
º ~ (~ P Ù ~ Q ) Ú ~ P
\ P Ù (P ® Q ) ® Q is a tautology. º (P Ú Q ) Ú ~ P Þ (Tautology)
Hence, option (a) is correct. For option (c),
30. From the truth table (~ P Ù ~ Q ) ® P
~(p Ú (~(p Ú (~ q))) º (P Ú Q ) Ú P ¹ Tautology
p q ~ p ~ q p Ú (~ q) p Ù q pÚq
(~ q)) ®pÚq For option (d),
T T F F T T F T T (~ P Ù ~ Q ) ® (P Ù Q )
T F F T T F F T T º (P Ú Q ) Ú (P Ù Q ) ¹ Tautology
F T T F F F T T T
T F T T T F F F T 34. p q ~p pÞq ~p Ù q (p Þ q) Û (~ p Ù q)

So, (~ ( p Ú (~ q))) ® p Ú q is a tautology. T T F T F F


Hence, option (d) is correct. T F F F F T
F T T T T T
31.
F F T T F F
p q pÙq p®q (p Ù q) ®( p ® q)
T T T T T 35. By truth table
T F F F T ~(p Þ q)
p q ~p ~q p Þ q ~(p Þ q) ~ p Ú ~ q
F T F T T Þ (~ p Ú ~ q)
F F F T T T T F F T F F T
( p Ù q) ® ( p ® q) is tautology. T F F T F T T T
32. For option (a), F T T F T F T T
F F T T T F T T
( p Ù q) ® ( p ® q)
= ~ ( p Ù q) Ú (~ p Ú q) Hence, given proposition is a tautology.
= (~ p Ú ~ q) Ú (~ p Ú q) 36. Since, S : ~ ( p Ú (~ p Ú q))
= ~ p Ú (~ q Ú q) Þ S : ~ p Ù ~ (~ p Ú q) (De-Morgan’s Law)
=~pÚt Þ S : ~ p Ù ( p Ù ~ q) (De-Morgan’s law)
=t 37. By truth table
For option (b),
( p Ù q) Ù ( p Ú q) = ( p Ù q) (Not a tautology) p q (p Ù q) ~(p Ù q) q Û p ~(q Û p) ~(p Ù q) Ú
~(q Û p)
For option (c)
( p Ù q) Ú ( p ® q) T T T F T F F
= ( p Ù q) Ú (~ p Ú q) T F F T F T T
= ~ p Ú q (Not a tautology) F T F T F T T
F F F T T F T
For option (d),
( p Ù q) Ù ( p ® q) It is clear that neither tautology nor contradiction.
486 JEE Main Mathematics

38. (~ p Ú q) 46. (a) ( p Ù q) ® (~ p) Ú q


p q ~p ~ q (~ p Ú q) (~ p Ù ~ q)
Ù(~ p Ù ~ q) º (~ ( p Ù q)) Ú ((~ p) Ú q)
º ((~ p) Ú (~ q)) Ú ((~ p) Ú q)
T T F F T F F º (~ p) Ú (~ q) Ú q [Q (~ p) Ú (~ p) º ~ p]
T F F T F F F º (~ p) Ú T [Q ~ q Ú q º T]
F T T F T F F ºT
F F T T T T T So, it is a tautology [Q ((~ q) Ú q) is tautology]
(b) ( p Ù q) ® p º ( p Ù q) Ú p
Hence, it is neither tautology nor contradiction. º ((~ p) Ú (~ q)) Ú p
[Q ~ ( p Ù q) º (~ p) Ú (~ q)]
39. p ~p p Þ~p ~p Þ p (p Þ ~ p) Ù (~ p Þ p) º (~ p Ú p) Ú (~ q) is tautology.
[Q ~ p Ú p is a tautology and (~ q) Ú T º T ]
F T T F F
(c) Q p ® ( p Ú q) º (~ p) Ú ( p Ú q)
T F F T F
[Q p ® q is equivalent to (~ p Ú q)]
\ Statements is contradiction. º (~ p Ú p) Ú q is tautology.
[Q (~ p Ú p) is tautology and q Ú T º T ]
40. p q ~q ~ p ~ q Þ ~ p pÛq (p Þ q) (d) ( p Ú q) ® ( p Ú (~ q))
Û (~ q Þ ~ p) º (~ ( p Ú q)) Ú ( p Ú (~ q))
T T F F T T T º ((~ p) Ù (~ q)) Ú ( p Ú (~ q))
T F T F F F T º ( p Ú (~ q) Ú ((~ p) Ù (~ q))
F T F T T T T º ( p Ú (~ q) Ú (~ p)) Ù ( p Ú (~ q) Ú (~ q))
F F T T T T T º (T Ú (~ q)) Ù ( p Ù (~ q))
Hence, it is tautology. º T Ù ( p Ù (~ q))
º p Ù (~ q), which is not a tautology.
41.
p q pÞq (~ p Ú q) (p Þ q) Û (~ p Ú q) 47. p Þ q is logically equivalent to ~ q Þ ~ p
T T T T T \ ( p Þ q) Û (~ q Þ ~ p) is a tautology but not a
T F F F T
contradiction.
F T T T T 48. (If the school bus does not come) Ú (I will not go to school)
F F T T T ® (I shall meet my friend) Ù (I shall go out for a movie)
Þ (~ p) Ú (~ q) Þ (r Ù s)
Since, all values of given proposition is true, hence it is
a tautology. Þ ~ ( p Ù q) Þ (r Ù s) (De-Morgan’s Laws)

42. p Þ q is logically equivalent to ~ q Þ ~ p. 49. We know that, ~ ( p ® q) @ p Ù ~ q

\ ( p Þ q) Û (~ q Þ ~ p) \~ ( p ® ~ (q Ù r ) @ p Ù (~ (q Ù r ))
is a tautology but not a contradiction. @ p Ù (~ q Ú ~ r ) [by De-Morgan’s laws]

43. ( p Ù ~ q) Ù (~ p Ù q) = ( p Ù ~ q) Ù (~ q Ù q) = f Ù f = f 50. (~ p Ù q) Ú ~ q º ~ q Ú (~ p Ù q) (by Commutative law)

(by using associative laws and commutative laws) º ~ q Ú (q Ù q ~ p) (by Commutative law)
\ ( p Ù ~ q) Ù (~ p Ù q) is a contradiction. º ~ q Ú q ( ~ q Ú ~ p) (by Distributive law)
º ~ ( q Ù p) º ~ ( p Ù q )
44.
51. ~ ( p Þ q) º p Ù ~ q
p ~p p Þ~p ~p Þ p (p Þ~ p) Ù (~ p Þ p)
\ ~ (~ p Þ ~ q) º ~ p Ù ~ (~ q) º ~ p Ù q
T F F T F
Thus, ~ (~ p Þ ~ q) º ~ p Ù q
F T T F F
52. We know that, ~ ( p ® q) º p Ù ~ q
Clearly, ( p Þ ~ p) Ù (~ p Þ p) is a contradiction.
\ ~ (( p Ù r ) ® (r Ú q)) º ( p Ù r ) Ù [~ (r Ú q)]
45. Since, ~ ( p Ú q) = (~ p) Ù (~ q)
º ( p Ù r ) Ù (~ r Ù ~ q)
So, negation of the statement
53. ~ ((T Ù ~ F ) Ú (~ T Ú F ))
‘ 5 is an integer and 5 is not irrational’.
Þ º ~ ((T Ù T ) Ú (F Ú F ))
Is 5 is not an integer and 5 is not irrational?
Þ º ~ ((T ) Ú (F )) º ~ (T ) º F
Hence, option (d) is correct.
Mathematical Reasoning 487

54. Since, ~ ( p Ú q) º (~ p Ù ~ q) and ~ ( p Ù q) º (~ p Ú q) Round II


So, option (b) and (d) are not true. 1. Given statement is
( p ® q) º p Ù ~ q, so option (c) is not true. ‘If you are born in India, then you are a citizen of India’.
Now, p ® q ~ p Ú q Now, let statement p : you are born in India
~ q ® ~ p º [~ (~ q) Ú ~ p] º q Ú ~ p º ~ p Ú q and q : you are citizen of India.
p ® q º ~ q ®~ p Then, given statement, ‘If you are born in India then
you are a citizen of India’ is equivalent to p Þ q.
55. Let S : ( p Ú q) Ù r
Q The contrapositive of statement p Þ q is ~ q Þ ~ p.
Þ ~ S : ~ ( p Ú q) Ù r ) \ The contrapositive of the given statement is
Þ ~ S : ~ ( p Ú q) Ú ~ r (De-Morgan’s Laws) ‘If you are not a citizen of India, then you are not
born in India.
\ ~ S : (~ p Ù ~ q) Ú ~ r
So, the contrapositive statement of given statement
56. Let S : (~ p Ù q) Ú ( p Ù ~ q) ‘For an integer n, if n3 - 1 is even, then n is odd’, is
Þ ~ S : ~ ((~ p Ù q) Ú ( p Ù ~ q)) ‘For an integer n, if n is even, then n3 - 1 is odd’.
Þ ~ S : ~ (~ p Ù q) Ù ~ ( p Ù ~ q) 2. Given statement
(De-Morgan’s Law) ( p Ù q) ¾® (~ q Ú r )
\ ~ S : ( p Ú ~ q) Ù (~ p Ú q) = ~ ( p Ù q) Ú (~ q Ú r ) = ((~ p) Ú (~ q)) Ú (~ q Ú r )
57. Let S : ( p Ú q) Ù ( p Ú ~ r ) = (~ p) Ú (~ q) Ú r, is give false, so ~ p , ~ q and r must
be false, so p, q must be true and r should be false, so
Þ ~ S : ~ ( p Ú q) Ú ~ ( p Ú ~ r ) truth value of p, q, r are T, T, F respectively.
\ ~ S : (~ p Ù ~ q) Ú (~ p Ù r ) Hence, option (c) is correct.
58. Given, ( p Ù q) « r is true. This is possible under two 3. As we know, ~ ( p « q) = ( p Ù ~ q) Ú (~ p Ù q)
cases So, ~ (x « ~ y) = (x Ù y) Ú (~ x Ù ~ y)
Case I When both p Ù q and r are true, which is not 4. The negation of Boolean expression p Ú (~ p Ù q) is
possible because q is false.
~ ( p Ú (~ p Ù q)) = ~ (( p Ú (~ p)) Ù ( p Ú q))
Case II When both ( p Ù q) and r are false.
= ~ ( p Ú q) = (~ p) Ù (~ q)
Þ p ºT
or F; q º F, r º F 5. The truth table of ( p Ù ~ q) Ù (~ p Ú q) is as given below
In this case, p q ~p ~q p Ú ~q ~p Ú q (p Ù ~ q) Ù (~ p Ú q)
(a) p Ú r is T or F
T T F F F T F
(b) ( p Ù r ) ® ( p Ú r ) is F® (T or F ), which always T F F T T F F
result in T.
F T T F F T F
(c) ( p Ú r ) ® ( p Ù r ) is (T or F) ® F, which may be T F F T T F T F
or F.
(d) p Ù r is F. The last column of the above truth table contains F only.
So, the given statement is a contradiction.
59. Key Idea Use De-Morgan’s and distributive law.
We have, ~ ( p Ú q) Ú (~ p Ù q) 6. The truth tables of p ® q and ~ p Ú q are given below
º (~ p Ù ~ q) Ú (~ p Ù q) p q ~p p®q ~(p Ú q)
[Q by DeMorgan’s law ~ ( p Ú q) = (~ p Ù ~ q)]
T T F T T
º ~ p Ù (~ q Ú q) [by distributive law] T F F F F
º ~pÙt [~ q Ú q = t] F T T T T
F F T T T
º~p
60. Q p Ú ( (~ p) Ù q) Clearly, truth tables of p ® q and ~ p Ú q are same.
So, p ® q is logically equivalent to ~ p Ú q.
= ( p Ú (~ p)) Ù ( p Ú q ) [by Distributive law]
Hence, option (a) is correct.
= pÚ q [Qp Ú (~ p) is tautology]
So, negation of p Ú ((~ p) Ù q) If the truth value of p, q, r are T, F, T respectively, then
the truth values of p Ú q and q Ú r are each equal to T.
= ~ [ p Ú (~ p) Ù q] = ~ ( p Ú q)
Therefore, the truth value of ( p Ú q) Ù (q Ú r ) is T.
= (~ p) Ù (~ q) [by De-morgan’s law]
Hence, option (b) is correct.
488 JEE Main Mathematics

We know that, ~ ( p Ú q Ú r ) @ (~ p Ù ~ q Ù ~ r ) 12. p q r ~p ~p Ú q (~ p Ú q) Þ r


So, option (c) is correct.
T F F F F F
If p is true and q is false, then p Ú q is true. T F T F F T
Consequently ~ ( p Ú q) is false and hence p Ù ~ ( p Ú q) is T T F F T F
false. T T T F T T
Hence, option (d) is wrong. F F F T T F
F F T T T T
7. We consider following truth table.
F T F T T F
(p Ù q) F T T T T T
p q ~p ~q p Ùq p Úq (~(p Ú q))
Ù ~(p Ú q) 13. Option (a) ( p Ú q) Ú ( p Ú (~ q))
T T F F T T F F º p Ú (qÚ ~ q) is tautology,
T F F T F T F F [Q q Ú (~ q) º T and p Ú T º T]
F T T F F T F F Option (b)
F F T T F F T F ( p Ù q) Ú ( p Ù (~ q)) º p Ù (q Ú ~ q)
not a tautology, [Q q Ú ~ q º T and p Ù T º p]
Clearly, last column of the above truth table contains Option (c)
only F. Hence, ( p Ù q) Ù ~ ( p Ú q) is a contradiction
( p Ú q) Ù ( p Ú (~ q)) º p Ú (q Ù ~ q)
8. The truth value of ~ (~ p) « p as follow not a tautology [Q q Ù ~ q º F and p Ú F º p]
Option (d)
p ~ p ~(~ p) ~(~ p) ® p p ® ~(~ p) ~(~ p) « p
( p Ú q) Ù ((~ p) Ú (~ q)) º ( p Ú q) Ù (~ ( p Ù q))
T F T T T T not a tautology.
F T F T T T 14. Key Idea Use formula :
Since, last column of above truth table contains only T. p ® q = ~p Ú q
Given statement is
Hence, ~ (~ p) ® p is a tautology.
p ® (~ q Ú r ) = ~ p Ú (~ q Ú r )
9. p q r pÙq pÙr (p Ù q) Ù (q Ù r ) Now, from the options
T F F F F F (a) When p = T, q = T and r = F
T F T F F F then ~ p Ú (~ q Ú r ) = F Ú (F Ú F) = F
T T F T F F (b) When p = T, q = F and r = F
T T T T T T then ~ p Ú (~ q Ú r ) = F Ú (T Ú F) = T
F F F F F F (c) When p = T, q = F and r = T
F F T F F F
then ~ p Ú (~ q Ú r ) = F Ú (T Ú T) = T
F T F F F F
F T T F T F (d) When p = F, q = T and r = T
then ~ p Ú (~ q Ú r ) = T Ú (F Ú T) = T
10. p q r ~r p Ù ~r pÚr (p Ù ~ r ) Þ (q Ú r ) 15. Given boolean expression is
T F F T T F F ~ ( p Þ (~ q))
T F T F F T T º ~ ((~ p) Ú (~ q)) [Q p Þ q º ~ p Ú q]
T T F T T T T º pÙq
T T T F F T T 16. We know that, contrapositive of
F F F T F F T
p ® q is ~ q ® ~ p
F F T F F T T
F T F T F T T Therefore, the contrapositive of the given statement is
F T T F F T T ‘‘If the squares of two numbers are equal, then the
numbers are equal’’.
11. p q ~p ~ q ~ p Ù q ~ q Ù p (~ p Ù q) Ú(~ q Ù p) 17. Since, the expression, p ® q º ~ p Ú q
T T F F F F F So, ~p®qº pÚq
T F F T F F F and therefore ~ (~ p ® q) º ~ ( p Ú q)
F T T F T F T º (~ p) Ù (~ q)
F F T T F F F [by De-Morgan’s law]
Mathematical Reasoning 489

18. (a) The truth table of the given expression is given below 20. ~ (~ s Ú (~ r Ù s)) º s Ù (~ (~ r Ù s))
p q x º p®q ~p ~p ® q yº p®q®q x ®y
º s Ù (r Ú ~ s) º (s Ù r ) Ú (s Ù ~ s)
º (s Ù r ) Ú F [Q s Ù ~ s is false]
T T T F T T T
ºsÙ r
T F F F T F T
F T T T T T T
21.
F F T T F T T p q ~p ~q p«q p « ~q ~p « q ~(p « ~ q)
T F F T F T T F
Hence, it is a tautology.
F T T F F T T F
19. Consider, ( p Ù ~ q) Ú q Ú (~ p Ù q) T T F F T F F T
º [( p Ù ~ q) Ú q] Ú (~ p Ù q) F F T T T F F T
º [( p Ú q) Ù (~ q Ú q)] Ú (~ p Ù q)
~ ( p « ~ q) is equivalent to ( p « q).
º [( p Ú q) Ù t ] Ú (~ p Ù q)
22. Suman is brilliant and dishonest, if and only if Suman is
º ( p Ú q) Ú (~ p Ù q) º ( p Ú q Ú ~ p) Ù ( p Ú q Ú q)
rich, is expressed as, Q « (P Ù ~ R)
º (q Ú t ) Ù ( p Ú q) º t Ù ( p Ú q) º p Ú q
\Negation of it will be, ~ (Q « (P Ù ~ R))

23. A B AÚB AÙB A Ú ( A Ú B) A Ú ( A Ù B) A®B A Ù ( A ® B) A Ù ( A ® B) ® B B ® ( A Ù ( A ® B))

T T T T T T T T T T
T F T F T T F F T T
F T T F F F T F T F
F F F F F F T F T T

Q The truth value of all the elements of the column A Ù ( A ® B ) ® B \ A Ù ( A ® B) ® B is a tautology.


20
Statistics
Representation of Data IN THIS CHAPTER ....
The data once collected, must be arranged or organised in a way so that Representation of Data
definite inferences can be made out from it. The most common organisational Measures of Central Tendency
schemes are Tabular representation and Graphical representation.
Mean
In tabular representation, the data can be organised as discrete frequency or
Geometric Mean (GM)
continuous frequency distribution and provides both economy of
representation and organisation. Harmonic Mean (HM)
On the other hand in case of Graphical representation the most common types Median
in which data can be represented are : Histogram, Pie diagram or an Ogive. Measures of Dispersion
The benefit of Graphical representation is that the data is often better
Range
understood and inferences/results can be made out of it easily and quickly.
Mean Deviation
Measures of Central Tendency Standard Deviation
Generally, average of a distribution is a measure of the middle or expected
value of distribution, such values are known as measures of central tendency.
These are five types of central tendency
(i) Mean (ii) Geometric mean
(iii) Harmonic mean (iv) Median (v) Mode.

Mean
The sum of all the observations is divided by the number of observations is
called mean and it is denoted by x.

Mean of Ungrouped or Individual Data


If x1 , x2 , x3 ,... , xn are n observations, then mean by
x + x2 + .... + xn 1 n
(a) Direct method x = 1
n
= å xi
n i =1
n
1
(b) Shortcut method x = A +
n
å di
i =1

where, A = Assumed mean and di = xi - A


Statistics 491

Mean of Grouped or Continuous Data Example 2. The weighted mean of first n natural numbers
If x1 , x2 , x3 , ... , xn are n observations whose corresponding whose weights are equal to the number of selections out of n
frequencies are f1 , f2 , f3 , ... , fn , then mean by natural numbers of corresponding numbers, is
n n × 2n - 1 3n(n + 1) (n + 1)(2n + 1) n(n + 1)
å fi xi (a)
2n - 1
(b)
2(2n + 1)
(c)
6
(d)
2
x1 f1 + x2 f2 + ... + xn fn i =1
(a) Direct method x = =
f1 + f2 + ... + fn n
Sol. (a) The required mean
å fi 1×n C1 + 2 ×n C 2 + 3 ×n C3 + ¼ + n ×n C n
i =1 x= n
n C1 + nC 2 + ¼ + nC n
å fi di n n
n n

(b) Shortcut method x = A +


i =1
n
å r ×n Cr år × r n -1
Cr - 1 nå n -1
Cr - 1
r =0 r =1 r =1
= = =
å fi n n n
i =1 å nC r å nC r å nC r
r =1 r =1 r =1
where A = Assumed mean and di = xi - A
n × 2n - 1
=
Example 1. If for some x Î R, the frequency distribution of 2n - 1
the marks obtained by 20 students in a test is
Marks 2 3 5 7
Combined Mean
Frequency ( x + 1) 2 2x - 5 2
x - 3x x If two sets of observations are given, then the combined
mean of both the sets can be calculated by the following
Then, the mean of the marks is (JEE Main 2019)
formula
(a) 3.0 (b) 2.8 (c) 2.5 (d) 3.2 n x + n 2 x2
x12 = 1 1
Sol. (b) The given frequency distribution, for some x Î R, of n1 + n 2
marks obtained by 20 students is where, x1 = mean of first set of observations
Marks 2 3 5 7 n1 = number of observations in first set
Frequency ( x + 1) 2 2x - 5 x2 - 3x x x2 = mean of second set of observations
Q Number of students = 20 = S fi n 2 = number of observations in second set
Þ ( x + 1) 2 + (2x - 5) + ( x2 - 3x) + x = 20
Þ ( x2 + 2x + 1) + (2x - 5) + ( x2 - 3x) + x = 20 Properties of Mean
Þ 2x2 + 2x - 24 = 0 Þ x2 + x - 12 = 0 (i) Algebraic sum of the deviations of a set of values
from their mean is zero.
Þ ( x + 4)( x - 3) = 0 Þ x = 3 [as x > 0 ]
S fi xi (ii) The sum of the squares of the deviations of a set of
Now, mean ( x) =
S fi values is minimum when taken about mean.
2( x + 1) 2 + 3(2x - 5) + 5( x2 - 3x) + 7x (iii) Mean is affected by the change or shifting of origin
= and scale.
20
2( 4) 2 + 3(1) + 5(0) + 7(3) 32 + 3 + 21
= = Example 3. If X1 and X 2 are the means of two distributions
20 20
56 such that X1 < X 2 and X is the mean of the combined
= = 2 .8
20 distribution, then
Hence, option (b) is correct. (a) X < X1 (b) X > X 2
X1 + X 2
Weighted Mean (c) X = (d) X1 < X < X 2
2
If corresponding weight of x1 , x2 , ¼ , xn are w1 , w2 , .... , wn
Sol. (d) Let n1 and n2 be the number of observations in two groups
respectively, then
w x + w2x2 + ..... + wn xn having means X1 and X 2 respectively. Then,
Weighted mean = 1 1 n1X1 + n2X 2
w1 + w2 + .... + wn X=
n n1 + n2
å wi xi n1X1 + n2X 2
i =1 Now, X - X1 = - X1
= n n1 + n2
å wi =
n2( X 2 - X1)
>0 [Q X 2 > X1]
i =1
n1 + n2
492 JEE Main Mathematics

Þ X > X1 …(i) Harmonic Mean for Ungrouped Data


n ( X1 - X 2)
and X - X2 = <0 [Q X 2 > X1] If x1 , x2 , . . . xn be n observations, then harmonic mean is
n1 + n2
given by
Þ X < X2 …(ii) n n
From Eqs. (i) and (ii), we get HM = = n
1 1 1 1
X1 < X < X 2
+
x1 x2
+ ... +
xn å x
i =1 i

Geometric Mean (GM) Harmonic Mean for Grouped Data


The nth root of the product of the values is called
If x1 , x2 , . . . xn be n observations, whose corresponding
geometric mean.
frequencies are f1 , f2 , . . , fn , then harmonic mean is given
Geometric Mean for Ungrouped Data by
n
If x1 , x2 , x3 , . . . , xn be n observations, then geometric mean
is given by G = ( x1 × x2 × x3 . . . xn )1/ n
å fi
i =1
HM = n
1 fi
Þ log G =
n
(log x1 + log x2 + log x3 + . . . + log xn ) å xi
i =1
1 n
Þ log G = å log xi
n i =1 Example 5. Find the harmonic mean of
1 2 3
, , , ...,
n
,
2 3 4 n +1
é1 n ù
Þ G = antilog ê å log xi ú occurring with frequencies 1, 2, 3, …, n, respectively.
êë n i =1 úû n -1 n +1
(a) (b)
3 -n 3+n
Geometric Mean for Grouped Data n +1
If x1 , x2 , . . . , xn be n observations whose corresponding (c) (d) None of these
3 -n
frequencies are f1 , f2 , . . . , fn , then geometric mean is given
by Sol. (b) We know that,
é1 n ù åf
GM = ( x1f1 × x2f2 . . . xnfn )1/ N = antilog ê å fi log xi ú, Harmonic mean =
æfö
êë N i = 1 úû å çè x ÷ø
n
where N = å fi \ å f =1+ 2 + 3 + ... + n =
n(n + 1)
i =1 2
f 1 2 3 n
Example 4. The geometric mean G of the product of n
and å x = 1/ 2 + 2 / 3 + 3 / 4 + . . . + n / (n + 1)
series of data with geometric means G1, G2, ........., Gn 3 ´2 4 ´3 n(n + 1)
respectively, then =2 + + + ... +
2 3 n
(a) G = G1G 2 ...... G n (b) G = G1G 2 ...... G n = 2 + 3 + 4 + . . . + n + (n + 1)
(c) G = - G1G 2 ...... G n (d) None of these which is an arithmetic progression with a = 2 and d = 1.
By the formula of sum of n term of an AP,
Sol. (b) Let x1, x2, ...... , xn be the variates corresponding to n sets f én ù
of data, each having the same number of observations say K å x = êë 2 { 2a + (n - 1) d} úû , we have
and x be their product. n
Then, x = x1x2 ...... xn = {2 ´ 2 + n - 1}
2
i.e. log x = log x1 + log x2 + .... + log xn n
\ = (3 + n)
S log x S log x1 S log x2 S log xn 2
or = + + ...... + n(n + 1)
K K K K
\ Harmonic mean = 2
or log G = log G1 + log G 2 + ...... + log G n n(3 + n)
Þ G = G1G 2 ...... G n 2
n(n + 1) ´ 2
=
Harmonic Mean (HM) n(3 + n) ´ 2
The harmonic mean of any series is the reciprocal of the n +1
=
arithmatic mean of the reciprocals of the observations. 3+n
Statistics 493

Example 6. The mean and the median of the following


Median ten numbers in increasing order 10, 22, 26, 29, 34, x, 42, 67,
A set of observations whenever arranged in order of y
magnitude (ascending or descending), then the 70, y are 42 and 35 respectively, then is equal to
x
middlemost or central value gives the median. It divides (JEE Main 2019)
the observation into two equal parts in such a way that 7 7 8 9
(a) (b) (c) (d)
the number of observations smaller than median is equal 3 2 3 4
to the number greater than it.
Sol. (a) Given ten numbers are
Median for Ungrouped or Individual Data 10, 22, 26, 29, 34, x, 42, 67, 70, y
Suppose n observations are arranged in ascending or and their mean = 42
descending order.
10 + 22 + 26 + 29 + 34 + x + 42 + 67 + 70 + y
(a) If n is an odd number, then \ = 42
10
æ n + 1ö 300 + x + y
Median = Value of ç ÷ th term Þ = 42
è 2 ø 10
(b) If n is an even number, then Þ x + y = 120 …(i)
and their median (arranged numbers are in increasing order)
Median
= 35
ænö æn ö
Value of ç ÷ th term + Value of ç + 1÷ th term 34 + x
è 2ø è2 ø Þ = 35
= 2
2
Þ 34 + x = 70
Median for Discrete Frequency Data Þ x = 36
First arrange the data in ascending or descending order On substituting x = 36 in Eq. (i), we get
N 36 + y = 120
and find cumulative frequency. Now find , where
2 Þ y = 84
N = S fi . See the cumulative frequency just greater than y 84 7
N \ = =
. The corresponding value of x is median. x 36 3
2

Median for Continuous Data Measures of Dispersion


In this case, the class corresponding to the A measure of dispersion is designed to state the extent to
which the individual observations vary from their
N
cumulative frequency just greater than is called the average. These are following types of dispersion are
2
median class and the value of median is obtained by the (i) Range
following formula. (ii) Quartile deviation
æN ö (iii) Mean deviation
ç - c÷
2 (iv) Standard deviation
Median = l + ç ÷´h
ç f ÷ (v) Variance
è ø
In this chapter, we shall study all of these measures of
where, l = lower limit of median class dispersion except the quartile deviation.
f = frequency of median class
h = size of median class Range
c = cumulative frequency of class before Range is the difference between two extreme
median class observations of distribution. If L and S are largest and
Note Cumulative frequency of a value is its frequency plus the smallest observations respectively, then
frequencies of all smaller values L-S
Range = L - S and coefficient of range =
e.g. L+S
x f cf
In a grouped frequency distribution, the range is
1 4 4
calculated by subtracting the lower limit of lower class
2 6 10 ( 4 + 6 )
3 4 14 ( 4 + 6 + 4 ) intervals from the upper limit of the highest class.
494 JEE Main Mathematics

Sol. (a)
Mean Deviation
Class fi cf Mid value xi |xi - M| fi|xi - M|
Mean deviation of a distribution is the arithmetic mean
of the absolute deviation of the terms of the distribution 0-10 6 6 5 |5 - 27.86|= 22.86 137.16
from its statistical mean (AM, median or mode). 10-20 8 (14)C 15 |5 - 27.86| = 12.86 102.88
(20-30) (14) 28 25 |25 - 27.86|= 2.86 40.04
For Ungrouped or Individual Series 30-40 16 44 35 |35 - 27.86|= 7.14 114.24
If x1 , x2 , .... , xn are n observations, then mean deviation 40-50 4 48 45 |45 - 27.86|= 17.14 68.56
from average, A (usually mean, median, mode) is 50-60 2 50 55 |55 - 27.86|= 27.14 54.28
n n
Total Sfi = 50 517.16
å ½ xi - A ½ å ½ d i ½
i =1 i =1
MD = = N 50
= = 25
n n 2 2
where, di = xi - A Þ C = 14, f = 14, l = 20 , h = 10
N
For Grouped Distribution -C
Median M = l + 2 ´h
If x1 , x2 , ... , xn are n observations whose corresponding f
frequencies are f1 , f2 , ... , fn , then mean deviation from 25 - 14 11 ´ 10
average A (usually mean, median or mode) is = 20 + ´ 10 = 20 +
14 14
n n
= 20 + 7.86 = 27.86
å fi½ xi - A½ å fi|di|
Sf| x - M|
i =1 i =1
MD = = \ Mean deviation about median = i i
n n Sfi
å fi å fi
517.16
i =1 i =1 = = 10.34
50
Mean deviation
Coefficient of MD =
Corresponding average
Standard Deviation
Example 7. The mean deviation from the mean of the AP The standard deviation of a variate is the square root of
the arithmetic mean of the squares of all deviations of
a, a + d , a + 2d , ¼, a + 2nd is
the values of the variate x from the arithmatic mean of
n(n + 1)d the observations and is denoted by s.
(a) n(n + 1)d (b)
2n + 1
n(n + 1)d n(n - 1)d Standard Deviation for Individual Series
(c) (d)
2n 2n + 1 If x1 , x2 , x3 , ... , xn are n observations, then
n n
Sol. (b) The mean of the series
å ( xi - x )2 å xi2
a, a + d , a + 2d , ¼ , a + 2nd is SD (s ) =
i =1
or (s ) =
i =1
- ( x )2
1 n n
X= [ a + ( a + d) + ( a + 2d) + ¼ + ( a + 2nd )]
2n + 1 2
1 n æ1 n ö
=
1 ì 2n + 1
í
ü
( a + a + 2nd)ý = a + nd
or s=
n
å di2 - çç å di ÷÷
2n + 1î 2 þ i =1 èn i =1 ø

Therefore, mean deviation from mean where, di = xi - x


2n 2n
1 1
= å
2n + 1 r = 0
|( a + rd) - ( a + nd)| = å |r - n|d
2n + 1 r = 0
Standard Deviation for Frequency Distribution
If x1 , x2 , x3 , ... , xn are n observations and corresponding
1 n(n + 1) frequencies are f1 , f2 , ... , fn respectively, then
= × 2d(1 + 2 + ¼ + n) = d
2n + 1 2n + 1 n n
Hence, option (b) is the correct answer. å fi ( xi - x )2 å fi xi2
i =1 i =1
s= or (s ) = - ( x )2
Example 8. Find the mean deviation about median for N N
the following data. 2
1 æ n ö 1 æ n ö
Marks 0-10 10-20 20-30 30-40 40-50 50-60 or s= çç å fi di2 ÷÷ - çç å fi di ÷÷
Number of girls 6
N èi = 1 ø N èi = 1 ø
8 14 16 4 2

(a) 10.34 (b) 10.50 (c) 10.40 (d) 10.37 where, di = xi - x and N = S fi
Statistics 495

x
Properties of Standard Deviation Þ 40 + = 48
6
(i) It is independent of origin. x
Þ = 8 Þ x = 48
(ii) It is dependent of change of scale. 6
(iii) It is not less than mean deviation from mean. Now, standard deviation of these marks
(iv) Let n1 and n 2 be the sizes of two series. Their means 412 + 45 2 + 43 2 + 54 2 + 57 2 + 48 2
and standard deviations are x1 , x2 and s1 , s 2 = - 48 2
6
respectively. Let x denote the combined mean of two
series, that is Sxi2
[Q standard deviation (SD) = - ( x) 2]
n x + n 2 x2 6
x= 1 1
n1 + n 2 ( 412 - 48 2) + ( 45 2 - 48 2) + ( 43 2 - 48 2)
\ The combined standard deviation of two series is + (54 2 - 48 2) + (57 2 - 48 2)
=
given by 6
n1 (s12 + d12 ) + n 2 (s 22 + d22 ) ( -7 ´ 89) + ( -3 ´ 93) + ( -5 ´ 91)
s=
n1 + n 2 + (6 ´ 102) + (9 ´ 105)
=
6
n1 s12 + n 2s 22 n1n 2 ( x1 - x2 )2
or s= + 945 + 612 - 455 - 279 - 623
n1 + n 2 ( n1 + n 2 )2 =
6
where, d1 = x1 - x and d2 = x2 - x 1557 - 1357 200
= =
5 (Mean deviation) 6 6
s=
4 100 10
= =
n n 3 3
Example 9. If å ( xi - a) = n and å ( xi - a) 2 = na, (n, a > 1),
i =1 i =1 Example 11. If both the mean and the standard deviation of
then the standard deviation of n observations x1, x2, K, xn is 50 observations x1, x2, K , x50 are equal to 16, then the mean
(JEE Main 2020) of ( x1 - 4) 2,( x2 - 4) 2, K , ( x50 - 4) 2 is (JEE Main 2019)
(a) a -1 (b) n a -1 (c) n( a -1) (d) a -1
(a) 480 (b) 400 (c) 380 (d) 525
n n
Sol. (d) It is given that å (xi - a) = n and å (xi - a) 2 = na, (n, a > 1) Sol. (b) It is given that both mean and standard deviation of
i =1 i =1 50 observations x1, x2, x3 , K , x50 are equal to 16,
\ Standard deviation of ‘n’ observations Sx
So, mean = i = 16 …(i)
n n 50
å (xi - a) 2 å (xi - a) Sxi2 æ Sxi ö
2
i =1 i =1
= - and standard deviation = - ç ÷ = 16
n n 50 è 50 ø
na n Sxi2
= - = a -1 Þ - (16) 2 = (16) 2
n n 50
Sxi2
Example 10. A student scores the following marks in five Þ = 2 ´ 256 = 512 …(ii)
50
tests 45, 54, 41, 57, 43. His score is not known for the sixth
Now, mean of ( x1 - 4) 2, ( x2 - 4) 2, K ,( x50 - 4) 2
test. If the mean score is 48 in the six tests, then the standard
deviation of the marks in six tests is (JEE Main 2019) S( xi - 4) 2 S( xi2 - 8xi + 16)
= =
10 10 50 50
(a) (b)
3 3 Sxi2 æ Sx ö 16
= -8ç i÷ + S1
100 100 50 è 50 ø 50
(c) (d)
3 3 æ 16 ö
= 512 - (8 ´ 16) + ç ´ 50 ÷
Sol. (b) Let the marks in sixth tests is ‘x’, so è 50 ø
41 + 45 + 43 + 54 + 57 + x [from Eqs. (i) and (ii)]
Mean = = 48 (given)
6 = 512 - 128 + 16
240 + x = 400
Þ = 48
6
496 JEE Main Mathematics

Sol. (c) Given, sets


Variance X = { x ÎN : 1 £ x £ 17}
The variance of a variate is the square of standard and Y = { ax + b : x Î X and a, b ÎR, a > 0}
deviation and it is denoted by s 2 or Var ( x ). 17
S ( ax + b)
x =1
2 2 Now, = 17
Var ( x ) = s = (standard deviation) 17
s 17
Coefficient of dispersion = ´ 100, x ¹ 0 Þ a S x + 17b = 17 ´ 17 Þ 9a + b = 17 … (i)
x x =1
17
Example 12. The mean and variance of 8 observations are S ( ax + b) 2
x =1
10 and 13.5 respectively. If 6 of these observations are 5,7,10, and - (17) 2 = 216
17
12, 14, 15, then the absolute difference of the remaining two 17
observations is (JEE Main 2020) Þ S ( ax + b) 2 = 17 [216 + 289]
x =1
(a) 9 (b) 3 (c) 7 (d) 5 = 17 ´ 505 = 8585
17 17
Sol. (c) Let remaining two observations are x and y, so mean 2 2
Þ a S x + 2ab S x + 17b = 8585
x =1 x =1
5 + 7 + 10 + 12 + 14 + 15 + x + y
= = 10 æ17 ´ 18 ´ 35 ö æ17 ´ 18 ö
8 Þ a2 ç ÷ + 2abç ÷ + 17b = 8585
è 6 ø è 2 ø
Þ x + y = 80 - 63
Þ x + y = 17 …(i) Þ 105a2 + 18ab + b 2 = 505 … (ii)
25 + 49 + 100 + 144 + 196 + 225 + x2 + y 2 From Eqs. (i) and (ii), we get
and, Variance =
8 105a2 + 18a(17 - 9a) + (17 - 9a) 2 = 505
-100 = 13.5 Þ105a2 + 306a - 162a2 + 289 + 81a2- 306a = 505
739 + x2 + y 2
Þ = 113.5 Þ 24a2 = 216 Þ a2 = 9
8
Þ a = 3 as a > 0
Þ x2 + y 2 = 908 - 739
So, b = - 10
Þ x2 + y 2 = 169 …(ii) \ a + b = 3 - 10 = - 7
From Eqs. (i) and (ii), we get
x = 12 and y = 5 Effect of Average and Dispersion on
Change of Origin and Scale
\ | x - y| = |12 - 5 | = 7
Hence, option (c) is correct. Change of origin Change of scale

Example 13. Let X = {x e N : 1£ x £ 17} and Mean Dependent Dependent


Y = {ax + b : x Î X and a, b Î R, a > 0}. If mean and variance Median Not dependent Dependent
of elements of Y are 17 and 216 respectively, then a + b Mode Not dependent Dependent
is equal to (JEE Main 2020) Standard deviation Not dependent Dependent
(a) 9 (b) 7 (c) –7 (d) –27 Variance Not dependent Dependent
Practice Exercise
ROUND I Topically Divided Problems
Mean (a) 2 n + 1 (b)
1
(2 n + 1)
2
1. If the sum of the deviations of 50 observations from 1 2n + 1
30 is 50, then the mean of these observations is (c) (2 n + 1) (d)
3 6
(JEE Main 2019)
(a) 50 (b) 30 (c) 51 (d) 31 9. If the mean of the set of numbers x1, x2 , . . . , xn is x,
2. The AM of n numbers of a series is x. If the sum of then the mean of the numbers xi + 2 i, 1 £ i £ n is
first ( n - 1) terms is k, then the nth number is (a) x + 2n (b) x + n + 1
(a) x - k (b) nx - k (c) x - nk (d) nx - nk (c) x + 2 (d) x + n

3. The mean of the series x1, x2 , . . . , xn is x. If x 2 is 10. The mean of a set of observations is x. If each
observation is divided by, a ¹ 0 and then is
replaced by l, then the new mean is
x - x2 - l
increased by 10, then the mean of the new set is
(a) x - x 2 + l (b) x x + 10 x + 10 a
n (a) (b) (c) (d) a x + 10
a a a
(n - 1)x + l nx - x 2 + l
(c) (d) 11. The mean of n observations is x. If one observation
n n
xn + 1 is added, then the mean remains same. The
4. Consider three observations a, b and c such that value of xn + 1 is
b = a + c. If the standard deviation of a + 2 , b + 2 ,
(a) 0 (b) 1 (c) n (d) x
c + 2 is d, then which of the following is true?
(JEE Main 2021) 12. In a class of 100 students, the average amount
(a) b2 = 3 (a 2 + c2) + 9d 2 (b) b2 = a 2 + c2 + 3d 2 of pocket money is ` 35 per student. If the average
(c) b2 = 3 (a 2 + c2 + d 2) (d) b2 = 3 (a 2 + c2) - 9d 2 is ` 25 for girls and ` 50 for boys, then the number
5. If the average of the numbers 148, 146, 144, 142, ... of girls in the class is
in AP, be 125, then the total numbers in the series (a) 20 (b) 40 (c) 60 (d) 80
will be 13. If the sum of deviations of a number of observations
(a) 18 (b) 24 (c) 30 (d) 48 about 4 is 30 and that about 3 is 40. Then, mean of
6. The mean of n items is x. If the first term is increased the observations is
by 1, second by 2 and so on, then the new mean is (a) 7 (b) 10
n (c) 11 (d) None of these
(a) x + n (b) x +
2 14. If a variate x is expressed as a linear function of
n+1
(c) x + (d) None of these two variates u and v in the form x = au + bv, then
2
mean x of x is
7. If the mean of n observations 12 , 2 2 , 32 , . . . , n2 is (a) au + bv (b) u + v
46 n (c) b u + a v (d) None of these
, then n is equal to
11
15. The average salary of male employees in a firm was
(a) 11 (b) 12 (c) 23 (d) 22
` 520 and that of females was ` 420. The mean
8. The weighted AM of first n natural numbers whose salary of all the employees was ` 500. The
weights are equal to the corresponding numbers is percentage of male employees is
equal to (a) 80% (b) 60% (c) 40% (d) 20%
498 JEE Main Mathematics

16. The AM of n observations is M. If the sum of ( n - 4) 24. Geometric mean of first group of 5 observations is 8
observations is a , then the mean of remaining four and that of second group of 4 observations is
observations is 128 2. Then, grouped geometric mean is
nM - a nM + a nM - a (a) 64 (b) 32 2 (c) 32 (d) None of these
(a) (b) (c) (d) nM + a
4 2 2
2n + 1 2n + 1 2n + 1 2n + 1
Mean Deviation, Standard Deviation
17. The AM of C0 , C1, C2 , . . . , Cn is
and Variance
2n 2n 2 2n 2 2n
(a) (b) (c) (d) 25. Following are the marks obtained by 9 students in
n n+1 n (n + 1)
a Mathematics test 50, 69, 20, 33, 53, 39, 40, 65, 59
18. If a variable takes values 0, 1, 2, …, n with The mean deviation from the median is
n n -1 n( n - 1) n - 2 2
frequencies qn , q p, q p , . . . pn, (a) 9 (b) 10.5 (c) 12.67 (d) 14.76
1 1× 2
where p + q = 1, then the mean is 26. Let in a series of 2n observations, half of them are
equal to a and remaining half are equal to - a.
(a) np (b) nq
(c) n ( p + q) (d) None of these Also, by adding a constant b in each of these
observations, the mean and standard deviation of
Geometric and Harmonic Mean new set become 5 and 20, respectively, then the
value of a 2 + b2 is equal to (JEE Main 2021)
19. Let G1 and G2 be the … geometric means of two …
(a) 425 (b) 650 (c) 250 (d) 925
series x1, x 2 , … , x n ; y1, y2 , . . . , yn . If G is the
x 27. The mean and variance of 20 observations are
geometric mean of i , i = 1, 2 , . . . , n. Then, G is found to be 10 and 4, respectively. On rechecking,
yi
it was found that an observation 9 was incorrect
equal to
and the correct observation was 11. Then, the
log G1 G1 æG ö
(a) G1 - G 2 (b) (c) (d) log ç 1 ÷ correct variance is (JEE Main 2020)
log G2 G2 è G2 ø
(a) 4.01 (b) 3.99 (c) 3.98 (d) 4.02
20. If the observations 2, 4, 8 and 16 occur 8, 6, 4 and 28. Let xi (1 £ i £ 10) be ten observations of a random
2 times respectively, then the geometric mean of 10 10
variable X. If S ( xi - p) = 3 and S ( xi - p) 2 = 9 p
the observations is i =1 i =1
(a) 8 (b) 4 2 where 0 ¹ p Î R, then the standard deviation of
(c) 4 (d) None of these
these observation is (JEE Main 2020)
5 9 3 7 4
21. If harmonic mean of first 5 observations is and (a) (b) (c) (d)
2 10 5 10 5
9
harmonic mean of another 5 observations is , then
2 29. If the mean and the standard deviation of the data
harmonic mean of all 10 observations is 3, 5, 7, a, b are 5 and 2 respectively, then a and b are
45 the roots of the equation (JEE Main 2020)
(a) 7 (b)
14 (a) x2 - 10x + 18 = 0 (b) 2x2 - 20x + 19 = 0
101 (c) x2 - 10x + 19 = 0 (d) x2 - 20x + 18 = 0
(c) (d) None of these
36
30. The mean and the standard deviation (s.d.) of 10
22. Geometric mean of 10 observations is 8. observations are 20 and 2 respectively. Each of
If geometric mean of first six observations is 4, these 10 observations is multiplied by p and then
then geometric mean of last four observations is reduced by q,where p ¹ 0 and q ¹ 0. If the new mean
(a) 16 2 (b) 8 2 and new s.d. become half of their original values,
(c) 16 (d) None of these then q is equal to (JEE Main 2020)

23. Two possible observations have arithmetic mean 3 (a) 10 (b) - 10 (c) - 5 (d) - 20
and geometric mean 2 2. If each observation is aX + b
31. If SD of X is S, then SD of the variable m = ,
multiplied by 2, then harmonic mean will be c
16 8 where a , b and c are constants, is
(a) (b) c a b c2
3 3 (a) s (b) s (c) s (d) s
(c) 12 (d) None of these a c c a2
Statistics 499

32. Let x1, x2 , . . . , xn be n observations. Let wi = lxi + k 41. The mean of the data set comprising of 16
for i = 1, 2, . . . , n, where l and k are constants. If the observations is 16. If one of the observation valued
mean of xi ’s is 48 and their standard deviation is 16 is deleted and three new observations valued 3,
12, the mean of wi ’s is 55 and standard deviation of 4 and 5 are added to the data, then the mean of the
wi ’s is 15, the values of l and k should be resultant data is (JEE Main 2015)
(a) l = 1.25, k = - 5 (b) l = - 1.25, k = 5 (a) 16.8 (b) 16.0
(c) l = 2.5, k = - 5 (d) l = 2.5, k = 5 (c) 15.8 (d) 14.0
9 9
33. Consider the numbers 1, 2, 3, 4, 5, 6, 7, 8, 9, 10.
If 1 is added to each number, then variance of the
42. If å (xi - 5) = 9 and å (xi - 5) 2 = 45, then the
i =1 i =1
numbers so obtained is standard deviation of the 9 items x1, x2 , ¼ , x9 is
(a) 6.5 (b) 2.87 (c) 3.87 (d) 8.25 (JEE Main 2018)
34. Let the observations xi (1 £ i £ 10) satisfy the (a) 9 (b) 4 (c) 2 (d) 3
10 10 49
43. The variance of the numbers 2, 3, 11 and x is .
equations, å ( xi - 5) = 10 and å ( xi - 5) 2 = 40. 4
i =1 i =1
Find the value of x.
If m and l are the mean and the variance of the 14 14
(a) 6, (b) 6,
observations, x1 - 3, x2 - 3, . . . . . , x10 - 3, then the 3 5
ordered pair (m, l) is equal to (JEE Main 2020) 16
(c) 6, (d) None of these
(a) (6, 3) (b) (3, 6) (c) (3, 3) (d) (6, 6) 3

35. If the standard deviation of the numbers -1, 0, 1, k 44. Mean and standard deviation of 100 observations
is 5 where k > 0, then k is equal to (JEE Main 2019) were found to be 40 and 10, respectively. If at the
10 5
time of calculation two observations were wrongly
(a) 2 (b) 2 6 (c) 4 (d) 6 taken as 30 and 70 in place of 3 and 27,
3 3
respectively, find the correct standard deviation.
36. The mean of five observations is 5 and their (a) 10.20 (b) 10.24
variance is 9.20. If three of the given five (c) 10.29 (d) 10.27
observations are 1, 3 and 8, then a ratio of other
45. The mean and the variance of five observations are
two observations is (JEE Main 2019)
4 and 5.20, respectively. If three of the observations
(a) 4 : 9 (b) 6 : 7 (c) 10 : 3 (d) 5 : 8
are 3, 4 and 4, then the absolute value of the
37. Coefficient of variation of two distributions are difference of the other two observations, is
50 and 60 and their arithmetic means are 30 and (JEE Main 2019)
25 respectively. Difference of their standard (a) 1 (b) 7 (c) 5 (d) 3
deviation is 46. The outcome of each of 30 items was observed 10
(a) 0 (b) 1 (c) 1.5 (d) 2.5 1
items gave an outcome - d each, 10 items gave
2
38. If for a distribution S( x - 5) = 3, S( x - 5) = 43 and 2
1
the total number of items is 18, find the mean and outcome each and the remaining 10 items gave
2
standard deviation. 1
(a) 94, 1.53 (b) 93, 1.53 outcome + d each. If the variance of this outcome
2
(c) 93, 1.55 (d) None of these 4
data is , then|d|equals (JEE Main 2019)
39. The mean and variance of n values of a variable x 3
are 0 and s 2 , respectively. If the variable y = x 2 , 2 5
(a) (b) (c) 2 (d) 2
then mean of y is 3 2
(a) s (b) s 2 47. The mean life of a sample of 60 bulbs was 650 h
(c) 1 (d) None of these and the standard deviation was 8 h. A second
sample of 80 bulbs has a mean life of 660 h and
40. If the standard deviation of the numbers 2, 3, a and
standard deviation 7 h. Find the over all standard
11 is 3.5, then which of the following is true?
(JEE Main 2016) deviation.
(a) 3a 2 - 26a + 55 = 0 (b) 3a 2 - 32a + 84 = 0 (a) 8.97 (b) 8.98
(c) 3a 2 - 34a + 91 = 0 (d) 3a 2 - 23a + 44 = 0 (c) 8.94 (d) None of these
500 JEE Main Mathematics

ROUND II Mixed Bag


Only One Correct Option 9. The first of two samples has 100 items with mean
1. The mean and variance of 7 observations are 8 and 15 and SD = 3. If the whole group has 250 items
16, respectively. If five observations are 2, 4, 10, 12, with mean 15.6 and SD = 13. 44 , the SD of the
14, then the absolute difference of the remaining second group is
two observations is (JEE Main 2020) (a) 4 (b) 5 (c) 6 (d) 3.52
(a) 1 (b) 3 (c) 4 (d) 2 10. If the data x1, x2 , ¼ , x10 is such that the mean of
2. For the frequency distribution Variate ( x) : x1 x2 first four of these is 11, the mean of the remaining
x3K x15 Frequency ( f ) : f1 f2 f3K f15 where six is 16 and the sum of squares of all of these is
15 2000, then the standard deviation of this data is
0 < x1 < x2 < x3 < K < x15 = 10 and S fi > 0, the (JEE Main 2019)
i =1
(a) 2 2 (b) 2 (c) 4 (d) 2
standard deviation cannot be (JEE Main 2020)
11. 5 students of a class have an average height 150 cm
(a) 4 (b) 1 (c) 6 (d) 2
and variance 18 cm 2 . A new student, whose height
3. The mean and variance of seven observations are 8 is 156 cm, joined them. The variance (in cm 2 ) of the
and 16, respectively. If 5 of the observations are 2, height of these six students is(JEE Main 2019, 9 January)
4, 10, 12, 14, then the product of the remaining two (a) 16 (b) 22 (c) 20 (d) 18
observations is (JEE Main 2019)
12. In a group of data, there are n observations,
(a) 45 (b) 49 (c) 48 (d) 40 n n
x, x2 , . . . . , xn . If S ( xi + 1) 2 = 9 n and S ( xi - 1) 2 = 5 n,
4. The marks of some students were listed out of 75. i =1 i =1

The SD of marks was found to be 9. Subsequently the the standard deviation of the data is (JEE Main 2019)
marks were raised to a maximum of 100 and (a) 2 (b) 7 (c) 5 (d) 5
variance of new marks was calculated. The new 13. If mean and standard deviation of 5 observations
variance is x1, x2 , x3, x4 , x5 are 10 and 3, respectively, then the
(a) 81 (b) 122 variance of 6 observations x1, x2 , . . . . . x5 and - 50 is
(c) 144 (d) None of these equal to (JEE Main 2019)
5. A batsman scores sums in 10 innings 38, 70, 48, 34, (a) 507.5 (b) 586.5
42, 55, 46, 63, 54 and 44, then the deviation from (c) 582.5 (d) 509.5
median is 14. There are 60 students in a class. The following is
(a) 8.6 (b) 6.4 (c) 9.6 (d) 10.6
the frequency distribution of the marks obtained by
18 18
the students in a test
6. If å ( xi - 8) = 9 and å ( xi - 8) 2
= 45, then the
i =1 i =1 Marks 0 1 2 3 4 5
standard deviation of x1, x2 , . . . , x18 is Frequency x-2 x x2 ( x + 1)2 2x x+1
4 9
(a) (b) where, x is a positive integer. Find the mean and
9 4
standard deviation of the marks.
3
(c) (d) None of these (a) 2.9, 1.14 (b) 2.8, 1.12
2
(c) 2.8, 1.18 (d) None of these
7. If the standard deviation of x1, x2 , . . . , xn is 3.5, then
15. If the mean deviations about the median of the
the standard deviation of - 2 x1 - 3, - 2 x2 - 3, . . . ,
numbers a, 2 a, . . . , 5 a is 50, then|a |is equal to
- 2 xn - 3 is
(a) 3 (b) 4 (c) 5 (d) 2
(a) -7 (b) -4 (c) 7 (d) 1.75
8. The mean deviation from mean of the observation 16. For two data sets, each of size 5, the variance are
a, a + d, a + 2 d, . . . , a + 2 nd is given to be 4 and 5 and the corresponding means
2 are given to be 2 and 4, respectively. The variance
n (n + 1)d n (n + 1) 2
(a) (b) d of the combined data set is
3 2
5 11 13
n (n + 1)d 2 (a) (b) (c) 6 (d)
(c) a + (d) None of these 2 2 2
2
Statistics 501

n
17. If the mean deviation of number C0 , nC1, n C2 , . . . . , nCn respectively. If the mean of
1, 1 + d, 1 + 2 d, . . . , 1 + 100 d from their mean is 255, 728
this data is n , then n is equal to (JEE Main 2020)
then the d is equal to 2
(a) 10.0 (b) 20.0 21. If the variance of the terms in an increasing AP,
(c) 10.1 (d) 20.2
b1, b2 , b3, ……, b11 is 90, then the common difference
18. The mean of the numbers a, b, 8, 5 and 10 is 6 and of this AP is ……… . (JEE Main 2020)

the variance is 6.80. Then, which one of the 22. If the variance of 10 natural numbers 1, 1, 1, ..., 1, k
following gives possible values of a and b? is less than 10, then the maximum possible value of
(a) a = 3, b = 4 (b) a = 0, b = 7 k is (JEE Main 2021)
(c) a = 5, b = 2 (d) a = 1, b = 6
19. The average marks of boys in a class is 52 and that 23. If the mean and variance of eight numbers 3, 7, 9,
of girls is 42. The average marks of boys and girls 12, 13, 20, x and y be 10 and 25 respectively, then
combined is 50. The percentage of boys in the class is x × y is equal to …… . (JEE Main 2020)

(a) 40% (b) 20% 24. Let X1, X 2 , . . . . . . X18 be eighteen observation such
(c) 80% (d) 60% 18 18
that å ( X i - a ) = 36 and å ( X i - b) 2 = 90, where
Numerical Value Type Questions i =1 i =1

20. Consider the data on x taking the values 0, 2, 4, 8, a and b are distinct real numbers. If the standard
……, 2 n with frequencies n C0 , nC1, n C2 , . . . . , nCn deviation of these observations is 1, then the value
of|a - b|is (JEE Main 2021)

Answers
Round I
1. (d) 2. (b) 3. (d) 4. (d) 5. (b) 6. (c) 7. (a) 8. (c) 9. (b) 10. (c)
11. (d) 12. (c) 13. (a) 14. (a) 15. (a) 16. (a) 17. (d) 18. (a) 19. (c) 20. (c)
21. (b) 22. (a) 23. (a) 24. (c) 25. (c) 26. (a) 27. (b) 28. (a) 29. (c) 30. (d)
31. (b) 32. (a) 33. (d) 34. (c) 35. (b) 36. (a) 37. (a) 38. (b) 39. (b) 40. (b)
41. (d) 42. (c) 43. (a) 44. (b) 45. (b) 46. (c) 47. (c)

Round II
1. (d) 2. (c) 3. (c) 4. (c) 5. (a) 6. (c) 7. (c) 8. (d) 9. (a) 10. (b)
11. (c) 12. (d) 13. (a) 14. (b) 15. (b) 16. (b) 17. (c) 18. (a) 19. (c) 20. (6)
21. (3) 22. (11) 23. (54) 24. (4)
Solutions
Round I 5. Given series is 148, 146, 144, 142, ... whose first term
1. Let the 50 observations are x1 , x2, x3 , ... , x50. and common difference is
a = 148, d = (146 - 148) = - 2
Now, deviations of these observations from 30 are
n
(x1 - 30), (x2 - 30), (x3 - 30), ... , (x50 - 30) S n = [2a + (n - 1 ) d ] = 125n (given)
50 2
Q å (xi - 30) = 50 (given)
Þ
n
125n = [2 ´ 148 + (n - 1 ) ´ (- 2)]
i =1 2
50
Þ n 2 - 24n = 0
Þ å xi - (30 ´ 50) = 50 Þ n (n - 24) = 0 Þ n = 24 (n ¹ 0)
i =1
50
6. Let x1 , x2, . . . xn be the n items.
Þ å xi = 50(30 + 1) = 50 ´ 31 x1 + x2 + . . . + xn
i =1
50
Given, x= …(i)
n
å xi (x + 1 ) + (x2 + 2) + . . . + (xn + n )
\ Mean of 50 observations = (x ) =
i =1
= 31 \ New mean = 1
50 n
(x1 + x2 + . . . + xn ) + (1 + 2 + . . . + n )
2. Let the n numbers be x1 , x2, . . . , xn. Then, =
n
1 n n (n + 1 )
x=å xi
n i=1
=x+
2n
[Q from Eq. (i)]

x + x2 + . . . + xn-1 + xn (n + 1 )
Þ x= 1 =x+
n 2
k + xn 7. According to the given condition,
Þ x= (Q x1 + x2 + . . . + xn-1 = k)
n 12 + 22 + 32 + . . . + n 2 46n
\ xn = nx - k =
n 11
x1 + x2 + . . . + xn-1 + xn n (n + 1 )(2n + 1 ) 46n
3. We have, x = Þ =
n 6n 11
Þ nx = x1 + x2 . . . + xn-1 + xn
Þ 11 (2n 2 + 3n + 1 ) = 276n
Let y be the new mean when x2 is replaced by l. Then,
Þ 22n 2 - 243n + 11 = 0
x + l + x3 + . . . + xn-1 + xn
y= 1 Þ (n - 11 )(22n - 1 ) = 0
n
(x1 + x2 + . . . + xn ) - x2 + l Þ n = 11
Þ y=
n 8. The required mean is
nx - x2 + l 1 × 1 + 2 × 2 + 3 × 3 + ... + n × n
\ y= X =
n 1 + 2 + 3 + ... + n
4. For a , b, c (2 n + 1 )
n (n + 1 )
a+ b+ c S n2 6
Mean = = =
Sn n (n + 1 )
3
2
Þ b=a+ c
2n + 1
2b =
Þ x= 3
3 n
1
SD (a + 2, b + 2, c + 2) = SD (a , b, c) = d 9. x =
n
å xi
i=1
a 2 + b2 + c2
Þ d2 = - ( x )2 n
3 Þ å x i = nx
i=1
2 a 2 + b2 + c2 4b2
Þ d = - 1 n
1 n
2(1 + 2 + . . . + n )
3 9 Let M =
n
å (xi + 2i ) =
n
å xi +
n
i=1 i=1
Þ 9d 2 = 3a 2 + 3b2 + 3c2 - 4b2
1 n (n + 1 )
Þ 9d 2 = 3a 2 - b2 + 3c2 Þ M = nx + 2
n 2n
\ b2 = 3 (a 2 + c2) - 9d 2
\ M = x + (n + 1 )
Statistics 503

10. Let x1 , x2, x3 , ... xn be n observations. Then, Hence, the percentage of male employees in the firm
1 n æ 4 ö
x= å xi =ç ÷ ´ 100 = 80%
n i =1 è4 + 1ø

1 n
æ xi ö 16. Let x1 , x2, ... , xn be n observations.
\ New mean, X =
n
å çè a + 10÷
ø x1 + x2 + x3 + ... + xn
i =1 \ M=
n
1 æ1 n ö 1
= ç å x i ÷ + × (10n ) x1 + x2 + ... + xn - 4 + xn - 3 + xn - 2 + xn - 1 + xn
a çn ÷ =
è i =1 ø n n
1 x + 10 a Þ nM = a + xn - 3 + xn - 2 + xn - 1 + xn
= x + 10 =
a a nM - a xn - 3 + xn - 2 + xn - 1 + xn
Þ =
4 4
11. Let x1 , x2, x3 , ... , xn be n observations.
2n + 1 2n + 1 2n + 1 2n + 1 2n + 1
S xi 17. C0 + C1 + C 2 + ... + C 2n + C 2n + 1
\ x= …(i) 2n + 1
n =2
S xi + xn + 1 Now, 2n + 1
C0 = 2n + 1
C 2n + 1 ,
New mean = 2n + 1 2n + 1 2n + 1 2n + 1
n+1 C1 = C 2n ... Cr = C 2n - r + 1
S x i + xn + 1 So, sum of first (n + 1 ) terms
According to the question, x =
n+1 = sum of last (n + 1 ) terms
2n + 1 2n + 1 2n + 1 2n + 1
Þ (n + 1 ) x = nx + xn + 1 Þ xn + 1 = x Þ C0 + C1 + C 2 + ... + C n = 22n
2n + 1 2n + 1
12. Let the number of girls in the class = y C0 + C1 + 2n + 1C 2 + ... + 2n + 1
Cn 22n
Þ =
n+1 (n + 1 )
\ Number of boys in the class = 100 - y
Now, x1 = 25, n1 = y, x2 = 50, n2 = 100 - y 18. Required mean,
and x = 35, n1 + n2 = 100 n n-1 n (n - 1 ) n- 2 2
0 × qn + 1 ×
q p + 2× q p + . . . + n × pn
25 ´ y + 50 ´ (100 - y) Þx = 1 2 !
\ 35 = n n (n - 1 ) n- 2 2
100 qn + qn-1 p + q p + . . . + pn
1 2!
Þ 3500 = 25 y + 5000 - 50 y
0 × nC qn p0 + 1 × nC1qn-1 p + . . . + n × nC nq0 pn
Þ 25 y = 1500 Þ y = 60 Þx = n 0 n 0 n
C 0q p + C1qn-1 p1 + . . . + nC nqn- n pn
\ Number of girls in the class = 60
n n
n
å r × nC rqn- r pr å n-1
C r-1qn- r p × pr-1
n

13. Given that, å (x i - 4) = 30
Þx = r= 0
= r= 0 r
i=1 n n
n
å n
C rq n- r r
p å n
C rqn- r pr
and å (x i - 3) = 40 r=0 r=0
i=1
é n ù
i.e., Sx i - 4n = 30 …(i) npê å n-1
C r-1 pr-1q( n-1) - ( r-1) ú
and Sx i - 3n = 40 …(ii) êë r = 1 úû
Þx = n
On solving Eqs. (i) and (ii), we get S nC rqn-r pr
r= 0
n = 10 and Sx i = 70
n-1
n np(q + p)
S xi Þx =
i =1 1 (q + p)n
\ Mean = = (70) = 7
n 10 \ x = np (Q q + p = 1 )
14. Since, x = au + bv 19. Given, G1 = (x1 × x2 . . . xn ) 1/ n
and G2 = ( y1 × y2 . . . yn )1/ n
\ Sx = aSu + bSv 1/ n
æx x x ö (x1 × x2 . . . xn )1/ n G1
Þ nx = a (nu ) + b(nv ) \ G = ç 1 . 2 ... n ÷ = =
è y1 y2 yn ø ( y1 × y2 . . . yn )1/ n G2
Þ x = au + bv
15. Since, x1 = 520, x2 = 420 and x = 500 20. Q We know that,
n1 x1 + n2x2 Geometric mean = n x1 ´ x2 ´ . . . ´ xn
Also, we know x=
n1 + n2 = 20 28 ´ 46 ´ 84 ´ 162
Þ 500(n1 + n2) = 520n1 + 420n2 = 20 28 ´ 212 ´ 212 ´ 28
Þ 20n1 = 80n2 40

Þ n1 : n2 = 4 : 1 = 20 240 = 2 20 = 22 = 4
504 JEE Main Mathematics

21. Let H 1 and n1 are the harmonic mean and number of 26. Let observations are denoted by xi for
observations of first group and H 2 and n2 are the
1 £ i £ 2n
harmonic mean and number of observations of another
S xi (a + a + K + a ) - (a + a + K + a )
group. x= = =0
2n 2n
\By using combined harmonic mean formula, we get
n + n2 5+5 S x2
Combined harmonic mean = 1 = and s x2 = i - (x )2
n1 n2 5 5 2n
+ +
H1 H 2 5 9 a2 + a2 + K + a2
= -0
2 2 2n
10 90 45 = a2
= = =
28 28 14
sx = a
9
Now, adding a constant b, then
22. We are given, here geometric mean of 10 observations is
y = x + b =5
8 and geometric mean of first six observations is 4, then
we have to find the geometric mean of last four
observations. Let G be the GM of last four observations. b =5
\By using grouped geometric mean, and s y = s x (No change in SD)
i.e., 810 = 46 ´ G 4 Þ a = 20
810 230 Þ a 2 + b2 = (20)2 + 52
Þ G 4 = 6 = 12 = 218
4 2 = 425
Þ G = 218/ 4 = 24 2 = 16 2
27. Let the observations are x1 , x2, x3 , …, x20, then mean
23. Since, we know that
= 10
(GM)2 = AM ´ HM
x1 + x2 + x3 + ¼ + x20
(2 2 )2 = 3 ´ HM Þ = 10
8 20
Þ HM =
3 Þ x1 + x2 + x3 + ¼ + x20 = 200 …(i)
Also, given here that, if each observation is multiplied and, variance = 4
by 2, then 2
8 16 x2 + x22 + x32 + ¼ + x20
2
æ x + x2 + x3 + ¼ + x20 ö
HM = ´ 2 = Þ 1 -ç 1 ÷ =4
3 3 20 è 20 ø
24. Suppose that, G1 is the GM of first 5 observations = 8 x12 + x22 + x32 + ¼ + x20
2
Þ - 100 = 4
and G2 is the GM of last 4 observations = 128 2 20
Also, let G be the grouped mean Þ x12 + x22 + x32 + ¼ + x20
2
= 2080 …(ii)
= 9 (G1n1 ´ G2n2 ) = 9 (G1n1 ´ G2n2 ) = (G15 ´ G24 )1/ 9 Let observation x1 = 9 was incorrect and its correct
1/ 9
value was 11.
é æ 7+ 1ö ù
4
x + x2 + x3 + ¼ + x20 - 9 + 11
5
= [8 ´ (128 2 ) ] 4 1/ 9
= ê (23 )5 ´ ç2 2 ÷ ú So, mean is x = 1
ê ç ÷ ú 20
è ø
ë û 200 - 9 + 11
45
= [from Eq. (i)]
20
= (215 ´ 230 )1/ 9 = 2 9 = 25 = 32 202
=
25. Firstly, given marks are arranged in ascending order 20
20, 33, 39, 40, 50, 53, 59, 65, 69 and variance is
æ9 + 1ö x2 + x22 + x32 + ¼ + x20
2
- 92 + 112 æ 202 ö
2
\ Median, M d = ç ÷ th term = 5th term = 50 s= 1 -ç ÷
è 2 ø 20 è 20 ø
9 2
å |xi - M d | =
2080 - 81 + 121 æ 202 ö
-ç ÷ [from Eq. (ii)]
i =1 20 è 20 ø
\Mean deviation, MD =
n 2
2120 æ 202 ö 2120 ´ 20 - (202)2
|20 - 50| + |33 - 50| + |39 - 50| + |40 - 50| + |50 = -ç ÷ =
20 è 20 ø (20)2
- 50| + |53 - 50| + |59 - 50| + |65 - 50| + |69 - 50|
= 42400 - 40804
9 =
30 + 17 + 11 + 10 + 0 + 3 + 9 + 15 + 19 400
= 1596
9 = = 3.99
114 400
= = 12.67
9 Hence, option (b) is correct.
Statistics 505

28. It is given, there are ten observations, 31. We know that,


10 10 var (aX + b) = a 2 var (X )
and å (xi - p) = 3 and å (xi - p)2 = 9, 0 ¹ p Î R. 2
a2 2
i =1 i =1 æ aX + b ö æ a ö
\ var ç ÷ = ç ÷ var (X ) = 2 s
è c ø è cø c
So, the standard deviation
10 æ 10 ö
2 æ aX + b ö a
\ SD = var ç ÷= s
å (xi - p)2 ç å (xi - p) ÷ è c ø c
i =1 çi =1 ÷
SD = Var (x) = -ç ÷
10 10 32. Given, wi = lx i + k
ç ÷
ç ÷
è ø M (xi ) = x = 48, s (x i ) = 12
2 M (w ) = 55 and s (w) = 15
9 æ3ö 9
= -ç ÷ = M (wi ) = lM (x i ) + M (k)
10 è 10 ø 10
55 = l ´ 48 + k …(i)
Hence, option (a) is correct.
and s (wi ) = ls (xi ) + s (k)
29. Given data 3, 5, 7, a, b having mean = S Þ 15 = l(12) + 0
3+5+ 7+ a + b 15
Þ =5 Þ l= = 1.25
5 12
Þ 15 + a + b = 25 \From Eq. (i), 55 = 1.25 ´ 48 + k
Þ a + b = 10 …(i) Þ k = 55 - 60
2 2
9 + 25 + 49 + a + b \ k = -5
and SD = 2 Þ – 25 = 2
5 33. We know SD of first 10 natural number is 2.87.
Þ a 2 + b2 – 42 = 20 i.e., s = 2.87
Þ a 2 + b2 = 62 …(ii) Variance, s 2 = 8.24
From Eqs. (i) and (ii), we have We know that, variance is independent of change of
origin.
(a + b)2 – (a 2 + b2) 100 – 62
ab = = = 19 Hence, after adding 1 on each observation new variance
2 2
will same i.e., 8.25
\Equation of having roots a and b is
x2 – (a + b)x + ab = 0 34. For the observations xi (1 £ i £ 10), we have
10
Þ x2 – 10x + 19 = 0
å (xi - 5) = 10
i =1
30. Let the 10 observations are xi (for i = 1, 2, 3, …, 10) and
10
mean and standard deviation is x = 20 and s 1 = 2,
respectively. Now, it is given that each of these 10
Þ å xi - 50 = 10
i =1
observations is multiplied by ‘p’ and then reduced by q , 10
where p ¹ 0 and q ¹ 0, so then new mean Þ å xi = 60 … (i)
20 i =1
= p( x ) - q
2 10

Þ 10 = 20 p - q …(i)
and å (xi - 5)2 = 40
i =1
2
and new standard deviation =| p|s 1 10 10
2 Þ å xi2 - 10 å xi + 10(52) = 40
1 i =1 i =1
Þ | p| = 10
2
1
Þ å xi2 - 10(60) + 250 = 40, [from Eq. (i)]
Þ p=± i =1
2 10

Now, at p=
1 Þ å xi2 = 390 … (ii)
2 i =1
10 10
q =0 [from Eq. (i)]
1 å (xi - 3) å xi - 30
at p=- Now, mean m =
i =1
=
i =1
2 10 10
q = - 20 [from Eq. (i)] 60 - 30 30
= = =3
Hence, option (d) is correct. 10 10
506 JEE Main Mathematics

10
Dividing Eq. (iii) by Eq. (iv), we get
å (xi - 3)2 x2(1 + t )2 169
i =1
and variance l = - (m )2 =
10 x2(1 + t 2) 97
10 10
Þ 97(t 2 + 2t + 1) = 169 (1 + t 2)
å xi2 - 6 å xi + 10(32) 2
i =1 i =1 Þ (169 - 97) t - 194 t + (169 - 97) = 0
= -9
10 Þ 36 t 2 - 97 t + 36 = 0
390 - 6(60) + 90 - 90 30 Þ (4 t - 9) (9 t - 4) = 0
= = =3
10 10 9 4
Þ t = or t =
So, the ordered pair (m , l ) = (3, 3) 4 9
Hence, option (c) is correct. 37. Given, coefficient of variation, C1 = 50
35. Given observations are -1, 0, 1 and k. and coefficient of variation, C 2 = 60
Also, standard deviation of these four observations = 5 x1 = 30 and x2 = 25
2
2 2 2 2
(-1) + (0) + (1) + k æ -1 + 0 + 1 + k ö s
\ -ç ÷ = 5 Q C=
´ 100
4 è 4 ø x
s
[Q if x1 , x2.... xn are n observation, then standard \ 50 = 1 ´ 100

30
1 n 2 æ1 n ö ú s
deviation = S xi - ç S xi ÷ Þ s 1 = 15 and 60 = 2 ´ 100
n i=1 è n i=1 ø ú 25
û
Þ s 2 = 15
2 + k2 k2
Þ - =5 [squaring both sides] \Required difference, s 1 - s 2 = 15 - 15 = 0
4 16
8 + 4k2 - k2 8 + 3k2 38. Given, S(x - 5) = 3 and S(x - 5)2 = 43, n = 18
Þ =5 Þ =5 S(x - 5) 3 1
16 16 \ Mean = 5 + =5 + =5 +
Þ 2
8 + 3k = 80 Þ 3k = 72 2 18 18 6
2 31
Þ k = 24 Þ k = 2 6 or -2 6 = = 5.17
6
Þ k =2 6 [Q k > 0]
Now, S(x - 5)2 = 43
36. Let 1, 3, 8, x and y be the five observations. 2
Þ Sx - S10x + S25 = 43
Sx Þ Sx2 - 10{ S (x - 5 + 5)} + 18 ´ 25 = 43
Then, mean x = i
n Þ Sx2 - 10S (x - 5) - 10 S5 + 450 = 43
1+3+8+ x+ y
Þ x= =5 (given) Þ Sx2 - 10(3) - 10 ´ 18 ´ 5 + 450 = 43
5
Þ Sx2 - 30 - 900 + 450 = 43
Þ x + y = 25 - 12 = 13 Þ Sx2 = 523
Þ x + y = 13 …(i) and S(x - 5) = 3
2
S(xi. - x ) Þ Sx - 5S1 = 3
and variance = s 2 =
n Þ Sx = 3 + 5 ´ 18 = 93
é (1 - 5)2 + (3 - 5)2 + (8 - 5)2 ù 2 2
ê ú S x2 æ S x ö 523 æ 93 ö
Q s= -ç ÷ = -ç ÷
êë + (x - 5)2 + ( y - 5)2úû n è n ø 18 è 18 ø
= = 9.2 (given)
5
= 29.06 - (5.17)2
Þ 16 + 4 + 9 + (x2 - 10x + 25) + ( y2 - 10 y + 25) = 46
= 29.06 - 26.73
Þ x2 + y2 - 10(x + y) = 46 - 79
= 2.33 = 1.53
Þ x2 + y2 - 10 ´ 13 = - 33 (Q x + y = 13)
39. Given that, mean = 0 and variance = s 2 (for x)
Þ x2 + y2 = 97 …(ii)
2
y Sx2 æ Sx ö
Let = t Þ y = xt Therefore, variance = s 2 = -ç ÷
x n è nø
Putting y = xt in Eq. (i), we get Sx2 Sx2
Þ s2 = - 0 Þ s2 =
x(1 + t ) = 13 n n
Þ x2 (1 + t )2 = 169 …(iii) Now, y = x2
Putting y = xt in Eq. (ii), we get Þ S y = S x2
S x2
x2(1 + t 2) = 97 … (iv) \ y= = s2
n
Statistics 507

2
40. We know that, if x1, x2, ..., xn are n observations, then S x2 æ S x ö
But we know that, variance = -ç ÷
their standard deviation is given by n è nø
2
1 æ Sx ö
2 134 + x2 æ 16 + x ö 49
Sxi2 - ç i ÷ Þ -ç ÷ = (given)
è n ø 4 è 4 ø 4
n
134 + x2 (256 + x2 + 32x) 49
(22 + 32 + a 2 + 112)
2 Þ - =
We have, (3.5) = 4 16 4
4
3x2 - 32x + 280 49
æ 2 + 3 + a + 11 ö
2
Þ =
-ç ÷ 16 4
è 4 ø 49
2
2
Þ 280 + 3x - 32x = ´ 16
49 4 + 9 + a 2 + 121 æ 16 + a ö 4
Þ = -ç ÷ 2
4 4 è 4 ø Þ 280 + 3x - 32x = 196
Þ 3x2 - 32x + 84 = 0
49 134 + a 2 256 + a 2 + 32a
Þ = - Þ (x - 6)(3x - 14) = 0
4 4 16 14
Þ x = 6, x =
49 4a 2 + 536 - 256 - a 2 - 32a 3
Þ =
4 16 14
Therefore, the values of x are 6 and .
Þ 49 ´ 4 = 3a 2 - 32a + 280 3
100 ´ 40 + 3 + 27 - 30 - 70
Þ 3a 2 - 32a + 84 = 0 44. \New mean, x =
100
x + x2 + x3 + ... + x16 4000 - 70 3930
41. Given, 1 = 16 = =
16 100 100
16
Þ å xi = 16 ´ 16 = 39.3
i=1 Q Sx = N (s 2 + x 2)
2

Sum of new observations \ = 100(100 + 1600)


18
= å yi = (16 ´ 16 - 16) + (3 + 4 + 5) = 252 = 170000
i=1 New, Sx2 = 170000 - (30)2 - (70)2 + (3)2 + (27)2
Number of observations = 18 = 170000 - 900 - 4900 + 9 + 729
18 = 164938
å yi 252 New Sx2
i=1
\ New mean = = = 14 \ New, SD = - (New x )2
18 18 N
42. (c) Key idea Standard deviation is remain unchanged, 164938
= - (39.3)2
if observations are added or subtracted by a fixed 100
number. = 1649.38 - 1544.49 = 104.89
9 9
We have, å (x1 - 5) = 9 and å (x1 - 5)2 = 45 = 10 . 24
i =1 i =1
45. Given mean x = 4
2
9 æ 9 ö variance s 2 = 5.20
å (x1 - 5)2 ç å (x1 - 5) ÷ and numbers of observation n = 5
i =1 çi =1 ÷
SD = -ç ÷
9
ç
9
÷ Let x1 = 3, x2 = 4, x3 = 4 and x4 , x5 be the five
ç ÷ observations
è ø
2 5

Þ SD =
45 æ 9 ö
-ç ÷ So, å xi = 5 × x = 5 ´ 4 = 20
9 è9ø i =1

Þ SD = 5 - 1 = 4 = 2 Þ x1 + x2 + x3 + x4 + x5 = 20
43. From given data, we make the following table Þ 3 + 4 + 4 + x4 + x5 = 20
x x2 Þ x4 + x5 = 9 …(i)
5
2
3
4
9
å xi2
2 i =1
11 121 Now, variance s = - ( x )2
5
x x2
Sx = 16 + x Sx2 = 134 + x2 x12 + x22 + x32 + x42 + x52
Þ - (4)2 = 5.20
5
508 JEE Main Mathematics

9 + 16 + 16 + x42 + x52 Round II


Þ = 16 + 5.20
5
1. Let remaining two observations are x and y,
Þ 41 + x42 + x52 = 5 ´ 21 .20 so the variance
Þ x42 + x52 = 106 - 41 22 + 42 + 102 + 122 + 142 + x2 + y2
= - 82
Þ x42 + x52 = 65 …(ii) 7
(x4 + x5 ) = 2
x42 + x52 + 2x4x5 4 + 16 + 100 + 144 + 196 + x2 + y2
Q Þ - 64 = 16 [given]
7
\ 81 = 65 + 2x4x5 [from Eqs. (i) and (ii)]
Þ 460 + x2 + y2 - 448 = 112
Þ 16 = 2x4x5
Þ x2 + y2 = 100 …(i)
Þ x4x5 = 8 …(iii) 2 + 4 + 10 + 12 + 14 + x + y
And, mean = =8 [given]
Now, (|x4 - x5|)2 = x42 + x52 - 2x4x5 7
= 65 - 16 [from Eqs. (ii) and (iii)] Þ 42 + x + y = 56 Þ x + y = 14 …(ii)
From Eqs. (i) and (ii), we get remaining observations
= 49
as 6 and 8.
Þ |x4 - x5| = 7 \ Absolute difference of remaining observations = 2
Sx2
2
46. We know, s = - m2 2. If the variate varies from m to M, then variance,
n 2 2
æ M - mö æ 10 - 0 ö
æ1 ö
2
1 æ1 ö
2 var(x) £ ç ÷ Þ var(x) £ ç ÷
10 ç – d ÷ + 10 ´ + 10 ç + d ÷ è 2 ø è 2 ø
è2 ø 4 è2 ø
= Þ var(x) £ 25 Þ standard deviation Î (- 5, 5).
30
2 \So, clearly standard deviation cannot be 6.
æ æ1 ö 1 æ1 öö
ç 10 çè - d ÷ø + 10 ´ + 10 ç + d÷ ÷
è2 ø
Hence, option (c) is correct.
2 2
-ç ÷
3. Let the remaining two observations are a and b.
ç 30 ÷
ç ÷
è ø According to the question,
Sxi ù 2 + 4 + 10 + 12 + 14 + a + b
é Mean = =8
êëQ m = n úû 7
Þ 42 + a + b = 56
æ1 ö
20 ç + d 2÷ + 5 / 2 Þ a + b = 14 …(i)
è4 ø æ1ö
= -ç ÷ 2 2
a + b + 4 + 16 + 100 + 144 + 196
30 è4ø and variance = - 82
15 7
+ 20 d 2 = 16
2 1 1 2d 2 1 2 2
= - = + - = d
30 4 4 3 4 3 a 2 + b2 + 460
Þ - 64 = 16
2 2 4 7
\ d =
3 3 a 2 + b2 + 460
Þ = 80
Þ d 2 = 2 Þ |d| = 2 7
47. Given, n1 = 60, x1 = 650, s 1 = 8, Þ a 2 + b2 + 460 = 560
Þ a 2 + b2 = 100 …(ii)
n2 = 80, x2 = 660 , s 2 = 7
We know that,
n1s 12 + n2s 22 n1n2 (x1 - x2)2 (a + b)2 = (a 2 + b2) + 2ab
\ Combined SD = +
n1 + n2 (n1 + n2)2 Þ (14)2 = 100 + 2ab [from Eqs. (i) and (ii)]
Þ 196 = 100 + 2ab
60 ´ 64 + 80 ´ 49 60 ´ 80 (650 - 660)2
= + Þ 2ab = 96 Þ ab = 48
60 + 80 (60 + 80)2
4. Given, s = 9
3840 + 3920 (4800 ´ 100)
= + Let a student obtains x marks out of 75. Then, his marks
140 (140)2 out of
7760 480000 776 4800 4x 4
= + = + 100 are . Each observation is multiply by .
140 19600 14 196 3 3
4
\ New SD, s = ´ 9 = 12
= 55 . 42 + 24 . 49 3
= 79. 91 = 8.94 Hence, variance is s 2 = 144
Statistics 509

5. The ascending order of the given data are Þ x5 + x6 + x7 + x8 + x9 + x10 = 96 … (ii)


44 + 96 140
34, 38, 42 , 44, 46, 48, 54, 55 , 63, 70 So, mean of given 10 observations = = = 14
46 + 48 10 10
Hence, Median, M = = 47 Since, the sum of squares of all the observations = 2000
2
S|x i - M | S|x i - 47| \ x12 + x22 + x32 + K + x10
2
= 2000 … (iii)
\ Median deviation = = 2
n n Sxi2 æ Sx ö
Now, s 2 = (standard deviation)2 = - ç i÷
13 + 9 + 5 + 3 + 1 + 1 + 7 + 8 + 16 + 23 10 è 10 ø
= = 8.6
10 2000
= - (14)2 = 200 - 196 = 4
6. Let di = xi - 8 10
2 So, s =2
1 æ1 ö
\ s 2x = s 2d = Sdi2 - ç Sdi ÷ 11. Let x1 , x2, x3 , x4 , x5 be the heights of five students.
18 è8 ø
2 Then, we have
1 æ9ö 5 1 9 3
= ´ 45 - ç ÷ = - = Þ s x = 5
18 è 18 ø 2 4 4 2 S xi 5
i =1
xi - A Mean, x= = 150 Þ S xi = 750 …(i)
7. If di = , then s x =|h|s d 5 i =1
h 5

xi +
3 S xi2
i =1
Now, -2 x i - 3 = 2 and variance = - ( x )2
1 n
- 5
2 S xi2
1 Þ
i =1
- (150)2 = 18
Here, h=-
2 5
5
1
\ sd = s x = 2 ´ 3.5 = 7 Þ S xi2 = 112590 …(ii)
|h| i =1
6
1 S xi
8. x = [a + (a + d ) + K + (a + 2nd )] i =1
2n + 1 Now, new mean =
1 6
= [(2n + 1)a + d (1 + 2 + ... + 2n )] 5
2n + 1 S xi + 156
i =1 750 + 156
æ 2n ö (2n + 1) = = [using Eq. (i)]
= ça + d ÷ = a + nd 6 6
è 2 ø 2n + 1
1 Þ xnew = 151
\ MD from mean = S|xi - x| 6
2n + 1 S xi2
i =1
1 and new variance = - (xnew )2
= 2|d|(1 + 2 + K + n ) 6
2n + 1 5
n (n + 1 )|d| S xi2 + (156)2
= i =1
(2n + 1 ) = - (151)2
6
n1 (s 12 + d12) + n2(s 22 + d22) 112590 + (156)2
9. We know, s 2 = = - (151)2
n1 + n2 6
[using Eq. (ii)]
where, d1 = m1 - a , d2 = m2 - a , a being the mean of the
whole group. = 22821 - 22801 = 20
n
100 ´ 15 + 150 ´ m2
\ 15.6 =
250
Þ m2 = 16 12. We have, å (xi + 1)2 = 9n ...(i)
i =1
é (100 ´ 9 + 150 ´ s 2) + ù n
ê 2 2
ê100 ´ (0.6) + 150 ´ (0.4) úû
ú and å (xi - 1)2 = 5n ...(ii)
Thus, 13.44 = ë Þs = 4 i =1
250
On subtracting Eq. (ii) from Eq. (i) is, we get
10. Given 10 observations are x1 , x2, x3 , K , x10 n

x1 + x2 + x3 + x4
Þ å {(xi + 1)2 - (xi - 1)2} = 4n
\ = 11 i =1
4 n

Þ x1 + x2 + x3 + x4 = 44 … (i) n n å xi
i =1
and
x5 + x6 + x7 + x8 + x9 + x10
= 16
Þ å 4xi = 4n Þ å xi = n Þ
n
=1
6 i =1 i =1
510 JEE Main Mathematics

\ Mean (x ) = 1 Sfixi 168


Mean x = = = 2.8
n N 60
å (xi - x )2 Sfixi2 546
Now, standard deviation =
i =1 \ Variance s 2 = - ( x )2 = - (2.8)2
n N 60
n = 9.1 - 7.84 = 1.26 = 1.12
å (xi - 1)2 15. Median of a , 2a , 3a , 4a , ¼ , 50a is
25a + 26a
= ( 25.5 ) a
i =1 5n
= = = 5 2
n n 50

13. The mean of five observation, x = 10 (given)


S xi - median
i =1
Mean deviation about median =
x1 + x2 + x3 + x4 + x5 n
Þ = 10
5 1
Þ 50 = × 0.5 + 1. 5 + 2 .5 + ¼ + 24 . 5)}
{2|a|(
Þ x1 + x2 + x3 + x4 + x5 = 50 …(i) 50
5 25 25
Þ 2500 = 2|a|× (2 ´ 0.5 + 24 ´ 1) = 2 a × (25)
å xi2 2 2
i =1
and standard deviation SD = - ( x )2 = 3 (given) Þ |a|= 4
5
5 16. Q s x2 = 4 and s y2 = 5
å xi2 5 Also, x = 2 and y = 4
i =1
Þ
5
- 100 = 9 Þ å xi2 = 5 ´ 109 Now,
Sxi
= 2 Þ Sxi = 10; Syi = 20
i =1
5
5
Þ å xi2 = 545 … (ii)
and
æ1 ö
s x2 = ç Sxi2÷ - (x )2
i =1 è5 ø
Now, variance of 6 observations x1 , x2, x3 , x4 , x5 and - 50, Þ Sxi2 = 5(4 + 4) Þ Sxi2 = 40
is equal to
æ1 ö
2 and s 2 y = ç Syi2÷ - ( y)2
5 æ 5 ö è5 ø
å xi2 + (- 50)2 ç å xi - 50 ÷
Syi2 = 5(5 + 16) Þ Syi2 = 105
i =1 çi =1 ÷ and
s2 = -ç ÷
6 6 2
ç ÷ 1 æ x + yö
ç ÷ Combined mean, s z2 = (Sxi2 + Syi2 - ç ÷
è ø 10 è 2 ø
2 1 145 - 90 55 11
545 + 2500 æ 50 - 50 ö = (40 + 105) - 9 = = =
= -ç ÷ [from Eqs. (i) and (ii)] 10 10 10 2
6 è 6 ø
n
=
3045
= 507.5 (a + l )
Sum of quantities 2
6 17. (x ) = =
n n
14. First find x as given total number of students is 60 1
= (1 + 1 + 100 d ) = 1 + 50 d
i.e. x - 2 + x + x2 + (x + 1)2 + 2x + x + 1 = 60 2
5x - 2 + x2 + x2 + 1 + 2x + 1 = 60
Þ
Þ 2x2 + 7x = 60
\ MD =
1
n
S|xi - x|
2 1
Þ 2x + 7x - 60 = 0 Þ 255 = (50 d + 49 d + 48 d + K + d
Þ 2x2 + 15x - 8x - 60 = 0 101
Þ x (2x + 15) - 4(2x + 15) = 0 + 0 + d + K + 50 d )
2 d æ 50 ´ 51 ö
Þ (2x + 15)(x - 4) = 0 = ç ÷
15 101 è 2 ø
Þ x=- ,x=4 255 ´ 101
2 Þ d= = 10.1
Þ x=4 (Q x is positive) 50 ´ 51

Marks ( xi ) Frequency (fi ) x2i fi xi fi x2i 18. According to the given condition,
é (6 - a ) 2 + (6 - b) 2 + (6 - 8) 2 ù
0 2 0 0 0
ê ú
1 4 1 4 4 ê + (6 - 5) 2 + (6 - 10) 2 + (6 - 6)2úû
2 16 4 32 64 6.80 = ë
5
3 25 9 75 225
Þ 34 = (6 - a ) 2 + (6 - b) 2 + 4 + 1 + 16
4 8 16 32 128
Þ (6 - a ) 2 + (6 - b) 2 = 13 = 9 + 4
5 5 25 25 125
Þ (6 - a ) 2 + (6 - b) 2 = 32 + 22
Total 60 168 546
Þ a = 3, b = 4
Statistics 511

2
19. Let the number of boys and girls be x and y. S X 2 æ S xö
22. s2 = -ç ÷
\ 52x + 42 y = 50(x + y) n è nø
2
Þ 52x + 42 y = 50x + 50 y (9 + k2) æ 9 + k ö
s2 = -ç ÷ < 10
Þ 2x = 8 y Þ x = 4 y 10 è 10 ø
\Total number of students in the class Þ (90 + k2) 10 - (81 + k2 + 18k) < 1000
= x + y = 4y + y = 5y Þ 90 + 10k2 - k2 - 18k - 81 < 1000
\ Required percentage of boys Þ 9k2 - 18k + 9 < 1000
æ4y ö 1000 10 10
=ç ´ 100÷% = 80% Þ (k - 1)2 < Þk -1 <
è5y ø 9 3
20. It is given that the data on ‘x’ taking the values 0, 2, 4, 8, 10 10
\ k< +1
..., 2n with frequencies n
C 0, n
C1, n
C 2, ..., n
Cn 3
respectively, Maximum integral value of k = 11
Sx f 23. Given eight numbers are 3, 7, 9, 12, 13, 20, x and y, then
so mean = i i
Sfi their mean
n
C0 (0) + nC1 (2) + nC2 (22 ) + nC3 (23 ) + K + nCn (2n ) x = 10 (given)
= n
C0 + nC1 + nC2 + K + nCn 3 + 7 + 9 + 12 + 13 + 20 + x + y
Þ = 10
C1 (2) + nC 2(22) + nC3 (23 ) + K + nC n (2n )
n 8
= Þ 64 + x + y = 80 Þ x + y = 16 …(i)
2n
and, variance = 25 (given)
Q (1 + x)n = nC 0 + nC1x + nC 2x2 + K + nC nxn 8
At x = 2, we get å (xi )2
i =1
3n = 1 + nC1 (2) + nC 2(22) + K + nC n (2n ) Þ - (x )2 = 25
8
Þ C1 (2) + C2 (2 ) + C3 (2 ) + K + Cn (2 ) = 3 - 1
n n 2 n 3 n n n
9 + 49 + 81 + 144 + 169 + 400 + x2 + y2
3n - 1 728 Þ - 100 = 25
\Mean = = n [given] 8
2n 2
Þ 852 + x2 + y2 = 1000 Þ x2 + y2 = 148
Þ 3n = 729 = 36 Þ n = 6
Þ (x + y)2 - 2xy = 148 [from Eq. (i)]
21. Let the AP is a, a + d, a + 2d, ..., a + 10 d. Þ 256 - 2xy = 148
According to the question, Þ 2xy = 256 - 148 = 108 Þ xy = 54
a 2 + (a + d )2 + (a + 2d )2 + ....+ (a + 10d )2 18
Var =
11
24. Given, å (Xi - a) = 36
i =1
2
é a + (a + d ) + ....+ (a + 10d ) ù 18 18

ë 11 úû Þ å xi - 18a = 36 Þ å xi = 18 (a + 2) …(i)
i =1 i =1
11a 2 + d 2 (12 + 22 + 32 + .... + 102) 18

+ 2ad (1 + 2 + K + 10) Also, å (Xi - b)2 = 90


= i =1
11
2 Þ S xi2 + 18b 2 - 2b S xi = 90
æ 11a + d (1 + 2 + 3 + .... + 10) ö
-ç ÷ Þ S xi + 18b 2 + 2b ´ 18 (a + 2) = 90
2
[using Eq. (i)]
è 11 ø
Þ S xi2 2
= 90 - 18 b + 36 b (a + 2)
10 ´ 11 ´ 21 2 10 ´ 11 2
11a 2 + d + 2ad 1 æ Sx ö
= 6 2 \ s2 = 1 Þ S xi2 - ç i ÷ = 1 [Q s = 1, given]
18 è 18 ø
11
2
æ 10 ´ 11 ö
2 1 æ 18 (a + 2) ö
Þ (90 - 18b 2 + 36ab + 72b ) - ç ÷ =1
ç 11a + d÷
18 è 18 ø
-ç 2 ÷
ç 11 ÷ Þ 90 - 18 b 2 + 36ab + 72b - 18 (a + 2)2 = 18
è ø
Þ 5 - b 2 + 2ab + 4b - (a + 2)2 = 1
= a 2 + 35d 2 + 10ad - a 2 - 25d 2 - 10ad = 10d 2 Þ 5 - b 2 + 2ab + 4b - a 2 - 4 - 4a = 1
Q Var = 90 [given] Þ - a 2 - b 2 + 2ab + 4b - 4a = 0
\ 10d 2 = 90 Þ d = ± 3 Þ (a - b ) (a - b + 4) = 0
Q AP is an increasing AP, so d = 3. Þ a -b = -4
Hence, answer is 3.00. \ |a - b| = 4 [Q a ¹ b ]
21
Fundamentals of
Probability
Basic Definition IN THIS CHAPTER ....
Basic Definition
Experiment
Event
An operation which results in some well defined outcomes is called an
experiment. Important Events
Probability
Random Experiment Additional Theorem of
If an experiment is performed many times under similar conditions and the Probability
outcome of each time is not the same, then this experiment is called a random
experiment.
e.g. Throwing of an unbiased die.

Sample Space
The set of all possible outcomes of a random experiment is called the sample
space for that experiment. It is usually denoted by S.
e.g. When two coins are tossed, then the sample space is
S = {( H 1 , H 2 ), ( H 1 , T2 ), (T1 , H 2 ), (T1 , T2 )}

Sample Point/Event Point


Each element of the sample space is called a sample point or an event point.
e.g. When a dice is thrown, the sample space is S = { 1, 2, 3, 4, 5, 6}, where 1, 2,
3, 4, 5 and 6 are the sample points.

Event
An event is associated with a subset of sample space. Events can be classified
into various types on the basis of the elements they have.
Fundamentals of Probability 513

Types of Events Important Events


(i) Impossible and sure events The emtpy set f is
called an impossible event. e.g. Getting two heads in Equally Likely Events
tossing a unbiased coin one time. Outcomes are said to be equally likely when we have no
The event which is certain to occur is said be the reason to believe that one is more likely to occur than the
sure event. e.g. In tossing a die any number will other e.g. When an unbiased die is thrown all the six
come is a sure event. faces, 1, 2, 3, 4, 5, 6 are equally likely to come up.
(ii) Simple event If an event E has only one sample
point of a sample space, it is called a simple (or Exhaustive Events
elementary) event. e.g. In the experiment of tossing A set of events is said to be exhaustive, if one of them must
a coin twicely getting both head i.e. E = { H , H }. necessarily happen every time the experiment is performed.
(iii) Compound event If an event has more than one e.g. When a die is thrown, events 1, 2, 3, 4, 5, 6 form an
sample point, it is called a compound event. exhaustive set of events.
e.g. In the experiment of tossing a coin thrice the
events. Note We can say that the total number of elementary events of a
E : Exactly one head appeared random experiment is called the exhaustive number of cases.
F : Atleast one head appeared
G : Atmost one head appeared, etc., are all Mutually Exclusive Events
compound events. Two or more events are said to be mutually exclusive if
As one of them occurs, others cannot occur. Thus, if two or
E = { HTT , THT , TTH } more events are said to be mutually exclusive, if no two
F = { HTT , THT , TTH , HHT , THH , HTH , HHH } of them can occur together.
G = { HTT , THT , TTH , TTT } Hence, A1 , A2 , A3 , . . . , An are mutually exclusive if and
Each of the above subsets contain more than one only if Ai Ç Aj = f , "i ¹ j.
sample point. Hence, they are all compound events.
Example 1. Three coins are tossed once. Let A denote the
event ‘three heads show,’ B denote the event ‘two heads and
Algebra of Events one tail show’, C denote the event ‘three tails show’ and D
Let A, B and C be events associated with an experiment denote the event ‘a head shows on the first toss’, which
whose sample space is S.
events are mutually exclusive?
Complementary Event (a) A Ç B, A Ç D (b) A Ç C , B Ç D
For every event A, there corresponds another event A¢ or (c) B Ç C , C Ç D (d) None of these
Ac or A called the complementary event to A. It is also
Sol. (c) When three coins are tossed, then there are 23 = 8
called the event not A.
possible outcomes.
e.g. Let S = { HHH , HHT , HTH , THH , HTT , THT ,
TTH , TTT } be a sample space i.e. S = {HHH, HHT, HTH, HTT, THH, THT, TTH, TTT }
and let A = { HTH , HHT , THH } be a subset of S, A = Three heads shows = {HHH }
then complementary of A is defined as B = Two heads and one tail show = {HHT , HTH , THH }
A = { HHH , HTT , THT , TTH , TTT } C = Three tails show = {TTT }
D = A head shows on the first toss
Event A or B
= {HHH , HHT , HTH , HTT }
It is the set of all those elements, which are either in A or
B or in both. And it is denoted by the symbol A È B. Here, A Ç B = f, A ÇC = f
e.g. Let A = { 1, 2, 3} and B = { 3, 4} , then A È B = { 1, 2, 3, 4} B Ç C = f, C ÇD = f
A ÇB ÇC = f
Events A and B
It is the set of all those elements, which are common in Here, A and B, A and C, B and C, C and D; A, B and C are
A and B. And it is denoted by the symbol A Ç B. mutually exclusive.
e.g. Let A = { 2, 3, 5} and B = { 1, 2, 3, 4, 5},
then A Ç B = { 2, 3, 5} Example 2. Two dice are thrown. The events A, B and C
are as follows
Event A but not B
A : Getting an even number on the first die.
It is the set of all those elements, which are in A but not
in B. And it is denoted by the symbol A - B or A Ç B¢. B : Getting an odd number on the first die.
C : Getting the sum of the numbers on the dice £ 5.
e.g. Let A = { 1, 2, 3, 4} and B = { 2, 4, 6} , then A - B = { 1, 3}.
514 JEE Main Mathematics

Which of the following statement is false? Odds in favour and odds against an event
(a) A and B are mutually exclusive
If a is the number of cases favourable to the event E , b is
(b) A and B are mutually exclusive and exhaustive
the number of cases favourable to the event E¢ . (i.e.
(c) A = B’
number of cases against to E). Then, odds in favour of E
(d) Aand C are mutually exclusive
are a : b and odds against of E are b : a .
Sol. (d) If two dice are thrown, then total number of possible a b
Then, P ( E ) = and P ( E ) = .
outcomes, S = 6 ´ 6 = 36 which are as follows a+b a+b
® 1 2 3 4 5 6 Thus, odds in favour of an event
¯
a a/( a + b) P ( E )
1 1, 1 1, 2 1, 3 1, 4 1, 5 1, 6 E= = =
2 2, 1 2, 2 2, 3 2, 4 2, 5 2, 6 b b/( a + b) P ( E )
3 3, 1 3, 2 3, 3 3, 4 3, 5 3, 6 and odds against an event
4 4, 1 4, 2 4, 3 4, 4 4, 5 4, 6 b P( E )
E= =
5 5, 1 5, 2 5, 3 5, 4 5, 5 5, 6 a P( E )
6 6, 1 6, 2 6, 3 6, 4 6, 5 6, 6
Example 3. Out of 11 consecutive natural numbers if three
A = getting an even number on the first die
numbers are selected at random (without repetition), then the
= {(2, 1), (2, 2), (2, 3), (2, 4), (2, 5), (2, 6), (4, 1), (4, 2), (4, 3), probability that they are in AP with positive common
(4, 4), (4, 5), (4, 6), (6, 1), (6, 2), (6, 3), (6, 4), (6, 5), (6, 6)} difference, is (JEE Main 2020)
B = getting an odd number on the first die
15 5 5 10
= {(1, 1), (1, 2), (1, 3), (1, 4), (1, 5), (1, 6), (3, 1), (3, 2), (3, 3), (a) (b) (c) (d)
101 101 33 99
(3, 4), (3, 5), (3, 6), (5, 1), (5, 2), (5, 3), (5, 4), (5, 5), (5, 6)}
C = {(1, 1), (1, 2), (1, 3), (1, 4), (2, 1), (2, 2), (2, 3), (3, 1), Sol. (c) We have to select three numbers at random out of
(3, 2), (4, 1)} 11 consecutive natural numbers such that they are in AP,
Here, B¢ = Event getting an odd number on the first die. then two numbers must be either even or odd and third will
(a) True be selected automatically, so we can do this in 6C 2 + 5C 2
Q A = getting an even number on the first die ways and total number of ways to select 3 numbers out of 11
B = getting an odd number on the first die consecutive natural numbers is 11C3 .
Þ A ÇB = f 6
C 2 + 5C 2
So, required probability =
\A and B are mutually exclusive events. 11
C3
(b) True 15 + 10
\ A È B = S i.e., exhaustive. Also, A Ç B = f =
165
(c) True 25 5
= =
\ B = getting an odd number on the first die 165 33
Þ B ¢ = getting an even number on first die = A
\ A = B¢ Example 4. If 10 different balls are to be placed in 4
(d) False distinct boxes at random, then the probability that two of
Q A Ç C = {(2, 1), (2, 2) ,(2, 3), ( 4, 1)} ¹ f, so A and C are these boxes contain exactly 2 and 3 balls is (JEE Main 2020)

not mutually exclusive. 945 945 965 965


(a) (b) (c) (d)
211 210 210 211
Probability Sol. (b) There are 10 different balls, then number of ways to
select 5 balls is 10C5 .
The probability of an event E to occur is the ratio of
the number of cases in its favour to the total number of Now, the number of ways to distribute these 5 balls in two
cases. boxes from 4 different boxes such that these two boxes gets
Number of cases favourable to event E exactly 2 and 3 balls is
\ P( E ) = 4 5!
Total number of cases C2 ´ ´2!
n( E ) 3 !2 !
= Now, the remaining 5 balls we can distribute in remaining
n(S )
2 boxes in 25 ways.
Probability of non-occurrence of event E is So, the number of ways to place 10 different balls in
P( E ) = 1 - P( E ) 4 different boxes such that two of these boxes contain exactly
and 0 £ P ( E ) £ 1. If P ( E ) = 1, then event E is known as 2 and 3 balls is
certain event and if P ( E ) = 0, then E is known as 10 5!2! 5
C5 ´ 4C 2 ´ ´2
impossible event. 3!2!
Fundamentals of Probability 515

and the total number of ways to place 10 different balls in 4 if b = 6, ac = 9, a = 3, c = 3


different boxes is 410 = 2 20 \ Total number of favourable case = 5
10 ! 4 ! 5 !2 ! 5
´ ´ ´2 5 \ Required probability =
5 ! 5 ! 2 !2 ! 3 !2 ! 216
So, required probability =
2 20
=
10 ´ 9 ´ 8 ´ 7 ´ 6 5 ´ 9 ´ 7 ´ 3
= Addition Theorem of Probability
215 210 (i) If A and B be any two events in a sample space
945 S,then the probability of occurrence of atleast one
= 10
2 of the events A and B is given by
P ( A È B) = P ( A) + P ( B) - P ( A Ç B).
Example 5. If the integers m and n are chosen at random
from 1 to 100, then the probability that a number of the form Note
7n + 7m is divisible by 5 equals • If A and B are mutually exclusive events, then A Ç B = f
1 1 1 1 and hence P( A Ç B) = 0\P( A È B) = P( A) + P(B)
(a) (b) (c) (d)
4 2 8 3 • Two events A and B are mutually exclusive iff P( A È B )

Sol. (b) Let I = 7 n + 7 m, then we observe that 71, 7 2, 73 and 7 4 = P( A) + P(B) .


• 1 = P(S ) = P( A È A¢ ) = P( A) + P( A¢ )
ends in 7, 9, 3 and 1, respectively. Thus, 7i ends in 7, 9, 3 or
1 according as i is of the form 4k + 1, 4k + 2, 4k - 1 or 4k, (Q A Ç A¢ = f ) or P( A¢ ) = 1 - P( A).
respectively. (ii) If A, B and C are any three events in a sample space
If S is the sample space, then n( S) = (100) 2. S, then
7 m + 7 n is divisible by 5, if P ( A È B È C ) = P ( A) + P ( B) + P (C ) - P ( A Ç B)
(i) m is of the form 4k + 1 and n is of the form 4k - 1 or - P( B Ç C ) - P( A Ç C ) + P( A Ç B Ç C )
(ii) m is of the form 4k + 2 and n is of the form 4k or Note
(iii) m is of the form 4k - 1 and n is of the form 4k + 1 or • If A, B, C are mutually exclusive events, then
(iv) m is of the form 4k and n is of the form 4k + 2. A Ç B = f, B Ç C = f, A Ç C = f, A Ç B Ç C = f
Thus, number of favourable ordered pairs (m, n) = 4 ´ 25 ´ 25 \ P( A È B È C ) = P( A) + P(B) + P(C )
4 ´ 25 ´ 25 1
\Required probability = = • If A and B are any two events, then
(100) 2 4 ( A - B) Ç ( A Ç B) = f and A = ( A - B) Ç ( A Ç B)
\ P( A) = P( A - B) + P( A Ç B) = P( A Ç B¢ ) + P( A Ç B)
Example 6. Let the quadratic equation ax 2 + bx + c = 0,
or P( A) - P( A Ç B) = P( A - B) = P( A Ç B¢ )
where a, b, c are obtained by rolling the dice thrice. What is
the probability that equation has equal roots? [Q A - B = A Ç B¢ ]
5 1 1 3 Similarly, P(B) - P( A Ç B) = P(B - A = P(B Ç A¢ )
(a) (b) (c) (d)
216 72 36 216 (iii) General form of addition theorem of
Sol. (a) Here, a, b, c are obtained by rolling a dice.
probability If A1 , A2 , . . . , An are n sample space,
then
\ a, b, c Î{1, 2, 3, 4, 5, 6}
P ( A1 È A2 È . . . È An )
n ( S) = 6 ´ 6 ´ 6 = 216 n
ax2 + bx + c = 0 has equal roots, then = å P ( Ai ) - å P ( Ai Ç Aj ) + å P ( Ai Ç Aj Ç Ak )
b2 i =1 i<i i< j<k
b 2 = 4ac Þ ac =
4 - . . . + ( -1)n - 1 P ( A1 Ç A2 Ç . . . Ç An )
if b = 2, ac = 1, a = 1, c = 1
if b = 4, ac = 4, a = 1, c = 4, Note For any number of (finite or infinite) mutually exclusive
a = 2, b = 2, events,
c = 4, b = 1 P( A1 È A2 È . . . È An ) = P( A1 ) + P( A2 ) + . . . + P( An )
516 JEE Main Mathematics

Relation between Set Theory and Set Probability


Set Theory Set Probability
1. When two events A and B (i) n (atleast one of the events occurs) (i) P (atleast one of the events occurs)
occurs n( A È B) = n( A ) + n( B) - n( A Ç B) P( A È B) = P( A ) + P( B) - P( A Ç B)
(ii) n (exactly one of the event occur), (ii) p (exactly one of the events occurs)
n( A Ç B ) n( A È B) = n( A ) + n( B) - 2 n( A Ç B) P( A Ç B ) + P( A Ç B) = P( A ) + P( B) - 2 P( A Ç B)

2. When three events (i) n (atleast one of the vents occur), (i) P (atleast one of the events occurs)
A, B and C occurs n( A È B È C ) = n( A ) + n( B) + n(C ) - n( A Ç B) P( A È B È C ) = P( A ) + P( B) + P(C ) - P( A Ç B)
- n( B Ç C ) - n(C Ç A ) + n( A Ç B Ç C ) - P( B Ç C ) - P(C Ç A ) + P( A Ç B Ç C )
(ii) n (atleast two of the events occur) (ii) P (atleast two of the events occur)
= n( A Ç B) + n( B Ç C ) + n(C Ç A ) - 2 n( A Ç B Ç C ) = P( A Ç B) + P( B Ç C ) + P(C Ç A ) - 2 P( A Ç B Ç C )
(iii) n (exactly two of the events occur) (iii) P (exactly two of the events occur)
= n( A Ç B) + n( B Ç C ) + n(C Ç A ) - 3n( A Ç B Ç C ) = P( A Ç B) + P( B Ç C ) + P(C Ç A ) - 3P( A Ç B Ç C )
(iv) n (exactly one of the events occur) (iv) P (exactly one of the events occur)
= n( A ) + n( B) + n(C ) - 2 n( B Ç C ) = P( A ) + P( B) + P(C ) - 2 P( B Ç C )
- 2 n(C Ç A ) - 2 n( A Ç B) + 3n( A Ç B Ç C ) - 2 P(C Ç A ) - 2 P( A Ç B) + 3P( A Ç B Ç C )

Important Results Example 8. Let A and B be two events such that the
Let A, B and C are three events. 2
probability that exactly one of them occurs is and the
(i) The probability of occurrence of atleast one of the 5
1
events A, B and C is given by probability that A or B occurs is , then the probability of both
2
P ( A È B È C ) = P ( A) + P ( B) + P (C ) - P ( A Ç B) of them occur together is (JEE Main 2020)
- P ( B Ç C ) - P (C Ç A) + P ( A Ç B Ç C ) (a) 0.10 (b) 0.20 (c) 0.01 (d) 0.02
(ii) P(atleast two of A, B and C occur)
Sol. (a) For two events A and B it is given that probability of
= P ( A Ç B) + P ( B Ç C ) + P (C Ç A) - 2P ( A Ç B Ç C ) 2
occurrence of exactly one of them is .
(iii) P(exactly two of A, B and C occur) 5
= P ( A Ç B) + P ( B Ç C ) + P (C Ç A) - 3P ( A Ç B Ç C ) 2
So, P( A) + P(B) - 2P( A Ç B) = …(i)
5
(iv) P(exactly one of A, B and C occur) 1
and probability that A or B occurs is ,
= P ( A) + P ( B) + P (C ) - 2P ( A Ç B) - 2P ( B Ç C ) 2
1 1
- 2P (C Ç A) + 3P ( A Ç B Ç C ) so P( A È B) = Þ P( A) + P(B) - P( A Ç B) = …(ii)
2 2
(v) Booley’s Inequality If events E1 , E2 , . . . , En are From Eqs. (i) and (ii), we get
associated with a random experiment, then 1 2 5-4 1
n n P( A Ç B) = - = =
2 5 10 10
(a) P ( Ç Ei ) ³
i =1
å P ( Ei ) - ( n - 1)
Probability of both of them occur together =
1
= 0.10
i =1
10
n n
(b) P ( È Ei ) £
i =1
å P ( Ei ) Example 9. The probabilities of three events A, B and C
i =1
are given by P( A) = 0.6, P(B) = 0.4 and P(C ) = 0.5. If
P( A È B) = 0.8, P( A Ç C ) = 0.3, P( A Ç B Ç C ) = 0.2,P(B Ç C ) = b
Example 7. In a class of 60 students, 40 opted for NCC, 30
and P( A È B È C ) = a , where 0.85 £ a £ 0.95, then b lies in the
opted for NSS and 20 opted for both NCC and NSS. If one of
interval (JEE Main 2020)
these students is selected at random, then the probability that
the student selected has opted neither for NCC nor for NSS is (a) [ 0.35, 0.36] (b) [0.25, 0.35]
(JEE Main 2019) (c) [0.20, 0.25] (d) [0.36, 0.40]
1 1 2 5
(a) (b) (c) (d) Sol. (b) As, we know that P ( A Ç B) = P ( A) + P (B) - P ( A È B)
6 3 3 6
Sol. (a) Let C and S represent the set of students who opted for Þ P ( A Ç B) = 0.6 + 0.4 - 0.8 Þ P ( A Ç B) = 0.2
NCC and NSS respectively. and, as
Then, n(C) = 40 , n( S) = 30 , n(C Ç S) = 20 and n( U) = 60 P ( A È B È C) = P ( A) + P (B) + P (C)
- P( A Ç B) - P(B Ç C) - P(C Ç A) + P( A Ç B Ç C)
Now, n(C È S ) = n(C È S) = n( È) - n(C È S)
Þ a = 0.6 + 0.4 + 0.5 - 0.2 - b - 0.3 + 0.2
= 60 - [n(C) + n( S) - n(C Ç S)]
Þ a = 1.2 - b
= 60 - [ 40 + 30 - 20 ] = 10
Q 0.85 £ a £ 0.95 Þ 0.85 £ 1.2 - b £ 0.95
10 1
So, required probability = = Þ 0.25 £ b £ 0.35
60 6
Fundamentals of Probability 517

Practice Exercise
ROUND I Topically Divided Problems
Probability of an Event 8. When a missile is fired from a ship, the probability
1
1. The probability that two randomly selected subsets that it is intercepted is and the probability that
3
of the set {1, 2, 3, 4, 5 } have exactly two elements in
their intersection, is (JEE Main 2021)
the missile hits the target, given that it is not
3
65 135 65 35 intercepted, is . If three missiles are fired
(a) (b) (c) (d) 4
27 29 28 27
independently from the ship, then the probability
2. In a college, 25% of the boys and 10% of the girls
that all three hit the target, is (JEE Main 2021)
offer Mathematics. The girls constitute 60% of the
1 3 1 3
total number of students. If a students is selected at (a) (b) (c) (d)
8 4 27 8
random and is found to be studying Mathematics.
The probability that the student is a girl is 9. If any four numbers are selected and they are
1 3 5 5 multiplied, then the probability that the last digit
(a) (b) (c) (d)
6 8 8 6 will 1,3,5 or 7, is
3. If birth to a male child and birth to a female child 4 18 16
(a) (b) (c) (d) None of these
are equal probable, then what is the probability 625 625 625
that atleast one of the three children born to a 10. A die is rolled three times. The probability of
couple is male ? getting a larger number than the previous number
4 7 8 1
(a) ](b) (c) (d) each time is
5 8 7 2 15 5 13 1
(a) (b) (c) (d)
4. A four-digit number is formed by the digits 1, 2, 3, 216 54 216 18
4 with no repetition. The probability that the
11. A fair coin is tossed repeatedly. If the tail appears
number is odd, is
1 1
on first four tosses, then the probability of the head
(a) zero (b) (c) (d) None of these appearing on the fifth toss is equal to
3 4
1 1 31 1
5. Five persons A, B, C, D and E are in queue of a (a) (b) (c) (d)
2 32 32 5
shop. The probability that A and E are always
together, is 12. A five digit number is formed by writing the digits
1 2 2 3 1, 2, 3, 4, 5, in a random order without repetitions.
(a) (b) (c) (d)
4 3 5 5 Then, the probability that the number is divisible
by 4, is
6. If two dice are thrown simultaneously, then the
3 18 1 6
probability that the sum of the numbers which (a) (b) (c) (d)
5 5 5 5
come up on the dice to be more than 5 is
5 1 5 13 13. Four candidates A, B, C and D have applied for the
(a) (b) (c) (d)
36 6 18 18 assignment to coach a school cricket team. If A is
7. Two numbers b and c are chosen at random (with twice as likely to be selected as B and B and C are
replacement from the numbers 1, 2, 3, 4, 5, 6, 7, 8 given about the same chance of being selected,
and 9). The probability that x 2 + bx + c > 0 for all while C is twice as likely to be selected as D, what
x Î R is are the probabilities that C will be selected?
17 32 82 45 1 2 5 4
(a) (b) (c) (d) (a) (b) (c) (d)
123 81 125 143 9 9 9 9
518 JEE Main Mathematics

14. In x = 33n , n is a positive integral value, then what 22. A seven-digit number is formed using digit 3, 3, 4,
is the probability that x will have 3 at its units 4, 4, 5, 5. The probability, that number so formed is
place? divisible by 2, is (JEE Main 2021)
1 1 1 1 6 4 3 1
(a) (b) (c) (d) (a) (b) (c) (d)
3 4 5 2 7 7 7 7

15. Two numbers are selected randomly from the set 23. A letter is taken out at random from ‘ASSISTANT’
S = {1, 2 , 3, 4, 5, 6 } without replacement one-by-one. and another is taken out from ‘STATISTICS’. The
The probability that minimum of the two number is probability that they are the same letters is
1 13
less than 4, is (a) (b)
45 90
1 14 1 4 19
(a) (b) (c) (d) (c) (d) None of these
15 15 5 5 90
16. A five digit number is chosen at random. 24. A bag contains 5 white and 3 black balls and 4 balls
The probability that all the digit are distinct and are successively drawn out and not replaced. The
digits at odd places are odd and digits at even place probability that they are alternately of different
are even, is colours, is
1 2 1 1 1 2 13 1
(a) (b) (c) (d) (a) (b) (c) (d)
60 75 50 75 196 7 56 7

17. A three digit number, which is a multiple of 11, is 25. If a committee of 3 is to be chosen from a group of
chosen at random. Probability that the number so 38 people of which you are a member. What is the
chosen is also a multiple of 9, is equal to probability that you will be on the committee?
1 2 1 9 (a) ( 38C3 ) (b) ( 37C 2)
(a) (b) (c) (d) (c) ( 37C 2)/(38C3 ) (d) 666/8436
9 9 100 100
26. A bag contains 5 brown and 4 white socks. A man
18. Three of the six vertices of a regular hexagon are
pulls out two socks. The probability that these are
chosen at random. The probability that the triangle
of the same colour, is
with these three vertices is equilateral is equal to 5 18 30 48
1 1 1 1 (a) (b) (c) (d)
(a) (b) (c) (d) 108 108 108 108
2 5 10 20
27. Out of 13 applicants for a job, there are 8 men and
19. If the integers m and n are chosen at random 5 women. It is desired to select 2 persons for the
between 1 and 100, then the probability that a job. The probability that atleast one of the selected
number of the form 7 m + 7 n is divisible by 5, is persons will be a woman, is
equal to 5 10 14 25
(a) (b) (c) (d)
1 1 1 1 13 13 39 39
(a) (b) (c) (d)
4 7 8 49 28. An integer is chosen at random from the numbers
20. If n integers taken at random are multiplied 1, 2,…,25. The probability that the chosen number
together, then the probability that the last digit of is divisible by 3 or 4, is
the product is 1,3, 7 or 9 is 2 11 12 14
(a) (b) (c) (d)
25 25 25 25
(a) 2n/ 5n (b) 4n - 2n/ 5n
(c) 4n/ 5n (d) None of these 29. If 12 identical balls are to be placed in 3 different
boxes, then the probability that one of the boxes
21. There are 9999 tickets bearing numbers
contains exactly 3 balls, is (JEE Main 2015)
0001, 0002, ... , 9999. If one ticket is selected from 11 10
55 æ 2 ö æ2ö
these tickets at random, the probability that the (a) ç ÷ (b) 55ç ÷
3 è3ø è3ø
number on the ticket will consists of all different 12 11
æ1ö æ1ö
digits, is (c) 220ç ÷ (d) 22ç ÷
è3ø è3ø
5040 5000
(a) (b)
9999 9999 30. Among 15 players, 8 are batsmen and 7 are
5030 bowlers. The probability that a team is chosen of 6
(c) (d) None of these
9999 batsmen and 5 bowlers, is
Fundamentals of Probability 519

8
C 6 ´ 7C5 8
C 6 + 7C5 38. In shuffling a pack of playing cards, four are
(a) 15
(b) 15
C11 C11 accidently dropped. The probability that missing
15 cards should be one from each suit, is
(c) (d) None of these
28 1 1
(a) (b)
31. Two cards are drawn without replacement from a 256 270725
2197
well-shuffled pack. The probability that one of (c) (d) None of these
20825
them is an ace of heart, is
1 1 39. If A and B are any two events, then P ( A Ç B) is
(a) (b)
25 26 equal to
1 (a) P ( A ) P (B )
(c) (d) None of these
52 (b) 1 - P ( A ) - P (B)
32. A box contains 3 white and 2 red balls. If we draw (c) P ( A ) + P (B) - P ( A Ç B)
(d) P (B) - P ( A Ç B)
one ball and without replacing the first ball, the
probability of drawing red ball in the second draw is 40. In class XI of a school, 40% of the students study
8 2 3 21 Mathematics and 30% study Biology. 10% of the
(a) (b) (c) (d)
25 5 5 25 class study both Mathematics and Biology. If a
33. In a lottery three were 90 tickets numbered 1 to 90. student is selected at random from the class, find
Five tickets were drawn at random. The probability the probability that he will be studying
Mathematics or Biology.
that two of the tickets drawn numbers 15 and 89, is
2 2 1 1 (a) 0.5 (b) 0.6
(a) (b) (c) (d) (c) 0.65 (d) None of these
801 623 267 623
34. A basket contains 5 apples and 7 oranges and 41. The probability that at least one of the events A
3
another basket contains 4 apples and 8 oranges. and B occurs is . If A and B occur simultaneously
5
One fruit is picked out from each basket. The 1
with probability , then P ( A) + P ( B) is
probability that the fruits are both apples or both 5
oranges, is 2 4 6 7
(a) (b) (c) (d)
24 56 68 76 5 5 5 5
(a) (b) (c) (d)
144 144 144 144
42. The probabilities that a student passes in
35. Out of 30 consecutive integers, 2 are chosen at Mathematics, Physics and Chemistry are m, p and
random. The probability that their sum is odd, is c, respectively. Of these subjects the student has
14 16 15 10 75% chance of passing in at least one, a 50% chance
(a) (b) (c) (d)
29 29 29 29 of passing in at least two and a 40% chance of
36. There are 10 prizes, five A¢s, three B¢s and two C¢s, passing in exactly two. Which of the following
placed in identical sealed envelopes for the top relation is true?
19 27
10 contestants in a Mathematics contest. The (a) p + m + c = (b) p + m + c =
20 20
prizes are awarded by allowing winners to select an 1 1
envelope at random from those remaining. When (c) pmc = (d) pmc =
5 4
the 8th contestant goes to select the prize, the
43. If A and B are arbitrary events, then
probability that the remaining three prizes are one
(a) P ( A Ç B) ³ P ( A ) + P (B)
A, one B and one C is
1 1 1 1 (b) P ( A È B) £ P ( A ) + P (B)
(a) (b) (c) (d) (c) P ( A Ç B) = P ( A ) + P (B)
4 3 12 10
(d) None of the above
Addition Theorem of Probability 44. An experiment yields 3 mutually exclusive and
37. Given two events A and B. If odds against A are as exhaustive events A, B and C. If
2 : 1 and those in favour of A È B are as 3 : 1, then P ( A) = 2 P ( B) = 3P (C), then P ( A) is equal to
(a) 1 /2 £ P (B) £ 3 /4 (b) 5 /12 £ P (B) £ 3 /4 1 2 3 6
(a) (b) (c) (d)
(c) 1 /4 £ P (B) £ 3 /5 (d) None of these 11 11 11 11
520 JEE Main Mathematics

ROUND II Mixed Bag


Only One Correct Option 7. There are 7 seats in a row. Three persons take
1. Let A denote the event that a 6-digit integer seats in a row. Three persons take seats at random.
formed by 0, 1, 2, 3, 4, 5, 6 without repetitions, The probability that the middle seat is always
be divisible by 3, then the probability of event A occupied and no two persons are consecutive, is
is equal to (JEE Main 2021) 9 9
(a) (b)
9 4 3 11 70 35
(a) (b) (c) (d)
56 9 7 27 4
(c) (d) None of these
35
2. A natural number x is chosen at random from the
first 100 natural numbers. The probability that 8. Number 1, 2, 3, …, 100 are written down on each of
100 the cards A, B and C. One number is selected at
x+ > 50 is
x random from each of the cards. The probability that
1 11 the numbers so selected can be the measures (in
(a) (b)
10 50 cm) of three sides of right-angled triangles no two
11
(c) (d) None of these of which are similar, is
20
4 3
(a) (b)
3. If four digit numbers greater than 5000 are 1003 503
randomly formed from the digits 0, 1, 3, 5 and 7, 36
what is the probability of forming number divisible (c) (d) None of these
1003
by 5 when
9. Let A be a set of all 4-digit natural numbers whose
(i) the digits are repeated ?
exactly one digit is 7. Then, the probability that a
(ii) the repetition of digits is not allowed ?
randomly chosen element of A leaves remainder 2
30 3 34 1
(a) , (b) , when divided by 5 is (JEE Main 2021)
83 8 83 8
33 3 1 2 97 122
(c) , (d) None of these (a) (b) (c) (d)
83 8 5 9 297 297

4. The coefficients of a quadratic equation 10. A class consists of 80 students, 25 of them are girls
2
ax + bx + c = 0 ( a ¹ b ¹ c) are chosen from first and 55 are boys. If 10 of them are rich and the
three prime numbers, the probability that roots of remaining are poor and also 20 of them are
the equation are real is intelligent, then the probability of selecting an
1 2 intelligent rich girl is
(a) (b)
3 3 5 25
(a) (b)
1 3 128 128
(c) (d)
4 4 5
(c) (d) None of these
512
5. A bag contains 7 red and 2 white balls and another
bag contains 5 red and 4 white balls. Two balls are 11. Three critics review a book. Odds in favour of the
drawn, one from each bag. The probability that book are 5 : 2, 4 : 3 and 3 : 4, respectively, for the
both the balls are white, is three critics. The probability that majority are in
2 2 favour of the book is
(a) (b)
9 3 35 125 164 209
(a) (b) (c) (d)
8 35 49 343 343 343
(c) (d)
81 81
12. An urn contains nine balls of which three are red,
6. In a class, there are 10 boys and 8 girls. When 3 four are blue and two are green. Three balls are
students are selected at random, the probability drawn at random without replacement from the
that 2 girls and 1 boy are selected, is urn. The probability that the three balls have
35 15
(a) (b) different colours, is
102 102 1 2 1 2
55 25 (a) (b) (c) (d)
(c) (d) 3 7 21 23
102 102
Fundamentals of Probability 521

13. A die is thrown. Let A be the event that the number which 4 marbles can be drawn so that at the most
obtained is greater than 3 and B be the event that three of them are red is ……… . (JEE Main 2020)
the number obtained is less than 5. Then,
16. Out of 21 tickets consecutively numbered, there are
P ( A È B) is
drawn at random. Find the probability that the
2 3
(a) (b) (c) 0 (d) 1 numbers on them are in AP is a / b, then (14a - b)
5 5
is ……… .
14. Two aeroplanes I and II bomb a target in succession.
The probabilities of I and II scoring a hit correctly 17. In throwing of a die, let A be the event ‘an odd
are 0.3 and 0.2, respectively. The second plane will number turns up’, B be the event ‘a number
bomb only, if the first misses the target. The divisible by 3 turns up’ and C be the event ‘a
probability that the target is hit by the second plane, number £ 4 turns up’. The probability that exactly
a
is two of A, B and C occur is , then a + b is ……… .
b
(a) 0.06 (b) 0.14 (c) 0.32 (d) 0.7
18. If the papers of 4 students can be checked by any
Numerical Value Based Questions one of the 7 teachers. If the probability that all the
15. An urn contains 5 red marbles, 4 black marbles and 4 papers are checked by exactly 2 teachers is A,
3 white marbles. Then the number of ways in then the value of 490 A must be ……… .

Answers
Round I
1. (b) 2. (b) 3. (b) 4. (d) 5. (c) 6. (d) 7. (b) 8. (a) 9. (c) 10. (b)
11. (a) 12. (c) 13. (b) 14. (b) 15. (d) 16. (d) 17. (a) 18. (c) 19. (a) 20. (a)
21. (a) 22. (c) 23. (c) 24. (d) 25. (c) 26. (d) 27. (d) 28. (c) 29. (a) 30. (a)
31. (b) 32. (b) 33. (a) 34. (d) 35. (c) 36. (a) 37. (b) 38. (c) 39. (d) 40. (b)
41. (c) 42. (b) 43. (d) 44. (d)

Round II
1. (b) 2. (c) 3. (c) 4. (a) 5. (c) 6. (a) 7. (c) 8. (d) 9. (c) 10. (c)
11. (d) 12. (b) 13. (d) 14. (c) 15. (490) 16. (7) 17. (7) 18. (60)

Round I It means, 16 students offer Mathematics.


1. Let A and B be two subsets. 6 3
\ Required probability = =
For each x Î{1, 2, 3, 4, 5}. There are four possibilities : 16 8
x Î A Ç B, x Î A ¢ Ç B, x Î A Ç B¢, x Î A ¢ Ç B¢ 3. S = { BBB, BBG , BGB, GBB, GGB, GBG , BGG , GGG }
So, the number of elements in sample space = 45 and E = { BBB, BBG , BGB, GBB, GGB, GBG , BGG }
(5C 2 ´ 33 ) 10 ´ 27 135 n (E ) = 7 and n (S ) = 8
Required probability = = = 9
45 210 2 n (E ) 7
\ P (E ) = =
2. Let total number of students be 100 in which 60% girls n (S ) 8
and 40% boys. 4. Given numbers are 1, 2, 3 and 4.
Number of boys = 40, number of girls = 60 Possibilities for unit’s place digit (either 1 or 3) = 2
25 Possibilities for ten’s place digit = 3
25% of boys offer Mathematics = ´ 40 = 10 boys
100 Possibilities for hundred’s place digit = 2
10 Possibilities for thousand’s place digit = 1
10% of girls offer Mathematics = ´ 60 = 6 girls \ Number of favourable outcomes = 2 ´ 3 ´ 2 ´ 1 = 12
100
522 JEE Main Mathematics

Number of numbers formed by 1, 2, 3, 4 2


8. Probability of not getting intercepted =
(without repetition) = 4 ! 3
12 1 3
\Required probability = = Probability of missile hitting target =
4 ´3 ´2 2 4
3
5. Total number of ways = 5 ! æ2 3ö 1
\ Probability that all 3 hit the target = ç ´ ÷ =
è3 4ø 8
and favourable number of ways = 2 × 4 !
2 × 4! 2 9. Total number of digits in any number at the unit place is
\ Required probability = =
5! 5 10.
6. Total number of outcomes in sample space, \ n (S ) = 10
n (S ) = 6 ´ 6 = 36 To get the last digit in product is 1, 3, 5 or 7, it is
Let E = Event of getting maximum sum of numbers necessary the last digit in each number must be 1, 3, 5
on two dice is 5 or 7.
= {(1, 1), (1, 2), (2, 1), (1, 3), (3, 1), (2, 2), n( A) = 4
(1, 4), (4, 1), (2, 3), (3, 2)} 4 2
\ P ( A) = =
\ n (E ) = 10 10 5
4
\ P (the maximum sum of numbers on two dice is 5) æ2ö 16
Hence, required probability = ç ÷ =
n (E ) 10 5 è5ø 625
= = =
n (S ) 36 18 10. The total number of ways = 63 = 216
\ P (sum of numbers on two dice is more than 5)
If the second number is i (i > 1 ), then the total number
= 1 - P (the maximum sum of numbers 5
on two dice is 5) of favourable ways = å (i - 1 ) (6 - i ) = 20
5 13 i =1
=1 - =
18 18 20 5
\ Required probability = =
2
7. Here, x + bx + c > 0 "x Î R 216 54

\ D <0 11. The event that the fifth toss results in a head is
independent of the event that the first four tosses result
Þ b2 < 4c
in tails.
1
\ Probability of the required event =
2
12. The number is divisible by 4, if last two digits are 12, 24,
x 32 and 52.
Value of b Possible values of c Remaining three place can be filled by 3! ways.
1 1 < 4c Þ c> 1 Þ {1, 2, 3, 4, 5, 6, 7, 8, 9}
\ Favourable cases = 3! ´ 4
4 3! ´ 4 3! ´ 4 1
Required probability = = =
2 4 < 4c Þ c >1 Þ {2, 3, 4, 5, 6, 7, 8, 9} 5! 5 ´4 ´3! 5
13. Let the probability of D selected is x.
9
3 9 < 4c Þ c> Þ {3, 4, 5, 6, 7, 8, 9} Then, P (C ) = 2x
4
P (B ) = 2x
4 16 < 4c Þ c>4 Þ {5, 6, 7, 8, 9} and P ( A ) = 4x
5 25 < 4c Þ c > 625
. Þ {7, 8, 9} Now, P ( A ) + P (B ) + P (C ) + P (D ) = 1
6 36 < 4c Þ c>9 Impossible [Q sum of all probabilities is equal to one]
7 Impossible \ 4x + 2x + 2x + x = 1
1
8 Impossible Þ 9x = 1 Þ x =
9
9 Impossible
1 2
\ P (C selected) = 2x = 2 ´ =
\Number of favourable cases = 9 + 8 + 7 + 5 + 3 9 9
= 32 14. Given that, x = 33n
Total ways = 9 ´ 9 = 81
where, n is a positive integral value.
32
\ Required probability = Here, only four digits may be at the unit place
81
Fundamentals of Probability 523

i.e. 1, 3, 7, 9. 20. In any number of last digits can be 0, 1, 2, 3, 4, 5, 6, 7,


\ n (S ) = 4 8, 9.
Let E be the event of getting 3 at its units place. Therefore, last digit of each number can be chosen in
10 ways.
n (E ) = 1
Thus, exhaustive number of ways is 10n.
n (E ) 1
\ P (E ) = = If the last digit be 1, 3, 7 or 9, then none of the
n (S ) 4
numbers can be even or end in 0 or 5.
15. Total ways = 6 ´ 5 = 30 Thus, we have a choice of four digits, viz. 1, 3, 7 or 9
Favourable events with which each of n numbers should end.
= The minimum of the two numbers is less than 4. So, favourable number of ways is 4n.
n (E ) = 6 ´ 4 = 24 Hence, the required probability is
[we can select one from {1, 2, 3, 4} and other n
4n æ2ö
from (1, 2, 3, 4, 5, 6)] =ç ÷
10n è 5 ø
24 4
\ Required probability = =
30 5 21. Total number of cases = 9999
16. Total number of 5-digit numbers Favourable cases = 10 ´ 9 ´ 8 ´ 7 = 5040
= 9 ´ 10 ´ 10 ´ 10 ´ 10 = 90000 5040
\ Probability =
Number of favourable numbers 9999
7!
= 5 ´ 5 ´ 4 ´ 4 ´ 3 = 12000 22. n (S ) =
1200 1 2 !3 !2 !
Thus, required probability = = 6!
90000 75 n (E ) = \
4
2 !2 !2 ! 3, 3, 4, 4, 5, 5
17. Three digit numbers multiple of 11 are 110, 121, ..., 990 n (E )
(81 numbers). P (E ) =
n (S )
Now, number also divisible by 9 are divisible by 99.
6!
So, numbers are 198, 297, ..., 990 (9 numbers). 2 ! 2 !2 !
9 1 =
So, required probability = = 7!
81 9 2 !3 !2 !
18. We can choose three vertices out of 6 in 6C3 = 20 ways. 3
=
Chosen vertices can form an equilateral triangle in just 7
two ways viz. A1 A3 A5 and A2A4 A6. 23. ASSISTANT : A A I N SSS TT
2 1 STATISTICS : A C II SSS TTT
\ Required probability = =
20 10 Same letters can be A, I, S, T
19. 71 = 7, 72 = 49, 73 = 343, 74 = 2401, . . . 2
C1 1
C 1
Probability of choosing, A = 9
´ 10 1 =
r
Therefore, for 7 , r Î N the number ends at unit place 7, C1 C1 45
9, 3, 1, 7, 1
C1 2
C 1
\ 7m + 7n will be divisible by 5, if it end at 5 or 0. Probability of choosing, I = 9
´ 10 1 =
C1 C1 45
But it cannot end at 5. 3
C1 3
C1 1
Probability of choosing, S = ´ =
Also, it cannot end at 0. 9
C1 10
C1 10
For this m and n should be as follows 2
C1 3
C1 1
Probability of choosing, T = 9
´ 10
=
m n C1 C1 15
1 4r 4r + 2 19
So, total probability =
2 4r + 1 4r + 3 90
3 4r + 2 4r
24. Required probability = P (WBWB ) + P (BWBW )
4 4r + 3 4r + 1
æ 5C ´ 3C ´ 4C ´ 2C ö æ 5C ´3C ´ 4C ´ 2C ö
For any given value of m, there will be 25 values of n. = ç 8 1 7 1 6 1 5 1÷ + ç 8 1 7 1 6 1 5 1÷
è C1 ´ C1 ´ C1 ´ C1 ø è C1 ´ C1 ´ C1 ´ C1 ø
Hence, the probability of the required event is
1 1 2 1
100 ´ 25 1 = + = =
= . 14 14 14 7
100 ´ 100 4
524 JEE Main Mathematics

2
25. From the given condition, it is clear that a particular C1 1C1 2 1
Probability =
5
´4 = =
person is always in a committee of 3 persons. It means, C1 C1 20 10
we have to select 2 person out of 37 person. 3 1 2
37 \ Required probability = + =
C2 10 10 5
\ Required probability = 38
C3 33. Out of 90 tickets, two tickets already considered,
26. Total number of socks = 5 + 4 = 9 instead of selecting 5 tickets we have to select only 3
tickets out of 88 tickets.
The number of ways to select 2 socks out of 9 = 9C 2 88 ´ 87 ´ 86
88
Number of ways to select both brown socks = 5 C2 C3 3 ´2 ´1
\ Required probability = 90 =
And number of ways to select both white socks = 4C 2 C5 90 ´ 89 ´ 88 ´ 87 ´ 86
5 5 ´4 ´3 ´2 ´1
C 2 + 4C 2
\ P(either both brown or white) = 5 ´4 2
9
C2 = =
90 ´ 89 801
5! 4!
+ 34. P (selecting an apple from both baskets)
3 ! × 2 ! 2 ! × 2 ! 10 + 6 16 3 48
= = = ´ = = P (apple from first basket) × P(apple from second
9! 36 36 3 108 basket)
5
7 !× 2 ! C1 4C1
= 12 × 12
C1 C1
27. 13 applicants = 8 men + 5 women
P (selecting a orange from both baskets)
2 persons are selected i.e. (1 men + 1 women) or 2 women
8
= P (orange from first basket) × P(orange from second
C1 ´ 5C1 5
C2 50 25 basket)
\Required probability = 13
+ 13
= 13 =
C2 C2 C 2 39 7
C1 8C1
= 12 × 12
28. Numbers divisible by 3 are 3, 6, 9, 12, 15, 18, 21, 24 C1 C1
and numbers divisible by 4 are 4, 8, 12, 16, 20, 24
5
C 4C 7
C 8C
Required probability = 12 112 1 + 12 112 1
and numbers divisible by (3 ´ 4 = 12) are 12, 24 C1 C1 C1 C1
\Probability that the chosen number is divisible by 3 20 + 56 76
8 6 2
= =
C C C 12 144 144
or 4 = 25 1 + 25 1 - 25 1 =
C1 C1 C1 25 35. The sum of the selected numbers is odd, if exactly one of
them is even and one is odd.
29. We have mentioned that boxes are different and one
particular box has 3 balls. \ Favourable number of cases = 15C1 × 15C1
15
12
C3 ´ 29 55 æ 2 ö
11 C1 × 15C1 15
Then, number of ways = = \ Required probability = =
ç ÷ 30
C2 29
312 3 è3ø
30. Total number of ways of selecting 11 players = 15C11 36. n (S ) = 10C7 = 120

Favourable cases = 8C 6 ´7C5 n ( A ) = 5C 4 ´ 3C 2 ´ 2 C1


5 ´3 ´2 1
8
C ´7C5 P (E ) = =
\ Required probability = 156 120 4
C11
1 3
37. P ( A ) = , P ( A È B ) =
31. There are two conditions arise. 3 4
(i) When first is an ace of heart and second one is Now, P ( A È B ) = P ( A ) + P (B ) - P ( A Ç B ) £ P ( A ) + P (B )
1 51 1 3 1 5
non-ace of heart, the probability = ´ = Þ £ + P (B ) Þ £ P (B )
52 51 52 4 3 12
(ii) When first is non-ace of heart and second one is an Again, we have B Í A È B
51 1 1 3
ace of heart, the probability = ´ = \ P (B ) £ P ( A È B ) =
52 51 52 4
1 1 1 5 3
\ Required probability = + = Hence, £ P (B) £ .
52 52 26 12 4

32. There are two cases arise. 38. Total cases = 52 C 4


Case I If Ist ball is white, then Favourable cases = (13C1 )4
3
C 2
C 6 3 (13 C )4
Probability = 5 1 ´ 4 1 = = So, probability = 52 1
C1 C1 20 10 C4
Case II If Ist ball is red, then 13 ´ 13 ´ 13 ´ 13 ´ 1 ´ 2 ´ 3 ´ 4 2197
= =
52 ´ 51 ´ 50 ´ 49 20825
Fundamentals of Probability 525

39. P ( A Ç B ) = P (B ) - P ( A Ç B ) 43. We know,


A B P ( A È B ) = P ( A ) + P (B ) - P ( A Ç B ) £ P ( A ) + P (B )
[Q P ( A Ç B ) ³ 0]
44. Clearly, P (A È B ÈC ) = 1
Þ P ( A ) + P (B ) + P (C ) = 1
1 1
Þ P ( A) + P ( A ) + P ( A) = 1
P(A Ç B) 2 3
11
40. Let M and B denote the students of Mathematics and Þ P ( A) = 1
Biology. 6
6
Given, P (M ) = 40%, P (B ) = 30% Þ P ( A) =
11
P (M Ç B ) = 10%
[Q 10% of the class study both subjects i.e. they Round II
are common in both subjects]
1. The 6-digit integer formed by 0, 1, 2, 3, 4, 5, 6 without
\ P (M or B ) = P (M È B ) repetition is 6 ´ 6 !
= P (M ) + P (B ) - P (M Ç B ) n (S ) = 6 ´ 6 !
40 30 10
= + - Favourable Cases
100 100 100
Number divisible by 3 = Sum of the digit must be
60
= = 60% = 0.6 divisible by 3
100
Case I
3 1
41. Here, P ( A È B) = and P ( A Ç B) = . If the six digit no formed by the digit 1, 2, 3, 4, 5, 6
5 5
Number of ways = 6 !
So, from the addition theorem,
3 1 Case II
= P ( A ) + P (B) -
5 5 Number used to formed six-digit numbers are
4 0, 1, 2, 4, 5, 6
or = 1 - P ( A ) + 1 - P (B)
5 Number of ways is 5 ´ 5 !
4 6 Case III
\ P ( A ) + P (B) = 2 - =
5 5 Number used to formed six-digit numbers are
3 0, 1, 2, 3, 4, 5
42. We have, P ( A È B È C ) =
4 Number of ways is 5 ´ 5 !
i.e. P ( A ) + P (B) + P (C ) - P ( A Ç B) - P (B Ç C ) 6! + 5 ´ 5! + 5 ´ 5!
P ( A) =
3 6 ´ 6!
-P ( A Ç C ) + P ( A Ç B Ç C ) = …(i)
4 4
=
P ( A Ç B) + P (B Ç C ) + P ( A Ç C ) 9
1 100
-2 P ( A Ç B Ç C ) = …(ii) 2. x + > 50
2 x
and P ( A Ç B) + P (B Ç C ) + P ( A Ç C ) Þ x2 + 100 > 50 x [Q x Î N ]
2 2
-3P ( A Ç B Ç C ) = …(iii) Þ x - 50x + 100 > 0
5
Þ (x - 25)2 > 525
From Eqs. (ii) and (iii), we get
Þ x - 25 < - 525 or x - 25 > 525
1 2 1
P(A Ç B ÇC ) = - = …(iv) Þ x < 25 - 22.9 or x > 25 + 22.9
2 5 10
1 Þ x £ 2 or x ³ 48
Þ P ( A ) P (B) P (C ) =
10 Hence, the number of favourable cases = 2 + 53 = 55
1 55 11
Þ pmc = Thus, required probability = =
10 100 20
From Eqs. (i), (ii) and (iii), we get 3. Case I When digits are repeated.
æ1 2 ö 1 3 Total four digit numbers which are greater than 5000
P ( A ) + P (B) + P (C ) - ç + ÷+ =
è 2 10 ø 10 4 from the digits 0, 1, 3, 5,7 i.e. in thousand place, the
27 number will be 5 or 7 and rest of three places any five
Þ p+m+ c=
20 numbers will come
526 JEE Main Mathematics

5 5 ´ 5 ´ 5 - 1 = 124 25 - 4ac ³ 0 Þ ac £
25
5
(here, one is subtracted because one case is possible in 4
\ ac = 1, 2 , 3, 4, 5, 6
which all the three places are occupied by the number
ac ¹ 1, 2, 3, 4, 5
zero.)
for ac = 6
7 5 ´ 5 ´ 5 = 125 (2, 3) (3, 2)
\Total four digit numbers, n (S) = 124 + 125 = 249 \ Favourable ways = 2
Total four digit numbers which are divisible by 5. Favourable ways
\Required probability =
i. e. , In unit place number will be 0 or 5 and thousand Total ways
place the number will be 5 to 7. And rest of the two 2 1
= =
places any five numbers will be come. 6 3
2
5 5 ´ 5 0 - 1 = 24 C1 4C1 8
5. Required probability = 9
´ =
C1 9C1 81
7 5 ´ 5 0 = 25
5 5 ´5 5 = 25
6. Total number of boys = 10

7 5 ´5 5 = 25 Total number of girls = 8


Number of students have to be selected at random = 3
Number of favourable cases, n(E)
= 24 + 25 + 25 + 25 = 99 If 2 girls and 1 boy are selected, then the required
8´7
n (E ) 99 33 8 ´ 10
\Required probability = = = C 2 ´ 10C1 2
n (S ) 249 83 probability = =
18
C3 18 ´ 17 ´ 16
Case II When digits are not repeated. 3 ´2
Total four digit numbers which are greater than 5000 é n n! ù
from the digits 0, 1, 3, 7, 5 i. e. , In thousand place, the êQ C r = ú
ë r ! (n - r )! û
number will be 5 or 7. Since, the digit are not repeated,
4 ´ 7 ´ 10 ´ 6 70
then rest of the place may be filled by 4, 3, 2. = =
18 ´ 17 ´ 16 3 ´ 17 ´ 4
5 4 ´ 3 ´ 2 = 24
35 35
7 4 ´ 3 ´ 2 = 24 = =
6 ´ 17 102
Total four digit number n (S ) = 24 + 24 = 48
7 ×6 ×5
Total four digit numbers which are divisible by 5 7. n (S ) = 7C3 ´ 3 ! = × 6 = 210
6
5 3 ´2 0 =6 n (E ) = 2C1 ´ 2C1 ´ 1C1 ´ 3 !
7 3 ´2 0 =6
P P P P
7 3 ´2 5 =6
Favourable number of cases n (E ) = 6 + 6 + 6 = 18 because one has to sit at any one of the two marked
18 3 seats on the left and the other has to sit at any one of
\ Required probability = = the two marked seats on the right.
48 8
n (E ) 2 ´ 2 ´ 6 4
4. First three prime numbers are 2, 3 and 5 \ P (E ) = = =
n (S ) 210 35
\ Total ways of choosing a, b, c = 3 ´ 2 ´ 1 = 6
8. n (S ) = 100 ´ 100 ´ 100
[Q a ¹ b ¹ c]
We know that, (2n + 1)2 + (2n 2 + 2n )2 = (2n 2 + 2n + 1)2
For roots to be real, b2 - 4ac ³ 0 for all n Î N .
Value of b Possible values of a and c \ For n = 1, 2, 3, 4, 5, 6, we get lengths of the three
2 4 - 4ac ³ 0 Þ ac £ 1 sides of a right angled trianle whose longest side £ 100.
a= c=1 e.g. When n = 1, sides are 3, 4, 5
(No values of a and c) and when n = 2, sides are 5, 12, 13 and so on.
9
3 9 - 4ac ³ 0 Þ ac £ The number of selections of 3, 4, 5 from the three cards
4
ac = 1, ac = 2
by taking one from each is 3!.
a = c = 1, a = 2 , c = 1 \ n (E ) = 6 (3 !)
2
a = 1, c = 2 6 (3 !) 1 æ3ö
Hence, P (E ) = = ç ÷
(no values of a and c) 100 ´ 100 ´ 100 100 è 50 ø
Fundamentals of Probability 527

9. n (S ) = n (when 7 appears on thousands place) and P (B ) = 0.2 Þ P ( A ) = 0.7


+ n (7 does not appear on thousands place) and P (B ) = 0.8
=1 ´9 ´9 ´9 + 8 ´9 ´9 ´3 \Required probability
= 729 + 1944 = 2673 = P ( A ) P (B) + P ( A ) P (B ) P ( A ) P (B ) + K
n (E ) = n (last digit 7 and 7 appears once) = (0.7)(0.2) + (0.7)(0.8)(0.7)(0.2)
+ n (last digit 2, when 7 appears once) + (0.7) (0.8)(0.7)(0.8) (0.7)(0. 2) + ...
= 8 ´ 9 ´ 9 ´ 1 + [(1 ´ 9 ´ 9 ´ 1) + (9 ´ 8 ´ 2C1 ´ 1)] = 0.14 [1 + (0.56) + (0.56)2 + ... ]
= 648 + 81 + 144 = 873 æ 1 ö 0.14
= 0.14 ç ÷= = 0.32
873 97 è 1 - 0.56 ø 0.44
P (E ) = =
2673 297
15. It is given that an urn contains 5 red marbles, 4 blacks
10. Total number of the students is 80. Total number of girls marbles and 3 white marbles, then the number of ways
is 25. Total number of boys is 55. There are 10 rich, 70 in which 4 marbles can be drawn so that at the most
poor, 20 intelligent students in the class. three of them are red is
Therefore, required probability is = (total number of ways to drawn 4 marbles)
1 1 25 5 - (numbers of ways to drawn 4 marbles
´ ´ =
4 8 80 512 so that all are red)
(I) (R) (G) 12 5 12 ´ 11 ´ 10 ´ 9
= C4 - C4 = -5
4 ´3 ´2
11. The probability that the first critic favours the book is
5 5 = (11 ´ 5 ´ 9) - 5
P (E1 ) = = = 5(99 - 1) = 5 ´ 98 = 490
5+2 7
The probability that the second critic favours the book is 16. Any three tickets out of 21 tickets can be chosen is
21
4 4 C3 ways.
P (E 2) = =
4+3 7 For the favourable choice, if the chosen numbers are
a+c
The probability that the third critic favours the book is a , b and c, a < b < c, then we should have = b.
3 3 2
P (E3 ) = = Obviously either both a and c are even or both are odd
3+4 7
and then b is fixed.
Majority will be in favour of the book, if atleast two
Hence, for the favourable choice, we have to choose
critics favour the book. two numbers from 1 to 21, which are either both even
Hence, the probability is or both odd.
P (E1 Ç E 2 Ç E3 ) + P (E1 Ç E 2 Ç E3 ) + P (E1 Ç E 2 Ç E3 ) This can be done in 11C 2 + 10C 2 ways.
11
+ P (E1 Ç E 2 Ç E3 ) C 2 + 10C 2 10
Hence, required probability = 21
=
= P (E1 )P (E 2)P (E3 ) + P (E1 )P (E 2)P (E3 ) C3 133
+ P (E1 Ç E 2 Ç E3 ) + P (E1 )P (E 2)P (E3 ) a
Þ Þ 14a - b = 140 - 133 = 7
5 4 æ 3ö 5 æ 4ö 3 b
= ´ ´ ç1 - ÷ + ´ ç1 - ÷ ´
7 7 è 7ø 7 è 7ø 7 17. Event A = {1, 3, 5}, event B = {3, 6}, event C = {1, 2, 3, 4}
æ 5 ö 4 3 5 4 3 209 \ A Ç B = {3}, B Ç C = {3}, A Ç C = {1, 3}, A Ç B Ç C = {3}
+ ç1 - ÷ ´ ´ + ´ ´ =
è 7 ø 7 7 7 7 7 343 Thus, P (exactly two of A, B and C occur)
12. Total number of cases = 9C3 = 84 = P ( A Ç B) + P (B Ç C ) + P (C Ç A ) - 3P ( A Ç B Ç C )
Number of favourable cases = 3C1 × 4C1 × 2C1 = 24 1 1 2 1 1 a
= + + -3 ´ = = Þ a + b = 7
24 2 6 6 6 6 6 b
\ p= =
84 7 18. Total number of ways in which papers of 4 students, can
13. Since, A = {4 , 5, 6} and B = { 1, 2, 3, 4 } be checked by seven teachers = 74
\ A Ç B = {4} Now, choosing two teachers out of 7 is 7C 2 = 21
\ P ( A È B) = P ( A ) + P (B ) - P ( A Ç B ) The number of ways in which 4 papers can be checked
3 4 1
= + - =1 by exactly two teachers = 24 - 2 = 14
6 6 6
\ Favourable ways = (21)(14)
14. Let the events be (21)(14) 6
A = Ist aeroplane hit the target \ Required probability = = =A
74 49
B = IInd aeroplane hit the target 6
and their corresponding probabilities are \ 490 A = 490 ´ = 60
49
P ( A ) = 0.3
22
Matrices
A rectangular array of mn numbers in the form of m horizontal lines (called IN THIS CHAPTER ....
rows) and n vertical lines (called columns), is called a matrix of order m ´ n.
Matrix
This type of array is enclosed by [ ] or ( )
Types of Matrices
Each of mn numbers of a matrix is known as element of a matrix. A matrix is
generally denoted by A, B, C , ... etc., and its element is denoted by aij , where Equality of Two Matrices
aij belongs to the ith row and jth column and is called ( i , j)th element of the Algebra of Matrices
matrix A = [aij ] . Trace of a Matrix
An m ´ n matrix is usually written as Transpose of a Matrix
é a11 a12 K a1n ù
Special Types of Matrices
êa a22 K a2n ú
A = ê 21 ú = [aij ]m ´ n Elementary Row Transformations
ê M M M ú
Elementary Matrix
êa ú
ë m1 am 2 K amn û
é3 2 7ù
e.g. A = ê 5 -4 6 ú is a matrix of order 3 ´ 3.
ê ú
êë 4 8 -12úû

Types of Matrices
Row Matrix
A matrix which has only one row and any number of columns, is called a row
matrix. e.g. A = [ 27 85 1 4 ] 1 ´ 4 is a row matrix.
Column Matrix
A matrix is said to be a column matrix, if it has only one column and any
number of rows.
é 1ù
e.g. A = ê 2ú is a column matrix.
ê ú
êë 3úû3 ´ 1
532 JEE Main Mathematics

Rectangular Matrix Scalar Matrix


A matrix in which number of rows is not equal to the A square matrix A = [aij ] is said to be scalar matrix, if
number of columns or vice-versa is called a rectangular (a) aij = 0, " i ¹ j
matrix. (b) aij = k, " i = j, where k ¹ 0
é 1 2 3ù
e.g. A = ê ú is a rectangular matrix of order 2 ´ 3. i.e. a diagonal matrix is said to be a scalar matrix, if the
ë 4 5 6û elements of principle diagonal are same.
Square Matrix é 5 0 0ù
A matrix in which number of rows is equal to the number e.g. A = ê 0 5 0ú is a scalar matrix.
ê ú
of columns, is called a square matrix. The elements aij of êë 0 0 5úû
a square matrix A = [aij ]m ´ m for which i = j i.e, the
elements a11 , a22 , . . . , amm are called the diagonal Unit Matrix or Identity Matrix
elements and the line along which they lie is called the A square matrix A = [aij ] is said to be a unit matrix or
principal diagonal or leading diagonal of the matrix. identity matrix, if
é 1 2 3ù (a) aij = 0, " i ¹ j (b) aij = 1, " i = j
e.g. A = ê 3 2 1ú i.e. A diagonal matrix, whose elements of principle
ê ú
êë 2 3 1 úû3 ´ 3 diagonal are equal to 1 and all remaining elements
are zero, is known as unit or identity matrix. It is
is a square matrix of order 3 in which diagonal elements denoted by I.
are 1, 2, 1. é 1 0 0ù
e.g. I = ê 0 1 0ú is a unit matrix of order 3.
Null Matrix ê ú
A matrix of order m ´ n whose all elements are zero, is êë 0 0 1 úû
called a null matrix of order m ´ n.
Upper Triangular Matrix
It is denoted by O.
A square matrix A = [aij ] is known as upper triangular
é 0 0ù é0 0 0ù matrix, if
e.g. ê 0 0ú and ê 0 0
ë û ë 0úû aij = 0, " i > j.
are two null matrices of order 2 ´ 2 and 2 ´ 3, respectively. é 1 4 5ù
e.g. A = ê 0 2 6ú is an upper triangular matrix.
Diagonal Matrix ê ú
A square matrix is called a diagonal matrix, if all its êë 0 0 3úû
non-diagonal elements are zero and diagonal elements
are not all equal.
Lower Triangular Matrix
If d1 , d2 , d3 , ... , dn are elements of principal diagonal of a A square matrix A = [aij ] is known as lower triangular
diagonal matrix of order n ´ n, then matrix is denoted as matrix, if aij = 0, " i < j.
diag [d1 , d2 , ... , dn ]. é 1 0 0ù
é a 0 0ù e.g. A = ê 4 2 0ú is a lower triangular matrix.
ê ú
e.g. A = ê 0 b 0ú is a diagonal matrix which is denoted êë 5 6 3úû
ê ú
êë 0 0 c úû Submatrices of a Matrix
by A = diag [a , b, c]. A matrix B obtained by deleting the row (s) or Column (s)
Note The number of zeroes in a diagonal matrix is given by or both of a matrix A is said to be a submatrix of A.
n 2 - n, where n is an order of the matrix. i.e. The matrix B constituted by the array of elements,
which are left after deleting some rows or columns or
Triple Diagonal Matrix both of matrix A is called submatrix of A.
A square matrix A is said to be a triple diagonal matrix, (a) Principle Submatrix A square submatrix B of a
if all its elements are zero except possibly for those lying square matrix A is called a principle submatrix, if
on the principle diagonal, the diagonal immediately the diagonal elements of B are aslo diagonal
above as well as below the principle diagonal. elements of A.
é 1 -1 0 0ù (b) Leading Submatrix A principle square submatrix
é 5 -3 0 ù ê ú B is said to be a leading submatrix of a square
e.g. ê -3 4 -3ú and ê -1 2 -1 0ú
ê ú matrix A if it is obtained by deleting only some of
ê0 1 2 3ú
the last rows and the corresponding columns such
êë 0 0 4 úû ê 0 0 4 5ú
ë û that the leading elements (i.e. a11) is not lost.
Matrices 533

Horizontal Matrix Algebra of Matrices


Any matrix in which the number of columns is
more than the number of rows is called a horizontal Addition and Subtraction of Two Matrices
matrix. Let A = [aij ]m ´ n and B = [bij ]m ´ n are two matrices whose
é2 3 4 5 ù orders are same, then
e.g. ê 8 9 7 -2ú is a horizontal matrix. A + B = [aij + bij ], " i = 1 , 2 , ... , m and j = 1 , 2 , ... , n
ê ú
êë 2 -2 -3 4 úû Also, A - B = A + ( - B)
é 1 2ù é 5 6ù
Vertical Matrix e.g. If A = ê ú and B= ê ú
Any matrix in which the number of rows is ë 3 4û ë 7 8û
more than the number of columns is called column é 1 2ù é 5 6ù é 1 + 5 2 + 6ù é 6 8ù
Then, A + B = ê ú+ê ú=ê =
matrix. ë 3 4û ë 7 8û ë 3 + 7 4 + 8úû êë10 12úû
é 2 3ù é 1 2ù é 5 6ù
e.g. ê 4 5ú is a column matrix. A- B= ê ú-ê ú
ê ú ë 3 4û ë 7 8û
êë 6 7úû
é 1 - 5 2 - 6ù é - 4 - 4ù
=ê ú=ê ú
ë 3 - 7 4 - 8û ë - 4 - 4û
Equality of Two Matrices
Two matrices A = [aij ]m ´ n and B = [bij ]m ´ n are said to be Properties of Matrix Addition
equal, if
Let A, B and C are three matrices of same order, then
(i) Number of rows in A is equal to number of rows in B .
(i) Matrix addition is commutative, i.e. A + B = B + A
(ii) Number of columns in A is equal to number of
columns in B . (ii) Matrix addition is associative,
(iii) aij = bij , " i = 1, 2 , ... , m and j = 1, 2 , ... , n i.e. ( A + B) + C = A + ( B + C )
(iii) If O is a null matrix of order m ´ n and
éx yù é x 6 ù é 4 x + yù
Example 1. If 3 ê ú = ê -1 2w ú + ê z + w , then the A + O = A = O + A , then O is known as additive
ë z w û ë û ë 3 úû identity.
value of z is
(iv) If for each matrix A = [aij ]m ´ n , a matrix ( - A) is
(a) 1 (b) 2 (c) 3 (d) 4
such that A + ( - A) = O = ( - A) + A,
éx yù é x 6 ù é 4 x + yù
Sol. (a) Given, 3 ê ú = ê -1 2w ú + ê z + w then matrix ( - A) is known as additive inverse of A.
ë z w û ë û ë 3 úû
(v) Matrix addition follows cancellation law,
é 3x 3y ù é x + 4 6 + x + yù
Þ ê3z 3w ú = ê -1 + z + w 2w + 3 ú i.e. A + B = A + C Þ B = C (left cancellation law)
ë û ë û
and B + A = C + A Þ B = C (right cancellation law)
By definition of equality of matrix as the given matrices are
equal, their corresponding elements are equal. Comparing Note Two matrices are said to be conformable for addition or
the corresponding elements, we get subtraction, if they are of the same order.
3x = x + 4 Þ 2x = 4 Þ x = 2 Example 2. If 1, w and w2 are the cube roots of unity and
and 3y = 6 + x + y Þ 2y = 6 + x
6+ x é1 + w 2 wù é a - wù é 0 wù
Þ y= ê - 2 w -b ú + ê3 w =
2úû êë w 1 úû
,
2 ë û ë
On putting the value of x, we get then a2 + b 2 is equal to
6+2 8
y= = =4 (a) 1 + w2 (b) w2 - 1 (c) 1 + w (d) (1 + w) 2
2 2
Now, 3z = - 1 + z + w, é1 + w 2 wù é a - wù é 0 wù
2z = -1 + w Sol. (c) Given, ê ú+ê ú=ê 1 úû
ë - 2 w - bû ë3 w 2 û ë w
-1 + w
z= …(i) é1 + w + a w ù é0 wù
2 Þ =
ê w 2 - búû êë w 1 úû
Now, 3w = 2w + 3 ë
Þ w =3 Þ 1+ w + a = 0 , 2 - b =1
On putting the value of w in Eq. (i), we get
Þ a = - 1 - w, b = 1
-1 + 3 2
z= = =1 \ a2 + b 2 = ( -1 - w ) 2 + 12 = 1 + w2 + 2 w + 12
2 2
Hence, the values of x, y, z and w are 2, 4, 1 and 3. =0 + w+1 [Q1 + w + w2 = 0 ]
534 JEE Main Mathematics

Scalar Multiplication (v) AB = O does not necessarily imply that A = O or


B = O or both A and B are O.
Let A = [aij ] be any m ´ n matrix and k be any scalar.
Then, the matrix obtained by multiplying each element of é 0 - 1ù é 1 1ù
e.g. If A= ê ú ¹ O and B = ê 0 0ú ¹ O
A by k is called the scalar multiple of A by k and is ë0 0 û ë û
denoted by kA. é 0 0ù
But AB = ê ú =O
Thus, if A = [aij ]m ´ n , then kA = [kaij ]m ´ n . ë 0 0û
é 1 2 3ù é 2 4 6ù
e.g. If A = 3 2 1 , then 2 A = ê 6 4 2ú
ê ú
Example 3. If A = ê
é cos q i sin q ù æ
, çq =

ê ú ê ú ÷
êë 1 3 1 úû êë 2 6 2úû ëi sin q cos q úû è 24 ø
éa bù
and A 5 = ê ú, where i = -1, then which one of the
Properties of Scalar Multiplication ëc d û
If A = [aij ]m ´ n and B = [bij ]m ´ n are two matrices and l following is not true? (JEE Main 2020)
and m are two scalars, then (a) a2 - d 2 = 0 (b) a2 - c2 = 1
(i) l ( A + B) = lA + lB (ii) ( l + m ) A = lA + mA (c) a2 - b 2 = 1 (d) 0 £ a2 + b 2 £ 1
(iii) ( l ) mA = m( l A) = l(m A) (iv) ( - l ) A = -( lA) = l( - A)
é cos q i sin qù
Sol. (c) Since, matrix A = ê
ëi sin q cos q úû
Multiplication of Two Matrices
é cos 2q i sin 2qù
If A = [aij ]m ´ n and B = [bij ]n ´ p are two matrices such Þ A2 = ê
ëi sin 2q cos 2q úû
that the number of columns of A is equal to the number
é cos 3 q i sin 3 qù
of rows of B, then a matrix C = [cij ]m ´ p of order m ´ p is Þ A3 = ê ú
known as product of matrices A and B, where ëi sin 3 q cos 3 q û
n é cos 4q i sin 4qù
Þ A4 = ê
cij = å ai kbk j = ai 1 b1 j + ai 2 b2 j + ....+ ain bn j ëi sin 4q cos 4q û
ú
k=1
On multiplication of matrices is denoted by C = AB é cos 5q i sin 5qù é a bù
Þ A5 = ê ú=ê ú
é 2 1ù é 5 2ù ëi sin 5q cos 5q û ë c d û
e.g. If A = ê ú and B = ê ú, then
ë 3 5û ë 1 7û Þ a = d = cos 5q, b = c = i sin 5q

é 2 ´ 5 + 1 ´ 1 2 ´ 2 + 1 ´ 7 ù é 11 11 ù \ a2 - b 2 = cos2 5q - i 2 sin 2 5q = 1
AB = ê ú=ê ú Hence, option (c) is correct.
ë 3 ´ 5 + 5 ´ 1 3 ´ 2 + 5 ´ 7û ë 20 41û
Note In multiplication of two matrices A and B, the order roles an Example 4. Let a be a root of the equation x 2 + x + 1 = 0
important role. é1 1 1ù
1 ê
and the matrix A = 1 a a 2 ú , then the matrix A31 is
Properties of Multiplication of Matrices 3ê 2 4
ú
Let A = [aij ]m ´ n , B = [bij ]n ´ and C = [cij ] p ´ m are three êë1 a a úû
p
equal to (JEE Main 2020)
matrices, then
(a) A3 (b) I3 (c) A2 (d) A
(i) Generally, matrix multiplication is not
commutative, Sol. (a) It is given that ‘a’ is the root of equation x2 + x + 1 = 0 , so
i.e. AB ¹ BA a 2 + a + 1 = 0 and a3 = 1.
(ii) Matrix multiplication is associative, é1 1 1 ù é1 1 1 ù
1 ê 2ú 1 ê
i.e. A( BC ) = ( AB) C Now, matrix A = 1 a a = 1 a a 2ú
3ê 2 4
ú 3ê 2
ú
(iii) Matrix multiplication is distributive over matrix êë1 a a úû êë1 a a úû
addition. [Q a3 = 1]
i.e. A( B + C ) = AB + AC 2 é1 1 1 ù é1 1 1 ù
æ 1 ö ê 2ú ê 2ú
(iv) If A is a m ´ n matrix and I n is an identity matrix of \ A2 = ç ÷ ê1 a a ú ê1 a a ú
è 3ø
order n ´ n and I m is an identity matrix of order êë1 a
2
a úû êë1 a 2
a úû
m ´ m, then é 1 + 1 + 1 1 + a + a2 1 + a2 + a ù
I m A = A and AI n = A 1ê ú
= ê1 + a + a 2 1 + a 2 + a 4 1 + a3 + a3 ú
3
In particular, if A is a square matrix of order n, then ê1 + a 2 + a 1 + a3 + a3 1 + a 4 + a 2ú
ë û
AI n = I n A = A
Matrices 535

é3 0 0 ù é 1 0 0 ù
1ê Transpose of a Matrix
=0 0 3 ú = ê0 0 1ú
3ê ú ê ú If A = [aij ]m ´ n is a matrix of order m ´ n, then the
êë0 3 0 úû êë0 1 0 úû
transpose of A can be obtained by changing all rows to
[Q a3 = 1 and a 2+ a + 1 = 0 ]
columns and all columns to rows i.e. transpose of
\ A4 = A2 × A2
A = [a ji ]n ´ m . It is denoted by A¢ , AT or At .
é1 0 0ùé1 0 0ù é1 0 0ù
= ê0 0 1ú ê0 0 1ú = ê0 1 0 ú = I é 1 3 5ù
é1 6ù
ê úê ú ê ú ê
êë0 1 0 úû êë0 1 0 úû êë0 0 1úû e.g. If A = ê ú , then A¢ = ê 3 9ú
ë 6 9 4û ú
\ A8 = I = A16 = A20 = A24 = A28 êë 5 4úû
Q A31 = A28 × A3 = I × A3 = A3 Þ A31 = A3
Properties of Transpose
If A and B are two matrices and k is a scalar, then
Trace of a Matrix (i) ( A¢ ) ¢ = A (ii) ( A + B) ¢ = A¢ + B¢
Let A = [aij ]n ´ n be a square matrix. Then, the sum of
(iii) ( kA)¢ = kA¢ (iv) ( AB)¢ = B¢ A¢ (reversal law)
all diagonal elements of A is called the trace of A and is
denoted by tr( A). Note If A, B and C are any three matrices conformable for
n multiplication, then ( ABC )¢ = C ¢ B¢ A¢
Thus, tr( A) = å aii = a11 + a22 + . . . + ann
i =1 Example 6. Let a, b, c ÎR be all non-zero and satisfy
é 2 -7 9ù æa b c ö
A = ê 0 3 2ú, then tr( A) = 2 + 3 + 4 = 9 ç ÷
e.g.
ê ú a + b + c = 2. If the matrix A = ç b c a ÷ satisfies AT A = I,
3 3 3

êë 8 9 4úû ç ÷
èc a b ø
then a value of abc can be (JEE Main 2020)
Properties of Trace of a Matrix
1 1 2
Let A = [aij ] and B = [bij ] are two square matrix of order (a) - (b) (c) 3 (d)
3 3 3
n, then
(i) tr( A + B) = tr( A) + tr( B) é a b cù
(ii) tr( AB) = tr( BA) Sol. (a) Given matrix, A = ê b c a ú
ê ú
(iii) tr( lA) = ltr( A) , where l is a scalar. êë c a búû
(iv) tr( A¢ ) = tr( A) é a b cù é a b cù
So, AT A = ê b c a ú ê b c a ú
é2 1ù é5 4ù ê úê ú
Example 5. Consider three matrices X = ê ú, Y = ê6 5 ú êë c a búû êë c a búû
ë 4 1û ë û é a2 + b 2 + c2 ab + bc + ca ac + ba + cbù
é 5 - 4ù ê ú
and Z = ê ú. Then, the value of the sum = ê ba + cb + ac b 2 + c2 + a2 bc + ca + abú
ë- 6 5 û ê ca + ab + bc cb + ac + ba c2 + a2 + b 2 ú
æ X(YZ) 2 ö æ X(YZ)3 ö ë û
æ XYZ ö
tr ( X) + tr ç ÷ + tr ç ÷ + tr ç ÷ + K to ¥ is AT A = I [given]
è 2 ø è 4 ø è 8 ø
Q
2 2 2
\ a + b + c = 1 and ab + bc + ca = 0 …(i)
(a) 6 (b) 9 (c) 12 (d) None of these 2 2 2 2
Now, as ( a + b + c) = a + b + c + 2 ( ab + bc + ca)
é5 4ù é 5 - 4ù é 1 0 ù
Sol. (a) Here, YZ = ê úê ú=ê ú Þ ( a + b + c) 2 = 1Þ a + b + c = ± 1 …(ii)
ë6 5 û ë -6 5 û ë0 1û
As we know,
æ XYZ ö æ X (YZ) 2 ö æ X (YZ)3 ö
\ tr( X) + tr ç ÷ + tr ç ÷ + tr ç ÷+K a3 + b3 + c3 - 3abc = ( a + b + c) ( a2 + b 2 + c2 - ab - bc - ca)
è 2 ø è 4 ø è 8 ø
Þ 2 - 3abc = ( a + b + c) (1 - 0)
æXö æXö
= tr( X) + tr ç ÷ + tr ç ÷ + K [from Eq. (i) and a3 + b3 + c3 = 2 (given)
è2ø è 4ø
2 - ( a + b + c)
1 1 Þ -3abc = ( a + b + c) - 2 Þ abc =
= tr( X) + tr ( X) + tr( X) + K 3
2 4
1
é 1 1 ù If a + b + c = 1Þ abc = , or
= tr( X) ê1 + + 2 + Kú 3
ë 2 2 û
1 If a + b + c = -1Þ abc = 1
= tr( X) 1
1 Now, from the options abc =
1- 3
2
= 2 tr ( X) = 2 (2 + 1) = 6 Hence, option (b) is correct.
536 JEE Main Mathematics

æ 0 2q r ö éa h gù
ç ÷
Example 7. Let A = ç p q -r ÷. If AAT = I3, then| p| is i.g. If A = êh b fú ,
ç ÷ ê ú
è p -q r ø êë g f c úû
(JEE Main 2019)
éa h gù
1 1 1 1
(a) (b) (c) (d) then A¢ = ê h b fú Þ A¢ = A
5 2 3 6 ê ú
êë g f c úû
Sol. (b) Given, AAT = I
Thus, matrix A is a symmetric matrix.
é0 2q r ù é0 p p ù é1 0 0ù
Þ ê p q -r ú ê2q q-q ú = ê0 1 0 ú
ê ú ê ú ê ú Skew-symmetric Matrix
êë p -q r úû êë r -rr úû êë0 0 1úû A square matrix A = [aij ]m ´ m is said to be a
é0 + 4q 2 + r 2 0 + 2q 2 - r 2 0 - 2q 2 + r 2ù é 1 0 0 ù skew-symmetric matrix, if
ê ú
Þ ê 0 + 2q 2 - r 2 p 2 + q 2 + r 2 p2 - q 2 - r 2 ú = ê0 1 0 ú (a) aij = - a ji , " i , j
ê ú
ê 0 - 2q 2 + r 2 p 2 - q 2 - r 2 p 2 + q 2 + r 2úû êë0 0 1úû (b) Each element of diagonal is zero.
ë
We know that, if two matrices are equal, then corresponding OR
elements are also equal, so A square matrix A is said to be a skew-symmetric
4q 2 + r 2 = 1 = p 2 + q 2 + r 2, … (i) matrix, if A¢ = - A.
2q 2 - r 2 = 0 Þ r 2 = 2q 2 … (ii) é 0 2 -3ù
e.g. If A = ê -2 0 3ú ,
and 2 2
p -q -r =0 2
… (iii) ê ú
êë 3 -3 0úû
Using Eqs. (ii) and (iii), we get
p 2 = 3q 2 … (iv) é 0 -2 3ù é 0 2 -3ù
Using Eqs. (ii) and (iv) in Eq. (i), we get then A¢ = ê 2 0 -3ú = - ê 2 0 -3ú = - A
ê ú ê ú
4q 2 + 2q 2 = 1 êë -3 3 0úû êë 3 -3 0úû
Þ 6q 2 = 1 Þ 2p 2 = 1 [using Eq. (iv)] Hence, A is a skew-symmetric matrix.
1 1
p 2 = Þ | p| = Note Every square matrix can be uniquely expressed as the sum of
2 2
symmetric and skew-symmetric matrix.
1 1
Special Types of Matrices i.e. A = ( A + A¢) + ( A - A¢)
2 2
Nilpotent Matrix
Orthogonal Matrix
If Ak = O and Ak + 1 ¹ O, where k is a positive integer and
If the product of a square matrix and its transpose A¢ is
O is a null matrix, then A is called nilpotent matrix, k is an identity matrix, then matrix A is said to be an
called the index of the nilpotent matrix A. orthogonal matrix.
Periodic Matrix i.e. AA¢ = I = A¢ A
If Ak + 1 = A, where k is a positive integer, then A is known 1 é 2 -2ù
e.g. Let A=
as periodic matrix, k is known as period of matrix A. 2 2 êë 2 2 úû
For k = 1 , A2 = A. 1 é 2 2ù
\ A¢ =
2 2 êë -2 2úû
Idempotent Matrix
If A2 = A, then square matrix A is known as idempotent 1 1 é 2 -2ù é 2 2ù
Also, AA¢ = ×
matrix. 2 2 2 2 êë 2 2úû êë -2 2úû
1 é 8 0ù é 1 0ù
Involutory Matrix = ê =
8 ë 0 8úû êë 0 1 úû
If A2 = I , where I is an identity matrix, then A is called
an involutory matrix. Þ AA¢ = I
Similarly, A¢ A = I
Symmetric Matrix Thus, A is an orthogonal matrix.
If for a square matrix A, A¢ = A, then A is known as If A and B are two orthogonal matrices, then AB will also
symmetric matrix. be an orthogonal matrix.
Matrices 537

Example 8. If A, B and A + B are idempotent matrices, then é2 5 ù


Þ AT + BT = ê ú … (ii)
AB is equal to ë3 - 1û
(a) BA (b) - BA (c) I (d) O Given, AT = A and BT = - B

Sol. (b) Since, A, B and A + B are idempotent matrix é2 5 ù


Þ A-B= ê ú
\ A2 = A; B2 = B and ( A + B) 2 = A + B ë3 - 1û
Now, consider ( A + B) 2 = A + B On solving Eqs. (i) and (ii), we get
Þ A2 + B2 + AB + BA = A + B é2 4 ù é0 - 1ù
A=ê ú and B = ê 1 0 ú
ë 4 - 1û ë û
Þ A + B + AB + BA = A + B Þ AB = - BA
é 2 4 ù é0 - 1ù é 4 - 2 ù
é ab b2 ù So, AB = ê úê ú=ê ú
Example 9. If A = ê 2 ú , then A is a/an ë 4 - 1û ë 1 0 û ë - 1 - 4û
ë- a - ab û (JEE Main 2021)
é cos a sin a ù
(a) idempotent matrix (b) nilpotent matrix Example 12. If A = ê , then the matrix A is
(c) involutory matrix (d) None of these ë - sin a cos a úû

é ab (a) symmetric matrix


b2 ù
Sol. (b) Since, A = ê 2 ú (b) skew-symmetric matrix
ë - a - abû (c) identity matrix
é ab b 2 ù é ab b2 ù (d) orthogonal matrix
\ A2 = ê 2 úê 2 ú
ë - a - abû ë - a - abû é cos a sin a ù
Sol. (d) Here, A=ê ú
é0 0 ù ë - sin a cos a û
Þ A2 = ê ú =O ¢
ë0 0 û é cos a sin a ù é cos a - sin a ù
Þ A¢ = ê ú =ê ú
Thus, A is a nilpotent matrix of order 2. ë - sin a cos a û ë sin a cos a û
¢
é cos a - sin a ù é cos a sin a ù
Example 10. If A and B are symmetric matrices of the same \ A¢ A = ê ú ê ú
order and X = AB + BA and Y = AB - BA, then ( XY )T is equal to ë sin a cos a û ë - sin a cos a û
é (cos a) (cos a) + (sin a) (sin a)
(a) XY (b) YX =ê
(c) - YX (d) None of these ë(sin a) (cos a) + (cos a) ( - sin a)
(cos a) (sin a) + ( - sin a) (cos a) ù
Sol. (c) Since, A and B are symmetric matrices
sin 2 a + cos2 a ú
û
\ X = AB + BA
é 2
cos a + sin a 2
sin a × cos a - sin a × cos a ù
will be a symmetric matrix and Y = AB - BA will be a =ê ú
skew-symmetric matrix. ë sin a × cos a - sin a × cos a cos2 a + sin 2 a û
Thus, we get X T = X and Y T = - Y é 1 0 ù
=ê ú =I [Q sin 2 q + cos2 q = 1 ]
Now, consider ( XY)T = Y T X T = ( -Y) ( X) = - YX ë0 1û
So, A is an orthogonal matrix.
Example 11. If A is a symmetric matrix and B is a
é2 3 ù
skew-symmetric matrix such that A + B = ê
Elementary Row Transformations
ú, then AB is
ë5 -1û Any one of the following operations on a matrix is called
equal to (JEE Main 2019) an elementary row (or column) transformation.
é -4 -2 ù é 4 -2 ù é 4 -2 ù é -4 2 ù (i) Interchanging any two rows (or columns). This
(a) ê ú (b) ê ú (c) ê ú (d) ê ú transformation is indicated by
ë -1 4 û ë -1 -4 û ë 1 -4 û ë 1 4û
Ri « R j ( or Ci « C j )
Sol. (b) Given matrix A is a symmetric and matrix B is a
(ii) Multiplication of the elements of any row (or
skew-symmetric.
column) by a non-zero scalar quantity. This
\ AT = A and BT = - B
transformation is indicated as
é2 3 ù Ri « kRi ( or Ci « kCi )
Since, A + B = ê ú (given) … (i)
ë5 - 1û (iii) Addition of a constant multiple of the elements of
On taking transpose both sides, we get any row to the corresponding element of any
T
é2 3 ù other row. This transformation is indicated as
( A + B)T = ê ú
ë5 - 1û Ri ® Ri + kR j .
538 JEE Main Mathematics

é 2 -3 3 ù é 1 0 0 ù
Elementary Matrix ê 2 2 3ú = ê0 1 0 ú A
\
A matrix obtained from an identity matrix by a single ê ú ê ú
elementary operation is called an elementary matrix. êë 3 -2 2úû êë0 0 1úû
é 1 1 4ù é 1 1 -1ù
Inverse of a Matrix using Elementary Þ ê2 2 3 ú = ê0 1 0 ú A [using R ® R + R - R ]
Row Transformation ê ú ê ú 1 1 2 3
êë3 -2 2 úû êë0 0 1 úû
Let A = IA
é1 1 4 ù é 1 1 -1ù
If matrix A (LHS) is reduced to I by elementary row ê
Þ 0 0 -5 ú = ê -2 - 1 2 ú A [using R2 « R3 ]
transformation, then suppose I (RHS) is reduced to P and ê ú ê ú
not change A in RHS i.e., after transformation, we get êë0 -5 -10 úû êë -3 -3 4 úû
I = PA, then P is the inverse of A,
é 1 1 4ù é 1 1 -1 ù
\ P = A-1 ê0 1 2 ú = ê3 / 5
Þ 3 / 5 -4 / 5 ú A
ê ú ê ú
é2 -3 3 ù êë0 0 1úû êë2 / 5 1 / 5 -2 / 5úû
Example 13. The inverse of A = ê2 2 3 ú by elementary 1 -1
ê ú [using R2 ® - R2 and R3 ® R3 ]
êë 3 -2 2 úû 5 5
row transformation is
é 1 1 0 ù é -3 / 5 1 / 5 3 / 5 ù
é 2 0 -3 ù Þ ê 0 1 0 ú = ê -1 / 5 1 / 5 0 ú A
(a) ê 1 -1 0 ú ê ú ê ú
ê ú êë0 0 1úû ëê 2 / 5 1/ 5 -2 / 5úû
êë -2 -1 2 úû
[using R2 ® R2 - 2R3 and R1 ® R1 - 4R3 ]
é 2 0 3ù
1
(b) ê 1 - 1 0 ú é 1 0 0 ù é -2 / 5 0 3 /5 ù
5ê ú ê 0 1 0 ú = ê -1 / 5 1 / 5 0 ú A
êë -2 - 1 2 úû ê ú ê ú
é 2 0 - 3ù êë0 0 1úû ëê 2 / 5 1 / 5 -2 / 5úû

(c) - 1 -1 0 ú [using R1 ® R1 - R2]
5ê ú
êë -2 -1 2 úû é 2 0 -3 ù

(d) None of the above Þ I3 = - 1 -1 0 ú A
5ê ú
é 2 -3 3 ù êë -2 - 1 2 úû
Sol. (c) Let A = ê 2 2 3ú .
ê ú é 2 0 -3 ù
êë 3 - 2 2úû 1ê
\ A- 1 = - 1 -1 0 ú [Q AA-1 = I3 ]
We know that, A = IA 5ê ú
êë -2 - 1 2 úû
Practice Exercise
ROUND I Topically Divided Problems
é x ù
Types of Algebra of Matrix sin - 1 ( x p) tan - 1 æç ö÷ ú
1ê èpø
1. The non-zero values of x satisfying the matrix 7. If A = ê ú,
p ê sin - 1 æ x ö cot - 1 ( px) ú
é2x 2ù é8 5 x ù é ( x 2 + 8) 24 ù ç ÷
ë èpø û
equation x ê ú +2ê ú =2ê ú is
ë 3 xû ë 4 4 xû ë (10) 6x û é -1 - 1æ x ö ù
- cos ( x p) tan ç ÷ ú
1ê èpø
(a) 1 (b) 2 (c) 3 (d) 4 B= ê ú,
p ê sin - 1 æ x ö -1
- tan ( px) ú
éa 0 ù é1 0 ù ç ÷
èpø
2. If A = ê ú and B = ê ú , then value of a for ë û
ë 1 1û ë5 1 û
which A 2 = B is then A - B is equal to
1
(a) 1 (b) - 1 (a) I (b) O (c) 2I (d) I
(c) 4 (d) No real values 2

æ cos a - sin a ö 8. If matrix A = [ aij ]2 ´ 2 , where aij = 1 ,if i ¹ j and


3. Let A = ç ÷, (a Î R) such that
èsin a cos a ø aij = 0 , if i = j, then A 2 is equal to
æ0 -1ö (a) I (b) A
A 32 =ç ÷. Then, a value of a is
è1 0 ø (JEE Main 2019, 8 April)
(c) O (d) None of these
p 9. If A is a square matrix such that A 2 = I , then
(a) (b) 0
32
p p ( A - I ) 3 + ( A + I ) 3 - 7 A is equal to
(c) (d) (a) A (b) I - A
64 16
(c) I + A (d) 3A
4. The minimum number of zero’s in an upper
é0 1ù é0 -1ù
triangular matrix of order n ´ n, is 10. If A = ê ú and B = ê 1 , then which of the
ë 1 1û ë 0 úû
n (n - 1) n (n + 1)
(a) (b)
2 2 following is correct?
n (n 2 - 1) (a) ( A + B) × ( A - B) ¹ A 2 - B2
(c) (d) None of these
2 (b) ( A + B) × ( A - B) = A 2 - B2
é0 2ù é 0 3a ù (c) ( A + B) × ( A - B) = I
5. If A = ê ú and kA = ê ú , then the values (d) None of the above
ë3 -4 û ë 2 b 24 û
of k, a , b are respectively 11. If A is square matrix such that A 2 = A, then
(a) - 6, - 12 , - 18 ( A + I ) 3 is equal to
(b) - 6, 4, 9 (a) A + 1 (b) 7 A + I
(c) - 6, - 4, - 9 (c) 3 A + I (d) A - I
(d) - 6, 12 , 18 é 0 aù
- tan
ê 2 ú and I is the identity matrix
6. If A = [ aij ]n ´ n be a diagonal matrix with diagonal 12. If A = ê ú
a
element all different and B = [ bij ]n ´ n be some êë tan 2 0 úû
another matrix. Let AB = [ cij ]n ´ n , then cij is equal é cos a - sin a ù
of order 2, then ( I - A) ê is equal to
to ësin a cos a úû
(a) a jj bij (b) aii bij (a) A (b) I
(c) aij bij (d) aij b ji
(c) I + A (d) None of these
540 JEE Main Mathematics

13. If w ¹ 1 is the complex cube root of unity and matrix é 1 2 0ù


éw 0ù 21. Let A + 2 B = 6 - 3 3 ú and
ê
H=ê 70 ê ú
ú, then H is equal to êë - 5 3 1 úû
ë 0 wû
(a) H (b) 0 (c) -H (d) H 2 é2 - 1 5ù
2 A - B = ê 2 - 1 6 ú. If tr ( A) denotes the sum of
14. For each real number x such that - 1 < x < 1, let ê ú
êë 0 1 2 úû
é 1 -x ù
ê1 - x 1 - xú x+ y all diagonal elements of the matrix A, then
A( x ) = ê ú and z = . Then,
ê -x 1 ú 1 + xy tr ( A) - tr ( B) has value equal to (JEE Main 2021)
êë 1 - x 1 - x ûú (a) 1 (b) 2
(a) A (z ) = A (x) + A ( y) (b) A (z ) = A (x) [ A ( y)]-1 (c) 0 (d) 3
(c) A (z ) = A (x) × A ( y) (d) A (z ) = A (x) - A ( y) é a bù
22. If A = ê ú satisfies the equation
15. If A = diag {2, -1 , 3 } and B = diag { -1 , 3 , 2 }, ë c dû
then A 2 B is x 2 - ( a + d) x + k = 0, then
(a) diag {5 , 4 , 11 } (b) diag { -4 , 3 , 18 } (a) k = bc (b) k = ad
(c) diag {3 , 1 , 8 } (d) B (c) k = a 2 + b2 + c2 + d 2 (d) k = ad - bc
é7 0 ù é3 0 ù é cos q sin q ù
16. If X + Y = ê ú and X - Y = ê ú, then the sum 23. If A = ê n
ë2 5û ë0 3 û ú , then A is equal to
ë - sin q cos q û
of the elements of the matrix 3 X - 4 Y is equal to é - cos n q sin n qù é sin n q sin n qù
(a) ê ú (b) ê ú
(a) 14 (b) 16 (c) 12 (d) 25 ë - sin n q sin n qû ë - sin n q sin n qû
é i 0 - iù é- i i ù ésin n q - sin n qù
(c) ê (d) None of these
ê ú
17. If P = 0 - i i and Q = ê 0 0 ú , then PQ is ësin n q sin n q û
ú
ê ú ê ú
êë - i i 0 úû êë i - i úû
é 1 0 -1ù
equal to 24. If A = ê 2 1 3 ú , then A 2 + A is equal to
é -2 2 ù é 2 -2 ù é1 0 0ù ê ú
(a) ê 1 -1ú (b) ê -1 1 ú (c) é 2 -2ù (d) ê0 1 0ú êë 0 1 1 úû
ê ú ê ú ê -1 1úû ê ú
ë (a) A + I (b) A 2 + I
êë 1 -1úû êë -1 1 úû êë0 0 1úû
(c) A ( A + I ) (d) None of these
18. A square matrix P satisfies P 2 = I - P ,where I is the (where I is 3 ´ 3 unit matrix)
identity matrix. If P n = 5 I - 8 P , then n is equal to 25. If A and B are square matrices of the same order
(a) 4 (b) 5 (c) 6 (d) 7 such that AB = BA, then AB n is equal to
é cos q 2
sin q cos q ù (a) A nB (b) ABn
19. Let A = ê ú (c) AB (d) Bn A
ë cos q sin q sin 2 q û
é cos2 f sin f cos f ù 26. Elements of a matrix A of order 10 ´ 10 are defined
and B = ê ú , then AB = O, if
ë cos f sin f sin 2 f û as aij = wi + j (where, w is cube root of unity), then
(a) q = nf , n = 0, 1, 2 ,... tr ( A) of the matrix is
(b) q + f = np , n = 0, 1, 2 ,... (a) 0 (b) w
p (c) 3 (d) None of these
(c) q = f + (2 n + 1) , n = 0, 1, 2 , K
2
p Transpose and Special
(d) q = f + n , n = 0, 1, 2 ,...
2 Types of Matrix
é1 0 ù é1 0 ù é 0 2y 1 ù
20. If A = ê ú and I = ê0 1 ú , then which one of the
ë1 1 û ë û 27. The total number of matrices A = ê 2 x y -1ú ,
ê ú
following holds for all n ³ 1, by the principle of êë 2 x - y 1 úû
Mathematical Induction ( x, y Î R, x ¹ y) for which A T A = 3I 3 is(JEE Main 2019)
(a) A n = nA + (n - 1)I (b) A n = 2n- 1 A + (n - 1)I (a) 2 (b) 4
(c) A n = nA - (n - 1)I (d) A n = 2n - 1 A - (n - 1)I (c) 3 (d) 6
Matrices 541

é 3/ 2 1/ 2 ù é 1 1ù T é 1 2 3ù
28. If P = ê ú , A = ê0 1ú and Q = PAP and
ë -1 / 2 3 / 2û ë û 35. The matrix 1 2 3ú is
ê
ê ú
X = P T Q 2005 P , then X is êë -1 -2 -3ûú
é1 2005ù (a) idempotent (b) nilpotent
(a) ê
ë0 1 úû (c) involutary (d) orthogonal
é4 + 2005 3 6015ù é1 2 2ù
(b) ê ú
ë2005 4 - 2005 3û 36. If A = ê 2 1 - 2 ú and A T A = AA T = I , then xy is
ê ú
é1 0ù êë x 2 y úû
(c) ê ú
ë 2005 1 û equal to
é1 0ù (a) -1 (b) 1 (c) 2 (d) - 2
(d) ê ú
ë0 1û 37. If A and B are symmetric matrices of the same
29. Let M = [ auv ]n ´ n be a matrix, where order and X = AB + BA and Y = AB - BA, then
auv = sin(q u - q v ) + i cos (q u - q v ), ( XY ) T is equal to
then M is equal to (a) XY (b) YX
(a) M (b) - M (c) M T
(d) - M T (c) - YX (d) None of these

30. If A and B are matrices of same order, then 38. An orthogonal matrix is
( AB ¢ - BA ¢ ) is a é cos a 2 sin a ù é cos a sin a ù
(a) ê (b) ê
(a) skew-symmetric matrix (b) null matrix ë -2 sin a cos a úû ú
ë - sin a cos a û
(c) symmetric matrix (d) unit matrix é cos a sin a ù é1 1ù
(c) ê ú (d) ê ú
é 1 1 ù ë sin a cos a û ë1 1û
ê 2 ú is
31. The matrix A = ê 2 é cos q - sin q ù
1 1 ú 39. If A = ê , then A T + A = I 2 , if
ê- - ú ësin q cos q úû
ë 2 2û
(a) unitary (b) orthogonal (a) q = np , n Î Z (b) q = (2n + 1) p /2, n Î Z
(c) nilpotent (d) involutory (c) q = 2np + p /3, n Î Z (d) None of these
é6 8 5 ù é1 -aù
40. If for the matrix, A = ê , AA T = I 2 , then the
32. If A = ê 4 2 3 ú is the sum of a symmetric matrix ëa b úû
ê ú
êë 9 7 1 úû value of a 4 + b 4 is (JEE Main 2021)

B and skew-symmetric matrix, C, then B is (a) 1 (b) 3 (c) 2 (d) 4


é 6 6 7ù é 0 2 -2 ù 41. A and B are two square matrices such that
(a) ê 6 2 5 ú (b) ê -2 5 -2ú A 2 B = BA and if ( AB)10 = A kB10 . Then, k is
ê ú ê ú
êë 7 5 1 úû êë 2 2 0 úû (a) 1001 (b) 1023
(c) 1042 (d) None of these
é 6 6 7ù é 0 6 -2 ù
(c) ê -6 2 -5ú (d) ê 2 0 -2 ú 42. If A and B are two square matrices such that
ê ú ê ú AB = B and BA = A , then for n Î N ( A + B) n is
êë -7 5 1 úû êë -2 -2 0 úû
(a) 2n ( A + B) (b) 2n-1 ( A + B)
é0 0 1 ù é1 0 0 ù (c) 2n + 1 ( A + B) (d) 2n/ 2 ( A + B)
33. If E = ê0 0 1 ú and F = ê0 1 0 ú, then éa bù
ê ú ê ú 43. Let A = ê
êë0 0 0 úû êë0 0 1 úû ú be a matrix such that
ë c dû
E2 F + F 2E a + d = 1 = ad - bc, then A 2010 is
(a) F (b) E (a) I 2 (b) -I 2
(c) 0 (d) None of these (c) A (d) null matrix

34. If A and B are two invertible matrices and both are 44. Let A, B, are square matrices of same order
symmetric and commute each other, then satisfying AB = A and BA = B , then
(a) both A -1B and A -1B-1 are symmetric ( A 2010 + B 2010 ) 2011 equals
(b) neither A -1B nor A -1B-1 are symmetric (a) A + B (b) 2010 ( A + B)
(c) A -1B is symmetric but A -1B-1 is not symmetric (c) 2011 ( A + B) (d) 22011 ( A + B)
(d) A -1B-1 is symmetric but A -1B is not symmetric
542 JEE Main Mathematics

ROUND II Mixed Bag


é1 0 0ù 8. Matrix A such that A 2 = 2 A - I , where I is the
1. Let P = ê 3 1 0 ú and Q = [ qij ] be two 3 ´ 3 matrices identity matrix. Then, for n ³ 2 , A n is equal to
ê ú
êë 9 3 1 úû (a) nA - (n - 1) I
q + q31 (b) nA - I
such that Q - P 5 = I 3. Then, 21 is equal to
q32 (c) 2n - 1 A - (n - 1) I
(JEE Main 2019) (d) 2n - 1 A - I
(a) 10 (b) 135 (c) 9 (d) 15 é 2 1 ù 2
9. If A = ê ú , then I + 2 A + 3 A + . . . ¥
é1 2 2 ù ë - 4 - 2 û
2. If A = ê 2 1 -2 ú is a matrix satisfying the é 4 1ù é 3 1ù
ê ú (a) ê ú (b) ê
êë a 2 b úû ë - 4 0û ë - 4 - 1úû
equation AA T = 9 I , where I is 3 ´ 3 identity matrix, é 5 2ù é 5 2ù
(c) ê ú (d) ê ú
then the ordered pair ( a, b) is equal to (JEE Main 2015) ë -8 -3û ë - 3 –8 û
(a) (2, - 1) (b) (-2, 1) é1 0 0ù
(c) (2, 1) (d) (-2, - 1) 10. Let A = ê 2 1 0 ú , if u1 and u2 are column matrices
ê ú
é 2 -1ù é 4 1ù T T êë 3 2 1 úû
3. If A = ê ú and B = ê 7 2 ú , then B A is
ë - 7 4 û ë û é 1ù é0 ù
(a) null matrix such that Au1 = ê0 ú and Au2 = ê 1 ú, then u1 + u2 is
ê ú ê ú
(b) an identity matrix êë0 úû êë0 úû
(c) scalar but not an identity matrix
equal to
(d) such that Tr (BT AT ) = 4
é - 1ù é - 1ù
é0 -1ù 16 (a) ê 1ú (b) ê 1ú
4. If A = ê ú and A = lI , then the value of l is ê ú ê ú
ë 1 0 û êë 0úû êë - 1úû
(a) 2 (b) 1 (c) 0 (d) 3 é - 1ù é 1ù
é i - iù (c) ê - 1ú (d) ê - 1ú
5. Let A = ê ê ú ê ú
ú, i = - 1, then the system of linear êë - 1úû
ë- i i û êë 0úû
é xù é 8 ù
equations A 8 ê ú = ê ú has 11. If M is a 3 ´ 3 matrix satisfying
ë y û ë64 û (JEE Main 2021) é0 ù é - 1ù é 1ù é 1ù é1ù é 0 ù
(a) a unique solution M ê 1 ú = ê 2 ú, M ê - 1ú = ê 1ú and M ê1ú = ê 0 ú
(b) Infinitely many solution ê ú ê ú ê ú ê ú ê ú ê ú
êë0 úû êë 3úû êë 0 úû êë - 1úû êë1úû êë12 úû
(c) No solution
(d) Exactly two solutions Then, the sum of the diagonal entries of M is
6. Let A is a 3 ´ 3 matrix and A = [ aij ]3´ 3. If for every (a) 9 (b) 8 (c) 10 (d) 11
é1 0 0ù
column matrix X, if X ¢ AX = 0 and a23 = - 2009,
then a32 = . . . 12. If A = ê 0 1 0 ú and I is the unit matrix of order
ê ú
(a) 2009 (b) -2009 êë a b - 1úû
(c) 0 (d) 2008 3, then A 2 + 2 A 4 + 4 A 6 is equal to
é -1 + i 3 -1 - i 3 ù (a) 7 A 8 (b) 7 A7 (c) 8I (d) 6I
ê ú éw 0ù
7. If A = ê 2 i 2i
ú , i = -1 and 13. If w is a complex cube root of unity and A = ê ú,
ê 1+ i 3 1- i 3 ú ë 0 wû
ë 2i 2i û then A 50 is equal to
2
f ( x) = x + 2, then f ( A) is equal to (a) w2A (b) w A (c) A (d) 0
æ 5 - i 3 ö é1 0ù æ 3 - i 3 ö é1 0ù é3 3 3ù
(a) ç ÷ê ú (b) ç ÷ê
è 2 ø ë0 1û è 2 ø ë0 1úû 14. If A = ê3 3 3ú , then A 4 is equal to
ê ú
é1 0ù é1 0ù êë3 3 3úû
(c) ê ú (d) (2 + i 3 ) ê
ë0 1û ë0 1úû
(a) 27 A (b) 81 A (c) 243 A (d) 729 A
Matrices 543

é 1 -1ù é 1 aù (a) symmetric


15. If A = ê ú and B = ê 4 b ú and
ë 2 - 1û ë û (b) skew-symmetric
2 2 2 (c) neither symmetric nor skew-symmetric
( A + B) = A + B . Then, a and b are respectively
(d) data not adequate
(a) 1, –1 (b) 2, –3 (c) –1, 1 (d) 3, –2
16. If A is a skew-symmetric matrix of order n and C is Numerical Value Type Questions
a column matrix of order n ´ 1, then C AC is T é x 1ù 4
20. Let A = ê ú, x Î R and A = [ aij ]. If a11 = 109,
(a) an identity matrix of order n ë1 0 û
(b) an identity matrix of order 1 then a22 is equal to .......... . (JEE Main 2020)
(c) a zero matrix of order 1
(d) None of the above 21. The number of all 3 ´ 3 matrices A, with enteries
17. Let A, B, C, D be (not necessarily square) real from the set { - 1, 0, 1} such that the sum of the
diagonal elements of AA T is 3, is ……… .
matrices such that
(JEE Main 2020)
A T = BCD, B T = CDA, C T = DAB, D T = ABC
éa b cù
for the matrix S = ABCD. Consider the two
22. If A = ê b c a ú, where a, b, c are real positive
statements. ê ú
êë c a b úû
I. S 3 = S II. S 2 = S 4
numbers, abc = 1 and A T A = I , then find the value
(a) II is true but not I (b) I is true but not II
of a 3 + b3 + c3 is ……… .
(c) Both I and II are true (d) Both I and II are false
23. Let M be any 3 ´ 3 matrix with entries from the set
18. Which of the following is an orthogonal matrix?
é 6 2 -3 ù é6 2 3 ù {0, 1, 2 }. The maximum number of such matrices, for
1ê 1ê
(a) 2 3 6ú (b) 2 -3 6 ú which the sum of diagonal elements of M T M is
7ê ú 7ê ú
êë3 -6 2 úû êë3 6 - 2úû seven, is (JEE Main 2021)

é -6 -2 -3 ù é 6 -2 3 ù é 1ù é -1ù
1ê 1ê 24. If A is 2 ´ 2 matrix such that A ê ú = ê ú
(c) 2 3 6ú (d) 2 2 -3 ú ë -1û ë 2 û
7ê ú 7ê ú
êë -3 6 2 úû êë -6 2 3 úû é 1ù é 1 ù
and A 2 ê ú = ê ú, the sum of the elements of A
19. If A1, A3, . . . , A2 n - 1 are n skew-symmetric matrices ë -1û ë0 û
n is ……… .
of same order, then B = å ( 2 r - 1) ( A2 r - 1) 2 r - 1 will
r =1

be

Round I Answers
1. (d) 2. (d) 3. (c) 4. (a) 5. (c) 6. (b) 7. (d) 8. (a) 9. (a) 10. (a)
11. (b) 12. (c) 13. (a) 14. (c) 15. (b) 16. (a) 17. (b) 18. (c) 19. (c) 20. (c)
21. (b) 22. (d) 23. (d) 24. (c) 25. (d) 26. (d) 27. (b) 28. (a) 29. (b) 30. (a)
31. (c) 32. (a) 33. (b) 34. (c) 35. (b) 36. (c) 37. (c) 38. (b) 39. (c) 40. (a)
41. (b) 42. (b) 43. (a) 44. (d)

Round II
1. (a) 2. (d) 3. (b) 4. (b) 5. (c) 6. (a) 7. (d) 8. (a) 9. (c) 10. (d)
11. (a) 12. (a) 13. (b) 14. (d) 15. (a) 16. (c) 17. (c) 18. (a) 19. (b) 20. (10)
21. (672) 22. (4) 23. (540) 24. (5)
544 JEE Main Mathematics

Solutions
Round I é 0 3a ù
5. Q kA = ê
1. Given that, ë2 b 24 úû
é2x 2ù é8 5xù é (x2 + 8) 24ù é0 2 ù é 0 3a ù
xê +2ê =2 ê Þ kê ú=ê
ë 3 xúû ë4
ú
4xû ë 10
ú
6xû ë 3 -4 û ë 2 b 24 úû
é 2 x2 2x ù é16 10xù é (2x2 + 16) 48ù é0 2k ù é 0 3a ù
Þ +ê Þ ê 3 k -4 k ú = ê 2 b
ê 2ú 8 8 xú= ê 12
ú
xû ë û ë 24 úû
ë 3x x û ë û ë 20
é2x2 + 16 2x + 10xù é2x2 + 16 48ù Þ 2k = 3a , 3k = 2b, - 4k = 24
Þ ê 2 ú =ê ú 2k 3k
ë3 x + 8 x + 8x û ë 20 12xû Þ a= , b= , k = -6
3 2
By equality of two matrices, \ a = - 4, b = - 9, k = - 6
3x + 8 = 20 Þ 3x = 12 Þ x = 4 n

é a 0ù
6. cij = å aik bkj (in general)
2. Given that, A = ê ú
k =1
ë 1 1û And in a diagonal matrix non-diagonal elements are
é a 0ù é a 0ù é a 2 + 0 0 + 0ù é a 2 0ù
2 ì 0, if i ¹ j
Þ A =ê úê ú=ê ú=ê ú zero. i.e. a ij = í
ë 1 1û ë 1 1û ë a + 1 0 + 1û ë a + 1 1û î a ii, if i = j
Also, B = A2 (given) So, cij = a ii b ij
é1 0ù é a 2 0ù é æ x öù
Þ sin -1 (px) tan -1 ç ÷ ú
ê5 1ú = ê ú 1ê èpø
ë û ë a + 1 1û 7. Given that, A = ê ú
p ê -1 æ x ö
Clearly, this is not satisfied by any real value of a. sin ç ÷ cot-1 (px) ú
ëê èpø úû
é cos a - sin a ù é æ xö ù
3. Given, matrix A = ê ú - cos -1 (px) tan -1 ç ÷ ú
ë sin a cos a û 1ê èpø
and B= ê ú,
pê -1 æ x ö
é cos a - sin a ù é cos a - sin a ù sin ç ÷ - tan (px)ú -1
\ A2 = ê ú ê sin a cos a ú ëê èpø ûú
ë sin a cos a û ë û
é æ x öù
é cos2 a - sin 2 a - cos a sin a - sin a cos aù sin -1 (px) tan -1 ç ÷ ú
=ê ú 1ê èpø
ë sin a cos a + cos a sin a - sin 2 a + cos2 a û then A-B = ê ú
p ê -1 æ x ö
é cos 2 a - sin 2 a ù sin ç ÷ cot (px) ú
-1

=ê êë èpø ûú
ú
ë sin 2 a cos 2 a û
é æ xö ù
é cos(na ) - sin(na )ù - cos -1 (px) tan -1 ç ÷ ú
Similarly, A n = ê ú, n Î N 1ê èpø
ë sin(na ) cos(na ) û - ê ú
pê æ x ö
sin -1 ç ÷ - tan -1 (px)ú
é cos(32 a ) - sin(32 a )ù é0 -1ù êë èpø úû
Þ A32 = ê ú=ê ú (given)
ë sin(32 a ) cos(32 a ) û ë1 0 û é æ xö æ x öù
sin -1 (px) + cos -1 (px) tan -1 ç ÷ - tan -1 ç ÷ ú
So, cos(32 a ) = 0 and sin(32 a ) = 1 1ê èpø èpø
= ê ú
p p p ê -1 æ x ö -1 æ x ö
Þ 32 a = Þ a = sin ç ÷ - sin ç ÷ cot (px) + tan (px) ú
-1 -1
2 64 êë èpø èpø úû
4. ép ù
The minimum number of zeros
Order 1 ê2 0ú
= ê
p 0 pú
in upper triangular matrix
ê ú
1 2 ´2 ë 2û
1+ 2 = 3 3´3 é -1 -1 p -1 -1 pù
êëQ sin x + cos x = 2 and tan x + cot x = 2 úû
1+ 2 + 3 = 6 4´4
1 + 2 + 3 + 4 = 10 5´5 é1 ù
ê 0 ú 1 é1 0ù 1
n (n - 1) n´n = ê2 = = I
1 + 2 + L + (n - 1) = 1 ú 2 êë0 1úû 2
2 ê0 ú
ë 2û
Matrices 545

ì1, if i ¹ j = I + A × A + 3 A (I + A ) [Q A 2 = A]
8. Given that, aij = í ,
î0, if i = j = I + A 2 + 3 ( A × I + A 2)
é1 0ù = I + A + 3 ( A + A) [Q A 2 = A]
then A = [aij ]2´ 2 = ê ú
ë0 1û = I + A + 3 (2 A )
= I + A + 6A = 7A + I
é1 0ù é1 0ù é1 + 0 0 + 0ù
\ A2 = A × A = ê ú×ê ú =ê ú é0 -t ù æ aö
ë0 1û ë0 1û ë0 + 0 0 + 1û 12. Here, A = ê ú , where t = tan çè 2 ÷ø
ë t 0 û
é1 0ù
=ê ú =I æ aö
ë0 1û 1 - tan 2 ç ÷
è 2 ø 1 - t2
Now, cos a = =
9. Given that, A2 = I …(i) æ a ö 1 + t2
1 + tan 2 ç ÷
è2ø
Now, we have
( A - I )3 + ( A + I )3 - 7 A æ aö
2 tan ç ÷
è2ø 2t
= ( A )3 - I3 - 3 AI ( A - I ) + ( A )3 + I3 and sin a = =
2 æ aö 1 + t2
+ 3 A × I(A + I ) - 7 A 1 + tan ç ÷
è2ø
3.
= 2( A ) - 3 A ( A - I ) + 3 A ( A + I ) - 7 A
é cos a - sin a ù
Now, we have (I - A ) ê
= 2 ( A2 × A ) - 3 A( A - I - A - I ) - 7 A
ë sin a cos a úû
= 2(I × A ) - 3 A (-2I ) - 7 A [from Eq. (i)] é 1 - t2 -2 t ù
= 2A + 6A× I - 7A é æ1 0ö æ 0 -t ö ù 1 + t 1 + t 2 ú
ê 2
= êç ÷-ç ÷ú ê ú
= 2A + 6A - 7A = 8A - 7A = A ë è0 1ø è + t 0ø û ê 2 t 1 - t2 ú
ê 1 + t2 1 + t2 ú
10. Given that, ë û
é0 1ù é0 - 1ù é 1 - t2 - 2t ù
A=ê ú and B = ê ê ú
ë1 1û ë1 0úû =ê
é 1 tù ê 1 + t2 1 + t2 ú
ú
é0 1ù é0 -1 ù é 0 0ù ë -t 1û ê 2t 1 - t2 ú
A+ B=ê + = ê 1 + t2 1 + t2 ú
ë1 1úû êë1 0úû êë2 1úû ë û
é 1 - t 2 + 2t 2 - 2t + t (1 - t 2) ù
é0 1ù é0 -1 ù é 0 2ù ê ú
A-B=ê - = 1 + t2 1 + t2
ë1 1úû êë1 0úû êë0 1úû =ê 2 2 2
ú
ê - t (1 - t ) + 2t 2t + 1 - t ú
é0 1ù é0 1ù ê 1 + t2 1 + t 2 úû
A2 = A × A = ê ë
ë1 1úû ê1
ë 1úû é 1 + t 2 - 2t + t - t3 ù
é0 + 1 0 + 1ù é1 1ù ê ú
1 + t2 1 + t2
=ê = =ê ú
ë0 + 1 1 + 1úû êë1 2úû ê - t + t3 + 2t 2t 2 + 1 - t 2 ú
ê 1 + t2 1 + t 2 úû
é0 - 1ù é0 - 1ù é0 -1 0 + 0ù ë
and B2 = B × B = ê . =
ë1 0úû êë1 0úû ê0 + 0
ë - 1 + 0úû é 1 + t 2 - t (1 + t 2) ù
ê ú
é-1 0ù 1 + t2 1 + t 2 ú é1 - t ù
=ê = … (i)

- 1úû
2
ê t (1 + t ) 1 + t ú êë t
2
1úû
ë 0 ê 1 + t2 ú
ë 1 + t2 û
é1 1 ù é -1 0ù é2 1ù
\ A 2 - B2 = ê - = é1 0ù é0 -t ù
2úû êë 0 - 1úû êë1 3úû I+ A=ê
ë1 Now, ú+ê ú
ë0 1û ë t 0û
é0 0ù é0 2ù
and ( A + B ) ( A - B ) = ê é0 + 1 - t + 0ù é1 -t ù
ë2 1úû êë0 1úû =ê = …(ii)
ët + 0 0 + 1úû êë t 1úû
é0 + 0 0 + 0ù é0 0ù On putting the value of t in both equations, we get
=ê =
ë0 + 0 4 + 1úû êë0 5úû é æ aöù é æ aöù
ê 1 - tan ç ÷ ú ê 1 - tan ç ÷ ú
Hence, ( A + B ) ( A - B ) ¹ A 2 - B2 è2ø è2ø
ê ú=ê ú
æ a
ê tan ç ÷ ö æ a ö
11. Given that, A 2 = A …(i) 1ú ê tan ç ÷ 1ú
êë è2ø úû êë è2ø úû
Now, we have (I + A )3 = (I )3 + ( A )3 + 3 I × A (I + A )
é cos a - sin a ù
= I + A 2. × A + 3 A (I + A ) \ (I - A ) ê ú =I+ A
ë sin a cos a û
[Q I3 = I , I × A = A ]
546 JEE Main Mathematics

é w 0 ù é w 0 ù é w2 0 ù 16. X + Y + X - Y = 2X
13. Clearly, H 2 = ê úê ú=ê 2ú
ë 0 wû ë 0 wû ë 0 w û é 7 0ù é3 0ù
\ 2X = (X + Y ) + (X - Y ) = ê ú + ê ú
éw 2
0ù éw 0ù éw 3
0ù ë 2 5û ë0 3û
H3 = ê ú ê 0 wú = ê ú
ë0 w2û ë û ë0 w3 û é 7 + 3 0 + 0ù é10 0ù 1 é10 0ù é5 0ù
=ê ú =ê ú \X= ê =
é w70 0 ù é w69 × w 0 ù ë 2 + 0 5 + 3û ë 2 8û 2 ë 2 8úû êë1 4úû
\ H 70 = ê 70 ú
=ê ú
ë 0 w û ë 0 69
w × wû é 7 0ù é3 0ù
Again, 2Y = (X + Y ) - (X - Y ) = ê ú -ê ú
é (w3 )23 × w 0 ù ë 2 5û ë0 3û
=ê 3 23 ú é 7 - 3 0 - 0ù é4 0ù
ë 0 (w ) × wû =ê ú =ê ú
ë 2 - 0 5 - 3û ë2 2û
éw 0ù
=ê ú=H [Q w3 = 1] 1 é4 0ù é2 0ù
ë 0 wû \ Y= ê =
2 ë2 2úû êë1 1úû
1 é 1 - xù
14. We have, A (x) = ê -x 1 ú …(i) é5 0ù é2 0ù é 7 0ù
1-x ë û 3X - 4Y = 3 ê ú - 4 ê 1 1 ú = ê -1 8 ú
ë 1 4 û ë û ë û
1 é 1 - yù
\ A ( y) = …(ii) Sum of element of 3X - 4Y = 7 + 0 - 1 + 8 = 14
1 - y êë - y 1 úû
(x + y) ù é i 0 - iù é- i iù
é
ê 1 - 17. Since, P = ê 0 - i i ú and Q = ê 0 0ú
1 1 + xy ú ê ú ê ú
and A (z ) = ê ú
(x + y) ê (x + y) ú êë - i i 0 úû êë i - i úû
1- - 1
1 + xy ëê 1 + xy úû é i 0 - iù é - i iù
é (x + y) ù ê
\ PQ = 0 - i i ú ê 0 0ú
ê 1 - ê úê ú
1 + xy 1 + xy ú êë - i i 0 úû êë i - i úû
= ê ú
1 + xy - x - y ê - (x + y ) ú
1 é - i 2 - i 2 i 2 + i 2 ù é 1 + 1 - 1 - 1 ù é 2 -2 ù
êë 1 + xy úû ê ú
(x + y) ù = ê i2 - i2 ú = ê - 1 1 ú = ê -1 1 ú
é ê ú ê ú
ê 1 - ê i 2
- i úû êë - 1
2
1 úû êë -1 1 úû
1 + xy 1 + xy ú ë
= ê ú
(1 - x) (1 - y) ê - (x + y) ú
1 18. Q P 3 = P (I - P ) (Q P 2 = I - P )
ëê 1 + xy úû
+ - + = PI - P 2 = PI - (I - P ) = P - I + P = 2 P - I
1 é 1 xy (x y)ù
= Now, P 4 = P × P3
(1 - x) (1 - y) êë - (x + y) 1 + xy úû
Þ P 4 = P (2 P - I )
…(iii)
Now, consider Þ P4 = 2 P2 - P
1 é 1 - xù é 1 - yù Þ P4 = 2 I - 2 P - P
A (x) × A ( y) = ×ê
(1 - x) (1 - y) ë - x 1 úû êë - y 1 úû Þ P 4 = 2I - 3P and P 5 = P (2I - 3P )
1 é 1 + xy - (x + y)ù Þ P5 = 2 P - 3(I - P )
= × …(iv)
(1 - x) (1 - y) êë - (x + y) 1 + xy úû Þ P5 = 5P - 3I
Also, P 6 = P (5 P - 3 I )
From Eqs. (iii) and (iv), we get
Þ P 6 = 5P 2 - 3P
A (z ) = A (x) × A ( y).
Þ P 6 = 5 (I - P ) - 3P
15. A = diag {2, –1, 3} and B = diag {–1, 3, 2}
Þ P 6 = 5I - 8P
é2 0 0ù é -1 0 0 ù So, n =6
Þ A = ê 0 -1 0 ú Þ B = ê 0 3 0 ú
ê ú ê ú Alternate Solution
êë0 0 3úû êë 0 0 2úû Q P n = 5 I - 8P = 5 (I - P ) - 3P
é4 0 0ù = P (5 P - 3 I ) [Q P 2 = I - P ]
Then, A = ê0 1 0ú
2
ê ú 2 2
= P (2 P - 3 P ) = P (2 I - 3 P )
êë0 0 9úû
= P 2 [2 (I - P ) - P ]
é 4 0 0 ù é -1 0 0 ù é -4 0 0 ù
= P 2 [2 P 2 - P ] = P3 [2 P - I ]
\ A 2B = ê0 1 0ú ê 0 3 0ú = ê 0 3 0 ú
ê úê ú ê ú = P 4 [I - P ] = P 4 × P 2 = P 6
êë0 0 9úû ëê 0 0 2úû êë 0 0 18úû
= diag {– 4, 3, 18} Þ n =6
Matrices 547

é cos 2 q sin q cos qù é cos 2 f sin f cos f ù é a 2 + bc ab + bd ù


19. AB = ê 2 ú ê ú =ê 2ú
ë cos q sin q sin q û ë cos f sin f sin 2 f û ë ac + cd bc + d û
é cos 2 q cos 2 f + sin q cos f cos q sin f é (a + d ) × a (a + d ) × b ù
=ê (a + d ) × A = ê ú
2 2
ë cos f cos q sin q + sin q sin f cos f ë (a + d ) × c (a + d ) × d û
é a 2 + ad ab + bd ù
cos 2 q sin f cos f + sin 2 f sin q cos qù =ê
ú 2ú
cos q sin q sin f cos f + sin 2 q sin 2 f û ë ac + cd ad + d û
é cos q cos f cos(q - f ) sin f cos q cos(q - f )ù Q x2 - (a + d ) x + k = 0
=ê ú é a 2 + bc ab + bd ù é a 2 + ad ab + bd ù é k 0 ù
ë sin q cos f cos(q - f ) sin q sin f cos(q - f ) û Þ ê -ê +ê
2ú 2ú ú
Q AB = O ë ac + cd bc + d û ë ac + cd ad + d û ë 0 kû
Þ cos (q - f ) = 0 é0 0ù
=ê ú
p ë0 0û
Þ cos (q - f ) = cos (2n + 1 )
2 é bc - ad + k 0 ù é0 0ù
p Þ ê ú = ê0 0ú
Þ q = (2n + 1 ) + f , where n = 0, 1, 2 ,... ë 0 bc - ad + k û ë û
2
Þ k = ad - bc
é1 0ù é1 0ù é1 0ù
20. A2 = ê = é cos q sin qù
ë1 1úû êë1 1úû êë2 1úû 23. Since, A = ê ú,
ë - sin q cos qû
é1 0ù é1 0ù é1 0ù
A3 = ê = \ A2 = A × A
ë2 1úû êë1 1úû êë3 1úû é cos q sin qù é cos q sin qù
=ê úê ú
é1 0ù ë - sin q cos qû ë - sin q cos qû
\ An = ê
ën 1úû é cos 2 q sin 2 qù
=ê ú
én 0ù én -1 0 ù ë - sin 2 q cos 2 qû
nA = ê ú , (n - 1 ) I = ê
ën nû ë n n - 1úû é cos n q sin n qù
Similarly, A n = ê ú
nA - (n - 1 )I = ê
é 1 0ù n ë - sin n q cos n qû
ú=A
ë n 1û é 1 0 -1 ù
é1 2 0ù 24. A = ê2 1 3 ú,
ê ú
21. A + 2B = ê 6 - 3 3ú …(i) êë0 1 1 úû
ê ú
êë - 5 3 1 úû é 1 0 -1 ù é 1 0 -1 ù
é 2 -1 5 ù A 2 = A ´ A = ê2 1 3 ú ê2 1 3 ú
ê úê ú
2 A - B = ê 2 -1 6 ú êë0 1 1 úû êë0 1 1 úû
ê ú
êë0 1 2úû é1 + 0 + 0 0 + 0 + (-1) -1 + 0 + (-1)ù
é4 -2 10ù = ê2 + 2 + 0 0+1+3 -2 + 3 + 3 ú
ê ú
4 A - 2B = ê4 -2 12ú …(ii) êë0 + 2 + 0 0+1+1 0 + 3 + 1 úû
ê ú
êë0 2 4 úû é1 -1 -2 ù
Adding Eqs. (i) and (ii), we get = ê4 4 4ú
ê ú
é5 0 10ù êë2 2 4 úû
5 A = ê 10 -5 15ú é1 0 -1 ù é 1 0 0 ù é 2 0 -1 ù
ê ú
êë -5 5 5 úû \ A + I = ê2 1 3 ú + ê0 1 0ú = ê2 2 3 ú
ê ú ê ú ê ú
é1 0 2ù é0 1 -1 ù êë0 1 1 úû êë0 0 1úû êë0 1 2 úû
A = ê 2 -1 3ú and B = ê 2 -1 0 ú é 1 -1 -2 ù é 1 0 -1 ù
ê ú ê ú
êë -1 1 1úû êë -2 1 0 úû A 2 + A = ê4 4 4 ú + ê2 1 3 ú
ê ú ê ú
Þ tr ( A ) = 1 - 1 + 1 = 1 êë2 2 4 úû êë0 1 1 úû
Þ tr (B) = 0 - 1 + 0 = - 1 é 1 + 1 -1 + 0 -2 + (-1)ù é2 -1 -3ù
\ tr ( A ) - tr (B) = 2 = ê4 + 2 4 + 1 4 + 3 ú = ê6 5 7ú
ê ú ê ú
éa bù êë2 + 0 2 + 1 4 + 1 úû êë2 3 5 úû
22. We have, A = ê
ëc d úû é 1 0 -1 ù é 2 0 -1 ù
éa bù éa bù A ( A + 1) = ê2 1 3 ú ê2 2 3 ú
Þ A2 = ê ê úê ú
ëc d úû êc
ë d úû êë0 1 1 úû êë0 1 2 úû
548 JEE Main Mathematics

25. Given that, AB = BA …(i) = [- sin (qu - qv ) - i cos (qu - qv )]


n n
We want to prove that AB = B A …(ii) = - [sin(qu - qv ) + i cos (qu - qv )] = - M
For n = 1, Eq. (ii) is obviously true. 30. Let P = AB¢ - BA ¢ be any matrix.
Let Eq. (ii) be true for a positive integer n = m.
Now, P ¢ = ( AB¢ - BA ¢ )¢ = ( AB)¢ - (BA ¢ )¢
i.e. ABm = Bm A …(iii)
m+ 1 m m = (B¢ )¢( A ¢ ) - ( A ¢ )¢ (B¢ ) [Q ( AB )¢ = B¢A ¢
Then, for n = m + 1, AB = A (B B) = ( AB )B
\ ( A ¢ )¢ = A ¢ ]
(associative law of matrix multiplication)
= - [ AB¢ - BA ¢] = - P
= (Bm A )B [using Eq. (iii)]
Q P¢ = - P
= Bm ( AB) = Bm (BA ) [using Eq. (i)]
P is a skew-symmetric matrix.
m m+ 1
= (B B) A = B A é 1 1 ù
Hence, by induction Eq. (ii) is true for all n Î N . ê 2 2ú
31. We have, A = ê
1 1 ú
26. tr ( A ) = (a11 + a 22 + . . . + a10a10 ) ê- - ú
ë 2 2û
= (w2 + w4 + w6 + . . . + w20 )
é 1 1 ù é 1 1 ù
= w2(1 + w2 + w4 + . . . + w18 ) ê
\ 2
A =ê 2 2ú ê 2 2ú
= w2[(1 + w2 + w) + . . . + (1 + w + w2) + 1] 1 1 ú ê 1 1 ú
ê- - ú ê- - ú
= w2 [0 + . . . + 0 + 1] = w2 ë 2 2û ë 2 2û
é 0 2y 1 ù é 1 1 1 1ù
- -
27. Given matrix A = ê2x y -1ú , (x, y Î R, x ¹ y) ê 2 2 ú = é0 0ù = O
ê ú =ê 2 2
1 1 1 1ú ê ú
êë2x - y 1 úû ê- + - + ú ë0 0û
ë 2 2 2 2û
for which AT A = 3I3
\ Matrix A is nilpotent.
é 0 2x 2x ù é 0 2 y 1 ù é3 0 0ù
ê2 y y - yú ê2x y -1ú = ê0 3 0ú é6 8 5ù
Þ
ê úê ú ê ú 32. Given, A = ê4 2 3ú
êë 1 -1 1 úû êë2x - y 1 úû êë0 0 3úû ê ú
êë9 7 1úû
é 8 x2 0 0 ù é 3 0 0 ù
ê ú ê A+ A¢
Þ 2 ú and symmetric matrix, B =
ê 0 6 y 0ú = ê0 3 0ú 2
ê 0 0 3úû êë0 0 3úû
ë ì é6 8 5ù é6 4 9 ùü é 6 6 7ù
1ï ï
Here, two matrices are equal, therefore equating the \ B = í ê4 2 3ú + ê8 2 7úý = ê 6 2 5 ú
2 ïê ú ê ú ê ú
corresponding elements, we get î êë9 7 1úû êë5 3 1 úûïþ êë 7 5 1 úû
8x2 = 3 and 6 y2 = 3
33. F is unit matrix Þ F 2 = F
3 1
Þ x=± and y = ± and E 2F + F 2E = E 2 + E
8 2
é0 0 1ù é0 0 1ù é0 0 0ù
Q There are 2 different values of x and y each.
Also, E 2 = ê0 0 1ú ´ ê0 0 1ú = ê0 0 0ú
So, 4 matrices are possible such that AT A = 3I3 . ê ú ê ú ê ú
êë0 0 0úû êë0 0 0úû êë0 0 0úû
28. Q 2 = Q × Q = PAPT × PAPT
\ E 2 + E = E.
= PA (PT × P ) APT = PA × I × APT = PA 2PT
Similarly, Q3 = PA3 PT
34. Consider, ( A -1B)T = BT ( A -1 )T
= BT ( AT )-1 = B A -1
Q 2005 = PA 2005 PT
[Q AT = A and BT = B]
Now, X = PTQ 2005 P
= A -1B [Q AB = BA Þ A -1 ( AB) A -1
= PT (PA 2005 PT )P
= A -1 (BA ) A -1 Þ BA -1 = A -1B]
= (PT P ) A 2005 (PT P ) -1
Þ A B is symmetric.
é1 2005ù
= I × A 2005 × I = A 2005 = ê Now, consider
ë0 1 úû T
( A -1B-1 )T = ((BA )-1 )T = (( AB)-1) [Q AB = BA ]
29. Given, M = [auv ]n ´ n = [sin (qu - qv ) + i cos (qu - qv )] -1 -1 T
= (B A ) = ( A ) (B ) -1 T -1 T

Þ M = [sin(qu - qv ) - i cos (qu - qv )] = ( AT )-1 (BT )-1 = A -1 B-1


-1 -1
Þ T
(M ) = [sin (qv - qu ) - i cos (qv - qu )] ÞA B is also symmetric.
Matrices 549

é 1 2 3ù é1 - a ù é 1 a ù é 1 + a 2 a - ab ù é1 0ù
40. ê =ê ú=
35. Let A=ê 1 2 3ú a
ë b úû êë - a b úû ë a - ab a 2 + b 2û êë0 1úû
ê ú
êë - 1 -2 - 3úû 1 + a2 = 1
é0 0 0ù a2 = 0
Then, A 2 = ê0 0 0ú a + b2 = 1
2
ê ú
êë0 0 0úû b2 = 1
a4 = 0
Hence, A is nilpotent matrix of index 2.
b4 = 1
36. Since, A is orthogonal, each row is orthogonal to the a + b4 = 1
4
other rows.
41. Here, ( AB) ( AB) = A (BA ) B = A ( A 2B) B = A3 B2
Þ R1 × R3 = 0
Now, ( AB) ( AB) ( AB) = ( A3 B2) AB
Þ x + 4 + 2y = 0
= A3 B2AB = A3 B (BA ) B
Also, R 2 × R3 = 0
= A3 B ( A 2B) B = A3 (BA ) × AB2
Þ 2x + 2 - 2 y = 0
= A3 ( A 2B) × AB
On solving, we get = A5 BAB2 = A5 × A 2B × B2
x = -2 , y = -1
= A7 × B3
\ xy = 2 n
-1
So, ( AB)n = A 2 × Bn
37. Since, A and B are symmetric matrices 10
-1
\ ( AB)10 = A 2 × B10 Þ k = 210 - 1 = 1023
\ X = AB + BA will be a symmetric matrix
and Y = AB - BA will be a skew-symmetric matrix. 42. We have, ( A + B)2 = A 2 + AB + BA + B2
Thus, we get X T = X and Y T = - Y éQ A 2 = BA × BA = B × B × A = B × A = A ù
ê ú
Now, consider (XY )T = Y T X T 2
êëand B = A B A B = A × AB = AB = Búû
= (-Y ) (X ) = - YX
= A+ B+ A+ B
T
é cos a sin a ù é cos a sin a ù = 2 ( A + B)
38. ê úê ú
ë - sin a cos a û ë - sin a cos a û Also, ( A + B)3 = ( A + B)2 × ( A + B)
é cos a sin a ù é cos a - sin a ù = 2 ( A + B) × ( A + B)
=ê úê
ë - sin a cos a û ë sin a cos a úû = 2 ( A + B )2 = 2 ´ 2 ( A + B )
= 4 ( A + B)
é cos 2 a + sin 2 a - cos a sin a + sin a cos a ù Hence, ( A + B )n = 2 n - 1 ( A + B )
=ê ú
ë - sin a cos a + cos a sin a sin 2 a + cos 2 a û é a bù é a bù
43. Here, A 2 = ê úê ú
é1 0ù ë c dû ë c dû
=ê ú=I
ë0 1û é a 2 + bc ab + bd ù
=ê 2ú
Q If A is an orthogonal matrix, then ë ac + cd bc + d û
AAT = I. é a 2 + ad - 1 b (a + d ) ù
=ê ú
é cos a sin a ù ë c (a + d ) d 2 + ad - 1û
\ ê - sin a cos a ú is an orthogonal matrix.
ë û é a (a + d ) - 1 b ù

é cos q - sin qù ë c d (a + d ) - 1úû
39. Given, A = ê ú
ë sin q cos q û éa - 1 b ù é a b ù é1 0ù
=ê = -
Q AT + A = I 2 ë c d - 1úû êë c d úû êë0 1úû
é cos q sin qù é cos q - sin qù é1 0ù \ A 2 = A - I Þ A3 = A 2 - A = ( A - I ) - A = - I
Þ ê - sin q cos qú + ê sin q cos q ú = ê0 1ú \ A3 = - I
ë û ë û ë û 2010
Þ A = ( A3 )670 = (- I )670 = I 2
é2 cos q 0 ù é1 0ù
Þ ê 0 = AB = Aü A 2 = Aü A n = Aü
ë 2 cos qúû êë0 1úû 44. ý Þ 2 ýÞ n ý
BA = Bþ B = Bþ B = Bþ
1
Þ cos q =
2 and ( A + B)2 = A 2 + B2 + AB + BA
p = A + B + A + B = 2 ( A + B)
\ q = 2 np + , n Î Z 2010 2010 2011
3 \ (A +B ) = ( A + B)2011 = 22011 ( A + B)
550 JEE Main Mathematics

Round II On comparing, we get


a + 4 + 2b = 0 Þ a + 2b = - 4 …(i)
1. Given matrix
é1 0 0ù é0 0 0ù é1 0 0ù 2a + 2 - 2b = 0 Þ a - b = - 1 …(ii)
P = ê3 1 0ú = ê3 0 0ú + ê0 1 0ú and a 2 + 4 + b2 = 9 …(iii)
ê ú ê ú ê ú
êë9 3 1úû êë9 3 0úû êë0 0 1úû On solving Eqs. (i) and (ii), we get
a = - 2, b = - 1
Þ P = X + I (let)
This satisfies Eq. (iii)
Now, P5 = (I + X )5
Hence, (a , b) º (-2, - 1)
= I + 5C1 (X ) + 5C 2(X 2) + 5C3 (X 3 ) + … T
æ é 2 -1 ù é 4 1 ù ö
n n
[Q I = I , I × A = A and (a + x) = C 0a + n n
3. Q BT AT = ( AB)T = ç ê ÷
è ë -7 4úû êë 7 2úû ø
n
C1a n - 1x + ...+T nC nxn]
T T
é0 0 0ù é0 0 0ù é0 0 0ù é 8-7 2 - 2ù é1 0ù é1 0ù
=ê ú = ê0 1ú = ê0 1ú = I
Here, X = ê3 0 0ú ê3 0 0ú = ê0 0 0ú
2
ë - 28 + 28 - 7 + 8 û ë û ë û
ê úê ú ê ú
êë9 3 0úû êë9 3 0úû êë9 0 0úû Hence, BT AT is an identity matrix.
é0 0 0ù é0 0 0ù é0 0 0ù é 0 -1 ù
4. A=ê
and X 3 = X 2 × X = ê0 0 0ú ê3 0 0ú = ê0 0 0ú ë1 0úû
ê úê ú ê ú
êë9 0 0úû êë9 3 0úû êë0 0 0úû é 0 -1 ù é 0 -1 ù é -1 0ù
A2 = ê ú ê1 ú = ê 0 -1 ú
é0 0 0ù ë 1 0 ûë 0 û ë û
Þ X = X = ê0 0 0ú
4 5
é - 1 0 ù é - 1 0 ù é 1 0 ù
ê ú Þ A4 = ê
êë0 0 0úû 0 - 1 ú ê 0 -1 ú = ê 0 1 ú
ë ûë û ë û
é0 0 0ù é0 0 0ù é 1 0 0ù Þ A8 = I × I = I
So, P5 = I + 5 ê3 0 0ú + 10 ê0 0 0ú = ê 15 1 0ú Þ A16 = I × I = I
ê ú ê ú ê ú
êë9 3 0úû êë9 0 0úû êë135 15 1úû Q A16 = lI
é 2 0 0ù Þ I = lI
and Q = I + P5 = ê 15 2 0ú = [q ij ] \ l =1
ê ú
êë135 15 2úû é i - iù
5. A=ê
Þ q21 = 15, q31 = 135 and q 32 = 15 ë- i i úû
q21 + q31 15 + 135 150 é-2 2 ù é-1 1 ù
Hence, = = = 10 A2 = ê ú =2 ê ú
q32 15 15 ë 2 - 2û ë 1 - 1û
é1 2 2ù é1 2 a ù é -1 1 ù é - 1 1 ù
A4 = 2 ê ú2ê ú
2. Given, A = ê 2 1 -2ú AT = ê2 1 2 ú ë 1 -1 û ë 1 - 1 û
ê ú ê ú
êë a 2 b úû êë2 -2 b úû é 1 - 1ù
=8 ê ú
é1 2 2 ù é1 2 a ù ë-1 1 û
AA = ê 2
T
1 -2 ú ê 2 1 2 ú é 1 - 1ù é 1 - 1ù
ê úê ú A8 = 8 ê
b úû êë2 -2 b úû ú8ê ú
êë a 2 ë-1 1 û ë-1 1 û
é 9 0 a + 4 + 2b ù é 1 - 1ù
= 128 ê
=ê 0 9 2a + 2 - 2bú ë-1 1 û
ú
ê 2 2
ú
êë a + 4 + 2b 2a + 2 - 2b a + 4 + b úû é xù é 8 ù
A8 ê ú = ê ú
It is given that ë yû ë64û
AAT = 9I é 1 - 1ù é x ù é 8 ù
a + 4 + 2b ù
128 ê úê ú=ê ú
é 9 0 é1 0 0ù ë - 1 1 û ë yû ë64û
Þ ê 0 9 2a + 2 - 2b = 9 ê0 1 0ú
ú
ê ú ê ú é x - yù é 8 ù
2 2
êë a + 4 + 2b 2a + 2 - 2b a + 4 + b úû êë0 0 1úû
128 ê ú=ê ú
ë - x + yû ë64û
é 9 0 a + 4 + 2b ù é9 0 0ù 1 1
ê x- y= and - x + y =
0 9 2a + 2 - 2bú = ê0 9 0ú 16 2
ê 2 2
ú ê ú
êë a + 4 + 2b 2a + 2 - 2b a + 4 + b úû êë0 0 9úû Hence, the linear equation has no solution.
Matrices 551

é a11 a12 a13 ù é X1 ù é1 0 0ù é a1 ù é1ù


6. Let A = ê a 21 a 22 a 23 ú and X = ê X 2 ú We have, ê2 1 0ú ê b ú = ê0ú
ê ú ê 3ú ê ú ê 1ú ê ú
êë a31 a32 a33 úû êë X úû êë3 2 1úû ëê c1 úû êë0úû
\ X¢ × A× X = 0 é1 0 0ù é a 2ù é0ù
ê2 1 0ú ê b ú = ê1ú
Þ a11 X12 + a 22 X 22 a33 X32 + (a12 + a 21 )X1 X 2 and
ê ú ê 2ú ê ú
+ (a13 + a31 ) X1 X3 + (a 23 + a32)X 2 X3 = 0 êë3 2 1úû êë c2 úû êë0úû
This is true Xi é a1 ù é1ù
a11 = a 22 = a33 = 0 Þ ê 2a + b ú = ê0ú
ê 1 1 ú ê ú
a12 + a 21 = 0 êë3a1 + 2b1 + c1 úû êë0úû
a13 + a31 = 0
é a2 ù é0ù
a 23 + a32 = 0 ê 2a + b ú = ê1ú
and
Þ -2009 + a32 = 0 ê 2 2 ú ê ú
êë3a 2 + 2b2 + c2úû êë0úû
Þ a32 = 2009
é w w2 ù Þ a1 = 1, 2a1 + b1 = 0, 3a1 + 2b1 + c1 = 0
ê ú wé 1 wù and a 2 = 0, 2a 2 + b2 = 1, 3a 2 + 2b2 + c2 = 0
7. Thus, A = ê i 2 i ú = ê ú
ê - w - w ú i ë - w -1 û Þ a1 = 1, b1 = - 2, c1 = 1
êë i i úû and a 2 = 0, b2 = 1, c2 = - 2
é 1 - w2 0 ù é 1ù é 0ù é 1ù
\ A 2 = - w2 ê ú
ë 0 1 - w2û \ u1 + u2 = ê - 2ú + ê 1ú = ê - 1ú
ê ú ê ú ê ú
é - w2 + w4 0 ù êë 1úû êë - 2úû êë - 1úû
=- ê 2 4ú
ë 0 -w + w û é a1 a 2 a3 ù
é - w2 + w 0 ù 11. Let M = ê b1 b2 b3 ú
=ê ú ê ú
ë 0 2
- w + wû êë c1 c2 c3 úû
f (x) = x2 + 2 é0ù é - 1ù é 1ù é 1ù é1ù é 0ù
é - w2 + w 0 ù é2 0ù Q M ê1ú = ê 2ú , M ê - 1ú = ê 1ú and M ê1ú = ê 0ú
\ f ( A) = A2 + 2 = ê ú+ê ú ê ú ê ú ê ú ê ú ê ú ê ú
ë 0 2
- w + wû ë0 2û êë0úû êë 3úû êë 0úû êë - 1úû êë1úû êë12úû
é1 0ù é a 2ù é - 1ù é a1 - a 2ù é 1ù
= [- w2 + w + 2] ê ú
ë0 1û Þ ê b2ú = ê 2ú Þ ê b1 - b2ú = ê 1ú
ê ú ê ú ê ú ê ú
é1 0ù é1 0ù êë c2úû êë 3úû êë c1 - c2úû êë - 1úû
= (3 + 2w) ê ú = (2 + i 3 ) ê0 1ú
ë 0 1 û ë û é a1 + a 2 + a3 ù é 0ù
ê b + b + b ú = ê 0ú
8. Given, A 2 = 2 A - I and
ê 1 2 3ú ê ú
Now, A3 = A 2 × A = 2 A 2 - IA êë c1 + c2 + c3 úû êë12úû
= 2 A 2 - A = 2(2 A - I ) - A Þ a 2 = - 1, b2 = 2, c2 = 3, a1 - a 2 = 1, b1 - b2 = 1,
= 3 A - 2I = 3 A - (3 - 1 )I c1 - c2 = -1, a1 + a 2 + a3 = 0, b1 + b2 + b3 = 0,
… … … … … c1 + c2 + c3 = 12
… … … … … Þ a1 = 0, b2 = 2, c3 = 7
A n = n A - (n - 1) I \ a1 + b2 + c3 = 0 + 2 + 7 = 9
é 2 1 ùé 2 1ù é0 0ù Hence, the sum of diagonal elements is 9.
9. A 2 = ê ú ê - 4 - 2ú =ê0 =O
ë - 4 - 2 ûë û ë 0úû é1 0 0ù é1 0 0ù
\ I + 2 A + 3 A 2 + ... = I + 2 A 12. A 2 = ê 0 1 0ú ê0 1 0ú
ê úê ú
é1 0ù é 4 2ù é 5 2ù êë a b - 1úû êë a b - 1úû
=ê ú+ ê - 8 - 4ú = ê - 8 - 3ú
ë0 1û ë û ë û é1 0 0ù
é a1 ù é a 2ù Þ A 2 = ê0 1 0ú = I3
ê ú
10. Let u1 = ê b1 ú and u2 = ê b2 ú êë0 0 1úû
ê ú ê ú
êë c1 úû êë c2 úû A 4 = A 2 × A 2 = I3 × I3 = I3
é1 0 0ù é1ù é0ù A 6 = A 2 × A 4 = I3 × I3 = I3
A = ê2 1 0ú Au1 = ê0ú and Au2 = ê1ú [given] \ A 2 + 2 A 4 + 4 A 6 = I3 + 2I3 + 4I3 = 7I3 = 7(I3 )4
ê ú ê ú ê ú
êë3 2 1úû êë0úû êë0úû = 7 ( A 2 )4 = 7 A 8
552 JEE Main Mathematics

éw 0ù 17. QS3 = S × S × S
13. Given, A=ê ú
ë 0 wû = ( ABCD ) ( ABCD ) ( ABCD )
é w 0 ù é w 0 ù é w2 0 ù = ( ABC ) (DAB) (CDA ) (BCD )
A2 = ê úê ú=ê 2ú
ë 0 wû ë 0 wû ë 0 w û = DTCT BT AT = (BCD )T AT = ( AT )T AT
é w2 0 ù é w 0 ù é w3 0 ù = AAT = ABCD = S
A3 = ê 2ú ê 0 wú = ê 3ú
ë0 wû ë û ë0 w û \ S = S and S 4 = S 2
3

é w50 0 ù 18. We know that a matrix


Similarly, A50 = ê ú
w50 û é a1 a 2 a3 ù
ë 0
A = ê b1 b2 b3 ú will be orthogonal if AA ¢ = I, which
é (w3 )16 w2 0 ù é w2 0 ù ê ú
=ê 3 16 2 ú
=ê ú =wA êë c1 c2 c3 úû
ë 0 (w ) w û ë 0 w2û
implies
é1 1 1ù S ai2 = S bi2 = S ci2 = 1
14. Given, A = 3 ê1 1 1ú
ê ú and S aibi = S bici = S ciai = 0
êë1 1 1úû é 6 2 -3 ù
1
é1 1 1ù é1 1 1ù Now, from the given options, only ê2 3 6ú
Now, A 2 = 3 ê1 ú
1 1 × 3 ê1 1 1ú = 9 A 7ê ú
ê ú ê ú êë3 -6 2 úû
êë1 1 1úû êë1 1 1úû satisfies these conditions.
\ A 4 = A 2 × A 2 = 9 A × 9 A = 81 × 9 A = 729 A é 6 2 -3 ù
1
é 1 -1 ù é 1 a ù é 2 -1 + a ù Hence, ê2 3 6 ú is an orthogonal matrix.
15. Q A + B = ê ú+ê ú=ê ú 7ê ú
ë 2 -1 û ë 4 b û ë 6 -1 + b û êë3 -6 2 úû
é 2 -1 + a ù é 2 -1 + a ù 19. Given, A1 , A3 , K A2n - 1 are skew-symmetric matrices of
Þ ( A + B )2 = ê úê ú
ë 6 -1 + b û ë 6 -1 + b û same order.
é -2 + 6 a -1 + a - b + abù Þ ( A1 )T = - A1 , ( A3 )T = - A3 , K ( A2n - 1 )T = - A 2n - 1
=ê 2ú
ë 6 + 6 b -5 + 6 a - 2 b + b û n

é1 -1 ù é1 -1 ù é - 1 0ù
Now, B = å (2r - 1) ( A2r - 1 )2r - 1
and 2
A =ê = r =1
ë2 -1úû ê2
ë -1úû êë 0 - 1úû
B = A1 + 3 ( A3 )3 + 5 ( A5 )5 + K + (2n - 1) ( A2n - 1 )2n - 1
é1 aù é1 a ù é1 + 4a a + abù
Also, B2 = ê BT = ( A1 )T + 3 [( A3 )3 ]T + 5 [( A5 )5 ]T + K
ë4 b úû ê4
ë b úû êë4 + 4b 4a + b2 úû
+ K (2n - 1)[( A2n - 1 )2n - 1 ]T
Given, ( A + B)2 = A 2 + B2 3
= - A1 - 3 ( A3 ) - 5 ( A5 ) - K5

é -2 + 6 a - 1 + a - b + ab ù - (2n - 1)( A2n - 1 )2n - 1


\ ê 2ú
ë 6 + 6 b -5 + 6 a - 2 b + b û =-B
é -1 0ù é1 + 4a a + ab ù Hence, B is a skew-symmetric matrix.
=ê ú+ê 2ú
ë 0 -1 û ë 4 + 4 b 4 a + b û é x 1ù
20. Given matrix, A = ê ú
é -2 + 6 a - 1 + a - b + ab ù é 4a a + ab ù ë1 0û
Þ ê =
ë 6 + 6b -5 + 6a - 2b + b2úû êë4 + 4b -1 + 4a + b2úû
é x 1ù é x 1ù é x2 + 1 xù
Þ A2 = ê úê ú=ê ú
On comparing both sides, we get ë1 0û ë1 0û ë x 1û
-2 + 6a = 4a and 6 + 6b = 4 + 4b
é x2 + 1 xù é x2 + 1 xù
\ a = 1 and b = - 1 Þ A4 = ê úê ú
ë x 1û ë x 1û
16. Here, C , A and CT are matrices of order
é (x2 + 1)2 + x2 x(x2 + 2)ù
n ´ 1, n ´ n and 1 ´ n, respectively. =ê ú
2
ë x(x + 2) x2 + 1 û
Let CT AC = k
Then, (C AC )T = CT AT (CT )T = CT ATC = CT (- A ) C
T Q A 4 = [aij ] and a11 = 109
T
= - C AC = - k Þ (x + 1) + x2 = 109
2 2

Þ k = -k Þ k =0 Þ x2 = 9
Hence, CT AC is a null matrix, which shows that \ a 22 = x2 + 1
CT AC is a zero matrix of order 1.
= 9 + 1 = 10
Matrices 553

21. Let a 3 ´ 3 matrix A = [aij ]3 ´ 3 ,i , j Î {1, 2, 3} a 2 a3 ù


é a1
23. Let M = ê a 4 a5 a 6 ú where a Î{0, 1, 2}
So, sum of diagonal elements of AAT ê ú
êë a7 a 8 a 9 úû
= trace of AAT = tr ([aij ][a ji ])
2 2 2 2 2 2 2 é a1 a 4 a7 ù é a1 a 2 a3 ù
= a11 + a12 + a13 + a 21 + a 22 + a 23 + a31
Then, M T M = ê a 2 a5 a 8 ú ê a 4 a5 a 6 ú
2
+ a32 2
+ a33 ê úê ú
êë a3 a 6 a 9 úû êë a7 a 8 a 9 úû
=3 (given)
Sum of the diagonal entries in M T M = 7
Q aij Î { -1, 0, 1}
Þ (a12 + a 42 + a72) + (a 22 + a52 + a 82 ) + (a32 + a 62 + a 92 ) = 7
So, possible combinations are
It is possible when
0, 0, 0, 0, 0, 0, 1, 1, 1
Case I Seven ais are 1 and two ais are zero, which can
0, 0, 0, 0, 0, 0, -1, 1, 1 be done in
0, 0, 0, 0, 0, 0, -1, -1, 1 9
C 2 = 36
0, 0, 0, 0, 0, 0, -1, -1, -1 Case II One ais is 2 and three ais are 1 and five ais
\Number of such matrices such taht tr ( AAT ) = 3 is are zero.
= 9C 6 ´ 2 ´ 2 ´ 2 9!
It can be done in 9C1 × 8C3 = = 504
9 ´8 ´ 7 5 !3 !
= ´2 ´2 ´2
3 ´2 \ Total = 540
= 84 ´ 8 é a b ù é 1 ù é -1 ù
24. ê c d ú ê -1 ú = ê 2 ú
= 672 ë ûë û ë û
Hence, answer is 672. é a - b ù é -1 ù
ê c - d ú = ê 2ú
22. Q AT A = I ë û ë û
é 1 ù é1ù
é a b c ù é a b c ù é1 0 0ù and A 2ê ú = ê ú
ê b c a ú ê b c a ú = ê0 1 0ú 1
ë û ë0û

ê úê ú ê ú éa bù é a b ù é 1ù é1ù
êë c a b úû êë c a b úû êë0 0 1úû Þ êc ê c d ú ê -1 ú = ê 0 ú
ë d úû
ë ûë û ë û
é a 2 + b2 + c2 ab + bc + ca ac + ab + bc ù
ê ú é a b ù é a - bù é1ù
= ê ab + bc + ac b2 + c2 + a 2 bc + ca + abú ê c d ú ê c - d ú = ê0ú
ë ûë û ë û
ê ca + ab + bc bc + ac + ab 2 2
c +a +b û 2ú
ë éa b ù é -1 ù é 1 ù
êc =
é1 0 0ù ë d úû êë 2úû êë0úû
= ê0 1 0ú é -a + 2bù é1ù
ê ú
êë0 0 1úû ê -c + 2d ú = ê0ú
ë û ë û
Q a3 + b3 + c3 - 3abc Now, a - b = -1 …(i)
= (a + b + c) (a 2 + b2 + c2 - ab - bc - ca ) …(i) c- d =2 …(ii)
and (a + b + c)2 = a 2 + b2 + c2 + 2 (ab + bc + ca ) -a + 2b = 1 …(iii)
=1 + 2 ´0=1 -c + 2d = 0 …(iv)
Þ a + b + c=1 …(ii) On adding Eqs. (i), (ii), (iii) and (iv), we get
[Q a , b, c are positive real numbers] b + d =2
On adding Eqs. (i) and (ii), we get
From Eqs. (i) and (ii), we get
(a + c) - (b + d ) = 1
a3 + b3 + c3 - 3 ´ 1 = 1 ´ (1 - 0) [Q abc = 1]
Þ a + c-2 =1Þa + c=3
Þ a3 + b3 + c3 = 4
Hence, a + b + c + d = 2 + 3 = 5
23
Determinants
The determinant is a value associated with a square matrix. It can be IN THIS CHAPTER ....
computed from the entries of the matrix by a specific arithmetic expression, Properties of a Determinant
while other ways to determine its value exist as well. The determinant
Applications of Determinant in
provides important information when the matrix is that of the coefficients of a
Geometry
system of linear equations or when it corresponds to a linear transformation.
Determinants arise in connection with simultaneous linear algebraic Minors and Cofactors of a
equations expressions, which are very complicated and lengthy can be easily Determinant
handled, if they are expressed as determinants. Adjoint of a Square Matrix
An expression which is related to a square matrix Inverse of a Matrix
é a11 a12 K a1n ù Solution of System of Linear
êa a22 K a2n úú Equations by Determinants
A= ê
21
ê M M M ú
ê ú
ë an1 an 2 K ann û
is known as a determinant of A. It is denoted by det( A), D or| A| and written as
a11 a12 K a1n
a21 a22 K a2n
det( A) =| A| =
M M K M
an 1 an 2 K ann
OR
Every square matrix can be associated to an expression or a number which is
known as its determinant.

Value of a Determinant of Order 1


If A = [ a11 ] is a square matrix of order 1 ´ 1,
then | A| = a11

Value of a Determinant of Order 2


éa a12 ù
If A = ê 11 ú is a square matrix of order 2 ´ 2 , then corresponding
ë a21
a22 û
a a12
determinant is| A| = 11 = a11a22 - a12a21
a21 a22
Determinants 555

Value of a Determinant of Order 3 Properties of a Determinant


é a11 a12 a13 ù
Properties of a determinant of order three only are stated
If A = êê a21 a22 a23 úú is a square matrix of order 3 ´ 3 , below. However, these properties hold for determinants
êë a31 a32 a33 úû of any order. These properties help a good deal in the
evaluation of determinants.
whose corresponding determinant is
(i) The value of the determinant remains unchanged, if
a11 a12 a13 rows are changed into columns and columns are
| A| = a21 a22 a23 changed into rows,
a31 a32 a33 a11 a12 a13 a11 a21 a31
a22 a23 a21 a23 a21 a22 i.e. a21 a22 a23 = a12 a22 a32
\ | A| = a11 - a12 + a13 a31 a32 a33 a13 a23 a33
a32 a33 a31 a33 a31 a32
1 2 3 1 3 2
= a11( a22a33 - a23 a32 ) - a12 ( a21a33 - a23 a31 )
e.g. 3 2 1 = 2 2 3
+ a13 ( a21a32 - a22a31 )
2 3 1 3 1 1
é1 2 3ù 1 2 3
e.g. If A = êê 3 ú
2 1 ú , then| A| = 3 2 1 (ii) If two adjacent rows (or columns) of a determinant
are interchanged, then the value of the
êë 1 1 1 úû 1 1 1 determinant, so obtained is the negative of the value
2 1 3 1 3 2 of the original determinant,
=1 -2 +3 a11 a12 a13 a21 a22 a23
1 1 1 1 1 1
i.e. a21 a22 a23 = - a11 a12 a13
= ( 2 - 1) - 2 ( 3 - 1) + 3 ( 3 - 2)
a31 a32 a33 a31 a32 a33
= 1 - 2 ( 2) + 3
a b c d e f
= 4- 4= 0
e.g. d e f =- a b c
Note The determinant of order greater than 2 can be expanded
g h i g h i
along any row or column.
(iii) If any two rows (or columns) of a determinant are
x sin q cos q identical, then its value is zero,
Example 1. If D1 = - sin q -x 1 a11 a11 a13 a a c
cos q 1 x i.e. a21 a21 a23 = 0 e.g. b b a = 0
x sin 2q cos 2q a31 a31 a33 c c b
and D 2 = - sin 2q -x 1 , x ¹ 0, then for all Here, first and second columns are same.
cos 2q 1 x (iv) Each element of a row (or column) of a determinant
æ pö is multiplied by a constant k, then the value of the
q Î ç0, ÷ new determinant is k times the value of the original
è 2ø (JEE Main 2019)
determinant,
(a) D1 + D 2 = - 2( x3 + x - 1) ka11 ka12 ka13 a11 a12 a13
(b) D1 - D 2 = - 2x3 i.e. a21 a22 a23 = k a21 a22 a23
(c) D1 + D 2 = - 2x3 a31 a32 a33 a31 a32 a33
(d) D1 - D 2 = x(cos 2q - cos 4q ) 2 4 6 1 2 3
x sin q cos q e.g. 4 2 3 =2 4 2 3
Sol. (c) Given determinants are D1 = - sin q - x 1 4 5 6 4 5 6
cos q 1 x
(v) If any two rows (or columns) of a determinants are
= - x3 + sin q cos q - sin q cos q + x cos2 q - x + x sin 2 q proportional, then its value is zero,
= - x3 a11 a12 a13 a11 a12 a13
x sin 2q cos 2q i.e. ka11 ka12 ka13 = k a11 a12 a13 = 0
and D 2 = - sin 2q -x 1 ,x¹0 a31 a32 a33 a31 a32 a33
cos 2q 1 x 2 4 6 1 2 3
3
= - x (similarly as D1) e.g. 1 2 3 =2 1 2 3 =0
So, according to options, we get D1 + D 2 = - 2x3 4 5 6 4 5 6
556 JEE Main Mathematics

(vi) If each element of a row (or column) of a determinant f1( x ) f2( x ) f3 ( x )


is the sum of two or more terms, then the (xii) If D( x ) = g1( x ) g2( x ) g3 ( x ) , then
determinant can be expressed as the sum of the two
a b c
or more determinants,
n n n
a11 a12 a13
i.e. a21 + c1 a22 + c2 a23 + c3
å f1( x ) å f2( x ) å f3 ( x )
x =1 x =1 x =1
n n n n
a31 a32 a33
(a) å D( x ) = å g1( x ) å g2( x ) å g3 ( x )
a11 a12 a13 a11 a12 a13 x =1 x =1 x =1 x =1

= a21 a22 a23 + c1 c2 c3 a b c


a31 a32 a33 a31 a32 a33
n n n
a b c a b c a b c
e.g. d + e f+g h+i = d f h + e g i
Õ f1( x ) Õ f2( x ) Õ f3 ( x )
x =1 x =1 x =1
n n n n
j k l j k l j k l
(b) Õ D( x ) = Õ g1( x ) Õ g2( x ) Õ g3 ( x )
(vii) If each element of a row (or column) of a x =1 x =1 x =1 x =1
determinant is multiplied by a constant k and then a b c
added to (or subtracted) from the corresponding
elements of any other row (or column), then the
value of the determinant remains the same,
Important Points
a11 a12 a13
● In D =|aij| is a determinant of order n, then the value
i.e. a21 a22 a23
of the determinant| Aij|, where Aij is the cofactor of aij ,
a31 a32 a33 is Dn - 1.
a11 a12 a13 ● If A = B + C, then it is not necessary that
= a21 a22 a23 det( A) = det( B) + det(C )
a31 + ka21 a32 + ka22 a33 + ka23 ● If A is a square matrix of order n ´ n, then
a b c a b c det( kA) = kn (det A)
e.g. d e f = d e f
● If A, B and C are three square matrix such that ith row
of A is equal to the sum of ith row of B and C and
g h i g + ak h + bk i + ck
remaining rows of A, B and C are same, then
(viii) If each element of a row (or column) of a
det( A) = det( B) + det(C )
determinant is zero, then its value is zero.
● det( An ) = (det A)n , where n is a positive integer.
(ix) If r rows (or r columns) become identical when a is
substituted for x, then ( x - a )r - 1 is a factor of given ● det ( A-1 ) =
1
determinant. det ( A)
(x) The sum of the products of elements of any row ● If the rows and columns are interchange, D is
(or column) of a determinant with the cofactors of unchanged i.e.| AT| =| A|.
the corresponding elements of any other row (or
column) is zero, ● Product of two determinants
a11 a12 a13 i.e. | AB| =| A||B| =|BA| =| ABT| =| AT B|=| AT BT|
i.e. if D = a21 a22 a23 , then
a31 a32 a33 ½x - 4 2 x 2x ½
½
Example 2. If 2 x x - 4 2 x ½ = ( A + Bx)( x - A) 2, then
a11C31 + a12C32 + a13C33 = 0 and so on. ½ ½
½ 2x 2x x - 4½
(xi) The sum of the products of elements of any row
(or column) of a determinant (where | A|n ´ n = D ) the ordered pair ( A, B) is equal to (JEE Main 2018)

with the cofactors of the corresponding elements of (a) ( -4, - 5) (b) ( -4, 3)
same row (or column) is equal to D. (c) ( -4, 5) (d) ( 4, 5)
a11 a12 a13 ½x - 4 2x 2x ½
i.e. D = a21 a22 a23 , then Sol. (c) Given, 2x x - 4 2x ½ = ( A + Bx )( x - A) 2
½
½ ½
a31 a32 a33 ½ 2x 2x x - 4½
Þ Apply C1 ® C1 + C 2 + C3
a11C11 + a12C12 + a13C13 = D
Determinants 557

½ 5x - 4 2x 2x ½ Sol. (a) Given, 2w + 1 = z


½ 5x - 4 x - 4 2x ½ = ( A + Bx )( x - A) 2 Þ 2w + 1 = - 3 [Q z = - 3]
½ ½
½ 5x - 4 2x x - 4½ - 1 + 3i
Þ w=
Taking common (5x - 4) from C1, we get 2
½1 2x 2x ½ Since, w is cube root of unity.
(5x - 4) 1 x - 4 2x ½ = ( A + Bx )( x - A) 2
½ - 1 - 3i
½ ½ \ w2 = and w3n = 1
½1 2x x - 4½ 2
Apply R2 ® R2 - R1 and R3 ® R3 - R1 1 1 1
½1 2x 0 ½ Now, 1 - w2 - 1 w2 = 3k
\ (5x - 4)½0 - x - 4 0 ½ = ( A + Bx )( x - A) 2 1 w2 w7
½ ½
½0 0 -x - 4½ 1 1 1
Expanding along C1, we get Þ 1 w w2 = 3k
(5x - 4)( x + 4) 2 = ( A + Bx )( x - A) 2 1 w2 w
Equating, we get [Q1 + w + w2 = 0 and w7 = ( w3) 2 × w = w]
A = - 4 and B = 5 On applying R1 ® R1 + R2 + R3 , we get
3 1 + w + w2 1 + w + w2
Example 3. A value of q Î(0, p / 3), for which
1 w w2 = 3k
1 + cos2 q sin2 q 4 cos 6q
2 2
1 w2 w
cos q 1 + sin q 4 cos 6q = 0, is
3 0 0
cos2 q sin2 q 1 + 4 cos6q
(JEE Main 2019) Þ 1 w w2 = 3k
p p 7p 7p 1 w2 w
(a) (b) (c) (d)
9 18 24 36
Þ 3( w - w4) = 3k
2
2 2
1 + cos q sin q 4 cos 6 q Þ ( w2 - w ) = k [Q w4 = ( w3) × w = w]
2 2
Sol. (a) Let D = cos q 1 + sin q 4 cos 6 q = 0 æ - 1 - 3i ö æ - 1 + 3i ö
cos2 q sin 2 q 1 + 4 cos 6 q \ k=ç ÷-ç ÷ = - 3i = - z
è 2 ø è 2 ø
Applying C1 ® C1 + C 2, we get
2 sin 2 q 4 cos 6 q Example 5. Let m and M be respectively the minimum and
D = 2 1 + sin q2
4 cos 6 q = 0 cos2 x 1 + sin2 x sin 2 x
2
1 sin 2 q 1 + 4 cos 6 q maximum values of 1 + cos x sin2 x sin 2 x .
cos2 x sin2 x 1 + sin 2 x
Applying R1 ® R1 - 2R3 and R2 ® R2 - 2R3 , we get
0 - sin 2 q - 2 - 4 cos 6 q Then, the ordered pair (m, M) is equal to (JEE Main 2020)
D = 0 1 - sin 2 q - 2 - 4 cos 6 q = 0 (a) ( -3, 3) (b) ( -3, - 1) (c) ( -4, - 1) (d) (1, 3)
1 sin 2 q 1 + 4 cos 6 q Sol. (b) Given determinant is
On expanding w.r.t. C1, we get ½ cos2 x 1 + sin 2 x sin 2x ½
½ 2
D = 1 + cos x 2
sin 2x ½
Þ sin 2 q (2 + 4 cos 6 q ) + (2 + 4 cos 6 q ) (1 - sin 2 q ) = 0 sin x
½ 2 2 ½
Þ 2 + 4 cos 6 q = 0 ½ cos x sin x 1 + sin 2x½
1 2p On applying, R2 ® R2 - R1 and R3 ® R3 - R1, we get
Þ cos 6 q = - = cos
2 3
½ cos2 x 1 + sin 2 x sin 2x ½
2p p é æ p öù
Þ 6q = Þq= êQ q Î çè0 , 3 ÷ø ú D =½ 1 -1 0 ½
3 9 ë û ½ ½
½ 0 -1 1 ½
Example 4. Let w be a complex number such that = cos2 x ( -1 - 0) - (1 + sin 2 x )(1 - 0) + sin 2x ( -1 - 0)
1 1 1
= - cos2 x - 1 - sin 2 x - sin 2x
2 w + 1 = z, where z = - 3. If 1 - w2 - 1 w2 = 3 k, then k is
= - (2 + sin 2x)
1 w2 w7 \Minimum value of D is ‘m’ = - (2 + 1) = - 3
equal to (JEE Main 2017) and maximum value of D is ‘M’ = - (2 - 1) = - 1
(a) - z (b) z (c) - 1 (d) 1 \The ordered pair (m, M) is ( -3, - 1) .
558 JEE Main Mathematics

Now, equation of altitude from vertex A is


Applications of Determinant in Geometry -1
y - ( - 6) = ( x - 2)
Area of Triangle æ 2 -2 ö
ç ÷
If ( x1 , y1 ), ( x2 , y2 ) and ( x3 , y3 ) are the vertices of a triangle, then è - 2 - 5ø
x1 y1 1 Þ x=2 …(i)
1
Area of triangle = x2 y2 1 Equation of altitude from vertex C is
2 -1
x3 y3 1 y -2 = [ x - ( - 2)]
é 2 - ( - 6) ù
1 ê 5 -2 ú
= [x1( y2 - y3 ) + x2( y3 - y1 ) + x3 ( y1 - y2 )] ë û
2
Þ 3x + 8y - 10 = 0 …(ii)
Condition of Collinearity of Three Points On solving Eqs. (i) and (ii), we get
Let three points are A( x1 , y1 ), B( x2 , y2 ) and C( x3 , y3 ), then these 1
x = 2 and y = .
points will be collinear, if 2
x1 y1 1 æ 1ö
\ Orthocentre = ç2, ÷
è 2ø
Area of DABC = 0 Þ x2 y2 1 = 0
x3 y3 1
Minors
Equation of Straight Line Passing Let A = [aij ] is a square matrix of order n, then
Through Two Points minor of aij is a determinant of a square matrix of
order ( n - 1) which is M ij . M ij is obtained on
Let two points are A( x1 , y1 ) and B( x2 , y2 ) and P ( x , y ) be a point leaving ith row and jth column of A.
on the line joining points A and B, then equation of line is
given by \Minors of elements a1 , a2 and a3 of determinant
x y 1 a1 b1 c1
b c b c b c
x1 y1 1 = 0 a2 b2 c2 are 2 2 , 1 1 and 1 1
b3 c3 b3 c3 b2 c2
x2 y2 1 a3 b3 c3
respectively.
Example 6. Let k be an integer such that the triangle with
1 2 3
vertices (k, - 3k), (5, k) and ( - k, 2) has area 28 sq units. Then, the
orthocentre of this triangle is at the point (JEE Main 2017) e.g. If A= -3 2 - 1 , then
æ 1ö æ 3ö æ 3ö æ 1ö 2 -4 3
(a) ç2, - ÷ (b) ç1, ÷ (c) ç1, - ÷ (d) ç2, ÷
è 2ø è 4ø è 4ø è 2ø 2 -1
M11 = = 6- 4= 2
Sol. (d) Given, vertices of triangle are (k, - 3k), (5, k) and ( - k, 2). -4 3
k - 3k 1 k - 3k 1 - 3 -1
1 M12 = = - 9+ 2= - 7
\ 5 k 1 = ± 28 Þ 5 k 1 = ± 56
2 2 3
-k 2 1 -k 2 1
-3 2
Þ k (k - 2) + 3k (5 + k) + 1(10 + k2) = ± 56 M13 = = 12 - 4 = 8
2 -4
Þ k2 - 2k + 15k + 3k2 + 10 + k2 = ± 56
Þ 5k2 + 13k + 10 = ± 56 2 3
M 21 = = 6 + 12 = 18
2 2 -4 3
Þ 5k + 13k + 66 = 0 or 5k + 13k - 46 = 0
Þ k=2 [Qk Î I] 1 3
M 22 = = 3- 6= - 3
Thus, the coordinates of vertices of triangle are A(2, - 6), B(5, 2) and 2 3
C( - 2, 2). 1 2
Y M 23 = = - 4- 4= - 8
2 -4
C (–2, 2) D 2 3
B (5, 2) M31 = = - 2- 6= - 8
(2, 1/2) 2 -1
E 1 3
X¢ X M32 = = -1+ 9= 8
O - 3 -1
A (2, –6)
1 2
and M33 = = 2+ 6= 8
Y¢ -3 2
Determinants 559

Note Adjoint of a square matrix of order 2 can be easily obtained


Cofactors by interchanging the diagonal elements and changing the signs of
The cofactor Cij of aij in D is equal to ( - 1)i + j times the the off diagonal elements.
determinant of order ( n - 1) obtained by leaving ith row
and j th column of D. é2 -3ù
Example 8. If A = ê , then adj (3A 2 + 12 A) is
It follows that, cofactor of an element aij of determinant ë -4 1 úû
a11 a12 a13 equal to (JEE Main 2016)
D = a21 a22 a23 is Cij . é 72 - 84ù é 51 63ù
(a) ê ú (b) ê ú
a31 a32 a33 ë - 63 51 û ë84 72û
Þ Cij = ( - 1) i+ j
M ij , where M ij is a minor of aij . é 51 84ù é 72 - 63ù
(c) ê ú (d) ê ú
ë63 72û ë - 84 51 û
a21 a23
Cofactor of a12 = C12 = ( - 1)1 + 2 M12 = - é 2 - 3ù
a31 a33 Sol. (b) We have, A = ê ú
ë- 4 1 û
4 -7
e.g. If A= , then cofactors are é 2 - 3ù é 2 - 3ù
-3 2 \ A2 = A × A = ê úê ú
ë- 4 1 û ë- 4 1 û
C11 = ( - 1)1 + 1 M11 = M11 = 2 é 4 + 12 - 6 - 3ù
=ê ú
C12 = ( - 1)1 + 2 M12 = - M12 = - ( - 3) = 3 ë - 8 - 4 12 + 1 û
C21 = ( - 1)2 + 1 M 21 = - M 21 = 7 é 16 - 9ù
=ê ú
and C22 = ( - 1) 2+ 2
M 22 = M 22 = 4 ë - 12 13 û
é 16 - 9ù é 2 - 3ù
Now, 3A2 + 12A = 3 ê ú + 12 ê - 4 1 ú
Example 7. If A1, B1, C1, … are respectively, the cofactors of ë - 12 13 û ë û
the elements a1, b1, c 1, … of the determinant é 48 - 27ù é 24 - 36ù
a1 b1 c 1 =ê ú+ê ú
B C2 ë - 36 39 û ë - 48 12 û
D = a2 b 2 c 2 , D ¹ 0, then the value of 2 is equal to
B3 C3 é 72 - 63ù
a3 b3 c 3 =ê ú
ë - 84 51 û
(a) a12 D (b) a1 D (c) a1 D2 (d) a12 D2
é 51 63ù
Sol. (b) B2 = a1c3 - a3 c1, C 2 = -( a1b3 - a3 b1) \adj (3A2 + 12A) = ê ú
ë84 72û
B3 = -( a1c2 - a2c1), C3 = a1b2 - a2b1
B C2 a1c3 - a3 c1 - a1b3 + a3 b1 Properties of Adjoint of a Square Matrix
\ 2 =
B3 C3 - a1c2 + a2c1 a1b2 - a2b1 (i) ( adj A) A = A ( adj A) =| A|× I n
a1c3 - a1b3 a1c3 a3 b1
= + (ii)|adj A| =| A|n - 1, if| A| ¹ 0
- a1c2 a1b2 - a1c2 - a2b1
(iii) adj ( AB) = ( adj B) ( adj A)
- a3 c1 - a1b3 - a3 c1 a3 b1
+ + (iv) If| A| = 0, then ( adj A) A = A ( adj A) = 0
a2c1 a1b2 a2c1 - a2b1
c3 - b3 c3 a3 (v) adj ( adj A) =| A|n - 2 A, where A is a non-singular
= a12 + a1b1
- c2 b2 - c2 - a2 matrix
- a3 - b3 - a3 a3 (vi) adj ( AT ) = ( adj A)T
+ a1c1 + b1c1
a2 b2 a2 - a2 (vii) Adjoint of a diagonal matrix is a diagonal matrix.
= a1 {a1 ( b2c3 - b3 c2) - b1 ( a2c3 - a3 c2) + c1 ( a2b3 - a3 b2)} 2
(viii)|adj( adj A)|=| A|( n - 1)
a1 b1 c1
(ix) adj( Am ) = ( adj A)m , m Î N
= a1 a2 b2 c2 = a1D
a3 b3 c3 (x) adj( KA) = K ( adj A), K Î R
(xi) adj( In ) = In
Adjoint of a Square Matrix (xii) adj0 = 0
Let A = [aij ]n ´ n is a square matrix of order n and let Cij (xiii) Adjoint of symmetric matrix is a symmetric matrix.
be the cofactor of aij in the determinant| A| . Then, the
(xiv) A is singular Þ|adj A|= 0
adjoint of A is denoted by adj( A) and is defined as the
transpose of the cofactor matrix. (xv) Adjoint of triangular matrix is a triangular matrix.
560 JEE Main Mathematics

é 3 - 3 4ù
Example 9. If A = êê2 - 3 4úú , then adj ( adj A) is equal to
Inverse of a Matrix
If two square matrices of same order are A and B, for
êë0 - 1 1úû
which
(a) A (b) I AB = BA = I n
(c) O (d) None of these Then, B is known as inverse of A,
i.e. A- 1 = B
Sol. (a) We know that,
adj ( adj A) = | A|n - 2 A, if| A| ¹ 0 If| A| ¹ 0 i.e. A is non-singular, then
adj A
é 3 - 3 4ù A- 1 = .
| A|
Since, A = ê 2 - 3 4ú
ê ú
êë0 - 1 1úû Properties of Inverse of a Matrix
Here, n = 3 (3 order matrix) If A, B and C are three matrices of same order and
3 -3 4 | A| ¹ 0,|B|¹ 0 and|C| ¹ 0, then
\ | A| = 2 - 3 4 (i) (a) AB = AC Þ B = C (left cancellation law)
0 -1 1 (b) BA = CA Þ B = C (right cancellation law)
= 3 ( - 3 + 4) + 3 (2 - 0) + 4 ( - 2 - 0) (ii) (a) ( AB)-1 = B-1 A-1
=1¹ 0 (b) ( ABC )-1 = C -1 B-1 A-1
\ A is non-singular. (iii) ( AT )-1 = ( A-1 )T
adj ( adj A) = | A|3 - 2×A = A 1 -1
(iv) ( kA)-1 = A , if k ¹ 0
k
é1 1 2 ù éa bù
(v) If A = ê ú and| A| ¹ 0, then
Example 10. If the matrices A = êê1 3 4úú, B = adj A and ë c dû
êë1 -1 3 úû 1 é d - bù
A- 1 = ê
| adj B| ad - bc ë - c a úû
C = 3A, then is equal to
|C| (JEE Main 2020) (vi) If A is a non-singular matrix, then
(a) 16 (b) 2 (c) 8 (d) 72 1
| A-1| =| A|-1 Þ | A-1| =
é1 1 2 ù | A|
Sol. (c) Given matrices A = ê1 3 4ú B = adj( A) and C = 3A (vii) If A is a symmetric matrix, then A-1 is also a
ê ú
êë1 - 1 3 úû symmetric matrix.
Now,| adj(B)| =| adj(adj(A))| [QB = adj(A)] (viii) A square matrix is invertible iff it is non-singular
( n - 1)2 and every invertiable matrix possesses a unique
and | adj(adj(A))| = | A| , where n is the order of square inverse.
matrix A.
2 é0 1 2 ù
\ | adj(adj(A))| = | A|(3 - 1) = | A|4
Example 11. The inverse of A = êê 1 2 3 úú is
and | C | = | 3A| = 33 | A| = 27 | A|
êë 3 1 1úû
Q | KA| = K n | A|, where K is a scalar and n is the order of
square matrix A. é -1 1 - 1ù é 1 -1 1ù
1ê 1ê
| adj B| | A|4 | A|3 (a) 8 -6 2ú (b) -8 6 - 2ú
\ = = 2ê ú 2ê ú
|C| 27 | A| 27 êë - 5 3 - 1úû êë 5 - 3 1úû
1 1 2
é1 1 1ù
| A| = 1 3 4 1ê
Q (c) 1 1 1ú (d) None of these
1 -1 3 2ê ú
êë1 1 1úû
= 1 (9 + 4) - 1 (3 - 4) + 2 ( - 1 - 3)
é0 1 2ù
= 13 + 1 - 8 = 6
| adj B| | A|3 Sol. (b) Given, A = ê 1 2 3ú
So, = ê ú
|C| 27 êë 3 1 1úû
63 Þ | A| = 0 (2 - 3) - 11
( - 9) + 2(1 - 6)
= = 23 = 8
27 = 0 + 8 - 10 = - 2 ¹ 0
Determinants 561

2 3 0 1 é cos q - sin q ù
Now, C11 = = - 1, C 23 = - = 3; Example 13. If A = ê , then the matrix A -50
1 1 3 1 ë sin q cos q úû
1 3 1 2 p
C12 = - = 8, C31 = = - 1; when q = , is equal to (JEE Main 2019)
3 1 2 3 12
é 1 3ù é 3 1ù
1 2 0 2 ê 2 ú ê 2 - 2ú
C13 = = - 5, C32 = - = 2; (a) ê 2 (b) ê
3 1 1 3 ú ú
ê- 3 1 ú ê 1 3ú
1 2 1 1 ë 2 2 û ë 2 2 û
C 21 = - = 1, C33 = = -1
1 1 0 2 é 3 1 ù é 1 3ù
ê 2 2 ú ê 2 - 2 ú
0 2 (c) ê ú (d) ê ú
C 22 = = -6
3 1 ê- 1 3ú ê 3 1 ú
ë 2 2 û ë 2 2 û
é - 1 8 - 5ù é cos q - sin qù
\ Matrix of cofactors, C = ê 1 - 6 3ú Sol. (c) We have, A = ê ú
ê ú ë sin q cos q û
êë - 1 2 - 1úû \ 2 2
| A| = cos q + sin q = 1
é -1 1 -1ù é cos q sin q ù
and adj A = ê
\ adj ( A) = C ¢ = ê 8 -6 2ú ë - sin q cos qû
ú
ê ú
êë -5 3 -1úû é a bù é d - bù
[Q If A = ê ú , then adj A = ê ú]
Hence, ( A) -1 =
adj A ëc dû ë -c a û
| A| é cos q sin q ù é adj A ù
Þ A- 1 = ê êQ A- 1 =
é -1 1 -1ù ë - sin q cos qû
ú
ë | A| úû

=- 8 -6 2ú Note that, A-50 = ( A-1)50
2ê ú
êë -5 3 -1úû Now, A-2 = ( A-1)( A-1)
é 1 -1 1ù é cos q sin q ù é cos q sin q ù
1 Þ A- 2 = ê úê ú
= ê -8 6 -2 ú ë - sin q cos qû ë - sin q cos qû
2ê ú
êë 5 -3 1 úû é 2
cos q - sin q2
cos q sin q + sin q cos qù
=ê ú
ë - cos q sin q - cos q sin q - sin 2 q + cos2 q û
Example 12. Let A be a 3 ´ 3 matrix such that é cos 2 q sin 2 q ù
=ê ú
é 2 -1 1 ù ë - sin 2 q cos 2 qû
adj A = êê -1 0 2 úú and B = adj ( adj A). If| A| = l and Also, A-3 = ( A-2)( A-1)
êë 1 -2 -1úû é cos 2 q sin 2 q ù é cos q sin q ù
A- 3 = ê úê ú
ë - sin 2 q cos 2 qû ë - sin q cos qû
|(B-1)T| = m, then the ordered pair, (| l|, m) is equal to
é cos 3 q sin 3 q ù
(JEE Main 2020) =ê ú
ë - sin 3 q cos 3 qû
æ 1ö æ 1ö æ 1ö
(a) (3, 81) (b) ç9, ÷ (c) ç9, ÷ (d) ç3, ÷ é cos 50 q sin 50 q ù
è 81ø è 9ø è 81ø Similarly, A-50 = ê ú
ë - sin 50 q cos 50 qû
é 2 -1 1 ù é 25 25 ù
cos p sin p
Sol. (d) It is given that, adj(A) = ê -1 0 2 ú ê 6 6 ú é pù
ê ú =ê when q =
êë 1 -2 -1úû 25 25 ú êë 12 úû
ê - sin p cos pú
ë 6 6 û
Þ | adj A| = 2 (0 + 4) + 1(1 - 2) + 1(2) = 9 p pù
é
Q | adj A| = | A|3 - 1 ê
cos
6
sin


Þ | A|2 = | adj A| Þ| A| = ± 3 Þ | l| = 3 p pú
ê - sin cos ú
Q B = adj ( adj A) ë 6 6û
2 é æ 25p ö æ pö pù
Þ |B| = | A|(3 - 1) = | A|4 = 81 ê Q cos çè 6 ÷ø = cosçè 4p + 6 ÷ø = cos 6 ú
1 1 1 ê 25p ö pö pú
Q |(B-1)T| = |B-1| = = Þ m= ê and sin æç æ
÷ = sinç 4p + ÷ = sin ú
|B| 81 81 êë è 6 ø è 6ø 6 úû
æ 1ö é 3 1 ù
\(| l|, m) is ç3, ÷
è 81ø ê 2 ú
=ê 2
ú
Hence, option (d) is correct. ê - 1 3 ú
ë 2 2 û
562 JEE Main Mathematics

a1 b1 c1
Solution of System of Linear Equation
\ D = a2 b2 c2
by Determinants
a3 b3 c3
In solution of system of non-homogeneous equation in
two or three variables we always use cramer’s rule. x y z 1
If D ¹ 0 , then = = =
Let a1x + b1 y + c1z = d1, a2x + b2 y + c2z = d2 D1 D2 D3 D
and a3 x + b3 y + c3 z = d3 d1 b1 c1
is a system of linear equations. where, D1 = d2 b2 c2 ;
A set of values of variables x , y and z which d3 b3 c3
simultaneously satisfy these three equations is called a a1 d1 c1
solution.
D 2 = a2 d2 c2
Consistent a3 d3 c3
If the system of equations has a unique solution or a1 b1 d1
infinite many solution, then the system of equations is
and D3 = a2 b2 d2
known as consistent.
a3 b3 d3
Inconsistent
If the system of equations has no solution, then system of Conditions for Consistency
equations is known as inconsistent. (i) If D ¹ 0, then system of equation is consistent and
has a unique solution given by
Trivial and Non-trivial Solution D D D
x = 1 , y = 2 and z = 3
If the value of all variables of system of equations is zero, D D D
i.e., x = 0, y = 0 and z = 0, then solution is known as (ii) If D = 0 and D1 = D2 = D3 = 0, then system of
trivial solution. equation is consistent with infinitely many solution.
If system of equations has infinite many solution, then (iii) If D = 0 and atleast one of D1 , D2 and D3 is non-zero,
the solution is known as non-trivial solution. then system of equation is inconsistent.
Homogeneous and Example 14. The system of linear equations
Non-homogeneous System lx + 2y + 2 z = 5, 2 lx + 3y + 5 z = 8, 4x + ly + 6 z = 10 has
If d1 = d2 = d3 = 0 in given system of equations, then (JEE Main 2020)
system of equations is said to be homogeneous otherwise (a) no solution when l = 2
it is said to be non-homogeneous system. (b) infinitely many solutions when l = 2
(c) no solution when l = 8
Cramer’s Rule (d) a unique solution when l = - 8
A system of simultaneous linear equations can be solved Sol. (a) Given, system of linear equations
by Cramer’s rule named for the Swiss Mathematician lx + 2y + 2z = 5, 2lx + 3y + 5z = 8
Gabriel Cramer. and 4x + ly + 6z = 10 , then
(i) The solution of the system of linear equations ½ l 2 2½
D = ½2l 3 5½= l(18 - 5l) - 2(12l - 20) + 2(2l2 - 12)
a1x + b1 y = c1 and a2x + b2 y = c2 ½ ½
D1 D ½ 4 l 6½
is given by x = ,y= 2 = - l2 - 6l + 16 = - [ l2 + 6l - 16]
D D
a b c b1 = - [ l2 + 8l - 2l - 16]
where, D = 1 1 , D1 = 1 = - [ l( l + 8) - 2( l + 8)] = - ( l - 2)( l + 8)
a2 b2 c2 b2
Now, when l = 2, then
a1 c1 ½ 5 2 2½
and D2 =
a2 c2 D1 = ½ 8 3 5½= 5(18 - 10) - 2( 48 - 50) + 2(16 - 30)
½ ½
provided that D ¹ 0. ½10 2 6½
(ii) Let the determinant of coefficient of equations = 40 + 4 - 28 = 16 ¹ 0
a1x + b1 y + c1z = d1, a2x + b2 y + c2z = d2 For, l = 2, then D = 0 but D1 ¹ 0, so there is no solution for
given system of linear equations if l = 2.
and a3 x + b3 y + c3 z = d3 is D.
Hence, option (a) is correct.
Determinants 563

Example 15. If the system of equations x + y + z = 5, Sol. (d) Given, system of linear equations
x + 2y + 3z = 9, x + 3y + az = b has infinitely many solutions, 7x + 6y - 2z = 0
then b - a equals (JEE Main 2019) Þ 3x + 4y + 2z = 0
x - 2y - 6z = 0
(a) 8 (b) 18 (c) 21 (d) 5
7 6 -2
Sol. (a) Since, the system of equations has infinitely many \ D=3 4 2 = 7( -24 + 4) - 6( -18 - 2) - 2( -6 - 4)
solution, therefore
1 -2 -6
D = D1 = D2 = D3 = 0
= 7( -20) + 6(20) + 2(10)
1 1 1
= - 140 + 120 + 20 = 0
Here, D = 1 2 3 = 1(2a - 9) - 1 ( a - 3) + 13 ( - 2)
Since, D = 0 for homogeneous systems of linear equations,
1 3 a then system has infinitely many solutions.
= a -5 Now, since (7x + 6y - 2z) + 3( x - 2y - 6z) = 0
1 1 5 Þ 10 x - 20 z = 0 Þ x = 2z
and D3 = 1 2 9 = 1 (2 b - 27) - 1(b - 9) + 5 (3 - 2) So, the given system of linear equations has infinitely many
solutions satisfying x = 2z.
1 3 b
Hence, option (d) is correct.
= b - 13
Now, D = 0 Þ a - 5 = 0 Þ a = 5 Example 17. The greatest value of c Î R for which the
system of linear equations x -cy - cz = 0, cx -y + cz = 0,
and D3 = 0 Þ b - 13 = 0
cx + cy - z = 0 has a non-trivial solution, is (JEE Main 2019)
Þ b = 13 1
\ b - a = 13 - 5 = 8 (a) -1 (b) (c) 2 (d) 0
2
Sol. (b) Given, system of linear equations is
Solution of Homogeneous System
x - cy - cz = 0,
of Equations cx - y + cz = 0
Let a1x + b1 y + c1z = 0 and cx + cy - z = 0
a2x + b2 y + c2z = 0 We know that, a homogeneous system of linear equations
and a3 x + b3 y + c3 z = 0 have non-trivial solutions if
D =0
be a homogeneous system of linear equations. Then,
½ 1 - c - c ½
(i) System of equations has a unique trivial ½ c -1
Þ c ½= 0
a1 b1 c1 ½ ½
½c c -1½
x = y = z = 0, if D = a2 b2 c2 ¹ 0.
a3 b3 c3 Þ 1(1 - c2) + c( - c - c2) - c( c2 + c) = 0
Þ 1 - c2 - c2 - c3 - c3 - c2 = 0
(ii) System of equations has non-trivial solution, if
Þ -2c3 - 3c2 + 1 = 0
D=0
Þ 2c3 + 3c2 - 1 = 0
Example 16. The following system of linear equations Þ ( c + 1)[2c2 + c - 1] = 0
7x + 6y - 2 z = 0, 3x + 4y + 2 z = 0 and x - 2y - 6 z = 0, has Þ ( c + 1)[2c2 + 2c - c - 1] = 0
(JEE Main 2020)
(a) infinitely many solutions, ( x, y , z) satisfying y = 2z. Þ ( c + 1)(2c - 1)( c + 1) = 0
1
(b) no solution. Þ c = -1or
2
(c) only the trivial solution.
1
(d) infinitely many solutions, ( x, y , z) satisfying x = 2z. Clearly, the greatest value of c is .
2
Practice Exercise
ROUND I Topically Divided Problems

Determinants and its Properties - sin 2 q -1 - sin 2 q 1


2 2
( a + 1) ( a + 2) a + 2 1 f (q ) = - cos q -1 - cos q 1 are m and M
1. The value of ( a + 2) ( a + 3) a + 3 1 is 12 10 -2
( a + 3) ( a + 4) a + 4 1 (JEE Main 2021) respectively, then the ordered pair ( m, M ) is equal
(a) - 2 to (JEE Main 2020)
(b) (a + 1) (a + 2) (a + 3) (a) (0, 2 2 ) (b) (-4, 4)
(c) 0 (c) (0, 4) (d) (-4, 0)
(d) (a + 2) (a + 3) (a + 4)
7. If a + x = b + y = c + z + 1, where a, b, c, x, y, z are
cos a cos b cos a sin b - sin a
non-zero distinct real numbers, then
2. - sin b cos b 0 is equal to ½x a + y x + a½
sin a cos b sin a sin b cos a ½ y b + y y + b½ is equal to
½ ½
(a) sin (a + b ) (b) cos (a + b ) ½ z c + y z + c½ (JEE Main 2020)
(c) 0 (d) 1
(a) y(b - a ) (b) y(a - b) (c) 0 (d) y(a - c)
a b 0
3. If 0 a b = 0, then 8. Let the numbers 2, b, c be in an AP and
b 0 a é1 1 1ù
A = ê2 b c ú. If det( A) Î [ 2, 16 ], then c
(a) a is one of the cube roots of unity ê 2
ú
(b) b is one of the cube roots of unity êë 4 b c2 úû
æaö lies in the interval (JEE Main 2019)
(c) ç ÷ is one of the cube roots of unity
è bø (a) [3, 2 + 23/ 4 ] (b) (2 + 23/ 4 , 4)
æaö (c) [4, 6] (d) [2, 3)
(d) ç ÷ is one of the cube roots of - 1
è bø
9. Let a and b be the roots of the equation
4. If x, y, z are in arithmetic progression with common x 2 + x + 1 = 0. Then, for y ¹ 0 in R,
difference d, x ¹ 3d, and the determinant of the y+1 a b
é3 4 2 xù a y+b 1 is equal to
ê ú b 1 y+a
matrix ê 4 5 2 y ú is zero, then the value of k2 is (JEE Main 2019)
êë 5 k z úû 2
(a) y( y - 1) (b) y ( y2 - 3)
(JEE Main 2021)
(c) y3 - 1 (d) y3
(a) 72 (b) 12 (c) 36 (d) 6
10. Let d Î R, and
5. Let A be a 3 ´ 3 matrix with det ( A) = 4. Let Ri
é -2 4+d (sin q ) - 2 ù
denote the ith row of A. If a matrix B is obtained by
performing the operation R2 ® 2 R2 + 5 R3 on 2 A, A=ê1 (sin q ) + 2 d ú,
ê ú
then det ( B) is equal to (JEE Main 2021) ëê 5 ( 2 sin q ) - d ( - sin q ) + 2 + 2 d úû
(a) 64 (b) 16 (c) 80 (d) 128 q Î[q , 2 p ]. If the minimum value of det(A) is 8, then
6. If the minimum and the maximum values of the a value of d is (JEE Main 2019)
p p (a) -5 (b) -7
function f : é , ù ® R, defined by
êë 4 2 úû (c) 2( 2 + 1) (d) 2( 2 + 2)
Determinants 565

é2 b 1ù Then, the number of elements in S, is (JEE Main 2019)


11. Let A = b b + 1 b ú, where b > 0. Then, the
ê 2
(a) 4 (b) 2
ê ú
êë 1 b 2 úû (c) 10 (d) infinitely many
det ( A) 18. The value of determinant
minimum value of is
b (JEE Main 2019) ( ax + a- x) 2 ( ax - a- x) 2 1
(a) - 3 (b) -2 3 (c) 2 3 (d) 3 ( bx + b- x ) 2 ( bx - b- x ) 2 1 is
é 1 sin q 1 ù ( cx + c- x ) 2 ( cx - c- x ) 2 1
ê
12. If A = - sin q 1 sin q ú; then for all
ê ú (a) 0 (b) 2 abc
êë - 1 - sin q 1 úû (c) a 2b 2c 2 (d) None of these
3p 5 p ö
q Î æç , ÷, det( A) lies in the interval 19. The arbitrary constant on which the value of the
è 4 4 ø (JEE Main 2019)
1 a a2
æ3 ù é5 ö æ 3ù æ 5ù determinant cos ( p - d) a cos pa cos ( p - d) a
(a) ç , 3ú (b) ê , 4÷ (c) ç0, ú (d) ç1, ú
è2 û ë2 ø è 2û è 2û
sin ( p - d) a sin pa sin ( p - d) a
a- b- c 2a 2a
does not depend, is
13. If 2b b- c- a 2b
(a) a (b) p
2c 2c c-a-b (c) d (d) a
= ( a + b + c) ( x + a + b + c) 2 , x ¹ 0 and a + b + c ¹ 0, 20. If a , b and c are unequal what is the condition
then x is equal to (JEE Main 2019)
that the value of the determinant,
(a) - (a + b + c) (b) - 2(a + b + c) a a2 a3 + 1
(c) 2(a + b + c) (d) abc
D º b b2 b3 + 1 is 0 ?
14. If a , b ¹ 0, f ( n) = a n + b n and c c2 c3 + 1
3 1 + f (1) 1 + f ( 2)
(a) 1 + abc = 0
1 + f (1) 1 + f ( 2) 1 + f (3) (b) a + b + c+ 1 =0
1 + f ( 2) 1 + f (3) 1 + f ( 4) (c) (a - b) (b - c) (c - a ) = 0
= K (1 - a ) 2 (1 - b) 2 (a - b) 2 , then K is equal to (d) None of the above
(JEE Main 2014)
1
21. The value of x obtained from the equation
(a) ab (b) (c) 1 (d) - 1 x+a b g
ab
g x+b a = 0 will be
x+ a x+2 x+1
a b x+g
15. Let a - 2 b + c = 1. If f ( x) = x + b x +3 x + 2 ,
(a) 0 and - (a + b + g )
x+c x+4 x +3 (b) 0 and (a + b + g )
then (JEE Main 2020) (c) 1 and (a - b - g )
(a) f (-50) = 501 (b) f (50) = 1 (d) 0 and (a 2 + b 2 + g 2)
1 4 20
(c) f (-50) = - 1 (d) f (50) = - 501
22. The roots of the equation 1 -2 5 = 0 are
16. The sum of the real roots of the equation
1 2 x 5 x2
x -6 -1
(a) -1, - 2 (b) -1, 2
2 - 3x x - 3 = 0, is equal to
(c) 1, - 2 (d) 1, 2
-3 2x x+2 (JEE Main 2019)
23. If a + b + c = 0, then the solution of the equation
(a) 0 (b) - 4 (c) 6 (d) 1
a-x c b
17. Let a1, a2 , a3. . . . . , a10 be in GP with ai > 0 for c b- x a = 0 is
i = 1, 2, . . . . . ,10 and S be the set of pairs ( r, k), b a c-x
r, k Î N (the set of natural numbers) for which 3 2
(a) 0 (b) ± (a + b2 + c2)
log e a1r a2k log e a2r a3k log e a3r a4k 2
log e a4r a5k log e a5r a6k log e a6r a7k = 0 3 2
(c) 0, ± (a + b2 + c2) (d) 0, ± a 2 + b2 + c2
log e a7r a8k log e a8r a9k log e a9r a10k 2
566 JEE Main Mathematics

24. If a , b and c are positive integers, then the 1+ a 1 1


2
a +x ab ac 31. The value of the determinant 1 1+ a 1 is
determinant D = ab b +x2
bc is divisible 1 1 1+ a
ac bc 2
c +x æ 2ö æ 3ö
(a) a3 ç1 - ÷ (b) a3 ç1 + ÷
è aø è aø
by æ 3ö æ 2ö
(a) x3 (b) x2 (c) a3 ç1 - ÷ (d) a3 ç1 + ÷
è aø è aø
(c) (a 2 + b2 + c2) (d) None of these
32. For positive numbers x, y and z, the numerical
25. If x, y and z are different from zero and
1 log x y log x z
a b- y c- z
value of the determinant log y x 1 log y z is
D= a-x b c - z = 0, then the value of the
log z x log z y 1
a- x b- y c
a b c (a) 0 (b) 1
expression + + is
x y z (c) log e xyz (d) None of these
(a) 0 (b) -1 (c) 1 (d) 2 1 a a2
1 x y 33. If f (a ) = a a2 1 , then f ( 3 3) is equal to
26. Determinant 2 sin x + 2 x sin y + 2 y is equal to a2 1 a
3 cos x + 3x cos y + 3 y (a) 1 (b) – 4 (c) 4 (d) 2
(a) sin (x - y) (b) cos (x - y) é r r - 1ù
34. If the matrix M r is given by M r = ê ,
(c) cos(x + y) (d) xy sin(x - y)
ër -1 r úû
27. The value of the determinant r = 1, 2 , 3, . . . , then the value of
a2 a 1 det (M1 ) + det (M 2) + . . . + det (M 2008 ) is
cos( nx) cos ( n + 1) x cos( n + 2) x is independent of (a) 2007 (b) 2008
sin ( nx) sin ( n + 1) x sin( n + 2)x (c) (2008)2 (d) (2007)2
(a) n (b) a a a+b a+ b+ c
(c) x (d) None of these 35. D = 3a 4 a + 3b 5 a + 4 b + 3c , where
2r - 1 3r - 1 4r - 1 6 a 9 a + 6 b 11a + 9 b + 6 c
28. If Dr = x y z , then the a = i, b = w, c = w2 , then D is equal to
(b) - w2
n n n
2 - 1 (3 - 1)/ 2 ( 4 - 1)/3 (a) i
n (c) w (d) - i
value of å Dr is equal to
36. If a , b and c are respectively the pth, qth and rth
r =1

(a) 1 (b) – 1
a p 1
(c) 0 (d) None of these terms of an AP, then b q 1 is equal to
n 1 5 c r 1
N
29. The value of å U n , if U n = n 2
2N +1 2N +1 , (a) 1 (b) - 1 (c) 0 (d) pqr
n=1
n3 3N 2 3N 37. If a , b and c are in AP, then the value of
is x+ 2 x+3 x+ a
(a) 0 (b) 1 x + 4 x + 5 x + b is
(c) - 1 (d) None of these x+6 x+7 x+c
1 1 1
(a) x - (a + b + c) (b) 9x2 + a + b + c
30. If f ( x) = 2x ( x - 1) x , then (c) a + b + c (d) 0
3x ( x - 1) ( x - 1)( x - 2) x ( x - 1)
38. The area of a triangle with vertices ( -3, 0), (3, 0)
f (50) is equal to
and (0, k) is 9 sq units. The value of k will be
(a) 0 (b) 1
(c) 100 (d) – 100 (a) 9 (b) 3 (c) –9 (d) 6
Determinants 567

39. If the coordinates of the vertices of an equilateral 47. Let A and B be two invertible matrices of order
triangle with sides of length a are ( x1, y1), ( x2 , y2 ) 3 ´ 3. If det( ABA T ) = 8 and det( AB - 1) = 8, then
x1 y1 1
2
det( BA - 1B T ) is equal to (JEE Main 2019)
1 1
and ( x3, y3), then x2 y2 1 is equal to (a) 1 (b) (c) (d) 16
4 16
x3 y3 1
é et e- t cos t e- t sin t ù
48. If A = ê e - e cos t - e sin t - e sin t + e cos t úú
ê t -t
4 2 -t -t -t
a 3a
(a) (b)
4 4
êë et 2 e- t sin t -2 e- t cos t úû
5a 4 3a 4
(c) (d)
4 4 then A is (JEE Main 2019)
(a) invertible only when t = p
Adjoint of Matrix (b) invertible for every t Î R
(c) not invertible for any t Î R
é3 4 ù
40. If A = ê p
ú , then A × (adj A) is equal to (d) invertible only when t =
ë5 7 û 2
(a) A (b) | A| é 1 1ù é 1 2 ù é 1 3ù é 1 n - 1ù é 1 78 ù
49. If ê ú.ê ú.ê ú . . . ê0 = ,
(c) | A| I (d) None of these
ë0 1û ë0 1 û ë0 1û ë 1 úû êë0 1 úû
é1 -1 1ù é 1 nù
41. If A = ê 0 2 -3 ú , B = (adj A) and C = 5 A,
then the inverse of ê ú is
ë0 1 û
ê ú (JEE Main 2019)
êë 2 1 0 úû é 1 0ù é1 -13ù é 1 0ù é1 -12ù
|adj B| (a) ê ú (b) ê0 1 ú (c) ê13 1ú (d) ê ú
then is equal to ë 12 1 û ë û ë û ë0 1 û
|C|
é 0 3ù -1
(a) 5 (b) 25 (c) –1 (d) 1 50. A = ê ú and A = l (adj A), then l is equal to
ë 2 0 û
é 1 2 -1 ù
1 1 1 1
42. If A = ê -1 1 2 ú , then det [adj (adj A)] is equal (a) - (b) (c) - (d)
ê ú 6 3 3 6
êë 2 -1 1 úû é x1 ù é 1 -1 2 ù é3ù
to 51. Let X = x2 , A = 2 0 1 and B = ê 1 ú. If
ê ú ê ú
ê ú ê ú ê ú
(a) 124 (b) 134 êë x3 úû êë 3 2 1 úû êë 4 ûú
(c) 144 (d) None of these
AX = B, then X is equal to
é -1 -2 -2 ù
é1ù é -1 ù é -1 ù é -1 ù
43. If A = ê 2 1 -2 ú , then adj A is equal to (a) ê2ú (b) ê -2 ú (c) ê -2ú (d) ê 2ú
ê ú ê ú ê ú ê ú ê ú
êë 2 -2 1úû êë3úû êë 3úû êë -3úû êë 3úû
(a) A (b) A¢ é cos x - sin x 0 ù
(c) 3 A (d) 3 A¢
52. If f ( x) = êsin x cos x 0 ú ,
é5 a - bù ê ú
44. If A = ê T
ú and A adj A = AA , then 5a + b is êë 0 0 1 úû
ë 3 2 û é cos y 0 sin y ù
equal to
G ( y) = ê 0 0 ú,
(JEE Main 2016)
1
(a) - 1 (b) 5 (c) 4 (d) 13 ê ú
êë - sin y 0 cos y úû
Inverse of Matrix
then [ F ( x) × G ( y)]-1 is equal to
(b) F -1 (x) G -1 ( y)
2
45. If A - A + I = 0, then the inverse of A is (a) F (x) G (- y)
(a) A - I (b) I - A (c) G -1 ( y) F -1 (x) (d) G (- y) F (- x)
(c) A + I (d) A é0 1 2 ù é 1 / 2 -1 / 2 1 / 2 ù
46. The element in the first row and third column of 53. If A = ê 1 2 3 ú and A -1 = ê -4 3 c ú,
ê ú ê ú
é 1 2 -3ù êë 3 a 1 úû êë5 / 2 -3 / 2 1 / 2 úû
the inverse of the matrix ê0 1 2 ú is then
ê ú
êë0 0 1 úû (a) a = 2, c = 1 /2 (b) a = 1, c = - 1
1 1
(a) - 2 (b) 0 (c) 1 (d) 7 (c) a = - 1, c = 1 (d) a = , c =
2 2
568 JEE Main Mathematics

é qù 61. If the system of linear equations


1 tan
ê 2 ú and AB = I , then B is equal
54. If A = ê ú x + y + 3z = 0
q
êë - tan 2 1 ú
û x + 3 y + k2 z = 0
to 3x + y + 3z = 0
q 2 2q T has a non-zero solution ( x, y, z) for some k Î R, then
(a) cos ×A (b) cos ×A
x + æç ö÷ is equal to
2 2 y
q è zø
(c) cos 2 q× 1 (d) sin 2 × A (JEE Main 2020)
2
(a) -3 (b) 9
(c) 3 (d) -9
Solution of Simultaneous
Linear Equations 62. If for some a and b in R, the intersection of the
following three planes
55. The system of equations
x + 4 y - 2 z = 1, x + 7 y - 5 z = b, x + 5 y + az = 5
(aa + b)x + ay + bz = 0
(ba + c)x + by + cz = 0 is a line in R 3 , then a + b is equal to (JEE Main 2020)

(aa + b) y + (ba + c)z = 0 (a) 0 (b) 10


(c) -10 (d) 2
has a non-trival solution, if
(a) a, b, c are in AP 63. If the system of linear equations
(b) a, b, c are in GP x - 4 y + 7z = g
(c) a, b, c are in HP 3y - 5z = h
(d) a is not a root of ax2 + 2bx + c = 0
- 2x + 5 y - 9z = k
56. The system of equations is consistent, then (JEE Main 2019)
ax + y + z = a - 1 (a) 2 g + h + k = 0
x+a y+ z=a -1 (b) g + 2h + k = 0
x+ y+a z=a -1 (c) g + h + k = 0
has no solution, if a is (d) g + h + 2k = 0
(a) not -2 (b) 1 64. If the system of linear equations
(c) -2 (d) either -2 or 1
x + y + z = 5, x + 2 y + 2 z = 6
57. If the system of equations x + ky - z = 0, x + 3 y + lz = m, (l, m Î R), has infinitely many
3x - ky - z = 0 and x - 3 y + z = 0 has non-zero solutions, then the value of l + m is (JEE Main 2019)
solution, then k is equal to (a) 7 (b) 12 (c) 10 (d) 9
(a) – 1 (b) 0 (c) 1 (d) 2
65. If the system of linear equations
58. The system of equations 3x - 2 y + z = 0, 2 x + 2 ay + az = 0
lx - 14 y + 15 z = 0, x + 2 y - 3z = 0 has a solution 2 x + 3by + bz = 0
other than x = y = z = 0, then l is equal to
2 x + 4 cy + cz = 0
(a) 1 (b) 2 (c) 3 (d) 5
where a, b, c Î R are non-zero and distinct; has a
59. The system of linear equations
non-zero solution, then (JEE Main 2020)
3x - 2 y - kz = 10 1 1 1
(a) a + b + c = 0 (b) , , are in AP
2x - 4 y - 2z = 6 a b c
x + 2 y - z = 5m (c) a , b, c are in AP (d) a , b, c are in GP
is inconsistent, if (JEE Main 2021) 66. An ordered pair (a , b) for which the system of linear
4
(a) k = 3, m = (b) k ¹ 3, m Î R equations
5
4 4 (1 + a ) x + by + z = 2
(c) k ¹ 3, m ¹ (d) k = 3, m ¹
5 5 ax + (1 + b) y + z = 3
ax + by + 2 z = 2
60. If the equations 2 x + 3 y + 1 = 0, 3x + y - 2 = 0 and
ax + 2 y - b = 0 are consistent, then has a unique solution, is (JEE Main 2019)

(a) a - b = 2 (b) a + b + 1 = 0 (a) (2, 4) (b) (- 4, 2)


(c) a + b = 3 (d) a - b - 8 = 0 (c) (1, - 3) (d) (-3, 1)
Determinants 569

67. If S is the set of distinct values of b for which the xz


has a non-zero solution ( x, y, z), then is equal to
following system of linear equations y2
(JEE Main 2018)
x + y + z = 1, x + ay + z = 1
(a) -10 (b) 10
and ax + by + z = 0 (c) -30 (d) 30
has no solution, then S is (JEE Main 2017)
69. Let A = { X = ( x, y, z) T : PX = 0 and x 2 + y 2 + z2 = 1},
(a) an infinite set
(b) a finite set containing two or more elements é1 2 1ù
(c) singleton set where P = ê -2 3 -4 ú, then the set A
ê ú
(d) an empty set êë 1 9 -1 úû (JEE Main 2020, 2 September)
68. If the system of linear equations (a) is a singleton
x + ky + 3z = 0 (b) is an empty set
3x + ky - 2 z = 0 (c) contains more than two elements
2 x + 4 y - 3z = 0 (d) contains exactly two elements

ROUND II Mixed Bag


Only One Correct Option
(JEE Main 2020)
1. If x, y, z are integers in AP, lying between 1 and 9 (a) - 1 (b) 1 (c) - 3 (d) 9
and x 51, y 41 and z 31 are three digited numbers, p é cos q sin q ù
5 4 3 5. Let q = and A = ê ú.
5 ë - sin q cos q û
then value of x51 y 41 z31 is
If B = A + A 4 , then det (B)
x y z (JEE Main 2020)
(a) x + y + z (b) x - y + z (a) is one (b) lies in (2, 3)
(c) 0 (d) None of these (c) is zero (d) lies in (1, 2)

2. Consider the following system of equations 6. Let A = [ aij ] and B = [ bij ] be two 3 ´ 3 real matrices
x + 2 y - 3z = a such that bij = (3) ( i + j - 2)
a ji , where i, j = 1, 2, 3.
2 x + 6 y - 11z = b If the determinant of B is 81, then the determinant
x - 2 y + 7z = c of A is (JEE Main 2020)
where a, b and c are real constants. Then, the 1 1 1
(a) (b) 3 (c) (d)
system of equations (JEE Main 2021) 9 81 3
(a) has a unique solution when 5a = 2b + c 7. The system of equations kx + y + z = 1, x + ky + z = k
(b) has infinite number of solutions when 5a = 2b + c
(c) has no solution for all a , b and c and x + y + zk = k2 has no solution,
(d) has a unique solution for all a , b and c if k is equal to (JEE Main 2021)
(a) 0 (b) 1 (c) - 1 (d) - 2
3. All the values of t for which the system of
equations. 8. If the system of equations
(t - 1)x + (3t + 1) y + 2tz = 0 x+ y+z=2
(t - 1)x + (4 t - 2) y + (t + 3) z = 0 2x + 4 y - z = 6
2x + (3t + 1) y + 3(t - 1)z = 0 3x + 2 y + lz = m
has infinitely many solutions, then (JEE Main 2020)
has a non-trivial solution, are
(a) 2l + m = 14 (b) 2l - m = 5
(a) 1 or 2 (b) 2 or 4
(c) l - 2m = - 5 (d) l + 2m = 14
(c) 0 or 3 (d) None of these
x-2 2x - 3 3x - 4 9. Suppose the vectors x1, x2 and x3are the solutions of
the system of linear equations, Ax = b when the
4. If D = 2 x - 3 3x - 4 4x - 5 =
vector b on the right side is equal to b1, b2 and b3
3x - 5 5 x - 8 10 x - 17
respectively.
Ax + Bx 2 + Cx + D, then B + C is equal to
3
570 JEE Main Mathematics

3
é1ù é0 ù é0 ù é 1ù é0 ù (a) a3 + b + c2 - 3abc (b) a 2b - b2c
If x1 = 1 , x2 = 2 , x3 = 0 , b1 = 0 , b2 = ê 2 ú
ê ú ê ú ê ú ê ú
(c) 0 (d) a 2 + b2 + c2
ê ú ê ú ê ú ê ú ê ú
êë1úû êë 1 ûú êë 1 ûú êë0 ûú êë 0 ûú 1 + sin 2 q cos2 q 4 sin 4 q
2 2
é0 ù 16. If sin q 1 + cos q 4 sin 4 q = 0, then q is
and b3 = ê 0 ú, then the determinant of A is equal to sin 2 q cos2 q 1 + 4 sin 4 q
ê ú
êë 2 úû (JEE Main 2020) equal to
3 1 7p 11p 5p 7p
(a) (b) 2 (c) (d) 4 (a) , (b) ,
2 2 24 24 24 24
11p p p 7p
10. The maximum value of (c) , (d) ,
24 24 24 24
1 + sin 2 x cos 2 x 4 sin 2x
2 2 x 2 + 3x x-1 x+3
f (x) = sin x 1 + cos x 4 sin 2x is 4 3 2
17. Let px + qx + rx + sx + t = x + 1 - 2x x - 4 ,
sin 2 x cos 2 x 1 + 4 sin 2x
x-3 x+ 4 3x
(a) – 2 (b) – 6
(c) 6 (d) 2 where p, q, r, s and t are constants, then t is equal
to
a2 ( s - a) 2 ( s - a) 2 (a) 0 (b) 1 (c) 2 (d) –1
11. If 2s = a + b + c and ( s - b) 2 b2 ( s - b) 2 18. If a , b and g are the roots of the equation
( s - c) 2 ( s - c) 2 c2
x 3 + px + q = 0, then the value of the determinant
3
= ks ( s - a)( s - b)( s - c), a b g
then the numerical quantity k should be b g a is
(a) 1 (b) 2 (c) 3 (d) 4 g a b

n n +1 n+2 (a) 0 (b) – 2 (c) 2 (d) 4


n ( n + 1) (n + 2) 19. If the system of equations x + ay = 0, az + y = 0 and
12. Pn P( n + 1) P( n + 2 ) is equal to
n
Cn ( n + 1)
C( n + 1) (n + 2)
C( n + 2 ) ax + z = 0 has infinite solutions, then the value of a
is
(a) n (n !) (b) (n + 1) (n + 1)! (a) 0 (b) – 1
(c) (n + 2) (n + 2)! (d) (n 2 + n + 1)n ! (c) 1 (d) no real values
13. If a, b and c are cube roots of unity, then 20. If [ ] denotes the greatest integer less than or equal
ea e2 a e 3a - 1 to the real number under consideration and
eb e2 b e3b - 1 is equal to -1 £ x < 0; 0 £ y < 1; 1 £ z < 2 , then the value of the
ec e2 c e 3c - 1 [x] + 1 [ y] [ z]
determinant [x] [ y] + 1 [ z] is
(a) 0 (b) e
(c) e2 (d) e3 [x] [ y] [ z] + 1

xp + y x y (a) [x] (b) [ y]


(c) [z ] (d) None of these
14. yp + z y z = 0, if
1+ a 1 1
0 xp + y y+z
21. If 1 + b 1 + 2 b 1 = 0, where
(a) x, y and z are in AP
1+ c 1+ c 1 + 3c
(b) x, y and z are in GP
(c) x, y and z are in HP a ¹ 0, b ¹ 0, c ¹ 0, then a -1 + b-1 + c-1 is
(d) xy, yz and zx are in AP (a) 4 (b) - 3 (c) - 2 (d) - 1

15. If w is an imaginary cube root of unity, then the a -1 0


22. If f ( x) = ax a - 1 , then f ( 2 x) - f ( x) is equal
a bw2 aw
ax 2 ax a
value of bw c bw2 is
to
cw2 aw c (a) ax (b) ax (2a + 3x)
(c) ax (2 + 3x) (d) None of these
Determinants 571

1 n n 29. If the system of linear equations


2 2
23. If D r = 2r n + n+1 n + n and 2 x + 2 y + 3z = a, 3x - y + 5 z = b, x - 3 y + 2 z = c
2r - 1 n2 n2 + n + 1
n where a, b, c are non-zero real numbers, has more
å D r = 56, then n is equal to than one solution, then (JEE Main 2019)
r =1 (a) b - c - a = 0 (b) a + b + c = 0
(a) 4 (b) 6 (c) 7 (d) 8 (c) b - c + a = 0 (d) b + c - a = 0
24. If a , b, g Î R, then the determinant 30. Let l Î R. The system of linear equations
ia - ia 2 ia -ia 2 2 x1 - 4 x2 + lx3 = 1,
(e + e ) (e - e ) 4
D = (eib + e–ib )2 (eib - e–ib )2 4 is x1 - 6 x2 + x3 = 2
ig – ig 2 ig – ig 2
(e + e ) (e - e ) 4 and lx1 - 10 x2 + 4 x3 = 3

(a) independent of a , b and g is inconsistent for (JEE Main 2020)

(b) dependent on a , b and g (a) exactly two values of l


(c) independent of a , b (b) exactly one positive value of l
(d) independent of a , g (c) every value of l
25. The value of l and m for which the system of linear (d) exactly one negative value of l
equations
Numerical Value Based Questions
x + y + z = 2, x + 2 y + 3z = 5, x + 3 y + lz = m
has infinitely many solutions are, respectively 31. Let S be the set of all integer solutions, ( x, y, z), of
(JEE Main 2020) the system of equations x - 2 y + 5 z = 0
(a) 6 and 8 (b) 5 and 7 - 2 x + 4 y + z = 0, - 7 x + 14 y + 9 z = 0 such that
(c) 5 and 8 (d) 4 and 9 15 £ x 2 + y 2 + z2 £ 150. Then, the number of
26. If the system of linear equations elements in the set S is equal to ............ .
(JEE Main 2020)
x - 2 y + kz = 1, 2 x + y + z = 2 , 3x - y - kz = 3
has a solution ( x, y, z), z ¹ 0, then ( x, y) lies on the 32. If the system of equations
straight line whose equation is (JEE Main 2019) x - 2 y + 3z = 9, 2x + y + z = b, x - 7 y + az = 24,
(a) 3x - 4 y - 4 = 0 has infinitely many solutions, then a - b is equal to
(b) 3x - 4 y - 1 = 0 ............. . (JEE Main 2020)
(c) 4x - 3 y - 4 = 0
33. If the system of linear equations,
(d) 4x - 3 y - 1 = 0
x + y + z = 6, x + 2 y + 3z = 10, 3x + 2 y + lz = m
27. Let l be a real number for which the system of
has more than two solutions, then m - l2 is equal to
linear equations
…… . (JEE Main 2020)
x + y + z = 6, 4 x + ly - lz = l - 2
and 3x + 2 y - 4 z = - 5 34. Let P = [ aij ] be a 3 ´ 3 matrix and let Q = [ bij ],
has infinitely many solutions. Then l is a root of where bij = 2 i + j aij for 1 £ i, j £ 3. If the
the quadratic equation (JEE Main 2019) determinant of P is 2, then the determinant of the
(a) l2 - 3l - 4 = 0 matrix Q is ……… .
(b) l2 + 3l - 4 = 0
35. The number of distinct real roots of
(c) l2 - l - 6 = 0 ½ sin x cos x cos x ½
(d) l2 + l - 6 = 0 ½ cos x p p
sin x cos x ½ = 0 in the interval - £ x £
é5 2a 1ù ½ ½ 4 4
½ cos x cos x sin x ½
28. If B = ê 0 2 1 ú is the inverse of a 3 ´ 3 matrix
ê ú is ……… .
êëa 3 -1úû
½ 1 x x+1 ½
A, then the sum of all values of a for which det
36. If f ( x) =½ 2 x x ( x - 1) ( x + 1) x ½,
( A) + 1 = 0, is (JEE Main 2019) ½ ½
½ 3x ( x - 1) x ( x - 1) ( x - 2) ( x + 1) x ( x - 1) ½
(a) 0 (b) -1
(c) 1 (d) 2 then f (100) is equal to ……… .
572 JEE Main Mathematics

37. For a real number a, if the system é 0 2k - 1 k ù


é 1 a a ù é xù é 1 ù
2 ê ú
and B = ê 1 - 2 k 0 2 kú
ê úê ú ê ú
ê a 1 a ú ê y ú = ê -1ú ê - k -2 k
ë
0 ú
û
êëa 2 a 1 úû êë z úû êë 1 úû If det (adj A) + det (adj B) = 10 6 , then [ k] is equal to
of linear equations, has infinitely many solutions, ……… .
then 1 + a + a 2 = ……… .
39. The total number of distincts x Î R for which
38. Let k be a positive real number and let x x2 1 + x3
é 2k - 1 2 k 2 k ù
ê ú 2 x 4 x2 1 + 8 x 3 = 10 is ……… .
A=ê 2 k 1 - 2k ú 3x 9 x2 1 + 27 x 3
ê - 2 k 2k -1 ú
ë û

Answers
Round I
1. (a) 2. (d) 3. (d) 4. (a) 5. (a) 6. (d) 7. (b) 8. (c) 9. (d) 10. (a)
11. (c) 12. (a) 13. (b) 14. (c) 15. (b) 16. (a) 17. (d) 18. (a) 19. (b) 20. (a)
21. (a) 22. (b) 23. (c) 24. (b) 25. (d) 26. (a) 27. (a) 28. (c) 29. (a) 30. (a)
31. (b) 32. (a) 33. (b) 34. (c) 35. (a) 36. (c) 37. (d) 38. (b) 39. (d) 40. (c)
41. (d) 42. (c) 43. (d) 44. (b) 45. (b) 46. (d) 47. (c) 48. (b) 49. (b) 50. (a)
51. (d) 52. (d) 53. (b) 54. (b) 55. (b) 56. (c) 57. (c) 58. (d) 59. (d) 60. (a)
61. (a) 62. (b) 63. (a) 64. (c) 65. (b) 66. (a) 67. (c) 68. (b) 69. (d)

Round II
1. (c) 2. (b) 3. (c) 4. (c) 5. (d) 6. (a) 7. (d) 8. (a) 9. (b) 10. (a)
11. (b) 12. (d) 13. (a) 14. (b) 15. (c) 16. (a) 17. (a) 18. (a) 19. (b) 20. (c)
21. (b) 22. (b) 23. (c) 24. (a) 25. (c) 26. (c) 27. (c) 28. (c) 29. (a) 30. (d)
31. (8) 32. (5) 33. (13) 34. (8192) 35. (1) 36. (0) 37. (1) 38. (4) 39. (2)
Determinants 573

Solutions
Round I Þ (k - 6 2 ) (3z - 5x) = 0
(a + 1) (a + 2) a + 2 1 If 3z - 5x = 0 Þ 3(x + 2d ) - 5x = 0
1. Given, (a + 2) (a + 3) (a + 3) 1 Þ x = 3d (Not possible)
(a + 3) (a + 4) (a + 4) 1 Þ k = 6 2 Þ k2 = 72
C1 ® C1 - C 2 C 2 ® C 2 - C3 é R11 R12 R13 ù
a (a + 2) a+1 1 5. A = ê R21 R22 R23 ú
ê ú
(a + 3) (a + 1) a+2 1 êë R31 R32 R33 úû
(a + 4) (a + 2) a+3 1 é 2R11 2R12 2R13 ù
a 2 + 2a a+1 1 2 A = ê2R21 2R22 2R23 ú
ê ú
2
a + 4a + 3 a + 2 1 êë 2R31 2R32 2R33 úû
a 2 + 6a + 8 a + 3 1 R2 ® 2R2 + 5R3 , we get
R2 ® R2 - R1 and R3 ® R3 - R1 é 2R11 2R12 2R13 ù
B = ê4R21 + 10R31 4R22 + 10R32 4R23 + 10R33 ú
a 2 + 2a a+1 1 ê ú
2a + 3 1 0 êë 2R31 2R32 2R33 úû
4a + 8 2 0 R2 ® R2 - 5R3 , we get
é 2R11 2R12 2R13 ù
Þ 4a + 6 - 4a - 8 = - 2
B = ê4R21 4R22 4R23 ú
cos a cos b cos a sin b - sin a ê ú
êë 2R31 2R32 2R33 úû
2. Given, determinant = - sin b cos b 0
sin a cos b sin a sin b cos a 2R11 2R12 2R13
|B| = 4R21 4R22 4R23
Expanding corresponding to R1, we get
= cos a cos b (cos a cos b - 0) 2R31 2R32 2R33
- cos a sin b (- cos a sin b - 0) R11 R12 R13
- sin a (- sin 2 b sin a - cos 2 b sin a ) |B| = 2 ´ 2 ´ 4 R21 R22 R23
= cos 2 a cos 2 b + cos 2 a sin 2 b + sin 2 a [sin 2 b + cos 2 b ] R31 R32 R33
= cos 2 a (cos 2 b + sin 2 b ) + sin 2 a [sin 2 b + cos 2 b ] = 16 ´ 4
= cos 2 a (1) + sin 2 a [1] = 64
= cos 2 a + sin 2 a [Q sin 2 q + cos 2 q = 1] 6. Given function
=1
½ - sin 2 q -1 - sin 2 q 1 ½
f (q) = ½ - cos 2 q -1 - cos 2 q 1 ½
a b 0
3. We have, D= 0 a b =0 ½
½ ½
½
½ 12 10 -2 ½
b 0 a
½- sin 2 q -1 - sin 2 q 1 ½
Þ D = a (a 2 - 0) - b(0 - b2) = a3 + b3
= 2 ½- cos 2 q -1 - cos 2 q 1 ½
Þ a3 + b3 = 0 ½
½ ½
½
3 ½ 6 5 -1 ½
æaö
Þ ç ÷ = -1
è bø On applying R1 ® R1 + R3 and R2 ® R2 + R3 , we get
æaö ½6 - sin 2 q 4 - sin 2 q 0 ½
\ ç ÷ is one of the cube roots of - 1.
è bø f (q) = 2½6 - cos 2 q 4 - cos 2 q 0 ½
½
½ ½
½
3 4 2 x ½ 6 5 -1 ½
4. 4 5 2 y =0 = 2(-1)[(6 - sin 2 q)(4 - cos 2 q) - (4 - sin 2 q)(6 - cos 2 q)]
5 k z = - 2 [24 - 6 cos 2 q - 4 sin 2 q + sin 2 q cos 2 q
R2 ® R1 + R3 - 2R2 , we get -24 + 4 cos 2 q + 6 sin 2 q - sin 2 q cos 2 q]
3 4 2 x = - 2 [-2 cos 2 q + 2 sin 2 q] = 4 cos 2 q
Þ 0 k -6 2 0 =0 ép pù ép ù
As q Î ê , ú Þ 2 q Î ê , p ú
ë4 2û ë2 û
5 k z
574 JEE Main Mathematics

æpö b = 2 + d and c = 2 + 2d
\ f (q) minimum = f ç ÷ = - 4 = m
è2ø So, | A| = d (2d )d = 2d3 Î [2, 16] [given]
æpö Þ d3 Î [1, 8]
and f (q) maximum = fç ÷ =0 = M
è4ø Þ d Î [1, 2]
\ The ordered pair (m, M ) is (-4, 0). \ 2 + 2d Î [2 + 2, 2 + 4] = [4, 6]
x a + y x+ a Þ c Î [4, 6]
7. Given determinant D = y b + y y + b 9. Given, quadratic equation is x2 + x + 1 = 0 having roots
z c+ y z+ c a , b.
x a+ y x x a+ y a Then, a + b = -1 and ab = 1
= y b+ y y + y b+ y b Now, given determinant
z c+ y z z c+ y c y+1 a b
x a a x y a x 1 a D= a y+b 1
=0+ y b b + y y b = yy 1 b b 1 y+ a
z c c z y c z 1 c On applying R1 ® R1 + R2 + R3 , we get
y+1+ a+b y+1+ a+b y+1+ a+b
On applying C1 ® C1 + C3 , we get
x+ a 1 a D= a y+b 1
D = y y+ b 1 b b 1 y+ a
z+ c 1 c y y y
= a y+b 1 [Q a + b = -1]
It is given that a + x = b + y = c + z + 1 = l (let), so
l 1 a b 1 y+ a
D=y l 1 b On applying C 2 ® C 2 - C1 and C3 ® C3 - C1,
l–1 1 c we get
y 0 0
On applying R1 ® R1 – R3 and R2 ® R2 – R3 ,
we get D = a y+b-a 1-a
1 0 a–c b 1 -b y + a -b
D=y 1 0 b – c = y(–1)[(b – c) – (a – c)] = y(a – b) = y[( y + (b - a )) ( y - (b - a )) - (1 - a ) (1 - b )]
l–1 1 c [expanding along R1]
é1 1 1 ù = y [ y2 - (b - a )2 - (1 - a - b + ab )]
8. Given, matrix A = ê2 b c ú , so = y [ y2 - b 2 - a 2 + 2ab - 1 + (a + b ) - ab ]
ê ú
2 2
êë4 b c úû = y [ y2 - (a + b )2 + 2ab + 2ab - 1 + (a + b ) - ab ]
= y[ y2 - 1 + 3 - 1 - 1] = y3
½1 1 1 ½
det( A ) = ½2 b c ½ [Q a + b = -1 and ab = 1]
½ 2 2
½ é -2 4+ d (sin q) - 2 ù
½4 b c ½ ê ú
10. Given, A = 1 (sin q) + 2 d
On applying, C 2 ® C 2 - C1 and C3 ® C3 - C1, we get ê ú
êë 5 (2 sin q) - d (- sin q) + 2 + 2d úû
½1 0 0 ½
det( A ) = 2 b - 2 c - 2 ½
½ -2 4+ d (sin q) - 2
½ 2 2
½ \ | A| = 1 (sin q) + 2 d
½4 b - 4 c - 4½
5 (2 sin q) - d (- sin q) + 2 + 2d
½ b -2 c-2½
=½ 2 2
½ -2 4+ d (sin q) - 2
½ b - 4 c - 4½
= 1 (sin q) + 2 d
b -2 c-2
= 1 0 0
(b - 2)(b + 2) (c - 2)(c + 2)
[R3 ® R3 - 2R2 + R1 ]
½ 1 1 ½
= (b - 2)(c - 2)½ ½ = 1 [(4 + d )d - (sin q + 2) (sin q - 2)]
½ b + 2 c + 2½ [expanding along R3 ]
[taking common (b - 2) from C1 and (c - 2) from C 2] = (d 2 + 4d - sin 2 q + 4)
= (b - 2)(c - 2)(c - b) = (d 2 + 4d + 4) - sin 2 q
Since, 2, b and c are in AP, if assume common = (d + 2)2 - sin 2 q
difference of AP is d, then
Determinants 575

Note that| A|will be minimum if sin 2 q is maximum, 1 1 1


i.e. if sin 2 q takes value 1. = (a + b + c) 2b b - c - a 2b
Q | A|min = 8, 2c 2c c-a -b
therefore (d + 2)2 - 1 = 8 Þ (d + 2)2 = 9
[taking common (a + b + c) from R1]
Þ d + 2 = ± 3 Þ d = 1, - 5
Applying C 2 ® C 2 - C1 and C3 ® C3 - C1 , we get
é2 b 1ù 1 0 0
11. Given matrix A = b b + 1 bú , b > 0
ê 2
D = (a + b + c) 2b - (a + b + c)
ê ú 0
êë 1 b 2 úû 2c 0 - (a + b + c)
2 b 1
Now, expanding along R1, we get
So, det ( A ) =| A| = b b2 + 1 b
D = (a + b + c) 1. {(a + b + c)2 - 0 }
1 b 2
= (a + b + c)3 = (a + b + c)(x + a + b + c)2 (given)
2 2 2 2
= 2 [2(b + 1) - b ] - b(2b - b) +1(b - b - 1) Þ (x + a + b + c)2 = (a + b + c)2
= 2[2b2 + 2 - b2] - b2 - 1 Þ x + a + b + c = ± (a + b + c)
= 2b2 + 4 - b2 - 1 = b2 + 3 Þ x = - 2(a + b + c) [Q x ¹ 0]
det( A ) b2 + 3 3 n n
14. Given, f (n ) = a + b , f (1) = a + b,
Þ = = b+
b b b
f (2) = a 2 + b 2, f (3) = a3 + b3 , f (4) = a 4 + b 4
Now, by AM ³ GM, we get
3 1 + f (1) 1 + f (2)
3
b+ 1/ 2 Let D = 1 + f (1) 1 + f (2) 1 + f (3)
b ³ æç b ´ 3 ö÷ [Q b > 0 ]
2 è bø 1 + f (2) 1 + f (3) 1 + f (4)
3 3 1 + a + b 1 + a2 + b2
Þ b+ ³2 3
b Þ D = 1 + a + b 1 + a 2 + b 2 1 + a3 + b3
det ( A )
So, minimum value of = 2 3. 1 + a 2 + b 2 1 + a3 + b3 1 + a 4 + b 4
b
é 1 sin q 1 ù 1 ×1 + 1 ×1 + 1 ×1 1 ×1 + 1 × a + 1 ×b
12. Given matrix A = ê - sin q 1 sin qú = 1 ×1 + a ×1 + b ×1 1 ×1 + a × a + b ×b
ê ú
êë -1 - sin q 1 úû 1 × 1 + 1 × a2 + 1 × b2 1 × 1 + a2 × a + b2 × b
1 sin q 1 1 × 1 + 1 × a2 + 1 × b2
Þ det( A ) =| A|= - sin q 1 sin q 1 × 1 + a × a2 + b × b2
-1 - sin q 1 1 × 1 + a2 × a2 + b2 × b2
1 1 1 1 1 1
= 1(1 + sin q) - sin q(- sin q + sin q) + 1(sin 2 q + 1)
2
= 1 a b 1 a a2
Þ | A| = 2 (1 + sin 2 q) …(i)
1 a2 b2 1 b b2
æ 3 p 5 p ö
As we know that, for q Î ç , ÷ 2
è 4 4 ø 1 1 1 1 1 1 1 1 1
æ 1 1 ö = 1 a b 1 a b = 1 a b
sin q Î ç - , ÷
è 2 2ø 1 a2 b2 1 a2 b2 1 a2 b2
é 1ö é 1 ö On expanding, we get
Þ sin 2 q Î ê0, ÷ Þ 1 + sin 2 q Î ê 0 + 1, + 1÷
ë 2ø ë 2 ø D = (1 - a )2 (1 - b )2 (a - b )2
é 3 ö But given, D = K (1 - a )2 (1 - b )2 (a - b )2
Þ 1 + sin 2 q Î ê1, ÷ Hence, K (1 - a )2 (1 - b )2 (a - b )2
ë 2ø
æ3 ù = (1 - a )2 (1 - b )2 (a - b )2
Þ 2(1 + sin 2q) Î [2, 3) Þ| A| Î [2, 3) Ì ç , 3ú \ K =1
è2 û
a-b-c 2a 2a x+ a x+2 x+1
13. Let D = 2b b-c-a 2b 15. Given, f (x) = x + b x + 3 x + 2
2c 2c c-a -b x+ c x+4 x+3
On applying R1 ® R1 - 2R2 + R3 , we get
Applying R1 ® R1 + R2 + R3 , we get
a - 2b + c 0 0
a+ b+ c a+ b+ c a+ b+ c
f (x) = x+ b x+3 x+2
D= 2b b-c-a 2b
x+ c x+4 x+3
2c 2c c-a -b
Q a - 2b + c = 1 so, (given)
576 JEE Main Mathematics

1 0 0 [Q a1 , a 2, a3 ....... , a10 are in GP, therefore put


f (x) = x + b x + 3 x + 2 a1 = a , a 2 = aR, a3 = aR2, ... , a10 = aR9 ]
x+ c x+4 x+3 æ a r + kRr + 2k ö
log e a r + kRk log e ç ÷
On applying R2 ® R2 - R3 , we get è a r + k Rk ø
1 0 0 æ a r + k R4 r + 5 k ö
Þ log e a r + kR3 r + 4k log e ç r + k 3 r + 4k ÷
f (x) = b - c -1 -1 èa R ø
x+ c x+4 x+3 æa r + k
R7 r + 8 k ö
log e a r + kR6r + 7k log e ç r + k 6r + 7k ÷
On applying C3 ® C3 - C 2, we get èa R ø
1 0 0 æ a r + kR2r + 3 k ö
f (x) = b - c -1 0 =1 log e ç ÷
è a r + k Rk ø
x + c x + 4 -1 æ a r + kR 5 r + 6k ö
log e ç r + k 3 r + 4k ÷ = 0
So, f (50) = 1 èa R ø
Hence, option (b) is correct. æ a r + k R8 r + 9 k ö
x -6 -1 log e ç r + k 6r + 7k ÷
èa R ø
16. Given equation 2 - 3x x - 3 = 0
log e (a r + kRk ) log e Rr + k log e R2r + 2k
- 3 2x x + 2
Þ log e a r+ kR3 r+ 4k log e Rr + k log e R2r + 2k = 0
On expansion of determinant along R1, we get log e a r+ kR6r+ 7k log e Rr + k log e R2r + 2k
x [(- 3x) (x + 2) - 2x(x - 3)] + 6 [2(x + 2) + 3(x - 3)]
- 1 [2(2x) - (- 3x) (- 3)] = 0 log e (a r + kRk ) log e Rr + k 2 log e Rr + k
Þ x [- 3x2 - 6x - 2x2 + 6x] + 6[2x + 4 + 3x - 9] Þ log e (a r + kR3 r + 4k ) log e Rr + k 2 log e Rr + k = 0
- 1 [4x - 9x] = 0 log e (a r + kR6r + 7k ) log e Rr + k 2 log e Rr + k
Þ x(- 5x2) + 6(5x - 5) - 1(- 5x) = 0 [Q log mn = n log m and here
Þ -5x3 + 30x - 30 + 5x = 0 log e R2r + 2k = log e R2( r + k) = 2 log e Rr + k ]
3 3
Þ 5x - 35x + 30 = 0 Þ x - 7x + 6 = 0 Q Column C 2 and C3 are proportional,
Since all roots are real So, value of determinant will be zero for any value of
coefficient of x2 (r , k), r , k Î N .
\ Sum of roots = - =0
coefficient of x3 \Set ‘S’ has infinitely many elements.
log e a1ra 2k log e a 2ra3k log e a3r a 4k (a x + a - x )2 (a x - a - x )2 1
17. Given, log e a 4r a5k log e a5r a 6k log e a 6r a7k =0 18. (bx + b-x )2 (bx - b-x )2 1
log e a7ra 8k log e a 8r a 9k log e a 9r a10
k (cx + c- x )2 (cx - c- x )2 1

On applying elementary operations Applying C1 ® C1 - C 2


C 2 ® C 2 - C1 and C3 ® C3 - C1, we get 4 (a x - a - x )2 1
log e a1ra 2k log e a 2ra3k - log e a1ra 2k = 4 (bx - b- x )2 1
log e a 4r a5k log e a5r a 6k - log e a 4r a5k 4 (cx - c-x )2 1
log e a7ra 8k log e a 8r a 9k - log e a7ra 8k 1 (a x - a - x )2 1
= 4 1 (bx - b- x )2 1 = 0
log e a3r a 4k - log e a1ra 2k
1 (cx - c- x )2 1
log e a 6r a7k - log e a 4r a5k = 0
log e a 9r a10
k
- log e a7ra 8k [Q two columns are identical]
19. Applying C3 ® C3 - C1, we get
æ a ra k ö æ arak ö
log e a1ra 2k log e ç 2r 3k ÷ log e ç 3r 4k ÷ 1 a a2 - 1
è a1 a 2 ø è a1 a 2 ø
æa a ö
r k æ arak ö D = cos ( p - d )a cos pa 0
Þ log e a 4r a5k log e ç 5r 6k ÷ log e ç 6r 7k ÷ = 0 sin ( p - d )a sin pa 0
è a 4a5 ø è a 4a5 ø
æ a 8r a 9k ö k ö
æ a 9r a10 = (a 2 - 1 ){ - cos pa sin ( p - d )a
log e a7ra 8k log e ç r k ÷ log e ç r k ÷ + sin pa cos ( p - d )a }
è a7 a 8 ø è a7 a 8 ø
= (a 2 - 1 ) sin { - ( p - d )a + pa }
é æmöù Þ D = (a 2 - 1 ) sin da
êQ log e m - log e n = log e çè n ÷ø ú
ë û which is independent of p.
Determinants 577

a a 2 a3 + 1 a-x c b
20. Given determinant, D = b b 2 3
b + 1 =0 23. Given, c b-x a =0
c c2 c3 + 1 b a c- x
Applying R1 ® R1 + R2 + R3
On splitting the determinant into two determinants,
we get 1 1 1
Þ (a + b + c - x ) c b - x a =0
1 a a2 1 a a2
b a c- x
D = abc 1 b b + 1 b b2 = 0
2
1 0 0
1 c c2 1 c c2
Þ (a + b + c - x ) c b - x - c a-c =0
Þ (1 + abc) [1 (bc2 - cb2) - a (c2 - b2) + a 2(c - b)] = 0 b a-b c- x- b
Þ (1 + abc) [(a - b) (b - c) (c - a )] = 0 Þ (a + b + c - x ) [1(b - x - c)(c - x - b)
Since, a , b and c are different, the second factor cannot - (a - c)(a - b)] = 0
be zero. Þ (a + b + c - x )[bc - xb - b2 - xc + x2 + bx
Hence, 1 + abc = 0. - c2 + cx + bc - (a 2 - ab - ac + bc)] = 0
x+ a b g Þ (a + b + c - x )[x2 - a 2 - b2 - c2 + ab + bc + ca ] = 0
21. We have, g x+b a =0 Þ x = a + b + c or x2 = a 2 + b2 + c2 + ab + bc + ca
a b x+ g 1
Þ x = 0 or x2 = a 2 + b2 + c2 + (a 2 + b2 + c2)
Applying C1 ® C1 + C 2 + C3 2
x+ a+b+ g b g 3 2
Þ x = 0 or x = ± (a + b2 + c2)
Þ x+ a+b+ g x+b a =0 2
x+ a+b+ g b x+ g a3 + ax a 2b a 2c
1 2 3
1b g 24. We have, D= ab b + bx b2c
abc
Þ (x + a + b + g ) 1 x + b a =0 c2a 2
cb 3
c + xc
1 b x+ g Taking a , b and c common in columns Ist, IInd and
IIIrd, we get
Applying R2 ® R2 - R1 and R3 ® R3 - R1
a2 + x a2 a2
1 b g 2 2
D= b b +x b2
Þ (x + a + b + g ) 0 x a - g = 0
c2 c2 c2 + x
0 0 x
Applying R1 ® R1 + R2 + R3
Þ (x + a + b + g ) (x2 - 0) = 0 1 1 1
Þ x = 0 or x = - (a + b + g ) = (a 2 + b2 + c2 + x ) b2 b2 + x b2
1 4 20 c2 c2 c2 + x
22. We have, 1 -2 5 =0 Applying C 2 ® C 2 - C1 , C3 ® C3 - C1
1 2 x 5 x2 1 0 0
0 6 15 = (a 2 + b2 + c2 + x ) b2 x 0
Þ 0 -2 - 2x 5(1 - x ) = 0 2
c2 0 x
1 2x 5 x2 = x2 (a 2 + b2 + c2 + x)
[R1 ® R1 - R2 and R2 ® R2 - R3 ] Hence, D is divisible by x 2 as well as x.
0 1 1 a b - y c- z
Þ 3 × 2 × 5 0 - (1 + x ) 1 - x2 = 0 25. a - x b c- z = 0
1 x x2 a -x b- y c
[taking common 3 from R1 , 2 from C 2, 5 from C3 ] Applying R2 ® R2 - R1 and R3 ® R3 - R2
0 1 1 a b - y c- z
Þ (1 + x) 0 -1 1 - x = 0 Þ -x y 0 =0
1 x x2 0 -y z
Þ a ( yz ) + x (bz - yz + cy - yz ) = 0
Þ (1 + x) (2 - x) = 0
Þ ayz + bzx + cyx = 2xyz
Þ x + 1 = 0 or x - 2 = 0
a b c
Þ x = - 1, 2 Þ + + =2
x y z
578 JEE Main Mathematics

1 x y Applying C3 ® C3 + C 2
26. Let D = 2 sin x + 2x sin y + 2 y 6 1 6
N (N + 1)
3 cos x + 3x cos y + 3 y = 4N + 2 2N + 1 4N + 2
12
1 x y 3N (N + 1) 3N 2 3N (N + 1)
= 0 sin x sin y =0 [Q two columns are identical]
0 cos x cos y 1 1 1
[R2 ® R2 - 2R1 and R3 ® R3 - 3R1 ] 30. Let f (x) = 2x (x - 1 ) x
= sin x cos y - cos x sin y = sin(x - y) 3x (x - 1 ) (x - 1 ) (x - 2) x (x - 1 )
2
a a 1 1 1 1
27. Let D = cos(nx ) cos(n + 1 )x cos(n + 2)x = (x - 1 ) 2x x - 1 x
sin(nx ) sin(n + 1 )x sin(n + 2) x 3x x - 2 x
Since, cos(nx) + cos(n + 2)x = 2 cos(n + 1 ) x cos x Applying C1 ® C1 - C3 and C 2 ® C 2 - C3
and sin(nx) + sin(n + 2) x = 2 sin(n + 1 )x cos x 0 0 1
Applying C1 ® C1 - 2 cos x × C 2 + C3 = (x - 1 ) x - 1 x = (x - 1 ) [-2x + 2x] = 0
2
a - 2 a cos x + 1 a 1 2x - 2 x
\ D= 0 cos(n + 1 )x cos(n + 2)x \ f (x) = 0 Þ f (50) = 0
0 sin(n + 1 )x sin(n + 2) x 31. Operating R1 ® R1 + R2 + R3 , we get the determinant
2
= (a - 2 a cos x + 1 ) [cos(n + 1 ) x sin(n + 2)x 3+ a 3+ a 3+ a 1 1 1
- cos(n + 2) x sin(n + 1)x] = 1 1+ a 1 = (3 + a ) 1 1 + a 1
2
= (a - 2 a cos x + 1 )sin x 1 1 1+ a 1 1 1+ a
\ D is independent of n. 1 0 0
2 r -1
3 r -1
4 r -1 Operate C3 ® C3 - C1 , C 2 ® C 2 - C1 = (3 + a ) 1 a 0
28. We have, Dr = x y z 1 0 a
2n - 1 (3n - 1 ) /2 (4n - 1 ) /3 æ 3ö
(3 + a )a 2 = a 3 ç1 + ÷
n n n
è aø
å 2r - 1 å 3r - 1 å 4r - 1 1 log x y log x z
n r =1 r =1 r =1
32. Let D = log y x 1 log y z
Þ å Dr = x y z
r =1
2n - 1 (3n - 1 ) / 2 (4n - 1 ) / 3 log z x log z y 1
= 1(1 - log z y log y z ) - log x y (log y x - log y z log z x )

2n - 1 (3n - 1 ) /2 (4n - 1 )/3 + log x z (log y x log z y - log z x )


n
= (1 - log z z ) - log x y (log y x - log y z log z x )
Þ å Dr = x y z
r =1
2n - 1 (3n - 1 )/2 (4n - 1 )/3 + log x z (log y x log z y - log z x )
n = (1 - 1 ) - (1 - log x y log y x ) + (log x z log z x - 1 ) = 0
Þ å Dr = 0 [Q two rows are same] (since, log x y log y x = 1 )
r= 1
= 0 - (1 - 1 ) + (1 - 1 ) = 0
Sn 1 5
N 1 a a2
29. å U n = Sn 2 2 N + 1 2 N + 1 33. Given, f (a ) = a a2 1
n =1
Sn3 3N 2 3N
a2 1 a
N (N + 1 ) = 1 (a - 1 ) - a (a 2 - a 2) + a 2(a - a 4 )
3
1 5
2 = a3 - 1 - 0 + a3 - a 6
N (N + 1) (2N + 1 )
= 2N + 1 2N + 1 Þ f ( 3 = 3 - 1 - 0 + 3 - 32 = 6 - 10 = - 4
3
6
2
ì N (N + 1)ü 34. Q det (M r ) = r 2 - (r - 1)2 = 2 r - 1
í ý 3N 2 3N
î 2 þ \det (M1 ) + det (M 2) + . . . + det (M 2008 )
6 1 5 = 1 + 3 + 5 + . . . + 4015
N (N + 1)
= 4N + 2 2N + 1 2N + 1 2008
12 = [2 + (2008 - 1) 2 ] = 2008(2008) = (2008)2
3N (N + 1) 3N 2 3N 2
Determinants 579

a a+b
a+ b+ c 39. If (x1 , y1 ), (x2, y2) and (x3 , y3 ) are the vertices of a
35. We have, D = 3a 4a + 3b 5a + 4b + 3c triangle, then
6a 9a + 6b 11a + 9b + 6c x1 y1 1
1
Applying R2 ® R2 - 3R1 , R3 ® R3 - 2 R2 area = x2 y2 1 ...(i)
2
a a a+ b+ c x3 y3 1
= 0 a 2a + b Also, we know that if a be the length of an equilateral
0 a a+b triangle, then
= a [a 2 + ab - 2a 2 - ab] 3 2
area = a ...(ii)
=-a =i 3
[Q a = i, given] 4
\ From Eqs. (i) and (ii), we get
36. Let the first term and common difference of an AP are A
and D, respectively. x1 y1 1
3 2 1
\ a = A + ( p - 1 ) D , b = A + (q - 1 ) D, a = x2 y2 1
4 2
and c = A + (r - 1 ) D x3 y3 1
a p 1 A + ( p - 1 )D p 1 x1 y1 1
3 2
Now, b q 1 = A + (q - 1 )D q 1 Þ a = x2 y2 1
2
c r 1 A + (r - 1 )D r 1 x3 y3 1
Applying C1 ® C1 - DC 2 + DC3 On squaring both sides, we get
2
A p 1 1 p 1 x1 y1 1
3 4
= A q 1 = A 1 q 1 =0 a = x2 y2 1
4
A r 1 1 r 1 x3 y3 1
[Q two columns are identical] é3 4 ù
40. Given, A=ê ú
x+2 x+3 x+ a ë5 7û
37. Let D = x + 4 x + 5 x + b Þ | A| = 1
x+6 x+ 7 x+ c é 3 4 ù é 7 -4 ù
\ A adj ( A ) = ê úê
Applying C 2 ® C 2 - C1, we get ë 5 7 û ë -5 3úû
x+2 1 x+ a
é1 0ù é1 0ù
D= x+4 1 x+ b =ê ú = 1 ê0 1ú = | A| I
ë 0 1 û ë û
x+6 1 x+ c
é 1 -1 1ù
Applying R2 ® R2 - R1 and R3 ® R3 - R1
41. Since, A = 0 2 -3 ú
ê
x+2 1 x+ a ê ú
êë2 1 0úû
D= 2 0 b-a
4 0 c-a é 3 1 1ù
\ B = adj ( A ) = ê -6 -2 3ú
= - 1(2 c - 2 a - 4b + 4a ) ê ú
êë -4 -3 2úû
Þ D = 2 (2b - c - a ) …(i)
Since, a , b and c are in AP. é 5 -5 5ù
a+c Þ adj (B) = 0 10 -15ú
ê
\ b= [from Eq. (i)] ê ú
2 êë10 5 0 úû
D = 2(a + c - c - a ) = 0 5 -5 5
38. Required area = 9 sq units Þ |adj (B)|= 0 10 -15 = 625
-3 0 1 10 5 0
1
Þ 3 0 1 =9
2 Given that, C = 5 A
0 k 1 1 -1 1
Þ - k (- 3 - 3) = 18 Þ 3
|C| = 5 | A| = 125 0 2 -3 = 625
Þ - k(-6) = 18 2 1 0
Þ 6k = 18 |adj (B)| 625
Hence, = =1
Þ k =3 |C| 625
580 JEE Main Mathematics

1 2 -1 é 1 2 -3 ù
42. | A|= -1 1 2 = 14 46. Let A = ê0 1 2ú
ê ú
2 -1 1 êë0 0 1úû

Q |adj (adj A )| = | A |( n - 1)
2
é 1 2 -3 ù
1 2
\ | A| = ê0 1 2 ú = =1
= 144 ê ú 0 1
êë0 0 1 úû
é -1 -2 -2 ù
é 1 -2 7 ù
43. Given, A=ê 2 1 -2 ú
ê ú and adj A = ê0 1 -2ú
êë 2 -2 1úû ê ú
êë0 0 1 úû
é C11 C12 C13 ù é -3 -6 -6ù
é 1 -2 7ù
\ B = êC 21 C 22 C 23 ú = ê 6 3 -6 ú 1
ê ú ê ú Hence, A -1 = adj A = ê0 1 -2 ú
êë C31 C32 C33 úû êë 6 -6 3úû | A| ê ú
êë0 0 1 úû
é -3 6 6ù é -1 2 2ù
-1
ê ú ê So, required element = A13 =7
Þ adj A = (B)¢ = -6 3 -6 = 3 -2 1 -2 ú
ê ú ê ú
êë -6 -6 3úû êë -2 -2 1úû 47. Given, | ABAT| = 8
= 3 A¢ Þ | A||B|| AT| = 8 [Q|XY | = |X ||Y |]
- bù \ | A|2|B| = 8 [Q| AT| = | A|] …(i)
é5a
44. Given, A = ê and A adj A = AAT
ë3 2 úû Also, we have
| AB-1| = 8 Þ| A||B-1| = 8
Clearly, A (adj A ) = A I 2
| A| é -1 -1 1 ù
[Q if A is square matrix of order n, then Þ =8 êëQ| A |=| A| = | A|úû …(ii)
|B|
A (adj A ) = (adj A ) × A = A I n ]
5a - b é1 0ù On multiplying Eqs. (i) and (ii), we get
= I 2 = (10a + 3b) I 2 = (10a + 3b) ê ú | A|3 = 8 × 8 = 43
3 2 ë0 1û
Þ | A| = 4
é10a + 3b 0 ù
=ê ...(i) | A| 4 1
ë 0 10a + 3búû Þ |B| = = =
8 8 2
é5a - bù é 5a 3ù é25a 2 + b2 15a - 2bù 1 æ1ö 1 æ1ö 1
and AAT = ê ê - b 2ú = ê ú Now, |BA -1BT| = |B| |B| = ç ÷ ç ÷ =
ë3 2 úû ë û ë 15a - 2b 13 û | A| è 2 ø 4 è 2 ø 16
...(ii) et e- t cos t e- t sin t
Q A (adj A ) = AAT 48. | A | = e t
-e -t
cos t - e -t
sin t -e -t
sin t + e- t cos t
-t
é10a + 3b 0 ù é25a 2 + b2 15a - 2bù e t
2e sin t - 2e- t cos t
\ ê =ê ú
ë 0 10a + 3búû ë 15a - 2b 13 û 1 cos t sin t
t -t -t
[using Eqs. (i) and (ii)] = (e ) (e ) (e ) 1 - cos t - sin t - sin t + cos t
2b 1 2 sin t - 2 cos t
Þ 15a - 2b = 0 Þ a = ...(iii)
15
[taking common from each column]
and 10a + 3b = 13 ...(iv)
Aplying R2 ® R2 - R1 and R3 ® R3 - R1, we get
On substituting the value of ‘a’ from Eq. (iii) in
Eq. (iv), we get 1 cos t sin t
20b + 45b = e- t 0 - 2 cos t - sin t - 2 sin t + cos t
æ 2b ö
10 × ç ÷ + 3b = 13 Þ = 13 0 2 sin t - cos t - 2 cos t - sin t
è 15 ø 15
65b [Q et - t = e0 = 1]
Þ = 13 Þ b = 3 -t
15 =e ((2 cos t + sin t ) + (2 sin t - cos t )2)
2

Now, substituting the value of b in Eq. (iii), we get [expanding along column 1]
5a = 2 = e- t (5 cos 2 t + 5 sin 2 t )
Hence, 5a + b = 2 + 3 = 5 = 5e- t [Q cos 2 t + sin 2 t = 1]
45. A 2 - A + I = 0 Þ I = A - A 2 Þ I = A (I - A ) Þ | A | = 5e- t ¹ 0 for all t Î R
\ A is invertible for all t Î R.
Þ A -1 = I - A
[Q if| A | ¹ 0, then A is invertible]
Determinants 581

é1 1ù é1 2ù é1 3ù ... é1 n - 1ù é1 78ù é cos x sin x 0ù


49. Given, ê úê =
ë0 1û ë0 1úû ê0 1ú ê0
ë û ë 1 úû êë0 1 úû Also, F -1 (x) = ê - sin x cos x 0ú = F (- x)
ê ú
é1 1ù é1 2ù é1 2 + 1ù êë 0 0 1úû
ê0 1ú ê0 = ,
1úû êë0 1 úû
Q
ë û ë é cos y 0 - sin yù
é1 2 + 1ù é1 3ù é1 3 + 2 + 1ù and G -1
( y) = ê 0 1 0 ú = G (- y)
= ê ú
ê0 1 úû ê0 1úû êë0 1 ú, êë sin y 0 cos y úû
ë ë û
: : : Þ [F (x) × G ( y)]-1 = G -1 ( y) F -1 (x) = G (- y) F (- x)
: : : 53. We must have AA -1 = I
é1 1ù é1 2ù é1 3ù é1 n - 1ù (3, 1)th entry of AA -1 = I = (1, 3)th entry of AA -1
\ê úê úê ú ... ê
ë0 1û ë0 1û ë0 1û ë0 1 úû 1 5 1 1
Þ 3 ´ + a ´ (-4) + 1 ´ = 0 = 0 ´ + 1 + c + 2 ´
é1 (n - 1) + (n - 2)+ ...+3 + 2 + 1ù 2 2 2 2
=ê ú
ë0 1 û Þ -4a + 4 = 0 and c + 1 = 0
é n (n - 1 ) ù é1 78ù Þ a = 1 and c = - 1
1
=ê 2 ú=ê ú 54. Given, AB = I Þ B = A -1
ê0 ú ë0 1 û
ë 1 û
é qù
ê 1 - tan
Since, both matrices are equal, so equating 2ú
corresponding element, we get ê q ú
n (n - 1) ê tan 1 ú
= 78 Þ n (n - 1) = 156 adj A ë 2 û
Now, A -1 = =
2 | A| q
1 + tan 2
= 13 ´ 12 = 13(13 - 1) Þ n = 13 2
é1 13ù é 1 - 13 ù AT q
A=ê -1 = = cos 2 AT
So, ú = A = ê0 1 ú q 2
ë0 1 û ë û sec2
2
éa bù é d -bù ù
[Q if| A|= 1 and A = ê ú , then A -1 = ê úú 55. The given system equations will have a non-trivial
ëc dû ë -c a û û solution, if
50. Given that, A -1 = l (adj A ) aa + b a b
1 D = ba + c b c =0
On comparing with A -1 = adj ( A ), we get
| A| 0 aa + b ba + c
1 Þ - (aa 2 + 2ba + c) (ac - b2) = 0
l=
| A| b2 = ac
0 3 1 \ a , b, c are in GP.
Now,| A| = =0 -6 = -6Þ l = -
2 0 6 56. For no solution or infinitely many solutions
é 1 -1 2 ù a 1 1
51. Since, A = ê2 0 1ú 1 a 1 = 0 Þ a = 1, a = - 2
ê ú
êë3 2 1úû 1 1 a
Now,| A |= 1 (0 - 2) + 1 (2 - 3) + 2 (4 - 0) = 5 But for a = 1, clearly there are infinitely many solutions
é -2 5 -1 ù and when we put a = - 2 in given system of equations
1
\ A -1 = ê 1 - 5 3ú and adding them together LHS ¹ RHS Þ No solutions.
5ê ú
êë 4 -5 2úû 57. It has a non-zero solution, if
é -2 5 -1 ù é 3 ù 1 k -1

Now, A B = -1
1 -5 3ú ê1ú 3 - k -1 = 0
5ê úê ú
êë 4 -5 2úû êë4úû 1 -3 1
é x1 ù é -1ù Þ 1(- k - 3) - k (3 + 1 ) - 1 (-9 + k ) = 0
Þ ê x ú = ê 2ú Þ - 6k + 6 = 0 Þ k = 1
ê 2ú ê ú
êë x3 úû êë 3 úû 58. Given system of equations are
52. { F (x) × G ( y)} -1
=G -1
( y) × F -1
(x), if|F (x)|¹ 0,|G (x)|¹ 0 3x - 2 y + z = 0
lx - 14 y + 15z = 0
Here, | f (x)|= 1,|G ( y)| = 1
and x + 2 y - 3z = 0
582 JEE Main Mathematics

The system of equations has infinitely many Þ 2k2 – 18 = 0


(non-trivial) solutions, if D = 0. Þ k2 = 9 Þ k = 3
3 -2 1
\The Eqs. (i) and (ii) provide us
Þ D = l - 14 15 = 0
x + y + 3z = 0
1 2 -3
x + 3 y + 9z = 0
Þ 3(42 - 30) - l (6 - 2) + 1(-30 + 14) = 0
Þ 2 y + 6z = 0
Þ 36 - 4l - 16 = 0 Þ l = 5
y
3 -2 - k Þ = –3 and x = 0
z
59. D = 2 -4 -2 = 0
y
1 2 -1 \ x + = 0 – 3 = –3
z
3 (4 + 4) + 2 (- 2 + 2) - k (4 + 4) = 0 62. It is given that the three planes
Þ k =3
x + 4 y - 2z = 1,
10 -2 -3
x + 7 y - 5z = b
D x = 6 -4 -2 ¹ 0
and x + 5 y + az = 5
5m 2 -1
having intersection is a line in R3 , so
10 (4 + 4) + 2 (- 6 + 10m) - 3 (12 + 20m) ¹ 0
D =0
80 - 12 + 20m - 36 - 60m ¹ 0
1 4 -2
4
40m ¹ 32 Þ m ¹ Þ 1 7 -5 =0
5
3 10 -3 1 5 a
D y = 2 6 -2 ¹ 0 Þ 1(7a + 25) - 4(a + 5) - 2(5 - 7) = 0
1 5m -1 Þ 7a + 25 - 4a - 20 + 4 = 0
3 (-6 + 10m) - 10 (- 2 + 2) - 3 (10m - 6) ¹ 0 Þ 3a + 9 =0
- 18 + 30m - 30m + 18 ¹ 0 Þ 0 Þ a = - 3 and D3 = 0
[the value of Dy] 1 4 1
3 -2 10 Þ 1 7 b =0
D z = 2 -4 6 ¹0 1 5 5
1 2 5m Þ 1 (35 - 5 b ) - 4(5 - b ) + 1(5 - 7) = 0
3 (- 20m - 12) + 2 (10m - 6) + 10 (4 + 4) = - 40m + 32 ¹ 0 Þ 35 - 5 b - 20 + 4 b - 2 = 0
4 Þ b = 13
Þ m¹
5 \ a + b = -3 + 13 = 10
60. Since, the given equations are consistent. -4 7
1
2 3 1 63. Here, D = 0 3 -5
\ 3 1 -2 = 0 -2 5 -9
a 2 -b = 1(- 27 + 25) + 4(0 - 10) + 7(0 + 6)
Þ 2 (- b + 4) - 3 (-3b + 2a ) + 1(6 - a ) = 0 [expanding along R1]
Þ -2 b + 8 + 9 b - 6 a + 6 - a = 0 = - 2 - 40 + 42 = 0
Þ 7b - 7a = - 14 Þ a - b = 2 \The system of linear equations have infinite many
solutions.
61. Given system of linear equations
[Q system is consistent and does not have unique
x + y + 3z = 0 …(i) solution as D = 0]
x + 3 y + k2z = 0 …(ii) Þ D1 = D2 = D3 = 0
3x + y + 3z = 0 …(iii) g -4 7
has a non-zero solutions, so Now, D1 = 0 Þ h 3 - 5 = 0
1 1 3 k 5 -9
D = 0 Þ 1 3 k2 = 0
Þ g (- 27 + 25) + 4(- 9h + 5k) + 7(5h - 3k) = 0
3 1 3
Þ - 2 g - 36h + 20k + 35h - 21k = 0
Þ 1(9 – k2) – 1(3 – 3k2) + 3(1 – 9) = 0 Þ - 2g - h - k = 0
Þ 9 – k2 – 3 + 3k2 – 24 = 0 Þ 2g + h + k = 0
Determinants 583

64. Given system of linear equations Apply R1 ® R1 - R3 and R2 ® R2 - R3


x+ y+ z =5 …(i) 1 0 -1
x + 2 y + 2z = 6 …(ii) 0 1 -1 ¹0
x + 3 y + lz = m …(ii) a b 2
(l, m Î R) Þ 1(2 + b ) - 0(0 + a ) - 1(0 - a ) ¹ 0
The above given system has infinitely many solutions, Þ a + b + 2 ¹0 … (i)
then the plane represented by these equations Note that, only (2, 4) satisfy the Eq. (i)
intersect each other at a line, means
\ Option (a) is correct.
(x + 3 y + lz - m ) = p(x + y + z - 5) + q (x + 2 y + 2z - 6)
1 1 1
= ( p + q)x + ( p + 2q) y + ( p + 2q)z - (5 p + 6q)
67. Q D= 1 a 1 = 1 (a - b) - 1 (1 - a ) + 1 (b - a 2)
On comparing, we get
a b 1
p + q = 1, p + 2q = 3, p + 2q = l and 5 p + 6q = m
= - (a - 1)2
So, ( p, q) = (-1, 2)
1 1 1
Þ l = 3 and m = 7 Þ l + m = 3 + 7 = 10
D1 = 1 a 1 = 1 (a - b) - 1 (1) + 1 (b) = (a - 1)
65. Given system of linear equations
0 b 1
2x + 2ay + az = 0 …(i)
1 1 1
2x + 3by + bz = 0 …(ii),
D 2 = 1 1 1 = 1 (1) - 1 (1 - a ) + 1 (0 - a ) = 0
and 2x + 4cy + cz = 0 …(iii)
a 0 1
where a , b, c Î R are non-zero and distinct, has a
non-zero solution, means 1 1 1
½2 2a a½ and D3 = 1 a 1 = 1 ( - b) - 1 (- a ) + 1 (b - a 2)
D = 0 Þ ½2 3b b½ = 0 a b 0
½ ½
½2 4c c½ = - a (a - 1)
C For a =1
On applying C1 ® 1 , we get
2 D = D1 = D 2 = D3 = 0
½1 2a a½ D for b = 1 only
Þ ½1 3b b½ = 0 x + y + z = 1, x + y + z = 1 and x + y + z = 0
½ ½
½1 4c c½ i.e. no solution [Q RHS is not equal]
On applying R2 ® R2 - R1 and R3 ® R3 - R1, we get Hence, for no solution b = 1 only
½1 2a a ½ 68. We have, x + ky + 3z = 0; 3x + ky - 2z = 0;
Þ ½0 3b - 2a b - a½ = 0
½ ½ and 2x + 4 y - 3z = 0
½0 4c - 2a c - a½ System of equation has non-zero solution, if
Expension with respect to first column ½1 k 3 ½
Þ (3b - 2a )(c - a ) - (b - a )(4c - 2a ) = 0 ½ 3 k -2 ½ = 0
½ ½
Þ (3bc - 3ba - 2ac + 2a 2) - (4bc - 2ba - 4ac + 2a 2) = 0
½ 2 4 -3 ½
Þ - bc - ab + 2ac = 0
Þ (-3k + 8) - k(-9 + 4) + 3(12 - 2k) = 0
Þ 2ac = ab + bc
2 1 1 Þ -3k + 8 + 9k - 4k + 36 - 6k = 0
Þ = + [on dividing by abc]
b c a Þ -4k + 44 = 0 Þ k = 11
1 1 1 Let z = l , then we get
Þ , , are in AP.
a b c x + 11 y + 3l = 0 …(i)
66. Given system of linear equations, 3x + 11 y - 2l = 0 …(ii)
(1 + a )x + by + z = 2 and 2x + 4 y - 3l = 0 …(iii)
ax + (1 + b ) y + z = 3 Solving Eqs. (i) and (ii), we get
ax + by + 2z = 2 5l -l
x= , y= ,z=l
has a unique solution, if 2 2
1+ a b 1 xz 5l2
a (1 + b ) 1 ¹ 0 Þ 2
= 2
= 10
y æ lö
a b 2 2 ´ ç- ÷
è 2ø
584 JEE Main Mathematics

69. Given set, A = { X = (x, y, z )T : PX = 0 = - 5 (5a - 2b - c)


2
and x + y + z 2 2
=1 1 2 a
é1 2 1ù D3 = 2 6 b
where, P = ê -2 3 -4 ú 1 -2 c
ê ú
êë 1 9 -1úû = 6c + 2b - 2 (2c - b) - 10a
é1 2 1 ù é xù = - 10a + 4b + 2c
Q PX = ê -2 3 -4ú ê yú = 0 [given] = - 2 (5a - 2b - c)
ê úê ú
êë 1 9 -1úû êë z úû For infinite solution
1 2 1 D = D1 = D2 = D3 = 0
Q |P | = -2 3 -4 Þ 5a = 2b + c
1 9 -1 t - 1 3t + 1 2t
= 1 (-3 + 36) - 2 (2 + 4) + 1 (-18 - 3) 3. For non-trival solution t - 1 4t - 2 t + 3 = 0
= 33 - 12 - 21 = 0 2 3t + 1 3(t - 1)
Now, according to Cramer's Rule, the homogeneous t - 1 3t + 1 2t
system of linear equations represented by PX = 0 have Þ 0 t -3 3 - t
infinitely many solution, so the planes represented by
3-t 0 t -3
system of linear equations contains a unique line
passes through origin. [R2 ® R2 - R1 and R3 ® R3 - R1 ]
And the equation x2 + y2 + z 2 = 1 is the equation of a t - 1 3t + 1 2t
sphere having centre (0, 0, 0) and radius 1.
Þ (3 - t )2 0 -1 1
So, the unique line intersect the sphere at two distinct
1 0 -1
points.
2
Therefore, set A contains exactly two elements. Þ (t - 3) [(t - 1 ) (1) - (3t + 1 ) (-1 ) + 2t (1 )] = 0
Hence, option (d) is correct. Þ (t - 3)3 [t - 1 + 3t + 1 + 2t ] = 0
Þ t = 0 or t = 3
Round II 4. Given determinant is
5 4 3
x - 2 2x - 3 3x - 4
1. D = 100x + 50 + 1 100 y + 40 + 1 100z + 30 + 1
D = 2x - 3 3x - 4 4x - 5
x y z
3x - 5 5x - 8 10x - 17
[operating R2 - 100 R3 - 10R1]
5 4 3 On applying R3 ® R3 - R2 - R1, we get
= 1 1 1 = x - 2y + z = 0 x - 2 2x - 3 3x - 4
x y z D = 2x - 3 3x - 4 4x - 5
1 2 -3 0 -1 3x - 8
2. D = 2 -11
6 On applying R2 ® R2 - 2R1, we get
1 -2 7 x - 2 2x - 3 3x - 4
D= 1 - x + 2 - 2x + 3
= 20 - 2 (25) - 3 (-10)
0 -1 3x - 8
= 20 - 50 + 30 = 0
a 2 -3 = (x - 2) [(2 - x) (3x - 8) + (- 2x + 3)]
D1 = b 6 -11 - 1 [(2x - 3) (3x - 8) + (3x - 4)]
c -2 7 = (x - 2) [- 3x2 + 14x - 16 - 2x + 3]
= 20a - 2 (7b + 11c) - 3 (-2b - 6c) - [6x2 - 25x + 24 + 3x - 4]
= 20a - 14b - 22c + 6b + 18c = (x - 2) [- 3x2 + 12x - 13] - [6x2 - 22x + 20]
= - 3x3 + 18x2 - 37x + 26 - 6x2 + 22x - 20
= 20a - 8b - 4c = 4 (5a - 2b - c)
1 a -3 = - 3x3 + 12x2 - 15x - 6
D2 = 2 b -11 = Ax3 + Bx2 + Cx + D [given]
On comparing the coefficient of different terms, we get
1 c 7
A = - 3, B = 12, C = - 15 and D = 6
= 7b + 11c - a (25) - 3 (2c - b)
\ B + C = -3
= 7b + 11c - 25a - 6c + 3b
Hence, option (c) is correct.
= - 25a + 10b + 5c
Determinants 585

é cos q sin qù For k = 1


5. Given matrix, A = ê ú
ë - sin q cos qû D = D1 = D 2 = D3 = 0
é cos q sin qù é cos q sin qù But for k = - 2, at least one out of D1 , D 2, D3 are not
So, A2 = ê úê - sin q cos qú zero
ë - sin q cos qûë û
é cos 2q sin 2qù Hence, for no solution,
=ê ú k = -2
ë - sin 2q cos 2qû
é cos 4q sin 4qù 8. Given system of linear equations
\ A4 = ê ú
ë - sin 4q cos 4qû x + y + z = 2, 2x + 4 y - z = 6 and 3x + 2 y + lz = m
Since, it is given matrix, B = A + A 4 has infinite number of solution
é cos q + cos 4q sin q + sin 4q ù 1 1 1
Þ B =ê
ë - (sin q + sin 4 q) cos q + cos 4qúû \ D = 0 Þ 2 4 –1 = 0
\det (B) = (cos q + cos 4q)2 + (sin q + sin 4q)2 3 2 l
= 2 + 2 cos (3q) Þ 1(4l + 2) – 1(2l + 3) + 1(4 – 12) = 0
p Þ 2l - 9 = 0 Þ l = 9 / 2
So, at q =
5 1 1 2
3p æ 3p ö
det(B) = 2 + 2 cos = 4 cos 2ç ÷ and D3 = 0 Þ 2 4 6 = 0
5 è 10 ø
3 2 m
2
æ 10 - 2 5 ö
= 4ç ÷ = 1 (10 - 2 5 ) Þ 1(4 m – 12) – 1(2 m – 18) + 2(4 – 12) = 0
ç 4 ÷ 4 Þ 2 m - 10 = 0 Þ m = 5
è ø
10 - 2(2.23) \ 2l + m = 9 + 5 = 14
= [Q 5 » 2.23]
4 é a1 a 2 a3 ù
10 - 4.46 5.54 9. Let matrix A = ê a 4 a5 a 6 ú
= = Î (1, 2) ê ú
4 4 êë a7 a 8 a 9 úû
6. It is given that there are two matrices of order 3 ´ 3 and Ax = b having solution x1 , x2 and x3 when the
each, A = [a ij ] and B = [b ij ], such that b ij = 3 i + j-2
a ij vector b on the right side is equal to b1 , b2 and b3
é 31 + 1 - 2a11 31 + 2 - 2a12 31 + 3 - 2a13 ù respectively, so
ê ú a1 + a 2 + a3 = 1, 2a 2 + a3 = 0 a3 = 0
\Matrix B = ê32 + 1 - 2a 21 32 + 2 - 2a 22 32 + 3 - 2a 23 ú
ê33 + 1 - 2a31 33 + 2 - 2a32 33 + 3 - 2a33 ú a 4 + a5 + a 6 = 0, 2a5 + a 6 = 2 and a 6 = 0
ë û
a7 + a 8 + a 9 = 0 2a 8 + a 9 = 0 a9 = 2
é a11 3a12 32a13 ù \a 2 = 0, a5 = 1, a 8 = –1 and a1 = 1, a 4 = –1, a7 = –1
ê ú
= ê 3a 21 32a 22 33 a 23 ú 1 0 0
ê3 a31 3 a32 3 a33 ú
2 3 4
\ A = –1 1 0 Þ| A|= 2
ë û
é a11 a12 a13 ù –1 –1 2
= (3 ´ 32) ´ (3 ´ 32)ê a 21 a 22 a 23 ú = 36 × A 10. Using C1 ® C1 + C 2 + C3 , we get
ê ú
êë a31 a32 a33 úû f (x ) = (1 + sin 2 x + cos 2 x + 4 sin 2x )
1 1 1 cos 2 x 4 sin 2x
Þ A = 6 B Þ det( A ) = 6 det(B)
3 3 1 1 + cos x 2
4 sin 2x
81 1 1
Þ det( A ) = 6 = 2 = [Q det (B) = 81 (given)] 1 cos 2 x 1 + 4 sin 2x
3 3 9
Using R2 ® R2 - R1 , R3 ® R3 - R1, we get
7. kx + y + z = 1
f (x ) = 2(1 + 2 sin 2x )
x + ky + z = k
max f (x ) = 6, min f (x ) = - 2
x + y + zk = k2
11. Let a = s - a , b = s - b, g = s - c, then determinant
k 1 1
D = 1 k 1 = k (k2 - 1) - 1 (k - 1) + 1 (1 - k) (b + g )2 a2 a2
1 1 k = b (g + a )2
2
b 2 = 2abg (a + b + g )2
g2 g 2 (a + b )2
= k3 - k - k + 1 + 1 - k
= k3 - 3k + 2 [standard determinant]
2
= (k - 1) (k + 2) = 2(s - a )(s - b)(s - c)s3 Þ k = 2
586 JEE Main Mathematics

n n+1 n+2 a+b+g b g 0 b g


12. Let D = n ! (n + 1 )! (n + 2)! \ D= a+b+g g a = 0 g a =0
1 1 1 a+b+g a b 0 a b
n 1 1 1 a 0
= n ! n × n ! (n + 1 )(n + 1 )! 19. Using Cramer’s rule, 0 1 a = 0 Þ 1 + a3 = 0
1 0 0 a 0 1
(C3 ® C3 - C 2 and C 2 ® C 2 - C1 ) \ a = -1
= (n + 1 )(n + 1 )! - n × n ! 20. Since, -1 £ x < 0
= [(n + 1 )2 - n ] n ! = (n 2 + n + 1 ) × n !
\ [x] = - 1
ea e2a e3 a ea e2a 1 Also, 0 £ y<1
13. D = eb e2b e3b - eb e2b 1 Þ [ y] = 0 and 1 £ z < 2
ec e2c e3 c ec e2c 1 Þ [z ] = 1
1 ea e2a ea 1 e2a \Given determinant becomes
2b
a b c
= e × e × e 1 eb e + eb 1 e2b 0 0 1
1 ec e2c ec 1 e2c -1 1 1 = 1 = [z ]
= 0, since a + b + c = 0 -1 0 2
14. Applying C1 ® C1 - pC 2 - C3
21. Take a , b and c common from R1 , R2, R3 respectively
0 x y
1 1 1
0 y z =0 +1
a a a
2
- (xp + 2 yp + z ) xp + y yp + z 1 1 1
\ D = abc +1 +2
b b b
Þ 0 = (xp2 + 2 yp + z )( y2 - xz ) Þ y2 = xz 1 1 1
\ x, y, z are in GP. +1 +1 +3
c c c
a (1 + w) bw2 aw Applying R1 ® R1 + R2 + R3
15. D = b (w + w2) c bw2 (C1 ® C1 + C3 )
c (w2 + 1) aw c 1 1 1
æ 1 1 1ö 1 1 1
2 2 2 D = abc ç3 + + + ÷ 1 + 2+
- aw bw aw - a b aw è a b cø b b b
= -b c bw2 = w2 × w - b c bw2 1 1 1
1+ 1+ 3+
- cw aw c - c a cw2 c c c
Now, applying C3 ® C3 - C 2 and C 2 ® C 2 - C1 and on
a b a
expanding, we get
= - w2 b c b =0
é 1 1 1ù
c a c D = 2 abc ê3 + + + ú = 0
ë a b cû
16. Applying R1 ® R1 – R3 and R2 ® R2 – R3 Q a ¹ 0, b ¹ 0, c ¹ 0
1 0 -1
\ a -1 + b-1 + c-1 = - 3
0 1 -1 =0
a -1 0
sin 2 q cos 2 q 1 + 4 sin 4 q
22. f (x ) = ax a -1
Þ sin 2 q + cos 2 q + 1 + 4 sin 4 q = 0 ax2 ax a
[expanding along R3 ]
1 7p 11p Applying R3 ® R3 - xR2
Þ sin 4 q = - Þ q = , a -1 0
2 24 24
f (x ) = ax a -1
17. Put x = 0 in the given equation, we get
0 0 a+x
0 -1 3
t= 1 0 - 4 = - 12 + 12 = 0 = (a + x ) (a 2 + ax )
-3 4 0 Þ f (x ) = a (a + x )2
\ f (2 x ) = a (a + 2x )2
18. Since, a + b + g = 0
Applying C1 ® C1 + C 2 + C3 in the given determinant. Þ f (2 x ) - f (x ) = ax (2 a + 3x )
Determinants 587

23. Putting r = 1, 2 , 3,..., n and using the formula Þ 4x - 3 y - 4 = 0


S1 = n This is the required equation of the straight line in
(n + 1) n which point (x, y) lies.
and Sr =
2 27. Given, system of linear equations
S(2 r - 1 ) = 1 + 3 + 5 + ... = n 2 x+ y+ z =6 … (i)
n n n 4 x + ly - lz = l - 2 …(ii)
n
\ å D r = n (n + 1 ) n 2 + n + 1 n2 + n = 56 and 3x + 2 y - 4z = - 5 …(iii)
r =1
n 2
n 2 2
n + n +1 has infinitely many solutions, then D = 0
1 1 1
Applying C1 ® C1 - C3 , C 2 ® C 2 - C3
Þ 4 l - l =0
0 0 n
2
3 2 -4
0 1 n +n = 56
Þ 1(- 4l + 2l ) - 1(- 16 + 3l) + 1(8 - 3l) = 0
- n - 1 - n - 1 n2 + n + 1
Þ - 8l + 24 = 0
Þ n (n + 1 ) = 56 Þ l =3
Þ n 2 + n - 56 = 0 Þ (n + 8)(n - 7 ) = 0 From, the option l = 3, satisfy the quadratic equation
Þ n = 7 (n ¹ - 8) l2 - l - 6 = 0.
(eia + e-ia )2 (eia - e-ia )2 4 28. Given matrix B is the inverse matrix of 3 ´ 3 matrix A,
24. Given, D = (eib + e-ib )2 (eib - e-ib )2 4 é 5 2a 1 ù
(eig + e-ig )2 (eig - e-ig )2 4 where B = ê 0 2 1 ú
ê ú
Applying C1 ® C1 - C 2 êë a 3 - 1ûú
4 (eia - e-ia )2 4 We know that, det( A ) =
1 é -1 1 ù
êQ det( A ) = det( A ) ú
= 4 (eib - e-ib )2 4 det(B) ë û
4 (eig - e-ig )2 4 Since, det( A ) + 1 = 0 (given)
1
=0 [Q two columns are same] + 1 = 0 Þ det(B) = - 1
det(B)
Hence, it is independent of a , b and g.
Þ 5(- 2 - 3) - 2a (0 - a ) + 1 (0 - 2a ) = - 1
25. The system of linear equations Þ - 25 + 2a 2 - 2a = - 1 Þ 2a 2 - 2a - 24 = 0
x+ y+ z =2 Þ a 2 - a - 12 = 0 Þ (a - 4) (a + 3) = 0
x + 2 y + 3z = 5 Þ a = - 3, 4
x + 3 y + lz = m
So, required sum of all values of a is 4 - 3 = 1
has infinitely many solutions, so
29. We know that, if the system of equations
½1 1 1½
D = 0 Þ½1 2 3½ = 0 a1x + b1 y + c1z = d1
½ ½ a 2x + b2y + c2z = d2
½1 3 l½
a3 x + b3 y + c3 z = d3
Þ 1 (2l - 9) - 1 (l - 3) + 1 (3 - 2) = 0 Þ l = 5
has more than one solution, then D = 0 and
½1 1 2 ½ D1 = D2 = D3 = 0. In the given problem,
and D3 = 0 Þ ½1 2 5 ½ = 0 a 2 3
½ ½
½1 3 m ½ D1 = 0 Þ b - 1 5 = 0
Þ 1 (2 m - 15) - 1 (m - 5) + 2 (3 - 2) = 0 Þ m = 8 c -3 2
26. Given system of linear equations Þ a (- 2 + 15) - 2(2b - 5c) + 3(- 3b + c) = 0
x - 2 y + kz = 1 …(i) Þ 13a - 4b + 10c - 9b + 3c = 0
2x + y + z = 2 …(ii) Þ 13a - 13b + 13c = 0
and 3x - y - kz = 3 …(iii) Þ a - b + c=0Þb - a - c=0
has a solution (x, y, z ), z ¹ 0. 30. Given system of linear equations
On adding Eqs. (i) and (iii), we get 2x1 - 4x2 + lx3 = 1
x - 2 y + kz + 3x - y - kz = 1 + 3 x1 - 6x2 + x3 = 2
4x - 3 y = 4 and lx1 - 10x2 + 4x3 = 3
588 JEE Main Mathematics

½ 2 -4 l ½ Þ 1(24 + 7b) + 2(48 - b) + 9(-14 - 1) = 0


is inconsistent, so D = 0 Þ½1 -6 1 ½ = 0 Þ 5b + 120 - 135 = 0
½ ½
½ l -10 4 ½ Þ b =3
Þ 2(-24 + 10) + 4(4 - l ) + l (-10 + 6l ) = 0 \ a - b =8 -3=5
Þ 6l2 - 14l - 12 = 0 33. The given system of linear equations
Þ 3l2 - 7l - 6 = 0 x + y + z = 6, x + 2 y + 3z = 10 and 3x + 2 y + lz = m
Þ 3l2 - 9l + 2l - 6 = 0 has more than two solutions, so system must have
2 infinite number of solutions.
Þ 3l (l - 3) + 2(l - 3) = 0 Þ l = -
,3
3 Now, on putting z = 0 in above equation, we get
On adding first and second given linear equations we x+ y=6 …(i)
have x + 2 y = 10 …(ii)
3x1 - 10x2 + (l + 1)x3 = 3 and 3x + 2 y = m …(iii)
And at l = 3, it is same as the third linear equation, so On solving Eqs. (i) and (ii), we are getting
for l = 3, there are infinitly many solution, but for
-2 -2 y = 4 and x = 2
l= both are different, so for l = , the system of
3 3 From Eq. (iii), we get
linear equations is inconsistent. m = 3(2) + 2(4) = 14
31. System of given linear equations Now, on putting y = 0 and m = 14, in the given system
x - 2 y + 5z = 0 of linear Eq., we get
-2x + 4 y + z = 0 and -7x + 14 y + 9z = 0 x+ z =6 …(iv)
½ 1 -2 5 ù x + 3z = 10 …(v)
D = ½-2 4 1ú and 3x + lz = 14 …(vi)
½ ú
½-7 14 9úû On solving Eqs. (iv) and (v), we get z = 2 and x = 4
= 1(36 - 14) + 2(-18 + 7) + 5(-28 + 28) From Eq. (vi), we get
= 22 - 22 + 0 = 0 3(4) + l (2) = 14
\The given planes intersects each other over a unique Þ l =1
line. \ m - l2 = 14 - 1 = 13
Now, put z = 0, so x = 2 y 34. It is a simple question on scalar multiplication,
Q 15 £ x2 + y2 + z 2 £ 150 k a1 k a 2 k a3 a1 a2 a3
Þ 15 £ 4 y2 + y2 + 02 £ 150 i.e. b1 b2 b3 = k b1 b2 b3
2
Þ 3 £ y £ 30 c1 c2 c3 c1 c2 c3
Þ y Î [- 30 , - 3 ] È [ 3 , 30 ] Construction of matrix,
\Integral values of y are - 5, - 4, - 3, - 2, 2, 3, 4, 5. é a11 a12 a13 ù
\Number of required integral solution is 8. i.e. if a = [aij ]3 ´ 3 = ê a 21 a 22 a 23 ú
ê ú
32. System of given linear equations is êë a31 a32 a33 úû
x - 2 y + 3z = 9, é a11 a12 a13 ù
2x + y + z = b Here, P = [aij ]3 ´ 3 = ê a 21 a 22 a 23 ú
ê ú
and x - 7 y + az = 24, êë a31 a32 a33 úû
has infinitely many solution, so é b11 b12 b13 ù
D = 0 = D1 = D 2 = D3 Q = [bij ]3 ´ 3 = ê b21 b22 b23 ú
ê ú
½1 -2 3½ êë b31 b32 b33 úû
So, D = 0 Þ ½2 1 1½= 0
½ ½ where, bij = 2i + j aij
½1 -7 a½ 4 a11 8 a12 16 a13
Þ 1(a + 7) + 2(2a - 1) + 3(-14 - 1) = 0 \ Q = 8 a 21 16 a 22 32 a 23
Þ 5a = 40 Þ a = 8 16 a31 32 a32 64 a33
½1 -2 9 ½ a11 a12 a13
and D3 = 0 Þ½2 1 b ½= 0 = 4 ´ 8 ´ 16 2 a 21 2 a 22 2 a 23
½ ½
½1 -7 24½ 4 a31 4 a32 4 a33
Determinants 589

a11 a12 a13 1 a a2


9
= 2 ´ 2 ´ 4 a 21 a 22 a 23 37. a 1 a = 0 Þ a 4 - 2a 2 + 1 = 0
a31 a32 a33 a2 a 1
= 212 × P = 212 × 2 = 213 = 8192 Þ a2 = 1 Þ a = ± 1
½ sin x cos x cos x ½ But a = 1 not possible [not satisfying equation]
35. Given, ½ cos x sin x cos x ½ = 0 \ a = -1
½ ½
½ cos x cos x sin x ½ Hence, 1 + a + a 2 = 1
Applying C1 ® C1 + C 2 + C3 38. | A | = (2k + 1)3 ,| B| = 0

½ sin x + 2 cos x cos x cos x ½ But det (adj A) + det (adj B) = 106
= ½ sin x + 2 cos x sin x cos x ½ Þ (2k + 1)6 = 106
½ ½ 9
½ sin x + 2 cos x cos x sin x ½ Þ k= Þ [k] = 4
2
½1 cos x cos x ½
x x2 1 + x3
= (2 cos x + sin x)½ 1 sin x cos x ½ = 0
½ ½ 39. Given, 2x 4x 1 + 8x3 = 10
2

½1 cos x sin x ½
3x 9x2 1 + 27x3
Applying R2 ® R2 - R1 , R3 ® R3 - R1
½1 cos x cos x ½ 1 1 1 + x3
2
½
Þ (2 cos x + sin x) 0 sin x - cos x 0 ½= 0 Þ x× x 2 4 1 + 8x3 = 10
½ ½
½0 0 sin x - cos x ½ 3 9 1 + 27x3

Þ (2 cos x + sin x) (sin x - cos x)2 = 0 Apply R2 ® R2 - 2R1 and R3 ® R3 - 3R1, we get
Þ 2 cos x + sin x = 0 or sin x - cos x = 0 1 1 1 + x3
Þ 2 cos x = - sin x or sin x = cos x x3
0 2 - 1 + 6x3 = 10
p p 0 6 - 2 + 24x3
Þ cot x = - 1 / 2 gives no solution in - £ x £
4 4
2 6x3 - 1
and sin x = cos x Þ tan x = 1 Þ x = p /4 Þ x3 × = 10
6 24x3 - 2
36. Given,
Þ x3 (48x3 - 4 - 36x3 + 6) = 10
½ 1 x x+1 ½
½ ½ Þ 12x6 + 2x3 = 10
f (x) = 2x x (x - 1) (x + 1) x
½ ½ Þ 6x6 + x3 - 5 = 0
½ 3 x (x - 1 ) x (x - 1) (x - 2) (x + 1) x (x - 1) ½
5
Þ x3 = , - 1
Applying C3 ® C3 - (C1 + C 2) 6
½ 1 x 0½ æ5ö
1/3

= ½ x (x - 1) 0 ½= 0 x=ç ÷ ,-1
2x è6ø
½ ½
½ 3 x (x - 1 ) x (x - 1)(x - 2) 0½ Hence, the number of real solutions is 2.
\ f (x) = 0 Þ f (100) = 0
24
Relation and
Functions
The relation we are going to study here is an abstraction of relations we see in IN THIS CHAPTER ....
our everyday life such as those between parent and child, between an object
Relation
and its owner, among name, social security number, address and telephone
number etc. Function
Function of a real variable is a mathematical function whose domain is the Explicit and Implicit Functions
real line. More loosely, a function of a real variable is sometimes taken to Types of Functions
mean any function whose domain is a subset of the real line.
Composite Function
Inverse Function
Relation Even and Odd Function
Let A and B be two non-empty sets, then relation R from A to B is a subset of
A ´ B. Periodic Function

Let R Í A ´ B and (a , b) Î R, then we say that a is related to b by the relation


R i.e., aRb. If ( a , b) Î R, then we write it as aRb.
If ( a , b) Ï R , then we write aRb and we say that a is not related to b by the
relation R.
Total number of relations Let A and B be two non-empty finite sets
consisting of m and n elements respectively. Then, A ´ B consists of mn
ordered pairs. So, total number of subsets of A ´ B is 2mn .

Domain and Range of a Relation


Let R be a relation from a set A to set B. Then, the set of all first components
or coordinates of the ordered pairs belonging to R is called the domain of R
while the set of all second components or coordinates of the ordered pairs in R
is called the range of R.
Thus, Dom ( R ) = { a : ( a , b) Î R }
and Range ( R ) = { b : ( a , b) Î R }
Note Domain of a relation from A to B is a subset of A and its range is a subset of B.
Relation and Function 591

Types of Relation x < y and y < z Þ x < z


i.e. xRy and yRz Þ xRz
Reflexive Relation (b) Let L be the set of all straight lines in a plane. Then,
A relation R on a set A is said to be reflexive, if every the relation is parallel to on L is a transitive
element of A is related to itself. relation, because for any l1 , l2 , l3 Î L
Thus, if R is reflexive, then ( a , a ) Î R , "a Î A. l1|| l2 and l2|| l3 Þ l1|| l3
A relation R on a set A is not reflexive, if there exists an
Note The identity and the universal relations on a non-void set are
element a Î A such that ( a , a ) Ï R.
transitive.
e.g.
(a) If A = {1, 2, 3} be a set. Anti-symmetric Relation
Then, R = {(1, 1), (2, 2), (3, 3), (1, 3), (2, 1)} is a Let A be any set. A relation R on set A is said to be an
reflexive relation on A. anti-symmetric relation iff
But R¢ = {(1, 1), (3, 3), (2, 1), (3, 2)} is not a reflexive, ( a , b) Î R and ( b, a ) Î R
because for 2 Î A,(2, 2) Ï R Þ a = b for all a , b Î A
(b) Let L be the set of all lines in a plane. Then, relation By definition, if ( a , b) Î R and ( b, a ) Ï R , then also R is an
R on L defined by ( l1 , l2 ) Î R such that l1 is parallel to anti-symmetric relation. The identity relation on a set A
l2 is reflexive, since every line is parallel to itself. is an anti-symmetric relation.
Note The universal relation on a non-void set A is reflexive.
Example 1. Let W denotes the words in the English dictionary
Symmetric Relation define the relation R by R = {( x, y) Î W ´ W : the words x and y
A relation R on a set A is said to be a symmetric relation have atleast one letter in common}. Then, R is
iff (a) reflexive, symmetric and not transitive
( a , b) Î R Þ ( b, a ) Î R , "a , b Î A (b) reflexive, symmetric and transitive
i.e. aRb Þ bRa ,"a, b Î A (c) reflexive, not symmetric and transitive
Also, a relation R on a set A is not a symmetric relation if (d) not reflexive, symmetric and transitive
there are atleast two elements a, b Î A such that ( a , b) Î R Sol. (a) Let W = {CAT , TOY , YOU , . . .}
but ( b, a ) Ï R
Clearly, R is reflexive and symmetric but not transitive.
e.g. [Q CAT RTOY , TOY RYOU Þ
/ CAT RYOU ]
(a) Let A = {1, 2, 3, 4} be any set and let R1 and R2 be
relations on A1 such that Example 2. Let R = {(3, 3), (6, 6), (9, 9), (12, 12), (6, 12),
R1 = {(1, 3), (1, 4), (3, 1), (2, 2), ( 4, 1)} (3, 9), (3, 12), (3, 6)} be a relation on the set
and R2 = {(1, 1), (2, 2), (3, 3), (1, 3)} A = {3, 6, 9, 12}. The relation is
Clearly, R1 is a symmetric relation on A. However, (a) reflexive and symmetric only
R2 is not so, because (1, 3) Î R2 but (3, 1) Î R2. (b) an equivalence relation
(c) reflexive only
(b) Let L be the set of all lines in a plane and let R be a
(d) reflexive and transitive only
relation defined on L by the rule ( x , y ) Î R , x is
perpendicular to y. Then, R is a symmetric on L, Sol. (d) Since, for every elements of A, there exists elements
because L1 ^ L2 Þ L2 ^ L1 e.g., if ( L2 , L1 ) Î R (3, 3), (6, 6), (9, 9), (12, 12) ÎR Þ R is reflexive relation.
automatically implies ( L2 , L1 ) Î R Now, (6, 12) ÎR but (12 , 6) ÏR , so it is not a symmetric
Note (i) The identity and the universal relations on a non-void set relation.
are symmetric relations. Also, (3, 6), (6, 12) ÎR Þ (3, 12) ÎR
(ii) A reflexive relation on a set A is not necessarily symmetric. \ R is transitive relation.
(iii) The relation R on a set A is symmetric iff R = R -1.
Example 3. Let R1 and R2 be two relations defined as follows
Transitive Relation R1 = {( a, b) Î R2 : a 2 + b 2 Î Q}
Let A be any set. A relation R on A is said to be a and R2 = {( a, b) Î R2 : a 2 + b 2 Ï Q},
transitive relation iff ( a , b) Î R and ( b, c) Î R
where Q is the set of all rational numbers. Then
Þ ( a , c) Î R for all a, b, c Î A.
(JEE Main 2020)
i.e. a R b and b R c Þ a R c for all a , b, c Î A.
(a) R1 and R2 are both transitive
e.g.
(b) Neither R1 nor R2 is transitive
(a) On the set N of natural numbers, the relation R
defined by xRy, such that x is less than y is (c) R1 is transitive but R2 is not transitive
transitive, because for any x , y , z Î N (d) R2 is transitive but R1 is not transitive
592 JEE Main Mathematics

Sol. (b) Given relation, R1 = {( a, b) Î R 2 : a2 + b 2 Î Q} (ii) Symmetric xRy Þ / yRx as 0R1


2 2
Let ( a, b) Î R1 Þ a + b Î Q and ( b, c) Î R1 But 1R0 Þ1 = w × (0)
Þ b 2 + c2 Î Q which is not true for any rational number.
The relation R is not symmetric.
\a2 + 2b 2 + c2 Î Q, but we can not say that a2 + c2 Î Q,
Thus, R is not equivalence relation.
so ( a, c) Ï R1
Now, for relation S which is defined as
\R1 is not transitive.
ìæm pö
And another given relation S = í ç , ÷ m , n, p and q Î integers such that
R2 = {( a, b) Î R 2 : a2 + b 2 Ï Q} îè n q ø
n , q ¹ 0 and qm = pn}
Let ( a, b) Î R2 Þ a2 + b 2 Ï Q
m m
and ( b, c) Î R2 Þ b 2 + c2 Ï Q (i) Reflexive R Þ mn = mn [true]
2 2 2 2 2 n n
\ a + 2b + c Ï Q, but we can not say that a + c Ï Q,
The relation S is reflexive.
so ( a, c) Ï R2
\ R2 is not transitive.
m p
(ii) Symmetric R Þ mq = np
Hence, option (b) is correct. n q
p m
Þ np = mq Þ R
Equivalence Relation q n
A relation R on a set A is said to be an equivalence
The relation S is symmetric.
relation on A iff
(a) it is reflexive, i.e., ( a , a ) Î R , " a Î A. m p p r
(iii) Transitive R and R
(b) it is symmetric, i.e., ( a , b) Î R. n q q s
Þ ( b, a ) Î R , " a , b Î A Þ mq = np
(c) it is transitive, i.e., ( a , b) Î R and ( b, c) Î R. and ps = rq
Þ ( a , c) Î R , " a , b, c Î A Þ mq × ps = np × rq
Þ ms = nr
Properties of Equivalence Relations m r m r
Þ = Þ R
(i) The inverse of an equivalence relation is also an n s n s
equivalence relation, i.e. if R is an equivalence The relation S is transitive.
relation on a set A, then R -1 is also an equivalence
Hence, the relation S is equivalence relation.
relation on A.
(ii) The union of two equivalence relations on a set is Example 5. Let R be the real line. Consider the following
not necessarily an equivalence relation, on a set. subsets of the plane R ´ R
(iii) The intersection of two equivalence relations on a S = {( x, y) : y = x + 1 and 0 < x < 2 }
set is an equivalence relation on the set.
T = {( x, y) : x - y is an integer}
Example 4. Consider the following relations Which one of the following is true?
R = {( x , y) | x , y are real numbers and x = wy for some rational (a) T is an equivalence relation on R but S is not
ìæ m p ö (b) Neither S nor T is an equivalence relation on R
number w} ; S = í ç , ÷ m , n, p and q are integers such
îè n q ø (c) Both S and T are equivalence relations on R
that n, q ¹ 0 and qm = pn} . Then, (d) S is an equivalence relation on R but T is not
(a) R is an equivalence relation but S is not an equivalence Sol. (a) Since, (1, 2) Î S but (2 , 1) Ï S
relation So, S is not symmetric.
(b) Neither R nor S is an equivalence relation Hence, S is not an equivalence relation.
(c) S is an equivalence relation but R is not an equivalence Given, T = {( x, y) : ( x - y) Î l }
relation
Now, x - x = 0 Î I, it is reflexive relation.
(d) R and S both are equivalence relations
Again now, ( x - y) Î I
Sol. (c) Given, relation R is defined as y - x Î I, it is symmetric relation.
R = {( x , y) | x , y are real numbers and Let x - y = I1 and y - z = I2
x = wy for some rational number w} Now, x - z = ( x - y) + ( y - z) = I1 + I2 Î I
(i) Reflexive xRx Þ x = wx So, T is also transitive.
\ w = 1 Î rational number Hence, T is an equivalence relation.
The relation R is reflexive.
Relation and Function 593

f
Function 1 2
Let A and B be two non-empty sets. Then, a function ‘f’ 2 4
from set A to set B is a rule or method or correspondence 3 6
which associates elements of set A to elements of set B A B
such that
(i) all elements of set A are associated to elements in Example 6. Let f : R ® R defined by
set B. f ( x) = x3 + ax 2 + 3x + 100, then the values of a for which f is a
(ii) an element of set A is associated to a unique element one one function, is
in set B. (a) -2 £ a £ 3 (b) -2 £ a £ 2
In other words, a function ‘f’ from a set A to a set B (c) -3 £ a £ 3 (d) None of these
associates each element of set A to a unique element of
set B. Sol. (c) f ( x) = x3 + ax2 + 3x + 100 Þ f ¢ ( x) = 3x2 + 2ax + 3.
For f ( x) to be one-one. f ¢ ( x) ³ 0 or £ 0.
Terms such as ‘map’ (or mapping), ‘correspondence’ are
used as synonyms for ‘function’. If f is a function from a But f ¢( x) is a quadratic expression and coefficient of x2 > 0 .
set A to set B, then we write f : A ® B or A ® B. So, that f ¢ ( x) ³ 0 ÞD £ 0 Þ 4 a 2 - 36 £ 0 Þ a 2 £ 9 Þ -3 £ a £ 3.
If an element a Î A is associated to an element b Î B, then
b is called the ‘f-image of a’ or ‘image of a under f’ or ‘the
Many One Function
value of the function f at a’. Also, a is called the If let f : A ® B. If two or more than two elements of set A
pre-image of b under the function f. We write it as have the same image in B, then f is said to be many one.
b = f(a) e.g. Let A = { -1, 1, 2, 3} and B = {1, 4, 9}
(i) all the independent elements (inputs) in A must where f : A ® B given by f ( x ) = x 2
have their images in set B. f
(ii) to each and every independent element in A there
–1 1
corresponds one and only one image in B.
1
(iii) every function is a relation but every relation may or 4
2
may not be a function. 3 9
Note A B
• Function cannot be multivalued.
• If A and B have m and n elements respectively, then the number of
Here, f( -1) = 1; f(1) = 1; f( 2) = 4 and f( 3) = 9.
functions from A to B is nm . Clearly, two elements 1 and –1 have the same image
1 Î B. So, f is a many one function.
Explicit and Implicit Functions x 2 + 4x + 7
Example 7. If f : R ® R be defined as f ( x) = ,
A function is said to be an explicit function, if the x2 + x + 1
dependent variable y is expressible completely in terms of then f ( x) is
independent variable.
(a) one one (b) many one
A function is said to be an implicit function, if the (c) many many (d) None of these
dependent variable is not directly expressible in terms of
the independent variable x. Sol. (b) For the given function f ( x1) = f ( x2)
e.g. y = 3ex - x 2 + 2 , x Î R is an explicit function while x12 + 4x1 + 7 x22 + 4x2 + 7
Þ = 2
2 2 2 x12 + x1 + 1 x2 + x2 + 1
yex + xe- y = ea is an implicit function.
Þ ( x1 - x2)(2x1 + 2x2 + 1 + x1x2) = 0.
Here, one of the solution of 2x1 + 2x2 + 1 + x1x2 = 0 is x1 = 0
Types of Functions and x2 = - 1/2.
One One Function æ 1ö
Hence, f (0) = f ç - ÷ = 7, so that f ( x) is many one.
è 2ø
A function f : A ® B is said to one one, if different
elements in A have different images in B. Remark Any function will be either one one or many one.

This type of functions are known as injective functions. Onto Function


e.g. Let A = { 1, 2, 3} and B = { 2, 4, 6}. Consider f : A ® B, If the function f : A ® B is such that each and every
f ( x ) = 2x. Then, f(1) = 2; f( 2) = 4 and f( 3) = 6. Clearly, f is a element in B is the f image of atleast one element in A,
one one function from A to B such that different elements then f is a function of A onto B. It is also called
in A have different images in B. Surjective function.
594 JEE Main Mathematics

OR One One Onto Function


If each element in codomain have atleast one pre-image A function which is one one function and onto function is
in the set of domain, then function is onto. called one one onto function.
e.g., Let A = { -1, 1, 2, - 2}, B = { 1, 4 } and f : A ® B, be a It is also called bijective function.
function defined by f ( x ) = x 2. Then, f is onto because
f ( A) = { f ( -1), f (1), f ( 2), f ( -2)} = { 1, 4 } = B. Example 9. Let f : ( -¥, ¥) ® [2, ¥) be a function defined
f by f ( x) = x 2 - 2 a + a 2 , a Î R, then the value of a for which f
–1 1 is onto
1 (a) 1 (b) 3 (c) 1 ± 3 (d) 1 ± 5
4
2
–2 Sol. (d) For f to be onto range of the function should be [2, ¥).
A B So, a2 - 2a = 4 Þ a = 1 ± 5.

Note If range is same as codomain, then function is onto. Example 10. If the function f : R - {1, - 1} ® A defined by
Example 8. The number of functions f from {1, 2, 3, … , x2
f ( x) = , is surjective, then A is equal to
20} onto {1, 2, 3, … , 20} such that f (k) is a multiple of 3, 1 - x2 (JEE Main 2019)
whenever k is a multiple of 4, is (JEE Main 2019) (a) R - {-1} (b) [0 , ¥) (c) R - [ -1, 0) (d) R - ( -1, 0)
(a) (15) ! ´ 6 ! (b) 5 6 ´ 15 Sol. (c) Given, function f : R – {1, - 1} ® A defined as
(c) 5 ! ´ 6 ! (d) 65 ´ (15) ! x2
f ( x) = = y (let)
Sol. (a) According to given information, 1 - x2
we have if k Î{4, 8, 12, 16, 20} Þ x2 = y(1 - x2) [Q x2 ¹ 1]
Then, f (k) Î {3, 6, 9, 12, 15, 18} y
Þ x2(1 + y) = y Þ x2 = [provided y ¹ -1]
[Q codomain ( f ) = {1, 2, 3, …, 20}] 1+ y
Now, we need to assign the value of f (k) for y
Q x2 ³ 0 Þ ³0
k Î{4, 8, 12, 16, 20} this can be done in 6C5 × 5 ! ways 1+ y
Þ y Î ( -¥ , - 1) È [0 , ¥ )
and remaining 15 element can be associated by15 ! ways.
Since, for surjective function, range of f = codomain
\Total number of onto functions = 15 16 = 15 ! 6 !
\ Set A should be R - [ -1, 0).
Into Function Example 11. Let A = { x Î R : x is not a positive integer}.
If the function f : A ® B is such that there is atleast one 2x
element in B which is not f image of any element in A, Define a function f : A ® R as f ( x) = , then f is
x -1
then f is called an into function. (JEE Main 2019)
OR (a) injective but not surjective
(b) not injective
If there is even single element in the set of codomain
(c) surjective but not injective
which does not have its pre-image, then it is known as
(d) neither injective nor surjective
into function.
2x
e.g. Let A = { 2, 3, 5, 7} and B = { 0, 1, 3, 5, 7}. Consider Sol. (a) We have a function f : A ® R defined as f ( x) =
f : A ® B; f ( x ) = x - 2. Then, f( 2) = 0; f( 3) = 1, f( 5) = 3 and x -1
f( 7) = 5. One-one Let x1, x2 Î A such that f ( x1) = f ( x2)
2x1 2x2
f Þ =
x1 - 1 x2 - 1
0
2
1
Þ 2x1x2 - 2x1 = 2x1x2 - 2x2 Þ x1 = x2
3 Thus, f ( x1) = f ( x2) has only one solution, x1 = x2
3
5 5 \ f ( x) is one-one (injective)
7 7 2 ´2
Onto Let x = 2, then f(2) = =4
A B 2 -1
Clearly, f is a function from A to B. Now, there exists an But x = 2 is not in the domain, and f ( x) is one-one function
element 7 Î B, having no pre-image in A. So, f is an into \ f ( x) can never be 4.
function. Similarly, f ( x) can not take many values.
Hence, f ( x) is into (not surjective).
Note If range of a function f is proper subset of codomain, then
function f is into. \ f ( x) is injective but not surjective.
Relation and Function 595

Example 12. Let a function f : (0, ¥) ¾¾® (0, ¥) be Important Points


1 (i) gof is defined only if for each x Î A, f ( x ) is an
defined by f ( x) = 1 - . Then, f is
x (JEE Main 2019) element of g so that we can take its g-image. Hence,
for the composition gof to exist, the range of f must
(a) injective only
be a subset of the domain of g.
(b) both injective as well as surjective
(ii) gof exists iff the range of f is a subset of domain
(c) not injective but it is surjective
of g. Similarly, fog exists if range of g is a subset of
(d) neither injective nor surjective
domain of f.
ì ( x - 1)
| x - 1| ï - x , if 0 < x £ 1 Properties of Composition of Functions
Sol. (d) We have, f ( x) = =í
x x -1 (i) If f and g are even functions Þ fog is an even
ï , if x > 1
î x function.
ì1 (ii) If f is an odd and g is an even function Þ fog is an
ï - 1, if 0 < x £ 1 odd function.
=íx
1 (iii) If f is an even and g is an odd function Þ fog is an
ï1 - , if x > 1
î x even function.
Now, let us draw the graph of y = f ( x) (iv) The composition of functions is not commutative.
Note that when x ® 0, then f ( x) ® ¥, when x = 1, then i.e., fog =/ gof
f ( x) = 0, and when x ® ¥, then f ( x) ®1 (v) The composition of functions is associative
Y i.e., If f, g, h are any three functions such that
( fog) oh and fo ( goh ) both exist, then
x=0 ( fog) oh = fo ( goh )
y=1 (vi) The composition of two bijective function is also a
bijective function.
O 1 y=0
X (vii) The composition of any function with the identity
function is the function itself.
Clearly, f ( x) is not injective because if f ( x) < 1, then f is many
i.e., If f : A ® B, then
one, as shown in figure.
foI A = I R of = f
Also, f ( x) is not surjective because range of f ( x) is [0 , ¥ [ and
but in problem co-domain is (0 , ¥), which is wrong. (viii) If f, g and h be functions from R to R then,
\ f ( x) is neither injective nor surjective ( f + g) oh = foh + goh
and ( f × g) oh = ( foh ) × ( goh )
Composite Functions ì2 + x, if x ³ 0
Example 13. If f ( x) = í , then ( fof )( x) is
Let f : A ® B and g : B ® C be two functions. Then, a î2 - x, if x < 0
function gof : A ® C defined by
(a) | x| (b) 4 -| x| (c) 4 + | x| (d) None of these
( gof )( x ) = gf ( x )}, for all x Î A
Sol. (c) f ( x) = 2 + | x|
is called the composition of f and g.
( fof )( x) = 2 + | f ( x)| = 2 + 2|( x)| = 4 + | x|
Let A, B and C be three non-void sets and let f : A ® B,
g : B ® C be two functions. Since, f is a function from A to 1
Example 14. For x Î R - {0, 1}, let f1( x) = , f2( x) = 1 - x and
B, therefore for each x Î A there exists a unique element x
f ( x ) Î B. Again, since g is a function from B to C, therefore 1
f3( x) = be three given functions. If a function, J (x) satisfies
corresponding to f ( x ) Î B there exists a unique element 1- x
g{ f ( x )} Î C. Thus, for each x Î A there exists a unique ( f2oJof1)( x) = f3( x), then J ( x) is equal to (JEE Main 2019)
element g{ f ( x )} Î C. 1
(a) f2( x) (b) f3( x) (c) f1( x) (d) f3( x)
It follows from the above discussion that f and g when x
considered together define a new function from A to C. 1 1
Sol. (b) We have, f1( x) = , f2( x) = 1 - x and f3( x) =
This function is called the composition of f and g and is x 1- x
denoted by gof. We define it formally as follows : Also, we have ( f2 o J o f1)( x) = f3( x)
gof Þ f2(( J o f1)( x)) = f3( x)
f g Þ f2( J ( f1( x)) = f3( x)
1
Þ 1 - J( f1( x)) =
x f (x) g [f (x)] 1- x
1
Z [Q f2( x) = 1 - x and f3( x) = ]
X Y 1- x
596 JEE Main Mathematics

æ 1ö 1 1 Sol. (c) Given, f ( x) = ln ( x2 + 3x + 1)


Þ 1- J ç ÷ = [Q f1( x) = ]
è xø 1 - x x 2x + 3
\ f ¢ (x) = 2 > 0 " x Î[1, 3],
æ 1ö 1 1 - x - 1 -x ( x + 3x + 1)
Þ J ç ÷ =1- = =
è xø 1- x 1- x 1- x which is a strictly increasing function. Thus, f ( x) is injective,
1 given that f ( x) is onto. Hence, the given function f ( x) is invertible.
Now, put = X , then
x Now, f {f -1( x)} = x Þ ln (( f -1( x)) 2 + 3( f -1( x)) + 1) = x
-1
Þ {f -1( x)} 2 + 3{f -1( x)} + 1 - ex = 0
-1 1 é 1ù
J( X) = X = = êëQ x = X úû
1 X -1 1- X ( - ex )
-3 ± 9 - 4 ×11
1- \ f -1( x) =
X 2
Þ J( X) = f3( X) or J( x) = f3( x) -3 ± (5 + 4ex)
=
2
Inverse Function
-3 + (5 + 4ex)
Let f : A ® B is a bijective function, then there exists a Þ f -1( x) = [Q f -1( x) Î [1, 3]]
2
unique function g : B ® A such that
f ( x ) = y Û g( y ) = x , " x Î A and y Î B, then g is called -3 + (5 + 4ex)
Hence, f -1( x) =
inverse function of f. 2
Hence, g = f -1: B ® A
Example 16. The inverse of the function
f
ì x, x <1
ï
a d f ( x) = í x 2, 1 £ x £ 4 is
b e ï8 x , x>4
c f î
ì 1/ x, x <1 ì x, x <1
g
A B ï ï
(a) í x , 1 £ x £ 16 (b) í x , 1 £ x £ 16
ï 2 x > 16 ï 2 x < 16
Properties of Inverse Function î x /64, î x /64,
(i) Inverse of bijective function is unique. ì x, x <1
ï
(ii) Inverse of bijective function is also bijective (c) í x , 1 £ x £ 16 (d) None of these
ï 2 x > 16
function. î x /64,
(iii) If f : A ® B is bijective function and g: B ® A is ì x, x <1
inverse of f , then fog = I B and gof = I A , where I A ï 2
Sol. (c) Given, f ( x) = í x , 1 £ x £ 4
and I B are identity function of sets A and B, ï
respectively. î8 x , x > 4
(iv) If f : A ® B and g: B ® A are two bijective functions, Let f ( x) = y Þ x = f -1(y) …(i)
then gof : A ® C is also bijective function and ì y, y <1
( gof )-1 = f -1og-1. ï
\ x =í y, 1£ y £ 4
(v) fog =/ gof , but if fog = gof , then either f -1 = g or ï 2 2
îy /64, y /64 > 4
g-1 = f and ( fog)( x ) = ( gof )( x ) = x. ì y, y <1
-1 ï
Note If the inverse of f exists, then f is called an invertible function. Þ f (y) = í y , 1 £ y £ 16 [from Eq. (i)]
ï 2
Example 15. The inverse of the function îy /64, y > 16
f ( x) = log ( x 2 + 3x + 1); x Î[1, 3] assuming it to be an onto ì x, x <1
-1 ï
function, is Hence, f ( x) = í x , 1 £ x £ 16
ï 2 x > 16
- 3 - 5 + 4 ex î x /64,
(a)
2
- 3 ± 5 + 4 ex Even and Odd Functions
(b)
2 A real function f ( x ) is an even function, if f ( - x ) = f ( x )
- 3 + 5 + 4 ex e.g. f ( x ) = cos x is an even function.
(c)
2 A real function f ( x ) is an odd function, if f ( - x ) = - f ( x )
(d) None of the above e.g. f ( x ) = sin x or x3 are odd functions.
Relation and Function 597

Properties of Even and Odd Functions Let g be the extension. Then, for even extension,
we define
(i) Product of two odd functions or two even functions is ì f ( x ), if x Î [0, a ]
an even function. g( x ) = í
(ii) Product of an even function and an odd function is î f ( - x ), if x Î [- a , 0]
an odd function. and for odd extension, we define
(iii) Each function can be expressed as the sum of an ì f ( x ), if x Î [0, a ]
g( x ) = í
even function and an odd function. î - f ( x ), if x Î [- a , 0]
Such that
1 1 Example 18. Let the function f ( x) = x 2 + x + sin x - cos x
f ( x ) = { f ( x ) - f ( - x )} + { f ( x ) - f ( -x)} be defined on the interval [0, 1], then odd and even extensions
2 1 44244 3 2 1 44244 3
Even function odd function of f ( x) in the interval [ -1, 1] are respectively
(iv) Derivative of an even function is an odd function ìï x2 + x + sin x - cos x 0 £ x £1
(a) í
and derivative of an odd function is an even ïî - x2 + x + sin x + cos x -1 < x < 0
function.
(v) Square of an even or an odd function is always an ìï x2 + x + sin x - cos x 0 £ x £1
and í
even function. ïî x2 - x - sin x - cos x -1 £ x < 0
(vi) The graph of an odd function is symmetrical about
ïì x + x + sin x - cos x
2
origin or symmetrical in opposite quadrants. 0 £ x £1
(b) í
(vii) The graph of an even function is symmetrical about îï x - x - sin x - cos x
2 -1 £ x < 0
y-axis.
ìï x2 + x + sin x - cos x 0 £ x £1
and í
Example 17. The functions f1( x) = log ( x + x + 1) and 2
ïî - x2 + x + sin x + cos x -1 < x < 0
æ a x - 1ö (c) does not exist
f2( x) = x ç x ÷ are respectively.
è a + 1ø (d) None of the above
(a) odd and even function (b) even and odd function Sol. (a) Odd extension To make f ( x) an odd function in the
(c) odd and odd function (d) even and even function interval [ -1,1], we define f ( x) as follows
ì f ( x) 0 £ x £1
Sol. (d) (i) f1( x) = log ( x + x2 + 1) f ( x) = í
î - f ( - x) - 1 £ x<0
Þ f1( - x) = log ( - x + x2 + 1)
ìï x2 + x + sin x - cos x 0 £ x £1
æ 1 ö =í
= log ç ÷ = log ( x + x2 + 1) -1 ïî - ( x2 - x - sin x - cos x) -1 £ x < 0
ç x + x2 + 1÷
è ø
ìï x2 + x + sin x - cos x 0 £ x £1
= - log ( x + x2 + 1) = - f1( x) =í
ïî - x2 + x + sin x + cos x -1 £ x < 0
So, f1( x) is an odd function
Even extension To make f ( x) an even function in the interval
æ ax - 1ö [ -1,1], we define f ( x) as follows
(ii) We have, f2( x) = x ç x ÷
è a + 1ø ì f ( x) 0 £ x £1
f ( x) = í
æ 1 ö î f ( - x) - 1 £ x<0
æ a-x - 1ö ç x - 1÷ æ 1 - ax ö
\ f2( - x) = - x ç - x ÷ =-xç a ÷ =-xç ÷ ìï x2 + x + sin x - cos x 0 £ x £1
è a + 1ø ç 1 + 1÷ è 1 + ax ø =í
è ax ø ïî x2 - x - sin x - cos x -1 £ x < 0
æ a - 1ö
x
=xç x ÷ = f2( x) Example 19. Let f ( x) = a x( a > 0) be written as
è a + 1ø
f ( x) = f1( x) + f2( x), where f1( x) is an even function and f2( x) is an
So, f2( x) is an even function.
odd function. Then f1( x + y) + f1( x - y) equals (JEE Main 2019)
(a) 2f1( x + y) × f2( x - y) (b) 2f1( x + y) × f1( x - y)
Even and Odd Extension
(c) 2f1( x) × f2(y) (d) 2f1( x) × f1(y)
A function f ( x ) defined on the interval [0, a ] can be
extended to [-a , a ], so that f ( x ) becomes an even or odd Sol. (d) Given, function f ( x) = ax, a > 0 is written as sum of an
function on the interval [-a , a ]. If this extension is an even and odd functions f1( x) and f2( x) respectively.
even function, it is called even extension and if this ax + a- x ax - a- x
extension is an odd function, it is called odd extension. Clearly, f1( x) = and f2( x) =
2 2
598 JEE Main Mathematics

So, f1( x + y) + f1( x - y) provided that there does not exist a positive number k < T
1 1
= [ ax + y + a- ( x + y ) ] + [ ax - y + a- ( x - y ) ] for which f ( k + x ) = g( x ) and g( k + x ) = f ( x ), else k will be
2 2 the period of ( f + g)( x ).
1é 1 ax ay ù f
= ê ax ay + x y + y + x ú This result is also true for functions , f - g and fg.
2ë aa a a û g
1é æ (v) If f ( x ) is a periodic function with period T and g( x ) is a
1ö 1 æ 1 öù
= ê ax ç ay + y ÷ + x ç y + ay ÷ ú monotonic function, then g[ f ( x )] is also a periodic function
2ë è a ø a èa øû
with same period T as that of f ( x ).
1æ x 1ö æ y 1ö æ ax + a- x ö æ ay + a- y ö (vi) If f1( x ), f2( x ) and f3 ( x ) are periodic functions with periods
= ça + x ÷ ça + y ÷ = 2 ç ÷ ç ÷
2è a øè a ø è 2 ø è 2 ø T1 ,T2 and T3 respectively also, if
= 2f1( x) × f1(y) h( x ) = af1( x ) ± bf2( x ) ± cf3 ( x ). then, period of
ìLCM of { T1, T2, T3 },
Periodic Function ï if h ( x ) is an odd function
{ h( x )} = í 1
A function f : X ® Y is said to be a periodic function, ïî 2 LCM of { T1 , T2 , T3 }, if h ( x ) is an even function
if there exist a positive real number T such that
f ( x + T ) = f ( x ), "x Î X. The least value of T is called
Example 20. If f ( x) = sin x + cos a x is a periodic function,
the fundamental period of function. In general, the
fundamental period (principal period) is called the then ‘ a’ is a
period of a function. Graphically graph gets (a) irrational number (b) rational number
repeated after each period of the function. (c) natural number (d) None of these
2p
Standard Results on Some Periodic Functions Sol. (b) Period of sin x = 2p and period of cos ax =
a
Function Period 2p 2p
\ Period of sin x + cos ax = LCM of and
sinn x,cos n x,sec n x,cosec n x p (if n is an even number) 1 a
2 p (If n is an odd number) LCM of 2p and 2p 2p
tann x,cot n x p
= = where, l is the HCF of 1 and a.
HCF of 1 and a l
|sin x|,|cos x|,|tan x|,|sec x|, p
1 a
|cosec x|,|cot x| Since, l is the HCF of 1 and a, and should be both integers.
x - [ x] 1 l l
-1 -1
a
sin (sin x), cos (cos x) 2p 1 a n n
Suppose l
= m and = n, then = , where n, m Î I , i.e., a =
cosec -1(cosec x), sec -1 (sec x) l l 1 m m
tan-1 (tan x), cot -1 (cot x) p l
Hence, a is rational number with period = 2
Properties of Periodic Functions Example 21. Which of the following function has period p?
(i) If f ( x ) is a periodic function with fundamental 5x
1 (a) tan 3x + cos (b) f ( x) = cos x + { x}
period T, then will also be a periodic 2
f(x) (c) f ( x) = cos (cos x) + cos (sin x) (d) None of these
function with same fundamental period T. [here { x} denotes the fractional part of x]
(ii) If f ( x ) is a periodic function with period T, then 5x
Sol. (a) Let f ( x) = tan 3x + cos
f ( ax + b), a ¹ 0 is also a periodic function with 2
T p 5x 2p 4p
fundamental period . Period of tan 3x = and period of cos = =
|a| 3 2 5/2 5
LCM of p and 4p
(iii) If f ( x ) is a periodic function with period T, then \ Period of f ( x) = = 4p
HCF of 3 and 5
af ( x ) + b is also a periodic function with same
fundamental period T. (b) f ( x) = cos x + { x} = cos x + x - [ x]
cos x is periodic with period 2p and x - [ x] is periodic with period
(iv) Let f ( x ) and g( x ) be two periodic functions such
m 1. But LCM of 2p and 1 does not exist. Hence, f ( x) is not periodic.
that period of f ( x ) = , where m , n Î N and (c) f ( x) = cos (cos x) + cos (sin x)
n
r Q cos {cos ( p + x)} = cos ( - cos x) = cos (cos x)
m , n are coprime and period of g( x ) = , where and cos {sin ( p + x)} = cos ( - sin x) = cos (sin x)
s
\cos (cos x) and cos (sin x) are periodic functions with period p. But
r , s Î N and r , s are coprime. Then, ( f + g)( x ) is
cos {cos ( p /2 + x)} = cos ( - sin x) = cos (sin x)
periodic with period T given by
and cos {sin ( p /2 + x)} = cos (cos x)
LCM of ( m , r ) p
T = \ f ( x) is periodic with period .
HCF of ( n , s) 2
Practice Exercise
ROUND I Topically Divided Problems
Types of Relation 8. Let f : N ® N be defined by
1. Let R be a relation on the set N of natural numbers ìn+1 if n is odd
,
ï
defined by nRm, if n divides m. Then, R is f ( n) = í 2 for all n Î N . Then, the
(a) reflexive and symmetric ï n, if n is even
(b) transitive and symmetric î 2
(c) equivalence function f is (JEE Main 2019, 10 January)
(d) reflexive transitive but not symmetric (a) onto
(b) one one
2. Relation R in set N of natural numbers defined as (c) bijective
R = {( x, y) : y = x + 5 and x < 4} are (d) None of the above
(a) reflexive
(b) symmetric
9. Let E = {1, 2, 3, 4} and F = {1, 2 }. Then, the number
(c) not reflexive not symmetric of onto functions from E to F is
(d) not reflexive not symmetric but transitive (a) 14 (b) 16
(c) 12 (d) 8
3. Given a non-empty set X. Consider P ( X ), which is
the set of all subset of X. Defined the relation R in 10. Let x denote the total number of one-one functions
P ( X ) as follows from a set A with 3 elements to a set B with 5
For subsets A and B in P ( X ), ARB if and only if elements and y denote the total number of one-one
A Ì B, then R is functions from the set A to the set A ´ B. Then,
(a) an equivalence relation on P (X ) (JEE Main 2021)
(b) not an equivalence relation on P (X ) (a) y = 273x (b) 2 y = 91x
(c) an anti-symmetric (c) y = 91x (d) 2 y = 273x
(d) None of the above
11. Let f : ( e, ¥) ® R be defined by f ( x) = log[log(log x)],
4. Let A = {1, 2, 3}. Then, number of relations then f is
containing (1, 2) and (1, 3) which are reflexive and (a) one-one onto
symmetric but not transitive is (b) many one onto
(a) 1 (b) 2 (c) 3 (d) 4 (c) one-one into
5. Let A = {1, 2, 3}. Then, number of equivalence (d) many one into
relations containing (1, 2) is x2 + e
12. The function f : R¢ ® (1, e) defined by f ( x) = is
(a) 1 (b) 2 (c) 3 (d) 4 x2 + 1

Types of Mapping (Function) (a) one one but not onto


(b) onto but not one one
x
6. The function f : R ® R defined by f ( x) = 2
, (c) both one one and onto
x +1
(d) neither one one nor onto
"x Î R is
(a) one one and onto (b) neither one one nor onto 13. If f ( x) = x 3 + 3x 2 + 12 x - 2 sin x, where f : R ® R,
(c) one one (d) on to then
7. Set A has 3 elements and the set B has 4 elements. (a) f (x) is many one and onto
Then, the number of injective mappings that can be (b) f (x) is one one and onto
defined from A to B is (c) f (x) is one one and into
(a) 144 (b) 12 (c) 24 (d) 64 (d) f (x) is many one and into
600 JEE Main Mathematics

1 1ù ì -1, x < 0
14. The function f : R ® é- , defined as ï
ëê 2 2 ûú 20. Let g ( x) = 1 + x - [ x ] and f ( x) = í 0, x = 0, then for
x ï 1, x > 0
f ( x) = is î
1 + x2 (JEE Main 2017)
all x, f [ g ( x)] is equal to
(a) invertible (a) x (b) 1 (c) f (x) (d) g (x)
(b) injective but not surjective 2
21. If fog =|sin x |and gof = sin x then f ( x) and g ( x)
(c) surjective but not injective
(d) neither injective nor surjective are
(a) f (x) = sin x , g (x) = x2
15. Let f : R ® R be defined as f ( x) = 2 x - 1 and
(b) f (x) = | x|, g (x) = sin x
1
x- (c) f (x) = x , g (x) = sin 2 x
g : R - {1} ® R be defined as g ( x) = 2 . Then, the
x -1 (d) f (x) = sin x , g (x) = x2

composition function f ( g ( x)) is (JEE main 2021) 22. If g ( x) = x 2 + x - 1 and ( gof ) ( x) = 4 x 2 - 10 x + 5,


5
(a) both one-one and onto then f æç ö÷ is equal to (JEE Main 2020)
(b) onto but not one-one è4ø
(c) neither one-one nor onto 1 3 3 1
(a) - (b) (c) - (d)
(d) one-one but not onto 2 2 2 2

Composition of Functions Inverse of Functions


1
16. Let f , g : R ® R be two functions defined as 23. If f : [1, ¥) ® [ 2 , ¥) is given by f ( x) = x + , then
f ( x) =|x |+ x and g ( x) =|x |- x, "x Î R. Then, fog x
and gof are respectively. f -1 ( x) equal to
ì0, x>0 x + x2 - 4 x x - x2 - 4
(a) í and 0 (a) (b) (c) (d) 1 + x2 - 4
2 1 + x2 2
î-4x, x<0
ì0, x>0 24. If the function f : [1, ¥) ® [1, ¥) is defined by
(b) í and 0
î2x, x<0 f ( x) = 2 x ( x -1) , then f -1 ( x) is equal to
x( x -1 )
ì0, x>0 æ1ö 1
(c) 0 and í (a) ç ÷ (b) (1 + 1 + 4 log 2 y)
î - 4 x, x<0 è2ø 2
1
(d) None of the above (c) (1 - 1 - 4 log 2 x ) (d) not defined
2
x
17. Let f : R ® R be defined by f ( x) = , then p 3p ù
1 + x2 25. If f : é , ® [ -1, 1] is defined by f ( x) = sin x, then
ëê 2 2 ûú
( fofof )( x) is f -1 ( x) is given by
x x
(a) (b) (c) x (d) 1 (a) sin -1 x (b) p + sin -1 x
2
1+ x 1 + 3 x2
(c) p - sin -1 x (d) None of these
( 4 x + 3) 2 p 2p
18. If f ( x) = , x ¹ , then the value of ( fof )( x) is, 26. Let f : é , ù ® [0, 4 ] be a function defined as
(6 x - 4) 3 êë 3 3 úû
2 f ( x) = 3 sin x - cos x + 2. Then, f -1 ( x) is given by
for all x ¹ .
3 æ x - 2ö p æ x - 2ö p
1 1 (a) sin -1 ç ÷- (b) sin -1 ç ÷+
(a) (b) x 2
(c) x (d) 2 è 2 ø 6 è 2 ø 6
x x 2p æ x - 2ö
(c) + cos -1 ç ÷ (d) None of these
19. Let f : R ® R be the Signum function defined as 3 è 2 ø
ì 1, x > 0 8 2 x - 8 -2 x
ï 27. The inverse function of f ( x) = , x Î ( -1, 1)
f ( x) = í 0, x = 0 and g : R ® R be the 8 2 x + 8 -2 x
ï -1, x < 0 is (JEE Main 2020)
î
greatest integer function given by g ( x) = [ x ] is 1 æ1 - xö 1 æ1 - xö
(a) (log 8 e) log e çç ÷÷ (b) log e çç ÷÷
greatest integer less than or equal to x. Then, fog 4 è 1 + xø 4 è 1 + xø
and gof do not coincide in 1 æ 1 + xö 1 æ 1 + xö
(c) (log 8 e) log e çç ÷÷ (d) log e çç ÷÷
(a) (0, 1) (b) [0, 1] (c) [0, 1) (d) (0, 1] 4 è1 - xø 4 è1 - xø
Relation and Function 601

Even and Odd Function Periodic Function


x
28. If ‘ f ’ is a real valued function not identically zero, 31. The period of the function f ( x) = sinæç sin ö÷ is
satisfying f ( x + y) + f ( x - y) = 2 f ( x) × f ( y), è 5ø
"x, y Î R, then f ( x) is (a) 2p (b) 2p / 5 (c) 10p (d) 5p
(a) odd 32. If f ( x + 2) + f ( x) = f ( x + 1), then
(b) even
(a) f (x + 1) = f (x)
(c) neither even nor odd
(b) f (x + 1) = 2 f (x)
(d) cannot say
(c) f (x + 2) = 2 f (x)
29. If f ( x) is a function that is odd and even (d) f (x) is a periodic function
simultaneously, then f (3) - f ( 2) is equal to
33. The function g ( x) = a |sin x |+ a 2|cos x |+ f ( a) has
(a) 1 (b) –1 (c) 0 (d) 2 p
cos x period equal to , then a is equal to
30. f ( x) = , (where x is not an integral multiple 2
é ù+1
x
(a) 2 (b) 3 (c) 1 (d) 4
êë p úû 2
|sin x|+|cos x|
of p and [´ ] denote the greatest integer function), is 34. The period of the function f ( x) = is
|sin x - cos x|
(a) an odd function (b) an even function
p p
(c) neither odd nor even (d) cannot be determined (a) (b) 2p (c) (d) p
4 2

ROUND II Mixed Bag


Only One Correct Option 3ö
6. For x Î æç 0, ÷, let f ( x) = x , g ( x) = tan x and
è 2ø
1. If f : R ® R be a function satisfying
1 - x2 p
f (2x + 3) + f (2x + 7) = 2 , "x Î R, then period of f (x) is h( x) = . If f( x) = (( hof ) og)( x), then f æç ö÷ is
1 + x2 è3ø
(a) 2 (b) 4 (c) 8 (d) 16
equal to (JEE Main 2019, 12 April)
2. If f ( x) and g ( x) are periodic functions with periods p 11p
(a) tan (b) tan
7 and 11 respectively. Then, the period of 12 12
7p 5p
x x (c) tan (d) tan
F ( x) = f ( x) gæç ö÷ - g ( x) f æç ö÷ is 12 12
è5 ø è3ø
(a) 177 (b) 222 (c) 433 (d) 1155 7. Let f : ( -1, 1) ® B be a function defined by
x |x | æ 2x ö
3. If f ( x) = - , then f -1 ( x) equals f ( x) = tan -1 çç ÷ , then f is both one-one and
2 ÷
1 + x2 è1 - x ø
|x| |x| onto when B is in the interval
(a) (b) - (sgn x)
1 - |x| 1 - |x| æ p pö é p pù
(a) ç - , ÷ (b) ê - , ú
x è 2 2ø ë 2 2û
(c) - (d) None of these é pö æ pö
1-x (c) ê0, ÷ (d) ç0, ÷
ë 2ø è 2ø
x2
4. Let f : N ® R; f ( x) = 2 x - 1, g : R ® R; g ( x) = ,
2 8. The function f ( x) = log ( x + x 2 + 1), is
then ( gof ) (0), is (a) an even function
(a) 1 (b) 1/2 (b) an odd function
(c) –1/2 (d) None of these (c) a periodic function
5. Let f ( x) = x 2 , x Î R. For any A Í R, define (d) neither an even nor an odd function
g ( A) = { x Î R : f ( x) Î A }. If S = [0, 4 ], then which one 9. The period of the function
of the following statements is not true? f ( x ) = sin 4 x + cos4 x is
(JEE Main 2019) p
(a) f ( g (S )) = S (b) g ( f (S )) ¹ S (a) p (b)
2
(c) g ( f (S )) = g (S ) (d) f(g(S)) ¹ f (S ) (c) 2p (d) None of these
602 JEE Main Mathematics

1
Numerical Value Type Questions 12. Let f ( x) = , then { fo ( fof )} (100) is equal to
1-x
10. Let A = { a, b, c} and B = {1, 2, 3, 4 }. Then the number ............ .
of elements in the set C = { f : A ® B|2 Î f ( A) and f
13. If P ( S) dentoes the set of all subsets of a given set S,
is not one-one} is (JEE Main 2020)
then the number of one-to-one functions from the set
11. If a + a = 1, b + b = 2 and S = {1, 2, 3} to the set P ( S) is ............ .
1 b |sin x|-|cos x|
af ( x) + af æç ö÷ = bx + , x ¹ 0, then the value of the 14. Let period of f ( x) = is l, then [ 4l] is
èxø x |sin x + cos x|
1
æ
f ( x) + f ç ö÷ equal to ............ where [×] dentoes greatest integer
expression è x ø is
1 function.
x+
x (JEE Main 2021)

Answers
Round I
1. (d) 2. (d) 3. (b) 4. (a) 5. (b) 6. (b) 7. (c) 8. (a) 9. (a) 10. (b)
11. (a) 12. (c) 13. (b) 14. (c) 15. (d) 16. (a) 17. (b) 18. (c) 19. (d) 20. (b)
21. (c) 22. (a) 23. (a) 24. (b) 25. (c) 26. (b) 27. (c) 28. (b) 29. (c) 30. (a)
31. (c) 32. (d) 33. (c) 34. (d)

Round II
1. (b) 2. (d) 3. (b) 4. (d) 5. (a) 6. (b) 7. (a) 8. (b) 9. (b) 10. (19)
11. (2) 12. (100) 13. (336) 14. (12)

Solutions
Round I Therefore, R is not reflexive.
1. Since, n divides n, "n Î N , R is reflexive, R is not Let ARB Þ A Ì B
symmetric, since, for 3, 6 Î N ,3R6 ¹ 6R3. This cannot be implied to B Ì A.
R is transitive, since for n, m, r whenever n/m and m/r For instance, if A = { 1, 2} and B = { 1, 2, 3}, then it cannot
Þ n/r, i.e., n divides m and m divides r, then n will be implied that B is related to A.
divide r. Therefore, R is not symmetric.
\ R is reflexive and transitive but not symmetric. Further, if ARB and BRC, then A Ì B and B Ì C.
2. Here, A = N , the set of natural numbers and Þ A Ì C Þ ARC. Therefore, R is transitive.
R = {(x, y) : y = x + 5, x < 4} Hence, R is not an equivalence relation since, it is not
= {(x, x + 5): x Î N and x < 4} symmetric.
= {(1, 6), (2, 7), (3, 8)} 4. This is because relation R is reflexive as
(a) For reflexive (x, x) Î R, " x putting y = x, x ¹ y + 5 (1, 1), (2, 2), (3, 3) Î R.
Þ (1, 1) Ï R. So, R is not reflexive. Relations R is symmetric since (1, 2), (2, 1) Î R and
(b) For symmetrical (x, y) Î R Þ ( y, x) Î R putting (1, 3), (3, 1) Î R.
y = x + 5, then x ¹ y + 5 Þ (1, 6) Î R but (6, 1) Ï R. So, But relation R is not transitive as (3, 1), (1, 2) Î R but
R is not symmetric. (3, 2) Ï R.
(c) For transitivity (x, y) Î R, ( y, z ) Î R Þ (x, z ) Î R if Now, if we add any one of the two pairs (3, 2) and (2, 3)
y = x + 5, z = y + 5,then z ¹ x + 5 . Since, (1, 6) Î R and (or both) to relation R, then relation R will become
there is no order pair in R which has 6 as the first transitive.
element same in the case for (2, 7) and (3, 8). So, R is Hence, the total number of desired relations is one.
transitive.
5. It is given that A = {1, 2, 3}.
3. Since, every set is a subset of itself, ARA for all An equivalence relation is reflexive, symmetric and
A Î P (X ). transitive.
Relation and Function 603

The smallest equivalence relation containing (1, 2) 4r


Then, there exists 4r Î N such that f (4r ) = = 2r
is given by, 2
R1 = {(1, 1), (2, 2), (3, 3), (1, 2), (2, 1)} Therefore, f is onto. Hence, f is not a bijective function.
Now, we are left with only four pairs i. e. , 9. For an onto function, the image set = Range. Now, for
(2, 3), (3, 2), (1, 3), and (3, 1). each element of the domain E, there exists two possible
If we add any one pair [say (2, 3)] to R1 , then for images 1 and 2 in the codomain.
symmetry we must add (3, 2 ). Thus, total number of functions = 24 = 16 (since, there
Also, for transitivity we are required to add (1, 3) and are 4 elements in domain). Two of these functions are
(3, 1). not onto (all the four elements have the same image
Hence, the only equivalence relation (bigger than R1 ) either 1 and 2). Thus, total number of onto functions
is the universal relation. = 16 - 2 = 14.
This shows that the total number of equivalence
10. Number of elements in A = 3
relations containing (1, 2) is two.
Number of elements in B = 5
6. For x1 , x2 Î R, consider Number of elements in A ´ B = 15
f (x1 ) = f (x2) A B
x1 x
Þ 2
= 2 2
x1 + 1 x2 + 1
3 5
Þ x1x22 + x1 = x2x12 + x2
Þ x1x2(x2 - x1 ) = x2 - x1
Þ x1 = x2 or x1x2 = 1
Number of one-one function
We note that there are point, x1 and x2 with x1 ¹ x2 and x = 5C3 ´ 3 !
1
f (x1 ) = f (x2), for instance, if we take x1 = 2 and x2 = , x = 5 ´4 ´3
2
2 2 1 x = 60
then we have f (x1 ) = and f (x2) = but 2 ¹ . A A× B
5 5 2
Hence, f is not one one. Also, f is not onto for if so then for
1 Î R $ x Î R such that f (x) = 1 3 15
x
which gives 2 = 1. But there is no such x in the
x +1
domain R, since the equation x2 - x + 1 = 0 does not give Number of one-one function
any real value of x. x = 15C3 ´ 3 !
7. The total number of injective mappings from the set Þ y = 15 ´ 14 ´ 13
containing 3 elements into the set containing 4 14
Þ y = 15 ´ 4 ´ ´ 13
elements is 4 P3 = 4 ! = 24. 4
7
8. Here, f : N ® N is defined as, Þ y = 60 ´ ´ 13
2
ìn + 1
ï , if n is odd Þ 2 y = (13) (7x)
f (n ) = í 2 for all n Î N . Þ 2 y = 91x
n
ï , if n is even
î 2 11. Let x, y Î (e, ¥ )
It can be observed that f (x) = f ( y)
1+1 2
f (1) = = 1 and f (2) = = 1 (by definition of f) log [log (log x)] = log [log (log y)]
2 2
log (log x) = log (log y)
\ f (1) = f (2), where 1 ¹ 2.
Therefore, f is not one one. Consider a natural number n log x = log y
in co domain N . x= y
Case I When n is odd. f (x) = log[log(log x)], x Î (e, ¥ )
Therefore, n = 2r + 1 for some r Î N . For x > e Þ log x > 1 Þ log(log x) > 0
Then, there exists 4r + 1 Î N such that Þ -¥ < log [log (log x)] < ¥
4r + 1 + 1 Þ f is onto function and one-one.
f (4r + 1) = = 2r + 1.
2 x2 + e
12. f (x) =
Therefore, f is onto. x2 + 1
Case II When n is even 2x (x2 + 1) - 2x (x2 + e) 2x3 + 2x - 2x3 - 2ex
Therefore, n = 2r for some r Î N f ¢ (x) = =
(x2 + 1)2 (x2 + 1)2
604 JEE Main Mathematics

2x - 2xe 2x(1 - e) ì 0, if x ³ 0
= = <0 g (x) = í
(x2 + 1)2 (x2 + 1)2 î-2x, if x < 0
f ¢ (x) < 0, f (x) is decreasing Therefore, gof gets defined as
Hence, f is one one function.
For x ³ 0, ( gof )(x) = g{ f (x)} = g (2x) = 0
x ® 0, f (x) ® e
and for x < 0, ( gof )(x) = g{ f (x)} = g (0) = 0
x ® ¥ , f (x) ® e
Consequently, we have ( gof )(x) = 0, "x Î R
Hence, range = (1, e) = codomain
Similarly, fog gets defined as
13. f (x) = x3 + 3x2 + 12x - 12x - 2 sin x
For x ³ 0, ( fog )(x) = f { g (x)} = f (0) = 0
f ¢ (x) = 3x2 + 6x + 12 - 2 cos x
and for x < 0, ( fog )(x) = f { g (x)} = f (-2x) = - 4x
f ¢ (x) > 0 for all values of x
ì 0, x>0
\ f (x) is an increasing function, so f (x) is one one and i.e., ( fog )(x) = í
î - 4 x, x<0
onto
x x
14. We have, f (x) = 17. Given that, f (x) =
1 + x2 1 + x2
1 ( fof )x = { f ( f (x))}
æ1ö x x
\ f ç ÷= x = = f (x)
è x ø 1 + 1 1 + x2 æ x ÷ ö 1 + x2 x
x2 =f ç = =
ç 1 + x2 ÷ x 2
1 + 2 x2
æ1ö æ1ö è ø 1+
\ f ç ÷ = f (2) or f ç ÷ = f (3) and so on. 2
è2ø è3ø 1+ x
So, f (x) is many-one function. Now, ( fofof )x = { f ( fof )x }x
x x
Again, let y = f (x) Þ y =
1 + x2 æ x ö 1 + 2 x2 x
=f ç ÷= =
Þ y + x2y = x Þ yx2 - x + y = 0 ç 1 + 2 x2 ÷ x 2
1 + 3 x2
è ø 1+
As, x ÎR 1 + 2 x2
é - 1 1ù
\ (- 1)2 - 4 ( y)( y) ³ 0 Þ 1 - 4 y2 ³ 0 Þ y Î ê , (4x + 3) 2
ë 2 2 úû 18. It is given that f (x) = ,x¹ .
(6x - 4) 3
é - 1 1ù
\ Range = Codomain = ê , æ (4x + 3) ö
ë 2 2 úû ( fof ) (x) = f ( f (x)) = f çç ÷÷
So, f (x) is surjective. è (6x - 4) ø
Hence, f (x) is surjective but not injective. æ 4x + 3 ö
4 çç ÷+3
6x - 4 ÷ø 16x + 12 + 18x - 12 34x
15. f ( g (x)) = 2 g (x) - 1 = è = = =x
æ 4x + 3 ö 24x + 18 - 24x + 16 34
æ 1ö 6ç ç ÷
÷ -4
çx - ÷ è 6x - 4 ø
è 2ø x
=2 = 2
x-1 x-1 Therefore, fof (x) = x, for all x ¹ Þ fof = I
3
1
f ( g (x)) = 1 + Hence, the given function f is invertible and the inverse
x-1 of f is itself.
one-one, into ì 1, x > 0
ï
19. It is given that f : R ® R is defined as f (x) = í 0, x = 0
ï- 1 , x < 0
î
(1, 1) Also, g : R ® R is defined as g (x) = [x], where [x] is the
1 greatest integer less than or equal to x. Now, let x Î (0, 1].
Then, we have
[x] = 1 if x = 1 and [x] = 0 if 0 < x < 1
ì f (1), if x = 1
\ fog (x) = f { g (x)} = f ([x]) = í
16. Here, f (x) = | x| + x which can be redefined as î f (0), if x Î (0, 1)
ì2x if x ³ 0 ì1, if x = 1
f (x) = í =í
î0 if x < 0 î0, x Î (0, 1)
Similarly, the function g defined by g (x) = | x| - x may gof (x) = g{ f (x)} = g(1) = [1] = 1 (Q x > 0)
be redefined as Thus, when x Î(0, 1), we have fog (x) = 0 and gof (x) = 1.
Relation and Function 605

But fog (1) ¹ gof (1) 1 ± 1 + 4 log 2 y


Þ x=
Hence, fog and gof do not coincide in (0, 1]. 2
20. g (x) = 1 + x - [x] = 1 + { x } ...(i) But as x ³ 1
1 + 1 + 4 log 2 y
where { x } is the fractional part of x. Þ x=
2
Thus, g (x) ³ 1 (Q 0 £ { x} < 1)
ì-1, g (x) < 0 25. Obviously f is one one and onto, thus f -1 exists. Since,
ï
Now, f [ g (x)] = í 0, g (x) = 0 p p p 3p
- £ sin -1 x £ , £ p - sin -1 x £
ï 1, g (x) > 0 2 2 2 2
î
æ pö
From Eq. (i), we get f [ g (x)] = 1 26. f (x) = 3 sin x - cos x + 2 = 2 sin ç x - ÷ + 2
è 6ø
21. fog = f { g (x)} = |sin x| = sin 2 x
Since, f (x) is one one and onto, f is invertible.
Also, gof = g{ f (x)} = sin 2 x æ pö
Now, fof -1 (x) = x Þ 2 sin ç f -1 (x) - ÷ + 2 = x
Obviously, sin 2 x = g (x ) and sin 2 x = sin 2 { f (x)} è 6ø
æ -1 pö x
i.e., g (x) = sin 2 x and f (x) = x. Þ sin ç f (x) - ÷ = - 1
è 6ø 2
22. Since the g (x) = x2 + x - 1 and gof (x) = 4x2 - 10x + 5 both -1 -1 æ x ö p
Þ f (x) = sin ç - 1 ÷ +
are quadratic equations, therefore f (x) must be a linear è2 ø 6
equation. Because|(x/2) - 1| £ 1 for all x Î [0, 4].
Now, let f (x) = ax + b, Hence, (b) is the correct answer.
So, gof (x) = 4x2 - 10x + 5 82x - 8-2x
Þ g (ax + b) = 4x - 10x + 5 2 27. Given function, f (x) = , x Î (-1, 1)
82x + 8-2x
Þ (ax + b)2 + (ax + b) - 1 = 4x2 - 10x + 5
84x - 1
[Q g (x) = x2 + x - 1] = =y (let)
84x + 1
Þ a x + a (2b + 1)x + (b + b - 1) = 4x2 - 10x + 5
2 2 2

On applying componendo and dividendo law, we get


On comparing the coefficients of terms, we get
1+ y
a2 = 4 …(i) 84x =
1- y
a (2b + 1) = - 10 …(ii)
On applying logarithm having base ‘8’ both sides, we get
b2 + b - 1 = 5 …(iii)
æ 1 + yö
From Eq. (i), we have a = ± 2 4x = log 8 çç ÷÷
è1 - yø
From Eq. (iii), we have b = - 3, 2
1 æ1 + yö
and the equation is getting satisfied by a = - 2 and b = 2, Þ x= log 8 çç ÷
4 è1 - y ÷ø
so f (x) = - 2x + 2
æ5ö æ5ö 5 1 1 æ1 + yö
\ f ç ÷ = - 2ç ÷ + 2 = - + 2 = - = (log 8 e) log e çç ÷
è4ø è4ø 2 2 4 è1 - y ÷ø
1 {by base change property of logarithm
23. Given, y = f (x) = x +
x log a b = log a e × log e b}
2
Þ xy = x + 1 Þ x2 - xy + 1 = 0 By interchanging the variables x and y, we get the
2
y± y -4 inverse function of f (x) and it is
Þ x=
2 1 æ 1 + xö
f -1 (x) = (log 8 e) log e çç ÷÷ .
y± y2 - 4 4 è1 - xø
Þ f -1 ( y) =
2 Hence, option (c) is correct.
-1 x ± x2 - 4
Þ f (x) =
2 28. f (x + y) + f (x - y) = 2 f (x) f ( y)
Since, f -1: [2 , ¥ ) ® [1, ¥ ), we get
Put x = 0, y = 0
x + x2 - 4
y= f -1
(x) = f (0) + f (0) = 2 f 2(0)
2
f (0) ¹ 0, f (0) = 1
24. Let y = 2x( x -1) where y ³ 1 as x ³ 1
Put x = 0,
Þ log 2 y = x(x - 1)
f ( y) + f (- y) = 2 f (0) f ( y)
Þ x2 - x - log 2 y = 0
606 JEE Main Mathematics

f ( y) + f (- y) = 2 f ( y) f (2x + 11) = f (2x + 3)


f (- y) = f ( y) \ Period of the function = 4
f (- x) = f (x), even 2. Period of f (x) is 7.
æ xö 7
29. f (x) = 0, " x Î R \ Period of f ç ÷ is = 21 and period of g (x) is 11.
è 3 ø 1 /3
Þ f (3) - f (2) = 0 11
æ xö
Hence, (c) is correct answer. \ Period of g ç ÷ is = 55
5
è ø 1 /5
cos (- x) cos x æ xö
30. f (-x) = = \ T1 = Period of f (x) g ç ÷ = 7 ´ 55 = 385
é xù 1 é xù 1
è5ø
êë - p úû + 2 - êë p úû - 1 + 2
æ xö
and T2 = Period of g (x) f ç ÷ = 11 ´ 21 = 231
æ x é xù é xù ö è3ø
ças x ¹ np Þ Ï I , so as ê - ú = - ê ú - 1÷
è p ë p û p
ë û ø \ Period of F (x) = LCM {T1 , T2}
cos x = LCM { 385,231 }
= = - f (x) Þ f (x) is an odd function.
é xù 1 = 7 ´ 11 ´ 3 ´ 5 = 1155
+
êë p úû 2 ì - x2
ïï 2
, if x ³ 0
Hence, (a) is correct answer. 3. Let y = f (x) = í1 + 2x
ì æ xö ü ï x , if x < 0
31. f (x) = sin ísin ç ÷ ý ïî1 + x2
î è5øþ
ì -y ì -x
ì æ x + 10p ö ü ï , y<0 ï , x<0
Þ f (x + 10p ) = sin ísin ç ï 1+ y ï 1+ x
÷ý f -1 ( y) = í Þ f -1 (x) = í
î è 5 øþ ï- y ï- æ x ö
, y>0 ç ÷, x>0
ì æ xö ü ì æ xö ü ïî 1 - y ïî è 1 - xø
Þ f (x + 10p ) = sin ísin ç2p + ÷ ý = sin ísin ç ÷ ý = f (x)
î è 5øþ î è5øþ ì æ -x ö
ï ç ÷, x<0
Therefore, the period of f (x) is 10p. ï è 1 + xø |x|
\ f -1 (x) = -í = - (sgn x)
32. f (x + 2) + f (x) = f (x + 1) ...(i) ï æ -x ö 1 - |x|
ç ÷, x>0
ïî è 1 - xø
Þ f (x + 3) + f (x + 1) = f (x + 2) ...(ii)
(putting x = x + 1) x2
4. f (x) = 2x - 1 Þ g (x) =
On adding Eqs. (i) and (ii), we get 2
f (x + 3) = - f (x) g{ f (0)} is undefined as 0 Ï N .
Þ f (x + 6) = - f (x + 3) 5. Given, functions f (x) = x2, x Î R
Þ f (x) = f (x + 6) and g ( A ) = { x Î R : f (x) Î A }; A Í R
So, f (x) is a periodic function. Now, for S = [0, 4]
33. If a = 1, g (x) = |sin x|+ |cos x|+ f (1), then period of g (S ) = { x Î R : f (x) Î S = [0, 4]}
p = { x Î R : x2 Î [0, 4]} = { x Î R: x Î [-2, 2]}
g (x) =
2 Þ g (S ) = [-2, 2]
p So, f ( g (S )) = [0, 4] = S
34. Since, period of |sin x|+ |cos x| is and period of
2 Now, f (S ) = { x2 : x Î S = [0, 4]} = [0, 16]
|sin x - cos x|is p so period of and g ( f (S )) = { x Î R : f (x) Î f (S ) = [0, 16]}
|sin x|+ |cos x| = { x Î R : f (x) Î [0, 16]}
is p.
|sin x - cos x| = { x Î R: x2 Î [0, 16]}

Round II = { x Î R : x Î [-4, 4]} = [-4 ,4]


1. Given, f (2x + 3) + f (2x + 7) = 2 …(i) From above, it is clear that g ( f (S )) ¹ g (S ).
On replacing x by x + 1 and x by x + 2 in Eq. (i), we get
6. Given, for x Î (0, 3 / 2), functions
f (2x + 5) + f (2x + 9) = 2 …(ii)
f (x) = x … (i)
and f (2x + 7) + f (2x + 11) = 2 …(iii)
g (x) = tan x … (ii)
Now, on subtracting Eq. (i) from Eq. (iii), we get
Relation and Function 607

1 - x2 10. The desired functions will contain either one element or


and h (x) = … (iii)
1 + x2 two elements in its co-domain and ‘2’ will be the element
of the each co-domain.
Also given, f(x) = ((hof )og )(x) = (hof ) ( g (x))
So, the possible co-domain are {2}, {1, 2,}, {2, 3} or {2, 4}
= h ( f ( g (x))) = h ( f (tan x))
\Total number of functions = 1 + 3 ´ (23 - 2)
1 - ( tan x )2
= h ( tan x ) = = 1 + (3 ´ 6) = 19.
1 + ( tan x )2
æ1ö b
1 - tan x æp ö 11. af (x) + af ç ÷ = bx + …(i)
= = tan ç - x÷ è xø x
1 + tan x è 4 ø
1
æ pö æ p pö æ 3p - 4p ö x®
Now, f ç ÷ = tan ç - ÷ = tan ç ÷ x
è3ø è4 3ø è 12 ø 1 b
æ ö
æ pö æ pö af ç ÷ + af (x) = + bx …(ii)
= tan ç - ÷ = - tan ç ÷ è xø x
è 12 ø è 12 ø
From Eqs. (i) and (ii), we get
æ pö æ 11p ö
= tan ç p - ÷ = tan ç ÷ é æ1öù æ 1ö
è 12 ø è 12 ø (a + a ) ê f (x) + f ç ÷ ú = ç x + ÷ (b + b )
ë è xø û è xø
7. Given, x Î (-1, 1) æ1ö
f (x) + f ç ÷
Þ tan -1
æ p pö æ p pö
x Î ç - , ÷ Þ 2 tan -1 x Î ç - , ÷ Þ è xø = 2 = 2
è 4 4ø è 2 2ø 1 1
x+
æ 2x ö x
Given that, f (x) = tan -1 çç ÷ = 2 tan -1 x [Q x2 < 1] 1

è1 - x ø 12. We have, f (x) =
1-x
æ p pö
So, f (x) Î ç - , ÷ æ 1 ö 1 x-1
è 2 2ø \ fof (x) = f ( f (x)) = f çç ÷÷ = =
è 1 - x ø 1- 1 x
Hence, function is one-one onto.
1-x
8. Given that, æ x - 1ö 1
Again, f {( fof )} (x) = f ç ÷= =x
f (x) = log (x + x2 + 1 ) è x ø 1 - x-1
x
Now, f (- x) = log (- x + x2 + 1 )
\ f {( fof )} (100) = 100
\ f (x) + f (- x) = log (x + x2 + 1 ) + log (- x + x2 + 1 )
13. Number of elements in P (S ) is 23 = 8
= log (1) = 0
\ Number of one-to-one functions from S to
Hence, f (x) is an odd function.
P (S ) = 8C3 ´ 3 ! = 336
4 4
9. Given that, f (x) = sin x + cos x
14. We observe that
\ f (x) = (sin 2 x + cos 2 x)2 - 2 sin 2 x cos 2 x |sin (p + x)| - |cos (p + x)|
f (x + p ) =
1 |sin (p + x) + cos(p + x)|
= 1 - (2 sin x cos x)2
2 |- sin x| - |- cos x| |sin x| - |cos x|
1 = = = f (x)
= 1 - (sin 2 x)2 |- sin x - cos x| |sin x + cos x|
2
\ f (x + p ) = f (x)
1 æ 1 - cos 4x ö
=1 - ç ÷ So, f (x) has period p.
2è 2 ø
3 1 \ l=p
= + cos 4x \ [4l ] = [4p ] = 12
4 4
2p p
\The period of f (x) = =
4 2
[Q cos x is periodic with period 2 p]
25
Inverse Trigonometric
Functions

Inverse Trigonometric Function IN THIS CHAPTER ....


If a function is one-one and onto from A to B, then function g which associates Inverse Trigonometric Function
each element y Î B to one and only one element x Î A, such that y = f ( x ), then Graphs of Inverse Trigonometric
g is called the inverse function of f , denoted by x = g ( y ). Functions
Q g = f -1 Domain and Range of Inverse
\ x = f -1( y ) Trigonometric Functions

Consider the sine function with domain R and range [- 1, 1] . It is clear that Properties of Inverse
this is not a one-one and onto (bijection), so it is not invertible. So, we have to Trigonometric Functions
restrict the domain of it in such a way that it becomes one-one, then it would Sum and Difference of Inverse
become invertible. Trigonometric Function
é p pù
Suppose we consider sine as a function with domain ê - , ú and codomain
ë 2 2û
[- 1, 1 ], then it is a bijection and therefore, invertible, such types of
trigonometric functions are called inverse trigonometric functions.

Principal Value of Inverse Trigonometric Functions


Since all the trigonometric function are many one functions over their
domains, their domains and codomains needs to be restrict to make them
one-one and onto.
Lets consider the sine function. Domain of sine function is the set of all real
numbers and range is [- 1, 1]. It can be restricted to any of the intervals
[- 3p/ 2, - p/ 2], [- p/ 2, p/ 2], [p/ 2, 3p/ 2], ... etc. to make it one-one and onto.
The inverse of the sine function can be defined in each of these intervals.
Therefore, sin-1 x is a function whose domain is [- 1, 1] and the range could be
any of the intervals [- 3p/ 2, - p/ 2], [- p/ 2, p/ 2], [p/ 2, 3p/ 2], ... corresponding to
each such interval, there is a branch of the sine function. The branch with
range [- p/ 2, p/ 2] is called the principal value branch.
Hence, the domain of sin-1 x function is [- 1, 1] and range is [- p/ 2, p/ 2].
Inverse Trigonometric Functions 609

Principal Value of Inverse Trigonometric Functions Y –1


y = sin x
S. No. Functions Interval of principal value p/2
y=x
- p /2 £ y £ p /2 ,
1. sin-1 x
where y = sin-1 x 1
0 £ y £ p,
2. cos -1 x y = sin x
where y = cos -1 x -p/2 -1
- p /2 < y < p /2 , X¢ X
3. tan-1 x 1 p/2
where y = tan-1 x
- p /2 £ y £ p /2 ,
4. cosec -1 x
where y = cosec -1 x, y ¹ 0 -1

0 £ y £ p,
5. sec -1 x
where y = sec -1 x, y ¹ p /2 -p/2
0 < y < p, Y¢
6. cot -1 x
where y = cot -1 x
Note sin -1 x is an increasing function in [–1, 1].
Note If no branch of an inverse trigonometric function is
(ii) cosec y = x Þ y = cosec-1 x ,
mentioned, then it means the principal value branch of the function.
é p ö æ pù
where y Î ê - , 0÷ È ç 0, ú
Example 1. The principal value of cos-1 (cos(2 cot -1( 2 - 1))) ë 2 ø è 2û
is equal to and x Î ( -¥ , - 1] È [1, ¥ )
2p p 3p 5p
(a) (b) (c) (d)
3 4 4 6 Y
y = cosec x
-1 -1
Sol. (c) We have, cos (cos (2 cot ( 2 - 1)))
æ æ 3p ö ö é 3p ù p/2 y=x
Þ cos-1 ç cos ç2 ´ ÷÷ êëQ cot 8 = 2 - 1úû
è è 8 øø
1
æ 3p ö 3p y = sin x
Þ cos-1 ç cos ÷= -p/2
è 4ø 4 X¢ -1
X
1 p/2
æ 5p ö -1æ 2p ö
Example 2. If tan- 1 ç tan ÷ = a , tan ç - tan ÷ = b, -1
è 4ø è 3 ø
then -p/2

(a) 4 a - 4b = 0 (b) 4 a - 3 b = 0
(c) a > b (d) None of these Y¢
é 5p ù é æ p öù Note cosec -1 x is a decreasing function in (-¥, - 1.)
Sol. (b) Clearly, tan - 1 ê tan = tan - 1 ê tan ç p + ÷ ú
ë 4 úû ë è 4 øû
It also decreases in [1, ¥ ).
æ pö p p
= tan - 1 ç tan ÷ = Þ a = (iii) cos y = x Þ y = cos-1 x ,
è 4ø 4 4
where y Î [0, p ] and x Î [-1, 1]
é æ 2p ö ù -1 é æ p öù
and tan - 1 ê - tan ç ÷ ú = tan ê - tan ç p - ÷ ú
ë è 3 ûø ë è 3 øû Y

æ pö p p p
= tan - 1 ç tan ÷ = Þ b= y = cos–1x
è 3ø 3 3
p
æ ö p
æ ö
Now, consider 4a - 3 b = 4 ç ÷ - 3 ç ÷ = 0 p/2 y=x
è 4ø è3ø
1

Graphs of Inverse p/2 p


Trigonometric Functions X¢
–1 y = cos x
X

(i) sin y = x Þ y = sin-1 x ,


é p pù Y¢
where y Î ê - , ú and x Î [-1, 1] -1
ë 2 2û Note cos x is a decreasing function in [-1, 1] .
610 JEE Main Mathematics

(iv) sec y = x Þ y = sec-1 x , Domain and Range of Inverse


é pö æp ù Trigonometric Functions
where y Î ê 0, ÷ È ç , p ú
ë 2 ø è2 û
Suppose we defined a sine inverse trigonometric function
and x Î ( -¥ , - 1] È [1, ¥ ) é p pù é p pù
as sin-1 ê - , ú ® [-1, 1]. Here, ê - , ú and [- 1, 1] are
Y ë 2 2û ë 2 2û
called domain and range of sine inverse trigonometric
y=x function.
p
Domain and Range of Inverse Trigonometric Functions
p/2
y = sec –1 x S. No. Functions Domain Range
1
X¢ X -1
–1 1 p/2 p 1. sin x [–1, 1] [-p /2 , p /2 ]
2. cos -1 x [–1, 1] [0, p ]
–1 3. tan -1
x R ( -p /2 , p /2 )
4. cosec -1 x ( -¥, - 1] È [1, ¥) [- p /2 , p /2 ] - { 0}
5. sec -1 x ( -¥, - 1] È [1, ¥) [0, p ] - { p /2}
y = sec x
Y¢ 6. cot -1 x R ( 0, p )
Note sec -1 x is an increasing function in (-¥, - 1]. It also
increases in [1, ¥ ).
Example 3. Range of f ( x) = sin-1 x + tan-1 x + sec-1 x is
æ p 3p ö é p 3p ù
(v) tan y = x Þ y = tan-1 x , (a) ç , ÷ (b) ê ,
è4 4 ø ë 4 4 úû
æ p pö ì p 3p ü
where y Î ç - , ÷ (c) í ,
è 2 2ø ý (d) None of these
î4 4 þ
and x Î ( -¥ , ¥ )
Sol. (c) We have, f ( x ) = sin -1 x + tan -1 x + sec-1 x
Y
Domain of sin -1 x is [ -1, 1]
y = tan x y=x
p/2 Domain of tan -1 x is ( -¥ , ¥)
Domain of sec-1 x is ( -¥ , - 1] È [1, ¥)
–1
y = tan x

\ Domain of f ( x) = {-1, 1}

–p/2 Thus, range of f ( x) = {f (1), f ( -1)}
X
p/2 p p 3p
i.e. sin -1 1 + tan -1 1 + sec-1 1= + + 0 =
2 4 4
-1 -1 -1 p p p
and sin ( -1) + tan ( -1) + sec ( -1) = - - + p =
–p/2 2 4 4
ì p 3p ü
\ Range is í , ý.
Y¢ î4 4 þ
-1
Note tan x is an increasing function in R. æ| x| + 5 ö
Example 4. The domain of the function f ( x) = sin-1ç ÷
(vi) cot y = x Þ y = cot -1
x, è x2 + 1 ø
where y Î( 0, p ) is ( -¥, - a] È [ a, ¥). Then, a is equal to (JEE Main 2020)

and x Î ( -¥ , ¥ ) 17 17 - 1 1 + 17 17
(a) +1 (b) (c) (d)
Y 2 2 2 2
p y = cot x y=x æ| x| + 5 ö
Sol. (c) Given function, f ( x) = sin -1ç 2 ÷
y = cot –1
x è x +1ø
p/2
For, domain of ‘f ’
| x| + 5 | x| + 5
X¢ X 2
£ 1, as 2 > 0 " x ÎR
p/2 p x +1 x +1
Þ | x| + 5 £ x2 + 1 as x2 + 1 is positive " x ÎR
Þ x2 - | x| - 4 ³ 0

Þ | x|2 - | x| - 4 ³ 0 [Q " x ÎR , x2 = | x|2 ] … (i)
Note tan -1 x is a decreasing function in R.
Inverse Trigonometric Functions 611

1 ± 1 + 16 Inverse Sum Identities


Q For| x|2 - | x| - 4 = 0 Þ| x| =
2 p
1 ± 17 1 + 17 (i) sin-1 x + cos-1 x = , "x Î [-1, 1 ]
Þ | x| = Þ | x| = as| x| ³ 0 2
2 2 p
17 + 1 (ii) tan-1 x + cot-1 x = , "x Î R
\ | x| ³ 2
2 p
æ 17 + 1ù é 17 + 1 ö (iii) sec-1 x + cosec-1 x = , "x Î ( -¥ , - 1] È [1, ¥ )
Þ x Î ç - ¥, - ú Èê , ¥÷ 2
è 2 û ë 2 ø
Conversion of One Inverse to Another
17 + 1
\ a= Inverse Trigonometric Function
2
æ x ö
(i) sin-1 x = cos-1 1 - x 2 = tan-1 ç ÷
Properties of Inverse ç 1 - x2 ÷
è ø
Trigonometric Functions æ 1 - x2 ö æ 1 ö
= cot-1 ç ÷ = sec-1 ç ÷
Self Adjusting Property ç x ÷ ç 1 - x2 ÷
è ø è ø
é p pù -1 æ 1 ö
(i) sin-1 (sin q ) = q ; " q Î ê - , ú = cosec ç ÷
ë 2 2û èxø
(ii) cos-1 (cos q ) = q ; "q Î [q , p ] æ 1 - x2 ö
(ii) cos-1 x = sin-1 1 - x 2 = tan-1 ç ÷
é p pù ç x ÷
(iii) tan-1 (tan q ) = q ; " q Î ê - , ú è ø
ë 2 2û æ x ö æ 1 ö
= cot-1 ç ÷ = sec-1 ç ÷
é p pù ç 1 - x2 ÷ èxø
(iv) cosec-1 ( cosec q ) = q ; " q Î ê - , ú , q ¹ 0 è ø
ë 2 2û æ ö
1
p = cosec-1 ç ÷
(v) sec-1 ( sec q ) = q ; " q Î [0, p ], q ¹ ç 1 - x2 ÷
2 è ø
(vi) cot-1 (cot q ) = q ; " q Î ( 0, p ) æ x ö æ 1 ö
(iii) tan-1 x = sin-1 ç ÷ = cos-1 ç ÷
(vii) sin (sin -1
x ) = x , "x Î [-1, 1] ç 1 + x2 ÷ ç 1 + x2 ÷
è ø è ø
-1
(viii) cos (cos x ) = x , "x Î [-1, 1] æ1ö
= cot-1 ç ÷ = sec-1 ( 1 + x 2 )
(ix) tan (tan -1
x ) = x , "x Î R èxø
æ 1 + x2 ö
(x) cosec ( cosec-1 x ) = x , "x Î ( -¥ , - 1] È [1, ¥ ) = cosec-1 ç ÷
ç x ÷
(xi) sec ( sec-1 x ) = x , "x Î ( -¥ , - 1] È [1, ¥ ) è ø
(xii) cot (cot-1 x ) = x , "x Î R
Example 5. If x = sin-1(sin 10) and y = cos-1(cos 10), then
Negative Arguments y - x is equal to (JEE Main 2019)
-1 -1 (d) p
(i) sin ( - x ) = - sin ( x ), "x Î [-1, 1] (a) 0 (b) 10 (c) 7p
(ii) cos-1 ( - x ) = p - cos-1 ( x ), "x Î [-1, 1] Sol. (d) The graph of y = sin - 1(sin x ) is
-1 -1
(iii) tan ( - x ) = - tan x , "x Î R Y
-1 -1
(iv) cosec ( - x ) = - cosec x , "x Î ( -¥ , - 1] È [1, ¥ ) y=p–x y=–2p+x y=–x+3p
x
y=

-1 -1
(v) sec ( - x ) = p - sec x , "x Î ( -¥ , - 1] È [1, ¥ ) –p/2 3p/2
X
(vi) cot-1 ( - x ) = p - cot-1 x , "x Î R
p/2 p 2p 5p/2 3p

Reciprocal Arguments \ x = sin - 1(sin 10) = - 10 + 3p ...(i)


-1
æ1ö and the graph of y = cos (cos x ) is
(i) sin-1 ç ÷ = cosec-1 x , "x Î ( -¥ , - 1] È [1, ¥ )
èxø Y
x

y=
y=

p+
x

æ1ö
–x
y=

2p

–2

(ii) cos-1 ç ÷ = sec-1 x , "x Î ( -¥ , - 1] È [1, ¥ )


+
–x

y=

4p

èxø
X
-1 O
æ 1 ö ì cot x ,
p 2p 3p 10 4p
-1 "x > 0
(iii) tan ç ÷ =í
è x ø î - p + cot-1 x , " x < 0
\ y = cos- 1(cos 10) = - 10 + 4p ...(ii)
612 JEE Main Mathematics

Now, from Eqs. (i) and (ii), Sum and Difference of


y - x = ( - 10 + 4p) - ( - 10 + 3p) = p Inverse Trigonometric Function
Example 6. The greatest and least values of (i) sin-1 x + sin-1 y
(sin- 1 x) 2 + (cos- 1 x) 2 are, respectively ì sin -1 { x 1 - y 2 + y 1 - x 2 }, if - 1 £ x , y £ 1 and
ï
p4 p -p ï x 2 + y 2 £ 1 or if xy < 0
(a) and 0 (b) and ï and x 2 + y 2 > 1
4 2 2 ï
ï p - sin -1 {x 1 - y 2 + y 1 - x 2 }, if 0 < x , y £ 1
5p 2 p2 p2 - p2 =í
(c) and (d) and and x 2 + y 2 > 1
4 8 4 4 ï
ï - p - sin -1 { x 1 - y 2 + y 1 - x 2 }, if - 1 £ x , y < 0,
Sol. (c) We have, (sin - 1 x) 2 + (cos- 1 x) 2 ï
ï and x 2 + y 2 > 1
= (sin - 1 x + cos- 1 x) 2 - 2 sin - 1 x × cos- 1 x ï
î
p2 æp ö
= - 2 sin - 1 x ç - sin - 1 x÷ (ii) sin-1 x - sin-1 y
4 è2 ø
ì sin -1 { x 1 - y2 - y 1 - x 2 }, if - 1 £ x , y £ 1 and
p2 ï
= - p sin - 1 x + 2 (sin - 1 x ) 2 ï x 2 + y2 £ 1 or if xy > 0
4 ï
ï and x 2 + y2 > 1
é p p2ù
= 2 ê(sin - 1 x ) 2 - sin - 1 x + ú ïï p - sin { x 1 - y - y 1 - x }, if 0 < x £ 1,

- 1 2 2

ë 2 8û -1 £ y £ 0 and x 2 + y2 > 1
ï
éæ ï
p2ù
2 -1
pö 2 2
ï -p - sin { x 1 - y - y 1 - x }, if - 1 £ x < 0,
= 2 ê ç sin - 1 x - ÷ + ú ï
êë è 4ø 16 úû
ï 0 < y £ 1 and x 2 + y2 > 1
ïî
æ p2ö p2
Thus, the least value is 2 ç ÷ i. e. and the greatest value
è 16 ø 8 Note
-1 p
éæ - p p ö p
2 2ù
5p 2 • If sin x + sin -1 y + sin -1 z = , then x 2 + y 2 + z 2 + 2 xyz = 1
is 2 ê ç - ÷ + ú i. e. . 2
è
êë 2 4 ø 16 úû 4 -1
• If sin x + sin -1 y + sin -1 z = p , then

pæ x 1 - x 2 + y 1 - y 2 + z 1 - z 2 = 2 xyz
æ2ö -1æ 3 ö 3ö
Example 7. If cos-1ç ÷ + cos ç ÷ = ç x > ÷, then x is
è 3x ø è 4x ø 2 è 4ø (iii) cos-1 x + cos-1 y
equal to (JEE Main 2019) ì cos-1 { xy - 1 - x 2 1 - y 2 }, -1 £ x , y £ 1
145 146 145 145 ï
(a) (b) (c) (d) ï and x + y ³ 0
10 12 12 11 =í
-1 2
ï 2p - cos { xy - 1 - x 1 - y },
2 -1 £ x , y £ 1
æ2ö æ3ö p
Sol. (c) We have, cos-1ç ÷ + cos-1ç ÷ = ï and x + y £ 0
è 3x ø è 4x ø 2 î
æ 2 ö p æ 3 ö (iv) cos-1 x - cos-1 y
Þ cos-1 ç ÷ = - cos-1 ç ÷
è 3x ø 2 è 4x ø
ì cos-1 { xy + 1 - x 2 1 - y 2 }, -1 £ x , y £ 1
-1 æ
2ö æ3ö ï
Þ cos ç ÷ = sin -1 ç ÷ ï and x £ y
è 3x ø è 4x ø =í
æ 9 ö é
-1 2
ï - cos { xy + 1 - x 1 - y },
2 -1 £ y£ 0
æ2ö
Þ cos-1 ç ÷ = cos-1 ç 1 - ÷ Q sin -1 x = cos-1 ( 1 - x2) ùú ï 0 < x £ 1 and x ³ y
è 3x ø è 16x2 ø êë û î
2 9 Note
Þ = 1-
3x 16x2 • If sin -1 x + sin -1 y = q, then cos -1 x + cos -1 y = p - q
4 9 -1
x + cos -1 y = q, then sin -1 x + sin -1 y = p - q
Þ = 1- • If cos
9x2 16x2
4 9 (v) tan-1 x + tan-1 y
Þ + = x2
9 16 ì -1 æ x + y ö
64 + 81 ï tan ç ÷, if xy < 1
Þ x2 = è 1 - xy ø
144 ï
ï æx+ yö
2 145 = í p + tan-1 ç ÷,
Þ x =
144 ï è 1 - xy ø if x > 0, y > 0 and xy > 1
145 ï -1 æ x + y ö
Þ x= ï - p + tan çè 1 - xy ÷ø , if x < 0, y < 0 and xy > 1
12 î
Inverse Trigonometric Functions 613

(vi) tan-1 x - tan-1 y æ 4 5 ö


ç -1 + -1 16
÷
ì = 2p - ç tan 3 12 + tan ÷
-1 æ x - y ö 5
ï tan ç ÷, if xy > - 1 ç 1- 63 ÷
è 1 + xy ø è ø
ï 9
ï æx- yö æ -1æ 21 ´ 3 ö -1 16 ö
= í p + tan-1 ç ÷, = 2p - ç tan ç ÷ + tan ÷
ï è 1 + xy ø if x > 0, y < 0 and xy < - 1 è è 4´4ø 63 ø
ï -1 æ x - y ö æ 63 16 ö
ï - p + tan çè 1 + xy ÷ø , if x < 0, y > 0 and xy < - 1 = 2p - ç tan -1 + tan -1 ÷
î è 16 63 ø
(vii) If x 1, x 2, x 3 , . . . , x n Î R, then æ 16 16 ö p 3p
= 2p - ç cot -1 + tan -1 ÷ = 2p - =
tan-1 x 1 + tan-1 x 2 + . . . + tan-1 xn è 63 63 ø 2 2
1 2
æ S 1 - S3 + S5 - ... ö Example 10. If sin-1 + sin-1 = sin-1 x, then the value of x is
= tan-1 ç ÷ 3 3
è 1 - S 2 + S 4 - S 6 + . . .ø 5+ 2 5+4 2 5+2 2
(a) (b) (c) (d) None of these
where, S 1 = x 1 + x 2 + . . . + x n = Sx 1 9 9 9
S 2 = x 1 x 2 + x 1 x 3 + . . . + x n - 1 x n = Sx 1 x 2 1 2
Sol. (b) sin -1 + sin -1 = sin -1 x
3 3
S 3 = Sx 1x 2x 3 . . . and so on.
æ 1 4 2 1ö
Þ sin -1 ç 1- + 1 - ÷ = sin -1 x
æ12 ö -1æ 3 ö è 3 9 3 9 ø
Example 8. The value of sin-1ç ÷ - sin ç ÷ is equal to
è 13 ø è5 ø æ1 5 2 8ö
(JEE Main 2019) Þ sin -1 ç × + × ÷ = sin -1 x
è3 3 3 3 ø
æ 63 ö p æ 56 ö
(a) p - sin -1ç ÷ (b) - sin -1ç ÷
è 65 ø 2 è 65 ø æ 5 + 4 2ö
Þ sin -1 ç ÷ = sin -1 x
p æ9ö æ 33 ö è 9 ø
(c) - cos-1ç ÷ (d) p - cos-1ç ÷
2 è 65 ø è 65 ø 5+4 2
Þ x=
æ12 ö æ3ö 9
Sol. (b) We have, sin - 1ç ÷ - sin - 1ç ÷
è13 ø è5ø
Example 11. If S is the sum of the first 10 terms of the series
æ12 2 2ö
æ 1ö æ 1ö æ 1ö æ 1ö
æ3ö 3 æ12 ö tan-1ç ÷ + tan-1ç ÷ + tan-1ç ÷ + tan-1ç ÷ + .... , then
= sin - 1ç 1- ç ÷ - 1- ç ÷ ÷
ç13
è
è5ø 5 è13 ø ÷
ø
è3ø è 7ø è13 ø è 21ø
tan(S) is equal to (JEE Main 2020)
[Q sin - 1 x - sin - 1 y = sin - 1( x 1 - y 2 - y 1 - x2),
6 5 10 5
if x2 + y 2 £ 1 or if xy > 0 and x2 + y 2 > 1"x, y Î [ - 1, 1]] (a) (b) (c) (d)
5 6 11 11
æ12 4 3 5 ö æ 48 - 15 ö
= sin - 1ç ´ - ´ ÷ = sin - 1ç ÷ Sol. (b) Given, series is
è13 5 5 13 ø è 65 ø
æ 1ö æ 1ö 1
S = tan -1ç ÷ + tan -1ç ÷ + tan -1
æ 33 ö æ 33 ö
2 è3ø è7ø 13
= sin - 1ç ÷ = cos- 1 1 - ç ÷
è 65 ø è 65 ø æ 1ö
+ tan -1ç ÷ + ¼+ up to 10 terms
è 21ø
3136
= cos- 1 [Q sin - 1 x = cos- 1 1 - x2] -1 1 -1 1 1
4225 = tan + tan + tan -1
1+ 2 1+ 6 1 + 12
æ 56 ö
= cos- 1ç ÷ 1
è 65 ø + tan -1 + ¼+ up to 10 terms
1 + 20
p æ 56 ö é pù
= - sin - 1ç ÷ êQ sin - 1 q + cot - 1 q = ú 2 -1 3 -2 4 -3
2 è 65 ø ë 2û = tan -1 + tan -1 + tan -1
1 + (1× 2) 1 + (2 × 3) 1 + (3 × 4)
æ 4 5 16 ö 5-4
Example 9. 2 p - ç sin-1 + sin-1 + sin-1 ÷ is equal to + tan -1 + ¼ + up to 10 terms
è 5 13 65 ø 1 + ( 4 × 5)
(JEE Main 2020) = [tan -1 2 - tan -1 1] + [tan -1 3 - tan -1 2]
p 5p 3p 7p + [tan -1 4 - tan -1 3] + [tan -1 5 - tan -1 4]
(a) (b) (c) (d)
2 4 2 4 + ¼¼ + upto 10 terms
æ -1 4 -1 5 -1 16 ö -1 -1 -1 æ 11 - 1 ö
Sol. (c) We have, 2p - ç sin + sin + sin ÷ = tan 11 - tan 1= tan ç ÷
è 5 13 65 ø è1 + (11 ´ 1) ø
æ 4 5 16 ö æ10 ö 5
= 2p - ç tan -1 + tan -1 + tan -1 ÷ Þ S = tan -1ç ÷ Þ tan( S) =
è 3 12 63 ø è 12 ø 6
614 JEE Main Mathematics

Inverse Trigonometric Ratios of æ 2x ö


Example 12. Let tan-1 y = tan-1 x + tan-1ç ÷, where
Multiple Angles è1 - x 2 ø
1
(i) 2 sin-1 x | x| < . Then, a value of y is
3 (JEE Main 2015)
ì -1 1 1
ï sin ( 2x 1 - x ),
2 if - £x£ 3x - x 3
3x + x3
2 2 (a) (b)
ïï 1 1 - 3x2 1 - 3x2
= í p - sin-1 ( 2x 1 - x 2 ), if £ x£1
2 3x - x3 3x + x3
ï (c) (d)
ï - p - sin-1( 2x 1 - x 2 ), if - 1 £ x £ - 1 1 + 3x2 1 + 3x2
ïî 2 æ 2x ö 1
ì sin-1 ( 3x - 4x3 ), 1 1 Sol. (a) Given, tan -1 y = tan -1 x + tan -1ç ÷ , where| x| <
if - £ x £ è1 - x2 ø 3
ï 2 2
ïï 1 \ tan -1 y = tan -1 x + 2 tan -1 x
(ii) 3 sin-1 x = í p - sin-1 ( 3x - 4x3 ), if < x £ 1 é ù
2 -1 -1 æ 2 x ö
ï êQ 2 tan x = tan ç ÷ ,| x| < 1ú
ï - p - sin ( 3x - 4x ), if - 1 £ x < - 1
2
- 1 3 êë è 1 - x ø úû
ïî 2
Þ tan -1 y = 3 tan -1 x
ì cos-1 ( 2x 2 - 1), if 0 £ x £ 1 æ 3x - x3 ö
(iii) 2 cos-1 x=í -1 2 Þ tan -1 y = tan -1 ç ÷
î 2p - cos ( 2x - 1), if - 1 £ x £ 0 è 1 - 3x2 ø
ì -1 3 1 é 3 ù
-1 æ 3 x - x ö
ï cos ( 4x - 3x ), if £ x £ 1 -1
2 êQ 3 tan x = tan ç ÷ú
ïï êë è 1 - 3x2 ø úû
1 1
(iv) 3 cos-1 -1 3
x = í 2p - cos ( 4x - 3x ), if - £ x £ 3x - x3
ï 2 2 Þ y=
ï 2p + cos-1 ( 4x3 - 3x ), if - 1 £ x £ - 1 1 - 3x2
ïî 2
æ 3a 2x - x3 ö
ì -1 æ 2x ö Example 13. The simplified form of tan-1 ç ÷,
ï tan ç ÷, if - 1 < x £ 1 è a3 - 3ax 2 ø
ï è 1 - x2 ø
-a a
ï æ 2x ö a > 0; £x£ is
(v) 2 tan-1 x = í p + tan-1 ç ÷, if x > 1 3 3
ï è 1 - x2 ø
ï æ xö æ xö
-1 æ 2x ö (a) 3 tan -1 ç ÷ (b) 2 tan -1 ç ÷
ï - p + tan ç ÷ , if x < - 1 è aø è aø
î è 1 - x2 ø
æ aö
(c) 3 tan -1 ç ÷ (d) None of these
ì -1 æ 3x - x ö
3
1 1 è xø
ï tan ç 2
÷, if - <x<
ï è 1 - 3x ø 3 3
Sol. (a) Let x = a tan q
ï
ï æ 3x - x ö 3
1
(vi) 3 tan-1 x = í p + tan-1 ç 2
÷, if x > Þ
x
= tan q
ï è 1 - 3x ø 3 a
ï æ 3 x - x 3ö
1 æ xö
ï - p + tan ç
-1
÷ , if x < - Þ q = tan -1 ç ÷
2 è aø
ïî è 1 - 3x ø 3
æ 3a2x - x3 ö -1 é 3 a ( a tan q ) - ( a tan q ) ù
2 3
ì -1 æ 2x ö \ tan -1 ç 3 ÷ = tan ê ú
ï sin ç ÷, if - 1 £ x £ 1 è a - 3ax2 ø ë a3 - 3a( a tan q ) 2 û
ï è 1 + x2 ø
ï æ 2x ö é a3(3 tan q - tan3 q ) ù
(vii) 2 tan-1 x = í p - sin-1 ç ÷, if x > 1 = tan -1 ê 3 2 ú
ï è 1 + x2 ø ë a (1 - 3 tan q ) û
ï -1 æ 2x ö æ 3 tan q - tan3 q ö
ï - p - sin ç ÷ , if x < - 1 = tan -1 ç ÷
î è 1 + x2 ø è 1 - 3 tan 2 q ø
ì -1 æ 1 - x 2 ö
ï cos ç ÷, if 0 £ x < ¥ = tan -1(tan 3q )
ï è 1 + x2 ø
(viii) 2 tan-1 x=í æ xö
= 3q = 3 tan -1 ç ÷
ï - cos-1 æç 1 - x ö÷ , if - ¥ < x < 0
2 [from Eq. (i)]
è aø
ï è 1 + x2 ø
î
Practice Exercise
ROUND I Topically Divided Problems
Domain and Range of Inverse p
8. If cos-1 x + cos-1 y = and tan -1 x - tan -1 y = 0,
2
Trigonometric Functions
then x 2 + xy + y 2 is equal to
1. If q = tan - 1 a , f = tan - 1 b and ab = - 1, then (q - f) 1
(a) 0 (b)
is equal to 2
p 3 1
(a) 0 (b) (c) (d)
4 2 8
p
(c) (d) None of these
2 Properties of Inverse
2. The principal value of sin -1
(sin 10) is Trigonometric Functions
p
(a) 10 (b) 10 - 3p 9. The solution of tan - 1 2 x + tan - 1 3x = is
4
(c) 3p - 10 (d) None of these
1
20 20 (a) (b) - 1
3. If å sin -1
xi = 10 p, then å xi is equal to 6
æ1 ö
i =1 i =1
(c) ç , - 1÷ (d) None of these
è6 ø
(a) 20 (b) 10
(c) 0 (d) None of these 10. The value of sin { 2 tan -1 (.75)} is equal to
ép æ 3 öù
4. The value of sin ê - sin - 1 ç - ÷ ú is
(a) 0.75 (b) 1.5
ë2 è 2 øû (c) 0.96 (d) sin 1 . 5
p
11. If a = cos-1 æç ö÷, b = tan -1 æç ö÷, where 0 < a , b < ,
3 3 1 1 3 1
(a) (b) - (c) (d) - è5 ø è3ø
2 2 2 2 2
2 ö pü then a - b is equal to
5. The value of tan ìí cos- 1 æç -
(JEE Main 2019)
÷- is
î è 7 ø 2 ýþ æ 9 ö æ 9 ö
(a) tan -1 ç ÷ (b) cos -1 ç ÷
2 2 1 4 è 5 10 ø è 5 10 ø
(a) (b) (c) (d)
3 5 3 5 5 æ9ö æ 9 ö
(c) tan -1 ç ÷ (d) sin -1 ç ÷
-1 2 è 14 ø è 5 10 ø
6. The value of x for which cos (cos 4) > 3x - 4 x is
y
æ 2 + 6p - 8 ö 12. If cos- 1 x - cos- 1 = a, where - 1 £ x £ 1, - 2 £ y £ 2,
(a) ç0, ÷ 2
è 3 ø y
æ 2 - 6p - 8 ö
x£ , then for all x, y, 4 x 2 - 4 xy cosa + y 2 is
(b) ç , 0÷ 2
è 3 ø equal to (JEE Main 2019)
(c) (–2, 2) (a) 2 sin 2 a (b) 4 cos 2 a + 2x2y2
æ 2 - 6p - 8 2 + 6p - 8 ö (c) 4 sin 2 a (d) 4 sin 2 a - 2x2y2
(d) ç , ÷
è 3 3 ø æ 2a ö -1 æ 1 - a ö
2
æ 2x ö
13. If sin -1 ç 2 ÷ + cos ç 2÷
= tan -1 ç 2÷
,
7. If q = sin -1 x + cos-1 x - tan -1 x ³ 0, then the è1 + a ø è1 + a ø è1 - x ø
smallest interval in which q lies, is given by where a, x Î ] 0, 1[, then the value of x is
p 3p p a
(a) £q£ (b) - £ q £ 0 (a) 0 (b)
2 4 4 2
p p p 2a
(c) 0 £ q £ (d) £q£ (c) a (d)
4 4 2 1-a2
616 JEE Main Mathematics

14. If we consider only the principal values of the 23. The value of
inverse trigonometric functions, then the value of æ xy + 1 ö - 1 æ yz + 1 ö - 1 æ zx + 1 ö
cot - 1 ç ÷ + cot ç ÷ + cot ç ÷ is
tan æç cos- 1
1 4 ö èx- yø è y-zø è z-x ø
- sin - 1 ÷ is
è 5 2 17 ø
(a) 0 (b) 1
29 29 3 3 (c) cot- 1 x + cot- 1 y + cot- 1 z (d) None of these
(a) (b) (c) (d)
3 3 29 29
24. If x 2 + y 2 + z2 = r 2 , then
15. If sin - 1 a + sin - 1 b + sin - 1 c = p, then the value of
- 1 æ xz ö
tan - 1 æç ö÷ + tan - 1 æç
xy yz ö
÷ + tan ç ÷ is equal to
a 1 - a 2 + b 1 - b2 + c 1 - c2 will be è zr ø è xr ø è yr ø
1 1 p
(a) 2 abc (b) abc (c) abc (d) abc (a) p (b)
2 3 2
1 (c) 0 (d) None of these
16. Given, 0 £ x £ , then the value of
2 25. If in a D ABC, Ð A = tan - 1 2 and Ð B = tan - 1 3,
é ìï x 1 - x 2 üï ù
tan êsin - 1 í + -1
ý - sin x ú is then ÐC is equal to
ê 2 2 ï ú p p
ë îï þ û (a) (b)
2 3
1
(a) 1 (b) 3 (c) - 1 (d) p
3 (c) (d) None of these
4
æ sin 2 - 1 ö
17. Value of tan -1 ç ÷ is 26. The value of the expression
è cos 2 ø
sin æç 2 tan -1

÷ + cos (tan -1 2 2 ) is
p p p p è 3ø
(a) -1 (b) 1 - (c) 2 - (d) -1
2 4 2 4 13 14
(a) (b)
-1 -1 -1 -1 15 15
18. If a sin x - b cos x = c, then a sin x + b cos x 11
(c) (d) None of these
is equal to 15
p ab + c (a - b)
-1 æ 6r ö
k
(a) (b) 0
a+b 27. Let Sk = å tan ç
è2
2r + 1
+32r + 1 ÷
ø
. Then, lim Sk
k®¥
p ab - c (a - b) p r =1
(c) (d)
a+b 2 is equal to (JEE Main 2021)
æ3ö p
p (a) tan -1 ç ÷
19. If tan - 1 æç ö÷ + tan - 1 æç ö÷ = , then x is equal to
a b (b)
è2ø 2
è ø x è xø 2
æ3ö
(a) ab (b) 2ab (c) cot-1 ç ÷ (d) tan -1 (3)
è2ø
(c) 2ab (d) ab
é1 æ 2a ö 1 2 ù
-1 æ 1 - a ö
æ x - 1ö -1 æ x + 1 ö p 28. tan ê sin -1 ç ÷ + ç 2 ÷ú
-1 cos is equal to
20. If tan ç ÷ + tan ç ÷ = , then x is equal 2
è1 + a ø 2 è1 + a øû
èx + 2ø èx + 2ø 4 ë2
to 2a 1 - a2
(a) (b)
1 1 5 1 1 + a2 1 + a2
(a) (b) - (c) ± (d) ± 2a
2 2 2 2 (c) (d) None of these
1 - a2
21. The value of the expression tan æç cos-1
1 2 ö
÷ is æ æ öö
è2 5ø 19 n
29. The value of cot ç å cot -1 çç1 + å 2 p ÷÷ ÷÷ is
(a) 2 + 5 (b) 5 -2 ç è ø ø(JEE Main 2019)
èn = 1 p =1
5+2
(c) (d) 5 + 2 23 21 19 22
2 (a) (b) (c) (d)
22 19 21 23
p pö
22. If x Î æç - , ÷, then the value of é 1 - sin x + 1 + sin x ù
è 2 2ø 30. The value of cot -1 ê ú is
æ tan x ö -1 æ 3 sin 2 x ö ë 1 - sin x - 1 + sin x û
tan - 1 ç ÷ + tan ç ÷ is
è 4 ø è 5 + 3 cos 2 x ø x
(a) p - x (b) p -
x 2
(a) (b) 2 x (c) 3x (d) x x
2 (c) p - (d) None of these
3
Inverse Trigonometric Functions 617

æ x2 x3 ö æ x4 x6 ö (a) a < b (b) a = b


31. If sin -1 ç x - + - K÷ + cos-1 ç x 2 - + - K÷
è 2 4 ø è 2 4 ø (c) a > b (d) None of these
p 39. If cos-1 x + cos-1 y + cos-1 z = p, then
= , for 0 <|x |< 2 , then x equals
2 (a) x2 + y2 = z 2 (b) x2 + y2 + z 2 = 0
(a) 1/2 (b) 1 (c) –1/2 (d) –1 2 2 2
(c) x + y + z = 1 - 2xyz (d) None of these
32. If a1, a2 , a3, K, an are in AP with common 3p
40. If cos-1 p + cos-1 1 - p + cos-1 1 - q = , then
difference 5 and if ai a j ¹ - 1 for i, j = 1, 2, K, n, 4
the value of q is
æ 5 ö -1 æ 5 ö
then tan -1 ç ÷ + tan ç ÷ (a) 1 (b)
1
(c)
1
(d)
1
è 1 + a1a2 ø è 1 + a2 a3 ø 2 3 2
æ 5 ö c1x - y c - c1 c - c2
+ L + tan -1 çç ÷ is equal to
÷ 41. tan -1 + tan -1 2 + tan -1 3
è 1 + an - 1 an ø c1 y + x 1 + c2 c1 1 + c3c2
æ ö æ 5a1 ö 1
(a) tan -1 çç
5
÷÷ (b) tan -1 ç ÷ + . . . + tan -1 is equal to
è 1 + a na n - 1 ø è 1 + a na1 ø cn
æ yö
æ 5n - 5 ö æ 5n - 5 ö (a) tan -1 ç ÷ (b) tan -1 ( yx)
(c) tan -1 ç ÷ (d) tan -1 çç ÷÷ è xø
è 1 + a na1 ø è 1 + a1a n + 1 ø æ xö
(c) tan -1 ç ÷ (d) tan -1 (x - y)
æ 3ö è yø
-1 æ 1 ö
33. If a = sin -1 ç ÷ + sin çè ÷ø
è 2 ø 3 n æ 2m ö
-1 æ 3ö -1 æ 1 ö
42. å tan -1 ç 4
è m + m 2
+ 2
÷ is equal to
ø
and b = cos ç ÷ + cos çè ÷ø, then m =1
è 2 ø 3 æ n2 + n ö æ n2 - n ö
(a) a > b (b) a = b (c) a < b (d) a + b = 2p (a) tan -1 ç 2 ÷ (b) tan -1 ç 2 ÷
è n + n + 2ø è n - n + 2ø
34. The number of positive integral solutions of the æ n2 + n + 2 ö
æ ö (c) tan -1 ç ÷ (d) None of these
-1 -1 ç y ÷ = sin -1 æç 3 ö÷ is è n2 + n ø
equation tan x + cos
ç 1 + y2 ÷ è 10 ø
è ø 43. If cos-1 a + cos-1 b + cos-1 g = 3p, then
(a) one (b) two a (b + g ) + b ( g + a ) + g (a + b) is equal to
(c) zero (d) None of these (a) 0 (b) 1 (c) 6 (d) 12
1 p
35. Solution of the equation cot -1 x + sin -1 = is 44. The number of real solutions of the equation
5 4
p
(a) x = 3 (b) x =
1 1 + cos 2 x = 2 cos-1 (cos x) in é , p ù is
5
êë 2 úû
(c) x = 0 (d) None of these (a) 0 (b) 1 (c) 2 (d) infinite
1- x 2 1 1
36. If 3 sin -1
2x
- 4 cos-1 45. 4 tan -1 - tan -1 is equal to
1+ x 2
1 + x2 5 239
p p p
2x p (a) p (b) (c) (d)
+2 tan -1 = , 2 3 4
1 - x2 3
ïì 1 x 2 ïü -1 x
then value of x is 46. cos-1 í x 2 + 1 - x 2 1 - ý = cos - cos-1 x
1 ïî 2 4 ïþ 2
(a) 3 (b)
3 holds for
(c) 1 (d) None of these (a) | x| £ 1 (b) x Î R
37. The sum of the infinite series (c) 0 £ x £ 1 (d) -1 £ x £ 0
cot -1 2 + cot -1 8 + cot -1 18 + cot -1 32 + . . . is ¥ æ ö
1
(a) p (b)
p
(c)
p
(d) None of these
47. The value of å tan -1 ç 2÷
è1 + r + r ø
is equal to
2 4 r=0

4 1 4 1 p 3p
38. If a = sin -1 + sin -1 and b = cos-1 + cos-1 , (a)
2
(b)
4
5 3 5 3
p
then (c) (d) None of these
4
ROUND II Mixed Bag
Only One Correct Option (a) (- ¥ , cot 5) È (cot 2, ¥ )
æ 3 1 ö 1 (b) (cot 5, cot 4)
1. If f ( x) = sin -1 ç x- 1 - x 2 ÷ , - £ x £ 1 , then (c) (cot 2, ¥ )
è 2 2 ø 2
(d) (- ¥ , cot 5) È (cot 4, cot 2)
f ( x) is equal to
æ1ö p 9. Considering only the principal values of inverse
(a) sin -1 ç ÷ - sin -1 (x) (b) sin -1 x - functions, the set
è2ø 6
p
(c) sin -1 x +
p
(d) None of these A = ìí x ³ 0 : tan -1 ( 2 x) + tan -1 (3x) = üý
6 î 4þ (JEE Main 2019)
(a) is an empty set
2. Sum of infinite terms of the series
(b) is a singleton
cot -1 æç12 + ö÷ + cot -1 æç 2 2 + ö÷ + cot -1 æç32 + ö÷ + . . .
3 3 3
è ø è ø è (c) contains more than two elements
4 4 4ø
(d) contains two elements
is
p 10. The number of solutions of the equation
(a) (b) tan -1 ( 2)
sin -1 é x 2 + ù + cos-1 é x 2 - ù = x 2 for x Î [ - 1, 1],
4 1 2
(c) tan -1 3 (d) None of these ëê 3 ûú êë 3 ûú
3. If x1, x2 , x3 and x4 are the roots of the equation and [ x ] denotes the greatest integer less than or
x 4 - x 3 sin 2 b + x 2 cos 2 b - x cos b - sin b = 0 , then equal to x, is (JEE Main 2021)

tan -1 x1 + tan -1 x2 + tan -1 x3 + tan -1 x4 is equal to (a) 2 (b) 0 (c) 4 (d) Infinite
p 11. If 3 + i = ( a + ib)( c + id) , then
(a) b (b) -b (c) p - b (d) - b
2 æ bö ædö
tan -1 ç ÷ + tan -1 ç ÷ has the value
4. If q and f are the roots of the equation èaø ècø
8 x 2 + 22 x + 5 = 0, then p p
(a) + 2 np , n Î I (b) np + , n ÎI
(a) -1
both sin q and sin f are equal -1 3 6
p p
(b) both sec -1 q and sec -1 f are real (c) np - , n Î I (d) 2np - , n Î I
3 3
(c) both tan -1 q and tan -1 f are real
(d) None of the above 12. cot -1 ( cos a ) - tan -1 ( cos a ) = x, then sin x is
5. If sin -1 x + sin -1 y + sin -1 z = p, then equal to
4 4 4 2 2 2 2 2 2 2 2 2 æ aö æ aö
x + y + z + 4 x y z = k( x y + y z + z x ) (a) tan 2 ç ÷ (b) cot2 ç ÷
è2ø è2ø
where, k is equal to
æ aö
(a) 1 (b) 2 (c) tan a (d) cot ç ÷
è2ø
(c) 4 (d) None of these
ì 23 æ n öü 13. The value of a for which ax 2 + sin -1 ( x 2 - 2 x + 2)
6. The value of cot í å cot -1 ç1 + å 2 k÷øý is
è + cos-1 ( x 2 - 2 x + 2) = 0 has a real solution, is
î n =1 k= 1 þ (JEE Advanced)
2 2 p p
23 25 23 24 (a) - (b) (c) - (d)
(a) (b) (c) (d) p p 2 2
25 23 24 23
3p
æ 1 + y2 ö 14. If sin -1 x + sin -1 y + sin -1 z = and f (1) = 2 ,
7. If sec-1 ( 1 + x 2 ) + cosec-1 ç ÷ + cot -1 æç 1 ö÷ = p, 2
ç y ÷ è zø f ( p + q) = f ( p) × f ( q), " p, q Î R, then
è ø
( x + y + z)
then x + y + z is equal to x f (1 ) + y f ( 2 ) + z f ( 3 ) - f (1 ) is equal to
(a) xyz (b) 2 xyz (c) xyz 2 (d) x2yz x + y f ( 2 ) + z f ( 3)

8. All x satisfying the inequality (a) 0 (b) 1 (c) 2 (d) 3


-1 2 -1
(cot x) - 7 (cot x) + 10 > 0, lie in the interval
(JEE Main 2019)
Inverse Trigonometric Functions 619

y
15. The sum of the infinite series 19. If cos- 1 x - cos- 1 = a, then 4 x 2 - 4 xy cosa + y 2 is
2
-1 æ 2 - 1 ö -1 æ 3 - 2 ö
sin -1 æç
1 ö
÷ + sin ç ÷ + sin ç ÷ +... equal to
è 2ø è 6 ø è 12 ø
(a) -4 sin 2 a (b) 4 sin 2 a
æ n - ( n - 1) ö (c) 4 (d) 2 sin 2 a
+ . . . + sin -1 çç ÷ + . . . is
÷
è { n( n + 1)} ø
p p p Numerical Value Based Questions
(a) (b) (c) (d) p
8 4 2 20. Number of solutions of the equation
æ x cos q ö -1 æ cos q ö æ 1 ö -1 æ 1 ö æ 2 ö is
16. The value of tan -1 ç ÷ - cot ç ÷ is tan -1 ç ÷ + tan ç ÷ = tan
-1
ç 2÷
è 1 - x sin q ø è x - sin q ø è 2 x + 1ø è 4x + 1ø èx ø
(a) 2 q (b) q ……… .
(c) q /2 (d) independent of q
21. The number of real solutions of
sin -1 x cos-1 x tan -1 y p
17. If = = ; 0 < x < 1, then the tan -1 x ( x + 1) + sin -1 x 2 + x + 1 = is ……… .
a b c 2
æ pc ö é 1 ì cos (tan -1 y) + y sin (tan -1 y) ü 2 ù
1/ 2
value of cos ç ÷ is 4
è a + bø 22. ê 2 í -1 -1 ý +y ú takes
(JEE Main 2021)
êë y î cot (sin y) + tan (sin y) þ úû
2 2
1- y 1- y
(a) (b) value ……… .
2y 1 + y2
1 - y2 23. If a, b, c are positive real numbers
(c) 1 - y2 (d)
y y a ( a + b + c) b ( a + b + c)
q = tan -1 + tan -1
18. If x, y and z are in AP and tan -1
x, tan -1
y and bc ca
-1 c ( a + b + c)
tan z are also in AP, then + tan -1 . Then, tan q equals …… .
(a) x = y = z (b) 2x = 3 y = 6z ab
(c) 6x = 3 y = 2z (d) 6x = 4 y = 3z
24. Let cos-1 x + cos-1 2 x + cos-1 3x = p. If x satisfies the
equation ax 3 + bx 2 + c = 0, then the value of
( a + b + c) is

Answers
Round I
1. (c) 2. (c) 3. (a) 4. (c) 5. (a) 6. (d) 7. (d) 8. (c) 9. (a) 10. (c)
11. (d) 12. (c) 13. (d) 14. (d) 15. (a) 16. (a) 17. (b) 18. (a) 19. (a) 20. (c)
21. (b) 22. (d) 23. (a) 24. (b) 25. (c) 26. (b) 27. (c) 28. (c) 29. (b) 30. (b)
31. (b) 32. (c) 33. (c) 34. (b) 35. (a) 36. (b) 37. (c) 38. (a) 39. (c) 40. (d)
41. (c) 42. (a) 43. (c) 44. (a) 45. (d) 46. (a) 47. (a) 48. (a)

Round II
1. (b) 2. (b) 3. (b) 4. (c) 5. (b) 6. (b) 7. (a) 8. (c) 9. (b) 10. (b)
11. (b) 12. (a) 13. (c) 14. (c) 15. (c) 16. (b) 17. (b) 18. (a) 19. (a) 20. (2)
21. (2) 22. (1) 23. (72) 24. (25)
Solutions
Round I 8. Q tan -1 x - tan -1 y = 0 Þ x = y
-1
1. Given that, q = tan a p
Also, cos -1 x + cos -1 y =
-1 2
and f = tan b
-1 p
and ab = -1 Þ 2 cos x =
2
\ tan q tan f = ab = - 1
-1 p
Þ cos x =
Þ tan q = - cot f 4
æp ö p
Þ x=
1
Þ tan q = tan ç + f ÷ Þ q - f =
è2 ø 2 2
1
2. sin -1 (sin 10) = sin -1 {sin(3p - 10)} = 3p - 10 Þ x2 =
2
p p
3. Since, - < sin -1 x £ Hence, x2 + xy + y2 = 3x2 =
3
2 2 2
sin -1 x1 + sin -1 x2 + K + sin -1 x20 = 10p p
9. tan -1 2x + tan -1 3x =
æpö
sin -1 xi contains maximum value i.e. ç ÷ . 4
è2ø 3x + 2x p
Þ = tan
-1 p 1 - 6 x2 4
\ sin x i = , 1 £ i £ 20
2
Þ 5x = 1 - 6x2
Þ x i = 1, 1 £ i £ 20
20
Þ 6 x2 + 5 x - 1 = 0
Thus, å xi = 20
Þ x = - 1,
1
i =1 6
ép æ 3 öù æ æ 3 öö But when x = - 1,
4. sin ê - sin -1 ç - ÷ ú = cos çsin -1 ç - ÷÷
êë 2 è 2 ø ûú è è 2 øø tan -1 2x = tan -1 (-2) < 0
æ and tan -1 3x = tan -1 (-3) < 0
3 ö
= cos ç cos -1 1 - ÷ This, value will not satisfy the given equation.
è 4 ø
1
æ æ1öö 1 Hence, x=
= cos ç cos -1 ç ÷ ÷ = 6
è è2øø 2
10. sin { 2 tan -1 (0.75)}
ì æ 2 ö pü ì æ 2 ö pü
5. tan í cos -1 ç - ÷ - ý = tan í p - cos -1 ç ÷- ý é æ 2(0.75) ö ù
î è 7 ø 2þ î è 7ø 2þ = sin ê tan -1 ç ÷ú
êë è 1 - (0.75)2 ø úû
ìp æ 2 öü
= tan í - cos -1 ç ÷ý ì
î2 è 7 øþ æ 1.5 öü
= sin í tan -1 ç ÷ý
ì æ 2 öü î è 1 - 0 . 5625 øþ
= tan ísin -1 ç ÷ý
î è 7 øþ é æ 1.5 ö ù é -1 æ 24 ö ù
= sin ê tan -1 ç ÷ = sin ê tan çè 7 ÷ø ú
ì æ 2 öü 2 ë è 0.4375 ø úû ë û
= tan í tan -1 ç ÷ý =
î è ø
3 5 þ 3 5 é æ öù
ç 24 ÷ú
= sin êsin -1
6. Now, cos -1
(cos 4) = cos -1
{cos (2p - 4)} = 2p - 4 ê ç (24)2 + (7)2 ÷ ú
ë è øû
Þ 2p - 4 > 3x2 - 4x é ù
æ xö x
Þ 3x2 - 4x - (2p - 4) < 0 êQ tan -1 ç ÷ = sin -1 ú
êë è yø x2 + y2 úû
2 - 6p - 8 2 + 6p - 8
Þ <x< æ 24 ö
3 3 = sin çsin -1 ÷
è 476 + 49 ø
7. We have, q = sin -1 x + cos -1 x - tan -1 x
æ 24 ö
p = sin çsin -1 ÷
- tan -1 x = cot-1 x
= è 25 ø
2
p p 24
Since, 0 £ x £ 1, therefore £ q £ . = = 0.96
4 2 25
Inverse Trigonometric Functions 621

æ3ö æ1ö y2
11. Given, a = cos -1 ç ÷ and b = tan -1 ç ÷ Þ x2 + - xy cos a = 1 - cos 2 a
è5ø è3ø 4
p Þ 4x2 - 4xy cos a + y2 = 4 sin 2 a
where 0 < a , b <
æ 2a ö -1 æ 1 - a ö æ 2x ö
2 2
13. Given, sin -1 ç 2
÷ + cos ç 2
÷ = tan -1 ç ÷
è1 + a ø è1 + a ø è 1 - x2 ø
5 \ 2 tan -1 a + 2 tan -1 a = 2 tan -1 x
52 – 32 = 4
Þ 4 tan -1 a = 2 tan -1 x
a Þ 2 tan -1 a = tan -1 x
3 2a
Þ tan -1 = tan -1 x
4 1 - a2
Clearly, a = tan -1
3 2a
Þ x=
-1 4 1 1 - a2
So, a - b = tan - tan -1
3 3
æ ö æ 1 4 ö
4 1 14. tan ç cos -1 - sin -1 ÷
ç - ÷ è 5 2 17 ø
= tan -1 ç 3 3 ÷
ç æ4 1ö ÷ = tan (tan -1 7 - tan -1 4 )
ç 1 + çè ´ ÷ø ÷
è 3 3 ø é æ 7 - 4 öù 3
= tan ê tan -1 ç ÷ú =
é -1 -1 -1 x - y ù ë è 1 + 28 ø û 29
êQ tan x - tan y = tan , if xy > - 1ú
ë 1 + xy û 15. Let sin - 1 a = A, sin - 1 b = B and sin - 1 c = C
-1 1 -1 9
= tan
4
= tan where, sin - 1 a + sin - 1 b + sin - 1 c = p
1+ 13
9 \ sin A = a , sin B = b, sin C = c and A + B + C = p , then
sin 2 A + sin 2B + sin 2 C = 4 sin A sin B sin C
250
2 =
Þ sin A cos A + sin B cos B + sin C cos C
3
2 1 = 2 sin A sin B sin C
9+ 9
a –b Þ sin A 1 - sin 2 A + sin B 1 - sin 2 B
13
+ sin C 1 - sin 2 C = 2 sin A sin B sin C
-1 9 -1 9
= sin = sin
92 + 132 250 Þ a 1 - a 2 + b 1 - b2 + c 1 - c2 = 2 abc

æ 9 ö 16. Given, for 0 £ x £ 1 /2,


= sin -1 ç ÷
è 5 10 ø é ù
ìï x 1 - x2 üï
-1 y -1 tan êsin - 1 í + ý - sin
-1

12. Given equation is cos x - cos = a, ê ïî 2 2 ï ú
2 ë þ û
y é ìï x + 1 - x2 üï ù
where - 1 £ x £ 1, - 2 £ y £ 2 and x £
2 = tan êsin - 1 í ý - sin
-1

ê ï 2 ï ú
æ y ö ë î þ û
\ cos - 1 ç x + 1 - x2 1 - ( y / 2)2 ÷ = a
è 2 ø é ìï sin q + 1 - sin 2 q üï ù
= tan êsin - 1 í ý - qú
[Q cos - 1 x - cos - 1 y = cos - 1 (xy + 1 - x2 1 - y2), ê 2 ú
ë îï þï û
| x|,| y| £ 1 and x + y ³ 0]
xy [put sin - 1 x = q Þ x = sin q]
Þ + 1 - x 1 - ( y / 2)2 = cos a
2
é ì 1 1 ü ù
2 = tan êsin - 1 í sin q + cos qý - qú
xy ë î 2 2 þ û
Þ 1 - x2 1 - ( y / 2)2 = cos a -
2 é ì æ p öü ù
= tan êsin - 1 ísin ç q + ÷ý - qú
On squaring both sides, we get ë î è ø
4 þ û
æ y2 ö x2 y2 xy é p ù
(1 - x2) ç1 - ÷ = cos 2 a + -2 cos a = tan ê q + - qú
è 4 ø 4 2 ë 4 û
y 2 x2 y2 x2 y2 p
Þ 1 - x2 - + = cos 2 a + - xy cos a = tan = 1
4 4 4 4
622 JEE Main Mathematics

sin 2 - 1 1 - sin 2 (cos 1 - sin 1)2 é 2x(x + 2) ù p


17. =- =- Þ ê 2 ú = tan
cos 2 cos 2 (cos 1 + sin 1)(cos 1 - sin 1) 2
ë x + 4 + 4x - x + 1 û 4
cos 1 - sin 1 1 - tan 1 2x (x + 2)
=- =- Þ =1
cos 1 + sin 1 1 + tan 1 4x + 5
æp ö æ pö Þ 2 x2 + 4 x = 4 x + 5
= - tan ç - 1÷ = tan ç1 - ÷
è4 ø è 4ø
5
æ sin 2 - 1 ö p öù p Þ x=±
-1 -1 é æ 2
Þ tan ç ÷ = tan ê tan ç1 - ÷ ú = 1 -
è cos 2 ø ë è 4 øû 4
é æ 2 öù
1 - cos ê cos -1 ç ÷
18. Given, a sin - 1 x - b cos - 1 x = c …(i) æ1 2 ö ë è 5 ø úû
21. tan ç cos -1 ÷=
æp ö è2 5ø é æ 2 öù
Þ a ç - cos - 1 x÷ - b cos - 1 x = c 1 + cos ê cos -1 ç ÷
è2 ø ë è 5 ø úû
ap
Þ (a + b) cos - 1 x = -c 2
1-
2 5 = 5 -2 5 -2
ap = ´
-c 2 5+2 5 -2
1+
Þ cos - 1 x= 2 5
a+b
( 5 - 2 )2
Again, from Eq. (i), = = 5 -2
5 -4
æp ö
a sin - 1 x - b ç - sin - 1 x÷ = c
è2 ø æ tan x ö -1 æ 3 sin 2 x ö
22. tan -1 ç ÷ + tan ç ÷
bp è 4 ø è 5 + 3 cos 2x ø
Þ (sin - 1 x) (a + b) = c +
2 æ 6 tan x ö
bp ç 2
÷
c+ æ tan x ö -1 ç 1 + tan x ÷
2 = tan -1 ç ÷ + tan ç
Þ sin - 1 x= è 4 ø 3 ( 1 - tan 2 x ) ÷
a+b ç5 + ÷
è 1 + tan 2 x ø
æ bp ö æa p ö
a çc + ÷ bç - c÷
è 2 ø è 2 ø æ tan x ö -1 æ 6 tan x ö
\a sin -1
x + b cos -1
x= + = tan -1 ç ÷ + tan ç ÷
a+b a+b è 4 ø è 8 + 2 tan 2 x ø
c (a - b) + a b p æ tan x ö -1 æ 3 tan x ö
= = tan -1 ç ÷ + tan ç ÷
a+b è 4 ø è 4 + tan 2 x ø
æaö æ bö p æ tan x 3 tan x ö
19. tan -1 ç ÷ + tan -1 ç ÷ = ç + ÷
è xø è xø 2
ç 4 4 + tan 2 x ÷ æ ½ tan x 3 tan x ½ ö
= tan -1 ç as½ × ½< 1÷
æ a bö ç 3 tan 2 x ÷è 4 tan 2 x½ ø
ç + ÷ p ç 1- ÷ ½ 4
Þ tan -1 ç x x÷ = è 4(4 + tan 2 x ) ø
ç 1 - ab ÷ 2
è x2 ø æ 16 tan x + tan3 x ö
= tan -1 ç ÷ = tan -1 (tan x ) = x
a b
+ è 16 + tan 2 x ø
Þ x x = tan p xy + 1 yz + 1 zx + 1
ab 2 23. cot-1 + cot-1 + cot-1
1- 2 x- y y-z z-x
x
ab = cot-1 y - cot-1 x + cot-1 z - cot-1 y + cot-1 x - cot-1 z
Þ 1 - 2 =0
x =0
Þ x2 = ab 24. Given that, x2 + y2 + z 2 = r 2
Þ x = ab æ xy ö æ yz ö æ xz ö
Now, tan -1 ç ÷ + tan -1 ç ÷ + tan -1 ç ÷
-1 x-1 x+1 p è zr ø è xr ø è yr ø
20. We have, tan + tan -1 =
x+2 x+2 4 é xy yz xz xyz ù
é ù ê + + - 3 ú
x-1 x+ 1 ê zr xr yr r ú
ê + ú = tan -1
Þ tan -1 ê x+2 x+2 ú=p ê æ x2 + y2 + z 2 ö ú
ê æ x - 1 ö æ x + 1ö ú 4 ê 1-ç ÷ ú
ê1 - ç ÷ç ÷ú êë è r2 ø úû
ë è x + 2ø è x + 2ø û
Inverse Trigonometric Functions 623

é xy yz xz xyz ù é æ2ö
r ù
ê zr + xr + yr - 3 ú ê ç ÷ ú
= tan -1 ê r ú k
-1 ê è 3 ø ú
ê r 2
ú
å tan
ê æ 2 ö 2r ú
1- 2 r =1
êë r úû ê çè ÷ø × 2 + 3 ú
ë 3 û
p
= tan -1 ¥ = é r ù
2 ê æ2ö ú
ç ÷
k
-1 ê è3ø ú
25. Given that, Ð A = tan -1 2 , Ð B = tan -1 3 Þ å tan ê æ 2r + 1 ö ú
We know that, Ð A + Ð B + ÐC = p
r =1 ê 3 ç æç ö÷
2
+ 1÷÷ ú
ê çè è 3 ø ø úû
Þ tan -1
2 + tan -1 3 + ÐC = p ë
r
æ2ö
æ 2+3 ö Let ç ÷ = t
Þ tan -1 ç ÷ + ÐC = p è3ø
è1 - 2 ´ 3ø
æ t ö
Þ tan -1 (-1 ) + ÐC = p k ç ÷
3p
Þ å tan -1
ç 3 ÷
Þ + ÐC = p r =1 ç 1 + 2 t2 ÷
4 è 3 ø
p æ 2t ö
Þ ÐC = k ç t- ÷
4 Þ å tan -1
ç 3 ÷
æ 1ö r =1 ç 1 + t × 2t ÷
26. Let E = sin ç2 tan -1 ÷ + cos (tan -1 2 2 ) è 3ø
è 3ø
k
é -1 æ 2 t ö ù
æ
ç 2´1 ÷
ö 2 Þ å ë
-1
ê tan (t ) - tan çè 3 ÷ø ú
û
r =1
1
Now, 2 tan -1 = tan -1 ç 3 ÷ = tan -1 3
é r + 1ù
3 ç æ1ö ÷
2 8 k
-1 æ 2 ö
r
-1 æ 2 ö
ç1 - ç ÷ ÷
è è3ø ø 9
Þ å ê tan ç ÷ - tan ç ÷
è3ø è3ø
ú
r =1 êë úû
k+1
é -1 -1 2 q ù æ2ö æ2ö
êQ 2 tan q = tan ú Þ S k = tan -1 ç ÷ - tan -1 ç ÷
ë 1 - q2 û è3ø è3ø
é k + 1ù
3 æ2ö æ2ö
3 Þ S ¥ = lim ê tan -1 ç ÷ - tan -1 ç ÷ ú
= tan -1 = sin -1 4 k®¥ê è3ø è3ø úû
ë
4 æ3ö
2
1+ ç ÷ æ2ö
è4ø = tan -1 ç ÷ - tan -1 (0)
è3ø
é æ x ö÷ ù æ2ö æ3ö
êQ tan -1 x = sin -1 ç ú \ S ¥ = tan -1 ç ÷ = cot-1 ç ÷
ç 1 + x2 ÷ ú è3ø è2ø
ê è ø
ë û
é1 æ 2a ö 1 æ 1 - a2 öù
æ3ö 28. tan ê sin -1 ç ÷ + cos -1 ç ÷ú
ç ÷
-1 4 3 êë 2 è1 + a2ø 2 è 1 + a 2 ø úû
= sin ç ÷ = sin -1
ç5÷ 5 é1 1 ù
= tan ê × 2 tan -1 a + × 2 tan -1 a ú
è4ø
ë2 2 û
and tan -1 (2 2 ) = cos -1
1 æ1ö
= cos -1 ç ÷ = tan (2 tan -1 a )
1 + (2 2 )2 è3ø é æ 2 a öù
= tan ê tan -1 ç ÷ú
é æ öù êë è 1 - a 2 ø úû
ç 1 ÷ú
êQ tan -1 x = cos -1 2a
ê ç 1 + x2 ÷ ú =
ë è øû 1 - a2
æ 1ö æ öö
\ E = sin ç2 tan -1 ÷ + cos(tan -1 2 2 ) 19 æ n
è 3ø 29. Consider, cot ç S cot-1 çç1 + pS= 1 2 p÷÷ ÷÷
çn =1
1 ö 3 1 9 + 5 14 è è øø
æ 3ö æ
= sin çsin -1 ÷ + cos ç cos -1 ÷ = + = =
è 5ø è 3ø 5 3 15 15 æ 19 ö
= cot ç S cot-1(1 + n (n + 1))÷
çn =1 ÷
k æ 6r ö è ø
27. S k = å tan -1 çè 22r + 1 + 32r + 1 ÷ø é n ù
S p = n (n + 1) ú
r =1
êQ
êë p =1 2 úû
Divide by 32r, we get
624 JEE Main Mathematics

æ 19 ö 2x 2 x2
= cot ç S cot-1 (1 + n + n 2)÷ Þ =
2 + x 2 + x2
çn =1 ÷
è ø
Þ 2x (2 + x2) = 2x2(2 + x)
æ 19 ö
= cot ç S tan -1
1 ÷ Þ 4x + 2x3 = 4x2 + 2x3
çn =1 1 + n (n + 1) ÷ø
è Þ x (4 + 2x - 4x - 2x2) = 0
2

1 Þ Either x = 0 or 4 - 4x = 0
[ Q cot-1 x = tan -1 , if x > 0 ]
x Þ x = 0 or x = 1
æ 19 æ (n + 1) - n ö ö 0 < | x|< 2
= cot ç S tan -1 ç ÷÷
Q
çn =1 è 1 + n (n + 1) ø ÷ø \ x = 1 and x ¹ 0
è
[put 1 = (n + 1) - n] 32. Since, a1 , a 2, a3 , K , a n are in AP with common
19 difference 5.
= cot S (tan -1 (n + 1) - tan -1 n )
n =1
Þ a 2 - a1 = a3 - a 2 = K = a n - a n - 1 = 5
5 a - a1
é -1 x - y -1 -1 ù Now, T1 = tan -1 = tan -1 2
êQ tan 1 + xy = tan x - tan yú 1 + a1a 2 1 + a 2a1
ë û
= tan -1 a 2 - tan -1 a1
= cot [(tan -1 2 - tan -1 1) + (tan -1 3 - tan -1 2) +
Similarly, T2 = tan -1 a3 - tan -1 a 2
......+ (tan -1 20 - tan - 1 19)]
-1 -1
T3 = tan -1 a 4 - tan -1 a3
= cot(tan 20 - tan 1)
M M M
éæ p ö æp öù
= cotê ç - cot-1 20÷ - ç - cot- 1 1÷ ú Tn - 1 = tan -1 a n - tan -1 a n - 1
ëè 2 ø è2 øû
On adding all, we get
[Q tan -1 x + cot-1 x = p / 2 ] a n - a1
-1 -1 Required sum = tan -1 a n - tan -1 a1 = tan -1
= cot(cot 1 - cot 20) 1 + a na1
cot(cot-1 1) cot (cot-1 20) + 1 a1 + 5(n - 1) - a1
= = tan -1
cot (cot-1 20) - cot (cot-1 1) 1 + a na1
é cot A cot B + 1 ù 5(n - 1)
êëQ cot ( A - B) = cot B - cot A úû = tan -1
1 + a na1
(1 ´ 20) + 1 æ 3ö æ 3ö
= [Q cot (cot-1 x) = x] æ1ö æ1ö
20 - 1 33. a + b = sin -1 ç ÷ + cos -1 ç ÷ + sin -1 ç ÷ + cos -1 ç ÷
è 2 ø è 2 ø è3ø è3ø
21
= p p
19 = + =p
2 2
éæ 1 1 ö æ 1 1 öù
ê çè cos 2 x - sin 2 x÷ + ç cos x + sin
ø è 2 2
x÷ ú
ø p æ1ö p æ1ö
Also, a = + sin -1 ç ÷ < + sin -1 ç ÷
30. cot-1 ê ú 3 è3ø 3 è2ø
ê æç cos 1 x - sin 1 ö æ 1
x÷ - ç cos x + sin
1 ö
x÷ ú
êë è 2 2 ø è 2 2 ø úû é pù
As sin q is increasing in ê0, ú .
ë 2û
æ 1 ö
= cot-1 ç - cot x÷ p p p
è 2 ø \ a< + =
3 6 2
æ 1 ö 1
= cot-1 cot ç p - x÷ = p - x é é p ùù
è 2 ø Similarly, b > p /2 êsince, cos q is decreasing in
2 êë0, 2 úû ú
ë û
p
31. We know that, sin -1 (a ) + cos -1 (a ) = p
2 Þ b > > a Þ a <b
2
Therefore, a should be equal in both functions.
æ ö
x2 x3 x4 x6 34. Given, tan -1 x + cos -1 ç
y ÷ = sin -1 æç 3 ö÷
\ x- + - K = x2 - + -K ç 1 + y2 ÷ è 10 ø
2 4 2 4 è ø
x x2 æ1ö æ 3 ö
Þ = Þ tan -1 x + tan -1 ç ÷ = sin -1 ç ÷
1+
x x2 è yø è 10 ø
1+
2 2 æ 1ö
x x2 çx+ ÷
Þ = -1 y÷
2 + x 2 + x2 Þ tan ç = tan -1 3
ç1 - x÷
ç ÷
2 2 è yø
Inverse Trigonometric Functions 625

1 æ xö = tan -1
1
Þ x+ = 3 ç1 - ÷
y è yø 2 n2
1 3x æ 2 ö é (2 n + 1 ) - (2 n - 1 ) ù
Þ x+ =3 - = tan -1 ç 2 ÷ = tan -1 ê +( ú
y y è 4n ø ë 1 2 n + 1 )(2 n - 1 ) û
Þ
1 3x
+ =3 - x = tan -1 (2 n + 1 ) - tan -1 (2 n - 1 )
y y ¥

1
\ Sn = å {tan -1 (2 n + 1 ) - tan -1 (2 n - 1 )}
Þ (1 + 3x) = 3 - x n =1
y
= tan -1 ¥ - tan -1 1
1 + 3x
\ y= p p p
3-x = - =
2 4 4
For positive values of x and y, we have æ4ö æ1ö
x = 1, y = 2 and x = 2, y = 7 38. Since, a = sin -1 ç ÷ + sin -1 ç ÷
è5ø è3ø
Hence, the number of solutions of given equation is 2. æ4 1 1 16 ö
1 p = sin -1 ç 1- + 1- ÷
35. Q cot -1
x + sin -1
= è5 9 3 25 ø
5 4
æ8 2 3ö æ8 2 + 3ö
-1 1 1 Þ a = sin -1 ç + ÷ = sin -1 ç ÷
Þ tan + tan -1 = tan -1 1 è 15 15 ø è 15 ø
x 2
1 1 8 2+3
Þ tan -1 = tan -1 1 - tan -1 Since, <1
x 2 15
p
æ 1ö \ a<
1 ç1 - ÷ 2
Þ tan -1 = tan -1 ç 2÷
ç1 + 1 ÷ æ 4 ö æ1ö
x Now, b = cos -1 ç ÷ + cos -1 ç ÷
è 2ø è5ø è3ø
1 1 p æ4ö p æ1ö
Þ tan -1 = tan -1 Þ b= - sin -1 ç ÷ + - sin -1 ç ÷
x 3 2 è5ø 2 è3ø
Þ x=3 æ 4 1ö
= p - çsin -1 + sin -1 ÷ = p - a
è 5 3ø
2x 1 - x2 2x p
36. 3 sin -1 2
- 4 cos -1 + 2 tan -1 = æ pö
1+ x 1 + x2 1 - x2 3 Þ b>a çQ a < ÷
è 2ø
On putting x = tan q, we get
39. cos -1 x + cos -1 y + cos -1 z = p
æ 2 tan q ö æ 1 - tan 2 q ö
3 sin -1 ç ÷ - 4 cos -1 ç ÷
2
è 1 + tan q ø è 1 + tan 2 q ø Þ cos -1 (xy - 1 - y2 1 - x2 ) = p - cos -1 z

æ 2 tan q ö p Þ xy - 1 - x2 1 - y2 = cos (p - cos -1 z )


+ 2 tan -1 ç ÷=
è 1 - tan 2 q ø 3 Þ xy - 1 - x2 1 - y2 = - z
p
Þ 3 sin -1 (sin 2q ) - 4 cos -1 (cos 2 q ) + 2 tan -1 (tan 2q) = Þ xy + z = 1 - x2 1 - y2
3
p On squaring both sides, we get
Þ 3 (2 q ) - 4 (2 q ) + 2 (2 q ) =
3 x2y2 + z 2 + 2xyz = 1 - x2 - y2 + x2y2
p Þ x2 + y2 + z 2 = 1 - 2xyz
Þ 6q-8q+ 4q=
3
p p 40. Let a = cos -1 p , b = cos -1 1 - p
-1
Þ q= Þ tan x =
6 6 and g = cos -1 1 - q
p 1 Þ cos a = p, cos b = 1 - p
Þ x = tan Þ x=
6 3
and cos g = 1 - q
37. Let S ¥ = cot-1 2 + cot-1 8 + cot-1 18 + cot-1 32 + . . . Therefore,
n
sin a = 1 - p, sin b = p and sin g = q
Sn = å cot-1 2k2 The given equation may be written as,
k =1
3p
\ Tn = cot-1 2 n 2 a+b+g=
4
626 JEE Main Mathematics

3p \ a (b + g ) + b (g + a ) + g (a + b )
Þ a+b= -g
4 = - 1 (- 1 - 1 ) - 1 (- 1 - 1 ) - 1 (- 1 - 1 )
æ 3p ö =2 + 2 + 2 =6
Þ cos (a + b ) = cos ç - g÷
è 4 ø
44. Given, 1 + cos 2x = 2 cos -1 (cos x)
Þ cos a cos b - sin a sin b
\ 2 cos 2 x = 2 x Þ 2 cos x = 2x
ì æp öü æp ö
= cos í p - ç + g ÷ý = - cos ç + g ÷
î è4 øþ è4 ø ép ù
For x Î ê , p ú , cos x = - cos x
æ 1 1 ö ë2 û
Þ p 1- p- 1- p p=-ç 1-q- q÷
è 2 2 ø - 2 cos x = 2 x
Þ - cos x = x Þ cos x = - x
Þ 0 = 1 - q - q Þ1 - q = q
Hence, no solution exist
1
Þ q= 1 é 1ù
2 45. 4 tan -1 = 2 ê2 tan -1 ú
5 ë 5û
æ c1x - y ö -1 æ c - c ö -1 æ c - c2 ö
41. tan -1 ç ÷ + tan ç 2 1 ÷ + tan ç 3 ÷ 2
è c1 y + x ø è 1 + c2c1 ø è 1 + c3 c2 ø -1 5 5
= 2 tan = 2 tan -1
1 1 12
+ . . . + tan -1 1-
cn 25
10
æ x 1 ö æ 1 1 ö æ 1 1 ö
ç - ÷ ç - ÷ ç - ÷ = tan -1 12 = tan -1 120
y c c c c c
= tan -1 ç 1 ÷
+ tan -1 ç 1 2 ÷
+ tan -1 ç 2 3 ÷ 1-
25 119
ç1 + x × 1 ÷ ç1 + 1 ÷ ç1 + 1 ÷ 144
ç ÷ ç ÷ ç ÷
è y c1 ø è c1c2 ø è c2c3 ø 1 1 120 1
So, 4 tan -1 - tan -1 = tan -1 - tan -1
1 5 239 119 239
+ . . . + tan -1
cn 120 1
-
x 1 1 1 1 = tan -1 119 239
= tan -1 - tan -1 + tan -1 - tan -1 + tan -1 120 1
y c1 c1 c2 c2 1+ ´
119 239
1 1 1 1
- tan -1 + . . . + tan -1 - tan -1 + tan -1 (120 ´ 239) - 119
c3 cn - 1 cn cn = tan -1
(119 ´ 239) + 120
æ xö
= tan -1 ç ÷ 28561 p
è yø = tan -1 = tan -1 1 =
28561 4
n æ 2m ö
42. We have, å tan -1 ç 4 2
÷
èm + m + 2ø
æ x2 x2 ö p
46. Since, 0 £ cos -1 çç + 1 - x2 1 - ÷÷ £
m=1
è2 4ø 2
n æ 2m ö
= å tan -1 ç ÷
è 1 + (m2 + m + 1 )(m2 - m + 1 ) ø
Because cos -1 x is in first quadrant when x is positive
m=1 x
and cos -1 - cos -1 x ³ 0
n æ (m2 + m + 1 ) - (m2 - m + 1 ) ö 2
= å tan -1 ç
è 1 + (m2 + m + 1)(m2 - m + 1 )ø
÷ x
m=1 So, cos -1 ³ cos -1 x
n
2
= å [tan -1 (m2 + m + 1 ) - tan -1 (m2 - m + 1 )] Also, ½
½x½ ½£ 1,|x| £ 1 Þ |x| £ 1
m=1 ½2½
= (tan -1 3 - tan -1 1 ) + (tan -1 7 - tan -1 3) é 1 ù æ r+1-r ö
47. Q tan -1 ê 2ú
= tan -1 ç ÷
+ (tan -1 13 - tan -1 7 ) + . . . + [tan -1 (n 2 + n + 1 ) ë 1 + r + r û è 1 + r (r + 1) ø
- tan -1 (n 2 - n + 1 )] = tan -1 (r + 1 ) - tan -1 (r )
2
n + n + 1 -1 æ n +n ö 2 n
= tan -1 2
= tan -1 ç ÷ \ å [tan -1 (r + 1 ) - tan -1 (r )]
1 + (n + n + 1) × 1 è 2 + n2 + n ø r=0

43. Given, cos -1 a + cos -1 b + cos -1 g = 3p = tan -1 (n + 1) - tan -1 (0)


Þ cos -1
a = cos -1
b = cos -1
g=p = tan -1 (n + 1)
¥ æ ö
Þ a = cos p, b = cos p , g = cos p 1 p
Þ å tan -1 ç 2
è1 + r + r ø
÷ = tan -1 (¥ ) =
2
Þ a = - 1, b = - 1, g = - 1 r=0
Inverse Trigonometric Functions 627

æ 1ö æ 5ö
Round II \ sin -1 ç - ÷ exists but sin -1 ç - ÷ does not exist.
è 4ø è 2ø
1
1. Let x = sin q, where - £ x£1
2 æ 5ö æ 1ö
sec-1 ç - ÷ exists but sec-1 ç - ÷ does not exist.
p p è 2ø è 4ø
Þ - £q£
6 2 æ 1ö æ 5ö
tan -1 ç - ÷ and tan -1 ç - ÷ both exist.
æ 3 1 ö è 4ø è 2ø
Then, f (x) = sin -1 ç x- 1 - x2 ÷
è 2 2 ø
5. We have, sin -1 x + sin -1 y = p - sin -1 z
æ 3 1 ö
= sin -1 ç sin q - cos q÷ Þ x 1 - y2 + y 1 - x2 = z
è 2 2 ø
Þ x2(1 - y2) = z 2 + y2(1 - x2) - 2 yz (1 - x2)
-1 ì æ p öü
= sin ísin çè q - ÷øý
î 6 þ Þ (x2 - z 2 - y2)2 = 4 y2z 2(1 - x2)
p p é p é -p p ù ù Þ x + y4 + z 4 - 2x2z 2 + 2 y2 z 2 - 2x2y2
4
=q- = sin -1 x - êQ q - Îê ,
6 6 ë 6 ë 3 3 úû úû + 4x2 y2z 2 - 4 y2z 2 = 0
æ 3ö æ 4 ö Þ x4 + y4 + z 4 + 4x2 y2z 2 = 2 (x2y2 + y2z 2 + z 2x2)
2. Here, Tn = cot-1 ç n 2 + ÷ = tan -1 ç ÷
è 4ø 2
è 4n + 3 ø \ k=2
é ù ì 23 æ n öüï
ê ú ï
-1 1 6. cot í å cot-1 çç1 + å 2k÷÷ý
= tan ê ú è øïþ
ê 1 + æç n + 1 ö÷ æç n - 1 ö÷ ú ïî n =1 k =1

êë è 2ø è 2 ø úû ì 23 ü
é æ 1ö æ 1ö ù = cot í å cot-1 (1 + 2 + 4 + 6 + 8 + L + 2n )ý
ê çè n + 2 ÷ø - çè n - 2 ÷ø ú î n=1 þ
= tan -1 ê ú é 23 ù
ê 1 + æç n + 1 ö÷ æç n - 1 ö÷ ú = cot ê å cot- 1 {1 + n (n + 1)}ú
ëê è 2ø è 2 ø úû êë n =1 úû
æ 1ö æ 1ö
= tan -1 ç n + ÷ - tan -1 ç n - ÷ é 23 1 ù
è 2ø è 2ø = cot ê å tan -1 ú
êë n =1 1 + n (n + 1) úû
æ1ö p æ1ö
\ S ¥ = T¥-1 - tan -1 ç ÷ = - tan -1 ç ÷
è2ø 2 è2ø é 23 ì (n + 1) - n ü ù
= cot ê å tan -1 í ýú
æ1ö
Þ S ¥ = cot-1 -1
ç ÷ Þ S ¥ = tan (2) êë n =1 î 1 + n (n + 1)þ úû
è2ø
é 23 ù
3. We have, Sx 1 = sin 2 b, Sx1 x2 = cos 2 b, Sx 1x 2x 3 = cos b = cot ê å {tan - 1 (n + 1) - tan - 1 n }ú
êë n =1 úû
and x 1x 2 x 3 x 4 = - sin b
\ tan -1 x1 + tan -1 x2 + tan -1 x3 + tan -1 x4 é æ 23 ö ù
= cot [tan - 1 24 - tan - 1 1] = cot ê tan - 1 ç ÷ ú
ë è 25 ø û
æ Sx1 - Sx1 x2 x3 ö
= tan -1 çç ÷÷ é æ 25 ö ù 25
è 1 - Sx1x2 + x1 x2 x3 x4 ø = cot ê cot- 1 ç ÷ ú =
ë è 23 ø û 23
æ sin 2 b - cos b ö æ 1 + y2 ö
= tan -1 ç ÷
è 1 - cos 2 b - sin b ø 7. Given, sec-1 æè 1 + x2 öø + cosec-1 çç ÷ + cot-1 æç 1 ö÷ = p
y ÷ è zø
è ø
é (2 sin b - 1 ) cos b ù
= tan -1 ê ú \ tan -1 x + tan -1 y + tan -1 z = p
ë sin b (2 sin b - 1 ) û
æ x + y + z - xyz ö
= tan -1 (cot b ) Þ tan -1 ç ÷ = p Þ x + y + z = xyz
è 1 - xy - yz - zx ø
ì æp öü
= tan -1 í tan ç - b ÷ý
î è 2 øþ 8. Given, (cot-1 x)2 - 7(cot-1 x) + 10 > 0
p Þ (cot-1 x - 2)(cot-1 x - 5) > 0 (by factorisation)
= -b
2 Þ cot-1 x < 2 or cot-1 x > 5
1 5
4. 8x2 + 22x + 5 = 0 Þ x = - , - cot-1 x Î (-¥ , 2) È (5, ¥ )
4 2
cot-1 x Î (0, 2) [Q range of cot-1 x is (0, p )]
1 5
Q -1 < - < 1 and - < - 1 \ x Î (cot 2, ¥ )
4 2
628 JEE Main Mathematics

9. Given equation is 2 5
Case II If £ x2 <
p 3 3
tan - 1 (2x) + tan -1 (3x) = , x³0
4 sin -1 (1) + cos -1 (0) = x2
5x p p p
Þ tan -1 = , 6x2 < 1 Þ + = x2
1 - 6 x2 4 2 2
Þ x2 = p
-1 -1 -1 æ x+ yö
[Q tan x + tan y = tan ç ÷ , xy < 1] é2 5ö
è 1 - xy ø but p Ï ê , ÷
ë3 3ø
5x 1
Þ 2
= 1 , x2 < Þ No value of ‘x’
1 - 6x 6
So, number of solutions of the equation is zero.
1
Þ 6x2 + 5x - 1 = 0, 0 £ x < [Q x ³ 0] 11. Given, ( 3 + i ) = (a + ib)(c + id ) = (ac - bd ) + i (ad + bc)
6
1 On comparing the real and imaginary parts on both
Þ 6x2 + 6x - x - 1 = 0, 0 £ x < sides, we get
6
1 ac - bd = 3 and ad + bc = 1
Þ 6x (x + 1) - 1 (x + 1) = 0, 0 £ x < æ bö æ dö æ bc + ad ö
6 Now, tan -1 ç ÷ + tan -1 ç ÷ = tan -1 ç ÷
èaø è cø è ac - bd ø
1
Þ (6x - 1)(x + 1) = 0, 0 £ x < æ 1 ö
6 = tan -1 ç ÷
1 1 è 3ø
Þ x = , - 1, 0 £ x < p
6 6 = np + , n Î I
1 1 6
Þ x= , [Q 0 £ x < ]
6 6 æ 1 ö
12. Given, tan -1 ç ÷ - tan -1 ( cos a ) = x
So ‘A’ is a singleton set. è cos a ø

10. Given equation æ 1 ö


ç - cos a ÷
ç cos a ÷=x
é 1ù
sin -1 ê x2 + ú + cos -1
é 2 2ù 2 Þ tan -1
ë 3û êë x - 3 úû = x ç
1 +
1
× cos a
÷
ç cos a ÷
è ø
é 1ù
Now, sin -1 ê x2 + ú is defined if 1 - cos a
ë 3û Þ = tan x
2 cos a
1 -4 5
- 1 £ x2 + < 2 Þ £ x2 <
3 3 3 2 cos a
Þ = cot x
2 5 1 - cos a
Þ 0£x <
3 1 + cos a
Þ cosec x =
é 2 ù 1 - cos a
and cos -1 ê x2 - ú is defined if
ë 3û 1 - cos a
Þ sin x =
2 -1 8 1 + cos a
- 1 £ x2 - < 2 Þ £ x2 <
3 3 3 æ aö
2 sin 2 ç ÷
8 è2ø æ aö
Þ 0 £ x2 < Þ sin x = = tan 2 ç ÷
3 2 æ aö è2ø
2 cos ç ÷
So, from Eqs. (i) and (ii), we can conclude è2ø
5
0 £ x2 < 13. Here, x2 - 2 x + 2 = (x - 1 )2 + 1 ³ 1
3
2 But -1 £ (x2 - 2 x + 2) £ 1
2
Case I If 0 £ x <
3 which is possible only when
sin -1 (0) + cos -1 (-1) = x2 x2 - 2 x + 2 = 1
Þ 0 + p = x2 Þ x=1
-1
Þ 2
x =p Then, 2
a(1 ) + sin (1 ) + cos -1 (1 ) = 0
æ 2ù p
but p Ï ç0, ú Þ a+ +0=0
è 3û 2
p
Þ No value of ‘x’ Þ a=-
2
Inverse Trigonometric Functions 629

p p p p sin -1 x cos -1 x tan -1 y


14. Q - £ sin -1 x £ , - £ sin -1 y £ 17. Let = = =r
2 2 2 2 a b c
p -1 p
and - £ sin z £ sin -1 x cos -1 x tan -1 y
2 2 = a, = b, =c
3p r r r
Given that, sin -1 x + sin -1 y + sin -1 z = p
2 So, a + b =
2r
which is possible only when
p æ ö
sin -1 x = sin -1 y = sin -1 z = æ pc ö ç p × tan -1 y ÷
2 cos ç ÷ = cos ç ÷
è a + bø ç p
Þ x= y= z =1 ×r ÷
è 2r ø
Put p= q =1
= cos (2 tan -1 y),
Then, f (2) = f (1 ) f (1 ) = 2 × 2 = 4
and put p = 1, q = 2 Let tan -1 y = q
Then, f (3) = f (1 ) f (2) = 2 × 22 = 8 = cos (2q)
x+ y+ z 1 - tan 2 q
\ xf (1) + yf ( 2) + z f (3 ) - f (1) =
x + yf ( 2) + z f (3 ) 1 + tan 2 q

=1 + 1 + 1 -
3
=3 -1 =2 1 - y2
=
1+1+1 1 + y2
é r - (r - 1 ) ù 18. Since, x, y and z are in AP.
15. Q Tr = sin -1 ê ú
ë r (r + 1 ) û
\ 2y = x + z
é r - (r - 1 ) ù Also, tan -1 x, tan -1 y and tan -1 z are in AP.
= tan -1 ê ú
ë 1 + r (r - 1 ) û \ 2 tan -1 y = tan -1 x + tan -1 z
n æ r - r -1 ö æ 2y ö æ x+ zö
S n = å tan -1 ç ÷ Þ tan -1 ç ÷ = tan -1 ç ÷
r =1 è1 + r r - 1 ø è 1 - y2 ø è 1 - xz ø
n
x+ z x+ z
= å {tan -1 r - tan -1 (r - 1 )} Þ =
r =1 1- y 2
1 - xz
= tan -1 n - tan -1 0 Þ y2 = xz
= tan -1 n -0
p Since, x, y and z are in AP as well as in GP.
\ S ¥ = tan -1 ¥= \ x= y=z
2
y
æ x cos q ö -1 æ cos q ö 19. Given, cos -1 x - cos -1 = a
16. tan -1 ç ÷ - cot ç ÷ 2
è 1 - x sin q ø è x - sin q ø
æ xy y2 ö÷
æ x cos q ö -1 æ x - sin q ö Þ cos -1 ç + 1 - x2 1 - =a
= tan -1 ç ÷ - tan ç ÷ ç2
è 1 - x sin q ø è cos q ø è 4 ÷ø
é x cos q x - sin q ù xy y2
ê - ú Þ + 1 - x2 1 - = cos a
ê 1 - x sin q cos q ú 2 4
= tan -1
ê æ x cos q ö æ x - sin q ö ú
ê1 + ç ÷ç ÷ú y2
ë è 1 - x sin q ø è cos q ø û Þ 2 1 - x2 1 - = 2 cos a - xy
4
æ x cos 2 q - x + sin q + x2 sin q - x sin 2 q ö
= tan -1 ç ÷ On squaring both sides, we get
è cos q - x cos q sin q + x2 cos q - x cos q sin q ø
æ 4 - y2 ö
æ - x sin q + sin q + x sin q - x sin q ö
2 2 2 4(1 - x2) ç ÷ = 4 cos 2 a + x2 y2 - 4xy cos a
= tan -1 ç ÷ è 4 ø
è cos q - 2x cos q sin q + x2 cos q ø
Þ (1 - x2)(4 - y2) = 4 cos 2 a + x2y2 - 4xy cos a
æ - 2x sin 2 q + sin q + x2 sin q ö
= tan -1 ç ÷ Þ 4 - y2 - 4x2 + x2y2 = 4 cos 2 a + x2y2 - 4xy cos a
è cos q - 2x cos q sin q + x2 cos q ø
ì sin q (- 2x sin q + 1 + x2)ü Þ 4x2 + y2 - 4xy cos a = 4 - 4 cos 2 a
= tan -1 í 2 ý
î cos q (1 - 2x sin q + x ) þ Þ 4x2 - 4xy cos a + y2 = 4 sin 2 a
= tan -1 (tan q ) = q
630 JEE Main Mathematics

æ 1 ö -1 æ 1 ö æ2ö é ì 2 ù
1/ 2
20. Given, tan -1 ç ÷ + tan ç ÷ = tan
-1
ç 2÷ 1 y2 ü
è2 x + 1ø è 4x + 1 ø èx ø ê ï + ï ú
ê 1 ï 1 + y2 1 + y2 ï ú
æ 1 1 ö ê
= 2í ý + y ú
4
ç + ÷ ê y ï 1 - y2 ú
+ + y ï
Þ -1 ç 2 x 1 4 x 1 ÷ = tan -1 æç 2 ö÷ ê ï + ú
tan
è x2 ø y 2ï
ç1 - 1 ´
1 ÷ ê î 1- y þ ú
ç ÷ ë û
è 2 x + 1 4x + 1 ø
1/ 2
é1 ù
æ 6x+ 2 ö æ2ö = ê 2 × y2(1 - y4 ) + y4 ú =1
Þ tan -1 ç 2 ÷ = tan -1 ç 2÷ ëy û
è 8 x + 6x ø èx ø
6x+ 2 2 a (a + b + c) b (a + b + c)
Þ = 2 23. Given, q = tan -1 + tan -1
2
8 x + 6x x bc ac
c (a + b + c)
Þ 6 x3 + 2 x2 = 16 x2 + 12 x + tan -1
ab
Þ 6 x3 - 14 x2 - 12 x = 0
é -1 -1 -1 -1 æ x + y + z - xyz ö ù
Þ 2 x(3x2 - 7x - 6) = 0 êQ tan x + tan y + tan z = tan çè 1 - xy - yz - zx ÷ø ú
ë û
Þ 2 x(3x + 2)(x - 3) = 0
2
\ x = 0, - ,3 é ù
3 ê ú
2 ê a + b + c æç a + b + c ö÷ ú
But x = - does not satisfy the given relation. ê ú
3 è bc ca ab ø
ê ú
21. Given function is ê a + b+ c ú
ê - (a + b + c) ú
p abc ú
tan -1 x(x + 1) + sin -1 x2 + x + 1 = = tan -1 ê
2 ê æ 1 1 1ö ú
Function is defined, if ê 1 - (a + b + c) çè a + b + c ÷ø ú
ê ú
(i) x (x + 1) ³ 0, since domain of square root function. ê ú
êë úû
(ii) x2 + x + 1 ³ 0, since domain of square root
function.
2 -1 é a + b+ c a + b+ cù
(iii) x + x + 1 £ 1, since domain of sin function. ê (a + b + c) - (a + b + c) ú
= tan -1 ê abc abc ú
From (ii) and (iii), 0 £ x2 + x + 1 £ 1 Ç x2 + x ³ 0 ê (a + b + c) (ab + bc + ca ) ú
1-
Þ 0 £ x2 + x + 1 £ 1 Ç x 2 + x + 1 ³ 1 êë abc úû
Þ x2 + x + 1 = 1 Þ q = tan -1 0
2
Þ x + x=0 Þ tan q = 0
Þ x (x + 1) = 0
24. cos -1 (x) + cos -1 (2x) + cos -1 (3x) = p
Þ x = 0, x = - 1
or cos -1 (2x) + cos -1 (3x) = p - cos -1 (x) = cos -1 (- x)
22. Here, innermost function is inverse.
or cos -1 [(2x)(3x) - 1 - 4x2 1 - 9x2 ] = cos -1 (- x)
Ö1 + y2 or 6 x2 - 1 - 4 x 2 1 - 9 x 2 = - x
y
or (6x2 + x)2 = (1 - 4x2)(1 - 9x2)
2
1 q or x2 + 12x3 = 1 - 13x2
or12x3 + 14x2 - 1 = 0
\ Put tan -1 y = q Þ tan q = y
Þ a = 12 ; b = 14 ; c = 0
1/ 2
é 1 ì cos (tan -1 y) + y sin(tan -1 y)ü 2 ù Þ a + b + c = 12 + 14 - 1 = 25
ê 2 ×í ý + y4ú
ê y î cot (sin -1 y) + tan(sin -1 y) þ ú
ë û
26
Continuity and
Differentiability
Continuity IN THIS CHAPTER ....
The geometrical significance of continuity is that if the function is continuous, Continuity
its graph does not bear a break otherwise it is discontinuous. The point where
Discontinuity
the graph of the function breaks is called the point of discontinuity.
Different Kinds of Discontinuity
Thus, a function is continuous at a point, if limit exist at this point and equal
to the value of the function at this point. Properties of Continuous
x Functions
e.g. Graphs of functions sin x , x and e etc. are continuous while tan x and sec x
Continuity of Composition of
etc. are discontinuous. Function
Y Y
Applications of Continuity
Differentiability
Relation Between Derivability and
–p/2 p 2p X¢ X
X¢ –p X Continuity
–2p O p/2 O
1 Properties of Differentiable
(0, –1) f (x) =
y = sin x x Functions
Y¢ Continuity and Differentiability of

Continuous function Discontinuous function Different Functions

Cauchy’s Definition of Continuity


A real valued function f ( x ) is said to be continuous at x = a, if for any
arbitrary positive number E, however small but different from zero, there
exists a positive number d, such that
| x - a| < d Þ | f ( x ) - f ( a )| < e
Þ - e < f(x) - f(a) < e
f(a) - e < f(x) < f(a) + e
632 JEE Main Mathematics

Heine’s Definition of Continuity æ é 4 - hùö


and lim f ( x) = lim ç[ 4 - h] - ê ÷
x ®4 - h ®0 è ë 4 úû ø
A real valued function f ( x ) is said to be continuous at
x = a, if and only if for every sequence { an } of real [Q put x = 4 - h, when x ® 4 - then h ® 0]
numbers for which lim an = a , = lim(3 - 0) = 3
n ®¥ h ®0

we have lim f ( an ) = f ( a ) é 4ù
n ®¥
and f( 4) = [ 4] - ê ú = 4 - 1 = 3
ë 4û
Types of Continuity Q lim f ( x) = f ( 4) = lim f ( x) = 3
x ®4 - x ®4 +
Continuity of a Function at a Point So, function f ( x) is continuous at x = 4.
A function f ( x ) is said to be continuous at a point x = a of ì sin ( p + 1) x + sin x
its domain if and only if it satisfies the given condition ï , x <0
x
(i) f ( a ) exists (a lies in the domain of f ) ï
Example 2. If f ( x) = í q, x = 0 is
(ii) lim f ( x ) exist i.e. lim f ( x ) = lim f ( x ) ï
x ®a x ®a - x ®a +
ï x + x2 - x x >0
or RHL = LHL î 3/ 2
,
x
(iii) lim f ( x ) = f ( a ).
x ®a continuous at x = 0 , then the ordered pair ( p, q) is equal to
(JEE Main 2019)
Continuity in an Open Interval
æ 3 1ö æ 1 3ö æ 5 1ö æ 3 1ö
A function f ( x ) is said to be continuous in an open (a) ç - , - ÷ (b) ç - , ÷ (c) ç , ÷ (d) ç - , ÷
è 2 2ø è 2 2ø è2 2ø è 2 2ø
interval ]a , b[ or a < x < b, if it is continuous at each point
of the interval. ì sin( p + 1) x + sin x
ï , x<0
ï x
Continuity from Left and from Right Sol. (d) Given function f ( x) = í q , x =0
Let f ( x ) be a function defined o an open interval I and let ï 2
x+x - x
a Î I , then f is continuous from the left at a, if lim f ( x ) ï , x>0
x ® a- î x3 / 2
exists and is equal to f ( a ). is continuous at x = 0, then
Similarly, f ( x ) is said to be continuous from the right at f (0) = lim f ( x) = lim f ( x) …(i)
x® 0- x® 0+
a, if lim f ( x ) exists and is equal to f ( a ).
x ® a+ sin( p + 1) x + sin x
lim f ( x) = lim
x ®0 - x ®0 - x
Continuity in a Closed Interval é sin( ax) ù
= p + 1+ 1= p + 2 êëQ xlim = aú
Let f ( x ) be a function on defined on the closed interval ®0 x û
[a , b]. Then, f ( x ) is said to be continuous on the closed
x + x2 - x
interval [a , b], if it is and lim f ( x) = lim
x ®0 + x ®0 + x3 / 2
(i) continuous from the right at a
x[(1 + x)1/ 2 - 1]
(ii) continuous from the left at b and = lim
x ®0 + x x
(iii) continuous on the open interval ]a , b[. æ 1 æ1 ö ö
ç ç - 1÷ ÷
éxù 1
ç1 + x + 2 è 2 ø
Example 1. If f ( x) = [ x] - ê ú, x Î R where [ x] denotes the x + .... - 1÷
2

ë4û ç 2 2! ÷
ç ÷
greatest integer function, then (JEE Main 2019) = lim è ø
(a) lim f ( x) exists but lim f ( x) does not exist x ®0 + x
x ®4 + x ®4 - n(n - 1) 2 n(n - 1(n - 2)) 3
[Q(1+ x) n = 1 + nx + x + x + ... ,| x| < 1]
(b) f is continuous at x = 4 1× 2 1× 2 × 3
(c) Both lim f ( x) and lim f ( x) exist but are not equal æ 1 æ1 ö ö
x ®4 - x ®4 + ç ç - 1÷ ÷
ç 1 2 è2 ø 1
(d) lim f ( x) exists but lim f ( x) does not exist = lim + x + ...÷ =
x ®4 - x ®4 + x ®0 + ç 2 2! ÷ 2
ç ÷
é xù è ø
Sol. (b) Given function f ( x) = [ x] - ê ú , x Î R
ë 4û From Eq. (i), we get
1
æ é 4 + hùö f (0) = q =
Now, lim f ( x) = lim ç[ 4 + h] - ê ÷ 2
x ®4 + h ®0 è ë 4 úû ø 1 3
and lim f ( x) = p + 2 = Þ p = -
[Q put x = 4 + h, when x ® 4 + , then h ® 0] x ®0 - 2 2
æ 3 1ö
= lim( 4 - 1) = 3 So, ( p, q) = ç - , ÷
h ®0 è 2 2ø
Continuity and Differentiability 633

ìé æ p 1/ x é (1 - cos h) + sin h ù
öù = lim ê (sin h + cos h) ú
ï êtan ç + x ÷ú , x ¹ 0 h ®0 ë 2 sin h
Example 3. Let f ( x) = í ë è 4 øû . û
ï x = 0 é 2h h h ù
î k, ê 2 sin 2 + 2 sin 2 cos 2 ú
For what value of k is f ( x) continuous at x = 0 ? = lim ê (sin h + cos h) ú
h ®0 h h
ê 4 sin cos ú
1
(a) 1 (b) e (c) (d) e2 ë 2 2 û
e é h h ù
é æp öù
1/ x
ê sin 2 + cos 2 ú
Sol. (d) lim f ( x) = lim ê tan ç + x÷ ú = lim ê ´ (sin h + cos h) ú
x ®0 x ®0 ë è4 øû h ®0 h
ê 2 cos ú
1/ x ë 2 û
æ1 + tan x ö
= lim çç ÷ 1
x ® 0 è 1 - tan x ÷
ø Þ k=
2
tan x / x
é ù é ù
êë xlim {(1 + tan x)1/tan x} ú êë xlim {(1 - tan x) -1/tan x} tan x/ x ú
®0 û ®0 û Example 5. The continuity of the function f ( x) = [ x 2] - [ x] 2,
= e ´ e = e2 "x Î R at the end points of the interval [ -1, 0], where [×] denotes
Since, f ( x) is continuous at x = 0 , lim f ( x) = f (0) the greatest integer function, is
x ®a
Þ e2 = k. (a) f ( x) is continuous at the end points of the interval [ -1, 0 ]
(b) f ( x) is not continuous at the end points of the interval
Hence, f ( x) is continuous at x = 0 when k = e2
[ -1, 0 ]
æ p pö (c) f ( x) is continuous only at x = 0
Example 4. If the function f defined on ç , ÷ by
è6 3ø (d) None of the above
ì 2 cos x - 1 p
ïï , x¹ Sol. (b) Continuity at x = -1,
f ( x) = í cot x - 1 4 is continuous, then k is equal to
p f( -1) = [( -1) 2] - [ -1]2
ï k, x=
ïî 4 (JEE Main 2019) = [1] - ( -1) 2 = 1 - 1 = 0
1 1 RHL = lim {[ x2] - [ x]2} = 0 - 1 = - 1
(a) (b) 2 (c) 1 (d)
2 2 x ® -1+

ì 2 cos x - 1 So, f(-1) ¹ RHL


p
ï ,x¹ Continuity at x = 0
Sol. (a) Given function is f ( x) = í cot x - 1 4
ï k , x=
p f(0) = [(0) 2] - [0 ]2 = 0 - 0 = 0
î 4
LHL = lim {[ x2] - [ x2]} = 0 - 1 = - 1
p x ® 0-
Q Function f ( x) is continuous, so it is continuous at x = .
4
So, f(0) ¹ LHL
æ pö lim 2 cos x - 1
\ f ç ÷ = lim f ( x) Þ k = p Hence, the function is not continuous at the end points of the
è 4 ø x® p x® cot x - 1
4 interval [ -1, 0 ].
4
p p
Put x = + h, when x ® , then h ® 0 Example 6. A function f is defined as follows
4 4
æp ö
ì
2 cos ç + h÷ - 1 ï , when - ¥ < x < 0
k = lim è 4 ø ïï1 p
h ®0 æp ö f ( x) = í1 + sin x , when 0 £ x <
cot ç + h÷ - 1 2
è4 ø ï 2
é 1 1 ù ï 2 + æç x - p ö÷ , when p £ x < ¥
2ê cos h - sin hú - 1 ïî è 2ø 2
ë 2 2 û
= lim
h ®0 cot h - 1 Continuity of f ( x) is
-1
cot h + 1 p
(a) f ( x) is continuous at x =
é 2
ê Q cos ( x + y) = cos x cos y - sin x sin y (b) f ( x) is continuous at x = 0
ë
cot x cot y - 1ù (c) f ( x) is discontinuous at x = 0
and cot ( x + y) = (d) f ( x) is continuous over the whole real number
cot y + cot x úû
cos h - sin h - 1 Sol. (a) Continuity at x = 0
= lim
h ®0 -2
LHL at x = 0 lim f ( x) = lim (1) = 1
1 + cot h x ® 0- x ® 0-
634 JEE Main Mathematics

RHL at x = 0 lim f ( x) = lim (1 + sin x) = 1 The function can be made continuous by defining it in
x ® 0+ x ® 0+
such a way that lim f ( x ) = f ( a ).
x ®a
f(0) = 1 + sin 0 = 1
= LHL = RHL = f(0) e.g. f ( x ) = [sin x ], where x Î( 0, p) has a removable
So, f ( x) is continuous at x = 0.
p
discontinuity at x =
p 2
Continuity at x =
2 æ pö
Q lim [sin x ] = 0 but f ç ÷ = 1
p x ®
p è 2ø
LHL at x = = lim f ( x) = lim (1 + sin x) = 1 + 1 = 2 2
2 p- p-
x® x®
2 2 To remove this we redefine f ( x ) as follows
2
RHL at x =
p æ p pö
= lim f ( x) = 2 + ç - ÷ = 2 ì [sin x ], x ¹ p
2 p+ è2 2ø ï 2
x® f(x) = í
2 p
ï 0, x =
æ pö æ p pö
2 î 2
f ç ÷ =2 + ç - ÷ =2
è2ø è2 2ø Now, f ( x ) is continuous for x Î( 0, p).
æ pö
\ LHL = RHL = f ç ÷ Discontinuity of First Kind
è2ø
A function f ( x ) is said to have a discontinuity of the first
æ pö
So, f ( x) is continuous at x = ç ÷ kind or ordinary discontinuity at a, if f( a + 0) and
è2ø f( a - 0) both exist but are not equal.
Hence, f ( x) is continuous over the whole real number.
Discontinuity of Second Kind
Discontinuity A function f ( x ) is said to have a discontinuity of the
second kind at a, if none of the limits f( a + 0) and f( a - 0)
A function f is said to be discontinuous at a point a in
exist.
its domain, if there is a break in the graph of the function
at x = a. Infinite Discontinuity
Then, the point x = a is called a point of discontinuity.
If either or both of f( a + 0) and f( a - 0) be infinite
Since, for the function to be continuous, we must have
( + ¥ or- ¥), then f ( x ) has infinite discontinuity at x = a.
LHL = RHL = Value of function at x = a
i.e. lim f ( x ) = lim f ( x ) = f ( a ) Important Points
x ® a- x ® a+
(i) If f ( x ) is continuous and g( x ) is discontinuous at
The discontinuity of f ( x ) at x = a may arise due to any of x = a , then the product function f( x ) = f ( x ) × g( x ) is
the following situations not necessarily be discontinuous at x = a.
(i) f ( a ) is not define. ì 1
ï sin , x¹0
(ii) L ¹ R i.e. lim f ( x ) ¹ lim f ( x ) e.g. f ( x ) = x and g( x ) = í x
- +
x ®a x ®a
ïî 0, x=0
Þ The limit of function at x = a does not exist.
(iii) L = R ¹ V i.e. lim f ( x ) = lim f ( x ) ¹ f ( a ) (ii) If f ( x ) and g( x ) both are discontinuous at x = a , then
x ® a- x ® a+ the product function f( x ) = f ( x ) × g( x ) is not
Þ The limit of the function exists but is not equal necessarily be discontinuous at x = a.
to the value of the function at x = a. ì 1, x³0 ì -1 , x³0
e.g. f ( x ) = í and g( x ) = í
(iv) Either lim f ( x ) or lim f ( x ) or both non-existing î -1 , x<0 î 1, x<0
x ® a- x ® a+
or infinite. (iii) Continuity of an inverse function If the
function y = f ( x ) is defined, continuous and strictly
Different Kinds of Discontinuity monotonic on the interval x, then there exist a
single valued inverse function x = j( y ) defined,
Removable Discontinuity
continuous and also strictly monotonic in the range
A function f ( x ) is said to have a removable discontinuity of the function y = f ( x ).
at a point a, if f ( a + 0), f ( a - 0) and f( a ) exist but
f ( a + 0) = f ( a - 0) ¹ f ( a ).
Continuity and Differentiability 635

Example 7. Let f : [ -1, 3] ® R be defined as ì a[ x] + x - 1


ï , x¹0
ì| x| + [ x], -1 £ x < 1 Example 9. f ( x) = í [ x] + x , a > 0 at x = 0 is
ï ï log a , x =0
f ( x) = í x + | x|, 1 £ x < 2 î e
ï x + [ x], 2 £ x £ 3 , (a) continuous (b) discontinuous
î
(c) cannot be determined (d) None of these
where, [t] denotes the greatest integer less than or equal to t.
Then, f is discontinuous at (JEE Main 2019) a[ h] + h - 1 ah - 1
Sol. (b) RHL = lim = lim = log e a
(a) four or more points (b) only two points
h ®0 [h] + h h ®0 h
(c) only three points (d) only one point a[ - h] - h - 1 a- h - 1 - 1
LHL = lim = lim [Q [ - h] = - 1 ]
h ®0 [ - h] - h h ®0 -1- h
Sol. (c) Given function f : [ -1, 3] ® R is defined as
a- 1 - 1
ì- x - 1 , - 1 £ x < 0 = = 1 - a- 1
ï -1
ì| x| + [ x], -1 £ x < 1 ï x, 0 £ x <1
ï ï Q RHL ¹ LHL = f(0)
f ( x) = í x + | x|, 1 £ x < 2 = í 2x, 1£ x < 2
ï x + [ x ], 2 £ x £ 3 ï x + 2, 2 £ x < 3 Hence, f ( x) is discontinuous at x = 0
î ï
ïî 6, x=3 Properties of Continuous Functions
[Q if n £ x < n + 1, " n Î Integer, [ x] = n]
(i) If f and g are continuous functions, then
Q lim f ( x) = - 1 ¹ f (0) [Q f(0) = 0 ]
x ®0 - (a) f ± g and fg are continuous.
Q lim f ( x) = 1 ¹ f (1) [Q f(1) = 2] (b) cf is continuous, where c is a constant.
x ®1-
f
Q lim f ( x) = 4 = f (2) = lim f ( x) = 4 [Q f(2) = 4]
(c) is continuous at those points, where g( x ) ¹ 0.
x ®2- x ®2+
g

and lim f ( x) = 5 ¹ f (3) [Q f(3) = 6] (ii) If f is continuous in [a , b] , then it is bounded in [a , b ]


x ®3 - i.e. there exist m and M such that
\ Function f ( x) is discontinuous at points 0, 1 and 3. m £ f (x ) £ M , " x Î [a , b ]

Example 8. Let [t] denote the greatest integer £ t and where, m and M are called minimum and maximum
values of f ( x ) respectively in the interval [a , b].
é4ù
lim x ê ú = A. Then, the function, f ( x) = [ x 2] sin( px) is (iii) If f is continuous in [a , b], then f assumes atleast
x® 0 ë x û
once every value between minimum and maximum
discontinuous, when x is equal to (JEE Main 2020) values of f ( x ). Thus, a £ x £ b Þ m £ f ( x ) £ M or
(a) A+1 (b) A + 21 range of f ( x ) = [m , M ], if x Î [a , b].
(c) A (d) A+5 (iv) If f is continuous in its domain, then | f| is also
continuous in its domain.
Sol. (a) As we know that x - 1 < [ x] £ x, (v) If f is continuous at a and f ( a ) ¹ 0, then there exist
where [ x] denote the greatest integer £ x. an open interval ( a - d, a + d) such that for all
4 é 4ù 4 x Î( a - d, a + d), f ( x ) has the same sing as f ( a ) .
So, -1 < ê ú £
x ë xû x (vi) If f is a continuous function defined on [a , b] such
æ4 ö é 4ù 4 that f ( a ) and f ( b) are of opposite signs, then there
\ lim xç - 1÷ < lim xê ú £ lim x
x ®0 è x ø x®0 ë x û x®0 x exists atleast one solution of the quation f ( x ) = 0 in
the open interval ( a , b).
é 4ù
Þ lim( 4 - x) < lim xê ú £ lim 4
x ®0 x ®0 ë x û x ®0 (vii) If f is continuous on [a , b] and maps [a, b] into [a, b],
then for some x Î[a , b], we have f ( x ) = x.
So, according to Squeeze theorem, we have
1
é 4ù (viii) If f is continuous in domain D, then is also
lim x ú = 4 = A (given) f
x ®0 ê
ë xû
continuous in D - { x : f ( x ) = 0}.
Now, the function f ( x) = [ x2] sin( px) is continuous at every (ix) A function f ( x ) is said to be everywhere continuous,
integral value of x, so according to given options if it is continuous on the entire real line ( - ¥ , ¥ ).
A + 1 = 5 ÎInteger.
/ Integral function of a continuous function is a
So, f ( x) is discontinuous when x = 5 = A + 1. continuous function.
636 JEE Main Mathematics

Continuity of Composition of Function Thus, f ( x ) is differentiable at x = c.


f ( x ) - f ( c)
If f is continuous at x = c and g is continuous at x = f ( c), Û lim exists finitely.
x ®c x-c
then the composite g[ f ( x )] is continuous at x = c,
x sin x
e.g. f ( x ) = 2 and g( x ) =| x| are continuous at x = 0, Differentiability of a Function at a Point
x +2
x sin x The function f ( x ) is differentiable at a point P , iff there
hence the composite ( gof )( x ) = 2 will also be exists a unique tangent at point P.
x +2
In other words, f ( x ) is differentiable at a point P, iff the
continuous at x = 0.
curve does not have P as a corner point i.e. the function is
1 not differentiable at those points on which function has
Example 10. The points of discontinuity of y = 2 ,
u + u -2 holes or sharp edges.
1 Let us consider the function f ( x ) =|x - 1|
where u = , is
x -1 Y
f (x) = – x + 1 f (x) = x – 1
1 3 3 1
(a) , 1, (b) 1, , 2 (c) , 1, 2 (d) 1, 2, 3 f ¢ (x) = – 1 f ¢ (x) = –1
2 2 2 2
1
Sol. (c) The function u = f ( x) = is discontinuous at the
x -1 X¢ X
O 1 2 3
point x = 1. …(i) Y¢
1 1 It is not differentiable at x =1. Since, f ( x ) has sharp edges
The function y = g ( x) = 2 = is
u + u - 2 (u + 2)(u - 1) at x = 1.
discontinuous at u = - 2 and u = 1
when u = - 2,
1
= -2 Þ x =
1 Progressive Derivative/Right Hand
x -1 2 Derivative
1
u =1 Þ =1 Þ x = 2 Let f ( x ) be a function and a be a point in the domain of f.
x -1 Then, right hand derivative is denoted by Rf ¢ ( a ) and
Hence, the composite function y = g [ f ( x )] is discontinuous at defined as
1 f(x) - f(a) f(a + h ) - f(a)
three points x = , 1, 2
2 RHD = Rf ¢ ( a ) = lim = lim
x ® a+ x-a h ®0 h
Applications of Continuity Regressive Derivative/Left Hand
Intermediate Value Theorem Derivative
Suppose f ( x ) is continuous on an interval I and a and b Let f ( x ) be a function and a be a point in the domain of f.
are any two points of I. Then, if y0 is a number between Then, left hand derivative is denoted by Lf ¢ ( a ) and
f ( a ) and f ( b), there exists a number c between a and b defined as
such that f ( c) = y0. f(x) - f(a) f(a - h ) - f(a)
LHD = Lf ¢ ( a ) = lim = lim
Extreme Value Theorem x ® a- x-a h ®0 -h
If f is continuous at every point of a closed interval I, Mathematically, a function f ( x ) is said to be
then f assumes both an absolute maximum value M and f(x) - f(a)
differentiable at a point a in its domain, if lim
an absolute minimum value m somewhere in I. That is, x ®a x-a
there are numbers x1 and x2 in I with f ( x1 ) = m , f ( x2 ) = M exist finitely or if and only if
and m £ f ( x ) £ M for every other x in I.
f(x) - f(a) f(x) - f(a)
lim = lim
Differentiability x ®a - x - a x ®a + x-a

Let f ( x ) be a real valued function defined on an open i.e. Left hand derivative (LHD)
interval ( a , b), where c Î( a , b). = Right hand derivative (RHD)
Then, f ( x ) is said to be differentiable or derivable at x = c or Lf ¢ ( a ) = Rf ¢ ( a )
f ( x ) - f ( c) If Lf ¢ ( a ) ¹ Rf ¢ ( a ),
iff lim exists finitely.
x ®c x-c then function f ( x ) is not differentiable at x = a.
This limit is called the derivative or differential A necessary condition for existence of a finite derivative
coefficient of the function f ( x ) at x = c and is denoted by is that the function must be continuous. But it is not the
d sufficient condition.
f ¢ ( c) or Df ( c) or [ f ( x )]x = c.
dx
Continuity and Differentiability 637

Example 11. The function Example 12. If a function f ( x) is defined as


ì| x |(3e1 /| x| + 4) ì -x, x £0
ï , x¹0 ï
f ( x) = í 2 - e1 /| x| is, f ( x) = í x 2, 0 < x £ 1 , then
ï 0 , x = 0 ï x 2 - x + 1, x >1
î î
(a) differentiable at x = 0 (b) non-differentiable at x = 0 (a) f ( x) is differentiable at x = 0 and x = 1
(c) not continuous at x = 0 (d) cannot be determined (b) f ( x) is differentiable at x = 0 but not at x = 1
(c) f ( x) is differentiable at x = 1but not at x = 0
Sol. (b) The given function may be written as (d) f ( x) is not differentiable at x = 0 and x = 1
ì - x (3e-1/ x + 4) ì - x, if x £ 0
ï , x<0
- 1/ x ï
ï 2-e
2
Sol. (d) f ( x) = í x, if 0 < x £ 1
ï ïx2 - x + 1 , if x > 1
f ( x) = í 0, x=0 î
ï 1/ x
ï x(3e + 4) , x>0 For differentiability at x = 0,
ïî 2 - e1/ x L f ¢(0 - ) = lim
f (0 - h) - f (0)
= lim
- (0 - h) - 0
= -1
h ®0 -h h ® 0 -h
- x (3e-1/ x + 4)
For continuity, lim f ( x) = lim f (0 + h) - f (0) (0 + h 2) - 0
x ®0 -
x ®0 -
2 - e-1/ x R f ¢(0 + ) = lim = lim =0
h ®0 h h ®0 h
- x (3 + 4e1/ x)
= lim =0 L f ¢ (0 - ) ¹ R f ¢ (0 + )
x ® 0- 2e1/ x - 1
\ f ( x) is not differentiable at x = 0.
x (3e1/ x + 4) For differentiability at x = 1,
lim f ( x) = lim
x ® 0+ x ® 0+ 2 - e1/ x f (1 - h) - f (1)
L f ¢(1- ) = lim
x (3 + 4e-1/ x) h ®0 -h
= lim =0
x ® 0+ 2e-1/ x - 1 (1 - h) 2 - 1
= lim
Since, lim f ( x) = f (0) = lim f ( x), f ( x) is continuous at x = 0.
h ®0 -h
x ® 0- x ® 0+ 1 + h 2 - 2h - 1
= lim =2
For differentiability, h ®0 -h
f (h) - f (0) -h (3e-1/ h + 4) f (1 + h) - f (1)
f ¢ (0 -) = lim = lim R f ¢ (1+ ) = lim
h ®0 - h h ®0 -
h (2 - e-1/ h) h ®0 h
(1 + h) 2 - (1 + h) + 1 - 1
- (3 + 4e-1/ h) = lim
= lim =3 h ®0 h
h ®0 -
2e1/ h - 1 (1 + h) (1 + h - 1)
= lim =1
h (3e1/ h + 4) h ®0 h
and f ¢ (0 + ) = lim
h ®0 +
h (2 - e-1/ h) Q - +
L f ¢ (1 ) ¹ R f ¢(1 )
-1/ h
(3 + 4e ) \f ( x) is not differentiable at x = 1.
lim = -3
h ® 0+ 2e-1/ h - 1
Example 13. Let f ( x) = 15 - x - 10 ; x Î R. Then, the set of
Since, f ¢ (0 - ) ¹ f ¢ (0 + ), f ( x) is not-differentiable at x = 0 all values of x, at which the function, g ( x) = f ( f ( x)) is not
differentiable, is (JEE Main 2019)
Differentiability in an Interval (a) {5, 10 , 15, 20} (b) {5, 10 , 15} (c) {10 } (d) {10 , 15}
A function y = f ( x ) defined on an open interval ( a , b) is Sol. (b) Given function is f ( x) = 15 - | x - 10 |, x Î R
said to be differentiable in an open interval ( a , b), if it is and g ( x) = f ( f ( x)) = f (15 -| x - 10 |)
differentiable at each point of ( a , b).
= 15 - |15 - | x - 10 | - 10 | = 15 - | 5 - | x - 10 ||
A function y = f ( x ) defined on a closed interval [a , b] is ì15 - | 5 - ( x - 10)| , x ³ 10
said to be differentiable in closed interval [a , b], if it is =í
differentiable at each point of an open interval ( a , b) î15 - | 5 + ( x - 10)| , x < 10
f(x) - f(a) ì15 - |15 - x| , x ³ 10
and lim =í
x ®a + (x - a) î 15 - | x - 5 | , x < 10
ì 15 + ( x - 5) = 10 + x , x<5
f ( x ) - f ( b) ï 15 - ( x - 5) = 20 - x , 5 £ x < 10
and lim both exist. ï
x ® b- ( x - b) =í
ï 15 + ( x - 15) = x , 10 £ x < 15
OR ïî15 - ( x - 15) = 30 - x , x ³ 15
A function f is said to be a differentiable function, if it is From the above definition it is clear that g ( x) is not
differentiable at every point of its domain. differentiable at x = 5, 10 , 15.
638 JEE Main Mathematics

p
Causes of non-derivability Q f(1) = , f(1+ ) = 0
2
(i) a corner, where function is continuous but LHD and Þ f is discontinuous at x = 1, so it is non-differentiable at x = 1
RHD differ and are finite. Now, at x = –1,
(ii) a cusp, where the slope approaches ¥ from one side p p
and -¥ from the other. f(–1) = – = 0
4 4
(iii) a vertical tangent where the slope approaches ¥ from f(–1+ ) = 0 and f(–1–) = 0
both sides or approaches -¥ from the both sides. Þ f is continuous at x = –1
(iv) discontinuity 1 1
Q f'(–1+ ) = and f'(–1–) = –
2 2
Relation between Derivability \ f is non-differentiable at x = -1
and Continuity \ f is continuous on R – {1} and differentiable on R – {–1, 1}.
(i) If f ¢ ( a ) exists, then f ( x ) is derivable at x = a Þ f ( x ) is
ì k1( x - p) 2 - 1, x £ p
continuous at x = a. In general a function f is Example 15. If the function f ( x) = í is
derivable at x, then f is continuous at x. î k2 cos x, x>p
i.e. if f ( x ) is derivable for every point of its domain of twice differentiable, then the ordered pair (k1, k2) is equal to
(JEE Main 2020)
definition, then it is continuous in that domain. The
æ1 ö æ1 ö
converse of the above result is need not be true (a) (1, 1) (b) ç , 1÷ (c) (1, 0)(d) ç , - 1÷
è2 ø è2 ø
e.g. The functions f ( x ) =| x| ìk1 ( x - p) 2 - 1, x £ p
ì 1 Sol. (b) The given function, f ( x) = í
ï x sin , x ¹ 0 î k2 cos x, x>p
and g( x ) = í x both are continuous at
Is twice differentiable, so f ( x) must be continuous at x = p,
ïî x, x = 0
so lim f ( x) = f ( p)
x = 0 but not derivable at x = 0. x ®p+
Þ -k2 = - 1Þ k2 = 1 …(i)
(ii) Let f+ ¢ ( a ) = l and f- ¢ ( a ) = m , where l and m are finite,
then ì2k1 ( x - p), x < p ì 2k1, x<p
Now, f ¢ ( x) = í ; f ¢ ¢ ( x) = í
(a) l = m Þ f is derivable at x = a Þ f is continuous at î -k2 sin x, x > p î-k2 cos x, x > p
x = a. \ lim f ¢ ¢ ( x) = lim f ¢ ¢ ( x) Þ k2 = 2k1
x ®p+ x ®p-
(b) l ¹ m Þ f is not derivable at x = a but f is
k2 1
continuous at x = a. If a function f not Þ k1 = = [from Eq. (i)]
differentiable but is continuous at x = a , it 2 2
æ1 ö
geometrically implies a sharp corner or kink at \The ordered pair (k1, k2) = ç , 1÷
è2 ø
x = a.
(c) If f is not continuous at x = a , then it is not Properties of Differentiable Functions
differentiable at x = a.
(i) Every polynomial function is differentiable at each
ìp -1 x Î R.
ï 4 + tan x | x| £ 1 (ii) The exponential function a x , a > 0 is differentiable
Example 14. The function f ( x) = í is
1
ï (| x| - 1), | x| > 1 at each x Î R.
î2
(iii) Every constant function is differentiable at each
(JEE Main 2020)
x Î R.
(a) both continuous and differentiable on R - {1}
(iv) The logarithmic function is differentiable at each
(b) both continuous and differentiable on R - {-1}
point in its domain.
(c) continuous on R - {-1} and differentiable on R - {-1, 1}
(v) Trigonometric and inverse-trigonometric functions
(d) continuous on R - {1} and differentiable on R - {-1, 1}
are differentiable in their respective domains.
ìp –1
ï 4 + tan x, | x| £ 1| (vi) The sum, difference, product and quotient of two
Sol. (d) Given function, f ( x) = í differentiable functions is differentiable.
1
ï (| x|–1), | x| > 1|
î2 (vii) The composition of differentiable function is a
ìp –1 differentiable function.
x Î [–1, 1]
ï 4 + tan x, (viii) Absolute functions are always continuous through-
ïï 1
Þ f ( x) = í ( x – 1), x > 1 at x = 1 out but not differentiable at their critical point.
ï 2 e.g. | x - a| is continuous throughout but not
1
ï (–x – 1), x < –1
ïî 2 differentiable at x = a.
Continuity and Differentiability 639

Continuity and Differentiability of Different Functions

Type of Functions Curve Domain and Range Continuity and Differentiability

Identity function f( x ) = x Domain = R, Range = ] - ¥ , ¥ [

Exponential function f( x ) = a x , a > 0, a ¹ 1 Domain = R, Range = ] 0, ¥ [


Continuous and differentiable in their domain
Logarithmic function f( x ) = log a x; x, a > 0 Domain = ( 0, ¥ ), Range = R
and a ¹ 1

Root function f( x ) = x Domain = [0, ¥ ), Range = [0, ¥ ) Continuous and differentiable in ( 0, ¥ )

Greatest integer function f( x ) = [ x ] Domain = R, Range = I


Other than integral value it is continuous and
Least integer function f( x ) = ( x ) Domain = R, Range = I
differentiable
Fractional part function f( x ) = { x} = x - [ x ] Domain = R, Range = [0, 1)

Signum function ì -1, x<0 Domain = R, Range = { - 1, 0, 1}


| x| ï Continuous and differentiable everywhere except at
f( x ) = = í 0, x=0
x ï x=0
î 1, x >0

Constant function f ( x )= c Domain = R, Range = {c} and where


c ® constant

Polynomial function Domain = R Continuous and differentiable everywhere.

Sine function y = sin x Domain = R,


Range = [ - 1, 1]

Cosine function y = cos x Domain = R, Range = [ - 1, 1]

Tangent function y = tan x é pù


Domain = R, = R - ê(2 n + 1) ú,
ë 2û
Range = R
Continuous and differentiable in their domain
Cosecant function y = cosec x Domain = R - np,
Range = ( - ¥, - 1] È [1, ¥ )

Secant function y = sec x ì pü


Domain = R - í(2 n + 1) ý,
î 2þ
Range = ( - ¥, - 1] È [1, ¥ )

Cotangent function y = cot x Domain = R - { np}, Range = R

Arc sine function y = sin-1 x Domain = [- 1, 1],


é p pù
Range = ê - , ú
ë 2 2û

Arc cosine function y = cos -1 x Domain = [- 1, 1], Range = [0, p ]

Arc tangent function y = tan-1 x æ p pö


Domain = R, Range = ç - , ÷
è 2 2ø

Arc cosecant function y = cosec -1 x Domain = ( - ¥, 1] È [1, ¥ ), Continuous and differentiable in their domain
æp pö
Range = ç , ÷ - { 0}
è2 2 ø

Arc secant function y = sec -1 x Domain = ( - ¥, - 1] È [1, ¥ ),


ìpü
Range = [0, p ] - í ý
î2 þ

Arc cotangent function y = cot -1 x Domain = R, Range = ( 0, p )


640 JEE Main Mathematics

1
Problems on Functional Equations Þ f ¢ ( x) = 2 ´
1 + x2
While solving problems based on this topic, always keep
2
in mind the concept of Þ f ¢ ( x) =
1 + x2
(i) Limits (ii) Continuity
On integrating both sides, f ( x) = 2 tan -1 ( x) + C
(iii) Differentiability (iv) Function
where f(0) = 0 Þ C = 0.
In this type of problems, firstly we simplify the given
condition and then use the derivative formula Thus, f ( x) = 2 tan -1 x
f(x + h ) - f(x) æ 1 ö -1 æ 1 ö p p
f ¢ ( x ) = lim Hence, f ç ÷ = 2 tan ç ÷ =2× =
h ®0 h è 3ø è 3ø 6 3
2 2
Further, if it is needed, then we use the integration and and f ¢ (1) = = =1
simplify it. 1 + 12 2

æ x+y ö Example 17. Let f : R ® R is a function satisfies condition


Example 16. If f ( x) + f (y) = f çç ÷÷ for all x, y Î ( -1, 1) and
è 1 - xy ø f ( x + y3) = f ( x) + [ f (y)]3 for all x, y Î R. If f ¢ (0) ³ 0, then f (10) is
f ( x) æ 1 ö (a) 8 (b) 9
lim = 2, then f ç ÷ and f ¢ (1) are
x®0 x è 3ø (c) 0 or 10 (d) 2 or 4
p p 1 p p
(a) and 1 (b) and (c) and 0 (d) and 3 Sol. (c) Given, f ( x + y3) = f ( x) + [ f (y)]3 …(i)
3 6 2 2 4
and f ¢ (0) ³ 0 …(ii)
æ x+y ö Replacing x, y by 0
Sol. (a) f ( x) = f (y) = f çç ÷÷ …(i)
è1 - xy ø f (0) = f (0) + f (0)3
Putting x = y = 0 , we get Þ f(0) = 0 …(iii)
f(0) = 0 f (0 + h) - f (0) f (h)
Putting y = - x, we get Also, f ¢ (0) = lim = lim …(iv)
h ®0 h h ®0 h
f ( x) + f ( - x) = f (0)
Þ f ( - x) = - f ( x) …(ii) f [0 + (h1/3)3 ] - f (0)
Let I = f ¢ (0) = lim
f ( x) h ®0 (h1/3)3
Also, lim =2
x ®0 x
f [(h1/3)3 ]
f ( x + h) - f ( x) = lim
Now, f ¢ ( x) = lim …(iii) h ®0 (h1/3)3
h ®0 h
3
f ( x + h) + f ( - x) é f (h1/3) ù
= lim = lim ê 1/3 ú = I3
h ®0 h h ® 0 (h )
ë û
[using Eq. (ii), f ( - x) = - f ( x)]
I = I3
æ x+h-x ö
f çç ÷ or I = 0 ,1, - 1 as f ¢ (0) ³ 0
1 - ( x + h)( - x) ÷ø
f ¢ ( x) = lim è [using Eq. (i)] \ f ¢ (0) = 0 , 1 …(v)
h ®0 h
( x + h) - f ( x)
é æ h öù Thus, f ¢ ( x) = lim f
ê f çç ÷ú h ®0 h
1 + x ( x + h) ÷ø ú
Þ f ¢ ( x) = lim ê è f [ x + (h1/3)3 ] - f ( x)
h ®0 ê h ú = lim
ê ú h ®0 (h1/3)3
ë û
æ h ö f ( x) - [ f (h1/3)]3 - f ( x)
f çç ÷ f ¢ ( x) = lim [using Eq. (i)]
1 xh x2÷ h ®0 (h1/3)3
Þ f ¢ ( x) = lim è + + ø ´ æç 1 ö
÷
h ®0 æ h ö ç 1 + xh + x ø2÷ é f (h1/3) ù
ç ÷ è Þ f ¢( x) = lim ê 1/3 ú = [ f ¢ (0)]3
ç1 + xh + x2 ÷ h ® 0 (h )
ë û
è ø
æ h ö Þ f ¢( x) = 0 , 1 [as f ¢ (0) = 0 , 1using Eq. (v)]
f çç ÷

è 1 + xh + x ø ´ lim 1 On integrating both sides,
Þ f ¢ ( x) = lim
h ®0 æ h ö h ® 0 1 + xh + x2 f ( x) = c or x + c as f(0) = 0
ç ÷
ç1 + xh + x2 ÷
è ø Þ f ( x) = 0 or x.
é f ( x) ù
êëusing xlim®0 x
= 2ú
û
Thus, f(10) = 0 or 10.
Continuity and Differentiability 641

Practice Exercise
ROUND I Topically Divided Problems
Problems Based on Continuity ì sin 3 ( 3) × log ( 1 + 3x)
ï , x¹0
ì a|p - x|+1, x £ 5 7. If f ( x) = í (tan -1 x ) 2 ( e5 x - 1) x is
1. If the function f ( x) = í is continuous ï a, x =0
î b|x - p|+3, x > 5 î
at x = 5, then the value of a - b is (JEE Main 2019) continuous in [0, 1], then a equals
-2 2 2 2 3 5
(a) (b) (c) (d) (a) 0 (b) (c) 2 (d)
p+5 p+5 p -5 5-p 5 3

2. If f : R ® R is a function defined by 8. The value of f at x = 0, so that function


æ 2x - 1 ö 2x - 2- x
f ( x) = [ x ] cos ç ÷ p, where [ x ] denotes the f ( x) = , x ¹ 0 is continuous at x = 0 , is
è 2 ø x
greatest integer function, then f is (a) 0 (b) log 4 (c) 4 (d) e4
(a) continuous for every real x 9. Let f : R ® R be defined as
(b) discontinuous only at x = 0
(c) discontinuous only at non-zero integral values of x ì æ px ö
ï 2 sin çè - 2 ÷ø, if x < -1
(d) continuous only at x = 0 ï
f ( x) = í|ax 2 + x + b|, if - 1 £ x £ 1.
ì x 2 - ( a + 2) x + 2 a ï sin ( px),
ï , x¹2 if x >1
3. If f ( x) = í x-2 is continuous ï
ï x=2 î
î 2,
at x = 2 , then a is equal to If f ( x) continuous on R, then a + b equals
(JEE Main 2021)
(a) 0 (b) 1 (c) -1 (d) 2
(a) 3 (b) - 1 (c) - 3 (d) 1
ì log (1 + 2 ax) - log (1 - bx)
ï , x¹0 x - ex + cos 2 x
4. If f ( x) = í x is 10. If f ( x) = , where x ¹ 0, is continuous at
ïî k, x = 0 x2
continuous at x = 0, then k is equal to x = 0, then (where, {×} and [ x ] denote the fractional
(a) 2a + b (b) 2a - b part and greatest integer)
5
(c) b - 2a (d) a + b (a) f (0) = (b) [ f (0)] = - 2
2
ì [ x ] + [ -x ], x ¹ 2 (c) { f (0)} = - 0.5 (d) [ f (0)] × { f (0)} = - 1.5
5. If f ( x) = í (where, [×] denotes
î l , x=2
ì - 4 sin x + cos x, for x £ - p/ 2
greatest integer function), f is continuous at x = 2 , ï
11. If f ( x) = í a sin x + b, for - p/ 2 < x < p/ 2 is
then l is equal to ï cos x + 2 , for p/ 2 £ x
î
(a) -1 (b) 0 (c) 1 (d) 2
continuous for all x, then the values of a and b are
ì e px + log e (1 + 4 x) + q
ï , x¹0 (a) -1 , 3 (b) 1 , - 3 (c) 1, 3 (d) -1 , - 3
6. If the function g ( x) = í x3
ïî r, x =0 12. Let f ( x) be a continuous function which satisfies
is continuous at x = 0, then find the values of p, q x2
f ( x 2 + 1) = and f ( x) > 0 for all x Î R, then
and r (JEE Main 2021) f ( 2 x) - 1
- 32 32
(a) - 4, 1, (b) 4, 1, lim f ( x) is equal to
3 3 x ®1
- 32 32 (a) 2 (b) 4
(c) 4, 1, (d) - 4, - 1,
3 3 (c) 1 (d) Does not exist
642 JEE Main Mathematics

13. Let function f be defined as f : R + ® R + and ì ( 4 x - 1) 3


ï 2
,x¹0
function g is defined as g : R + ® R + . Functions f ï sinæ x ö log æç 1 + x ö÷
21. If f ( x) = í ç ÷ ç is a continuous
and g are continuous in their domain. Suppose, the èaø è 3 ÷ø
ï
x n f ( x) + x 2 ï 9 (log 4) 3 , x =0
function h( x) = lim , x > 0. If h( x) is î
n ® ¥ x n + g ( x)
function at x = 0, then the value of a is equal to
continuous in its domain, then f (1) × g (1) is equal to (a) 3 (b) 1 (c) 2 (d) 0
(a) 2 (b) 1 (c) 1/2 (d) 0 3
ì sin ( x ) log(1 + 3x)
ì - 2 sin x, - p £ x £ - p ï , x¹0
ï 22. The function f ( x) = í (tan -1 x ) 2 ( e5 x - 1) x
2 x =0
ï p p ï a,
14. If f ( x) = í a sin x + b, - < x < and it is î
ï 2 2 is continuous in [0, 1], if
p
ï cos x, £x£p 3
î 2 (a) a = 0 (b) a =
5
continuous on [ - p, p], then 5
(c) a = 2 (d) a =
(a) a = 1 and b = 1 (b) a = - 1 and b = - 1 3
(c) a = - 1 and b = 1 (d) a = 1 and b = - 1
ì cos2 x - sin 2 x - 1
( ex - 1) 2 ï , x¹0
15. If f ( x) = for x ¹ 0 and f (0) = 12 . If 23. Let f ( x) = í x2 + 4 - 2 , then the
x x
sinæç ö÷ logæç 1 + ö÷ ï
èaø è 4ø î a , x =0
f is continuous at x = 0, then the value of a is value of a in order that f ( x) may be continuous at
(a) 1 (b) - 1 (c) 2 (d) 3 x = 0 is
log e (1 + x) - log e (1 - x) (a) - 8 (b) 8 (c) - 4 (d) 4
16. For the function f ( x) = to be
x ì x, if x is rational
continuous at x = 0, the value of f (0) should be 24. If f ( x) = í , then
î -x, if x is irrational
(a) -1 (b) 0 (c) -2 (d) 2
1
(a) f (x) is an odd function
p p 1
17. If f ( x) = (sin x) p - 2x
, x ¹ is continuous at x = , (b) f (x) is continuous at x =
2 2 2
p (c) f (x) is continuous at x = 0
then f æç ö÷ is
è2ø (d) f (x) is a periodic function
(a) 0 (b) 1 ( -e x + 2 x )
(c) e (d) None of these 25. The value of f (0), so that may be
x
2 x - sin -1 x continuous at x = 0 , is
18. If the function f ( x) = , ( x ¹ 0) is
2 x + tan -1 x æ1ö
(a) log ç ÷ (b) 0
continuous at each point of its domain, then the è2ø
value of f (0) is (c) 4 (d) -1 + log 2
1 2 1 ln (1 + ax) - ln (1 - bx)
(a) 2 (b) (c) (d) - 26. The function f ( x) = is not
3 3 3 x
sin 2 x + A sin x + B cos x defined at x = 0. The value which should be
19. If f ( x) = is continuous at
x3 assigned to f at x = 0, so that it continuous at x = 0,
x = 0, then the values of A, B and f (0) are is
(a) A = - 2, B = 0 and f (0) = - 1 (a) a - b (b) a + b
(b) A = 0, B = - 2 and f (0) = 1 (c) ln a - ln b (b) ln a + ln b
(c) A = 1, B = - 1 and f (0) = 0
(d) None of the above Problems Based on Discontinuity
ì 1 + kx - 1 - kx ì 1 + x, 0£x£2
ï , for - 1 £ x < 0 27. Let f ( x) = í , then the points of
20. If f ( x) = í x is î 3 - x, 2 < x £3
ï 2 x 2 + 3x - 2, for 0 £ x £ 1
î discontinuity of g ( x) = f [ f ( x)] is/are
continuous at x = 0, then k is equal to (a) only at x = 1 (b) x = 2, 3
(a) - 4 (b) - 3 (c) - 2 (d) - 1 (c) only at x = 3 (d) x = 1, 2
Continuity and Differentiability 643

ì 1 (a) lim f (x) = lim f (x)


ï , x¹0 x ®1 x ®3
28. If f ( x) = í e4 x + 1 , then
(b) f (x) is discontinuous at x = 3
ïî 0, x =0
(c) f (x) is discontinuous at x = 1
(a) lim f (x) = 1 (d) f (x) is discontinuous at x = 1 and 3
x ® 0+
(b) lim f (x) = 0 1
35. Given f (0) = 0 and f ( x) = for x ¹ 0. Then,
x ® 0- (1 - e-1/ x )
(c) f (x) is discontinuous at x = 0
only one of the following statements on f ( x) is true.
(d) f (x) is continuous at x = 0
That is f ( x) is
29. The number of points of discontinuity of (a) continuous at x = 0
f ( x) = [ 2 x 3 - 5 ] in [1 , 2) is equal to (b) not continuous at x = 0
(where, [×] denotes greatest integer function) (c) both continuous and differentiable at x = 0
(a) 14 (b) 13 (c) 10 (d) 8 (d) not defined at x = 0
ì|x + 1|, x<-2 ì x 2 cos e1/ x when x ¹ 0
ï 2 x + 3, 36. Let f ( x) = í . Then, f ( x) is
ï -2 £ x < 0 î 1 when x = 0
30. Function f ( x) = í 2 is
x +3 0 £ x <3 (a) discontinuous at x = 0
ï
ï x 3 - 15, x ³3 (b) continuous but not differentiable at x = 0
î
(c) differentiable at x = 0
(a) continuous at all points in R.
(d) lim f (x) exist
(b) continuous only at x = - 2 x ®0
(c) continuous at all points in R except at x = - 2 ì 1 + x2 , x< 3
(d) None of the above
ï
ï , 3£x<4
1 1 37. Let f ( x) = í 3x - 1 , where [ x ] is the
31. If f ( x) = and g ( x) = 2 , then the , 4 £ x <5
ï [x]
( x + 1) ( x - 2) x ï |1 - x |,
î x ³5
points of discontinuity of f [ g ( x)] are
greatest integer £ x. The number of points of
ì 1 ü ì 1 1 ü
(a) í-1 , 0 , 1 , ý (b) í- , -1 ,0 ,1 , ý discontinuity of f ( x) is R is
î 2þ î 2 2þ
(a) 3 (b) 0 (c) infinite (d) 1
ì 1 ü
(c) {0, 1} (d) í0, 1 , ý
î 2þ
Problems Based on Differentiability
32. Let f : [ a, b] ® R be any function which is such that
38. Let f : R ® R be a function defined by
f ( x) is rational for irrational x and f ( x) is irrational
for rational x. Then, in [ a, b] f ( x) = max { x, x 2 }. Let S denote the set of all points
(a) f is discontinuous everywhere in R, where f is not differentiable. Then,
(JEE Main 2020)
(b) f is continuous only at x = 0
(a) {0, 1} (b) {0}
(c) f is continuous for all irrational x and
(c) f {an empty set} (d) {1}
discontinuous for all rational x
(d) f is continuous for all rational x and discontinuous 39. Let f : R ® R be differentiable at c Î R and f ( c) = 0.
for all irrational x
If g ( x) =|f ( x)|, then at x = c, g is (JEE Main 2019)
33. Let the functions f : R ® R and g : R ® R be (a) not differentiable
defined as (b) differentiable if f ¢ (c) ¹ 0
(c) not differentiable if f ¢ (c) = 0
ì x + 2, x < 0 ì x 3, x <1
f ( x) = í 2 and g ( x) = í (d) differentiable if f ¢ (c) = 0
î x , x ³ 0 î 3x - 2, x ³1
ì max {|x|, x 2 }, |x|£ 2
40. Let f ( x) = í
Then, the number of points in R where ( fog)( x) is î 8 - 2 |x|, 2 <|x |£ 4
NOT differentiable is equal to (JEE Main 2021)
Let S be the set of points in the interval ( -4, 4) at
(a) 3 (b) 1 (c) 0 (d) 2 which f is not differentiable. Then, S (JEE Main 2019)
ì x + 2 , when x < 1 (a) equals {-2, - 1, 0, 1, 2}
ï
34. If f ( x) í 4 x - 1 , when 1 £ x £ 3 , then the correct (b) equals {-2, 2}
ï x 2 + 5 , when x > 3 (c) is an empty set
î
(d) equals {-2,-1, 1, 2}
statement is
644 JEE Main Mathematics

41. If f is a real-valued differentiable function 49. The set of points of differentiability of the function
2 ì x + 1 -1 x¹0
satisfying|f ( x) - f ( y)|£ ( x - y) ; x, y Î R and
ï ,
f (0) = 0, then f (1) is equal to f ( x) = í ( x) 3/ 2 is
(a) 1 (b) 2 (c) 0 (d) –1 ïî 0, x =0

42. If f ( x) is differentiable at x = 1 and (a) R (b) [0, ¥ ] (c) (0, ¥ ) (d) R - {0}
1 x
lim f ( 1 + h) = 5, then f ¢ ( 1) is equal to 50. If f ( x) = for x Î R, then f ¢ (0) is equal to
h®0 h 1 + |x |
(a) 6 (b) 5 (c) 4 (d) 3 (a) 0 (b) 1 (c) 2 (d) 3
ì g ( x) cos (1/ x), if x ¹ 0 51. The number of points of non-differentiability for
43. Let f ( x) = í , where g ( x) is an p
î 0, if x = 0 the function f ( x) =|x |+ |cos x |+ tanæç x + ö÷ in the
è 4ø
even function, differentiable at x = 0, passing
interval ( - 2 , 2) is
through the origin. Then, f ¢ (0) is equal to
(a) 1 (b) 2 (c) 3 (d) 4
(a) 1 (b) 0
(c) 2 (d) Does not exist ì ( x - 1) 2 sin 1 -|x |,
ï if x ¹ 1
52. Let f ( x) = í ( x - 1) be a
44. The function f ( x) = ( x 2 - 1)|x 2 - 3x + 2|+ cos (|x|) is if x = 1
ï - 1,
not differentiable at x = î
(a) -1 (b) 0 real-valued function. Then, the set of points where
(c) 1 (d) 2 f ( x) is not differentiable is
(a) {0} (b) {0, 1} (c) {1} (d) null set
45. If f ( x) = |x|- 1 , then points, where f ( x) is not
ì x
ï , x¹0
differentiable, is/(are) 53. For the function f ( x) = í 1 + e1/ x , the
(a) 0, ± 1 (b) ± 1 ï x =0
î 0,
(c) 0 (d) 1
derivative from the right, f ¢ (0 + ) and the derivative
46. Let the function f , g and h be defined as follows:
from the left f ¢ (0 - ) are
ì æ1ö (a) {0, 1} (b) {1, 0} (c) {1, 1} (d) 0, 0
ï x sin ç ÷ , for - 1 £ x £ 1 and x ¹ 0
f ( x) = í èxø ,
ïî 0, for x = 0 Problems Based on Continuity and
ì 2 æ1ö Differentiability
ï x × sin ç ÷ , for - 1 £ x £ 1 and x ¹ 0
g ( x) = í èxø ì ex , x £0
ïî 0 , for x = 0 54. If f ( x) = í , then
î|1 - x|, x >0
and h( x) =|x|3, for -1 £ x £ 1 (a) f (x) is differentiable at x = 0
(b) f (x) is continuous at x = 0, 1
Which of these functions are differentiable at x = 0 ?
(c) f (x) is differentiable at x = 1
(a) f and g (b) f and h
(d) None of the above
(c) g and h (d) None of these
ì - æç 1 + 1 ö÷
ï ç ÷
47. Let f ( x) = [ n + p sin x ], x Î (0, p), n Î Z , p is a prime 55. If f ( x) = í xe è|x| x ø , x ¹ 0 , then f ( x) is
number and [×] denotes the greatest integer ï 0 ,x =0
î
function. The number of points at which f ( x) is not (a) continuous as well as differentiable for all x
differentiable is (b) continuous for all x but not differentiable at x = 0
(a) p (b) p - 1 (c) neither differentiable nor continuous at x = 0
(c) 2 p + 1 (d) 2 p - 1 (d) discontinuous everywhere
ì x 2 cos p/ x , x ¹ 0, x Î R ì x + { x } + x sin { x }, for x ¹ 0
48. If f ( x) = í , then f is 56. If f ( x) = í ,
î 0, x =0 î 0, for x = 0
(a) differentiable both at x = 0 and at x = 2 where {×} denotes fractional part of x, then
(b) differentiable at x = 0 but not differentiable at x = 2 (a) f is continuous and differentiable at x = 0
(c) not differentiable at x = 0 but differentiable at x = 2 (b) f is continuous but not differentiable at x = 0
(c) f is neither continuous nor differentiable at x = 0
(d) differentiable neither at x = 0 nor at x = 2
(d) None of the above
Continuity and Differentiability 645

2
57. Let f ( x) = degree (ux + u2 + 2u + 3), then at x = 2, 63. Let f : R ® R be a function satisfying
f ( x) is f ( x + y) = f ( x) + l xy + 3x 2 y 2 for all x, y Î R. If
(a) continuous but not differentiable f (3) = 4 and f (5) = 52, then f ¢ ( x) is equal to
(b) differentiable (a) 10x (b) -10x (c) 20x (d) 128 x
(c) discontinuous
(d) None of the above 64. If f ( x), g ( x) be differential functions and
f (1) = g (1) = 2 , then
58. If f ( x) = x 5 × Sgn ( x), then at x = 0, f ( x) is
f (1) g (x) - f (x) g (1) - f (1) + g (1)
(a) everywhere continuous and differentiable lim is equal to
x ®1 g (x) - f (x)
(b) everywhere continuous but not differentiable
(c) everywhere differentiable but not continuous (a) 0 (b) 1
(d) neither continuous nor differentiable (c) 2 (d) None of these
ì e2 x - 1 , x <0 65. Let f : R ® R be a function defined by
ï
59. The function f ( x) = í bx 2 is f ( x) = max( x, x 3). The set of all points where f ( x) is
ï ax + - 1, x ³ 0
î 2 not differentiable, is
continuous and differentiable for (a) { - 1, 1} (b) { -1, 0} (c) {0, 1} (d) { - 1, 0, 1}
(a) a = 1, b = 2 (b) a = 2 , b = 1 f ( x) - 3
(c) a = 2 , any b (d) any a , b = 4 66. If f ( 9) = 9, f ¢ ( 9) = 4, then lim , equals
x®9 x -3
ì x n sin 1 , x ¹ 0
ï (a) 2 (b) 4 (c) 6 (d) 0
60. Let f ( x) = í x , then f ( x) is
ïî 0, x =0 67. If ( x + y) = f ( x) f ( y) for all x, y Î R, f (5) = 2 and
continuous but not differentiable at x = 0, if f ¢ (0) = 3. Then, f ¢ (5) equals
(a) n Î (0, 1) (b) n Î [1, ¥ ) (a) 6 (b) 5 (c) 4 (d) 3
(c) (-¥ , 0) (d) n = 0
68. If f ( a) = 2, f ¢ ( a) = 1, g ( a) = - 1, g¢ ( a) = 2, then the
ì 1 for |x |³ 1
ï g ( x) f ( a) - g ( a) f ( x)
61. Let f ( x) = í|x | . If f ( x) is continuous value of lim is
x®a x-a
ï ax 2 + b for |x |< 1
î 1
(a) –5 (b) (c) 5 (d) 1
and differentiable everywhere, then 5
1 3 1 3
(a) a = ,b=- (b) a = - ,b= 69. Let f : R ® R be a function given by
2 2 2 2
(c) a = 1, b = - 1 (d) a = b = 1 f ( x + y) = f ( x) × f ( y) for all x, y Î R.
If f ( x) = 1 + x × g ( x) + x 2 g ( x) × f( x) such that
62. Let f ( 2) = 4 and f ¢ ( 2) = 4. Then,
lim g ( x) = a and lim f( x) = b, then f ¢ ( x) is equal to
x f ( 2) - 2 f ( x) x®0 x®0
lim is given by
x®2 x-2 (a) (a + b) f (x) (b) af (x)
(c) b f (x) (d) ab f (x)
(a) 2 (b) –2 (c) – 4 (d) 3

ROUND II Mixed Bag


Only One Correct Option (a)
e
2
e + 3e + 13
1. If a function f ( x) defined by
e
ì aex + be- x , -1 £ x < 1 (b) 2
e - 3e - 13
ï
f ( x) = í cx 2 , 1 £ x £3 1
(c) 2
ï ax 2 + 2 cx , 3 < x £ 4 e - 3e + 13
î
e
be continuous for some a, b, c Î R and (d) 2
e - 3e + 13
f ¢ (0) + f ¢ ( 2) = e, then the value of a is (JEE Main 2020)
646 JEE Main Mathematics

2. Let f : R ® R be a function defined as (a) 2 (b) 3 (c) 5 (d) 4


ì 5, if x £1 9. If f ( x) = log x , then
ï a + bx, if 1 < x <3
ï (a) f (x) is continuous and differentiable for all x in its
f ( x) = í
ï b + 5 x, if 3 £ x <5 domain
ïî 30, if x ³5 (b) f (x) is continuous for all x in its domain but not
differentiable at x = ± 1
Then, f is (JEE Main 2019)
(c) f (x) is neither continuous nor differentiable at
(a) continuous if a = - 5 and b = 10
x=±1
(b) continuous if a = 5 and b = 5
(d) None of the above
(c) continuous if a = 0 and b = 5
(d) not continuous for any values of a and b 10. The value of f (0), so that
3. Let K be the set of all real values of x, where the a 2 - ax + x 2 - a 2 + ax + x 2
f ( x) = becomes
function f ( x) = sin|x |-|x |+ 2 ( x - p) cos|x |is not a+ x - a-x
differentiable. Then, the set K is equal to continuous for all x, is given by
(JEE Main 2019) (a) a a (b) - a
(a) {0} (b) f (an empty set) (c) a (d) -a a
(c) { p } (d) {0, p }
11. Let f ( x + y) = f ( x) + f ( y) for all x and y. If the
4. Let S = ( t Î R : f ( x) =|x - p|× ( e|x| - 1) sin|x |is not
differentiable at t }. Then, the set S is equal to function f ( x) is continuous at x = 0, then f ( x) is
(JEE Main 2018) continuous
(a) f (an empty set) (b) {0} (a) only at x = 0 (b) at x Î R - { 0}
(c) { p } (d) {0, p } (c) for all x (d) None of these
5. For x Î R, f ( x) =|log 2 - sin x| 1
12. Let f ( x) = , then the points of discontinuity of
and g ( x) = f ( f ( x)), then x -1
(JEE Main 2016)
(a) g is not differentiable at x = 0 the composite function y = f [ f { f ( x)}], are
(b) g¢ (0) = cos (log 2) 1 1
(a) x = , 1 and 2 (b) x = 0, ,1
(c) g¢ (0) = - cos (log 2) 2 2
(d) g is differentiable at x = 0 and g¢ (0) = - sin (log 2) 3
(c) x = 0, 1, 2 (d) x = 1, , 2
2
ìk x + 1 , 0 £ x £3
6. If the function g ( x) = í is 13. A function f : R ® R satisfies the equation
î mx + 2 , 3 < x £ 5
differentiable, then the value of k + m is f ( x + y) = f ( x) f ( y) for all x, y Î R, f ( x) ¹ 0. Suppose
(JEE Main 2015) that the function is differentiable at x = 0 and
16 10 f ¢ (0) = 2 . Then, f ¢ ( x) is equal to
(a) 2 (b) (c) (d) 4
5 3 (a) f (x) (b) 2 f (x)
7. Let f : R ® R be a function defined as 1
(c) f (x) (d) None of these
ì sin ( a + 1) x + sin 2 x 2
ï , if x <0 ì -4, -4 £ x < 0
ï 2x 14. Let f ( x) = í 2
,
f ( x) = í b, if x =0 î x - 4, 0£x£4
ï 3
x + bx - x then g ( x) = f (|x |) + | f ( x)|is
ï , if x >0 (a) continuous as well as differentiable at x = 0, 2
î bx 5 / 2
(b) continuous but not differentiable at x = 0, 2
If f is continuous at x = 0, then the value of a + b is (c) Neither continuous nor differentiable at x = 0, 2
equal to (JEE Main 2021) (d) Continuous as well as differentiable at x = 0 and
5 3 continuous at x = 2 but not differentiable at x = 2
(a) - (b) - 2 (c) - 3 (d) -
2 2 15. The function defined by
8. If f ( x) = [sin x ] + [cos x ], x Î [0, 2p], where [. ] ìï |x - 3|, x ³1
denotes the greatest integer function. Then, the total f ( x) = í 1 2 3 13 is
x - x+ ,x <1
number of points, where f ( x) is non-differentiable, îï 4 2 4
is (a) continuous at x = 1 (b) continuous at x = 3
(c) differentiable at x = 1 (d) All of these
Continuity and Differentiability 647

ì ex , x £0 24. Let f : R ® R be a function such that


16. If f ( x) = í , then
î|1 - x|, x > 0 æ x + y ö f ( x) + f ( y)
fç , f (0) = 3 and f ¢ (0) = 3, then
÷=
(a) f (x) is differentiable at x = 0 è 3 ø 3
(b) f (x) is continuous at x = 0, 1 f (x)
(a) is differentiable in R
(c) f (x) is differentiable at x = 1 x
(d) None of the above (b) f (x) is continuous but not differentiable in R
(c) f (x) is continuous in R
æ 3x - x 3 ö -1æ 1 - x
2
ö
17. If f ( x) = cot -1çç ÷
2 ÷
and g ( x) = cos ç
ç 1 + x2
÷,
÷
(d) f (x) is bounded in R
è 1 - 3x ø è ø tan p [( 2 x - 3p) 3 ]
f ( x) - f ( a) 1 25. If f ( x) = ([×] denotes the greatest
then lim , 0 < a < is 1 + [ 2 x - 3p]2
x ® a g ( x) - g ( a) 2
integer function), then
3 3
(a) (b) (a) f (x) is continuous in R
2 (1 + a 2) 2 (1 + x2)
(b) f (x) is continuous in R but not differentiable in R
3 3 (c) f ¢ (x) exists everywhere but f ¢ (x) does not exist at
(c) (d) -
2 2 some x Î R
ì -1 , x < 0 (d) None of the above
ï
18. Let f ( x) = í 0 , x = 0 and g ( x) = sin x + cos x, then 26. The values of p and q so that
ï 1 , x >0 ì x 2 + 3x + p, if x £ 1
î f ( x) = í is differentiable at
points of discontinuity of f { g ( x)} in (0, 2p) is î qx + 2, if x > 1
ì p 3p ü ì3p 7p ü x = 1 are
(a) í , ý (b) í , ý
î2 4 þ î4 4 þ (a) p = 3 and q = 5 (b) p = 5 and q = 3
ì2p 5p ü ì5p 7p ü (c) p = 2 and q = 4 (d) p = 4 and q = 2
(c) í , ý (d) í , ý
î3 3 þ î4 3 þ
27. If the function
19. The domain of the derivative of the function ì x + a 2 2 sin x, 0 £ x < p/ 4
ï
ì tan -1 x , |x|£ 1 f ( x) = í x cot x + b, p / 4 £ x < p/ 2 is
ï
f ( x) = í 1 is ï b sin 2 x - a cos 2 x, p/ 2 £ x k £p
(|x|-1) , |x|> 1 î
îï 2
(a) R - { -1} (b) R
continuous in the interval [0, p], then the values of
(c) R - { -3} (d) None of these ( a, b) are
(a) (-1, - 1) (b) (1, 2) (c) (-1, 1) (d) (1, 1)
20. f ( x) = [ x ] + { x }, where [. ] and {. } denotes the
A sin x + sin 2 x
greatest integer function and fractional part 28. A function f ( x) is defined as f ( x) = ,
x3
respectively, then ( x ¹ 0). If the function is continuous at x = 0, then
(a) f (x) is continuous but non-differentiable at x = 1
(a) A = - 2, f (0) = -1 (b) A = -1, f (0) = - 1
(b) f (x) is differentiable at x = 1
(c) A = 1, f (0) = 0 (d) A = 0, f (0) = 1
(c) f (x) is discontinuous at x = 1
(d) None of the above 29. If f ( x) =|log x |, then
(a) f ¢ (1+ ) = 2 (b) f ¢ (1- ) = - 1
21. f ( x) = [sin x ] + [cos x ], x Î [0, 2p], where [. ] denotes
(c) f ¢ (1) = 1 (d) f ¢ (1) = - 1
the greatest integer function. Total number of
points, where f ( x) is non-differentiable is equal to 30. f ( x) is differentiable function and [ f ( x) × g ( x)] is
(a) 2 (b) 3 (c) 5 (d) 4 differentiable at x = a, then
22. Let f ( x) = sin x, g ( x) = [ x + 1] and h( x) = gof ( x), (a) g (x) must be differentiable at x = a
where [×] is greatest integer function, then h¢ ( p/ 2) is (b) if g (x) is discontinuous, then f (a ) ¹ 0
(a) 1 (b) -1 (c) f (a ) ¹ 0, then g (x) must be differentiable
(c) non-existent (d) None of these (d) nothing can be said

23. The left hand derivative of f ( x) = [ x ] sin ( px) at 31. If x + | y |= 2 y, then y as a function of x is
x = k, k an integer, is (a) not defined for all real x (b) continuous at x = 0
(a) (- 1)k (k - 1) p (b) (- 1)k - 1 (k - 1) p (c) differentiable for all x
(c) (- 1)k kp (d) (- 1)k - 1 kp (d) such that
dy 1
= for x < 0
dx 4
648 JEE Main Mathematics

x 1
ì ( x - 1) sin
32. The set of points where the function f ( x) = ï , if x ¹ 1
(1 + |x |) 39. Let f ( x) = í x -1
is differentiable, is ïî 0 , if x = 1
(a) Only (-¥ , ¥ ) (b) Only [0, ¥ ) Then, which one of the following is true?
(c) Only (-¥ , 0) È (0, ¥ ) (d) Only (0, ¥ ) (a) f is differentiable at x = 1 but not at x = 0.
(b) f is neither differentiable at x = 0 nor at x = 1.
p p
33. If f ( x) = cos é ù cos æç ( x - 1) ö÷ , where [×] denotes the (c) f is differentiable at x = 0 and x = 1.
êë x úû è2 ø (d) f is differentiable at x = 0 but not at x = 1.
greatest integer function, then f ( x) is continuous at
(a) x = 0 (b) x = 3 40. Let f : R ® R be function defined by f ( x) = min
(c) x = 2 (d) None of these { x + 1, x + 1}. Then, which of the following is true?
(a) f (x) ³ 1 for all x Î R
34. Let f ( x + y) = f ( x) + f ( y) + 2 xy - 1 "x, y Î R. If f ( x) (b) f (x) is not differentiable at x = 1
is differentiable and f ¢0) = sin f, then (c) f (x) is differentiable everywhere
(a) f (x) < 0, "x Î R (d) f (x) is not differentiable
(b) f (x) = 0, "x Î R 41. Let f : [1, 10 ] ® Q be a continuous function and
3 f (1) = 10, then f (10) is equal to
(c) f (x) ³ , "x Î R
4 1
(a) (b) 10
(d) -1 £ f (x) £ 1, "x Î R 10
35. Let h( x) = min { x, x 2 } for every real number x. Then, (c) 1 (d) Cannot be obtained

(a) h is not continuous for all x


(b) h is differentiable for all x
Numerical Value Type Questions
x
(c) h ¢ (x) = 2 for all x > 1 42. Let f ( x) = x é ù, for -10 < x < 10, where [ t ] denotes
(d) h is not differentiable at two values of x ëê 2 ûú
the greatest integer function. Then, the number of
36. Let a Î R be such that the function
points of discontinuity of f is equal to ….
ì cos-1 (1 - { x }2 ) sin -1 (1 - { x }) (JEE Main 2020)
ï , x¹0
f ( x) = í { x } - { x }3 43. Let S be the set of points where the function,
ï a, x =0 f ( x) =|2 -|x - 3||, x Î R, is not differentiable. Then,
î
is continuous at x = 0, where { x } = x - { x }, { x } is the å f ( f ( x)) is equal to …. . (JEE Main 2020)
xÎS
greatest integer less than or equal to x. Then,

You might also like